You are on page 1of 1080

Anul IV, Nr.

Ianuarie Iunie 2002

RECREATII

MATEMATICE
REVIST| DE MATEMATIC| PENTRU ELEVI {I PROFESORI

iS

= 1

Editura Crengua Gldu


IAI, 2002

Anul IV, Nr. 1

Ianuarie - Iunie 2002

RECREATII
M A T E M A T I C E
REVIST| DE MATEMATIC| PENTRU ELEVI {I PROFESORI

eiS = 1

Apare cu sprijinul
FILIALEI IAI a SOCIETII de TIINE MATEMATICE

IASI, 2002

Semnificaia formulei de pe copert:


ntr-o form concis, formula e iS
matematicii:
ARITMETICA
GEOMETRIA
ALGEBRA
ANALIZA MATEMATIC

1 leag cele patru ramuri fundamentale ale

reprezentat de 1
reprezentat de S
reprezentat de i
reprezentat de e

Redacia revistei :
Petru ASAFTEI , Temistocle BRSAN, Dan BRNZEI, Ctlin CALISTRU,
Constantin CHIRIL, Constantin COCEA, Eugenia COHAL, Adrian CORDUNEANU,
Paraschiva GALIA, Mihai GRTAN, Paul GEORGESCU,
Dumitru GHERMAN (Pacani), Gheorghe IUREA, Lucian Georges LDUNC,
Gabriel MRANU, Gabriel POPA, Dan POPESCU (Suceava),
Florin POPOVICI (Braov), Maria RACU, Petru RDUCANU, Alin SPUM

Coordonatorul numrului : Temistocle BRSAN


Tehnoredactare computerizat:
Ionu AILINCI, Ctlin CALISTRU, Paul GEORGESCU, Gabriel POPA,
Vlad ROTARIU i un grup de elevi de la Liceul Teoretic Gr. Moisil
i Liceul Teoretic G. Ibrileanu

Adresa redaciei:
Catedra de Matematic Universitatea Tehnic Gh. Asachi Iai
Bd. Carol I, nr.11, 6600, Iai
Tel. 032 213737 / int. 123
E-mail: acord@math.tuiasi.ro

EDITURA CRENGUA GLDU


Toate drepturile rezervate
ISSN 1582 - 1765
Bd. N. Iorga, Bl. K2, ap. 4
Tel. / Fax: 032 - 230598
IAI, 6600

Alin Spum
Spum
Alin
Cu mult tristee i prere de ru anunm dispariia n luna decembrie, anul 2001,
a unuia dintre cei mai ataai i entuziati membri ai redaciei revistei noastre.
S-a nscut n Iai , la 22 noiembrie 1974. n acest ora drag lui, a parcurs n mod
strlucit toate treptele nvmntului, de la cel elementar i pn la cel superior: a absolvit
Liceul de informatic Gr. Moisil i a obinut licena n matematic la Universitatea Al.
I. Cuza. A ocupat prin concurs un post de preparator la Catedra de algebr, Facultatea de
matematic a universitii ieene.
Pasionat de matematic, s-a ndreptat spre domeniul Algebrei abstracte i
aplicaiilor acesteia: dup doi ani de masterat, s-a nscris la doctorat i a susinut examenele
i referatele programate. Lucra la teza de doctorat i urma s plece n Germania, beneficiind
de o burs de studii, n momentul dispariiei sale.
Timpul a pus capt irului de proiecte n care se avnta i pentru realizarea crora
era att de nzestrat.
A muncit cu mult pasiune i pricepere i a obinut de timpuriu frumoase rezultate.
Ca elev, a participat la fazele finale ale Olimpiadelor de matematic, Concursurilor
rezolvitorilor Gazetei Matematice i multor concursuri interjudeene i a fost rspltit cu
premii i meniuni. Ca student, a fost premiat la mai multe ediii ale Concursului studenesc
Traian Lalescu i la Sesiunile de comunicri tiinifice studeneti. Cadru didactic n
nvmntul superior fiind, a participat cu regularitate la Conferinele naionale de Algebr
din ultimii ani.
A dorit mult ca Iaul s aib o revist de matematic elementar i chiar a avut o
iniiativ personal n acest sens. ntors n ar, dup o perioad de studii la Universitatea
din Udine (Italia), s-a ataat imediat de revista Recreaii matematice, care n acel moment
pregtea apariia primului su numr.
Cu competen i fr a-i menaja energia i timpul, a contribuit la creterea
calitii revistei cu fiecare numr nou aprut. Priceperea sa n tehnoredactarea pe calculator
a fost o ans a revistei la primele sale apariii. A fost neobosit n promovarea i distribuirea
revistei, folosind orice prilej pe care-l avea: concursuri, tabere de matematic etc.
Era att de tnr i era o prezen att de vie printre noi, membrii redaciei i
colaboratorii revistei, nct vestea dispariiei lui a fost primit ca un fapt absurd i
incredibil.
Va rmne mereu n amintirea noastr ca un exemplu de pasiune i druire,
hrnicie i modestie, entuziasm i competen.
i nu avem alt mod mai potrivit de a cinsti memoria celui ce a fost Alin Spum,
dect asigurnd apariia revistei, la care a inut att de mult, fcnd-o ct mai interesant i
atractiv.

REDACIA REVISTEI

Petre Osmatescu ( 1925 2001 )


Petre Osmatescu s-a nscut n Romnia la 20 iunie 1925 n satul Vdeni, comuna
Moldova, judeul Cetatea Alb. Ca o nedreptate istoric, oraul omonim din antichitate,
nfiinat n secolul VI nainte de Christos sub denumirea de Tyras i fcnd parte din
Moldova lui tefan cel Mare, este rebotezat astzi Bielgorod Dnestrovski, iar comuna
Moldova - Krutoiarovska i aparin amndou Ucrainei. coala primar a terminat-o n
comuna natal n 1938 (care, la acea dat, mai aparinea Romniei).
n perioada 1945-1947, urmeaz cursurile pregtitoare de pe lng Institutul
Pedagogic Ion Creang din Chiinu i apoi (1947-1951) cursurile institutului propriuzis, ncepndu-i cariera ca profesor de matematic i fizic la coala Medie nr. 2 din Orhei
(1951-1956), unde este ales preedintele seciei metodice de fizic i matematic (19541956). n 1956, este transferat la Institutul Pedagogic din Tiraspol.
n 1962, nfiineaz la Tiraspol Seminarul de Topologie General , Geometrie
Analitic i Funcii Reale pentru studeni i tinerele cadre. n 1964, i susine teza de
doctorat n matematic i fizic .
n 1965, devine confereniar, iar n 1979 profesor la Catedra de matematici
superioare nr. 1 de la Institutul Politehnic din Chiinu. Cu aceast ocazie, introduce
metodele analizei vectoriale n cursul de analiz matematic. Tot n 1965, nfiineaz i
conduce Seminarul de Topologie General de la Universitatea de Stat din Moldova. El
este membru al Societii Matematice din Moldova i Preedinte al Societii East-West.
A fost numit n 1996 Persoan Eminent a Educaiei Publice i a primit
Mileniul 2000. Medalie de Onoare. El a fost fondatorul i organizatorul celor apte
Simpozioane Tiraspoliene de Topologie i Aplicaii, pe care le-a editat. Cercetrile i
rezultatele sale sunt de Topologie general i Teoria funciilor .
A inut conferine n Canada, Egipt, Elveia, Germania, Romnia, Spania i
Turcia. Un ntreg ciclu de conferine l-a inut la Universitatea Tehnic din Iai, cu care s-a
semnat i un acord de cooperare interuniversitar.
O maladie necrutoare a ntrerupt firul vieii sale n noiembrie 2001, punnd astfel
capt unei activiti prodigioase.
Regretatul Profesor Petre Osmatescu , al crui prieten am fost, s-a impus pe
dou fronturi, care se ntreptrund : pe de o parte, pe plan matematic, n topologie, prin
introducerea spaiilor subtile , unde elementul de baz este diviziunea punctului (ceva
analog cu divizarea atomului din fizic), teorie care i-a gsit aplicaii n economie, finane
i sociologie, ct i prin nfiinarea la Chiinu a revistei Scripta Scientiarum
Matematicarum, pe care a condus-o pn n ultima clip, iar pe de alt parte, pe plan
romnesc, s-a dedicat luptei, prin mijloace culturale, pentru reunificarea Basarabiei cu
Romnia, deziderat, care sperm s nu rmn doar un vis al nostru.

Prof. dr. Petru CARAMAN

Niels Henrik Abel 200 de ani de la natere


A fost una dintre fiinele de excepie ce natura
rareori d natere n cursul unui secol.
August Leopold Crelle
N. H. Abel (1802-1829) s-a nscut la Finde ntr-o familie cu frumoase tradiii.
Moartea timpurie a tatlui i falimentul bunicului su au dus familia la o existen precar.
Se cunosc puine lucruri despre Abel. Ocupaiile lui favorite au fost teatrul i matematica.
La 19 ani, student fiind, N. H. Abel propune o rezolvare algebric a ecuaiei de gradul
a cincilea. Aceasta coninea o eroare pe care tot el o gsete doi ani mai trziu. n
Mmoires sur les l'quations algbriques o on dmontre l'impossibilit de la rsolution
de l'quation gnrale du cinquime degr (1824), Abel pune capt ncercrilor de
continuare a lucrrilor lui Tartalia, Cardano i Ferrari, ce au durat mai bine de un secol.
n 1825 face o cltorie de studii n Germania, Frana i Italia. La Berlin cunoate pe
A. L. Crelle, fondatorul revistei Journal fr die reine und angewandte Mathematik. Multe
din lucrrile lui Abel au fost publicate n aceast revist.
n iulie 1826 se afl la Paris, unde cunoate pe cei mai importai membri ai Academiei
de tiine: Cauchy, Laplace, Legendre .a. Pregtete i depune la acest nalt for de tiin
cea mai important lucrare a sa Mmorie sur une proprit gnrale dune classe trs
tendue des fonctions transcendentes. A ateptat zadarnic un rspuns din partea Academiei
de tiine, n particular din partea lui Cauchy i Legendre. Aici, n Paris, apar primele
semne ale bolii sale. Rspunsul mult ateptat vine la cteva luni dup moartea sa. n 1830 i
s-a acordat postum, pentru aceast lucrare, marele premiu al Academiei de tiine
(mpreun cu Jacobi). Memoriul a disprut pentru un timp i a fost publicat abia n 1841.
Din Paris se ntoarce acas unde i triete ultimii doi ani de via mpovrat de griji.
Moare la numai 26 de ani, rpus de tuberculoz, boal incurabil n acea vreme.
n cinci ani de creaii originale i profunde, Niels Henrik Abel a reformat o parte a
analizei matematice i a deschis drumuri noi n teoria grupurilor i n geometria analitic i
algebric. A adus contribuii n teoria ecuaiilor algebrice i ecuaiilor abeliene, teoria
seriilor binomiale i a seriilor n general, teoria funciilor eliptice i, mai general, a
funciilor algebrice. Un numr mare de noiuni i teoreme importante poart numele lui :
integrale abeliene, grupuri abeliene, teorema Abel-Ruffini privind imposibilitatea rezolvrii
prin radicali a ecuaiilor algebrice de grad mai mare dect patru etc.
Recent guvernul norvegian a hotrt crearea unui premiu care s poarte numele
marelui matematician, Premiul Abel. Acesta este destinat domeniului matematicilor i se
acord anual ncepnd cu anul 2002. Valoarea premiului este de 200.000.000 coroane
norvegiene ( 27.100.000 euro). Crearea unui Premiu Abel a fost propus i n 1902 de ctre
regele Oscar II al Suediei i Norvegiei. Datorit separrii celor dou ri n 1905, proiectul
a fost abandonat. Pn acum, cel mai prestigios premiu pentru matematicieni a fost medalia
Fields, acordat din patru n patru ani, ncepnd din 1936. Premiul Abel este ca valoare i
importan comparabil cu Premiul Nobel (ce nu se acord matematicienilor).
Prof. dr. Temistocle BRSAN
4

ARTICOLE SI NOTE MATEMATICE


Asupra ipotezei lui Goldbach
Petru MINU 1
Una din problemele care au impulsionat considerabil dezvoltarea teoriei numerelor
i care nu este nc rezolvat, n ciuda eforturilor fcute n ultimii 250 de ani de
matematicieni dintre cei mai renumii, este aa numita ipotez a lui Goldbach.
Problema a fost pus pentru prima dat n corespondena dintre Christian
Goldbach i Leonhard Euler, la vremea respectiv matematicieni la Academia din
Sankt Petersburg. La 7 iunie 1742, Goldbach i scrie lui Euler: Evident, orice numr
(natural) este suma a trei numere prime. La 30 iunie 1742, Euler i rspunde:
Consider ca o teorem pe deplin adevrat c orice numr par este suma a dou
numere prime, dei nu pot s o demonstrez. Prin tradiie s-a pstrat sub denumirea de
ipoteza lui Goldbach urmtoarea afirmaie:
Propoziia 1. Orice numr natural par, mai mare ca 2 , este suma a dou numere
prime (de exemplu, 4 = 2 + 2 , 6 = 3 + 3 , 8 = 3 + 5 , 10 = 3 + 7 = 5 + 5 etc.).
A fost formulat i o ipotez mai tare:
Propoziia 2. Orice numr natural par, mai mare ca 6, este suma a dou numere
prime diferite.
Propoziia 2 a fost verificat de Pipping pentru toate numerele pare pn la 100000.
Teorema 1. Propoziia 2 este echivalent cu afirmaia: orice numr natural mai
mare ca 17 este suma a trei numere prime diferite.
Demonstraie. Observm c dac n se reprezint sub forma n p  q  r , p, q, r
prime, n cazul cnd n este impar toate trei numerele p, q, r sunt impare, iar n cazul
cnd n este par unul dintre ele este par (deci 2) i celelalte dou impare.
S mai observm c nu exist numere n N care s admit patru reprezentri de
forma: n 3  3  r1 , n 5  5  r2 , n 7  7  r3 , n 11  11  r4 ( r1 , r2 , r3 , r4 prime).
ntr-adevr, n caz contrar ar rezulta c r1 r4  16 , r2 r4  12 , r3 r4  8 . Ca
urmare, dac r4 este de forma r4 3k , atunci r2 este multiplu de 3; dac r4 3k  1 ,

atunci r3 este multiplu de 3 i dac r4 3k  2 , atunci r1 este multiplu de 3, absurd.


Presupunem c Propoziia 2 este adevrat fie n ! 17 , n impar. Numerele pare
n  3 , n  5 , n  7 , n  11 sunt mai mari ca 6 i se scriu sub forma unei sume de dou
numere prime diferite: n  3 p1  q1 , n  5 p 2  q 2 , n  7 p 3  q 3 , n  11 p 4  q 4 .
Conform observaiei de mai sus, din aceste patru reprezentri ale lui n ca sume de trei
numere prime exist cel puin una n care toi termenii sunt diferii.
Dac n este par, n  2 este par i se poate scrie sub forma n  2 p  q , unde p
i q sunt dou numere prime impare diferite, deci n 2  p  q .

Prof. dr. , Catedra de algebr, Univ. Al. I. Cuza, Iai

Invers, presupunem c orice numr mai mare ca 17 este suma a trei numere prime
diferite. Dac n este par, n ! 15 , avem n  2 par , n  2 ! 17 , n  2 2  p  q , deci
n p  q , p i q numere prime diferite. Pentru 6  n d 15 avem: 8=3+5, 10=3+7,
12=5+7, 14=3+11. Cu aceasta demonstraia este complet.
Teorema 2. Dac ipoteza lui Goldbach este adevrat, orice numr natural impar,
mai mare ca 7, este suma a trei numere prime impare.
Demonstraie. Dac 2n  1 ! 7 , 2n  1  3 2 n  1 ! 4 i conform ipotezei lui
Goldbach 2n  1  3 p  q , p, q prime, impare. Deci 2n  1 3  p  q .
Observaie. I.M. Vinogradov a demonstrat, n 1937, c orice numr natural impar
16

mai mare ca 3 3 este sum de trei numere prime.


Considerm propoziia:
Propoziia 3. Orice numr natural impar, mai mare ca 7, este sum de trei numere
prime impare.
Pentru a arta c Propoziia 3 este o teorem (propoziie adevrat) ar trebui
16

verificat c orice numr natural impar n, 7  n  3 3


numere prime impare.

se poate scrie ca o sum de trei

Teorema 3. Dac ipoteza lui Goldbach este adevrat, atunci orice numr ntreg
impar n se poate reprezenta, ntr-o infinitate de moduri sub forma n p  q  r , unde
p, q, r sunt numere prime.
Demonstraie. Fie n un numr ntreg impar. Putem alege, ntr-o infinitate de
moduri, un numr prim impar r astfel nct n  r ! 4 . Conform ipotezei lui Goldbach
exist dou numere prime impare p i q astfel nct n  r p  q .
Teorema 4. Orice numr natural mai mare ca 11 este suma a dou numere
compuse.
Demonstraie. Dac n este par, n  4 este par (deci compus) i n 4  n  4 .
Dac n este impar, n  9 este par (deci compus) i n 9  n  9 .
Observaie. G.H. Hardy i J.E. Littlewood au formulat ipoteza c orice numr
natural n, suficient de mare este suma unui numr prim i a unui ptrat : n p  k 2 , p
prim, k N . Ipoteza nu a putut fi nc confirmat sau infirmat. O alt ipotez a lui
Hardy i Littlewood a devenit teorem prin demonstraia dat de I.V. Linnik n 1959 :
Teorema 5. Orice numr natural, suficient de mare, este suma ntre un numr
prim i dou ptrate: n p  k 2  h 2 , p prim , k , h N .
Demonstraia acestei teoreme nu poate fi fcut cu mijloace elementare.
Bibliografie
1. D. A. Buhtab Teoria cisel, Moskva, 1960.
2. C. Creang, C. Cazacu, P. Minu, Gh. Opai, C. Reischer Introducere n teoria
numerelor, Editura Didactic i Pedagogic, Bucureti , 1965.
3. P. Minu Teoria numerelor. Capitole introductive, Ed. Crengua Gldu, Iai, 1997.
4. W. Sierpinski Ce tim i ce nu tim despre numerele prime, Ed. tiinific, Bucureti,
1966.

Generalizarea teoremei de omologie a lui Barbilian


Constantin COCEA 1
Strlucitul matematician Dan Barbilian a demonstrat c dou triunghiuri echilaterale,
de acelai centru, sunt triomoloage. Vom extinde n cele ce urmeaz acest rezultat. Are loc
urmtoarea
Teorem. Fie ABC un triunghi cu centrul cercului nscris I; A 1 , B 1 , C 1 punctele de
contact ale cercului nscris cu laturile, iar A 2 B 2C 2 un triunghi avnd centrul cercului
circumscris n I i invers asemenea cu A 1B 1C 1 . Atunci dreptele AA 2 , BB 2 , CC 2 sunt
concurente (adic triunghiurile ABC i A 2B 2C 2 sunt omologice)
Demonstraie. Fie 0, a, b, c afixele punctelor I , A, B, C , iar C (I,1) cercul nscris de
raz unitate. Atunci:
a1 b1 c1 1
(1)

Triunghiul A 2B 2C

fiind invers asemenea cu A 1 B 1C 1 , cu centrul cercului

circumscris I , rezult c exist O C nct


a 2 O a1 , b 2 Ob1 , c 2 Oc 1
(2)
n loc s demonstrm concurena dreptelor AA 2 , BB 2 , CC 2 , vom demonstra (avnd
n vedere teorema lui Desargues) c punctele ^D ` BC B2 C 2 , ^E ` CA C 2 A2 i
^J ` AB A2 B2 sunt coliniare.
a1
. Cum BC este tangent cercului nscris,
Panta complex a dreeptei IA1 este k IA1
a1
rezult c ecuaia lui BC este
a
(BC) z  a1  1 z  a1 sau z a1  z a1  2 0 .
(3)
a1

Se tie c raportul n care o dreapt de ecuaie D z  E z  J


(BC), afixele lui B i C fiind b respectiv c, este
R

D b  Eb  J

Dc  E c  J
Prin urmare, innd seama de (2) i (3), avem :

DB 2

Ob1 a1  Ob1a1  2

DC 2

Oc 1 a1  Oc 1a1  2

Profesor , Liceul Teoretic D.Cantemir, Iai

(4)

Avnd n vedere expresiile analoage cu (4) ale rapoartelor

0 mparte un segment

EC 2
EA2

JA2
, obinem:
JB2

DB 2 EC 2 JA2

DC 2 EA2 JB 2
ceea ce probeaz coliniaritatea punctelor D , E , J , deci triunghiurile ABC i A2 B 2 C 2 sunt
omologice.
Observaie. Dac ABC este echilateral , I este centrul comun al triunghiurilor ABC i
A 1B1C 1 i se obine:
Teorema lui BARBILIAN. Dou triunghiuri echilaterale de acelai centru, ABC i
A2 B 2 C 2 , sunt n trei moduri omologice:
A B C
A B C
A B C

,
,
.
A
B
C
B
C
A
2 2 2
2 2 2
C 2 A2 B 2
Demonstraia decurge din teorema anterioar, deoarece dou triunghiuri echilaterale
sunt n trei moduri invers asemenea.

Observaie. Teorema se poate extinde i astfel:


Teorem. Fie ABC un triunghi cu I a centrul cercului exnscris corespunztor laturii
BC, iar A1 , B1 , C1 punctele de contact ale acestui cerc exnscris cu laturile BC, CA
respectiv AB. Fie A2 B 2 C 2 un triunghi invers asemenea cu triunghiul A 1B1C 1 avnd
centrul cercului circumscris n I a . Atunci triunghiurile ABC si A2 B 2 C 2 sunt omologice.
Consecina 1. Fie ABC un triunghi, iar A 1 , B 1 , C1 punctele de contact ale cercului
nscris C (I,r) cu laturile. Paralelele prin B 1 i C1 la BC retaie cercul C n B 2 , C 2 . S
se arate c dreptele AA 1 , BB 2 , CC 2 sunt concurente.
Demonstraie. Triunghiurile A 1B1C 1 i A2 B 2 C 2 au acelai centru al cercului
circumscris (punctul I) i sunt invers egale, deci invers asemenea.

Consecina 2. Fie ABC un triunghi, iar A 1 , B1 , C1 punctele de contact ale cercului


nscris C (I,r) cu laturile. Notm cu A2 , B 2 , C 2 simetricele punctelor A 1 , B1 , C1 fa de
un diametru oarecare al cercului C . Atunci dreptele AA

, BB 2 , CC 2 sunt concurente.

Demonstraie. Centrul cercului circumscris triunghiului

A2 B2 C 2

este I, iar

triunghiurile A2 B2 C 2 i A 1B1C 1 sunt invers egale, deci invers asemenea.


Bibliografie
1. D. Barbilian - I. Barbu - Pagini inedite, Editura Albatros, Bucureti, 1981.
2. C. Cocea - Proprieti remarcabile ale triunghiurilor invers asemenea, S nelegem
matematica, Bacu, 1992.
3. C. Cocea - Teoreme de triortologie i triparalelogie, 1992.
4. P. S. Modenov - Probleme de geometrie , Editura Nauka, Moscova, 1979.
5. N. Mihileanu - Utilizarea numerelor complexe n geometrie, Ed. Tehnic, Bucureti,
1968.

Asupra unor iruri de integrale


Iuliana GEORGESCU 1 i Paul GEORGESCU 2
Articolul de fa prezint un mod de calcul al limitelor unor iruri x n n t1 de termen
general x n

x dx , cu

f satisfcnd anumite ipoteze ce vor fi precizate ulterior. n

particular, se pot determina limitele irurilor a n n t1 , bn n t1 , c n n t1 , d n nt1 de termen


general a n

S /2

S /2

sin n xdx , bn

S /2

cos n xdx , a n

tg n xdx , a n

ln

xdx .

Insistm mai nti asupra unei soluii eronate date n [2] pentru faptul c lim a n
n of

Soluie. Aplicnd teorema de medie, c 0, S / 2 astfel nct

S 2 sin n c . Deoarece sin c 0,1 , avem lim sin n c


n of

0 i, deci,

S /2

S
lim

0.

sin n xdx

/2

n of 0

sin n xdx

Totui, c nu este constant, ci depinde de n , i nu putem deduce c lim sin n c


n of

0.
0.

n cele ce urmeaz vom indica un mod de calcul al unor limite de acest tip.

Teorema 1. Dac f : >a, b@ o >0,1@ este continu, iar U f


finit, atunci lim

n of a

x dx

^x >a, b @; f x 1` este

0.

Demonstraie. Presupunem c U f

a i , bi

Considerm I i

intervalului I i (daca x1

cu x i I i
a sau x k

* >ai , bi @

Fie D1

i 1

^x1 , x 2 , ! x k ` i fie H ! 0 arbitrar, dar fixat.


i l I i  H / k  1 , i 1, k i l I i = lungimea
b , atunci I 1 >a1 , b1 , respectiv I k a k , bk @ ).
k

i D 2

>a, b@ \ * a i , bi .

Cum D 2 este o reuniune finit de

i 1

intervale nchise i mrginite, i f este continu, exist M

sup f x , iar M  1 . De aici,


xD2

rezult c

x dx d M n S / 2

. Deoarece f x d 1 , x >a, b @ , se deduce c

D2

D1

1
2

x dx d b i

 ai S / 2 . n consecin ,

i 1

Profesor, Liceul cu Program Sportiv, Iai


Profesor, Liceul Teoretic Grigore Moisil, Iai

x dx

d k H / k  1  M

S / 2 .

Alegnd acum n 0 H astfel nct M n S / 2  H / k  1 , n t n 0 H , se obine c


b

x dx

 H , n t n 0 H , deci lim

n of a

x dx

0.

Observaia 1. Ipoteza U f este finit din enun poate fi nlocuit cu U f este


Jordan neglijabil, teorema rmnnd valabil cu aceeai demonstraie.
Dac f : >a, b@ o >0,1@ este continu i strict cresctoare (sau strict

Corolar 1.

descresctoare), iar f b 1 (respectiv f a 1 ), atunci lim

n of a

Demonstraie. Suntem n ipotezele teoremei , cu U f


Aplicaia 1. Are loc relaia

lim a n

lim bn

n of

n of

lim c n

n of

x dx

0.

{b } , respectiv U f

{a} .

0.

lim d n

nof

Soluie. Se aplic Corolarul 1, inndu-se seama de monotonia funciilor de sub


semnul de integral.
n cele ce vor urma vom nota cu D f mulimea punctelor de discontinuitate ale unei
funcii f . Observm c este valabil deasemenea urmtoarea mbuntire a Teoremei 1.

Teorema 2. Dac f : >a, b@ o > 1, 1@ , iar D f


atunci lim

n of a

x dx

i U f sunt

0.

Demonstraie. Deoarece f este integrabil Riemann, f , f

i f

integrabile Riemann. Fie acum H ! 0 arbitrar, dar fixat. Deoarece D f

* >ai , bi @ ,

D2

i 1

>a, b@ \ * a i , bi . Mai departe,


i 1

este continu pe D 2 ,

x dx

D1

sunt de asemenea

 ai  H / 2 .

i 1

i 1

este Jordan-negli-

a i , bi , D f * I k

jabil , k N * i I i 1di d k astfel nct I i


Notm D1

Jordan-neglijabile,

x dx

d H / 2 , i cum

se poate majora cu ajutorul Teoremei 1. Rezult de

D2
b

aici c lim

n of a

x dx

0 , deci lim

lim

n of a

n of a

Aplicaia 2. Fie b ! 0 , iar

Riemann, iar

x dx

x dx

0.

1
sin , x 0, b@
f x
.
x
0
,
x
0

0.

10

Atunci f este integrabil

Soluie.

Se observ c 1 d f d 1, x >0, b @ i D f

^0` .

Atunci f este

integrabil Riemann, conform criteriului Lebesgue de integrabilitate Riemann. Mai departe,


U f ^2 2k  1 S ; k N` este de msur Jordan nul i se poate aplica Teorema 2.
Observaia 2. Folosind noiuni de teoria msurii i integralei Lebesgue, mai precis
teorema lui Egorov, se poate demonstra urmtorul rezultat:
Teorema 3. Fie f : >a, b@ o I continu, g : >a, b@ o R integrabil Riemann i
g n nt1 un ir de funcii continue, g n : I o R , astfel nct g n D f x o g x x >a, b@ ,
i g n D f x d M , x >a, b @ . Atunci

Aplicaia 3. Pentru a

g n D f x dx o g x dx .

0, x >0, S 2
, g n x
0 , b S 2 , g x
1, x S 2

remarcnd c sunt ndeplinite condiiile Teoremei 3, iar

S 2

lim a n

n of

0 . Analog deducem c

lim bn

n of

lim c n

nof

lim d n

n of

g x dx

x n , f x sin x ,

0 , obinem c

0.

Aplicaia 4 (V. Drul i I. Paralescu, Problema 24154, G.M. 7-8/1999 ). Calculai:


S 2
S 2
sin 2 x
1
dx
b) lim
dx .
a) lim
2
2n
n of 0
n
o
f
0
1  sin x  sin x
1  sin x  sin 2 n x
Soluie. Aplicm Teorema 3. Lum
sin 2 x
, x >0, S 2
2x 1 x 2





g
x
,
f
x
sin
x
,
,
a) a 0 , b S 2 , g n x
1  sin 2 x
2
n
1 x  x
1 / 3,
2
x
S

b) a

0 , b S 2 , g n x

1
1  x 2  x 2n

1 1  x 2 , x >0, S 2
, f x sin x , g x
.
1 3,
,x S 2

Bibliografie
1. D. M. Btineu et al. Primitive i integrale , Ed. Brchi, Timioara, 1998.
2. V. Schneider Culegere de probleme de analiz matematic, Ed. Hyperion, Craiova, 1993.

11

O generalizare a lemei lui Riemann


Dan POPESCU 1 i Florin POPOVICI 2
n aceast not sunt generalizate urmtoarele dou rezultate:
Teorema 1 (Lema lui Riemann [3]). Dac f : >0, 2S @ o R este o funcie
2S

2S

integrabil Riemann, atunci irurile 0 f ( x) sin nx dx nN * i 0 f ( x) cos nx dx nN *


2S

sunt convergente i avem

lim 0 f ( x) sin nx dx 0

n of

2S

lim 0 f ( x) cos nx dx .

n of

Teorema 2 (Problema XII.19 [1]). Dac f : >0, T @ o R este o funcie continu i


g :>0, f o >0, f este o funcie continu i periodic de perioad T, atunci
1 T
T
T
lim f ( x) g (nx) dx
f ( x) dx) 0 g ( x) dx) .

T 0
n of 0
Amintim mai nti un rezultat la care vom face referire.

Teorema 3[2]. Dac

f : >a, b @ o R este o funcie integrabil Riemann i

M : >c, d @ o >a, b @ este o funcie bijectiv, derivabil i cu derivata integrabil Riemann,


atunci funcia f D M M c este integrabil Riemann i are loc formula schimbrii de
variabil

M 1(b )

a f ( x)dx M 1(a) f (M (t ))M c(t )dt .

D R*

astfel nct funcia de gradul nti M : >c, d @ o >a , b @ ,


M ( x) D x  E este o funcie bijectiv. Dac f : >a, b@ o R este o funcie integrabil
Riemann atunci funcia f D M este integrabil Riemann i are loc formula

Corolar.

Fie

1

b
M (b )
a f ( x)dx D M 1(a) f (M (t ))dt.

Rezultatul principal este dat de

Teorema 4 (Lema lui Riemann generalizat). Dac f : >0, T @ o R este o funcie

>

integrabil Riemann i g : 0, f o R este o funcie periodic de perioad T, astfel nct


restricia g [0,T ] este integrabil Riemann, atunci avem:
T
lim
nof 0

1
2

f ( x) g (nx) dx

Profesor, Colegiul Naional tefan cel Mare, Suceava,


Profesor, Liceul Teoretic N. Titulescu ,Braov

12

1 T
T
f ( x) dx) 0 g ( x) dx) .

T 0

(1)

Demonstraie. Conform Corolarului, funcia dat de x o g (nx ) , x[0,T ] este

este corect definit.


integrabil Riemann. Urmeaz c irul T f ( x) g (nx) dx
0
nN *
Pentru
orice
n N*
fie
'n 0 x n0  x n1  !  x nn T ,

unde

T
,  i ^0,1,! n`. Conform primei teoreme de medie, pentru orice i ^0,1,! n`
n
exist J ni inf f ( x ) x [ x n , i 1 , x ni ] , sup f ( x ) x [ x n , i 1 , x ni ] astfel nct
x ni

> ^

`@

x
xnni, i 1 f ( x)dx J ni x ni  x n, i 1

T
J ni
n

(2)

Evident, avem
n

i 1

i 1

i 1

x
x
x
T
0 f ( x) g (nx) dx xnni, i 1 f ( x) g (nx)dx xnni, i 1 ( f ( x) J ni ) g (nx)dx  J ni xnni, i 1 g (nx)dx. (3)

Deoarece funcia g este periodic de perioad T i restricia g


este integrabil
[0,T ]
Riemann, rezult c funcia g este mrginit, deci exist M 0, f astfel nct
g ( x ) d M ,  x [0, f ) . Urmeaz c
n

i 1

i 1

x
x
xnni, i 1 ( f ( x)  J ni ) g (nx) d xnni, i 1 f ( x)  J ni g (nx) dx d

n x
d M x ni
i 1 n, i 1

unde

f ( x)  J ni dx d M S'n ( f )  s'n ( f ) .

S ' n ( f ), s ' n ( f ) noteaz sumele Darboux superioar i inferioar relative la ' .


n

De aici rezult c

lim

x
xnni, i 1 ( f ( x)  J ni ) g (nx)dx 0.

(4)

n of i 1

innd cont de relaia (2) i de periodicitatea funciei g, se obin egalitile:


T
n
n
n
1 iT
x
i
J ni xnni,i 1 g (nx)dx J ni n T g (nx)dx J ni (i 1)T g (t )dt
n
i 1
i 1
i 1
i 1
n

T T
1 n
J ni 0 g (t )dt
Ti 1
n

(5)

n
1 T
1 T
T
0 g (t )dt 0 f ( x)dx .
0 g (t )dt J ni ( x n,i 1  x ni )

i 1
T
T

Din (3), (4) i (5) rezult c are loc (1).

Observaie. Teoremele 1 i 2 sunt particularizri ale Teoremei 4.


Bibliografie
1.D. Popescu - Problema XII.19, Recreaii matematice 1/2001, p. 77.
2.F. Popovici, M. Bencze - Asupra schimbrii de variabile in integrala Riemann,
G.M. metodic, 3/1996, 161-164.
3.Gh. Sirechi - Calcul diferenial i integral, vol I, Ed. tiinific i Enciclopedic,
Bucureti, 1985.

13

Cteva relaii metrice deduse vectorial


Marian TETIVA 1
Ne propunem s prezentm n cele ce urmeaz o modalitate de deducere a unor relaii
metrice n triunghi sau n tetraedru cu ajutorul calculului vectorial. Ideea (simpl, dar
eficient) am ntlnit-o n lucrarea [3] i este exprimat n
Propoziia 1. Fie K, M, L trei puncte necoliniare. Pentru un punct P oarecare,
urmtoarele afirmaii sunt echivalente:
a) Punctele L, M, P sunt coliniare;
b) Exist numerele reale l, m astfel nct l+m = 1 i KP
Demonstraie. P LM l R astfel nct MP

l KL  m KM

l ML l R astfel nct

KP  KM l ( KL  KM ) l R astfel nct KP l KL  (1  l ) KM , q.e.d .


Mai departe vom rezolva cteva probleme folosind rezultatul din Propoziia 1.
ncepem cu problema din [3] care ne-a condus la aceast not.

Problema 1. Fie ABC un triunghi, iar E, D puncte situate pe laturile AB, respectiv AC.
Considerm punctul M, intersecia dreptelor BD i CE; P i N sunt interseciile dintre AM
PN
PM
i BC, respectiv AM i DE. S se arate ca are loc egalitatea:
2
.
NA
MA
AE
AD
AM
AN
Soluie. S notm p=
.
Cum P BC , exist
,q
, m
, n
EB
DC
NP
MP

numerele x, y astfel nct

imediat

AE

x+y=1, i

p
AB ; la fel , AD
p 1

AP

x AB  y AC . Din

AE

p EB rezult

q
AC ,
q 1

m
n
AP i AN
AP . Egalitatea de
m 1
n 1
mai sus poate fi scris n urmtoarele forme
echivalente :
q 1
m 1
AM x AB 
y AD ,
m
q
AM

n 1
AN
n

m 1
AM
m

q 1
p 1
y AD,
x AE 
q
p

p 1
x AE  y AC . innd seama de unicitatea scrierii unui vector n bazele
p

( AE , AD), ( AE , AC ) respectiv ( AB, AD) i de Propoziia 1,


obinem egalitile:
m 1
m 1 p 1
q 1
n 1 p 1
q 1
x
x  y (2);
y (3). Adunnd (2)
x
y (1);
m
m
p
q
n
p
q

Profesor, Colegiul Naional Gh. Roca Codreanu, Brlad

14

cu (3) i innd seama de (1) i de faptul c x+y=1 , obinem c

2
m

1
, q.e.d.
n

Problema 2 (relaia lui Van Aubel). Cu notaiile din problema precedent, are loc
AE AD AM
.
egalitatea

EB DC MP
Soluie. Relaia (2) de mai sus se scrie: 1 

din (3) obinem my=q . Atunci

AE AD

EB DC

1
m

pq

1
1  x  1  x , adic mx=p . Analog,
p

AM
, q.e.d.
m( x  y ) m
MP

Problema 3 ([1]). Pstrm notaiile din Problema 1. n plus, considerm punctele T,


p q
TA
SA
S pe laturile AB, AC astfel nct
D,
E . S se arate c M TS 
1.
TB
SC
D E

D
E
AB, AS
AC. Din relaia
D 1
E 1
E 1
D 1
x
AT  y
AS . Conform Propoziiei 1
D
E

Soluie. Vom avea, ca mai sus, AT

AP

x AB  y AC rezult c

m 1
AM
m

i innd seama de unicitatea scrierii unui vector n baza AT , AS , avem:


E 1
p
q
p q
m 1 D 1
1 x y
1
M TS
x
y


1
 , q.e.d.
D
E
D E
m
m D E
m mD mE
Dou cazuri particulare mai des ntlnite ale Problemei 3 sunt acelea n care punctul M
este centrul de greutate G , respectiv centrul cercului nscris I pentru triunghiul ABC;
cititorul se poate convinge c este valabil urmtorul enun:
Problema 3. Fie ABC un triunghi i punctele T, S pe laturile AB, AC. Atunci:
TB SC
a) G TS

1;
TA SA
SC
TB
b) I TS b
a, unde a, b, c reprezint lungimile laturilor triunghiului.
c
SA
TA

Iat n continuare i variantele n spaiul cu trei dimensiuni ale chestiunilor discutate


anterior; rezultatul similar celui din Propoziia 1 este coninut n:
Propoziia 2. Fie K, L, M, N patru puncte necoplanare. Un punct P este situat n
acelai plan cu L, M, N dac i numai dac exist numerele reale l, m, n astfel nct

l+m+n=1 i KP

l KL  m KM  n KN . Altfel spus, dac x,y,z sunt coordonatele vectorului

KP n baza, KL, KM , KN , atunci P LMN dac i numai dac x+y+z=1.


Demonstraia este asemantoare. Avem P LMN  m, n R astfel nct
LP

m LM  n LN  m, n R astfel nct KP  KL

l , m, n R , l=1-m-n (deci cu l+m+n=1), astfel nct KP


Folosind Propoziia 2 putem rezolva:

15

m KM  KL  n KN  KL
l KL  m KM  n KN , q.e.d.

Problema 4 ([3]). Fie ABCD un tetraedru i F, G, H puncte pe AB, AC, AD respectiv;


fie M punctul comun planelor (FCD), (GBD), (HBC), iar P i N punctele de intersecie ale
PN PM
dreptei AM cu planele (BCD) respectiv (FGH). Atunci are loc:
.
3
NA MA
AG
AM
AF
AH
Soluie. Ca n Problema 1, s notm p
, q
, r
, m
,
GC
FB
HD
MP
AN
. innd cont de unicitatea scrierii unui vector ntr-o baz i de Propoziia 2,
n
NP
obinem mai nti existena numerelor reale x, y, z astfel nct x+y+z=1 i
AP

x AB  y AC  z AC , iar apoi echivalenele:


p 1
q 1
1
r 1
z
x
y 
p
q
r
n

n 1
n
m 1
m
m 1
m

x y 

p 1
1
x yz
p
m
r 1
1
z
r
m

x m 1
;
p
m

z
3
. De aici
r
m

x

x y z
  ;
p q r
q 1
1
yz
q
m

x y z
 
p q r

y
;
q

1
, adic relaia din enun.
n

p q  r
p  q  r m , deci am rezolvat i
m
AF AG AH
AM
Problema 5. Cu aceleai notaii din Problema 4 avem
.


FB GC HD
MP
n ncheiere propunem cititorului s rezolve problemele :
Problema 6. Pstrm notaiile din Problemele 4 i 5; n plus fie punctele S, T, U pe
AS
AT
AU
D,
E,
J . S se arate c:
AB, AC, respectiv AD astfel nct
SB
TC
UD
p q r
1.
M STU  

De asemenea vom avea 1 x  y  z

Caz particular. Centrul de greutate al tetraedrului se afl n planul (STU) dac i


SB TC UD


1 ([4]).
numai dac
AS AT AU
Problema 7. Fie ABCD un patrulater convex i punctele M ,N, P, Q pe laturile sale
BN
AQ
AM
DP
AB, BC, CD, DA respectiv, astfel nct
b . Fie S intersecia
a,
NC QD
MB
PC
MS
QS
dreptelor MP i NQ. S se calculeze rapoartele
i
(O frumoas soluie sintetic a
SN
SP
acestei probleme poate fi gsit n [2]).
Bibliografie
1. M. Andronache Problema 4 (p. 158) , G.M. 4/2000.
2. D. Brnzei, R. Brnzei Metodica predrii matematicii, Editura Paralela 45, 2000.
3. V.N. Dubrovski Soluia problemei M1062, Kvant 1/1988.
4. Gh. Szllsy Problema C: 2275, G.M. 4/2000.

16

Inegaliti geometrice. Aplicaii


Dan-tefan MARINESCU 1 i Ioan ERDEAN 2
Scopul propus este de a demonstra, prin mijloace mai puin folosite, dou
inegaliti geometrice care au un numr mare de aplicaii.
Propoziia 1. Fie nN, nt2 i M, A1, A2, , An puncte din spaiu date. Dac a1,
a2, , anR, atunci
n
n

a i a i MA i2 t a i a j Ai A 2j .
(1)

i 1 i 1
1d i  j d n
n

Egalitatea se obine dac i numai dac

ai MAi

0.

i 1

n
n

a i MA i a i MA i t 0 , de unde

i 1
i 1

Demonstraie. Avem:
n

ai2 MA i2  2 ai a j MAi MA j t 0 sau ai2 MA i2  2 ai a j MA i MAj cos(A i MA j ) t 0.


1di jdn

i 1

1di jdn

i 1

Cum din teorema cosinusului n MAiAj , eventual degenerat, avem

2MA i MA j cos( A i MA j )

MA i2  MA j2  A i A 2j , urmeaz c

ai2 MAi2  ai a j MAi2  MA2j t ai a j Ai A2j


1d i  j d n

i 1

i, deci, are loc inegalitatea (1).

1d i  j d n

Observaie. Exist i alte demonstraii ale inegalitii (1) pentru cazul n care
punctele sunt coplanare i a1, a2, , an>0.
Urmtorul caz particular este util n aplicaii :
Corolar. Fie nt3, A1 A2 An un poligon i M un punct dat. Atunci () a1, a2, ,
anR are loc inegalitatea (1).
Aplicaia 1. Dac ABC este un triunghi oarecare i D,E,JR, atunci
(D+E+J) 2R 2 t a2E J+b2J D+c2D E.
L. Panaitopol
Soluie. n (1) lum a1=D, a2=E, a3=J i M punctul O (centru cercului
circumscris). Ca un caz particular al acestei aplicaii se obine
Aplicaia 2. Dac
a2EJ+b2JD+c2DE0.

este un triunghi i D, E, JR, atunci

ABC
*

Aplicaia 3. Fie x, y, z R . Atunci


xy

x  z y  z
2

yz

y  x z  x
2

zx

z  y x  y
2

1 1
1
1


 .
16 xy yz zx

D-t. Marinescu, I. erdean (etapa local 1998)


1
2

Profesor, Liceul teoretic Iancu de Hunedoara, Hunedoara


Profesor, Colegiul Naional Aurel Vlaicu, Ortie

17

Soluie. n Aplicaia 1 se va lua D=x, E=y, J=z i a=y+z, b=z+x, c=x+y. Din
(x  y )( y  z )(z  x )
abc
, obinem R
etc.
4S
4 xyz (x  y  z )

Aplicaia 4 . Pentru orice priamid triunghiular MABC avem :


G.M. 2/1995, p.88

aMA 2  bMB 2  cMC 2 t abc (a=BC, b=AC, c=AB).


Soluie. Se obine din (1) pentru n=3 i a1=a, a2=b, a3=c.

Aplicaia 5. Fie ABC un triunghi i M un punct al planului su. Atunci


MA  MB 4  MC 4 t a 2b 2c 2 /(a 2  b 2  c 2 ) .
I. Tomescu, G.M. 6/1972
Soluie. Conform inegalitii Cauchy Buniakovski Schwarz, avem:
4

MA

 MB 4  MC 4 a 2  b 2  c 2 t aMA2  bMB 2  cMC 2 . .


Combinnd aceasta cu inegalitatea din Aplicaia 4, deducem inegalitatea cerut.

Aplicaia 6 . Fie ABC un triunghi, P un punct n planul su, iar PA=x, PB=y,
PC=z. S se arate c ayz+bxz+cxytabc.
C.
Cocea
Soluie. Lum n Corolar n=3, M=P, a1 a / x , etc. Obinem:
a b c
a 2bc ab 2c abc 2 abc
  ax  by  cz t
ax  by  cz


yz
xz
xy
xyz
x y z
ayz  bxz  cxy
abc

t
ayz  bxz  cxy t abc.
xyz
xyz

Aplicaia 7. Dac A1 A2 An este un poligon i M un punct oarecare, atunci


n

MA
i 1

2
i

A j2 .

G.M. 5/1995, p.193

1d i  j d n

Soluie. Lum ai

1, i

1, n n Corolar.

Aplicaia 8. Dac A1 A2, An este un poligon nscris n cercul C(O,R), atunci


n2R2 t

Ai A2j .

M. Chiri, M. Dinc Numere complexe

1d i  j d n

Soluie. n Corolar se consider ai

1, i

1, n i punctul M n O.

Cel de-al doilea rezultat general este urmtorul:


Propoziia 2. Pentru orice nN, nt3 i orice x1, x2, , xnR avem:
n
n
n
2S
S
2 x k2  2 x k  2 x k 1  x k 1 cos
 n t n cos2 (x n 1 x 1 ) .
n
n
k 1
k 1
k 1
Egalitatea are loc dac i numai dac x1= x2= = xn=1/2.
Demonstraie. Pentru n=3 inegalitatea revine la
3

2 x k2  3 x k 
k 1

k 1

x k x k 1  4 t 0

(cu x4= x1). Deoarece

k 1

18

xk2 t xk xk 1 ,

(2)

avem:

A t x k2  2 x k x k 1  3 x k 

9
4

x k 2  3 x k  9

xk  t 0 ,
2

4
deci (2) are loc pentru n=3.
Pentru n t 4 inegalitatea (2) revine la
S
2S
2S

B 2x k2  2x k 1  cos  2x k x k 1 cos  n sin2 t 0.


n
n
n

2
S
Cum  2 xk xk 1 t  xk2  xk2 1 , k 1, n , prin multiplicare cu cos
(ce este t0 pentru
n
2S
2S
.
nt4) i apoi sumare dup k, obinem:  2 xk xk 1 cos
t  2 xk2 cos
n
n
2S
2S

2S
Ca urmare, avem: B t 2 xk2 1  cos
 n sin
 2 xk 1  cos
n
n
n

4 xk2 sin 2

S
n

 4 xk sin 2

S
n

 n sin 2

S
n

2 xk  1 2 sin 2 n t 0

q.e.d

Aplicaia 9. Se consdider triunghiul echilateral ABC de latur 1 i punctele


3
A1(BC), B1(CA), C1(AB). S se arate c A1B12  B1C 12  C 1A12 t .
4
Concurs de matematic 1988 etapa final
Soluie. Notnd x1=BA1, x2=CB1, x3=AC1 i , conform teoremei cosinusului ,
inegalitatea revine la
3
2 x12  x22  x32  2 x1  x2  x3  x1 (1  x2 )  x2 (1  x3 )  x3 (1  x1 )  3 t , adic (2)
4
pentru n=3.

Aplicaia 10. Pe laturile unui ptrat ABCD de latur 1 se consider punctele


M(AB), N(BC), P(CD), Q(DA) astfel nct MN 2  NP 2  PQ 2  QM 2 2.
S se arate c MNPQ este ptrat.
C. Nsturic, Cardinal 3/1991, p.55
Soluie. Notm AM=x1 , BN=x2, CP=x3, DQ= x4. Egalitatea se scrie:

2 x12  x22  x32  x42  2 x1  x2  x3  x4  4 2 , adic n (2) pentru n=4 are loc egalitate,
deci x1=x2=x3=x4=1/2 i M, N, P, Q, vor fi mijloacele laturilor ptratului ABCD. n
consecin MNPQ este ptrat.

Aplicaia 11. Fie A1 A2, An , nt4, un poligon regulat de latur 1 i P1( A1 A2),
2S
P2( A2 A3), , Pn( An A1). S se arate c P1P22  P2 P32  ...  Pn P12 t n cos
.
n
R.M.T 1,2 / 1989 , Problema 6539
Soluie. Notnd cu P1A2 =x1 ,P2A3 =x2 ,, PnA1 =xn , teorema cosinusului aplicat
triunghiurilor P1A2P2 , P2A3P3 , ,PnA1P1 ne conduce la
(n  2)S
(n  2)S
,", Pn P12 xn2  (1  xn )2  2xn (1  xn ) cos
P1P22 x12  (1  x1 )2  2x1(1  x1 ) cos
n
n
n
n
n
2S
2
2
2
2
Prin sumare obinem: P1P2  P2 P3  ...  Pn P1 2 xk  2 xk  2 xk (1  xk ) cos  n
n
k 1
k 1
k 1

i, n conformitate cu (2), deducem inegalitatea cerut.


19

Asupra unei clase de iruri recurente


Dan POPESCU 1
Scopul acestui articol este prezentarea unei metode de abordare a unei clase de iruri
recurente de primul ordin. Mai precis, dat irul ( x n ) nt1 prin
x1 D R

i xn 1

f ( xn ),  n N *,

unde f : D o D , se pune problema existenei limitei

(1)

lim nx n n R .

nof

lim x n R  {0} , irul (nx n ) n t1 are limita evident; rmne de

n ipoteza c

nof

investigat cazul n care lim x n


nof

0 . Rezultatul principal este cuprins n urmtoarea

Teorem. Fie D R * avnd originea ca punct de acumulare, funcia f : D o D i


irul ( xn ) n t1 definit prin (1) astfel nct sunt ndeplinite condiiile: 10 lim x n
nof

x  f ( x)
x o 0 xf ( x )

2 0 lim

0 i

l R . Au loc implicaiile:

(a) dac l R  {0} , atunci

1
;
l

lim nx n

n of

(b) dac l=0 i exist o vecintate V1 a originii nct

x  f ( x)
! 0,  x V1 D ,
xf ( x)

f ;

atunci lim nxn


nof

(c) dac l=0 i exist o vecintate V2 a originii nct

x  f ( x)
 0,  x V2 D ,
xf ( x )

f .

atunci lim nxn


nof

1
Demonstraie. Se aplic criteriul Stolz-Cesro irului
nx n
1/ x 1/ x n
1/ x n

. n ipotezele impuse avem: lim n1


nof (n 1) 1
n n t1
Urmeaz c lim

n o f nx n

scris sub forma


n t1

x n  f (x n )
x  f (x )
lim
l R. .
nof xf (x )
nof x nf (x n )
lim

l i afirmaiile (a), (b), (c) decurg imediat.

Prezentm mai nti cteva aplicaii directe ale acestei teoreme.


1 (G.M. 11,12/1986, C: 649, M. Bencze). Fie irul ( xn ) n t1 definit prin x1 ! 0 i
xn 1 ln(1  xn ),  n N * . S se demonstreze c lim nx n 2 .
nof

(G.M. 3/1987, 21056, F. Dumitrel). Dac irul ( x n ) n t1 se definete prin

x1 ! 0 i xn 1
1

xn 2  x n ,  n N * , s se calculeze lim nx n .
nof

Profesor, Colegiul Naional tefan cel Mare, Suceava

20

x n 1

3 (G.M. 10/1987, 21253, M. Lascu). Fie irul ( xn ) n t1 definit prin x1 ! 0 i


ln(1  arctg ( xn )),  n N * . S se arate c irul nx n n t1 este convergent la 2.
4 (R.M.T. 2/1987, 6256, V. Bivolaru). Se consider irul ( xn ) n t1 nct x1 ! 0 i

xn2
, n N * . S se arate c irul este convergent i s i se calculeze
2xn  ln(1  xn )

xn1

limita.
5 (G.M. 4/1995, 23241, V. Nicula). Fie p N * , x 1 (0, 1 / p )
xn 1 xn (1  xn )(1  2 xn )...(1  px n ),  n N * . S se calculeze limita lim nx n .

n of

6 (G.M. 1/1997, 23668, A. Vernescu). Dac irul ( xn ) n t1 este definit prin x 1 ! 0 i


1 1
 ,  n N * , s se studieze natura irului
4 2
convergen, s se calculeze lim nx n .

xn 

xn 1

nxn n t1 i,

n caz de

nof

Condiia 10 din teorem se verific uor pentru fiecare dintre irurile precedente. n
privina condiiei 2 0 , pentru funcia aferent f : (0, f) o (0, f) indicm:
x  ln(1  x )
x o0 x ln(1  x )

1. lim

x  x 2 x
x o0 x 2 2 x

1
,
2

2. lim

x

x  ln(1  arctg x )
x o0 x ln(1  arctg x )

3. lim

4. lim

1
,
2

ln 2 ,

x2
2 x  ln(1  x )

1
,
2

x3
2 x  ln(1  x )

x o0

1 1

4 2
1 1
x 
4 2

x x

6. lim

x  x (1 x )(1 2x )...(1 px) p( p 1)


x o0 x 2(1 x )(1 2x )...(1 px)
2

x o0

5. lim

1.

Observaie. n privina problemei 5, menionm c pentru p= 1 se obine o problem


din revista Matematika v kole, 5/1984.
Prezentm acum cteva probleme care sunt sub incidena Teoremei, fapt care nu
este ns evident.
7 (G.M. 5,6/1988, 21458, M. Banyai). Fie k>0, x1>k i x n 1 x n2 /( x n  k ),
n
 n N * . S se calculeze lim
.
n of x n
Soluie. Se constat c ( xn ) n t1 este cresctor i lim xn
n of

irului ( y n ) n t1 definit prin yn

1
,  n N * . Avem y n 1
xn

21

f . Aplicm Teorema

y n (1  ky n ) ,  n N * , i

x (1  kx ) are lim

0. Funcia f : (0,1) o (0,1) dat de f ( x )

lim y n

n of

n
n o f xn

. Ca urmare, lim

lim ny n

x o0

x  f ( x)
xf ( x )

1
.
k

8 (G.M. 1/1989, 21668, M. Lascu). Fie irul ( x n ) n t1 definit de x1


xn 

x n 1

1
,  n N * . S se calculeze
xn
n of

(G.M.

2
n

6,7/1990,

2 / x n2 ,  n N * , i deducem c

xn

M.

2n

nof

Dac

Mihail).

P.

lim

ny n

1.

a, b, x1 ! 0

/(a  bx n ),
x o0

Soluie.
y

22115,

1 i

(enun parial).

2n

nof

 n N * , s se calculeze lim

y n 1

xn

4 y n /( y n2  4 y n  4). Utiliznd Teorema, obinem: lim

9
xn

lim

nof

f . Notm y n

Soluie. Se constat c lim xn


y n 1

k.

2
n

Se

introduce
2

1
n xn

irul

/(a  b y n ) , n t 1, f (x )

( y n ) n t1
2

prin

x n2 , n t 1 .

yn

Urmeaz

x /(a  b x ) , x ! 0 , etc. Discuie dup b i x1 .

10 (G.M. 11/1993, C: 1463, L. Panaitopol). Se consider irul (x n ) n t1 definit prin


x1 ! 1 i xn 1

xn  xn  1,  n N * .S se calculeze lim

Soluie. Cu

xn

nof n 2

yn

1 / x n , n t 1 , obinem y n 1

y n / 1  y n  y n2 , n t 1 , i

xn
nof n2

1
.
4
11 (R.M.T. 2/1997, X127, V.Bivolaru). Fie irul ( xn ) n t1 definit prin x1 ! 0 ,
x1  x2  ...  xn
x n 1 x n a  x n , n N * , unde a>1. Dac y n
, n N * , s
ln n
calculeze lim y n .

gsim, n cele din urm, lim

se

nof

Soluie. Cu criteriul Stolz-Cesro, avem:


xn
1
lim nx n
lim y n lim
lim nx n
ln n  ln(n  1) nof
n of
nof ln n  ln( n  1) nof
ultima egalitate stabilindu-se cu ajutorul Teoremei (ca n Problema 3).

1
ln a

12 (G.M. 1/1998, C:2005, M. Bencze). Fie irul ( x n ) n t1 astfel ca x1 ! 0 i


(1  x n ) x n 1 1  x n2 , n N * . S se demonstreze c ( x n ) n t1 este convergent i s se
calculeze lim n(1  x n ).
nof

Soluie. Considerm y n

1  x n , n t 1 . Avem y n 1

i f (x ) (x  x 2) /(2  x ), x  0 , etc. Limita este egal cu zero.


22

( y n  y n2 ) /(2  y n ), n t 1 ,

Un criteriu de concuren a dreptelor


Temistocle BRSAN 1
n revista Recreaii tiinifice, IV (1886), pag. 48, este enunat urmtoarea
Problem. n orice triunghi, dreptele ce unesc picioarele nlimilor corespunztoare
la dou laturi, picioarele bisectorelor alturate cu nlimile i picioarele
perpendicularelor duse din centrul cercului nscris pe cele dou laturi trec prin acelai
punct.
n acelai volum, la pag. 118, este prezentat o soluie sintetic a problemei, dat de
N.Gr. Blnescu, elev la coala de Poduri i osele din Paris.
ncepem prezenta not cu un criteriu de concuren a trei drepte determinate de
punctele lor de intersecie cu dou dintre laturile unui triunghi. Cu ajutorul acestuia vom da
apoi o soluie tehnic problemei de mai sus.
Fie un triunghi ABC i dreptele d1, d2 i d3 ce intersecteaz n M, P i respectiv R
dreapta AB i n N, Q i respectiv S dreapta AC. Considerm c poziiile acestor puncte sunt
determinate de rapoartele urmtoare:
MB
NC
PB
QC
RB
SC
m,
n;
p,
q;
r,
s.
(1)
MA
NA
PA
QA
RA
SA
Menionm c aceste puncte au poziii oarecare pe dreptele AB i AC, dar c, n cele ce
urmeaz, vom exclude tacit anumite cazuri triviale, cum ar fi: dou dintre drepte sunt
paralele sau coincid, cele trei drepte sunt concurente ntr-un punct situat pe AB sau AC, una
dintre drepte trece prin vrful A etc.
Propoziia 1. Dreptele d1, d2 i d3 determinate de numerele m,n,p,q,r,s date de (1) sunt
concurente dac i numai dac aceste numere ndeplinesc condiia
1 1 1
ps  rq  rn  ms  mq  pn 0 m p r 0 .
(2)
n q s
Demonstraie. Din (1) obinem uor relaiile:
m
1
1
AB ; NA
AC , NB
MA
AB , MB
m 1
n 1
m 1
Fie {X}= d1 d2 i ^X c` d 1 d 3 . Utiliznd
teorema lui Menelaus relativ la 'AMN i
transversalele PQ i RS i innd seama de (3),
avem: d1, d2, d3 concurente X i X c coincid

XM

X 'M

XN

X 'N

PB  MB
PA

PM QA

PA QN

QA

RB  MB

QA  NA

RA

RM
RA

SA
SN

SA
SA  NA

Prof. dr., Catedra de matematic, Univ. Tehnic Gh. Asachi, Iai

23

n
AC
n 1

i analoagele. (3)

p
m
1

p 1 m 1
q 1

1
1
1

p 1
q 1 n 1

1
r
m

s 1
r 1 m 1
ps  rq  rn  ms  mq  pn
1
1
1

r 1
s 1 n 1

Soluia Problemei. Fie d1, d2, d3 dreptele ce unesc picioarele nlimilor, picioarele
bisectoarelor i, respectiv, punctele de contact ale cercului nscris (puncte pe dreptele AB i
p b
pc
a cos B
a cos C
a
a
,n 
; p  ,q  ; r 
AC). Atunci m 
,s 
i
b cos A
c cos A
b
c
pa
pa
condiia (2) se verific prin calcul direct.
Observaie. Fie A0 punctul de concuren a dreptelor din problem i B0, C0 punctele
analoage acestuia. Vom arta ntr-o not urmtoare c AA0, BB0, CC0 sunt concurente.
n particular, dac M coincide cu B i S coincide cu C (i.e. m=s=0), vom obine:
Propoziia 2. Fie ABC un triunghi oarecare. Dreapta PQ trece prin punctul de
intersecie a cevienelor BN i CR dac i numai dac numerele p,q,n,r definite ca n (1)
1 1 1
p q
(4)
satisfac condia

1 0 p r 0.
r n
n q 0
Observaie. Acest ultim rezultat este cunoscut ([3, Teorema 3], [2, Teorema 1 o
form diferit a condiiei] etc.). n locurile citate (ct i n alte locuri!) sunt indicate
condiiile n care dreapta PQ trece prin diferite puncte remarcabile ale triunghiului:
a
a
a cos B
a cos C
sin 2 A
sin 2 A
G ( r n 1), I ( r  , n  ), H ( 
,
), O ( 
,
),
sin 2C
b
c
b cos A c cos A
sin 2B
a2

a2

p a p a
p b
p c
,
), N (
,
) , unde K, , N sunt punctele
p a
p a
p b
p c
b
c
lui Lemoine, Gergonne i respectiv Nagel. Se obin rezultatele:
QC
PB QC
PB
1. G PQ

1 ,
2. H PQ tgB
 tgC
tgA ,
PA QA
PA
QA
K( 

,

3. I PQ b

), * ( 

PB
PA

5. K PQ b 2

c

PB
PA

QC
QA

 c2

 a,
QC
QA

4. O PQ sin 2B
a 2 ,

6. * PQ

PB
PA

 sin 2C

QC

 sin 2 A

QA

1 PB
1 QC

p  b PA p  c QA

1
p a

QC
 ( p  c)
( p  a ) etc.
PA
QA
Foarte adesea aceste condiii sunt date pentru cazul restrictiv cnd P(AB) i Q(AC), fapt
care justific renunarea la lucrul cu segmente orientate.

7. N PQ ( p  b)

PB

Bibliografie
1. Colecia revistei Recreaii tiinifice (1883-1886).
2. C. Chiser Condiii necesare i suficiente ca o dreapt s treac prin puncte importante
dintr-un triunghi, G.M. 9/2000.
3. N. Oprea Un punct i o dreapt remarcabil din planul unui triunghi, G.M. 11/1996.
24

0.

Cteva consecine ale unei relaii a lui Gergonne


Ioan SCLEANU 1
n triunghiul ABC considerm cevienele A A c , B B c , CC c concurente n M.
AM
BM
CM
Notm x
, y
, z
; constatm c dac M este centrul de greutate
MC c
MA c
MB c
al triunghiului, atunci x, y, z sunt numere naturale. Ne propunem s determinm toate
punctele M Int ABC cu proprietatea urmtoare:
rapoartele

x, y , z

corespunztoare lui

sunt numere naturale.

(*)
n particular, vom aeza ntr-un cadru firesc i vom extinde rezultatul obinut n [1].
Pornim de la urmtoarea
Teorem(Gergonne).
concurente n M. Atunci

triunghiul

MA c MB c MC c


A A c B B c CC c

ABC,

cevienele

AA ' , BB ' , CC ' sunt

1 (vezi [2]).

1
1
1


1 ; n mod
x 1 y 1 z 1
1
. Vom presupune
necesar, unul dintre termenii sumei din stnga trebuie s fie cel puin
3
1
1
c
t , de unde x d 2 . Cum x N*, rezult c x 1 sau x 2.
x 1 3
1
1
1
Dac x 1 , atunci
, i.e. ( y  1)(z  1) 4 i deoarece

y 1 z 1 2

Cu notaiile iniiale, putem rescrie concluzia

y, z N*, rezult c ( y , z ) {(2,5), (3,3), (5,2)} .


1
1
2
Dac x 2 , atunci
. Unul dintre termenii din stnga trebuie s

y 1 z 1 3
1
1
1
; s presupunem c
fie cel puin
t , adic y d 2 . Dac y 1 , obinem z 5 ,
3
y 1 3
iar dac y 2 , rezult z 2 .

n concluzie, ( x , y , z ) poate lua valorile (2,2,2), (1,3,3), (1,2,5), precum i


toate permutrile posibile ale acestor triplete. Avem astfel 3 tipuri de puncte M cu
proprietatea (*), pe care le vom nota corespunztor M[2,2,2], M[1,3,3], M[1,2,5]. Ne
propunem n continuare s caracterizm geometric fiecare dintre aceste tipuri de puncte.
Propoziia 1. Un punct M este de tipul M[2,2,2] dac i numai dac este centrul
de greutate al triunghiului ABC.

Profesor, Liceul Teoretic tefan cel Mare, Hrlu

25

Demonstraie. Suficiena este evident. Pentru a demonstra necesitatea, aplicm


teorema lui Menelaus n triunghiul AB cM cu transversala BAcC ; obinem
B 'C 1
BM B cC A A c
BM
2
AA '
i atunci
, adic B c
1 . Din ipotez,
3,

BB ' 3
CA
2
A'M
B B c CA A cM
este mijlocul lui [AC]. Rezult c M se afl situat pe mediana [ BB ' ] astfel nct
MB '
BB '

1
, deci M este centrul de greutate al triunghiului ABC.
2

Propoziia 2. Un punct M este de tipul M[1,3,3] dac i numai dac este


mijlocul medianei [ AA ' ] (analog pentru M[3,1,3] i M[3,3,1]).
C 'M
B 'M 1
, deci. B ' C ' // BC
Demonstraie. Dac M[1,3,3], atunci
MC
MB
3
conform reciprocei teoremei lui Thales aplicat n triunghiul MBC. Rezult de aici c
AC ' AB '
BA '
i folosind teorema lui Ceva obinem c
1 , deci [ AAc ] este median,
A 'C
C ' B B 'C
iar M este mijlocul su.
Reciproc, dac M este mijlocul medianei >AAc@ , evident c x 1 . Din teorema lui
AC ' AB '
, deci B ' C ' // BC , de unde 'C ' MB ' ~ 'CMB i
Ceva obinem c
C ' B B 'C
'AB 'C ' ~ 'ABC . Obinem de aici c

teorema lui Menelaus n triunghiul

1
y

1 B 'C ' AB '


. Pe de alt parte, aplicnd
z
BC
AC
AAcC cu transversala BMB ' , rezult c

CB '
BA ' CB ' MA
BA ' 1 MA

1 i cum
,
1 , avem c
BC B ' A MA '
BC 2 MA '
B 'A
concluzie, y z 3 , deci M este de tipul M[1,3,3].

2 , i.e.

AB '
AC

1
. n
3

Propoziia 3. Un punct M este de tipul M[1,2,5] dac i numai dac aparine


liniei mijlocii [C 1 B 1 ] i o mparte n dou segmente avnd raportul 2 (au loc i afirmaiile
analoage pentru M[1,5,2] etc.).
Demonstraie.
A
Fie M[1,2,5]; cum x 1 , rezult c M aparine
liniei mijlocii [C 1 B 1 ] . Ducem MT AC ,
Bc
T BC; atunci MB 1CT este paralelogram
i deci >MB 1 @ { >TC @ . Aplicnd teorema lui

C1

B1

BT BM
2,
Thales n triunghiul BCB ' obinem
TC MB'

deci CT

1
BC
3

2
B 1C 1 , adic MB
3

2
B 1C 1 .
3

26

Ac

Reciproc, fie M >C1 B1 @ astfel nct

MB 1
C 1M

2 . Cum M aparine liniei mijlocii,

2
2 1
1
C 1B 1
BC
BC , deci
3
3 2
3
BT
BM
BT
2 . Din teorema lui Thales aplicat n 'BCB ' rezult c
2 , adic
TC
MB ' TC
y 2. Folosind acum relaia lui Gergonne, se deduce c z 5 .

evident c x 1 . Cu notaiile precedente, CT

MB 1

Aplicaia 1. Centrul cercului circumscris triunghiului ABC are proprietatea (*)


dac i numai dac triunghiul este echilateral.
Demonstraie. Presupunem c O are proprietatea (*). Dac, prin absurd,
O B 1C 1 , atunci C1O A AB i B 1O A AC , deci A, B, C ar fi coliniare. Rezult c
O B 1C 1 i de aici urmeaz c O este de tipul O [2,2,2], adic O { G, deci triunghiul
ABC este echilateral. Reciproca este imediat.
Aplicaia 2. Centrul cercului nscris are proprietatea (*) dac i numai dac
triunghiul ABC este echilateral.
Demonstraie. Presupunem c I are proprietatea (*). Dac, prin, absurd,
I B 1C 1 , atunci < C1 IB { < IBC i deci < C1 IB { < C1 BI , adic triunghiul C1 BI este
isoscel cu [ BC 1 ] { [ C 1I ]. Analog, [ CB 1 ] { [ B 1 I ], de unde BC 2 B1C1 2C1 I 
2 B1 I 2 BC1  2CB1 AB  AC , imposibil. Rezult c I este punct de tipul I[2,2,2],
adic I { G, deci triunghiul ABC este echilateral. Reciproca este evident.
Observaia 1. Acest rezultat este demonstrat pe o alt cale n [1].
Observaia 2. Exist triunghiuri neechilaterale al cror ortocentru are proprietatea
(*). Caracterizarea acestora este dat mai jos (Aplicaia 4) i o propunem spre rezolvare,
mpreun cu dou probleme ajuttoare:
Aplicaia 3. Fie triunghiul ABC, M si N mijloacele laturilor [AB], respectiv [AC]
iar H intersecia inlimilor AA ' i BB ' . Au loc echivalenele:

1)

BC
3

AB

AC

5
AB

2 2
AC

H [ MN ] i HN=2HM

HB
HA
1 i
HA '
HB '

2;

HA
BC
HA
1 i
3.
H este mijlocul lui [MN]
'
HA
2
'
HB
3
3
Aplicaia 4. Ortocentrul triunghiului ABC are proprietatea (*) dac i numai dac

2)

laturile triunghiului sunt direct proporionale fie cu 1,1,1, fie cu 3, 2 2 , 5 , fie cu 2,


3, 3.

Bibliografie
1. Iulica Georgescu Asupra unei clase de triunghiuri, G.M. 2-3/1982.
2. Liviu Nicolescu, Vladimir Boskoff Probleme practice de geometrie, Ed. Tehnic,
Bucureti, 1990.

27

Unele iruri monotone cu limita e sau e-1


Gheorghe COSTOVICI 1
n Propoziiile care urmeaz, vom demonstra prin metode elementare monotonia
unor iruri cu limita e sau e-1.
Propoziia 1. irul e n (x ) (1 
Demonstraie. Din a n 1  b n 1
a ! b t 0 , se obine c
a

1

1
nx

i b

1

1 n
) este cresctor, x t 0 fixat.
nx
(a  b )(a n  a n 1b  ....  ab n 1  b n ) , n ipoteza

a n [a  (a  b )(n  1)]  b n 1 . Lund n aceast inegalitate

1
, gsim c
n 1 x

e n (x )[1 

x
]  e n1 ( x) ,
(n  x)(n  x  1)

care atrage e n (x )  e n 1 (x ) , n t 1 , x t 0 fixat.


1 n 1
)
este descresctor, x (1,0] fixat.
nx
1 t 1
Demonstraie. Considerm funcia f : [1, f) o R , f (t ) (1 
) , cu
t x
1
t 1
x (1,0] arbitrar fixat. Avem f c(t ) f (t )[ln(1 
)
] , t t1.
(t  x  1)(t  x )
t x

Propoziia 2. irul a n (x ) (1 

Folosind ln 1  y  y,


t 1
(t  x  1)(t  x)

y ! 0,

rezult ln(1 

x
d 0 , adic
(t  x  1)(t  x)

1
t 1
1
)


t  x (t  x  1)(t  x) t  x

f ' t  0, t t 1

deci

este

descresctoare. Atunci f ( n ) ! f (n  1), i.e. an (x ) ! an 1 (x ) , n t 1 , ceea ce ncheie


demonstraia.

1 n
) este descresctor, x t 1 fixat.
n x
1 n
1
1
Demonstraie. Observm c b n ( x ) (1 
,
)
1
nx
e n (x  1)
n
(1 
)
n  x 1
n t 1 . irul e n ( x  1) este cresctor pentru x  1 t 0 x t 1 i are termeni pozitivi,
deci bn (x) este descresctor pentru orice x t 1 fixat.

Propoziia 3. irul b n ( x )

(1 

Propoziia 4. irul c n (x ) (1 

1 n 1
)
este cresctor, x (0,1] .
nx

Demonstraie. Observm c

Conf. dr., Catedra de matematic, Univ. Tehnic Gh. Asachi, Iai

28

1
1
, n t 1 .
1
a
(
n x  1)
(1 
) n 1
n  x 1
Conform cu Propoziia 2, irul an (x  1) este descresctor pentru x  1 (1,0]
x (0,1] i are termenii pozitivi, deci c n (x ) este cresctor pentru x (0,1] .
c n ( x)

(1 

1 n 1
)
n x

1
Propoziia 5. irul E n p (1  ) n  p este descresctor, p N*.
n
Demonstraie. Din identitatea
a n  p  b n  p (a  b)(a n  p 1  a n  p  2 b  ...  ab n  p  2  b n  p 1 ), a ! b ! 0,
1
1
rezult a n  p  b n  p ! (a  b )(n  p )b n  p 1 . Punnd aici a 1 
i b 1 
,
n
n 1
np
1
1 np
se obine (1  ) n  p ! (1 
)
(1 
) pentru n t 1 i p fixat. innd
n
n 1
n (n  2)

seama

1

np
1
,
! 1
n ( n  2)
n 1

va

rezulta

1
1 n 1 p
(1  ) n  p ! (1 
)

n
n 1

E n ( p ) ! E n 1 ( p ) , ceea ce ncheie demonstraia.


1 n x
)
este descresctor, x t 1 fixat.
n
1
Demonstraie. Se consider funcia f : [1, f) o R, f (t ) (1  )t  x , unde x t 1
t
t x
1
fixat. Atunci f c(t ) f (t )[ln(1  ) 
] i folosind inegalitatea ln(1  y )  y
t
t (t  1)
t x
1 t x
1 x
1
pentru y t 0 , avem ln(1  ) 
, de unde f ' t  0 i
 
t
t (t  1) t t (t  1) t (t  1)

Propoziia 6. irul E n (x ) (1 

deci f este descresctoare pentru t t 1 . Avem f (n) ! f ( n  1) E n ( x) ! E n 1 ( x)


pentru n t 1 , ceea ce ncheie demonstraia.

Observaie. Propoziia 5 este un caz particular al Propoziiei 6, avnd ns o


demonstraie elementar.
1 n x
este cresctor, x t 0 fixat.
)
n
1
1
1 n 1 x
Demonstraie. Putem scrie u n 1 (x ) (1 
.
)
1 n  x 1 E n (x  1)
n 1
(1  )
n
Conform Propoziiei 6, irul E n ( x  1) este descresctor pentru x  1 t 1 x t 0 i
are termenii pozitivi. Rezult ca irul u n 1 (x ) este cresctor, adic

Propoziia 7. irul u n ( x ) (1 

u 2 (x )  u 3 ( x )  ...  u n (x )  ... . Dar u1 ( x ) =0, deci avem i u 1 (x )  u 2 (x ) , ceea ce


ncheie demonstraia.

29

Propoziia 8. irul v n ( x ) (1 

1 n x
)
n 1

este descresctor, x d 0 fixat.

Demonstraie. Se consider funcia g : (0, f) o R,

g (t )

(1 

1 t x
) , unde
t 1

1
t x
)
] i folosind inegalitatea
t  1 t (t  1)
1
t x
1
t x
x
ln(1  y )   y,  0  y  1, gsim ln(1 
)


d0
t  1 t (t  1)
t  1 t (t  1) t (t  1)
pentru t (0, f). Deci g c(t )  0 , t (0, f) , adic g este descresctoare pe (0, f) .
xd0

fixat. Avem

g c(t )

g (t )[ln(1 

Vom avea g ( n ) ! g ( n  1) v n (x ) ! v n 1 (x ) , n t 1 , ceea ce ncheie demonstraia.


1 n x
este cresctor, x t 1 fixat.
)
n 1
1 n x
1
1
Demonstraie. Avem v n ( x ) (1 
, n t 1 .
)
1
n 1
E n (x )
(1  ) n  x
n
Conform Propoziiei 6, irul E n (x ) este descresctor pentru x t 1 i are termenii

Propoziia 9. irul v n ( x ) (1 

pozitivi, deci v n x este cresctor, x t 1 fixat.

Observaii. n demonstraia Propoziiei 1, ne-am inspirat din [1;p. 24-25].


Relativ la irul E n (x ) , n [2; p. 223-224] se arat pe cale neelementar c este
1
1
descresctor pentru x t
i cresctor pentru x  . In [3; p. 13-14] se arat pe cale
2
2
1
elementar ca irul E n ( ) este descresctor. irurile studiate n Nota de fa au
2
aplicaie n studiul unor serii de funcii.

Bibliografie
1. Fr. Junker Hhere Analysis I, 1920.
2. G. Klambauer Problems and Propositions in Analysis, New York, 1979.
3. A. Vernescu iruri de numere reale, Bucureti, 2000.

30

Studiu comparativ privind cteva medii uzuale


Claudiu-tefan POPA 1
Vom folosi n cele ce urmeaz urmtoarele convenii de notare: dac x, y ! 0, atunci
x2  y2
2 xy
x y
xy , m a
, mp
sunt mediile armonic,
, mg
x y
2
2
geometric, aritmetic, respectiv ptratic ale numerelor x i y. De asemenea, introducem
mh

x2  y2
media ponderat a numerelor x i y cu ponderile reale x, respectiv y. n
x y

acest context, m h poate fi privit ca media ponderat a numerelor x i y cu ponderile y,


respectiv x. Au loc urmtoarele inegaliti ntre medii:
mh d m g d ma d m p d m
(1)
cu egalitate pentru x

y.

Se observ uor prin calcul c m  m a m a  m h ,


(2)
deci media aritmetic a numerelor x i y este medie aritmetic i pentru numerele m h i m.
De asemenea, aa cum m g este i medie geometric ntre m h i m a , la fel m p este medie
geometric ntre m a i m, deoarece

x2  y2
2

x  y x2  y2

.
x y
2

Vom presupune n continuare x  y i, ca n figura de mai jos, notm A


B

m p  ma , C

m g  mh , D

(3)
ma  m g ,

m  mp :

A
C  B  D  A.
2
Demonstraie. Pentru prima inegalitate, avem echivalent:
A
 C m a  m g  2m g  2m h m a  2m h  3m g .
2
a  b 4ab
a  b 4ab
ns m a  2m h

d2

2 2ab  3 ab 3m g . La fel,
2
2
ab
ab

Propoziie. Cu notaiile de mai sus, avem:

C  B m g  mh  m p  ma

x  y 2
2 x  y

x  y 2 xy


x y
2

x2  y2
 xy
2

x2 y2
x2 y2
x  y 2 1 x 2  y 2  xy 1 ! 0
 xy 
 xy

x y
2
2
2
2

Profesor, coala Alecu Russo, Iai

31

(4)

x2 y2
1
x2 y2
1!
 xy
x  y !
 xy A ! B .

x y
2
2

Pentru a arta c A ! B, s observm mai nti c m 2p  m g2 2ma2 , relaie ce rezult n


x zy

urma unui calcul de rutin; deci m a


numerelor m p i m g , avem c
mg x

mp

(m p2  m g2 ) / 2 . Pe de alt parte, aplicnd (1)

(m p2  m g2 ) / 2 t (m p  m g ) / 2 , cu egalitate pentru

y. Cum x z y, rezult c m a ! (m p  m g ) / 2 , adic A ! B.

n continuare s aplicm inegalitatea

xy  x d y  xy (echivalent cu m g d m a )

numerelor ma i m. . innd seama de (3) rezult c m p  m a d m  m p B d D,


egalitatea fiind atins pentru cazul exceptat x y.
n fine, inegalitatea D  A rezult din faptul c A  C
(2)) i C  B.

B  D (relaie echivalent cu

Observaia 1. Rezultatul demonstrat generalizeaz unele probleme aprute n Gazeta


Matematic. Astfel problemele
x y
xy
E: 11 997. Dac x, y ! 0, demonstrai c
xy d

.
4
x y
Gh. Neculeu i Ion Neculeu
x2  y2
 xy .
2
Claudiu-tefan Popa
sunt simple transcripii ale inegalitilor A ! C , respectiv A ! B.

E: 12 162. Dac x, y ! 0, s se arate c x  y t

Observaia 2. Se pot obine inegaliti mai interesante, n care s apar mai multe
dintre medii. Prezentm un exemplu: adunnd membru cu membru inegalitile A t B,
x y
1 x2  y2
2 xy
,
 xy t

2
3
2
x y

inegalitate a crei demonstraie direct este destul de laborioas.

AtC

At A

obinem

3A t A  B  C

Observaia 3. Relaiile (4) arat c oricum am alege trei dintre numerele A, B, C, D,


ele pot constitui laturile unui triunghi (n cazul x z y ).
Observaia 4. Continund ideea, aezm pe o ax numerele C  B  D  A i
comparnd lungimile intervalelor care apar, obinem noi inegaliti n care apar mediile. n
acest sens, problema:
x y
xy
x y
E: 12 177. Dac x, y ! 0, demonstrai c
.

 xy d
4
x y
2
Manuela Prajea
este banal, ntruct se reduce la A  C d x  y , inegalitate grosier.

32

NOTA ELEVULUI
Aplicaii ale monotoniei mediilor n raport cu ordinul lor
Codrin ANDREI i tefan RUSU 1
Fie date numerele x1, x2, ..., xn R*+ (n N*) i R. Se numete media de
ordin a numerelor x1, x2, ..., xn numrul M (x1,...,xn) (pe scurt, M) definit prin

MD

Observm c

x 1D  x 2D  "  x nD

n x  x " x
n
1
2

M0= n x1 x 2 ... x n = Gn

1/D

, D z0

(1)

este media geometric, M1=

x1  x 2 ...  x n
n

=An -

n
1
1 =Hn - media armonic a numerelor date.
 ... 
x1 x 2
xn

media aritmetic i c M-1= 1

Mai general, se numete media ponderat de ordin a numerelor x1, x2, ..., xn cu
ponderile (pozitive) p1, p2, ..., pn numrul M(x, p) (pe scurt, M) definit prin

M D (x , p )

D
D
D 1 / D
p 1 x 1  p 2 x 2  ...  p n x n
,
D z0

p 1  p 2  ...  p n

p
p
p 1 /( p1  p 2 ... p n ) , D 0
x 1 1 x 2 2 ... x n n

>

(2)

( pentru p1=p2==pn=1, (2) devine (1) ).


Vom prezenta mai nti un rezultat binecunoscut, anume :

Propoziie. Mediile (ponderate sau nu) sunt monoton cresctoare n raport cu


ordinul lor, adic , R, < M d M .
Demonstraie. S presupunem mai nti c avem 0<<. Ca urmare, exist t >1
astfel nct = t . Utilizm inegalitatea lui Jensen pentru funcia convex f (x)=x t,
x 0,f , i scriem :
p1 x1  p 2 x 2  ...  p n x n

p1  p 2  ...  p n

t
t
t

p x  p x  ...  p n x n
d 1 1 2 2
.
p1  p 2  ...  p n

nlocuim n aceasta x1, x2, ..., xn respectiv cu x1 , x 2 ,..., x n ; obinem :

Elevi, Liceul Teoretic Gr. Moisil, Iai

33

(3)

D t

p1 x1  p 2 x 2  ...  p n x n
p1  p 2  ...  p n

i , prin ridicare la puterea pozitiv

1
tD

tD
tD
tD
p1 x1  p 2 x 2  ...  p n x n
p1  p 2  ...  p n

deducem c M d Mt , adic M d M .

Dac <<0, atunci exist t >1 astfel nct =t . Se procedeaz n mod


similar: n (3) se nlocuiesc x1, x2, ..., xn cu x1E , x 2E ,..., x nE , se ridic la puterea negativ
1
ambii membri ai inegalitii rezultate i se obine M t Mt , adic M d M .
tE

n sfrit, dac n inegalitatea M < M ( 0< < ) facem ca s tind la zero,


vom obine M0 < M ( >0). Tot ca un caz limit se obine i M< M0 ( <0). Q.e.d.
Indicm cteva aplicaii ale propoziiei de mai sus.
y

1
1

1. S se rezolve ecuaia x   x 
2 x y , x t 1 i y ! 0 .
x
x

Soluie. Avem trei cazuri :


1 1
I. y=1. Ecuaia devine x   x  2 x i aceasta este verificat de orice x t 1 .
x
x

Aadar perechile (x,1),  x t 1 , sunt soluii ale ecuaiei date.

II. 0<y<1. Utiliznd inegalitatea My d M1 pentru numerele distincte x 


y
y

1
1
1
1

obinem : x   x   2 x   y  2

x
x
x
y

nu are soluii n acest caz.

1
x 
x

2x y

, deci ecuaia dat

III. y>1. Utilizm de aceast dat inegalitatea M1 d My pentru aceleai numere distincte i
obinem c membrul nti al ecuaiei este strict mai mare ca 2x . Deci nu avem soluii.
Rezumnd, ecuaia dat are ca soluii perechile (x,1),  x t 1 .

2. Fie a, b, c >0 cu abc=1. Demonstrai inegalitatea 2(a+b+c) d a2b+ab2+a2c+


+ac2+b2c+bc2

(V. Popa, G : 615, R. M. Galai, nr. 17-18).

Soluie. Prin calcul direct constatm c a2b + ab2 +a2c +ac2+ b2c+ bc2-2 (a+b+c)
=(a+b+c)(ab+bc+ca-2)-3abc i, deci,avem de artat c (a+b+c)(ab+bc+ca-2) t 3 (1) (am
utilizat condiia abc=1). Inegalitatea A t G pentru numerele a, b, c revine la a+b+c t 3 (2)
iar G t H conduce la 1 t 3 /(ab  bc  ca ) sau ab+bc+ca-2 t 1 (3) (s-a inut seama de relaia

abc=1). Din (2) i (3) , prin nmulire, se obine (1).

34

1
d sin 2n D  cos 2n D d 1,  D R i n N*.
2 n 1
Soluie. Partea dreapt se dovedete astfel : sin 2nD  cos2nD d (sin2 D  cos2D )n 1 .
Pentru a dovedi inegalitatea din partea stng, utilizm M d M pentru numerele sin 2 D i

3. S se arate c

cos 2 D ; obinem : sin 2n D  cos 2n D t 2[(sin 2 D  cos 2 D ) / 2]n

1 / 2 n 1

, q.e.d.

4. Fie ABC un triunghi dreptunghic n A i numrul R. Au loc inegalitile :


D

(i) b  cD d 21D / 2 a D , dac D d 2


(ii) bD  cD t 21D / 2 a D , dac D t 2 .
Soluie. Avem D d 2 MD (b, c) d M 2 (b, c)

>(b

cD )/2

1/D

>

d (b 2  c 2 ) / 2

1/ 2

a/

2 b D  c D d 2 1D / 2 a D

Similar se procedeaz pentru a dovedi (ii).


5. Fie ABC un triunghi echilateral nscris n cercul C(O,R) i MC(O,R)
oarecare. Artai c D , E , J R astfel nct D d 2, 2 d E d 4 i J t 4 au loc
inegalitile:
(i) MA D  MB D  MC D d 3 2 D / 2 R D ; (ii ) MA J  MB J  MC J t 3 6 J / 4 R J

(iii) 3 2 E / 2 R E d MA E  MB E  MC E

3 6E /4 R E .

Soluie. Sunt cunoscute relaiile urmtoare : MA2+MB2+MC2=6R2 i

MA4+MB4+MC4=18R4 ([1], pp. 27-28). Ca urmare, M2(MA, MB, MC)=R 2

M4(MA,MB,MC)=R 4 6 . Pe de alt parte, avem : M d M2 d M d M4 d M. Din M d M2


D

urmeaz c MA  MB  MC =3(M) d 3(M2) = 3 2D / 2 R D , adic (i). Analog se


obin (ii) i (iii).
6. Fie ABCD un ptrat nscris nscris n cercul C(O, r). Fie MC(O,r) i
D , E , J R astfel nct D d 2, 2 d E d 4 i J t 4 . Artai c au loc inegalitile :

(i) MA D  MB D  MC D  MD D d 4 (3/ 2)D / 2 r D ;

(ii ) MA J  MB J  MC J  MD J t 4 6J / 4 r J

(iii) 4 (3 / 2) E / 2 r E d MA E  MB E  MC E  MD E
Soluie.
4

Se
4

utilizeaz

MA  MB  MC  MD
inegalitile cerute.

24r

egalitile:
4

4 6E /4 r E .

MA 2  MB 2  MC 2  MD 2

6r 2

[4] i , procednd ca n problema precedent, se obin

Bibliografie
1.C. Cocea - 200 de probleme din geometria triunghiului echilateral, Ed. Gh. Asachi,
Iai, 1992 .
2.L. Pran, C.-G. Lazanu -Probleme de algebr i trigonometrie, Ed. Facla,
Timioara,1983.
3.V. tefnescu, N. Deval - Asupra unor inegaliti ntre medii generalizate i aplicaiile lor,
G.M. 1/1985, 8-11.
4.G. ieica Culegere de probleme de geometrie, Ed. Tehnic, Bucureti, 1956.

35

CHESTIUNI METODICE
Funcii care admit / nu admit primitive
Dumitru GLEAT 1 i Gabriel POPA 2
Ne propunem n ceea ce urmeaz o abordare ct mai intuitiv a unor probleme
considerate n general ca fiind dificile, grupate n urmtoarele clase:
I. Dat o funcie, s se stabileasc faptul c admite primitive;
II. Dat o funcie, s se arate c nu admite primitive;
III. Dat o funcie a crei expresie depinde de anumii parametri, s se determine
valorile acestora astfel nct funcia s admit primitive.
n redactarea acestui articol am pornit de la constatarea c soluiile date n diverse
cri unora dintre problemele din [2], pag. 13-14, sunt artificiale i greu de urmrit de ctre
elevi. Propunem n cele ce urmeaz o prezentare pe care o credem logic i adecvat
predrii la clas.
I. (i) Orice funcie continu admite primitive.
1

1 e x2
Problema 1. Funcia f : R o R , f(x)= 5
x 0

1
x2

Soluie. Substituia

x ! 0 , deci lim f ( x)

t conduce la

1
x5

,xz0
,x
5
2

t2

t 2

admite primitive.

dac x ! 0 i

1
x5

t

5
2

dac

0 ; lim f ( x) lim t = 0, de unde lim f ( x) = 0 =


xo 0
xn0
t of e
et
f(0), adic f este continu n 0. Cum f este evident continu pe (-f , 0) i pe (0 , f),
rezult c f continu pe R , deci f admite primitive.
1

x sin
,x z 0
i g : R o R ,
Problema 2. Funciile f : R o R , f(x)=
x
0
,x 0
lim

t of

xp0

x cos
g(x)=
x
0

,x z 0
,x

admit primitive.

1
x o0
 0 d x 
o 0 , deci lim f ( x ) = 0 = f(0),
xo 0
x
adic f este continu . Analog se procedeaz pentru g .

Soluie. Pentru f, avem

x sin

(ii) Anumitor funcii li se pot construi efectiv primitivele.


1
2

Profesor, Grup colar tefan Procopiu, Iai


Profesor, Liceul Teoretic Garabet Ibrileanu, Iai

36

1
1

2 x sin  cos
Problema 3. Funciile f : R o R , f(x)=
x
x

0
1
1

2 x cos  sin
,x z 0
admit primitive.
g(x) =
x
x

0
,x 0

,x z 0
,x

i g : R o R ,

1
2
x sin
,x z 0
, care este derivabil pe R

x
0
,x 0

i Fc(x) = f(x), x R . S mai artm c F este derivabil n origine, iar Fc(0) = 0:
1
x 2 sin
F ( x)  F (0)
x lim x sin 1 0
lim
F ' (0) lim
x o0
x o0
x o0
x0
x
x
(vezi Problema 2), deci F este primitiv a lui f . Analog se procedeaz pentru g .
1
1
n 1
nx sin  x n  2 cos
,x z 0
Generalizare 1. Funciile fn : R o R , fn(x)=
i
x
x

0
,x 0
1
1
n 1
nx cos  x n  2 sin
,x z 0
gn: R o R , gn(x)=
admit primitive,  nt 2.
x
x

0
,x 0

Soluie. Considerm funcia F ( x)

Generalizare
2.
1
1

n
(n  1) x sin n  n cos n
fn(x)=
x
x

Funciile

fn

,xz0
,x

1
1

(n  1) x n cos n  n sin n
i gn : R o R , gn(x)=
x
x

,x z 0
,x

admit primitive,  n t 1.

(iii) O combinaie liniar a unor funcii ce admit primitive, admite primitive.


1
sin
,x z 0
i g : R o R , g(x)=
Problema 4. Funciile f : R o R , f(x)= x
0
,x 0
1

cos
=
x
0

,x z 0
,x

admit primitive.

Soluie. Funcia f se poate scrie ca o combinaie liniar cu coeficienii 2, 1 a


funciilor g din Problemele 2 i 3, deci admite primitive. Analog pentru g .
x
Observaia 1. Dac F este o primitiv a funciei f, atunci funcia f ( ) admite

x
primitiva DF .
D
37

Observaia 2. Funciile f i g snt exemple de funcii necontinue (cu


discontinuitate de specia a doua n origine) care admit primitive.
2 1
sin x , x z 0
Problema 5. Funciile f : R o R , f(x)=
i g : R o R , g(x)=
1

,x 0
2
2 1
cos x
=
1

,xz0

admit primitive.
,x

Soluie. Avem f ( x)

2
1
2
2 (1  cos x ) , x z 0
1 1 cos
= 

x
1
2 2 0

,x 0

,x z 0
,x

i analog

pentru g.
31
sin
Problema 6. Funciile f : R o R , f(x)=
x
0
31
cos
=
x
0

,x z 0
,x

,x z 0
,x

i g : R o R , g(x)=

admit primitive.

1
3
3 sin
,xz0
, x z 0 1 sin
,


x
x
4
4 0
0
,x 0
,x 0

fiecare dintre funciile din dreapta admind primitive cf. Problemei 4 i Observaiei 1.
2 n 1 1
sin
,x z 0
Generalizare. Funcia f : R o R , f(x)=
admite primitive
x

0
,x 0
(O.M.,etapa judeean, 1982).

Soluie. sin

3
x

3 sin

1
1
 4 sin 3 , deci f ( x)
x
x

II. (i) Funciile care nu au P.D. nu admit primitive. n acest sens, innd seama
de proprietile unei funcii cu P.D., putem proceda n mai multe moduri:
x Dac Im f nu este interval, atunci f nu admite primitive;
x Dac f are discontinuiti de specia I , atunci f nu admite primitive.
Problema 7. Funcia f : R o R, f(x) = >x @ nu admite primitive.
Soluie. Im f = Z , care nu este interval.
Problema 8. Funcia f : R o R , f(x) = x  >x @
Soluie.

Pentru

k Z,

avem

^x `

nu admite primitive.

lim f ( x ) 1, lim f ( x)
xn k

xp k

0, deci

are

discontinuiti de specia I.
f ( x ) , x ( a, b) Q
Problema 9. Funcia h : (a,b) o R , h(x)=
, cu f i g
g ( x ) , x ( a, b) / Q

38

funcii continue distincte, nu admite primitive (O.M., etapa final, 1981).


Soluie. Cum f z g, exist x0 (a, b) a.. f(x0) z g(x0) . Cum R este separat
Hausdorff, putem alege Vf V(f(x0)) i Vg V(g(x0)) a.. Vf Vg = . Deoarece f, g sunt
continue, exist Uf ,Ug V(x0) (pe care le putem presupune intervale deschise) a..  x
(a, b) Uf f(x) Vf i  x (a, b) Ug g(x) Vg. Observm acum c (a, b) Uf
Ug este un interval, iar imaginea sa prin h este o reuniune de mulimi disjuncte incluse
n Vf respectiv Vg ,care nu poate fi interval.
Observaie. Particulariznd convenabil funciile f i g, obinem soluii pentru
problemele II.5, 6, pag. 13 din [2].
(ii) Pentru anumite funcii, putem demonstra efectiv c nu admit primitive.
1
1 1
sin  cos
,x z 0
nu admite
Problema 10. Funcia f1 : R o R , f1(x)= x x
x

0
,x 0
primitive (v. i Problema 3).
c
1
1 1
1

Soluie. Observm c x sin sin  cos , deci o eventual primitiv a


x
x x
x

x sin  c 1 , x z 0
lui f 1 are n mod necesar forma F(x) =
. Cum F trebuie s fie
x

c2
,x 0
continu, atunci c1 = lim F ( x) F (0) c2 , deci c1 = c2. Funcia F trebuie s fie
x o0

F ( x )  F ( 0)
derivabil, deci exist limita lim
x o0
x 0
posibil. Urmeaz c f 1 nu admite primitive.

x sin
lim

x o0

1
x

lim sin

x o0

1
, ceea ce nu este
x

(iii) Reducere la absurd.


1
sin
Problema 11. Funcia f : R o R , f(x) = x
1

,x z 0

nu admite primitive.
,x

1
sin
,x z 0
Soluie. Presupunem c f ar admite primitive. Cum funcia g(x) = x
0
,x 0
admite primitive (v. Problema 4), atunci diferena lor admite primitive, deci funcia h(x) =
0 , x z 0
1
f(x) g(x) = 1
admite primitive. ns Im h = ^0, ` nu este interval , deci h nu
,x 0
2
2
admite primitive. Presupunerea fcut este fals.
Observaia 1. Problema de mai sus furnizeaz un exemplu de funcie cu P.D. care
nu admite primitive.
Observaia 2. Putem da, folosind acest raionament, nenumrate exemple de
funcii cu ramuri care nu admit primitive. De altfel, n unele culegeri de teste gril n
vederea pregtirii a diverse examene, se ntlnesc probleme de forma:

39

1
sin
,xz0
Funcia f : R o R , f(x) =
admite primitive d.n.d. a =
x
a
,x 0
Elevii trebuie obinuii s simt valoarea potrivit a constantei nc nainte de a trece la
rezolvarea propriu-zis a problemei !

III. Apar deseori aplicaii de tipul:


ae x
,x d 0

Problema 12. Se consider funcia f : R o R , f(x) = 1  x  b


cu
,x ! 0

a, b R. Determinai a, b a.. f s admit primitive.


Metoda uzual de abordare a unor astfel de probleme este cutarea primitivelor lui
f pe ramuri, considerarea unei primitive F sub forma general

ae x  c1
,x  0

F(x)=
c2
, x 0 i determinarea apoi a constantelor a i b

x
 c3 , x ! 0
2 x  1  ln b
x (1  x  1) 2

a.. F s fie continu i derivabil.


Propus pentru prima oar n clas, problema primete ns de obicei urmtoarea
rezolvare: Pentru ca f s admit primitive, trebuie ca f s fie continu i putem afla a, b
din continuitatea lui f . Lsnd la o parte confuzia care se face ntre condiia necesar i
cea suficient, n contextul dat un astfel de rspuns nu este chiar att de greit. n >4@ este
enunat i parial demonstrat urmtorul rezultat (Problema 10, pag. 118):
Teorem. Fie , un interval, x0 punct interior lui ,, iar f : , o R o funcie cu P.D.
Dac f are limite laterale n x0, atunci f este continu n x0.
Demonstraia dat n >4@ este valabil numai n cazul limitelor laterale finite
(adic al discontinuitilor de specia I), ns ea poate fi adaptat i n cazul general.
Presupunem, prin reducere la absurd, c f(x0 - 0) < f(x0) (limita putnd fi finit sau -f i
fie D un numr ntre cele dou valori. Atunci  V V(f(x0 0)),  U V(x0) a..  x U ,
x < x0 f(x) V. Considernd V = (f(x0 0), D), exist U pe care o putem lua
U = >a, x0) a..  x U f(x) V ; altfel spus, f(x) < D, x >a, x0). Rezult c f
nu ia valoarea D pentru x >a, x0@, fals.
Revenind la Problema 12, f admite primitive, deci are P.D. n plus, f are limite
r f , b z 1

laterale n 0 : f(0 0) = a, iar f(0 + 0) = 1


. Din teorema de mai sus, rezult c
,b 1
2
1
a= , b =1.
2
Bibliografie
1. V. Arsinte Probleme elementare de calcul integral , Ed. Univ. Bucureti, 1995.
2. N. Boboc, I. Colojoar Analiz matematic , manual cl. a XII-a, E.D.P., 1999.
3. M. Ganga Elemente de analiz matematic , cl. a XII-a, Ed. Mathpress, 1999.
4. Gh. Gussi et al. Analiz matematic , manual cl. a XI-a, E.D.P., 1999.

40

CHESTIUNI COMPLEMENTARE MANUALELOR


Construcii geometrice cu echerul
Eugenia COHAL 1
Nota Redaciei. S-au primit de la mai muli elevi rezolvitori soluii necorespunztoare la problema VII.20
din Rec.Mat-1/2001 : acetia au confundat echerul abstract cu echerul colar ce are un unghi de 30q. Este
binevenit aceast lucrare ce aduce clarificri n aceast privin.

Geometria constructiv sau teoria construciilor geometrice este o parte a geometriei


fascinant prin problematica abordat , vechimea i istoria ei, printr-un numr de probleme
celebre formulate n antichitate i care i-au gsit rezolvarea dup multe secole de cercetri
i cu utilizarea cunotinelor de algebr i analiz matematic. Muli i strlucii
matematicieni au fost atrai de acest domeniu de care i-au legat numele prin contribuiile
aduse.
Rezolvarea problemelor de construcii (geometrice) depinde de instrumentele admise
a fi utilizate. Pe lng rigl i compas , de-a lungul timpului au fost folosite i alte
instrumente : rigla bilateral, rigla cu etalon, distanierul, echerul etc. sau au fost
inventate instrumente speciale cu care puteau fi trasate anumite curbe.
Fiecare instrument (sau grup de instrumente) se definete prin operaiile fundamentale
ce pot fi efectuate cu acesta (acestea).
n prezenta Not atenia se concentreaz asupra unui singur instrument: echerul.
Acesta trebuie imaginat ca dou rigle fixate sub un unghi drept n dou capete ale lor. I se
mai spune echerul drept n contrast cu echerul oblic (colarul). Evident, echerul se
deosebete de instrumentul colar cu acelai nume.
n fapt, echerul este definit de operaiile (construciile) ce se pot face cu el pornind de
la elementele date de problem (puncte, drepte etc.) ct i de la anumite elemente luate
arbitrar. Operaiile fundamentale ce definesc echerul sunt n numr de trei:
1q construirea dreptei ce trece prin dou puncte i determinarea punctului de
intersecie a dou drepte ;
2q construirea perpendicularei la o dreapt ce trece printr-un punct;
3q construirea unui punct pe o drept din care dou puncte se vd sub un unghi
drept.
Dm acum cteva construcii ce pot fi fcute cu ajutorul echerului.
1. Construcia paralelei la o dreapt d printr-un punct P exterior ei. Se construiete
cu echerul d c perpendicular pe d i care trece prin P (2q). Apoi se construiete d cc
perpendicular n P dreptei d c (2q). Evident, d cc este drepta ce trebuia construit.
2. Multiplicarea segmentelor. Pentru a dubla, tripla etc. un segment AB dat procedm
astfel (fig. 1):
Prin punctul A costruim dou drepte d i d1 (1q), d oarecare iar d1 perpendicular pe
d (2q). Perpendiculara n B pe dreapta AB (2q) taie dreptele d1 i d n punctele B1 ,
respectiv B2 (1q). Fie d2 perpendiculara n B2 pe d (2q). Perpendiculara n B1 pe drepta d1
se intersecteaz cu d2 n punctul C2 (2q, 1q). Perpendiculara pe drepta AB ce trece prin C2
1

Profesor, Liceul teoretic Mihai Eminescu, Iai

41

intersecteaz dreptele d1 i AB n punctele C1 i repectiv C. Obinem similar punctele D2 ,


D1, D . a. m. d. Segmentele AB, BC, CD etc. sunt congruente ca proiecii pe dreapta AB
ale segmentelor congruente AB1, B1C1, C1D1 etc.
d1

D1

P
C1

d2

B1

d
A

Q R

C
B
C2

D2
D

Ac

B2
Fi g. 1

Fi g. 2

Fi g. 3

3. mprirea segmentelor. S urmrim pe fig. 2, mprirea segmentului AB n trei


pri congruente ; avem d AB (construcia 1), segmeentele PQ, QR, RS congruente
(construcia 2) i AS BP {I } . Punctele C i D , ce mpart segmentul AB n trei
pri congruente, se obin ca interseciile dreptei AB cu dreptele RI i QI.
njumtirea se face printr-o construcie mai simpl : se proiecteaz pe AB centrul
unui dreptunghi oarecare construit (cu echerul) pe segmentul AB.
4. Construcia bisectoarei unui unghi . Procedeul este indicat de fig.3, unde s-au luat:
A arbitrar, A ' simetricul lui A fat de O (construcia 2), B se obine punnd vrful
echerului pe latura Oy astfel ca laturile lui s treac prin punctele A i A ' (operaia 3q) i
OM A ' B . Faptul c semidreapta OM este bisectoarea unghiului xOy decurge din
congruena unghiurilor marcate.
Observaie. Dat un triunghi ABC se pot construi cu echerul punctele H, G, O, I.
5. Construcia centrului unui cerc. Fie A un punct pe cercul dat. Punem vrful
echerului n A i marcm cu B i C punctele n care laturile lui intersecteaz cercul.
Dreapta BC trece prin centrul cercului. Lum pe cerc un alt punct A ' i obinem o dreapt
B ' C c n acelai mod. Centrul cercului este punctul de intersecie a acestor drepte.
6. Intersecia unei drepte cu un cerc. Fie dat un cerc printr-un diametru AB
(echivalent, prin centru i un punct al su) i o dreapt d. Conform cu 3q , gsim cu echerul
un punct P d din care diametrul AB se vede sub unghi drept. Atunci, P se afl i pe cerc.
Al doilea punct de intersecie cutat se poate construi (cu echerul) ca simetricul lui P n
raport cu dreapta perpendicular pe d i care trece prin centrul cercului.
Rezolvarea urmtoarelor probleme (cu echerul) o propunem cititorilor ( eventual se
gsete n [2] , p.73 ).
7. Date o dreapt d i un punct O d , s se determine un punct P d astfel nct
OP AB , segmentul AB fiind dat.
8. Date o dreapt d i un punct P , s se duc prin P o dreapt care s formeze cu
d un unghi de mrime egal cu a unui unghi cunoscut.
Observaie . Orice construcie cu rigla i compasul poate fi efectuat cu echerul.
Bibliografie
1. N.F. Cetveruhin Metodele construciilor geometrice (l. rus), Moscova, 1952.
2. A. Tth Noiuni de teoria construciilor geometrice, Ed. Did. i Ped., Bucureti, 1963.
42

DIN ISTORIA MATEMATICII


Teorema celor patru culori
Silviana IONESEI 1
Cum se face c matematica produs
prin excelen al gndirii umane,
independent de experien poate fi
att de admirabil adaptat obiectelor
lumii reale?
Albert Einstein
Este binecunoscut faptul c marile probleme ale matematicii, cum ar fi Marea
Teorem a lui Fermat sau Conjectura lui Goldbach, au contribuit enorm la dezvoltarea
acestei tiine. Din eforturile matematicienilor (timp de zeci sau chiar sute de ani) de a gsi
o rezolvare la ntrebri n aparen simple s-au nscut noi discipline n matematic, cu
aplicaii spectaculoase.
Problema celor patru culori are toate valenele unei probleme de mare carier:
n primul rnd formularea ei este extrem de simpl, nu presupune cunotine matematice; n
al doilea rnd, ea a rmas nerezolvat timp de peste un secol, fiind surprinztor de grea i a
suscitat preocuparea multor matematicieni de prestigiu.
Iat cteva repere istorice.
n 1852 un geograf din Edinburgh (istoria nu i-a reinut numele) l-a informat pe
prietenul su, student n matematici, c folosete cel mult patru culori pentru o hart
mprit n regiuni, fr ca dou regiuni vecine s aib aceeai culoare (precizm c este
vorba despre hri plane, cu regiuni nchise, iar vecine sunt regiunile cu o linie de
frontier comun; dou regiuni care se ntlnesc ntr-un numr finit de puncte nu sunt
considerate vecine).
Tnrului matematician, pe nume Francis Guthrie, i-au plcut cele aflate i a cerut
informaii mai ample, ns geograful l-a ncredinat c acest procedeu e foarte rspndit i
aplicat pretutindeni din cauza economiei care-l prezint. Rspunsul nu a fost mulumitor
pentru Guthrie ; el i-a propus s demonstreze acest fapt dar nu a reuit.
Fratele su, Frederick, studia chimia la Londra i aflnd de problema care-l
preocupa pe Francis a cerut ajutorul profesorului August De Morgan, dar nici acesta nu a
gsit o demonstraie satisfctoare.
n civa ani, problema a ajuns la mod printre matematicieni. Astfel, A. Cayley
nefiind nici el capabil s demonstreze valabilitatea teoremei, a propus-o Societii
Matematice din Londra.
S trecem n revist cteva din rezultatele pariale ale demonstrrii teoremei.
Faptul c trei culori nu sunt suficiente pentru colorarea oricrei hri plane a fost
repede constatat (vezi fig.1).
1

Profesor, Colegiul Naional C. Negruzzi , Iai

43

De Morgan a demonstrat c nu exist hart format din 5 regiuni astfel nct s fie
dou cte dou vecine, deci aceasta poate fi colorat cu patru culori.
A. B. Kempe, un avocat din Londra, membru al Societii Matematice din Londra
i deosebit de pasionat de matematic a publicat n 1879 un articol n care susinea c a
demonstrat teorema. Raionamentul lui era deosebit de ingenios; el redusese problema la
hri normale, adic hri n care nu exist ri nchise complet n alte ri i nici puncte

4
Fig.1

n care se ntlnesc mai mult de trei regiuni. Dei raionamentul s-a dovedit incomplet, el
coninea ideile de baz ce au condus la demonstraia corect un secol mai trziu. Astfel,
oricrei hri i se poate asocia un graf n care fiecare regiune este reprezentat printr-un
punct i dou puncte vor fi legate printr-o muchie dac i numai dac punctele corespund la
dou regiuni vecine (vezi fig.2).

A
A

B
C

B
C

E
E
E

D
Fig.2

n acest mod problema colorrii regiunilor de pe hart revine la problema colorrii


punctelor din graful asociat astfel nct punctele legate printr-o muchie s fie colorate
diferit.
Problema celor patru culori a contribuit la cercetari importante n teoria grafurilor,
cum ar fi numerele cromatice ale grafurilor.
Cu ajutorul unui graf special, matematicianul P. J. Heawood a artat n 1890 c
demonstraia lui Kempe are o eroare nu tocmai uor de nlturat.

44

Mai trziu, n 1913, matematicianul Ph. Franklin de la Massachusetts Institute of


Technology ridic limita numrului de regiuni pentru care problema este rezolvat de la 5
la 21, iar n 1940 Winn reuete s ajung la 35 de regiuni.
Un alt rezultat deosebit de interesant se datoreaz lui P. J. Heawood, care i-a
consacrat 60 de ani din via studierii problemei. Iat-l: probabilitatea de a gsi o hart cu
mai mult de 36 regiuni care s nu poat fi colorat cu patru culori este mai mic dect
10-10000 ! (de remarcat c 1010000 este un numr mai mare dect numrul atomilor din
ntreaga galaxie ).
O teorem a celor cinci culori (faptul c cinci culori sunt eficiente pentru a
colora o hart) a fost obinut relativ uor; o demonstraie elementar a acestui rezultat
poate fi gsit n Despre numere i figuri de H. Rademacher i O. Toeplitz.
Pe la mijlocul sec. XX s-a conturat ideea de rezolvare a problemei prin mrirea
numrului de regiuni pentru care patru culori sunt suficiente i examinarea unor aa-zise
configuraii inevitabile. Dac ar fi fost posibil s se produc toate aceste configuraii i s se
arate c ele pot fi colorate cu patru culori, atunci demonstraia ar fi fost complet.
Cea mai eficient metod de producere a configuraiilor s-a dovedit a fi un
algoritm implementat pe calculator de W. Haken i K. Appel , Universitatea Illinois, SUA ,
care au lucrat aproape 1200 ore i, n fine, demonstraia a fost ncheiat [1].
Un an mai trziu, folosind o alt procedur de reducere a configuraiilor
inevitabile, F. Allaise de la Universitatea Waterloo, Ontario, CA, a reuit s obin
demonstraia teoremei n numai 50 de ore de dialog om-calculator.
Entuziasmul firesc strnit n lumea matematicienilor de acest reuit neobinuit
pn atunci a fost temperat de voci sceptice care susineau c aceasta nu e o teorema de
matematic n sensul clasic. Astfel, T. Tymoczko n articolul Problema celor patru culori
i semnificaia ei filozofic (Journal of Philosophy, 1979) afirm c teorema exprim un
adevr a posteriori i nu a priori, ca orice adevr matematic. Argumentele sale se bazeaz
n principal pe imposibilitatea de a verifica manual ( cu creionul ) demonstraia, dat fiind
faptul c nu exist un unic algoritm care s verifice toate programele posibile pe calculator.
n replic, E. R. Swart scrie n [5] c inconvenientul semnalat de preopinentul su
este aparent, deoarece calculele foarte numeroase efectuate pe calculator erau de rutin, iar
programul utilizat poate fi verificat. Sarcina calculatorului a fost copleitoare prin
dimensiuni, sarcin pe care omul tia cum s o abordeze, dar n-ar fi putut-o termina
niciodat.
A fost prima situaie memorabil n urma creia lumea matematicienilor a trebuit
s admit existena unor demonstraii parial accesibile omului, ct i dreptul calculatorului
de a ne sprijini n stabilirea adevrurilor matematice.

Bibliografie
1. K. Appel, W. Haken Every Planar Map Is Four Colorable , Bulletin of the American
Mathematical Society, 82(1976), 711-712.
2. F. Cmpan Probleme celebre, Ed. Albatros, Bucureti, 1972.
3. Gh. Pun Din spectacolul matematicii, Ed. Albatros, Bucureti, 1983.
4. L.A. Steen Mathematicians Today, Twelve Informal Essays, SpringerVerlag, 1978.
5. E.R Swart The Philosophical Implications of the Four-Color Problem, The American
Mathematical Monthly, 87(1980), 697-707.

45

MATEMATICA N CLASELE PRIMARE


Introducerea operaiei de adunare la clasa I
Petru ASAFTEI 1
Introducerea operaiei de adunare la clasa I se poate face fie folosind reuniunea a dou
mulimi disjuncte, fie folosind rigletele.
n prezenta not metodic utilizm prima variant, considernd c aceasta este mai
puternic ancorat n experiena de via. Vom putea folosi astfel un vocabular mai bogat,
contribuind astfel la o mai bun nelegere a operaiei de adunare.
n predarea-nvarea noiunilor cu coninut matematic la colarii mici, conform
cerinelor psiho-pedagogice, trebuie s parcurgem urmtoarele etape:
I. Etapa operrii cu mulimile de obiecte concrete (etapa perceptiv). n aceast etap se
realizeaz aciunea nemijlocit cu obiecte concrete din mediul nconjurtor.
Exemplu. Pe o farfurie punem trei mere i patru pere, apoi prin numrare constatm c
avem apte fructe. (Se recomand s folosim cel puin o fraz n care s fie inclui termenii
premergtori operaiei de adunare ; exemplu : Copii, dac la trei mere adugm patru pere,
obinem apte fructe.)
II. Etapa formrii reprezentrilor imaginativ-concrete (etapa semi-abstract). n
aceast etap se construiesc mulimi cu mere i pere
(jetoane), apoi se face reuniunea lor (fig. 1). Aceast
construcie se efectueaz simultan la tabl i pe
caiete. Fiecrei mulimi care intr n reuniune i
atam cifra care indic numrul de fructe (3,
respectiv 4). Mulimii obinut prin reuniune i
atam cifra 7 (prin numrare). Aceast etap putem
3
7
4
s
o ncheiem cu o fraz de genul : Copii dac
Fig. 1
punem la un loc 3 mere i 4 pere obinem 7 fructe .
III. Etapa scrierii i efecturii adunrii (etapa abstract). Acum reprezentm att merele
ct i perele cu aceleai simboluri (stelue, cerculee etc.) (fig. 2). Se explic elevilor c
pentru a arta faptul c am pus la un loc o mulime cu trei elemente i alta cu patru
elemente se folosete semnul ( simbolul matematic ) ,,+ , numit plus, i scriem 3+4.
Scrierea 3+4 se citete trei plus patru sau trei adunat cu patru.
Din etapele precedente, prin numrare, elevii au
constatat c mulimea obinut prin punere la un loc
are apte elemente. Atenionm elevii ca 3+4 i 7
reprezint tot att. Pentru a exprima acest lucru se
folosete simbolul ,,=" i scriem 3+4=7 se citete
trei plus patru este egal cu apte . Spunem elevilor
c scrierea 3+4 reprezint adunarea neefectuat a
3
3+4
4
numerelor
3 i 4 iar scrierea 3+4=7 nseamn
Fig. 2
efectuarea adunrii numerelor 3 i 4. Explicm
1

Profesor, coala Normal Vasile Lupu, Iai

46

elevilor c trecerea de la scrierea 3+4=7 s-a fcut prin operaia de adunare.


Trebuie s dm elevilor un algoritm eficient de efectuare a operaiei de adunare n acest
stadiu. S scriem adunarea 5+2 i s numrm de la 5, nainte, nc dou numere n irul
numerelor naturale. Ajungem astfel la numrul 7 i spunem elevilor c 7 este rezultatul
adunrii numerelor 5 i 2. Scriem i n acest caz 5+2=7. Pentru verificarea corectitudinii
rezultatului obinut putem s apelm la procedeul punerii la un loc. n continuare putem s
dm terminologia specific operaiei de adunare. Referindu-ne la ultima scriere 5+2=7
spunem elevilor c numerele 5 i 2 sunt numere care se adun i se numesc termenii
adunrii, primul termen, respectiv, al doilea termen. Despre 7 spunem c este rezultatul
adunrii sau totalul.
Un caz special l reprezint adunarea cnd unul
dintre termeni este zero. Folosim o nou reprezentare
(fig. 3) din care elevii constat c dac punem la un
loc o mulime cu dou elemente i o mulime fr nici
un element, obinem tot o mulime cu dou elemente.
Faptul c am pus elementele celor dou mulimi la un
loc se scrie 2+0. Numrnd elementele noii mulimi
2
2+0=2
0
gsim 2 elemente. Cum 2+0 i 2 reprezint tot att
Fig. 3
putem scrie 2+0=2. Spunem elevilor c 0 l las pe 2
neschimbat prin operaia de adunare. Cu un demers asemntor se pot justifica i scrierile
0+2=2 i 0+0=0. Deoarece 0 are aceste proprieti l numim element neutru la adunare.
Suntem pregtii s punem n eviden proprietatea de simetrie a relaiei de egalitate
folosind relaia tot att. Exemplu. Deoarece 7 reprezint tot att ca i 5+2, putem s scriem
7=5+2 i 5+2=7.
Dup ce am consolidat algoritmul de adunare, putem s dm i cele dou proprieti
importante ale ei: comutativitatea i asociativitatea. Propunnd elevilor s efectueze
adunrile 5+4 i 4+5 vor constata c obin acelai rezultat, numrul 9, deci 5+4 i 4+5
reprezint tot att. Din acest motiv scriem 5+4=4+5 . Spunem elevilor c rezultatul unei
adunri nu se schimb dac schimbm termenii ntre ei ; n aceasta const proprietatea de
comutativitate. Pe baza proprietii de comutativitate se introduce conceptul de prob a
adunrii prin adunare.
Exemplu. Propunem elevilor s efectueze adunarea 5+3. Numrnd de la 5, nainte, nc
trei numere consecutive, vor obine rezultatul 8. Le spunem elevilor c putem verifica acest
rezultat efectund adunarea 3+5, adic numrnd de la 3, nainte, nc cinci numere
consecutive din irul numerelor naturale. Elevii vor constata c obin acelai rezultat :8 .
Pentru a justifica proprietatea de asociativitate, propunem elevilor s adune numerele 2,
4 i 3 astfel: nti s adune numerele 2 i 4 iar rezultatul obinut s l adune cu 3, apoi s
adune numerele 3 i 4 iar rezultatul s l adune cu 2. n primul caz elevii au efectuat
(2+4)+3 iar n al doilea caz au efectuat (4+3)+2 care este tot att ca i 2+(4+3). Cum n
ambele cazuri elevii au obinut acelai rezultat, numrul 9, putem scrie (2+4)+3=(4+3)+2
i spunem elevilor c au utilizat asocierea, nti a lui 2 cu 4, a doua oar a lui 4 cu 3. Putem
s introducem scrierea 2+4+3, numit sum cu trei termeni, nsemnnd (2+4)+3 sau
(4+3)+2.
Observaii. 1) nvtorul trebuie s explice rolul parantezelor n astfel de scrieri.
2) Dup ce elevii pot s scrie cu uurin aceste proprieti pe exemple numerice, se poate
trece la scrierea literal a lor, fcndu-se meniunea c literele pot reprezenta numere.

47

CONCURSURI SI EXAMENE
Concurs de admitere 2001 , Iai
Facultatea de Informatic , Universitatea Al. I. Cuza
Elemente de analiz matematic
I. a)
1. Privitor la numrul de asimptote, care dintre afirmaiile de mai jos pot fi adevrate
pentru o funcie f: RoR ?
i) f are dou asimptote orizontale, dou asimptote oblice i o asimptot vertical ;
ii) f are o asimptot orizontal, o asimptot oblic i o asimptot vertical ;
iii) f are o asimptot orizontal, o asimptot oblic i nici o asimptot vertical ;
iv) f are dou asimptote oblice, dou asimptote verticale i nici una orizontal.
Justificai rspunsul (rspunsurile) ales(e).
2. Care din propoziiile de mai jos exprim o proprietate adevrat privind continuitatea
i derivabilitatea?
i) Orice funcie continu ntr-un punct este derivabil n acel punct ;
ii) Orice funcie derivabil ntr-un punct este continu n acel punct ;
iii) Orice funcie continu pe intervalul [a,b] este derivabil pe intervalul (a,b) ;
iv) Orice punct n care o funcie oarecare este derivabil i continu este punct de
extrem al acelei funcii.
3. Pentru o funcie f : IoR, I interval deschis, f derivabil pe I, fie aI un punct de
minim local sau global. Atunci:
i) f s' (a ) z f d' (a ) ;

ii) f ' (a ) ! 0 ;

iii) f ' (a)  0 ;

iv) f ' (a) 0 .

4. Fie o funcie oarecare f :[a,b]oR, a<b, f derivabil pe intervalul (a,b). Atunci:


i) ntre dou rdcini consecutive ale lui f exist cel mult o rdcin a lui f ' ;
ii) ntre dou rdcini consecutive ale lui f ' exist cel mult o rdcin a lui f ;
iii) ntre dou rdcini consecutive ale lui f exist cel puin o rdcin a lui f ' ;
iv) ntre dou rdcini consecutive ale lui f ' exist cel puin o rdcin a lui f .
Justificai rspunsul (rspunsurile) ales(e).
5. Care dintre afirmaiile de mai jos sunt adevrate pentru orice ir de numere reale?
i) Orice ir convergent este marginit ;
ii) Orice ir convergent este monoton ;
iii) Orice ir mrginit este convergent;
iv) Orice ir monoton este convergent .
Not. Rspunsurile se vor da numai pe teza cu coluri nnegrite, sub forma: I.a) 1.x, unde x { i, ii, iii,
iv} etc. Fiecare ntrebare are cel puin una din cele patru variante corect. Dac la o ntrebare sunt
adevrate mai multe variante, atunci se vor indica toate variantele adevrate. Punctajul la o ntrebare nu se
acord dac indicai i variante false. Acolo unde se cere, se va aduga justificarea rspunsului dat.

  x , x  0
b) Fie funcia f : RoR, f (x )
.
1  x , x t 0
1. S se studieze continuitatea funciei n punctul x=0.
2. S se studieze derivabilitatea funciei n punctul x=0.
3. S se comenteze rezultatele de la puntele I.b)1. i I.b)2. prin prisma rspunsului
care l-ai dat la punctul I.a) 2 .

48

II. a) Fie aR, a>0, fixat i


xn

x n n t 0

a
1
x n 1 
, n t 1, cu x 0
2
x n 1

irul definit prin relaia de recuren:


2. S se studieze convergena acestui ir.


b) S se reprezinte grafic funcia f : RoR , f ( x) e

( x a )2
2b 2

, a i b constante reale.

Algebra
III. a) Fie (G, x) un grup cu proprietatea c x y 2 x 2 y 2 pentru oricare x, y din G.
Demonstrai c G este grup abelian.
b) S se discute dup valorile parametrilor a Z 8 , b Z 8 i s se rezolve n Z8 ecuaia
a x  b 0. Fiecare caz identificat va fi ilustrat printr-o ecuaie (a=?, b=?) i prin soluiile

respective.
IV. a) Definiia corpului comutativ.
b) Fie a, b, c numere reale. Definim pe R legile de compoziie:
x y ax  by  2 , oricare ar fi x i y numere reale; x y xy  2 x  2 y  c , oricare ar fi
x i y numere reale. Determinai a, b i c astfel nct (R, , ) s fie corp.

Facultatea de automatic i calculatoare, Univ. Tehnic Gh. Asachi


1. Suma cuburilor rdcinilor trinomului
3x 2  3(a  1) x  a 2 , aR, are valoarea
maxim dac:
1
3
(a) a
(e) a [ 2,  1] .
; (b) a
; (c) a  3 ; (d) a ! 0 ;
4
4
4
2
2
2.Mulimea rdcinilor ecuaiei a  b
a 2  b 2 , a, b C, ab0, este:
x
x 1

(a) {0, 1};

a 2  i ab a 2  i ab
(b)
,

;
2
2
a  b

a 2  iab a 2  iab
(c) 2
;
, 2
2
2

a 2  b 2

2a 2  i ab 2a 2  i ab
(d)
, 2
2
2
a  b2
a  b

a  b

a b

2a 2  iab 2a 2  iab
, 2
(e) 2
a  b 2
a b2

: (0, 1) (1,  f) o R, definit de f ( x ) x ln x , este:


strict descresctoare pe (0, 1) i strict cresctoare pe (1, +f);
strict cresctoare pe (0, 1) i strict descresctoare pe (1, +f);
injectiv;
(d) constant;
(e) surjectiv.
ax

y
1

a

4. Fie sistemul
cu a R. S se determine a, astfel nct sistemul s fie
x  ay 1  a
compatibil determinat i soluia s satisfac condiia x ! 0, y ! 0.

3. Funcia f
(a)
(b)
(c)

49

(b)  1  a  2  1 ;
(c)  1  2  a  1
(a)  2  a  1  2 ;
(d)  2  a  2  1 ;
(e)  1  2  a  1  2 .

1
(1  xn 1 ) pentru n t 1 , n care x0  1
2
este un numr real fixat. S se studieze monotonia irului i s se afle lim xn .

5. Se consider irul ( xn ) , dat prin relaia xn

nof

(a) ( xn ) cresctor i lim xn


nof

(c) ( xn ) descresctor i lim xn


nof

(b) ( xn ) cresctor i lim xn

2;

nof

1;
2

(d) ( xn ) descresctor i lim xn


nof

(e) ( xn ) cresctor i lim x n


nof

6. Se d funcia f : RoR prin relaia

f ( x)

1;

xm
x2  1

2;

3
.
2

e  x , n care m este un parametru

real. S se afle valoarea lui m pentru care f are extrem n punctul x = 1 i s se precizeze
natura acestui extrem.
(b) m
(a) m 3 , x 1 punct de maxim;
3, x 1 punct de minim;
2
(d) m  1 , x 1 punct de maxim;
(c) m 1 , x 1 punct de maxim;
2
2
1
(e) m
, x 1 punct de minim.
e
7. Valoarea integralei

x dx

( x  1) ( x 2  1)

este:

3
; (b)
2

1
1
1  S ; (c) 1
(S  ln 2) ; (e)
(S  2 ln 2) ; (d)
(S  3 ) .
4
4
2
8
8. Se consider dreptunghiul ABCD unde AB=3BC. Pe latura CD se ia punctul E astfel
c EC{BC , iar AE DB={O}. Se cere msura unghiului DOA.
(c) 45o; (d) arcsin 5 ;
(a) 30o;
(b) arccos 5 ;
(e) 60o.
3
3
9. S se determine soluiile ecuaiei sin10 x  cos10 x 29 cos 4 2 x .
16
S
S
S (kZ) .
S
S
3
S
S
S
(a)
 k ; (b)   k ; (c)
 kS ; (d)  k ; (e) k
8
2
8
4
8
8
8
2
10. O mrgea se obine dintr-un corp sferic sfredelind o gaur de forma unui cilindru
circular drept. tiind c nlimea cilindrului este 6 se cere volumul mrgelei.
(b) 36S ;
(c) 48S ;
(d) 100S ;
(e) 50S .
(a) 24S ;
3
3
(a)

Not. Timp de lucru 3 ore. Fiecare problem are numai una din cele cinci variante corect.

50

Capacitate - teste pregtitoare


Ion SECRIERU 1 i Cezar Marius ROMACU 2
Testul 1
A.
1. tiind c viteza luminii este 300 000 km/s, atunci distana de 1 an-lumin este egal
cu. km.
2. n timpul semestrului I al anului colar 2001-2002, un elev a obinut la matematic
notele 7, 7, 9, 7, 7, 7, iar la tez nota 8. Media la sfritul semestrului este .
36% din 12
3. Valoarea raportului
este ..
12% din 36
4. Un avion pleac de la Londra la ora 540 G.M.T. i ajunge la Bucureti la ora 1200
ora local. Timpul n care a parcurs avionul distana Londra-Bucureti este..
2
2
10
( x  1)  ( y  3)
5. Sistemul
are soluiile.
( x  y )( x  y ) ( x  1) 2  y 2  9
6. Fie punctele A(-2, -2), B(1, 2). Distana AB este.
7. tiind c raza Pmntului este 6400 km, atunci:
a) lungimea Ecuatorului estekm;
b) lungimea paralelei 450 este.km.
8. Un teren are forma unui dreptunghi cu lungimea de 150 m i limea de 50 m.
a) Suprafaa acestui teren este..m2;
b) Costul aratului pentru un hectar este 1 000 000 lei; atunci aratul terenului
cost lei.
9. O piramid patrulater regulat are latura bazei de 12 cm, iar unghiul diedru dintre o
fa lateral i planul bazei este de 60 0.
a) Aria seciunii diagonale a piramidei estecm2;
b) Volumul piramidei este.cm3.
B. 10. La un concurs de matematic, valoarea cumulat a premiilor primilor patru clasai a
fost 3 200 000 lei. Premierea a fost fcut dup cum urmeaz: meniunea primete o optime
din sum, iar premiile III, II i I au fost rspltite direct proporional cu numerele 2,5; 5,
respectiv 6,5. Ce sum a primit fiecare premiant?
11. Fie funciile f, g: R o R, f(x) = ax+b, g(x) = 2 x  4 .
a) Determinai funcia f tiind c graficul acesteia trece prin punctele A(1, 1),
B(0, 2);
b) Reprezentai grafic, folosind acelai sistem de axe, funciile f i g;
c) Determinai aria suprafeei cuprinse ntre graficele celor dou funcii i axa Oy.
12. Fie trapezul isoscel ABCD nscris ntr-un cerc de raz 24. tiind c bazele AB i CD
sunt de o parte i de alta a centrului cercului i subntind unghiuri de 120 0 , respectiv 60 0,
s se calculeze:
a) lungimile bazelor;
b) aria i perimetrul trapezului;
1
2

Profesor, Grupul colar Economic nr. 2 Virgil Madgearu, Iai


Profesor, coala Picioru Lupului, Ciurea (Iai)

51

c) raportul volumelor corpurilor obinute prin rotirea trapezului o dat n jurul


bazei mici, apoi n jurul bazei mari.
Testul 2
A.

1. Numerele naturale 4 x3 y divizibile cu 45 sunt


2. Soluia inecuaiei (x  2)(x  5)  4x  20 d 0 este ..
3. Calculnd (1  3 )( 4  2 3  2 3 ) , obinem..
4. Dac numerele naturale a, b sunt astfel nct

a
b

4
i [a, b] =308, atunci
7

a+b=..
x 2

5. Pentru A= ^x N ; x  1 d 4`; B= ^x R; x  1 ! 3` i C= x Z;
d 0 , avem
x

3

c A B C .
6. Aria lateral a unui cilindru avnd ca seciune axial un ptrat este egal cu aria
lateral a unui con echilater. Raportul volumelor este
7. Un romb are un unghi de 120 0 i nlimea 4 3 cm.
a) Perimetrul rombului este.cm;
b) Latura unui hexagon regulat echivalent cu rombul estecm.
8. ntr-un cub ABCDAcB cC cD c distana de la vrful Ac la mijlocul E al laturii [BC]
este 15 cm.
a) Aria cubului estecm2;
b) Volumul cubului este..cm3.
4
3
53 3
dintr-un numr adugm
din alt numr obinem
;
din primul
9. Dac la
11
5
55 5
13
4
mai mare dect
din al doilea numr.
numr este cu
55
11
y
x
a)

..;
13 53
b) x + y =
a b
b c
B. 10. Se dau proporiile
i
, cu a, b, cZ .
2 3
4 5
a b c
a) S se determine k i p aa nct
;
8 k p
b) Pentru k =12 i p=15, s se determine a, b, c n fiecare din urmtoarele cazuri:
(i) a+b+c=70; (ii) a2+b2+c2=433.
11. Pe laturile [BC] i [AD] ale paralelogramului ABCD se construiesc n exterior
ptratele BCMN i ADPQ. S se demonstreze c:
a) [BP] { [ND];
b) PN, MQ, AC i BD sunt drepte concurente.
12. O piramid patrulater regulat SABCD are toate muchiile de lungimi egale. tiind

c apotema piramidei este de 6 3 cm , s se calculeze:

52

a) aria lateral i volumul piramidei;


b) unghiul diedru format de planele (SAB) i (ABC);
c) distana de la centrul bazei la o faa lateral.
Testul 3
A.

1. Dac

xyz

yz , atunci xyz

2. Pentru n Z, fracia

n2  n  6
Z dac n{.}.
n 1

3 2 2
este .

3 2 2
3 2 2
4. ntre bazele trapezului ABCD exist relaiile ma=10 i mg=5; lungimea
segmentului paralel cu bazele care trece prin punctul de intersecie a diagonalelor este
5. ntr-un paralelogram ABCD (BC>CD), mediatoarea diagonalei [BD] intersecteaz
dreptele AB i CD n E, respectiv F. Patrulaterul BFDE este
6. Lungimea i limea paralelipipedului dreptunghic ABCDAcB cC cD c sunt valorile
x  a x  a a ( 4a  3 x)
absolute ale soluiilor ecuaiei

, a R+ , iar nlimea reprezint
xa xa
x2  a2
jumtate din lungime. Dac M, N sunt mijloacele segmentelor >AD@ , respectiv >AcB c@ ,
atunci d(M, N)=.

3. Rezultatul calcului

3 2 2

7. Seciunea axial a unui con echilater are nlimea de 6 3 cm.


a) Aria total a conului este..cm2;
b) Volumul conului estecm3.

6
Z ,  6 d x d 4 i B= ^x Z  2 d 3x  4 d 13` .
8. Fie mulimile A= x Z
x 1

Atunci A={..} i B={}.


9. Fie mulimile A={-2, 0, 2}i B={-2, 4, 10}. O funcie liniar f: AoB are legea de
compoziie f(x)=. sau f(x)=.
3111
3 291
B. 10.a) S se compare numerele a= 333 i b= 453 .
5
5
n 3
n
b) S se arate c numrul a= 5
2  125 este divizibil cu 45 pentru orice n1

Fie funciile f, g: RoR, f(x) = ax+b, g(x) = cx+d.
a) S se determine f i g tiind c f x  1 3 x  5  f 2 i f 2 x  3 g 2 x  5 ;
b) Determinai valoarea parametrului real m pentru care ecuaia mx2+f(x)=0 are
soluiile egale; aflai aceste soluii.
12. Doi elevi privesc simultan i din aceeai parte un turn. Primul vede vrful turnului
sub un unghi de 45 0, iar al doilea sub un unghi de 30 0. tiind c nlimea copiilor este de

1,50 m iar distana de la turn pn la al doilea elev este 9 3 m, aflai:


a) nlimea turnului;
b) distana dintre cei doi copii.

53

Testul 4
1. S se determine numerele naturale a i b tiind c (a, b)=15 i ab=3150.

2. Aflai elementele mulimii A= abc abc  c N , tiind c azbzcza.

3. Gsii numerele naturale a, b, c tiind c a reprezint 20% din b, suma dintre a i c


este 24, iar b i c sunt direct proporionale cu 5 i 7.
ac
4. Dac a2+b2+c2+14=4a+2b+6c, calculai
.
b

A.

5. Determinai funciile liniare f: RoR cu proprietatea c f(x-2) f(x+2)=x2-2x-3,


 xR.
6. ntr-un triunghi ascuitunghic n care m B este cu 50 0 mai mare dect m C se

construiesc nlimea AD i bisectoarea CE. tiind c m BA D i m BE A sunt direct


proporionale cu 3 i 4, s se calculeze msurile unghiurilor triunghiului ABC.
7. O prism din lemn are ca baz un ptrat cu diagonala 12 2 cm. tiind c masa
prismei este 30,24 kg, aflai masa unui cilindru din acelai material nscris n prisma dat.
8. Un trapez isoscel are aria 85 cm2 i nlimea 5 cm. tiind c diferena bazelor
trapezului este 24 cm, aflai perimetrul su.
9. Un paralelogram are laturile de 10 cm, respectiv 14 cm, iar msura unghiului ascuit
este 60 0. Aflai lungimile nlimilor paralelogramului.
B. 10. Preul unui produs a fost redus de dou ori succesiv, prima oar cu 10% , iar a doua
oar cu 30%, ajungnd n final de 69 300 lei. S se afle:
a) preul iniial al produsului;
b) preul dup dou scumpiri succesive cu 10% i 30%, pornind de la preul final;
c) preul n dolari la un curs al zilei de 32 000 lei, n toate cele trei variante.
( x  y  1) 2  ( x  y )( x  y )  63 2 y ( y  x)

11. S se rezolve sistemul 2 x  y 2 y  x


.

9 y

4
7
12. Apotema bazei unei prisme triunghiulare regulate este 3 , iar diagonala unei fee
laterale este 10 S se calculeze:
a) aria total a prismei;
b) nlimea unui tetraedru regulat ce are o fa echivalent cu o baz a prismei;
c) raportul dintre volumul tetraedrului astfel determinat i volumul prismei.
Testul 5
1. tiind c x i y sunt direct proporionale cu 2 i 3, atunci

3 x 2  xy  2 y 2

............
x 2  xy  3 y 2
2. Dac (a, b)=6, [a, b]=336 i a< 10, atunci a+b=..............
1 1
3. Dac x, y, z sunt invers proporionale cu , i 3, iar 2x-y+3z=9, atunci y+z
5 8
reprezint ............% din x.

A.

54

4. n triunghiul isoscel ABC, [AB] { [AC], m ( BA C ) =360, BC=4, iar BD este


bisectoare cu D (AC). Atunci ABCD=
5. Cercurile C1(O1 , 3) i C2(O2 , 2) sunt tangente exterior, iar tangentele lor comune
exterioare se intersecteaz n P. Atunci PO2 =................
6. Aria unui dreptunghi este 60 cm2, iar lungimea este cu 7cm mai mare dect limea.
Lungimea diagonalei dreptunghiului este .................
7. Un triunghi dreptunghic are o catet de 12 cm, iar suma dintre cealalt catet i
ipotenuz este 24 cm.
a) Lungimea ipotenuzei este.............cm;
b) Aria triunghiului este ............cm2.
1
1
1
1 3 1 3 2  5 2  5


8. Fie A=
, B=
.



2 1
3 2
4 3
2 3 2 3 3 5 3 5
a) Numrul A este un numr...........;
b) Produsul AB este egal cu
2x  m
9. Se d ecuaia mx=
 6 , m R.
2
a) Dac ecuaia are soluia x=3, atunci m=.............;
b) Pentru m=2, ecuaia are soluia x=..............
1
1

y
y  3 y 2  3y  9
x
1
x
x

1

 1 :
B. 10. Fie E1(x)=
.
: ; E2(y)=
1
1
2
y
3y
5
y 3



x 1 x 1 1 x 2
a) Aducei expresiile la forma cea mai simpl;
2
5 E1 ( x)  E 2 ( y ) 4
b) Rezolvai sistemul
.
E ( x)  1 E ( y ) 1
2
1
2
11. Msurile unghiurilor A, B, C ale triunghiului ABC sunt direct proporionale cu
numerele 6,5; 5 i 0,5, iar mediatorea laturii [AC] taie latura [BC] n E. Aflai m BA E .

12. Fie VABC o piramid triunghiular regulat cu nlimea de 6 2 i unghiul diedru


dintre o fa lateral i planul bazei de msur 45 0.
a) Aflai aria total a piramidei;
b) Determinai aria lateral i volumul trunchiului obinut prin secionarea

piramidei printr-un plan paralel cu baza aflat la o distan de 2 2 cm fa de vrf;


c) Aflai raportul dintre volumele piramidei i cel al unui con circular drept n care
poate fi nscris aceasta.

55

Bacalaureat - teste pregtitoare


Gabriel MRANU 1
Testul 1
I.

1. a) Se consider funciile f : A o B, g : B o C unde A, B, C sunt submulimi


ale lui R. S se arate c:
(i) dac g D f este injectiv, atunci f este injectiv;
(ii) dac g D f este surjectiv, atunci g este surjectiv.
b) Fie f : (0, f) o (0, f) o funcie cu proprietatea (f D f )(x )
S se arate c :
(i) f este o funcie bijectiv;
(ii) f (x 2 ) f 2 (x ), x (0, f) ;
(iii)

f (x )

x 2 , x (0, f) .

f ( x ), x (0, f) .

2. Fie irul ( x n ) nt0 definit prin : x 0

0, x n 1
6  xn .
a) S se studieze monotonia i mrginirea irului ( x 2n ) nt0 .
b) S se studieze monotonia i mrginirea irului ( x 2n 1 ) n t0 .
c) S se arate c irul ( x n ) n t0 este convergent i s se calculeze lim x n .
n of

1 0 .
a
b2
a) S se arate c dreapta y mx  n este tangent elipsei dac i numai dac

3. Fie elipsa de ecuaie :

r a 2m2  n 2 .
b) S se determine locul geometric al punctelor din plan din care se pot duce tangente
perpendiculare la elips.
1
1
1
1
a
b
c
x
II. 1. Se consider determinantul '
, unde a, b, c, x R.
a 2 b2 c 2 x 2
a 4 b4 c 4 x 4
a) Dezvoltai determinantul dup coloana a patra punnd rezultatul sub forma unui
polinom p(x).
b) Determinai coeficientul dominant i rdcinile polinomului p(x).
c) Scriei polinomul sub form de produs.
2. Fie funcia continu f : [a, b] o R. S se arate c pentru orice cuplu de numere
reale pozitive D , E se poate determina un numr c [a, b] , astfel nct
D f ( a )  E f (b) (D  E ) f ( c ) .
n

Profesor, Liceul Teoretic Gr. Moisil , Iai

56

III. Fie G

(  k , k ) , unde kR, k>0. Pe G se definete o operaie: (x, y) ox y

k2(x  y)
k2  x y

a) Artai c (G, ) este grup abelian.


b) Artai c funcia

f :G oR

kx
1
este un izomorfism ntre
ln
2k k  x

, f ( x)

grupurile (G, ) i (R, +).

IV. a) Fie f : RoR o funcie continu i periodic de perioad principal T. S se arate c


a  nT

f ( x ) dx

n f ( x ) dx ,  a R i nN.
0

b) Folosind punctul a) , s se calculeze :


1H sin n arccos x
lim
dx .
H o0
1 x2
H ! 0 1H
Testul 2
I.
1. Se d o mulime A care are n elemente. mprim toate submulimile lui A n clase
(disjuncte), punnd n aceeai clas toate submulimile lui A care au acelai numr de
elemente. Care dintre aceste clase este cea mai numeroas ?
2. Fie f : R \ {1, 2}oR, f ( x )

i a n

n2

k! f ( k ) (n  3) .

x  3x  2
k 0
a) S se determine determine derivata de ordin n a lui x.
b) S se calculeze lim a n
n of

3. Se consider punctele A(-2,1), B(2,-1), C(-1,8).


a) S se calculeze perimetrul triunghiului ABC.
b) S se determine coordonatele punctului D , al patrulea vrf al paralelogramului
ABCD , pentru care BC este o diagonal.
c) S se scrie ecuaia cercului circumscris triunghiului ABC.
II.

1. Fie familia de funcii de gradul al doilea :

f m ( x ) (m  1) x 2  m x  1, , MR\{-1}
a) S se arate c parabolele asociate acestor funcii trec prin dou puncte fixe.
b) S se determine locul geometric al punctelor descris de vrfurile parabolelor.
2. S se determine o primitiv a funciei f : [0, 1] o R , f (x )
4

1
0

1 .
ln x

ln( 1  x  1  x ) .

III. Fie polinomul fZ6[X] , f 4 x  2 x  2 x  4 x  3 .


a) S se arate c f nu admite rdcini n Z6.
b) S se arate c f se poate descompune n produsul a dou polinoame , din care unul
de gradul nti.
x ln x
0
IV.

Fie funcia f : D o R , f ( x )

0
1

a) S se determine domeniul maxim de definiie D i apoi s se studieze continuitatea


i derivabilitatea funciei f.
57

b) S se arate c restricia lui f la (0, f ) este bijectiv i apoi s se calculeze


c
s
f 1 (0), f 1 (0) .
c) S se determine primitivele lui f pe R*.
d).S se calculeze aria suprafeei plane determinate de graficul funciei f , axa Ox i
dreptele x=1, x=3.

Testul 3
I.

1. S se rezolve inecuaia : 2 3 x  8  x  3 2 x  8  3 2  x d 0 .
2. Fie xR \ {1} i S ( x ) x  x 2  !  x n .
a) S se calculeze S(x) i S c(x ) .
x  2 x 2  3x 3  !  nx n .

b) Folosind a) s se calculeze T ( x )

c) S se calculeze limita irului (a n ) n t1 cu a n

k 12

II. 1. Fie M mulimea matricilor de tipul (m, n) n care toate elementele sunt egale cu 1
sau (1) i astfel nct produsul numerelor din fiecare linie, respectiv coloan s fie egal
cu (1). S se calculeze numrul de elemente al mulimii M .
2. a) Enunai teorema lui Lagrange.
b) Aplicai teorema lui Lagrange funciei f : [e, S ] o R, f ( x ) ln x .
c) Demonstrai inegalitatea : eS ! S e .
III. 1. Fie G

^f

D : ( b, f ) o R

fD ( x )

b  ( x  b)D , D R*, b ! 0 .

a) S se arate c (G , D) este grup abelian.


b) S se demonstreze c (G , D) # ( R, ) .
2. Fie PZ[X] un polinom pentru care P(0), P(1) sunt impare. S se arate c ecuaia
P(x)=0 nu are rdcini ntregi.
IV. 1. Fie funcia f : I o R , I interval inclus n R aI i funcia f este derivabil n a.
1
2
k

S se calculeze lim n f ( a  )  f ( a  )  " f ( a  )  kf ( a ) , unde kN* este fixat.


n
n
n
n of

2. Fie irul ( a n ) nt1 cu termenul general a n

0 x(1  x )

dx , nN*.

a) S se determine an.
n

b) S se calculeze bn

ak .

k 1

c) S se calculeze lim bn .
n of

Testul 4
I. 1. Fie x1 , x 2 ,!, x n ,! un ir de numere reale nenule. S se arate c acest ir este o
progresie aritmetic , dac i numai dac pentru orice n t 2 , avem relaia :

58

1
1
1

"
x1 x 2 x 2 x 3
x n 1 x n

n 1
.
x1 x n

2. Se consider irul ( x n ) n t0 , x n nS  2n arctgn .


a) S se arate c ( x n ) n t0 este un ir monoton i mrginit.
b) S se calculeze lim x n .
n of

3. S se demonstreze c produsul distanelor unui punct oarecare al hiperbolei


x2
a

y2
b

 1 0 , la cele dou asimptote , este egal cu

a 2b2
a 2  b2

II. 1. Determinai prile stabile finite ale lui Z n raport cu nmulirea. Este R\Q parte
stabil a lui R n raport cu adunarea , respectiv cu nmulirea ?
x 1 t
2. Fie funcia f : [0, x ] o R , f ( x ) ln
dt , x (0, 1) .
0 1 t
a) S se calculeze f c( x ) .
f ( x)
b) S se calculeze lim
.
xo0 arcsin x 2
III. Fie (G , ) i (G, c) dou grupuri, f : G o G c un morfism de grupuri i mulimea

Ker f

{x G f ( x )

e c} , e c fiind elementul neutru al lui G c .

a) S se arate c Kerf este subgrup al lui G.


b) S se arate c morfismul f este injectiv dac i numai dac Kerf={e}.

IV. 1. Fie funcia g : [0, 1] o R , g ( x )

1
1
3
x , x [0,1] \ {1, 2 , ! n }
nN*.

1
1
3x ,
x {1, ,! }
2
n

S se arate c g este integrabil pe [0, 1] i s se calculeze

0 g ( x )dx .

p ln x
, nN*.

x
x
S se calculeze S(m) , aria suprafeei mrginit de graficul funciei f, axa Ox, i dreptele
x=1 , x=m (m >1). S se determine m, astfel nct S ( m ) p ln m  1 / 2 .

2. Se consider funcia f : (0, f) o R, f ( x )

Testul 5
I.

1. Fie irul de numere complexe ( z n ) n t1 definit prin z1 i, z2 i, zn  zn1 q (zn1  zn2)
pentru n t 2 , q fiind un numr complex dat diferit de 1.
a) Calculai z3 i z4.
b) Artai c z n  z n  1

2 iq n  2 i deducei c z n  z1

c) Artai c afirmaiile z n
echivalente.

2i

1  q n 1

.
1 q
i i q este rdcin de ordinul n-1 a unitii sunt

59

2. a) Definii noiunea de punct de inflexiune al unei funcii i dai interpretarea


geometric.
b) S se determine punctele de inflexiune ale funciei : f : RoR, f (x)

x 1  3 x 1 .

x2 y2

1 0 .
4
3
a) S se scrie ecuaiile tangentelor la elips, duse din punctul A(-16, 9).
b) Dac se noteaz cu T1, T2 punctele de contact ale celor dou tangente de la
punctul a) cu elipsa (E) , s se determine ecuaia dreptei T1T2.
c) Calculai distana de la punctul A la dreapta T1T2.

3. Fie elipsa (E) :

II.

1. a) Rezolvai n R inecuatia

1 1 x2
3
! .
x
4
2002

6 1
b) S se gseasc rangul celui mai mare termen din dezvoltarea 
.
7 7
x Q
1,
2. a) Artai c funcia f : RoR, f ( x )
nu admite primitive pe R.
0
,
x

R\Q

b) Gsii o funcie g : RoR care nu admite primitive pe R , astfel nct funcia


compus f D g s admit primitive pe R (f fiind funcia de la punctul a)).

III. 1.Artai c polinomul f x 2  3 x  3 divide polinomul, g ( x  1)3n2  x  2, n N*.


2. Fie a , b Zn, i ecuaia a x b . Artai c:
a) dac (a, n)=1, atunci ecuaia a x

b admite soluie unic.

b) dac (a, n)=d > 1 i d nu divide pe b, atunci ecuaia a x


IV. 1. Fie funcia f : [0, f) o R , f ( x )

x ln x  x ,

0,

b nu are soluii.

x (0, f)
x 0

a) Studiai continuitatea i derivabilitatea lui f pe [0, f)


b) Studiati variatia funciei f i reprezentai graficul ei.
c) Artati c restricia lui f la [e, f) are invers, f- 1, i studiai derivabilitatea lui f- 1 .
d) Calculai f c( e 3 ) i ( f 1 ) cc( 2e 2 ) .
e) Calculati aria suprafeei plane mrginit de graficul funciei f , axa Ox, i dreptele
x

e, x

e2 .
2

2. Fie f : (0, f) o R, f ( x ) e x  1  x 2  x 4 .
a) S se arate c f c( x ) ! 0, x ! 0.
b) S se arate c f ( x ) ! 0, x ! 0.
c) S se demonstreze c :

1 x2

0 e

dx ! 1,43 .

60

PROBLEME SI SOLUTII
Soluiile problemelor propuse n nr. 1/2001
Clasele primare
P.7. Crizantema are cu 38 timbre mai puin dect colega ei, Maria. Cte timbre
trebuie s mai cumpere Crizantema pentru a avea cu cel mult 4 timbre n plus fa de
Maria?
Crizantema Mironeanu, elev, Iai
Soluie. Pentru a o egala pe Maria, Crizantema mai are nevoie de 38 timbre. Pentru a
o depi cu cel mult 4 timbre, ea trebuie s mai cumpere, pe lng cele 38, nc 1,2,3 sau 4
timbre. Deci Crizantema trebuie s mai cumpere 39,40,41 sau 42 timbre.
P.8. S se arate c din numerele 1,2,3,,10 nu se pot forma dou iruri de numere,
cu acelai numr de numere, astfel nct adunnd numerele din fiecare ir s obinem sume
egale.
Maria Mursa, elev, Iai
Soluie. Suma numerelor 1,2,3,,10 este 55. Dac ar exista cele dou iruri de
numere cu sume egale, atunci cele dou sume adunate ar trebui s dea 55. Acest lucru este
imposibil deoarece nu exist dou numere naturale egale care s dea suma 55.
P.9. Aflai vrsta n prezent a tatlui unui biat tiind c biatul are 7 ani, iar atunci
cnd biatul va avea vrsta tatlui, tatl va avea 55 ani.
nv. Elena Marchitan, Iai
Soluie. S figurm cele dou vrste innd cont de relaiile dintre ele. Notm cu t
vrsta tatlui i cu f vrsta fiului.
7
Se observ c vrsta de 55 de ani este format din
f
vrsta fiului i din dublul diferenei dintre vrsta tatlui
t
i a fiului n prezent.
55 ani
1. Care este dublul diferenei dintre cele dou vrste n prezent?
55 7 = 48
2. Care este diferena dintre cele dou vrste n prezent?
48 : 2 = 24
3. Care este vrsta tatlui n prezent?
24 + 7 = 31
R : 31
P. 10. George i-a propus s citeasc n cinci zile o carte ce are 42 file. Numrul
filelor citite n primele trei zile este reprezentat de numere pare consecutive. n a patra i a
cincea zi a citit 12 file. tiind c n ultima zi a citit de dou ori mai mult dect n ziua
precedent, s se afle cte file a citit George n fiecare zi.
nv. Geta Dragnea, Iai
Soluie. S figurm numrul de file citite de George n fiecare zi
I zi
2
a II-a zi
(42-12) file
2
a III-a zi
a IV-a zi
12 file
a V-a zi
1. Cte file a citit n primele trei zile?
42 12 = 30
61

2. Care este triplul numrului de file citit n prima zi?


30 6 = 24
3. Cte file a citit n prima zi?
24 : 3 = 8
4. Cte file a citit n a doua zi?
8 + 2 = 10
5. Cte file a citit n a treia zi?
10 + 2 = 12
6. Cte file a citit n a patra zi?
12 : 3 = 4
7. Cte file a citit n a cincea zi?
4 . 2 =8
R : 8 file, 10 file, 12 file, 4file, 8 file.
P. 11. Ci spectatori au fost asear la Teatrul Naional Vasile Alexandri, din Iai,
dac la balcon au fost 160 de spectatori, la loj un sfert din restul spectatorilor, iar la stal
cu 80 spectatori mai mult dect la loj i balcon mpreun?
nv. Rodica Agrici, Iai
Soluie. S figurm repartiia spectatorilor, n fiind numrul total.
n

balcon

restul spectatorilor
stal
loj
loj

80+160

1. Ct reprezint jumtate din restul spectatorilor?


2. Ci spectatori au stat la loj i stal?
3. Ci spectatori au fost la teatru?

80 + 160 = 240
240 . 2 = 480
480 + 160 = 640
R : 640 spectatori
P.12. Mo Crciun mparte daruri elevilor clasei a IV-a. Dac ar da fiecrui copil
cte 2 pachete, ar rmne n sac 2 pachete. Dac ar oferi fiecrui copil cte 3 pachete, ar
rmne 9 copii fr daruri. Cte pachete are Mo Crciun n sac?
nv. Fnic Dragnea, Iai
Soluie. Figurm cele dou situaii din problem folosind simbolurile E (elev) i P
(pachet) Primul rnd de simboluri sugereaz fiecare elev a primit cte 2 pachete 2 pachete
au rmas nerepartizate.
PP PP
PP PP
PP
Al doilea rnd de simboluri sugereaz
, PP
E E ...............E
E .........
E


c
primii
elevi au primit cte 3 pachete iar
9 elevi
ultimii 9 elevi nu au nici un pachet. Putem
PP PP
PP
PP
PPE .........
PPE
EPPE ......... EPP
E
considera c al treilea pachet a fost oferit

P P
P
9 elevi ,
..........
.....
.........
PP
ultimilor 9 elevi. Acum putem scrie:


1.Ci elevi au primit cte 3 pachete?


2 . 9 + 2 . 1 = 20
9elevi
2.Cte pachete are Mo Crciun?
20 . 3 = 60
R : 60 pachete
P.13. Scriitorul Ion Creang a publicat povestea Capra cu trei iezi n 1875. Se
spune c pe atunci capra ar fi avut o vrst egal cu dublul sumei vrstelor ieziorilor ei,
anii acestora fiind exprimai prin numere naturale consecutive. Peste un an, cnd s-a
abtut necazul asupra caprei, lupul avea vrsta egal cu dublul sumei vrstelor de atunci
ale iezilor, iar toi cinci aveau mpreun 40 ani. Ce vrst avea fiecare n anul publicrii
acestei poveti?
nv. Mihai Agrici, Iai
Soluie. Figurm datele corespunztoare anilor 1875 i 1876.

EE

I1
I2
I3
C

EE E

1
1
1 1 1 1 1 1

62

de 9 ori segmentul I1
i nc 9 ani

1
I1+1
1
I2+1
1
40 ani
I3+1
1 1 1 1 1 1 1
C+1
1 1 1 1 1 1 1 1 1 1 1 1
L+1
Unde I1, I2, I3, C, L reprezint vrstele celor cinci vieuitoare n anul 1875. Analiznd
figurarea corespunztoare anului 1876, putem scrie:
1. Care este numrul segmentelor ce reprezint vrsta I1?
3 . 1+ 2 . 6 = 15
2. Ci ani reprezint aceste segmente?
40 25 = 15
3. Ce vrst avea mezinul?
15 : 15 = 1
4. Ce vrst avea iedul mijlociu?
1+1=2
5. Ce vrst avea iedul cel mare?
2+1=3
6. Ce vrst avea capra?
(1+2+3) . 2 = 12
7. Ce vrst avea lupul?
(2+3+4) . 2 1 = 17
R : 1an, 2ani, 3ani, 12ani, 17ani.
Clasa a V-a
V.16. Un automobilist vede la un moment dat pe kilometrajul de la bord numrul
12921. Dup dou ore de mers cu vitez constant, pe kilometraj a aprut urmtorul
numr care se citete la fel n ambele sensuri. Aflai viteza de deplasare a automobilului.
Gabriel Mranu, Iai
Soluie. Urmtorul numr care se citete la fel n ambele sensuri este 13031. n dou
ore de mers automobilistul a parcurs cu vitez constant distana de 110 km = 13031 km
12921 km, deci viteza sa a fost de 55 km/h.

V.17. S se arate c fiecare termen al irului: 19204, 9012004, 900120004, ... este un
ptrat perfect.
Constantin Chiril, Iai
Soluia 1. Se observ c 9120 = 3022, 9012004 = 30022 etc. n general, se pare c
2
900
.....
.....
.....


01200


04 300


02 . Se verific prin ridicare la ptrat c egalitatea este


p

p 1

p 1

adevrat; ca urmare, numerele din ir sunt ptrate perfecte.


Soluia 2 (Schibinschi Greta, Botoani). Scriem:
2 p4
90
.....
.....
 12 10 p  2  4 9 10 2 p  4  6 10 p  2  6 10 p  2  4

0120

04 9 10
p

p 1

3 10 p  2 (3 10 p  2  2)  2(3 10 p  2  2)

(3 10 p  2  2) 2

2
30
.....

02 .
p 1

V.18. Artai c, dac suma a n numere naturale nenule este un numr prim, atunci
aceste numere sunt prime ntre ele.
Cristiana Artenie, elev, Iai
Soluie. Fie p = a1 +a2 +...+an, p numr prim i d = (a1, a2, ... , an). Atunci d |a1, d |
a 2,
, d | an i deci d | a1+a2 + ... + an, adic d | p. Ca urmare sau d = 1, sau d = p; artm c nu
putem avea d = p i de aici va rezulta concluzia. ntr-adevr, dac d = p, atunci numerele de
date se scriu: a1 pa1c ,..., a n pa cn i vom avea pa1c  pa 2c  ...  pa nc p sau
a1c  a 2c  ...  a cn 1 , ceea ce nu se poate!
63

V.19. Fie a, b, p, q N*. Aflai valorile pe care le poate lua numrul (1+A)B,
unde A a
b
 a

b  a

b
... i B b
a
 b

a  b

a
...

p termeni

q termeni

Cristiana Constanda, elev, Iai


Soluie. Observm c numrul A are valorile 0, a, a + b sau b, dup cum p = M4, p =
M4 + 1, respectiv p = M4 + 3. Ca urmare, 1+A{1,1+a,1+a+b, 1+b}. n mod analog, B
{0, b, a+b, a}. n consecin, avem: (1+A)B {1, (1+a)a, (1+b)b, (1+a)a+b, (1+b)a, (1+b)b,
(1+b)a+b, (1+a+b)0, (1+a+b)b, (1+a+b)a+b}.
V.20. S se aranjeze 12 puncte pe 6 drepte astfel nct pe fiecare dreapt s fie
situate 4 puncte (indicai cel puin dou aranjamente de acest fel).
Andrea Balla, elev, Braov
Soluie. Reproducem trei dintre numeroasele soluii primite de redacie.

Clasa a VI-a
VI.16. Fie a i b dou numere ntregi. Artai echivalena afirmaiilor urmtoare: 1q
1000a+b # 43;
2q a+4b # 43; 3q 11b-8a # 43; 4q 7b-9a # 43.
Gheorghe Costovici, Iai
Soluie. Vom arta c 1q 2q 3q 4q 1q. Avem:
1q 1000a+b = 43k (2343+11)a+b = 43k 2343a+11a+b = 43k

42343a+44a+4b = 443k 42343a+43a+(a+4b) = 443k a+4b # 43;


2q a+4b = 43k -8a32b = -843k (11b8a) 43b = -843k 11b-8a # 43;
3q 11b8a = 43k 99b72a = 943k 43b+56b72a = 943k 43b+8(7b 9a) = 943k 7b9a # 43;
4q 7b9a = 43k 1357b1359a = 13543k 945b1215a = 13543k 946bb
1000a 215a = 13543k 43(22b5a)(1000a+b) =13543k 1000a+b # 43.
VI.17. Fie E = 24n+2 + 34n + 44n + 52n + 62n, n N.
1) Artai c E nu este ptrat perfect.
2) Aflai n astfel nct E # 9.
Cristiana Constanda, elev, Iai
Soluie. 1) Dac n = 0, atunci E = 8 i nu este ptrat perfect. Dac n t 1, atunci ultima
cifr a numrului E este ultima cifr a sumei 4+1+6+5+6 = 22, deci E se termin n 2 i de
aici rezult c nu poate fi ptrat perfect.
2) Dac n = 0, E = 8 nu se divide prin 9. Dac n t 1, atunci 62n # 9 i deci avem:

64

E # 9 24n+2 +44n+52n # 9 2n+223n + 162n + (9-4)2n # 9 2n+2(9-1)n + 22n(9-1)2n + 42n # 9


(-1)n2n+2+22n+2n(9-1)n # 9 (-1)n2n4+2n2n+(-1)n2n # 9 5(-1)n+2n # 9.
Dac n = 6k (k t 1), atunci 5(-1)n+2n # 9 (5+26k) # 9 5+(9-1)2k # 9 (5+1) # 9,
fapt care nu este adevrat. La fel se arat c E nu se divide cu 9 dac n = 6k+1, n = 6k+3
sau n = 6k+4 (k t 0).
Dac n = 6k+2 sau n = 6k+5 (k t 0), atunci E se divide cu 9, deoarece:
5(-1)6k+1+26k+2 # 9 5+4(9-1)2k # 9 (5+4) # 9 etc.
V.18. S se descompun n factori primi numrul S dat de:
S = 123456789 + 234567891 + 345678912 + ... +12345678.
Paraschiva Brsan, Iai
Soluie. Scriind reprezentarea n baza 10 al fiecrui termen din suma S, obinem :
S = (1+2+ ... +9)108 + (1+2+ ... +9)107 + ... + (1+2+ ... +9) =
=45(108+107+ ... +10 + 1) = 45[106(102+10+1) + 103(102+10+1) + (102+10+1)]=
= 45(102+10+1)(106+103+1) = 3251111001001 = 34537333667,
aceasta fiind descompunerea n factori primi a numrului S.
VI.19. S se afle cinci numere raionale tiind c:a) suma lor este 351; b) primele trei
sunt invers proporionale cu primele trei numere prime; c) ultimele trei sunt direct
proporionale cu 7, 11 i 13.
Cristiana Artenie, elev, Iai.
Soluie. Fie a, b, c, d i e cele cinci numere. Condiiile din enun se scriu:
5c
c d
e
a+b+c+d+e = 351, 2a = 3b = 5c i
. Din ultimele relaii deducem c a
,
7 11 13
2
5c
11c
13c
b
, d
, e
i nlocuind n prima egalitate obinem o ecuaie cu necunoscuta
3
7
7
c etc.
VI.20. Fie a, b N i c Q direct proporionale cu p1, p2, p3, unde p1 < p2 < p3 sunt
numere prime.
a) Artai c c N*.
b) Determinai p1, p2, p3 dac a+b < 35 = c.
Gheorghe Iurea, Iai
a
b
c
a
deducem ap2 = bp1 i c
p 3 . Din prima egalitate
Soluie. a) Din
p1 p 2
p3
p1

rezult p1 | ap2 i deci p1 | a, adic

a
p1

k N*. Folosind a doua egalitate, obinem c

N*.

b) Avem: a+b < 35 = c k (p1+ p2) < 35 = kp3 35 = kp3 i p1+ p2 < p3 p3{5,
7} i p1+p2 < p3. Dac p3 = 3, atunci p1 = 2 i p2 = 3 (deoarece p1 < p2 < p3) i nu avem p1+p2
< p3. Rmne p3=7, caz n care inegalitatea p1+p2 < p3 este ndeplinit numai de numerele
prime p1 = 2, p2 = 3.
Clasa a VII-a
VII.16. S se cerceteze care dintre elementele mulimii A={(x,y)QuQ; 4x2+12x+9 +
2
|y -25|=0} aparin graficului funciei f(x) = -2x+2, xR.
65

Cristiana Constanda, elev, Iai


Soluie. Avem: A ={(x,y)QuQ; (2x+3)2+|y2-25| = 0} = {(x,y)QuQ; 2x+3 = 0 i y2-

25 = 0} = {(x,y)QuQ; x = 

3
2

i y = r5 }= ( - , - 5), (- , 5) . Cum f( 

) = 5, numai

al doilea punct al lui A aparine graficului lui f.


VII.17. a) Fie x, y, z [2, +). Artai c (x2+y)(y2+z)(z2+x) t 27xyz.
b) Fie x, y, z [3, +). Artai c (x2+y)(y2+z)(z2+x) t 64xyz.
Lucian Tuescu, Craiova
Mai general, pentru orice nN fixat are loc inegalitatea:
(x2+y)(y2+z)(z2+x) t (n+1)3xyz,  x, y, z [n, +).
(1)
(pentru n = 2 i n = 3 se obin inegalitile din enun).
Soluia 1 (n maniera autorului). Cum x t n x2 t nx x2+y t nx+y i cum nx+y =
x++x+y t (n+1) n 1 x n y , obinem x2+y t (n+1) n 1 x n y . Analog, y2+z t
t(n+1) n 1 y n z , yn+x t (n+1) n 1 z n x . Prin nmulirea membru cu membru a ultimelor trei

inegaliti se obine (1).


Soluia 2 (n maniera mai multor elevi din Braov). Are loc x2+y t (n+1)x,  x, y
[n, +) cci x2 + y t (n+1)x x2 (n+1)x + n + (y-n) t 0 (x-1)(x-n) + (y-n) t 0.
nmulind membru cu membru inegalitatea x2 + y t (n+1)x cu analoagele ei, obinem (1).
VII.18.

se

determine

numerele

reale

pentru

expresia

x 2  y 2  ( x  a ) 2  ( y  a ) 2 + ( x  b) 2  ( y  a ) 2  x 2  ( y  c) 2 (a,b,c R+) este minim

i s se afle apoi aceast valoare minim.

Cristiana Artenie, elev, Iai


Soluie. n raport cu un sistem Oxz de axe coordonate figurm punctele O(0,0),
A(-a,a), B(b,a), C(0,c) i M(x,y). Deoarece
( x  b) 2  ( y  a ) 2 =MB i

x 2  y 2 = MO,

( x  a ) 2  ( y  a ) 2 = MA,

x 2  ( y  c) 2 =MC,

problema revine la determinarea minimului


expresiei E = MO+MA+MB+MC, atunci cnd
M este un punct oarecare din plan.
Avem inegalitile: MO + MC t OC
('MOC), MA+MB t AB ('MAB), deci E t t
AB+OC, cu egalitate pentru M (OC)
(AB), adic M coincide cu P(0,a). Ca urmare,
Emin se obine pentru x = 0, y = a i are valoarea Emin = 2a+b+|a c|.
VII.19. Se consider un triunghi dreptunghic isoscel ABC cu vrful n A i se noteaz
cu E i F punctele de intersecie ale cercurilor C(C,
punctele B, E i F sunt coliniare.

66

3
1
BC) i C(A, BC). S se arate c
4
4

Adrian Zanoschi, Iai

Soluie. Fie AB = AC = a. Avem: AE = =AF


3BC
3a 2
BC a 2
=
=
i CE = CF =
=
. Se
4
4
4
4
observ de aici c
2a 2
18a 2
AE2 + AC2 =
 a2 =
=EC2, deci 'AEC
16
16
este dreptunghic n A, adic AE A AC , de unde
rezult c EAB. Analog se arat c FAB, deci
B, E, F sunt coliniare.
VII.20. S se mpart cu ajutorul unui echer negradat un segment [AB] n trei pri
de lungimi egale.
Constatin Cocea, Iai
Soluie. Vezi E. Cohal - Construcii geometrice cu echerul, p.41 din acest
numr.
Clasa a VIII-a
VIII.16. Fie nN* i An= x1 x 2 ...x n . Definim funcia d: An u AnoN prin d(x, y) =

Card i / i 1, n , x i z y i , unde x = x1 x 2 ...x n i y = y1 y 2 ... y n .


S se arate c d este o distan, adic satisface condiiile: 1) d(x, y) = 0 x = y; 2)
d(x,y) = d(y,x),  x,y An; 3) d(x,y) d d(x,z)+d(z,y),  x,y,z An.
Petru Asaftei, Iai
Soluie. Verificm pe rnd 1), 2) i 3):
1. d(x, y) = 0 x1 = y1, x2 = y2, ... , xn = yn x = y.
2. d(x, y) = d(y, x),  x,yAn, cci perechile de numere (x, y) i (y, x) au aceleai cifre
distincte.
3. S presupunem c d(x, y) = k = Card{i1, i2, , ik}(deci xi1 z y i1 , , xin z y in i xj =
yj pentru j{i1, ..., ik}) i fie z = z1 , z 2 , ... , z n . Fie p numrul indicilor din {i1, ..., ik} pentru
care coincid cifrele lui z i x i fie q numrul acelora pentru care coincid cifrele lui z i y.
Evident, avem p+qd k. Deoarece z i1 xi1 z i1 z y i1 i z i1 y i1 z i1 z xi1 etc., rezult c
d(x, z) t k-q i d(z, y) t k-p. Ca urmare, d(x, z) + d(z, y) t 2k- (p+q) t k = d(x, y), adic este
verificat 3).

VIII.17. Artai c D(0,1), tiind c numrul real D este o soluie a ecuaiei


2x5 + x3 1= 0.
Dumitru Neagu, Iai
Soluia 1. Avem: 2D5 + D3 1 = 0 D3(2D2+1) = 1 D3 =

1
2D 2  1

0 < D3 < 1

0 < D < 1 (s-a utilizat faptul c D z 0, zero nefiind soluie a ecuaiei date).
Soluia 2. Dac x d 0, atunci 2x5+x31 d -1 < 0, deci x d 0 nu este soluie a ecuaiei
67

date. Dac x t 1, atunci 2x5 +x3 1 t 2 + 1 1 > 0, deci x t 1 nu este soluie a ecuaiei date.
Cum numrul D este soluie a acestei ecuaii, rezult c D (0,1).
VIII.18. Dac x,y [0,1], atunci avem:
x+ yd 1  1  x 2 1  1  y 2  1  1  x 2 1  1  y 2 .

Constantin Cocea, Iai


Soluie. Ridicnd inegalitatea dat la ptrat (avem voie!), obinem: x2+y2+2xy d

(1+ 1  y 2 - 1  x 2 - (1  x 2 )(1  y 2 ) ) + (1- 1  y 2 + 1  x 2 - (1  x 2 )(1  y 2 ) )+2xy,


echivalent cu (1-x2) + (1-y2) t 2 (1  x 2 )(1  y 2 ) , adevrat conform inegalitii mediilor.
VIII.19. S se determine numerele naturale n t 2 tiind c mulimea

x 2  x 1
N {1,2,..., n} este format dintr-un singur element.
x N /
n

Cristinel Mortici, Constana


Soluie. Fie n t 2 un numr cu proprietatea dorit i fie x {1, 2, , n} unicul numr
pentru care n | x2+x+1. Cum n+1-x {1, 2, , n} i n | (n+1-x)2 (n +1-x)-1 (dup cum se
vede din egalitatea (n+1-x)2 (n+1-x)-1= n(n-2x+1) + (x2x+1), din unicitatea lui x rezult
n 1
i atunci n este impar: n = 2k+1, x = k+1. n consecin, n | x2
c n+1-x=x, deci x =
2
x1 2k+1 | (k+1)2 (k+1)1 2k + 1 | k2+k1 2k + 1 | 4k2+4k4 2k+1 | (2k+1)2 5
2k + 1 | 5 k {1, 5}. Se verific direct c numai n = 5 satisface cerinele problemei.
VIII.20. Fie cubul ABCD A cB cC cD c cu O c centrul feei A cB cC cD c . Calculai
tangenta unghiului dintre B O c i A cD .
Cristiana Constanda, elev, Iai
Soluie. Dublnd cubul ca n figur se obine paralelipipedul EFCDE cF cC cD c , n
care F cB A cD (ambele paralele cu E cA ). Observm c <( A cD , O cB ) = <( F cB , O cB ) =
=< F cB O c =D.

Pentru

= O ' B '2  B ' B 2

a 6
2

laturile

F cB O c

triunghiului

unde
= 1

10a 2
6a 2
a 6
= 2a2 +
-2 a 2
cosD, de
4
4
2
3
.
6

cosD

3
36

33
i deci tgD =
6

Ca

urmare,

sin

11 .

68

F cB =a 2 ,

a 10
(unde p este mijlocul
2

i O cF c = O ' P 2  F ' P 2

muchiei B cC c ) .
Aplicnd teorema cosinusului n ' F cB O c ,
obinem:

avem:

O cB =

Clasa a IX-a
IX.16. n ipoteza c ecuaia cu coeficieni reali x 3  ax 2  bx  c 0 are rdcinile
x1 , x 2 , x 3 reale, s se demonstreze echivalena:
x i ! 0 , i 1, 2, 3 , a  0 , b ! 0 , c  0 , ab  c .
Adrian Corduneanu, Iai
Soluie. Conform relaiilor lui Vite, avem:
x1  x 2  x 3  a , x1 x 2  x 2 x 3  x 3 x1

b , x1 x 2 x 3

c

(1)

Ecuaia din enun se poate scrie i sub forma ( x  b)( x  a) ab  c (2).


Dac x i ! 0 , i 1, 2, 3 , atunci din (1) rezult c a  0 , b t 0 i c  0 . Tot din (1) se
poate deduce c x1  a  (x 2  x 3 )  0 . De aici i din egalitatea obinut punnd x1 n
locul lui x n (2) deducem c ab  c  0 .
Reciproc, s presupunem c sunt ndeplinite condiiile din enun privind coeficienii.
Din (1) avem x1 x 2 x 3  c ! 0 i atunci nu toate numerele x1 , x 2 , x 3 pot fi negative. Fie
x1 ! 0 ; rezult c are loc i x 2 x 3 ! 0 (3). Pe de alt parte, din ab  c i din (2) cu x 1 n
locul lui x , se obine c x1  a  0 . ns x1  a  (x 2  x 3 ) , deci x 2  x 3 ! 0 (4). Din
(3) i (4) decurge c x 2 ! 0 i x 3 ! 0 .

IX.17. Artai c ecuaia {x 2 }  { y 2 } {z 2 } are o infinitate de soluii n Q \ Z ({a}


reprezint partea fracionar a numrului real a).
R. Brbulescu i M.B. Ion, elevi, Lucian Tuescu, prof. , Craiova
Soluie (dat de un grup de elevi din Braov). Scriem egalitatea 3 2  4 2 5 2 sub
2

5
4
3
forma: n  n
n , n N * .
7
7
7
2
3 3 2 4 2 4 2 5 2 5 2
Deoarece: n n , n n , n n , n N * ,
7 7
7 7
7 7
3 4 5
urmeaz c tripleta x, y, z = n , n , n este o soluie a ecuaiei din enun n N * .
7 7 7

IX.18. Determinai funciile

f: R o R pentru care avem: f(x 3 + 3x 2 + 3x) d x d

f 3 (x) + 3f 2 (x) + 3f(x) x R. Generalizare.


Gabriel Popa, Iai
Soluie. Vom rezolva direct urmtoarea problem mai general:
Dac M: R o R este o funcie surjectiv i strict cresctoare, s se determine funciile
f: R o R pentru care: f( M x d x d M f x , x R. (1)

Din ipotez, M este inversabil i avem M  x t x M 1 t Atunci, innd seama i


de monotonia lui M, au loc:
69

1

(1) f M x d x, x R f t d M t , t R f x d M 1 x d f x , x R f
1

x d M f x , x R

x d f x , x R

M 1 .

n cazul particular considerat, M  x x 3 x 2 x  x 3   i este evident strict


cresctoare i surjectiv, deci f(x) = M 1 (x)= 3 x  1  1 , x R.
IX.19. Artai c n orice triunghi are loc inegalitatea: (a8 +b8 + c8 +3) R4 t 8r2a2b2c2.
Mihai Bogdan Ion, elev, Craiova
Soluie. Se tie c n orice triunghi au loc R t 2r i a 4  b 4  c 4 t 16S 2 , cu egalitate
pentru triunghiul echilateral. Atunci:
a8 1 b8 1 c8 1 4
R t 2 a 8  b 8  c 8 R 4
a 8  b 8  c 8  3 R 4 2


2

2
2

2 a 4  b 4  c 4 R 4 t 2 16 S 2 R 4 t 2

a 2b 2c 2

R 2 4 r 2 8r 2 a 2 b 2 c 2 .
R2
IX.20. Pe laturile AB i AC ale tringhiului ABC se consider punctele M, respectiv N,
astfel nct BM { CN = k (constant). Dac B i C sunt fixe i m A este constant, s se

afle locul geometric al mijlocului segmentului MN.


Ctlin Calistru, Iai
Soluie. Fie P, Q, R, S respectiv mijloacele
segmentelor [MN], [BC], [BN], [CM]. Laturile
patrulaterului PRQS sunt linii mijlocii n triunghiurile
NMB, BNC, CMB, respectiv MNC i cum BM { CN
=k, rezult c PRQS este romb de latur k . n plus,
2

unghiurile acestui romb au msuri constante (D,


respctiv S-D), deci indiferent de poziia punctului A,
diagonala PQ a rombului are lungime constant

QP

k cos

D . Cum Q este fix, rezult c P descrie


2

un arc de cerc de centru Q i raz r

k cos

D , delimitat de dreptele CA i CA
1
2

2
corespunztoare poziiilor limit ale punctului A pe arcul capabil de unghi D (BAtk, CAtk).
Dac A descrie arcul simetric din semiplanul inferior, P parcurge un alt arc de cerc,
simetricul primului fa de BC.
Clasa a X-a
X.16. S se determine numrul funciilor f : ^1,2, ! n` o ^ 1,1` cu proprietatea c
n

f(i) = k, k, n N , n t 2, k < n.

i 1

70

Petru Asaftei, Iai


Soluie. Fie X {1,2,...,n} mulimea acelor elemente a cror imagine prin f este 1.
n

Evident c X z , altfel

f i

n ! k . Notm x

card X . Atunci

i 1

f i f (i)  f ( j)
i X

i 1

 x  n  x 2 x

nk x

jX

nk
.
2

Dac n i k au pariti diferite, nu exist funcii ca n enun. Pentru n i k de aceeai


paritate, funciile cutate sunt bine determinate de submulimea X; numrul lor va fi deci
nk

egal cu numrul submulimilor cu n  k elemente ale mulimii ^1,2,...,n` , adic C n 2 .


2
X.17. Fie f : R o 0, f definit prin :
x
f(x) = a , x Q

b x , x R \ Q

a,b N\{0,1}, a z b.

Notm cu Ma,b mulimea acestor funcii i cu I = { f Ma,b | f injectiv }, S ={ f Ma,b |


f surjectiv }.
1) I z , S z i I = S.
2) dac (a, b) = 1, atunci f nu-i nici injectiv i nici surjectiv.
3) este adevarat reciproca afirmaiei de la punctual 2)?
Dumitru Gherman, Pacani

Soluie. Vom demonstra c f surjectiv f injectiv  r Q * astfel nct b a r .


a) f surjectiv f injectiv. ntr-adevr, s presupunem prin absurd c f nu este
injectiv; atunci x1 Q, i x 2 R\Q astfel nct f(x1) = f(x2), de unde a x1 b x2 , adic
x1 x 2 log a b , deci log a b R\Q, prin urmare ecuaia f x b nu are soluie, ceea ce
contrazice faptul c f este surjectiv.
b) f injectiv r Q* astfel nct b a r . Vom proceda tot prin reducere la
i, ca urmare,
absurd;
presupunem c b z a r , r Q*. Atunci log b a Q
f log b a b logb a
*

f 1 . Cum f injectiv, urmeaz c log b a 1 , imposibil.

c) r Q astfel nct b

a r f surjectiv. Pentru y 0, f , considerm ecuaia

f(x) y . Dac log a y Q, atunci x1

log a y este soluie a acestei ecuaii; n caz

1
log a y R \ Q este soluie a ecuaiei.
r
Acum cerinele problemei sunt imediate. Pentru a=2, b=4, avem f I iar I=S
conform primei echivalene dovedite. Punctul 2) rezult din ultima echivalen, iar
reciproca sa este fals: de exemplu, f I pentru a=2 i b=6, ns ( 2,6 ) z 1.

contrar, x

71

X.18. Fie ABCD un patrulater convex, orientat pozitiv, ale crui vrfuri au afixele a,
b, c, d .S se arate c ABCD este ptrat dac i numai dac d  a i (b  a) i a+c=b+d.
Adrian Corduneanu, Iai
Soluie. Prin translaia z o z - a , obinem patrulaterul A cB cC cD c congruent cu
ABCD, unde Ac 0 , B c b  a , C c c  a , D c d  a . Avem: ABCD ptrat A cB cC cD c
ptrat A cB cC cD c paralelogram i OD c se obine din OB c printr-o rotaie de unghi
sens direct c  a

(b  a)  (d  a). i d  a

i(b  a) a+c=b+d i. d  a

S n
2

i(b  a) .

X.19. Fie x, y, z, a, b, c C. Notm = xa + yb + zc, = xb + yc + za i = xc + ya


+ zb. S se arate c dac numerele complexe , , sunt afixele vrfurilor unui triunghi
echilateral, atunci cel puin unul din tripletele (x, y, z) i (a, b, c) reprezint afixele
vrfurilor unui triunghi echilateral.
Constantin Cocea, Iai
Soluie. Concluzia problemei se obine imediat folosind urmtoarele observaii :
2

(i) u, w, z sunt afixele vrfurilor unui triunghi echilateral dac i numai dac u +
2

w + z uw uz wz = 0;
(ii) Are loc relaia (ce se poate verifica printr-un calcul de rutin):
D 2  E 2  J 2  DE  DJ  EJ x 2  y 2  z 2  xy  xy  yz a 2  b 2  c 2  ab  bc  ac .

X.20. Stabilii natura triunghiului n care au loc simultan relaiile:


= 2 2 cos A + 2, 2 cos B  cos C 1 = 2 2 cos B + 2, 2 cos C  cos A 1 = 2 2 cosC + 2.
Neculai Roman, Mirceti (Iai)

cos A  cos B 1

Soluie. Cu notaiile x 2cos A , y = 2 cos B i z = 2 cos C , ipoteza problemei se scrie


2xy x 2  2 , 2 yz y 2  2 , 2zx z 2  2 . Avem: 2xy d x 2  y 2 i analoagele i atunci
se obine c 2 d y 2 , 2 d z 2 , 2 d x 2 . Deci, 1 d 2 cos B , 1 d 2 cos C , 1 d 2 cos A , de unde
m( A ) d 60, m( B ) d 60, m( C ) d 60. Deoarece m() + m( B ) + m() = 180,
inegalitile precedente se transform n egaliti, adic triunghiul ABC este echilateral.

Clasa a XI-a
XI.16. Fie numerele a, b C i matricele A, B M n (C) astfel nct aAB  bBA I n ,
In fiind matricea unitate de ordin n t 1 . S se demonstreze c det AB  BA 0 sau exist

Z U n

{z C | z n

1} nct a  bZ

0.

Dan Popescu, Suceava


Soluie. Din ipotez obinem c a AB  BA  a  b BA I n , deci are loc egalitatea
a AB  BA I n  a  b BA . Analog  b AB  BA I n  a  b AB . Pe de alt parte,
det I n  tAB det I n  tBA t C, cci matricele AB i BA au acelai polinom caracteristic. Rezult c det a AB  BA det  b AB  BA . Presupunnd c det( AB  BA) z 0,

72

urmeaz c a n

(b) n , relaie care mpreun cu ipoteza asigur c ab

i atunci  a b

z 0 (deci b z 0)

1 , ceea ce ncheie demonstraia.

XI.17. Fie 0  x1  x 2 i D , E 0,1 . S se arate c irul x n n t1 dat de

x 2n 1

x D2 n 1 x 12nD , x 2 n  2

x 2En x 12nE1 , n t 1 , este convergent.

Gheorghe Costovici, Iai


Soluie. Pornind de la observaia c 0  x1  x 3  x 4  x 2 , se arat prin inducie
matematic faptul c 0  x 2n -1  x 2n 1  x 2n  2  x 2n , n t 1 . Atunci subirul x 2n -1 n t1

este cresctor i mrginit superior de x2 ,deci este convergent la l1, subirul x 2n n t1 este
descresctor i mrginit inferior de x1, deci este convergent la l2 i n plus l 1 d l 2 . Trecnd
la limit n relaiile de recuren, obinem c l 1 = l 1D l 21-D , deci l11D = l 21D , adic l1= l2,
aadar x n n t1 este un ir convergent.
99

XI.18. S se demonstreze inegalitatea

arcsin k 100

k 1

1999 99

.
S7
Lucian-Georges Ldunc, Iai

Soluie. Plecnd de la inegalitatea cunoscut sin x  x, x (0,1] i folosind monotonia

funciei arcsin, rezult c x < arcsin x, x (0,1 ), de unde e arcsin x ! e x , x 0,1@ . Pe de


x2
x2
, x ! 0 , deci e arcsin x ! 1  x 
, x
2
2
(0,1) . Facem, pe rnd, x 1 100, 2 100 , ! , 99 100 i summ inegalitile obinute. Se

alt parte, se arat imediat c e x ! 1  x 

99

deduce c

k 1

e arcsin k 100 ! 99 

1
100

99

k 1

k

1
2 100 2

99

k
k 1

99 1999 99 1999
.
!
1200
S7

XI.19. Determinai funciile f: R o R continue n x0 = 1 i care verific relaia:


f ( a 2 x  a 2  1)  2 f (ax  a  1)  f ( x)
unde a>1 este un numr real dat.

( a  1) 2 ( x  1) , x R,

Dumitru - Dominic Bucescu, Iai


Soluie. Cu substituia x 1 t , ecuaia funcional dat devine:
f ( a 2 t  1)  2 f ( at  1)  f (t ) (a  1) 2 t , t R
Evident c funcia identic 1R verific (1); fie f o alt soluie. Avem
( f  1R )(a 2 t  1)  2( f  1R )(at  1)  ( f  1R )(t ) 0, t R ,
echivalent
2

(1)
cu

g(a t )  2 g (at )  g (t ) 0 t R, unde funcia g: R o R, g (t ) ( f  1R )(t  1) este


continu n t 0 0 . Definim h : R o R , h t g at  g t , funcie continu n t 0 0 ,
cu proprietaile: h 0 0 , h at  h t 0 , t R. Inductiv, obinem

73

1
h t h t
a

i fcnd n o f , rezult c h t

1
1
h 2 t ! h n t , t R, t N*
a
a
1
lim h n t
a

nof

h lim n t
o
f
n
a

h 0 0 , t R, deci

g at  g t 0 , t R. Relund raionamentul anterior obinem c g t g lim n t


o
f
n
a

h 0 c , deci g 1R  c c R . Reciproc, aceste funcii verific evident relaia (1).

XI.20. ntr-un plan dat se consider punctele fixe A, A' i punctul mobil P. Se noteaz
cu P' proiecia punctului P pe mediatoarea segmentului A A' i cu P" simetricul lui P fa
de dreapta A A'. S se afle locul geometric al punctului P tiind c dreapta P'P" este
paralel cu dreapta lui Euler a triunghiului PAA'.
Paraschiva Brsan, Iai
Soluie. Fie ^G` = PO P'P", unde O este mijlocul lui AA'. Din ' OGP' ~ ' PGP",
OG OP c 1
obinem
, deci G este centrul de greutate al ' PAA'. Cum dreapta lui Euler
GP PP c c 2
conine punctul G, rezult din ipotez c aceasta esta chiar P'P". Deoarece centrul cercului
circumscris ' AA' P este intersecia dreptei lui Euler cu o mediatoare, urmeaz c P' este
punctul din plan egal deprtat de P, A, A'. Atunci P'P 2=P'A2=P'O 2+OA 2. Raportnd
planul la un reper ortogonal cu originea n O i avnd pe AA' drept ax a absciselor, fie
x, y coordonatele lui P. Relaia precedent conduce la x 2  y 2 a 2 , unde AA'=a. Deci
locul geometric al punctului P este hiperbola echilater de semiaxe a i avnd ca axe de
simetrie dreptele AA' i mediatoarea segmentului [AA'].
Clasa a XII-a
XII.16. Cercetai dac exist funcii continue f : R* R* pentru care
F (2x  1 / x ) 2F (x ) F (1 / x ), x R ,
unde F este o primitiv a funciei f.
Gabriel Popa, Iai
Soluie. Pentru nceput, s observm c relaia din enun are sens, ntruct
2x  1 / x z 0 x R * , adic putem defini F (2x  1 / x ) . Cum F este derivabil, derivnd
membru cu membru relaia dat, se obine: (2 1/ x 2 )f (2x  1/ x )
2f (x )F (1/ x )  (2 / x 2 )F (x ) f (1/ x ),x R*. Lund n aceast egalitatea x 1 R*, obinem

c f (3)

0 Im f , deci nu exist funcii f cu proprietile cerute.

XII.17. Dac funcia f : RR* este continu, impar i periodic de perioad


T

principal T, calculai I

f ( f ( x)  kx)dx, k Z .
0

Dumitru Gherman, Pacani


Soluie. Cu schimbarea de variabil x=T- u obinem:

74

f ( f (T  u )  k (T  u ))du

XII.18. Calculai I

T
0

T
0

f ( f (u )  ku  kT )du

f ( f (u )  ku )du

x 2000
2668

1

dx i J

I I

T
0

f ( f (u )  ku )du

0.

x 666
2668

dx , x (0, f) .
1
Lucian-Georges Ldunc, Iai

Soluie.
I J

x 2000  x 666
x

2668

1

x 666  1 / x 668

x 13341 / x

dx

dx
1334

2
x
arctg
1334
c
x 667  1 / x 667

667

c
1
x 667  1 / x 667
dx
667 x 667  1 / x 667 2  2

1 / x
2

667

C .

x 667  1 / x 667  2
1
2
dx
ln
C .
667 x 667  1 / x 667 2  2
2668 x 667  1 / x 667  2
Formnd un sistem din relaiile obinute, aflm valorile lui I i J.
I J

...

XII.19. Dac T > 0 este perioad pentru funcia continu f : [0, f) R i


g : [0,T] R este continu, s se demonstreze relaia:
T
T

1 T
lim g (x )f (nx )dx
f (x )dx g (x )dx .

n of
T
0
0
0

Dan Popescu, Suceava


Soluie. A se vedea nota D. Popescu i F. Popovici O generalizare a lemei lui
Riemann , n acest numr al revistei, p.12.
n1

XII.20.

Fie (G,) un grup comutativ de ordin n.

S se arate c: (ai )n2


i 1

n 1

(ai )n 2
i 1

e , unde e, a1 , a 2 , ! a n 1 sunt elementele grupului G, e- elementul neutru.


Cristian Frsinaru, Iai
n
Soluie. Deoarece G n , rezult c ai e, i 1, n  1 (fie din teorema lui
Lagrange, fie cf. pb. R-4, cap.III, 3 din manualul n vigoare n cazul comutativ), deci
n 1

( a i ) n

e (1). Aplicaia f : G o G , f ( x)

x 1 , x G , este injectiv i cum G este

i 1

finit, rezult c f este bijectiv. Atunci: {e , a1 ,..., a n 1 } G

{e 1 , a11 ,..., a n11 } i deci


n 1

are loc egalitatea e a 1.... a n 1

e 1 a11 ... a n11 , ceea ce implic ( a i ) 2


i 1

(1) i (2), concluzia problemei urmeaz imediat.

75

e (2). Din

Probleme propuse 1
Clasele primare
P.24. Aflai numerele a, b, c, d tiind c verific n acelai timp urmtoarele egaliti:
a + 3 = b, b + 3 = c, c + 3 = d, a + 3 = 10.
( Clasa I )
nv. Maria Racu, Iai
P.25. Un elev din clasa I, fixnd un numr din irul numerelor naturale, constat c
suma numerelor din faa lui nu este mai mic dect 55, iar suma aceasta adunat cu
numrul fixat nu depete pe 66. Despre ce numr este vorba?
( Clasa I )
Luminia Popa, elev, Iai
P.26. Pe trei borcane de compot, unul de ciree, altul de viine i al treilea cu amestec
de ciree i viine, toate etichetele au fost puse greit. Scond un singur fruct dintr-un
singur borcan, determinai coninutul fiecruia.
( Clasa a II-a )
***
P.27. S se scrie numrul 31 folosind cele patru operaii aritmetice i numai cifra 3
(se cer cel puin dou soluii).
( Clasa a II-a )
Andrea Balla, elev, Braov
P.28. Cte pagini are o carte dac pentru paginarea ei s-a folosit cifra 9 de 117 ori?
( Clasa a III-a )
Crizantema Mironeanu, elev, Iai
P.29. Ioana i Alina au cules mpreun 165 de nuci. Ioana a cules mai puine nuci
dect Alina; ea face un calcul i observ c triplul diferenei dintre numrul nucilor culese
de ele reprezint tocmai numrul nucilor culese de Alina. Cte nuci a cules fiecare fat?
( Clasa a III-a)
nv. Maria Racu, Iai
P.30. Artai c dintre oricare patru numere naturale diferite, mai mici dect 1000000,
se pot alege dou a cror diferen s se mpart exact la 3.
( Clasa a IV-a )
Roxana Bolocan, elev, Iai
P.31. O veveri descoper un alun ncrcat cu fructe i i face provizii pentru iarn
transportnd la scorbura sa alternativ: o dat dou alune, o dat trei alune. Dup ce
transport 47 de alune, face o pauz pentru a se odihni. S se calculeze ce distan a parcurs
veveria n total, dac de la alun la scorbura ei este o distan de x hm x dam x m, unde x are
ca valoare cel mai mic numr natural posibil.
( Clasa a IV-a )
nv. Mihai Agrici, Iai
P.32. Un printe i mparte averea astfel: la primul copil 10 milioane plus o cincime
din rest, la al doilea copil 20 de milioane plus o cincime din noul rest, la al treilea 30 de
milioane plus o cincime din noul rest i aa mai departe. S se afle suma mprit de
printe, precum i numrul copiilor, tiind c toi au moteniri egale.
( Clasa a IV-a )
Mihai Grtan, Iai

Clasa aV-a

a0b
c0d
V.26. S se determine cifrele distincte i
e*f
nenule a, b, c, d, e, f, g pentru care rezultatul
g**
nmulirii alturate este cel mai mare posibil:
g***f
Ioan Scleanu, Hrlu
V.27. Trei apicultori au tras mpreun 700 kg miere de albine. Cnd au mprit
mierea, primul apicultor a luat jumtate, al doilea jumtate din rest, al treilea jumtate din
1

Se primesc soluii pn la data de 15. 01. 2003.

76

noul rest, apoi operaiunea se repet pn se mparte toat mierea. S se afle ct miere a
luat fiecare.
Ctlin-Cristian Budeanu, Iai
2001
2001
2002
2002
i N 2 3
sunt numere divizibile cu 5.
V.28. Artai c N1 3
2
2
Dorina Carapanu, Iai
V.29. S se afle numerele abc pentru care abc ac b 2 .
Romana Ghi i Ioan Ghi, Blaj
V.30. Dac xi, i= 1,500 , sunt numere naturale nedivizibile cu 5, atunci numrul
N

2000
este divizibil cu 5.
4x 14  8x 28  12x 312  ...  2000x 500

Tamara Culac, Iai

Clasa aVI-a
VI.26. Fie A 4a  6b  c, B 4a  3b  c, C
(A, B)=23, artai c (A, B, C)=23.

3a  11b  28c , unde a,b,c Z . Dac

Cristiana Constanda, elev, Iai


VI.27. S se rezolve n Z sistemul
3x  2 y d 8;

x  y d 1; 3x  1 1. .
Mihai Crciun, Pacani

VI.28. S se rezolve n N ecuaia


1 2  2 3  n (n  1)  (n  1)  ( n  2)  ...  2n 2  4  ...  2n .
Dumitru - Dominic Bucescu, Iai
VI.29. n triunghiul ascuitunghic ABC, bisectoarea interioar a unghiului B
intersecteaz nalimea din A n E. Fie F (DC astfel nct AE=EF. Artai c BE A AF.
Tamara Culac, Iai
VI.30. Pe ipotenuza (BC) a triunghiului dreptunghic ABC se consider punctele N i
M astfel nct BN=AB, CM=AC. Dac P i Q sunt proieciile punctelor M i N pe dreptele
AN, respectiv AM, demonstrai c segmentele (MP), (NQ) i (PQ) se pot constitui n laturile
unui triunghi.
Ctlin Calistru, Iai

Clasa a VII-a
VII.26. Determinai a Q tiind c

a 2 

2 Q.
Gheorghe Iurea, Iai

VII.27. Determinai a R astfel nct sistemul

a2
x 2,
4
s admit numai soluii ntregi.
x12  (a  1)x1 

...

, x n21  (a 1)x n 1 

a2
4

x n , x n2  (a 1)x n 

a2
4

x1

Ctlin Calistru, Iai


VII.28. Fie zece numere naturale nenule care au suma egal cu 55. S se arate c
printre ele exist trei care pot fi lungimile laturilor unui triunghi.
Adrian Zanoschi, Iai
VII.29. Fie ABCD un ptrat, O centrul su, iar M i P mijloacele segmentelor (OA),

77

respectiv (CD). S se arate c triunghiul BMP este dreptunghic isoscel.258


Constantin Cocea i Dumitru Neagu, Iai
VII.30. Fie ABCD un ptrat de latur 1 i punctele M (AD), N (BC),
4
4
1
i AM BN d .
{P}=BM AN. Dac SDCNPM = , demonstrai c 1  AM  BN d
2
3
9
Emil Vasile, Ploieti

Clasa aVIII-a
VIII.26. Demonstrai c ecuaia (t 2  1) x 2  4t 2 x  4t 2  t
n ZuZ.

0 are numai dou soluii

Mihai Crciun, Pacani


5
VIII.27. Determinai x,y Z pentru care fracia 2
este echivalent cu
2
19
x  xy  y
(n legtur cu E: 9314* din G.M. 11-12/1987).
Gabriel Popa, Iai
VIII.28. Fie a,b numere naturale de pariti diferite. Aflai valorile lui n pentru care
Sn=an+ an-1b + an-2b2 ++ abn-1 + bn este divizibil cu a+b.
Mihaela Predescu, Piteti
VIII.29. Se consider piramida triunghiular regulat VABC cu latura bazei a, iar
3a
.
muchia lateral 2a. Fie M mijlocul lui (VA), iar N un punct pe (VB) astfel nct VN=
4
Aflai distana de la V la planul (MNC).
Adrian Corduneanu, Iai
x y

VIII.30. Fie A, B, C, D patru puncte necoplanare astfel nct AB

4 73 ,

CD 4 29 . Notm cu E, F mijloacele segmentelor (AB), respectiv (CD). S se arate c


mijloacele segmentelor (AF), (BF), (CE), (DE) sunt vrfurile unui paralelogram i s se

calculeze aria acestuia tiind c are o latur de lungime 194 .


Romana Ghi i Ioan Ghi, Blaj

Clasa a IX-a
a
a
^ x`  . Discuie.
x
^ x`
> @
D. M. Btineu-Giurgiu, Bucureti
IX.27. S se determine funciile f, g : > 0, f o > 0, f , unde g este surjectiv i

IX.26. Dac a 0, f , s se rezolve ecuaia

aditiv i g y  g f x

> x@ 

f x  g y ,  x , y > 0, f .

Ioan Scleanu, Hrlu


IX.28. S se determine funciile f : R o R pentru care ( f D f D ! D f ) x x  D ,



n ori

D R fixat, iar funcia g

f  1R este monoton.

Mihail Bencze, Braov


IX.29. S se arate c n orice triunghi ABC are loc inegalitatea

78

l a3 l b3 l c3 3R


d
p ( p 3  3abc) .
ha hb hc 2r
Viorel Cornea i Dan tefan Marinescu, Hunedoara
IX.30. n patrulaterul ABCD considerm punctele R i S pe diagonala BD, n
interioarele triunghiurilor ABC, respectiv ACD. Notm ^M ` CR AB , ^N ` AR BC ,

^P`

AM 2

AS CD i ^Q` CS AD . tiind c

arate c

AM n
MB n

BM n

MC n

CP n
PD n

DQ n
QA n

MB 2

BM 2
MC 2

CP 2
PD 2

DQ 2
QA 2

4 , s se

4 , n N .

Ctlin Calistru, Iai

Clasa a X-a
X.26. Fie ecuaia x 4  S1 x 3  Sx 2  mx  m  1 0 , unde S este aria unui triunghi
neechilateral ABC , iar S1 este aria triunghiului A1 B1C1 determinat de punctele de
intersecie ale bisectoarelor interioare cu cercul circumscris triunghiului ABC . S se
determine m R tiind c ecuaia admite un numr impar de rdcini n 0,1 .
X.27. Fie r >1, f , D

^z C; z d r`

Dumitru Gherman, Pacani


i P C>X @ , P X aX 2  bX  c . S se

arate c dac P z D , z D , atunci a, b, c D .


D.M. Btineu-Giurgiu, Bucureti
2

X.28. Rezolvai ecuaia z (2

 1)  z (2

z 1

 1)  1 0 , z C\R.

Emil Vasile, Ploieti


X.29. Un motan scoate cu ajutorul unui pahar un numr de petiori dintr-un acvariu.
Ci petiori trebuie s conin acvariul astfel nct motanul s aib matematic sperana c
va scoate 5 dintre ei?
Gabriel Popa, Iai
X.30. Fie M ^1,2,..., n` . S se afle numrul de k-uple A1 , A2 ,..., Ak de submulimi
k

ale lui M astfel nct

*
i 1

Ai

M i Card (

A

l , l d n fixat.

i 1

Lucian-Georges Ldunc, Iai

Clasa a XI-a
XI.26. Fie A M n R . Dac Tr (t A A  (t A )* A * ) 2n det A , atunci t A A * .
Iuliana Georgescu i Paul Georgescu, Iai
XI.27. Fie a >0,1 i x n n t0 un ir de numere reale astfel nct

Artai c irul x n nt0

x n2 d a max x n21 , x n2  x n21 , n N * .


2

este convergent i determinai limita sa.

Aurel Muntean, Sibiu

79

XI.28. S se determine p R pentru care limita irului a n n t1 definit prin termenul


n

general a n

k 1

np

este finit i nenul.


2

k  k 1

Constantin Chiril, Iai


XI.29. S se arate c

1
1
n n
lim
1   .....   1 1 .

n of ln ln n
2
n

Marian Tetiva, Brlad


XI.30. Fie f : R o R o funcie discontinu i care are proprietatea lui Darboux. Dac
exist o funcie g : R u R o R astfel nct f x  y g f x , y , pentru orice x, y R ,
atunci funcia f nu are limit la f .
tefan Alexe, Piteti

Clasa a XII-a
1  a 2

2
a

a 2

XII.26. Se consider mulimea M

 a 2

 a 2 ; a A , unde A = Z

a2
1 a 2
a 2

sau A = Q sau A = R. Artai c M , este grup; este acesta izomorf cu A* , ?


Gheorghe Costovici, Iai
XII.27. Fie G , un grup cu Z G z ^e` i H un subgrup netrivial al lui G . S se
demonstreze c exist x, y G \ H , x z y 1 , astfel nct xy H i yx H . Dai exemplu
de grup care nu are aceast proprietate.
Ovidiu Munteanu, student, Braov
S

XII.28. Calculai n tgx dx, x 0, , pentru n ^2,3,4` .


2
Daniel Jinga, Piteti
XII.29. Fie f : R o R o funcie continu i t ! 0 . Pentru a, b ! 0 , s se calculeze

lim

n of

nt

k 1

XII.30. S se arate c

k t  k  1 t

x2

ln 1  x

e 1  x
0

2 2

f n t k a k  1 b

t
a b

Mihail Bencze, Braov

eS
S
.
dx ln 2,
16
8

Cristian Moan, Craiova

80

   
  

  





  


   
   
     

 

! 


  
 
  
  
 
 

   

  


 

  



" # + " #
  
   


  

   
" +
 !

 "   # $%
' $ 
 %
    %!  '
  


  

 
  (
  
    

 % %  %!   

(
  
    

 % %  %!
) 
  


     %

 *      


 

 
   %
 
   
  +
 

  
    
  
   ,


      
! 

 
  )  
 
# $%
() -
 
! '   
+

(( .! '  

(1

  
(3


  

(. $   
 
(0

+

+

+  +



 

+  +

= + 

*+ , #  -

= /00/ 

  $  #

[ ] + [  ] +  + [ ] =

# $%


/ 

+# $%



  # %  #  
 %

 

+ + $ # $%
$ 

1  

 ,2 
 ,2 ')    



  ! ,

2
 

2  # $%
$ 

  "  
3

   0   + / +

   4   +  

 4

2  
 
 # # $%
( 5 
   


 ,2   2 
   )
  6   + 

,2  7   * 
!

2
 " /- 
# %

( 5 


 '1 

  ,   
 
  % 

     

  '    
 %  
     

  ' 


$ 
    )

  /"

#

  

  $ 


   



=
%  
      3      


2  / !5 7

# $%
7
( 5 

 ,2

 8 -
   
    %
   )  %    
       

,2   9




{  } =   %

 ,2    
  !
   %   
  ' 

{ } =     
  

8 
   # " %
(' 5   

 ,2  2 

 

  % 

  
 "   %
! ! ,   


   ')

   
  ! 
 $
 "  "

 

    
 
  " + "  + " " =  
  ,  ,     
 
,   "9 
# $%
.) $ 



  ,
 


   '
 


2 




 
  

2

')


  % 
  ')

 
  

  ! 
2
 %

 $ 
     


 
 
 ,  '  

 
 

 "


   '
2
 

2+  # $%

()

" 

 
< 5

+  +

#

= 

 = :;

  =




=



 


"

+




 
"


+  +



"

+





+


4
"  # # 2  
     %   
 
< <
 ,2 
7 4   

{ } =    {#} =    
     % #  
   >
       
$ 
> 
# ?@ 
< 5
 ,2  %  ?     (  ) = {    }    ( ) =

()

  


+
+
4
 

   4 
# *  %  A$%B
% $ % ( ' & (  )  ( ' & 
<' 5 

 ,2  ,2
() + ') + &) = ( ( + ' + &)    )   
 ,
 
 ,2  

  
! 
 


  
 ,2     2 
 * 
!

   %  
# $%
1  
,
 
   


<()  5  @    $ 





   
  

  
3 {    }    (  ) = {    }  $ 


#


 

 
,
 
   


   

  


 
 
,
           
    

/ 

+# $%
<(( $ 
! '    
 " = "  + A0 

  $  # # $%
<(. 5 


 '1+ 
: ;3B  C    
 (  
   +  (  +
C    -

! 
" (   
 (
   " +   "  +  "   
,


 
* +,  *D " 
C
2
 % 
5 7
* 
 # ?
/  
+ + - . +  
<(0 5  4[E] . = . + .
 >  %
 
    ! ' %

   

<(1 -

 



 $ 


+ ( )

>

. +

)
     +



  
    

   
 
 

     %    
' ( ] 
,   "9 
# $%
  


<(3 5 4 =  % ( $ ) 
)    

={ +$ }

)

+ + * # 2  /  %
. +





. + 

 (.) =


  

!
  
 


  
 
 ,

 
  

%
 '
, 
 

   
$ 
 



  $ 

 %  

%

 # $%

< % %  = {  ( [  ] [  ] +     F }  
 1 / 
<(  5
/ ( )  [  % ] % 
      

 

  
 1
/$ [ + % / ( $ ) ] = $( + % $ ) 
5 7
 D #  % 


 ! ' % / ( [  ) [  )    
'  

/   $ [  % ]   )

  

<( 5 

/ ( ) =  %
/

= /

/ (

/

/  

( )

 /    

<( 5   (  ) 

(

[  ) 



 


 

 




+ 0)   (  ) 

  0 {
 $ 




> 
% 
# 2  
  )  + 0    '
 %

 }

 + 1    '
 

  1  % (  + 1)   (  ) +

#"

) = {  + 1+ 1 2 }   ,
 


  =   






  
%   
    ,

   ) 
* 
$
 #  % 
,
 ,
  
(   ) 
888   
 
3
=
888   
 
3 =  
%  3    

< 
!  </
# $%
<.) 5 4  >   $  
  / ( [ ] 4   
 
'   $

 
  4 ( [ ] 4  4 ( ) = / ( )     
 
  
/ (")
/

5" 

"
*  2+ -# "  % 
<(' 5 3

( )

<$>, *E*"4$<F4 8$<$<E$ $5$  >>*


6  

 

4# > I>J J 


48 *.?*I.K P 
A R?DISJ *2
,2
A R.> 5?. ? 
A T<.$> -
2'
A" T<.$> R   
A0 T<.$> D

A/ 9.->U 

A4 D?.I> >
AA U9   >  
A# 9 ?K D2  >

A8 -Q*S?U D

# -QVJKU M
 
# 9IMQ9?U9 D
,

# 9IMQ9?U9


#" D?SQ9SJ M  
#0 $?D?. 9 
#/ $?D?. 9
 
#4 -MQ?U9 9
 
#A S?M$ D2 
## $U$>U *
#8 *JI.*J$9U ? 
@

Q '   


 

 H %
 H 6
6 K  LM   9 N ? %
6 Q  R
  LM     
N ? %
6 Q  R
  LJ  . ' N ? %
6 Q  R
  LJ  . ' N ? %
6 Q  R
  L*
 ?
  N ? %
6 U '
  R2  L*2  2N ? %
6 U '
  L  ? 9 ! N ? %
6 Q  R
  L*
 ?
  N ? %
6 Q  R
  LJ  . ' N ? %
6 Q   
 LI ' T  N ? %
6 *
 K 
J 
,  ? %
6 *
 K 
J 
,  ? %
6 K  LR D 
 N ? %
6 K  LR D 
 N ? %
6 K  LR D 
 N ? %
6 9 , >  L9 >,
!!N ? %
6 K 
  ? %
6 Q  R
  L  ? 9 ! N ? %
6 ?   

 ? %
6 Q  R
  L*
 D  N ? %
6 Q  R
  L*
 D  N ? %




#0

Pagina rezolvitorilor
BRAOV
coala general nr.5. Clasa a V-a. POSTEUC Raluca: P(12,21), V(21-23);
POSTEUC Bogdan: P(12,21), V(21-23).
Liceul Teoretic N. Titulescu. Clasa a IX-a. BORO Olivia-Dana: VII.16, VIII(17,
23,24), IX.17; BOSCORNEA Ionu Bogdan: VII.16, VIII(17,23,24), IX.17; BURSUCANU
Lucian: VII.16, VIII(17,23,24), IX.17; CENU Nicoleta Veronica: VII.16, VIII(17,23,24),
IX.17; EDU Vladimir: VII.16, VIII(17,23,24), IX.17; GHEORGHE Liana Elena: VII.16,
VIII(17,23,24), IX.17; HALIPPA Andra: VII.16, VIII(17,23,24), IX.17; LITR Anatolie:
VII.16, VIII(17,23,24), IX.17; MOARCS Liviu George: VII.16, VIII(17,23,24), IX.17;
ONCIOIU Cristina: VII.16, VIII(17,23,24), IX.17; PAN Andreea: VII.16, VIII(17,23,24),
IX.17; PASCAL Andreea: VII.16, VIII(17,23,24), IX.17; PLATON Alexandru: VII.16,
VIII(17,23,24), IX.17; RCHITEANU Lavinia: VII.16, VIII(17,23,24), IX.17; RUSEN
Adrian Nicolae: VII.16, VIII(17,23,24), IX.17; OIMU Adelina: VII.16, VIII(17,23,24),
IX.17; TUDOR Ioana: VII.16, VIII(17,23,24), IX.17.Clasa a X-a. BRAOVEANU Adina:
VIII.24, IX.23, X(20,22-24); CHELMEA Andreea: VIII(17,24), IX.23,X(22,23); CHI
Cristina: VIII.24, IX.23, X(20,22-24); CURCAN Diana: VIII.24, IX.23, X(20,22-24);
DINU Cristina: VIII.24, IX.23, X(20,22-24); DIOR Mihaela: VIII(17,24), IX.23,
X(22,23); ELEKE Alexandra: VIII(17,24), IX.23, X(22,23); EPURE Alexandru: VIII.24,
IX.23, X(20,22-24); EZARU Lucian: VIII.24, IX.23, X(20,22-24); GAVRIL Ramona:
VIII(17,24), IX.23, X(22,23); HOOLEANU Cristina: VIII.24, IX.23, X(20,22-24);
JEGAN Alina: VIII(17,24), IX.23, X(22,23); LUNGU Cezar: VIII(17,24), IX.23, X(22,23);
NEAGOE Lcrmioara: VIII(17,24), IX.23, X(22,23); OANCEA Vlad: VIII.24, IX.24,
X(20,22,23); OSTAFI Andrei: VIII.24, IX.23, X(20,22-24); PAHONU Radu: VIII(17,24),
IX.23, X(22,23); PETRIORU Doris-Georgiana: VIII.24, IX.23, X(20,22-24); PUCAU
Andreea: VIII.24, IX.23, X(20,22-24); PUCOIU Lucian: VIII(17,24), IX.23, X(22,23);
SAVEI Ionela: VIII(17,24), IX(17,23), X(22,23); TODORU Anca: VIII.24, IX.23,
X(20,22-24); UIC Tatiana: VIII(17,24), IX.23, X(22,23); VRREANU Andrei:
VIII.24, IX.23, X(22-24). Clasa a XII-a. BCLEA Timea: X.24, XI(22,23,25), XII(20,24):
CPRARU Andrei: X(20,24), XI(22,25), XII.20; CERCELEA Rare: X.24, XI(22,25),
XII(20,24); DOBO Claudiu: X(22-24), XI(22,23,25), XII.24; DUMITRU Andrei:
X(20,24), XI(22,23,25), XII(20,24); FERSTOARU Valentin: X(20,23), XI(22,23,25),
XII(20,24); FORSEA Adrian: X(22-24), XI(22,23,25), XII(20,24); GHERGHE Mihai:
X(20,24), XI(22,25), XII.20; IORDACHE Florin: X(20,23,24), XI(22,23,25), XII(20,24);
MUOIU Rare: X(23,24), XI(22,23,25), XII(20,24); NSTAS Gabriel: X(22,23),
XI(22,25), XII.20; ROCA Silviu: X(20,24), XI(22,23,25), XII(20,24); ROTOPNESCU
Roxana: X(20,22-24), XI.25, XII.20; AGU Marian: X(23,24), XI(22,23,25), XII(20,24);
RU Ana-Maria: X(20,22-24), XI(22,25), XII.20.

CRAIOVA
Colegiul Na. Fraii Buzeti. Clasa a V-a. TUESCU Anca-tefania:V(2125),VI.22.

IAI
coala nr. 3 Alexandru Vlahu. Clasa a IV-a (nv. GALIA Gabriela). CHITIC
Andreea: P(14-18,20); CIOBANU Diana Luciana: P(14-18,20); SOFICU Corina-Maria:
P(14-17,19,20); VRABIE Corina-Roxana: P(14-16,19,20).
coala nr. 7 Nicolae Tonitza. Clasa a IV-a ( nv. MELINTE Rodica). ANDRONIC
85

Mihaela: P(14, 16-23); DUMITRIU Alexandru: P(14, 16-19).Clasa a IV-a (nv. TUDOSE
Elena). BOTEZ Alin: P(14-23); CHILCO Claudia: P(14-23); CHIRIAC Alexandra:P(1423); CIUBOTARIU Remus: P(14-18,21); IONESCU Mihai-Bogdan: P(14-21,23); ZORICI
Iulian: P(14-18,20-23). Clasa a IV-a (nv. PANAINTE Adriana). CURC Bogdan P(1418,21); ONU Adina: P(14,15,17,19-21); SRGHE Nicoleta: P(14,15,17, 19-21);
TRUFANDA Alexandru: P(14-17,20,21).
coala nr. 11 Otilia Cazimir. Clasa a III-a. (nv. PRIAL D-tru). TIBA Marius:
P(14-22).
coala nr.17 Ion Creang Clasa a VI-a BDI Vasilic Valentin: V(16, 19-21,
23),VI.17; CASIAN Manuela: P(9,12), V(16,18) VI.17; IFTODE Andreea P(9,12),
V(16,18,20,21); LUPU Andreea-Mdlina: P(9,12), V(20,21), VI.17; SOFRONEA
Gabriela: P.9, V(20,21), VI(16,17); TANANA Irina-Eliza: P(9,12), V(16,19,21) VI.17.
coala nr. 22 B.P. Hadeu Clasa a IV-a (nv. TRZIORU Iuliana). BARAN LigiaMaria P(14-23); BLUA Bogdan-Alexandru: P(14-23); CHIHAIA Mihai: P(14-18, 2023); MACOVEI Smaranda-Teona: P(14-23); PRODAN Andreea: P(14-23); RAI
Bogdan: P(14-23); SILION Ctlina: P(14-23); SPNU Drago: P(14-23).
coala nr. 23 Titu Maiorescu .Clasa a II-a (nv. CHIRIL Beatrice).
TUDORACHE Alexandru-Gabriel: P(7-11, 14-18, 20,21).
coala nr. 43 Dimitrie Sturdza. Clasa a IV-a (nv. OBREJA Rodica). OPRIC
Adelina: P(14,16,17,21,22)
Colegiul Naional C. Negruzzi. Clasa a IX-a. ARTENIE Cristiana: VII(16,22),
VIII(17, 18, 20-24), IX(16,18,22,24).
Liceul Teoretic Gr. Moisil Clasa a VII-a. COSTIN Ciprian : V(21-23, 25), VI(2123,25), VII(22,23,25), G(2,3). Clasa a XII-a. ALECSANDRESCU Ioana: XI(21-24),
XII.24.
Liceul Teoretic Garabet Ibrileanu Clasa a V-a . BABIUC Sanda: P(9,12,13,21),
V(16,20,21,23); BUDEANU tefana: P(9,13,22), V(16,20,21); FUIOREA Bogdan:
P(9,12,21-23), V.16; GAVRILESCU Ioana: P(10,19,20,22), V.16; TURCHIN Ariadna:
P(9,12,13,21-23), V(20,21); UNGUREANU Drago: P(9,10,13,21,22), V(20,21).
Clasa a VI-a. PETRUC Magda: P(9,12,13), V(19,21). Clasa a VII-a. CHIRVASE
Ariadna : V.23, VI(22,23,25), VII.22; JUVERDEANU George: V(21,23), VI(22,23,25),
VII.23; PRUTEANU Irina: : V.23, VI(22,25), VII(22,23); STANCIU Filip: V(16, 20, 21,
23), VI(18,19,22).
Liceul Teoretic M. Eminescu. Clasa a VII-a. AVRAM Mircea: V(21-23,25),
VI(23,25), VII.22; CIUCANU Radu: V(21-23,25), VI.25; DNIL MIHAI: V(21-23),
VI(22,23,25), VIII.23; EANU Ruxandra: V(16,21,23,25), VI.19, VIII.23; GRAMSCHI
Raluca: V(21,23,25), VI(19,25), VIII.23; TOFAN Andrei: V(21-23,25), VI(23,25);
TUDOSE tefan: V(16,20,21,23,25), VI.25; TURLIUC Clin Rare: V(22-25), VI(21,25);
Clasa a VIII-a. BUZDUGA Alexandru: VI(21,25), VIII(21,24,25).

Redacia revistei Recreaii matematice acord cte un premiu n cri elevilor:


COSTIN Ciprian (Lic. Teoretic Gr. Moisil): 1/2001(8pb), 2/2001(8pb), 1/2002(12pb).
CIUCANU Radu (Lic. M. Eminescu): 1/2001(8pb), 2/2001(6pb), 1/2002(5pb).
DNIL Mihai (Lic. M. Eminescu): 1/2001(5pb), 2/2001(7pb), 1/2002(7pb).
TOFAN Andrei (Lic. M. Eminescu): 1/2001(6pb), 2/2001(8pb), 1/2002(6pb).
TURLIUC Clin Rare (Lic. M. Eminescu): 1/2001(6pb), 2/2001(6pb), 1/2002(6pb).
86

Revista RECREAII MATEMATICE apare de dou ori pe an


(la datele de 1 martie i 1 septembrie) i se adreseaz elevilor, profesorilor,
studenilor i tuturor celor pasionai de matematicile elementare.
n atenia tuturor colaboratorilor
Materialele trimise redaciei spre publicare (note i articole,
chestiuni de metodic, probleme propuse etc.) trebuie prezentate ngrijit,
clar i concis; ele trebuie s prezinte interes pentru un cerc ct mai larg de
cititori. Se recomand ca textele s nu depeasc patru pagini. Evident, ele
trebuie s fie originale i s nu fi aprut sau s fi fost trimise spre
publicare altor reviste.
Problemele originale destinate rubricii Probleme propuse vor fi
redactate pe foi separate cte una pe fiecare foaie, cu enun i
demonstraie/rezolvare, fiind nsoite de numele autorului, coala i
localitatea unde lucreaz/nva.
Redacia revistei va decide asupra oportunitii publicrii
materialelor primite.
n atenia elevilor
Numele elevilor care vor trimite redaciei soluii corecte la exerciiile
i problemele din rubrica Probleme propuse vor fi menionate n Pagina
rezolvitorilor. Elevii vor ine seama de urmtoarele reguli:
1. Pot trimite soluii la minimum cinci probleme propuse n
numrul prezent i cel anterior al revistei; pe o foaie va fi redactat
soluia unei singure probleme.
2. Elevii din clasele VI-XII au dreptul s trimit soluii la
problemele propuse pentru clasa lor, pentru orice clas mai mare, din dou
clase mai mici i imediat anterioare. Elevii din clasa a V-a pot trimite
soluii la problemele propuse pentru clasela a IV-a, a V-a i orice clas mai
mare, iar elevii claselor I-IV pot trimite soluii la problemele propuse pentru
oricare din clasele primare i orice clas mai mare. Orice elev poate trimite
soluii la problemele de concurs (de tip G i L).
3. Vor fi menionate urmtoarele date personale: numele i
prenumele, clasa, coala i localitatea.
4. Plicul cu probleme rezolvate se va trimite prin pot (sau va fi
adus direct) la adresa Redaciei:
Prof. dr. Temistocle Brsan
Catedra de Matematic
Universitatea Tehnic Gh. Asachi Iai
Bulevardul Carol I nr. 11, 6600, Iai
(pentru Recreaii Matematice)
E-mail: tbi@math.tuiasi.ro

CUPRINS
Alin Spum

.1

Petre Osmatescu (1925 2001).2


Academicianul Petru Soltan la 70 ani..3
Niels Henrik Abel 200 de ani de la natere4
ARTICOLE I NOTE MATEMATICE
P. MINU Asupra ipotezei lui Goldbach.5
C. COCEA Generalizarea teoremei de omologie a lui Barbilian.7
I. GEORGESCU, P. GEORGESCU Asupra unor iruri de integrale...9
D. POPESCU, F. POPOVICI O generalizare a lemei lui Riemann.12
M.TETIVA Cteva relaii metrice deduse vectorial....14
D.-T. MARINESCU, I. ERDEAN Inegaliti geometrice. Aplicaii...17
D. POPESCU Asupra unei clase de iruri recurente....20
T. BRSAN Un criteriu de concuren a dreptelor...23
I. SCLEANU Cteva consecine ale unei relaii a lui Gergonne....25
GH. COSTOVICI Unele iruri monotone cu limita e sau e-1 ..28
C.-T. POPA Studiu comparativ privind cteva medii uzuale.31
NOTA ELEVULUI
C. ANDREI, T. RUSU Aplicaii ale monotoniei mediilor n raport
cu ordinul lor..33
CHESTIUNI METODICE
D. GLEAT, G. POPA Funcii care admit / nu admit primitive..36
CHESTIUNI COMPLEMENTARE MANUALELOR
E. COHAL Construcii geometrice cu echerul....41
DIN ISTORIA MATEMATICII
S. IONESEI Teorema celor patru culori..43
MATEMATICA N CLASELE PRIMARE
P. ASAFTEI Introducerea operaiei de adunare la clasa I ..46
CONCURSURI I EXAMENE
Concurs de admitere 2001, Iai...48
Capacitate teste pregtitoare (I. SECRIERU, C.M. ROMACU)...51
Bacalaureat teste pregtitoare (G. MRANU)...56
PROBLEME I SOLUII
Soluiile problemelor propuse n nr.1/2001....61
Probleme propuse..76
Probleme pentru pregtirea concursurilor...81
Pagina rezolvitorilor...85

Recreatii stiintifice - cea nt


ai brazd
a
La 15 ianuarie 1883 apare la Iasi primul numar al revistei "Recreatii stiintifice", revista
ce va dura timp de sase ani, cu cte un numar pe luna.
Obiectivele urmarite, racordate la cerintele din acele timpuri ale nvatamntului, sunt
"de o n
telepciune ce face cinste fondatorilor revistei" [6] si apar expuse cu deosebita
claritate n "Catra cetitori" - cuvtul de nceput din 15 ianuarie 1883:

Aparitia revistei "Recreatii stiintifice" este strns legata de conditiile istorice din acea
epoca. Unirea Principatelor si Proclamarea Independentei Romniei, ct si reformele
1

structurale din timpul domniei lui Al. I. Cuza si apoi a regelui Carol I au creat cadrul
politic si legislativ al formarii statului romn modern si afirmarii acestuia. Invatamntul
romnesc, ntocmai ca si societatea romneasca n general, a trecut prin mari prefaceri si
framntari: nfiintarea celor doua universitati din Iasi si Bucuresti, reforma nvatamntului
din 1964, multele regulamente menite sa organizeze reteaua de scoli si sa stabileasca
programele acestora etc.
Pe de alta parte, licentiatii romni ai universitatilor apusene (din Franta, Germania
sau Italia) odata reveniti n tara realizau imediat faptul ca, n climatul existent, greu ar
fi putut ntreprinde cercetari originale proprii. Acestora le revenea obligatia cu mult mai
importanta pentru acel moment de a contribui la edificarea nvatamntului romnesc,
de a tine lectii si a elabora manuale si cursuri n limba romna, de a pregati generatia
viitoare ce urma sa faca pasul catre creatia stiintifica originala.
Fondatorii reviste sunt: N. Culianu, C. Climescu si I. Melik - profesori la Facultatea
de stiinte din Iasi, G. I. Lucescu si V. Paladi - profesori de matematica la Liceul National
din Iasi, G. I. Rosiu si I. D. Rallet - profesori de matematica la S coala Militara din Iasi,
G. Zarifopol - profesor de fizica si chimie la S coala Militara din Iasi, I. V. Praja - profesor
de matematica la S coala Normala "Vasile Lupu" din Iasi si I. M. Dospinescu - profesor de
matematica la Gimnaziul "
Stefan cel Mare" din Iasi.
Este prima revista din tara cu profil stiin
tific, materialele publicate acoperind diversele
ramuri ale stiintei: matematica, fizica, chimie, mineralogie, geografie, astronomie, cosmografie etc. Se adreseaza cu precadere elevilor din scolile secundare, studentilor si
profesorilor.
Majoritatea fondatorilor erau licentiati n matematica sau profesau aceasta disciplina;
se explica astfel faptul ca "Recrea
tiile stiin
tifice" au un con
tinut predominant matematic;
multe numere de revista au un continut exclusiv matematic.
Un numar are n medie 25 pagini; doar n anii II, V si VI numerele 7 si 8 (de vacanta)
au fost tiparite mpreuna, ca o singura revista de 40 pagini. Structura de baza a unui
numar este: articole, probleme rezolvate si probleme propuse. Date fiind conditiile n
care a aparut, materialele publicate n paginile revistei, articole si probleme, erau preluate sau erau prelucrari din tratatele si revistele de circulatie din acea vreme. Articolele
de matematica vizeaza diversele ei ramuri: aritmetica, algebra, geometrie (sintetica,
analitica, descriptiva si diferentiala), mecanica teoretica si astronomie, matematica actuariala, istoria matematicii, problemele nvatamntului matematic. Problemele propuse au
trezit un viu interes printre elevii din Iasi si din toata tara; plecati la studii n strainatate,
unii dintre fostii colaboratori ai revistei au continuat sa trimita solutii. Pentru stimularea
elevilor, redactia publica solutiile corecte primite si mentioneaza numele tuturor celor care
au dat-o. Sunt tiparite liste de rezolvitori n ordinea numarului de probleme rezolvate. n
"Catra cetitori" din nr. 1/1885, redactia revistei apreciaza ca rezultatele obtinute n primii
doi ani sunt pozitive: "[...] rezultatele la care am ajuns snt n destul de mul
tamitoare.
O miscare n aceasta direc
tie, ntre elevii eminen
ti din scoalele noastre, putem zice ca
s-a determinat. Un numar nsemnat de tineri ne trimet regulat solu
tii, dintre care unele
2

destul de ingenioase".
Nu ne propunem sa facem o analiza a continutului revistei (v. [6],[7],[10]), vom
selecta doar cteva aspecte ce consideram ca sunt semnificative si interesante.
C. Climescu, sufletul revistei "Recreatii stiintifice", a publicat multe articole si din
domenii diverse. n ciclul "Cteva curbe celebre si importante", nceput n vol.II si
ncheiat n vol.III, expune principalele curbe plane clasice: cisoida lui Diocles, concoide,
cicloide, spirale etc.
I. Melik publica n nr. 3/1883 articolul "Despre scrierea numerelor cu litere chirilice".
G. I. Rosiu traduce (dupa o editie italiana) prima carte a "Elementelor" lui Euclid si
publica n vol.II si III ale revistei. Precizam ca traducerea completa n limba romna a
"Elementelor" a fost facuta mult mai trziu de Victor Marian si publicata n Biblioteca
Gazetei Matematice n trei volume, 1939-1941.
I. D. Rallet contribuie cu articole variate: maxime si minime geometrice, proprietati
ale patrulaterului circumscriptibil, formulele fundamentale ale trigonometriei sferice, determinanti, echilibrul unui punct material etc.
G. I. Lucescu publica printre altele un studiu amplu si documentat despre masurarea
timpului si calendar, iar I. V. Praja abordeaza chestiuni de aritmetica (probleme de amestec
s. a.), geometrie (transversale, poli si polare s.a.), analiza etc.
Revista "Recreatii stiintifice" a reusit sa atraga colaborarea unor eminenti profesori
din acele timpuri: Miltiade Tzony - Facultatea de stiinte din Iasi, a publicat "Un curs de
probleme", ce este prima culegere de probleme de mecanica teoretica din tara (98 probleme); P. Tanco - profesor din Nasaud, cu chestiuni de filozofia matematicii si de calendar;
Constantin Gogu - Universitatea din Bucuresti, cu cteva scrisori despre calendar; Iacob
Solomon - inginer, cu chestiuni de istoria matematicii din antichitate; profesorii din Iasi
V. Bu
tureanu (mineralogie) si August Scriban (geografie) s.a.
Nu putini sunt aceia care, n drum catre o stralucita cariera, au fost n tineretea
lor activi rezolvitori ai "Recreatiilor stiintifice": Ermil Pangrati - profesor de geometrie
descriptiva si rector al Universitatii din Bucuresti, Anastasie Obreja - creatorul scolii de
chimie organica din Iasi, Vasile Cristescu - unul din cei patru "stlpi" ai Gazetei matematice, Dimitrie Pompeiu - ilustrul matematician romn, Petre Culianu, Gr. G. Stratilescu
si multi altii.
Rezolvitorii erau mai ales elevi ai liceelor si scolilor militare sau studenti si proveneau
din toate colturile tarii (de atunci!): Dorohoi, Bacau, Brlad, Focsani, Bucuresti, Craiova.
Apar si rezolvitori cu profesii mai departate de matematica: un preot din Bucuresti, un
profesor de limba franceza din Bacau, o persoana ce semneaza cu "Vrfu cu dor" etc.
Cu o munca sustinuta si mari sacrificii materiale, redactorii au scos la timp si au
asigurat revistei un nivel de calitate nalt. Ei sunt pe deplin constienti de rezultatele
obtinute si de faptul ca prin munca lor au apropiat momentul aparitiei lucrarilor originale.
Acest lucru rezulta fara echivoc din cuvntul redactiei "Catra cetitori" la nceputul anului
al VI-lea:

Greutatile materiale fac ca odata cu publicarea numarului din decembrie 1888, adica
exact dupa sase ani, revista sa nceteze sa apara. Se cuvine sa mentionam ca sustinatorul
principal al "Recreatiilor stiintifice" a fost C. Climescu. Pe coperta revistei din al VI-lea
an este scris: "Redac
tia si Administra
tia la Dl. C. Climescu, Profesor la Facultatea de
te, Strada Butu 22". Aceeasi adresa apare si n casetele ce urmeaza titlul n fiecare
Stiin
numar din ultimul an de aparitie (cum se poate vedea si n reproducerea de mai sus).
4

Fosti rezolvitori ai "Recreatiilor stiintifice" si vor aduce aminte cu recunostinta de


aceasta. Peste timp, mari matematicieni romni vor avea cuvinte de apreciere pentru
curajul, sacrificiile si fapta celor care "au tras cea ntai brazda".
Peste nici sapte ani de la disparitia lor, "Recreatiile stiintifice" si afla o continuare
n "Gazeta matematica", ai carei fondatori si colaboratori au stiut si au reusit sa nvinga
imense greutati si obstacole si sa faca din visul cercetarii originale o realitate; aceasta
si-a serbat un veac de existenta nentrerupta si este mereu tnara.
Prof. dr. Temistocle BRSAN

Recreatii stiintifice prezent


a n constiinta posterit
atii
Rezolvirea problemelor este unul din cele mai bune stimulente pentru a atrage pe
cineva catre studiul matematicilor. Experienta noastra personala ne probeaza lucrul
tiintifice ce a
acesta. Mai multi dintre noi datoresc acest gust revistei Recreatii S
aparut n timp de 6 ani la Iasi si pe care noi ncercam a o continua.
Redactia ["Gazetei matematice"] [2, p.1]
Pe cnd la Bucuresti se petreceau aceste prefaceri de societati stiintifice, profesorii de
stiinte din Iasi se hotarasc sa scoata o revista stiintifica, si anume Recreatii stiintifice
care a aparut cu mari greutati sase ani, de la 1883-1888. Revista consacra cea mai mare
parte matematicilor si propunea probleme pentru folosul liceenilor. Aceasta revista a
contribuit mult la raspndirea gustului pentru studiul matematicilor la noi n tara.
Ion Ionescu [3, p.11]
Cea dinti ncercare de a iesi din acest impas, de a rupe inertia, de a determina un
curent de preocupare stiintifica si de a crea astfel un nceput de atmosfera prielnica dezvoltarii stiintei matematice, a fost facuta la Iasi prin publicarea Recreatiilor stiintifice.
teica [4, p.69]
Gheorghe Ti
Omagiu pios primilor pionieri ai studiilor matematice la noi; omagiu pios acelei vechi
tiintifice si vrednicilor ei fondatori si colaboratori;
reviste matematice Recreatii S
omagiu Gazetei Matematice si acelor care au ntemeiat-o si sustinut-o prin colaborarea
lor pna azi.
Gr. G. Stratilescu [5, p.364]
"Recreatiile stiintifice" au aparut n 1883 viznd obiective de o ntelepciune ce face
cinste fondatorilor revistei. [...] Discret si oarecum neasteptat cum a aparut, apune, la 15
decembrie 1888, prima revista stiintifica romneasca menita sa deschida drum cercetarii
originale.
Ilie Popa [6, pp.492,493]
5

Pe lnga nvatamntul matematic secundar si superior, o contributie importanta la


dezvoltarea stiintei matematice la noi n tara si a interesului fata de aceasta stiinta
au adus-o, n perioada aceasta de pregatire, cele doua reviste care au aparut la Iasi
si Bucuresti: "Recreatii stiintifice", care a durat din 1883 pna n 1888, si "Gazeta
matematica", nfiintata la 15 septembrie 1895.
George S
t. Andonie [8, p.236]
n tara noastra, publicatiile periodice destinate propagarii gustului si competentei pentru propunerea si rezolvarea de probleme matematice aniverseaza un secol de existenta.
n 1883 a aparut la Iasi, prin dragostea si devotamentul unui grup de intelectuali de
seama, profesori universitari sau secundari, ingineri, fizicieni, medici, revista "Recreatii
S
tiintifice", careia Universitatea din Iasi si Societatea de S
tiinte Matematice din R. S.
Romnia i-au aniversat centenarul aparitiei la sfrsitul anului 1983.
Nicolae Teodorescu [9, p.7]
Revista si-a ntrerupt activitatea pe neasteptate. Ea a reusit sa trezeasca un viu
interes pentru matematici n rndul elevilor si studentiolor. Mare parte din corespondentii
formati de "Recreatii stiintifice" au devenit profesori si ingineri cu o serioasa pregatire n
matematicile elementare.
Nicolae Mih
aileanu [11, p.177]
Bibliografie
1. Recreatii stiintifice (1883 - 1888) - colectia revistei.
2.
***
- Introducere, Gazeta matematica, an.I, nr.1, septembrie 1895, (Mihail Roco
este autorul acestei "Introduceri" - sarcina ncredintata de redactia G.M.).
3. I. Ionescu - Constituirea, administrarea si redactarea Gazetei Matematice, articol
aparut n volumul Gazeta Matematica, 1895-1935. Istoric - nvataminte (volum jubiliar),
Biblioteca Gazetei Matematice, vol. XI, Bucuresti, 1935.
teica - Rolul Gazetei Matematice n dezvoltarea stiin
4. Gh. Ti
tei matematice n Romnia, ibidem.
5. Sarbatorirea celor 40 ani ai "Gazetei matematice". Cuvntarea D-lui profesor Gr. G. Stratilescu, G.M. XLI (1936), 361-374.
6. I. Popa - Recrea
tii stiin
tifice - precursoare a Gazetei Matematice, G.M.F., seria
A, 9/1955, 492-493.
7. I. Popa - Dezvoltarea matematicii, aparut n Contribu
tii la istoria dezvoltarii Universitatii
din Iasi, vol. II, Bucuresti, 1960.
t. Andonie - Istoria matematicii n Romnia, vol. I, Ed. S tiintifica, Bucuresti, 1965.
8. G. S
9. N. Teodorescu s.a. - Probleme din Gazeta Matematica, Ed. Tehnica, Bucuresti, 1984
(citat din Prefa
ta, semnata de acad. N. Teodorescu, presedintele S.S.M.R.).
10. Gh. Bantas - O pagina din istoria matematicii romnesti: centenarul revistei "Recrea
tii
stiin
tifice", Probleme de istoria si filozofia stiintei, vol. X, Acad. R.S.R., filiala Iasi, 1984.
11. N. Mih
aileanu - Revistele de matematici elementare din Romnia (pna la 1848), Ed.
Gil, Zalau, 1995.
6

Cteva curbe celebre si importante


1. Cisoida lui Diocles1
Se d
a un cerc, pe care se ia un punct A; fie AB
y
diametrul ce trece prin acest punct si T T 0 tangenta
n B; prin A se duce o secant
a care taie cercul n H
Q
si tangenta n G; pe secant
a se ia, cu ncepere de
la punctul A, o lungime AM egal
a cu HG - portiM
unea de secant
a dintre cerc si tangent
a - ; locul
geometric al punctelor M este Cisoida lui Diocles. A
P
Sa nsemnam prin R raza cercului; originea de coordonate sa fie A; directia diametrului AB sa fie luata
ca axa de x si perpendiculara n A pe acest diametru
sa fie axa de y.
Fie M un punct al Cisoidei, ale carui coordonate snt
x = AP si y = M P . Prin ipoteza avem AM = HG,
de unde rezulta AP = DB. n triunghiul dreptunghiu AHB, avem

T
G

L
x
B

K D

HD2 = AD DB = x (2R x) ;

apoi triunghiurile asemenea AM P si AHD dau


HD
AD
HD
2R x
=
sau
=
.
MP
AP
y
x
Eliminnd HD ntre relatiunile (1) si (2), avem
(2R x) y 2 = x3

sau x3 + xy 2Ry 2 = 0.

(1)

(2)

(3)

Aceasta este ecvatia Cisoidei.


Curba este simetrica n privirea axei de x, caci la fiecare valoare data lui x, corespunde pentru y doua valori egale si de semne contrare. Ea se compune din doua ramuri
indefinite, egale ntre ele, situate de o parte si de alta a axei de x.
Originea A este un punct de napoiere de specia nt
aia. n adevar, daca ne raportam
la teoria punctelor multiple, stim ca coordonatele unui asemenea punct, satisfac ecvatiei
curbei si ntailor derivate partiale; avem
f (x, y) = x3 + xy 2 2Ry 2 ,

fx0 = 3x2 + y 2 ,

fy0 = 2xy 4Ry;

coordonatele punctului A snt x = 0 si y = 0, care substituite n aceste relatiuni dau


f (0, 0) = 0,

f00 = 0,

f00 = 0,

tiintifice", an II (1884), nr. 1, 19-23.


Articol preluat din "Recreatii S
Ciclul "Cteva curbe celebre si importante" cuprinde 9 lectii prezente n numerele din anii II si III n
care sunt expuse principalele curbe plane: Cisoida lui Diocles (care deschide ciclul), concoida lui Nicomede, melcul lui Pascal , strofoida, ovalele lui Cassini, cicloide, epicicloide, conice si spirale.
S-au pastrat termenii de matematica din textul original, dar s-au facut modificari n privinta lexicului
si ortografiei (astfeli
u - astfel, valor - valori, da
u - dau, adeca - adica, sa - s-a etc).

ceea ce nsemneaza ca originea este un punct duplu; apoi daca formam ecvatia tangentelor n acest punct gasim y 2 = 0; adica tangentele n acest punct se confunda cu axa
de x. Asadar, originea este un punct duplu de napoiere si de nt
aia specie, caci curba
este de o parte si de alta a tangentei.
Cisoida admite o asimptota paralela cu axa de y, caci stim ca aceste asimptote se
capata egalnd cu zero coeficientii celei mai nalte puteri a lui y, ceea ce da 2R x = 0,
de unde x = 2R, adica tangenta la cercul T T 0 este asimptota Cisoidei.
Asimptote neparalele cu axele nu snt.
Cercul dat este numit cercul director al Cisoidei.
T
Newton a dat Cisoidei urmatoarea descriptie mecanica.
K
H
Fie un punct fix A si o dreapt
a fix
a T T 0;
N
din A se duce perpendiculara AD pe dreapta
fix
a; apoi se imagineaz
a un unghi drept care
I
se misc
a astfel c
a una din laturile lui trece prin
M
punctul A, iar extremitatea celeilalte laturi, - A
O
R
D
L
luat
a egal
a cu AD -, se razim
a pe dreapta
fix
a; dac
a G este vrful unghiului drept si H
G
extremitatea laturii a doua, punctul M din
mijlocul laturii GH descrie Cisoida.
T
Sa luam mijlocul O al dreptei AD si din punctul D ca centru cu DO ca raza sa descriem un cerc; sa unim AH, apoi OM , M fiind
mijlocul laturii GH.
Vom demonstra mai nti ca dreptele AH si OM snt paralele ntre ele. n adevar,
triunghiurile dreptunghe AGH si ADH snt egale caci AH = AH, apoi GH = AD
conform enunciului. De aici rezulta AG = DH si fiindca unghiurile ARG si HRD snt
egale ca opuse la vrf, apoi rezulta ca si triunghiurile dreptunghe AGR si HDR snt
egale ntre ele.
Din egalitatea acestor doua din urma triunghiuri rezulta GR = RD si fiindca OD =
= GM , apoi mai rezulta OR = RM si AO = HM si prin urmare dreptele AH si OM
snt paralele.
Fiindca punctul O este mijlocul dreptei AO, din paralelismul acestor drepte, urmeaza
ca OM taie pe HD ntr-un punct care-i la mijlocul dreptei HD.
Sa aratam acum ca triunghiurile HIM si DIM snt egale; n adevar, avem mai nti
HM = DO = DN ; apoi ungh.N ID = ungh.HIM ; pe urma succesiv ungh.DN I =
= ungh.M OR = ungh.HAD = ungh.AHG = ungh.HM I, asadar IM = IN si
fiindca OI = IK, apoi rezulta IM = IN si prin urmare OM = N K.
Asadar, n miscarea unghiului drept AGH, punctul M descrie o Cisoida al carei cerc
director este acel descris din D ca centru cu DO ca raza.
Cisoida a fost imaginata de Diocles (500 a. Ch.) pentru a rezolvi problema a dou
a
medii proportionale1 . Iata cum se rezolveste aceasta problema.
1

Sublinierele din aceast


a fraz
a nu apar si n textul original.

Ecvatia (3) fiind rezolvita n privirea lui y da:


x2
.
(4)
y=p
x (2R x)
Luam semnul + naintea radicalului fiindca consideram ramura de deasupra axei de x.
Sa nsamnam prin z ordonata punctului de pe cerc al cariu abscisa este x; avem
z 2 = x (2R x) .
(5)
Comparnd ecvatiile (4) si (5) avem raporturile
2R x
z
x
= = .
(6)
z
x
y
Fie a si b liniile ntre care se cere a se afla doua medii proportionale.
b
Sa nmultim terminii raporturilor (6) prin , ceea ce da
y
(2R x) b
bz
bx
y
y
y
=
=
.
(7)
bz
bx
b
y
y
Sa luam pe Cisoida un punct astfel ca sa avem
(2R x) b
= a,
(8)
y
ceea ce revine a considera punctul comun Cisoidei, - reprezentata prin ecvatia (3) -, si
dreptei - reprezentata prin ecvatia (8) -; atunci raporturile (7) se pot scrie
bz
bx
a
y
y
=
=
.
bz
bx
b
y
y
bx
a

bz
= si
= , vom avea = = . Aceste trei raporturi ne dau doua
Sa punem
y
y

b
bz
bx
si
ecvatii, din care vom scoate pe si . Astfel cantitatile
vor fi cunoscute si
y
y
aceste vor fi cele doua medii proportionale ntre a si b.
Daca Cisoida este construita, pentru a gasi punctul de pe ea cu ajutorul caruia putem
rezolvi problema, trebuie sa construim dreapta (8), care este o dreapta ce trece prin
punctul B, pentru aceea luam o lungime BK = a, radicam perpendiculara KL = b,
dreapta BL este dreapta (8). Punctul M n care aceasta dreapta taie Cisoida este
punctul cautat, si avem
AP = x, M P = y, QP = z;
QP
AP
si b
prin urmare cele doua medii proportionale ntre a si b vor fi b
.
M
P
M
P
(Va urma)

C. CLIMESCU
9

Scrierea numerelor cu litere chirilice2


Toate popoarele, afara de vechii chinezi si de un trib putin cunoscut de care vorbeste
Aristot3 au adoptat sistema de numeratie zecimala, n care numerele snt mpartite n
perioade de cte zece unitati. Baza, n aceasta sistema de numeratie, este zece, adica
zece unitati de un ordin oarecare trebuiesc ca sa formeze o unitate de ordinul imediat
mai nalt.
Scrierea oricarui numer se face astazi numai cu zece semne, numite cifre, dintre care
noua reprezinta pe cele dinti noua numere ntregi, si care snt :
1
unu,

2
doi,

3
trei,

4
patru

5
cinci,

6
sese,

7
septe,

8
opt,

9
noua.

Semnul al zecelea este 0 sau zero, prin care se arata lipsa de unitati de un ordin
oarecare.
S-a admis ca orice cifra pusa la stnga unei alte cifre reprezinta unitati de zece ori
mai mari dect aceasta din urma, adica unitati de ordinul imediat superior. Astfel nct,
dnd acestor semne, diferite locuri se poate exprima, ntr-un chip nu se poate mai usor,
orice numer, fie orict de mare numerul unitatilor din care el se compune.
Semnele acestea le-am mprumutat, pe la mijlocul secolului al X, de la arabi care si
dnsii pare ca le-au luat de la indieni, adevaratii ntemeietori ai stiintei ctimilor.
n vechime, era obeciul de a se reprezenta numerele cu litere.
Grecii nsemnau numerele cu literele din alfabetul lor; diferite ordine de unitati: unimi,
zeci, sute, . . . , se deosebeau unele de altele prin accente puse deasupra literelor.
Romanii, pentru scrierea numerelor, ntrebuintau septe semne principale:
I
unu,

V
cinci,

X
zece,

L
cincizeci,

C
o suta,

D
cinci sute,

M
o mie.

Cu ajutorul acestor litere, si prin chipul combinarei lor facuta dupa niste reguli bine
statornicite, se pot scrie numere din o mie si orice numer de sute, zeci si unimi.
n cartile romnesti vechi, n cronici precum si n cartile religioase, pe nscriptii,
hrisoave, se gasesc numere scrise cu litere chirilice.
n reprezentarea chirilica a numerelor, unimilor, zecile si sutele se nseamna, dupa cum
se vede n tabela de mai la vale, fiecare cu cte un semn deosebit, o litera a alfabetului
chirilic avnd deasupra lor un semn, poate spre a arata ca litera este luata drept numer.
Astfel, pentru a scrie un numer oarecare, mai mic dect o mie, n notatia chirilica,
trebuiesc 27 semne.
Aceleasi litere, avnd semnul
n stnga lor si putin mai jos, servesc a reprezenta
unitatile din clasa miilor, adica unimile de mii, zecile de mii si sutele de mii.
Alte rnduri de semne conventionale, puse alaturi cu aceleasi litere, ar fi putut servi
spre a reprezenta unitatile din celelalte clase mai nalte, clasa milioanelor, a miliardelor,
2
3

Articol preluat din "Recreatii S


tiintifice", an I (1883), nr. 3, 57-60.
Bossut, Istoria generala a matematicilor.

10

11

. . . ; totusi, numerele exprimnd milioanele precum si ordine mai nalte urmatoare se scriu
n cuvinte.
Caracteristic este modul cum se potriveste de bine scrierea numerelor n notatia chirilica cu numirea lor. Se stie ca, numerele cuprinse ntre zece si douazeci prezinta o exceptie,
n ceea ce priveste numirea lor, de la regula generala ntemeiata pe modul formarii lor.
Pe cnd la numerele de la douazeci nainte, compuse din zeci si unimi, se enunta mai
nti zecile si apoi unimile, daca snt , ca de exemplu n: patruzeci si patru, septezeci si
noua etc., la numerele cuprinse ntre zece si douazeci, se enunta mai nti unimile si apoi
zecile: treisprezece, optsprezece etc. n reprezentarea chirilica a numerelor, se urmeaza
ntocmai dupa cum se enunta: unitatile se scriu nainte sau dupa zeci, dupa cum este si
numirea numerului.
Este lesne de vazut ca numerele exprimate cu litere se pot supune calculului ca si
numerele scrise cu cifre arabe; singura deosebire sta ntr-aceea ca calculul va fi, n cazul
nti, cu atta mai lung si mai greu, cu ct numerul va fi mai mare.

I. M. MELIK

12

Acad. Radu Miron la a 75-a aniversare


Academician profesor doctor docent Radu Miron.....ce se mai poate oare spune
n cteva rnduri!? S
i totusi...profesorul Radu Miron este preferatul multor generatii de studenti ai Facult
atii de Matematic
a din Iasi. Prezenta carismatic
a n amfiteatru, reuseste sa captiveze n mod natural auditoriul. Cu mult calm si cu o dictie
remarcabila, cele mai ntunecate capitole ale matematicii se limpezesc, iar fereastra opac
a din spatele oglinzii cap
at
a o transparenta de cristal chiar si pentru orbul
disimulat n student.
S
i totusi...profesorul R. Miron arunca peste noi, ca un prestidigitator, o plasa
imensa care este opera sa, opera care contine peste 250 de lucrari stiintifice, note
bibliografice, mongrafii etc.
Tr
aind peste 50 de ani n atmosfera Seminarului Matematic Al. Myller, un adevarat laborator de creatie stiintifica n cmpul abstract al matematicii, o scoala academica de nalta tinuta, profesorul R. Miron este un continuator al acelei generatii de
aur a matematicienilor romni. Mention
am aici numele lui Alexandru Myller, fondatorul Seminarului Matematic, Octav Mayer, Gheorghe Vranceanu, Grigore Moisil,
Mendel Haimovici, Adolf Haimovici, Dimitrie Mangeron, Constantin Climescu, Ilie
Popa, Gheorghe Gheorghiev s. a.
Remarcat de profesori nc
a din primii ani de studentie este numit asistent n
anul II, iar dup
a absolvirea facult
atii este ncadrat ca cercet
ator la Institutul de
Matematica, Filiala Iasi a Academiei. In 1957 si sustine teza de doctorat cu titlul Problema geometrizarii sistemelor mecanice neolonome, sub conducerea academicianului Mendel Haimovici, lucrare publicat
a n ntregime n revista Studii si
Cercet
ari Matematice.
Parcurgnd ntreaga ierarhie universitara este numit n 1969 profesor la Catedra
de Geometrie a Facultatii de Matematica din Universitatea Al. I. Cuza, iar n 1973
director al Institutului de Matematic
a. Decan al Facult
atii de Matematic
a ntre anii
1972-1976, conduc
ator de doctorat din 1972, sef al Catedrei de Geometrie, membru
al Consiliului Profesoral si al Senatului, profesorul R. Miron a desfasurat o bogata
activitate didactica si educativa.
Activitatea de cercetare stiintific
a a profesorului R. Miron este bine cunoscut
a n
lumea ntreag
a. Este stabilit
a si recunoscut
a contributia sa important
a si original
a
la dezvoltarea Geometriei Diferentiale moderne si a aplicatiilor ei n Fizica Teoretica.
Profesorul R. Miron a creat si dezvoltat n matematica noi ramuri ca: geometria
configuratiilor Myller, teoria invarianta a spatiilor Finsler, spatii Lagrange, spatii
Lagrange generalizate, teoria subspatiilor Lagrange, geometria spatiilor Lagrange de
ordin superior, spatii Hamilton, spatii Hamilton generalizate, teoria geometrica a
spatiilor fibrate, teoria lagrangeana a relativitatii si electromagnetismului, ecuatii
Einstein si Maxwell. A rezolvat multe probleme deschise ca: prelungirea structurilor
riemanniene, finsleriene, lagrangeene, spatii Finsler de ordin superior etc.
Cercetarile initiate si dezvoltate de profesorul R.Miron au avut un mare impact
asupra specialistilor n geometrie si nu numai. Profesorul Makoto Matsumoto de la
Universitatea din Kyoto, n cartea sa Fundamentele Geometriei Finsler si Spatii
13

Finsler speciale se refer


a la conceptul de reper Miron si ecuatii fundamentale ale
reperului Miron. Profesorul Masao Hashiguchi atribuie numele de "spatiu Miron
unui caz remarcabil de spatii Hamilton, introduse pentru prima dat
a de profesorul
R. Miron, iar G. S. Asanov, de la Universitatea din Moscova, aplica modelele Lagrange
ale profesorului R. Miron n cosmologie si obtine cea mai bun
a deviatie teoretic
aa
periheliului planetelor Marte, Venus si Mercur. Bazndu-se pe teoria profesorului
R. Miron, G. Beil (S.U.A.) a obtinut o bun
a teorie gauge si P. L. Antonelli (Canada)
a aplicat-o n biologie.
Profesorul R. Miron a creat n Romnia o scoal
a de matematic
a de nalt nivel,
care a cooperat pe parcursul multor ani cu oameni de stiinta din Japonia, Rusia,
S.U.A., Germania, Italia, Anglia, Canada, Ungaria, Egipt etc. Un num
ar mare de
doctoranzi din tara si strainatate (Japonia, Italia, Ungaria, Vietnam) au obtinut
titlul de doctor n matematic
a sub conducerea d-sale.
Avnd un renume deosebit n lumea Matematicii, profesorul R. Miron a fost invitat ca visiting professor la prestigioase universit
ati ca: Universitatea din Tsukuba
(Japonia, 1988, 1990, 1992 ), Bary (Italia, 1987 ), Freiburg si Mnchen (Germania,
1975, 1990 ), Edmonton (Canada, 1992 ).
Profesorul R. Miron a publicat o parte din lucr
arile sale n colaborare cu geometri japonezi: M. Matsumoto, M. Hashiguchi, Y. Ichijio, S. Kikuchi, S. Watanabe, S. Ikeda sau cu membrii ai Seminarului National de Geometrie Finsler si Lagrange, initiat de dnsul n 1980 la Universitatea din Brasov.
Este primul presedinte al Societ
atii Balcanice a Geometrilor constituit
a la initiativa sa si a profesorului G. Tsagas de la Universitatea Aristotel din Thessaloniki
(Grecia) n 1994-1995. Instituto per la Ricerca di Base din Italia i-a oferit profesorului R. Miron titlul de Full Professor in the Division of Mathematics of the
I.R.B..
Profesorului R. Miron i s-a acordat Premiul Ministerului Educatiei (1963), premiul Gh.Ti
teica al Academiei (1968) si a fost ales membru al Academiei Romne
(1991).
Profesorul R. Miron a scris n colaborare cu profesorul M. Anastasiei o carte
de pionerat Geometria spatiilor Lagrange: teorie si aplicatii , publicata n 1994 de
Kluwer Academic (S.U.A.) n prestigioasa serie Fundamental Theories of Physics.
Mai amintim dou
a titluri importante de monografii care concentreaz
a ideile matematice ale profesorului R.Miron: Geometria spatiilor Lagrange de ordin superior.
Aplicatii n Mecanica si Fizica (Kluwer Academic, 1997) si Geometria spatiilor
Finsler de ordin superior (Hadronic Press, U.S.A., 1998).
Impreun
a cu profesorul P. L. Antonelli de la Universitatea din Alberta, profesorul
R. Miron este editorul c
artii Geometrie Finsler si Lagrange.Aplicatii n Fizica si
Biologie, publicata deasemeni de Kluwer Academic n 1996.
Chiar daca pe 3 octombrie 2002, profesorul Radu Miron a mplinit frumoasa
vrst
a de 75 de ani, sunt convins c
a "surprizele matematice" vor continua s
a ne
uimeasc
a.
Prof. dr. Alexandru NEAGU

14

Numere prime din progresii aritmetice


Petru Minu
t1
Un numar natural p, p > 1, se numeste numar prim daca nu are alti divizori
nafara de 1 si p.
Lem
a. Un numar natural n, n > 1, are un divizor prim.
Demonstratie. Fie M multimea tuturor numerelor naturale care sunt divizori
ai lui n diferiti de 1. M 6= , deoarece n M . n M exist
a un num
ar care este cel
mai mic, p. Ar
at
am, prin reducere la absurd, c
a p este prim. Presupunem c
a p este
compus: p = ab, 1 < a < p. Din a | p si p | n rezulta ca a | n. Am gasit un divizor al
lui n mai mic ca p ceea ce contrazice alegerea lui p.
Teorema 1. n multimea numerelor naturale exista o infinitate de numere prime.
Demonstratie. Exist
a numere prime. De exemplu 2, care nu poate avea alti
divizori n afar
a de 1 si 2. Folosim metoda reducerii la absurd. Presupunem c
a exist
a
o multime finita de numere prime n N, P = {p1 , p2 , . . . , pk }. Consideram numarul
ajutator N = p1 p2 . . . pk + 1. Deoarece N > 1, exista un numar prim p, p | N . Din
p P rezult
a c
a p | p1 p2 . . . pk . Dac
a dou
a numere sunt multipli de p, atunci diferenta
lor este multiplu de p. Rezult
a c
a p | 1, ceea ce implic
a p = 1 si contrazicem definitia
numarului prim.
Observatie. Teorema 1 o g
asim enuntat
a si demonstrat
a pentru prima oar
a n
opera lui Euclid "Elemente" (sec. III . Ch.) si este cunoscuta sub denumirea de
teorema lui Euclid. Se cunosc numeroase demonstratii ale acestei teoreme.
Singurul numar prim par este 2. Aranjam numerele impare n doua siruri:
3, 7, 11, 15, . . . , 4k 1, . . .
1, 5, 9, 13, . . . , 4k + 1, . . .

(1)
(2)

Constatam ca n aceste siruri (progresii aritmetice), mergnd pna la termeni de rang


tot mai mare, g
asim noi termeni care sunt numere prime. Dac
a lu
am si alte progresii
aritmetice, de exemplu:
3, 13, 23, 33, 43, 53, 63, 73, 83, . . .
(3)
2, 7, 12, 17, 22, 27, 32, 37, 42, 47, . . .
(4)
constat
am acelasi lucru. Este usor de demonstrat c
a n progresia (1) exist
a o infinitate
de numere prime.
Teorema 2. Exista o infinitate de numere prime de forma p = 4k 1, k N.
Demonstratie. Procedam prin reducere la absurd. Am pus deja n evidenta
cteva numere prime de aceast
a form
a (sirul (1)). Presupunem c
a exist
a un num
ar
arul ajut
ator N =
finit de numere prime de acest fel: p1 , p2 , . . . , pn . Construim num
= 4p1 p2 . . . pn 1. Deoarece N > 1, exist
a p prim, p | N . Orice num
ar prim p diferit
de 2 este de forma p = 4k 1 sau p = 4k + 1. Daca toti divizorii primi ai lui N sunt
de forma p = 4k + 1, num
arul N este de forma N = 4h + 1, deci 4 | N 1 si 4 | N + 1,
ceea ce implic
a 4 | 2. Contradictie! Exist
a divizori primi ai lui N de forma 4k 1. Fie
1

Prof. dr., Univ. "D. Cantemir", Tg. Mures

15

p un asemenea divizor. Rezult


a c
a p {p1 , p2 , . . . , pn }, deci p | 4p1 p2 . . . pn si cum
p | N rezult
a p = 1. Contradictie! Presupunerea c
a exist
a un num
ar finit de numere
prime de forma p = 4k 1 nu poate fi adev
arat
a.
Teorema 2 se generalizeaza dupa cum urmeaza:
Teorema 3. Pentru orice numar natural n, n 6= 0, exista o infinitate de numere
prime p de forma p = nk 1, k N.
Demonstratie. Pentru n = 1, {nk 1 | k N} = N {1} si afirmatia teoremei este adev
arat
a (teorema lui Euclid). Pentru n = 2, {nk 1 | k N} =
= {1, 1, 3, 5, 7, . . . } si afirmatia teoremei este adev
arat
a (exist
a o infinitate de numere prime impare).
Pentru demonstratia teoremei n cazul n > 2 vom folosi lema urmatoare:
Lem
a. Pentru orice numa
r natural n, n > 1, avem:
Y
(5)
1r<n r 1 (mod n) .
(r,n)=1

Demonstratie. Pentru n = 2 congruenta este evidenta. Pentru n > 2 si r fixat,


1 r < n, (r, n) = 1, stim ca exista solutie unica pentru congruenta rx 1 (mod n).
Deci, exist
a un singur r0 , 1 r0 < n, (r0 , n) = 1 astfel nct rr0 1 (mod n). n
0
membrul nti al congruen
tei (5) nlocuim
Q produsele rr cu 1 pentru toti r pentru
Q
0
care r 6= r. Rezulta ca (r,n)=1 r
r. Observam ca pentru r cu
(r,n)=1
r2 1(mod n)

proprietatea c
a r2 1 (mod n) avem r (n r) 1 (mod n). Rezult
a c
a membrul
k
arul acelor r cu
nti al congruentei (5) este congruent cu (1) , unde 2k este num
proprietatea r2 1 (mod n) (r si n r au ambii aceasta proprietate). Lema este
demonstrata.
Revenim la demonstratia teoremei n cazul n > 2. Vom ar
ata prin reducere la
absurd, c
a exist
a o infiniate de numere prime p de forma p = nak 1, k N, unde
a este produsul numerelor naturale mai mici ca n si prime cu n luat cu semnul +
sau dupa cum produsul acestor numere este congruent cu +1 sau 1 modulo n.
Presupunem c
a exist
a un num
ar finit de numere p de forma p = nak1: p1 , p2 , . . . , ps .
Consider
am num
arul ajut
ator N = nap1 p2 . . . ps 1. N > 1 deoarece chiar n cazul
s = 0, N = na 1 > n 1 1. Exista p prim, p | N ; p este de forma p = nau + r,
(r, N a) = 1. Din p = N au + r nu + r (mod n) rezulta ca p si nu + r dau acelasi
rest la mp
artirea cu n. Rezult
a c
a r < n si nu putem avea 1 < r < n 1 deoarece ar
rezulta c
a r | n sau r | a, deci r | p si contrazicem faptul c
a p este prim. Prin urmare,
p este de forma p = nau 1. Daca toti divizorii lui N ar fi de forma p = nu + 1, N
ar fi si el de aceasta forma. Prin urmare, exista un divizor prim p al lui N de forma
p = nau 1. Rezult
a c
a p {p1 , p2 , . . . , ps }, p | nap1 p2 . . . ps si cum p | N am avea
p | 1, deci p = 1 si contrazicem definitia num
arului prim.
Teorema 2 se obtine din Teoremei 3 lund n = 4. Din Teorema 3 rezulta ca exista
o infinitate de numere prime p de forma p = 6k 1, k N sau p = 8k 1, k N
s.a.m.d.
Pentru a ar
ata c
a progresia (2) contine o infinitate de numere prime vom folosi
urmatoarea
2
Lem
a. Oricare ar fi numarul natural n, n > 1, numarul (n!) + 1 are divizori
primi si acestia sunt de forma p = 4k + 1.
16

Demonstratie. Pentru n > 1, num


arul (n!) + 1 este impar, mai mare ca 1.
Exist
a p | (n!)2 + 1, p 6= 2. Deci p este de forma p = 4k + 1 sau p = 4k + 3. Dac
ap
2
2(2k+1)
p1
a p | (n!)
+1, adic
a p | (n!)
+1
este de forma p = 4k+3, din p | (n!) +1 rezult
si apoi p | (n!)p + n!. Conform cu mica teorema a lui Fermat p | (n!)p n!. Rezult
a
p | 2n!, deci p n, p | n! ceea ce implica p | 1, contradictie!
Teorema 4. Exista o infinitate de numere prime p de forma p = 4k + 1, k N.
Demonstratie. Folosim din nou metoda lui Euclid. Presupunem c
a exist
a un
am
num
ar finit de numere prime de forma 4k + 1: p1 = 5 < p2 < < ps si consider
num
arul ajut
ator N = [(p1 p2 . . . ps )!]2 + 1. Acesta admite, conform lemei, un divizor
prim p de forma p = 4k + 1 si ajungem din nou la contradictia p | 1.
Pentru generalizarea Teoremei 4 avem nevoie de cteva chestiuni preg
atitoare. Fie
2h
2h
k
i
k un num
ar natural, k 1. Ecuatia x 1 = 0 are r
+
ad
acinile xh = e k = cos
k

Q
2h
2h
i sin
, h = 0, 1, . . . k 1. Consideram polinomul Fn (x) = (h,n)=1 x e k i ,
k
unde produsul se face dupa numerele h {0, 1, . . . , n 1} care sunt prime cu n.
Gradul lui Fn (x) este (n) ( (n) = numarul numerelor naturale mai mici ca n si
prime cu n, este
a sub numele de functia indicatoare a lui Euler ). Observ
am
Q cunoscut
c
a xk 1 = n | k Fn (x) (produsul se face dup
a divizorii pozitivi ai lui k). Fie
xk 1 = Fk (x) Gk (x), unde Gk (x) este cel mai mic multiplu comun al polinoamelor
xn 1, n | k, n < k, avnd coeficientul termenului de grad cel mai nalt egal cu
1. Deoarece Gk (x) este un polinom cu coeficienti ntregi, atunci si Fk (x) este un
polinom cu coeficienti ntregi. Observam ca pentru orice numar ntreg x, x 6= 1,
avem Fk (x) Gk (x) 6= 0.
Lema 1. Fie n un divizor
propriu al
lui k ( n 6= 1, n 6= k). Pentru orice numar

1
x
ntreg x, x 6= 1, avem: xn 1, n
| k.
x 1
n
Demonstratie. Notam k = nd, x 1 = y. Vom avea:
d

(y + 1) 1
xk 1
= y d1 + Cd1 y d2 + + d d (mod y) .
=
xn 1
y

xk 1
Daca = xn 1, n
, din | y rezulta | d si, cum d | k, rezulta | k.
x 1
Lema 2. Fie x Z, x 6= 1. Orice divizor prim, comun lui Fk (x) si Gk (x)
este un divizor al lui k.
Demonstratie. Fie p prim, p | Fk (x), p | Gk (x). Din p | Gk (x)
a c
a exist
a
Q rezult
n N , n | k, n < k, astfel nct p | Fn (x) (deoarece Gk (x) = n | k, k<x Fn (x) si
dac
a un num
ar prim divide un produs atunci el divide
tin unul
cel pu
dintre factori).
xk 1
xk 1
n
n1
si p | x
. Conform Lemei
, n
Din p | x 1 si p | Fk (x) rezulta p | n
x 1
x 1
1, p | k.
Teorema 5. Pentru orice numar natural k, k 1, exista o infinitate de numere
prime de forma p = nk + 1, n N.
Demonstratie. Pentru k = 1 enuntul teoremei este adev
arat (teorema lui Euclid). Pentru k > 1, ar
at
am mai nti c
a exist
a numere prime de forma p = nk + 1.
17

Pentru x = ky, y Z vom avea: Fk (x) Gk (x) = xk 1 1 (mod k). Deoarece


ar finit de r
ad
acini, putem alege y astfel nct
ecuatiile Fk (x) = 1 au un num
Fk (x) 6= 1. Exist
a numere prime p care sunt divizori ai lui Fk (x). Deoarece p - k
(p | k p | x p |1), rezulta (conform Lemei 2) ca p - Gk (x) si deci p - xn 1,
oricare ar fi num
arul natural n, n | k, n < k. Deci xn 6 1 (mod n), n | k, n < k si
k
x 1 (mod n). Fie n = (k, p 1). Exist
a dou
a numere ntregi s si t astfel nct
s p1 t
x
1 (mod p). Nu putem avea n < k,
n = sk+t (p 1). Rezulta ca xn = xk
deci n = k. Conform cu mica teorema a lui Fermat xp1 1 (mod p). Rezult
a c
a
p 1 este multiplu de k. ntr-adev
ar, dac
a este cea mai mic
a putere ntreag
a, strict
pozitiva a lui x astfel nct x 1 (mod p), atunci xa 1 (mod p) | a. Daca
b
a = b, atunci xa = x 1 (mod p). Daca xa 1 (mod p), a = b + r, 0 r < ,
nu putem avea > 0 deoarece xa xr 1 (mod p) si contrazicem alegerea lui .
Deci, p 1 = nk, adic
a p = nk + 1.
ar prim de forma p1 = n1 k + 1. Lu
am k1 = p1 k. Conform primei
Fie p1 un num
parti a demonstratiei exista un numar prim p2 de forma p2 = np1 k + 1, adica pentru
orice numar prim p1 de forma p1 = nk + 1 exista un numar prim p2 de aceeasi forma,
p2 > p1 . Rezult
a c
a exist
a o infinitate de numere prime p de forma p = nk + 1,
n N.
Enuntul cel mai general, care cuprinde drept cazuri particulare toate teoremele
prezentate, l constitue teorema urm
atoare, cunoscut
a n literatura matematic
a sub
denumirea de teorema lui Dirichlet.
Teorema 6. Oricare ar fi numerele l Z si k N , (l, k) = 1, progresia
aritmetica
l, l + k, l + 2k, . . . l + nk, . . .
contine o infinitate de numere prime.
Conditia (l, k) = 1 este necesar
a . Dac
a (l, k) = d > 1, toti termenii progresiei
sunt multipli de d. Demonstratia Teoremei 6, n cazul general, nu poate fi facuta
prin metode ale matematicii elementare.
Problema num
arului de numere prime dintr-o progresie aritmetic
a a fost pus
a
pentru prima oar
a n 1775 de Leonard Euler n cazul particular l = 1. n cartea
sa "Thorie des nombres" A. M. Legendre a dat o demonstratie Teoremei 6 bazata
pe o ipotez
a, care ulterior s-a dovedit a fi fals
a. Prima demonstratie a teoremei a
fost dat
a n 1837 de Lejeune P. G. Dirichlet care a creat un aparat analitic special
(seriile Dirichlet). Demonstratia lui Dirichlet este considerat
a actul de nastere al
teoriei analitice a numerelor.
Bibliografie
1. I. Creang
a, C. Cazacu, P. Minut, Gh. Opait, C. Reischer - Introducere n teoria
numerelor, Editura didactic
a si pedagogic
a, Bucuresti, 1965.
2. Hua Loo Keng - Introduction to Number Theory, Springer Verlag, Berlin, Heidelberg, 1982.
3. P. Minut - Teoria numerelor. Capitole introductive, Editura "Crenguta Gldau",
Iasi, 1997.
4. C. P. Popovici - Teoria numerelor, Ed. didactic
a si pedagogic
a, Bucuresti, 1973.
5. W. Sierpinski - Ce stim si ce nu stim despre numerele prime, Editura stiintifica,
Bucuresti, 1966.
18

Recreatie matematic
a si nu numai
Horea BANEA1
Este cunoscuta urmatoarea problema - joc: Sa se descompuna poligonul din figura
alaturata prin doua linii drepte astfel nct din poligoanele obtinute prin realipire sa
se realizeze un patrat. Solutia este indicat
a n figur
a.

Sugerat de aceasta, propunem urm


atoarea problem
a:
Dintr-o bucata de carton, de forma poligonului de mai sus, printr-o taietura dupa
o dreapta si realipirea bucatilor obtinute se realizeaza diferite figuri de forma unor
poligoane convexe. Sa se gaseasca toate situatiile distincte. Sa se calculeze lungimile
laturilor poligoanelor obtinute.
n leg
atur
a cu enumerarea propus
a facem urm
atoarele preciz
ari:
Situatii distincte sunt cele n care T = taietura si/sau P = poligonul obtinut
difera ntre ele.
Cazurile n care cu aceeasi T obtinndu-se componente simetrice, se poate
realiza acelasi P n diferite moduri, vor fi considerate doar variante echivalente ale
aceleiasi situatii cu exceptia cazurilor n care alipirea aceleiasi componente se face
la alt segment al componentei de baza (=cea mai mare) care vor fi considerate ca
situatii distincte.
Cnd T este variabil
a, obtinndu-se acelasi tip de P dar cu dimensiuni variabile
depinznd de un parametru, se considera ca o singura situatie, dar cazurile particulare
ale parametrului care conduc la P cu anumite particularitati se enumera distincte de
cazul general.
Nu se enumer
a, fiind socotite variante echivalente, figurile obtinute prin ntoarcerea pe verso a ntregii figuri obtinute ntr-un caz.
Pentru a usura urmarirea efectuarii taieturilor indicam gruparea lor n raport cu anumite puncte remarcabile prin care au
fost duse: M : T116 ; N : T1722 ; P : T23 ; Q : T25,26 .
Enumerarea lungimilor laturilor se face ncepnd cu cea superioara, n sens matematic. Justificarea calculelor, bazndu-se
doar pe teorema lui Pitagora si pe asem
anarea triunghiurilor, se las
a pentru cititori.
Trapez dreptunghic variabil:
p
5, y, 4y 2 8y + 29, 2 y; 0 < y < 1.
Triunghi dreptunghic: 5,

Conf. dr., Univ. Transilvania, Brasov

19

29, 2.

Patrulater inscriptibil:

Pentagon: 3,

19 4 3 29 2 29
, ,
,
.
5 5
5
5

4 3 29 4 2 29
,
, ,
.
5
5
5
5

Trapez dreptunghic variabil:


p
3 + x, 4x2 + 4x + 5, 2 x, 2; 0 < x < 2.

Hexagon:
p
p
2x + 2
3, 4 x2 ,
, 2 x, x, 4 x2 ;
x
x 1, 84, x4 + x3 2, 75x2 4x 1 = 0.
Obs. Nu se consider
a si P5 .
Hexagon:
p
p
2x + 2
3, 4 x2 ,
, 2 x, 4 x2 , x;
x
x ca la P6 .
Obs. Aceeasi ca la P6 .
2
8
Trapez dreptunghic: 3, , 13, .
3
3

Trapez dreptunghic ortogonal: 4,

13, 1, 2.

x+2
Pentagon: 3, 2 1 x,
, 2 1 x, 2;
x
x 0, 89 , 4x3 + x2 3x 1 = 0.
Obs. Nu se considera si P5 .

Trapez dreptunghic circumscriptibil:

5+ 5
5 5
51
, 3,
, 2; x =
.
2
2
2

Trapez dreptunghic: 7/2, 2 2, 3/2, 2.


Obs. Realizat prin alipirea altor triunghiuri
fata de P5 .
Trapez dreptunghic: 3,

20

5, 2, 2.

Trapez dreptunghic variabil:


p
3 x, 4x2 4x + 5, 2 + x, 2; 0 < x < 1/2.
5
5
Dreptunghi: , 2, , 2.
2
2
Obs. Are variante echivalente.

Trapez dreptunghic variabil:


p
x, 4x2 20x + 29, 5 x, 2; 0 < x < 5/2.
Trapez dreptunghic circumscriptibil:

5 5
5+ 5
x=
, 3,
, 2.
2
2

Trapez dreptunghic ortogonal: 1, 13, 4, 2.

Triunghi dreptunghic:

29, 5, 2.

Trapez dreptunghic variabil:


p
4y 2 8y + 29, y, 5, 2 y; 0 < y < 1.
Paralelogram:

Trapez isoscel:

5
5
10, , 10, .
3
3

2
8
10, , 10, .
3
3


Triunghi dreptunghic: 5, 2 5, 5.

Trapez: 4, 2 5, 1, 5.

21

Pentagon: 3,

1 1
, 2 5, , 5.
2
2

Pentagon:

4 3
9+2 3 3 3 6+2 3
,
,
, 2 3,
.
3
3
3
3
Pentagon:

3 3 6+2 3 6 3 4 3
3,
,
,
,
.
3
3
3
3


Pentagon: 3, 2 2, 1, 2, 2.
Obs. Cele dou
a triunghiuri si pot schimba
locurile. Sunt si variante echivalente.
5
5
Dreptunghi: , 2, , 2.
2
2

Paralelogram:

Trapez isoscel:

5 5
, 5, , 5.
2
2
7 3
, 5, , 5.
2
2


5 5
5 3 5
Patrulater inscriptibil: ,
,
, .
2
2
2 2

5
5 3 5 1
Pentagon: ,
, ,
, 2.
2
2
2 2

Hexagon inscriptibil cu axa de simetrie:

1, 2, 1, 3, 1, 2.
Hexagon cu centru de simetrie:

2, 2, 1, 2, 2, 1.

22

Hexagon cu centru de simetrie:

1, 2, 2, 1, 2, 2.

Hexagon inscriptibil cu axa de simetrie:

1, 2, 1, 2, 1, 3.

Pentagon: 3, 1,

5, 1, 2.

Pentagon: 3, 2 2, 2, 3 2, 2,.
Obs. Cele doua truinghiuri si pot schimba
locurile. Sunt si variante echivalente.
Pentagon cu axa de simetrie: 2,
Obs. Are variante echivalente.


2, 2, 2, 2.

Pentagon variabil cu axa de simetrie:


p
p
3x, 4x2 4x + 2, 4x2 4x + 2, 3x, 2;
1/2 < x < 1.
Hexagon cu centru de simetrie:

2, 1, 2, 2, 1, 2.

Obs. Are variante echivalente.


Hexagon inscriptibil cu axa de simetrie:

3, 1, 2, 1, 2, 1.
Obs. Are variante echivalente.
Dreptunghi: 5, 1, 5, 1.
Obs. Are variante echivalente.
Dreptunghi: 5, 1, 5, 1.
Obs. Are variante echivalente.
*
* *
Aceast
a list
a lung
a de poligoane (adres
am cititorilor provocarea de a mai g
asi
altele remarcabile) permite utilizarea ei n diferite moduri adaptate la nivelul cunostintelor celor carora li se adreseaza si la scopurile urmarite de propunator. De exemplu, propunem urmatoarele:
1) S
a se g
aseasc
a doar situatiile care dau triunghiuri sau patrulatere (eventual
23

particulare: trapeze s. a.)


2) n ce situatie se obtine figura cu perimetrul maxim sau minim?
3) Ce tipuri distincte de poligoane se pot obtine? (triunghi dreptunghic, dreptunghi, paralelogram, trapez, trapez isoscel, trapez dreptunghic, trapez circumscriptibil, trapez ortodiagonal, patrulater inscriptibil, pentagon, pentagon cu ax
a de simetrie, hexagon, hexagon cu centru de simetrie, hexagon cu ax
a de simetrie; s-au luat
n considerare doar particularit
atile clasice nelund n considerare particularit
ati
ca: pentagon cu doua unghiuri drepte s. a.).
4) Care t
aietur
a d
a cel mai mare num
ar de variante echivalente? (adic
a s
a se
obtin
a acelasi poligon; de exemplu T20 d
a 16 variante c
aci dac
a not
am triunghiurile
decupate prin I cel de sus si II cel de jos, respectiv prin F (fata) si V (verso), atunci
ele pot fi asezate pentru a forma pentagonul n cele doua pozitii S (sus) si J (jos)
astfel:
S : IF
IF
IV
IV IIF IIV IIF IIV
J : IIF IIV IIF IIV IF
IF
IV
IV
si toate aceste combinatii nc
a o dat
a num
arate dac
a ntoarcem ntreg pentagonul pe
verso.
5) Daca se considera o singura fata a cartonului care situatii nu se pot realiza?
(De exemplu T2 cu P4 ).
6) Care situatii duc la poligoane congruente? (De exemplu P35 , P39 , P45 ).
7) De ce nu se pot realiza poligoane convexe cu mai mult de sase laturi?
8) Rezolvarea aproximativa a ecuatiilor de gradul 3 si 4 care au aparut. (Acestea
pot constitui un pretext pentru a prezenta la o activitate suplimentar
a formulele lui
Cardano, respectiv Ferrari).
9) Relativ la taieturi: care este cea mai mica? cea mai mare? cea care mparte
poligonul n doua bucati echivalente (de aceeasi arie)?
*
* *
n ncheiere sa revenim la problema nitiala adaptnd-o la multimea de situatii de
mai sus si anume:
Se poate ca printr-o singur
a t
aietur
a n linie dreapt
a s
a realiz
am din figura initial
a
un p
atrat?
Raspunsul este afirmativ doar daca ndoim n prealabil cartonul asa cum este indicat mai jos (dupa bisectoarea unghiului drept format de cele doua taieturi prezentate
n solutia sa).

MATEDar aceast
a problem
a cu ndoire si t
aiere poate initia o alt
a RECREATIE

MATICA.

24

Asupra unor perechi de siruri liniar recurente


1

D. M. BATINE
TU-GIURGIU

n aceasta nota matematica vom evidentia proprietatile unor perechi de siruri,


fiecare satisf
acnd o anumit
a recurenta liniar
a omogen
a de ordinul al doilea cu coeficienti constanti.
Spunem ca un sir (xn )n0 de numere reale, satisface o recurenta liniara (reala),
omogena de ordinul al doilea, daca exista a, b R, b 6= 0 si exista k N astfel
nct:
xn+2 + axn+1 + bxn = 0, n k.
(1)
n functie de valorile coeficientilor a, b R si de conditiile initiale xk = u R,
xk+1 = v R se obtin diferite siruri, dintre care unele cunoscute si de elevii de liceu.
De exemplu, dac
a a = b = 1, k = 0 se obtine recurenta:
(2)
xn+2 xn+1 xn = 0, n N,
pe care o vom numi recurenta Fibonacci-Lucas.
Daca n recurenta (2) consideram x0 = F0 = 0, x1 = F1 = 1, xn = Fn se obtine
sirul lui Fibonacci care satisface recurenta:
(3)
Fn+2 = Fn+1 + Fn , n N.
Dac
a n recurenta (2) consider
am x0 = L0 = 2, x1 = L1 = 1, xn = Ln se obtine
sirul lui Lucas, sir care satisface recurenta:
Ln+2 = Ln+1 + Ln , n N.
(4)
Daca n (1) luam a = 2, b = 1, k = 1 se obtine recurenta liniara:
xn+2 2xn+1 + xn = 0, n N xn+2 xn+1 = = x2 x1 = r R, n N ,
(5)
numit
a recurenta progresiilor aritmetice de ratie r R.
n fine, dac
a n recurenta (1) lu
am a = 2, b = 1, k = 0 obtinem recurenta
liniara cu coeficienti constanti de tip Pell :
xn+2 2xn+1 xn = 0, n N.
(6)
Dac
a n recurenta (6) consider
am x0 = P0 = 0, x1 = P1 = 1, xn = Pn se obtine
sirul lui Pell, care satisface recurenta:
Pn+2 = Pn + 2Pn+1 , n N.
(7)
De asemenea, daca n (6) luam x0 = Q0 = 1, x1 = Q1 = 1, xn = Qn obtinem
sirul lui Pell asociat (Qn )n0 , sir care satisface recurenta:
(8)
Qn+2 = 2Qn+1 + Qn , n N.
Mai departe, vom enunta si demonstra unele propozitii care scot n evidenta,
anumite propriet
ati pe care le verific
a unele perechi formate dintr-un sir care verific
a
recurenta (2) si un sir care verifica recurenta (5).
Propozitia 1. Daca (xn )n0 satisface recurenta (2) n care x0 = c R+ ,
x1 = d R+ , iar (un )n1 este o progresie aritmetica de ratie r R, atunci:
n
X
uk xk = un xn+2 r (xn+3 x4 ) x2 u1 , n N .
(9)
k=1

Profesor, Colegiul National Matei Basarab, Bucuresti

25

Demonstratie. Vom demonstra afirmatia prin metoda inductiei matematice.


Pentru n = 1 relatia (9) devine u1 x1 = u1 x3 r (x4 x4 ) x2 u1 x2 u1 =
= u1 (x3 x1 ) = u1 x2 ceea ce arat
a c
a pentru n = 1, enuntul este adev
arat.
Pentru n = 2 relatia (9) devine u1 x1 + u2 x2 = u2 x4 r (x5 x4 ) x2 u1
u2 (x4 x2 ) = u1 (x1 + x2 ) + r (x5 x4 ) u2 x3 = (u1 + r) x3 = u2 x3 de unde se
deduce c
a enuntul este adev
arat si pentru n = 2.
Presupunem c
a enuntul este adev
arat pentru n 2, (adic
a relatia (9) este verificata) si sa demonstram ca ea este verificata si pentru n + 1. Avem de aratat ca:
n+1
X
uk xk = un+1 xn+3 r (xn+4 x4 ) x2 u1 .
(10)
k=1

ntr-adevar, relatia (10) este echivalenta cu


n
X
uk xk + un+1 xn+1 = un+1 xn+3 r (xn+4 x4 ) x2 u1
k=1

relatie care (n conditiile verificarii conditiei (9)) este echivalenta cu:

un xn+2 r (xn+3 x4 ) x2 u1 + un+1 xn+1 = un+1 xn+3 r (xn+4 x4 ) x2 u1


un+1 (xn+3 xn+1 ) = un xn+2 + r (xn+4 xn+3 )
xn+2 un+1 = un xn+2 + rxn+2 = (un + r) xn+2 = un+1 xn+2
de unde (daca tinem seama ca xn R+ , n N ) deducem ca relatia (10) este adevarata. Conform principiului inductiei matematice rezulta ca enuntul este adevarat
pentru orice n N si astfel propozitia este demonstrat
a.
Observatie. Dac
a x0 = 0 = F0 , x1 = 1 = F1 iar (un )n1 este o progresie
aritmetica de ratie r din relatia enuntului deducem ca
n
X
uk Fk = un Fn+2 r (Fn+3 F4 ) u1 ,
k=1

adic
a am obtinut Problema C:2310 propus
a de Florin Rotaru n G.M.-9/2000,
p.360. Dac
a aici lu
am r = 0 si un = 1, n N deducem c
a sirul lui Fibonacci
verifica relatia
n
X
Fk = Fn+2 1, n N .
k=1

Propozitia 2. Daca (xn )n0 este un sir de numere reale strict pozitive care
satisface recurenta (2) iar sirul (un )n1 are proprietatea ca exista r R astfel
nct
n
X
uk xk = un xn+2 r (xn+3 x4 ) x2 u1 , n N ,
(11)
k=1

atunci sirul (un )n1 este o progresie aritmetica de ratie r.


Demonstratie. Vom face si aici demonstratia prin metoda inductiei matematice.
Pentru n = 2 relatia enuntului devine:

u1 x1 +u2 x2 = u2 x4 r (x5 x4 )x2 u1 u1 x1 +r (x4 + x3 x4 )+u1 x2 = u2 (x4 x2 )


u1 (x1 + x2 ) + rx3 = u2 x3 u1 x3 + rx3 = u2 x3 (u1 + r) x3 = u2 x3
de unde dac
a tinem seama c
a xn > 0, n N deducem c
a u2 = u1 + r.
26

Presupunem c
a un = un1 + r, n 2 si s
a demonstr
am c
a un+1 = un + r.
ntr-adev
ar, pentru n + 1 relatia (11) se scrie:
n+1
X
k=1

n
X

k=1

uk xk = un+1 xn+3 r (xn+4 x4 ) x2 u1

uk xk + un+1 xn+1 = un+1 xn+3 r (xn+4 x4 ) x2 u1

n care dac
a tinem seama de relatia (11) obtinem:
un xn+2 r (xn+3 x4 ) x2 u1 + un+1 xn+1 = un+1 xn+3 r (xn+4 x4 ) x2 u1

un+1 (xn+3 xn+1 ) = un xn+2 + r (xn+4 xn+3 ) un+1 xn+2 = un xn+2 + rxn+2
de unde prin simplificare cu xn+2 > 0, n N deducem ca un+1 = un + r.
Conform principiului inductiei matematice rezult
a c
a un+1 = un + r, n N
ceea ce arat
a c
a (un )n1 este o progresie aritmetic
a de ratie r.
Propozitia 3. Daca (un )n1 este o progresie aritmetica de ratie r > 0, u1 > 0
iar (xn )n0 este un sir de numere reale astfel nct x0 0, x1 > 0, x2 = x1 + x0 si
daca
n
X
uk xk = un xn+2 r (xn+3 x4 ) x2 u1 , n N ,
(12)
k=1

atunci xn+2 = xn+1 + xn , n N.


Demonstratie. Proced
am si acum prin inductie matematic
a. Conform enuntului pentru n = 0, avem x2 = x1 + x0 . Pentru n = 1 relatia (12) devine u1 x1 =
u1 x3 r (x4 x4 ) x2 u1 u1 x1 + u1 x2 = u1 x3 de unde, dac
a tinem seama c
a
un > 0, n N , obtinem c
a x3 = x2 + x1 , adic
a afirmatia enuntului este adev
arat
a
si pentru n = 1. Presupunem c
a xk+2 = xk+1 + xk , k = 0, n si s
a demonstr
am c
a
xn+3 = xn+2 + xn+1 .

(13)

Dac
a n (12) nlocuim n cu n 1 deducem c
a
n1
X
k=1

uk xk = un1 xn+1 r (xn+2 x4 ) x2 u1 , n 2.

(14)

Daca n (12) tinem seama de (14) obtinem ca


un xn +

n1
X
k=1

uk xk = un xn+2 r (xn+3 x4 ) x2 u1

un xn + un1 xn+1 r (xn+2 x4 ) x2 u1 = un xn+2 r (xn+3 x4 ) x2 u1


un (xn + xn+1 xn+2 ) + r (xn+3 xn+2 xn+1 ) = 0,
dar xn + xn+1 xn+2 = 0 n baza ipotezei de inductie si deci r
amne
r (xn+3 xn+2 xn+1 ) = 0 xn+3 = xn+2 + xn+1 .

Conform principiului inductiei matematice rezulta ca xn+2 = xn+1 + xn , n N si


astfel propozitia este demonstrata.
Bibliografie
Editura Albatros, Bucuresti, 1979.
1. M. D. B
atinetu - Siruri,

2. Gazeta Matematica, Colectia 1895-2001.


27

Asupra unei probleme de constructie


Anca TIMOFTE, Alexandru T
URCANU 1
Punctul de plecare al acestei note a fost una dintre problemele propuse spre
rezolvare absolventilor clasei a VII-a n cadrul Concursului "Recreatii Matematice" din 27 august 2002. Enuntul acestei probleme este urm
atorul:
Fie dat un segment [M N ]. Construiti cu rigla si compasul un patrat ABCD
astfel nct M [AB], AM = M B, iar N [AC], AN = 3N C. (Descrieti toate
constructiile care trebuie efectuate.) (Gabriel Popa)
Prezent
am n continuare solutia dat
a de autorul problemei, asa cum a reiesit din
baremul de corectare:
A
D
Solutia 1. Sa presupunem problema rezolvata si fie P
mijlocul segmentului [BC]. Atunci N P kBD, P M kAC (ca
\
linii mijlocii), deci N P M P si m(M
P B) = 45 . Daca a
a 2
M
este lungimea laturii patratului, atunci M P =
, NP =
2

N
a 10
1
a 2
, deci M N =
, de unde N P = M N .
=
4
4
5
B
C
Constructia. Vom lua ca unitate un segment u de
P
lungime egala cu cea a segmentului [M N ].

u 5
u 5
5u

u
u

x=

1
u
5

u
1
Ca n figurile de mai sus, construim un segment de lungime u. Intersectam
5
1
cercul de diametru [M N ] cu cercul de centru N si raza u, obtinnd punctul P .
5
Construim triunghiul dreptunghic isoscel de ipotenuz
a [M P ] si afl
am astfel vrful B
al patratului. Apoi, A si C sunt simetricele lui B fata de M , respectiv P . Vrful D
se obtine ca intersectie a paralelelor duse prin A si C la BC, respectiv AB.
Demonstrarea faptului c
a ABCD astfel determinat este p
atrat cu propriet
atile
dorite, este imediat
a. Evident c
a problema are solutie, unic
a pn
a la o izometrie a
planului.
Vom da mai jos nc
a dou
a solutii ale acestei probleme. Prima are avantajul c
a nu
foloseste nici un punct auxiliar; este ns
a necesar
a o bun
a cunoastere a constructiilor
1

Elevi, S
coala nr.7 "Octav B
ancil
a", Botosani

28

cu rigla si compasul. A doua are la baz


a un rationament mai elaborat, dar utilizeaz
a
numai constructii la nivelul manualelor.
Solutia 2. Pe figura si notatiile din prima solutie, aplic
am teorema cosinusului
n 4AM N :
MN2

deci a =

2 10
MN.
5

\
= AM 2 + AN 2 2AM AN cos(M
AN ) =
!2

a 2
3a 2
5a2
a 3a 2
2
+
=

=
,
2
2
4
2
4
2
8

2 10
u,
Rezulta urmatoarea constructie: determinam un segment de lungime a =
5
unde u = M N . Vrful A al patratului este la intersectia arcului capabil de 45
a
construit pe [M N ] drept coard
a, cu cercul de centru M si raz
a . Afl
am apoi B ca
2
fiind simetricul lui A fata de M etc.
D
Solutia 3. Presupunem problema rezolvata
A
si aplic
am teorema lui Menelaus n 4ABC cu
transversala M N P ; obtinem:
AM BP CN
M

=1
MB P C NA
BP
T
N

= 3 BC = 2P C.
PC
Aplicnd acum Menelaus n 4M N P cu transver- B
P
C
sala C N A, g
asim:
MN
P C BA M N

=1
= 1 M N = N P.
CB AM N P
NP
\.
\ CDP
Sa observam ca 4DAM 4DCP (C.C.), de unde M D = DP si ADM
Ultima relatie arat
a c
a
\
\
\ ) = m(M
\
\ ) = m(ADC)
\ = 90 ,
m(M
DP ) = m(M
DC) + m(CDP
DC) + m(ADM
asadar 4M DP este dreptunghic isoscel. Fie {T } = M P CD; teorema fundamentala
PT
PC
1
a semanarii aplicata n 4P BM cu CT kBM arata ca
=
= .
PM
PB
3
Constructia. Afl
am P ca simetric al lui M fata de N . Intersect
am cercul de
diametru [M P ] cu mediatoarea acestui segment, determinnd vrful D al p
atratului.
PT
1
Afl
am punctul T [M P ] care mparte segmentul n raportul
= , apoi fie C
PM
3
intersectia dreptei DT cu semicercul de diametru [DP ] aflat n semiplanul delimitat
de dreapta DP ce contine punctul N . Vrfurile A si B ale patratului se construiesc
acum cu usurinta.
Observatie. Problema se poate generaliza considernd c
a punctele M si N sunt
luate astfel nct AM = m M B si AN = n N C.

29

Cteva aplicatii ale teoremei lui Casey


Marius PACHI T
ARIU 1
O generalizare remarcabil
a a teoremei lui Ptolemeu este teorema lui Casey. Se
numeste distanta tangentiala dintre cercurile C1 si C2 , notata d12 , lungimea tangentei
lor comune exterioare.
Teorema lui Casey. Daca cercurile C1 , C2 , C3 , C4 sunt tangente (toate interior sau toate exterior) la cercul C, ordinea punctelor de tangenta fiind data de
numerotarea acestor cercuri, atunci are loc relatia:
d12 d34 + d23 d41 = d13 d24 .

Rezultatul r
amne adev
arat dac
a cercurile Ci (toate sau o parte din ele) degenereaz
a n puncte sau dac
a cercul C devine dreapt
a.
Aplicatia 1. Fie ABC un triunghi nscris n cercul C si cercurile C1 , C2 , C3
tangente la C interior precum si laturilor (BC), (CA) si respectiv (AB) astfel nct
A si C1 , B si C2 , C si C3 sa fie de parti diferite fata de BC, CA, respectiv AB.
Notam cu l1 , l2 , l3 lungimile tangentelor din A, B, C la cercurile C1 , C2 , respectiv C3 .
Are loc echivalenta:
b+c
c+a
a+b
, l2 =
, l3 =
.
d12 = d23 = d31 l1 =
2
2
2
A
Solutie. Observam mai nti ca C1 , C2 , C3 sunt tangente
C
la laturi n mijlocul acestora. Aplicnd teorema lui Casey
pentru cercurile C si A, C2 , C1 , C3 ; C si B, C3 , C2 , C1 ; C si
C3
C, C1 , C3 , C2 , obtinem:
B
b
c
c
a
a
b
C2
d13 + d12 = l1 d23 , d12 + d23 = l2 d13 , d23 + d13 = l3 d12 .
2
2
2
2
2
2
Din acestea, rezult
a imediat implicatia "". Invers, dup
a
C1
C
b+c
nlocuirea lui l1 cu
etc., aceste relatii se scriu:
2
b (d13 d23 ) = c (d23 d12 ) , c (d12 d13 ) = a (d13 d23 ) , a (d23 d12 ) = b (d12 d13 ) ,
d12 d13
d23 d12
d13 d23
0
i.e.
=
=
=
a
b
c
a+b+c
(suma num
ar
atorilor fiind nul
a). Deducem c
a d12 d13 = 0, d23 d12 = 0, d13 d23 = 0,
deci d12 = d23 = d31 , q.e.d.
b = 60 . Fie
Aplicatia 2. Fie ABC un triunghi nscris n cercul C si cu m(A)
0
0
0
C (O , R ) cercul tangent la C interior si la laturile [AB] si [AC]. Sa se arate ca
4
A
R0 = r, unde r este raza cercului nscris n triunghiul dat.
3
C
Solutie. Fie {X} = AB C 0 , {Y } = AC C 0 si l = AX =
C
b = 60 ).
= AY = XY (4AXY este echilateral, caci m(A)
X
Relativ la C si cercurile A, C, C 0 , B aplic
am teorema lui
Y
Casey:
O'
B
C
b (c l) + c (b l) = al,
1

Elev, cl. a IX-a, Colegiul National, Iasi

30

2bc
bc
bc
2bc
4
a parte,
=
= r=
r = r. Pe de alt
a+b+c
p
S
bc sin A
3
XY
l
4
R0 = O 0 X =
= . Ca urmare, R0 = r.

2 sin 60
3
3
Aplicatia 3. Cercurile Ci (Oi , ri ), i = {1, 2, 3, 4} sunt tangente (n ordinea numerotarii) la cercul C (O, r) si, pentru orice i {1, 2, 3, 4}, Ci este tangent la Ci1
si Ci+1 ( C1 fiind C4 , iar C5 fiind C1 ). Atunci, n conditia ca punctele O, O1 , O3
ct si O, O3 , O4 sunt coliniare, avem:
a) 4r1 r2 r3 r4 = (r r1 ) (r r2 ) (r r3 ) (r r4 ), daca Ci sunt tangente interior
la C;
b) 4r1 r2 r3 r4 = (r + r1 ) (r + r2 ) (r + r3 ) (r + r4 ), daca Ci sunt tangente exterior
la C.
Solutie. Se stabileste usor c
a dou
a cercuri de raze a
O1
si b tangente exterior au
lungimea

a
tangentei comune

exterioare data de d = 2 ab. Ca urmare, teorema lui Casey


ne conduce la relatia:

O O2
O4
2 r1 r2 2 r3 r4 + 2 r2 r3 2 r1 r4 = d13 d24 .
de unde obtinem c
al=

Datorit
a coliniarit
atii punctelor O, O1 O3 , avem d213 =
O3
2
= (2r r1 r3 ) (r1 r3 )2 , adica d213 = 4 (r r1 ) (r r3 );
2
analog d24 = 4 (r r2 ) (r r4 ). nlocuind n relatia precedent
a obtinem formula de la punctul a). Punctul b) se dovedeste n mod asem
an
ator.
Aplicatia 4. Fie dat un cerc C si pe el punctele A si B. De o parte si de alta
a dreptei AB consideram cercurile C1 , C2 tangente interior la C si tangente coardei
[AB] n punctele X si respectiv Y . Sa se determine pozitia punctelor X si Y pe AB
1
pentru care d12 = AB.
2
Solutie. Cu teorema lui Casey aplicat
a lui C si cerA
curilor C1 , A, C2 , B, obtinem AX BY +AY BX = AB d12 .
d12
Conditia din enunt este echivalenta cu
X
1
AX BY + AY BX = AB AB
C1
2
Y
2AX BY + 2AY BX = (AX + BX) (AY + BY )
C2
C
AX BY + AY BX AX AY BX BY = 0
B
(AX BX) (BY AY ) = 0 AX = BX sau AY = BY,
adica unul dintre punctele X si Y trebuie sa fie mijlocul coardei [AB] (celalat putnd
fi oriunde pe [AB]) pentru a fi ndeplinit
a conditia problemei.

Bibliografie.
1. M. Dr
agusin - Despre utilitatea unui rezultat prea putin folosit: teorema lui Casey,
G.M. 12/1995, 716-720.
2. N. Roman - Asupra unor probleme date la O.I.M., G.M. 3/2000, 99-102.

31

Extinderi de inele si corpuri o posibil


a lectie de recapitulare final
a
Dumitru GHERMAN 1
Pentru ca recapitularea sa aiba eficienta, trebuie ca n organizarea ei sa se tina
de unele principii:
la recapitulare nu se parcurge din nou ntreaga materie;
trebuie sa se urmareasca, pe ct este posibil, realizarea unei legaturi ntre diversele ramuri ale matematicii scolare;

recapitularea trebuie s
a aduc
a elemente noi, probleme care pot fi rezolvate prin
prelucrarea creatoare a cunostintelor anterioare;
se are n vedere stimularea lucrului individual al elevului, folosind bibliografia
indicata de profesor si / sau cautnd noi surse;
recapitularea trebuie s
a tin
a cont de structura si cerintele examenelor scolare.

n cele ce urmeaz
a, vom prezenta un proiect didactic pentru o posibil
a lectie de
recapitulare finala la clasa a XII-a. Nu ne propunem sa rezolvam toate problemele
sau sa demonstram toate teoremele ce vor aparea; majoritatea apartin fondului clasic
si poate fi consultat
a bibliografia.
I. Inelul ntregilor p
atratici. Fie d un num
ar ntreg liber de p
atrate; definim
o
h i n

Z d = x C | x = m + n d , m, n Z .
h i

1) Z d ; +, este un subinel al corpului numerelor complexe, chiar domeniu


de integritate.

h i
m n
2) Z d este izomorf cu inelul matricelor de forma
, m, n Z, n
dn m
raport cu operahtiile iuzualehcu matrice.

i
a si numai dac
a d = d 0 (se arat
3) Inelele Z d si Z d 0 sunt izomorfe dac
a
n primul rnd c
a un izomorfism f ntre celedou
a inele invariaz
a elementele lui Z si

atunci el este bine determinat de valoarea f
d ).
h i
4) Subinelele unitare ale lui Z d sunt de forma
o
n

An = a + bn d | a, b Z , n N.
h i


5) Definim aplicatia norma N : Z d Z, N m + n d = m2 d n2 . Dac
a

notam cu x
= m n d conjugatul ntregului patratic x = m + n d , se arata ca N
are propriet
an
atoare modulului: N (x) = x x
, N (xy) = N (x) N (y). De
atihasem
i
aici, x U Z d
N (x) U (Z) = {1}.
6) Grupul multiplicativ al elementelor inversabile din Z [i] este U (Z [i]) = {1, i}.
1

Profesor, Liceul Teoretic Mihail Sadoveanu, Pascani

32

7) Fie C\Q astfel nct multimea A = {m + n | m, n Z} este inel fata


de operatiile uzuale din C. Dac
a A are exact patru elemente inversabile, atunci
A = Z [i].
8) Daca d {2, 3, 5}, atunci U (Z [d ]) contine o infinitate de elemente si putem
g
asi n U (Z [d ]) elemente pozitive orict de mici (este suficient s
a g
asim un singur
element, considernd apoi puterile acestuia si conjugatele lor).
Problemele 1-6 sunt rezolvate n [3]; problema 7 a fost propus
a de Marcel Tena

la etapa finala a Olimpiadei de Matematica n 1997, iar 8 poate fi gasita n variantele


examenului de bacalaureat din ultimii ani.
II Corpul numerelor p
atratice. Fie d un numar ntreg liber de patrate;
definim
n
o

Q
d = z C | z = a + b d, a, b Q .

1) Q
d ; +, este subcorp al lui C (inversul elementului nenul a + b d



1
a

b
d
d , deoarece a2 db2 6= 0; altfel, se ajunge la
este 2

Q
2
a

db

a
d=
/ R\Q!).
b

a b
d este izomorf cu multimea matricelor de forma
, a, b Q, care
2) Q
db a
formeaz
a corp n raport
cu opera
uzuale.
tiile
3) Corpurile Q
a si numai dac
a d = d0 ; sind si Q
d0 sunt izomorfe dac

gurele automorfisme ale corpului Q
d sunt aplicatia identica si cea de conjugare,
ambele invariind elementele lui Q.
4) Daca un subcorp K C este astfel nct End K = {f, g} si f (x)= g(x)

x Q, atunci exist
a un ntreg liber de p
atrate d 6= 1 pentru care K = Q
d .

5) Dac
a f Q [x], atunci f (
z ) = f (z), z Q
d ; de aici urmeaz
a c
a

orice polinom cu coeficienti rationali, are eventualele radacini din Q
d n perechi
conjugate.
Problemele 1-3 pot fi gasite n [3], problema 4 a fost propusa la etapa finala a
Olimpiadei de Matematic
a din 1988 de c
atre Marcel Tena,

iar 5 poate fi rezolvat


a
urmnd pas cu pas demonstrarea unor rezultate analoage din manuale.

III Extinderi p
atratice.Corpuri pitagorice. Fie r Q+ astfel nct r
/ Q;
definim camai sus corpul Q ( r), care este subcorp al lui R din pozitivitatea
lui
r.

1) Q ( r) este cel mai mic


subcorp
al
lui
R
care
include
Q
{
r}.

2) Putem gndi
pe Q ( r) ca un Q-spatiu vectorial, definind nmultirea "vectorilor"
din Q ( r) cu "scalari" din Q prin restrictionarea nmultirii obisnuite din
a fiind
Q ( r) (de fapt, din R). Dimensiunea acestui spatiu vectorial este 2, o baz
asi si alte baze!).
{1, r} (a se g
f = X 2 r Q [X] este ireductibil peste Q, dar

admite radacina
3) Polinomul
r n Q ( r); orice alt polinom g Q [X] care
admite
r
a
d
a
cina
r se divide prin f .

Spunem c
a f este polinomul minimal al lui r.
33


d se face utiliznd
4) Din puncte de vedere geometric, extinderea lui Q la Q

compasul. De exemplu, r este abscisa unuia dintre punctele de intersectie ale


1
1
cercului de centru O si raz
a (r + 1) cu dreapta y = (r 1).
2
2

r2 , . . .
5) Considernd
r1 , r2 , Q+ , definim Qr1 = Q r1 , Qr2 = Qr1
rn , . . . ; spunem ca am adjunctionat la Q, pe rnd, elementele
, Qrn = Qrn1

r1 , r2 , . . . , rn , . . . . Am construit astfel sirul de extinderi


Q Qr1 Qr2 Qrn R.

Cum Q este multime num


arabil
a, putem alege r1 , r2 , . . . , rn , . . . astfel nct orice
num
ar obtinut, pornind de la Q, prin efectuarea unui num
ar finit de adun
ari, sc
aderi,
nmultiri, mpartiri si extrageri de radacini patrate (un astfel[
de numar se numeste
numar pitagoric) s
a apartin
a unui anumit Qrn . Not
am K =
Qrn - multimea nun

merelor pitagorice; se arata ca aceasta nu depinde de alegerea sirului r1 , r2 , . . . , rn , . . .


si ca formeaza un subcorp al lui R, nchis la operatiile aritmetice si la extragerea
r
ad
acinii p
atrate si care este "cel mai mic" (n sensul incluziunii) cu aceste propriet
ati.
IV Constructii cu rigla si compasul.
1) Daca L este un subcorp al lui R iar (D) este o drepta ce trece prin doua puncte
avnd coordonatele n L L, atunci ecuatia dreptei are coeficienti din L. Analog,
un cerc care are centrul de coordonate din L L si trece printr-un astfel de punct,
are coeficientii ecuatiei sale din L.
2) Fie L subcorp n R, iar M (x, y) un punct n plan; spunem ca M este constructibil cu rigla si compasul plecnd de la L dac
a el poate fi obtinut prin intersectii
de drepte si cercuri avnd coeficientii n L. Dac
a M este
un astfel de punct, atunci
fie x, y L, fie exist
a u L, u > 0 astfel nct x, y L ( u).
3) Numim numar constructibil cu rigla si compasul un numar real ce este coordonat
a a unui punct constructibil. Se arat
a c
a orice num
ar real constructibil este
totodat
a si num
ar pitagoric. Ca o consecinta, polinomul minimal al unui num
ar constructibil are gradul putere a lui 2. De aici rezult
a imposibilitatea dublarii cubului,
trisectiei unghiului si cuadraturii cercului (pentru amanunte, v.[1],[2],[4]).
4) Corpul ordonat K R se bucur
a de un anumit tip de completitudine, numit
a completitudine euclidiana : cercetat cu rigla si compasul, nu se va putea spune
niciodat
a c
a lipseste vreun punct. Tocmai confuzia dintre aceast
a completitudine
si completitudinea Cantor - Dedekind a lui R a ntrziat solutionarea problemelor
clasice ale antichit
atii.
Bibliografie
1. T. Brsan - Trisectia unghiului, Recreatii Matematice, 2/2001, 38-41.
2. E. Moise - Geometrie elementara dintr-un punct de vedere superior, E.D.P., Bucuresti, 1980.
3. C. Nita
, T. Spircu - Probleme de structuri algebrice, Ed. Tehnica, Bucuresti, 1974.
4. I. Tofan, C. Volf - Algebra - Inele, Module, Teorie Galois, Ed. Matrix-Rom, Bucuresti, 2001.
34

Metode si procedee de rezolvare a problemelor


de maxim sau de minim
Gheorghe CROITORU 1
Ne propunem n cele ce urmeaza sa prezentam, prin exemple, o serie de procedee
si metode prin care pot fi solutionate problemele de maxim sau de minim. Rezolvarea
pe mai multe c
ai a unei aceleiasi probleme va permite cititorului s
a compar
a eficienta
acestora, precum si s
a aleag
a contextul cel mai potrivit pentru aplicarea uneia sau
alteia dintre ele.
a3
Problema 1. Sa se afle minimul expresiei E (x) = x2 +
, x R+ , unde
x

a R+ este dat.
Solutia 1. Aplicnd inegalitatea mediilor, obtinem c
a
r
a3 a3
a3 a3
3

3
3
3
x

3
x
+
+
3
3
2
x +a
2 2 = 3 2a ,
2
2
E (x) =
=
x
x
x
2
3
a
a
egalitatea fiind atins
a atunci cnd x3 =
. Urmeaz
a c
a Emin =
, i.e. x =
3
2
2

3 3 2a2
=
.
2
Solutia 2. Se stie c
a, dac
a x, y R+ si suma x + y = const, atunci produsul
x
y
m n

x y (m, n N ) este maxim pentru


= ; dual, dac
a xm y n = const, atunci
m
n
x
y
suma x + y este minima pentru
= (aceste afirmatii se extind la un numar finit
m
n
de termeni / factori si la cazul n care exponentii sunt din Q+ ).
3 2
a
= a6 = const, urmeaza ca E (x) are valoare minima atunci
ntruct x2
x

3 3 2a2
a3
a
a
2
=
. Se obtine Emin = E
cnd x =
x=
.
3
3
2x
2
2
2
Solutia 3. Fie functia f : (0, ) R,
a

x 0

f (x) = E (x). Extremele acestei functii se pot


3
2
g
asi folosind prima derivat
a. Avem c
a f 0 (x) =
0
f
0
+ +
2x3 a3
a
f

&
E
%

=
, care se anuleaza pentru x =
.
min
3
x2
2
Tabelul de variatie este prezentat al
aturat.
sin x 3
Problema 2. Aflati valorile extreme ale expresiei E (x) =
, x R.
cos x + 2
(Paul Georgescu, Gabriel Popa, Problema 24739, G.M. 9/2002)
x
a E =
Solutia 1. Pentru x 6= (2k + 1) , k Z, notnd t = tg , obtinem c
2
2
2
3t + 2t 3
3t + 2t 3
, t R. Pentru a afla multimea valorilor lui E, fie y =
,
=
t2 + 3
t2 + 3
t R, deci t2 (y + 3) 2t + (3y + 3) = 0, unde t R. Se impune conditia 0, ceea
1

Profesor, Liceul Teoretic Al. I. Cuza, Iasi

35

"

#
2 3
2 3
ce conduce la y 2
, 2 +
. Pe de alt
a parte, E ((2k + 1) ) = 1
3
3

2 3
2 3
2 3
2 3
, 2 +
, iar Emax = 2 +
2
. Urmeaz
a c
a Emin = 2
3
3
3
3
(se dovedeste usor faptul ca aceste valori extreme sunt efectiv atinse).
Solutia 2. Definim punctele M (cos x, sin x), A (2, 3); atunci E (x) este tocmai
panta dreptei AM . Pentru x R, M parcurge cercul trigonometric si cum A este n
exteriorul acestui cerc, urmeaz
a c
a valorile extreme ale lui E (x) sunt atinse atunci
cnd AM este una dintre tangentele duse din A la cerc. Fie d : y 3 = m (x + 2)
ecuatia unei drepte prin A; aceasta este tangent
a la C (0, 1) cnd dist (O, d) = 1.
Folosind formula care d
a distanta de la un punct la o dreapt
a, obtinem
(
)
2 3
|2m + 3|
2

= 1 3m + 12m + 8 = 0 m 2
.
3
m2 + 1

2 3
2 3
n concluzie, Emin = 2
, iar Emax = 2 +
.
3
3
Solutia 3. Putem evident utiliza derivata nti n studiul functiei atasate expresiei E (x).
Problema 3. Dintr-o bara metalica de forma cilindrica se obtine prin strunjire o
bara paralelipipedica. Sa se determine dimensiunile dreptunghiului de sectiune astfel
nct pierderea de material sa fie minima.
Solutia 1. Notnd cu x si y dimensiunile dreptunghiului si cu R raza cilindrului,
problema revine la a g
asi maximul functiei f (x, y) = xy, n conditiile x, y > 0,
x2 + y 2 = 4R2 . nsa, cum x, y sunt pozitive, produsul xy este maxim odata cu
produsul x2 y 2 . Deoarece suma x2 + y 2 este constanta, x2 y 2 este maxim pentru
x2 =
autat sunt, prin urmare, x = y = R 2.
= y 2 = 2R2 . Dimensiunile dreptunghiului c
2
2
2
Solutia 2. Studiul func
tiei f (x, y) = xy pentru x, y > 0, x + y = 4R revine la
studiul functiei g(x) = x 4R2 x2 , x (0, 2R) si acesta se face apelnd la derivata g 0 .
Solutia 3. Cnd se cauta extremele unei functii f (x, y), ntre variabile existnd
o leg
atur
a de forma (x, y) = 0, se aplic
a n general metoda multiplicatorilor lui
Lagrange. Dac
a f, sunt de clas
a C 1 , consider
am functia auxiliar
a
F (x, y) = f (x, y) + (x, y) ,

R.

Punctul (x0 , y0 ) din domeniul lui f este punct de extrem al acestei functii daca si
numai dac
a (x0 , y0 ) este solutie a sistemului
Fx0 (x, y) = 0,
0x

Fy0 (x, y) = 0,

(x, y) = 0,

0y

(x0 , y0 ) = 0,
(x0 , y0 ) = 0 nu sunt satisfacute simultan.
iar egalitatile

n cazul nostru, F (x, y) = xy + x2 + y 2 4R2 si avem de rezolvat sistemul


y + 2x = 0,

x + 2y = 0,

x2 + y 2 4R2 = 0,

neadmitnd solutiile pentru care 2x = 2y = 0. Obtinem imediat c


a x = y = R 2.
Problema 4. Doua orase A, B sunt situate respectiv la 10 km si 15 km de un
ru rectiliniu, iar proiectia lungimii AB pe directia rului este de 20 km. Cele doua
orase trebuie alimentate cu apa de la o uzina amplasata pe marginea rului. Se cere
36

pozitia uzinei pentru care lungimea conductelor ce o leaga de cele doua orase sa fie
minima.
Solutia 1. Dac
a A00 este simetricul lui A fata de directia rului, iar M este pozitia
uzinei, evident ca [AM ] [A00 M ], deci AM + M B = A00 M + M B. Aceasta din urma
sum
a este minim
a cnd A00 , M, B sunt puncte coliniare; punctul M de amplasare
a uzinei astfel obtinut se caracterizeaz
a prin congruenta unghiurilor , f
acute de
M A, respectiv M B cu normala la directia rului (v. figura 1).
Notnd x = A0 M , din 4M AA0 4M BB 0 obtinem ca
AA0
10
A0 M
x
=
=
x = 8 (km).

0
0
BB
MB
15
20 x
B

TA

TB

Fig.1

Fig.2

Solutia 2. Un fir inextensibil de lungime suficient de mare este fixat n A, trecut


printr-un inel M ce poate culisa pe directia d si apoi printr-un mic scripete aflat n B.
Capatul liber are atasata o greutate P , care la echilibru se va afla ct mai aproape de
sol, minimiznd astfel lungimea M A+M B. n aceasta pozitie de echilibru, tensiunile

TA si TB care actioneaz
a n fire sunt egale n modul, paralelogramul fortelor este romb,
deci punctul c
autat M este determinat din nou de congruenta unghiurilor si .
Solutia 3. Locul geometric al punctelor X pentru care XA + XB = const este
o elipsa de focare A si B. Considernd fasciculul de elipse omofocale (de focare A si
B), punctul c
autat M este dat de intersectia cu d a acelei elipse din fascicul ce este
tangent
a la d. Proprietatea optic
a a elipsei asigur
a din nou congruenta unghiurilor
si .
q

Solutia 4. Cu notatiile din Solutia 1, AM = 100 + x2 , M B = 225 + (20 x)2


q

si avem de determinat minimul functiei f (x) = 100 + x2 + 225+(20x)2 , x [0, 20].


Not
a. Exista multe alte procedee si metode de abordare a problemelor de extrem; mention
am, pentru importanta lor, metodele program
arii liniare si pe cele ale
grafurilor. Pentru alte aplicatii, poate fi consultat
a bibliografia.
Bibliografie.
1. M. Cerchez - Aplicatii ale matematicii n practica, E.D.P., Bucuresti, 1975.
2. A. Leonte, C. Niculescu - Culegere de probleme de algebra si analiza matematica,
Ed. "Scrisul Romnesc", Craiova, 1981.
3. C. Udriste, E. T
an
asescu - Minime si maxime ale functiilor reale de variabile reale,
Ed. Tehnic
a, Bucuresti, 1980.
4. Gazeta Matematica (colectie).
37

Comentarii asupra unui exercitiu


SU 1
Dan PLAE
Exercitiu. Fie numerele naturale nenule a, b. Sa se demonstreze echivalenta:
19 | (5a + 4b) 19 | (9a + 11b) .

Solutie (tip culegere). 1) 19 | (5a + 4b) 19 | 11 (5a + 4b)


(19,4)=1

19 | (55a + 44b) 19 | [19a + 4 (9a + 11b)] 19 | 4 (9a + 11b)


2) 19 | (9a + 11b) 19 | 5 (9a + 11b) 19 | (45a + 55b)
19 | [19b + 9 (5a + 4b)] 19 | 9 (5a + 4b)

(19,9)=1

19 | (9a + 11b) .

19 | (5a + 4b) .

Comentariu metodic
Se poate pune, firesc, ntrebarea: prin care rationament s-a ajuns la concluzia ca
expresia 5a + 4b trebuie nmultita cu 11, iar 9a + 11b cu 5? Prezent
am n cele ce
urmeaz
a un punct de vedere n aceast
a privinta.
Determinam numerele naturale n, x, y, x 6= 0 astfel nct sa avem:
n (5a + 4b) = 19 (xa + yb) + z (9a + 11b) .
Comparnd coeficientii lui a si b obtinem:

19x + 9z = 5n
.
19y + 11z = 4n
nmultind prima relatie cu 4 si a doua cu 5 si sc
aznd membru cu membru, obtinem
76x 95y = 19z, de unde, prin np
artirea la 19, rezult
a:
4x 5y = z.

D
am lui x si y acele valori pentru care expresia x + y + z este minim
a si z 6= 0. (Minimalitatea expresiei precedente nu este necesara ci recomandata pentru simplificarea
calculelor!). Astfel, considernd x = 1, y = 0 obtinem z = 4. Rezult
a n = 11 si, deci,
egalitatea:
11 (5a + 4b) = 19a + 4 (9a + 11b) .
n mod analog, pentru demonstrarea implicatiei reciproce, determinam numerele
naturale n, x, y, z 6= 0 astfel nct s
a avem:
n (9a + 11b) = 19 (xa + yb) + z (5a + 4b) .

Efectund calculele, se obtine z = 11x + 9y. Rezult


a, lund x = 0, y = 1, c
az=9
si n = 5. Deci, avem:
5 (9a + 11b) = 19b + 9 (5a + 4b) .
Observatie. Rationamentele sunt valabile si pentru numere ntregi!
n ncheiere, propunem
1)
2)
3)
1

cititorilor demonstrarea urm


atoarelor echivalente:
11 | (2a + 5b) 11 | (3a + 2b) ,
23 | (2a + 3b) 23 | (9a + 2b) ,
19 | (11a + 2b) 19 | (18a + 5b) .

Profesor, S
coala Normal
a Vasile Lupu, Iasi

38

Cteva probleme privind triplete pitagoreice


1

Mircea CR
SMAREANU

Subiectul triplete pitagoreice are o istorie bogat


a, fiindu-i dedicate zeci de articole (a se vedea n acest sens capitolul IV din [2], unde la pagina 189 sunt citate si
cteva tabele cu astfel de triplete).
Definitie. Tripletul de numere naturale nenule (x, y, z) cu max (x, y) < z se
numeste pitagoreic daca x2 + y 2 = z 2 .
Se stie c
a forma general
a a unui triplet pitagoreic ([1], [2, ex.5.8, p.125-127])
este:
x = 2 2 , y = 2, z = 2 + 2 ,
(1)
cu , numere naturale nenule si prime ntre ele, adic
a (, ) = 1.
n cele ce urmeaz
a prezent
am cteva generaliz
ari ale unor rezultate referitoare la
triplete pitagoreice, rezultate aflate n bibliografia romna.
1. 60 | xyz.

Demonstratie. Avem xyz = 2 2 2 2 + 2 = 2 4 4 .


(i) Divizibilitatea cu 3 (5). Dac
a sau este multiplu de 3 (5) am terminat. Dac
a
nu, conform teoremei lui Fermat, avem 2 2 1 (mod 3) 4 4 1 (mod 5)
de unde rezulta divizibilitatea cu 3 si 5.
(ii) Divizibilitatea cu 4. Deoarece (, ) = 1 cel mult unul dintre si poate fi
par.
2

2
(ii21 ) =2 2k, = 2l + 1 xyz = 4k (2l + 1) 4k 4l 4l 1
4k + 4l + 4l + 1
2

2
(ii
)

=
2k
+
1,

=
2l
+
1

xyz
=
16
(2k
+
1)
(2l
+
1)
k
+
k

2k 2 + 2k + 2l2 + 2l + 1 .
n concluzie avem si divizibilitatea cu 4.
Observatie. Divizibilitatea cu 4 constituie Problema C:827, G.M.-10/1988, autor
Augustin Stan, iar divizibilitatea cu 5 Problema E:6303, G.M.-8/1978, f
ar
a autor.
n [2] la pagina 171 este citat P. Lenthric ca fiind autor al acestui rezultat, n jurul
anului 1830!
2. z si orice putere a sa este suma a doua patrate diferite.
Demonstratie. Pentru z avem concluzia datorit
a relatiei (1) cu (, ) = 1. Pen tefan Kleitsch
tru puterile lui z aplicam Problema E:5888 , G.M.-5/1977, autor S
(pentru rezolvare a se vedea G.M.-10/1977, p.405-406):
Dac
a un num
ar natural este suma a k p
atrate diferite atunci orice putere a sa
este suma a k p
atrate diferite.
3. Se cer lungimile catetelor unui triunghi dreptunghic asa nct produsul lor sa
fie de p ori perimetrul, cu p un numar prim dat.
1

Lector dr., Facultatea de matematic


a, Univ. Al. I. Cuza, Iasi

39

Demonstratie. Din xy = p (x + y + z) rezult


a 2 2 2 = p 22 + 2
adic
a ( ) = p. Cum p este num
ar prim rezult
a c
a avem solutiile (, ) =
(1, p) , (p, 1) deci (, ) = (p + 1, 1) , (p + 1, p). n concluzie avem:
2
(i) x = (p + 1) 12 = p (p + 2), y = 2 (p + 1),
(ii) x = (p + 1)2 p2 = 2p + 1, y = 2p (p + 1).
Observatie. Pentru p = 2 se obtine Problema OG:111, G.M.-1/1991, autor
Valer Pop.
4. (G.M.-5/1979, Problema O:35, Bucur B. Ionescu) Exista triplete pitagoreice
cu x, y, z numere prime?
Solutie. Din y = 2 rezult
a singura posibilitate = = 1 dar atunci
x = 2 2 = 0, imposibil. Deci r
aspunsul este negativ.
5. (G.M.12/1979, Problema E:6736 , I. Joldis) x + y + z | xy.
Demonstratie. x + y + z = 2 ( + ), iar xy = 2 2 2 .

6. (Problema 7.8, [3, p.190 + p.199]) x2 xy + y 2 este suma a doua patrate.


2

2
Demonstratie. x2 xy+y 2 = 2 2 2 2 2 +42 2 = 2 ( + ) +
+2 ( )2 .
7. Daca p si q sunt numere naturale nenule si prime ntre ele, sa se rezolve
ecuatia diofantica p2 x2 + q 2 y 2 = 2p2 q 2 z 2 .
Solutie. Considernd x = qu si y = pv obtinem u2 + v 2 = 2z 2 de unde rezulta
2
2

uv
u+v
u+v
uv
si deci
z2 =
+
= 2,
= 2 2 , z = 2 + 2 . n
2
2
2
2
concluzie:

x = q 2 + 2 2 , y = p 2 2 2 , z = 2 + 2
cu (, ) = 1.

Observatie. Pentru p = 2, q = 3 se obtine Problema 5.9 din [3, p.119].

Bibliografie
1. V. Claudian - Analiza diofantica, G.M.-1/1970, 1-9.
2. L. E. Dickson - History of the theory of numbers, vol. II - Diophantine Analysis,
Chelsea, N. Y., 1952.
3. P. Radovici-M
arculescu - Probleme de teoria elementara a numerelor, Ed. Tehnic
a,
Seria Culegeri de probleme de matematic
a si fizic
a, Bucuresti, 1986.

40

Cteva aplicatii ale inegalit


atii Cauchy-Buniakowski
1

Ioana CRACIUN
si Gheorghe CRACIUN

Daca a1 , a2 , . . . an si b1 , b2 , . . . bn sunt numere reale, n N , n 2, atunci are loc


inegalitatea Cauchy-Buniakowski:

2
(a1 b1 + a2 b2 + + an bn ) a21 + a22 + + a2n b21 + b22 + + b2n ,
a2
an
a1
cu egalitate daca si numai daca
sau ai = bi = 0, i = 1, n.
=
= =
b1
b2
bn
n
n

P
P
a2k , S (ab) =
ak bk . Avem:
Demonstratie. Notam S a2 =
k=1

k=1

0 (ai bj aj bi )2 = a2i b2j 2 (ai bj ) (aj bi ) + a2j b2i , i, j = 1, n.


2
2
2
2
S
b
b
S
(ab)
+
b
S
a , i = 1, n. Sumam
Sumam dupa j si ob
t
inem:
0

2a
i
i
i
i


a S 2 (ab) S (a) S (b),
acum dup
a i: 0 S a2 S b2 2S 2 (ab) + S a2 S b2 , adic
q.e.d.
Aplicatii
1. Fie patratul ABCD si M , N doua puncte pe cercul nscris n acest patrat. Sa
se arate ca
1
AABCD (AM AN + BM BN + CM CN + DM DN ) .
3
Fie P, R, S, T mijloacele laturilor AB, BC, CD,DA. Din teorema medianei avem:
M A2 + M B 2 AB 2
M B 2 + M C 2 BC 2

, M R2 =

,
MP2 =
2
4
2
4
M C 2 + M D2
M D2 + M A2 DA2
CD2

, MT 2 =

.
M S2 =
2
4
2
4
Adunnd obtinem
M P 2 + M R2 + M S 2 + M T 2 = M A2 + M B 2 + M C 2 + M D2 AB 2 .
Triunghiurile P M S si T M R sunt dreptunghice, deci
M P 2 + M S 2 = P S 2 = AB 2 si M T 2 + M R2 = T R2 = AB 2 .
Adunnd obtinem M P 2 +M S 2 +M T 2 +M R2 = 2AB 2 , deci
M A2 +M B 2 +M C 2 +M D2 = 3AB 2 (egalitate cunoscuta).
Analog, N A2 + N B 2 + N C 2 + N D2 = 3AB 2 . Folosind inegalitatea Cauchy-Buniakowski, avem:

M A2 + M B 2 + M C 2 + M D2 N A2 + N B 2 + N C 2 + N D2
2

(AM AN + BM BN + CM CN + DM DN ) sau

9AB 2 (AM AN + BM BN + CM CN + DM DN )2
3AABCD AM AN + BM BN + CM CN + DM DN.
2. Fie a1 , a2 , . . . an R. Sa se afle x1 , x2 , . . . xn stiind ca a1 x1 + a2 x2 + +
n
n
P
P
+an xn =
a2i si x21 + x22 + + x2n =
a2i .
i=1

i=1

Profesori, Plopeni (Prahova)

41

Conform inegalit
atii C-B, avem:

(a1 x1 + a2 x2 + + an xn )2 a21 + a22 + + a2n x21 + x22 + + x2n ,

cu egalitate daca xi = kai , i = 1, n. Tinnd


cont de conditiile din enunt, rezulta
n

n
P 2 2
P 2
P 2
c
a
ai
ai
ai , care are loc cu semnul de egalitate. Atunci si

i=1

i=1

i=1

relatia precedent
a este o egalitate si rezult
a c
a xi = kai , i = 1, n. nlocuind n prima
n
n
P
P
2
2
conditie din enunt, obtinem:
ai = k
a2i , deci k2 = 1 si xi = ai , i = 1, n.
i=1

i=1

Observatie. Pentru n = 3 si a1 = 2, a2 = 2, a3 = 3 obtinem problema C:2331


(Alfred Eckstein, G.M. - 11/2000).
3. Fie A1 , A2 , . . . An un poligon convex si M un punct n interiorul sau. Se
proiecteaza M pe laturile A1 A2 , A2 A3 , . . . , An A1 n punctele M1 , M2 , . . . , Mn . Sa se
arate ca:

1
A1 A2 2 + A2 A3 2 + + An A1 2 .
A1 M1 2 + A2 M2 2 + + An Mn 2
4
am teoFie i {1, 2, 3, . . . , n} fixat. n triunghiurile M Ai Mi si M Ai+1 Mi aplic
rema lui Pitagora: Mi M 2 = Ai M 2 Ai Mi 2 = Ai+1 M 2 Ai+1 Mi2 (consider
am
An+1 = A1 ). Scriind a doua egalitate pentru i = 1, n si sumnd, gasim:
n
n
X
X
Ai Mi 2 =
Ai+1 Mi 2 .
i=1

i=1

Dac
a not
am Ai Mi = xi , iar Ai Ai+1 = ai , aceast
a ultim
a relatie se scrie: x21 +
2
2
2
2
2
+x2 + + xn = (a1 x1 ) + (a2 x2 ) + + (an xn ) sau, efectund calculele,
x21 +x22 + +x2n = x21 +x22 + +x2n +a21 +a22 + +a2n 2 (a1 x1 + a2 x2 + + an xn )

1 2
a1 + a22 + + a2n . Aplicnd inegalitatea C-B,
2

avem: (a1 x1 + a2 x2 + + an xn )2 x21 + x22 + + x2n a21 + a22 + + a2n , cu


egalitate daca si numai daca ai = kxi , i = 1, n. n consecinta, rezulta ca

2 2

1 2
2
2
a + a2 + + an
x1 + x22 + + x2n a21 + a22 + + a2n
2 1

1 2
a1 + a22 + + a2n x21 + x22 + + x2n ,

4
adic
a relatia dorit
a. Pentru a avea egalitate punem conditia ai = kxi , i = 1, n;
gasim k = 2. Deci M1 , . . . Mn sunt pe mediatoarele laturilor poligonului, care va fi
inscriptibil, iar M va fi centrul cercului circumscris lui.
(
)
a+b+c
4. Sa se determine suma elementelor multimii M = abc | p
N .
3 (a2 + b2 + c2 )

Inegalitatea (a + b + c)2 3 a2 + b2 + c2 se deduce imediat cu ajutorul inegaa+b+c


lit
atii C-B. Deci p
1, cu egalitate pentru a = b = c. Rezult
a
3 (a2 + b2 + c2 )
ca M = {111, 222, 333, . . . , 999} si 111 + 222 + + 999 = 111 (1 + 2 + + 9) =
= 111 45 = 4995.
a1 x1 + a2 x2 + + an xn =

42

Concursul "Recreatii Matematice"


Editia a II-a, Iasi, 27 August 2002
Clasa a VII-a
1. Fie x, y, z (0, ) cu x + y + z = 1. Sa se determine cele mai mici valori pe
care le pot lua expresiile
xy 2 yz 2 zx 2
xy yz
zx
E=
+
+
.
+
+
; F =
z
x
y
z
x
y
a, Focsani si Lucian Tutescu, Craiova
Cornel Noan
2. Fie n N si x = n2 + n.
a) S
a se arate c
a x R\Q si s
a se afle [x].
b) S
a se determine primele dou
a zecimale de dup
a virgul
a ale lui x pentru
n = 20022002 .
Cornel Noan
a, Focsani
3. Fie dat un segment [M N ]. Construiti numai cu rigla si compasul un p
atrat
ABCD astfel nct M [AB], AM = M B, iar N [AC], AN = 3N C. (Descrieti
toate constructiile care trebuie efectuate.)
Gabriel Popa, Iasi

Clasa a VIII-a
1. Daca suma, produsul si ctul a doua numere irationale sunt, fiecare, numere
rationale, calculati suma cuburilor celor doua numere.
Claudiu-
Stefan Popa, Iasi (Recreatii Matematice 2/2002)
2. S
a se rezolve n Z Z ecuatia x2 (y + 1) + y 2 (x + 1) + 1 = 0.
Gabriel Popa, Iasi
3. Fie ABCA0 B 0 C 0 un trunchi de piramida oarecare. Notam cu G, G0 centrele de
greutate ale bazelor, iar {D} = BC 0 CB 0 , {E} = AC 0 CA0 , {F } = AB 0 BA0 .S
a
se arate c
a dreptele BE, CF si GG0 sunt concurente.
Dan Brnzei, Iasi

Clasa a IX-a
1. S
a se rezolve n R ecuatia xn 3 [x] + 2 = 0, n N .
Cornel Noan
a, Focs
ani
3
2. S
a se arate c
a pentru orice (0, 2), exist
a n Z astfel nct sin n
.
2
Gheorghe Iurea, Iasi
3. Fie Db , Dc , Fa , Ea puncte de tangenta ale cercurilor exnscrise triunghiului ascutitunghic ABC cu dreptele suport ale laturilor, astfel nct B (Dc C), C (BDb ),
A
B
C
B (AFa ), C (AEa ). Sa se arate ca: Dc Fa kDb Ea sin2 = sin2 + sin2 .
2
2
2
Temistocle Brsan, Iasi

Clasa a X-a
1. Fie P R [X], P (X) = X n + an1 X n1 + + a1 X + a0 .
a) S
a se arate c
a dac
a toate r
ad
acinile polinomului sunt reale si mai mari dect
n
2, atunci (1) P (1) + an1 + 2n 1.
43

b) S
a se arate c
a dac
a toate r
ad
acinile polinomului sunt reale, pozitive, mai mici
2n + 2n2 an2 + 2n4 an4 + . . .
dect 2, atunci n1
< 1.
2
an1 + 2n3 an3 + 2n5 an5 + . . .
Carmen Nejneru si Vlad Martinusi, Iasi

2. Sa se arate ca cos n arctg 2 2 Q, n N.


Gheorghe Iurea, Iasi
3. Fie A, B dou
a puncte fixate, iar M un punct variabil n plan. Fie A0 si B 0

imaginile lui M prin rotatiile n jurul punctului A, respectiv B, de unghi , respectiv


2

astreaz
a aceeasi directie, ar
atati c
a M parcurge o dreapt
a.
. Dac
a vectorul A0 B 0 p
2
Reciproca este adevarata?
Gabriel Popa, Iasi

Clasa a XI-a
apolinoame, fiecare avnd
cte o r
ad
acin
a real
a. Dac
a
1. Fie P, Q R [X]
dou

1
1
4
4
P
atati c
a P = Q.
+ x + Q (x) = Q
+ x + P (x) , x R, ar
2002
2002
Lucian Tutescu, Craiova
2. Fie A Mn (R) astfel nct exist
a m N, m > n 3 si R, || 1,
pentru care Am+1 Am A + In = On . Sa se arate ca |det A| = 1.
Lucian-Georges L
adunc
a, Iasi (Recreatii Matematice 2/2002)
3. Determinati func
t
iile
continue
f
:
(0, 1) (0, ) pentru care f (x) f (y) =

xy
, x, y (0, 1).
=f
2xy x y + 1
Lucian Laz
ar, Bac
au

44

Concurs de admitere 2002, Iasi


Facultatea de Informatic
a, Universitatea "Al. I. Cuza"
Analiz
a matematic
a
1. Fie (an )nN , (bn )nN doua siruri de numere reale.
i) Dac
a (an )nN converge c
atre a si (bn )nN converge c
atre b, ce se poate spune
despre convergenta sirului a0 , b0 , a1 , b1 , . . . , an , bn , . . . ? S
a se justifice r
aspunsul dat.

(1)n
2n+1
si bn = (1)
+ n+2 n, n N, s
a se studieze convergenta
ii) Dac
a an =
n+1
sirului a0 , b0 , a1 , b1 , . . . , an , bn , . . . .

2. Fie functia f : [1, 1] R, f (x) = ln x2 + 1 . S


a se arate c
a:
i) |f (x2 ) f (x1 )| < |x2 x1 |, x1 , x2 [1, 1], x1 6= x2 ;
ii) exist
a un singur x0 (1, 1), astfel nct f (x0 ) = x0 .

Algebr
a
1. Fie (G, ) si (, ) dou
a grupuri. S
a se demonstreze c
a dac
a f : G este
izomorfism, atunci si f 1 : G este izomorfism.
2. Fie dat q Q . Sa se arate ca:
i) functia f : Z Q, f (k) = q k , este morfism de la grupul (Z, +) la grupul

(Q , );
ii) dac
a q
/ {1, 1}, atunci exist
a un subgrup al lui (Q , ) izomorf prin f cu
(Z, +). Sa se precizeze acest subgrup.
3. Fie f Z3 [X], f = b
1 X X 2 X n1 , n N . Aratati ca f se divide
2, dac
a si numai dac
a n este multiplu al lui 3.
prin X b
4. S
a se descompun
a n factori ireductibili peste Q, R si respectiv C, polinomul
g = X 4 + X 3 X 2 2X 2, stiind ca g se divide prin X , unde este o radacina
de ordinul trei a unitatii.

Algebr
a - colegiu
1. Fie A M2 (R), t (A) suma elementelor de pe diagonala principala a matricii
A si det (A) determinantul matricii A. Sa se arate ca:
i) A2 t (A)A+det (A)I2 = O2 , unde I2 si O2 sunt matricea unitate si respectiv
matricea nul
a din M2 (R);
ii) daca A2 =
O2 , atunci t (A)
= 0.
a11 a12 a13
2. i) Fie A = a21 a22 a23 M3 (R). S
a se scrie explicit toti termenii care
a31 a32 a33
apar n expresia determinantului matricii A si care sunt de forma (1)i+j+1 a1i a23 a3j .
ii) fiind un parametru real, sa se discute si sa se rezolve sistemul:

2x 3y z = 1
.
4x + 6y + 2z = 2

atati c
a H este parte stabil
a
3. Fie H = x + y 2 | x, y Q, x2 2y 2 = 1 . Ar
a lui R n raport cu nmultirea si c
a toate elementele lui H sunt simetrizabile n
raport cu operatia indus
a.
4. Sa se determine polinoamele f, g Z [X], de gradul 1, astfel nct
2

X + 2X + 2 f + X 2 + 3X + 3 g = 1.
45

Fac. de Electronic
a si Telecomunicatii, Univ. Tehnic
a "Gh. Asachi"
Matematic
a

1. Ce relatie exista ntre numerele reale A = n n+1 si B = (n + 1) n , n 8?


a) A > B; b) A = B; c) A < B; d) A B; e) A B.
k2
k
2. Multimea solutiilor inecuatiei C16
> C16
este
a) {4, 5, . . . , 9}; b) ; c) {17, 18, 19}; d) {10, 11, . . . , 16}; e) {1, 2, . . . , 9}.
3. Fie inecuatia logx x + 30 1. Solutiile acestei inecuatii sunt
a) x (, 5]; b) x [6, ); c) x (1, 6]; d) x (1, ); e) x .
Z 3
dx
4. Fie I (a) =
, a R si L = lim I (a). Atunci:
a2
1 |x a| + 1
a) L = 2;
b) L = 2 ln 2; c) L = 4 ln 2; d) L = 8 ln 2; e) L = ln 2.
x + y + 2z = 1
x + (2 1) y + 3z = 1
5. Sistemul
cu , R este compatibil nede
x + y + ( + 3) z = 2 1
terminat pentru
a) R, = 1; b) = 0, = 5 sau R, = 1; c) = 0, = 2;
d) = 0, = 1 sau R, = 5; e) R, R.

sin2 x
cos2 x
6. Sa se afle solutiile ecuatiei

= 2.
cos x (1 + tg x) sin x (1 + ctg x)

a) x = k ; b) x = 2k ; c) x ; d) x = k ; e) x = k, k Z.
2
3
6
(1 + i)2002
7. Num
arul complex
n este real pentru n N de forma
(1 i)
a) n N; b) n = 4k; c) n = 4k + 1; d) n = 4k + 2; e) n = 4k + 3, k N.
2
8. Se consider
a polinoamele f, g R [X], f = X 2n X n + X 4 + 1, g = (X 1) .
Sa se determine restul r al mpartirii polinomului f la polinomul g.
a) r = (n + 4) X n 2; b) r = nX; c) r = (n + 2) X + n 1;
d) r = (n + 4) X + n 2; e) r =2nX.

x2
x+1
9. Se consider
a functia f care satisface relatia: 2f
+f
= x,
x+1
x2
pentru orice x R\ {1, 2}. Valoarea derivatei f (n) (2) este
n!
n!
n!
n!
n!
; d) (1)n n ; e) n .
a) (1)n n+1 ; b) n+1 ; c) (1)n
n
3
3
2

3
3
3
xn + 1
10. Functia f : D R, D R, f (x) = 3
, n N, are cel putin o asimptota
x +1
vertical
a si nu admite asimptote orizontale sau oblice pentru
a) n = 2k + 1, k N; b) n N; c) n < 4; d) n = 4; e) n = 2k, k 3.

Fac. de Automatic
a si Calculatoare, Univ. Tehnic
a "Gh. Asachi"
Matematic
a - ingineri
1.
real a pentru care r
ad
acinile x1 , x2 ale ecuatiei 8ax2

2 Valorile parametrului
2
2 a + 2a + 1 x + a + 1 = 0 satisfac x1 + x2 4x1 x2 sunt:
a) a > 0; b) a = 0; c) a = 1; d) a = 1; e) a 6= 2.
2
2. Fie polinoamele f = X 2n X n + X 4 + 1, n > 4 si g = (X 1) . S
a se
determine restul mp
artirii lui f la g.
46

a) (n + 4) X n 2; b) nX + n 2; c) (n + 2) X n 2;
d) (n + 4) X + n 2; e) (n 4) X + n
2.

1 1 1
3. Daca a este o radacina a ecuatiei x2 + x + 1 = 0 si A = 1 a a2 atunci:
1 a2 a

3 0 0
2 0 0
1 0 0
a) A2 = 0 0 3; b) A2 = 0 0 2; c) A2 = 0 0 1;
0 3 0
0 2 0
0 1 0
2 2 0
3 3 0
d) A2 = 0 2 2; e) A2 = 0 3 3.
2 0 2
3 0 3
1
4. Legea de compozitie x y = (x + y xy + 1) este o lege de grup comutativ
2
pe multimea:
a) R; b) R\ {0}; c) R\ {0, 1}; d) R\ {1}; e) R\ {1}.

1
1
1
.
5. Sa se afle lim
+
+ +
n 2n + 1
2n + 2
2n + n
3

1
3
a) ln 2; b) ln ; c) ; d) e; e) 1 + ln .
2
2
2
2
x + m x
6. Se d
a functia f : R\ {2} R, definit
a prin f (x) =
e , n care m
x+2
este parametru real. Sa se precizeze valorile lui m pentru care f are doua puncte de
extrem.
a) m [2, 6]; b) m (, 2/3]; c) m (2/3, 6); d) m (, 2) (6, );
e) m (, 2/3) (6, ).
Z 1 2
x +a
7. S
a se determine parametrul a, astfel nct s
a avem 1 <
dx < 2.
2
0 x + 3

!
2 3
2 3
3
a) 3 < a < 3 +
; b) 3
< a < 3; c) 3 < a < 3 1 +
;

.
d) 3 < a < ; e) 3 < a <
6
2
8. Se d
a un triunghi de arie S si de laturi a, b, c. Fie M, N, P proiectiile centrului
cercului nscris pe laturi. Se cere aria 4M
NP .

abc (a + b + c)
4S 3
S2 3
3S 2 3
a+b+c 2
; e)
S ; b)
; c)
; d)
.
a)
2
3abc
16S
3abc
abc (a + b + c)
(a + b + c)
9. S
a se rezolve ecuatia sin (2x + 1) = cos (2x 1).

a) +k ; b) +k; c) +k ; d) (1)k +k ; e) (1)k +k, k Z.


8
2
8
8
2
8
2
8
10. Unghiul diedru dintre doua fete adiacente ale unui octaedru regulat are
m
asura, exprimat
a n radiani, egal
a cu:

2
5
3
3

; c)
; d) 2 arccos
; e) 2 arcsin
.
a) ; b)
2
3
8
3
3
Matematic
a - colegiu
1. Solutiile sistemului

xy + x + y = 11
sunt:
x2 y + xy 2 = 30
47

a) (x, y) {(2, 3) , (3, 2)}; b) (x, y) {(1, 5) , (5, 1);


c) (x, y) {(2, 3) , (1, 5)}; d) (x, y) {(3, 2) , (5, 1)};
e) (x, y) {(2, 3) , (3, 2) , (1, 5) , (5, 1)}.
2. Sa se rezolve inecuatia log2 (9 2x ) > 3 x.
a) x < 8; b) 0 < x <
< x < 4; d) x > 3; e) nu exist
a solutii.
3; c) 2
1 0 1
3. Dac
a matricea A = 0 1 0 satisface A3 = aA2 + bA, atunci:
1 0 1
a) (a, b) = (3, 2); b) (a, b) = (3, 3); c) (a, b) = (2, 2); d) (a, b) = (3, 2);
e) (a, b) = (2, 3).
4. Pe R se defineste legea de compozitie prin relatia: x y = xy + ax + 2by + 1,
x, y R. Sa se determine a, b R astfel nct legea sa fie comutativ
a si asociativ
a.

1
1+ 5
1+ 5
1
,b =
sau
a) a = 1, b = ; b) a = 0, b = 0 sau a = 1, b = ; c) a =
2
2
2
4

1 5
1 5
,b =
; d) a = 4, b = 2; e) nu exist
a solutie.
a=
2
4
r
n2 + 1 n + 1
ln
.
5. Sa se afle lim
n
n+2
n

1
a) ; b) 1; c) e; d) e; e) .
2
x2 + mx + 2
6. Se considera functia f (x) = 2
, unde m R este un parametru. Sa
x + 2x + m
se determine m, astfel nct domeniul ei de definitie s
a fie R si s
a admit
a exact dou
a
puncte de extrem.
a) m(1, 2)(2, ); b) m(2, ); c) m(3, ); d) m(1, 2); e) m(, 1).
R
xdx
, x (a, ) (a 6= 0).
7. Sa se calculeze
3/2
(x
+
a)

x 2a
1
a
+ C; b) 2
+ C; c)
a) 2
x+a
x+a
+ C;
x+a
x+a
x+a
x + 2a
2x+a+1

+ C; e) 2
+ C.
d)
3
x+a
x+a
8. ntr-un patrulater convex se cunosc diagonalele d1 , d2 si un unghi dintre ele.
Se cere aria patrulaterului ale carui vrfuri sunt mijloacele laturilor celui dat.

1
1 2
1
d1 + d22 d1 d2 sin ;
a) d1 d2 cos ; b) d1 d2 sin ; c)
4
4
4
1
1
d) d1 d2 (sin + cos ); e)
(d1 + d2 )2 sin .
4
16
9. Se dau numerele x = cos 3, y = tg 3, z = ctg 3. Atunci
a) x < y < z; b) y < x < z; c) z < y < x; d) x < z < y; e) z < x < y.
10. Locul geometric al centrelor sferelor ce trec prin dou
a puncte distincte date
este
a) o sfera; b) o dreapta; c) doua drepte perpendiculare; d) un plan;
e) doua plane perpendiculare.

48

Capacitate - teste preg


atitoare
Testul 1 (prof. Gheorghe TIMOHE)


I. 1. Valorile rationale ale numerelor x si y pentru care x 2 2 3 +

+y 2 + 2 3 = 4 2 3 sunt .........................
2. Descompus
a n produs de doi factori expresia E (x) = x2 9y 2 8x + 16 este
egala cu .........................

98
97
96
95
94
93
92
91
99
+
+
+
+
+
+
+
+
,
3. Fie E =
98
97
96
95
94 93
92
91
90
unde [x] - partea ntreag
a a num
x R; atunci
E = .........................
p
parului

4. Solutia n N a inecuatiei 3 + 2 2 x + 3 2 2 x < 2 este .........................


5. Daca numerele prime x si y satisfac relatia 2x + 3y = 16, atunci x =
........................., y = .........................
6. ntr-un vas n form
a de cub se toarn
a 6 l ap
a, ceea ce reprezint
a 75% din
capacitatea vasului. Diagonala cubului este egala cu .........................
7. Dimensiunile unui paralelipipeddreptunghic sunt direct proportionale cu 6, 8
si 10, iar diagonala are lungimea de 10 2 cm. Volumul paralelipipedului este ............
8. Fie ABCD un trapez cu bazele AB = 13 cm si CD = 15 cm, M [AD] astfel
nct M D = 2M A si M N kAB, N [BC].
BN
a) Valoarea raportului
este ......................... b) M N = .........................
BC
9. Fie AC si BC doua tangente la un cerc C (O, R) n A si B, m(ACB) = 60 ,
AO = 8 cm.
a) m(AOB) = ......................... b) lungimea coardei AB = .........................
II. 1. a) Reprezentati ntr-un sistem de coordonate carteziene toate perechile
(x, y) care verifica relatiile |x 1| = 3 si |x y| 3.
b) Fie E (x) = (3 |x| 1) / |x|. Determinati x R pentru care E (x) N.
2. Demonstrati c
a suma distantelor de la vrfurile unui triunghi la o dreapt
a
exterioar
a lui este egal
a cu suma distantelor de la mijloacele laturilor triunghiului la
aceeasi dreapta.

3. Pe planul patratului ABCD de latura a se ridica perpendiculara OA = a 2.


Fie M si N proiectiile lui A pe (OBC) si (OCD), iar P si L proiectiile lui M si A
pe (AOD), respectiv M H. S
a se determine: a) M P si AL; b) unghiul diedru al
planelor (AM H) si (ABC); c) pozitia punctului Q pe OC astfel nct perimetrul
4OBD sa fie minim.
Testul 2 (prof. Gheorghe TIMOHE)

I. 1. Propozitia "n N, n (n + 3) + 3 este numar prim" este .........................


1
1
2
1
+
++
. Dintre S si este mai mare .........................
2. Fie S =
100 101
299
3
3. Solutiile ecuatiei ||3x 1|
12| =
4 sunt .........................

ab, a (b, (a + b) /2) = .........................


4. a) Dac
a 0 < b < a, atunci

b) Dac
a A = 21999 , 22000
Z, atunci card A = .........................
|x + 2| + |y 3| = 7
5. Rezolvnd sistemul
, obtinem (x, y) .........................
|x + 2| + 3 = y 3
6. Fie ABCD - trapez de baze AB = 7 cm, CD = 2 cm si laturile neparalele
49

BC = 4 cm si AD = 3 cm. M
asura unghiului dintre AD si BC este egal
a cu
......................... grade.
7. Fie ABCDA0 B 0 C 0 D0 - cub. Atunci m( (A0 C 0 , AD0 )) = .........................
8. Aria 4ABC este 268 cm2 . Aria triunghiului format de mijloacele laturilor
sale este .........................
9. Fie 4ABC cu m(A) = 60 , m(B) = 80 , (AA0 , (BB 0 , (CC 0 - bisectoarele
unghiurilor 4ABC (A0 , B 0 , C 0 apartin cercului circumscris 4ABC).
a) m(B 0 C 0 A0 ) = ......................... b) m(A0 B 0 C 0 ) = .........................
II. 1. Fie f : R R, f (x) = 3x + 1, g : R R, g (x) = x + 1.
a) Calcula
a de
graficele
ti aria suprafetei determinat

functiilor si Ox.
b) A = (x, y) | x, y R+ , f (x + y) f 2 xy . Reprezentati grafic elementele
lui A.

c) B = n N | |g (n)| 8 2 . Determinati suma elementelor din B.





2. Fie numerele a =
2+1
3 2 . . . 100 + 99
101 100 si



21
3 + 2 . . . 100 99
101 + 100 .
b=
a) Ar
atati c
a a + b > 2. b) Comparati numerele (1 a) (1 + b) si b a.
3. Tetraedrul ABCD se sectioneaz
a cu un plan paralel cu muchiile [AB] si
[CD], AB = a, CD = b. Planul intersecteaz
a muchiile [BD] , [BC] , [AD] , [AC] n
N, M, P si respectiv Q.
a) Demonstrati c
a patrulaterul M N P Q este paralelogram.
BM
b) Daca
= x, x > 0, exprimati aria paralelogramului M N P Q n functie de
MC
a, b, x si masura unghiului dintre AB si CD.
c) Determinati pozitia punctului M [BC] pentru care aria paralelogramului
M N P Q este maxim
a.

Testul 3 (prof. Lidia BOSNCIANU )


1. Cel mai mic numar natural nenul divizibil cu 88 si care poate fi scris ca
produsul a trei numere naturale consecutive este .........................
2. Dac
a a, b, c, d R, si cd = 1, valoarea minim
a a expresiei a2 + b2 + c2 + d2 +
+2ab 2ac 2ad 2bc 2bd + 14 este .........................
3. Numerele 5 x, 2x + 3 si 5x 2, x R, reprezinta lungimile laturilor unui
triunghi. Atunci x .........................
2
2
2
4. Ecuatiile (m 1) (x + 2)+9 = (m 1) 3 (m 1) (x + 2), (m 1) (x + 1) =
= 3 (3x + m + 5), m R sunt echivalente daca m .........................
5. Fie f : R R, f (x) = 2 |x| f (1) 3, x R. Atunci f (x) = ....................
6. Fie ABCD un
dreptunghi, M A (ABC), N mijlocul lui (BC), M B = BD =
= 6 cm si M D = 4 3 cm, atunci d (M, DN ) = .........................
1
7. Fie 4ABC oarecare, M (BC) astfel nct M C = BC, N AC astfel
3
1
AP
nct AN = AC, iar AM BN = {P }. Atunci
= .........................
4
PM
8. ABCDA0 B 0 C 0 D0 este un paralelipiped dreptunghic cu perimetrul bazei 28
cm. Daca diagonala paralelipipedului are lungimea de 20 cm si formeaza cu o muchie
lateral
a un unghi cu m
asura de 30 , volumul [ABCDA0 B 0 C 0 D0 ] este .........................
9. Fie un con circular drept cu sectiunea axial
a un triunghi isoscel cu baza 8 cm
50

si perimetrul 18 cm.
a) M
asura unghiului corespunz
ator sectorului de cerc obtinut prin desf
asurarea
conului este .........................
b) Volumul conului este .........................
II. 1. S
a se g
aseasc
a ultimele dou
a cifre ale num
arului

A = 22n + 22n+1 + 22n+2 + 22n+3 42n + 42n + 42n+1 + 42n+2 + 42n+3 2n 8n

22n+1 + 22n+3
, n N .
4n + 4n+1
a se demonstreze inegalitatea:
2. Fie x1 , x2 , . . . , xn R+ cu x1 x2 . . . xn = 1. S

(x1 + x2 + 1) (x2 + 2 x2 + 1) . . . (xn + n xn + 1) > 3 4 n (n + 1) (n + 2) .


3. V ABCD este o piramida patrulatera regulata avnd toate muchiile de lungime
a. Fie Q mijlocul lui (CV ) si (C) cercul nscris n triunghiul V BC.
a) S
a se arate c
a (BDQ) (V BC).
b) Dac
a M este un punct oarecare pe (C) si N (BD), s
a se determine lungimea
minima posibila pentru (M N ).

Testul 4 (prof. Lidia BOSNCIANU )

631
. Atunci a = .........................
1. Fie a = 21 22 23 2100 162
a

c
2. S
tiind c
a 6 3 + bb + 2 = 493, atunci abc = .........................
a
a13
a
3. Dac
a
=
, a, b N , atunci cea mai mare valoare pentru este ..........
b
b+7
b

ab + 36 / ab 36 N = .........................
4. Multimea A = ab N |

5. Fie f : R R, f (x 1) = 3x + 1 f (2), x R. Atunci f (x) = ...............


6. Fie M mijlocul laturii (AB) a dreptunghiului ABCD, M N AC, N (AC)
\ = .........................
si 8M N = AC. Atunci m(BAC)
b
b = 40 . Fie AD BC si (AE bisectoarea lui BAC,
\
7. Fie 4ABC cu m(B)m(
C)
\
D, E (BC). Daca (AD si (AE mpart BAC n trei unghiuri cu masurile direct
proportionale cu 1, 2, 3, atunci masurile unghiurilor 4ABC sunt .........................
8. ntr-un cilindru cu diametrul de 4 cm si n
altimea de 25 cm se aseaz
a niste
bile cu raza de 20 mm.
a) Care este numarul maxim de bile ce ncap n cilindru?
b) Cte procente din volumul cilindrului ocupa bilele?
9. ntr-o piramid
a patrulater
a regulat
a, sectiunea diagonal
a este un triunghi
dreptunghic isoscel. Atunci At /Al = .........................

II. 1. Fie a, b Z. Aratati ca a b2 + a2 + a = 0 a = b = 0.


x2 x3 . . . xn + x1 x3 . . . xn + + x1 x2 . . . xn1
2. Rezolvati n N ecuatia y =
.
x1 x2 . . . xn (x1 + x2 + + xn )
3. Fie 4ABC dreptunghic isoscel, (AB) (AC) si AB = a. Se duce CS
(ABC), CS = a.
a) Calculati aria totala si volumul piramidei SABC n functie de a.
\
b) Aflati m((SAB)
, (ABC)).
c) Daca CP SB si Q este mijlocul muchiei (SA), sa se demonstreze ca patrulaterul ABP Q este inscriptibil.
51

Solutiile problemelor propuse n nr. 1/2002


Clasele primare
P.24. Aflati numerele a, b, c, d stiind ca verifica n acelasi timp urmatoarele egalitati: a + 3 = b, b + 3 = c, c + 3 = d, a + 3 = 10.
( Clasa I )
nv. Maria Racu, Iasi
Solutie. Din ultima relatie afl
am pe a, a = 10 3 = 7, apoi b = 7 + 3 = 10,
c = 10 + 3 = 13, d = 13 + 3 = 16.
P.25. Un elev din clasa I, fixnd un numar din sirul numerelor naturale, constata
ca suma numerelor din fata lui nu este mai mica dect 55, iar suma aceasta adunata
cu numarul fixat nu depaseste pe 66. Despre ce numar este vorba?
( Clasa I )
Luminita Popa, elev
a, Iasi
Solutie. Suma numerelor din fata numarului cautat poate fi 55, 56, . . . . A
doua suma poate fi 66, 65, 64, . . . . Daca a doua suma nu este 66, atunci cea mai
mare valoare posibil
a a num
arului c
autat este 65 55 = 10. Suma primelor nou
a
numere nenule este 45, ceea ce nu corespunde datelor problemei. Deducem c
a a doua
suma este 66. Daca prima suma nu este 55, atunci cea mai mare valoare posibila
a numarului cautat este 66 56 = 10 si iarasi ajungem la o contradictie. Numarul
c
autat este 66 55 = 11.

P.26. Pe trei borcane de compot, unul de cirese, altul de visine si al treilea cu


amestec de cirese si visine, toate etichetele au fost puse gresit. Scotnd un singur
fruct dintr-un singur borcan, determinati continutul fiecaruia.
( Clasa a II-a)
***
Solutie. Se scoate un fruct din borcanul cu eticheta CV. Dac
a fructul este
cireasa, atunci n borcanul cu eticheta V nu putem avea numai visine sau numai
cirese. Rezulta ca avem cirese si visine. n acest caz avem corespondenta etichetacontinut CV C, C V, V CV. Dac
a fructul extras este visin
a, atunci avem
corespondenta CV V, C CV, V C.

P.27. Sa se scrie numarul 31 folosind cele patru operatii aritmetice si numai


cifra 3 (se cer cel putin doua solutii).
( Clasa a II-a)
Andrea Balla, elev
a, Brasov
Solutie. 1) [(3 + 3) 3 3] (3 3 : 3) + 3 : 3 = 15 2 + 1 = 31.
2) [(3 + 3 : 3) 3 (3 3 : 3)] 3 + 3 : 3 = 10 3 + 1 = 31.

P.28. Cte pagini are o carte daca pentru paginarea ei s-a folosit cifra 9 de 117
ori?
( Clasa a III-a)
Crizantema Mironeanu, elev
a, Iasi
Solutie. De la pagina 1 la pagina 100 se foloseste cifra 9 de 20 ori. nseamn
a c
a
de la pagina 1 la pagina 600 se foloseste cifra 9 de 120 de ori. Pentru a folosi de 117
ori cifra 9 trebuie sa eliminam paginile: 600, 599, 598. Cartea are 597 pagini.
P.29. Ioana si Alina au cules mpreuna 165 de nuci. Ioana a cules mai putine
nuci dect Alina; ea face un calcul si observa ca triplul diferentei dintre numarul
nucilor culese de ele reprezinta tocmai numarul nucilor culese de Alina. Cte nuci a
52

cules fiecare fata?


( Clasa a III-a)
nv. Maria Racu, Iasi
Solutie. Examinnd textul se constat
a c
a Alina are trei p
arti iar Ioana dou
a
parti din cele cinci parti egale. Alina a cules 165 : 5 3 = 33 3 = 99 (nuci) iar Ioana
a cules 165 99 = 66 (nuci).

P.30. Aratati ca dintre oricare patru numere naturale diferite, mai mici dect
1 000 000, se pot alege doua a caror diferenta sa se mparta exact la 3.
( Clasa a IV-a)
Roxana Bolocan, elev
a, Iasi
Solutie. La mp
artirea cu 3 resturile posibile sunt 0, 1, 2. nseamn
a c
a cel putin
doua numere din cele patru vor da acelasi rest la mpartirea cu 3. Fie a = 3c + r,
b = 3d + r, a > b. Avem a b = 3c 3d = 3 (c d).

P.31. O veverita descopera un alun ncarcat cu fructe si si face provizii pentru


iarna transportnd la scorbura sa alternativ: o data doua alune, o data trei alune.
Dupa ce transporta 47 de alune, face o pauza pentru a se odihni. Sa se calculeze ce
distanta a parcurs veverita n total, daca de la alun la scorbura ei este o distanta de
x hm x dam x m, unde x are ca valoare cel mai mic numar natural posibil.
( Clasa a IV-a)
nv. Mihai Agrici, Iasi
Solutie. Numarul x nu poate fi 0. nseamna ca x este 1. Distanta de la scorbura
la alun este de 1hm 1dam 1m= 111m. Pentru prima grup
a de 5 alune parcurge traseul
Alun-Scorbur
a-Alun-Scorbur
a, deci 3 111m. Pentru fiecare grup
a de 5 alune, din
restul de 42, parcurge traseul Scorbura-Alun-Scorbura-Alun-Scorbura, deci 4 111m.
Deoarece sunt 8 grupe, veverita va parcurge 8 4 111m. Pentru restul de 2 alune va
parcurge traseul Scorbur
a-Alun-Scorbur
a, deci 2 111m.
n total veverita parcurge (3 + 32 + 2) 111m = 37 111m = 4107m.

P.32. Un parinte si mparte averea astfel: la primul copil 10 milioane plus o


cincime din rest, la al doilea copil 20 de milioane plus o cincime din noul rest, la
al treilea 30 de milioane plus o cincime din noul rest si asa mai departe. Sa se afle
suma mpartita de parinte, precum si numarul copiilor, stiind ca toti au mosteniri
egale.
( Clasa a IV-a)
Mihai Grtan, Iasi
Solutie. Din faptul ca primii doi copii au primit sume egale rezulta ca (R1 R2 ) :
4
5 = 10, adic
a R1 R2 = 50. Al doilea copil primeste din suma de din R1 , ceea
5
4
ce nseamna ca din R1 depaseste pe R2 cu 20 milioane. Avem R1 R2 = 50 si
5
4 R1 : 5 R2 = 20 de unde rezulta R1 : 5 = 30. Obtinem R1 = 5 30 = 150 si
S = 10 + 150 = 160 (milioane). Deci suma mpartita este de 160 milioane. Primul
copil a primit 10 + 150 : 5 = 40 (milioane). Num
arul copiilor este 160 : 40 = 4.

Clasa a V-a
V.26.
Sa se determine cifrele distincte si nenule
a, b, c, d, e, f, g pentru care rezultatul nmultirii alaturate este
cel mai mare posibil:
Ioan Sac
aleanu, Hrl
au
Solutie. Avem ad = e < 10 si ac = g < 10. Deoarece
53

g
g

a
c
e

0
0

b
d
f

cifrele a, b, c, . . . , g sunt distincte, rezult


a c
a ad, ac {1 2, 1 3, . . . , 1 9, 2 1, 2 3,
2 4, 3 1, 3 2}. Cum a 6= 1 (c
aci altfel am avea d = e) si c 6= 1, d 6= 1 (din motive
similare), urmeaz
a c
a ad, ac {2 3, 2 4, 3 2}. Ca urmare, pentru ca produsul din
enunt, sa fie maxim, luam g = 8, e = 6, a = 2, d = 3, c = 4. Au ramas de aflat cifrele
b si f . Observ
am c
a b, f {1, 5, 7, 9}. Dar b 6= 1 (c
aci altfel f = 3 = d) si b 6= 5
(altfel f = 5 = b). Deci b = 7 si f = 1 sau b = 9 si f = 7. n consecinta produsul cel
mai mare este 209 403 = 84227 si cifrele c
autate sunt: a = 2, b = 9, c = 4, d = 3,
e = 6, f = 7, g = 8.
V.27. Trei apicultori au tras mpreuna 700 kg miere de albine. Cnd au mpartit
mierea, primul apicultor a luat jumatate, al doilea jumatate din rest, al treilea jumatate din noul rest, apoi operatiunea se repeta pna se mparte toata mierea. Sa se
afle cta miere a luat fiecare.
C
at
alin-Cristian Budeanu, Iasi
Solutie. Se observ
a c
a primul apicultor ia de dou
a ori mai mult
a miere dect al
doilea, iar al doilea de doua ori mai multa dect al treilea. Daca x noteaza cantitatea
de miere luat
a de al treilea apicultor, atunci al doilea ia 2x, iar primul 4x si obtinem
relatia x + 2x + 4x = 700. Rezult
a c
a x = 100; deci al treilea ia 100 kg de miere, al
doilea 200 kg, iar primul 400 kg.
V.28. Aratati ca N1 = 32001 + 22001 si N2 = 32002 22002 sunt numere divizibile
cu 5.
Dorina Carapanu, Iasi
Solutie. Ultima cifra a unui numar de forma 24n este 6, iar a unuia de forma 34n
este 1. Prin urmare, 22000 = 24500 se termina n 6, iar 22001 = 22000 2 are ultima
a n 3. Ca urmare, N1 se termin
a n 5 si
cifr
a 2. La fel obtinem c
a 32001 se termin
este, deci, divizibil cu 5.
n privinta numarului N2 , observam ca 32002 se termina n 9, 22002 n 4, iar nsusi
.
N n 9 4 = 5. Deci, N .. 5.
2

V.29. Sa se afle numerele abc pentru care abc = ac b2 .


Romanta Ghita
si Ioan Ghita
, Blaj
Solutie. Valorile b = 0 si b = 1 nu sunt posibile deoarece conditia din enunt se
scrie a0c = 0 si respectiv a1c = ac si aceste egalitati sunt false. Nici b = 2 nu-i o
valoare posibil
a, c
aci 100a + 20 + c = 40a + 4c 60a + 20 = 3c, ceea ce este evident
fals (3c poate fi cel mult 27).
Pentru b = 3 avem: 100a + 30 + c = 90a + 9c 5a + 15 = 4c. Ultima relatie
.
implic
a c .. 5, adic
a c = 5, precum si a = 1. Obtinem c
a abc = 135 este o solutie a
problemei.
2
2
Aratam ca nu putem
b 4. ntr-adev
avea
2 ar, abc = acb

2 = b
2100a+10b+c
2
(10a + c) 10b = 10ab 100a + b c c 10b = 10 b 10 a + b 1 c
(1). Dac
a b 4, atunci b2 10 > 0, b2 1 > 0 si din (1) rezult
a c
a 10b 10 b2 10
(am minorat a cu 1 si c cu 0). Constat
am c
a aceast
a inegalitate nu-i verificat
a de
valorile b = 4, 5, . . . , 9.
Numarul abc = 135 este singura solutie.
V.30. Daca xi , i = 1, 500 , sunt numere naturale nedivizibile cu 5, atunci
54

2000
numarul N = 4x41 + 8x82 + 12x12
3 + + 2000x500 este divizibil cu 5.
Tamara Culac, Iasi
Solutia I (a autorului). Num
arul xi , i = 1, 500, este de una din formele:
M5 + 1, M5 + 2, M5 + 3, M5 + 4. Aratam ca x4i = M5 + 1. ntr-adevar avem:
(M5 + 1)4 = (M5 + 1) (M5 + 1) (M5 + 1) (M5 + 1) = M5 +1, (M5 + 2)4 = M5 +24 =
= M5 + 1, (M5 + 3)4 = M5 + 34 = M5 + 1, (M5 + 4) = M5 + 44 = M5 + 1.
2000
Evident, avem si faptul ca numerele x8i , x12
sunt de forma M5 + 1. Atunci,
i , . . . , xi
N = 4 (M5 + 1) + 8 (M5 + 1) + + 2000 (M5 + 1) = M5 + 4 (1 + 2 + + 500) =
501 500
= M5 + 2 500 501 = M5 .
= M5 + 4
2
Solutia II (dat
a de eleva Tutescu Anca S
tefania, Craiova). Avem
8

4
N = 4x1 4 + 4 + 8x2 8 + 8 + + 2000 x2000
500 2000 + 2000 sau

(1)
N = 4 x41 1 + 8 x82 1 + + 2000 x2000
500 1 + 4 (1 + 2 + + 500) .
501 500
Cum 4 (1 + 2 + + 500) = 4
= 2 501 500, rezult
a c
a acest termen
2
al num
arului N se divide cu 10. Pe de alt
a parte, pentru
ar x N ce
2 orice num

{1,
4,
6,
U x4
nu-i divizibil
cu
5,
avem:
U
(x)

{1,
2,
3,
4,
6,
7,
8,
9},
U
x
8

9},
4
2000
{1, 6}, U x {1, 6}
. , U x500 1
etc. Ca
urmare,U x1 1 , U x2 1 , . .

1
{0, 5}; iar U 4 x41 1 , U 8 x82 1 , . . . , U 2000 x2000
{0}.
500
n consecinta, toti termenii din scrierea (1) a lui N sunt divizibili cu 10 si, deci,
.
N .. 10.

Clasa a VI-a
VI.26. Fie A = 4a + 6b c, B = 4a 3b c, C = 3a 11b 28c, unde
a, b, c Z. Daca (A, B) = 23, aratati ca (A, B, C) = 23.
Cristiana Constanda, elev
a, Iasi
..
.
Solutie. Nu trebuie s
a ar
at
am, de fapt, dect c
a C . 23. Avem c
a A B .. 23,
.
.
.
deci 9b .. 23 si cum (9, 23) = 1, atunci b .. 23. Din A = 4a + 6b c .. 23, urmeaz
a acum
..
c
a 4a c . 23, deci c = 4a 23k, k Z. n aceste conditii, C = 3a 11b 28c =
= 3a 11b 28 (4a 23k) = 115a 11b + 28 23k, fiecare termen fiind multiplu
de 23.
VI.27. Sa se rezolve n Z sistemul: 3x + 2y 8;

x y 1; 3x y = 1.
Mihai Cr
aciun, Pascani
7
Solutie. Avem: y = 3x 1 8 2y 1 = 7 2y 3y 7 y ,
3
y = 3x 1 3 (1 + y) 1 = 2 + 3y 2y 2 y 1; deoarece y Z, rezult
a c
a
y+1
si singurele solutii convenabile sunt (0, 1) si (1, 2).
y {1, 0, 1, 2}. nsa x =
3
VI.28. Sa se rezolve n N ecuatia
1 2 + 2 3 + + n (n + 1) (n + 1) (n + 2) 2n = 2 + 4 + + 2n.
Dumitru - Dominic Bucescu, Iasi
55

1
[k (k + 1) (k + 2) (k 1) k (k + 1)]
3
n care d
am valori lui k de la 1 la n, obtinem prin sumare 12+23+ +n (n + 1) =
1
= n (n + 1) (n + 2). Scaznd n ambii membri ai ecuatiei date suma 1 + 2 + + n,
3
obtinem echivalent:
Solutie. Plecnd de la identitatea k (k + 1) =

n (n + 1)
n (n + 1) (2n + 1)
2n (2n + 1)
n (n + 1) (n + 2) 2n (2n + 1)

=
,
3
2
2
6
6
n+1
altfel spus
= 1, ceea ce antreneaza n = 5.
6
b
VI.29. n triunghiul ascutitunghic ABC, bisectoarea interioara a unghiului B
intersecteaza naltimea AD n E, D [BC]. Fie F (DC astfel nct AE = EF .
Aratati ca BEAF .
Tamara Culac, Iasi
Solutie. Fie E 0 [AB] astfel nct EE 0 AB. Cum
b este egal departat de laE se afla pe bisectoarea lui B,
turile unghiului: ED = EE 0 . Atunci 4AEE 0 4F ED
\ , de unde rezult
\0 DEF
a c
a E 0 , E, F
(C.I.), deci AEE
0
sunt coliniare, adica F E AB. Urmeaza ca E este ortocentrul 4ABF , asadar BEAF .

VI.30. Pe ipotenuza (BC) a triunghiului dreptunghic


ABC se considera punctele N si M astfel nct BN =
AB, CM = AC. Daca P si Q sunt proiectiile punctelor
M si N pe dreptele AN , respectiv AM , demonstrati ca
segmentele (M P ), (N Q) si (P Q) se pot constitui n laturile
unui triunghi.
C
at
alin Calistru, Iasi
Solutie. Fie {R} = M P N Q ortocentrul 4AM N ;
atunci ARBC. Avem:
b
\ ) = m(BAR)
[ m(M
\
m(BAM
AR) = 90 m(B)

b =
\
\
90 m(AM
C) = m(AM
C) m(B)

b m(B)
b = m(M
b = m(M
\
\
\
AR).
= m(M
AC) m(B)
AR) + 90 m(C)
1
b = 45 . Atunci
\ ) = m(N
\
\
Analog se arata ca si m(CAN
AR), deci m(M
AN ) = m(A)
2
4P AM si 4QAN sunt triunghiuri isoscele, de unde M P = AP si N Q = AQ.
Urmeaza ca (M P ), (N Q), (P Q) se pot constitui n laturile unui triunghi, anume
4AP Q.

Clasa a VII-a

a + 2 2 Q.
Gheorghe Iurea, Iasi
p

Solutie. Fie x = a + 2 2 Q. Atunci a + 2 = 2 + 2x 2 + x2 si cum


1
9
a, x Q, urmeaza ca a = 2 + x2 si 1 = 2x. De aici, x = si a = . Reciproc, daca
2
4
VII.26. Determinati a Q stiind ca

56

9
2 2+1
1
a = avem ca a + 2 2 =
2 = Q.
4
2
2
VII.27. Determinati a R astfel nct sistemul
a2
a2
a2
x21 +(a + 1) x1 + = x2 , . . . , x2n1 +(a + 1) xn1 + = xn , x2n +(a + 1) xn + = x1
4
4
4
sa admita numai solutii ntregi.
C
at
alin Calistru, Iasi
Solutie. Adunnd membru cu membru ecuatiile, efectund reducerile si grupnd,
n
X
a
a 2
obtinem c
a
xi +
= 0, de unde n mod necesar x1 = x2 = = xn = .
2
2
i=1

Cum dorim ca sistemul s


a aib
a solutii ntregi, rezult
a a = 2k, k Z. n acest caz,
este imediat c
a x1 = x2 = = xn = k constituie solutie a sistemului dat.
VII.28. Fie zece numere naturale nenule care au suma egala cu 55. Sa se arate
ca printre ele exista trei care pot fi lungimile laturilor unui triunghi.
Adrian Zanoschi, Iasi
Solutie. S
a ordon
am cresc
ator numerele: 1 a1 a2 a10 . Evident
c
a ai+1 < ai + ai+2 , i = 1, 8; ar mai trebui s
a ar
at
am c
a exist
a un indice i
{1, 2, . . . , 8} pentru care ai+1 > ai+2 ai . Pentru aceasta, sa presupunem contrariul:
ai+1 ai+2 ai , i = 1, 8, i.e. ai+2 ai + ai+1 , i = 1, 8. Avem succesiv:
a3 a1 + a2 1 + 1 = 2, a4 a2 + a3 1 + 2 = 3 si n continuare a5 5, a6 8,
a7 13, a8 21, a9 34, a10 55. Atunci a1 +a2 + +a10 1+1+ +55 = 143,
adica 55 143, absurd.
VII.29. Fie ABCD un patrat, O centrul sau, iar M si P mijloacele segmentelor
(OA), respectiv (CD). Sa se arate ca triunghiul BM P este dreptunghic isoscel.
Constantin Cocea si Dumitru Neagu, Iasi
Solutia I (dat
a de elevul Mihul Andrei, Iasi). Fie
M 0 , M 00 proiectiile punctului M pe BC si respectiv CD. Avem:
1
1
M M 0 = (AB + OO0 ) si M M 00 = (AD + OP ) (ca linii mi2
2
jlocii n trapezele ABO0 O si ADP O) si deci M M 0 = M M 00 .
Evident, M 0 B = M 00 P . Deducem c
a 4BM 0 M 4P M 00 M ,
0
\
\
deci M B M P si BM M P M M 00 . Ultima relatie conduce
0 M M 00 . Unghiul M\
0 M M 00 fiind drept, urmeaz
\
a
la BM
P M\
\
ca BM
P este unghi drept si, deci, 4BM P este dreptunghic isoscel.
Solutia II. Fie a latura p
atratului. Avem c
a P A2 =
2
5a
. Aplicnd teorema medianei n 4P AO
AD2 + DP 2 =
4
10a2
10a2
si n 4BOA, obtinem P M 2 =
, BM 2 =
, deci
16
16
5a2
si atunci lungimile
P M = BM . Pe de alt
a parte, BP 2 =
4
P M , M B, P B sunt numere pitagoreice, de unde concluzia.
VII.30. Fie ABCD un patrat de latura 1 si punctele M (AD), N (BC),
1
4
si
, demonstrati ca 1 < AM + BN
{P } = BM AN . Daca SDCN P M =
2
3
57

AM BN

4
.
9

Emil Vasile, Ploiesti


Solutie. Fie x = AM , y = BN , z = P P 0 (unde P P 0 AB).
1
Din ipoteza, SABM +SABN SABP = , prin urmare x+yz =
2
z
BP 0
P 0A
xy
x+y
z
+
= 1, deci z =

,
1. Dar + =
x
y
AB
AB
x+y
4
unde am tinut seama de inegalitatea ntre media armonica si
x+y
, adica
cea aritmetica. Rezulta ca x + y = 1 + z 1 +
4
3
xy
x + y . Pe de alt
a parte, x + y
= 1 implic
a
4
x+y
4 1
4
xy = (x + y) (x + y 1) = ; am folosit faptul evident ca x + y > 1.
3 3
9

Clasa a VIII-a

VIII.26. Demonstrati ca ecuatia t2 + 1 x2 + 4t2 x + 4t2 5 = 0 are numai doua


solutii n Z Z.
Mihai Cr
aciun, Pascani
2
a x = 2, atunci
Solutie. Ecuatia se scrie echivalent t2 (x + 2) = t x2 . Dac
t x2 = 0, deci t = 4. Daca x Z\ {2}, atunci t2 (x + 2)2 t, iar tx2 t. Egalitatea este atinsa daca si numai daca t = x = 0. n concluzie, S = {(0, 0) , (2, 4)}.
VIII.27. Determinati a R stiind ca ecuatia x4 2x3 + 3x2 2x + a = 0 are o
singura solutie reala.
Gabriel Popa, Iasi
Solutie. Dac
a P (x) este expresia din membrul stng al ecuatiei, putem scrie:
P (x) = x4 2x3 + 3x2 2x + a = x2 (1 x)2 2x (1 x) + a
si de aici se observa ca P (x) = P (1 x), x R. Cum ecuatia are o unica solutie
1
a x0 = . nlocuind, obtinem n mod necesar
real
a x0 , trebuie ca x0 = 1 x0 , adic

2 2
7
7
1
7
2
= 0, care admite
x x+
c
aa=
. Dac
aa=
, ecuatia devine x
16
16
2
4
1
singura solutie real
a (dubl
a) x = .
2
VIII.28. Fie a, b numere naturale prime ntre ele. Aflati valorile lui n pentru
care Sn = an + an1 b + an2 b2 + + abn1 + bn este divizibil cu a + b.
Mihaela Predescu, Pitesti
Solutie. Dac
a n impar, atunci Sn are un
num
a
r
par
de termeni si avem c
a Sn =

(an + bn )+ an1 b + abn1 + = (a + b) an1 an2 b + + bn1 +ab (a + b)

.
an3 an4b + + bn3 + = (a + b) A, deci Sn .. a + b. Dac
a n este par,
n1
n2
n1
n
+ b si cum n 1 impar, paranteza se
atunci Sn = a a
+a
b + + b
..
.
n.
divide cu a + b. Rezulta ca Sn . a + b b . a + b; vom arata ca aceasta divizibilitate
este imposibil
a n conditiile date. Avem c
a (b, a + b) = 1, deoarece dac
a d | b si
d | a + b, atunci d | a si d | b, adic
a d | (a, b), deci d | 1. Urmeaz
a c
a n descompunerile
58

lor, numerele b si a + b nu au nici un factor prim comun, afirmatie valabil


a si pentru
numerele bn si a + b. n concluzie, pentru n par, Sn nu se divide cu a + b.
VIII.29. Se considera piramida triunghiulara regulata V ABC cu latura bazei a,
iar muchia laterala 2a. Fie M mijlocul lui (V A), iar N un punct pe (V B) astfel
3a
nct V N =
. Aflati distanta de la V la planul (M N C).
4
Adrian
Corduneanu, Iasi
a 6
. Folosind relatia
Solutie. Cu teorema medianei n 4V AC, obtinem CM =
2

a 31
. Din teorema cosinusului n 4V M N ,
lui Stewart n 4V BC, rezult
a CN =
4
a
obtinem M N = . Cunoastem prin urmare laturile 4CM N si cu formula lui Herron
2

10a2 15
. Calculnd acum volumul tetraedrului V M N C n
afl
am aria sa: SCMN =
128

a3 11
, iar cnd
doua moduri, cnd folosim drept baza 4V M N gasim ca VV MN C =
64
lu
am drept baz
a 4CM N ,volumul fiind cunoscut, obtinem c
a distanta de la V la
a 165
.
planul (M N C) este h =
25

VIII.30.
Fie A, B, C, D patru puncte necoplanare astfel nct AB = 4 73,

CD = 4 29. Notam cu E, F mijloacele segmentelor (AB), respectiv (CD). Sa


se arate ca mijloacele segmentelor (AF ), (BF ), (CE), (DE) sunt vrfurileunui
paralelogram si sa se calculeze aria acestuia stiind ca are o latura de lungime 194.
Romanta Ghita
si Ioan Ghita
, Blaj
Solutie. Fie M, N, P, Q mijloacele segmentelor (EC), (AF ), (ED) si respectiv
[BF ]. n 4ECD, (M P ) este linie mijlocie, iar (EF ) este mediana; atunci (M P )
si (EF ) se njumatatesc. Rationnd analog n 4F AB, deducem ca (N Q) si (EF )
se njum
at
atesc. Rezult
a c
a M P si N Q sunt concurente n mijlocul lui (EF ) si
se njum
at
atesc, deci M, N, P, Q sunt coplanare si M N P Q este paralelogram; fie

1
1
O centrul acestuia. Avem ca OQ = AB = 73, OP = CD = 29. Daca
4
4

P Q = 194, aria 4OP Q se poate calcula cu formula lui Herron sau aflnd naltimea;
1
1
obtinem SOP Q = si deci SMON = . Cum (P O) este mediana n 4P N Q, avem
2
2
1
1
ca SP ON = SP OQ = si deci SOMQ = . n final, SM N P Q = 2.
2
2

Clasa a IX-a

a
a
. Discutie.
= {x} +
[x]
{x}
D. M. B
atinetu-Giurgiu, Bucuresti
si x
Solutie. Pentru existen
t
a
numitorilor,
x

/
[0,
1)
/ Z. Ecuatia se scrie

a
echivalent ([x] {x}) 1
= 0, iar prima paranteza nu se poate anula.
[x] {x}
a
Ramne ca [x] {x} = a, deci {x} = , unde n = [x] este din N deoarece a > 0 si
n
IX.26. Daca a (0, ), sa se rezolve ecuatia [x] +

59

a
, n N , n > [a].
n
IX.27. Sa se determine functiile f, g : [0, ) [0, ), unde g este surjectiva si
aditiva si g (y) + g (f (x)) = f (x + g (y)), x, y [0, ).
Ioan S
ac
aleanu, Hrl
au
a g (f (x)) =
Solutie. Fie y0 [0, ) pentru care g (y0 ) = 0; atunci obtinem c
f (x), x [0, ) (1). Prin urmare, g (y) + f (x) = f (x + g (y)), x, y [0, ),
relatie care pentru x = 0 arata ca f (g (y)) = f (0) + g (y), y [0, ) (2). Cum g
este surjectiv
a, pentru orice z [0, ), g
asim y [0, ) astfel nct g (y) = z. Din
(2) deducem f (z) = f (0) + z, z [0, ). nlocuind n (1) si folosind faptul c
ag
este aditiva, obtinem ca g (z) = z, z [0, ).
{x} 0. Atunci x = n +

IX.28. Sa se determine functiile f : R R pentru care (f f f ) (x) =


|
{z
}
n ori

= x + , R fixat, iar functia g = f 1R este monotona.


Mihail Bencze, Brasov
Solutie. Aplicnd f n ambii membri ai egalitatii din enunt obtinem ca f (x + ) =
= f (x) + , x R, de unde g (x + ) = f (x + ) (x + ) = f (x) x = g (x),
x R.Cum g monoton
a, rezult
a c
a g constant
a: g (x) = k, x R. Atunci

ak= ,
f (x) = x + k, x R, deci (f f f ) (x) = x + nk = x + . Urmeaz
|
{z
}
n
n ori

adic
a f (x) = x + , x R.
n
IX.29. Sa se arate ca n orice triunghi ABC are loc inegalitatea
3R p
l3
l3
la3
+ b + c
p (p3 3abc)
ha hb
hc
2r
Viorel Cornea si Dan S
tefan Marinescu, Hunedoara
Solutie. Din egalitatea Cauchy-Buniakowski-Schwarz, obtinem ca
X 3 2 X X
1
la
.
(1)

la6
ha
h2a

1
Se stie c
a OG2 = R2 a2 + b2 + c2 0, deci a2 +b2 +c2 9R2 si atunci major
am
9
2
2
2
2
X 1
a +b +c
9R
=
. Acum

al doilea factor:
h2a
4S 2
4S 2
r
2bc
A
p (p a) p
la =
cos bc
= p (p a), deci
b+c
2
bc
X
X

3
(p a) = p3 3p3 3p2 (a + b + c) + 3p a2 + b2 + c2 a3 + b3 + c3 .
la6 p3

Un calcul de rutin
a arat
a c
a a3 + b3 + c3 = 3p a2 + b2 + c2 + 3abc 4p3 , de unde
X

rezulta ca
la6 p3 p3 3abc . Revenind n (1), deducem concluzia.
IX.30. n patrulaterul ABCD consideram punctele R si S pe diagonala BD, n
interioarele triunghiurilor ABC, respectiv ACD. Notam {M } = CR AB, {N } =
60

= AR BC, {P } = AS CD si {Q} = CS AD. Stiind


ca

AM 2 BN 2 CP 2
+
+
+
M B 2 N C 2 P D2

AM n
BN n
CP n
DQn
DQ2
=
4,
s
a
se
arate
c
a
+
+
+
= 4, n N.

QA2
M Bn
N Cn
P Dn
QAn
C
at
alin Calistru, Iasi
Solutie. Aplicnd teorema lui Ceva n 4ABC si 4ACD si combinnd relatiile
AM BN CP DQ
obtinute, rezult
a c
a

= 1. Putem scrie:
M B N C P D QA
s
BN 2
CP 2
DQ2
AM 2 BN 2 CP 2 DQ2
AM 2
+
+
+
=44
,

2
2
2
2
MB
NC
PD
QA
M B 2 N C 2 P D2 QA2
AM
BN
CP
DQ
deci este atins
a egalitatea n inegalitatea mediilor; atunci
=
=
=
.
MB
NC
PD
QA
Concluzia este acum imediat
a.

Clasa a X-a
X.26. Fie ecuatia x4 S1 x3 + Sx2 + mx m 1 = 0, unde S este aria unui
triunghi neechilateral ABC, iar S1 este aria triunghiului A1 B1 C1 determinat de
punctele de intersectie a bisectoarelor interioare cu cercul circumscris triunghiului
ABC. Sa se determine m R stiind ca ecuatia admite un numar impar de radacini
n (0, 1).
Dumitru Gherman, Pascani
Solutie. Dac
a not
am f (x) = x4 S1 x3 + Sx2 + mx m 1, atunci conditia ca
ecuatia data sa admita un numar impar de radacini n (0, 1) este echivalenta cu f (0)
f (1) < 0, adica (m 1) (S S1 ) < 0 (). Sa determinam acum semnul diferentei
A+B
A+C
B+C
SS1 . Avem S = 2R2 sin A sin B sin C si S1 = 2R2 sin
sin
sin
=
2
2
2
A
A
B
C
B
C
= 2R2 cos cos cos , de unde rezult
a c
a SS1 = 8 sin sin sin 1, cu egali2
2
2
2
2
2
tate daca si numai daca triunghiul ABC este echilateral. Deci, n conditiile problemei,
are loc S < S1 si atunci, avnd n vedere relatia (), obtinem m (, 1).
X.27. Fie r [1, ), D = {z C; |z| r} si P C [X], P (X) = aX 2 +
+bX + c. Sa se arate ca daca P (z) D, z D, atunci a, b, c D.
D.M. B
atinetu-Giurgiu,
Bucuresti

2
Solutie. Fie 1, si 2 r
ad
acinile ecuatiei x3 = 1. Deoarece
1,
,

D, rezult
a
2
2

=
a
+
b
D
s
i
P

+
c

D.
c
a P (1) = a + b + c D, P () = a2 + b +c


De aici, obtinem c
a 3c = P (1)
+ P () + P 2 , 3a = P (1) + P () + 2 P 2

si 3b = P (1) + 2 P ()+ P 2 . Folosind aceste


egalit
ati deducem c
a: 3 |c| =
2
2

= P (1) + P () + P
|P (1)| + |P ()| + P
3r si, analog, 3 |a| 3r si
3 |b| 3r. Deci, a, b, c D.
2

X.28. Rezolvati ecuatia z 2 2|z| 1 + z 2|z1| 1 + 1 = 0, z C\R.


Emil Vasile, Ploiesti
|z|2
si 2|z1| 1 = R.

1
=

R
Solutie. Fie z = a + bi (a, b R, b 6= 0),
2

2
2
Cu aceste nota
a devine: a b + 2abi + (a +bi) + 1 = 0 sau
tia noastr
t2ii ecua
2

+
a
+
1 = 0, de unde obtinem a2 b2 + 1 = 0, adic
a
2ab
+
b
=
a

b
2 2

a +b
2
2
2
1 a + b = 1 ().
61

Deoarece functia f (t) = t (2t 1) este strict cresc


atoare pe (0, ) rezult
a c
a
c
a

=
1
ecuatia () are o solutie unic
a si anume a2 + b2 = 1. De aici deducem

si nlocuind n egalitatea 2a + = 0, obtinem = 2a, adic


a 2 22a = 1 2a.
Dac
a facem notatia c = 1 2a, cum |a| 1, rezult
a c 3. Pe de alt
a parte din

2 1+c = c 0, obtinem c 2, deci c = 2 1+c 2 3 > 25/3 > 3. Contradictia la


care am ajuns arata ca ecuatia data nu are solutie.
X.29. Un motan scoate cu ajutorul unui pahar un numar de pestisori dintrun acvariu. Cti pestisori trebuie sa contina acvariul astfel nct motanul sa aiba
matematic speranta ca va scoate 5 dintre ei?
Gabriel Popa, Iasi
Solutie. Notam cu n numarul de pestisori din acvariu. Fie X variabila aleatoare
care ia ca valori num
arul de pestisori care se afl
a n paharul motanului. S
a calcul
am pk = P ({X = k}). Deoarece sunt n total 2n cazuri egal posibile (num
arul
submultimilor unei multimi cu n elemente), dintre care sunt favorabile Cnk , avem
Ck
pk = nn . Asadar, tabloul de repartitie al variabilei aleatoare X este:
2

0
1
2 ...
n
X : Cn0 Cn1 Cn2
Cnn .
.
.
.
2n 2n 2n
2n
Motanul poate spera c
a va extrage un num
ar de pestisori egal cu speranta matematic
a
(sau media) variabilei X, adica
n
n
X
X
kCnk
n2n1
n
E (x) = m =
pk xk =
=
= .
n
n
2
2
2
k=1
k=1
Prin urmare, avem n/2 = 5, deci n = 10.
X.30. Fie M = {1, 2, . . . , n}. Sa se afle numarul de
kk-uple
! (A1 , A2 , . . . , Ak ) de
k
[
\
Ai = M si Card
Ai = l, l n fixat.
submultimi ale lui M astfel nct
i=1

i=1

Lucian-Georges L
adunc
a, Iasi
Solutie. Fie N = {(0, 0, . . . 0, 1), (0, 0, . . . , 1, 0), . . . , (1, 1, . . . , 1, 1)} Rk
multimea k-uplelor formate din 0 si 1, f
ar
a elementul (0, 0, . . . , 0, 0). Multimea N are
2k 1 elemente.
A partitiona M = {1, 2, . . . , n} n k submultimi (A1 , A2 , . . . , Ak ) astfel nct
k
[
Ai = M este totuna cu a defini o functie f : M N prin legea f (j) =
i=1

/ Ai (de
= (j1 , j2 , . . . , jk ) N, unde ji = 1 daca j Ai si ji = 0 daca j
exemplu, f (3) = (1, 1, 0, . . . , 0, 1) daca si numai dac
a
3

A
,
3

A
,
3
Ak si

1
2

n
arul acestor functii este 2k 1 , dar nu toate satisfac
3
/ A3 , A4 , . . . , Ak1 ). Num
ultima conditie din ipotez
a.
Observam ca j A1
Ak2 ! Ak daca si numai daca f (j) = (1, 1, . . . , 1)
\
N . Deci, conditia Card
Ai = l este echivalenta cu Card ({j M | f (j) =
i=1

= (1, 1, . . . , 1)}) = l. Cum num


arul de moduri n care l elemente din M sunt duse

nl
prin f n (1, 1, . . . , 1) este Cnl , rezult
a c
a r
aspunsul problemei este Cnl 2k 2
.
62

Clasa a XI-a
XI.26. Fie A Mn (R). Daca tr (tA A + (tA) A ) = 2n det A, atunci tA = A .
Iuliana Georgescu si Paul Georgescu, Iasi
Solutie. Se stie ca tr (X + Y ) = tr X + tr Y ( R), tr (XY ) = tr (Y X),

XX = (det X) In si (tX) = t (X ), X, Y Mn (R). Egalitatea dat


a se rescrie
astfel:



tr tA A + tA A = tr A A + tA tA

t
tr A A + A t (A ) = tr A A + tA t (A )



tr A tA A + A tA t (A ) = 0

tr A t (A ) tA A = 0,

a
de unde, notnd cu C = A t (A ), obtinem tr (C t C) = 0. Ultima relatie, mpreun
cu observatia C Mn (R), ne conduce la conditia C = On , adica tA = A .
XI.27. Fie a [0, 1) si (xn ) n0 un sir de numere reale astfel nct

1 2
xn + x2n1 , n N .
x2n a max x2n1 ,
2
Aratati ca sirul (xn ) n0 este convergent si determinati limita sa.
Aurel Muntean, Sibiu

1
x2 + x2n1
Solutie. Daca max x2n1 ,
= x2n1 , atunci x2n ax2n1 , sau
2 n

1 2
xn + x2n1
=
|xn | k1 |xn1 |, unde k1 = a [0, 1). Daca max x2n1 ,
2

a 2
a 2
a
1 2
xn x2n1 , adic
xn + x2n1 , avem x2n
xn + x2n1 sau 1
a x2n
=
2
2
2
2
r
a
a

x2 si deci |xn | k2 |xn1 |, unde k2 =


[0, 1).
2 a n1
2a
Fie k = max {k1 , k2 }. Deoarece k [0, 1) si |xn | k |xn1 | k 2 |xn2 |
k n |x0 |, rezult
a c
a sirul (xn )n0 converge la zero.
XI.28. Sa se determine p R pentru care limita sirului (an )n1 definit prin
n
X
np
p
este finita si nenula.
termenul general an =

k + k2 1
k=1
Constantin Chiril
a, Iasi
Solutie. Avem succesiv:

n
n
X
X

np
p 2
p
n
k
+
1

k =
=
an =

2
k + k2 1 k=1
k=1
n

np 2

np 2 X
=
k+1 k =
n+11 =
2
2
k=1
!
r

1
2 p+ 1
1
=
1+
.
n 2
2
n
n
1
Deci, sirul (an ) are limita finita nenula daca si numai daca p = .
2
63

!
1
1
1 + + + 1 = 1.
2
n
Marian Tetiva, Brlad
n (un 1) ln an
, unde

Solutie. Scriem termenul general xn sub forma: xn =


ln an
ln bn
r
1
1
1
1
si bn = ln n. Din relatia 1 <
un = n 1 + + + , an = 1 + + +
2
n
2
n
r

1
1
< n 1 + + + < n n, n 2 deducem c
a lim un = 1. Atunci, avem:
n
2
n
n
XI.29. Sa se arate ca lim
n ln ln n

r
n

n (un 1)
un 1
un 1
= lim
= lim
= 1.
(1)
n ln un
n ln [1 + (un 1)]
ln an
an
= 1. n consecinta,
Folosind criteriul lui Stolz-Cesro, g
asim c
a lim
n bn

ln (an /bn )
ln an
= lim
+ 1 = 1. De aici si din (1), obtinem c
a lim xn = 1.
lim
n ln bn
n
n
ln bn
XI.30. Fie f : R R o functie discontinua si care are proprietatea lui Darboux.
Daca exista o functie g : R R R astfel nct f (x + y) = g (f (x) , y), pentru orice
x, y R, atunci functia f nu are limita la .
S
tefan Alexe, Pitesti
Solutie. Dac
a f ar fi injectiv
a, cum f are proprietatea lui Darboux, ar nsemna
c
a f este continu
a, ceea ce contrazice ipoteza. Deci f nu este injectiv
a si atunci exist
a
a, b R, a < b astfel nct f (a) = f (b). Asadar, avem f (a + x) = g (f (a) , x) =
= g (f (b) , x) = f (b + x), x R, de unde rezult
a c
a f (x) = f (x + b a), x R,
adic
a f este periodic
a si T = b a este o perioad
a a ei. Cum f este discontinu
a
rezult
a c
a f nu este constant
a si deci exist
a , R, 6= astfel nct f () 6= f ().
Considernd sirurile xn = + nT si yn = + nT , n N, care tind la +, avem
f (xn ) = f ( + nT ) = f () f () si f (yn ) = f ( + nT ) = f () f (), ceea
ce demonstreaz
a c
a f nu are limit
a la +.
lim

Clasa a XII-a

a2
a2

1 a2
2
2
; a A ,
XII.26. Se considera multimea M = a
1
+
a
2
a

1
a 2 a 2
unde
A = Z sau A = Q sau A = R. Aratati ca (M, ) este grup; este acesta izomorf
cu A+ , ?
Gheorghe Costovici, Iasi
Solutie. Not
am cu M (a), a A, un element oarecare al multimii M . Deoarece
M (a) M (b) = M (a + b) (), a, b A, rezulta ca nmultirea este lege de compozitie intern
a pe M . Folosindu-ne de relatia () si avnd n vedere c
a adunarea este
asociativ
a si comutativ
a pe A rezult
a c
a nmultirea este asociativ
a si comutativ
a pe
M . Mai mult, se observ
a c
a M (0) este element neutru pentru nmultirea pe M si
orice M (a) M admite un simetric, si anume, M (a) M . Prin urmare, (M, )
a

este grup comutativ. n fine, se verific


a usor c
a func
tia f : M A+ , f (M (a)) = e
este un izomorfism ntre grupurile (M, ) si A+ , .
XII.27. Fie (G, ) un grup cu Z (G) 6= {e} si H un subgrup netrivial al lui G.
64

Sa se demonstreze ca exista x, y G\H, x 6= y 1 , astfel nct xy H si yx H.


Dati exemplu de grup care nu are aceasta proprietate.
Ovidiu Munteanu, student, Brasov
Solutie. Oricare ar fi x G\H si oricare ar fi u H\{e}, avem y = x1 u G\H
(ntr-adev
ar, dac
a y = x1 u H, atunci y 1 H si deci x = uy 1 H, ceea ce
a
este fals). De aici, deducem c
a xy = u H. Deoarece yx = x1 ux, trebuie s
mai demonstr
am c
a exist
a x G\H si u H\{e} astfel nct x1 ux H. Cum
Z (G) = {a G | ab = ba, b G} 6= {e}, rezulta ca exista x G\H si u H\{e}
astfel nct xu = ux, deci x1 ux = u H. Cu aceasta prima parte a demonstratiei
este ncheiat
a.

Pentru a doua parte, consider


am S3 = e, , , 2 , , 2 si H = {e, }. Se
observ
a c
a nu exist
a x, y G\H astfel nct xy = yx = . ntr-adev
ar, dac
a,
prin absurd, ar exista x, y G\H astfel nct xy = yx = , atunci y = x1 si
x1 x = , deci x = x. Cum nu comuta cu nici un element din G\H, nseamna
ca ultima egalitate este falsa.
Z

n
, pentru n {2, 3, 4}.
tg xdx, x 0,
XII.28. Calculati
2
Daniel Jinga, Pitesti
Z
Z

n
n
Solutie. Notnd In =
tg xdx, Jn =
ctg xdx si efectund schimbarea
Z
Z
n

t
tn2
n
tg x = t, obtinem: In = n
dt,
J
=
n
dt.
n
1 + t2n
1 + t2n
Pentru n = 3,

Z
Z
Z
1
u
1
u+1
t3
t2 =u 3
du etc.
dt =
du =
I3 = 3
+
1 + t6
2
1 + u3
2
u + 1 u2 u + 1
Pentru n = 2, consider
am
0

1
1
Z
Z 2
Z t2 1 2
t
t 1
t
t

dt = 2
I2 J2 = 2
dt = 2
dt.
2
1
1 + t4
1
t2 t2 + 2
t

2
t
t
Prin urmare,

tg x + ctg x 2

1
tg x ctg x

+C
+C, I2 J2 = ln
I2 +J2 = 2 arctg

2
2 2 tg x + ctg x + 2

si se calculeaz
a usor valoarea lui I2 .
Pentru n = 4, procedam la fel:
I4 J4

Z 1 1
t2 dt =
= 4
1
t4 + 4
t
0

1
Z
Z
t
du
t
= 4
dt = 4
etc.
4
2

4
u 4u2 + 2
1
1
t
+2
4 t
t
t
Z

t4 t2
dt = 4
1 + t8

65

XII.29. Fie f : R R o functie continua si t > 0. Pentru a, b > 0, sa se


calculeze

t
X
k t (k 1)t
b a+b
1
a
.
k (k 1)
lim
f n
n
nt
k=1
Mihail Bencze, Brasov
t t

0 1
nt
, , . . . , t o diviziune a intervalului [0, 1]. Atunci
Solutie. Fie n =
nt nt
n
k t (k 1)t
nt (n 1)t
=
max
0 pentru n . Lu
am drept
kn k =
nt
nt
k{0,1,...,n}
puncte intermediare media geometrica ponderata a punctelor de diviziune, adica
1

! a+b
t a
t b

t
k
(k 1)
b a+b
a
t

=
n
(k

1)
, k = 1, n.
k
k = t
n
nt
Atunci

n
t

t
X
k t (k 1)
b a+b
a
t
.
k
f
n
(1

k)
nt
k=1
Z 1
Cum f : [0, 1] R este continu
a, rezult
a c
a lim n (f, ) =
f (x) dx.
n
0
Z t
h
i
2 ln (1 + x)
e
XII.30. Sa se arate ca
ex
dx

ln
2,
ln
2
.
8
16
(1 + x2 )2
0
Cristian Moanta
, Craiova
2
ex
.
Solutie. Not
am cu I integrala din enunt si fie f : [0, 1] R, f (x) =
1 + x2
Deoarece ex 1 + x2 , x R, avem c
a f (x) 1; deoarece f este cresc
atoare pe [0, 1]
3 x2
2x
e
e
(c
aci f 0 (x) =
0, x [0, 1]), rezult
a c
a f (x) f (1) = . Ca urmare
2
(1 + x2 )2
Z 1
Z 1
e
ln (1 + x)
ln (1 + x)
dx I
dx
2
1
+
x
2
1 + x2
0
0
Z 1
ln (1 + x)

si r
dx = ln 2.
amne de ar
atat c
aJ=
2
1+x
8
0

ntr-adevar, cu schimbararile x = tg t si t = u, vom avea


4
Z 0
Z /4

ln (1 + tg t) dt =
ln 1 + tg
u du =
J =
4
0
/4
Z /4
Z /4
2
ln
ln 2du J,
du =
=
1 + tg u
o
0

de unde J = ln 2.
8

n (f, ) =

66

Solutiile problemelor pentru preg


atirea concursurilor
din nr. 1/2002
A. Nivel gimnazial
G6. Daca un numar natural se poate scrie ca suma a doua patrate perfecte
nenule distincte, atunci orice putere a sa se poate scrie, de asemenea, ca suma de
doua patrate perfecte nenule.
***
Solutie. Fie a = b2 + c2 , cu b, c N , iar n N . Dac
a n = 2k + 1, atunci

2 2
an = a2k a = a2k b2 + c2 = ak b + ak c , cu ak b, ak c N . Daca n = 2k, vom
demonstra
k = 1. ntr-adev
ar, a2 = b4 + c4 + 2b2 c2 =
2
2
mai nti2 afirmatia pentru
2 2
2

= b c
+ (2bc) , cu B = b c , C = 2bc N . Pentru k 2, avem c
a
2
k1 2 k1 2
n
2k2 2
2(k1)
2
k1
k1

B +C = a
a =a
a =a
B + a
C , cu a
B, a
CN .
G7. Aratati ca numarul aa . . . a (2001 cifre) nu poate fi patrat perfect, oricare
ar fi cifra a n baza 10.
***
Solutie. Afirmatia este adev
arat
a n cazul general al unui num
ar aa . . . a cu
n 2 cifre. Numerele 22 . . . 2, 33 . . . 3, 77 . . . 7 si 88 . . . 8 nu pot fi patrate perfecte
din cauza ultimei cifre. Cum orice p
atrat perfect este fie de forma 4k, fie de forma
4k + 1, k N, nu pot fi p
atrate perfecte numerele 11 . . . 1, 55 . . . 5, 66 . . . 6 si 99 . . . 9.
n sfrsit, dac
a 44 . . . 4 = 4 11 . . . 1 ar fi p
atrat perfect, atunci 11 . . . 1 ar fi p
atrat
perfect, absurd.
3n (18n + 13) 28
este fractie reductibila.
G8. Determinati n Z pentru care
3n + 1
Dumitru - Dominic Bucescu, Iasi
Solutie. Deoarece 3n (18n + 13) 28 = (3n + 1) (18n + 7) 35, fractia dat
a se
simplific
a prin d N\ {0, 1} dac
a d este un divizor al lui 35. Pentru d = 5, obtinem
ca 3n + 1 = 5l, l Z, ecuatie diofantica cu solutia particulara l = 1, n = 2 si cea
generala l = 1+3k, n = 2+5k, k Z. Pentru d = 7, gasim l = 3p+1, n = 7p+2,
p Z. n concluzie, valorile c
autate ale lui n sunt {5k 2 | k Z}{7p + 2 | p Z}.
G9. Se dau trei fisicuri de monede asezate vertical, asupra carora putem efectua
una dintre operatiile O1 : luam cele doua monede de deasupra unui fisic si le asezam
peste altul, sau O2 : luam cele doua monede de deasupra unui fisic si le asezam cte
una peste fiecare dintre celelalte doua fisicuri.
a) Gasiti o conditie necesara pentru ca, dupa un numar de operatii, toate fisicurile
sa contina la fel de multe monede;
b) Aratati ca aceasta conditie nu este suficienta daca este permisa o singura operatie, nsa este suficienta n cazul n care sunt permise amndoua.
Gabriel Popa, Iasi
Solutie. Deoarece num
arul total de monede r
amne constant pe parcursul efectu
arii operatiilor, acest num
ar trebuie s
a fie n mod necesar un multiplu de 3 mai
mare sau egal cu 6.
Presupunnd ca distributia initiala a monedelor este (3, 2, 1), n conditiile n care
este permis
a o singur
a operatie, se arat
a c
a egalizarea celor trei fisicuri nu este posibil
a
considernd toate misc
arile ce pot fi efectuate. n cazul n care ambele operatii sunt
67

permise, aseznd n mod repetat cte dou


a monede din fisicul cel mai nalt peste
cel mai mic, ajungem fie ca fisicurile s
a se egalizeze, fie ca n vrfurile lor s
a apar
a
O
O
o situatie de tipul (3, 2, 1). n aceast
a situatie, succesiunea (3, 2, 1) 2 (4, 0, 2) 1
(2, 2, 2) rezolv
a problema.
G10. Pentru n N, n 2, rezolvati ecuatia
r
r
r

n+1
n+1
n+1
x1 +
x2 + . . .
xn + x1 + x2 + + xn = n + 1.
n
n
n
Mihai Totolici, Galati
r

n+1
Solutie. Cu notatiile ui =
xi , x = 1, n, un+1 = x1 + x2 + + xn
n
obtinem ca u1 + u2 + + un+1 = n + 1, iar u21 + u22 + + u2n+1 = n + 1. De aici,

(u1 + u2 + + un+1 )2 = (n + 1) u21 + u22 + + u2n , deci este atinsa egalitatea n


inegalitatea Cauchy-Buniakowski-Schwarz aplicata numerelor u1 , u2 , . . . , un+1 ; 1,
1
1, . . . , 1. Urmeaza ca u1 = u2 = = un+1 = 1, de unde x1 = x2 = = xn = .
n
G11. Rezolvati n N N ecuatia x2 + y 2 = 5445.
Daniela Iosub, elev
a, Iasi
Solutie. Vom folosi urmatorul rezultat din teoria numerelor: daca p = 4k+3 este
un numar prim si p | a2 + b2 , a, b N , atunci p | a si p | b. Din ipotez
a, 3 | x2 + y 2
2
2

si 11 | x + y , x, y N , iar 3 sau 11 sunt numere prime de forma 4k + 3. Prin


urmare, 33 | x si 33 | y, deci x = 33l, y = 33m, l, m N . nlocuind n ecuatie,
obtinem ca l2 + m2 = 5, l, m N , adica (l, m) {(1, 2) , (2, 1)}. De aici, (x, y)
{(33, 66) , (66, 33)}.

G12. Sa se determine n, m N pentru care


1 + 2 + . . . [ n] = nm .
Adrian Zanoschi, Iasi
Solutie. Pentru n = 1, m N relatia data se verifica. Cautam solutii cu n 2.
Nu putem avea m 2, deoarece
h i h i

1 +
2 + . . . n 1 + 2 + + n < n n < n2 nm .
Ramne de cercetat cazul m = 1; se observa ca n = 2 si n = 3 dau solutii ale ecuatiei,
iar pentru n 4 obtinem
h i h i

1 +
2 + . . . n 1 + 1 + 1 + 4 + + n > 3 + 2 (n 3) = 2n 3 > n,
adic
a nu mai g
asim solutii. n concluzie, (n, m) {(1, a) | a N } {(2, 1) , (3, 1)}.

G13. Aratati ca numerele 18n si 2n + 18n , n N, au acelasi numar de cifre.


Gheorghe Iurea, Iasi
Solutie. S
a presupunem prin reducere la absurd c
a 18n are k cifre, iar 2n + 18n
are mai mult de k cifre, deci 2n + 18n 10k > 18n 10k1 . Evident, k > n si atunci
mpartind prin 2n aceasta inegalitate, obtinem:
kn k

1 + 9n 5k 2kn > 9n 2kn 5k = 1 + 9n ,


n

(1)
n

deoarece 2
5 N. Pe de alt
a parte, 1 + 9 = 1 + (8 + 1) = M 4 + 2, deci
2kn 5k = M 4 + 2, de unde k n = 1; relatia (1) devine 2 5n+1 = 1 + 9n (2).
68

Numerele n = 0, 1, 2, 3 nu verific
a (2), iar pentru n 4 avem c
a
n
9
= (1, 8)n (1, 8)4 = (3, 24)2 > (3, 2)2 = 10, 24 > 10,
5

a pentru n 4. Contradictia
adic
a 9n > 10 5n = 2 5n+1 , deci (2) nu este verificat
obtinut
a ncheie demonstratia.
G14. Sa se arate ca nu exista nici un triunghi dreptunghic avnd catetele numere
rationale, iar ipotenuza egala cu 2001.
Constantin Cocea, Iasi
x2
z2
Solutie. Pentru a ar
ata c
a ecuatia 2 + 2 = 2001 nu are solutii n N , este
y
t
suficient s
a demonstr
am c
a ecuatia m2 + n2 = 2001p2 (1) nu are solutii n N .
Folosind rezultatul amintit n solutia problemei G11 si observnd c
a 3 | 2001, obtinem

ca n mod necesar
m
s
i
n
sunt
multipli
de
3;
m
=
3m
,
n
=
3n
,
m
tia
1
1
1 , n1 N . Ecua

(1) devine 3 m21 + n21 = 667p2 si cum (3, 667) = 1, urmeaza ca p = 3p1 , p1 N .
Dup
a nlocuire, m21 + n21 = 2001p21 (2).
Dac
a presupunem c
a ecuatia (1) admite solutii, fie o asemenea solutie cu p minim.
Din (2) se obtine nsa o noua solutie cu p1 < p, contradictie! Urmeaza ca (1) nu are
solutii n N , de unde concluzia.
Not
a. Metoda folosit
a se numeste metoda coborrii si a fost utilizat
a n demonstrarea Marii Teoreme a lui Fermat n cazurile n = 3 si n = 4.

2
pG15. Sa se arate ca E(x, y, z) 3, daca E(x, y, z) = x 2x sin z 4 cos z + 5+
+ y 2 2y sin z 6 cos z + 10, x, y, z R.
Cristiana Artenie, elev
a, Iasi
Solutie. Se observa ca
q
q
E (x, y, z) = (x sin z)2 + (2 cos z)2 + (y sin z)2 + (3 cos z)2 = M P +M Q,
unde M (cos z, sin z), P (2, x), Q (3, y), x, y, z R. Punctul M parcurge cercul unitate C, iar punctele P si Q parcurg dreptele verticale d1 : x = 2, respectiv d2 : x = 3.
Minimul lui E (x, y, z) se atinge pentru {M } = C[Ox, {P } = d1 Ox, {Q} = d2 Ox;
n acest caz E (x, y, z) = 3, de unde concluzia.
G16. Fie M un punct n interiorul triunghiului echilateral ABC astfel nct

\
M A2 = M B 2+M C 2 2M BM C ; calculati masura unghiului BM
C. Generalizare.
Corneliu Br
ad
ateanu, Pascani
Solutie. n general, vom arata ca daca M A2 =
\
= M B 2 + M C 2 2M B M C cos , atunci m(BM
C) =

a
= + 60 . n situatia problemei date, va rezulta c
\
m(BM
C) = 105 .
Fie D n semiplanul determinat de BC opus lui A ast\) =
fel nct 4M BD este echilateral. Atunci m(ABM

\ = 60 m(M
\
= m(CBD)
BC), de unde 4ABM
4CBD (LUL), deci AM = DC. Cum M D = M B,
relatia de mai sus se scrie CD2 = M D2 +M C 2 2M D
\
\
M C cos , ceea ce arat
a c
a m(DM
C) = , adic
a m(BM
C) = 60 + .
69

G17. Fie ABCD un patrulater convex ce nu are diagonalele perpendiculare, B1 si


D1 proiectiile punctelor B, respectiv D pe AC, iar A1 si C1 proiectiile punctelor A,

2
BD
SBB1 DD1
2
respectiv C pe BD. Sa se arate ca
si SABCD
=
cos2 (BD, AC) =
SCC1 AA1
AC
= SBB1 DD1 SCC1 AA1 .
Claudiu-
Stefan Popa, Iasi
Solutie. Avem c
a SABCD = SACD + SACB =
AC BB1
BB1 + DD1
AC DD1
+
=
AC. Pe
=
2
2
2
de alt
a parte, BB1 DD1 este trapez sau paralelogram (BB1 , DD1 B1 D1 ) cu naltimea [B1 D1 ],
BB1 + DD1
deci SBB1 DD1 =
Atunci
B1 D1 .
2
B1 D1
SBB1 DD1
=
.
SABCD
AC
B1 O
BO
B1 D1
BD
Observ
am c
a 4DOD1 4BOB1 si de aici
=
, adic
a
=
,
D
O
DO
D
O
DO
1
1

SBB1 DD1
D1 O
BD

\
a c
a
BD). Rezult
deci B1 D1 = BD
=
= BD cos(AC,

DO
S
AC
ABCD

SCC1 AA1
AC

\
\
BD). Analog se obtine ca
BD). mpartind,
cos(AC,
=
cos(AC,
SABCD
BD
apoi nmultind membru cu membru ultimele dou
a egalit
ati, g
asim relatiile din concluzie.
b 90 . Pe latura (BC) se considera
G18. Fie ABC un triunghi cu m(A)
punctele M si N astfel nct AM si AN sa fie simetrice fata de bisectoarea unghiului
A. Cercul circumscris triunghiului AM N intersecteaza laturile AB si AC n E,
BC
respectiv F . Daca {I} = BF CE si {P } = AI BC, demonstrati ca AP
.
2
Florin Nicolaescu, Bals
\
\ N
AF , deci n
Solutie. Din ipotez
a, EAM
cercul C avem c
a EM F N , de unde EF kM N .
Fie {D} = AP EF ; atunci 4AED 4ABP si
ED
AD
4AF D 4ACP si va rezulta c
a
=
=
BP
AP
DF
ED
BP
=
, i.e.
=
(1). Din asem
an
arile
PC
FD
CP
4EID 4CIP si 4DIF 4P IB obtinem, ca
CP
ED
=
(2). Din (1) si (2) urmeaza
mai sus,
FD
BP
ca (BP ) (CP ).
BC
Presupunem prin reducere la absurd ca AP <
, adica AP < BP si AP < P C.
2
b si m(P[
b deci m(\
b
\) > m(B)
Atunci m(BAP
AC) > m(C),
BAP ) + m(P[
AC) > m(B)+
b i.e. m(A)
b > 90 , ceea ce contrazice ipoteza;
b de unde m(A)
b > 180 m(A),
+m(C),
problema este astfel rezolvat
a.
G19. Fie A1 A2 A3 un triunghi echilateral nscris n cercul C(O, R) si cercurile
70

Ci (i = 1, 2, 3) de aceeasi raza r, tangente interior cercului C n vrfurile Ai corespunzatoare. Sa se arate ca pentru orice P C(O, R) are loc relatia t21 + t22 + t23 =
constant, unde ti (i = 1, 2, 3) este lungimea tangentei dusa din P la cercul Ci .
Temistocle Brsan, Iasi
Solutie. Fie Oi centrul cercului Ci , i = 1, 3. Evident c
a 4O1 O2 O3 este echilateral, iar centrul s
au este
punctul O. Avem:
X
X
X

(1) X
(2)
P Oi2 r2 = 3r2 +
t2i =
P Pi2 =
P Oi2 =

X
2
= 3r2 + 3P O2 +
OOi2 = 3r2 +3R2 +3 (R r) =

= 6R2 6Rr = constant,


unde (1) se justific
a prin aplicarea teoremei lui Pitagora
n triunghiurile dreptunghice P Pi Oi , iar (2) prin relatia
lui Leibniz.
G20. Sa se arate ca pentru orice alegere a 12 numere naturale consecutive nu se
pot numerota muchiile unui cub astfel ca suma numerelor aflate pe trei muchii care
au un vrf comun sa fie aceeasi pentru toate vrfurile cubului (nu se numeroteaza
doua muchii cu acelasi numar). Sa se arate ca este posibila numerotarea descrisa
daca se aleg convenabil 12 numere dintre oricare 13 numere naturale consecutive.
Constantin Chiril
a, Iasi
Solutie. Fie n + 1, n + 2, . . . , n + 12, n N si sa presupunem prin absurd ca
suma numerelor de pe oricare trei muchii adunate adiacente este s. Obtinem ca 8s =
= 2[(n + 1) + (n + 2) + + (n + 12)], de unde, dup
a calcule, g
asim 2s = 3 (2n + 13).
Am ajuns evident la o contradictie, deoarece n stnga avem un num
ar par, iar n
dreapta unul impar.
Pentru partea a doua, fara a restrnge generalitatea,
putem considera numerele 1, 2, . . . , 13; cazul general se reduce imediat la acesta. Fie c num
arul pe care l vom elimina. Cu rationamentul de mai sus, obtinem 4s = 91 c
91 c
si cum s =
N, n mod necesar c {3, 7, 11},
4
deci s {22, 21, 20}. Pentru c = 7, vom da o asezare a
numerelor 1, 2, 3, 4, 5, 6, 8, 9, 10, 11, 12, 13 care s
a respecte
cerintele problemei: fiecare pereche de numere simetrice
fata de 7 de forma (p, 14 p) se scriu pe muchii simetrice fata de centrul cubului,
astfel nct suma ntr-un vrf s
a fie 21.

B. Nivel liceal
L6. Fie x1 , x2 , . . . , xn , n N\{0, 1}, numere reale cu proprietatea
x1
x2
xn
+
+ +
= 1,
S x1
S x2
S xn
P
unde S = ni=1 xi . Aratati ca
x32
x3n
x31
S2
+
+ +
.
S x1
S x2
S xn
n
R
azvan B
arbulescu, elev, Craiova
71

Solutie. Din relatia dat


a n ipotez
a, deducem succesiv:
x2
xn
Sx1
Sx2
Sxn
x1
+
+ +
=1
+
+ +
=S
S x1
S x2
S xn
S x1 S x2
S xn
xn (S xn ) + x2n
x1 (S x1 ) + x21 x2 (S x2 ) + x22
+
+ +
=S

S x1
S x2
S x2n
x21
x22
x2n
+ x2 +
+ + xn +
=S
x1 +
S x1
S x2
S xn
x21
x22
x2n
+
+ +
=0

S x1
S x2
S xn
x31
x32
x3n
+ x22 +
+ + x2n +
=0
x21 +
S x1
S x2
S xn

x32
x3n
x31
+
+ +
= x21 + x22 + + x2n .

S x1
S x2
S xn
r

x21 + x22 + + x2n x1 + x2 + + xn


nsa

, de unde rezulta imediat concluzia.


n
n

b > 60 , consideram medianele CN , BN 0 si bisecL7. n triunghiul ABC, m(A)


0
toarele BE, CE . Notam {P } = CN BE, {Q} = CE 0 BN 0 . Aratati ca punctele
P si Q nu pot fi ambele pe naltimea din A.
Ioan S
ac
aleanu, Hrl
au
Solutie. Sa presupunem prin absurd ca P si Q apartin naltimii (AD). Aplicnd
teorema lui Menelaus n 4ABD cu transversala N P C, apoi teorema bisectoarei
n acelasi triunghi, obtinem
AN BC DP
BC BD
a
c cos B
a
cos C

=1

=1

=1 =
.
N B CD P A
CD BA
b cos C
c
b
cos B
a
cos C
a
b
Repetnd rationamentul n 4ACD, obtinem c
a
=
, deci
= , adic
a
c
cos B
c
a
2
2
2
2
2
2
a = bc. nsa a = b + c 2bc cos A, de unde b + c bc (1 + 2 cos A) = 0, prin
2
b
b
urmare
(1 + 2 cos A) + 1 = 0 si cum = 4 cos2 A + 4 cos A 3 < 0 pentru
c
c
b
1
, deducem ca
cos A 1,
/ R, absurd.
2
c
Not
a. Solutie corect
a s-a primit de la Marius Pachitariu, elev, Iasi.
L8. Fie triunghiul ABC si M Int ABC, M A C (M BC) = {M, A1 },
M B C (M CA) = {M, B1 }, M C C (M AB) = {M, C1 }. Sa se arate ca
M A1 M B1 M C1
+
+
6.
MA
MB
MC
Neculai Roman, Mircesti, Iasi
\
\
\
B1 ), z = m(AM
C1 ); evident ca x+
Solutie. Fie x = m(BM
A1 ), y = m(CM
+y + z = 180 . Dac
a R1 este raza cercului prin M, B, C, avem: A1 B = 2R1 sin x,
A1 C = 2R1 sin z, BC = 2R1 sin (x + z) = 2R1 sin y. Aplicnd teorema lui Ptolemeu
72

n patrulaterul inscriptibil M BA1 C, obtinem succesiv:


M A1 BC = M B A1 C + M C A1 B M A1 sin y = M B sin z + M C sin x
M A1
M B sin z M C sin x

=
+
.
MA
M A sin y
M A sin y
Scriind relatiile analoage si adunndu-le, concluzia urmeaza imediat din faptul ca
1
a + 2, (0, ).
a
n cazul particular M = O, obtinem inegalitatea remarcabil
a
OA1 + OB1 + OC1 6R,

unde R este raza cercului circumscris 4ABC. S


a mai observ
am c
a egalitatea este
atins
a n triunghiul echilateral.
L9. Fie ABC un triunghi ascutitunghic cu a b c si u, v, w (0, ),
u v w. Daca uGA + vGB + wGC = (u + v + w) R, unde G este centrul de
greutate al triunghiului, iar R este raza cercului circumscris, atunci triunghiul ABC
este echilateral.
Paul Georgescu si Gabriel Popa, Iasi
Solutie. Fie f : P R, f (M ) = u |zM zA | + v |zM zB | + w |zM zC |.
2z0 + zH
2
, din inegalitatea modulului obtinem c
a f (G) f (O) +
Deoarece zC =
3
3
1
+ f (H). Din ipotez
a, f (G) = f (O), deci f (H) f (O). Pe de alt
a parte, aplicnd
3
h i
[0, 1], gasim ca
inegalitatea lui Jensen functiei concave cos : 0,
2

uA + vB + wC
.
f (H) = 2R (u cos A + v cos B + w cos C) 2R (u + v + w) cos
u+v+w
1
Din inegalitatea lui Cebsev, uA + vB + wC (u + v + w) (A + B + C), deci
3

f (H) 2R (u + v + w) cos = f (O). Am obtinut c


a f (H) = f (O) si atunci este
3
atins
a egalitatea n inegalit
atile Jensen si Cebsev, adic
a 4ABC este echilateral.

L10. a) Fie n N , n 2. Sa se arate ca exista o progresie aritmetica de


numere naturale care nu are nici un termen de forma xn , x N.
b) Daca o progresie aritmetica de numere naturale contine un termen de forma
xn , x N, atunci sa se arate ca progresia contine o infinitate de termeni de aceasta
forma.
Adrian Zanoschi, Iasi
Solutie. a) S
a demonstr
am c
a progresia aritmetic
a ak = 4k+2, k N, nu contine
nici un termen de forma xn , x N. ntr-adev
ar, acest fapt rezult
a din observatiile:
(4m)n = M4 , (4m + 1)n = M4 + 1, (4m + 2)n = M4 , (4m + 3)n = M4 1, m N.
b) Fie o progresie de numere naturale cu ratia r care contine un termen xn , x N.
n
Num
arul natural (x + r) este termen al progresiei, deoarece

(x + r)n = xn + nxn1 r + + rn = xn + nxn1 + + rn1 r.


r

Analog se demonstreaz
a c
a orice num
ar de forma (x + kr) , k N, este termen al
progresiei.
73

L11. Sa se rezolve n N ecuatia 2 3x = 3 2y + 174.


Daniela Iosub, elev
a, Iasi
Solutie. Cum x, y N , atunci a = x 1, b = y 1 sunt numere naturale.
mpartind ecuatia prin 6, obtinem ecuatia echivalenta 3a = 2b + 29, a, b N. Atunci
b
b
3a = (3 1) + 29, deci 3a = M3 + (1) + 29, prin urmare b trebuie s
a fie par,
b = 2k, k N (deoarece este evident c
a b = 0 nu convine). Obtinem 3a = 4k + 29,
i.e. (4 1)a = 4k +29, de unde M4 +(1)a = 4k +29, adic
a a trebuie
s
a fie par, a = 2l,

l N (a = 0 nu convine). n aceste conditii, ecuatia devine 3l 2k 3l + 2k = 29


si cum 29 este prim, iar 3l 2k < 3l + 2k , g
asim c
a 3l 2k = 1, 3l + 2k = 29. Sistemul
astfel format nu are solutii n N si atunci ecuatia initial
a nu are solutii n N .
L12. Fie ABCD un patrulater convex; notam {O} = AC BD, M mijlocul
lui (AB), N mijlocul lui (CD). Pentru propozitiile P1 : ABCD inscriptibil; P2 :
OM CD; P3 : ON AB, sa se arate ca: a) P1 P2 P3 ; b) P2 P3 P1 ;
c) P3 P1 P2 (n legatura cu problema C:2265 din G. M. 3/2000).
Viorel Cornea si Dan S
tefan Marinescu, Hunedoara
Solutie. a) Daca ABCD inscriptibil, din puterea punctului O fata de cercul
circumscris obtinem c
a OA OC = OB OD. Avem succesiv:

1
OA + OB OD OC = 0
P2 OM CD = 0
2

OA OD OA OC + OB OD OB OC = 0

OA OD OB OC + (OA OC OB OD) = 0
(1)

OA OD OB OC (OA OC OB OD) = 0


OA OB OD + OC = 0 2 AB ON = 0 AB ON.
b) Se procedeaza analog.
c) Dac
a OM CD, se obtine relatia (1). Din ON AB deducem analog

OD OB + OC OB OD OA + OA OC = 0

(2)

Adunnd (1) si (2), g


asim c
a OA OC = OB OD, adic
a ABCD este inscriptibil.

L13. Fie P R [X], P (X) = a0 X n + a1 X n1 + + an1 X + an , n 2, cu


a0 > 0 si cu toate radacinile pozitive si subunitare. Sa se arate ca (n 1) a0 + a1 +
n
+ (1) an > 0.
Gheorghe Molea, Curtea de Arges
a1
n
n an
Solutie. Avem: (n 1) a0 + a1 + (1) an > 0 (1)
< n1
a
a0
0
Pn
Qn
radacinile polinomului.
i=1 xi i=1 xi < n 1, unde xi (0, 1), i = 1, n, suntQ
P
Prin inductie completa, se dovedeste usor inegalitatea: 1 ni=1 (1 bi ) < ni=1 bi ,
bi (0, 1), i = 1, n, n 2.
Lu
am n aceasta bi = 1 xi , i = 1, n, si obtinem:
1
q.e.d.

n
Y

i=1

xi <

n
X
i=1

(1 xi ) 1

n
Y

i=1

xi < n

74

n
X
i=1

xi

n
X
i=1

xi

n
Y

i=1

xi < n 1,

X+1 1
1 ...
1

2 X 2 +2 2 . . .
2
.
L14. Pentru n N consideram polinomul Pn (X)=
... ... ...
. . .
...
n
n
n . . . X n +n
n (n + 1)
a) Aratati ca zero este radacina multipla de ordin
a acestui polinom;
2
b) Daca n este par, Pn nu are radacini reale nenule, iar daca n este impar, Pn are
o singura radacina reala nenula, care este simpla si situata n intervalul (2, 1].
Temistocle Brsan, Iasi
Solutie. Considernd ultima linie ca o sum
a de dou
a linii, avem:

X + 1
1
...
1
1

2
X2 + 2 . . .
2
2

...
...
...
. . . .
Pn (X) = X Pn1 (X) + n . . .
n1
n 1
n 1 ... X
+ (n 1) n 1

1
1
...
1
1
Sc
aznd ultima linie nmultit
a respectiv cu 1, 2, . . . , n1 din celelalte, obtinem relatia
de recurenta
Pn (X) = X n Pn1 (X) + nX n(n1)/2 ,
(1)
din care se deduce, prin calcule de rutin
a, c
a
Pn (X) = X n(n1)/2 Qn (X)
(2)
(3)
cu Qn (X) = X n + X n1 + 2X n2 + + (n 1) X + n.
Afirmatia a) rezulta direct din (2) si (3). Afirmatia b) n cazul n par rezulta
scriind polinomul Qn sub forma

1
3
3
Qn (X) = X n + X n1 + X n2 + X n2 + X n3 + X n4 +
2
2
2

5 n4
5
n3 2
n

3
+ X
+ 5X n5 + X n6 + +
X 4 + (n 3) X 3 +
X +
2
2
2
2

n1 2
+
X + (n 1) X + n
2
si observnd ca parantezele patrate au discriminantul nul, iar cele rotunde strict
negativ.
Dac
a n este impar, verific
am mai nti c
a Q0m are valori pozitive pentru x < 0;
ntr-adev
ar, proced
am ca mai sus, observnd c
a
n1
n2
0
Qn (X) = nX
+ (n 1) X
+ 2 (n 2) X n3 + + (n 2) 2X + (n 1) =

1 (n 1) n3
+ X n3 +
= nX n1 + 1 (n 1) X n2 +
X
2

3 (n 3) n5
3 (n 3) n3
+ X n5 + +
+
+ 3 (n 3) X n4 +
X
X
2
2

(n 4) 4 2
(n 4) 4 4
+
X + (n 4) 4X 3 +
X + X 2+
2
2

(n 2) 2 2
+
X + (n 2) 2X + (n 1) .
2
75

Partea a doua a afirmatiei b) rezult


a din faptul c
a Qn (x) > 0 pentru x > 0,
Qn (2) Qn (1) < 0 si functia x Qn (x) este strict cresc
atoare pentru x < 0.
L15. Fie R\Z si (cn )n1 un sir convergent de numere reale. Sa se arate ca
sirul (xn )n1 definit prin xn = {n + cn } nu este monoton.
Iuliana Georgescu si Paul Georgescu, Iasi
Solutie. S
a observ
am c
a {x} {y} {x y} = {x} {y}. Presupunem c
a
(xn )n1 este crescator. Atunci, deoarece acest sir este marginit, el va fi convergent
si xn+1 xn 0. Pe de alta parte, xn+1 xn = {(n + 1) + cn+1 } {n + cn } =
= { + cn+1 cn } .Dar + cn+1 cn si R\Z, deci { + cn+1 cn }
{} > 0. Prin urmare, xn+1 xn {} > 0. Absurd.
L16. a) Fie a < b si M = {f : [a, b] [a, b] ; f monotona}. Aratati ca exista
f M cu f (x) 6= x, x [a, b] si ca orice asemenea functie nu are proprietatea lui
Darboux.
b) Demonstrati ca f M , c [a, b] astfel nct f (c) [a + b f (c)] = c (a + b c).
S
tefan Alexe, Pitesti
Solutie. a) Functia f : [a, b] [a, b] definit
a prin f (x) = a + b x, dac
a
x [a, (a + b) /2) si f (x) = a, dac
a x [(a + b) /2, b], satisface conditiile enuntului.
Presupunem ca exista o functie f M fara puncte fixe si cu proprietatea lui Darboux pe [a, b]. Fiind monotona, f poate avea discontinuitati doar de prima speta;
avnd proprietatea lui Darboux, f nu are nici discontinuit
ati de acest fel. Rezult
a
c
a f este continu
a pe [a, b] si tot asa este si g : [a, b] R, g (x) = f (x) x. Cum
g (a) g (b) = [f (a) a] [f (b) b] 0, deducem ca c [a, b] astfel nct g (c) = 0,
adica f (c) = c. Atunci c este un punct fix al functiei f , ceea ce contrazice presupunerea f
acut
a.
b) Fie f M . Presupunem c
a f este cresc
atoare si not
am E = {x [a, b] ; f (x) x}.
Observam ca E este nevida si marginita (a f (a) si E [a, b]). Ca urmare,
c = sup E si, evident, c [a, b]. Din x c, x E, deducem ca x f (x) f (c),
x E. Deci f (c) este un majorant al multimii E si avem c f (c). Cum f
este cresc
atoare, rezult
a c
a f (c) f (f (c)), de unde deducem c
a f (c) E si, deci,
f (c) c. Asadar, f (c) = c (1).
Daca f este descrescatoare, atunci h : [a, b] [a, b], h (x) = a + b f (x) este
cresc
atoare si, procednd c
a mai sus, d [a, b] astfel nct h (d) = d, adic
a f (d) =
= a + b d (2).
Din (1) si (2) rezulta ca f M ecuatia [f (x) x] [a + b x f (x)] = 0 sau
f (x) [a + b f (x)] = x (a + b x) are solutii n [a, b], deci c [a, b] astfel nct
f (c) [a + b f (c)] = c (a + b c).
L17. Fie A un numar real pozitiv si f : [0, ) [0, ) o functie derivabila
pentru care f (0) = 0 si |f 0 (x)| Af n (x), x [0, ), unde n este un numar
natural dat, n 1, iar f n = f f f . Atunci f este identic nula.
Sorin Puspan
a, Craiova
Solutie. Este suficient s
a ar
at
am c
a f este identic nul
a pe orice interval de
forma [0, ], > 0. Presupunem ca > 0 astfel nct f nu-i identic nula pe [0, ],
adic
a avem M > 0, unde M = sup f (x). Cu teorema cresterilor finite obtinem:
x[0,]

x [0, ] are loc relatia f (x) = xf 0 (c), unde c (0, x); deci f (x) = x |f 0 (c)|
76

xA |f n (c)| AM , x [0, ]. Ca urmare, f (x) AM , x [0, ], de unde


M AM sau A 1.
Fie = (0, 1 , 2 , . . . , m1 , ) o diviziune a intervalului [0, ] cu A kk < 1
(). Daca f nu-i identic nula pe [0, 1 ], atunci M1 = sup f (x) > 0 si ca mai sus
x[0,1 ]

obtinem f (x) 1 AM1 , x [0, 1 ]; deducem ca M1 1 AM1 , adica 1 A 1,


ceea ce contrazice (). n concluzie f este identic nula pe [0, 1 ]. n particular, avem
f (1 ) = 0 si cu intervalul [1 , 2 ] proced
am la fel ca si cu [0, 1 ] etc.
Dup
a un num
ar finit de pasi, deducem c
a f este identic nul
a pe ntregul interval
[0, ]. Presupunerea initiala facuta este falsa. n concluzie, f este identic nula pe
[0, ).

L18. Fie A Mn (Z), n N, n 2 astfel nct In + sA este inversabila si


(In + sA)1 Mn (Z) pentru orice s {1, 2, . . . , n} .
1
a) Sa se arate ca In + kA este inversabila pentru orice k Z si (In + kA)
Mn (Z);
b) Daca A2 = On , sa se arate ca G = {In + kA; k Z} este grup n raport cu
nmultirea matricelor si sa se determine toate subgrupurile lui G.
Marian Ionescu, Pitesti
Solutie. a) Se arata usor afirmatia: C Mn (Z) este inversabila n Mn (Z)
det C = 1. Fie P (x) = det (In + xA), grad P n, P Z (X). Pentru orice
s {0, 1, 2, . . . , 2n}, avem: Cs = In + sA este inversabil
a n Mn (Z) det Cs =
= 1 P (s) = 1. Prin urmare, exist
a cel putin n + 1 numere s n care P ia
valoare 1 sau cel putin n + 1 numere s n care P este 1.
Consideram ca exista u1 , u2 , . . . , un+1 {0, 1, 2, . . . , 2n} astfel nct P (u1 ) =
= P (u2 ) = = P (un+1 ) = 1; analog se procedeaz
a n cel
alalt caz. Polinomul
Q (X) = P (X) 1, de grad cel mult n, se anuleaz
a pentru n + 1 valori distincte, deci
Q = 0 si P = 1. Rezulta ca det (In + kA) = 1, k Z si, n consecinta, concluzia
dorita.
b) Dac
a Cs = In + sA, Ct = In + tA, s, t Z, atunci, n ipotezele problemei,
Cs Ct = Cs+t . Se verific
a usor c
a G este grup n raport cu nmultirea matricelor
si ca (G, ) ' (Z, +) prin f : Z G, f (k) = In + kA. Deoarce subgrupurile lui
(Z, +) sunt de forma H = mZ, m 0, rezulta ca subgrupurile lui (G, ) au forma
{In + mkA; k Z} cu m N.
L19. Fie H un subgrup al grupului altern (A2002 , ). Daca

1 2 . . . 1999 2000 2001 2002


=
H
1 2 . . . 1999 2001 2002 2000

si 1 H, A2002 , sa se arate ca H = A2002 .


Lucian-Georges L
adunc
a, Iasi
Solutie. Se stie c
a grupul altern (An , ), n 3, este generat de cicluri de lungime
3. Pentru a demonstra ca H = A2002 este suficient sa aratam ca orice astfel de ciclu se
afla n H.
(, , {1, 2, . . . , 2002}).
Pentru aceasta, fie (, , ) un 3-ciclu oarecare
. . . a . . . b . . . 2000 2001 2002
, unde a, b, a0 , b0 sunt alese astFie =

. . . a0 . . . b0 . . .
a c
a (, , ) = 1 H.
fel nct A2002 . Se constat
77

L20. Fie a R, a > 1. Se considera functia f : [1, a] R de doua ori derivabila.


Sa se arate ca daca functia g : [1, a] R, g (x) = xf 0 (x) este monoton crescatoare,
atunci
Z a

f (t)
a ln a
f
dt.
t
1
Marcel Chirita
, Bucuresti
Solutie. Cum g este functie crescatoare, rezulta ca g 0 0, adica f 0 (x)+xf 00 (x)
0, x [1, a].
Fie functia h : [0, 1] R, h (x) = f (ax ). Avem: h0 (x) = ax ln a f 0 (ax ) si
00
h (x) = ax ln2 a [f 0 (ax ) + ax f 00 (ax )] 0, x [1, a], de unde rezult
a c
a h este
convexa. Conform inegalitatii lui Jensen, avem:

x1 + x2 + + xn
h (x1 ) + h (x2 ) + + h (xn )
h

, x1 , x2 , . . . , xn [0, 1] .
n
n
k
Pentru xk = , k = 1, n si trecnd la limita pentru n obtinem
n
Z 1
Z 1

1
h
h (t) dt, adic
a f
a
f (ax ) dx.

2
0
0
n ultima relatie efectuam schimbarea ax = t si obtinem inegalitatea ceruta.

LISTA MEMBRILOR FILIALEI IA


SI a S. S. M.1
- continuare din nr. 1/2000, 1/2001 si 1/2002 90. GALL Eduard
91. URSACHE Felicia-Camelia
ATIC

92. LAM
Lidia-Carmen
93. MACSIMIUC Delia
94. FARCASANU Ana-Corina
95. BAICAN Tatiana

96. BUCATARU
Mihaela

97. BUCATARU
Ion
Ines
98. CRETU
99. ASIMINOAIEI Ana
100. NAZARIE Elena
101. PSLARU Margareta Adriana
Mihaela
102. BOTRCA

Lucian-Georges
103. LADUNC
A
104. GOSMAN Neculai
105. ONICIUC Carmen-Elena
106. LUPULEASA Iuliana
Ecaterina
107. S
TIURCA
Alice
108. ANITA
109. PREDA Anisoara
1

Inginer, S.C. Easten, Iasi


S
coala gen. nr.36, Iasi
Grupul S
colar Agricol Holboca, Iasi
S
coala "Otilia Cazimir", Iasi
S
coala gen. nr.36, Iasi
Colegiul "C.Negruzzi", Iasi
Colegiul "E.Racovita", Iasi
Fac. de matematic
a, Univ. "Al.I.Cuza", Iasi
S
coala gen. nr.42, Iasi
Liceul de chimie, Iasi
Liceul de chimie, Iasi
S
coala prof. speciala, Tg. Frumos (Iasi)
S
coala gen. nr.10, Iasi
Liceul de informatic
a "Gr.Moisil", Iasi
S
coala "G.Ibr
aileanu", Tg.Frumos (Iasi)
S
coala nr.6 "M.Busuioc", Pascani
Grupul S
colar "M.Sturza", Iasi
Colegiul National, Iasi
S
coala "D.D.P
atr
ascanu", Tomesti (Iasi)

Lista va fi continuat
a n numerele urm
atoare.

78

Probleme propuse
Clasele primare
P.44. Un vecinul al unui vecin al numarului 81 este egal cu un vecin al unui vecin
al num
arului 77. Despre ce num
ar este vorba?
( Clasa I )
Mihaela Rusu, elev
a, Iasi
P.45. Adunnd trei numere naturale a, b, c obtinem suma 62. Primul numar este
mai mare dect al treilea si mpreun
a au suma 12. Care sunt cele trei numere?
( Clasa a II-a)
nv. Maria Racu, Iasi
P.46. Mihai, Dan si Petru practica fiecare un alt fel de sport si anume: tenis,
fotbal sau volei. Mihai si voleibalistul locuiesc n acelasi bloc. Cel care joaca volei si
cel care joac
a fotbal l-au urm
arit pe Petru la un meci. Ce sport practic
a fiecare?
( Clasa a II-a)
Adina Dohotaru, elev
a, Iasi
P.47. Diferenta a doua numere este 48. Aceasta diferenta este cu 22 mai mare
dect jum
atatea unuia dintre ele. Determinati numerele.
( Clasa a III-a)
nv. Rodica Rotaru, Brlad
P.48. Un agricultor mparte un teren n trei parcele. n fiecare an, fiecare parcela
este cultivat
a numai cu una din culturile: gru, porumb sau legume. ncepnd cu
anul 2003, agricultorul se hot
ar
aste ca pe fiecare parcel
a s
a fie alt
a cultur
a n trei ani
consecutivi.
a) Care este primul an dupa 2003 n care se repeta culturile pe cele trei parcele?
b) Se poate preciza care este ordinea culturilor pe cele trei parcele n anul 2019?
( Clasa a III-a)
Andreea Surugiu, elev
a, Iasi
P.49. La un moment dat, cernd unei persoane anul nasterii, aceasta raspunde:
"anul acesta mplinesc 25 ani, iar dac
a as scrie toate numerele ncepnd cu 1 si
terminnd cu anul nasterii si apoi toate numerele ncepnd cu 1 si terminnd cu anul
n care ne afl
am mi-ar trebui 13710 cifre. n ce an ne aflam cnd am pus ntrebarea?
( Clasa a III-a)
Prof. C
at
alin - Cristian Budeanu, Iasi
P.50. a) Cte numere trebuie ad
augate sirului 1, 2, 4, 5, 7, 8, . . . , 97, 98 pentru a
obtine toate numerele de la 1 la 98?
b) Efectuati 1 + 2 + 4 + 5 + 7 + 8 + + 97 + 98 2 (3 + 4 + 5 + + 34).
( Clasa a IV-a)
Georgiana Ciobanu, elev
a, Iasi
P.51. Produsul a dou
a numere naturale este 913 368. Unul din numere are cifra
unitatilor si cifra zecilor mai mare ca 2 si mai mica dect 8. Daca la acest numar
marim cifra zecilor cu 2 si micsoram cifra unitatilor cu 1, obtinem un produs egal cu
951 425. Aflati cele dou
a numere.
( Clasa a IV-a)
nv. Elena Z
arnescu, Iasi
P.52. n trei cutii sunt 212 bile. Din prima cutie se scoate un numar de bile, din a
doua de 2 ori mai mult si nc
a dou
a bile, din a treia se scoate ct triplul num
arului de
bile scos din a doua cutie. n fiecare cutie r
amne un num
ar de bile egal cu num
arul
total al bilelor scos din cele trei cutii la un loc. Cte bile au fost n fiecare cutie?
( Clasa a IV-a)
nv. Maria Racu, Iasi
P.53. Efectund o singur
a cnt
arire, s
a se ia 475g dintr-un kilogram de zah
ar
79

utiliznd dou
a greut
ati, una de 200g si cealalt
a de 150g.
( Clasa a IV-a)
Prof. Petru Asaftei, Iasi

Clasa a V-a
artite
V.36. Fie n un num
ar impar, iar a1 , a2 , . . . , an , n N numere care mp
la n dau cturi distincte si resturi distincte. Ar
atati c
a valoarea minim
a a sumei
S = a1 + a2 + + an este multiplu de 12.
Dragos Ungureanu, elev, Iasi
333331
222221
si b =
V.37. Comparati fractiile a =
.
333334
222223
Maria Cojocaru, Iasi
V.38. S
a se arate c
a 2a + 2b + 2c + 2d + 2e 6= 2003, a, b, c, d, e N.
Irina Ispas, student
a, Iasi
V.39. Sa se determine numerele prime p1 < p2 < p3 < p4 astfel nct numerele
p1 + p2 + p3 + p4 , p3 p2 , p4 p3 sa fie, de asemenea, prime.
Petru Minut, Iasi
V.40. Este posibila o partitionare a multimii {1, 2, . . . , 12n + 9} n 2n + 3 submultimi disjuncte, fiecare cu cte trei elemente, astfel nct n fiecare submultime un
element s
a fie suma celorlaltor dou
a?
Titu Zvonaru, Bucuresti

Clasa a VI-a
VI.36. Fie k N, k 3. Aratati ca printre valorile naturale ale lui n care fac
.
a cel putin trei p
atrate perfecte.
adev
arat
a propozitia n2 + k .. n + k, exist
Claudiu S
tefan Popa, Iasi
VI.37. Numerele 1160, 1604 si 2270 dau acelasi rest la mp
artirea prin n. Aflati
mpartitorul n.
Cristian Laz
ar, Iasi
VI.38. Demonstrati c
a nu exist
a numere naturale x, y, z direct proportionale cu
trei numere naturale consecutive, astfel nct x + y + z sa fie numar prim.
Alexandru Negrescu, elev, Botosani
VI.39. Radu si Mihai joaca de mai multe ori un joc n urma caruia cstigatorul
primeste a puncte, iar cel care pierde primeste b puncte (a, b N , a > b). Dac
a
scorul final este 6149 n favoarea lui Radu, iar Mihai a cstigat 4 partide, aflati a si b.
Adrian Zanoschi, Iasi

b
VI.40. Fie 4ABC cu m(A) = 120 . Perpendiculara n C pe AC intersecteaz
a
mediatoarea lui [AB] n D; notam {E} = CD AB. Sa se arate ca AB = 2AC daca
\ = 90 si BE = 2AB.
si numai dac
a m(BDE)
Ioan S
ac
aleanu, Hrl
au

Clasa a VII-a
VII.36. Sa se arate ca

1
+
n

2
+ +
n

2n 1
< 2n 1, n N, n 2.
n
C
at
alin Calistru, Iasi

VII.37. Ar
atati c
a n baza de numeratie 7 printre numerele ce se scriu cu cifrele
0, 1, 2 exist
a o infinitate care sunt p
atrate perfecte si o infinitate ce nu sunt p
atrate
80

perfecte. Aceste afirmatii r


amn valabile dac
a se folosesc cifrele 3, 5, 6?
Ruxandra Ioana Vlcu, elev
a, Iasi
abb . . . bc
ac
=
ca
cbb . . . ba
(termenii primei fractii continnd cte 2003 cifre b), atunci b = a + c.
Mihaela Buc
ataru, Iasi
VII.38. Fie a, b, c cifre nenule, a 6= c. Sa se arate ca daca

VII.39. Dac
a x < y < z sunt lungimile laturilor unui triunghi dreptunghic,
atunci xn + y n 6= z n , n N, n 3.
Dumitru Neagu, Iasi
b = 60 , iar M Int ABC
VII.40. Fie ABC un triunghi ascutitunghic cu m(A)
\
astfel nct m(BM
C) = 150 . Not
am cu P, Q, R proiectiile lui M pe BC, CA si
respectiv AB. Sa se arate ca 4P QR este dreptunghic.
Constantin Cocea, Iasi

Clasa a VIII-a
VIII.36. Determinati cardinalul minim al unei multimi B pentru care putem
defini functii f : R B astfel nct f (1) < 0 si f (xy) = f (x) f (y), x, y R.
Iulia Zanoschi, elev
a, Iasi
VIII.37. If a, b, c (0, ) prove
the following inequalities:
a) (a + b + c)3 a3 + b3 + c3 24 where abc = 1;

3
8 3
3
3
3
where ab + bc + ac = 1.
b) (a + b + c) a + b + c
3
Zdravko Starc, Vr
sac, Jugoslavia
VIII.38. Fie n N fixat. Ar
atati c
a exist
a o infinitate de numere x, y, z Z
astfel nct x2n + y 2n + z 2n = x2n+1 + y 2n+1 + z 2n+1 .
Lucian Tutescu, Craiova
VIII.39. Fie ABCD un patrulater strmb cu [AD] [BC]. Sa se construiasca
dreptele paralele d1 , d2 , d3 , d4 astfel nct A d1 , B d2 , C d3 , D d4 si
dist (d1 , d4 ) = dist (d2 , d3 ).
Horia Mihail Teodorescu, elev, Iasi
VIII.40. Fie ABCDA0 B 0 C 0 D0 un cub, iar O (BB 0 ). Dreptele A0 O si C 0 O
intersecteaz
a (ABC) n E, respectiv F , iar AO si CO intersecteaz
a (A0 B 0 C 0 ) n E 0 ,
0
respectiv F .
a) Aratati ca EF E 0 F 0 nu depinde de pozitia lui O;
b) Ar
atati c
a SBB 0 E0 E SABCD si determinati O pentru care se atinge egalitatea.
Monica Nedelcu, Iasi

Clasa a IX-a

y
IX.36. Determinati x < 0 < y astfel nct xy + = y 3 5y + 2.
x
Cezar Lupu, elev, Constanta

IX.37. Pentru x [1, ), n N , demonstrati inegalitatea

n+1
+ 1 (xn 1) 2nxn (x 1) .
x

Marius Pachitariu, elev, Iasi

81

xn+1 y n+1 z n+1


+ n + n x + y + z, x, y, z > 0, n N.
yn
z
x
Gigel Buth, Satu Mare
1
2
1
.
+ q
=
IX.39. S
a se rezolve ecuatia q
[x] [x + 2]
2 [x]3
3 3 [x] [x + 1]3
Daniel Jinga, Pitesti
IX.40. Fie M 6= G n planul 4ABC si D, E, F mijloacele laturilor [BC],

[CA] si respectiv [AB]. Consider


am punctele X, Y, Z astfel nct XD = mXM ,

Y E = mY M , ZF = mZM , m 6= 1.
2m
3
a) Dac
a m 6= , atunci AX, BY, CZ sunt concurente n S, cu SG =
SM .
2
3
3
b) Dac
a m = , atunci AX, BY, CZ sunt paralele cu GM .
2
Virgil Nicula, Bucuresti
IX.38. S
a se arate c
a

Clasa a X-a

X.36. S
a se rezolve inecuatia alogb x + xlogb x a + b, unde a, b (1, ).
Daniela Dodan, elev
a, Iasi
X.37. Fie a, b (0, 1) (1, ) si functia injectiva f : (0, ) R astfel nct
a. S
a se arate c
a ab = 1 si c
a
functia g : R R, g (x) = f (ax ) + f (bx ) este constant
exist
a functii f care satisfac ipotezele problemei.
Dan Popescu, Suceava
X.38. Fie a, b, c, d R cu a > b > c > d. S
a se arate c
a a, b, c, d sunt n progresie

3
ad
aritmetica daca si numai daca (a b) (b c) (c d) =
.
3
A. V. Mihai, Bucuresti
0 0 0 0
X.39. Fie ABCDA B C D un paralelipiped dreptunghic cu dimensiunile AB = a,
AD = b, AA0 = c. Dac
a M Int A0 B 0 C 0 D0 , not
am cu , , m
asurile unghiurilor
pe care AM le face cu AB, AD si respectiv AA0 . S
a se arate c
a
AM < a cos + b cos + c cos < AC 0 .
C
at
alin Calistru, Iasi
X.40. a) Pentru x, y, z 0, demonstrati inegalitatea
p

x + y + x + z + y + z xy + xz + yz 3 6xyz.
b) Cu notatiile uzuale, n orice triunghi are loc inegalitatea

2
2
2

a b + ( a c) +
b c
9
R
.
2

2
r
4
a+ b+ c
Marian Tetiva, Brlad

Clasa a XI-a
XI.36. Fie D, M doua matrice nesingulare de ordin n, D diagonala, iar M
a se arate c
a M este tot o matrice diagonal
a,
triunghiular
a. Dac
a D = t M DM , s
avnd 1 pe diagonala principal
a.
Adrian Corduneanu, Iasi
82

XI.37. Fie A M3 (C) astfel nct det (A + tA) = 0, unde R\{1, 0, 1}.
2 ( 1)2
det A.
Sa se arate ca det (A + tA) =

Marian Ionescu, Pitesti si Lucian Tutescu, Craiova


XI.38. S
a se determine functiile continue f : [0, ) [0, ) pentru care
f (f (x)) + 2f (x) = 3x, x 0.
Mihail Bencze, Brasov
n
P
XI.39. Fie sirul (yn )n1 astfel nct sirul
yi
este convergent. Dac
a
i=1

n1

(xn )n1 R+ are proprietatea c


a xn xn+1 (1 + xn yn+1 ), n 1, ar
atati c
a sirul

1
este convergent.
xn n1
Gheorghe Molea, Curtea de Arges
XI.40. Fie x0 [1, 1]; ar
atatic
a pentru orice n N, ecuatia 3x 4x3 = xn
1 1
a sirurile (xn )n0 si (3n xn )n0
are o singur
a solutie xn+1 , . Demonstrati c
2 2
sunt convergente si calculati limitele lor.
Marian Tetiva, Brlad

Clasa a XII-a

1 are solutie
XII.36. Sa se determine n N, n 2 pentru care ecuatia x2 = x + b
unic
a n Zn ; rezolvati ecuatia n acest caz.
Andrei Nedelcu, Iasi
XII.37. Fie (G, +) un subgrup al grupului (R, +). S
a se determine morfismele
cresc
atoare de la (G, +) la (R, +).
Dan S
tefan Marinescu si Viorel Cornea, Hunedoara
XII.38. Determinati functiile derivabile f, g : R R astfel nct f 0 (x) = g (x)+x
si g 0 (x) = f (x) x, x R.
Gheorghe Iurea, Iasi
XII.39. Fie f, g : (0, ) R astfel nct lim f (x) = lim g (x) = , iar
x
Z 1x
f (x)
xg(n)
lim
= R. S
a se calculeze lim f (n)
dx, unde [1, ).
x g (x)
n
0 x+
Adrian Sandovici, Piatra Neamt
XII.40. Fie f : [0, 1] R o functie derivabil
a cu derivata continu
a astfel nct
f (x)
0
exist
a si este finit
a. S
a se arate c
a
xf (x) f (x), x [0, 1], iar lim
x0 x
x>0
Z 1

Z 1
f (x)
f (x) dx,
f (1) min 2
dx .
x
0
0
Marcel Chirita
, Bucuresti

83

Probleme pentru preg


atirea concursurilor
A. Nivel gimnazial
G36. Fie x, n N astfel nct x divide 10n 1, ns
a x nu divide 10k 1 pentru
.
k < n. Sa se arate ca x divide 10m 1 daca si numai daca m .. n.
N. N. Hrtan, Iasi
G37. 2n muzicieni (n > 2) particip
a la un festival. La fiecare concert, o parte
dintre ei cnta iar ceilalti asculta. Sa se determine numarul minim de concerte astfel
nct fiecare muzician sa-i asculte pe toti ceilalti.
Titu Zvonaru, Bucuresti
G38. Multimea A Z are cinci elemente. Adunnd n toate modurile posibile
cte trei elemente din multime, obtinem urm
atoarele 10 sume: 3, 6, 8, 10, 11, 13,
15, 16, 18, 20. Determinati multimea A. (n leg
atur
a cu o problem
a de concurs din
Iugoslavia.)
Gabriel Popa, Iasi
G39. Fie xi R, i = 1, n, unde n 2003, astfel nct

x1 (n + 1) x2 + nx3 n 1

....................................
xn2 (n + 1) xn1 + nxn n 1

xn1 (n + 1) xn + nx1 n 1 n2

xn (n + 1) x1 + nx2 2n 1.
Dac
a x1 = 1, s
a se calculeze x2003 .
Romeo Cernat, Iasi
2
G40. Comparati numerele reale a si b, stiind c
a a 14a + b2 + 6b + 33 = 0.
Bogdan R
aducanu, elev, Iasi
G41. Daca 0 < x y z, sa se arate ca
x z
x y
x
x2 y 2
z2
y
z
z
3 + + + + + 1 + 2 + 2 + 2.
z
y x
y
z
x
z
x
y
z
x
Ovidiu Pop, Satu Mare
G42. Determinati a, b R dac
a [x] + [x + a] = [bx], x R.
Gheorghe Iurea, Iasi
d
G43. Fie xOy un unghi oarecare si P un punct n interiorul s
au. Se consider
a
punctele A, B [Ox cu A (OB) si C, D [Oy cu C (OD) astfal nct triunghiurile P AB si P CD sa fie echilaterale. Aratati ca dreptele OP , AD si BC sunt
concurente daca si numai daca P se afla pe bisectoarea unghiului dat.
Temistocle Brsan, Iasi
G44. Fie V ABC o piramida, iar G centrul de greutate al 4ABC. Un plan
ce trece prin G taie dreptele V A, V B, V C n A0 , B 0 si respectiv C 0 . Sa se arate ca
VA
VB
VC
+
+
= 3.
V A0
V B0
V C0
Constantin Cocea, Iasi
G45. Fie SABC un tetraedru n care 4ABC nu este echilateral, iar muchiile [SA] , [SB] , [SC] nu sunt toate congruente. Demonstrati ca exista sase puncte
A1 , B1 , C1 , A2 , B2 , C2 pe dreptele SA, SB, SC, BC, AC si respectiv AB astfel ca patrulaterele A1 B1 A2 B2 , B1 C1 B2 C2 si A1 C1 A2 C2 s
a fie trapeze izoscele (A1 B1 kA2 B2 ,
84

A1 C1 kA2 C2 , B1 C1 kB2 C2 ) dac


a si numai dac
a

SA2 AB 2 AC 2 + SB 2 BC 2 BA2 + SC 2 CA2 CB 2 = 0.


Daly Marciuc, Satu Mare

B. Nivel liceal

L36. Fie 4ABC si M triunghiul sau median. Daca P este un punct aflat n
interiorul sau pe laturile lui M, iar A0 , B 0 , C 0 sunt intersectiile dreptelor AP , BP ,
1
AP BP CP
8
CP cu laturile BC, CA si respectiv AB, atunci <
.

4
AA0 BB 0 CC 0
27
Marian Ionescu, Pitesti
L37. Fie cercurile C1 , C2 si C astfel nct C1 si C2 sunt tangente exterior n D, iar
cercurile C1 si C2 sunt tangente interior lui C n B, respectiv C. Tangenta comun
a
interioar
a cercurilor C1 si C2 taie cercul C n A si A1 , dreapta AB taie C1 n K, iar
1
2
1
=
AC taie C2 n L. Sa se arate ca
+
.
DA DA1
KL
Neculai Roman, Mircesti (Iasi)
L38. Fie 4ABC si punctele D, D0 BC conjugate armonic n raport cu vrfurile
a AB n M si AC n N . Ar
atati
B si C. Cercul circumscris 4ADD0 intersecteaz
b (interioar
c
a, dac
a M N BC, atunci [AD si [AD0 sunt bisectoarele unghiului A
a si

b
exterioar
a) sau m(A) = 90 .
Temistocle Brsan, Iasi
an (an + 2)
L39. Determinati toate numerele naturale nenule n pentru care
este
p (p + 1)

p
atrat perfect, unde a, p N .
Mihai Haivas, Iasi

2
2
a se arate
L40. Fie A, B Mn (Z) astfel nct det A B + AB este impar. S
c
a A + B este inversabil
a pentru orice Q.
Marian Urs
arescu, Roman
L41. Demonstrati c
a grupul simetric S32 nu are elemente de ordin 2002.
Paul Georgescu si Gabriel Popa, Iasi
L42. Fie
(A,
+,
)
un
inel
finit
cu
cel
pu
tin 5 elemente si cu 1 + 1 A inversabil.

a se arate c
a card M = card I <
Fie M = x A | x2 = 1 , I = x A | x2 = x . S
< card A /2.
Ovidiu Munteanu, Brasov
L43. Determinati polinoamele P R [X] pentru care P (z) C\R, z C\R.
Gheorghe Iurea, Iasi
L44. Fie n 2 numar natural, iar f0 , f1 , f2 , . . . un sir de polinoame definit
prin: f0 = (X + 1)n , fp+1 = X fp0 , p 0. Definim nca hp = fp p1
fp1 +
1
X
p1 p1
n
+ + (1)
p1 f1 , p 1, unde k =
i1 i2 . . . ik , k {1, 2, . . . , n},
1i1 <<ik n

sunt sumele simetrice fundamentale ale numerelor 1, 2, . . . , n. S


a se arate c
a
hp = n (n 1) . . . (n p + 1) X p (X + 1)np , p = 1, 2, . . .
Marian Tetiva, Brlad
L45. Fie f : [0, ) [0, ) continua. Daca functia F : [0, ) R, F (x) =
Z 1
Z x
f (t) dt este marginita, sa se arate ca lim n xf (nx) dx = 0.
=
n
0
0
Adrian Zanoschi, Iasi
85

Pagina rezolvitorilor
BOTO
SANI
Scoala
nr. 7 "Octav Bancila". Clasa a VIII-a. NEGRESCU Alexandru: VI(26
28,30,32,34), VII(26,32), VIII(26,27,32,34), G(7,11,32).
BRA
SOV
Raluca: V(26,31,33), VI(34,
Scoala
generala nr. 5. Clasa a VI-a. POSTEUCA

Bogdan: V(26,31,33), VI(34,35), VII(34).


35), VII(34). Clasa a VII-a. POSTEUCA
Scoala
generala nr. 20. Clasa a VII-a. BOERIU Adela: VI(31-35), VII(32,35).

Liceul "N. Titulescu". Clasa a IX-a. ANDRAS Cristian: VII(26,29,32,34,35),


VIII(32); BORICEAN Mihai: VII(26,29,32,34,35), VIII(32,33); BURLACU Alen:
Andreea: VII(26,29,32,34,35), VIII
VII(26,29,32,34,35), VIII(27,32,33); CIOBOTA
Cristian: VII(26,29,32,34,35), VIII(32,33); CMPEAN Simona:
(32,33); CIOBOTA
VII(26,29,32,34,35), VIII(27,32,33); COSTEA Rodica: VII(26,29,34), VIII(32,33);
FERAR Achim: VII(26,29,32,34,35), VIII(27,32,33); FUNDUREANU Alexandra:
VII(26,29,32,34,35), VIII(32,33); GHILIFTOIU Mirela: VII(26,29,32,34,35); GOICEA Ovidiu: VII(26,29,32,34,35), VIII(27,32, 33); MANEA George: VII(26,29,32,34,
35), VIII(32); MIHALCEA C
at
alin: VII(26, 29,32,34,35), VIII(32); MNZAT Mihai: VII(26,29,32,34,35), VIII(27,32,33); MUNTEAN Alexandru: VII(26,29,32,35),
VIII(32); MUNTEANU Luminita: VII(26,32,24), VIII(32,33); PUCHEANU Bogdan:
VII(26,29,32, 34,35), VIII(27,32,33); RSCU Laura: VII(26,27,29,34,35), VIII(32,33);
VLAD Daniel: VII(26,29,32,34,35), VIII(27).
CRAIOVA

Colegiul National "Fratii Buzesti". Clasa a VI-a. TUTESCU


Anca-Stefania:
V(31,35), VI(31,32,34), VII(31,32), G(21); Clasa a VIII-a. DINU Lavinia: VII(31),
VIII(31,32), G(21,23,26).
FOC
SANI
Andreea: VI(31-35),
Colegiul National "Unirea". Clasa a VIII-a. SECARA
VII(31,32), VIII(32,34,35), G(21,23,25).
(IA
HRLAU
SI)
Liceul Teoretic "Stefan
cel Mare". Clasa a VII-a. ANTOCI Bogdan: V(27,28,

30), VI(29,33,34,35), VII(29,32); BURICAN Bogdan Alexandru: V(28,30), VI(26,29),


Lucian: V(27,28,30,32), VI(29,34,35), VII(29,31,32); PASCARIU
VII(29); MIHULCA
Marian-Dragos: V(27,28,30,32,33), VI(29,33-35), VII(29); ROTARU Lucian: V(2628,32), VI(29,32-35), VII(29).
IA
SI
Colegiul National "C.Negruzzi". Clasa a VI-a. ROSU Eugenia: V(31-35),
VI(31-35), VII(32), VIII(32), G(21,23,27). Clasa a X-a. IACOB Alin: IX(32,34,35),
L(21,28,29).
Liceul "Garabet Ibraileanu". Clasa a VI-a. BUDEANU S
tefana: P(41,42),
V(31-33), VI(31); FUIOREA Bogdan: P(42), V(31,32), VI(31,32); UNGUREANU
Dragos: P(42), V(31-33), VI(31,32). Clasa a VIII-a. ANDRIESCU Alina: VI(33 STEANU S
35), VII(32), VIII(34); BRANI
tefana:VI(35), VII(31,32), G(22,23); JUVERDEANU George: VI(33,34), VII(32,35), VIII(34); MOROSANU Mircea: VI(33
35), VII(31,32); TANASE
Ioana: VI(33-35), VII(31,32); TUDORACHI Lucia: VI(34,
86

35), VII(31,32), VIII(32). Clasa a X-a. TONU Constantin: VIII(34), IX(28),


X(33), G(32,33).
Liceul Teoretic "M.Eminescu". Clasa a VIII-a. AVRAM Mircea: VI(25,27,3335), VII(26,28,29,32),VIII(27); CIUCANU Radu: VI(27,34,35), VII(26,29,32), VIII
AIL
A
Mihai: VI(27,33-35), VII(26,29,32), VIII(27); DUSA Cristian: VI
(27); DAN
(27,34), VII(31,32), VIII(33); TOFAN Andrei: VI(27,34,35), VII(26,29,32), VIII(27);
TUDOSE S
tefan: VI(27,34,35), VII(26,32), VIII(27); TURLIUC Rares: VI(31,3335), VII(29); GRAMSCHI Raluca: VI(26,27,34,35) (4 probleme). Clasa a IX-a.
DUMITRESCU Roxana: VII(26,29,31,32), VIII(31,32,34,35), IX(31).
OanaScoala
"G.Cosbuc". Clasa a II-a (nv. GALIA Paraschiva). CIOABA

C
at
alina: P(24,27,34-36); MIHAILESCU
Laura-Ioana: P(24,27,34-36); SCUTARU
Ioana:
Constantin: P(24,27,34-36). Clasa a II-a (nv. RACU Maria). BARABULA
P(24,27,34-36); BURLACU Claudiu: P(24,27,34-36); CALOIAN Andrei: P(24,27,34

36); CALIN
Georgiana: P(24,27,34-36); CRACIUN
M
ad
alina: P(24,27,34-36); LEA Crina-Alexandra: P(24,27,34-36); MOISA Bogdan: P(24,27,34-36); PINTILIE
GAN
Razvan: P(24,27,34-36); RUSU Flavia: P(24,27,34-36).
Scoala
"Al. Vlahuta". Clasa a IV-a (nv. MAXIM Gabriela). CIOCOIU Adrian
Florin: P(34-42); MUNTEANU Ioana-Alexandra: P(34-42); SOFICU Crina-Maria:
P(35-37,39,40); STURZU Tudor-Nicolae: P(33-42).
Scoala
"Alexandru cel Bun". Clasa a II-a (nv. SPNU Doinita). BURLACU

Ionut-Mihai: P(24,25,27,33-35); COJOCARIU Oana-Alexandra: P(24, 25,27,33-37);


COJOCARU Veronica: P(24,25,27,33-36); DAMIAN Daniel: P(24,25,27,33-36); FLO Marta: P(24, 25,27,33,35); IFTEREA Roxana-Maria: P(24,25,27,33-37); FURTUNA

A
NIE Ioana: P(24,25,27,34-36); IVANOV Alla: P(24,25,27,33-36); MARINCU
T
A
Tofana-Maria: P(24,25,27,33-37); MOCANU
Mihai: P(24,25,27,33-36); MIHAIL

Ciprian: P(24,25,27,33-37); PATRA


SC Ilinca: P(24,25,27,33,36,37); RUSU Alexandru: P(24,25,27,33-37); SRBU Silviu Alexandru: P(24,25,27,33,35,37); SITARU
Andreea: P(24,25,27,33-37); URSU Gina-Ioana: P(24,25,27,33-36).
Scoala
"B.P.Hasdeu". Clasa a IV-a (nv. S
TEFAN Liviu). PINTILIE Mina
Liviu: P(24-31); PINTILIE Nicoleta: P(24-30); S
TERBULEAC Daniel: P(24,2631). Clasa a IV-a (nv. TRZIORU Iuliana). CHIHAIA Mihai-Sebastian: P(34 Bogdan: P(34-43); SILION Catalina: P(34-42); SPNU Dragos-Andrei:
A
43); RAIT
P(33-43).
Scoala
"N.Tonitza". Clasa a III-a (nv. MARCU Monica). BUTNARU Valentin:

Alin: P(35-37,39,40). Clasa a IV-a (nv. ZARNESCU

A
P(35-37,39,40); ONUT
Loredana: P(34,38-42); BONCU Andrei: P(34,38-41).
Elena). ANDRUSCA
Beatrice). TUDORACHE
Scoala
"T.Maiorescu". Clasa a III-a (nv. CHIRILA

Alexandru-Gabriel: P(34-41).
Dumitru). TIBA Marius:
Scoala
"O.Cazimir". Clasa a IV-a (nv. PRIALA

P(38-43).
PLOIE
STI
Colegiul National "I.L.Cargiale". Clasa a VI-a. JELEA Anca: P(23,31),
V(21,23,28) (solutiile au fost primite nainte de aparitia nr. 1/2002).

87

IMPORTANT
n scopul unei legaturi rapide cu redactia revistei, pot fi utilizate urmatoarele adrese e-mail: tbi@math.tuiasi.ro, popagabriel@go.com .
Pe aceast
a cale colaboratorii pot purta cu redactia un dialog privitor la
materialele trimise acesteia, procurarea numerelor revistei etc.
La problemele de tip L se primesc solutii de la orice iubitor de matematici
elementare (indiferent de preocupare profesionala sau vrsta ). Fiecare
dintre solutiile acestor probleme - ce sunt publicate n revist
a dup
a un
an - va fi urmat
a de numele tuturor celor care au rezolvat-o.
Adres
am cu insistenta
amintea ca materialele trimise re rug
vistei s
a nu fie (s
a nu fi fost) trimise si altor publicatii.

Redactia revistei "Recreatii matematice" acord


a cte o diplom
a si un
premiu n c
arti urm
atorilor elevi:
ANDRIESCU Alina (Lic. "G. Ibraileanu", cl. a VIII-a): 2/2001 (5pb), 2/2002
(5pb), 1/2003 (5pb);
STEANU S
BRANI
tefana (Lic. "G.Ibraileanu", cl. a VIII-a): 2/2001 (5pb), 2/2002
(11pb), 1/2003 (5pb);
BUDEANU S
tefana (Lic. "G. Ibr
aileanu", cl. a VI-a): 1/2002 (6pb), 2/2002 (5pb),
1/2003 (6pb);
CHIHAIA Mihai - Sebastian (Sc. "B. P. Hasdeu", cl. a IV-a): 1/2002 (9pb),
2/2002 (9pb), 1/2003 (10pb);
JUVERDEANU George (Lic. "G. Ibraileanu", cl. a VIII-a): 1/2002 (6pb), 2/2002
(5pb), 1/2003 (5pb);
Bogdan (Sc. "B. P. Hasdeu", cl. a IV-a): 1/2002 (10pb), 2/2002 (9pb),
A
RAIT
1/2003 (10pb);
SILION C
at
alin (Sc. "B. P. Hasdeu", cl. a IV-a): 1/2002 (10pb), 2/2002 (9pb),
1/2003 (9pb);
SPNU Dragos - Andrei (Sc. "B. P. Hasdeu", cl. a IV-a): 1/2002 (10pb), 2/2002
(9pb), 1/2003 (11pb);
TUDORACHE Alexandru - Gabriel (Sc. "T. Maiorescu", cl. a IV-a): 1/2002
(12pb), 2/2002 (7pb), 1/2003 (8pb);
TUDOSE S
tefan (Lic. "M. Eminescu", cl. a VIII-a): 1/2001 (5pb), 1/2002 (5pb),
1/2003 (5pb);

TUTESCU
Anca S
tefania (Colegiul National "Fratii Buzesti", Craiova, cl. a VI-a):
1/2002 (6pb), 2/2002 (8pb), 1/2003 (8pb);
UNGUREANU Bogdan (Lic. "G. Ibraileanu", cl. a VI-a): 1/2002 (7pb), 2/2002
(7pb), 1/2003 (6pb), autor al problemei V.36.
C
artile au fost oferite de revista "Recreatii matematice" si

88

Al V-lea Congres international


al matematicienilor romni
Pitesti, 22 - 28 iunie, 2003
Incepnd cu anul 1929, s-au organizat, pna n prezent, cinci congrese international ale matematicienilor romni.
Primul Congres a avut loc n anul 1929 la Cluj avnd ca promotor pe Petre
Sergescu, dar avnd concursul celor mai reputati matematicieni romni din acea
perioada.
Al II-lea Congres a avut loc la Turnu Severin n 1932, bucurndu-se ca si
primul de participarea unor mari matematicieni ai timpului, Paul Montel, Arnaud
Denjoy, Waclaw Sierpinski si altii.
Al III-lea Congres s-a desfasurat la Bucuresti n 1945, cu o participare modesta
a matematicienilor straini, date fiind conditiile dificile de la sfrsitul celui de-al doilea
r
azboi mondial.
Cel de-al IV-lea Congres a fost organizat tot la Bucuresti, cu o preg
atire speciala, n anul 1956. Statul romn a facut un efort special si prestigiul de care se
bucurau matematicienii romani (toti formati n marile scoli din occident) au permis
invitarea unui num
ar nsemnat de matematicieni str
aini, de faim
a international
a.
Din Franta au participat Jacques Hadamard si Arnaud Denjoy, din Japonia Masuo Hukuhara, din Germania - W. Blaschke, din Statele Unite - Einar Hille
si S. Eilenberg, din Uniunea Sovietica - I. Vekua, din Polonia - K. Kuratowski si
T. Wazewski. A fost un prilej de rentlnire a matematicienilor romni din generatia
lui Grigore Moisil, Gheorghe Vr
anceanu, Miron Nicolescu, Simion Stoilow,
Nicolae Teodorescu, Tiberiu Popoviciu cu fostii lor mentori sau colegi.
Dupa o ntrerupere de 47 de ani, s-a organizat cel de-al V-lea Congres la Universitatea din Pitesti, al c
arei Rector dr. Gheorghe Barbu este el nsusi matematician
(format la Iasi si Bucuresti). Domnia sa si-a asumat sarcinile dificile ale organiz
arii
congresului, eveniment organizat sub egida Academiei Romne, a Universitatii Bucuresti si a Institutului de Matematica "S. Stoilow" al Academiei Romne. Organizarea congresului a fost reusit
a, datorit
a n primul rnd comitetului local de organizare, autorit
atilor locale si sprijinului acordat de la Bucuresti. Ca o nou
a caracteristica a acestui congres, subliniem prezenta unui numar nsemnat de matematicieni
romni care-si desfasoara acum activitatea n tari straine (a se vedea Libertas Mathematica, vol. XXIII, n care se afl
a numele si adresele a peste 300 de matematicieni
romni ce detin catedre n universit
ati din str
ain
atate, pe toate continentele Terrei).
"Ziarul de Azi" din Pitesti, n timpul desfasurarii congresului, a publicat numeroase
relatari si informatii privind participarea unor reputati matematicieni straini, dar si
a multor matematicieni romni care au activat sau activeaz
a n alte tari. Printre cei
din ultima categorie vom aminti academicienii Nicolae Cristescu si Nicolae Dinculeanu, Sergiu Klainerman (Princeton), Dan Burghelea (Columbus - Ohio),
Daniel Tataru (Berkeley, CA), Henri Moscovici (Columbus - Ohio), M. Epstein
(Tel Aviv), Radu Theodorescu (Laval, Canada).
Spre deosebire de multe alte congrese sau conferinte cu participare international
a,
congresele internationale ale matematicienilor romni au fost ntotdeauna caracteri1

zate printr-o larg


a reprezentare a tuturor domeniilor de baz
a din cercetarea matematica. Astfel, cele peste 400 de comunicari anuntate pentru Congresul al V-lea,
au fost distribuite n 15 sectii, ncepnd cu Logica, Algebra si Teoria numerelor,
mergnd pn
a la Istoria si Filozofia matematicii si Pedagogia matematicii. Au fost
reprezentate Geometria, Analiza clasica si moderna, Ecuatiile diferentiale, Teoria
controlului optimal, Teoria probabilitatilor si Statistica matematica, Cercetarea operationala, Mecanica si Astronomia, Fizica matematica. Lucrarile s-au desfasurat att
n plenul congresului (ncepnd cu sedinta de deschidere la care Ambasadorul Frantei
la Bucuresti, E. S. Philippe tienne, el nsusi matematician si admirabil vorbitor,
a captivat audienta), precum si n numeroase sectii pe specialitati.
Pe lnga matematicienii straini care au participat la Congres, venind din Statele
Unite, Canada, Franta, Germania, Rusia, Ungaria, Italia si alte tari, trebuie s
a
remarc
am prezenta destul de nsemnat
a a matematicienilor din Republica Moldova.
Este destul de dificil sa prezentam o vedere de ansamblu asupra desfasurarii Congresului al V-lea al matematicienilor romni, data fiind varietatea domeniilor abordate de c
atre participanti. Vom sublinia totusi faptul c
a programul si desfasurarea
lucrarilor congresului s-au ncadrat n standardele internationale. O critic
a ce s-a
adus organizatorilor a fost aceea ca data congresului a coincis cu multiple activitati
academice, cum ar fi: examenele studentesti, examenul de licenta si altele. n felul
acesta, multi doritori din tar
a de a participa au fost absenti.
Vom ncheia subliniind faptul c
a acest al V-lea Congres a ilustrat vitalitatea
matematicii romnesti, ncadrarea ei reusita n comunitatea matematica internationala. S
a sper
am c
a urm
atorul congres va avea loc dup
a o perioad
a nu att de
ndelungat
a ca pn
a acum.
Constantin CORDUNEANU
University of Texas at Arlington

Observatorul din Iasi 90 de ani de la nfiintare


nfiintarea observatoarelor astronomice din Bucuresti (n 1908) si apoi din Iasi
(n 1913) face parte dintr-un proces mai amplu de modernizare a nv
atamntului
universitar si a cercet
arii stiintifice, proces impulsionat de Legea Haret din 1898 si
care se va maturiza n conditiile social-politice si culturale din Romnia ntregit
a.
nca din momentul nfiintarii n 1860 a Universit
atii din Iasi, n programa
"sectiei stiintelor pozitive din facultatea de filozofie" sunt prev
azute si cursuri de
mecanic
a si astronomie, dar catedrele aferente vor c
ap
ata fiinta mai trziu. Prin
legea nvatamntului din 1864, care se pune n aplicare ncepnd cu data de 25
febr. 1865, se creeaza Facultatea de stiinte, desprinsa din Facultatea de filozofie
si avnd trei sectii distincte: fizic
a, matematic
a si stiinte naturale; una din cele 12
catedre ale noii facult
ati este cea de geodezie teoretica si astronomie. La 15 febr. 1865
este numit profesor titular al acestei catedre Neculai Culianu, care o va ocupa
pn
a n 1906, anul pension
arii sale. N. Culianu trece licenta n stiinte matematice
la Sorbona, este atras de astronomie si de Observatorul din Paris, cunoaste aici
si r
amne prieten pentru toat
a viata cu astronomul francez Camille Flammarion.
N. Culianu este autor al unui Curs de cosmografie pentru liceu (doua editii, 1893 si
1902). Universitatea din Iasi a primit, chiar din momentul nfiintarii, de la Societatea
de Medici si Naturalisti din Iasi un bun instrument de observatii astronomice, care
apartinuse poetului moldovean Costache Conachi si pe care mostenitorii l donaser
a
acesteia.
Dup
a nfiintarea Catedrei de astronomie (n 1864) au fost achizitionate si alte
instrumente; ele au fost depozitate ntr-o c
am
aruta a vechiului local al universit
atii
nct nici nu puteau fi ar
atate studentilor. Cu toate insistentele nu s-a reusit timp
ndelungat obtinerea fondurilor pentru construirea unui observator astronomic.
Constantin Popovici este licentiat al Facult
atii de stiinte din Iasi (1900). Pleac
a
la Paris cu o burs
a "Adamachi" unde obtine din nou licenta n matematici (1905) si
apoi doctoratul la Sorbona (1908) n domeniul ecuatiilor diferentiale. n 1909 este
numit la Catedra de geometrie analitica a universitatii iesene, iar n 1910 este trimis
n Franta pentru specializare n astronomie si documentare n privinta construirii
viitorului observator din Iasi. Se rentoarce si este numit n 1911 la Catedra de
astronomie, geodezie si mecanica cereasca, Universitatea din Iasi.
C. Popovici este fondatorul Observatorului astronomic din Iasi, amplasat pe
dealul Copou; piatra de temelie a cl
adirii a fost pus
a la 12 sept. 1912, iar receptia
s-a f
acut la mijlocul lui decembrie 1913.
C. Popovici este primul director al Observatorului (n perioada 1913-1937). Primele instrumente intrate n dotarea acestuia au fost cele provenite de la cabinetul de
astronomie nfiintat de N. Culianu. Prin str
adaniile lui C. Popovici si ale elevului
si colaboratorului s
au, Vintil
a S
iadbei, au fost achizitionate noi instrumente: o
luneta meridiana, un ecuatorial Ressel, doua cronometre (pentru timpul mediu si cel
sideral), un fotometru Gra si altele necesare procesului didactic.
n anul 1938 Catedra de astronomie este transformat
a ntr-o conferinta iar
C. Popovici se transfer
a la Bucuresti. n perioada 1938-1944, Vintil
a S
iadbei a
suplinit conferinta de astronomie.
3

Ca urmare a evacu
arii Observatorului, prilejuit
a de cel de-al doilea r
azboi mondial,
o buna parte a aparaturii din dotarea acestuia s-a deteriorat sau a fost sustrasa.
n 1948 Victor Nadolschi ocupa prin concurs conferinta de astronomie si devine
directorul Observatorului din Iasi, functie detinuta pna n anul 1966. V. Nadolschi
este un eminent continuator al lui C. Popovici si al lui V. S
iadbei. Acesta reorganizeaz
a si relanseaz
a activitatea si pune bazele astronomiei fotografice la Iasi.
V. Nadolschi achizitioneaza un astrograf Zeiss (1956), un fotometru fotoelectric (1959),
un ecuatorial Zeiss cotit (1960), un aparat pentru masurat clisee (1963), un teodolit
zenital Meopta (1963) etc.
ncepnd cu anul 1966 activitatea didactic
a si de cercetare este coordonat
a de
Iulian Breahn
a, absolvent al Universitatii din Bucuresti, sectia de astronomie.
Din 1966 functioneaza n cadrul Observatorului din Iasi un atelier de mecanica
fina si un laborator electronic necesare ntretinerii si cercet
arii. A fost achizitionat
un orologiu cu cuart care, completat ulterior cu alte anexe, constituie si n prezent
un cronograf digital de precizie.
n anul 1980 a fost achizitionat un planetariu Zeiss destinat nvatamntului astronomiei, care a fost instalat n incinta Universit
atii din Iasi. Studentii au astfel
posibilitatea de a-si nsusi mai usor o multitudine de fenomene privind cinematica si
dinamica sistemului planetar al Soarelui. Planetariul a atras pna n prezent cteva
mii de vizitatori.
Cu prilejul eclipsei totale de Soare din 11 august 1999 s-a achizitionat un astrograf
CCD (dispozitiv cu cuplaj de sarcin
a) de performanta si o camera Astrovid pentru
nregistrari continue de imagini.
n perioada 1951-1999, pe lnga Observator si prin grija personalului acestuia a
functionat o statie seismica. Observatiile efectuate de aceasta au pus n evidenta
dou
a focare seismice: unul la circa 25 km dincolo de Prut si al doilea n zona BrladZorleni.
Activitatea de cercetare desfasurata pe lnga Observatorul din Iasi s-a concretizat
n peste 140 lucr
ari. C. Popovici a generalizat legea Newton-Coulomb prin considerarea unei forte neconservative, rezultat
a dintr-o combinatie a gravitatiei newtoniene
cu presiunea luminii. V. S
iadbei obtine rezultate noi privind traiectoriile meteorilor
si cometelor si face observatii asupra eclipselor de Luna si Soare, stabilind relatii mai
simple pentru calculul acestora. V. Nadolschi s-a preocupat de teoria statisticii
grupurilor de pete solare, continu
a traditia observ
arii eclipselor si are meritul de
a fi pus bazele astronomiei fotografice la Iasi. Abordarea unor teme din domeniul
radioastronomiei s-a dovedit deosebit de dificila, desi s-au depus eforturi sustinute
pentru crearea bazei materiale necesare unei astfel de cercet
ari.
Cu toate c
a de-a lungul timpului au fost de nl
aturat multe dificult
ati si obstacole, la Observatorul din Iasi s-a reusit sa se desfasoare o activitate care l-a facut
cunoscut n tara si n strainatate. Aceste afirmatii sunt dovedite si de acordarea titlului de membru al Uniunii Astronomice Internationale urm
atorilor astronomi ieseni:
Constantin Popovici, Vintila Siadbei,
Victor Nadolschi si Iulian Breahna.

Redactia revistei
4

Marea teorem
a a lui Fermat pentru polinoame
Temistocle BRSAN 1
1. Odata cu caderea Constantinopolului (1453), multi nvatati bizantini s-au
ndreptat spre Europa de Vest aducnd cu ei manuscrise pretioase - manuscrisele
care supravietuiser
a devast
arii Bibliotecii din Alexandria se adunaser
a de-a lungul
timpului n aceasta capitala a lumii.
Prin hazardul mprejurarilor, sase din cele 13 volume ale Aritmeticii lui Diofant
au ajuns n Franta. nv
atatul si amatorul de matematic
a francez Claude Gaspar
Bachet de Mziriac si d
a seama de importanta c
artii lui Diofant si public
a n 1621
o versiune n limba latina a Aritmeticii, care cuprinde peste o suta de probleme si
rezolvarile detaliate ale lui Diofant.
Pentru Pierre Fermat (1601-1665) Aritmetica lui Diofant a fost cartea care
l-a pus n contact cu bogatele cunostinte ale popoarelor antice n directia teoriei
numerelor si sursa de inspiratie pentru noi si subtile probleme pe care singur si le
formula. Fermat obisnuia sa noteze pe marginile cartii lui Diofant comentarii, calcule si schite de demonstratii. Nu s-a preocupat s
a-si publice rezultatele si demonstratiile, dar se amuza comunicndu-si rezultatele altor matematicieni ai timpului si
provocndu-i la rezolvarea acestora.
n Cartea a II-a a Aritmeticii, Fermat gaseste informatii bogate relativ la tripletele
pitagoreice, adic
a trei numere naturale ce verific
a ecuatia lui Pitagora
x2 + y 2 = z 2 .

(1)

S
tia c
a Euclid demonstrase c
a exist
a o infinitate de astfel de triplete. Ce se ntmpl
a,
ns
a, dac
a n loc de (1) se consider
a ecuatia
xn + y n = z n ,

(2)

unde n 3? R
aspunsul lui Fermat, notat ca observatie pe marginea c
artii lui Diofant,
este cu totul surprinz
ator: nu exista nici o solutie a ecuatiei (2) cu numere x, y, z
nenule, daca n = 3, 4, . . . . Urmeaza notat urmatorul comentariu:
Cuius rei demonstrationem mirabilem sane detex hanc marginis exiguitas non
caperet [4]. (Ma aflu n posesia unei demonstratii minunate a acestei afirmatii, dar
marginea paginii este prea strmta pentru a o cuprinde.)
Aceasta extraordinara descoperire, care astazi poarta numele de Marea teorem
a
a lui Fermat, ct si alte rezultate, ar fi putut sa ramna necunoscute lumii matematicienilor si s
a se piard
a, dac
a, dup
a moartea lui Fermat, fiul s
au cel mai mare n-ar
fi examinat nsemn
arile scrise de tat
al s
au pe margini si n-ar fi publicat Aritmetica
lui Diofant continnd si observatiile lui Pierre de Fermat (Toulouse, 1670).
Pe parcursul ctorva secole, cele mai sclipitoare minti de matematicieni au ncercat si si-au adus contributia la rezolvarea acestei enigme (si, totodat
a, provoc
ari)
l
asat
a de Fermat: Euler, Sophie Germain, Dirichlet, Legendre, Lam, Cauchy,
Kummer s. a. Drumul ce duce la demonstrarea Marii teoreme a lui Fermat este
1

Prof. dr., Catedra de matematic


a, Univ. Tehnic
a "Gh. Asach", Iasi

pres
arat cu reusite partiale, ambitii, nfrngeri, deceptii, orgolii, intrigi, tentative de
sinucidere etc. [4].
n anul 1995, dupa opt ani de munca nentrerupta, n completa izolare fata de
colegii s
ai si p
astrnd o discretie total
a asupra cercet
arilor sale, englezul Andrew
Wiles pune cap
at enigmei de peste 350 de ani: Marea teorema a lui Fermat este
demonstrata! Demonstratia data de Wiles este, nsa, accesibila unui numar restrns
de specialisti; n fapt, Wiles pentru a atinge scopul a dovedit justetea Conjecturii
Taniyama - Shimura utiliznd o aparatur
a matematic
a modern
a si sofisticat
a: curbe
eliptice, forme modulare, reprezent
ari Galois s. a. [5].
2. Este cunoscut faptul c
a inelul Z al numerelor ntregi si inelul C [X] al polinoamelor cu coeficienti numere complexe au propriet
ati asem
an
atoare. De aceea
apare ca fireasca problema rezolvarii ecuatiilor (1) si (2) n C [X].
n privinta ecuatiei (1) constat
am usor, ca si n cazul numeric, c
a are o infinitate
de solutii: p, q C [X], lu
am
2

x (X) = [p (X)] [q (X)] , y (X) = 2p (X) q (X) , z (X) = [p (X)] + [q (X)]


si verificam direct ca tripleta (x (X) , y (X) , z (X)) este o solutie a ecuatiei (1) n
C [X].
Similar cu Marea teorem
a a lui Fermat se formuleaz
a
Teorema lui Fermat pentru polinoame ([3], [5]). Daca n este un ntreg,
n 3, atunci ecuatia (2) nu are solutii n C [X] cu polinoame neconstante si relativ
prime.

Surprinz
ator, spre deosebire de Marea teorem
a a lui Fermat, pentru acest rezultat
se cunoaste o demonstratie elementara si simpla, accesibila unui elev de liceu. Rezultatul este cunoscut din sec. al XIX-lea si a fost demonstrat utiliznd cunostinte de
geometrie algebric
a. Demonstratia elementar
a la care ne-am referit se sprijin
a pe
o teorem
a de dat
a recent
a datorat
a matematicienilor W. Stothers (1981) si, independent, R. C. Mason (1983), teorema foarte importanta si n sine. Sunt necesare
cteva (putine!) pregatiri.
Fie p C [X] un polinom neconstant avnd r
ad
acinile a1 , a2 , . . . , ak cu ordinele
de multiplicitate respective m1 , m2 , . . . , mk ; deci p se scrie sub forma
k
Y
(X ai )mi , C .
(3)
p (X) =
i=1

Not
am gradul polinomului p si num
arul r
ad
acinilor sale distincte cu deg p si respectiv
a
n0 (p), adic
deg p = m1 + m2 + + mk , n0 (p) = k.
Mention
am c
a, dac
a p, q C sunt neconstante, avem
deg (pq) = deg p + deg q, n0 (pq) n0 (p) + n0 (q) ,
cu egalitate dac
a si numai dac
a p si q sunt relativ prime.
Derivata formal
a a polinomului p dat de (3) este
p0 (X) = [m1 (X a1 )
+ mk (X a1 )

m1 1

m1

(X a2 )

m2

(X ak1 )
6

(X ak )

mk1

(X ak )

mk

+ +

mk 1

si, ca urmare, cel mai mare divizor comun al polinoamelor p si p0 are forma
(p, p0 ) = (X a1 )m1 1 (X a2 )m2 1 (X ak )mk 1 .
Atunci
deg (p, p0 ) = (m1 1) + (m2 1) + + (mk 1) = deg p n0 (p) ,
de unde obtinem relatia
deg p = deg (p, p0 ) + n0 (p) .

(4)

Teorema Mason - Stothers. Fie p, q, r C [X] neconstante si relativ prime.


Daca are loc egalitatea p + q = r, atunci
max {deg p, deg q, deg r} n0 (pqr) 1.

(5)

Demonstratie (dat
a de Noah Snyder [3], p.30). Vom ncepe cu dou
a observatii
utile. Mai nti, n prezenta conditiei p + q = r, polinoamele p, q, r sunt relativ prime
daca si numai daca sunt prime doua cte doua. Apoi, ntruct enuntul teoremei este
simetric n p, q, r (c
aci putem scrie egalitatea si sub forma p + q + r = 0), nu
restrngem generalitatea dac
a vom presupune c
a polinomul r are gradul cel mai
ridicat. Ca urmare, inegalitatea de demonstrat se scrie
deg r n0 (pqr) 1.

(50 )

Avem
p0 q pq 0 = p0 (p + q) p (p0 + q 0 ) = p0 r pr0 .
Constat
am c
a (p, p0 ) si (q, q 0 ) divid membrul stng, iar (r, r0 ) divide membrul drept,
deci si pe cel stng. Cum p, q, r sunt prime doua cte doua, urmeaza ca produsul
(p, p0 ) (q, q 0 ) (r, r0 ) divide p0 q pq 0 . n consecinta,
deg (p, p0 ) + deg (q, q 0 ) + deg (r, r0 ) deg (p0 q pq 0 ) deg p + deg q 1

sau, datorita relatiei (4) si analoagelor ei,


deg p n0 (p) + deg q n0 (q) + deg r n0 (r) deg p + deg q 1,
deci
deg r n0 (p) + n0 (q) + n0 (r) 1.
Cum p, q, r sunt prime doua cte doua, obtinem n final
deg r n0 (pqr) 1,

care este tocmai relatia (50 ) de demonstrat.

Demonstratia Teoremei lui Fermat pentru polinoame. Presupunem ca


ecuatia (2) pentru n 3 ar avea o solutie (x (X) , y (X) , z (X)) cu polinoame neconn
stante relativ prime. Aplic
am teorema Mason - Stothers polinoamelor p(X) = [x(X)] ,
n
n
q (X) = [y (X)] si r (X) = [z (X)] . Obtinem
deg [x (X)]n n0 ([x (X)]n [y (X)]n [z (X)]n ) 1
sau
n deg x (X) n0 (x (X) y (X) z (X)) 1.
7


Tinnd
seama c
a x (X), y (X) si z (X) sunt prime dou
a cte dou
a si de faptul c
a
n0 (p) deg p, p C [X], vom avea
n deg x (X) n0 (x (X)) + n0 (y (X)) + n0 (z (X)) 1
deg x (X) + deg y (X) + deg z (X) 1.

Obtinem astfel inegalitatea

n deg x (X) deg x (X) + deg y (X) + deg z (X) 1,

precum si inegalitatile analoage scrise pentru y (X) si z (X), care adunate dau
n (deg x (X) + deg y (X) + deg z (X)) 3 (deg x (X) + deg y (X) + deg z (X)) 3,

adic
a

(n 3) (deg x (X) + deg y (X) + deg z (X)) 3.


Evident, dac
a n 3, aceast
a relatie ne conduce la o absurditate, ceea ce ncheie
demonstratia.
3. Analogia care exist
a ntre inelele Z si C [X] pune n mod firesc problema
"translarii" teoremei Mason - Stothers de la polinoame la numerele ntregi astfel
nct Marea teorem
a a lui Fermat s
a poat
a fi demonstrat
a elementar.
D. Masser si J. Oesterle (1986) au ajuns la asa - numita conjectura abc ca
urmare a unor consideratii de geometrie algebric
a si teoria functiilor modulare (si nu
n legatura cu teorema Mason - Stothers).
k
Q
i
Daca m N are descompunerea n factori primi m =
pm
i , atunci vom numi
i=1
k
Q
pi .
radicalul lui m numarul N0 (m) =
i=1

Conjectura abc ([2], [3]). Dat > 0, exista o constanta C () astfel nct
pentru orice ntregi a, b, c nenuli si relativ primi cu a + b = c avem inegalitatea
1+

max {|a| , |b| , |c|} C () (N0 (abc))

Aceast
a conjectur
a spune c
a, dac
a n descompunerea numerelor a, b, c exist
a
factori primi cu exponenti mari, acesti factori sunt compensati prin factori primi mai
multi, dar cu exponentul 1.
S
a enuntam acum asa - numita
Teorema lui Fermat asimptotic
a. Exista un ntreg pozitiv n1 cu proprietatea
ca, daca n n1 , atunci ecuatia (2) nu are solutii cu x, y, z ntregi si xyz 6= 0.
Cu aceleasi argumente ca n cazul polinoamelor se poate dovedi urmatoarea
Teorem
a ([2], [3]). Conjectura abc implica Teorema lui Fermat asimptotica.
Demonstratie. Fie date x, y, z pozitive si relativ prime astfel nct tripleta
(x, y, z) sa fie solutie a ecuatiei (2), adica xn + y n = z n .
Not
am a = xn , b = y n si c = z n si observ
am c
a
N0 (abc) = N0 (xn y n z n ) = N0 (xyz) xyz.

Utiliznd conjectura abc obtinem


xn C () (xyz)1+ ,

y n C () (xyz)1+ ,
8

z n C () (xyz)1+ .

Prin nmultire, rezult


a c
a
n

de unde
si cum xyz > 2, obtinem

3+3

(xyz) [C ()] (xyz)

(n 3 3) log (xyz) 3 log C ()

3 log C ()
+ 3 + 3.
log 2
Not
am

3 log C ()
+ 3 + 3 .
(6)
n1 =
log 2
Urmeaz
a c
a ecuatia (2) nu are solutii ce verific
a conditiile specificate dac
a n n1 ,
ceea ce trebuia demonstrat.
n<

Observatie. Aceast
a cale nu ofer
a o demonstratie a Marii teoreme a lui Fermat.
ntr-adevar, numarul n1 definit de (6) depinde de C () (putem considera = 1 si
C (1) pentru a fixa ideile). Determinarea efectiv
a a constantei C () nu este cunoscut
a. Dac
a, de exemplu, C (1) s-ar putea efectiv determina, atunci demonstratia
Marii Teoreme a lui Fermat s-ar reduce la un num
ar finit de cazuri, care ar putea fi
abordate prin calcul direct.
4. Interesul pentru Marea teorema a lui Fermat nu s-a stins nici dupa demonstrarea ei. Au ramas ntrebari fara raspuns, sunt formulate altele noi. Daca Fermat
nu a dat dect o demonstratie eronat
a, care ar putea fi natura greselii f
acute? Dac
a
aceast
a demonstratie ar fi corect
a, care este acel argument ingenios produs de geniul
lui Fermat ce a scapat attor matematicieni ilustri? Este posibila o demonstratie
elementara, accesibila si unor persoane cu cunostinte obisnuite de matematica?
n 1966, Andrew Beal instituie un premiu pentru demonstrarea sau infirmarea
asa - numitei Conjecturi Beal, care este o generalizare a problemei lui Fermat:
Ecuatia xp + y q = z r , p, q, r numere ntregi mai mari ca 2, nu are nici o solutie
cu x, y, z ntregi pozitivi si relativ primi ([6], [1]).
Bibliografie
1.
2.
3.
4.
5.
6.

A. Corduneanu - Despre Marea teorema a lui Fermat, Recreatii Matematice,


1 (1999), nr.1, 37-39.
S. Lang - Old and new conjectured diophantine inequalities, Bull. AMS, 23 (1990),
37-75.
S. Lang - Math Talks for Undergraduates, Springer, 1999.
S. Singh - Marea teorema a lui Fermat, Humanitas, Bucuresti, 1998.
A. Wiles - Modular elliptic curves and Fermats Last Theorem, Annals of Math.,
142 (1995), 443-551.
***
- Beals Conjecture, The New Zealand Math. Mag., 35 (1998), no.2, 38.

De la o problem
a cu matrice la transform
ari
elementare
Marian TETIVA1
1. Introducere. Problema la care ne referim n titlu este urmatoarea:
Sa se arate ca nu exista matrice patratice X, Y Mn (C) astfel nct XYY X = In ,
In fiind matricea unitate de ordinul n.
Este o problem
a cunoscut
a, care poate fi ntlnit
a n mai multe manuale sau
culegeri, care s-a dat la concursuri etc. si nu este tocmai simpl
a: un elev mediu este
ntotdeauna descurajat de enunturi de tipul "sa se arate ca exista /nu exista. . . ".
Mai mult, n aceasta situatie nu prea avem alta cale de abordare n afara celei care
utilizeaz
a notiunea de urm
a a unei matrice si propriet
atile sale. Istoria problemei
este cam asa: prin anii 70 ai secolului trecut ea era propus
a la olimpiad
a, prin anii
80 a patruns n manuale pentru ca n anii 90 sa ajunga a fi parte din diverse teste
de bacalaureat sau admitere la facultate; aceasta spune ceva despre felul n care au
evoluat programele nv
atamntului matematic elementar n Romnia. Noi credem
c
a elevul mediu din ziua de azi se afl
a n acelasi impas ca si cel de acum dou
azeci sau
treizeci de ani (sau poate chiar mai rau) atunci cnd este confruntat cu asemenea
probleme. De aceea aceasta nota i se adreseaza, dar numai daca este cu adevarat
interesat de matematic
a.
Amintim c
a urma matricei A = (aij )1i, jn Mn (C) este, prin definitie,
numarul Tr (A) = a11 + a22 + + ann (suma elementelor situate pe "diagonala
principal
a" a matricei). Sunt cunoscute urm
atoarele propriet
ati ale urmei:
1 Tr (A + B) = Tr (A) + Tr (B), A, B Mn (C),
2 Tr (A) = Tr (A), C, A Mn (C),
3 Tr (AB) = Tr (BA), A, B Mn (C).
Primele dou
a egalit
ati se mai pot scrie condensat n forma Tr (A + B) =
= Tr (A) + Tr (B), oricare ar fi , C si A, B Mn (C) si exprim
a liniaritatea urmei: Tr : Mn (C) C este aplicatie liniara (sau morfism de C - spatii
vectoriale). De aici deducem Tr (XY Y X) = Tr (XY ) Tr (Y X) = 0 pentru orice
X, Y Mn (C) si aceasta explic
a de ce egalitatea din enunt nu poate avea loc pentru
nici o pereche de matrice X, Y : matricea XY Y X are urma nul
a, deci nu poate fi
egala cu In , a carei urma este n.
Remarc
am c
a matricea In din enunt poate fi nlocuit
a cu orice matrice de ordinul
n avnd urma nenul
a, enuntul si rezolvarea r
amnnd valabile; problema poate fi
usor reformulat
a astfel:
Daca pentru o matrice A Mn (C) exista X, Y Mn (C) astfel nct A =
XY Y X, atunci Tr (A) = 0.
Atunci se naste n mod natural ntrebarea dac
a reciproca acestei afirmatii este
adev
arat
a, adic
a se pune problema valabilit
atii urm
atorului enunt:

Fie A o matrice patratica de ordin n cu elemente numere complexe. Daca urma


matricei A este nula, atunci exista X, Y Mn (C) astfel nct A = XY Y X.
1

Profesor, Colegiul National "Gh. Rosca Codreanu", Brlad

10

n cele ce urmeaz
a ne propunem s
a rezolv
am aceast
a problem
a; mai precis, s
a
aratam ca raspunsul la ntrebare este afirmativ.
Ideea rezolvarii este sa cautam niste matrice Y, Z Mn (C) astfel nct A sa poata
fi scris
a n forma A = Z Y ZY 1 (Y fiind inversabil
a, desigur);
atunci
problema

va

fi rezolvat
a: e suficient s
a alegem X = ZY 1 si avem A = ZY 1 Y Y ZY 1 =
= XY Y X.
Aici cititorul poate avea o nemultumire: de unde si pna unde aceste matrice Y, Z
n locul lui Xsi Y
a remarc
am c
a din proprietatea 3 rezult
a c
a avem si
din enun
t? S
4 Tr(A) =Tr CAC 1 , C Mn (C), C inversabil
a (se dovedeste imediat, c
aci
1
1
A = AC
sunt asemenea, iar proprietetea
C). Matricele de forma A si CAC
4 spune ca acestea au aceeasi urma.
Desigur, problemele abia ncep. Sunt necesare cteva pregatiri.
2. Matrice asemenea si transform
ari elementare. Fie K un corp comutativ;
cititorul mai putin familiarizat cu aceast
a notiune abstract
a poate considera K o
notatie pentru unul dintre corpurile numerice uzuale Q, R sau C.
Dou
a matrice X, Y Mn (K) se numesc matrice asemenea ( sau similare) dac
a
exista U Mn (K) cu det U 6= 0 astfel nct Y = U XU 1 (vom nota X Y ).
Cititorul poate verifica usor faptul c
a relatia de asem
anare (similaritate) este o relatie
de echivalenta pe multimea Mn (K).
Transformarile elementare care se fac asupra unei matrice sunt, n principiu,
cele mai simple modificari care nu i afecteaza rangul, adica interschimbarea a doua
linii (sau coloane), adunarea unei linii (coloane) nmultite cu un numar (n general:
element al corpului K) la alt
a linie (respectiv coloan
a), sau chiar nmultirea unei
linii (coloane) cu un num
ar nenul. Una din cele mai simple aplicatii ale lor este
calculul rangului unei matrice; de asemenea, se pot folosi aceste transformari pentru
rezolvarea sistemelor de ecuatii liniare.
S
a ncepem prezentarea transform
arilor elementare cu asa numitele matrice elementare; leg
atura va ap
area curnd. Vom nota cu Eij matricea p
atratic
a de ordin
n peste corpul K ale carei elemente sunt toate nule, cu exceptia elementului de pe
linia i si coloana j care este egal cu 1. Se verifica usor ca matricele Eij , 1 i n,
1 j n formeaz
a o baz
a a spatiului vectorial Mn (K) peste K, precum si relatiile
Eij Ekl = 0n , j 6= k si Eij Ejl = Eil .
Se numesc matrice elementare urmatoarele tipuri de matrice patratice de ordinul
n, cu elemente din K:
1) Matricele Tij (a) = In +aEij ; aici a K si i, j {1, 2, . . . n} sunt indici diferiti.
Matricea Tij (a) se obtine din matricea unitate f
acndu-i o singur
a modificare: elementul de pe linia i si coloana j devine a. Se constata imediat ca (vezi proprietatile
matricelor Eij )
Tij (0) = In , Tij (a) Tij (b) = Tij (a + b) ,
Tij (a) GLn (K) , Tij (a)

= Tij (a) , a, b K,

unde GLn (K) = {U Mn (K) | det U 6= 0} - grupul general liniar de ordin n peste
corpul K. Deci {Tij (a) | a K} formeaz
a, pentru i 6= j fixate, un grup izomorf cu
grupul (K, +). Sa vedem ce efect are nmultirea unei matrice oarecare cu o matrice
11

Tij (a). Fie A = (akl )1k,ln o matrice din Mn (K), care mai poate fi scris
a si
n
X
akl Ekl . Atunci
A=
k,l=1

Tij (a) A = (In + aEij )


n
X

X
n

akl Ekl +

k,l=1

akl Ekl

k,l=1
n
X

n
X

akl Ekl +

k,l=1

n
X

aakl Eij Ekl =

k,l=1

aajl Eil .

l=1

amn aceleasi ca
Ce nseamn
a asta? nseamn
a c
a elementele matricei Tij (a) A r
ale matricei A, cu exceptia celor de pe linia i: aici, n locul elementului ail apare
acum ail + aajl , adic
a matricea Tij (a) A se obtine din A prin adunarea la linia i
a liniei j nmultite cu a, cu alte cuvinte nmultirea la stnga cu o matrice Tij (a)
realizeaz
a o transformare elementar
a a matricei A. De asemenea, se poate verifica n
acelasi fel c
a matricea ATij (a) se obtine din A prin adunare la coloana j a coloanei
i nmultite cu a.
2) Matricele Qij = Tij (1) Tji (1) Tij (1) (i, j {1, 2, . . . , n} , i 6= j) intra si ele
n categoria matricelor elementare. Avem
Qij = Tij (1) Tji (1) Tij (1) = (In Eij ) (In + Eji ) (In Eij ) =
= (In + Eji Eij Eii ) (In Eij ) = In + Eji Eij Eii Eij Ejj + Eij =
= In Eii Ejj Eij + Eji ,

deci Qij este matricea care se obtine din matricea unitate prin schimbarea a patru
elemente: elementele de pe diagonala principala, de pe linia i, coloana i si de pe linia
j, coloana j se nlocuiesc cu zerouri; n locul elementului de pe linia i si coloana j
avem 1, iar n locul celui de pe linia j si coloana i se g
aseste 1. La fel ca mai sus,
s
a calcul
am
X

n
Qij A = (In Eii Ejj Eij + Eji )
akl Ekl =
=

n
X

k,l=1

akl Ekl
=

n
X

k,l=1

n
X

k,l=1

akl Eii Ekl

akl Ekl

n
X
l=1

n
X

k,l=1

akl Ejj Ekl

ail Eil

n
X
l=1

ajl Ejl

k,l=1
n
X

akl Eij Ekl +

k,l=1
n
X

ajl Eil +

l=1

n
X

akl Eji Ekl =

k,l=1
n
X

ail Eil ;

l=1

asadar, matricea Qij A se obtine din A prin nlocuirea liniei i, respectiv j, cu linia j
nmultita cu 1, respectiv cu linia i. Asemanator, se poate observa ca schimbarile pe
care le produc asupra lui A nmultirea cu matricea Qij la dreapta sunt urm
atoarele:
coloana i se nlocuieste cu coloana j, iar coloana j se nlocuieste cu coloana i nmultit
a
cu 1. Sa mai spunem ca, fiind produs de matrice inversabile, Qij este, de asemenea,
matrice inversabila; avem
1

Q1
ij = Tij (1)
si, deci,

Q1
ij

Tji (1)

Tij (1)

= Qij .
12

= Tij (1) Tji (1) Tij (1)

3) Un alt tip de matrice elementare sunt matricele Di (d) = In + (d 1) Eii ,


1 i n, d K fiind nenul; o astfel de matrice se obtine din matricea unitate
modificndu-i un singur element: n locul lui 1 de pe linia i si coloana i punem d. Nu
e greu de v
azut c
a nmultirea unei matrice A oarecare cu Di (d) la stnga (respectiv
la dreapta) i modific
a doar linia (respectiv coloana) i, anume o nlocuieste pe aceasta

cu linia (respectiv coloana) i nmul


tita cu d. De asemenea, Di (d) este inversabila si
1
1
are inversa Di (d) = Di d
.
4) Vom folosi matricele Pij pe care le definim prin
Pij = Di (1) Qij = Qij Dj (1) = In Eii Ejj + Eij + Eji .

Verificati aceste egalit


ati! Observati forma matricei Pij si efectul s
au la nmultire:
matricea APij (respectiv Pij A) se obtine din A prin interschimbarea liniilor (respectiv
coloanelor) i si j. S
i nu n ultimul rnd, aratati ca Pij este inversabila si Pij1 = Pij .
3. Rezolvarea problemei. Sa demonstram asadar urmatoarea
Propozitie. Fie K un corp comutativ infinit si A Mn (K) o matrice a carei
urma este zero. Atunci exista matricele X, Y Mn (K) astfel nct A = XY Y X.
Demonstratie. Pentru nceput vom presupune ca nu exista nici o submultime
a multimii {a11 , a22 , . . . , ann } a elementelor de pe diagonala principal
a a matricei A
pentru care suma elementelor s
a fie nul
a, desigur, cu exceptia ntregii multimi (vom
vedea imediat la ce ne foloseste aceast
a presupunere, iar la sfrsit ne vom da seama c
a
nu este prea restrictiva). Deoarece, conform ipotezei, avem a11 + a22 + + ann = 0,
se pot determina elementele a1 , a2 , . . . , an K astfel nct
a11 = a1 a2 , a22 = a2 a3 , . . . , an1,n1 = an1 an , ann = an a1 .

E clar ca, datorita ipotezei suplimentare pe care am facut-o, oricare doua dintre
elementele a1 , a2 , . . . , an sunt distincte; vom folosi acest lucru mai departe.
Putem scrie pe A n forma A = B C, unde

a1
a2 a12 . . . a1,n1 a1n
0
...
0
0
a21
0
a2
...
0
0
a3 . . . a2,n1 a2n

...
...
...
. . . , C = . . . . . . . . .
...
...
B = ...
.
an1,1 an1,2 . . . an1 0
0
0
...
an an1,n
an2 . . . an,n1 an
0
0
...
0
a1
an1

Asa cum am ar
atat la nceput, pentru rezolvarea problemei ar fi suficient s
a
ar
at
am c
a matricele B si C sunt asemenea. n acest scop, vom ar
ata c
a B B0,
C C 0 , unde

a1
a2
0 ... 0
0 ... 0
0
0
a2 . . . 0
a3 . . . 0
0

B0 =
. . . . . . . . . . . . , C = . . . . . . . . . . . .
0
0 . . . an
0
0 . . . a1

si B 0 C 0 . Folosind faptul c
a asem
anarea matricelor este o relatie de echivalenta,
obtinem c
a B C.
Sa le luam pe rnd. Ne amintim ca nmultirea unei matrice cu matricea Pij la
stnga (respectiv la dreapta) schimba ntre ele liniile (respectiv coloanele) i si j ale
acelei matrice. De aceea, pentru M Mn (K), matricea Pij M Pij1 = Pij M Pij
13

are schimbate ntre ele elementele de pe diagonala principal


a situate pe liniile (si
coloanele) i si j; de asemenea, mai sunt afectate si celelalte elemente de pe liniile si
coloanele i si j. Aceasta nu are nsa importanta n cazul unor matrice precum B 0
sau C 0 , la care toate elementele din afara diagonalei principale sunt zerouri; astfel
Pij B 0 Pij1 = Pij B 0 Pij este o matrice care difer
a de B 0 doar prin aceea c
a si-au
schimbat ntre ele locurile doua elemente de pe diagonala principala, anume ai si
aj . Cum orice permutare e produs de transpozitii, e clar ca dupa un numar finit
de asemenea transform
ari o putem aduce pe B 0 la orice form
a n care pe diagonala
principal
a apar elementele a1 , a2 , . . . , an permutate cumva (si n rest, zerouri). n
particular, B 0 este asemenea cu C 0 .
Sa aratam acum ca B B 0 (si nu vom mai face demonstratia pentru C C 0 , ea
fiind ntru totul asem
an
atoare). ncepem prin a observa urm
atorul calcul:

1 0
0
1 0

0
0
=
=
,
a 1

1
a ( ) +
0
daca a este ales convenabil, adica daca a = / ( ); desigur, asta se poate face
numai n cazul n care 6= .
Un calcul asem
an
ator se poate face si pentru matrice de ordin n. Dac
a vom
considera Tij (a) BTij (a)1 = Tij (a) BTij (a), unde i > j, elementul aij din pozitia
(i, j) se nlocuieste cu a (aj ai ) + aij si a poate fi ales astfel nct acest element sa
devin
a nul (c
aci am presupus c
a ai 6= aj ). Mai sunt afectate si celelalte elemente ale
liniei i (la care se adun
a linia j nmultit
a cu a) si ale coloanei j (la care se adun
a
coloana i nmultita cu a). Remarcam ca aceste transformari oricum nu pot modifica
zerourile de deasupra diagonalei principale, care ramn intacte, si nici elementele de
pe diagonala principal
a.
1
Acum e clar ce avem de f
acut: mai nti calcul
am T21 (a) BT21 (a) care, pentru
un a bine ales reprezinta o matrice B1 asemenea cu B care are n pozitia (2, 1) pe
zero (asta daca nu era dinainte; de altfel se poate vedea usor ca, daca a21 = 0,
atunci a care ne trebuie este a = 0 deci T21 (a) = T21 (0) este, de fapt, matricea
identica). Apoi, pentru aceasta matrice calculam T31 (a) B1 T31 (a)1 , care pentru un
anumit a este o matrice asemenea cu B1 (deci si cu B) si are 0 n pozitia (3, 1); se
poate vedea c
a elementul 0 obtinut la pasul anterior nu va fi afectat. Continu
am
astfel, lucrnd cu matrice de forma Ti1 (a) pn
a cnd toate elementele de pe prima
coloana "de sub" a1 devin zerouri, apoi trecem si facem zerouri pe coloana a doua,
"sub" a2 , folosind transformari de tip T32 (a) , . . . , Tn2 (a) (adica nmultim cu acestea
la stnga si cu inversele lor la dreapta; la fiecare pas similaritatea matricelor se
p
astreaz
a), n ordine, alegnd, desigur, de fiecare dat
a valoare care trebuie pentru a.
Elementele nule obtinute pe prima coloana nu vor fi afectate, la fel cele de pe sau de
deasupra diagonalei principale, Tot asa vom proceda pna cnd, la urma, ajungem
la o matrice care are partea de deasupra diagonalei principale neschimbat
a, la fel
diagonala principal
a, iar sub diagonala principal
a are numai zerouri, adic
a ajungem
la B 0 si la concluzia dorita ca aceasta este asemenea cu B.
n concluzie, am aratat ca matricele B si C sunt asemenea, deci am ajuns acolo
unde ne-am propus: exist
a V GLn (K) astfel nct C = V BV 1 ; atunci A =
1
= B C = B V BV
si notnd X = BV 1 , Y = V avem A = XY Y X.
Demonstratia ar fi ncheiata, daca n-ar mai fi un mic amanunt de lamurit: ce
14

facem cu ipoteza suplimentar


a pe care am impus-o (si de care, s-a dovedit pe parcurs,
avem mare nevoie, caci daca elementele de pe diagonala nu sunt distincte, nu-l putem
alege pe a astfel nct Tij (a) sa produca un zero n locul lui aij )? Raspunsul nu e att
de greu si arat
a, cum spuneam, c
a restrictia dat
a de aceast
a ipotez
a nu este chiar att
de ... restrictiv
a. E suficient s
a mp
artim multimea elementelor de pe diagonal
a n
submultimi disjuncte doua cte doua, fiecare dintre acestea avnd suma elementelor
0 si fiecare nemaiavnd alta submultime (stricta) pentru care suma elementelor este
0. S
a numim b1 , . . . , bk elementele unei asemenea submultimi (a c
aror sum
a este,
asadar, zero); pentru acestea putem determina c1 , c2 , . . . , ck astfel nct b1 = c1 c2 ,
b2 = c2 c3 , . . . , bk1 = ck1 ck , bk = ck c1 . Mai mult, oricare doua dintre
c1 , c2 , . . . , ck sunt distincte doua cte doua si proprietatile lor se pastreaza daca le
asim cte o grupare de asemenea c-uri
nlocuim cu c1 + t, c2 + t, . . . , ck + t, t K. G
distincte dou
a cte dou
a pentru fiecare submultime de b-uri a multimii elementelor
de pe diagonala principala, iar apoi alegem cte un t pentru fiecare astfel de grupare
nct toate c-urile sa fie distincte doua cte doua (ceea ce sigur se poate face n cazul
n care corpul K este infinit; gnditi-v
a de ce!). Mai departe totul decurge la fel,
deoarece putem scrie matricea noastr
a ca diferenta a dou
a matrice, una inferior, alta
superior triunghiulara, fiecare dintre aceste elemente sunt distincte doua cte doua.
Propozitia este complet demonstrat
a.
Noi ne-am propus s
a rezolv
am problema n cazul corpurilor uzuale de numere:
Q, R, C, de aceea ipoteza pe care am facut-o asupra infinitatii corpului K nu ne
deranjeaz
a foarte mult; totusi, se prea poate ca aceast
a presupunere s
a fie strict
legat
a de rezolvarea pe care am dat-o aici si s
a nu fie esential
a. Asadar, ramne
ntrebarea daca este valabil enuntul propozitiei demonstrate si n cazul unui corp
finit.
n ncheiere, s
a mai spunem c
a nu exist
a nici o pretentie de originalitate n elaborarea acestei note; este foarte posibil ca aceast
a solutie s
a fie cunoscut
a, atta doar c
a
autorul nu are nici un fel de referinta pentru problema discutat
a, pe care o cunoaste
doar din folclor (n urma cu ctiva ani aceasta problema mi-a fost comunicata "prin
viu grai" de c
atre un elev, actualmente student str
alucit al Facult
atii de Matematic
a
din Bucuresti; asa c
a i multumesc pe aceast
a cale lui Dragos Deliu, care m-a f
acut
s
a caut s
a rezolv aceast
a problem
a, c
aut
ari din care s-a n
ascut si aceast
a not
a).

Recreaii matematice
1. S
a se ndep
arteze patru segmente din figura al
aturat
a
(alc
atuit
a din sase p
atrate) astfel nct noua figur
a s
a fie formata din trei patrate.
Not
a. Solutia problemei se poate g
asi la pagina 39.

15

Trei perle ale olimpiadelor de matematic


a
Gabriel DOSPINESCU 1
Problemele propuse la testele de selectie pentru OIM sau la fazele nationale din
diverse tari se remarca prin profunzimea (si uneori simplitatea) ideilor care le rezolva.
n cele ce urmeaz
a, vom rezolva trei probleme propuse la astfel de teste de selectie
n anii 2002 si 2003, demonstrnd dificultatea rezolv
arii problemelor de "matematic
a
elementara", precum si tendinta accentuata de a mbina algebra, teoria numerelor si
analiza matematica n actul de concepere si rezolvare a unor asemenea perle matematice.
1. Un prim exemplu este urmatoarea problema propusa la unul din testele de
selectie pentru OIM n anul 2002, in Vietnam. n rezolvare vom folosi doar cteva
rezultate legate de ecuatia de gradul al doilea. Dup
a cum se stie, multe probleme
dificile se rezolv
a relativ usor folosind trinomul de gradul al doilea (metoda coborrii).
Vom da doar doua exemple, fara a insista prea mult.
a
b
c
1) Aratati ca daca numarul d =
+
+
este ntreg, iar a, b, c sunt numere
bc ca ab
naturale, atunci d este 1 sau 3.
2) Aratati ca, daca numerele naturale distincte si nenule a1 , a2 , . . . , an verifica
a21 + a22 + + a2n = na1 a2 an , atunci ele sunt prime ntre ele doua cte doua.
ncercati sa rezolvati aceste doua probleme nainte de abordarea problemei 1.
PROBLEMA 1. Sa se demonstreze ca exista un numar m 2002 si m numere
m
n
Y
X
a2i 4
a2i sa fie patrat
naturale nenule a1 , a2 , . . . , am , distincte, astfel nct
i=1

i=1

perfect.
Solutie. Vom folosi trinomul pentru a crea solutii pentru anumite ecuatii diofantice, deci n mod constructiv.
m
Y
Ar fi bine s
a dispar
a
a2i . Deci, s
a scriem expresia sub forma
i=1

m
Y

i=1

a2i

n
X

a2i

i=1

m
Y

i=1

ai k

!2

Pentru a "scapa" si de 4, luam k = 2. Asadar am adus problema la o forma mai


"acceptabila" (dar nu mai putin dificila):
Aratati ca exista m 2002 si a1 , a2 , . . . , am N distincte astfel nct
1 + a21 + a22 + + a2m = a1 a2 am .

(1)

Sa cautam m astfel nct m 3 dintre necunoscutele ecuatiei (1) sa fie 1. Aceasta


revine la ecuatia
(2)
x2 + y 2 + z 2 + m 2 = xyz.
Privind aceasta ecuatie ca una de gradul al doilea
n
z,
vom
ncerca
s
a
lu
a
m
dis

criminantul nul. Deci x2 y 2 = 4 x2 + y 2 + m 2 . Luam x = 2a, y = 2b si obtinem


1

Student, Facultatea de Matematic


a-Informatic
a, Bucuresti

16

m = 4 a2 1 b2 1 2.
a concluzion
am: putem alege b > a > 2002 diferite si
2S

2
putem lua m = 4 a 1 b 1 2 > 2002. Atunci ecuatia (2) va avea solutiile
(x, y, z) = (2a, 2b, 2ab). Rezult
a c
a ecuatia (1) are solutia (2a, 2b, 2ab, 1, 1, . . . , 1).
Dar putem scrie (1) si sub forma
12 1 2a 2b 2ab + (2a)2 + (2b)2 + (2ab)2 + m 3 = 0.

(3)

Din relatiile lui Vite rezult


a c
a si 2a 2b 2ab 1 este solutie a ecuatiei (3), n
care n loc de 1 punem t. Asadar am redus cu o unitate num
arul celor m 3 de 1 si
am obtinut o noua solutie a ecuatiei (1): (2a, 2b, 2ab, 2a 2b 2ab 1, 1, 1, . . . , 1).
Analog, scriem
2

12 1 2a 2b 2ab (2a 2b 2ab 1) + (2a) + (2b) + (2ab) +


2

+ (2a 2b 2ab 1) + m 4 = 0.

Deci obtinem o alta solutie a ecuatiei (1), cu numar si mai mic de 1:


(2a, 2b, 2ab, 2a 2b 2ab 1, 2a 2b 2ab (2a 2b 2ab 1) 1, 1, 1, . . . , 1) .

Astfel, rezulta ca putem elimina pe rnd fiecare 1 din m-upla (2a, 2b, 2ab, 1, 1, . . . , 1).
Riguros, aceasta nseamna ca folosind succesiv relatiile lui Vite, obtinem cte o ma 2a = x1 < 2b = x2 < 2ab = x3 <
upl
a (x1 , x2 , . . . , xk , 1, 1, . . . , 1) n care este clar c
< < xk . La sfrsit (c
aci dup
a cel mult m pasi am eliminat toti de 1), obtinem o
solutie (a1 , a2 , . . . , am ) a ecuatiei (1), n care a1 < a2 < < am . Aceasta m-upla
va satisface conditiile enuntului.
2. Continuam cu o frumoasa problema propusa la ultima runda a olimpiadei
poloneze n anul 2003. Simplitatea solutiei care urmeaza nu are nsa nici o legatura
cu dificultatea problemei, c
aci multe metode de atacare a problemei nu duc la nici
un rezultat.
PROBLEMA 2. Determinati polinoamele cu coeficienti ntregi f cu proprietatea ca pentru orice n natural avem f (n) | 2n 1.
Solutie. Evident, problema ar fi banala daca s-ar demonstra ca exista o infinitate
de numere n pentru care 2n 1 este numar prim. Dar, dupa cum vom vedea, problema
accept
a si solutii mai "blnde".
Cum este clar c
a nu putem afla prea multe despre divizorii si factorii primi ai lui
2n 1, vom ncerca sa lucram cu divizori ai numerelor de forma f (n). Primul lucru
care ne vine n minte, tinnd seama ca f are coeficienti ntregi, este sa folosim
rezultatul urm
ator: m n | f (m) f (n). Deci, va trebui s
a c
aut
am m si n
astfel nct f (m) | f (n). Dup
a c
aut
ari mai mult sau mai putin lungi, g
asim c
a
f (n) = n + f (n) n | f (n + f (n)) f (n). Deci f (n) | f (n + f (n)).
n acest moment, jumatate din problema este rezolvata. ntr-adevar, schimbnd
f cu f , putem presupune c
a f are coeficientul dominant pozitiv. Atunci exist
aM
astfel nct pentru n > M s
a avem f (n) N. Fix
am un n > M . Avem f (n) | 2n 1 si
f (n) | f (n + f (n)) | 2n+f (n) 1 = (2n 1) 2f (n) + 2f (n) 1 (evident, n + f (n) N),
deci f (n) | 2f (n) 1. Daca am putea demonstra ca singurul numar natural n pentru
care n | 2n 1 este 1, atunci ar rezulta c
a pentru n > M avem f (n) = 1, adic
af
ar fi constanta 1. Dar faptul c
a n | 2n 1 implic
a n = 1 este binecunoscut si destul
de simplu. Sa presupunem ca n > 1 si sa luam p cel mai mic factor prim al lui n.
17

Atunci este clar c


a (n, p 1) = 1. Dar p | n | 2n 1 si p | 2p1 1 (teorema lui
Fermat). Deci p | 2n 1, 2p1 1 . Se stie ca sirul (xn )n1 , xn = 2n 1 este sir
Mersenne (adica (xm , xn ) = x(m,n) ). Rezulta ca p | (xn , xp1 ) = x(n,p1) = x1 = 1,
contradictie. Asadar n = 1 si f este constanta 1. Cum, dac
a f este solutie, atunci si
f este solutie, deducem c
a polinoamele cerute sunt constantele 1 si 1.
3. ncheiem scurta incursiune prin matematica elementara cu o problema extrem
de dificila, propusa la un test de selectie n Vietnam, 2002. Dificultatea problemei
const
a mai ales n faptul c
a admite multe solutii (care nici nu se intrez
aresc usor), iar
frumusetea const
a n mbinarea algebrei cu analiza matematic
a si teoria numerelor.
Nu exageram daca afirmam ca urmatoarea problema este una dintre cele mai dificile
si frumoase probleme referitoare la polinoame, propuse la vreun concurs pentru elevi.
PROBLEMA 3. Determinati toate polinoamele
p Z [X] cu proprietatea ca

exista un polinom q Z [X] pentru care q 2 (X) = X 2 + 6X + 10 p2 (X) 1.


Solutie. Evident,
orice rezolvitor "srguincios" va scrie relatia din enunt sub
forma q 2 (X 3) = X 2 + 1 p2 (X 3) 1 si va nota f (X) = p(X 3), g(X) =
= q (X 3). Deci

2
(1)
X + 1 f 2 (X) = g 2 (X) + 1.
Aici este ns
a punctul de oprire, c
aci orice ncercare ulterioar
a de rezolvare esueaz
a.
Ca de obicei, vom putea presupune ca f si g au coeficientii dominanti pozitivi (caci
putem schimba f cu f sau g cu g, f
ar
a a se modifica nimic). Deci exist
a M astfel
nct pentru orice n > M s
a avem f (n) , g (n) N.
Apel
am acum la teoria numerelor. Este binecunoscut faptul c
a toate solutiile n
numere naturale ale ecuatiei Pell x2 + 1 = 2y 2 sunt date de
2n1
2n1
2n1
2n1

+ 1 2
1 2
1+ 2
1+ 2

xn =
.
, yn =
2
2 2

Ce se ntmpla daca substituim xn n (1)? Obtinem g 2 (xn ) + 1 = 2 (yn f (xn ))2 .


Da, si perechea (g (xn ) , yn f (xn )) este solutie a ecuatiei Pell si aceasta se ntmpl
a
pentru orice n > M . Deci exist
a sirurile (an )n>M , (bn )n>M astfel nct g (xn ) = xan ,
yn f (xn ) = ybn .
Acum ncepe partea analizei matematice. Fie grad g = k , grad f = m. Avem

2an 1k(2n1)
2xan
= lim
lim 1 + 2
k(2n1) =
n
n
1+ 2

k
xn
g (xn )
= 2 lim
(2n1) = finit.

n xk
n
1+ 2

Rezulta ca sirul de numere ntregi (2an 1 k (2n 1))n>M este convergent, deci
stationar.
Asadar, exist
a n0 > M astfel nct pentru n > n0 s
a avem
2an 1 k (2n 1) = u, pentru o constant
a ntreag
a u. Ca urmare, pentru n > n0
avem

2n1 !
k(2n1)+u
k(2n1)+u
2n1
+ 1 2
+ 1 2
1+ 2
1+ 2
=
.
g
2
2
18

Rezult
a c
a

u
u

1
1
k
x 1+ 2 +
1 2
x
x

x
g
(2)
=
2
2
n
o
2n1
| n > n0 . Aducnd la acelasi numitor
pentru orice x din multimea 1 + 2
n (2), obtinem o identitate polinomiala adevarata pentru o infinitate de valori ale
variabilei, deci (2) este adev
arat
a pentru orice x nenul. Dup
a ce aducem
la
u acelasi

numitor si egalam coeficientii dominanti n (2), deducem ca 2k1 1 + 2 = k ,


unde k este coeficientul dominant al lui g. Dar aceasta implic
a u = 0. Asadar,
pentru orice x nenul, avem

1
1
k
x +
x
x

x
g
.
(3)
=
2
2

t2 + 1, din (3) obtinem c


a pentru orice t avem

k
k
t + 1 + t2 + t t2 + 1
g(t) =
.
(4)
2
Lu
am n (1) x = i si obtinem c
a g 2 (i) = 1. Deci, folosind (4), obtinem i2k = 1,
adic
a k este impar. Din (4) si (1) rezult
a prin calcul c
a
"

k
k #2
X + X2 + 1 + X2 + 1 X
2

f (X) =
, k impar.
(5)
2 X2 + 1
Dac
a not
am x = t +

Cum f este polinom si are coeficientul dominant pozitiv, deducem din (5) ca

k
k
X + X2 + 1 + X2 + 1 X

f (X) =
.
(6)
2 X2 + 1
Dar, daca f verifica (1), atunci si f verifica aceeasi relatie. Mai mult, polinomul
din membrul drept al relatiei (6) are coeficienti ntregi. Rezulta ca exista doua tipuri
de polinoame care verific
a relatia (1)

k
k

X + X2 + 1 + X2 + 1 X

, k impar.
(7)

2 X2 + 1
n sfrsit, obtinem c
a polinoamele p cerute se obtin din polinoamele (7) nlocuind
X cu X + 3.
Ce-ar mai fi de ad
augat dup
a prezentarea acestor trei nestemate din siragul nesfrsit al problemelor elementare de matematic
a? S
lefuite cu r
abdarea bijutierului,
cele trei probleme adaug
a o palet
a de lumini ncepnd cu actul creator al conceperii
lor si terminnd cu solutiile propuse. Fiecare dintre noi are nevoie de asemenea perle,
iar aceast
a scurt
a prezentare se nscrie pe aceast
a linie.

19

n leg
atur
a cu o problem
a de concurs
Dan Stefan
MARINESCU 1

La etapa finala a Olimpiadei de matematica din anul 1989 prof. univ. dr. T. Precupanu a propus urmatoarea problema:
Z b
Daca f : [a, b] R este o functie integrabila, continua pe (a, b) si

f (x) dx 6= 0,

a
x1 , x2 , . . . , xn

atunci pentru fiecare n N exista n numere distincte


Z b
ca
n (b a)
f (x) dx =
.
1
1
1
a
+
+ +
f (x1 ) f (x2 )
f (xn )

(a, b) astfel
(1)

(enunt partial)
Enuntul si o solutie a problemei pot fi aflate n [3]. n cele ce urmeaz
a vom
prezenta o generalizare a acestei frumoase probleme.
Pentru ceea ce ne-am propus, avem nevoie de
Propozitia 1. Fie f, g : [0, 1] R doua functii cu urmatoarele proprietati:
i) f , g continue pe [0, 1],
ii) f , g derivabile pe (0, 1),
iii) f (1) 6= f (0) si g 0 (x) 6= 0, x (0, 1).
Atunci pentru orice n N si orice 1 , 2 , . . . , n > 0 cu 1 + 2 + + n = 1
exista x1 , x2 , . . . , xn (0, 1) cu x1 < x2 < < xn astfel nct
n
X
g 0 (xi )
g (1) g (0)
i 0
=
.
(2)
f (xi )
f (1) f (0)
i=1

f (x) f (0)
; evident h este continu
a
f (1) f (0)
pe [0, 1], derivabila pe (0, 1) si h (0) = 0, h (1) = 1.
Pentru orice k {1, 2, . . . , n 1} consideram functia continua hk : [0, 1] R,
k
P
hk (x) = h (x)
i . Cum h1 (0) = 1 < 0, h1 (1) = h (1) 1 = 1 1 > 0,
Demonstratie. Fie h : [0, 1] R, h (x) =

i=1

conchidem, din continuitatea functiei h1 , ca exista c1 (0, 1) cu h1 (c1 ) = 0


h (c1 ) = 1 . Analog, h2 (c1 ) = h (c1 ) 1 2 = 2 < 0, h2 (1) = h (1) 1 2 =
= 1 1 2 > 0, de unde acelasi rationament conduce la existenta unui c2 (c1 , 1)
astfel nct h2 (c2 ) = 0 h (c2 ) = 1 + 2 . Inductiv, gasim 0 < c1 < c2 < <
< cn1 < 1 astfel nct
h (c1 ) = 1 ,

h (c2 ) = 1 + 2 ,

...

, h (cn1 ) = 1 + 2 + . . . + n1 .

(3)

a
Fie c0 = 0 si cn = 1, atunci pentru orice k {1, 2, . . . , n} functiile h si g verific
conditiile din teorema lui Cauchy pe intervalul [ck1 , ck ]; ca urmare, deducem c
a
exista x1 (c0 , c1 ), x2 (c1 , c2 ), . . . , xn (cn1 , cn ) astfel nct
h0 (x1 )
h (c1 ) h (c0 ) h0 (x2 )
h (c2 ) h (c1 )
h0 (xn )
h (cn ) h (cn1 )
=
,
=
,
.
.
.
,
=
,
g 0 (x1 )
g (c1 ) g (c0 ) g 0 (x2 )
g (c2 ) g (c1 )
g 0 (xn )
g (cn ) g (cn1 )
de unde, mpreun
a cu (3), avem:
1

Profesor, Liceul Teoretic "Iancu de Hunedoara", Hunedoara

20

h0 (x1 )
2
h0 (x2 )
n
h0 (xn )
1
= 0
,
= 0
, ...,
= 0
g (c1 ) g (c0 )
g (x1 ) g (c2 ) g (c1 )
g (x2 )
g (cn ) g (cn1 )
g (xn )
ceea ce, tinnd seama de faptul ca 1 , 2 , . . . , n > 0, conduce la relatiile
g 0 (x1 )
g 0 (x2 )
g 0 (xn )
1 0
= g (c1 )g (c0 ) , 2 0
= g (c2 )g (c1 ) , . . . , n 0
= g(cn )g(cn1 ) .
h (x1 )
h (x2 )
h (xn )
n
X
g 0 (xi )
De aici, prin adunare, obtinem
i 0
= g (cn ) g (c0 ) = g (1) g (0). Cum
h (xi )
i=1
f 0 (x)
h0 (x) =
, x (0, 1), conchidem c
a are loc relatia (2).
f (1) f (0)
Corolarul 1 [1]. Daca f : [0, 1] R este o functie continua pe [0, 1], derivabila
pe (0, 1), f (0) = 0, f (1) = 1, atunci pentru orice n N si orice k1 , k2 , . . . , kn > 0
n
n
X
X
ki
exista x1 , x2 , . . . , xn (0, 1) distincte doua cte doua astfel nct
ki
=
f 0 (xi )
i=1
i=1
(vezi si [2], [4]).
n
.X
Demonstratie. Consideram n Propozitia 1, i = ki
ki , pentru orice
i=1
i {1, 2, . . . , n} si g : [0, 1] R, g (x) = x.
Corolarul 2. Fie f, g : [a, b] R dou
a functii integrabile, continue pe (a, b),
Z b
f (x) dx 6= 0 si g (x) 6= 0, x (a, b). Atunci, pentru fiecare n N si 1 , 2 , . . . ,
a

a n numere distincte x1 , x2 , . . . , xn (a, b)


n > 0 cu 1 + 2 + + n = 1, exist
astfel ca
Z b
Z
n
b
X g (xi )
i
g (x) dx
f (x) dx.
(4)
=
f (xi )
a
a
i=1
Z x
Demonstratie. Fie f1 , g1 : [0, 1] R, f1 (x) =
f ((1 t) a + tb) dt, g1 (x) =
Z x
0
g ((1 t) a + tb) dt. n mod evident, f1 si g1 sunt bine definite, verific
a conditiile
=
Z b
Z b
0
1
1
g (x) dx, f1 (1)f1 (0) =
f (x) dx.
din Propozitia 1 si g1 (1)g1 (0) =
ba a
ba a
De unde exist
a c1 < c2 < < cn din (0, 1) astfel nct
Rb
Rb
n
n
X
X
g (x) dx
g (x) dx
g10 (ci )
g ((1 ci ) a + ci b)
a
i 0
i

.
= Rb
= R ab
f
(c
)
f
((1

c
)
a
+
c
b)
i
i
f (x) dx
f (x) dx
1 i
i=1
i=1
a
a
a x1 < x2 < < xn si obtinem
Lund xi = (1 ci ) a + ci b, i = 1, n, evident c
formula (4).
Observatie. Egalitatea (1) se obtine lund n (4) 1 = 2 = = n = 1/n si
g (x) = 1, x [a, b].
1.
2.
3.
4.

Bibliografie
G. G. Z. Giang - Problema 1125, Math. Mag.
P. Orno - Problema 1053, Math. Mag.
I. Tomescu (coordonator) - Probleme date la olimpiadele de matematica pentru
licee (19501990), Ed. stiintifica, Bucuresti, 1992.
***
- Problema C:1791, G. M. 3/1996.
21

Asupra unei probleme propuse la O. I. M. - 1982


Neculai ROMAN 1
La O. I. M. n anul 1982 a fost propus
a problema B3 GB:
Fie ABC un triunghi si P un punct n interiorul lui astfel ca P AC P BC.
Fie L, M picioarele perpendicularelor din P pe BC, CA respectiv. Fie D mijlocul
lui [AB]. Sa se demonstreze ca DL = DM .
Enuntul si o solutie a acestei probleme se poate g
asi n [1], pag. 322 si 333. Problema are si o solutie mai simpl
a, accesibil
a si elevului de gimnaziu si care merit
a
a fi cunoscuta. De asemenea, vom arata ca problema are loc pentru o multime mai
variat
a de puncte din planul triunghiului. n acest scop, vom demonstra urm
atoarea
0

Teorem
a. Fie ABC un triunghi, D mijlocul lui [AB] si punctele A , B pe
0
dreptele AC, respectiv BC astfel ca C (AA ) si C (BB 0 ). Fie P un punct
n planul triunghiului si L, M picioarele perpendicularelor din P pe BC, respectiv AC. Sa se demonstreze ca daca P Int (ACB) Int (A0 CB 0 ) astfel ca
P AC P BC sau daca P Int (BCA0 ) Int (ACB 0 ) astfel ca m (P AC) +
+m (P BC) = 180 atunci DM = DL.
Demonstratie. Fie punctele D0 si D00 mijloacele segmentelor [P A], respectiv
[P B] (fig. 1, 2 si 3).

D
M

D
B

CB

D
A

P
M

Fig. 1

L B L

Fig. 3

Fig. 2

PA
atoare ipotenuzei n
= DD00 , ([M D0 ] mediana corespunz
2
00
4AM P si [DD ] linie mijlocie n 4AP B).
Deci
[M D0 ] [DD00 ] .
(1)
Analog,
[LD00 ] [DD0 ] .
(2)
0
00
Din P D DD paralelogram, rezulta ca
P D0 D P D00 D
(3)
Avem M D0 =

Profesor, S
coala "V. Alecsandri", Mircesti, Iasi

22

Din P Int (ACB) Int (A0 CB 0 ) astfel ca P AC P BC (fig. 1 si 2) deducem


ca P D0 M P D00 L (teorema unghiului exterior).
Daca P Int (BCA0 ) astfel ca m (P AC) + m (P BC) = 180 (fig. 3), atunci
P AC P BL si deci P D0 M P D00 L (teorema unghiului exterior). Dac
a
P Int (ACB 0 ) astfel ca m (P AC)+m (P BC) = 180 , atunci P BC P AM
si deci P D0 M P D00 L.
n concluzie,
P D0 M P D00 L.
(4)
Din relatiile (3) si (4) rezult
a c
a
M D0 D LD00 D.
0

(5)
00

Acum din relatiile (1), (2) si (5) rezulta ca 4M D D 4DD L, de unde obtinem
[DM ] [DL] si deci DM = DL.

Teorema reciproc
a. Fie ABC un triunghi, D mijlocul lui [AB] si punctele A0 ,
B pe dreptele AC respectiv BC astfel ca C (AA0 ) si C (BB 0 ). Fie punctele L,
M pe dreptele BC respectiv AC astfel ca DM = DL. Perpendicularele n M si L pe
AC respectiv BC se ntlnesc n P . Sa se demonstreze afirmatiile:
a) daca P Int (ACB) Int (A0 CB 0 ), atunci P AC P BC;
b) daca P Int (ACB 0 )Int (BCA0 ), atunci m (P AC)+m (P BC) = 180 .
Demonstratie. a) Fie punctele D0 si D00 mijloacele segmentelor [P A] respectiv
[P B].
Se arata usor ca 4M D0 D 4DD00 L, de unde rezulta ca
0

00

M D0 D DD00 L.

(6)

P D0 D P D00 D.

(7)

P D0 M P D00 L.

(8)

Din P D DD paralelogram, rezult


a
Din relatiile (6) si (7) obtinem

a) Daca P Int (ACB)Int (A CB ), atunci din (8) rezulta P AC P BC


(fig.1 si 2).
b) Dac
a P Int (BCA0 ) (fig. 3), atunci din relatia (8) rezult
a c
a P AC P BL
si, deci, m (P AC) + m (P BC) = 180 .
Dac
a P Int (ACB 0 ), atunci din relatia (8) rezult
a: P BC P AM . Deci
m (P AC) + m (P BC) = 180 .
0

Bibliografie
1. I. Cuculescu - Olimpiadele internationale de matematica ale elevilor, Ed.
Tehnic
a, Bucuresti, 1984.

23

Asupra unei ecuatii functionale


Loredana AGORE 1
Scopul acestei note este rezolvarea ecuatiei functionale
f (axy + x + y) = bf (x) f (y) + c [f (x) + f (y)] + d (a, b, c, d R)

(1)

f (x + y) = f (x) + f (y) ,

(2)

n multimea functiilor f : R R sau f : R R sau f :


n ecuatia (1) este cuprins
a ecuatia lui Cauchy

R+

R.

ale c
arei solutii se numesc functii aditive, precum si urm
atoarele ecuatii clasice ce
sunt reductibile la ecuatia lui Cauchy:
f (x + y) = f (x) f (y) ,
(3)
f (xy) = f (x) + f (y) ,
(4)
f (xy) = f (x) f (y) .
(5)
Rezolvarea n detaliu a ecuatiilor (2) (5) se poate gasi n [1]. Tot n [1], p. 23, este
studiat
a si ecuatia functional
a obtinut
a considernd n (1) a 0, c = 1, d = 0.
Rezolvarea ecuatiei functionale (1) const
a n reducerea ei, potrivit cu parametrii
a, b, c si d, la una dintre ecuatiile (2) (5). Distingem cteva cazuri.
I a 6= 0, b 6= 0. nmultim ecuatia (1) cu b si apoi punem rezultatul obtinut sub
forma

(ax + 1) 1
(ax + 1) (ay + 1) 1
+ c = bf
+c
bf
a
a

(ay + 1) 1
bf
+ c + bd + c c2 . (6)
a

t1
2
Cu notatiile = bd + c c , u = ax + 1, v = ay + 1 si g(t) = bf
+ c, t R,
a
ecuatia (6) se scrie
g (uv) = g (u) g (v) + .
(7)
Dac
a bd = c2 c, adic
a = 0, atunci (7) este de tipul (5) si se reduce la ecuatia
lui Cauchy [1]. Solutiile se exprim
a cu functiile aditive sau sunt functii constante.
Daca bd 6= c2 c, deci 6= 0, luam u = v = 1 n (7) si obtinem
g (1) = [g (1)]2 + .

1
1
1
De aici, g (1) =
1 1 4 =
1 (1 2c)2 4bd , cu conditia ca
2
2
4
1
bd c2 c + . Pe de alt
a parte, ecuatia (7) cu v = 1 devine
4

g (u) = g (1)
g (u) = g (u) g (1) + g (u) =
1 g (1)
(evident, g (1) 6= 1). Ca urmare,ecuatia (1) are, n cazul considerat,
doua solutii

q
1
1
2
date de f (x) = [g (1) c] =
(1 2c) (1 2c) 4bd .
b
2b
1

Elev
a, cl. a XI-a, Colegiul National "Mihai Viteazul", Bucuresti

24

II a = 0, b 6= 0. Ecuatia (1) devine

f (x + y) = bf (x) f (y) + c [f (x) + f (y)] + d

(8)

si se poate scrie n forma


bf (x + y) + c = [bf (x) + c] [bf (y) + c] + bd + c c2

(9)

g (x + y) = g (x) g (y) + .

(10)

sau, notnd = bd + c c si g (t) = bf (t) + c, t R,


2

Dac
a bd = c c, atunci (10) este de tipul (3) etc.
2
Daca bd 6= c2c, n (8) luam x = y = 0 si ob
tinem f (0) = b [f (0)] +2cf (0)+d (),
q
1
deci f (0) =
(1 2c) (1 2c)2 4bd (n mod necesar, (1 2c)2 4bd 0
2b
1 4 0). Dar, daca n (8) luam y = 0 si apoi grupam convenabil, avem
()

[1 bf (0) c] f (x) = cf (0) + d


[1 bf (0) c] f (x) = [1 bf (0) c] f (0) f (x) = f (0)
()

(1 bf (0) c 6= 0 bd 6= c2 c). Avem dou


a solutii:

q
1
f (x) = f (0) =
(1 2c) (1 2c)2 4bd .
2b
III a 6= 0, b = 0. n acest caz, (1) se scrie

f (axy + x + y) = c [f (x) + f (y)] + d.

(11)

Dac
a c = 1, punem (11) n forma

(ax + 1) (ay + 1) 1
(ax + 1) 1
(ay + 1) 1
f
+d = f
+d + f
+d
a
a
a
t 1
+ d, t R,
sau, notnd u = ax + 1, v = ay + 1 si g (t) = f
a
g (uv) = g (u) + g (v) ,
care este o ecuatie de tipul (4).
Dac
a c 6= 1, pentru x = y = 0 luat n (11) obtinem

f (0) = 2cf (0) + d (1 2c) f (0) = d.

(12)

Cum, pentru y = 0 n (11), avem

(12)

f (x) = cf (x) + cf (0) + d f (x) = cf (x) + f (0) cf (0)


d
(12)
(1 c) f (x) = (1 c) f (0) f (x) = f (0) f (x) =
1 2c
1
dac
a mai presupunem n plus c 6= . Este banal
a verificarea faptului c
a, n conditiile
2
d
impuse, f (x) =
este solutie a ecuatiei (11).
1 2c
1
Daca c = , avem de rezolvat ecuatia
2
1
(13)
f (axy + x + y) = [f (x) + f (y)] + d.
2
25

1
[f (0) + f (0)] + d, deci d = 0. Lund acum
2

1
1
1
f (x) + f
, de unde rezulta
y = n (13) cu d = 0, vom avea f (0) =
a
2
a
ca f (x) = k (constant), x R. Se verifica usor ca aceasta functie este ntr-adevar
solutie pentru orice k R.
Pentru x = y = 0, obtinem f (0) =

IV a = 0, b = 0. Este vorba de ecuatia


f (x + y) = c [f (x) + f (y)] + d.
(14)
Daca c = 1, (14) se poate scrie
f (x + y) + d = [f (x) + d] + [f (x) + d] ,
care este o ecuatie Cauchy n g (t) = f (t) + d, t R.
Dac
a c 6= 1, lu
am x = y = 0 n (14) si obtinem, ca si n cazul precedent,
relatiile echivalente (12). Se continua tot ca n cazul amintit si se obtine solutia
1
d
, x R, dac
a c 6= .
f (x) = f (0) =
1 2c
2
1
Dac
a c = , (14) se scrie
2
1
(15)
f (x + y) = [f (x) + f (y)] + d.
2
Lund x = y = 0, constat
am c
a d = 0. Punnd n (15) d = 0 si fixnd y = 0,
1
obtinem f (x) = [f (x) + f (0)]. Deci f (x) = f (0) = constant, x R, functie ce
2
este solutie a ecuatiei (15).

Bibliografie
1. V. Pop - Ecuatii functionale. Ecuatii clasice si probleme, Ed. Mediamira, ClujNapoca, 2002.

Recreaii matematice
2. Un calator, care nu avea la el dect un lant cu sapte verigi de aur, poposeste
ntr-o zi la un han. El se ntelege cu hangiul sa-l plateasca pentru fiecare zi petrecuta
la han cte o verig
a de aur. Dac
a st
a sapte zile si plata trebuie f
acut
a n fiecare zi,
care este num
arul minim de t
aieturi care trebuie f
acute n lant pentru a putea pl
ati
pretul convenit? (Se accepta ca, atunci cnd este cazul, hangiul sa dea calatorului
ca rest un numar de verigi (posibil toate!) pe care le-a primit deja.)
3. Care este eroarea n "demonstratia" de mai jos a egalitatii 3 = 0?

x2 x + 1 = 0 | x
x3 (1) = 0

3
2

x x
x3 = 1

2+ x =
0
3

x = 1
x x x =0
2
Punnd x = 1 n x x + 1 = 0, obtinem 3 = 0.
Not
a. Solutiile problemelor 2 si 3 se pot g
asi la pagina 39.
26

Asupra unei inegalit


ati conditionate
Cezar LUPU 1
La OBM - 2001 a fost dat
a problema urm
atoare:
Fie a, b, c numere
reale
strict
pozitive
astfel
nct a + b + c abc. Sa se arate ca

a2 + b2 + c2 abc 3.
Cristinel Mortici, Romnia
Solutia autorului utilizeaza metoda reduceriila absurd. Presupunem ca are loc
inegalitatea contrar
a, adic
a a2 + b2 + c2 < abc 3. Aplicnd inegalitatea Cauchy

1
1
2
2
Schwarz, obtinem abc 3 > a2 +b2 +c2 (a + b + c) (abc) , de unde abc < 3 3.
3
3
Pe de
alt
a parte, aplicnd
inegalitatea mediilor, avem abc 3 > a2 + b2 + c2

3
2
2
2
3 a b c si, deci, abc > 3 3. Se obtine astfel o contradictie.
Alte solutii ale acestei probleme sunt prezentate n [1] si [2].
1. Problema de mai sus poate fi nt
arit
a astfel:
Problema 1. Sa se arate ca, daca a, b, c > 0 si a + b + c > abc, atunci
ab + bc + ca abc 3.
1
1
1
Solutia I. Ipoteza si concluzia se pot scrie n felul urm
ator:
+ +
1 si
ab bc ac

1 1 1
2
am binecunoscuta inegalitate (x + y + z) 3 (xy + yz + zx),
+ + 3. Utiliz
a b c

1 1 1
1
1
1
x, y, z R (1) pentru a obtine
3
+ +
+
+
, din care rezult
a
a b
c
ab bc ac

1 1 1
ca + + 3.
a b
c
Solu
t
ia
II. Inegalitatea ceruta rezulta direct din inegalitatea ab + bc + ca
p
3abc (a + b + c), a, b, c R+ (2) (aceasta se reduce la (1), daca notam x = ab,
y = bc si z = ca).
Problema 2. Se considera a, b, c >
0 astfel nct a + b + c abc. Sa se arate ca
bc
3
ca
ab
+ 2
+ 2

.
Cezar Lupu
2
a (b + c) b (c + a) c (a + b)
2
X
Solutie. Pentru prescurtarea scrierii folosim nsumarea ciclica
. Avem

2
2
X
1 X (bc)
1
(bc + ca + ab)
ab + bc + ca
3
bc
=

2
a (b + c)
abc
a (b + c)
abc 2 (bc + ca + ab)
2abc
2
X (bc)2
2
(s-a folosit (bc + ca + ab) 2 (bc + ca + ab)
, adev
arat
a conform inegaa (b + c)
litatii Cauchy-Schwarz).
2. Avnd ca punct de plecare inegalitatea conditionata data la OBM, se pot
obtine inegalit
ati geometrice ntr-un triunghi. S
a observ
am mai nti c
a avem:
Daca a, b, c sunt lungimile laturilor unui triunghi oarecare nscris ntr-un cerc de
raza egala cu unitatea, atunci a + b + c abc.
n [2] sunt date patru demonstratii. Reproducem una dintre ele. Formulele
abc
4RS = abc si S = pr conduc la relatia
= 2Rr. Utiliznd inegalitatea
a+b+c
lui Euler si faptul c
a R = 1, obtinem inegalitatea dorit
a.
1

Elev, cl. a X-a, Colegiul National "Mircea cel B


atrn", Constanta

27

Ca urmare, n conditia impus


a triunghiului, are loc si inegalitatea
a2 + b2 + c2 abc 3. De
altfel, aceasta din urma rezulta direct din cunoscuta
inegalitate a2 + b2 + c2 4S 3 (Weitzenbck, 1919) pentru R = 1.
O nt
arire a acestor inegalit
ati este dat
a de
Problema 3.
n oricetriunghieste satisfacuta inegalitatea
b ca +c ab 4S
ab + bc + ca
a bc +
3. n particular, daca R = 1, avem
ab + bc + ca a bc + b ca + c ab abc 3.
X
X
Solutie. Prima parte a dublei inegalit
ati se dovedeste astfel:
ab
a bc
1
1
1
1
1
1
+ + + + A2 + B 2 + C 2 AB + BC + CA (s-a notat
a
b
c
ca
ab
bc
1
1
1
A = , B = si C = ), care este adevarata.

a
c
b
X 1
X

3

.
Demonstr
am acum partea a doua, adic
a
a bc 4S 3 sau
R
bc
Cu inegalitatea Cauchy-Schwarz sau utiliznd inegalitatea (2) de mai sus, obtinem

X 1
9
9
9
3
X p
. Este suficient ca p

R
bc
3 (ab + bc + ca)
3 (ab + bc + ca)
bc

sau, echivalent, ab + bc + ca 9R2 . Aceasta decurge din ab + bc + ca a2 + b2 + c2


si faptul cunoscut c
a ntr-un triunghi are loc a2 + b2 + c2 9R2 .
Problema 4. Sa se arate ca n orice triunghi nscris ntr-un
cerc de raza egala

a
1 1 1
b
c
cu 1 are loc inegalitatea
+
+
+ 32
+ +
.
Cezar Lupu
bc ca ab
a b
c

2
Solutie. Este binecunoscut
a inegalitatea a2 + b2 + c2 4S 3 + (a b) +
si Hadwiger, 1938). Este echivalenta cu 2 (ab + bc + ca)
+ (b c)2 +(c a)2 (Finsler

a2 + b2 + c2 4S 3 si tinnd seama c
a R = 1, conduce la inegalitatea cerut
a.

Problema 5. n orice triunghi nscris ntr-un cerc de raza 1 are loc urmatoarea
a
b
c
3
inegalitate:
+
+

.
Cezar Lupu
bc (b + c) ca (c + a) ab (a + b)
2 (a + b + c)
Solutie. Utiliznd rezultatul
din Problema
3 si inegalitatea
Cauchy-Schwarz,

X (a bc)2
(a bc + b ca + c ab)2
3a2 b2 c2

, de unde, prin
putem scrie:
2 (a + b + c)
2 (a + b + c)
b+c
a.
mp
artire cu a2 b2 c2 , obtinem inegalitatea dorit
3. Propunem spre rezolvare urmatoarele probleme:
1. Se consider
a a, b, c trei numere reale strict pozitive
astfel nct a + b + c abc.

b2
c2
3 3
a2
+
+

.
Cezar Lupu, G.M.
Aratati ca
bc (b + c) ca (c + a) ab (a + b)
2
2. Fie ABC un triunghi
oarecare nscris ntr-un cerc de raza egala cu 1. Aratati

3
AB BC CA, P Int (ABC).
Cezar Lupu
c
a PA + PB + PC
3
Bibliografie
1. M. B
alun
a si M. Becheanu (prezentare de) - A 18-a OBM, 3-9 mai 2001, Belgrad,
GM - 5-6/2001, 229-236.
2. Cezar Lupu - Asupra unei probleme de concurs, Rev. Mate(matic
a), 2003, 17-20.
28

O metod
a de demonstrare a concurentei unor drepte
Gabriel POPA, Paul GEORGESCU 1
Vom exemplifica n cele ce urmeaza aplicabilitatea unei metode de demonstrare a
concurentei unor drepte, prea putin utilizata n contextul introducerii noii programe
scolare de geometrie.
Date dou
a puncte A, B avnd vectorii de pozitie rA si respectiv rB , vectorul de
pozitie al unui punct al dreptei AB este de forma
rM = rA + (1 ) rB , R

(ecuatia vectoriala a dreptei AB). Avnd o dreapt


a atasat
a unui triunghi, vectorul de
pozitie al unui punct curent M al s
au poate fi exprimat functie de vectorii de pozitie
ai vrfurilor si de un parametru real . Considernd nca o dreapta (cu parametrul
notat ), pentru a afla punctul comun celor doua drepte vom avea de rezolvat un
sistem liniar n si .
Dac
a dorim s
a prob
am concurenta a trei drepte, le vom intersecta dou
a cte
doua si vom urmari daca vectorii de pozitie ai punctelor obtinute coincid. Metoda
presupune, n general, un important volum de calcule, nsa este "sigura" si permite,
n plus fata de alte metode, pozitionarea punctului de concurenta.
Problema 1. Fie ABC un triunghi si M, N (BC), P, Q (AC), R, S (AB)
puncte astfel nct BM = CN = CP = AQ = AR = BS = x, unde 0 < 2x <
< min {AB, BC, CA}. Daca A1 , B1 , C1 sunt respectiv mijloacele segmentelor (SP ),
(RN ), (M Q), aratati ca dreptele AA1 , BB1 , CC1 sunt concurente.
Constantin Cocea
x
SB
=
Solutie. Punctul S mparte segmentul orientat BA n raportul
;
c

x
SA
atunci

cx
x
x
cx
rS =
rB +
rA =
rB + rA .
c
cx
c
c
bx
x
Analog, rP =
rC + rA si atunci
b
b
1
x (b + c)
cx
bx
rA +
rB +
rC .
rA1 = (rS + rP ) =
2
2bc
2c
2b
Vectorul de pozitie al unui punct curent X al dreptei AA1 va fi

x (b + c)
(c x)
(b x)
rX = rA1 + (1 ) rA =
+ (1 ) rA +
rB +
rC ,
2bc
2c
2b
unde R. Cu totul analog, vectorul de pozitie al unui punct curent Y al dreptei
BB1 va fi

(c x)
x (a + c)
(a x)
rY =
rA +
+ (1 ) rB +
rC .
2c
2ac
2a
1

Profesori, Colegiul National si Liceul de Informatic


a "Gr. Moisil", Iasi

29

Intersectia celor dou


a drepte se obtine rezolvnd sistemul

x (b + c)
(c x)

+ (1 ) =

2bc
2c

(c x)
x (a + c)
=
+ (1 )

2c
2ac

(b x) = (a x) .
2b
2a
Sistemul este compatibil determinat, cu solutia
2bc (a x)
;
= 2
x (a + b + c) 2x (ab + bc + ac) + 3abc
2ac (b x)
= 2
.
x (a + b + c) 2x (ab + bc + ac) + 3abc
Punctul comun al dreptelor AA1 si BB1 este T , unde
1
rT = 2
[a (b x) (c x) rA +
x (a + b + c) 2x (ab + bc + ac) + 3abc
+ b (a x) (c x) rB + c (a x) (b x) rC ] .

Scriind acum ecuatia vectorial


a a dreptei CC1 si aflnd intersectia acesteia cu AA1 ,
obtinem acelasi punct T . Urmeaza ca AA1 , BB1 , CC1 sunt concurente.

Observatii.
1) Calcule foarte asemanatoare rezolva problema L.25.a) din R. M. T. 2/1990,
autor Constantin Cocea. Legat de punctul b) al acestei probleme, comparnd
notele ap
arute n R. M. T. numerele 2/1991 si 1/1996, putem observa cum uneori
calculul vectorial ajut
a la simplificarea solutiilor (a se vedea si [4]).
2) n [6] se demonstreaza concurenta naltimilor si bisectoarelor unui triunghi
folosind aceasta metoda; aceste demonstratii au constituit punctul de plecare al articolului de fata.
3) Calculele pot fi simplificate atunci cnd, din considerente geometrice, intuim
anumite simetrii verificate de punctul de concurenta.
Problema 2. Fie H ortocentrul 4ABC, M , N si P mijloacele laturilor [BC],
[CA] respectiv [AB], iar A1 (AH), B1 (BH), C1 (CH) astfel nct
AA1
BB1
CC1
=
=
. Sa se arate ca dreptele A1 M , B1 N si C1 P sunt concurente.
A1 H
B1 H
C1 H
Gabriel Popa, Paul Georgescu
Solutie. Raportam planul la un reper cu originea n centrul cercului circumscris
A1 H
a
= k, atunci
triunghiului si fie rA , rB , rC vectorii de pozitie ai vrfurilor. Dac
AA1
1
rH = rA + rB + rC , rM = (rB + rC ) ,
2
1
k
1
1
rA1 =
rH +
rA = rA +
rB +
rC .
1+k
1+k
1+k
1+k
A1 Q
Cautam un punct Q (A1 M ) astfel nct
= l, iar rQ sa se exprime simetric
QM
30

functie de rA , rB si rC :


1
l
1
l
l
1
1
rA1 +
rM =
rA +
+
rB +
+
rC ;
rQ =
1+l
1+l
1+l
2 1+k
2 1+k
l
1+k
1
2k
pentru +
=1l=
, obtinem ca rQ =
(rA + rB + rC ). Analog,
2 1+k
1+k
1 + 3k
0
00
cautam Q (B1 N ) si Q (C1 P ) care sa se exprime simetric functie de rA , rB si
asi c
a Q0 = Q00 = Q, deci cele trei drepte sunt concurente.
rC ; vom g
Problema 3. Laturile (AB), (BC), (AC) ale triunghiului ABC sunt tangente
cercului nscris de centru I n punctele C1 , A1 respectiv B1 . Daca B2 este mijlocul
laturii (AC), demonstrati ca dreptele B1 I, A1 C1 si BB2 sunt concurente.
Olimpiad
a Rep. Moldova
Solutie. Functie de vectorii de pozitie ai vrfurilor 4ABC, vectorii de pozitie ai punctelor care
A
apar n problem
a sunt:
B1
a+bc
a+cb
rA1 =
rB +
rC ;
C1
B2
2a
2a
I
b+ac
b+ca
rB1 =
rA +
rC ;
2b
2b
c+ab
c+ba
rC1 =
rA +
rB ;
B
C
A1
2c
2c
1
a
b
c
rB2 = (rA + rC ) ; rI =
rA +
rB +
rC .
2
a+b+c
a+b+c
a+b+c
Fie X un punct pe IB1 ; atunci

(b + a c) (1 ) a
+
rA +
rX = rB1 + (1 ) rI =
2b
a+b+c

(1 ) b
(b + c a)
(1 ) c
+
rB +
+
rC , R.
a+b+c
2b
a+b+c
Cautam o valoare a lui pentru care rX sa aiba o exprimare simetrica n rA si rC ,
deci
(b + a c) (1 ) a
(b + c a)
(1 ) c
b
+
=
+
=
;
2b
a+b+c
2b
a+b+c
a+c
pentru aceasta valoare a lui ,
a+cb
b
a+cb
rX =
rA +
rB +
rC .
2 (a + c)
a+c
2 (a + c)
Fie acum Y punct pe A1 C1 ; atunci
c+ab
rY = rA1 + (1 ) rC1 = (1 )
rA +
2c

(a + c b)
(a + b c) (1 ) (c + b a)
+
+
rB +
rC .
2a
2c
2a
Cautnd o valoare pentru astfel nct rY sa aiba o exprimare simetrica n rA si rC ,
a
si, pentru aceast
obtinem =
a valoare,
a+c
a+cb
b
a+cb
rA +
rB +
rC ,
rY =
2 (a + c)
a+c
2 (a + c)
31

adic
a Y = X. S
a observ
am n final c
a, datorit
a simetriei n rA si rC , acest punct se
afla si pe mediana BB2 .
Probleme propuse
1. Fie GA , GB , GC , GD centrele de greutete ale fetelor tetraedrului ABCD, iar
M un punct interior tetraedrului. Daca A0 , B 0 , C 0 , D0 sunt situate respectiv pe
semidreptele (M GA , (M GB , (M GC , (M GD , n exteriorul tetraedrului, astfel nct
M GA
M GB
M GC
M GD
=
=
=
, s
a se arate c
a dreptele AA0 , BB 0 , CC 0 , DD0
0
0
0
GA A
GB B
GC C
GD D0
sunt concurente.
Gabriel Popa, Paul Georgescu
2. Fie ABC un triunghi nscris n cercul C, A1 , B1 , C1 punctele de pe C diametral
opuse vrfurilor, iar GA , GB , GC centrele de greutate ale triunghiurilor A1 BC, B1 CA,
respectiv C1 AB. Aratati ca dreptele AGA , BGB , CGC sunt concurente ntr-un punct
situat pe dreapta lui Euler a 4ABC.
Gabriel Popa, Paul Georgescu
\
3. Fie M n interiorul 4ABC. Bisectoarele interioare ale unghiurilor BM
C,
\
\
a se
CM
A, AM
B taie laturile [BC], [CA], respectiv [AB] n A1 , B1 , respectiv C1 . S
arate c
a AA1 , BB1 si CC1 sunt concurente.
Gheorghe Neagu
4. Fie D, E, F punctele de tangenta ale cercului nscris n 4ABC cu laturile
[BC], [CA], respectiv [AB]. Paralela prin E la AB taie F D n Q, iar paralela prin
D la AB taie EF n T . S
a se arate c
a dreptele CF , DE si T Q sunt concurente.
Marcel Chirita

5. Fie tetraedrul ABCD si punctele M (AB), N (CD), P (BC), Q (AD)


AM
DN BP
AQ
astfel nct
=
,
=
. Notam {A1 } = BN DP , {B1 } = AN CQ,
MB
NC PC
QD
{C1 } = BQ DM , {D1 } = AP CM . Sa se arate ca dreptele AA1 , BB1 , CC1 si
DD1 sunt concurente.
Bibliografie
1. C. Cocea - Problema L.25, R. M. T. - 2/1990.
2. C. Cocea - Problema X.8, R. M. T. - 1/1996.
3. P. Georgescu, G. Popa - Structuri fundamentale n algebra liniara, geometria vectoriala si geometria analitica, Ed. MatrixRom, Bucuresti, 2003.
4. G. Popa - Aplicatii ale dimensiunii dreptei vectoriale, planului vectorial si a spatiului
vectorial, Matematica pentru elevi, Galati, 17-18/2001.
5. G. Popa, P. Georgescu - Dreapta lui Euler privita ca loc geometric, Recreatii Matematice - 2/2002.
6. E. Murgulescu, N. Donciu - Culegere de probleme de geometrie analitica si diferentiala (vol. I), E. D. P., 1971.
7. *** - A 46-a Olimpiada de Matematica a Rep. Moldova, R. M. T - 3/2002.

32

Teorema ariciului si cteva aplicatii


Dumitru MIHALACHE 1
n aceasta not
a ne propunem s
a prezent
am un rezultat mai putin vehiculat n
literatura matematic
a romneasc
a din ultimii ani, precum si o aplicatie oarecum
neasteptata a sa; credem ca cititorii interesati vor gasi destule alte probleme care
sa-l foloseasca. Vom prezenta teorema ariciului n mod gradat (denumirea este
justificat
a de aspectul configuratiilor ce vor ap
area); n plan mai nti pentru triunghi
si apoi pentru poligon oarecare, iar n spatiu pentru tetraedru si pe urm
a pentru un
poliedru arbitrar, cu demonstratii ntre care exista analogii.
Propozitia 1. Fie ABC un triunghi si i, j, k versori perpendiculari pe dreptele
BC, CA, respectiv AB, ndreptati spre exteriorul triunghiului. Cu notatiile uzuale,
are loc egalitatea ai + bj + ck = 0.
Demonstatia I. Notam S = ai + bj + ck; avem ca S ai = a2 + ab i j + ac i k.
b obtinem ca i j = cos C si,
Deoarece unghiul dintre i si j are masura 180 m(C),
procednd la fel, analoagele. Folosind identitatea b cos C + c cos B = a, g
asim c
a
S ai = a2 ab cos C ac cos B = a (a b cos C c cos B) = 0.

Similar, S bj = 0, prin urmare S este ortogonal pe doi vectori necoliniari, deci S = 0.


Demonstratia II. Construim, ca n figur
a,
F
reprezentanti cu originea n A ai vectorilor bj si ck,

fie acestia AE, respectiv AD; fie nca F al patrulea


D
vrf al paralelogramului construit pe acesti vectori. Se
observ
a atunci c
a 4ABC 4EF A (L.U.L.), deci
\
AF = BC = a si F[
AE \
ACB. De aici, m(M
AC) =
ck

[ m(EAF
[ ) = 90 m(ACB),
\
E
= 180 m(CAE)
A

\
adic
a m(AM C) = 90 , unde {M } = AF BC.
bj

Urmeaza ca AF este ortogonal pe BC, are lungimea

a si sens opus lui i, deci AF = ai. Pe de alt


a parte,

AF = AE + AD = bj + ck, de unde concluzia.
Sa observam ca putem considera ca teorema ariciu- B
M C
lui a fost demonstrata cu prima metoda (sau cu alta,
vom vedea c
a mai exist
a); atunci relatia ai + bj + ck = 0 conduce, avnd n vedere

figura de mai sus, la AF = ai, i.e. AF BC, AF = BC, plus o conditie privind
sensul. Prin urmare, putem afirma c
a, din punct de vedere logic, teorema ariciului
pentru triunghi este echivalent
a cu urm
atorul enunt (pb. 45, pg. 49 din [2]):
Problema 1. Se considera 4ABC pe ale carui laturi [AB] si [AC] se construiesc n exterior patratele ABGD si ACKE. Daca O este mijlocul lui DE, atunci
AO = BC/2 si AO BC.
Altfel spus, mediana din A n 4ADE este n
altime n 4ABC; atentie c
a si invers!
\ ACB
\
S
i nca un amanunt: nu trebuie ignorat cazul n care unul din unghiurile ABC,
este drept sau obtuz.
1

Profesor, Colegiul National "Gh. Rosca Codreanu", Brlad

33

Propozitia 2. Fie A1 A2 . . . An un poligon cu laturile de lungimi A1 A2 = a1 ,


A2 A3 = a2 , . . . , An A1 = an . Pentru fiecare k = 1, n, pe latura de lungime ak se
construieste un versor ik orientat spre exteriorul poligonului;
atunci
a1 i1 + a2 i2 + + an in = 0.
Demonstratie. S
a remarc
am c
a, desi poate s
a
a4 i4
a3 i3
nu fie convex, se subntelege ca poligonul nu trebuie
sa aiba autointersectii; va convingeti usor ca pentru o
A4
"fundita" format
a cu dou
a laturi opuse ale unui dreptunghi si cu diagonalele sale, proprietatea nu are loc
A5
(asta daca reusiti sa stabiliti care este interiorul si care
A3
este exteriorul ei!).
mp
artim poligonul n triunghiuri cu interioarele
A2
disjuncte, prin diagonale care nu se intersecteaz
a. a5 i5 A1
a2 i2
Aplicam apoi Propozitia 1 fiecarui triunghi, nsumam
relatiile obtinute si concluzia urmeaza daca tinem
a1 i1
seama de faptul c
a pe laturile comune pentru cte dou
a
triunghiuri (diagonale ale poligonului!) sunt construiti cte doi vectori cu suma nul
a.
Binenteles, demonstratia poate capata si o forma mai tehnica, utiliznd metoda
inductiei matematice; l
as
am acest demers n seama cititorului. S
a spunem c
a am dat
aceste demonstratii deoarece n cazul poliedrelor se va observa o temeinic
a analogie
n argumentare. Cazul plan poate fi rezolvat mult mai simplu, chiar n urm
atoarea
forma mai generala:
Propozitia 20 . Fie A1 A2 . . . An un poligon si v1 , v2 , . . . , vn vectori n planul sau,
orientati catre exteriorul poligonului, nct, pentru fiecare k = 1, n, vk are lungimea

ct [Ak Ak+1 ] si formeaza un acelasi unghi cu Ak Ak+1 . Atunci v1 +v2 + +vn = 0.


Demonstratie (aflata de autor de la prof. Marian Tetiva, Brlad). Sa ob

serv
am c
a v1 , v2 , . . . , vn se obtin din A1 A2 , A2 A3 , . . . , respectiv An A1 printr-o

rotatie de acelasi unghi . Cum A1 A2 + A2 A3 + + An A1 = 0, la fel si v1 + v2 +


+ + vn = 0.
Pentru teorema ariciului n spatiu avem nevoie de urm
atoarea
Lem
a. Fie A1 A2 A3 A4 un tetraedru; notam cu Sk aria fetei opuse vrfului Ak si
cu hk unghiul fetelor de arii Sh si Sk , format spre interiorul tetraedrului. Atunci
are loc egalitatea S1 = S2 cos 12 + S3 cos 13 + S4 cos 14 .
Demonstratie. Consider
am nti c
a A1 se proiecteaz
a pe planul (A2 A3 A4 ) n
punctul H interior 4A2 A3 A4 . Atunci SHA3 A4 = S2 cos 12 , SHA2 A4 = S3 cos 13 ,
SHA2 A3 = S4 cos 14 si SHA3 A4 + SHA2 A4 + SHA2 A3 = S1 , de unde concluzia. Rationamentul este analog n cazul n care H
/ Int A2 A3 A4 .
S
a observ
am analogia cu egalitatea b cos C + c cos B = a din cazul triunghiului.
Propozitia 3. Cu notatiile din lema, fie versorii ik , k = 1, 4, ortogonali respectiv
pe fetele de arii Sk si orientati spre exteriorul tetraedrului. Atunci
S1 i1 + S2 i2 + S3 i3 + S4 i4 = 0.
Demonstratia pe care o d
am este dup
a [3] si decurge la fel cu aceea a Propozitiei 1. Notam asadar S = S1 i1 + S2 i2 + S3 i3 + S4 i4 si, folosind Lema, obtinem
34

c
a

S S1 i1 = S12 S1 S2 cos 12 S1 S3 cos 13 S1 S4 cos 14 = 0


si nc
a trei relatii analoage. Fiind ortogonal pe trei vectori necoplanari, vectorul S
este n mod necesar 0.
Propozitia 4. Fie un poliedru convex cu ariile fetelor S1 , S2 , . . . , Sn ( n 4).
Pe planul fetei de arie Sk se construieste versorul ik perpendicular, orientat spre
exteriorul poliedrului. Are loc relatia S1 i1 + S2 i2 + + Sn in = 0.
Demonstratie. Partitionam poliedrul n tetraedre cu interioarele disjuncte, oricare dou
a tetraedre avnd n comun cel mult o fata. Pentru fiecare tetraedru construim vectorii perpendiculari pe planele fetelor, spre exterior, de lungimi egale cu
ariile fetelor respective. Aplicam pentru fiecare tetraedru Propozitia 3 si tinem seama
ca pe fiecare fata a tetraedrelor care nu este fata a poliedrului initial sunt construiti
doi vectori care se anuleaz
a reciproc.
O alta demonstratie a teoremei ariciului pentru tetraedre poate fi gasita n [3]
si utilizeaz
a produsul vectorial, iar o frumoas
a demonstratie n cazul general apare
n [4], bazat
a pe ideea c
a suma proiectiilor vectorilor pe orice dreapt
a este nul
a
(pb. M119 din Kvant). n spatiu, o demonstratie analoag
a cu cea a Propozitiei 20
nu se poate gasi.
Folosind Propozitia 3, putem obtine valabilitatea urm
atorului enunt (de altfel,
credem c
a ele sunt echivalente), care reprezint
a extinderea n spatiu a Problemei 1:
Problema 2. Cu notatiile din lema, construim punctul B2 de cealalta parte a
planului (A1 A2 A3 ) dect A2 si astfel nct A1 B2 (A1 A3 A4 ), iar A1 B2 este numeric egal cu S2 . Analog construim B3 si B4 , apoi paralelipipedul A1 B2 B30 B4 B3 B40 A01 B20

pe vectorii A1 B2 , A1 B3 , A1 B4 . Atunci A1 A01 (A2 A3 A4 ) si A1 A01 = S1 .
n ncheiere, propunem rezolvarea urm
atoarelor probleme:
1. Deduceti, cu teorema ariciului, ca fiecare latura a unui poligon este mai mica
dect suma celorlaltor laturi; generalizare n spatiu. Este reciproca adev
arat
a?
2. Demonstrati teoremele cosinusurilor pentru tetraedru:
S12 = S22 + S32 + S42 2S2 S3 cos 23 2S2 S4 cos 24 2S3 S4 cos 34 ;
S12 + S22 2S1 S2 cos 12 = S32 + S42 2S3 S4 cos 34 .

Ar
atati c
a aceste egalit
ati sunt valabile si dac
a S1 , S2 , S3 , S4 sunt lungimile laturilor
unui patrulater (redefinind hk ).
3. Rezolvati Problema 2 sintetic (sau pe orice alt
a cale) si obtineti astfel echivalenta logic
a dintre Problema 2 si Propozitia 3.
Bibliografie
1. D. Brnzei, S. Anita, C. Cocea - Planul si spatiul euclidian, Ed. Academiei,
Bucuresti, 1986.
2. J. Hadamard - Lectii de geometrie elementara. Geometrie plana, Ed. Tehnica,
Bucuresti, 1960.
3. M. Miculita - Introducere n geometria tetraedrului, Ed. Mined, Iasi, 1994.
4. Probleme din revista KVANT (traduse si selectate de H. Banea), E. D. P., Bucuresti,
1983.
35

Num
arul polinoamelor ireductibile din Zp [X]
1 , Lucian-Georges LADUNC

2
Elena ROGOJINA
A
Problema 3 propus
a la Berkeley Preliminary Exams, Fall 1985, cere determinarea
numarului polinoamelor ireductibile de grad 3 si coeficientul dominant b
1 din Z5 [X]
([2], p. 230). Mai general, Problema 150 din G. M. (seria A), nr. 1/2003, cere determinarea numarului polinoamelor ireductibile de grad 3 din Zp [X], p prim (Gabriel
Popa, [3]).n nota de fata vom urm
ari rezolvarea acestor probleme si vom ar
ata cum
poate fi aflat numarul polinoamelor ireductibile de grad n din Zp [X], p prim.
n
P
S
a observ
am mai nti c
a polinomul
ak X k este ireductibil peste Zp dac
a si
k=0
n1
P
k
numai dac
a polinomul X k +
a1
a an 6= b
0
n ak X este ireductibil (unde evident c
k=0

este inversabil peste corpul Zp ); este deci suficient sa gasim numarul polinoamelor
normate (monice) ireductibile, prin nmultirea acestui num
ar cu p1 aflnd r
aspunsul
la problem
a.
Numarul polinoamelor de forma f = X 3 + aX 2 + bX + c, a, b, c Zp , este p3 .
Ca n [2], sa vedem nti cte dintre aceste polinoame sunt reductibile. Polinoamele
reductibile din Zp sunt
fie de forma
f = (X i) (X j) (X k), i, j, k Zp , fie de
forma f = (X i) X 2 + mX + n , i, m, n Zp si X 2 + mX + n ireductibil peste
Zp . Prima dificultate care trebuie depasita n trecerea de la p = 5 la cazul general
este num
ararea polinoamelor de primul tip: observ
am c
a num
arul lor este egal cu
num
arul tipurilor de cuvinte de lungime 3 formate cu elementele multimii Zp , care
este dat de num
arul combin
arilor cu repetitie
p (p + 1) (p + 2)
3
Cp3 = Cp+2
=
.
6
a. Dac
a
Afl
am acum cte polinoame normate ireductibile de grad 2 peste Zp exist
p = 2, n Z2 [X] exista patru polinoame de grad 2, dintre care singurul ireductibil este
X2 + X + b
1. Fie p 3 prim; n Zp [X] exista p2 polinoame de forma X 2 + mX + n,
4n este p
atratul unui element
dintre care sunt reductibile cele pentru care = m2 b
p+1
a Zp . Numarul acestor "patrate perfecte" este
. ntr-adevar (v., de exemplu,
2

0}Im f ,
[2], Problema 12, Spring 1977), Q (p) = {b
0}{x Zp | x = a2 , a Zp } = {b

2
1
unde f : Zp Zp , f (a) = a este morfism de grupuri. Deoarece p 3, ecuatia a2 = b
Z / Ker f ,
are exact doua solutii, b
1 si p[
1, deci Ker f = {b
1, p[
1} si cum Im f =
p
p1
p1
p+1
atunci Card (Im f ) =
, prin urmare Card Q (p) =
+1=
. Num
arul
2
2
2
p (p + 1)
perechilor (m, n) Z2p pentru care este "patrat perfect" este
(pentru
2
p+1
fiecare valoare dat
a lui m, n ia
valori, dat fiind faptul c
ab
4 este inversabil n
2
1
2

Student
a, Universitatea "Ovidius", Constanta
Profesor, Liceul de Informatic
a "Gr. C. Moisil", Iasi

36

Zp , p fiind impar). Prin urmare, num


arul polinoamelor normate ireductibile de grad
p
(p
+
1)
p
(p

1)
2 este p2
=
, relatie adev
arat
a si pentru p = 2.
2
2
n final, numarul polinoamelor normate ireductibile de grad 3 din Zp [X] este
p (p + 1) (p + 2)
p (p 1)
p (p 1) (p + 1)
p3
p
=
.
6
2
3
Evident, aceasta metoda de numarare este sortita esecului n cazul general al
polinoamelor ireductibile de grad n din Zp [X], dat fiind faptul ca exista nu numai
cele dou
a tipuri de polinoame reductibile din cazul n = 3. Modalitatea de rezolvare a
problemei poate fi urm
arit
a detaliat n [1], pp. 188-191 si foloseste rezultate profunde
de teoria corpurilor; vom prezenta mai jos numai desfasurarea ideilor.
Pentru un polinom normat ireductibil de gradul d din Zp [X], are loc echivalenta
n

f | X p X d | n.
n
Se arata ca polinomul X p X nu are radacini multiple, deci n descompunerea sa ca
produs de polinoame normate ireductibile nu exist
a factori care s
a se repete. Conform
echivalentei anuntate, aceast
a descompunere cuprinde ca factori toate P
polinoamele
normate ireductibile din Zp [X] al c
aror grad divide pe n, de unde pn =
d (d, p);
d|n

am notat cu (k, p) numarul polinoamelor normate ireductibile de grad k din Zp [X].


Aplicatia : N {1, 0, 1}, (1) = 1, (n) = (1)r daca n este produs de
r numere prime distincte si (n) = 0 dac
a n > 1 si n nu este liber de p
atrate se
numeste functia lui Mbius. Aceast
a functie aritmetic
a are proprietatea c
a pentru
orice aplicatie f : N C, avem ca
n
X n
X

(d) F
f (n) =
F (d) =
,
d
d
d|n
d|n
P
f (d); relatia de mai sus poarta numele de formula
unde f : N C, F (n) =
d|n

de inversiune a lui Mbius. Aplicnd aceasta formula functiei f : N C, f (n) =


= n (n, p), avem c
a F (n) = pn si atunci
1 X n d
1X
(n, p) =

(d) pn/d .
p =
n
d
n
d|n

n|d

Asadar, numarul polinoamelor normate ireductibile de grad n din Zp [X] este


1P
(d) pn/d . n cazul particular n = 3, avem
n n|d

p (p 1) (p + 1)
1
1 3
1X
(d) p3/d =
(1) p3 + (3) p =
p p =
,
(3, p) =
3
3
3
3
d|3

adic
a reg
asim rezultatul problemei [3].

Bibliografie
1. T. Albu, I. D. Ion - Itinerar elementar n algebra superioara, Ed. ALL, Bucuresti,
1997.
2. C. Costara, D. Popa - Berkeley Preliminary Exams, Ed. Ex Ponto, Constanta,
2000.
3. G. Popa - Problema 150, G. M. (seria A), nr. 1/2003.
37

Functiile lui Smarandache proprieta


ti elementare
Prezenta Not
a este rezultatul unei selectii din materialul
trimis Redactiei de c
atre Minh Perez, Rehoboth, NM, SUA.
Functia Smarandache apare n literatura matematic
a cu mult timp n urm
a (date
istorice pot fi g
asite n J. Sndor - The Smarandache function introduced more
than 80 years ago! Octogon Mathematical Magazine, 9 (2001), no. 2, 920921).
F. Smarandache redescopera si cerceteaza aceasta functie si are meritul de a fi
generat un curent de preocup
ari n privinta acesteia.
Definim functia Smarandache S (n) pe multimea N prin: S (1) = 1, iar pentru
n 2, S (n) este cel mai mic numar natural pentru care S (n)! se divide cu n.

n cele ce urmeaz
a, sunt adunate o serie de propriet
ati ale functiei Smarandache
si ale unor generalizari ale ei.
1. Daca p este prim, atunci S (p) = p. Reciproca este adevarata? (Anthony
Begay)
Solutie. Dac
a p este prim, atunci r! nu este divizibil cu p pentru r < p. Pe de
alt
a parte, p! se divide cu p si, cum este cel mai mic num
ar cu aceast
a proprietate,
rezult
a c
a S (p) = p. Reciproca nu este adev
arat
a: l
asnd la o parte cazul S (1) = 1,
cu 1 neprim, avem contraexemplul S (4) = 4. Pot fi gasite alte contraexemple?
2. Daca n este liber de patrate, iar p este cel mai mare factor prim din descompunerea sa, atunci S (n) = p. (Leonardo Motta)
Solutie. Fie n = a b . . . p descompunerea n factori primi a lui n, unde
a < b < < p. Atunci p! contine n scrierea sa toti divizorii primi ai lui n, deci
S (n) p. Pentru r < p, observam ca r! nu se divide cu p, deci S (n) p. Ramne
ca S (n) = p, ceea ce doream.
n
n
n particular, S (n) = p =

(deoarece n scrierea n = p q, avem q 2)


q
2
(T. Yau)
3. Daca p este prim, atunci S (pp ) = p2 . (Alecu Stuparu)
Solutie. Deoarece S (pp ) trebuie s
a se divid
a cu p, iar p este prim, rezult
a c
a
S (pp ) trebuie s
a fie un multiplu nenul al lui p, fie acesta kp . Mai mult, fiindc
a S (pp )
se divide cu pp , trebuie sa avem kp p (se vede ca p (p 1)! se divide cu pp1 , dar
nu si cu pp ). Atunci p2 este cel mai mic numar al carui factorial se divide cu pp , de
unde concluzia.
Asem
an
ator pot fi definite a doua si a treia functie Smarandache: S2 (n) este cel
mai mic num
ar natural pentru care S2 (n)!! se divide cu n (unde m!! este produsul
numerelor nenule cel mult egale cu m, de aceeasi paritate ca si m); S3 (n) este cel
mai mic numar natural pentru care S3 (n)!!! se divide cu n (unde m!!! este produsul
numerelor nenule cel mult egale cu m, care dau acelasi rest ca si m la mp
artirea cu 3).
4. Daca n 3 este un numar par liber de patrate, iar p este cel mai mare factor
prim din descompunerea sa, atunci S2 (n) = 2p. (Gilbert Johnson)
Solutie. Fie n = 2 a b . . . p, cu 2 < a < b < < p numere prime. Dac
a
S2 (n) = 2p k, unde 1 k < 2p, atunci (2p k)!! nu se divide prin p. Evident ca
38

(2p)!! = 2 4 . . . (2a) . . . (2b) . . . (2p) se divide la n si, cum este cel mai mic
numar cu aceasta proprietate, urmeaza concluzia.

5. Fie p numar prim impar; sa se determine S2 pk+2 , unde p = 2k + 1. (Ivan


Godunov)

Solutie. Ca n rezolvarea problemei 3, se arat


a c
a S2 pk+2 = p2 .
6. Daca n este multiplu nenul al lui 3, atunci S3 (n) este tot multiplu de 3.
(K. L. Ramsharan)
Solutie. Fie m = S3 (n); daca m nu ar fi multiplu de 3, atunci m!!! = m (m 3)
(m 6) . . . nu s-ar divide nici el cu 3 si atunci m!!! nu se divide cu n. R
amne c
a
S3 (n) este multiplu de 3.
7. Sa se rezolve ecuatia diofantica S2 (x) = p, unde p este un numar prim.
(Gilbert Johnson)
Solutie. Pentru p prim fixat, vom determina numarul de numere naturale x
astfel nct S2 (x) = p. Avem c
a p!! se divide cu x, iar p este cel mai mic ntreg cu
aceast
a proprietate. Cum p este prim, x trebuie s
a fie multiplu de p.
a) Daca p = 2, atunci x = 2.
b) Daca p > 2, atunci x este produsul dintre p si o combinatie de 0, 1 sau mai multi
p3
dintre factorii 3, 5, . . . , p 2. Notnd k =
, avem Ck0 = 1 solutie cu un singur
2
factor (x = p), Ck1 solutii cu doi factori (x = p 3, p 5, . . . , p (p 2)), Ck2 solutii cu
trei factori etc. Num
arul total de solutii este cel mult egal cu Ck0 +Ck1 + +Ckk = 2k .

Recreaii matematice
Solutiile problemelor enuntate la paginile 15 si 26.
1. nl
aturnd segmentele marcate se obtine o figur
a format
a
din trei p
atrate.
2. Cu o t
aietur
a f
acut
a n veriga a treia obtinem trei buc
ati de lant formate din
o veriga, doua verigi si patru verigi. n prima zi calatorul plateste o veriga; a doua zi
da bucata formata din doua verigi si ia napoi o veriga; a treia zi da hangiului veriga
izolat
a; a patra zi d
a bucata din patru verigi si primeste ca rest celelalte trei verigi
de la hangiu; a cincea zi d
a iar
asi veriga izolat
a; a sasea zi d
a bucata din dou
a verigi
si ia veriga napoi; n sfrsit, n a saptea zi da hangiului si veriga ramasa.
Asadar, este suficient
a o singur
a n lant pentru a putea fi f
acut
a plata convenit
a
zilnic.
3. Iat
a interpretarea corect
a a calculului efectuat: dac
a exist
a o solutie n R a
ecuatiei x2 x + 1 = 0, aceasta poate fi 1. Egalitatea 3 = 0, obtinuta punnd
x = 1 n ecuatie, arata ca 1 nu este solutie si, deci, ecuatia nu are solutii n R.
39

n leg
atur
a cu o problem
a de aritmetic
a
propus
a la BAC99.
si Ioan GHIT
1
Roman
ta GHIT
A
A
n august 1999, la bacalaureat, profilul pedagogic, a fost propus
a problema:
ntr-un depozit erau 185 t carbuni, iar n altul 237 t. Din primul depozit se iau
cte 15 t carbuni pe zi, iar din al doilea cte 18 t pe zi. Dupa cte zile a ramas n
1
depozitul al doilea de 1 ori mai mult carbune dact n primul?
2
2
Solutie ([1]). Dac
a n cel de-al doilea depozit ar fi din 237 t (adic
a 158 t) si
3
2
a 12 t), dup
a num
arul de zile cerut cantit
atile
din el s-ar scoate zilnic din 18 t (adic
3
ar fi egale. Diferenta de 185 t 158 t = 27 t este anulata de diferenta de 3 t dintre
cantit
atile scoase zilnic n 9 zile.
2
Consider
am c
a aceast
a solutie necesit
a unele clarific
ari. nti,
reprezint
a in3
1
versul lui 1 . Apoi, diferenta de 3 t nu este cea dintre 18 si 15, ci dintre 15 si 12;
2
coincidenta ntre diferentele de tone scoase zilnic este una nefericita.
a n depozitul I se scade zilnic cte a, iar dintr-o
Practic, dintr-o cantitate x aflat
cantitate y aflat
a n depozitul II se scade zilnic b (x < y, a < b). Ne intereseaz
a
m
> 1 ori mai
num
arul p de zile dup
a care n depozitul II r
amne o cantitate de
n
n
n
mare dect cea ramasa n depozitul I (a > b, x > y).
m
m
n
n
Daca n depozitul II ar fi y si s-ar scoate zilnic b, dupa cele p zile ar ramne
m
m
n
n
n
cantitatea C = y p b =
(y pb), egal
a cu cea r
amas
a n primul depozit
m
m
m
m
n
(deoarece y pb =
(x pa) din ipoteza problemei). Diferenta y
este anulat
a
n
my
n
n
n
de diferenta a b n x y : a b zile.
m
m
m
Putem formula probleme asem
an
atoare, cu conditia de a alege datele n asa fel
nct s
a fim condusi la operatii cu numere naturale.
Probleme propuse.
1. ntr-o tab
ar
a scolar
a sunt 792 elevi, iar n alta 531. Din fiecare tab
ar
a pleac
a
din 5 n 5 minute grupuri de cte 36 si respectiv 23 elevi n drumetii. Dup
a cte
minute n prima tabara se vor afla de 9/7 ori mai multi elevi dect ntr-a doua?
2. n doua cosuri se gasesc 405 si respectiv 800 bomboane. n fiecare zi se vnd
cte 15, respectiv 32 bomboane. Dup
a cte zile n cosul al doilea vor fi cu 60% mai
multe bomboane dect n primul?
Bibliografie
1. Gh. Andrei si colab. - Admiterea 1999, Ed. GIL, Zal
au, 1999.
1

Profesori, Col. Nat. "I. M. Clain", Blaj

40

Concursul "Recreatii Matematice"


Editia a III-a, Iasi, 28 August 2003
Clasa a VII-a
a
b
+
= 1.
b+1 a+1
Alexandru Negrescu, Botosani (RecMat-2/2003)
2. Un triunghi are doua mediane perpendiculare, iar suma lungimilor lor constant
a. S
a se determine maximul ariei triunghiului.
Mihai Gavrilut, Roman
3. Fie XOY un unghi oarecare si P un punct n interiorul lui. Se considera
punctele A, B OX cu A (OB) si C, D OY cu C (OD) astfel nct triunghiurile P AB si P CD s
a fie echilaterale. Ar
atati c
a, dac
a dreptele OP , AD, BC sunt
concurente, atunci P se afla pe bisectoarea unghiului XOY .
Temistocle Brsan, Iasi (RecMat-1/2003)
1. Rezolvati n N N ecuatia

Clasa a VIII-a
1. Fie n N fixat. Aratati ca exista o infinitate de numere x, y, z Z astfel nct
x2n + y 2n + z 2n = x2n+1 + y 2n+1 + z 2n+1 .
Lucian Tutescu, Craiova (RecMat-1/2003)
2. Gasiti ntregii pozitivi n, x1 , x2 , . . . , xn astfel nct x1 + x2 + + xn = 2003
si produsul x1 x2 . . . xn sa fie maxim.
Agnes Constantinescu, Harghita
0 0 0 0
3. Fie ABCDA B C D un cub. Cubul este patat cu cafea pe mai putin de
jumatate din suprafata lui totala. Aratati ca exista doua puncte pe suprafata cubului
coliniare cu centrul cubului care nu sunt p
atate cu cafea.
Valerica Benta, Iasi si Mugur Rosca, Craiova

Clasa a IX-a

2
1
1
=
1. Rezolvati n R ecuatia q
+ q
, unde [x] este
[x] [x + 2]
3
3
3
2 [x]
3 [x + 1] [x]
partea ntreag
a a lui x.
Daniel Jinga, Pitesti (RecMat-1/2003)
2. Fie f : R R o functie care satisface

2
n + 3n + 3 f (n + 2) 2 n2 + n 1 f (n + 1) + n2 n + 1 f (n) = 0,

pentru orice n natural. S


tiind ca f (0) = 0 si f (1) = 1, calculati f (2003).
Andrei Nedelcu, Iasi
3. Fie p
atratul ABCD, E mijlocul lui (AB), M (CD), N (AD) astfel nct
BM k EN . S
a se arate c
a M N este tangenta cercului C (S, r) nscris n p
atrat.
Nicu Miron, Iasi

Clasa a X-a
1. Fie a, b (0, 1) (1, ) si functia injectiv
a f : (0, ) R astfel nct functia
g : R R, g (x) = f (ax ) + f (bx ) este constanta. Sa se arate ca ab = 1 si ca exista
41

functii f care satisfac ipotezele problemei.


Dan Popescu, Suceava (RecMat-1/2003)
sin + i
2. Sa se afle locul geometric al imaginilor numarului complex z =
,
sin i
(0, ).
Mihai Gavrilut, Roman
3. Un triunghi de arie S se proiecteaz
a pe trei plane perpendiculare dou
a cte
,
S
,
respectiv
S
,
s
a
se
demonstreze
c
a
dou
a. Dac
a ariile proiec
t
iilor
sunt
S
1
2
3

S S1 + S2 + S3 < S 3.
Gheorghe Iurea, Iasi

Clasa a XI-a
1. Fie D, M dou
a matrice nesingulare de ordin n, D diagonal
a, iar M triunghiular
a. Dac
a D = t M DM , s
a se arate c
a M este tot o matrice diagonal
a, avnd 1
pe diagonala principala.
Adrian Corduneanu, Iasi (RecMat-1/2003)
2. Fie (xn )n1 , (yn )n1 dou
a siruri de numere naturale mai mari ca 1. S
a se
xn yn
pxn pyn
arate c
a lim
= 0 lim
= 0, unde pn este al n-lea num
ar prim.
n
n
yn
pyn
Gabriel Mrsanu, Iasi
3. n tetraedrul ABCD se consider
a notatia (ab) = m [ (ABC; ABD)], corespunz
atoare muchiei AB si analoagele, corespunz
atoare la celelalte muchii. Ar
atati c
a

1
cos (cd) cos (bd) cos (bc)

cos (cd)
1
cos (ad) cos (ac)

= 0.
cos (bd) cos (ad)
1
cos (ab)

cos (bc) cos (ac) cos (ab)


1

Silviu Boga, Suceava


4. O pat
a de ulei curge pe un ru. La un moment dat ea intersecteaz
a umbra
unui fir de telegraf. Sa se demonstreze ca exista un moment n care umbra firului
mparte pata de ulei n doua portiuni de aceeasi arie.
Vlad Martinusi, Iasi

Clasa a IX-a (BARAJ)


1. Determinati functiile f : R R care verific
a egalitatea
3

2
xf x + x + 1 + f x + 3x 4x + 3 = x + 1,

x R.

Silviu Boga, Suceava


2

2.
Se
dau
mul
t
imile:
A
=
x
+
x
|
x

Z
,
B
=
x
+
x
|
x

Z
, C =
4

3
2
= x + x + x + x | x Z , D = 2x | x Z . Determinati multimile A C,
B D.
Andrei Nedelcu, Iasi (RecMat-2/2002)

42

Concursul interjudetean "Octav Onicescu"1


Editia a VII-a, 31 oct. - 2 nov. 2003, Botosani
Aceast
a editie a Concursului de matematic
a "Octav Onicescu" a cunoscut o participare numeroas
a si entuziast
a, antrennd elevi din 5 judete: Botosani, Iasi, Suceava,
Vaslui si Vrancea.
Ceea ce particularizeaz
a n mod deosebit acest concurs este faptul c
a se propun spre
rezolvare aceleasi subiecte pentru toti participantii de la clasa a IX-a pn
a la clasa a XII-a.
Subiectele propuse nu sunt axate pe materia studiat
a de fiecare elev la nivelul s
au de studiu,
ci ncearc
a s
a pun
a n valoare abilit
atile matematice pure ale concurentilor.
Deschiderea festiv
a a concursului si premierea s-au desf
asurat n Aula Magna a C. N.
"A. T. Laurian" din Botosani, iar al
aturi de elevi si profesori au participat si autorit
atile
locale. De partea organizatoric
a s-a ocupat I. S. J. Botosani si C. N. "A. T. Laurian".
Sarcina elabor
arii subiectului de concurs a revenit, ca n fiecare an, domnilor profesori Adrian Botan si Adrian Panaete, iar misiunea corect
arii lucr
arilor scrise, membrilor catedrei de matematic
a de la C. N. "A. T. Laurian". Presedintele comisiei a fost
prof. univ. dr. Eugen Popa de la Facultatea de Matematic
a, Universitatea "Al. I. Cuza"
din Iasi.

Public
am n continuare problemele propuse concurentilor si lista premiatilor:
1. Fie a1 , a2 , a3 , . . . , a2003 numerele 1, 2, 3, . . . , 2003 n alta ordine. Aratati
ca macar doua din numerele |a1 1|, |a2 2|, . . . , |a2003 2003| sunt egale. (20p)
2. De pe o tabla de sah 7 7 scot un patrat; aratati ca patratele ramase:
a) nu pot fi acoperite cu 24 de dominouri 1 2 daca patratul scos e A2;
b) pot fi acoperite cu 24 de dominouri dac
a p
atratul scos e D4 si indicati o
acoperire cu num
ar minim de dominouri orizontale (justificare).
(20p)
n
3. Dac
a n este natural, g
asiti restul mp
artirii lui 10 prin 999 si ar
atati c
a un
num
ar natural divizibil cu 999 are m
acar 3 cifre nenule. Cte numere cu cel mult 16
cifre fiecare au fix 3 cifre nenule si se divid cu 999?
(30p)
4. Cte p
atrate ale unei table de sah 340 121 sunt t
aiate n interior de una din
diagonalele tablei? Dar pentru o tabla 340 120?
(30p)
5. Ali Baba si cei 40 de hoti stau n cerc n jurul focului si vor s
a mpart
a n
mod egal 4100 de galbeni care initial se afla mpartiti la ntmplare la ctiva dintre
ei (posibil la unul singur). Ali Baba bate din palme si la comanda lui fiecare din cei
41 d
a un galben vecinului din stnga sa, dac
a acesta are mai putin dect el (dac
a
vecinul are egal sau mai mult nu primeste nimic!). Dac
a nu au realizat egalitatea,
Ali Baba bate din palme din nou etc. Justificati ca dupa un timp sumele se egaleaza
(toti 100 de galbeni).
(30p)
Premiatii sunt: premiul I - Chiril
a Cezar (C. N. "M. Eminescu", Botosani),
premiul II - Istrate Carmen Maria (C. N. "Unirea", Focsani), premiul III Pachitariu Marius (Colegiul National Iasi). Au fost acordate 21 mentiuni.
1 Selectiuni din materialul trimis redact iei de c
atre elevul Alexandru Negrescu si prof. Liliana
Tomita
, C. N. "A. T. Laurian", Botosani

43

Concurs de admitere 2003, Iasi


Facultatea de Informatic
a, Universitatea "Al. I. Cuza"
Algebr
a
I. 1. Se da matricea A M3 (R),
unde M
3 (R) este inelul matricelor patratice
0 0 1
de ordin 3 cu elemente reale, A = 1 0 0. S
a se arate c
a A3 = I3 si c
a are loc
0 1 0
2

relatia (A I3 ) A + A + I3 = 0.
2. Fie S3 o permutare din grupul simetric de grad 3, astfel nct 2 = e
(e noteaz
a permutarea identic
a). Demonstrati c
a exist
a k {1, 2, 3} astfel nct
(k) = k.
1
3. Demonstrati ca polinomul P = X 3 + X + 1 este ireductibil n Q [X].
2
II. 1. Fie G un grup cu n elemente, n N . Aratati ca n orice coloana a tablei
operatiei lui G apar n elemente distincte.
2. Fie (Z, +, ) inelul numerelor ntregi. Determinati toate morfismele de inele
f : Z Z.
3. Fie (C, +, ) corpul numerelor complexe. Sa se arate ca f : C C definita prin
f (z) = z este izomorfism de corpuri (z noteaza conjugatul numarului complex z).
Analiz
a matematic
a
1 1
1
I. 1. Fie Hn = 1 + + + + , n N . Demonstrati ca sirul (Hn )nN este
2 3
n
nemarginit.
2. Fie f : R R o functie continua si marginita. Demonstrati ca exista x0 R
astfel nct f (x0 ) = x0 .
1
3. Fie f : R \ {1} R, f (x) =
. Sa se calculeze f (n) (0), unde n N ,
x+1
a derivata de ordin n a functiei f .
iar f (n) noteaz
II. Pentru n N consider
am fn : R R, fn (x) = xn + xn1 + + x 1.
f3 (x) f2 (x)
a) Reprezentati grafic functia g : D R, g (x) =
, unde D este
f1 (x)
domeniul maxim de definitie al functiei g.
b) Aratati ca pentru orice n N , ecuatia fn (x) = 0 are o unica solutie reala,
un , n intervalul [0, 1].
c) Demonstrati c
a sirul (un )nN este convergent.
d) S
a se determine lim un .
n

Fac. de Electronic
a si Telecomunicatii, Univ. Tehnic
a "Gh. Asachi"

13
3
a
1. Rangul termenului din dezvoltarea
care l contine pe a4 este
+
3
3
a
a) 8
b) 6
c) 3
d) 4
e) 9
n
k
P
k
k
2. Suma
2 + 3 /6 este egala cu
k=1

1
1
1
1
a) 1 n n b) 2 n+1 n+1
2
3
2
3
3
1
1
e) n
2 2
2 3n

c)

44

3
1
1
+
+
2 2n 2 3n

d)

1
1
+
2n 3n

3. Se consider
a inecuatia (m + 1) e2x + 2 (m + 1) ex + m > 0, x R, unde
m R este un parametru. Valorile lui m pentru care inecuatia este verificata x R
sunt
a) (, 0] b) [0, +) c) [1, 0] d) (0, 1) e) (, 1)
4. Multimea tuturor valorilor m R astfel ca sistemul

mx + y z = 0
x + (m + 1) y + z = 2 + m m2

x 2y mz = 2 + 3m m2
sa fie compatibil este
a) {2} b) {2, 1, 2} c) R \ {2, 1} d) R \ {1} e) R \ {2, 1, 2}
10
5. Numarul complex z, care satisface |z| + z =
este
2i
3
3
1
3
a) 2 + i b) 2 + 5i c) 2i d) + 3i e) + 2i
2
2
2
2
x sin x
a fie finit si diferit de zero.
6. S
a se determine m astfel ca l = lim
m s
x0 (1 cos x)
Sa se precizeze si valoarea lui l.

3
2
2
2
2
3
3
c) m = , l =
b) m = 3, l =
d) m =
,l =
a) m = , l =
4
3
2
3
2
3
3
1
e) m = 2, l =
6
2 ln x 1
este
7. Multimea valorilor functiei f : (0, ) R, f (x) =
2 x2

2
b) (0, ) c) R d) (0, e) e) 1/e ,
a) , 1/e
8. Daca x1 , x2 , . . . , xn sunt radacinile ecuatiei xn + 1 = 0, atunci valoarea sumei
1
1
1
+
+ +
este
1 x1 1 x2
1 xn
a) n/2 b) n2 c) n d) n (n + 1) e) n

Z 2
x3
max x , arctg x dx.
9. Sa se calculeze I =
3
1
1
5
5
2 arctg 3
a) arctg 2 + ln 3 b) + 2 arctg 2 ln 5 c)
12
2
12
5
5
2 arctg 2 + ln 3 e) + arctg 2
d)
11
12
1
1

10. S
a se afle solutia ecuatiei arcsin
+ arcsin
= .
x

1
x
+
1
p
p
p
p

2 p

a) 3 + 5 b) 3 + 5 c) 2 + 5 d) 2 + 5 e) 1 + 5

Fac. de Automatic
a si Calculatoare, Univ. Tehnic
a "Gh. Asachi"
1. S
a se determine parametrul real m astfel nct ecuatia x2 + 2mx + (m + 4) = 0
s
a admit
ad
acinile reale
x si x2 verificnd
< x2 .
a r
! x1 < 1
! 1

1 17
1 + 17
5
3
a) m ,

,
b) m ,
c) m ,
2
2
3
5

1 17 3
d) m e) m
,
2
5
45

2. Aflati num
arul termenilor rationali din dezvoltarea binomial
a
a) 1 b) 2 c) 3 d) 4 e) 5
3. Fie sistemul
x+y+z =1
ax + ay + 2az = b
a, b R, b 6= 0.
2
a x + a2 y + 2a2 z = b2

23

3
7+ 53 .

Care din urm


atoarele afirmatii este fals
a?
a) Dac
a a = 0, sistemul este incompatibil b) Dac
a a = b, sistemul este compatibil nedeterminat c) Exist
a a, b R, b 6= 0 astfel nct sistemul are solutie unic
a
d) Daca a 6= 0 si a 6= b, sistemul este incompatibil e) Daca a = 1 si b 6= 1, atunci
sistemul este incompatibil
4. Fie M = (, 1) (1, ) si legea de compozitie intern
a pe M dat
a prin
x y = 3ax + by + xy, x, y M , unde a, b R, b 6= 0. S
a se afle a si b astfel nct
(M, ) sa fie grup abelian si sa se precizeze simetricul x0 al unui element oarecare
x M.
1
x
x
x
1
a) a = , b = 1, x0 =
b) a = 1, b = 3, x0 =
c) a = , b = 1, x0 =
3
x+1
x+1
3
x+1
1
x
1
1
d) a = 1, b = , x0 =
e) a = , b = 1, x0 =
3
x+1
3
x+1 n
5. Se d
a sirul definit prin relatia xn+1 = xn + (a) , n N , x1 = 0, unde
0 < a < 1. Care din urm
atoarele afirmatii este adev
arat
a:
a) sirul este strict cresc
ator cu limita + b) sirul este strict descresc
ator cu
a
limita c) sirul nu este monoton, dar are limita
d) sirul este strict crescaa+1
tor cu limita 1 e) sirul nu este monoton, deci nu are limita
1
3
5
1
+
+
+ . Num
arul
6. Se d
a f : R\ {0, 2, 4, 6} R, f (x) = +
x x2 x4 x6
punctelor n care graficul functiei intersecteaza axa Ox este
a) 0 b) 1 c) 2 d) 3 e) 4
1
a se precizeze intervalul pentru
7. Fie ecuatia diferential
a y 0 + y = 6x, x > 0. S
x
care y (x) > 0, unde y (x) este solutia care satisface conditia y (1) = 1.
1

a) x (1, 2) b) x 3 2, c) x
,

d)
x

(2,
3)
e)
x

0,
3
3
2
2
8. Se dau triunghiurile ABC si A0 B 0 C 0 ce au centrele de greutate G si G0 . Atunci

vectorul GG0 este egal cu


1
1
2
a) (AA0 + BB 0 + CC 0 ) b) (AA0 + BB 0 + CC 0 ) c) (AB + BC + CA + A0 B 0 +
3
4
3

1
1
B 0 C 0 + C 0 A0 ) d) (AB 0 + BA0 + AC 0 + CA0 + BC 0 + CB 0 ) e) (AB 0 + BC 0 + CA0 )
6
3
9. S
a se determine multimea punctelor din planul complex care sunt imaginile
numerelor z care verific
a ecuatia z 2 z |z| + |z|2 = 0.
a) doua drepte perpendiculare b) un cerc cu centrul n origine c) doua drepte
paralele d) dou
a semidrepte e) dou
a cercuri concentrice
1
1
1

10. Num
arul solutiilor ecuatiei arctg
+ arctg
arctg 2
= este
x1
x+1
x 1
4
a) 1 b) 2 c) 3 d) 4 e) o infinitate
46

Solutiile problemelor propuse n nr. 1 / 2003


Clasele primare
P.44. Un vecin al unui vecin al numarului 81 este egal cu un vecin al unui vecin
al numarului 77. Despre ce numar este vorba?
( Clasa I )
Mihaela Rusu, elev
a, Iasi
Solutie. Acest num
ar trebuie s
a fie mai mare ca 77 si mai mic dect 81. Num
arul
se afl
a n secventa 77 81. Este vorba despre num
arul 79.
P.45. Adunnd trei numere naturale a, b, c obtinem suma 62. Primul numar este
mai mare dect al treilea si mpreuna au suma 12. Care sunt cele trei numere?
( Clasa a II-a)
nv. Maria Racu, Iasi
Solutie. Numarul b = 62 12 = 50. Perechea (a, c) poate fi: (12, 0); (11, 1);
(10, 2); (9, 3); (8, 4) sau (7, 5). Tripletul (a, b, c) poate lua valorile: (12, 50, 0); (11, 50, 1);
(10, 50, 2); (9, 50, 3); (8, 50, 4) sau (7, 50, 5).
P.46. Mihai, Dan si Petru practica fiecare un alt fel de sport si anume: tenis,
fotbal sau volei. Mihai si voleibalistul locuiesc n acelasi bloc. Cel care joaca volei si
cel care joaca fotbal l-au urmarit pe Petru la un meci. Ce sport practica fiecare?
( Clasa a II-a)
Adina Dohotaru, elev
a, Iasi
Solutie. Din textul problemei se deduce c
a Petru nu joac
a volei sau fotbal, deci
el joac
a tenis. Mihai si voleibalistul locuiesc n acelasi bloc. Aceasta nseamn
a c
a
Mihai nu joaca volei. Solutia problemei este: Petru joaca tenis, Mihai joaca fotbal si
Dan joaca volei.
P.47. Diferenta a doua numere este 48. Aceasta diferenta este cu 22 mai mare
dect jumatatea unuia dintre ele. Determinati numerele.
( Clasa a III-a)
nv. Rodica Rotaru, Brlad
Solutie. Fie a b = 48. Avem dou
a cazuri: 1) 48 = b : 2 + 22 de unde obtinem
b = 52 si a = 100. 2) 48 = a : 2 + 22 de unde obtinem a = 52 si b = 4.
P.48. Un agricultor mparte un teren n trei parcele. n fiecare an, fiecare parcela
este cultivata numai cu una din culturile: gru, porumb sau legume. ncepnd cu
anul 2003, agricultorul se hotaraste ca pe fiecare parcela sa fie alta cultura n trei
ani consecutivi.
a) Care este primul an dupa 2003 n care se repeta culturile pe cele trei parcele?
b) Se poate preciza care este ordinea culturilor pe cele trei parcele n anul 2019?
( Clasa a III-a)
Andreea Surugiu, elev
a, Iasi
Solutie. Presupunem c
a n anul 2003 avem ordinea (gru, legume, porumb).
n anul 2004 putem avea (legume, porumb, gru) sau (porumb, gru, legume). n
anul 2005 putem avea (porumb, gru, legume) sau (legume, porumb, gru). n 2006
avem din nou ordinea (gru, legume, porumb). Raspunsul la a) este anul 2006. b)
Ordinea culturilor se mai repet
a n 2009, 2012, 2015, 2018. Nu putem preciza ordinea
culturilor n anul 2019.
P.49. La un moment dat, cernd unei persoane anul nasterii, aceasta raspunde:
"anul acesta mplinesc 25 ani, iar daca as scrie toate numerele ncepnd cu 1 si
terminnd cu anul nasterii si apoi toate numerele ncepnd cu 1 si terminnd cu
47

anul n care ne aflam mi-ar trebui 13710 cifre. n ce an ne aflam cnd am pus
ntrebarea?
( Clasa a III-a)
Prof. C
at
alin - Cristian Budeanu, Iasi
Solutie. Pentru scrierea numerelor de la 1 999 sunt necesare 2889 cifre. Rezult
a
c
a anul nasterii nu poate fi format din trei cifre. ntr-adev
ar, 2 2889 + 25 n < 13710,
n 4. Anul nasterii este de forma abcd. Fie x numarul cifrelor pentru scrierea
numerelor de la 1 la abcd. Transpunnd n ecuatie ceea ce a spus persoana, obtinem:
x + (x + 4 25) = 13710, cu solutia x = 6805. Pentru scrierea numerelor de la 1000
la abcd sunt necesare 6805 2889 = 3916 cifre, ceea ce nseamn
a c
a de la 100 la abcd
sunt 3916 : 4 = 979 numere. nseamna ca anul abcd este 1978. ntrebarea a fost pusa
n anul 1978 + 25 = 2003.
P.50. a) Cte numere trebuie adaugate sirului 1, 2, 4, 5, 7, 8, . . . , 97, 98 pentru a
obtine toate numerele de la 1 la 98?
b) Efectuati 1 + 2 + 4 + 5 + 7 + 8 + + 97 + 98 2 (3 + 4 + 5 + + 34).
( Clasa a IV-a)
Georgiana Ciobanu, elev
a, Iasi
Solutie. a) Lipsesc numerele: 3, 6, 9, . . . 96 care pot fi scrise: 3 1, 3 2, 3 3, . . . ,
3 32. Se observa ca lipsesc 32 numere.
b) Expresia de calculat se poate scrie:
1 + 2 + (4 3) + (5 3) + (7 4) + (8 4) + + (97 34) + (98 34) =
= 1 + 2 + (1 + 2 + 3 + 4 + + 63 + 64) = 3 + 64 65 : 2 = 3 + 2080 = 2083.

P.51. Produsul a doua numere naturale este 913 368. Unul din numere are cifra
unitatilor si cifra zecilor mai mare ca 2 si mai mica dect 8. Daca la acest numar
marim cifra zecilor cu 2 si micsoram cifra unitatilor cu 1, obtinem un produs egal
cu 951 425. Aflati cele doua numere.
( Clasa a IV-a)
nv. Elena Z
arnescu, Iasi
Solutie. Fie a si b numerele c
autate. Obtinem
(a + 20 1) b = 951425 ab + 19b = 951425
913368 + 19b = 951425 b = 2003 a = 913368 : 2003 = 456.
P.52. n trei cutii sunt 212 bile. Din prima cutie se scoate un numar de bile, din
a doua de 2 ori mai mult si nca doua bile, din a treia se scoate ct triplul numarului
de bile scos din a doua cutie. n fiecare cutie ramne un numar de bile egal cu
numarul total al bilelor scos din cele trei cutii la un loc. Cte bile au fost n fiecare
cutie?
( Clasa a IV-a)
nv. Maria Racu, Iasi
Solutie. Not
am cu p num
arul bilelor scos din prima cutie. Rezult
a c
a n fiecare
cutie ramn 9p + 8 bile. Deducem ca n toate cutiile au fost 36p + 32 bile. Asadar,
36p + 32 = 212, de unde p = 5. n cele trei cutii au fost 58, 65, respectiv 89 bile.
P.53. Efectund o singura cntarire, sa se ia 475 g dintr-un kilogram de zahar
utiliznd doua greutati, una de 200 g si cealalta de 150 g.
( Clasa a IV-a)
Prof. Petru Asaftei, Iasi
Solutie. Utiliz
am o balanta cu brate egale. Distribuim kilogramul de zah
ar
si cte una din cele dou
a greut
ati, pe cele dou
a talere, pn
a realiz
am pozitia de
echilibru. Pe fiecare taler vom avea 675 g. Masa cautata este pe talerul n care se
48

afl
a greutatea de 200 g: 675 g 200 g = 475 g zah
ar.

Clasa a V-a
V.36. Fie n un numar impar, iar a1 , a2 , . . . , an , n N numere care mpartite
la n dau cturi distincte si resturi distincte. Aratati ca valoarea minima a sumei
S = a1 + a2 + + an este multiplu de 12.
Dragos Ungureanu, elev, Iasi
Solutie. Conform ipotezei, avem: a1 = nc1 +r1 , a2 = nc2 +r2 , . . . , an = ncn +rn ,
unde {r1 , r2 , . . . , rn } = {0, 1, 2, . . . , n 1}. Astfel, suma
n (n 1)
S = a1 + a2 + + an = n (c1 + c2 + + cn ) +
2
este minim
a dac
a {c1 , c2 , . . . , cn } = {0, 1, 2, . . . , n 1}, deci
n (n 1) n (n 1)
n (n 1) (n + 1)
Smin = n
+
=
.
2
2
2
.
.
Cum n este impar, rezulta ca (n 1) (n + 1) .. 8, deci Smin .. 4. Pe de alta parte,
..
deoarece n, n 1, n + 1 sunt numere consecutive, rezulta ca S
. 3. Prin urmare,
min

Smin este multiplu de 12.

V.37. Comparati fractiile a =

222221
333331
si b =
.
333334
222223

Maria Cojocaru, Iasi


3
1
2
1
si = 1 +
. Cum 3 222221 > 2 333331,
Solutie. Avem = 1 +
a
333331
b
222221
1
1
rezult
a c
a > , deci b > a.
a
b
V.38. Sa se arate ca 2a + 2b + 2c + 2d + 2e 6= 2003, a, b, c, d, e N.
Irina Ispas, student
a, Iasi
Solutie. Presupunem c
a exist
a cinci numere naturale a b c d e astfel
nct
2a + 2b + 2c + 2d + 2e = 2003.
(1)
Daca a 6= 0, atunci termenul din stnga al egalitatii (1) este par si atunci avem o
contradictie. Pentru a = 0, relatia (1) devine: 2b + 2c + 2d + 2e = 2002. Deoarece
210 = 1024, rezult
a c
a numai e ar putea avea, eventual, valoarea 10.
Daca e = 10, atunci 2b + 2c + 2d = 978. n acest caz, daca b, c, d 8, atunci
2b + 2c + 2d 3 256 < 978. Asadar, d = 9 si 2b + 2c = 466, ceea ce nu este posibil.
Dac
a toate numerele b, c, d, e sunt strict mai mici ca 10, se observ
a c
a cel mult
trei numere pot fi 9 (altfel avem 2b + 2c + 2d + 2e 4 512 > 2002) si cel putin trei
trebuie sa fie 9 (deoarece, n caz contrar, avem 2b + 2c + 2d + 2e < 29 + 29 + 28 + 28 <
< 2002). Prin urmare, c = d = e = 9 si atunci 2b = 2002 3 29 = 476, absurd.
V.39. Sa se determine numerele prime p1 < p2 < p3 < p4 astfel nct numerele
p1 + p2 + p3 + p4 , p3 p2 , p4 p3 sa fie, de asemenea, prime.
Petru Minut, Iasi
Solutie. Deoarece p1 + p2 + p3 + p4 este un numar prim mai mare ca 2, rezulta
c
a el este impar si atunci unul dintre numerele p1 , p2 , p3 , p4 trebuie s
a fie par, deci
p1 = 2. Cum p2 , p3 si p4 sunt impare, nseamn
a c
a p3 p2 si p4 p3 sunt pare si
avnd n vedere ca sunt prime, rezulta ca p3 p2 = p4 p3 = 2. De aici, deducem
49

c
a p3 = p2 + 2 si p4 = p2 + 4. Se observ
a c
a p1 = 2, p2 = 3, p3 = 5 si p4 = 7 este
o solutie a problemei (2 + 3 + 5 + 7 = 17 este numar prim). Daca p2 > 3, atunci
p2 = 3k + 1 sau p2 = 3k + 2, k N . n cazul p2 = 3k + 1, avem p3 = 3k + 3 care
nu este prim, iar n cazul p2 = 3k + 2, avem p4 = 3k + 6, care nu este prim. Asadar,
p1 = 2, p2 = 3, p3 = 5, p4 = 7 este singura solutie.
V.40. Este posibila o partitionare a multimii {1, 2, . . . , 12n + 9} n 4n + 3 submultimi disjuncte, fiecare cu cte trei elemente, astfel nct n fiecare submultime un
element sa fie suma celorlaltor doua?
Titu Zvonaru, Bucuresti
Solutia I. Fie {a, b, c} o multime astfel nct a = b + c. De aici, rezult
a c
a
elementele multimii {a, b, c} sunt ori toate pare, ori dou
a impare si unul par. Asadar,
pentru ca sa fie posibila o partitie ca n problema, trebuie ca multimea data sa contina
un num
ar par de numere impare. Deoarece multimea dat
a are 6n + 5 numere impare,
rezult
a c
a partitionarea nu este posibil
a.
Solutia II. S
a presupunem c
a ar fi posibil
a o partitie n conditiile impuse. Atunci,
fiecare din cele 4n + 3 submultimi de trei elemente are suma elementelor egala cu un
num
ar par, deci suma elementelor multimii {1, 2, . . . , 12n + 9} ar trebui s
a fie num
ar
(12n + 10) (12n + 9)
par. Cum, 1 + 2 + + 12n + 9 =
= (6n + 5) (12n + 5), care
2
este un numar impar, rezulta ca partitionarea ceruta nu este posibila.

Clasa a VI-a
VI.36. Fie k N, k 3. Aratati ca printre valorile naturale ale lui n care fac
.
adevarata propozitia n2 + k .. n + k, exista cel putin trei patrate perfecte.
Claudiu S
tefan Popa, Iasi
Solutie. Din n2 + k = n2 k 2 + k2 + k = (n k) (n + k) + k 2 + k, rezult
a c
a

.
.
n2 + k .. n + k dac
a si numai dac
a k 2 + k .. n + k. Cum A = k, k + 1, k2 + k Dk2 +k ,
putem lua n + k din multimea A si atunci obtinem n 0, 1, k 2 . Astfel, am g
asit
trei patrate perfecte care verifica cerinta problemei.
VI.37. Numerele 1160, 1604 si 2270 dau acelasi rest la mpartirea prin n. Aflati
mpartitorul n.
Cristian Laz
ar, Iasi
Solutie. Conform ipotezei, avem: 1160 = nc1 +r, 1604 = nc2 +r, 2270 = nc3 +r,
unde r < n si r, c1 , c2 , c3 N. Sc
aznd aceste egalit
ati dou
a cte dou
a, obtinem

444 = n (c2 c1 ), 666 = n (c3 c2 ) si 1110 = n (c3 c1 ), deci n este divizor comun
al numerelor 444, 666, 1110. Cum (444, 666, 1110) = 222 rezulta ca n {1, 2, 3, 6,
37, 74, 111, 222}, valori care verific
a ipoteza problemei.
VI.38. Demonstrati ca nu exista numere naturale x, y, z direct proportionale cu
trei numere naturale consecutive, astfel nct x + y + z sa fie numar prim.
Alexandru Negrescu, elev, Botosani
Solutie. Dac
a presupunem contrariul, avem
x
y
z
x+y+z
(1)
=
=
=
, cu n N .
n
n+1
n+2
3n + 3
De aici, obtinem c
a 3y = x + y + z, deci 3 | x + y + z, care mpreun
a cu faptul c
a
x + y + z este prim ne conduce la concluzia ca x + y + z = 3 si deci y = 1. nlocuind
50

x
1
n
=
, adic
ax=
, care nu apartine lui N.
n
n+1
n+1
VI.39. Radu si Mihai joaca de mai multe ori un joc n urma caruia cstigatorul
primeste a puncte, iar cel care pierde primeste b puncte ( a, b N , a > b). Daca
scorul final este 61 49 n favoarea lui Radu, iar Mihai a cstigat 4 partide, aflati
a si b.
Adrian Zanoschi, Iasi
Solutie. Dac
a not
am cu x num
arul partidelor cstigate de Radu, avem: xa+4b =
= 61, 4a + xb = 49, de unde obtinem ca (x + 4) (a + b) = 110. De aici, avnd n
vedere ca x + 4 9 si a + b 3, rezulta ca x + 4 = 22 si a + b = 5 sau x + 4 = 11
si a + b = 10 sau x + 4 = 10 si a + b = 11. n primul caz, avem x = 18, dar atunci
xa + 4b este un num
ar par, diferit de 61, deci aceast
a situatie nu convine. Procednd
la fel, constatam ca nici al treilea caz nu convine. n al doilea caz, gasim x = 7, a = 7
si b = 3, care este solutia problemei.
b = 120 . Perpendiculara n C pe AC intersecteaza
VI.40. Fie 4ABC cu m(A)
mediatoarea lui [AB] n D; notam {E} = CD AB. Sa se arate ca AB = 2AC
\ = 90 si BE = 2AB.
daca si numai daca m(BDE)
Ioan S
ac
aleanu, Hrl
au
Solutie. Fie M mijlocul lui AB.
Presupunem ca AB = 2AC. n acest caz
D
\ = ADM
\ =
rezult
a c
a AM = AC, deci CDA
\
= M
DB = . Cum suma unghiurilor patrulaterului DM AC este 360 , obtinem ca
C
\ = 90 . Triunghiul DAB
= 30 , deci BDE
\ de 60 , adica
este isoscel si are unghiul BDA
este echilateral si, prin urmare, DA = AB. n B
M
A
E
\ = 60 , deci AEC
[ = 30 . Atunci
plus DBA
4ACD 4ACE (C.U.), de unde AD = AE. n concluzie, BA = AD = AE, adica
BE = 2AB.
\ = 90 si A mijlocul lui BE. Cum AC k BD, rezult
Fie acum BDC
a c
a [AC] este
1
[ = 60 si CA k BD,
linie mijlocie n triunghiul BDE, deci AC = BD. Din CAE
2
\ = 60 , deci triunghiul DBA este echilateral, ceea ce conduce la
obtinem ca DBA
1
concluzia BD = AB. Asadar, avem AC = AB sau AB = 2AC.
2
n relatia (1), g
asim

Clasa a VII-a

2
+ +
n

2n 1
< 2n 1, n N, n 2.
n
C
at
alin Calistru, Iasi
Solutia I (un grup de elevi de r
la Colegiul National din Iasi si Alexandru
1 + k/n
k+n
k
<
=
, k = 1, 2n 1. Ca
Negrescu, elev, Botosani). Avem
n
2
2n
urmare,
r
r
r

1
1
1
2
2n 1
+
+ +
<
2n 1 + (1 + + (2n 1)) =
n
n
n
2
n
VII.36. Sa se arate ca

1
+
n

51

1
1 (2n 1) 2n
2n 1 +
= 2n 1.
2
n
2
Solutia II. r
Membrul r
din stnga al inegalit
atii date se poate scrie grupnd ternk
n+k
menii de forma
,
, k {1, 2, . . . , n 1}. n acest fel, obtinem n 1
nr
n
n
paranteze si termenul
= 1. Deoarece
n
nk+n+k
r
!2
r
r
r
nk
n+k
nk n+k
n
=2+2
+
<2+2
= 4,
n
n
n
n
2
r
r
nk
n+k
+
< 2, de unde concluzia.
rezult
a c
a
n
n
VII.37. Aratati ca n baza de numeratie 7 printre numerele ce se scriu cu cifrele
0, 1, 2 exista o infinitate care sunt patrate perfecte si o infinitate ce nu sunt patrate
perfecte. Aceste afirmatii ramn valabile daca se folosesc cifrele 3, 5, 6?
Ruxandra Ioana Vlcu, elev
a, Iasi
2
n
2n
n
2
=
(7
+
1)
=
7
+27
+1
=
10
.
.
.
020
Solutie. Se observa ca 100
.
.
.
01
{z . . . 1} (7)
|
| {z } (7)
=

n+1 cifre

2n+1 cifre

este p
atrat perfect, n N , iar 10
a{z . . . 2} (7) = |10 . . . 020
{z . . . 1} (7) + 1 nu este p
| . . . 020
2n+1 cifre

2n+1 cifre

trat perfect pentru nici un n N , deoarece este cuprins ntre (7n + 1)2 si (7n + 2)2 .
Daca n N, atunci putem scrie n = 7k + r, unde k, r N, r < 7. Deoarece
n2 = 7k 0 + r0 , cu r0 {0, 1, 2, 4}, rezult
a c
a nici un p
atrat perfect scris n baza 7 nu
se termin
a cu 3, 5 sau 6. Prin urmare, r
aspunsul la ultima ntrebare este negativ.
abb . . . bc
ac
=
ca
cbb . . . ba
(termenii primei fractii continnd cte 2002 cifre b), atunci b = a + c.
Mihaela Buc
ataru, Iasi
Solutie. Daca notam n = |11 {z
. . . 1} , avem succesiv:
VII.38. Fie a, b, c cifre nenule, a 6= c. Sa se arate ca daca

2002 cifre

a 102003 + 10nb + c
10a + c
=

2003
c 10
+ 10nb + a
10c + a

102003 a2 c2 + 100nb (c a) + 10nb (a c) + 10 c2 a2 = 0

102003 (a + c) 100nb + 10nb 10 (c + a) = 0

2002
1 90bn = 0 (a + c) 10 9n 90bn = 0 a + c = b.
(a + c) 10 10

VII.39. Daca x < y < z sunt lungimile laturilor unui triunghi dreptunghic,
atunci xn + y n 6= z n , n N, n 3.
Dumitru Neagu, Iasi
Solutie. Din relatia z > y > x, rezult
a c
a z n2 > y n2 si z n2 > xn2 , oricare
ar fi n 3. De aici, obtinem ca, pentru orice n 3, avem:

z n = z n2 z 2 = z n2 x2 + y 2 > xn2 x2 + y n2 y 2 = xn + y n .
b = 60 , iar M Int ABC
VII.40. Fie ABC un triunghi ascutitunghic cu m(A)
52

\
astfel nct m(BM
C) = 150 . Notam cu P , Q, R proiectiile lui M pe BC, CA si
respectiv AB. Sa se arate ca 4P QR este dreptunghic.
Constantin Cocea, Iasi
A
Solutie. Deoarece patrulaterele M P BR si M P CQ

\
\
\
sunt inscriptibile, avem: M P R = RBM = 90 RM B
60
\
\ = 90 QM
\
si M
P Q = QCM
C. Astfel, obtinem:
\
\
\
\
\
RP
Q=M
PR + M
P Q = 180 ( RM
B + QM
C)=
\
\
= 180 (360 RM
Q BM
C)=

= 180 (360 120 150 ) = 180 90 = 90 .

Clasa a VIII-a

M
B

VIII.36. Determinati cardinalul minim al unei multimi B pentru care putem


defini functii f : R B astfel nct f (1) < 0 si f (xy) = f (x) f (y), x, y R.
Iulia Zanoschi, elev
a, Iasi
Solutie. Vom demonstra c
a multimea B trebuie s
a aib
a cel putin trei elemente
si c
a exist
a o functie care are codomeniul B format din trei elemente si ndeplineste
restul conditiilor din enunt.
Avem f (1) = f ((1) (1)) = f (1) f (1) > 0. Pe de alt
a parte, din f (0) =
f (0 (1)) = f (0) f (1), rezult
a c
a f (0) [f (1) 1] = 0, deci f (0) = 0. Prin
urmare, f (1), f (0) si f (1) sunt trei numere distincte, ceea ce nseamn
a c
a B are
cel putin
trei
elemente.
n
fine,
se
observ
a
c
a
f
:
R

{1,
0,
1},
definit
a prin

1, x < 0
0, x = 0 , verific
f (x) =
a toate conditiile cerute.

1, x > 0
VIII.37. If a, b, c (0, ) prove
the following inequalities:
a) (a + b + c)3 a3 + b3 + c3 24 where abc = 1;

3
8 3
3
3
3
b) (a + b + c) a + b + c
where ab + bc + ac = 1.
3
Zdravko Starc, Vr
sac, Serbia and Montenegro
Solutie. a) Se stie ca, oricare ar fi numerele a, b, c, are loc egalitatea:

(1)
(a + b + c)3 a3 + b3 + c3 = 3 (a + b) (b + c) (c + a) .

Avnd n vedere identitatea (1) si inegalitatea mediilor, putem scrie:

3
(a + b + c) a3 + b3 + c3 = 3 (a + b) (b + c) (c + a)

3 2 ab 2 bc 2 ca = 24abc = 24.
b) Solutia I (Irina Mustata
, eleva, Iasi). Prin nmultirea ultimelor doua paran
teze din partea dreapt
a a rela
t
cont
c
a ab + bc + ca = 1, obtinem

iei (1) si tinnd


3
3
3
3
2
(a + b + c) a + b + c = 3 (a + b) c + 1 ; similar, avem si (a + b + c)3

a + b3 + c3 = 3 (b + c) a2 + 1 si (a + b + c)3 a3 + b3 + c3 = 3 (c + a) b2 + 1 .
Prin adunarea acestora avem

(a + b + c)3 a3 + b3 + c3 = (a + b) c2 + 1 + (b + c) a2 + 1 + (c + a) b2 + 1 =
= 2 (a + b + c) + ab (a + b) + bc (b + c) + ca (c + a) =
= 2 (a + b + c) + (a + b + c) (ab + bc + ca) 3abc, adica
53

3
(a + b + c) a3 + b3 + c3 = 3 (a + b + c) 3abc.
(2)

Observam ca din 3 = 3 (ab + bc + ca) (a + b + c)2 rezulta ca a + b + c 3, iar

1
3
a abc . Revenind la (2) vom obtine
din 1 = ab + bc + ca 3 a2 b2 c2 deducem c
3
3

1
8
3
3
(a + b + c) a3 + b3 + c3 3 3 =
.
3
3
Solutia II (Marius Pachitariu, elev, Iasi). Cum ab + ac + bc = 1, avem:

a3 + b3 + c3 3abc = (a + b + c) a2 + b2 + c2 ab ac bc =
i
h
2
3
= (a + b + c) (a + b + c) 3 (ab + ac + bc) = (a + b + c) 3 (a + b + c) .

8 3
Astfel, inegalitatea de la punctul b) se va scrie 3 (a + b + c) 3abc
sau

3
8 3
a + b + c abc
.
(2)
9
Pentru a justifica inegalitatea (2), vom demonstra dubla inegalitate:
r

ab + bc + ca
a+b+c
3
(3)

abc, a, b, c > 0.
3
3
Pentru prima parte a relatiei (3), observam ca

2
a+b+c
ab + bc + ca
2

(a + b + c) 3 (ab + ac + bc)
3
3
(a b)2 + (b c)2 + (c a)2 0,

evident adevarata. Pentru partea a doua, folosim inegalitatea mediilor:


r
r
q
2

ab + bc + ca
3
3
3
ab bc ca =
abc = abc.

3
Revenind la inegalitatea (2), avem:
r
!3
r

8 3
ab + bc + ca
ab + bc + ca
=

.
a + b + c abc 3
3
3
9
Solutia III (data de autor). Din inegalitatea lui Carlson:
r
r
ab + bc + ca
3 (a + b) (b + c) (c + a)

, a, b, c > 0
8
3
si identitatea (1), rezult
a c
a:

3
(a + b + c) a3 + b3 + c3 = 3 (a + b) (b + c) (c + a)
r
!3

1
8 3
ab + bc + ca
=38 =
38
.
3
3
3 3
VIII.38. Fie n N fixat. Aratati ca exista o infinitate de numere x, y, z Z
astfel nct x2n + y 2n + z 2n = x2n+1 + y 2n+1 + z 2n+1 .
Lucian Tutescu, Craiova
54

Solutie. Dac
a lu
am z = y, atunci din relatia dat
a, obtinem:
2n
2n
2y = x (x 1) .
(1)
2n
O solutie a acestei din urm
a ecuatii putem g
asi alegnd x 1 = 2a , a Z .

2n
ntr-adev
ar, n acest caz egalitatea (1) devine 2y 2n = 2a2n + 1
2a2n , de unde
2n
g
asim y = a 2a + 1 . Deci,
a o infinitate
a:
exist
de numere cu proprietatea dat
x = 1 + 2a2n , y = a 2a2n + 1 , z = a 2a2n + 1 , a Z .
VIII.39. Fie ABCD un patrulater strmb cu [AD] [BC]. Sa se construiasca
dreptele paralele d1 , d2 , d3 , d4 astfel nct A d1 , B d2 , C d3 , D d4 si
dist (d1 , d4 ) = dist (d2 , d3 ).
Horia Mihail Teodorescu, elev, Iasi
Solutie. Fie d o dreapt
a care face unghiuri egale cu AD si BC (evident, putem
gasi o astfel de dreapta). Dreptele d1 , d2 , d3 si d4 , duse prin A, B, C, respectiv
D si paralele cu d, satisfac conditiile problemei. ntr-adevar, daca notam cu E si F
picioarele perpendicularelor din A si B pe d4 , respectiv d3 avem c
a 4AED 4BF C
(I. U.), deci AE = BF , adic
a dist (d1 , d4 ) = dist (d2 , d3 ).
VIII.40. Fie ABCDA0 B 0 C 0 D0 un cub, iar O (BB 0 ). Dreptele A0 O si C 0 O
intersecteaza (ABC) n E, respectiv F , iar AO si CO intersecteaza (A0 B 0 C 0 ) n
E 0 , respectiv F 0 .
a) Aratati ca EF E 0 F 0 nu depinde de pozitia lui O;
b) Aratati ca SBB 0 E 0 E SABCD si determinati O pentru care se atinge egalitatea.
Monica Nedelcu, Iasi
Solutie. a) Cum (A0 B 0 C 0 ) k
E
(ABC) si (EOF ) (A0 B 0 C 0 ) = A0 C 0 ,
C
B
(EOF ) (ABC) = EF , rezulta ca
F
EF k A0 C 0 , deci 4A0 OC 0 4EOF ,
E
D
A
de unde deducem c
a
EF
EO
BO
O
=
= 0 .
(1)
0
0
0
AC
OA
BO
Analog, putem demonstra ca 4AOC
C
B
F
4E 0 OF 0 , deci
0 0
0
0
EF
EO
BO
=
=
.
(2)
A
D
AC
OA
OB
0 0
EF E F
= 1, deci EF E 0 F 0 = AC 2 = const.
Din (1) si (2), obtinem
AC A0 C 0 0 0
BE
x
BE
ax
si
b) Fie B 0 O = x. Atunci, avem
=
=
. De aici rezulta ca
a ax
a
x
0
0 0
2
2
BB (B E + BE)
a
x
ax
a
SBB 0 E 0 E =
=
+

2 = a2 = SABCD .
2
2 ax
x
2
x
a
Egalitatea are loc dac
a si numai dac
a
= 1, adic
a x = , ceea ce nseamn
a c
a
ax
2
0
O este mijlocul segmentului [BB ].

Clasa a IX-a

y
IX.36. Determinati x < 0 < y astfel nct xy + = y 3 5y + 2.
x
Cezar Lupu, elev, Constanta
55

Solutie. Ecuatia dat


a este echivalent
a cu:
2
1
x + + 5 = y2 + .
(1)
x
y
1
Cum x < 0, rezult
a c
a x + + 5 2 + 5 = 3, cu egalitate numai pentru x = 1.
x
Pe de alta parte, avnd n vedere ca y > 0, putem scrie:
r
2
1 1
1 1
y 2 + = y 2 + + 3 3 y 2 = 3,
y
y y
y y
cu egalitate numai pentru y = 1. Asadar, egalitatea (1) este posibil
a dac
a si numai
2
1
2
daca x + + 5 = 3 = y + , adica pentru x = 1 si y = 1.
x
y
IX.37. Pentru x [1, ), n N , demonstrati inegalitatea

n+1
+ 1 (xn 1) 2nxn (x 1) .
x
Marius Pachitariu, elev, Iasi
Solutia I. Inegalitatea dat
a se transform
a succesiv astfel:
x2n+1 xn+1 + xn 1 2nxn+1 2nxn

x2n+1 1 (2n + 1) xn+1 xn

(1)

Inegalitatea (1) este adevarata pentru x = 1, iar pentru x > 1 este echivalenta cu
1 + x + x2 + + x2n
x2n+1 1
a din inegali (2n + 1) xn sau
xn , care rezult
x1
2n + 1
tatea mediilor n felul urm
ator:

(2n+1)2n
1 + x + x2 + + x2n
2n+1
1 x x2 x2n = x 2(2n+1) = xn .

2n + 1
Solutia II (Irina Mustata
a, Iasi). Prin inductie complet
a.
, elev
n+1
n+1
n+1
y
z
x
+ n + n x + y + z, x, y, z > 0, n N.
IX.38. Sa se arate ca
yn
z
x
Gigel Buth, Satu Mare
Solutie. n GM - 4/2002, p. 146, L. Panaitopol enunta si demonstreaz
a rezultatul
urm
ator:
Daca p 1 si ai 0, bi > 0 pentru i 1, n, atunci
p
Xn
n
ai
X
api
i=1
p1 .
p1 Xn
b
i=1 i
b
i=1

Inegalitatea din enunt rezulta imediat din aceasta.


1
2
1
+ q
=
IX.39. Sa se rezolve ecuatia q
.
[x] [x + 2]
3
3
3
2 [x]
3 [x] [x + 1]
Daniel Jinga, Pitesti
Solutie. Ecuatia are sens daca [x] > 0, adica [x] 1. Daca facem notatia
a devine:
[x] = y N , ecuatia dat
1
1
2
.
(1)
+
=
2y y 3 (y + 1) 3 y
y (y + 2)
56

y+1
y+2

Deoarece y = y 1
(2) si 3 y = 3 y 1 1
(3), rezult
a c
a
2
3
1
1
1
1
2
+
=
. Prin urmare, ecuatia
+

2y y 3 (y + 1) 3 y
y (y + 1) (y + 1) (y + 2)
y (y + 2)
(1) are solutie dac
a si numai dac
a (2) si (3) sunt simultan egalit
ati, adic
a y = 1. Deci
solutia ecuatiei date este x [1, 2).
IX.40. Fie M 6= G n planul 4ABC si D, E, F mijloacele laturilor [BC],

[CA] si respectiv [AB]. Consideram punctele X, Y, Z astfel nct XD = mXM ,

Y E = mY M , ZF = mZM , m 6= 1.
2m
3
SM .
a) Daca m 6= , atunci AX, BY, CZ sunt concurente n S, cu SG =
2
3
3
b) Daca m = , atunci AX, BY, CZ sunt paralele cu GM .
2
Virgil Nicula, Bucuresti
Solutie. a) Avem:
2m
2m
2m 3
SM SM + M G =
SM
SM = M G
SG =
3
3
3


3
1
MS =
MG MS =
MA + MB + MC .
3 2m
3 2m

1
M A + M B + M C . Se poate
Fie punctul S 0 definit prin egalitatea M S 0 =
3 2m
0
verifica, prin calcul, faptul c
a S apartine dreptelor AX, BY , CZ, deci acestea vor

2m
0
SM . S
a
arat
a si egalitatea SG =
fi concurente n S S si atunci este adev
3
0
demonstram, de exemplu, ca S AX. Pentru aceasta vom demonstra ca vectorii

XS 0 si S 0 A sunt coliniari:

0 0 M A + M B + M C
MB + MC
XS = M S M X =

=
3 2m
2 2m

(2 2m) M A M B M C
,
=
(3 2m) (2 2m)

0 0 M A + M B + M C

S A = MA MS = MA
= (2 2m) XS 0 .
3 2m
3
b) Pentru m = , avem:
2
3


3
XD = XM XM + M D = XM M X = 2M D = (M B + M C)
2
2

si atunci XA = M A M X = M A + M B + M C = 3M G. Analog se obtine


Y B = ZC = 3M G, deci dreptele AX, BY , CZ sunt paralele.

Clasa a X-a

X.36. Sa se rezolve inecuatia alogb x + xlogb x a + b, unde a, b (1, ).


Daniela Dodan, elev
a, Iasi
2
Solutie. Din egalitatea x = blogb x , x > 0, rezult
a c
a xlogb x = blogb x , x > 0.
Deci, inecuatia data este echivalenta cu
2

alogb x + blogb x a + b.
57

(1)

Dac
a facem notatia log2b x = 0 si avem n vedere obsevatiile > 1 a + b >
> a + b, 1 a + b a + b, obtinem ca inecuatia (1) este echivalenta cu
log2b x 1, deci x b1 , b .

X.37. Fie a, b (0, 1) (1, ) si functia injectiva f : (0, ) R astfel nct


functia g : R R, g (x) = f (ax ) + f (bx ) este constanta. Sa se arate ca ab = 1 si
ca exista functii f care satisfac ipotezele problemei.
Dan Popescu, Suceava
Solutie. Fie
= f(ax ) +f (bx ) = k, x R, unde k R. Atunci,
avem
logg (x)

k = f (x) +f b a x = f alogb x + f (x), x > 0, de unde rezulta ca f bloga x =


= f alogb x . Cum f este functie injectiv
a, deducem c
a bloga x = alogb x , x > 0, deci
g (x)
a a = b sau ab = 1. Dac
a a = b, atunci f (ax ) =
log2a b = 1, adic
, x R,
2
k
1
sau f (x) = , x > 0, ceea ce contrazice injectivitatea functiei f . Pentru b = si
2
a
f (x) = loga x, se obtine g (x) = 0, x R.

X.38. Fie a, b, c, d R cu a > b > c > d. Sa se arate ca a, b, c, d sunt n progresie

3
ad
aritmetica daca si numai daca (a b) (b c) (c d) =
.
3
A. V. Mihai, Bucuresti
Solutie. Dac
a a, b, c, d sunt n progresie aritmetic
a de ratie r, atunci egalitatea
3
3r
data este echivalenta cu r r r =
, care este, evident, adevarata.
3
p
Reciproc, dac
a are loc egalitateap
din enunt, atunci ad = 3 3 (a b) (b c) (c d),
sau (a b) + (b c) + (c d) = 3 3 (a b) (b c) (c d), adica media aritmetica a
numerelor a b, b c si c d este egala cu media lor geometrica. De aici rezulta ca
a b = b c = c d, deci a, b, c, d sunt n progresie aritmetic
a.
X.39. Fie ABCDA0 B 0 C 0 D0 un paralelipiped dreptunghic cu dimensiunile AB = a,
AD = b, AA0 = c. Daca M Int A0 B 0 C 0 D0 , notam cu , , masurile unghiurilor
pe care AM le face cu AB, AD si respectiv AA0 . Sa se arate ca
AM < a cos + b cos + c cos < AC 0 .
C
at
alin Calistru, Iasi
Solutie. Fie E, F si G proiectiile punctului M
D
C
pe laturile AB, AD si respectiv AA0 . Astfel avem
AE
AF
AG
si cos =
cos =
, cos =
, de A
B
AM
AM
AM
unde deducem ca AE cos + AF cos + AG cos =
M
AE 2 + AF 2 + AG2
AM 2
=
=
= AM . De aici, G
AM
AM
C
D
avnd n vedere ca AE < AB = a, AF < AD = b,
F
AG < AA0 = c si cos > 0, cos > 0, cos > 0,
rezult
a c
a AM < a cos + b cos + c cos . Pentru
A
E
B
a doua parte a inegalitatii vom folosi inegalitatea lui
Cauchy-Buniakovski-Schwarz si identitatea cos2 + cos2 + cos2 = 1:
p
p
a cos + b cos + c cos < a2 + b2 + c2 cos2 + cos2 + cos2 = AC 0 .
58

X.40. a) Pentru x, y, z 0, demonstrati inegalitatea


p

x + y + x + z + y + z xy + xz + yz 3 6xyz.

b) Cu notatiile uzuale, n orice triunghi are loc inegalitatea

2
2
a

b
+
(
a

c)
+
b

c
R
9
.
2

r
4
a+ b+ c

Marian Tetiva, Brlad


Solutie. a) Din relatiile
p
p

x + y + x + z + y + z 3 6 (x + y) (x + z) (y + z) 3 6 8xyz = 3 2 6 xyz si
q p
p

xy + xz + yz 3 3 x2 y 2 z 2 = 3 6 x2 y 2 z 2

rezult
a c
a

p

x + y + x + z + y + z xy + xz + yz 3 6 xyz = 3 6xyz.

b) Vom aplica inegalitatea de la punctul a) pentru x = pa, y = pb si z = pc.


Cu notatiile facute, avem:
x + y = c, x + z = b, y + z = a,
X
X

xy + xz + yz =
(p a) (p b) =
p2 (a + b) p + ab =
X

= 3p2 4p2 +
ab = p2 + p2 + r2 + 4Rr = r2 + 4Rr,

S2
= pr2 .
p

2
a+ b+ c
r + 4Rr 3 6pr2 sau
Astfel, inegalitatea de la a) devine

2 2
R
27 (a + b + c)
a+ b+ c
. De aici
r + 4Rr 54pr2 , deci 1 + 4

r
a+ b+ c
obtinem c
a

2
a+ b+ c
9
R
27 (a + b + c)
9 3 (a + b + c)
=
=
2

2 4
2
r
4
4
a+ b+ c
a+ b+ c

2
2
a

b
+
(
a

c)
+
b

c
9
.
=

2
4
a+ b+ c
xyz = (p a) (p b) (p c) =

Clasa a XI-a

XI.36. Fie D, M doua matrice nesingulare de ordin n, D diagonala, iar M


triunghiulara. Daca D = t M DM , sa se arate ca M este tot o matrice diagonala,
avnd 1 pe diagonala principala.
Adrian Corduneanu, Iasi
Solutie.
59

1
m11 m12 . . . m1n
0 ... 0
0
0
2 . . . 0
m22 . . . m2n

Fie D =
. . . . . . . . . . . ., cu i 6= 0, i = 1, n, M = . . .
... ...
...
0
0 . . . n
0
0
. . . mnn
mii 6= 0, i = 1, n si Mij complementul algebric al lui mij n matricea M . Not
am cu
d = det M 6= 0. Relatia dat
a este echivalent
a cu DM 1 = t M D. Deoarece

0 ... 0
M11 M21 . . . Mn1
1

1 0
2 . . . 0
M22 . . . Mn2
0
=
DM 1 =

...
... ...
...
d ... ... ... ...
0
0 . . . n
0
0
. . . Mnn

1 M11 1 M21 . . . 1 Mn1


1
0
2 M22 . . . 2 Mn2

si
=

...
...
...
...
d
0
0
. . . n Mnn

0
...
0
0 ... 0
1
m11

m12 m22 . . .
2 . . . 0
0
t
0
=
MD =

...
... ...
...
. . . . . . . . . . . .
m1n m2n . . . mnn
0
0 . . . n

0
...
0
1 m11
1 m12 2 m22 . . .
0

=
...
...
...
...
1 m1n 2 m2n . . . n mnn

i Mii
= i mii , i = 1, n. Din mij = 0,
d
Mii
1
pentru i < j, deducem ca M este matrice diagonala si atunci
, i = 1, n,
=
d
mii
a mii = 1, i = 1, n. Cazul n care M este inferior
deci avem m2ii = 1, i = 1, n, adic
triunghiulara se trateaza n mod analog.
rezulta ca mij = 0, oricare ar fi i < j si

XI.37. Fie A M3 (C) astfel nct det (A + tA) = 0, unde R\{1, 0, 1}.
2 ( 1)2
det A.
Sa se arate ca det (A + tA) =

Marian Ionescu, Pitesti si Lucian Tutescu, Craiova


a cu gradul
Solutie. P (x) = det (A + x tA), x C este o functie polinominal
mai mic sau egal cu 3. Deoarece

1
1
t
3
t
P (x) = det x
= x det
A+ A
A+ A =
x
x


1t
1
= x3 det
A + A = x3 P
, x C ,
x
x

rezult
a c
a P este polinom reciproc, deci P (x) =
(detA) x3 + ax2 + ax + det A.
1
Cum, prin ipotez
a, P () = 0, nseamn
a c
a si P
= 0. De aici, avnd n

1
vedere c
a P (1) = 0, obtinem c
a P (x) = (det A) (x ) x
(x + 1). Asadar,

60

1
2 ( 1)2
det (A + A) = P (1) = (det A) (1 ) 1
2=
det A.

XI.38. Sa se determine functiile continue f : [0, ) [0, ) pentru care


f (f (x)) + 2f (x) = 3x, x 0.
Mihail Bencze, Brasov
Solutie. Fie fn (x) = f f f (x), x 0, n N . Pentru orice n 3
{z
}
|
n ori

si x 0, avem c
a fk (x) + 2fk1 (x) = 3fk2 (x), k = 2, n, de unde prin sumare
deducem c
a fn (x) + 3fn1 (x) = f1 (x) + 3x, n 3, x 0. De aici, obtinem:
f3 (x) + 3f2 (x) = f1 (x) + 3x,
f4 (x) + 3f3 (x) = f1 (x) + 3x,
f5 (x) + 3f4 (x) = f1 (x) + 3x,
....................................
fn (x) + 3fn1 (x) = f1 (x) + 3x, n 3, x 0.

0
1
1
1
, a doua cu
, a treia cu
Mai departe, nmultind prima ecuatie cu
3
3

2
1

etc. si apoi adunndu-le, g


asim relatia
3

n3

2
n3 !
1
1
1
1

fn (x)+3f2 (x) = (f1 (x) + 3x) 1 +


+ +
+
3
3
3
3
sau

n3
n2 !
3
1
1
fn (x) + 9x 6f (x) = (f1 (x) + 3x) 1
, n 3, x 0.

3
4
3
(1)
n
3x
3
Din ipoteza rezulta ca f (x)
x, n N ,
, x 0, si atunci fn (x)
2
2
x
fn (x)
n , n N , x 0, adic
a
x 0. De aici, obtinem c
a0
3n
2
fn (x)
= 0, x 0.
(2)
lim
n 3n
3
Din (1) si (2), rezulta ca 9x6f (x) = (f (x) + 3x), x 0, deci f (x) = x, x 0.
4
Observatie. Nu este nevoie de continuitatea functiei f .
n
P
yi
este convergent. Daca
XI.39. Fie sirul (yn )n1 astfel nct sirul
i=1

n1

(x )
R+ are proprietatea ca xn xn+1 (1 + xn yn+1 ), n 1, aratati ca sirul

n n1
1
este convergent.
xn n1
Gheorghe Molea, Curtea de Arges
Solutie. Deoarece xn > 0, n 1, rezulta ca inegalitatea din enunt este echiva61

lent
a cu

1
xn+1

1
yn+1 , n 1, de unde deducem c
a
xn
n
X
1
1

y2 + y3 + + yn =
yi y1 ,
xn x1
i=1

n 1.

Cum n
partea
a a ultimei relatii este un sir convergent, deci m
arginit, rezult
a
dreapt
1
este si el m
arginit.
c
a sirul
xn n1
n
n1
P
P
1
1

yn =
yi
yi este echivalent
a cu
Pe de alt
a parte, relatia
xn
xn1
i=1
i=1
n
n1
n
P
P
P
1
1
1

yi

yi , n 2, sau, cu notatia zn =

yi , zn zn1 ,
xn i=1
xn1
xn i=1
i=1
n 2. De aici, avnd n vedere ca sirul (zn )n1 este marginit, fiind diferenta a
dou
a siruri m
arginite, rezult
a c
a sirul (zn )n1 este convergent. Prin urmare, sirul cu
n
P
1
= zn +
yi este convergent.
termenul general
xn
i=1
3
XI.40. Fie x0 [1, 1];
aratati ca pentru orice n N, ecuatia 3x4x = xn are
1 1
o singura solutie xn+1 , . Demonstrati ca sirurile (xn )n0 si (3n xn )n0
2 2
sunt convergente si calculati limitele lor.
Marian Tetiva, Brlad
Solutie. S
a ar
at
am, pentru nceput,
c
a
dac
a
a

[1,
1],
atunci ecuatia 3x4x3 = a

1 1
are o singur
a solutie n intervalul , . Pentru aceasta, consider
am functia
2 2


1
1
f : R R , definit
a prin f (x) = 3x 4x3 a. Deoarece f
f
=
2
2
= (1 a) (1 a) 0 si f este continua, rezulta ca f se anuleaz
a cel putin o

1 1
1 1
0
2
data n intervalul , . Cum f (x) = 3 1 4x 0, x , , nseamna
2 2
2 2

1 1
ca f este strict crescatoare pe , , deci ecuatia f (x) = 0 are o singura solutie
2 2

1 1
n intervalul , .
2 2

Mai putem observa c


a, dac
a = arcsin a, avem 3 sin 4 sin3 = sin = a
3
3

h i

1 1

si ,
implic
a sin , . Deci sin este tocmai solutia din in3
2 2
3
2 2
1 1
tervalul ,
a ecuatiei 3x 4x3 = a. Astfel, am demonstrat ca, pentru orice
2 2

1 1
3
si anume
a [1, 1], ecuatia 3x 4x = a are o singura solutie n intervalul ,
2 2

arcsin a
sin = sin
.
3
3
Revenind la problema noastr
a, rezult
a, din cele ar
atate mai sus, c
a, pentru
arcsin xn
. De aici, obtinem c
a
orice n N, xn+1 este bine definit si xn+1 = sin
3
62

arcsin xn
arcsin x0
, n N. Astfel avem
, n N, deci arcsin xn =
3
3n
lim xn = lim sin (arcsin xn ) = 0 si

arcsin xn+1 =
n

lim 3n xn = lim (3n arcsin xn )

xn
= arcsin x0 .
arcsin xn

Clasa a XII-a

XII.36. Sa se determine n N, n 2 pentru care ecuatia x2 = x + b


1 are solutie
unica n Zn ; rezolvati ecuatia n acest caz.
Andrei Nedelcu, Iasi
Solutie. Dac
ab
a Zn este solutie a ecuatiei x2 = x + b
1, atunci si b
1b
a este
2
2
b
b
c
b
c
solutie a acestei ecuatii ((1 b
a) = 1 2a + b
a = 1 2a + b
a+b
1 = (b
1b
a) + 1). Cum
\
ecuatia trebuie sa aiba solutie unica, este necesar sa avem b
a=b
1b
a, sau 2a
1=b
0.
cb
Deoarece b
a2 = b
a + 1 implica b
4b
a2 = b
4b
a+b
4, sau (2a
1)2 = b
5, rezulta ca b
5=b
0. De
aici, obtinem c
a n = 5 si atunci ecuatia dat
a are solutia unic
ab
a=b
3.

XII.37. Fie (G, +) un subgrup al grupului (R, +). Sa se determine morfismele


crescatoare de la (G, +) la (R, +).
Dan S
tefan Marinescu si Viorel Cornea, Hunedoara
Solutie. Daca G = {0}, atunci f : G R, f (0) = 0 este functia cautata.

f (x0 )
,
x0
observam ca a 0. Folosind definitia morfismului de grupuri se poate demonstra
prin inductie ca f (nx) = nf (x), n Z, x G. De aici, deducem ca f (nx0 ) =
= nf (x0 ) = nax0 , n Z.
Fie un element oarecare y G. Dac
a x0 > 0, avem succesiv:

n (y + x0 )
n (y + x0 )
n (y + x0 )
<

+ 1, n N
x0
x0
x0


n (y + x0 )
n (y + x0 )
f
x0 f (n (y + x0 )) f
+ 1 x0 , n N
x0
x0

n (y + x0 )
n (y + x0 )
ax0 n (f (y) + ax0 )
+ 1 ax0 , n N
x0
x0

1 n (y + x0 )
1 n (y + x0 )
1
ax0 f (y) + ax0
+
ax0 , n N ,
n
x0
n
x0
n
Daca G 6= {0}, atunci exista x0 G \ {0} si atunci daca notam a =

de unde, trecnd la limit


a pentru n , obtinem
y + x0
y + x0
ax0
f (y) + ax0 ax0
,
x0
x0
deci f (y) = ay. Daca x0 < 0, se ajunge, n mod analog, la acelasi rezultat.
n sfrsit, observ
am c
a functia f : G R, f (y) = ay, a > 0, este un morfism
cresc
ator de grupuri.
XII.38. Determinati functiile derivabile f, g : R R astfel nct f 0 (x) = g (x)+x
si g 0 (x) = f (x) x, x R.
Gheorghe Iurea, Iasi
63

Solutie. Adunnd cele dou


a relatii date, obtinem (f + g) (x) = (f + g) (x),
0
x R, sau (ex (f + g)) (x) = 0, de unde gasim f (x) + g (x) = Cex , x R, unde
C R este o constanta arbitrara. Revenind la prima ecuatie, avem
f 0 (x) = Cex + x f (x) ,

x R,
C x
sau (ex f (x))0 = Ce2x + xex ,x R, deci f (x) =
e + k1 ex + x 1, x R.
2
C x
Analog, obtinem g (x) =
a usor c
a aceste
e k2 ex x + 1, x R. Se verific
2
functii satisfac sistemul de ecuatii dat.
XII.39. Fie f, g : (0, ) R astfel nct lim f (x) = lim g (x) = , iar
x
Z 1x
f (x)
xg(n)
lim
= R. Sa se calculeze lim f (n)
dx, unde [1, ).
x g (x)
n
0 x+
Adrian Sandovici, Piatra Neamt
Solutie. Din ipoteza rezulta ca exista n0 astfel nct f (n) > 0 si g (n) > 0,
n n0 . Pentru n n0 , avem:
Z 1 g(n)
Z
x
dx
f (n) 1 g(n) 0 x
In = f (n)
x
=
dx =
x+
g (n) 0
x+
0
#
"
1
Z 1
f (n) xg(n)+1
xg(n)

=
2 dx .
g (n)
x + 0
0 (x + )
Deoarece

xg(n) dx

1
xg(n) dx =
g (n) + 1
(x + )2
0
Z 1 g(n)

x
dx
si lim g (n) = , rezulta ca lim
= 0. Asadar, avem lim In =
.
2
n
n 0 (x + )
n
1+
XII.40. Fie f : [0, 1] R o functie derivabila cu derivata continua astfel nct
f (x)
0
exista si este finita. Sa se arate ca
xf (x) f (x), x [0, 1], iar lim
x0 x
x>0
Z 1

Z 1
f (x)
f (x) dx,
f (1) min 2
dx .
x
0
0
Marcel
Chirita
,ZBucuresti
Z
0

Solutie. Din xf 0 (x) f (x), x [0, 1] rezult


a c
a
xf 0 (x) dx
0
Z 1
Z 1
Z 1
1
f (x) dx
f (x) dx, deci f (1) 2
f (x) dx (1).
sau xf (x)|0
0

f (x) dx,

f (x)
exista si este finita, rezulta ca lim f (x) = 0. Astfel, avem
Deoarece lim
x&0 x
x&0
Z 1
Z 1
Z 1
f (x)
f (x)
f 0 (x) dx = lim (f (1) f ()) = f (1) ,
dx = lim
dx lim
&0
&0
&0
x
x
0

ceea ce, mpreun


a cu relatia (1), conduce la inegalitatea din enunt.

64

Solutiile problemelor pentru preg


atirea concursurilor
din nr. 1 / 2003
A. Nivel gimnazial
G36. Fie x, n N astfel nct x divide 10n 1, nsa x nu divide 10k 1 pentru
.
k < n. Sa se arate ca x divide 10m 1 daca si numai daca m .. n.
N. N. Hrtan, Iasi
..

Solutie. Daca m . n, atunci m = 0 sau exista q N astfel nct m = nq. n


.
primul caz, avem 10m 1 = 0 .. x, iar n al doilea avem:
.
10m 1 = (10n )q 1 = (10n 1) ((10n )q1 + + 1) .. x.
.
Sa presupunem acum ca 10m 1 .. x, m 6= 0 si m = nq + r, cu 0 < r < n. De aici

si din ipoteza, obtinem ca x | 10m 1 (10nq 1) = 10nq+r 10nq = 10nq (10r 1).
Deoarece din x | 10n 1 rezult
a c
a (x, 10) = 1, deci (x, 10nq ) = 1, din relatia
r
precedent
a deducem c
a x | 10 1, ceea ce contrazice ipoteza. Prin urmare, dac
a
.
.
10m 1 .. x, atunci m .. n.
G37. 2n muzicieni (n > 2) participa la un festival. La fiecare concert, o parte
dintre ei cnta iar ceilalti asculta. Sa se determine numarul minim de concerte astfel
nct fiecare muzician sa-i asculte pe toti ceilalti.
Titu Zvonaru, Bucuresti
Solutie. Fie a1 , a2 , . . ., a2n cei 2n muzicieni. Dac
a la un concert, unul dintre
ei asculta pe un coleg care cnta, spunem ca are loc o "auditie". pAstfel, la un
concert la care cnta p muzicieni, exista p (2n p) auditii. Deoarece p (2n p)
p + 2n p
a c
a num
arul maxim de auditii are
= n, adic
a p (2n p) n2 , rezult

2
loc atunci cnd n muzicieni cnt
a si n ascult
a.
Sa presupunem ca la primul concert cnta muzicienii a1 , a2 , . . ., an . Pentru a
putea fi ascultat de a2 , a3 , . . ., an , muzicianul a1 trebuie sa mai cnte la un concert
n care s
a nu cnte a2 , a3 , . . ., an , apoi a2 trebuie s
a cnte ntr-un concert n care
nu cnt
a a1 , a3 , . . ., an si asa mai departe. Deci, num
arul minim de concerte este
cel putin n + 1. Sa aratam ca acest numar este n + 1 indicnd o aranjare a celor
n + 1 concerte astfel nct sa fie ndeplinita cerinta problemei. Pentru aceasta, facem
notatiile:
Ak = {an+1 , an+2 , . . . , a2n } {an+k } ,

Bk = {a1 , a2 , . . . an } {ak } ,

k = 1, n

si repartiz
am muzicienii astfel:
Muzicieni care cnta Muzicieni care asculta
1)
a1 , a2 , . . ., an
an+1 , an+2 , . . ., a2n
2)
a1 , A1
B1 , an+1
3)
a2 , A2
B2 , an+2
...
................
....................
n)
an1 , An1
Bn1 , a2n1
n + 1) an , An
Bn , a2n
G38. Multimea A Z are cinci elemente. Adunnd n toate modurile posibile
65

cte trei elemente din multime, obtinem urmatoarele 10 sume: 3, 6, 8, 10, 11, 13,
15, 16, 18, 20. Determinati multimea A. (n legatura cu o problema de concurs din
Iugoslavia.)
Gabriel Popa, Iasi
Solutie. Fie A = {a, b, c, d, e} cu a < b < c < d < e si fie S = a + b + c + d + e.
Adunnd cele 10 sume din enunt obtinem 120. Deoarece n aceasta suma fiecare
element al multimii A apare exact de 6 ori, rezult
a c
a 6S = 120, deci S = 20.
Prin urmare, adunnd n toate modurile posibile cte dou
a elemente din multime,
obtinem urmatoarele 10 rezultate: 17 (= 20 3), 14 (= 20 6), 12, 10, 9, 7, 5, 4,
2, 0. Acum, din a + b = 0 si a + c = 2, gasim b = a si c = 2 a. Apoi, din
e + d = 17 si e + c = 14, deducem c
a d = e + 17 si c = e + 14, deci e = a + 12.
Asadar, putem scrie A = {a, a, a + 2, a + 5, a + 12}. A treia sum
a (n ordine
crescatoare) este a + d sau b + c si este egala cu 4. Cum a + d = a a + 5 = 5, ramne
doar b + c = a a + 2 = 2a + 2 = 4, de unde rezulta ca a = 1. n concluzie,
avem A = {1, 1, 3, 6, 11}.
G39. Fie xi R, i = 1, n, unde n 2003, astfel nct

x1 (n + 1) x2 + nx3 n 1

................................................
xn2 (n + 1) xn1 + nxn n 1

xn1 (n + 1) xn + nx1 n 1 n2

xn (n + 1) x1 + nx2 2n 1.
Daca x1 = 1, sa se calculeze x2003 .

Romeo Cernat, Iasi


Solutie. Cu notatiile yk = xk k + 1, k = 1, n, sistemul dat devine:

y1 (n + 1) y2 +ny3 0, y2 (n + 1) y3 +ny4 0, . . . , yn (n + 1) y1 +ny2 0, (1)

de unde, prin adunare, obtinem 0 0. Asadar, toate inegalit


atile din sistemul (1)
trebuie sa fie egalitati, adica
y1 y2 = n (y2 y3 ) , y2 y3 = n (y3 y4 ) , . . . , yn y1 = n (y1 y2 ) . (2)

nmultind membru cu membru relatiile din sistemul (2), obtinem

(y1 y2 ) (y2 y3 ) . . . (yn1 yn ) (yn y1 ) =


= nn (y1 y2 ) (y2 y3 ) . . . (yn1 yn ) (yn y1 ) ,

de unde rezulta ca una din paranteze este zero si atunci conform cu (2), avem y1 =
= y2 = = yn . Prin urmare, 1 = x1 = x2 1 = x3 2 = = xn n + 1, deci
x2003 = 2003.
G40. Comparati numerele reale a si b, stiind ca a2 14a + b2 + 6b + 33 = 0.
Bogdan R
aducanu, elev, Iasi
Solutia I (un grup de elevi de la Colegiul National din Iasi si Alexandru
Negrescu, elev, Botosani). Relatia din enunt se scrie (a 7)2 + (b + 3)2 = 25.
Deducem c
a |a 7| 5 si |b + 3| 5, de unde a [2, 12] si b [8, 2]. Deci
b 2 a si cum a 6= b (prin verificare direct
a), rezult
a c
a a > b.
Solutia II. Dac
a facem notatia a = b + t problema revine la compararea lui t cu
zero. nlocuind pe a n egalitatea dat
a, obtinem 2b2 + 2b (t 4) + t2 14t + 33 = 0,
ecuatie care are solutie daca si numai daca = t2 8t + 16 2t2 + 28t 66 =
66

= t2 + 20t 50 0. De aici, rezult


a c
a t 10 5 2, 10 + 5 2 , deci t > 0.
Asadar, avem a > b.
G41. Daca 0 < x y z, sa se arate ca
x z
y2
z2
y
x y
z
x
z
x2
3 + + + + + 1+ 2 + 2 + 2.
z
y x
y
z
x
z
x
y
z
x
Ovidiu Pop, Satu Mare
Solutie. Prima inegalitate rezulta din inegalitatea mediilor. Inegalitatea a doua
este echivalent
a cu x2 y + z 2 x + y 2 z x2 z + y 2 x+ z 2 y, sau (y x) (z x) (z y) 0,
care este adev
arat
a n virtutea ipotezei 0 < x y z. Inegalitatea a treia este
x
y
x
echivalenta cu + + 1, adica (z y) (x y) 0, care este adevarata. n
y
z
z
sfrsit, pentru a demonstra ultima inegalitate vom folosi din nou inegalitatea medi 2
2 2 2
2 2
y
x
y
z
z
z
x
x
+
2 ,
+
+
3,
+1 2 .
ilor. Avem:
y
z
z y
z
x
x
x
x
x2
y2
z2
z
Adunnd aceste relatii, obtinem 2
+
+

2
+
1
+
, q.e.d.
y2
z2
x2
z
x
G42. Determinati a, b R, daca [x] + [x + a] = [bx], x R.
Gheorghe Iurea, Iasi
Solutie. Lund x = 0 n egalitatea data gasim [a] = 0, deci a [0, 1). Din relatiile
x 1 + x + a 1 < [x] + [x + a] = [bx] bx, x R, rezult
a c
a a 2 x (b 2),
x R, deci b = 2. Asadar,
avem
[x]
+
[x
+
a]
=
[2x],
x

R,

de unde, avnd
1
1
n vedere c
a [x] + x +
= [2x], deducem c
a [x + a] = x + , x R (1). De
2

1
1
1 3
1
aici, lund x = , obtinem a +
[1, 2), adic
a a
,
[0, 1) =
,1 .
2
2

2
2 2
1
1
1
si avem 0 < x0 + <
Dac
aa
, 1 , atunci putem alege un x0 1 a,
2
2
2

1
a x0 +
< 1 < x0 + a. n acest caz ns
= 0, iar [x0 + a] 1, deci relatia (1) nu
2
1
este valabil
a. Prin urmare, avem a = , b = 2, valori care verific
a egalitatea dat
a.
2
d un unghi oarecare si P un punct n interiorul sau. Se considera
G43. Fie xOy
punctele A, B [Ox cu A (OB) si C, D [Oy cu C (OD) astfel nct triunghiurile P AB si P CD sa fie echilaterale. Aratati ca dreptele OP , AD si BC sunt
concurente daca si numai daca P se afla pe bisectoarea unghiului dat.
Temistocle Brsan, Iasi
Solutie. Fie OP BD = {M }, P R CD,
D
P Q AB (R CD, Q AB) si P Q = d1 , P R = d2 ,
R
OP = d. Avem:
C d2
q
MB
OB d1
OB sin 1
=
, OQ = d2 d21 ,
=
M
MD
OD sin 2
OD d2
P
q

2
d
OA = OQ AQ = d2 d21 d1 / 3,
d1

q
1

2
2
OB = OQ + QB = d d1 + d1 / 3,
O
A Q B
67

q
q
q

OR = d2 d21 , OC = d2 d22 d2 / 3, OD = d2 d22 + d2 / 3.


Cu aceste observatii, putem scrie succesiv: OP , AD si BC sunt concurente

M B CD AO
OB d1 2d2 / 3
OA
= 1 OA OB = OC OD

=1

M D CO AB
OD d2
OC
2d1 / 3
q
q
q

2
2
2
2
2
2
2
2
d d1 d1 / 3
d d1 + d1 / 3 = d d2 d2 / 3
d d2 + d2 / 3
d2 d21 d21 /3 = d2 d22 d22 /3 d1 = d2 1 = 2 .

G44. Fie V ABC o piramida, iar G centrul de greutate al 4ABC. Un plan


ce trece prin G taie dreptele V A, V B, V C n A0 , B 0 si respectiv C 0 . Sa se arate ca
VB
VC
VA
+
+
= 3.
0
0
VA
VB
V C0
Constantin Cocea, Iasi
Solutie. Fie {N } = B 0 C 0 BC si {M } = A0 C 0 AC.
V
Aplicnd teorema lui Menelaus n triunghiurile V AC
A0 A
C 0V
MC
si V BC, obtinem:
0
= 1 si
C
0
A
V
C
C
MA
0
0
B B C V NC
M
A
C

= 1, de unde rezult
a c
a
B0V C 0C N B

G
CC 0 AM
NB
CC 0
A0 A B 0 B
+
=
sau
+
=
N
B
0
0
0
0
VA
VB
V C MC
NC
VC
0
0
0
VA VA VB VB
VC VC
+
=
sau
A
V A0
V B0
V C0
B
VA
VB
VC
AM
BN
+
+
=
3.
(Am
folosit
c
a
G

M
N
implic
a
+
= 1.)
0
0
0
VA
VB
VC
MC
NC
G45. Fie SABC un tetraedru n care 4ABC nu este echilateral, iar muchiile [SA] , [SB] , [SC] nu sunt toate congruente. Demonstrati ca exista sase puncte
A1 , B1 , C1 , A2 , B2 , C2 pe dreptele SA, SB, SC, BC, AC si respectiv AB astfel ca patrulaterele A1 B1 A2 B2 , B1 C1 B2 C2 si A1 C1 A2 C2 sa fie trapeze isoscele (A1 B1 kA2 B2 ,
A1 C1 kA2 C2 , B1 C1 kB2 C2 ) daca si numai daca

SA2 AB 2 AC 2 + SB 2 BC 2 BA2 + SC 2 CA2 CB 2 = 0.


Daly Marciuc, Satu Mare
Solutie. S
a presupunem c
a A1 B1 A2 B2 , B1 C1 B2 C2 si A1 C1 A2 C2 sunt trapeze
isoscele n modul indicat. Din A1 B1 k A2 B2 rezulta ca B2 A2 k AB si apoi, analog,
rezulta ca B2 C2 k BC si A2 C2 k AC. De aici, deducem ca AB2 A2 C2 si BA2 B2 C2
sunt paralelograme, deci C2 este mijlocul lui AB. Analog, obtinem c
a B2 si A2 sunt
mijloacele laturilor AC si BC.
Din A1 B1 k AB, A1 C1 k AC si B1 C1 k BC rezulta ca
A1 A
B1 B
C1 C
=
=
= k.
(1)
SA
SB
SC
Notnd BC = a, AC = b si AB = c, avem: A1 B22 = A2 B12
b2
a2
SA2 + b2 SC 2
SB 2 + a2 SC 2
k 2 SA2 +
k
= k 2 SB 2 +
k
, deci
4
2
4
2
2
2
2
2
2
2
(2)
A1 B2 = A2 B1 2k SB SA = a b .
68

n mod analog, g
asim echivalenta:

B1 C22 = C1 B22 2k SB 2 SC 2 = c2 b2 .

n fine, din (2) si (3) rezult


a c
a
2

2
2
SA c b + SB 2 a2 c2 + SC 2 b2 a2 = 0.

(3)

(4)

Reciproc, relatia (4) poate fi scrisa astfel:

a2 b2
c2 b2
not
= k.
=
2
2
2 (SB SA )
2 (SB 2 SC 2 )

(5)

Alegem A1 , B1 , C1 pe SA, SB, SC astfel nct s


a avem relatia (1). n acest caz din
(5) rezulta ca A1 B1 A2 B2 si B1 C1 B2 C2 sunt trapeze isoscele, unde A2 , B2 si C2 sunt
mijloacele laturilor BC, AC si AB (A1 B1 k AB k A2 B2 etc.). Daca A1 B1 A2 B2 si
a c
a A1 A2 = B1 B2 si B1 B2 = C1 C2 , deci
B1 C1 B2 C2 sunt trapeze isoscele nseamn
A1 A2 = C1 C2 , adic
a si A1 C1 A2 C2 este isoscel.

B. Nivel liceal
L36. Fie 4ABC si M triunghiul sau median. Daca P este un punct aflat n
interiorul sau pe laturile lui M, iar A0 , B 0 , C 0 sunt intersectiile dreptelor AP , BP ,
1
8
AP BP CP
CP cu laturile BC, CA si respectiv AB, atunci <

.
4
AA0 BB 0 CC 0
27
Marian Ionescu, Pitesti
Solutie. Not
am S1 = (P BC), S2 = (P CA), S3 = (P AB) si S = (ABC).
S2 + S3
AP
si analoagele si se deduce relatia lui
=
Se stabilesc cu usurinta relatia
AA0
S
AP
BP
CP
Gergonne
+
+
= 2.
Cu inegalitatea mediilor obtinem
BB 0
CC 0
r AA0
AP BP CP
2 33

, de unde deducem a doua parte a dublei inegalitati din


AA0 BB 0 CC 0
enunt. Pentru prima parte, observ
am mai nti c
a, dac
a P se afl
a n interiorul sau pe
laturile triunghiului M, au loc inegalit
atile S2 +S3 S1 , S3 +S1 S2 si S1 +S2 S3 .
1
1
1
Notnd x =
(S2 + S3 S1 ), y =
(S3 + S1 S2 ), z =
(S1 + S2 S3 ),
2
2
2
t = x + y + z si observnd ca x 0, y 0, z 0 (numai unul poate fi nul),
t > 0, avem:
AP BP CP
(S2 + S3 ) (S3 + S1 ) (S1 + S2 )

=
=
AA0 BB 0 CC 0
S3
t3 + t2 (x + y + z)
2t3
1
(t + x) (t + y) (t + z)
>
=
= .
=
8t3
8t3
8t3
4
Not
a. Solutie corect
a s-a primit de la Marius Pachitariu, elev, Iasi.
Not
a. Aceasta problema apare n articolul "About elementary inequalities in triangle" (M. Dinc
a, M. Bencze) din revista Octogon Math. Magazine, 9 (2001),
no. 1B, p. 472. Aici nu se cere ca punctul P s
a fie n interiorul sau pe laturile triunghiului M, dar solutia prezentat
a este incorect
a.

L37. Fie cercurile C1 , C2 si C astfel nct C1 si C2 sunt tangente exterior n


D, iar cercurile C1 si C2 sunt tangente interior lui C n B, respectiv C. Tangenta
69

comuna interioara cercurilor C1 si C2 taie cercul C n A si A1 , dreapta AB taie C1


1
2
1
n K, iar AC taie C2 n L. Sa se arate ca
=
+
.
DA DA1
KL
Neculai Roman, Mircesti (Iasi)
Solutie. Fie {M } = C1 BC, {N } = C2 BC si T
A
un punct pe tangenta n B la cercurile C1 si C2 .
C
Ar
at
am c
a dreapta KL este tangenta comun
a
exterioara cercurilor C1 si C2 . ntr-adevar, avem
K
\
\
[ ) = m(CAB),
\ deci
m(M
KB) = m(M
BT ) = m(CBT
L
\
\ Cum
M K k CA. Ca urmare, M
KL KLA.
D
\ CBA,
\ deoarece 4KLA 4CBA (fapt ce
KLA
N C
2
M
decurge din AK AB = AL AC = AD ), rezulta ca
C2
\
\ Deci M
\
\
M
KL CBA.
KL M
BA, adic
a KL este
B
C1
tangenta la cercul C1 . Analog se arata ca drepta KL
A1
T
este tangenta la C2 .
Aplic
am teorema lui Casey pentru cercurile C1 , C2 , cercurile degenerate A, A1
tangente interior la C si obtinem relatia AD A1 D + AD A1 D = AA1 KL sau
AA1
2
1
1
2
=
, adic
a
=
+
.
KL
AD A1 D
KL
AD A1 D
L38. Fie 4ABC si punctele D, D0 BC conjugate armonic n raport cu vrfurile B si C. Cercul circumscris 4ADD0 intersecteaza AB n M si AC n N .
b
Aratati ca, daca M N BC, atunci [AD si [AD0 sunt bisectoarele unghiului A

b
(interioara si exterioara) sau m(A) = 90 .
Temistocle Brsan, Iasi
Solutie. Avem M N DD0 M D2 + N D02 = M D02 + N D2 (1). Daca notam
DB
D0 B
a
= 0 = , atunci BD =
, CD =
M
DC
DC
1+
a
a
a
0
0
, BD =
, CD =
(2). Ex=
A
1+
1
1
primnd puterea punctelor B si C fata de cercul
(ADD0 ), vom obtine relatiile: c BM = BD BD0
D
si b CN = CD CD0 sau
B
D
C
2 2
2
a
a
N
si CN =
BM =
. (3)
c (2 1)
b (2 1)
Utiliznd teorema cosinusului n 4BM D, 4CN D0 , 4BM D0 si 4CN D, (1) se scrie

BM 2 + BD2 2BM BD cos B + CN 2 + CD02 2CN CD0 cos C =


= BM 2 + BD02 2BM BD0 cos B + CN 2 + CD2 + 2CN CD cos C

43 a3
4a3
si, tinnd seama de (2) si (3), g
cos B
cos C = 0.
asim 4 2 1 a2 +
c
b
Din nou utiliznd teorema cosinusului, obtinem

2 1 2b2 c2 + 2 b2 a2 + c2 b2 c2 a2 + b2 c2 = 0 sau

b2 a2 b2 c2 2 c2 a2 b2 c2 = 0,
c
a concluzia.
ultima echivalent
a cu = sau a2 = b2 + c2 , de unde rezult
b
70

L39. Determinati toate numerele naturale nenule n pentru care


patrat perfect, unde a, p N .

an (an + 2)
este
p (p + 1)

Mihai Haivas, Iasi


a2 n2 + 2an
2
2

Solutie. Fie
= y , y N . Avem (an + 1) p (p + 1) y 2 = 1, de
p (p + 1)
unde, cu x = an + 1, obtinem ecuatia lui Pell: x2 p (p + 1) y 2 = 1, care are solutia
fundamentala (x0 , y0 ) = (2p + 1, 2) si solutia generala

k
k
p
p
1
xk =
=
x0 + y0 p (p + 1) + x0 y0 p (p + 1)
2

k
k
p
p
1
=
=
2p + 1 + 2 p (p + 1) + 2p + 1 2 p (p + 1)
2

1 p
2k p
2k
=
p+1+ p
+
p+1 p
,
2

k
k
p
p
1
x0 + y0 p (p + 1) x0 y0 p (p + 1)
yk = p
=
2 p (p + 1)

p
1
2k p
2k
p+1+ p

p+1 p
.
= p
2 p (p + 1)
Prin urmare, avem:

1 p
2k p
2k
nk =
p+1+ p
+
p+1 p
2
2a
h
i
2k
2k
care este solutie dac
a 2a |
p+1+ p
+
p+1 p
2 .
Not
a. Solutie corect
a s-a primit de la Marius Pachitariu, elev, Iasi.

L40. Fie A, B Mn (Z) astfel nct det A2 B + AB 2 este impar. Sa se arate


ca A + B este inversabila pentru orice Q.
Marian Urs
arescu, Roman

Solutie. Deoarece det A2 B + AB 2 = det A det (A + B) det B este un numar


impar, rezult
a c
a det A, det (A + B) si det B sunt numere impare. Fie polinomul p (X) = det (A + XB) = det A + a1 X + + an1 X n1 + (det B) X n . Cum
p (1) = det (A + B) = det A + a1 + + an1 + det B este numar impar, nseamna
ca si a1 + a2 + + an1 este numar impar. Sa presupunem acum ca polinomul
p
p are o r
ad
acin
a rational
a = , cu p, q Z, q 6= 0, (p, q) = 1. n acest caz,
q
avem p | det A si q | det B, deci p si q sunt impare. Din p () = 0, rezult
a c
a
n1
n1
n
n
n
+

+
a
p
q
+
(det
B)
p
=
0,
sau
(det
A)
q
+
(det
B)
p
+
(det A) q n + a1 pq
n1

+a1 pn1 q 1 + + an1 pn1 q 1 = (a1 + + an1 ), egalitate care este


fals
a deoarece membrul din stnga este par, iar cel din dreapta este impar. Prin
urmare p () = det (A + B) 6= 0, pentru orice num
ar rational , adic
a matricea
A + B este inversabila oricare ar fi Q.
L41. Demonstrati ca grupul simetric S32 nu are elemente de ordin 2002.
Paul Georgescu si Gabriel Popa, Iasi
Solutie. Presupunem c
a exist
a S32 un element de ordin 2002. Fie =
= 1 2 n descompunerea sa n produs de cicli disjuncti cu ordinele k1 , k2 , . . . ,
71

kn . Avem k1 + k2 + + kn = 32 si [k1 , k2 , . . . , kn ] = 2002. Cum 2002 = 2 7 11 13,


rezulta ca exista ki1 , ki2 , ki3 , ki4 , nu neaparat distincte, astfel nct 2 | ki1 , 7 | ki2 ,
11 | ki3 , 13 | ki4 . Daca ki1 , ki2 , ki3 , ki4 sunt distincte, atunci ki1 +ki2 +ki3 +ki4 33,
ceea ce este fals. Dac
a dou
a, sau mai multe, din cele patru ordine coincid, atunci
ordinul corespunz
ator se divide cu produsul factorilor ce-i corespund, fiind mai mare
sau egal dect produsul acelorasi factori si deci mai mare sau egal dect suma lor.
Astfel, n acest caz obtinem iarasi ca suma ordinelor este mai mare sau egala cu 33,
ceea ce este fals.
L42. Fie (A, +, ) un inel comutativ
si finit, cu celputin 5 elemente

si cu
1 + 1 A inversabil. Fie M = x A | x2 = 1 , I = x A | x2 = x . Sa se
arate ca card M = card I < card A /2.
Ovidiu Munteanu, Brasov
Solutie. Dac
a a A, atunci 21 (1 + a) A si avem: 21 (1 + a) I
2
1

2 (1 + a) = 21 (1 + a) 22 1 + 2a + a2 = 21 (1 + a) 1 + 2a + a2 =
= 2 (1 + a) a2 = 1 a M , de unde rezult
a egalitatea card M = card I.
Sa demonstram acum partea a doua a relatiei date. Daca avem card I = 2, atunci
card A 5 > 2 card I. n continuare ne ocupam de cazul n care card I > 2. n
aceast
a situatie, fie a I \ {0, 1} si atunci 1 a I \ {0, 1, a}. ntr-adev
ar, dac
a
1a = a, rezult
a c
a a = 21 , adic
a a este inversabil si din a2 = a obtinem a = 1, ceea
ce este fals. Avem deci card I > 3. Fie J = {x A | x I} si atunci I J = {0},
pentru ca x I J nseamna x = x = x2 , deci 2x = 0, adica x = 0. Pe de alta
parte, avem I M = {1} si J M = {1}. Cum I, J, M au acelasi num
ar de
elemente, rezult
a c
a are loc card A 3 card I 3 > 2 card I.
L43. Determinati polinoamele P R [X] pentru care P (z) C\R, z C\R.
Gheorghe Iurea, Iasi
Solutie. Polinoamele de gradul 1, P (X) = aX + b (a, b R, a 6= 0) verific
a
ipoteza, deci sunt solutii ale problemei. Aratam ca acestea sunt singurele solutii.
Fie P R [X] cu grad P = n 2 si f : R R, f (x) = P (x) = a0 xn + a1 xn1 +
+ + an (a0 6= 0) functia polinomial
a asociat
a acestuia. Fie a0 > 0 (la fel se va
proceda dac
a a0 < 0). Observ
am c
a m R ecuatia f (x) = m are numai solutii
reale (n solutii), n caz contrar ar exista z C\R si f (z) = m R.
Daca n este par, atunci lim f (x) = . De aici si din continuitatea lui f , rezulta
x

c
a Im f = [m, ), unde m = inf {f (x) ; x R}. Pentru k < m ecuatia f (x) = k nu
are solutii reale, fals.
Dac
a n este impar, avem f 0 (x) = na0 xn1 + , deci lim f 0 (x) = + si
x

f 0 (x) > 0 pentru |x| suficient de mare. Deci f este strict cresc
atoare pe intervalele
(, ) si (, ) (, R convenabil alesi). De aici, din continuitatea functiei
f (deci marginirea ei pe orice interval [, ]) si din faptul ca lim f (x) = +,
x+

deducem ca R astfel nct f () f (x), x (, ]. Ca urmare ecuatiile


f (x) = k, cu k > f () au solutie real
a unic
a, fals.
L44. Fie n 2 numar natural, iar f0 , f1 , f2 , . . . un sir de polinoame definit
prin f0 = (X + 1)n , fp+1 = X fp0 , p 0. Definim nca hp = fp 1p1 fp1 +
X
p1 p1
+ + (1)
p1 f1 , p 1, unde nk =
i1 i2 . . . ik , k {1, 2, . . . , n}
1i1 <<ik n

72

sunt sumele simetrice fundamentale ale numerelor 1, 2, . . . , n. Sa se arate ca


hp = n (n 1) (n p + 1) X p (X + 1)np , p = 1, 2, . . .
Marian Tetiva, Brlad
p
X
Solutie. Sa aratam ca hp+1 = Xh0p php . Avem hp+1 =
(1)k pk fp+1k =
k=0

p
p1
p

X
X
X
k p1
p1
=
(1)k p1
+
p
=
(1)

f
+
(1)k p p1
f
p+1k
p+1k
k
k1
k
k1 fp+1k =
k=0
p1
X

= X

k=0

k=0

0
(1) p1
fpk
p
k

p
X

k=1

k1

(1)

k=1

0
p1
k1 fp+1k = Xhp php (am considerat

n0 = 1 si nk = 0, pentru k < 0 sau k > n).


Demonstratia se poate face prin inductie si se bazeaz
a pe formula stabilit
a. Direct
n1
din enunt, deducem c
a h1 = f1 = Xf00 = nX (X + 1)
. S
a presupunem acum c
a
are loc egalitatea hp = n (n 1) (n p + 1) X p (X + 1)np . n acest caz, putem
scrie:
np
hp+1 = Xh0p php = n (n 1) (n p + 1) pX p (X + 1)
+
+n (n 1) (n p + 1) (n p) X p+1 (X + 1)np+1
pn (n 1) (n p + 1) X p (X + 1)np =

= n (n 1) (n p + 1) (n p) X p+1 (X + 1)
n

np+1

n1

Sa mai observam ca, deoarece hn = n! X , hn+1 = Xn! nX


n n! X n = 0, rezulta
c
a hp = 0, pentru orice p n + 1.
L45. Fie f : [0, ) [0, ) continua. Daca functia F : [0, ) R, F (x) =
Z x
Z 1
=
f (t) dt este marginita, sa se arate ca lim n xf (nx) dx = 0.
n
0
0
Adrian Zanoschi, Iasi
Z 1
Solutie. Dac
a n integrala In =
nxf (nx) dx facem schimbarea de variabil
a
0
Z n
t
nx = t, obtinem In =
f (t) dt. Fie (0, 1). Avem
n
0
Z n
Z n
Z n
Z n
n
n
f (t) dt +
f (t) dt.
(1)
f (t) dt +
f (t) dt =
|In |
n
0
n n
0
n
Deoarece F este m
arginit
a, exist
a M > 0 astfel nct F (x) < M , x R+ (2). Cum
f este continua, rezulta ca F este derivabila si F 0 (x) = f (x) 0, x R+ , deci F
este crescatoare. De aici, avnd n vedere marginirea functiei F , deducem
ca exista
Z
x

lim F (x) si este finita. Prin urmare, lim (F (x) F (x)) = lim

f (t) dt = 0,

de unde rezult
a c
a exist
a n0Z N astfel nct
Z nn n0 are loc:
n

f (t) dt =
f (t) dt < .
(3)

n
nZ n
f (t) dt + < (M + 1), n n0 ,
Din relatiile (1), (2), si (3) obtinem |In | <
0
ceea ce nseamn
a c
a lim In = 0.
n

73

Probleme propuse
Clasele primare
P.64. ntr-o pies
a de teatru sunt 12 personaje, copii si adulti. Cti copii joac
a n
pies
a, dac
a la fiecare doi adulti corespunde un copil?
( Clasa I )
Alexandra Radu, elev
a, Iasi
P.65.

Se dau jetoanele AT

II

CRE

TII

ATII

RECR

EA

RE

REC . Care este num


arul cel mai mare de jetoane cu care se poate forma cuvntul

"RECREATII"?
( Clasa I )
Oxana Pascal, elev
a, Rep. Moldova
P.66. ntr-o livad
a sunt tot attia peri ct si meri. Sunt 6 rnduri cu peri si 4
rnduri cu meri. Numarul merilor de pe un rnd ntrece cu 5 numarul perilor de pe
un rnd. Cti pomi sunt n acea livad
a?
( Clasa II-a)
nv. Maria Racu, Iasi
P.67. Dintr-o multime de 5 copii, orice grupare de trei contine cel putin o fat
a.
Cti b
aieti pot fi n multime?
( Clasa II-a)
Andreea Surugiu, elev
a, Iasi
P.68. Dac
a Ina ar mp
arti num
arul nucilor culese de ea la num
arul nucilor culese
de sora sa, ar obtine 7 rest 6. S
tiind c
a Ina a cules cu 78 nuci mai mult dect sora
sa, aflati cte nuci a cules fiecare.
( Clasa III-a)
nv. Doinita Spnu, Iasi
P.69. ntr-o mpartire cu rest, n care mpartitorul este mai mare ca noua, marind
mpartitorul cu o unitate si efectund din nou mpartirea obtinem ctul 9 si restul
0. Aflati ctul si restul mp
artirii initiale.
( Clasa III-a)
nv. Mariana Toma, Muncelu de Sus (Iasi)
P.70. ntr-o tab
ar
a international
a de matematic
a sunt elevi din patru tari: Bulgaria, Grecia, Republica Moldova si Romnia. Dac
a 21 elevi nu sunt din Bulgaria,
23 nu sunt din Grecia, 22 elevi nu sunt din Republica Moldova si 21 elevi nu sunt
din Romnia, cti elevi sunt din fiecare tara?
( Clasa III-a)
Georgiana Ciobanu, elev
a, Iasi
P.71. Fiecare patrat din figura alaturata se coloreaza cu o alta culoare. n
cte moduri putem face acest lucru avnd la dispozitie patru culori?
( Clasa IV-a)
nv. C
at
alina Rata
, Coarnele Caprei (Iasi)
P.72. Arunc
am dou
a zaruri si adun
am punctele de pe cele dou
a fete de deasupra.
a) Cte sume diferite putem obtine? b) Cte sume se pot forma n trei moduri
diferite?
( Clasa IV-a)
nv. Gheorghe Toma, Muncelu de Sus (Iasi)
P.73. n figura alaturata este pus n evidenta un
B
drum format din sase segmente care pleaca din A si
ajunge n B. Cte drumuri de felul acesta se pot A
construi?
( Clasa IV-a)
nv. Constantin Rata
, Coarnele Caprei (Iasi)
74

Clasa a V-a
V.46. Aflati n N pentru care 11n + 9n si 11n 9n sunt simultan patrate
perfecte.
Andrei - Sorin Cozma, elev, Iasi
V.47. Sa se arate ca numarul 51a51a nu poate fi scris ca produsul a patru numere
prime.
C
at
alin Budeanu, Iasi
9
10
11
V.48. Se consider
a fractiile x1 =
, x2 =
, x3 =
, . . . . Scrieti fractia
14
21
28
ator primele 1000 de fractii.
x1000 si apoi ordonati cresc
Dumitru Gherman, Pascani
V.49. Determinati numarul tripletelor (a, b, c) N3 daca 3a + 2b + c = 598 si
a + 2b + 3c = 602. Dac
a n plus a < b < c, determinati a, b si c.
Gheorghe Iurea, Iasi
V.50. Cte numere de 7 cifre se pot scrie folosind cifrele 1, 2 si 3, astfel nct 1
sa apara de 2 ori, 2 sa apara de 3 ori si 3 sa apara de 2 ori? Dar daca n locul cifrelor
1, 2 si 3 consider
am cifrele 0, 1 si respectiv 2?
Petru Asaftei, Iasi

Clasa a VI-a
VI.46. Suma dintre opusul unui numar natural si inversul altui numar natural
este 119, 992. S
a se determine numerele.
Ciprian Baghiu, Iasi
VI.47. Aflati restul mpartirii numarului N = 28442844 + 41074107 + 63986398
prin 79.
Tamara Culac, Iasi
VI.48. a) ntr-o proportie cu termeni nenuli, un extrem este suma celorlalti
trei termeni daca si numai daca celalalt extrem are inversul egal cu suma inversilor
celorlalti trei termeni.
b) Dac
a din patru numere rationale nenule distincte unul este suma celorlalti trei,
iar altul are inversul egal cu suma inverselor celorlaltor trei, atunci numerele sunt
termeni ai unei proportii.
Claudiu - S
tefan Popa, Iasi
VI.49. Sa se arate ca orice numar natural relativ prim cu 10 admite un multiplu
care se scrie folosind numai cifra 3.
Lucian - Georges L
adunc
a, Iasi
VI. 50. Fie 4ABC cu [AC] [BC], D mijlocul lui [AB], P un punct pe dreapta
AB, iar M si L picioarele perpendicularelor din P pe AC, respectiv BC. Sa se arate
c
a [DM ] [DL].
Neculai Roman, Mircesti (Iasi)

Clasa a VII-a
VII.46. Sa se rezolve n R inecuatiile:
a) x100 + x77 + x50 + x21 + x10 + x5 + 1 > 0;
b) x100 x77 + x50 x21 + x10 x5 + 2 < 0.
Vasile Solcanu, Bogd
anesti (Suceava)
75

VII.47. S
a se rezolve n Z2 ecuatia u2 v + uv 2 = 2u2 + 2v 2 40.
Mihai Cr
aciun, Pascani

VII.48. Dac
a ai = i + i, i = 1, 2004, precizati dac
a num
arul
N = a1 a2 a3 + a4 + a5 a6 a7 + a8 + + a2001 a2002 a2003 + a2004
este negativ, pozitiv sau nul.
Viorel Cornea si Dan S
tefan Marinescu, Hunedoara
VII.49. Fie 4ABC echilateral si D (BC). Not
am cu M1 , M2 mijloacele
segmentelor [BD], respectiv [CD]. Paralela prin M1 la AC intersecteaza AB n F ,
iar paralela prin M2 la AB intersecteaza AC n E. Sa se arate ca dreptele AD, M1 E
si M2 F sunt concurente.
Nicolae Gross si Lucian Tutescu, Craiova
VII. 50. Fie ABCD un trapez cu bazele [AB] si [CD]. O paralela la baze
intersecteaz
a AD, AC, BD si BC n punctele E, F , G si respectiv H. S
a se arate
c
a EH = 3F G dac
a si numai dac
a DF , CG si AB sunt drepte concurente.
Adrian Zanoschi, Iasi

Clasa a VIII-a

m
n
VIII.46. Sa se demonstreze ca nu exista m, n N pentru care
+
= 2003.
n
m
Alexandru Negrescu, elev, Botosani

VIII.47. Pentru x (0, ), s


a se demonstreze inegalitatea

5 3 2 3 2 4 3
x +x +x +1 x +x +2 + x +x +x+1 x3 +x+2 + x3 +x2 +x+1 x2 +x+2
6.
x6 + x5 + x4 + 2x3 + x2 + x + 1
Mircea Cosbuc, elev, Iasi
VIII.48. Gasiti numerele prime p si q pentru care p2 + q = 37q 2 + p.
Liviu Smarandache, Craiova
VIII.49. Fie 4ABC dreptunghic n A cu AB = AC = a. Consideram M A

\
(ABC), M A = a 2 si N AM astfel nct m(CN,
BM ) = 60 . S
a se afle
lungimea segmentului [AN ].
Romanta Ghita
si Ioan Ghita
, Blaj
b = m(C)
b = 90 ,
VIII.50. Fie patrulaterul convex ABCD cu AB = BC, m(A)
b 90 si fie O mijlocul lui [BD]. Pe perpendiculara n O pe planul (ABC) se
m(B)
ia un punct V astfel nct OV = OB. S
a se arate c
a d (D, (V AB)) = 2 d (D, (V AC))
\ = 60 .
dac
a si numai dac
a m(ABC)
Monica Nedelcu, Iasi

Clasa a IX-a
IX.46. S
a se rezolve ecuatia

x 1 + 3 x 2n x 2 = 2, unde n N .
Dan Popescu, Suceava

IX.47. Sa se determine sirul (an )n1 de numere strict pozitive pentru care
a21 a22 + a23 + (1)

n1

a2n = (1)

n1

(a1 + a2 + + an ) , n 1.
Marian Urs
arescu, Roman

76


IX.48. Fie a, b, c (0, ) cu a + b + c + abc = 4. S
a se arate c
a
2
2
2
b
3
a
c
+
+
.
2
b
+
ca
a + bc
c + ab
Cezar Lupu, elev, Constanta
IX.49. Sa se arate ca 4ABC este isoscel n fiecare din ipotezele:
a) 2ma + b = 2mb + a; b) 2ma + a = 2mb + b.
Marius Pachitariu, elev, Iasi
IX.50. Fie I centrul cercului nscris n triunghiul ascutitunghic ABC. Daca A, B,

C sunt masurile n radiani ale unghiurilor triunghiului, iar A IA+B IB+C IC = 0 ,


s
a se arate c
a 4ABC este echilateral.
Constantin Micu, Melinesti (Dolj)

Clasa a X-a
2

X.46. S
a se determine a R astfel nct ecuatia 2x1 + 2x

y 2 + ay + a2
s
a
y 2 + a2

aib
a solutii n Z Z.

Petru R
aducanu, Iasi
X.47. Fie z1 , z2 , z3 C distincte, cu z2 +z3 = 2 si astfel nct |z1 1| = |z2 1| =
= |z3 1|. S
a se arate c
a (z1 z2 ) (
z1 z3 ) este num
ar complex pur imaginar.
Lidia Nicola, Craiova
X.48. Se considera planele paralele si aflate la distanta h unul de celalalt si
4ABC echilateral inclus n planul .
a) S
a se afle locul geometric al punctelor M pentru care M A2 + h2 =
2
= M B + M C2.
b) Sa se determine M astfel nct suma M A2 + M B 2 + M C 2 sa fie minima.
Viorel Cornea si Dan S
tefan Marinescu, Hunedoara
3
3
3
X.49. S
a se arate c
a sin x + sin y + sin z 3 sin x sin y sin z
3
[sin x (1 cos (y z)) + sin y (1 cos (z x)) + sin z (1 cos (x y))] ,
4
x, y, z [0, /3].
Marian Tetiva, Brlad
X.50. Fie ak , bk , ck N, k 1, n; notam cu f (p) numarul tripletelor (A, B, C)
de submultimi (nu neap
arat nevide)
a
Pcu reuniunea
P M = {1,
P 2, . . . , n}, oricare dou
disjuncte si astfel nct num
arul
ai +
bi +
ci p s
a fie multiplu
iM\A
iM\B
iM\C
P
de 3 (convenim ca
xi = 0). Ar
atati c
a dac
a f (0) = f (1) = f (2), atunci exist
ai
i
..
pentru care ai + bi + ci . 3.
Gabriel Dospinescu, student, Bucuresti

Clasa a XI-a
XI.46. Determinati A, B Mn (Z) pentru care det (A + B) = 2 si det (A + 3B) = 5.
Cezar Lupu, elev, Constanta

a
a, dac
ai=j
XI.47. Fie A Mn (R) matrice cu aij =
, unde b 6= 0 si
/ Z.
b, daca i 6= j
b
Ar
atati c
a A este inversabil
a si determinati A1 .
Gheorghe Iurea, Iasi
77

XI.48. Se defineste sirul (xn )n0 prin xn = x2n1 [xn1 ], n 1; x0




0, 1 + 5 /2 . S
a se arate c
a lim xn = 0.
n
aeru, Suceava
C
at
alin Tig

P
XI.49. Fie (xn )n0 , (an )n0 siruri de numere reale astfel nct
an < ,
n=1

a se arate c
a (xn )n0
|xn+1 2xn + xn1 | + |xn+1 3xn + 2xn1 | an , n 1. S
este convergent.
Paul Georgescu si Gabriel Popa, Iasi

nx + y
XI.50. Fie n 2 N, iar f : R R o functie cu proprietatea c
af

n+1

f ( n+1 xn y), x, y R. Sa se arate ca functia este descrescatoare pe (, 0] si


crescatoare pe [0, ). (n legatura cu Problema 2819 din Crux Mathematicorum,
nr. 2/2003.)
Titu Zvonaru, Bucuresti

Clasa a XII-a
XII.46. Sa se determine functia f : R R daca (R, ) este grup abelian
cu proprietatea c
a simetricul oric
arui element x [1, 1] se afl
a n [1, 1], unde
x y = f (x) + f (y), x, y R.
Ioan S
ac
aleanu, Hrl
au
XII.47. Fie G = (a,b), a,b R, iar "" nmultirea numerelor reale. Sa se
determine a, b astfel nct R+ ,
= (G, ) printr-un izomorfism de forma f : R+ G,
x +
f (x) =
, x R+ , cu , , , R.
x +
Alexandru Blaga si Ovidiu Pop, Satu Mare
XII.48. Fie (G, ) grup de element neutru e si x, y G penrtru care avem:
a) k N \ {1} a. . xk = e;
b) p N \ {1} a. i. xy = y p x.
S
a se arate c
a:
1) xy n xk1 = y np , n N ;
2) xy = yx y n(p1) = e, n N .
Mihai Haivas, Iasi
XII.49. Se considera numerele reale b > a 0, c 1 si functiile continue
Z nb
f, g : R+ R+ astfel nct lim
g (x) dx = d R. S
a se arate c
a sirul (un )n1 ,
n na
Z b
1
un =
dx este convergent si sa se afle limita sa.
c
+
f
(x)
+ g (nx)
a
D. M. B
atinetu - Giurgiu, Bucuresti
s (n!)
,
XII.50. Fie s (n) suma cifrelor num
arului natural n. S
a se calculeze lim k
n ln ln n
unde k N este fixat.
Gabriel Dospinescu, student, Bucuresti

78

Probleme pentru preg


atirea concursurilor
A. Nivel gimnazial
G56. Fie m Z, n 2 Z + 1 fixate. Sa se arate ca ecuatia nx + y = m, x, y Z
are o unica solutie (x0 , y0 ) cu proprietatea ca |y0 | < |n| /2.
Petru Asaftei, Iasi
G57. Un seic a l
asat mostenire celor doi fii ai s
ai cinci c
amile, cu conditia ca
unul sa primeasca jumatate, iar celalalt o treime. Mostenitorii nu si-au putut mparti
averea, asa ca au apelat la un ntelept care trecea pe acolo, calare pe o camila. Cum
a procedat nteleptul?
Cte probleme asem
an
atoare mai putem formula (n care mostenirea este de n
camile, iar fiii primesc a p-a si a q-a parte)?
Gabriel Popa, Iasi
2
x
y
G58. S
a se rezolve n N ecuatia 2 + 1 = 5 .
Irina Mustata
a, si Valentina Blendea, Iasi
, elev
G59. Fie A = {n N | s (2000n) + s (2002n) = 2s (2001n)}, unde prin s (x) am
notat suma cifrelor lui x. Demonstrati ca orice numar natural nenul are un multiplu
ce apartine lui A.
Gabriel Dospinescu, student, Bucuresti
G60. Sa se demonstreze ca pentru orice a, b, c (0, ) are loc
ab
bc
ca
1
4abc
2 +
2 +
2 4 + (a + b) (b + c) (c + a) .
(a + b)
(b + c)
(c + a)
Gabriel Dospinescu, student, Bucuresti
G61. S
a se demonstreze c
a pentru orice a, b, c (0, ) are loc
p

(a2 +b2 )(b2 +c2 )(c2 +a2 )


a+b b+c c+a
(a+b)(b+c)(c+a)
54 2
+
+
27
.
c
a
b
abc
abc
Marian Tetiva, Brlad
G62. Fie ABCD un patrulater convex n care se poate nscrie patratul M N P Q
de centru O (M (AB), N (BC), P (CD), Q (AD)). S
a se arate c
a
AB + BC + CD + DA 2 (AO + BO + CO + DO). Cnd are loc egalitatea?
Lucian Tutescu, Craiova si Ioan S
erdean, Or
astie
b = 10 si m(B)
b = 100 construim M (AB) si
G63. n 4ABC cu m(A)
\
\
\
N (AC) astfel ca m(M
CB) = 40 si m(N
a se afle m(AM
N ).
BC) = 75 . S
Octavian Bondoc, Pitesti
G64. Prin punctul P al laturii (AC) a 4ABC se duc paralele la medianele AA0
si CC 0 , care intersecteaz
a laturile (BC) si (AB) n E, respectiv F . Fie {M } =
= EF AA0 , {N } = EF CC 0 , iar L si Q mijloacele segmentelor [F P ], respectiv
[P E]. Sa se arate ca dreptele M L, N Q si A0 C 0 sunt concurente.
Andrei Nedelcu, Iasi
G65. Fie SABCD o piramida cu baza ABCD dreptunghi, M proiectia lui D
pe SB si N proiectia lui C pe SA, iar {P } = AM N B. S
tiind c
a M (SB) si
N (SA), s
a se arate c
a N P SA M B = SM AN P B.
Daniel S
tefan Ninu, elev, Iasi
79

A. Nivel liceal
L56. Fie ABCD patrulater convex si {P } = AB CD, {Q} = AD BC.
Consideram J (AQ), L (BQ), K (DP ), N (AP ) astfel nct QJ = AD,
QL = CB, P K = DC si P N = AB. S
a se arate c
a JL k N K.
Carmen Nejneru, Iasi
L57. Fie 4ABC nscris n cercul C si punctele D (CB, D0 (BC astfel nct
\ Se mai considera cercul C1 tangent la AD, BD si la
\ ABC,
\ BAD
\0 ACB.
CAD
C, cercul C2 tangent la AD0 , CD0 si la C, iar {E} = C1 [BD], {F } = C2 [D0 C]. S
a
se arate c
a cercul circumscris 4AEF si cercul nscris n 4ABC sunt concentrice.
Neculai Roman, Mircesti (Iasi)
L58. Pe muchiile (Ox, (Oy si (Oz ale unui triedru oarecare se consider
a punctele
A, L (Ox, B, M (Oy si C, N (Oz astfel nct OA = OB = OC = a si OL =
\
\
\
= OM = ON = b (a < b). Not
am = m(Oy,
Oz), = m(Oz,
Ox), = m(Ox,
Oy)
si {P } = (AM N ) (BN L) (CLM ), {Q} = (LBC) (M CA) (N AB). Sa se
calculeze distanta P Q n functie de a, b, , , .
Temistocle Brsan, Iasi
L59. Care este probabilitatea ca latura si diagonalele unui romb, luate la ntmplare, s
a fie laturile unui triunghi?
Petru Minut, Iasi
L60. Fie A1 A2 . . . An si B1 B2 . . . Bn (n > 2) dou
a poligoane nscrise n acelasi
cerc de centru O si avnd centrele de greutate tot n O. S
a se arate c
a putem renumerota vrfurile poligonului A1 A2 . . . An pentru a obtine un nou poligon Ai1 Ai2 . . . Ain
n care Aij 6= Bj pentru j {1, 2, . . . , n}.
Gabriel Dospinescu, student, Bucuresti
L61. Fie n 3. Sa se determine maximul expresiei E = x31 x22 + x32 x23 + +
2(n1) 3 3
3 2
x1 x2 x3n , cnd numerele nenegative x1 , x2 , . . . , xn au suma 1.
+xn x1 +(n1)
Gabriel Dospinescu, student, Bucuresti
2
L62. Rezolvati ecuatia 2x = y (y + 1); x, y N.
Mircea Brsan, Iasi
L63. Fie G Mn (R) un grup netrivial n raport cu produsul uzual al matricelor.
Presupunem ca exista X G astfel nct pe fiecare linie, respectiv coloana a sa sa
existe cel mult un element nenul si acesta egal cu 1. Sa se demonstreze ca exista
k {1, 2, . . . n} astfel nct G este izomorf cu un subgrup al lui GLk (R) (s-a notat
GLn (R) = {A Mn (R) | det A 6= 0}).
Ovidiu Munteanu, Brasov
[xn+1 , xn ]
L64. Fie sirul (xn )n1 definit prin: x1 , x2 N , xn+2 =
, n 1. Daca
xn+1
a sirul nu este convergent.
x2003 = 2004, demonstrati c
Iuliana Georgescu si Paul Georgescu, Iasi
L65. Fie n N si functiile f, g : R R, unde f (x) = x2n cos (1/x), x < 0,
f (0) = 0, f (x) = x2n sin (1/x), x > 0, iar g (x) = x2n+1 sin (1/x), x < 0, g (0) = 0
si g (x) = x2n+1 cos (1/x), x > 0. Sa se afle cel mai nalt ordin de derivabiliate al
acestor functii si s
a se studieze problema continuit
atii acestor derivate n origine.
Gheorghe Costovici, Iasi
80

Pagina rezolvitorilor
BOTO
SANI
Colegiul National "A. T. Laurian". Clasa a IX-a. NEGRESCU Alexandru:
VII(39,40,42,44), VIII(36,40,42), IX(36,37,39), X(39,40), G(42,54).
BRA
SOV
Bogdan: V.37, VI (37,38,40),
Scoala
gen. nr. 5. Clasa a VII-a. POSTEUCA

VII.39; POSTEUCA Raluca: V.37, VI (37,38,40), VII.39.

CONSTANTA
Colegiul National "Mircea cel Batrn". Clasa a X-a. LUPU Cezar: VIII.37,
IX(38,39,40), X(38,40), XI(37,38).
CRAIOVA
Angela). STANCIU
Scoala
nr. 22 "M. Eliade". Clasa a IV-a (nv. VANTU

Ioan: P(54-63).
(Iasi)
HRLAU
Liceul "Stefan
cel Mare". Clasa a VI-a. CIOFU Alexandra: P(50,52), V.37,

VI.39, VII.41; SAVA Cristina Amelia: P(52,61,63), V(41,44); SCRIPCARIU Gabriela:


P.61, V(41,44), VI(37,42); SPIRIDON Florin: P(50,61), V(37,41), VII.41; SURUGIU
Ionut: V(37-39), VI(38,39). Clasa a VIII-a. ANTOCI Bogdan: VI(39,40,44,45),
VII.44; BURICAN Bogdan - Alexandru: VI(37,38,42,44,45), VII(41,44); ROTARU
Lucian: VI(39,40,44,45), VII.44.
HUNEDOARA

Liceul "Iancu de Hunedoara". Clasa a VII-a. CRACIUN


Maria: V(43,44),
VI(42,44), VII.41.
IA
SI
Colegiul National "C. Negruzzi". Clasa a V-a. OLARIU Tudor: P(51,52,61-63),
V(43,45), VI.39; TIBA Marius: P(58,61-63), V(42,45), VI.42.
Colegiul National. Clasa a V-a. ANDRONIC Adrian: V(36,38,39,41-45), VI.42;
ANDRONIC Adrian Constantin: V(36,38,39,41-45), VI.42; BALAN Elena-Lavinia:
V(36,38,39,41-44); BARBACARIU Ioana: V(36,38,39,41,42); BERCU Tudor: V(36,
Andreas: V(36,
38,39,41-44); CAPRARU M
ad
alina: V(36,38,41-44); CHELSAU
38,39,40-45); CHIDIU Alexandru: V(36,38,39,41-45); DOBROVICEANU C
at
alina:

V(36,38,39,41-43); GAFITANU
Oana: V(36,39,41-43); GEORGESCU Anca: P(61,
TEANU

62), V(36,38,41,42,44); MNZA


Maria-Adelina: V(36,38,39,41-44); MIHAI
C
Monica: V(36,38,39,41-44); MOSNEGUTU
at
alina Elena: V(36,38,39,41-44); PALAGHIA Irina: V(36,38,39,41-45); POPA Ana-Maria: V(36,38,39,41-43,45); POTUR George: V(36,38,39,41-44); SMARANDA Sava: V(36,38,40-44); TOMA Alexandra: V(36,38,39,41,42). Clasa a IX-a. CAZACU Roxana: VII.41, IX(37,38),
Marta: VII.41, VIII.42, IX(38,39), G.40; HAMCIUC Adrian:
G(40,47); CHIRUTA
VII(41,42), VIII.42, IX(36,39); PRODAN Diana: VII.41, IX(36,38), G(40,47); TIMOFTE Diana: VII.41, IX(36,38), G(40,47). Clasa a X-a. DUMITRESCU Ro
xana: VIII(37,42), IX(37,39), X(36-38,42), G(40,50); PACHITARIU
Marius: G(46 Irina: X.42, XI(41,43), XII.45,
A
50,52), L(46,47,49,50). Clasa a XI-a. MUSTAT
G(46,47,52), L(46,47).
Liceul "M. Eminescu". Clasa a V-a. BOHOTIN Alexandru: P(48,49,51,53,61,62),
81

V.37; COHAL C
alin: P(48,58,60,63), V(38,39,42), VI.38. Clasa a VI-a. CIMPOI
Mihaela: V(42-44), VI(37,42); CIURARU Ionela Alexandra: V(42-44), VI(37,42);
IPATE Cristina Alexandra: V(42-44), VI(37,42).
Liceul "G. Ibraileanu". Clasa a VII-a. UNGUREANU Dragos: V(39,42,45),
VI(39,42).
Scoala
nr. 7 "N. Tonitza". Clasa a II-a (nv. TUDOSE Elena). CRNU Alina:

P(54-57,60); DOBRIN Diana - Maria: P(54-57,60); LEONTE Anca: P(54-57,60);


Simona - Alexandra: P(54-57,60); ROTARIU Larisa: P(54-57,60); SAVIN
POSTICA
R
azvan: P(54-57,60). Clasa a II-a (nv. MELINTE Rodica). BACIU Ciprian: P(5457,60); BRZU Constantin: P(54-57,60); BOTOSANU Bianca - Mihaela: P(54 Danut - Vasilica: P(5457,60); BUZDUGAN Petru - Catalin: P(54-57,60); CEUCA
57,60); CONSTANTINESCU Diana - Gabriela: P(54-57,60); CUCUTEANU Paul C
at
alin: P(54-57,60); GUSOVATE Diana - S
tefana: P(54-57,60); LEOGAN Larisa Diana: P(54-57,60); MIRON Vlad - S
tefan: P(54-57,60); MOTAN Geanina - Diana:
P(54-57,60); ROTARIU Marian: P(54-57,60); SUCIUC Raluca: P(54-57,60); TEIU
- COSTIN Andrada - Mihaela: P(54-57,60). Clasa a IV-a (nv. MARCU Monica).
Alin: P(52,58-62).
A
BUTNARU Valentin: P(52,58-62); ONUT
Scoala
nr. 13 "Alexandru cel Bun". Clasa a III-a (nv. SPNU Doinita).

Marta:
BURLACU Ionut: P(54-57,61); DAMIAN Daniel: P(54-57,61); FURTUNA
P(54-57,61); IFTENIE Ioana - C
at
alina: P(54-57,61); RUSU Alexandru: P(5457,61); URSU Gina - Ioana: P(54-57,61).
A
Scoala
nr. 22 "B. P. Hasdeu". Clasa a II-a (nv. TRZIORU Iuliana). ADASC

LITEI
Victor: P(54-57,60); APOSTOL Ana - Maria: P(54-57,60,61); BALAN An C
drei: P(54-57,60); BURUIANA
at
alina: P(54-57,60,61); CUBERSCHI Paula: P(5457,60,61); ESANU Geogiana: P(54-57,60); GREIEROSU Claudia: P(54-57,60,61);
ATIC

GNDU Alexandra - Livia: P(54-57,60,61); LAM


Ioana: P(47,54-57,60); REBEGEA Andrada: P(54,56,57,60,61); UNGUREANU Teofana: P(54-57,60,61). Clasa a II-a. (nv. TUTU Laura). ANDRONICIUC Ana - Miruna: P(54-57,60,61);
Sebastian: P(54-57,60,61); BUZA
Eduard
BUHU Vlad: P(54-57,60); BURUIANA

- Andrei: P(54-57,60); CEOBANU Andrei - Nicolae: P(54-57,60); CHICHIRAU

Alexandra - Elena: P(54-57,60,61); COSTACHESCU


Ioana: P(54-57,60); DOROHOI Ovidiu: P(54-57,60,61); GELIP Ioana: P(54-57,60); GHERAN Ana - Maria:
Raluca - Claudia: P(54P(54-57,60); GRIGORE Georgiana: P(54-57,60); GURAU
57,60); HATESCU Iustina: P(54-57,60); HORBOVANU Bianca - Alexandra: P(54
57,60); NASTASE
Andrei Ionut: P(54-57,60,61). Clasa a II-a (nv. DOHOTARU
Liliana). TURCU Andrei - Daniel: P(54-57,60,61).
Beatrice). TUDOScoala
nr. 23 "T. Maiorescu". Clasa a IV-a (nv. CHIRILA

RACHE Alexandru - Gabriel: P(54-63).

Scoala
nr. 26 "G. Cosbuc". Clasa a III-a (nv. RACU Maria). BARABULA

Ioana - Mioara: P(54-57,61); BULGARU Ionela - Alexandra: P(54-57,61); BURLA


CU Claudiu: P(54-57,61); CALOIAN Andrei: P(54-57,61); CALIN
Georgiana - An
Adriana: P(54dreea: P(54-57,61); CRACIUN
Madalina: P(54-57,61); IFROSA
Petru - Alexandru: P(54-56,59,61); LEAGAN
Crina - Alexandra:
57,61); IOJA
P(54-57,61); MOISA Bogdan: P(54-57,61); PINTILIE R
azvan - Florin: P(54-57,61);
RUSU Flavia: P(54-57,61). Clasa a III-a (nv. GALIA Paraschiva). ALUPEI An82

Oana - C

dra - M
ad
alina: P(54-57,61); CIOABA
at
alina: P(54-57,61); GHERCA
Marius - Catalin: P(54-57,61); HOMEA Liviu: P(54-57,61); HUIDES Gina: P(54
57,61); MANOLIU Madalina: P(54-57,61); MIHAILESCU
Laura: P(54-57,61); PISI Alexandru: P(54-57,61); POPA Florin: P(54-57,61); SCUTARIU Constantin:
CA
P(54-57,61).

Premii acordate rezolvitorilor


Pentru aparitia de trei ori la rubrica "Pagina rezolvitorilor" redactia revistei
"Recreatii matematice" acord
a cte o diplom
a si un premiu n c
arti urm
atorilor
elevi:
Ioana (Sc. nr. 26 "G. Cosbuc", cl. a III-a): 1/2003 (5pb), 2/2003
BARABULA
(5pb), 1/2004 (5pb);
BURLACU Ionut (Sc. nr. 13 "Alexandru cel Bun", cl. a III-a): 1/2003 (6pb), 2/2003
(5pb), 1/2004 (5pb);
BUTNARU Valentin (Sc. nr. 7 "N. Tonitza", cl. a IV-a): 1/2003 (5pb), 2/2003
(6pb), 1/2004 (6pb);
CALOIAN Andrei (Sc. nr. 26 "G. Cosbuc", cl. a III-a): 1/2003 (5pb), 2/2003 (5pb),
1/2004 (5pb);

CALIN Georgiana (Sc. nr. 26 "G. Cosbuc", cl. a III-a): 1/2003 (5pb), 2/2003 (5pb),
1/2004 (5pb);
Oana - C
CIOABA
at
alina (Sc. nr. 26 "G. Cosbuc", cl. a III-a): 1/2003 (5pb),
2/2003 (5pb), 1/2004 (5pb);

CRACIUN
M
ad
alina (Sc. nr. 26 "G. Cosbuc", cl. a III-a): 1/2003 (5pb), 2/2003
(5pb), 1/2004 (5pb);
DAMIAN Daniel (Sc. nr. 13 "Alexandru cel Bun", cl. a III-a): 1/2003 (6pb), 2/2003
(5pb), 1/2004 (5pb);
Marta (Sc. nr. 13 "Alexandru cel Bun", cl. a III-a): 1/2003 (5pb),
FURTUNA
2/2003 (5pb), 1/2004 (5pb);
IFTENIE Ioana - C
at
alina (Sc. nr. 13 "Alexandru cel Bun", cl. a III-a): 1/2003
(6pb), 2/2003 (5pb), 1/2004 (5pb);

LEAGAN Crina - Alexandra (Sc. nr. 26 "G. Cosbuc", cl. a III-a): 1/2003 (5pb),
2/2003 (5pb), 1/2004 (5pb);

MIHAILESCU
Laura - Ioana (Sc. nr. 26 "G. Cosbuc", cl. a III-a): 1/2003 (5pb),
2/2003 (5pb), 1/2004 (5pb);

MOISA Bogdan (Sc. nr. 26 "G. Cosbuc", cl. a III-a): 1/2003 (5pb), 2/2003 (5pb),
1/2004 (5pb);
NEGRESCU Alexandru (C. N. "A. T. Laurian", Botosani, cl. IX-a): 1/2003 (15pb),
2/2003 (17pb), 1/2004 (14pb);
Alin (Sc. nr. 7 "N. Tonitza", cl. a IV-a): 1/2003 (5pb), 2/2003 (6pb),
A
ONUT
1/2004 (6pb).
PINTILIE R
azvan - Florin (Sc. nr. 26 "G. Cosbuc", cl. a III-a): 1/2003 (5pb),
2/2003 (5pb), 1/2004 (5pb);
83

Bogdan (Sc. nr. 5, Brasov, cl. a VII-a): 1/2002 (5pb), 1/2003 (6pb),
POSTEUCA
1/2004 (5pb).

POSTEUCA Raluca (Sc. nr. 5, Brasov, cl. a VII-a): 1/2002 (5pb), 1/2003 (6pb),
1/2004 (5pb).
RUSU Alexandru (Sc. nr. 13 "Alexandru cel Bun", cl. a III-a): 1/2003 (8pb), 2/2003
(5pb), 1/2004 (5pb);
RUSU Flavia (Sc. nr. 26 "G. Cosbuc", cl. a III-a): 1/2003 (5pb), 2/2003 (5pb),
1/2004 (5pb);
SCUTARU Constantin (Sc. nr. 26 "G. Cosbuc", cl. a III-a): 1/2003 (5pb), 2/2003
(5pb), 1/2004 (5pb);
URSU Gina - Ioana (Sc. nr. 13 "Alexandru cel Bun", cl. a III-a): 1/2003 (7pb),
2/2003 (5pb), 1/2004 (5pb).

LISTA MEMBRILOR FILIALEI IA


SI a S. S. M.
110.
111.
112.
113.
114.
115.
116.
117.
118.
119.
120.
121.
122.
123.
124.
125.

continuare din nr. 1/2000, 1/2001, 1/2002 si 1/2003

MIHAILA Marcela
S
coala "D.D.Patrascanu", Tomesti (Iasi)
BOBOC Romela
S
coala "D.D.P
atr
ascanu", Tomesti (Iasi)
TEMNEANU Mitica
Univ. Tehnica "Gh. Asachi", Iasi
MIRON Mirel
Liceul "C. Negruzzi", Iasi
ROTUNDU Raluca
S
coala gen. Gropnita, jud. Iasi
APETREI Laura
NAVROTESCU Mariana
Gr. sc. "Al. I. Cuza", Iasi
CHIORESCU Daniela Marinela Gr. sc. "D. Mangeron", Iasi

AVADANI
Adela
S
coala gen. nr.37, Iasi
Monica
STRACHINA
S
coala gen. nr.37, Iasi
BRGHISAN Mariana
Gr. sc. "Tehnoton", Iasi
SPIRIDON Ana - M
arioara
S
c. nr. 3 "Iordache Cantacuzino", Pascani
TUDORACHE Nelu
Liceul "V. Alecsandri", Iasi

DASCALU
Cristina
Liceul "M. Eminescu", Iasi
CORDUNEANU Adrian
Univ. Tehnic
a "Gh. Asachi", Iasi
ROSU M
arioara
Liceul de art
a, Iasi

IMPORTANT
n scopul unei leg
aturi rapide cu redactia revistei, pot fi utilizate urm
atoarele adrese
e-mail: tbi@math.tuiasi.ro, popagabriel@go.com . Pe aceasta cale colaboratorii
pot purta cu redactia un dialog privitor la materialele trimise acesteia, procurarea
numerelor revistei etc.
La problemele de tip L se primesc solutii de la orice iubitor de matematici elementare
(indiferent de preocupare profesionala sau vrsta ). Fiecare dintre solutiile acestor
probleme - ce sunt publicate n revista dupa un an - va fi urmata de numele tuturor
celor care au rezolvat-o.
Adres
am cu insistenta
amintea ca materialele trimise revistei s
a nu
rug
fie (s
a nu fi fost) trimise si altor publicatii.
84

Anul VII, Nr. 1

Ianuarie Iunie 2005

RECREAII
MATEMATICE
REVIST DE MATEMATIC PENTRU ELEVI I PROFESORI

e i = 1

Editura Crengua Gldu


IAI - 2005

Semnificaia formulei de pe copert:


i
ntr-o form concis, formula e = 1 leag cele patru ramuri fundamentale ale matematicii:
ARITMETICA
GEOMETRIA
ALGEBRA
ANALIZA MATEMATIC

reprezentat
reprezentat
reprezentat
reprezentat

de
de
de
de

i
e

Redacia revistei :
Petru ASAFTEI , Temistocle BRSAN, Dan BRNZEI, Ctlin - Cristian BUDEANU,
Constantin CHIRIL, Eugenia COHAL, Adrian CORDUNEANU, Mihai CRCIUN
(Pacani), Gabriel DOSPINESCU (student, Paris), Marius FARCA, Paraschiva GALIA,
Paul GEORGESCU, Dumitru GHERMAN (Pacani), Gheorghe IUREA, Lucian Georges
LDUNC, Mircea LUPAN, Dan tefan MARINESCU (Hunedoara), Gabriel
MRANU, Andrei NEDELCU, Gabriel POPA, Dan POPESCU (Suceava), Florin
POPOVICI (Braov), Maria RACU, Ioan SCLEANU (Hrlu), Ioan ERDEAN
(Ortie), Dan TIBA (Bucureti), Lucian TUESCU (Craiova), Adrian ZANOSCHI.

Adresa redaciei:
Catedra de Matematic Universitatea Tehnic Gh. Asachi Iai
Bd. Carol I, nr.11, 700506, Iai
Tel. 032 213737 / int. 123
E-mail: acord@math.tuiasi.ro
EDITURA CRENGUA GLDU
Toate drepturile rezervate
ISSN 1582 - 1765
Bd. N. Iorga, Bl. K2, ap. 4, IAI
Tel. / Fax: 032 230598
TIPRIT LA SL&F IMPEX IAI
Bd. Carol I, nr. 3-5
Tel. 0788 498933
E-mail: simonaslf@yahoo.com

Anul VII, Nr. 1

Ianuarie Iunie 2005

RECREAII
MATEMATICE
REVIST DE MATEMATIC PENTRU ELEVI I PROFESORI

e i = 1
Revist cu apariie semestrial
publicat de

ASOCIAIA RECREAII MATEMATICE

IAI - 2005

200 de ani de la nasterea lui Dirichlet


Johan Peter Gustav Lejeune -Dirichlet
a fost unul dintre marii matematicieni ai secolului XIX si ai tuturor timpurilor.
S-a n
ascut la 13 februarie 1805, n localitatea Dren, oras situat la jum
atatea distantei
dintre Kln si Aachen, n Germania (pe atunci
n Imperiul Francez al lui Napoleon), n familia
unui functionar postal. Numele de familie al lui
Dirichlet, mai precis Lejeune-Dirichlet, provine
de la bunicul s
au care a locuit n orasul Richelet
n apropiere de Lige, n Belgia ("Le jeune de
Richlet").
Dirichlet a fost un elev str
alucit, terminnd studiile secundare la vrsta de 16 ani.
nc
a nainte de a intra la gimnaziul din Bonn
dovedeste pasiune pentru matematic
a, cheltuind banii de buzunar pe c
arti de matematic
a.
Dup
a absolvirea colegiului, la Kln, se hot
ar
aste s
a urmeze cursurile universitare
n Franta. Soseste la Paris n 1822, purtnd cu el ca pe o biblie,Disquisitiones Arithmeticae, cunoscutul tratat al lui Gauss. Aici va urma cursurile Facult
atii de stiinte
de la Collge de France. ncepnd din vara lui 1823 este bine primit n casa generalului
Foy, unde locuieste si d
a lectii de limb
a german
a sotiei si fiicei acestuia. Generalul
Maximilien Sbastien Foy a fost una dintre figurile remarcabile ale r
azboaielor lui
Napoleon si din 1819 lider al opozitiei liberale din Camera deputatilor.
La Paris, Dirichlet intr
a n contact cu mari matematicieni precum Legendre,
Poisson, Laplace si Fourier. Ultimul l-a impresionat n mod deosebit, fapt ce are drept
consecinta interesul s
au pentru seriile trigonometrice si fizica matematic
a. n aceast
a
perioad
a redacteaz
a prima sa contributie originala n matematica: demonstreaza
marea teorema a lui Fermat pentru cazul n = 5. Aceast
a teorem
a afirm
a c
a pentru
orice num
ar natural n, n > 2, nu exist
a numere ntregi diferite de 0 astfel nct
xn +y n = z n . Demonstratia complet
a a acestei teoreme a fost dat
a abia n ultimii ani
ai secolului XX (v. [1], [2], [4], [5]). Ulterior, Dirichlet a fost primul matematician
care a observat c
a unele demonstratii date pentru cazuri particulare ale teoremei
lui Fermat, de c
atre mari matematicieni, erau gresite deoarece se bazau pe ipoteza
c
a n inele de extensiune a lui Z, descompunerea unui num
ar ca produs de factori
ireductibili (care nu se mai pot descompune) este unic
a, ipotez
a care este fals
a pentru
unele dintre aceste inele. Aceast
a observatie a impus diferentierea ntre notiunea
de num
ar ireductibil si cea de num
ar prim (un num
ar diferit de zero si unit
ati se
numeste num
ar prim dac
a ori de cte ori divide un produs de numere, divide cel putin
unul dintre factori) si a avut implicatii profunde n dezvoltarea teoriei numerelor si
algebrei.
La sfrsitul anului 1825, dup
a moartea generaluli Foy, Dirichlet se hot
ar
aste s
a se
ntoarc
a n Germania. La recomandarea lui Alexander von Humboldt obtine un
doctorat onorific (nu cunostea limba latin
a, conditie obligatorie n acel timp pentru
1

obtinerea unui doctorat) ceea ce i permite s


a-si sustin
a teza de docenta si s
a obtin
a
titlul de profesor. Pred
a la Universitatea din Breslau (ast
azi Wroklaw, Polonia),
la Liceul militar din Berlin si, timp de 27 de ani nentrerupt, la Universitatea din
Berlin. Dintre cei mai str
aluciti elevi ai s
ai, din aceast
a perioad
a, mention
am pe
L. Kronecker si B. Riemann.
n 1831 se c
as
atoreste cu Rebeca Mendelssohn, sor
a cu celebrul compozitor.
Contemporanii s
ai l apreciau ca pe un excelent matematician si profesor care nu
era lipsit de anumite defecte: se mbr
aca neglijent, era mereu cu o tigar
a n gur
a si
o cafea n fata, putin preocupat de imaginea sa si mereu n ntrziere.
n 1843, maestrul si prietenul s
au Karl Jacobi este diagnosticat ca fiind bolnav
de diabet si Dirichlet l nsoteste pentru o perioad
a de 18 luni n Italia, unde viziteaz
a
Roma, Florenta, Sicilia. Climatul blnd din Italia amelioreaz
a starea de s
an
atate a
lui Jacobi. C
al
atoria efectuat
a n 1844 - 1845 a fost posibil
a datorit
a unei subventii
obtinute de Alexander von Humboldt de la Friedrich Wilheem al IV-lea.
Dup
a moartea lui Gauss, n 1855, succede acestuia la catedra de la Universitatea
din Gttingen. Activitatea sa n acest mare centru matematic al lumii este scurt
a;
la 5 mai 1859 Dirichlet trece n lumea umbrelor ca urmare a unei maladii cardiace.
Evantaiul lucr
arilor lui Dirichlet ilustreaz
a profunzimea culturii matematice germane din perioada de nceput a epocii de aur a acesteia, inaugurat
a de Karl Friedrich
Gauss, cel mai mare matematician al timpurilor moderne. Lucr
arile lui acoper
a multe
aspectele ale matematicii; totusi cele de teoria numerelor, analiz
a si teoria potentialului sunt cele mai importante. Multe notiuni si rezultate i poart
a acum numele.
n teoria numerelor a demonstrat c
a, dac
a a si b sunt numere ntregi si (a, b) = 1,
n sirul (an + b)nN exist
a o infinitate de numere prime, rezultat cunoscut sub numele
de teorema lui Dirichlet. Demonstratia dat
a de Dirichlet acestei teoreme, n 1837,
este considerat
a actul de nastere a teoriei analitice a numerelor. Dirichlet a adus o
contributie important
a la elaborarea instrumentelor de lucru pentru teoria moderna a
numerelor prin introducerea seriilor atasate functiilor aritmetice, numite ast
azi seriile lui Dirichlet, crearea teoriei unitatilor si preocup
arile sale privind reprezentarea
numerelor ntregi prin forme patratice aritmetice (v. [4]).
De asemenea, lui Dirichlet dator
am principiul sertarelor, ce afirm
a c
a, dac
a sunt
n + 1 obiecte n n sertare, atunci cel putin un sertar contine cel putin dou
a obiecte;
Dirichlet utilizeaz
a acest principiu n studiul corpului numerelor algebrice.
n teoria potentialului se ocup
a cu problema Dirichlet privind existenta functiilor
armonice. Tot el a dat conditia Dirichlet pentru convergenta seriilor trigonometrice.
Ideile lui Dirichlet nu au pierdut str
alucirea odat
a cu trecerea timpului; dezvoltarea matematicii n ultimii 200 de ani a pus n evidenta profunzimea acestor idei.
Bibliografie
1.
2.
3.
4.
5.

A. Corduneanu - Despre Marea teorema a lui Fermat, RecMat - 1/1999, 37-39.


P. Minut - Pierre Fermat - Patru secole de la nasterea sa, RecMat - 2/2001, 4-5.
P. Minut - Numere prime din progresii aritmetice, RecMat - 1/2003, 15-18.
P. Minut - Teoria numerelor, Ed. Matrix-Rom, Buc., 2001 (pp. 35-50, 215-224).
M. M. Postnikov - Despre teorema lui Fermat, Ed. did. si ped., Bucuresti, 1983.
Prof. dr. Petru MINUT

Fractali

(II)

Stefan

FRUNZ
A1 , Irina FRUNZ
A2
Fractal este un obiect fractionat la infinit (cuvntul deriv
a din latinul fractus,
derivat, la rndul s
au, din verbul frangere - a rupe, a sparge). Termenul a fost
introdus de americanul de origine polonez
a Benoit Mandelbrot n 1975. O definitie
n acelasi timp precis
a si general
a a unui obiect fractal este dificil
a. Pentru a da o idee
vag
a, fractalii sunt, dup
a Mandelbrot, multimi care prezint
a neregularit
ati la toate
sc
arile. O structur
a fractal
a e aceeasi n mic ca si n mare, aproape sau departe.
n natur
a se pot ntlni structuri prefractale (termenul i apartine de asemenea
lui Mandelbrot). Ca exemple ar putea servi frunzele de spini, cele de ferig
a, fulgii
de z
apad
a, norii etc. Structurile fractale sunt abstractiuni matematice obtinute, de
regul
a, din structuri prefractale prin trecere la limit
a. S
irul de multimi prefractale
care ne d
a la limit
a o structur
a fractal
a este, de regul
a, un sir recurent. El se
obtine dintr-o structur
a prefractal
a initial
a (initiator) aplicndu-i succesiv aceeasi
transformare (generator ). Exemple n acest sens se g
asesc n [1]-[6]: curba lui Koch si
curba fulgului de zapada, praful lui Cantor, sita lui Sierpinski, carpeta lui Sierpinski,
buretele lui Menger etc.
n [1], pe care o continu
am n nota de fata, au fost stabilite sau indicate dimensiunile n sens Richardson ale ctorva fractali: curba lui Koch are dimensiunea fractal
a
log 4
log 2
log 3
D=
, praful lui Cantor
, sita lui Sierpinski
, sita tridimensional
a
log 3
log 3
log 2
log 4
log 20
a lui Sierpinski
, buretele lui Menger
etc.
log 2
log 3
O alt
a caracteristic
a a unei structuri fractale, pe care ne propunem s
a o prezent
am
n continuare, este autosimilaritatea.
Majoritatea structurilor fractale sunt autosimilare ntr-un sens care va fi explicat
n continuare. S
a ncepem prin a considera covorul lui Sierpinski. El se poate obtine
pornind de la triunghiul echilateral initial ABC si aplicnd succesiv trei omotetii:
omotetia de centru A si raport 1/2 si analoagele cu centrele B si C (s
a le not
am
respectiv cu f1 , f2 , f3 ).
A

f1

T0

f2

f1 (T0 )

T0

f3

f 2 (T0 )

Astfel, dup
a primul pas iterativ, putem scrie:
S1 = f1 (T0 ) f2 (T0 ) f3 (T0 ) ,
1
2

Prof. dr., Facultatea de matematic


a, Univ. "Al. I. Cuza", Iasi
Profesor, Grupul S
colar Agricol "M. Kog
alniceanu", Miroslava (Iasi)

T0

f3 (T0 )

cele trei multimi ale reuniunii putnd avea n comun dou


a cte dou
a cel mult un
punct. S
a not
am f1 (T0 ) = T01 si s
a urm
arim efectul aplic
arii celor trei omotetii
asupra lui.
f1

T01

f2

f3

T01

T01

Urm
arind analog transform
arile lui f2 (T0 ) = T02 si f3 (T0 ) = T03 constat
am c
a
S2 = f1 (S1 ) f2 (S1 ) f3 (S1 ) .
Se poate demonstra prin inductie c
a
Sn+1 = f1 (Sn ) f2 (Sn ) f3 (Sn ) ,

n 1,

(1)

unde Sn este aproximanta de ordin n a covorului lui Sierpinski S [1]. Multimile din
membrul drept al relatiei (1) nu pot avea n comun dou
a cte dou
a dect cel mult
un punct.
Din relatia (1) se poate deduce (elementar) c
a
S = f1 (S) f2 (S) f3 (S) .

(2)

Multimile din membrul drept al lui (2) pot avea dou


a cte dou
a cel mult un punct
1
n comun si fiecare este similar
a cu S cu raportul de asem
anare .
2
O asemenea situatie am mai ntlnit-o doar la figurile cele mai simple din geometria elementar
a (segmente de dreapt
a sau arce de cerc).

Un segment de dreapt
a poate fi mp
artit n orict de multe segmente adiacente similare fiecare cu segmentul initial.
Rezultatul precedent are o strns
a leg
atur
a cu notiunile de m
asur
a si dimensiune.
S
a amintim rezultatul din plan c
a raportul ariilor a dou
a figuri asemenea este egal
cu p
atratul raportului de asem
anare. Un rezultat similar are loc n spatiu: raportul
volumelor a dou
a corpuri asemenea este egal cu puterea a treia a raportului de
asem
anare.
n [1] s-a ar
atat c
a si obiectele fractale pot fi "m
asurate" si c
a m
asura lor are
leg
atur
a cu dimensiunea fractal
a. Dac
a rezultatul de geometrie clasic
a amintit mai
sus s-ar extinde corespunz
ator la obiecte fractale, atunci din relatia (2) am obtine
m (S) = m (f1 (S)) + m (f2 (S)) + m (f3 (S)) =
D
D
D
1
1
1
=
m (S) +
m (S) +
m (S) ,
2
2
2

(3)

unde m (S) este m


asura lui S iar D este dimensiunea fractal
a a lui S. Admitnd c
a
4

0 < m (S) < (ceea ce e destul de rezonabil), obtinem din (3) relatia
D
1
1=3
(4)
2
log 3
. Reg
asim astfel
care permite determinarea direct
a a lui D: 2D = 3, D =
log 2
dimensiunea determinat
a prin metoda lui Richardson.
O proprietate analoag
a de "autosimilaritate" o prezint
a si multimea lui Cantor,
ale c
arei aproximante succesive sunt:

2
1
C0 = [0, 1] ,
,1 ,
C1 = 0,
3
3


2 3
6 7
8 9
1
C2 = 0, 2 2 , 3 2 , 2 2 , 2 , . . .
3
3 3
3 3
3 3

1
x
x+2
2
Se observ
a c
a 0,
= f1 (C0 ), unde f1 (x) = si , 1 = f2 (C0 ), unde f (x) =
.
3
3
3
3
Se poate demonstra c
a
Cn+1 = f1 (Cn ) f2 (Cn ) , n 1,
de unde se deduce (elementar) c
a
C = f1 (C) f2 (C) .
1
ntruct f1 si f2 sunt similarit
ati de raport , pentru determinarea dimensiunii
2
D
log 2
2
n concordanta cu valoarea
, de unde D =
fractale se obtine ecuatia 1 = 2
3
log 3
determinat
a prin metoda lui Richardson.
Sistemele de functii {f1 , f2 , f3 } (implicat n covorul lui Sierpinski) si {f1 , f2 } (implicat n multimea lui Cantor) se numesc sisteme iterate de functii. Curba lui Koch se
poate obtine printr-un sistem iterat de 4 similarit
ati (implicnd omotetii si rotatii),
1
toate de raport
[2]. Pentru determinarea dimensiunii fractale a curbei lui Koch
3
D
log 4
1
n concordanta iar
asi cu valoarea
, de unde D =
se obtine ecuatia 1 = 4
3
log 3
determinat
a prin metoda lui Richardson.
Bibliografie
S
t. Frunz
a - Fractali, RecMat - 2/2002, 1-5.
S
t. Frunz
a - Fractali, curs optional, anul IV, Fac. Matematic
a - Informatic
a, 2004.
J. - F. Gornyet - Physique et structures fractales, Masson, Paris, 1992.
B. B. Mandelbrot - Les objects fractales: forme, hasard et dimension, Flammarion,
Paris 1975.
5. A. Le Mhaut - Les gomtries fractales, Herms, Paris, 1990.
6. H. Takayasu - Fractales in the physical sciences, Manchester University Press,
Manchester and New York, 1990.
1.
2.
3.
4.

Asupra monotoniei unor siruri


Dumitru MIHALACHE, Marian TETIVA1
n aceast
a not
a ne propunem s
a investig
am monotonia sirurilor care apar la precizarea ordinului de convergenta al ctorva siruri uzuale. Mai precis, dac
a limita
sirului (an )n1 este a (finit
a), iar (zn )n1 este un sir cu limita zero astfel nct exist
a
an a
,
lim
n
zn
finit
a si nenul
a, atunci spunem c
a (zn )n1 d
a ordinul de convergenta al sirului (an )n1
c
atre limita sa (este un fel de a m
asura "rapiditatea" cu care (an )n1 tinde la limit
a cu
ajutorul unor siruri cunoscute si/sau mai accesibile; a se vedea si [2]). Ne propunem
s
a cercet
am dac
a sirul cu termenul general (an a)/zn este sau nu monoton n cazul
a.
unor siruri (an )n1 des ntlnite si importante n analiza matematic
Primul sir de care ne ocup
am este cel cu termenul general
1
1
1
En = 1 + + + + , n N .
1! 2!
n!
Asa cum se stie, acesta este convergent, are limita e, iar ordinul s
au de convergenta
este dat de relatia
lim n n!(e En ) = 1,
n

ceea ce se verific
a usor cu teorema lui Cesro - Stolz (cazul 00 ). Pentru a vedea monoa compar
am
tonia sirului (xn )n1 cu termenul general xn = n n!(e En ) trebuie s
xn cu xn+1 . Avem

n+1
1
2
xn ? xn+1 n(e En ) ? (n + 1) e En

? e En .
(n + 1)!
n!(n2 + n + 1)
n+1
1
Este bine cunoscut
a si inegalitatea e En <
(v. [3]); cum
<
n n!
n!(n2 + n + 1)
1
<
, de aici nu putem obtine nimic. Atunci vom proceda astfel:
n n!
p
X
1
1
lim
>
e En =
n! p
(n + 1)(n + 2) (n + k)
k=1

1
1
1
1
1
+
+
+
.
>
n! n+1 (n+1)(n+2) (n+1)(n+2)(n+3) (n+1)(n+2)(n+3)(n+4)

Un calcul simplu (chiar dac


a, poate, nepl
acut!) arat
a c
a expresia de pe ultimul rnd
este egal
a cu
n+1
n3 + 10n2 + 34n + 41
>
,
n!(n + 1)(n + 2)(n + 3)(n + 4)
n!(n2 + n + 1)
ultima inegalitate fiind echivalent
a cu
n5 + 11n4 + 45n3 + 85n2 + 75n + 41 > n5 + 11n4 + 45n3 + 85n2 + 74n + 24,
adic
a evident
a pentru orice num
ar natural n. Astfel am demonstrat
1

Profesori, Colegiul National "Gheorghe Rosca Codreanu", Brlad

Propozitia 1. Sirul

(xn )n1 cu termenul general xn = n n!(e En ) este strict


crescator.
O alt
a metod
a de a demonstra Propozitia 1 este prezentat
a n
Exercitiul 1. Aratati ca sirul (un )n1 cu termenul general
n+1
un = En +
, n N ,
n!(n2 + n + 1)
este strict crescator cu limita e si deduceti de aici inegalitatea xn < xn+1 , n N .

S
i nc
a o observatie: avem acum inegalit
atile
n+1
1
< e En <
, n N ,
n!(n2 + n + 1)
n n!
care implic
a, si ele, faptul c
a lim n n!(e En ) = 1 (cum?).
n

Urm
atorul de care ne ocup
am este sirul lui Euler, care defineste num
arul e; asadar
consider
am
n

1
, n 1.
en = 1 +
n
Iar facem apel la o relatie cunoscut
a (a se vedea [4], sau [6]), anume
e
lim n (e en ) = ;
n
2
deci ne intereseaz
a acum (din punct de vedere al monotoniei) sirul (xn )n1 cu termenul general xn = n(e en ), n 1. Dar, nainte, s
a demonstr
am

x
1
Lema 1. Functia definita prin f (x) = x 1 +
, pentru orice x > 0 este strict
x
convexa pe (0, ).
Demonstratie. Vom folosi prima dintre inegalit
atile

1
1
2
< ln 1 +
<p
2x + 1
x
x(x + 1)

(pentru care se poate vedea tot [6]; de altfel, sunt niste inegalit
ati destul de vehiculate, care arat
a c
a, pentru functia logaritm natural, punctul intermediar din teorema
lui Lagrange este cuprins ntre mediile aritmetic
a si geometric
a ale extremit
atilor
intervalului pe care se aplic
a, aici intervalul [x, x + 1]), desigur, dup
a ce vom calcula
derivata a doua.
Cititorul
este
a verifice
a
invitat
x
s
c

1
1
1
x ln 1 +
+
, x > 0 si
f 0 (x) = 1 +
x
x
x+1

1
1
2
1
x+3
2
00
x ln 1 +
, x > 0.
+
ln 1 +

f (x) = 1 +
x
x
x+1
x
(x + 1)2
Atunci, procednd cum am anuntat, vom putea scrie

4
2
x+3
1
2
00
f (x) > 1 +
x
sau

+
x
(2x + 1)2 x + 1 2x + 1 (x + 1)2
x

3x + 1
1
, x > 0,
f 00 (x) > 1 +
x
(x + 1)2 (2x + 1)2
7

de unde f 00 (x) > 0, x > 0, si lema este dovedit


a.
Propozitia 2. Sirul

(xn )n1 definit prin

n
1
xn = n e 1 +
= n(e en ), n N ,
n
este strict crescator.
Demonstratie. Inegalitatea de demonstrat este n(e en ) < (n + 1)(e en+1 ) si
este echivalent
a cu (n + 1)en+1 nen < e, n N . Cum
lim [(n + 1)en+1 nen ] = lim [(n + 1)(en+1 e) n(en e) + e] = e,

nseamn
a c
a ar fi suficient s
a ar
at
am c
a sirul ((n + 1)en+1 nen )n1 este strict
cresc
ator. Dar monotonia acestui sir decurge din inegalitatea
(n + 1)en+1 nen < (n + 2)en+2 (n + 1)en+1 f (n + 2) + f (n) > 2f (n + 1);

aceast
a ultim
a form
a nu este nimic altceva dect inegalitatea lui Jensen aplicat
a
functiei convexe f si numerelor n si n + 2. Demonstratia este ncheiat
a.
Exercitiul 2. Studiati monotonia sirului (xn )n1 definit prin
"
#
n+1
1
e , n N .
xn = n 1 +
n
Mai departe consider
am sirul cu termenul general
1
1
n () = 1 + + + , n N ,
2
n
unde > 1 este un num
ar fixat. Se vede imediat c
a acest sir este strict cresc
ator
si (ceva mai greu) m
arginit superior, deci are o limit
a finit
a, pe care o not
am cu
(). n parantez
a fie spus, functia care asociaz
a fiec
arui num
ar real > 1 pe ()
se numeste functia zeta a lui Riemann si are mare importanta n multe domenii ale
matematicii. Se mai stie c
a ordinul de convergenta al sirului ( n ())n1 este dat de
egalitatea
1
lim n1 (() n ()) =
n
1
(vezi, de exemplu, [8]). Asadar, ne intereseaz
a monotonia sirului (xn )n1 dat prin
xn = n1 (() n ()) ,

n leg
atur
a cu aceasta avem

n N ;

Propozitia 3. Sirul

(xn )n1 , cu termenul general xn = n1 (() n ()),


n N , este strict crescator.
a cteva calcule,
Demonstratie. Inegalitatea xn < xn+1 devine n acest caz (dup
1
tinnd cont de n+1 () = n () +
)
(n + 1)
1
+ n () < ().
(n + 1)[(n + 1)1 n1 ]
Notnd
1
+ n (), n N ,
un =
(n + 1)[(n + 1)1 n1 ]
8

observ
am c
a este suficient s
a demonstr
am c
a sirul (un )n1 este strict cresc
ator, ca
s
a rezulte inegalitatea pe care o avem de demonstrat (deoarece lim un = (), ceea
n

ce se stabileste usor). Avem


1
1
1
un < un+1

<
+
(n+1)[(n+1)1 n1 ] (n+2)[(n+2)1 (n+1)1 ] (n+1)
1
1
1
<

(n + 1) n1 (n + 1) (n + 1)
(n + 2) (n + 1)1 (n + 2)
n1 (n + 1)(n + 2) n1 (n + 1) < (n + 1)2
n1 [(n + 2) (n + 1) ] < (n2 + 2n + 1) (n2 + 2n) .

Pentru a demonstra aceast


a ultim
a inegalitate, aplic
am teorema lui Lagrange functiei
x 7 x , pe intervalele [n + 1, n + 2] si [n2 + 2n, n2 + 2n + 1]; obtinem
2

,
(n + 2) (n + 1) = c1
n

(n + 2n + 1) (n + 2n) =

d1
,
n

cn [n + 1, n + 2] si

dn [n2 + 2n, n2 + 2n + 1].

a n1 c1
< d1
, care
Cum ncn < n(n + 2) = n2 + 2n < dn si > 1, rezult
n
n
este tocmai inegalitatea de demonstrat.
n fine, ne vom ocupa de sirul care d
a ordinul de convergenta al sirului ce defineste
constanta lui Euler. S
tim c
a aceasta este

1
1
c = lim 1 + + + ln n
n
2
n
si c
a

1
1
1
lim n 1 + + + ln n c =
n
2
n
2
(a se vedea [5]). Avem si aici nevoie, mai nti, de un mic rezultat ajut
ator.
Lema 2. Functia f : (0, ) R data de

1
, x > 0,
x+1
este strict descrescatoare si strict pozitiva pe intervalul (0, ).
Demonstratie. Functia f este strict descresc
atoare pe intervalul (0, ), c
aci

1
1
f 0 (x) = ln 1
+
< 0, x > 0,
(x + 1)2
(x + 1)2
f (x) = x ln x + (x + 2) ln(x + 2) 2(x + 1) ln(x + 1)

conform inegalit
atii ln(1 + t) < t, t > 1, t 6= 0. Cum lim f (x) = 0 (calcul de
x

rutin
a!) rezult
a c
a f (x) > 0, x (0, ).

Exercitiul 3. Functia f din Lema 2 este strict convexa pe intervalul (0, ).


Propozitia 4. Sirul

(xn )n1 cu termenul general

1
1
xn = n 1 + + + ln n c
2
n
este strict crescator.
9

Demonstratie. Inegalitatea xn < xn+1 se rescrie n forma


1
1
1 + + + + 1 + n ln n (n + 1) ln(n + 1) > c
2
n
si ar fi demonstrat
a dac
a am putea ar
ata c
a sirul (un )n1 , dat de
1
1
un = 1 + + + + 1 + n ln n (n + 1) ln(n + 1), n N ,
2
n
este strict descresc
ator (deoarece (un )n1 are limita c - demonstrati!). Dar inegalitatea un > un+1 se reduce la
1
n ln n + (n + 2) ln(n + 2) 2(n + 1) ln(n + 1)
>0
n+1
si decurge din lema anterioar
a, pentru orice n N ; astfel, demonstratia este
ncheiat
a.
S
i, odat
a cu ea, este ncheiat
a si aceast
a not
a. Dar calculele pot continua... De
exemplu:
Exercitiul 4.
ti monotonia
sirului (xn )n1 , pentru
Studia

1 n
a) xn = (n + 1) e 1 +
;
n

1 n+1
b) xn = (n + 1) 1 +
e ;
n

1
1
1
1
c) xn = n 1+ + + 2 nl , unde l = lim 1+ + + 2 n .
n
n
n
2
2
Bibliografie
1. D. Andrica, V. Berinde, L. Toth, A. Vernescu -Ordinul de convergenta al unor
siruri, G. M. A, 7-8/1998.
2. V. Berinde - Despre ordinul de convergenta al sirurilor de numere reale, G. M.
4/1998.
3. Gh. Gussi, O. St
an
asil
a, Gh. Stoica - Analiza matematica, manual pentru clasa
a XI-a, Editura Didactic
a si Pedagogic
a, 1983.

n
1
4. E. P
alt
anea - Asupra vitezei de convergenta a sirului 1 +
, G. M. A, 3/2001.
n
5. A. Vernescu - Ordinul de convergenta al sirului de definitie a constantei lui Euler,
G. M. 10-11/1983.
6. A. Vernescu - O demonstratie simpla a unei inegalitati relative la numarul e, G.
M. 5-6/1988.
7. A. Vernescu - Asupra convergentei unui sir cu limita ln 2, G. M. 10-11/1997.
8. A. Vernescu - Asupra seriei armonice generalizate, G. M. A, 3/1997.

10

O teorem
a uitat
a - inegalitatea lui Surnyi
Gabriel DOSPINESCU 1
Cu mult timp n urm
a, pe lista problemelor propuse pentru prestigiosul Concurs
"Miklos Schweitzer" a ap
arut si urm
atoarea inegalitate datorat
a lui Surnyi:
Teorem
a. Pentru orice numere reale nenegative a1 , a2 , . . . , an are loc inegalitatea
(n 1) (an1 + an2 + + ann ) + na1 a2 an

+ an1
+ + an1
. (1)
(a1 + a2 + + an ) an1
n
1
2

Departe de a fi o simpl
a aplicatie a unor inegalit
ati cunoscute cel putin, nu din
cte cunoastem pn
a n prezent aceast
a teorem
a dificil
a reprezint
a o inegalitate
foarte tare, ce permite rafin
ari ale multor inegalit
ati clasice. Fiind mai putin cunoscut
a, vom prezenta pentru nceput demonstratia ei, deloc facil
a, urmnd ca ulterior
s
a subliniem cteva consecinte interesante. Asadar,
Demonstratia teoremei. Vom folosi inductia matematic
a. Pentru n = 2,
inegalitatea este trivial
a. S
a presupunem c
a am reusit s
a o demonstr
am pentru n
variabile si s
a consider
am a1 , a2 , . . . , an , an+1 0. Datorit
a simetriei si omogenit
atii
acestei inegalit
ati, putem presupune c
a a1 a2 . . . an+1 si a1 +a2 + +an = 1.
S
a observ
am ns
a c
a inegalitatea pe care trebuie s
a o demonstr
am se rescrie
!
n
n
n
n
X
Y
Y
X
n+1
n+1
n
n
ai +nan+1 +nan+1
ai +an+1
ai (1+an+1 )
ai + an+1 0. (2)
n
i=1

i=1

i=1

i=1

Ipoteza de inductie asigur


a valabilitatea inegalit
atii
n
n
n
Y
X
X
n1
nan+1
ai an+1
ai (n 1)an+1
ani .
i=1

i=1

Ca urmare, mai r
amne de demonstrat inegalitatea
n
!
n
!
n
n
X
X
X
X
n+1
n1
n
n
n
+
ai
ai an+1 n
ai
ai
i=1

i=1

i=1

+ an+1

i=1

n
Y

i=1

ai + (n 1)ann+1 an1
n+1

Desigur, va fi suficient s
a demonstr
am inegalit
atile
n
!
n
!
n
n
X
X
X
X
n
0,
an+1

ani an+1 n
ani
an1
i
i
i=1

an+1

i=1
n
Y

i=1

(3)

i=1

i=1

ai + (n

1)ann+1

0. (4)

(5)

i=1

an1
n+1

(6)

pentru a finaliza demonstratia pasului inductiv. Din fericire, (5) si (6) nu sunt dificile.
ntr-adev
ar, (5) rezult
a combinnd trei observatii simple. Prima este inegalitatea
1

Student, cole Normale Suprieure, Paris

11

n
P

i=1

ani

n
P

i=1

an1
0, ce rezult
a imediat din inegalitatea lui Cebsev, a doua este
i
1
si, n sfrsit, a treia observatie este inegalitatea
n
!
n
n
n
n
X
X
X
X
1
n+1
n1
n
n
,
n
ai
ai
ai
ai
n
n
i=1
i=1
i=1
i=1

relatia evident
a an+1

1 n1
a
2ani ,
n i
i = 1, n. (6) este si mai usor de demonstrat; rezult
a din urm
atorul sir de inegalit
ati
si identit
ati evidente:
n
n
Y
Y
ai + (n 1)ann+1 an1
(ai an+1 + an+1 ) + (n 1)ann+1 an1
n+1 =
n+1
ce se poate obtine adunnd toate inegalit
atile de forma nan+1
+
i

i=1

i=1

ann+1 + an1
n+1

n
X
i=1

(ai an+1 ) + (n 1)ann+1 an1


n+1 = 0.

Astfel, am reusit s
a dovedim (2) si demonstratia se ncheie.
Odat
a demonstrat
a (1), s
a vedem ce obtinem pentru valori mici ale num
arului de
variabile.
Pentru n = 3 obtinem
a31 + a32 + a33 + 3a1 a2 a3 a21 (a2 + a3 ) + a22 (a3 + a1 ) + a23 (a1 + a2 ) ,

adic
a binecunoscuta inegalitate a lui Schur.
Pentru n = 4, dup
a calcule de rutin
a, rezult
a inegalitatea
4
!
4
X
X
Y

2
a4i + 2
ai
ai aj a2i + a2j ,
i=1

i=1

1i<j4

care este mai tare dect inegalitatea lui Turkevici [1]:


4
X
i=1

a4i + 2

4
Y

i=1

ai

a2i a2j .

1i<j4

Continu
am cu simple observatii ce decurg din (1). Astfel, lund s = a1 + a2 +
+ + an , rezult
a c
a
n
X
n a1 a2 an
an1
(s (n 1)ai ) .
(7)
i
i=1

s
S
a observ
am c
a n ipoteza ai <
, i = 1, n, obtinem o nt
arire a clasicei inegalit
ati
n1
a lui D. S. Mitrinovic si D. D. Adamovic [2]:
a1 a2 an (s (n1)a1 )(s (n1)a2 ) (s (n1)an ),

s = a1 +a2 + +an ,
(8)
, i = 1, n. (Pentru a vedea c
a (7) este mai tare dect (8), nu avem

s
n1
dect s
a aplic
am inegalitatea mediilor n (7).)
dac
a ai <

12

Putem obtine usor din (1) si urm


atoarea rafinare a inegalit
atii mediilor:

n
an1 + an2 + + ann
n
a1 + a2 + + an
a1 a2 an .
a1 a2 an
n
n1
n
(9)
ntr-adev
ar, s
a remarc
am c
a inegalitatea lui Jensen implic
a imediat inegalitatea

n
n1

a1 + a2 + + an
n1
n1
2
(a1 + a2 + + an ) a1 + a2 + + an
. (10)
n
n

Din (1) si (10), dupa cteva calcule simple, obtinem (9). Din p
acate, prin aceast
a
n
din membrul drept al
metod
a nu obtinem cea mai bun
a constant
a n locul lui
n
1
n1
n
, dar demoninegalit
atii (9). Am reusit s
a demonstr
am c
a aceasta este
n1
stratia dep
aseste cadrul acestei scurte note.
O aplicatie surprinz
atoare a inegalit
atii lui Surnyi o constituie si urm
atoarea
Propozitie (Vasile Crtoaje). Pentru orice numere a1 , a2 , . . . , an > 0 are loc
inegalitatea
an1 + an2 + + ann + n(n 1)a1 a2 an
a1 a2 an (a1 + a2 + + an )

1
1
1
+
+ +
a1 a2
an

. (11)

Demonstratie. Din nou, vom folosi inductia, dar vom vedea c


a pasul inductiv
se reduce exact la inegalitatea lui Surnyi. Pentru n = 3 , (11) coincide cu (1) si
cu inegalitatea lui Schur. S
a presupunem acum c
a (11) este adev
arat
a pentru n 1
a pentru fiecare grup
a de
variabile si fie a1 , a2 , . . . , an > 0. Aplicnd ipoteza inductiv
n 1 numere dintre a1 , a2 , . . . , an , obtinem

X
X
X 1
, j = 1, n.
an1
+ (n 1)(n 2)a1 a2 an a1 a2 an
ai
aj
i
ai
i6=j

i6=j

i6=j

(12)

nsumnd relatiile din (12), rezult


a inegalitatea
n ! n
!
n
X
X
X

ai
an1
ani + n(n 1)(n 2)a1 a2 an
i
i=1

i=1

Fie acum A =

n
P

i=1

i=1

X
X 1

.
a1 a2 an
ai
a
i
j=1
n
X

ai , B =

i6=j

(13)

i6=j

n 1
P
. Este evident lantul de egalit
ati urm
ator:
i=1 ai

n
X
X
X 1
1

=
ai
(Aaj ) B
= nAB AB AB +n = (n2)AB +n,
ai
aj
j=1
j=1

n
X

i6=j

i6=j

13

care, mpreun
a cu (13), implic
a
! n
n ! n
X
X
X

ai
an1
ani +n(n1)(n2)a1 a2 an a1 a2 an (n+(n2)AB) .
i
i=1

i=1

i=1

(14)

Combinnd acum (1) cu (14), obtinem


(n 2)

n
X
i=1

ani

+ n(n 1)(n 2)a1 a2 an (n 2)a1 a2 an

n
X

! n
!
X 1
,
ai
a
i=1
i=1 i

care nu este altceva dect (11) nmultit


a cu n 2 . Pasul inductiv fiind demonstrat,
propozitia este dovedit
a.
S
a observ
am c
a si (11) este o rafinare a inegalit
atii mediilor. Scriind-o sub forma
an1 + an2 + + ann
a1 a2 an
n

a1 a2 an
1
1
1

+
+ +
n2 ,
(a1 + a2 + + an )
n
a1 a2
an

ne ntreb
am dac
a nu cumva este mai slab
a dect (9). R
aspunsul este negativ, c
aci
inegalitatea

n
n2
a1 + a2 + + an
a1 a2 an
n1
n

1
1
1
2
a1 a2 an (a1 + a2 + + an )
n
+
+ +
a1 a2
an
a imediat lund
nu este adevarat
a pentru orice a1 , a2 , ..., an > 0 . Aceasta se observ
a1 0 si a2 = a3 = ... = an = 1 . ns
a nici (11) nu este mai tare dect (9), ceea ce
rezult
a iar
asi usor.
n ncheiere, mention
am c
a, folosind aceeasi tehnic
a precum cea utilizat
a pentru
a demonstra (11), se poate ar
ata c
a inegalitatea
(a1 + a2 + + an )n
n
a1 a2 an

1
1
1
n1
2
(a1 + a2 + + an )
n
(14)
+
+ +
(n 1)
a1 a2
an
este adev
arat
a pentru a1 , a2 , ..., an > 0 si c
a este mai tare dect (11). Dar despre
aceasta si multe altele, ntr-o alt
a poveste. . .
Bibliografie
1. Colectia revistei Kvant.
2. D. S. Mitrinovic, J. E. Pe
caric, A. M. Fink - Classical and New Inequalities in
Analysis, Kluwer Academic Publishers, 1993.
3. T. Andreescu, V. Crtoaje, G. Dospinescu, M. Lascu - Old and New Inequalities, Gil Publishing House, 2004.
14

Rapoarte determinate de o cevian


a
si o secant
a ntr-un triunghi
Titu ZVONARU 1 , Bogdan IONIT

A2
n triunghiul ABC consideram ceviana AD, cu D (BC). Daca o secanta intersecteaza laturile AB, AC si ceviana AD n punctele M, N , respectiv P , atunci
sunt adevarate urmatoarele doua relatii:
AM AN
BC

AP
AM BD AC P N
MB N C

= 1 (R1 ) ,
=
(R2 ) .
AN
AM
AB DC AN P M
PD
+ DC
BD
MB
NC
0
0
0
0
Demonstratie. (1) Fie M , N , B , C
A
proiectiile punctelor M, N , B, respectiv C pe
dreapta AD. Folosind triunghiuri dreptunghice
N
asemenea, avem:

N
AM
MM0
BB 0
BD
P
=
=
;
;
0
0
AB
BB
DC
CC
M B
M
AC
CC 0
NN0
PN
=
=
;
,
AN
NN0
PM
MM 0
de unde, prin nmultire, obtinem relatia (R1 ).
D
B
C
AM
AN
(2) Not
am
= x,
= y. Dac
a
C
MB
NC
AM
AN
AP
=
=
si relatia (R2 ) este adev
arat
a.
M N k BC, atunci
PD
MB
NC
Dac
a x 6= y, fie {S} = MN BC; presupunem
A
c
a punctul B este situat ntre S si C. Cu teorema lui Menelaus aplicat
a la 4ABC si transverN
SB N C M A

= 1, de
sala SMN obtinem
SC N A MB
P
y
ay
SB
M
= , adic
a SB =
.
unde rezult
a
SB + a
x
xy
Aplicnd acum teorema lui Menelaus n 4ABD cu
SB P D M A

= 1, adic
a S
transversala SMP avem
D
C
B
SD P A MB
PA
SB
PA
BC xy
=
x si obtinem
=
, care este tocmai (R2 ).
PD
SB + BD
PD
BD x + DC y

Prezent
am n continuare cteva aplicatii ale relatiilor (R1 ) si (R2 ).
Problema 1. Laturile (AB), (BC), (AC) ale triunghiului ABC sunt tangente
cercului nscris de centru I n punctele D, E, respectiv F . Bisectoarea interioara a
[ intersecteaza latura BC n punctul M. Notam {P } = F E AM.
unghiului BIC
\
Sa se demonstreze ca (DP este bisectoarea unghiului F
DE. (Propus
a de Spania la
Olimpiada Mediteraneana de Matematica n 1998.)
1
2

Profesor, Com
anesti (Bac
au)
Profesor, Bucuresti

15

A
Solutie. Cu notatiile obisnuite ntr-un triunghi,
avem AE = AF = p a, BD = BF = p b,
B
E
CD = CE = p c, DF = 2 (p b) sin , DE =
2
P
C
F
2 (p c) sin . Cu teorema bisectoarei si teorema
I
2
C
sin
BM
BI
2.
sinusurilor, n 4BIC obtinem
=
=
B
MC
CI
sin
C
B
D
M
2
AF MB AC P E

= 1, de unde rezult
a c
a
Aplicnd relatia (R1 ), avem
AB M C AE P F
B
B
B
C
sin
c sin
c sin
sin2
PE
c
pa
2
2
2
2 =
=

C
C
B
PF
pa
b
2 C
sin
b sin
b sin
sin
2
2
2
2
C
C
c sin
(p c) sin
ab
(p a) (p c)
2
2 = DE
=

B
B
ac
(p a) (p b)
DF
b sin
(p b) sin
2
2
si, conform reciprocei teoremei bisectoarei, (DP este bisectoarea unghiului F DE.
Problema 2. Fie ABC un triunghi si M, N (BC), P, Q (AC), R, S (AB)
puncte astfel nct BM = CN = CP = AQ = AR = BS = x, unde 0 < 2x <
min {AB, BC, CA}. Fie A1 , B1 , C1 puncte apartinnd segmentelor (SP ), (RN ),
(MQ) astfel nct AA1 , BB1 , CC1 sunt ceviene de rang k n triunghiurile ASP ,
BRN , respectiv CQM. Demonstrati ca dreptele AA1 , BB1 , CC1 sunt concurente.
(Generalizarea unei probleme propuse de Constantin Cocea n RMT - 1/1996, care
se obtine pentru k = 0.)
A
Solutie. Not
am cu A0 intersectia dreptelor
AA1 si BC. Avem AS = c x, AP = b x,

k
A1 S
cx
=
si cu relatia (R1 ) obtinem
A1 P
bx
S
0
A1
AS A B AC A1 P
P
0

= 1, de unde rezult
a c
a
AB A C AP A1 S

k+1
c bx
A0 B
.
=
B
C
A0 C
b cx
A
a utiliznd
Consideratii analoage relativ la punctele B 0 si C 0 . Concluzia rezult
reciproca teoremei lui Ceva.
Problema 3. Laturile (AB), (BC), (AC) ale triunghiului ABC sunt tangente
cercului nscris de centru I n punctele C1 , A1 , respectiv B1 . Daca B2 este mijlocul laturii (AC), demonstrati ca dreptele B1 I, A1 C1 si BB2 sunt concurente.
(Olimpiada, Republica Moldova)
am cu M punctul de
Solutie. Fie B 0 proiectia vrfului B pe latura AC. Not
intersectie al medianei BB2 cu B1 I. Deoarece B1 I AC, rezult
a c
a M B1 k BB 0 si
16

obtinem

BM
B 0 B1
=
. Dar
M B2
B1 B2

B 0 B1 = p a c cos A, B1 B2 =

A
b
(p a), deci
2

B
C1

B1

B2

b2 + c2 a2
I
pac
BM
2bc
=
=
M
b
M B2
(p a)
2
A1
C
B
b2 + c2 a2
b+ca
2
2
a
a

b
+
c

b
(a

c)
b
=
=
.
=
b (b + c a)
b (a c)
b
Dac
a A1 C1 BB2 = {M 0 } cu relatia (R2 ) avem
pb pb

b
0
BM
pb
2 (p a) (p c)
ab+c
pc pa
=
=

=
b pb b pb
M 0 B2
(p a) (p c)
pa+pc
b

+
2 pc 2 pa
si rezult
a c
a M M 0 ; dreptele A1 C1 , B1 I, BB2 sunt concurente.

Problema 4. Fie ABC un triunghi cu AB < AC, I centrul cercului nscris


si M mijlocul laturii BC. Notam cu D intersectia dintre IM si AB, iar cu E
intersectia lui CI cu perpendiculara din B pe AI. Sa se demonstreze ca DE k AC.
(Problema 2915, Crux Mathematicorum - 2/2004, propus
a de Toshio Seimiya)
Solutie. Not
am {B1 } = BE AC, {B 0 } =
A
BI AC. n triunghiul ABB1 , bisectoarea AI este si
n
altime, deci AB1 = AB = c, B1 C = b c. Cu teoD
BE
a
rema bisectoarei n 4BCB1 obtinem
.
=
B
EB1
bc
I
BD
Fie acum
a c
a
= x. Cu relatia (R2 ) rezult
B1
DA
BM
AC
x
E
BI
MC
=
;
B
C
0
M
BM
IB
+ B0A x
CB 0
MC
ab
bc
a
BI
BM
0
=
, deci
= 1, CB =
, B0A =
,
dar
0
MC
a+c
a + c IB
CB 0
bx
a
a+c
bx
=
x=
.
1=
bc
ab
b
a + cx
bc
+
x
a+c a+c
BD
BE
si rezult
a c
a DE k AC.
=
Deci
EB1
DA
Observatie. Alte solutii pentru Problema 2 (cazul k = 0) si Problema 3 pot fi
g
asite n [1].

Bibliografie
1. G. Popa, P. Georgescu - O metoda de demonstrare a concurentei unor drepte,
RecMat - 1/2004, 2932.
17

Asupra unor ecuatii diofantice p


atratice
Gheorghe MOLEA1
Scopul acestui material este de a prezenta cteva procedee de rezolvare a unor
ecuatii diofantice de gradul al doilea cu dou
a sau trei necunoscute accesibile nivelului
gimnazial si a le aplica n rezolvarea ctorva probleme ntlnite n Gazeta Matematica
sau alte publicatii.
Teorema 1. Solutiile ntregi ale ecuatiei
ax2 + bxy + cy + d = 0,

(1)

unde a, b, c Z si d Z, sunt

p c q ap + 2ac
2

Z;
p,
q

Z,
pq
=

ac
,
+
b
d
.
(x, y)
b
b2

(2)

aznd ac2 din ambii membri,


Demonstratie. nmultind ecuatia cu b2 si apoi sc
obtinem

ab2 x2 + b3 xy + cb2 y + db2 ac2 = ac2 (bx + c) abx + b2 y ac = ac2 b2 d.

Ca urmare, solutiile ntregi ale ecuatiei (1) sunt solutiile ntregi ale sistemelor
bx + c = p, abx + b2 y ac = q; p, q Z,

2
unde pq = ac + b2 d . Rezolvarea acestor sisteme conduce la solutiile
pc
q ap + 2ac
x=
,
, y=
b
b2
deci solutiile ntregi ale ecuatiei (1) sunt date de (2).

Aplicatie. Rezolvati n numere ntregi ecuatia 9x2 4xy y = 1 (Loredana


Agore E:12418, G.M. - 10/2002).

Solutie. Avem a = 9, b = 4, c = 1, d = 1, deci ac2 + b2 d = 7, de unde


(p, q) {(1, 7) , (7, 1) , (1, 7) , (7, 1)}. Tinnd

seama de (2), obtinem solutiile


(x, y) {(2, 5) , (0, 1)}.
Teorema 2. Solutiile ntregi ale ecuatiei

a, b, c Z , sunt

ax2 + bx ay 2 + c = 0,

(3)

p + q 2b q p
(4)
,
Z Z; p, q Z, pq = b2 4ac .
4a
4a
Demonstratie. nmultind (3) cu 4a si adunnd apoi b2 la ambii membri, obtinem

2 2
4a x + 4abx + b2 4a2 y 2 = b2 4ac (2ax + b 2ay) (2ax + b + 2ay) = b2 4ac.
(x, y)

Revine la a g
asi solutiile sistemelor liniare
2

2ax + b 2ay = p,

2ax + b + 2ay = q;

p, q Z,

cu pq = b 4ac. Solutiile ntregi ale acestora, deci si ale ecuatiei (3), sunt date de
(4).
1

Profesor, S
coala "Basarab I", Curtea de Arges

18

Aplicatie. Sa se rezolve n numere ntregi ecuatia x2 + 13x = y 2 26 (Gh.


Achim C:2487, G.M. - 3/2002).
Solutie. Avem a = 1, b = 13, c = 26 si b2 4ac = 65. Deci (p, q) {(1, 65),
(65, 1) , (5, 13) , (13, 5) , (1, 65) , (65, 1) , (5, 13) , (13, 5)} si, din (4), solutiile c
autate sunt (x, y) {(10, 16) , (10, 16) , (2, 2) , (2, 2) , (23, 16) , (23, 16),
(11, 2) , (11, 2)}.
Teorema 3. Solutiile n Z3 ale ecuatiei

x2 + ay 2 = a + b2 z 2 ,

a, b Z ,

(5)

sunt date de

x = t 2amnbm2 +abn2 , y = t an2 2bmnm2 , z = t an2 +m2 , t, m, n Z. (6)

Demonstratie. Ecuatia (5) se mai scrie (x + bz) (x bz) = a (z y) (z + y) si


este echivalent
a cu ansamblul sistemelor liniare
m (x + bz) = an (z y) ,

n (x bz) = m (z + y) ;

m, n Z.

Rezolvate n raport cu x si y, acestea conduc la solutiile

z 2amn bm2 + abn2


z an2 2bmn m2
x=
, y=
; m, n Z.
an2 + m2
an2 + m2

Notnd z = t an2 + m2 , t Z, obtinem ca posibile solutii ale ecuatiei (5) tripletele


(x, y, z) cu x, y, z dati de (6). Se verific
a usor c
a toate aceste triplete sunt solutii ale
ecuatiei date.
Aplicatie. Sa se arate ca exista o infinitate de numere ntregi x, y, z astfel nct
x2 + y 2 = 2z 2 (Dana si Eugen Radu Probleme de matematica pentru concursuri
si examene).
Solutie. Lund n (6) a = b = 1, g
asim c
a solutiile ecuatiei x2 + y2 = 2z 2 sunt
date de

x = t n2 2mn m2 , y = t n2 + 2mn m2 , z = t m2 + n2 , t, m, n Z.
Exercitiu. Rezolvati n Z Z ecuatia x2 y 2 = 3z 2 .
Teorema 4. Solutiile ecuatiei
ax2 + bxy + c2 y 2 = z 2 ,
n multimea Z3 sunt date de

x = t bm2 2cmn , y = t n2 am2 ,

a, b, c Z,

(7)

z = t bmncn2 acm2 ,

t, m, n Z. (8)

Demonstratie. Scriind ecuatia dat


a n forma x (ax + by) = (z cy) (z + cy) si
procednd ca n cazul ecuatiei (5) obtinem sistemele liniare
nx + cmy = mz,

amx + (bm cn) y = nz;

si n cele din urm


a rezultatul dorit.

m, n Z,

Aplicatie. Se considera a, b, c Z . Demonstrati ca ecuatia n x si z:


ax + bx + c2 = z 2 are o infinitate de solutii n multimea numerelor rationale (Gigel
Buth C:2690, G.M. - 12/2003).
2

19

Solutie. Ecua
tia din enunt se obtine din (7) pentru y = 1. Tinnd

seama de (8),

urmeaz
a c
a t n2 am2 = 1 si, deci,

bmn cn2 acm2


bm2 2cmn
, z=
; m, n Z cu n2 am2 6= 0.
2
2
n am
n2 am2
Asadar, ecuatia dat
a are o infinitate de solutii (x, y) cu x, y Q.
x=

Aplicatie. Sa se arate ca ecuatia x2 + xy + y 2 = 1 are o infinitate de solutii


numere rationale (L. Panaitopol, D. S
erb
anescu Probleme de teoria numerelor
si combinatorica pentru juniori, Problema 153).
Solutie. Proced
am ca n aplicatia precedent
a. Lund a = b = c = 1 si z = 1 n

(7), se obtine ecuatia dat


a. Din (8), avem t mn m2 n2 = 1 si apoi
x=

m2 2mn
Q,
mn m2 n2

y=

n2 m2
Q;
mn m2 n2

m, n Z.

Observatie. Pentru a = c = 1 si b = 0 ecuatia (7) devine ecuatia pitagoreic


a
x2 + y 2 = z 2, iar (8) conduce la solutiile acesteia: x = 2mnt, y = m2 n2 t,
z = m2 + n2 t cu t, m, n Z.

Truelul
Un truel este asem
an
ator cu un duel, dar exist
a trei participanti n loc de doi.
Domnii X, Y si Z se hot
ar
asc s
a rezolve un conflict truelndu-se cu pistoalele
pn
a cnd va r
amne n viata doar unul dintre ei. X este cel mai prost tr
ag
ator,
nimereste n medie tinta doar o dat
a din trei. Y este un tr
ag
ator mai bun, nimereste
tinta de dou
a ori din trei. Z este cel mai bun tr
ag
ator, nimereste tinta de fiecare
dat
a. Pentru a face truelul mai echitabil, X trage primul, urmat de Y (dac
a mai
este n viata), urmat de Z (dac
a mai este n viata) s. a. m. d., lund ostilit
atile de la
nceput, pn
a cnd r
amne n viata numai unul singur dintre ei.
ntrebarea este urm
atoarea: asupra cui ar trebui s
a trag
a X primul s
au foc?
Not
a. R
aspunsul se g
aseste la p. 70.

20

O caracterizare a functiilor convexe cu ajutorul


derivatelor laterale
Florin POPOVICI 1
Vom presupune cunoscute propriet
atile elementare ale functiilor convexe; n aceast
a
privinta pot fi consultate [1] sau [3]. Un rol aparte l va juca urm
atoarea
Teorem
a (O. Stolz; v. Teorema 1.3.3 din [1] sau Propozitia 7.1.4 din [3]). Daca
f : I R este o functie convexa pe intervalul deschis I, atunci f este derivabila la
stnga si la dreapta n orice punct din I si, daca x, y I si x y, avem
0
0
0
0
f
(x) f+
(x) f
(y) f+
(y) .
(1)
Nota de fata are strns
a leg
atur
a cu lucrarea [2] si o presupune cunoscut
a. Utiliznd derivatele laterale n locul derivatei, n [2] este presentat
a o generalizare a
teoremei de medie a lui Lagrange si, ca o consecinta, se face urm
atoarea leg
atur
a cu
functiile convexe:
Teorema lui Lagrange pentru functii convexe ([2], Corolarul 1 ). Fie f :
I R o functie convexa, I deschis. Atunci, a, b I cu a < b, c (a, b) nct
f (b) f (a)
0
0
f
f+
(c)
(c) .
(2)
ba
Scopul nostru este de a dovedi o reciproc
a a Teoremei lui Stolz si, astfel, de a
obtine o caracterizare a functiilor convexe Corolarul de mai jos.
Teorem
a. Daca f : I R, I interval deschis, este derivabila la stnga si la
dreapta pe I si x, y I, x < y, au loc inegalitatile (1), atunci f este convexa pe I.
Demonstratie. Fie x, y, z I cu x < y < z. Teorema precedent
a asigur
a c
a
c (x, y) si d (y, z) astfel nct
f (y) f (x)
f (z) f (y)
0
0
0
0
f
f+
f+
(c)
(c) si f
(d)
(d) .
yx
zy
0
0
(c) f
(d), rezult
a c
a
Cum, datorit
a ipotezei, c < d implic
a f+

1
1
1

f (y) f (x)
f (z) f (y)
y
z 0

x
yx
zy
f (x) f (y) f (z)

si, conform Corolarului 3 al Propozitiei 1.4.3 din [3], functia f este convex
a.
Corolar. Fie I R un interval deschis. O functie f : I R este convexa daca
si numai daca f este derivabila la stnga si la dreapta pe I si are loc (1) pentru orice
x, y I cu x < y.

Bibliografie
1. C. P. Niculescu - Convex Function, Basic Theory and Applications, Universitaria
Press, Craiova, 2003.
2. F. Popovici - O generalizare a teoremelor de baza ale calculului diferential, RecMat 2/2004, 104-105.
3. Gh. Siretchi - Calcul diferential si integral, vol. I, Ed. S
t. si Encicl., Buc., 1985.
1

Profesor, Liceul "N. Titulescu", Brasov

21

Asupra problemei G67


Adrian ZAHARIUC 1
n RecMat - 2/2004, p. 157, am propus urm
atoarea problem
a:
Problema 1. Spunem ca un numar natural este decompozabil daca se poate
scrie ca suma a doua numere naturale cu aceeasi suma a cifrelor. Sa se arate ca
exista o infinitate de numere naturale care nu sunt decompozabile.
n original, problema se referea la o baz
a de numeratie oarecare b. Un rationament simplu (un exemplu trivial si un argument de paritate) ne arat
a c
a n cazul
b impar, multimea numerelor decompozabile coincide cu multimea numerelor pare,
deci problema este epuizat
a. n cazul b par, numerele nedecompozabile sunt mult
mai rare. n cele ce urmeaz
a, ne vom referi la cazul decimal, b = 10, pentru a evita
complicatii inutile. Cititorul poate extinde folosind exact aceeasi tehnic
a rezultatul
obtinut n cazul b = 10 pentru orice num
ar b par.
Scopul acestei Note este de a da o form
a general
a simpl
a tuturor numerelor nedecompozabile. Voi da r
aspunsul la aceast
a problem
a nc
a din enunt:
Problema 2. Un numar natural n este nedecompozabil daca si numai daca are
una din formele: 19 . . . 99, 39 . . . 99, 59 . . . 99, 79 . . . 99, 99 . . . 99, cu un numar impar
de cifre sau 29 . . . 99, 49 . . . 99, 69 . . . 99, 89 . . . 99, cu un numar par de cifre.
Solutie. S
a rezolv
am nti partea mai delicat
a: dac
a n nu are nici una dintre formele mentionate mai sus, atunci n este decompozabil. Aceasta rezult
a din
urm
atoarele dou
a leme:
Lema 1. Pentru orice astfel de n, exista a n astfel nct
s(a) s(n a)(mod 2).

Demonstratie. Dac
a s(n) este par, atunci lu
am direct a = 0, deci ne r
amne
s(n) impar. n acest caz, n trebuie s
a aib
a o cifr
a, n afar
a de prima, diferit
a de
9 deoarece altfel ar avea una dintre formele interzise. Fie c valoarea acestei cifre
si p pozitia ei (de la dreapta la stnga). Evident, putem alege aceast
a cifr
a astfel
nct naintea ei s
a nu fie cifra zero. Atunci lu
am a = 10p1 (c + 1). La adunarea
a + (n a) = n se face un singur transport, deci
s(n a) + s(a) = s(n) + 9 0(mod 2) s(a) s(n a)(mod 2).

Lema 2. Daca exista a n astfel nct s(a) s(n a)(mod 2), atunci exista
A n astfel nct s(A) = s(n A).
Demonstratie. Fie k num
arul de cifre ale lui n. Not
am
a = a1 a2 . . . ak ,

n a = b1 b2 . . . bk ,

unde cteva dintre primele cifre pot fi 0. S


tim c
a

0 a1 + + ak + b1 + + bk = (a1 + b1 ) + + (ak + bk )(mod 2),


1

Elev, cl. a X-a, Colegiul National "Ferdinand I", Bac


au

22

deci num
arul elementelor multimii I = {i {1, 2, . . . , k}; 2 nu divide ai + bi } este
par. Atunci exist
a I = I1 I2 o partitie a lui I n dou
a clase cu acelasi num
ar de
elemente. Fie

i
/I

(ai + bi ) /2,
(ai + bi + 1) /2, i I1
Ai =

(ai + bi 1) /2, i I2

si Bi = ai + bi Ai , i = 1, k. Este clar c
a Ai , Bi {0, 1, . . . , 9}. Avem
A1 A2 . . . Ak +B1 B2 . . . Bk =

k
X

(Ai + Bi ) 10ki =

i=1

Avem c
a

k
X

Ai =

i=1

dar

k
X

Ai +

i=1

k
X
i=1

(ai + bi ) 10ki = a+na = n.

i=1

k
X
ai + bi
i=1

k
X

Bi =

k
X
i=1

|I1 | |I2 | X ai + bi

=
,
2
2
2
i=1

(ai + bi )

k
X
i=1

Ai =

k
X

Bi ,

i=1

a.
deci s(A) = s(n A), unde A = A1 A2 . . . Ak si prima parte este rezolvat

S
a presupunem acum c
a n are una dintre formele interzise si s
a demonstr
am c
a
este nedecompozabil. S
a presupunem prin absurd c
a exist
a a, b n cu a + b = n
astfel nct s(a) = s(b). Observatia esential
a este c
a la adunarea a + b = n nu se fac
transporturi. S
a presupunem prin absurd c
a totusi am avea transporturi. S
a lu
am
cifra cea mai nesemnificativ
a (cea mai din dreapta) la care se face transport. Din
faptul c
a este cea mai nesemnificativ
a cifr
a cu aceast
a proprietate rezult
a c
a la cifra
din dreapta sa nu s-a facut transport. Atunci aceast
a cifr
a este obtinuta doar prin
adunarea unei cifre m a lui a cu o cifr
a n a lui b, dar m + n 18, iar restul mp
artirii
lui m + n la 10 trebuie s
a fie 9, deci m + n = 9. Rezult
a c
a la acea cifr
a nu s-a f
acut
transport, contradictie. Atunci s(n) = s(a) + s(b) = 2s(a), deci s(n) este par, ns
a
nici unul dintre numerele care intr
a n discutie nu are suma cifrelor par
a si am ajuns
la o contradictie, deci n este nedecompozabil.

1. n "egalitatea"
XXIII
= II
V II
mutati un betisor astfel nct s
a obtineti o egalitate aproximativ
a ct mai bun
a.
Roxana C
ap
atn
a, elev
a, Iasi
2. Ad
augati o cifr
a par
a la dreapta unui num
ar, astfel nct s
a obtineti un num
ar
impar.
Gabriel Popa, Iasi
Not
a. R
aspunsurile se g
asesc la p. 26.
23

Matematic
a si algoritmi
Irina MUSTAT

A1
Majoritatea problemelor de informatic
a se rezolv
a cu ajutorul unei serii de algoritmi, care se adapteaz
a de la caz la caz. Din problemele de mai jos, se va vedea c
a
uneori anumite observatii matematice fie usureaz
a abordarea acestor probleme, fie
nt
aresc rezultatul.
1. La Concursul Info-Oltenia din 2004 s-a dat urm
atoarea problem
a:
Sa se afiseze numarul de descompuneri ale unui numar dat n suma de numere
naturale nenule. O descompunere si o permutare a ei nu se vor numara de doua ori.
(Enunt adaptat).
Un programator "constiincios" va aplica probabil backtracking (algoritm care
genereaz
a toate solutiile de descompunere) si apoi le va contoriza. Desi o astfel
de abordare este corect
a, se stie c
a backtracking-ul are o complexitate (num
ar de
operatii efectuate) exponential
a, determinnd un timp de executie mai ndelungat.
S
a privim problema altfel: vom ncerca s
a g
asim o functie recursiv
a care returneaz
a num
arul c
autat. Pentru a evita num
ararea simultan
a a unei solutii si a permut
arii ei, vom lua numerele care apar n descompunerea lui n n ordine cresc
atoare.
Presupunem c
a putem construi o functie recursiv
a f (n) (cu un singur parametru)
care returneaz
a num
arul cerut. Atunci f (n) = f (n 1) + f (n 2) + + f (1).
(Adic
a din n sc
adem k iar num
arul de descompuneri posibile va fi f (n k), apoi
facem suma dup
a k). Numai c
a, n acest caz apare o problem
a major
a: dac
a scriem
n = 2+n2 si calcul
am f (n 2), la un moment dat primul termen al descompunerii
ar fi 1, deci am obtine n = 2 + 1 + . . . , ceea ce contrazice ordonarea aleas
a pentru
numerele din descompunerea lui n.
Se impune, asadar, necesitatea construirii unei functii cu doi parametri f (n, k),
cu urm
atoarea semnificatie: f (n, k) reprezint
a num
arul de descompuneri distincte
ale lui n n sum
a de termeni k. Este clar c
a enuntul problemei cere f (n, 1). De
asemenea se observ
a c
a f (n, n) = 1 si f (n, k) = 0 pentru k > n.
Acum putem scrie definitia functiei f (n, k), c
areia i putem da ulterior parametrii
n, 1:

1,
n = k,

0,
n < k,
f (n, k) =
n1

f (n l, l) , n > k.

l=k

Se observ
a c
a att codul, ct si resursele consumate sunt mult mai restrnse dect
n cazul backtracking-ului.
2. O alt
a problem
a de acelasi gen ar fi:
Afisati toate solutiile ecuatiei 3x + y + 4xz = 100 cu x, y, z N.
1

Elev
a, cl. a XII-a, Colegiul National, Iasi

24

S
i n acest caz este de asteptat o aplicare de backtracking, dar putem reformula
ecuatia: x (4z + 3) = 100 y. Acum ration
am n
felul urm
ator. Alegem z de la 0 la
100
24 si, pentru z astfel luat, alegem x de la 1 la
. Apoi l obtinem pe y din
4z + 3
100 x (4z + 3) si tip
arim tripletul (x, y, z). n final tip
arim si solutiile (0, 100, z) cu
z N. Prin metoda de mai sus, obtinem ntr-adev
ar toate solutiile (z poate lua orice
valoare de la 0 la 24, deoarece pentru z 25 si x nenul
a
avem
4z + 3 > 100; totodat
100
100
si, cum x N, x
).
100 y 100, deci x
4z + 3
4z + 3
Observatie. Num
arul 100 poate fi nlocuit cu orice n N .
3. Un alt exercitiu interesant este urm
atorul:
Se dau m numere ntregi nenule b1 , . . . , bm si n numere ntregi nenule a1 , a2 , . . . ,
an . Sa se determine o submultime a multimi B = {b1 , . . . , bm } care sa maximizeze
valoarea expresiei
E = a1 x1 + a2 x2 + + an xn ,
stiind ca m n si xi {b1 , . . . , bm }, i = 1, n.
Sort
am cresc
ator ambele siruri (ai ) si (bi ). Evident, la ambele siruri vom avea
numerele negative n partea stng
a si cele pozitive n partea dreapt
a.
Vom enunta inegalitatea rearanjarilor, pe care o vom aplica la aceast
a problem
a:
Fie a1 a2 an , b1 b2 bn siruri de numere reale si
n
P
S () =
ai b(i) , unde este o permutare a multimii {1, 2, . . . , n}. Atunci S ()
i=1

este maxim pentru (i) = i, i = 1, n, si minim pentru (i) = n + 1 i, i 1, n.

S
irul (ai ) este alc
atuit din k numere strict negative si n k numere ntregi strict
pozitive. Pentru a maximiza suma este necesar s
a nmultim numerele pozitive din
(ai ) cu cele mai mari numere din (bi ) si numerele negative cu cele mai mici numere
din (bi ) (deoarece cele mai mici numere strict negative din (bi ) au modulul mai mare,
deci ai bi > 0 va putea fi maximizat, iar cele mai mici numere pozitive nmultite cu
numere negative vor da cele mai mici n modul numere negative, deci cei mai mari
termeni). Asadar vom nmulti k numere negative ale sirului (ai ) cu cele mai mici k
numere ale sirului (bi ), iar cele n k numere pozitive ale sirului (ai ) le vom nmulti
a submultimi nu se vor
cu cele mai mari n k numere ale sirului (bi ). Aceste dou
suprapune, ntruct m n, iar num
ar
atoarea ncepe din capetele opuse ale sirului
arilor obtinem c
a permutarea lui {b1 , . . . , bk }
(bi ) sortat. Din inegalitatea rearanj
k
P
pentru care
ai bi este maxim
a e permutarea identic
a si analog pentru multimile
i=1

{ak+1 , . . . , an } si {bmn+k+1 , . . . , bm }.
S
a remarc
am c
a programul propriu-zis va fi astfel redus la niste pasi extrem de
arului k de elemente
simpli: sortarea cresc
atoare a sirurilor (ai ) si (bi ), aflarea num
negative ale lui (ai ) si, n final, tip
arirea solutiei: primele k numere din stnga sirului
(bi ) si ultimele n k numere din dreapta din (bi ).
25

4. Un colier este format din margele rosii si albastre. La un moment dat taiem
colierul ntr-un punct si l ntindem n linie dreapta. Apoi scoatem de pe fir margelele
din capatul stng pna cnd ntlnim una din cealalta culoare. Analog procedam
pentru capatul drept. Dndu-se datele despre colier sub forma unui sir rarra. . . (care
indica culoarea margelelor succesive), sa se calculeze numarul maxim de margele pe
care l putem scoate, precum si locul unde trebuie taiat colierul. (Enunt adaptat).
(Olimpiada International
a de Informatic
a, 1993)
Consider
am un vector cu n componente (adic
a un sir a1 , . . . , an ). Initializ
am cu
0 toate elementele vectorului, ne plas
am pe primul element al vectorului si citim sirul
colierului caracter cu caracter, ct
a vreme caracterul citit este acelasi cu cel anterior;
m
arim cu 1 valoarea retinut
a de primul element al vectorului. Cnd caracterul se
schimb
a ne pozition
am pe urm
atoarea component
a a vectorului si continu
am pn
a
la sfrsitul sirului cu acelasi procedeu. De fapt, vectorul retine lungimile tuturor
secventelor succesive de m
argele colorate la fel.
Cnd t
aiem colierul, o putem face n dou
a feluri: fie n interiorul unei secvente
colorate la fel, fie desp
artind dou
a secvente colorate diferit. n primul caz, dup
a
ce ntindem colierul, num
arul maxim de m
argele ce pot fi scoase este num
arul de
m
argele al acelei secvente, n timp ce n al doilea caz, num
arul maxim obtinut este
suma numerelor de m
argele din dou
a secvente consecutive. Este evident c
a pentru
noi e mai convenabil s
a t
aiem colierul ntre dou
a secvente.
Maximul care poate fi obtinut este maximul dintre sumele numerelor de m
argele
a dou
a secvente succesive. Acesta se poate g
asi uitndu-ne la vectorul completat
anterior. Fie k num
arul de elemente nenule al acestui vector. Dac
a k este par,
compar
am sumele v [1]+v [2], v [2]+v [3], . . . , v [k 1]+v [k]. Dac
a k este impar, cum
oricare dou
a secvente consecutive au culori diferite, rezult
a c
a secventele num
arate
de v [1] si v [k] au aceeasi culoare, deci la "nchiderea" colierului se vor uni ntr-o
singur
a secventa de lungime v [1] + v [k]. n acest caz, trebuie aflat maximul dintre
v [k] + v [1] + v [2], v [1] + v [k] + v [k 1], v [i] + v [i + 1], i = 2, k 2.
Locul unde trebuie t
aiat colierul se obtine usor. Presupunem c
a facem t
aietura
i
P
ntre v [i] si v [i + 1]. Atunci pozitia unde vom t
aia este
v [j].
j=1

Solutiile problemelor enuntate la p. 23.


1. Mutnd un betisor de la num
ar
ator deasupra num
arului din membrul doi, se
obtine
XXII
= ,
V II
22
XXII
=
3, 14285 . . .,
care este o egalitate aproximativ
a cu o eroare foarte mic
a(
V II
7
iar 3, 14159 . . .).
2. Se adaug
a cifra 0, dar . . . la exponent, obtinndu-se num
arul impar 1 (de
exemplu, 70 = 1 etc.).
26

Asupra unei probleme de concurs


Alexandru NEGRESCU 1
La cea de-a VI-a editie a Concursului interjudetean de matematica "Radu Miron",
care a avut loc la Vaslui n perioada 5 - 7 noiembrie 2004, a fost propus
a elevilor de
clasa a X-a urm
atoarea problem
a:
Daca A, B, C sunt masurile unghiurile unui triunghi, atunci

1
1
1
+
+
2 3.
sin A sin B sin C
Precizati cnd are loc egalitatea.
V
a prezent
am n continuare cinci solutii pentru aceast
a inegalitate, l
asndu-v
a pe
dumneavostr
a s
a decideti care este cea mai frumoas
a.
Solutia I (prezentat
a n baremul de corectare). Deoarece sin A, sin B, sin C
sunt strict pozitive, avem
X
X 1
X 1
9
sin A
9 sau
P
.
sin A
sin A
sin A
Conform inegalitatii Cauchy - Buniakowski - Schwarz, avem
X
2
X
X 1 cos 2A
9 3X
=
cos 2A.
sin2 A = 3
sin A 3
2
2 2
Cum
X
cos 2A = 2 cos2 C 1 2 cos (A B) cos C =
i
1h
3
=
(2 cos C cos (A B))2 + sin2 (A B) 3 ,
2
2
rezult
a c
a

X
2 9 9
X
27
3 3
sau
sin A
.
sin A + =
2 4
4
2

P 1
2
9 = 2 3, adic
a inegaliCombinnd rezultatele precedente obtinem
sin A
3 3
tatea dorit
a. Se constat
a usor c
a egalitatea are loc dac
a si numai dac
a triunghiul
este echilateral.
Solutia a II-a (Alexandru Negrescu). Conform teoremei sinusurilor, avem:
a
sin A =
etc. Inegalitatea de demonstrat devine
2R

2R 2R 2R
1 1 1
3
+
+
2 3R
+ +
3 1 1 1 3R.
a
b
c
a b
c
a + b + c

1 1 1
a+b+c
2p
2p
Cum 3/
+ +

=
, r
amne s
a ar
at
am c
a
3R sau
c
3
3
3
a b
3 3
p
R, ceea ce este o cunoscut
a inegalitate a lui Mitrinovic.
2
1

Elev, cl. a X-a, Colegiul National "August Treboniu Laurean", Botosani

27

Solutia a III-a (Alexandru Negrescu si prof. Liliana Tomita). Conform


inegalit
atii mediilor avem
1
1
3
1
+
+

.
3
sin A sin B sin C
sin A sin B sin C

3 3
([1], pag. 123-124), cu egalitate pentru triunghiul
S
tim c
a sin A sin B sin C
8
echilateral. nlocuind mai sus, obtinem
s
. 3 33
. 3

1
1
1
+
+
3
=3
= 2 3.
sin A sin B sin C
8
2
Solutia a IV-a (Alexandru Turcanu

si S
erban Vatavu, elevi, C.N. "M. Eminescu", Botosani). Conform inegalit
atii mediilor, avem
1
1
1
9
+
+

.
sin A sin B sin C
sin A + sin B + sin C
Dar, conform inegalitatii lui Jensen pentru functii concave are loc

A+B+C
sin A + sin B + sin C
3 3
sin

sau sin A + sin B + sin C


,
3
3
2
care, nlocuit
a n relatia precedent
a, conduce la inegalitatea dorit
a.
Solutia a V-a (Alexandru Negrescu). Ridicnd relatia din enunt la p
atrat,
obtinem
X 1
X
1
12
+
2
2
sin
A
sin B
sin A
sau
P
X 1
sin A
12.
()
+2 Q
sin A
sin2 A
Q
1
Cu inegalitatea mediilor si tinnd seama c
a cos A , rezult
a c
a
8
X 1
9
9
9
Q

= 4.
P 2 =
2 + 2 cos A
2 + 2 18
sin2 A
sin A

Q
3 3
Pe de alt
a parte, cu inegalitatea mediilor si tinnd seama c
a sin A
, avem
8
si
v
u !2
p
Q
P
.
3
u
23
sin A
3 3
sin A
6
3
Q
2Q
6 t
= 8.

= qQ
8
sin A
sin A
3
2
( sin A)
Prin adunare, obtinem inegalitatea (), care este echivalent
a cu cea de demonstrat.
Bibliografie
1. T. Cohal - Probleme de trigonometrie, Editura Moldova, Iasi, 1994.
2. M. Ganga - Manual pentru clasa a IX-a, profil M1, M2, Editura Mathpress, Ploiesti,
2003.
3. M. Ganga - Manual pentru clasa a X-a, profil M1, Editura Mathpress, Ploiesti,
2003.
28

Din nou asupra unei probleme de concurs


Constantin APOSTOL1
n revista "Recreatii matematice" nr. 2, iulie - decembrie 2004, a ap
arut articolul
profesorilor D. Mihalache si M. Tetiva, care "generalizeaz
a" o problem
a pe care
am propus-o la Concursul National de Matematica "Laurentiu Duican" din 15-17
mai 2003, cu urm
atorul enunt:
b B,
b C
b si D
b sunt pron patrulaterul convex ABCD, masurile unghiurilor A,
portionale cu numerele 8, 12, 5 si 11. Sa se arate ca daca (BD estre bisectoarea
b atunci (AC este bisectoarea unghiului A.
b
unghiului B,

b
b
b
b
m(A)
m(B)
m(C)
m(D)
Printr-un calcul simplu, din faptul c
a
=
=
=
, deducem
8
12
5
11

b = 120 , m(C)
b = 50 , m(D)
b = 110 .
b = 80 , m(B)
c
a m(A)
Problema am propus-o pentru clasa a VII-a, asa c
a o rezolvare trigonometric
a nu
era de asteptat din partea elevilor. Profesorii nominalizati mai sus, au dat o elegant
a
solutie trigonometric
a, care dep
aseste nivelul de preg
atire al elevilor, dar au dat si o
solutie geometric
a, la nivelul programei scolare, care le-a permis si o "generalizare",
n sensul elabor
arii unor probleme rezolvabile pe aceeasi idee.
n rndurile care urmeaz
a mi propun s
a exprim solutiile cu care am trimis problema comisiei de concurs, care se ncadreaz
a cerintelor programei de concurs si care
este aceeasi cu a Olimpiadei Nationale pentru clasa a VII-a.
Solutia I. Prelungind latura (AB), obtinem
C
\ are m
c
a suplementul unghiului ABC
asura de 60
b care are
D
si cum (BD este bisectoarea unghiului B,
a (BC este bisectoarea
m
asura de 120 , deducem c
b din triunghiul ABD.
exterioar
a a unghiului B
n triunghiul ABD, prin calcul, deducem c
a

\
\
m(ADB) = 40 si, deci, m(BDC) = 70 . Suple\ care se obtine prelungind
mentul unghiului ADC,
B
latura (AD), are 70 . Asadar, (DC este bisec- A
b
toarea exterioar
a a unghiului D n triunghiul ABD.
Astfel, pentru triunghiul ABD, (BC si (DC sunt bisectoarele exterioare ale
b respectiv D;
b acestea sunt concurente n C. Deducem c
unghiurilor B,
a (AC este
b
bisectoarea interioar
a a unghiului A.

Solutia a II-a. Pe semidreapta (AB lu


am punctul E astfel nct AE = AD. Vom
b = 80 , deducem
ar
ata c
a 4ADC 4AEC. Din triunghiul isoscel ADE, cu m(A)
\ = 60 , c
\ = 110 . Dar
\ = m(AED)
\ = 50 . Rezult
a m(EDC)
aci m(ADC)
m(ADE)
\
\
\
\ = 60 .
(BD este bisectoarea unghiului ABC, deci m(ABD) = m(DBC) = m(CBE)
1

Profesor, Colegiul National "Al. Vlahuta", Rmnicu S


arat

29

\ =
Avem, n patrulaterul convex DBEC, m(EDC)

\
= m(EBC) = 60 , deci acest patrulater este
\ =
inscriptibil, de unde deducem c
a m(CED)

\
= m(CBD) = 60 .
\ = 60 , de\ = 60 si m(EDC)
Din m(CED)
ducem c
a triunghiul CDE este isoscel cu vrful C si
deci, CD = CE. Rezult
a 4ADC 4AEC (LLL);
b
asadar, (AC este bisectoarea unghiului A.

C
D

n continuare, voi prezenta o problem


a care se poate rezolva cu ideea problemei
anterioare:
b = 60 , m(B)
b = 105 ,
n patrulaterul convex ABCD masurile unghiurilor sunt: m(A)

b = 60 , m(D)
b = 135 . Sa se arate ca daca (BD este o trisectoare a unghiului
m(C)
b
b sau (CA este bisectoarea unghiului C.
b
D, atunci (AC este bisectoarea unghiului A
Solutie. Vom deosebi dou
a cazuri:
b
m(
D)
135
\ =
I. cnd m(CDB)
=
= 45 ;
3
3

b
\ = m(D) = 135 = 45 .
II. cnd m(ADB)
Y
C
3
3
60D
n cazul I, prelungind latura (AB), obtinem
D
45
\ = 180 105 = 75 , deci (BC este biD 45D
m(CBX)
90D
\ (1). Prelungind latura
sectoarea unghiului DBX
\ ) = 180 135 = 45 , deci
(AD), obtinem m(CDY
75D 75D
60D
30D
\ (2).
(DC este bisectoarea unghiului BDY
X
B
Din (1) si (2), deducem c
a (AC este bisectoarea A
b
unghiului A.
n cazul II, prelungind latura (CB), obtinem
C
D D 60D
[ = 180 105 = 75 , deci (BA este bim(ABZ)
T
90
45D D
\ (3).
sectoarea unghiului DBZ
45
Prelungind latura (CD), obtinem
[ ) = 180 135 = 45 ,
m(ADT

\ (4).
deci (DA este bisectoarea unghiului BDT
Din (3) si (4), deducem c
a (CA este bisectoarea
b
unghiului C.
Bibliografie

30D
60D

75D
75D

B
Z

1. F. Diac - A XI-a editie a Concursului National de Matematica "Laurentiu Duican",


Brasov, 2003, G.M. - 11/2003, 433-438.
2. D. Mihalache, M. Tetiva - Asupra unei probleme de concurs, RecMat - 2/2004,
111-113.
3. C. Apostol - Preocupari matematice, Ed. "Radical", 1966.
30

Dou
a functii cu aceeasi derivat
a pe un interval
nu difer
a neap
arat printr-o constant
a
Paul GEORGESCU 1 , Gabriel POPA2
Printre "cunostintele" dobndite de unii elevi n urma studierii Analizei Matematice de liceu se num
ar
a, din p
acate, si urm
atoarea "teorem
a", mentionat
a n manualul
[1], pag. 282:
Daca f, g : (a, b) R au aceeasi derivata pe (a, b), atunci ele difera printr-o
constanta.
"Demonstratia" urmeaz
a linia de mai jos:
Dac
a f 0 (x) = g 0 (x), x (a, b), atunci (f g)0 (x) = f 0 (x) g 0 (x) = 0,
x (a, b), deci f g este constant
a pe (a, b).
Desigur, nu avem neap
arat c
a (f g)0 (x) = f 0 (x) g 0 (x), x (a, b), egalitate
ce nici m
acar nu are sens pentru f 0 (x) = g 0 (x) = si deci demonstratia de mai
sus este invalid
a.
Observ
am c
a "f are derivat
a pe (a, b)" nu este acelasr
i lucru cu "f este derivabil
a pe
a+b
3
, care are derivat
a
(a, b)"; un exemplu este dat de f : (a, b) R, f (x) = x
2

a+b
a+b
= fd0
= +.
pe (a, b) f
ar
a a fi derivabil
a pe (a, b), deoarece fs0
2
2
Dac
a, n plus, f si g sunt presupuse a fi derivabile pe (a, b), atunci rationamentul
de mai sus este corect, conducnd la urm
atorul binecunoscut rezultat:
Daca f, g : (a, b) R derivabile au aceeasi derivata pe (a, b), atunci ele difera
printr-o constanta.
Prezent
am n continuare un exemplu, datorat matematicianului polonez Stanislaw Ruziewicz, de un num
ar infinit de functii cu aceeasi derivat
a pe un interval,
diferenta oric
aror dou
a nefiind constant
a, [3]. Vezi, de asemenea, [2] pag. 60.
Fie s 1. mp
artim segmentul [0, s] n trei p
arti n asa fel nct segmentul mij1
lociu are lungimea , iar centrul s
au este centrul segmentului [0, s]. Acoperim apoi
3
1
segmentul mijlociu cu semicercul superior de diametru determinat de acesta si apoi
3
stergem acest segment (exclusiv capetele). mp
artim apoi fiecare dintre cele dou
a
segmente r
amase n cte trei segmente n asa fel nct segmentele mijlocii s
a aib
a
1
artite s
a coincid
a cu centrele seglungimea 2 , iar centrele segmentelor care sunt mp
3
mentelor mijlocii. De asemenea, acoperim segmentele mijlocii cu semicercurile de di1
ametru 2 determinate de acestea si apoi stergem aceste segmente, exclusiv capetele.
3
Repetnd aceast
a procedur
a, la pasul n vom avea de construit 2n1 semicercuri de
1
atoare, exclusiv capetele, s. a. m. d.
diametru n si de sters diametrele corespunz
3
Obtinem deci o infinitate num
arabil
a de semicercuri si fie Ms reuniunea segmentelor
r
amase neacoperite si a semicercurilor.
1
2

Lector dr., Catedra de matematic


a, Univ. Tehnic
a "Gh. Asachi", Iasi
Profesor, Colegiul National, Iasi

31

a Ms poate fi privit ca graficul unei functii continue fs : [0, s]


evident c
Este
1
a a unui anumit sir
0, ; demonstratia acestui fapt foloseste convergenta uniform
6
de functii continue.
h i
Rt
, Fs (t) = 0 fs (x) ds, pentru 0 t s. Atunci
Fie atunci Fs : [0, s] 0,
56
a cu derivata continu
a, iar deoarece fs (x) 0, x [0, s], Fs este si
Fs este derivabil
a c
a pentru orice
cresc
atoare. Mai mult, din modul de constructie a lui Ms se observ
0 t1 < t2 s intervalul [t1 , t2 ] are un subinterval comun cu un interval acoperit
s
de un semicerc, deoarece dup
a pasul n orice subinterval al lui [0, s] de lungime n
2
are un subinterval comun cu un interval acoperit de un semicerc.
Cum pe aceste subintervale f ia valori strict pozitive si este continu
a, obtinem c
a
Fs (t1 ) < Fs(t2 ), deci Fs este
strict
cresc
a
toare.
Mai
mult,

1
22

+
+ . . . = , iar Fs (0) = 0.
Fs (s) =
8 32 34
56
h i

atoare si surjectiv
a, ea este inversabil
a si fie s : 0,
Cum Fs este strict cresc
56
a si strict cresc
atoare, s
[0, s] inversa sa. Observ
am c
a, deoarece Fs este continu
0
este hde asemenea
continu
a

s
i
strict
cresc
a
toare.
Determin
a
m
acum

(u)
pentru
s
i
.
u 0,
56
1
1
=
.
Dac
a u = Fs (ts ), 0 ts s, cu fs (ts ) 6= 0, atunci 0s (u) = 0
Fs (ts )
fs (ts )
0
Dac
a u = Fs (ts ), 0 ts s, cu fs (ts ) = 0, atunci Fs (ts ) = 0 si nu putem aplica
formula de derivare a functiei inverse. Totusi, utiliznd definitia derivatei si faptul
c
a Fs este strict cresc
atoare, obtinem c
a 0s (u) = +.
a a integralei
Fie acum 1 s1 < s2 < . Utiliznd semnificatia geometric
Riemann, remarc
am c
a are loc urm
atoarea proprietate:
Daca 0 t1 s1 si 0 t2 s2 , iar Fs1 (t1 ) = Fs2 (t2 ), atunci fs1 (t1 ) = fs2 (t2 )

(este important de notat c


a razele semicercurilor construite pentru cele 2 valori
ale lui s nu depind de s, iar dac
a Fs1 (t1 ) = Fs2 (t2 ), atunci M1 (t1 , fs1 (t1 )) si
))
trebuie
s
a
se
afle
la
aceea
si n
altime, pe semicercuri corespunz
atoare).
M2 (t2 , fs2 (t
2
h i
. Dac
a u = fs1 (ts1 ), 0 ts1 s1 , cu fs1 (ts1 ) 6= 0, fie 0 ts2 s2
Fie u 0,
56
astfel ca u = fs2 (ts2 ). Atunci Fs1 (ts1 ) = Fs2 (ts2 ), deci fs1 (ts1 ) = fs2 (t2 ), conform
1
propriet
atii de mai sus, si deci 0s1 (u) = 0s2 (u) =
. Dac
a u = Fs1 (ts1 ),
fs1 (ts1 )
0 ts1 s1 , cu fs1 (ts1 ) = 0, fie 0 ts2 s2 astfel ca u = Fs2 (ts2 ). Atunci
Fs1 (ts1 ) = Fs2 (ts2 ), deci fs1 (ts1 ) = fs2 (ts2 ) = 0, conform propriet
atii de mai sus, si
deci 0s1 (u) = 0s2 (u) = +.
h i
. Cum Fs1 (0) = Fs2 (0) = 0,
a pe 0,
n concluzie, s1 si s2 au aceeasi derivat
56

iar Fs1 (s1 ) = Fs2 (s2 ) =


, deducem c
a s1 (0) = s2 (0) = 0, iar s1
= s1 ,
56
56

s2
a printr-o constant
a. De aici, multimea
= s2 , deci s1 si s2 nu difer
56
h i
F = {s ; s 1} este o multime de functii cu aceeasi derivat
, diferenta
a pe 0,
56
32

oric
aror dou
a nefiind constant
a, ceea ce ncheie constructia exemplului.
*
*
*
Prezent
am n cele ce urmeaz
a, pe scurt, definitia derivatelor Dini ale unei functii
reale [2].
Fie f : (a, b) R si x0 (a, b). Numim derivata Dini superioara (respectiv
f (x) f (x0 )
arul D+ f (x0 ) = lim sup
inferioara ) la dreapta a lui f n x0 num
x&x0
x x0
f (x) f (x0 )
(respectiv D+ f (x0 ) = lim inf
). Analog se pot defini derivata Dini
x&x0
x x0
superioara, respectiv inferioara, la stnga a lui f n x0 , notate D f (x0 ), respectiv
D f (x0 ).
Privitor la relatia dintre derivatele Dini si derivatele clasice ale unei functii ntr-un
a dac
a D+ f (x0 ) = D+ f (x0 ), atunci f are derivat
a la
punct x0 , se poate observa c
0
an
adreapta n x0 n sens clasic si f+ (x0 ) = D+ f (x0 ) = D+ f (x0 ), un rezultat asem
tor avnd loc si pentru derivata la stnga, iar dac
a toate cele patru derivate Dini au
o valoare comun
a n x0 , atunci f are derivat
a n x0 . Este ns
a de remarcat c
a oric
arei
functii f i se pot asocia cele patru derivate Dini n x0 , spre deosebire de derivatele
laterale clasice.
Indic
am acum extinderile unor rezultate clasice folosind derivate Dini.
Teorema 1. Fie f : (a, b) R continua pe (a, b). Daca Df = 0 pe (a, b)
cu exceptia unei multimi cel mult numarabile, unde D poate fi orice derivata Dini,
atunci f este constanta pe (a, b).
Teorema 2. Fie f : (a, b) R continua pe (a, b). Daca Df 0 pe (a, b)
cu exceptia unei multimi cel mult numarabile, unde D poate fi orice derivata Dini,
atunci f este crescatoare pe (a, b).
Teorema 3. Fie f : (a, b) R continua pe (a, b) si x0 (a, b). Daca una dintre
derivatele Dini este continua n x0 , la fel sunt si celelalte trei. n acest caz, toate
cele patru derivate Dini n x0 sunt egale, iar f este derivabila n x0 .
ncheiem cu o extindere a rezultatului privitor la functiile derivabile cu aceeasi
derivat
a pe un interval .
Teoema 4. Fie f, g : (a, b) R continue pe (a, b). Daca Df si Dg au valori
egale si finite pe (a, b) cu exceptia unei multimi cel mult numarabile, unde D poate
fi orice derivata Dini, atunci f si g difera printr-o constanta pe (a, b).
Bibliografie
1. M. Ganga - Manual de matematica: clasa a XI-a, Mathpress, Ploiesti, 2001.
2. R. Kannan, C. K. Krueger - Advanced Analysis on the Real Line, Springer Verlag,
New York, 1996.
3. S. Ruziewicz - Sur les fonctions qui ont la mme drive et dont la dirence nest
pas constante, Fundamenta Mathematicae 1(1920), 148151 (accesibil si n format
electronic la adresa http://matwbn.icm.edu.pl/wyszukiwarka.php).
33

Probleme selectate de la Olimpiadele de Matematic


a
ale Republicii Moldova
Not
a. Material trimis pentru publicare Redactiei de catre Dr. Valeriu GUTU,

Facultatea de Matematica si Informatica, Universitatea de Stat din Chisinau.

Enunturi

hni
hni hni
+
+ +
, n N , unde [x]
1. Se consider
a sirul (an )nN , an =
1
2
n
este partea ntreag
a a num
arului x. S
a se arate c
a
an = 2 + an1 ,

(1)

dac
a si numai dac
a n este un num
ar prim. (O. R. M. 1997 )
2. S
a se demonstreze
c
a
,
pentru
orice numere
naturale m, n 2, cel mai mic

aseste num
arul 3 3. (O. R. M. 1996 )
dintre numerele n m si m n nu dep
3. Polinomul P (X) de grad n 5, cu coeficienti ntregi, are n r
ad
acini ntregi
a se g
aseasc
a toate r
ad
acinile ntregi ale
distincte 1 , 2 , . . . , n , unde 1 = 0. S
polinomului P (P (X)). (O. R. M. 1997 )
4. Fie triunghiul ABC cu n
altimea CD. Se stie c
a AB = 1999, BC = 1998
si AC = 2000. Cercurile nscrise n triunghiurile ACD si BCD sunt tangente la
segmentul CD n punctele M si N respectiv. S
a se afle M N . (O. R. M. 1999 )
5. Fie ABCD un p
atrat de latur
a 1. Pe laturile AB si CD se iau punctele
interioare X si Y . Fie M punctul de intersectie a dreptelor XD si Y A si N punctul
de intersectie a dreptelor XC si Y B. Pentru ce pozitie a punctelor X si Y aria
patrulaterului XN Y M este maxim
a? (O. R. M. 1997 )
6. Doi frati au vndut n pui cu cte n lei fiecare. Banii i-au mp
artit astfel:
fratele mai mare a luat 10 lei, apoi cel mai mic 10 lei, apoi din nou cel mai mare,
s. a. m. d. Fratelui mai mic i-a revenit la sfrsit o sum
a mai mic
a dect 10 lei. El a
luat acest rest si nc
a briceagul fratelui mai mare, ambii acceptnd c
a au obtinut n
final acelasi cstig. Ct cost
a briceagul? (O. R. M. 1996 )
7. Fie n un num
ar natural astfel nct num
arul 2n2 are 28 de divizori (pozitivi)
distincti, iar num
arul 3n2 are 24 de divizori distincti. Cti divizori distincti are
num
arul 6n2 ? (O. R. M. 1999 )
8. Din cubulete de latur
a 1 se construieste un cub de latur
a 45. n cubul obtinut
1998 de cubulete sunt populate de bacterii. n fiecare secund
a bacteriile se extind
n orice alt cubulet, care are cel putin trei fete comune cu cubuletele deja populate.
Este posibil ca bacteriile s
a ocupe toate cubuletele? (O. R. M. 1998 )
9. ntr-o scoal
a primar
a rural
a nvata 20 de copii. Fiecare doi copii au cel putin
un bunic comun. S
a se demonstreze c
a unul dintre bunici are n aceast
a scoal
a cel
putin 14 nepoti. (O. R. M. 1996 )
10. n timpul unei b
at
alii comune fiecare dintre cei 2001 de cocosi a rupt cte o
pan
a de la un alt cocos si fiecare cocos a r
amas f
ar
a o pan
a. Se stie c
a printre oricare
34

3 cocosi se g
aseste unul care nu a rupt nici o pan
a de la ceilalti doi. S
a se determine
cel mai mic num
ar k cu proprietatea: t
aind cel mult k cocosi putem aseza ceilalti
cocosi n dou
a cotete astfel nct nici un posesor de pan
a str
ain
a s
a nu nimereasc
a
n acelasi cotet cu st
apnul penei. (O. R. M. 2001 )

Solutiile problemelor
1. Scriem egalitatea (1) sub forma

hni
hni hni
n1
n1
n1
+
+ +
=2+
+
+ +
.
1
2
n
1
2
n1
Dup
a reducerea termenilor obtinem

hni hni
n
n1
n1
n1
+
+ +
=
+
+ +
.
2
3
n1
2
3
n1
hni n 1
Pentru orice 2 k n 1 avem

. Deci,
k
k

hni hni
n
n1
n1
n1
+
+ +

+
+ +
.
2
3
n1
2
3
n1

(2)

(3)

Presupunem adev
arat
a egalitatea (1), deci si (2). Dac
a n este un num
ar compus si
hni
n1
=b>
= b 1. Ca urmare, inegalitatea (3)
n = ab, 2 a < n, atunci
a
a
este strict
a, ceea ce contrazice (2).
Reciproc, fie n un num
ar prim. Atunci pentru orice
2 k n1 avem n = kt+r
hni
n1
=t=
. Prin urmare, egalitatea
cu t, r N si 1 r < k. n acest caz
k
k
(2), deci si (1), este adev
arat
a.

n
m <n n < m n. Pentru m > n avem m n < n n <
2. Dac
a m < n, atunci

< n m. Deci, min { m n, n m} n n,oricare


ar fi m, n 2. Vom demonstra prin
inductie c
a pentru orice n 2 avem n n 3 3 sau n3 3n . Pentru n = 2 si n = 3
inegalitatea este adev
arat
a. Presupunem c
a ea este adev
arat
a pentru k 3. Atunci

(k + 1)3 = k3 + 3k2 + (3k + 1) k3 + k3 + k3 = 3k3 3 3k = 3k+1 ,



ceea
ce finalizeaz
a demonstratia. Deci, pentru orice m, n 2 avem min{ m n, n m}

3
3.

3. Num
arul k este o r
ad
acin
a a polinomului P (P (X)), dac
a el este o solutie a
a ar
at
am c
a P (k) 6= i , oricare
uneia din ecuatiile P (x) = i , unde i = 1, 2, . . . , n. S
ar fi k Z si i 2.
Presupunem c
a exist
a k Z astfel nct P (k) = 2 . Atunci k 6= 0. Din enunt
rezult
a c
a polinomul P (X) are forma P (X) = aX(X 2 )(X 3 ) (X n ) cu
a
a Z . Prin urmare, ak(k 2 )(k 3 ) (k n ) = 2 . Relatia k | 2 implic
2 = kt, t Z . Dar ak(1 t)(k 3 ) (k n ) = t implic
a (1 t) | t.
Rezult
a c
a t = 2, pentru care avem ak(k 3 )(k 4 )(k 5 ) (k n ) = 2.
Observ
am c
a numerele k, k 3 , k 4 si k 5 sunt distincte. Dar num
arul 2
nu poate fi scris ca produs de cteva numere ntregi, dintre care cel putin patru s
a
35

fie distincte. Deci, presupunerea f


acut
a este fals
a si P (k) 6= i , oricare ar fi k Z si
i 2.
Deci, r
ad
acinile ntregi ale polinomului P (P (X)) sunt solutiile ntregi ale ecuatiei
P (x) = 0, adic
a numerele 1 , 2 , . . . , n .
4. Fie E si F punctele de tangenta a dreptei AB cu cercurile nscrise respectiv
n triunghiurile ADC si BCD. Aplicnd teorema lui Pitagora, avem
AC 2 BC 2 = (AC 2 CD2 ) (BC 2 CD2 ) = AD2 BD2 =
(AD BD)(AD + BD) = (AD BD) AB.

nlocuind valorile date, obtinem c


a AD BD = 4.
Pe de alt
a parte, avem DE = DM, DF = DN si CD + AD = 2DM + AC,
CD + BD = 2DN + BC. De aici rezult
a c
a
AD BD = 2DM + AC 2DN BC = 2(DM DN ) + 2.
Prin urmare, 2(DM DN ) = AD BD 2 = 2. Rezult
a c
a MN = 1.

5. Observ
am c
a 4BN X 4Y N C. Presupunem c
a BX CY . Atunci
XN N C (mention
am c
a XN = N C, dac
a si numai dac
a BX = CY ). Consider
am pe [XN ] punctul P astfel nct N P = N C. Atunci: SBN P = SBCN ,
SY P N = SY N C , SBP X SY XP (egalitatea are loc doar n cazul cnd BX = CY ).
Din trapezul XBCY avem SXNY = SBCN . Prin urmare,
SBN X + SY NC = SBP X + SBNP + SY N C
SY XP + SBCN + SY P N = SXNY + SBCN = 2 SXNY .
1
1
a c
a SXY M SAXY D . Ca
Rezult
a c
a SXN Y SXBCY . Prin analogie, se arat
4
4
urmare,
1
1
SXNY M = SXNY + SXY M (SXBCY + SAXY D ) = SABCD .
4
4
1
1
Dac
a BX = CY , atunci aria este maxim
a: SXNY M = SABCD = .
4
4
6. Din enunt rezult
a c
a suma ncasat
a este de n2 lei. Fie n = 10a + b, unde
a, b N, 0 b < 10. Atunci n2 = 100a2 + 20ab + b2 = 20a(5a + b) + b2 . Fratii au
luat n total un num
ar impar de cte 10 lei. Rezult
a c
a num
arul zecilor num
arului
n2 este impar, ceea ce implic
a b2 = 16 sau b2 = 36. n ambele cazuri restul este egal
cu 6. Deci, ultima dat
a fratelui mai mic i-a revenit 6 lei, cu 4 lei mai putin dect
fratelui mai mare. Deoarece mp
artirea a fost corect
a, rezult
a c
a pretul briceagului
este de 2 lei.
7. Se stie c
a pentru orice num
ar natural m cu descompunerea canonic
a
k
1

p
,
num
a
rul
divizorilor
(pozitivi)
distinc
t
i
ai
num
a
rului
m
este
egal
m = p
1
k
cu d(m) = (1 + 1) (k + 1) (se demonstreaz
a prin inductie n raport cu num
arul
k de factori primi).

Scriem descompunerea canonic


a a num
arului n = 2 3 p1 1 pk k , unde p1 , . . . , pk
sunt factori primi distincti si diferiti de 2 si 3 cu exponenti ntregi , 0, i 1
(i = 1, . . . , k). Atunci, num
arul divizorilor (pozitivi) distincti ai num
arului
2 k
2
2+1 2 2 1
2n = 2
3 p1 pk este egal cu
d(2n2 ) = (2 + 2)(2 + 1)(2 1 + 1) (2 k + 1).
36

Analog, avem
d(3n2 ) = (2 + 1)(2 + 2)(2 1 + 1) (2 k + 1),

d(6n2 ) = (2 + 2)(2 + 2)(2 1 + 1) (2 k + 1).

Not
am c = (2 1 + 1) (2 k + 1). Din conditiile problemei avem

(2 + 2) (2 + 1) c = 28
.
(2 + 1) (2 + 2) c = 24

(4)

Num
arul c este un divizor comun al numerelor 28 si 30. n plus, c este impar. Deci,
c = 1. Sistemul (4) pentru c = 1 are solutia n numere naturale = 1, = 3.
Rezult
a c
a d(6n2 ) = (2 + 2)(2 + 2)c = 32.
8. Dup
a ocuparea de c
atre bacterii a unui cubulet nou num
arul fetelor cubuletelor
de pe frontier
a ale volumului ocupat de bacterii nu creste. Num
arul maxim al fetelor
de acest fel este egal cu 1998 6 = 11988, iar num
arul fetelor cubuletelor de frontier
a
ale cubului construit este egal cu 45 45 6 = 12150. Deci, bacteriile nu pot ocupa
toate cubuletele.
9. Not
am cu A si B cei doi bunici ai unui elev oarecare. Fie X multimea elevilor
care au A si B ca bunici. n conditiile problemei, restul elevilor vor avea ca unul
dintre bunici sau pe A sau pe B. Fie Y si Z multimile acelora pentru care A si
respectiv B este bunic. Not
am cu C al treilea bunic al unui elev din multimea Y
(primul bunic fiind A). Ca urmare, toti elevii din Z vor avea ca bunici B si C. n
acelasi mod, deducem c
a si toti elevii din Y au pe C ca bunic. Asadar, exist
a doar
trei bunici: A, B, C. Dac
a a, b, c noteaz
a num
arul de nepoti pe care i are n acea
clas
a A, B si respectiv C, rezult
a c
a a + b + c = 40. Observ
am c
a a 13, b 13,
c 13 implic
a a + b + c 39 < 40. Deci, m
acar unul dintre a, b, c este 14, adic
a
m
acar un bunic are 14 nepoti.
10. Vom spune c
a n (n > 1) cocosi formeaz
a un n-ciclu, dac
a ei pot fi aranjati
astfel nct primul cocos a rupt o pan
a de la al doilea, al doilea a rupt o pan
a de la al
treilea etc., iar ultimul cocos a rupt o pan
a de la primul. Astfel, multimea cocosilor
poate fi partitionat
a n cicluri.
Toti cocosii din orice n-ciclu par (n este num
ar par) se asaz
a n cotete astfel:
cocosii cu un num
ar de ordine par ntr-un cotet, iar cei cu num
ar de ordin impar n
cel
alalt.
Pentru orice n-ciclu impar (n este un num
ar impar) este necesar si suficient s
a
t
aiem un cocos, de exemplu, ultimul. n acest caz ceilalti cocosi se asaz
a n dou
a
cotete dup
a metoda precedent
a. Mention
am c
a din ipotez
a rezult
a c
a nu exist
a nici
un 3-ciclu de cocosi.
Deoarece 2001 = 5 400 + 1, rezult
a c
a nu exist
a mai mult de 400 cicluri impare.
Dar deoarece un cocos nu poate forma un ciclu, rezult
a c
a exist
a cel mult 399 de
cicluri impare (de exemplu, 399 de 5-cicluri si un 6-ciclu).
Prin urmare, se pot t
aia oricnd cel mult k = 399 de cocosi (cte unul n ciclurile
impare), pentru a aseza ceilalti cocosi conform cerintelor din enunt.

37

Probleme pentru clasa a VIII-a Holger STEPHAN


Enunturi si solutii
1. Patru numere adunate doua cte doua dau sumele 4, 7, 9, 14, 16, 19. Care
sunt cele patru numere?
Solutie. Fie x1 , x2 , x3 si x4 cele patru numere. Vom presupune, f
ar
a a restrnge
generalitatea, c
a x1 x2 x3 x4 . Observ
am c
a x1 + x2 x1 + x3 x1 + x4 si
x2 + x3 x2 + x4 x3 + x4 . Relativ la sumele x1 + x4 si x2 + x3 putem avea att
x1 +x4 x2 +x3 ct si x2 +x3 x1 +x4 . Ca urmare, suntem condusi la urm
atoarele
dou
a sisteme:
x1 +x2 = 4, x1 +x3 = 7, x1 +x4 = 9, x2 +x3 = 14, x2 +x4 = 16, x3 +x4 = 19; (1)
x1 +x2 = 4, x1 +x3 = 7, x1 +x4 = 14, x2 +x3 = 9, x2 +x4 = 16, x3 +x4 = 19. (2)
Rezolv
am mai nti sistemul (1). Adunnd primele dou
a ecuatii si din rezultat
asim
sc
aznd a patra, obtinem 2x1 = 3. Prin nlocuire n primele trei ecuatii, g
3
11
17
21
a si ultimele dou
a ecuatii.
x1 = , x2 = , x3 = , x4 = , care verific
2
2
2
2
n mod similar, sistemul (2) conduce la solutia x1 = 1, x2 = 3, x3 = 6, x4 = 13.
2. Demonstrati ca prin "rotirea catre dreapta" a unui numar de 8 cifre, divizibil
cu 73, se obtine tot un numar divizibil cu 73. (Se spune ca un numar natural este
"rotit catre dreapta", daca ultima cifra este mutata n fata primei cifre; exemplu:
1234 4123.)
Solutie. Fie x = x7 x6 x5 x4 x3 x2 x1 x0 un astfel de num
ar si y = x0 x7 x6 x5 x4 x3 x2 x1
rotitul s
au c
atre dreapta. Avem

7x + 3y = 7 107 x7 + 106 x6 + + 10x1 + x0 + 3 107 x0 + 106 x7 + + 10x2 + x1 =

= 3 107 + 7 x0 + 7 107 + 3 106 x7 + 7 106 + 3 105 x6 + +

+ 7 102 + 3 10 x2 + (7 10 + 3) x1 =

= 3 107 + 7 x0 + 73 106 x7 + 73 105 x6 + + 73 10x2 + 73x1 .

Dar 3107 + 7 = 73 410956 si, deci, 7x + 3y este divizibil cu 73. Deoarece coeficientii
lui x si y nu se divid cu 73, rezult
a c
a x si y pot fi doar simultan divizibile cu 73.

3. Aflati cifrele necunoscute x, y, z din egalitatea 20 058 47311! = x00yz0055046400.


Solutie. Egalitatea din enunt se scrie 28 34 52 71120 058 473 = x00yz0055046400.
Observ
am c
a puterile lui 5 sau ale lui 2, cu care este divizibil num
arul, dau informatii
doar despre ultimele lui 8 cifre (care, ns
a, sunt cunoscute). Apel
am la criteriile de
divizibilitate cu 7, 9 si 11, aplicate num
arului x00yz00550464, care vor antrena cifrele
x, y, z; obtinem
x + y + z + 24 = 9i,

x y + z + 2 = 11j,

x y + 2z + 5 = 7k

(1)

(pentru ultima egalitate s-a folosit faptul c


a un num
ar este divizibil cu 7 dac
a suma
ponderat
a a cifrelor sale cu ponderile 1, 3, 2, 1, 3, 2, utilizate periodic ncepnd
de la unit
ati spre puterile mai mari ale lui 10, este divizibil
a cu 7).
a c
a avem 25 9i 51, 7 11j 20,
Cum 1 x 9 si 0 y, z 9, din (1) rezult
4 7k 32, adic
a
i {3, 4, 5} , j {0, 1} , k {0, 1, 2, 3, 4} .
(2)
38

Solutia sistemului (1) n x, y, z este


1
1
(3)
x = (9i + 33j 14k 20) , y = (9i 11j 22) , z = 7k 11j 3.
2
2
Expresia lui y din (3) arat
a c
a i si j trebuie s
a aib
a aceeasi paritate. Ca urmare,
exist
a trei cazuri:
1 i = 3 si j = 1, caz ce conduce la y = 3, ceea ce este imposibil;
a x = 29 7k, y = 6 si z = 7k 14; pentru k = 3 se
2 i = 5 si j = 1, care d
obtine solutia x = 8, y = 6, z = 7;
a x = 8 7k, y = 7, z = 7k 3; pentru k = 1 obtinem
3 i = 4 si j = 0, care d
solutia x = 1, y = 7, z = 4.
Asadar, sunt posibile dou
a numere divizibile cu 9, 7 si 11: 800 670 055 046 400 si
100 740 055 046 400. Pentru a decide care este num
arul corect este necesar nc
a un
test de divizibilitate, si anume, cu 27. Puterile lui 10 mp
artite la 27 pot da resturile
1, 10 sau 8, n mod periodic. Aplicnd aceast
a observatie la primul num
ar, avem
1 (0 + 6 + 5 + 0 + 0) + 10 (0 + 4 + 5 + 7 + 0) 8 (4 + 0 + 0 + 6 + 8) = 27,

deci acest num


ar este divizibil cu 27. Relativ la al doilea num
ar avem

1 (0 + 6 + 5 + 0 + 0) + 10 (0 + 4 + 5 + 4 + 0) 8 (4 + 0 + 0 + 7 + 1) = 45,

adic
a acest num
ar nu se divide cu 27 si trebuie exclus.
n final, singura solutie valabil
a este num
arul 800 670 055 046 400.
4. Un numar x format din cinci cifre diferite si nenule este divizibil cu 9. Aratati
ca suma tuturor numerelor de cinci cifre distincte ce se pot forma cu aceste cinci
cifre (inclusiv x) este divizibila cu 2399976.
Solutie. Fie x = x1 x2 x3 x4 x5 . Cifra x1 este exact de 4! = 24 ori pe prima pozitie.
Adunarea tuturor acestor prime pozitii egale cu x1 d
a num
arul 24 10000x1 . S
i pe
pozitia a doua, a treia etc. apare cifra x1 tot de 24 ori. Cifra x1 va contribui la suma
total
a S cu
24 10000 + 24 1000 + 24 100 + 24 10 + 24 1 = 24 11111 = 266 664.

Aceasta e valabil pentru orice cifr


a. Deci pentru suma S avem
S = 266664 (x1 + x2 + x3 + x4 + x5 ) .

Deoarece x e divizibil cu 9, atunci si x1 + x2 + x3 + x4 + x5 este divizibil cu 9. n


consecinta, S este divizibil
a cu 266 664 9 = 2 399 976.
5. Gasiti toate perechile de numere ntregi x si y care sunt solutii ale ecuatiei
diofantice 2x2 + 7xy + 3y 2 = 228.
Solutie. Scriem ecuatia n forma (x + 3y) (2x + y) = 228. Dac
a num
arul 228
este descompus ntr-un produs de doi factori ntregi p q = 228, atunci putem scrie
sistemul
2x + y = p, x + 3y = q
cu solutia
3p q
2q p
x=
, y=
.
5
5
228 are divizorii 1, 2, 3, 4, 6, 12, 19, 38, 57, 76, 114, 228 si cei corespunz
atori cu
semnul minus. Dac
a nlocuim p si q cu acesti divizori si calcul
am, obtinem
39

p
q

1
228

45

91

2
114
108

3
76
67

4
57

6
38

22

14

12
19
17
5

19
12
9
1

38
6
108
5

57
4
167
5

76
3

114
2

45

68

14

22

228
1
683
5

n concluzie, avem perechile de solutii (45, 91), (9, 22), (4, 14), (9, 1), (45, 14)
si (68, 22), mpreun
a cu variantele negative (45, 91), (9, 22), (4, 14), (9, 1),
(45, 14) si (68, 22).
6. Gasiti toate perechile de numere ntregi x si y care sunt solutii ale ecuatiei
diofantice 2x2 + 3y 2 = 77.
Solutie. Dac
a perechea (x, y) este solutie, atunci si perechile (x, y), (x, y)
si (x, y) sunt solutii. E suficient s
a consider
am solutiile nenegative. Deoarece
x2 0 avem 0 3y 2 77 sau 0 y 5. y nu poate fi par, deoarece 2x2 este par
si 77 este impar. E destul s
a d
am lui y valorile 1, 3 si 5. Ultimele dou
a furnizeaz
a
valori ntregi pentru x. Obtinem (x, y) = (1, 5), (5, 3) si (x, y) = (1, 5), (1, 5),
(1, 5), (5, 3), (5, 3), (5, 3).
7. Consideram numarul natural n, 1000 n < 5000. Formam numarul (de 12
sau 13 cifre) obtinut scriind n ordine cifrele lui 3n, 2n si respectiv n. Aratati ca
acest numar este divizibil cu 28 + 1.
Solutie. Num
arul 2n are patru cifre deoarece n < 5000. Deci num
arul m, obtinut
ca n enunt, are 12 sau 13 cifre. Atunci avem
m = 300 000 000n + 20 000n + n = 300 020 001n = 3 41 257 9491 n.

Deci m este divizibil cu 28 + 1 = 257.


8. Sase

numere prime 7 < p1 < p2 < p3 < p4 < p5 < p6 formeaza un "sextet de
numere prime", daca p2 , p3 si p4 , p5 sunt numere prime gemene (adica p3 p2 =
= p5 p4 = 2), iar p2 p1 = p4 p3 = p6 p5 = 4. Demonstrati ca suma lor este
divizibila cu 630.
Solutie. Fie M num
arul care satisface
p1 = M 8, p2 = M 4, p3 = M 2, p4 = M + 2, p5 = M + 4, p6 = M + 8.

Deoarece pi sunt numere prime mai mari ca 7, nu pot fi divizibile prin 2, 3, 5 sau 7.
Ca urmare, M este impar si divizibil cu 3, 5 si 7. M se poate reprezenta, deci, astfel:
M = 210k + 105. Atunci, avem
p1 = 210k + 97, p2 = 210k + 101, p3 = 210k + 103,
p4 = 210k + 107, p5 = 210k + 109, p6 = 210k + 113
si, n concluzie, p1 + p2 + p3 + p4 + p5 + p6 = 1260k + 630 = 630 (2k + 1).
9. Este posibil ca suma a sapte patrate perfecte succesive sa fie un patrat perfect?
Solutie. Suma a sapte p
atrate perfecte consecutive se poate scrie astfel:

2
2
(n 3) + (n 2) + (n 1)2 + n2 + (n + 1)2 + (n + 2)2 + (n + 3)2 = 7 n2 + 4 .

a fie divizibil
Pentru ca rezultatul s
a fie p
atrat perfect este necesar ca n2 + 4 s

cu 7.
2
Dar n d
a doar resturile 0, 1 sau 3 la mp
artirea cu 7. Ca urmare, 7 n2 + 4 este
divizibil cu 7, dar nu cu 49 si nu poate s
a fie p
atrat perfect.
40

Concursul "Recreatii Matematice"


Editia a IV-a, Muncel - Iasi, 28 August 2004
Clasa a VII-a
1. S
a se rezolve n Z Z ecuatia x2 + y 2 xy + 2x 2y + 1 = x2 y 2 .
C
at
alin Budeanu, Iasi
999
2. S
a se arate c
a num
arul 4
se scrie n baza 10 cu cel putin 601 cifre.
Gabriel Popa, Iasi

b
b
3. Fie 4ABC cu m(B) = 60 , m(C) = 50 si punctele M (BC), B (AN )
\ = m(BCN
\ ) = 5 . Dac
a I este centrul cercului nscris n
astfel nct m(BAM)
4ABC, s
a se arate c
a punctele M, N , I sunt coliniare.
Gheorghe Iurea, Iasi
4. Fie ABCD un dreptunghi de centru O. Consider
am N (AO), M mijlocul
lui [AD], {P } = MN CD, {E} = OP BC. S
a se arate c
a N E BC.
Andrei Nedelcu, Iasi (RecMat - 2/2004)

Clasa a VIII-a
1. Consider
am multimile A =
se demonstreze c
a A = B.

1
x [x]
, x 1 , B = 0,
. S
a
y | y R, y =
x
2

Vasile Nechita, Iasi


2. S
a se arate c
a ecuatia x + y = t are o infinitate de solutii n (N )3 .
Artur B
al
auc
a, Botosani
3. Fie ABCDA0 B 0 C 0 D0 un cub de muchie a, iar P un punct pe segmentul [AA0 ].
Determinati pozitiile lui P pentru care cu distantele de la P la planele (A0 BD),
(B 0 BD), respectiv (C 0 BD) se poate construi un triunghi.
Gabriel Popa, Paul Georgescu, Iasi
4. Fie b N, b 2. Spunem c
a un num
ar natural este decompozabil dac
a se
poate scrie ca suma a dou
a numere cu aceeasi sum
a a cifrelor n baza b. S
a se arate
c
a exist
a o infinitate de numere care nu sunt decompozabile.
Adrian Zahariuc, Bac
au (RecMat - 2/2004)
3

Clasa a IX-a
1. Fie n N fixat. Determinati x, y R pentru care xn +yn = xn+1 +y n+1 = 1.
Gheorghe Iurea, Iasi
2. Fie 0 < ni < 144, i = 1, 2, . . . , 12, dou
asprezece numere naturale distincte. S
a
se demonstreze c
a multimea S = {n1 , n2 , . . . , n12 } contine dou
a submultimi disjuncte
ale c
aror elemente au aceeasi sum
a.
C
at
alin Budeanu, Iasi
3. Fie 4ABC de laturi a, b, c; not
am cu la , lb lungimile bisectoarelor interioare
b respectiv B.
b Dac
ale unghiurilor A,
a la = a si lb = b, s
a se calculeze m
asurile
unghiurilor 4ABC.
Vasile Nechita, Iasi
4. Exist
a functii f : R R pentru care
41

|f (x + y + z + t) + cos x + cos y + cos z + cos t| < 4,

x, y, z, t R ?

Lucian Tutescu, Craiova (RecMat - 2/2004)

Clasa a X-a
2

1. S
a se determine a R astfel nct ecuatia 2x1 + 2x

y 2 + ay + a2
s
a
y 2 + a2

aib
a solutii n Z Z.

Petru R
aducanu, Iasi (RecMat-1/2004)
2. Ar
atati c
a exist
a o infinitate de numere naturale n pentru care n2 + n + 1
divide n!.
Lucian Tutescu, Craiova
3. Fie S1 (O1 , r1 ), S2 (O2 , r2 ) dou
a sfere, iar Ai S1 , Bi S2 , i = 1, 4 puncte
astfel nct Ai Bi , i = 1, 4 s
a fie tangente comune la cele dou
a sfere. Fie Mi mijlocul
a se determine un punct P O1 O2 pentru care suma
segmentului [Ai Bi ], i = 1, 4. S
P M1 + P M2 + P M3 + P M4 s
a fie minim
a.
Gabriel Popa, Paul Georgescu, Iasi
4. Exist
a triunghiuri pentru care IH = 2004 si GH = 2003? (notatii clasice)
Lucian L
adunc
a, Andrei Nedelcu, Iasi

Clasa a XI-a
1. a) Fie f : R R functie derivabil
a pe R si a R \ Im f 0 . S
a se arate c
a
pentru orice functii g, h : R R, ecuatiile
g (x) = h (x) ,

(f g) (x) ag (x) = (f h) (x) ah (x)

au aceleasi multimi de solutii.


b) S
a se rezolve ecuatia 2x + sin (sin x) = 3 sin x.

Silviu Boga, Suceava


2. Se d
a cercul (C) si dreapta (d) tangent
a lui. Din punctul M , mobil pe dreapta
(d), se duce tangenta MT la cercul (C). Se cere:
a) locul geometric al punctului care mparte segmentul [M T ] n raportul k.
b) s
a se reprezinte grafic acest loc pentru k = 1.
Gabriel Mrsanu, Iasi
3. Fie k, n, p, q N , n, p, q impare, iar A Mn (Z). S
a se demonstreze c
a
2k
A + pA + qIn 6= On .
Romeo Ilie, Brasov

n+k
n
n1
a se arate c
a ecuatia x
x x
x 1 = 0 are o
4. Fie k N ; s
singur
a solutie pozitiv
a (pe care o not
am cu xn ). S
a se arate c
a sirul (xn )n1 este
convergent si s
a se afle limita sa.
Dumitru Mihalache, Marian Tetiva, Brlad (RecMat - 2/2004)

"POIANA" (dir.
FUNDA
TIA CULTURALA

Dan Tiba)

a oferit dou
a premii n valoare de cte 1 000 000 lei, pentru cele mai bune teze de
la gimnaziu si liceu, urm
atorilor elevi:
1. Hurmuz Daniel, cl. a VIII-a, Scoala

nr. 7, Botosani,
2. S
avescu Cristian, cl. a IX-a, Colegiul National "Unirea", Focsani.
42

Concursul interjudetean "Octav Onicescu"


Editia a VIII-a, Botosani, 30 octombrie 2004
1. Pe data de 30 octombrie o veverita lacom
a are 20042004 alune. n zilele cnd
are un num
ar par de alune ea m
annc
a jum
atate din ele, iar n celelalte zile nu
m
annc
a nimic si mai culege 3 alune. Ar
atati c
a ncepnd cu o anumit
a zi veverita
va mnca doar cte 3 alune odat
a la 2 zile.
2. Broscutele stau n cerc n jurul lacului de la Ipotesti. n prima secund
a or
ac
aie
una din ele. n fiecare din secundele urm
atoare or
ac
aie doar acelea ce au m
acar o
vecin
a care a or
ac
ait n secunda anterioar
a. Dovediti c
a 2005 broscute vor or
ac
ai
toate deodat
a de la un moment dat ncolo, dar 2004 broscute nu vor or
ac
ai niciodat
a
toate deodat
a.
3. Pe o tabl
a de sah n n sunt asezate n2 numere egale fiecare cu 1 sau 1
(nu toate egale). Prin mutare se ntelege c
a alegem o linie si o coloan
a oarecare si
schimb
am semnele tuturor numerelor de pe linie si apoi tuturor celor de pe coloan
a.
Se stie c
a dup
a cteva astfel de mut
ari toate numerele pot fi f
acute 1. Ar
atati c
a
dac
a n este par, plecnd de la configuratia initial
a putem face toate numerele 1 iar
dac
a n este impar nu putem face niciodat
a toate numerele 1.
4. De aceeasi parte a unei drepte se consider
a 2003 puncte distincte A1 , A2 , . . . ,
a s
a contin
a cel mult dou
a dintre
A2003 astfel nct orice cerc cu centrul pe dreapt
puncte. Se numeste "cerc bun" un cerc cu centrul pe dreapt
a, ce trece printr-unul
din puncte si las
a 1001 puncte n interiorul s
au si 1001 n exterior. Ar
atati c
a:
a) orice punct de pe dreapt
a (cu exceptia eventual a unui num
ar finit de puncte)
este centrul unui "cerc bun";
b) dac
a absolut toate "cercurile bune" trec prin A1002 (deci prin acelasi punct)
atunci cele 2003 puncte sunt situate pe o perpendicular
a pe dreapta dat
a.

5. a) Se dau n 2004 sertare asezate n linie. n primele 2003 din stnga avem
cte o bil
a. La un pas se alege o bil
a oarecare ce are sertar gol n dreapta si se mut
a
n acesta, astfel c
a bilele "migreaz
a" una cte una spre dreapta. Ar
atati c
a dup
a fix
2003 (n 2003) pasi nici o bil
a nu mai poate fi mutat
a.
b) Se dau n 2004 numere reale distincte astfel nct suma oric
aror 2003 din ele
s
a fie tot unul din cele n numere. Ar
atati c
a n = 2004, c
a jum
atate din numere sunt
pozitive si celelalte sunt opusele lor.
Rezultatele obtinute au fost urm
atoarele:
Premiul I Pachitariu Marius (Colegiul National Iasi).
Premiul II Chiril
a Cezar (Colegiul National Tudor Vianu, Bucuresti).
Premiul III Turcanu

Alexandru, Vatavu S
erban (Colegiul National Mihai
Eminescu, Botosani).
Mentiuni Rosu Eugenia (Iasi), Istrate Carmen (Focsani), Berea Bogdan
(P. Neamt), Milatinovici Bianca (Iasi), Hurmuz Daniel (Botosani), Mandici
Ciprian (Suceava), Galea Lucian (Botosani), Cepoi Alexandru (Suceava), Dasc
alu Sorin (P. Neamt), Georgescu Flavian (P. Neamt), Cosbuc Mircea (Iasi),
Mihalcea Marcel (Vaslui), Pl
amad
a Andrei (Botosani).
43

Solutiile problemelor propuse n nr. 1 / 2004


Clasele primare
P.64. ntr-o piesa de teatru sunt 12 personaje, copii si adulti. Cti copii joaca
n piesa, daca la fiecare doi adulti corespunde un copil?
( Clasa I )
Alexandra Radu, elev
a, Iasi
Solutie. Form
am grupe de forma (copil, adult, adult) pn
a epuiz
am personajele.
Putem forma patru grupe de acest fel. Rezult
a c
a n pies
a joac
a patru copii.
P.65. Se dau jetoanele AT
II CRE TII

ATII

RECR EA RE REC .
Care este numarul cel mai mare de jetoane cu care se poate forma cuvntul "RECREATII"?

( Clasa I )
Oxana Pascal, elev
a, Rep. Moldova
Solutie. Cuvntul "RECREATII"

poate fi format astfel: RE


REC RE AT
II si RECR
utilizate este patru.

EA

CRE

ATII
,

TII
. Num
arul cel mai mare de jetoane

P.66. ntr-o livada sunt tot attia peri ct si meri. Sunt 6 rnduri cu peri si 4
rnduri cu meri. Numarul merilor de pe un rnd ntrece cu 5 numarul perilor de pe
un rnd. Cti pomi sunt n acea livada?
( Clasa a II-a)
nv. Maria Racu, Iasi
Solutie. Utiliz
am metoda figurativ
a.
Num
arul perilor din cele 6 rnduri:
5 5
5 5
Num
arul merilor din cele 4 rnduri:
Din figurarea m
arimilor se deduce c
a un rnd de peri are 5+5 = 10 pomi. Num
arul
perilor este 6 10 = 60. Num
arul tuturor pomilor din livad
a este 60 + 60 = 120.
P.67. Dintr-o multime de 5 copii, orice grupare de trei contine cel putin o fata.
Cti baieti pot fi n multime?
( Clasa a II-a)
Andreea Surugiu, elev
a, Iasi
Solutie. n multime nu putem avea mai mult de 2 b
aieti, altfel am g
asi o grupare
de trei n care nu avem cel putin o fat
a. n concluzie, putem avea 2 b
aieti, 1 b
aiat
sau nici unul.

P.68. Daca Ina ar mparti numarul nucilor culese de ea la numarul nucilor culese
de sora sa, ar obtine 7 rest 6. Stiind

ca Ina a cules cu 78 nuci mai mult dect sora


sa, aflati cte nuci a cules fiecare.
( Clasa a III-a)
nv. Doinita Spnu, Iasi
78
Solutie. Utiliz
am metoda figurativ
a.
Num
arul nucilor culese de Ina:
6
Num
arul nucilor culese de sora Inei:
Num
arul nucilor culese de sora Inei este (78 6) : 6 = 72 : 6 = 12. Num
arul
nucilor culese de Ina este 7 12 + 6 = 84 + 6 = 90.

P.69. ntr-o mpartire cu rest, n care mpartitorul este mai mare ca noua,
marind mpartitorul cu o unitate si efectund din nou mpartirea obtinem ctul 9
44

si restul 0. Aflati ctul si restul mpartirii initiale.


( Clasa a III-a)
nv. Mariana Toma, Muncelu de Sus (Iasi)
Solutie. mp
artirea initial
a este D = I C + R, R < I. A doua mp
artire este
exact
a si avem D = (I + 1) 9. Rezult
a D = I 9 + 9. Cum I > 9 obtinem C = 9
si R = 9.
P.70. ntr-o tabara internationala de matematica sunt elevi din patru tari: Bulgaria, Grecia, Republica Moldova si Romnia. Daca 21 elevi nu sunt din Bulgaria,
23 nu sunt din Grecia, 22 elevi nu sunt din Republica Moldova si 21 elevi nu sunt
din Romnia, cti elevi sunt din fiecare tara?
( Clasa a III-a)
Georgiana Ciobanu, elev
a, Iasi
Solutie. Dac
a 21 elevi nu sunt din Bulgaria, nseamn
a c
a sunt din Grecia, Republica Moldova si Romnia.
Analog, 23 elevi sunt din Bulgaria, Republica Moldova si Romnia;
22 elevi sunt din Bulgaria, Grecia si Romnia;
21 elevi sunt din Bulgaria, Grecia si Republica Moldova.
Triplul elevilor din cele patru tari este 21 + 23 + 22 + 21 = 87. Num
arul elevilor
din cele patru tari este 87 : 3 = 29. Rezult
a 29 21 = 8 elevi din Bulgaria, 29 23
elevi din Grecia, 29 22 = 7 elevi din Republica Moldova si 29 21 = 8 elevi din
Romnia.
P.71. Fiecare patrat din figura alaturata se coloreaza cu o alta culoare. n
cte moduri putem face acest lucru avnd la dispozitie patru culori?
( Clasa a IV-a)
nv. C
at
alina Rata
, Coarnele Caprei (Iasi)
Solutie. Dac
a alegem culorile C1 , C2 , C3 din cele patru, putem s
a color
am p
atratele n sase moduri diferite: (C1 , C2 , C3 ), (C1 , C3 , C2 ), (C2 , C1 , C3 ), (C2 , C3 , C1 ),
(C3 , C1 , C2 ), (C3 , C2 , C1 ). Cele trei culori pot fi alese n patru moduri diferite:
(C1 , C2 , C3 ), (C1 , C2 , C4 ), (C1 , C3 , C4 ) si (C2 , C3 , C4 ). Pentru fiecare alegere avem
sase moduri diferite de colorare. n total avem 6 4 = 24 moduri diferite de colorare.

P.72. Aruncam doua zaruri si adunam punctele de pe cele doua fete de deasupra.
a) Cte sume diferite putem obtine? b) Cte sume se pot forma n trei moduri
diferite?
( Clasa a IV-a)
nv. Gheorghe Toma, Muncelu de Sus (Iasi)
Solutie. a) Suma minim
a care se poate forma este 1 + 1 = 2, iar cea maxim
a
este 6 + 6 = 12. Toate numerele de la 2 la 12 sunt sume posibile. b) Singurele
sume care se pot forma n trei moduri diferite sunt: 6 = 1 + 5 = 2 + 4 = 3 + 3,
7 = 1 + 6 = 2 + 5 = 3 + 4 si 8 = 2 + 6 = 3 + 5 = 4 + 4. n trei moduri diferite se pot
forma trei sume.

P.73. n figura alaturata este pus n evidenta un


B
drum format din sase segmente care pleaca din A si
ajunge n B. Cte drumuri de felul acesta se pot con- A
strui?
( Clasa a IV-a)
nv. Constantin Rata
, Coarnele Caprei (Iasi)
Solutie. Orice drum de acest fel contine numai
B
9 10
6
7
8
dou
a segmente verticale. Num
arul drumurilor coin5
2
3
4
cide cu num
arul perechilor distincte de segmente ver- A 1
45

ticale prin care trec drumurile cu sase segmente. Aceste perechi sunt: (1, 10), (1, 9),
(1, 8), (1, 7), (1, 6), (2, 9), (2, 8), (2, 7), (2, 6), (3, 8), (3, 7), (3, 6), (4, 7), (4, 6), (5, 6).
n total se pot construi 15 drumuri formate din sase segmente.

Clasa a V-a
V.46. Aflati n N pentru care 11n + 9n si 11n 9n sunt simultan patrate
perfecte.
Andrei - Sorin Cozma, elev, Iasi
Solutie. Dac
a n este par, U (11n + 9n ) = 1 + 1 = 2, deci 11n + 9n nu poate fi
p
atrat perfect. Dac
a n este impar, U (11n 9n ) = U . . . 1 . . . 9 = 2, deci 11n 9n
nu poate fi p
atrat perfect. Rezult
a c
a nu exist
a n N cu propriet
atile dorite.

V.47. Sa se arate ca numarul 51a51a nu poate fi scris ca produsul a patru numere


prime.
C
at
alin Budeanu, Iasi
Solutie. Avem c
a 51a51a = 51a 1001 = 7 11 13 51a. ns
a 51a este num
ar
compus, oricare ar fi cifra a n baza 10, de unde concluzia.
9
10
11
, x2 =
, x3 =
, . . . . Scrieti fractia
V.48. Se considera fractiile x1 =
14
21
28
x1000 si apoi ordonati crescator primele 1000 de fractii.
Dumitru Gherman, Pascani
Solutie. Num
ar
atorii fractiilor sunt 9, 9 + 1, 9 + 2, . . . Num
ar
atorul lui x1000 va
. . . Numitorul lui x1000
fi 9+999 = 1008. Numitorii fractiilor sunt 14, 14+7, 14+ 27,
1008
1 1
1 1
. Observ
am c
a x1 = + ; x2 = + ;
va fi 14 + 999 7 = 7007; deci x1000 =
7007
7 2
7 3
1 1
1
1
. Atunci x1000 < x999 < < x3 < x2 < x1 .
x3 = + ; . . . ; x1000 = +
7 4
7 1001
Observ
am c
a dac
a indicelui i adun
am 8 obtinem num
ar
atorul. Dac
a indicelui i
1000 + 8
1008
=
.
adun
am 1 si l nmultim cu 7 obtinem numitorul, deci x1000 =
(1000 + 1) 7
7007

V.49. Determinati numarul tripletelor (a, b, c) N3 daca 3a + 2b + c = 598 si


a + 2b + 3c = 602. Daca n plus a < b < c, determinati a, b si c.
Gheorghe Iurea, Iasi
Solutie. Avem c
a (a + 2b + 3c) (3a + 2b + c) = 602 598, deci c a = 2. De
aici, c = a + 2 si apoi b = 298 2a, unde a {0, 1, 2, . . . , 149}. Prin urmare, exist
a
150 de triplete, avnd forma (a, 298 2a, a + 2). Dac
a n plus a < b < c, atunci
a < 298 2a < a + 2, de unde 296 < 3a < 298, deci a = 99 si apoi b = 100, c = 101.

V.50. Cte numere de 7 cifre se pot scrie folosind cifrele 1, 2 si 3, astfel nct
1 sa apara de 2 ori, 2 sa apara de 3 ori si 3 sa apara de 2 ori? Dar daca n locul
cifrelor 1, 2 si 3 consideram cifrele 0, 1 si respectiv 2?
Petru Asaftei, Iasi
Solutie. Dac
a prima cifr
a 1 se g
aseste pe primul loc, a doua cifr
a 1 poate ocupa
locurile 2, 3, . . . , 7, deci 6 pozitii. Dac
a prima cifr
a 1 se g
aseste pe locul al doilea, a
doua cifr
a 1 poate ocupa locurile 3, 4, . . . , 7, deci 5 pozitii etc. n total, cele 2 cifre
1 pot fi asezate n 6 + 5 + + 1 = 21 moduri. Pentru fiecare pozitionare a cifrelor
1, r
amn 5 locuri libere care pot fi ocupate n 4 + 3 + 2 + 1 = 10 moduri de cifra 3,
46

r
amnnd astfel exact trei locuri libere pentru cifrele 2. Folosind regula produsului,
obtinem 21 10 1 = 210 numere de 7 cifre.
n al doilea caz, cifrele 0 pot fi asezate n 5 + 4 + 3 + 2 + 1 = 15 moduri, cifrele 2
n 4 + 3 + 2 + 1 = 10 moduri, r
amnnd libere exact trei pozitii pentru 1; obtinem
15 10 1 = 150 numere.

Clasa a VI-a
VI.46. Suma dintre opusul unui numar natural si inversul altui numar natural
este 119, 992. Sa se determine numerele.
Ciprian Baghiu, Iasi
1

a numere, adic
a n + = 119, 992.
Solutie. Fie n N si k N cele dou
k

1
1
1
N. ns
a
Atunci k 6= 1 si 0, 008 = n 120, de unde
k
k 125

1 1 + 1 1 + 1 < 1,
k 125 k 125
2 125

1
1
=
, adic
a n = 120, k = 125.
k
125
VI.47. Aflati restul mpartirii numarului N = 28442844 + 41074107 + 63986398
prin 79.
Tamara Culac, Iasi
Solutie. Se stie c
a (a + b)n = Ma + bn ; atunci 28442844 = M79, 41074107 =
= (4108 1)4107 = M79 1, iar 63986398 = (6399 1)6398 = M79 + 1. Prin
urmare, N = M79, deci restul cerut este 0.
deci

VI.48. a) ntr-o proportie cu termeni nenuli, un extrem este suma celorlalti


trei termeni daca si numai daca celalalt extrem are inversul egal cu suma inversilor
celorlalti trei termeni.
b) Daca din patru numere rationale nenule distincte unul este suma celorlalti trei,
iar altul are inversul egal cu suma inverselor celorlaltor trei, atunci numerele sunt
termeni ai unei proportii.
Claudiu - S
tefan Popa, Iasi
Solutie. a) n conditiile a, b, c, d Q , ad = bc, avem c
a a=b+c+d
b+c
1 1
1 1
1
1 1 1
ad
=
= + = + + .
ad=b+c
ad
bc
d a
c
b
d
a b
c
1 1 1
1
b) Dac
a a, b, c, d Q si a = b + c + d, = + + , atunci a d = b + c 6= 0 si
d
a b c
1 1
1 1
ad
b+c
a
c
= + , adic
a a d = b + c 6= 0 si
=
, deci ad = bc, i.e. = .
d a
b
c
ad
bc
b
d
VI.49. Sa se arate ca orice numar natural relativ prim cu 10 admite un multiplu
care se scrie folosind numai cifra 3.
Lucian - Georges L
adunc
a, Iasi
Solutie. Fie n N cu (n, 10) = 1. Consider
am numerele 3, 33, 333, . . . , 33
. . . 3};
| {z
n+1

exist
a printre acestea m
acar dou
a care dau acelasi rest la mp
artirea prin n, conform
principiului cutiei. Evident c
a n dividediferen
t
a
acestor
numere,
iar aceast
a diferenta

este de forma 33 . . . 3 10k . Deoarece n, 10k = 1, urmeaz


a c
a n | 33 . . . 3.
47

VI.50. Fie 4ABC cu [AC] [BC], D mijlocul lui [AB], P un punct pe dreapta
AB, iar M si L picioarele perpendicularelor din P pe AC, respectiv BC. Sa se
arate ca [DM] [DL].
Neculai Roman, Mircesti (Iasi)
Solutie. Deosebim trei cazuri, dup
a cum P [AB],
P [BA \ [AB] sau P [AB \ [AB]. Vom trata
A
numai prima situatie, celelalte rezolvndu-se asem
an
aD
tor.
Ne situ
am cu P [DA], ca n figur
a si
M
P
fie D0 , D00 mijloacele segmentelor [P A], respectiv
D
[P B]; demonstr
am c
a 4DD0 M 4LD00 D. Avem
1
1
D
c
a DD0 =
(AB P A) =
P B = D00 L, iar
2
2
1
1
(AB P B) = P A = D0 M . n plus,
DD00 =
2
2
0 M) = 2 m(BAC)
00 D)
\
\ = 2 m(ABC)
\ = m(LD
\
B
m(DD
C
L
si atunci congruenta de triunghiuri anuntat
a urmeaz
a conform LUL. Rezult
a c
a
[DM] [DL].

Clasa a VII-a

VII.46. Sa se rezolve n R inecuatiile:


a) x100 + x77 + x50 + x21 + x10 + x5 + 1 > 0;
b) x100 x77 + x50 x21 + x10 x5 + 2 < 0.
Vasile Solcanu, Bogd
anesti (Suceava)
Solutie. a) Fie E (x) = x100 + x77 + x50 + x21 + x10 + x5 + 1. Dac
a x 0,
2k
2k+1
evident c
a E (x) > 0. Pentru
x

(1,
0),
avem
x
>
0

s
i
x
+
1
>
0, deci

a x = 1, atunci
E (x) = x100 + x50 x27 + 1 + x10 x11 + 1 + x5 + 1 > 0. Dac
E (1) = 1 >
n sfr
a x (, 1),
x2k+1 + 1 < 0, x2k+1 < 0, deci
sit,21dac

0.
29
5 atunci
77
23
5
E (x) = x x + 1 + x x + 1 + x x + 1 + 1 > 0. n concluzie, E (x) > 0,
x R.
b) Dac
a F (x) = x100 x77 +x50 x21 +x10 x5 +2, atunci F (x) = E (x)+1 > 0,
x R, deci inecuatia dat
a nu are solutie n R.
VII.47. Sa se rezolve n Z2 ecuatia u2 v + uv 2 = 2u2 + 2v 2 40.
Mihai Cr
aciun, Pascani
2
Solutie. Ecuahtia se scrie echivalent
uv
(u
+
v)
=
2
(u
+
v)
4uv 40
i
uv (u + v + 4) = 2 (u + v)2 16 8 (u + v + 4) (uv 2u 2v + 8) = 8, iar
uv 2u 2v + 8 = (u 2) (v 2) + 4. Considernd toate cazurile posibile, g
asim n
final solutiile (u, v) {(2, 8) , (8, 2)}.

VII.48. Daca ai = i + i, i = 1, 2004, precizati daca numarul


N = a1 a2 a3 + a4 + a5 a6 a7 + a8 + + a2001 a2002 a2003 + a2004
este negativ, pozitiv sau nul.
Viorel Cornea si Dan S
tefan Marinescu, Hunedoara
Solutie. Avem c
a N = N1 + N2 , unde
N1 = (1 2 3 + 4) + (5 6 7 + 8) + + (2001 2002 2003 + 2004) , iar

N2 = 1 2 3+ 4 + 5 6 7+ 8 + + 2001 2002 2003+ 2004 .


48

Evident c
a N1 = 0 si cum vom ar
ata c
a fiecare parantez
a din
scrierealui N2 este
nega
tiv
a, va rezulta c
a N < 0. Pentru a demonstra c
a p + p + 3 < p + 1 + p + 2,
p 1, este suficient s
a ridic
am la p
atrat n ambii membri, obtinnd dup
a reduceri
p2 + 3p < p2 + 3p + 2, fapt adev
arat.
VII.49. Fie 4ABC echilateral si D (BC). Notam cu M1 , M2 mijloacele
segmentelor [BD], respectiv [CD]. Paralela prin M1 la AC intersecteaza AB n F ,
iar paralela prin M2 la AB intersecteaza AC n E. Sa se arate ca dreptele AD,
M1 E si M2 F sunt concurente.
Nicolae Gross si Lucian Tutescu, Craiova
Solutie. Deoarece 4BF M1 si 4CEM2 sunt echilaA
terale, avem BM1 = BF = F M1 , CE = CM2 = EM2 ,
AF = CM1 si AE = BM2 . Fie {S} = AD EM1 ,
{T } = AD F M2 . Aplicnd teorema lui Menelaus n
4ABC cu transversala M1 S E si n 4ABD cu
transversala F T M2 , obtinem
M2 D BF AT
M1 D CE AS

=1=

,
M1 C EA SD
M2 B F A T D
AT
AS
=
, adic
a S = T.
de unde
SD
TD

T=S
B

M1

M2

VII. 50. Fie ABCD un trapez cu bazele [AB] si [CD]. O paralela la baze
intersecteaza AD, AC, BD si BC n punctele E, F , G si respectiv H. Sa se arate
ca EH = 3F G daca si numai daca DF , CG si AB sunt drepte concurente.
Adrian Zanoschi, Iasi
Solutie. Aplic
am teorema fundamental
a a
D
C
asem
an
arii n 4ADC si n 4BDC, obtinem c
a
EF
AE
GH
BH
=
si
=
de unde, avnd n
DC
AD
DC
BC
BH
AE
=
, rezult
a c
a EF = GH = u.
vedere c
a
H
AD
BC
E
Not
am nc
a v = F G si s
a presupunem c
a
F
G
EH = 3F G; atunci 2u + v = 3v, adic
a u = v. Astfel, n 4DEG avem DF median
a si AB k EG, deci
DF intersecteaz
a AB n mijlocul M al segmentului A
M=P
B
[AB]. Analog rezult
a c
a CG intersecteaz
a AB n
M , de unde urmeaz
a c
a DF , CG si AB sunt concurente. Reciproc, dac
a cele trei
a v
a
u
drepte sunt concurente n P , not
am AP = a, P B = b si obtinem c
a = si = ,
v
b
u
b
de unde u = v, adic
a EH = 2u + v = 3v = 3F G.

Clasa a VIII-a

m n
+
= 2003.
VIII.46. Sa se demonstreze ca nu exista m, n N pentru care
n m
Alexandru Negrescu, elev, Botosani
m
2
Q. n aceste
Solutie. Relatia dat
a se scrie t 2003t + 1 = 0, unde t =
n
conditii, discriminantul ecuatiei n t trebuie s
a fie p
atratul unui num
ar rational, chiar
atrat perfect.
p
atrat perfect. ns
a = 20032 4 = 2001 2005 nu este p
49

n
n
m
m
+
N implic
+
= 2.
a
n
m
n
m
VIII.47. Pentru x (0, ), sa se demonstreze inegalitatea

5 3 2 3 2 4 3
x +x +x +1 x +x +2 + x +x +x+1 x3 +x+2 + x3 +x2 +x+1 x2 +x+2
6.
x6 + x5 + x4 + 2x3 + x2 + x + 1
Mircea Cosbuc, elev, Iasi
Solutie. Cu notatiile a = x + 1, b = x2 + 1, c = x3 + 1, inegalitatea se scrie
succesiv
a b a c b c
ac (a + c) + bc (b + c) + ab (a + b)
6 + + + + + 6.
abc
b a c a c b
a b
Ultima inegalitate rezult
a din cunoscutele + 2 si analoagele.
b a
VIII.48. Gasiti numerele prime p si q pentru care p2 + q = 37q 2 + p.
Liviu Smarandache, Craiova
Solutie. Evident p 6= q si scriem ipoteza sub forma p (p 1) = q (37q 1), unde
(p, q) = 1. Urmeaz
a c
a 37q 1 = tp, t N , deci p (p 1)= qtp, adic
a p = qt + 1 si
nlocuind n ipotez
a g
asim c
a qt2 + t = 37q 1. De aici, q 37 t2 = t + 1 2, deci
a ncerc
ari, singurul caz favorabil
37 t2 0, prin urmare t {1, 2, 3, 4, 5, 6}. Dup
r
amne t = 6, cnd q = 7, p = 43.
Not
a. Se poate ar
ata c
a relatia

VIII.49. Fie 4ABC dreptunghic n A cu AB = AC = a. Consideram MA (ABC),

\
M A = a 2 si N AM astfel nct m(CN,
BM ) = 60 . Sa se afle lungimea segmentului [AN ].
Romanta Ghita
si Ioan Ghita
, Blaj
Solutie. Fie N de aceeasi parte a planului (ABC) ca
N
si M; not
am AN = x. Construim d k M B, cu N d;
evident c
a N P (AMB) si fie {P } = AB d.
Din
M
x 2
x 6
4ABM 4AP N , obtinem AP =
, NP =
.
2
2
Cu teorema
lui Pitagora n 4AP C si 4N AC, g
asim
C
r

x2
2
2
2
\
P C = a + , N C = a + x . Deoarece m(P N C) = A
2
\
4N P C duce
= m(CN,
BM ) = 60 , teorema cosinusului n
a 15
la o ecuatie n x, cu solutia admisibil
ax=
. Analog
B
5
se trateaz
a cazul cnd N se afl
a de cealalt
a parte a planului
P
(ABC), obtinnd acelasi rezultat.
b = m(C)
b = 90 ,
VIII.50. Fie patrulaterul convex ABCD cu AB = BC, m(A)
b 90 si fie O mijlocul lui [BD]. Pe perpendiculara n O pe planul (ABC) se
m(B)
ia un punct V astfel nct OV = OB. Sa se arate ca d (D, (V AB)) = 2 d (D, (V AC))
\ = 60 .
daca si numai daca m(ABC)
Monica Nedelcu, Iasi
Solutie. Observ
am c
a ABCD este patrulater nscris n cercul de diametru [BD]
si fie r = OA = OB = OC = OD = OV . Not
am {P } = AC BD si x = OP ;
evident c
a AP = P C si AP BD. Calculnd n dou
a moduri volumul tetraedrului
50


SABD
2r 5
.
VABD, obtinem c
a h1 = d (D, (VAB)) = V O
=
SVAB
5

2
2
Deoarece V P AC, V P = r + x si AC =
2
2
2 r x , calculnd n dou
a moduri volumul lui
V DAC, g
asim
V O SDAC
2r (r x)
=
.
SVAC
r2 + x2

Atunci h1 = 2h 5 (r x) = r2 + x2
3r
r
2x2 5rx+2r2 = 0 si x [0, r] x = BP =
2
2

si
r 3
\= 3
AP =
tg ABD
2
3

\
\
m(ABD) = 30 m(ABC) = 60 .

h2 = d (D, (VAC)) =

O
P

A
B

Clasa a IX-a

x 1 + 3 x 2n x 2 = 2, unde n N .
Dan Popescu, Suceava
Solu
t
ie.
Din
condi
t
iile
de
existen
t
a
a
radicalilor
ob
t
inem

x [2, 3], de unde

2n + 1 +
1].
Ecua
t
ia
se
scrie
atunci
t
1 t2n = 2 + t. ns
a
t= 2nx 2 [0,

2n
2n
a + b 2 a + b, deci 2 + t 2 t + 1 + 1 t = 2, adic
a t = 0. n
consecinta, singura solutie a ecuatiei este x = 2.
IX.46. Sa se rezolve ecuatia

IX.47. Sa se determine sirul (an )n1 de numere strict pozitive pentru care
a21 a22 + a23 + (1)n1 a2n = (1)n1 (a1 + a2 + + an ) , n 1.
Marian Urs
arescu, Roman
a a1 = 1. Pentru n = 2, obtinem
Solutie. Pentru n = 1, obtinem a21 = a1 , adic
a21 a22 = a1 a2 , deci a22 a2 2 = 0 si, cum a2 > 0, g
asim a2 = 2. Intuim c
a
a prin inductie matematic
a. Presupunem
an = n, n 1, fapt care se demonstreaz
c
a a1 = 1, a2 = 2, . . . , ak = k, avem c
a
12 22 + 32 + (1)k1 k2 + (1)k a2k+1 = (1)k (1 + 2 + + k) + (1)k ak+1 ,
de unde, dup
a calcule, folosind faptul c
a

k
P

i=1
k
P

(1)i1 i2 = (1)k1

k (k + 1)
, iar
2

k (k + 1)
, obtinem c
a a2k+1 ak+1 k (k + 1) = 0. Unica solutie pozitiv
i=
aa
2
i=1
acestei ecuatii de grad II este ak+1 = k + 1, ceea ce ncheie demonstratia.

IX.48. Fie a, b, c (0, ) cu a + b + c + abc = 4. Sa se arate ca


c2
a2
b2
3
+
.
+
2
b
+
ca
a + bc
c + ab
Cezar Lupu, elev, Constanta
51

4
Solutie. Conform inegalit
atii mediilor, 4 = a + b + c + abc 4 abc abc,
de unde abc 1, prin urmare a + b + c 3. Folosind acum inegalitatea Cauchy Schwartz si cunoscuta (a + b + c)2 3 (ab + bc + ac), avem:

h
i

c2
b2
a2
+

+
(a + bc) + (b + ca) + (c + ab)
(a + b + c)2
a+ bc b+ ca c+ ab
X a2
3
(a + b + c)2
(a + b + c)2
P
.

P
2
(a
+
b
+
c)
2
a + bc
bc
a+

IX.49. Sa se arate ca 4ABC este isoscel n fiecare din ipotezele:


a) 2ma + b = 2mb + a; b) 2ma + a = 2mb + b.
Marius Pachitariu, elev, Iasi
Solutie. Folosind torema medianei, avem c
a

4 m2a m2b = 2 b2 + c2 a2 2 a2 + c2 b2 = 3 b2 a2 .

n ipoteza a), obtinem c


a (a b) [2ma + 2mb + 3 (a + b)] = 0 si, cum paranteza p
atrat
a ia valori strict pozitive, r
amne c
a a = b. n ipoteza b), g
asim (a b) [2ma +
a 2ma < b + c, 2mb < a + c, deci 2 (ma + mb ) < a + b + 2c.
+2mb 3 (a + b)] = 0. ns
n plus, c < a + b, prin urmare 2 (ma + mb ) < 3 (a + b), adic
a paranteza p
atrat
a ia
valori strict negative si din nou a = b.
IX.50. Fie I centrul cercului nscris n triunghiul ascutitunghic ABC. Daca A,

B, C sunt masurile n radiani ale unghiurilor triunghiului, iar A IA + B IB + C


IC = 0 , sa se arate ca 4ABC este echilateral.


Constantin Micu, Melinesti (Dolj)


b
c
a

Solutie. Deoarece a IA+b IB+c IC = 0 , relatia din ipotez


a arat
a c
a = = .
A B C
a
b
c
sin A
sin B
sin C
Pe de alt
a parte,
=
=
, prin urmare
=
=
.
sin A sin
B
sin
C
A
B
C

sin x
Consider
am functia f : 0,
R, f (x) =
; vom demonstra c
a f este strict
x
2
descresc
atoare. Fie x, y 0,
, x < y; avem:
2
sin y sin x
x (sin y sin x) (y x) sin x
f (y) f (x) =

=
=
y
x
xy
yx
x+y
2x sin
cos
(y x) sin x
2
2
<
=
xy
yx
2x
cos x (y x) sin x
(y x) (x tg x) cos x
2
=
< 0.
<
xy
xy

x+y
(Am folosit faptul c
a 0 < sin x < x < tg x, x 0,
, apoi c
a
> x
2
2

x+y
< cos x, deoarece functia cosinus este descresc
atoare pe 0,
.) Urmeaz
a
cos
2
2
c
a f este injectiv
a si atunci faptul c
a f (A) = f (B) = f (C) conduce la A = B = C,
deci 4ABC este echilateral.
52

Clasa a X-a
2

X.46. Sa se determine a R astfel nct ecuatia 2x1 + 2x

y 2 + ay + a2
y 2 + a2

sa aiba solutii n Z Z.

Petru R
aducanu, Iasi
Solutie. Observ
am c
a pentru orice a R ecuatia dat
a are solutii n Z Z (de
exemplu, perechea (0, 0) pentru a 6= 0 si perechile (0, y), y Z , pentru a = 0).
X.47. Fie z1 , z2 , z3 C distincte, cu z2 +z3 = 2 si astfel nct |z1 1| = |z2 1| =
z1 z3 ) este numar complex pur imaginar.
= |z3 1|. Sa se arate ca (z1 z2 ) (
Lidia Nicola, Craiova
Solutia I. S
a observ
am nti c
a (z1 z2 ) (
z1 z3 ) 6= 0, dat fiind faptul c
a z1 , z2 ,
z3 sunt distincte. Atunci faptul c
a (z1 z2 ) (
z1 z3 ) este pur imaginar este succesiv
echivalent cu:
z1 z3 ) + (
z1 z2 ) (z1 z3 ) = 0
(z1 z2 ) (
z1 z2 ) + (
z2 z3 )] + [(
z1 z3 ) + (
z3 z2 )] (z1 z3 ) = 0
(z1 z2 ) [(
z1 z2 ) + (
z1 z3 ) (z1 z3 ) = z2 z2 z2 z3 z2 z3 + z3 z3
(z1 z2 ) (
|z1 z2 |2 + |z1 z3 |2 = |z2 z3 |2 .

(1)

Fie A1 , A2 , A3 , C punctele de afixe z1 , z2 , z3 si respectiv 1. Deoarece z2 + z3 = 2


implic
a (z2 1) + (z3 1) = 0 CA2 + CA3 = 0 , rezult
a c
a [A2 A3 ] este diametru
a
si, deci 4A1 A2 A3 este dreptunghic n A1 . Atunci A1 A2 2 + A1 A3 2 = A2 A3 2 , adic
tocmai relatia de demonstrat scris
a sub forma (1).
Solutia a II-a (prof. Dumitru G
aleat
a, Iasi; Diana Timofte, elev
a, Iasi). Din
conditia z2 +z3 = 2 rezult
a c
a z3 = 2z2 si, deci, |z3 1| = = |2 z2 1| = |z2 1|;
asadar, egalitatea a 2-a din conditie este superflu
a. Notnd zk = ak + ibk , k = 1, 3,
din z2 + z3 = 2 deducem c
a a2 + a3 = 2 si b2 + b3 = 0, iar din |z1 1| = |z2 1|

seama de aceste relatii, prin


obtinem a21 2a1 + 1 + b21 = = a22 2a2 + 1 + b22 . Tinnd
calcul direct, se arat
a c
a Re (z1 z2 ) (
z1 z3 ) = 0.
X.48. Se considera planele paralele si aflate la
distanta h unul de celalalt si 4ABC echilateral inclus n
A
planul .
a) Sa se afle locul geometric al punctelor M pentru
care M A2 + h2 = MB 2 + MC 2 .
C
B
P
b) Sa se determine M astfel nct suma MA2 +
+MB 2 + MC 2 sa fie minima.
Viorel Cornea si
D
Dan S
tefan Marinescu, Hunedoara
Solutie. a) Fie M 0 = Pr M; atunci MA2 = h2 + M 0 A2
M
si analoagele, deci relatia care caracterizeaz
a pe M devine
M 0 A2 = M 0 B 2 +M 0 C 2 . Fie D simetricul lui A fata de BC si
fie a latura 4ABC. Aplicnd teorema medianei n 4M 0 AD si n 4M 0 BC, obtinem

4M 0 P 2 = 2 M 0 A2 + M 0 D2 AD2 = 2 M 0 B 2 + M 0 C 2 BC 2 ,
de unde, dup
a calcule, M 0 D2 = a2 . Atunci locul punctului M 0 este inclus n cercul
53

de centru D si raz
a a. Se arat
a usor c
a orice punct de pe acest cerc apartine locului
lui M 0 . Rezult
a c
a locul lui M este proiectia locului lui M 0 pe planul .
b) Dac
a T este punctul lui Torricelli al 4ABC, avem:
MA2 + M B 2 + MC 2 = h2 + M 0 A2 + h2 + M 0 B 2 + h2 + M 0 C 2

(M 0 A + M 0 B + M 0 C)2
(T A + T B + T C)2
3h2 +
,
3
3
a parte, M 0 = T .
cu egalitate dac
a M 0 = O - centrul cercului circumscris si, pe de alt
ns
a 4ABC este echilateral, deci O = T si atunci suma este minim
a dac
a M = Pr O.
3h2 +

X.49. Sa se arate ca sin3 x + sin3 y + sin3 z 3 sin x sin y sin z


3
[sin x (1 cos (y z)) + sin y (1 cos (z x)) + sin z (1 cos (x y))] ,
4
x, y, z [0, /3].
Marian Tetiva, Brlad
Solutie. Functia sinus este concav
a pe intervalul [0, ]; aplicnd inegalitatea lui
Jensen cu argumentele 3x, 3y, 3z [0, ], obtinem c
a

sin 3x + sin 3y + sin 3z 3 sin (x + y + z)


X
X
sin x cos y cos z 3 sin x sin y sin z
3
sin x 4
sin3 x 3
X
X
X
sin x 3
sin x cos y cos z 9 sin x sin y sin z.
4
sin3 x 12 sin x sin y sin z 3
X

Dup
a rearanjarea termenilor n membrul drept si mp
artirea prin 4, obtinem inegalitatea dorit
a. Egalitate se obtine pentru x = y = z.

X.50. Fie ak , bk , ck N, k 1, n; notam cu f (p) numarul tripletelor (A, B, C)


de submultimi (nu neaparat nevide)Pcu reuniunea
P M = {1,
P 2, . . . , n}, oricare doua
disjuncte si astfel nct numarul
ai +
bi +
ci p sa fie multiplu
iM\A
iM \B
iM\C
P
xi = 0). Aratati ca daca f (0) = f (1) = f (2), atunci exista i
de 3 (convenim ca
i
.
pentru care ai + bi + ci .. 3.
Gabriel Dospinescu, student, Bucuresti
Solutie. Vom folosi metoda descris
a n articolul Combinatorica ... algebrica
2
2
publicat de autorul problemei n nr. 2/2003 al revistei. Fie = cos
+ i sin ;
3
3
avem c
a
S
S
S
n
X
Y
(ai +bi )+
(ai +ci )+
(ci +bi )
ai +bi

iB
iA
+ bi +ci + ci +ai =
iC
,

i=1

ABC=M
AB=BC=CA=

fapt care se obtine desf


acnd parantezele n stnga si grupnd termenii.
a
p = p(mod 3) si atunci, utiliznd ipoteza, obtinem c
n
Y

ai +bi + bi +ci + ci +ai = f (0) + f (1) + f (2) 2 .

ns
a

i=1

Dac
a f (0) = f (1) = f (2),produsul din stnga este zero, deci exist
a i M pentru
a si numai dac
a numerele
care ai +bi + bi +ci + ci +ai = 0. Aceasta este posibil dac
54

ai + bi (mod 3), bi + ci (mod 3) si ci + ai (mod 3) reprezint


a o permutare a numerelor
0, 1, 2 si atunci 3 | ai + bi + bi + ci + ci + ai , deci 3 | ai + bi + ci .

Clasa a XI-a

XI.46. Determinati A, B Mn (Z) pentru care det (A+B) = 2 si det (A+3B) = 5.


Cezar Lupu, elev, Constanta
Solutie. Fie A, B ca n enunt si fie P (X) = det (A + XB) Z [X], polinom de
grad n. Din ipotez
a, P (1) = 2 si P (3) = 5. Se stie c
a, dac
a P Z [X] si a, b Z
..
.
sunt distincte, atunci P (a) P (b) . a b; n cazul nostru, P (3) P (1) .. 3 1, deci
..
3 . 2, absurd. n concluzie, nu exist
a matrice cu propriet
atile dorite.

a, daca i = j
, unde b 6= 0 si
XI.47. Fie A Mn (R) matrice cu aij =
b, daca i 6= j
a

/ Z. Aratati ca A este inversabila si determinati A1 .


b
Gheorghe Iurea, Iasi
Solutie. Adunnd toate liniile la prima si apoi sc
aznd, pe rnd, coloana 1
din celelalte coloane, obtinem c
a det A = [a + (n 1) b] (a b)n1 6= 0, deoarece
a
a
a + (n 1) b = 0 = 1 n Z, iar a b = 0 = 1 Z, situatii contradictorii.
b
b
Rezult
a c
a A este inversabil
a. Pentru determinarea inversei, fie A = (a b) I + bB,
unde I este matricea unitate, iar B matricea avnd toate elementele egale cu 1.
Avem:
A2 = (a b)2 I + 2b (a b) B + b2 B 2 =
1
1
= (a b)2 I + 2b (a b) [A (a b) I] + b2 n [A (a b) I] =
b
b
h
i
= I (a b)2 2 (a b)2 bn (a b) + A [2 (a b) + bn] =

= [2a + b (n 2)] A (a b) [a + b (n 1)] I

A = [2a + b (n 2)] I (a b) [a + b (n 1)] A1


1
2a + b (n 2)
I
A
A1 =
(a b) (a + bn b)
(a b) (a + bn b)
XI.48. Se defineste sirul (xn )n0 prin xn = x2n1 [xn1 ], n 1;


x0 0, 1 + 5 /2 . Sa se arate ca lim xn = 0.
n
C
at
alin Tig
aeru, Suceava
Solitie. Dac
a x0 [0, 1), atunci [x0 ] = 0 si se demonstreaz
a usor prin inductie
n
a x0 = 1, se observ
a imediat c
a xn = 0,
c
a xn = x20 ; prin urmare, lim xn = 0. Dac
n
n 1, adic
a lim xn = 0 si n acest caz.
n

1+ 5
1+ 5
. Se arat
a prin inductie c
a xn 0,
,
Presupunem c
a x0 1,
2
2
n 1. n plus, xn poate lua valorile x2n1 , 0 sau x2n1 1, dup
a cum xn1 [0, 1),

1+ 5
xn1 = 1, respectiv xn1 1,
; n fiecare caz, xn xn1 0, deci sirul
2
este descresc
ator. Rezult
a c
a sirul este convergent si fie l limita sa. Vom demonstra
55

nn0

c
a exist
a n0 N astfel nct [xn0 ] = 0; atunci xn = x2n0

, n n0 (prin inductie),

1+ 5
de unde concluzia. S
a presupunem deci prin absurd c
a xn 1,
, n N.
2
2

a n aceast
a relatie, obtinem
n acest caz, xn = xn1 1, n N si, trecnd la limit

1+ 5
c
a l =
. Aceasta contrazice ns
a faptul c
a sirul este descresc
ator, ceea ce
2
ncheie demonstratia.

P
an < ,
XI.49. Fie (xn )n0 , (an )n0 siruri de numere reale astfel nct
n=1

|xn+1 2xn + xn1 | + |xn+1 3xn + 2xn1 | an , n 1. Sa se arate ca (xn )n0


este convergent.
Paul Georgescu si Gabriel Popa, Iasi
Solutie. Demonstr
am nti c
a, dac
a (zn )n0 si (an )n0 sunt siruri de numere

P
an < , astfel nct |zn+1 zn | an , n 0, atunci (zn )n0 este
reale cu
n=1
n1
n1
P
P
convergent. ntr-adev
ar, deoarece |zn | |z0 |+
|zk+1 zk | |z0 |+
ak , obtinem
k=0

k=0

c
a (zn )n0 este m
arginit. Apoi, fiindc
a an zn+1 zn an , n 0, atunci

P
P
P
ak zn +
ak , deci sirul yn = zn +
ak este monoton descresc
ator
zn+1 +

k=n+1
k=n
k=n

P
si m
arginit, adic
a (yn )n0 este convergent. ns
a sirul
ak
este convergent
k=n

n0

la 0, deci (zn )n0 este sir convergent.


Aplic
am acest rezultat sirurilor zn1 = xn+1 xn si zn2 = xn+1 2xn si rezult
a
convergenta lor; atunci (xn )n0 este convergent ca diferenta de siruri convergente.

nx + y

XI.50. Fie n 2 N, iar f : R R o functie cu proprietatea ca f


n+1

f ( n+1 xn y), x, y R. Sa se arate ca functia este descrescatoare pe (, 0] si


crescatoare pe [0, ). (n leg
atur
a cu Problema 2819 din Crux Mathematicorum,
nr. 2/2003.)
Titu Zvonaru, Bucuresti
Solutie. Fie a, b [0, ) astfel nct a > b. Deoarece

x + x + + x + y
nx + y
=
n+1 xn y, x, y > 0,
n+1
n+1

nx + y
ncerc
am s
a g
asim x, y astfel nct a =
, b = n+1 xn y. Prin substitutie,
n+1
x trebuie s
a fie solutie a ecuatiei nxn+1 (n + 1) axn + bn+1 = 0. Cu notatia
a g este functie continu
a de x, g (a) =
g (x) = nxn+1 (n + 1) axn + bn+1 , avem c
= bn+1 an+1 < 0 si lim g (x) = , deci ecuatia g (x) = 0 admite o unic
a solutie
n

ator g
asim pe y0 . n aceste conditii,
x0 > a > 0; corespunz

p
nx0 + y
f (a) = f
f n+1 xn0 y0 = f (b) ,
n+1
adic
a f este cresc
atoare pe [0, ).

56

Dac
a a, b (, 0] cu a > b proced
am asem
an
ator, cu observatia c
a n acest caz

nx + y
a n este par.
n+1 xn y, iar radicalul are sens dat fiind faptul c
n+1

Clasa a XII-a
XII.46. Sa se determine functia f : R R daca (R, ) este grup abelian
cu proprietatea ca simetricul oricarui element x [1, 1] se afla n [1, 1], unde
x y = f (x) + f (y), x, y R.
Ioan S
ac
aleanu, Hrl
au
Solutie. Not
am cu e elementul neutru al grupului si cu x0 simetricul lui x. Din
x = x e, x R, rezult
a c
a f (x) = x f (e), x R. Pentru x = e, obtinem
e
e
a f (x)+f (x0 ) = e,
f (e) = , deci f (x) = x , x R. Deoarece xx0 = e, avem c
2
2
a
prin urmare x0 = 2e x. Pentru x = 1, x0 = 2e 1 [1, 1], de unde e [0, 1]. Dac
amne c
a e = 0, deci f (x) = x,
x = 1, x0 = 2e + 1 [1, 1], de unde e [1, 0]. R
functie care verific
a toate conditiile din problem
a.
XII.47. Fie G = (a,
deter b), a, b R, iar "" nmultirea numerelor reale. Sa se
mine a, b astfel nct R+ ,
= (G, ) printr-un izomorfism de forma f : R+ G,
x +
, x R+ , cu , , , R.
f (x) =
x +
Alexandru Blaga si Ovidiu Pop, Satu Mare
Solutie. Deoarece (G, ) este grup si 1 este unitatea fata de nmultire, rezult
a c
a
a < 1 < b si f (1) = 1 + = + .
Dac
a b R, fie x, y G, 1 < x < b, 1 < y < b. Cum (G, ) este grup, urmeaz
a
c
a xy < b2 b, de unde b2 b sau b (0, 1), ceea ce este n contradictie cu 1 < b.
Rezult
a c
ab
/ R, deci b = +.

Deoarece f 0 (x) =
, functia f este strict monoton
a pe (0, ) .
(x + )2
Cazul I. f strict cresc
atoare pe (0, ) > 0. Este necesar ca lim f (x) = a
x0+

si lim f (x) = +, adic


a = a si = 0, > 0. Asadar,
x

x + a
f (x) =
= (1 a) x + a, x R+

(am utilizat faptul c


a relatia + = + revine la = (1 a) . Cum pentru
x = y = 2 n conditia de morfism obtinem f (4) = [f (2)]2 , rezult
a c
a
4 3a = (2 a)2 a2 a = 0 a {0, 1} .

Convine doar a = 0. n concluzie, a = 0, b = + si f (x) = x, x R+ .


Cazul II. f strict descresc
atoare pe (0, ) < 0. Urm
am pas cu pas
1

aceeasi cale si obtinem a = 0, b = + si f (x) = , x R+ .


x
XII.48. Fie (G, ) grup de element neutru e si x, y G penrtru care avem:
a) k N \ {1} a. . xk = e;
b) p N \ {1} a. i. xy = yp x.
Sa se arate ca:
1) xy n xk1 = y np , n N ;
2) xy = yx y n(p1) = e, n N .
Mihai Haivas, Iasi
57

Solutie. 1) Demonstr
am afirmatia prin inductie dup
a n. Pentru n = 1, xyxk1 =
= y p xxk1 = y p xk = y p . Presupunem concluzia adev
arat
a pentru n si s
a o justific
am
pentru n + 1; avem:

xy n+1 xk1 = (xy) yn xk1 = y p x y n xk1 = yp y np = y (n+1)p .

2) Dac
a xy = yx, atunci y p x = yx, deci y p = y si prin urmare y p1 = e; evident
a y n(p1) = e, n N , n
avem si y n(p1) = en = e, n N . Reciproc, dac
particular y p1 = e, deci xy = y p x = yp1 yx = e yx = yx, ceea ce ncheie
rezolvarea.
XII.49. Se considera numerele reale b > a 0, c 1 si functiile continue
Z nb
g (x) dx = d R. Sa se arate ca sirul (un )n1 ,
f, g : R+ R+ astfel nct lim
n na
Z b
1
un =
dx este convergent si sa se afle limita sa.
a c + f (x) + g (nx)
D. M. B
atinetu - Giurgiu, Bucuresti
Solutie. Din ipotezele problemei, avem c
a
1
g (nx)
1

=
g (nx)
0
c + f (x) c + f (x) + g (nx)
[c + f (x)] [c + f (x) + g (nx)]
a
pentru x [a, b] si n N . Prin integrare, deducem c
Z b
Z b
1
0
un
g (nx) dx.
a c + f (x)
a
Z
Z b
ns
a
1 bn
1
g (nx) dx = lim
g (t) dt = d lim = 0,
0 lim
n a
n n an
n n
Z b
1
prin urmare exist
a limita sirului (un )n1 , egal
a cu
dx.
a c + f (x)
s (n!)
XII.50. Fie s (n) suma cifrelor numarului natural n. Sa se calculeze lim k
,
n ln ln n
unde k N este fixat.
Gabriel Dospinescu, student, Bucuresti
Solutie. Vom ar
ata nti c
a orice multiplu nenul al lui A = |11 {z
. . . 1} are suma
m

cifrelor cel putin m. ntr-adev


ar, s
a presupunem c
a exist
a B cel mai mic multiplu
nenul al lui A cu s (B) < m. Cum s (iA) m, i 1, 9 se impune s
a avem
am num
arul
B 10A > 10m ; fie B = ar 10r + + a1 10 + a0 , unde r m. Consider
C = B 10rm (10m 1).
Evident c
a C < B si A divide C. n plus, dac
a arm < 9 atunci s (C) = s(B), iar
dac
a arm = 9, atunci s (C) < s (B), prin urmare C este un num
ar mai mic dect
B si avnd propriet
atile acestuia, absurd. R
amne deci c
a s (nA) m, n N .
Acum, deoarece 11
. . . 1} | 10[lg n] 1, iar 10[lg n] 1 | n! fiindc
a 10[lg n] 1 < n,
| {z
[lg n]

aplicnd rezultatul demonstrat mai sus urmeaz


a c
a s (n!) [lg n]. Se stie ns
a c
a
[lg n]
s (n!)
lim
= , prin urmare lim k
= .
n lnk ln n
n ln ln n
58

Solutiile problemelor pentru preg


atirea concursurilor
din nr. 1 / 2004
A. Nivel gimnazial
G56. Fie m Z, n 2 Z + 1 fixate. Sa se arate ca ecuatia nx + y = m, x, y Z
are o unica solutie (x0 , y0 ) cu proprietatea ca |y0 | < |n| /2.
Petru Asaftei, Iasi
Solutie. Pentru existenta solutiei, s
a observ
am c
a exist
a q, r Z, 0 r < |n| cu
|n|
m = nq + r (din teorema mp
artirii cu rest). Dca
a0r<
, lu
am x0 = q, y0 = r.
2
|n|
Dac
a
a
< r < n, lu
am x0 = q + sgn (n) si y0 = r |n|; avem evident c
2
nx0 + y0 = n (q + sgn (n)) + r |n| = nq + |n| + r |n| = nq + r = m,
iar |y0 | = |n| r < |n|

|n|
|n|
=
.
2
2

Pentru demonstrarea unicit


atii, fie nc
a (x1 , y1 ) solutie a ecuatiei cu |y1 | <

atunci n (x0 x1 ) = y0 y1 , de unde

|n|
;
2

|n| |x0 x1 | = |y0 y1 | |y0 | + |y1 | < |n|


si cum |x0 x1 | N, n mod necesar trebuie s
a avem |x0 x1 | = 0, adic
a x1 = x0 si
apoi y1 = y0 .
G57. Un seic a lasat mostenire celor doi fii ai sai cinci camile, cu conditia ca
unul sa primeasca jumatate, iar celalalt o treime. Mostenitorii nu si-au putut mparti
averea, asa ca au apelat la un ntelept care trecea pe acolo, calare pe o camila. Cum
a procedat nteleptul?
Cte probleme asemanatoare mai putem formula (n care mostenirea este de n
camile, iar fiii primesc a p-a si a q-a parte)?
Gabriel Popa, Iasi
Solutie. Problema este clasic
a, dar cu 3 fii si 17 c
amile. Totul se bazeaz
a pe
1 1
5
faptul c
a + = 6= 1. nteleptul aseaz
a c
amila sa lng
a cele cinci l
asate mostenire
2 3
6
1
1
amile, adic
a 3; al doilea ia , adic
a 2 c
amile, iar
de seic. Primul fiu ia din cele 6 c
2
3
nteleptului i r
amne c
amila sa.
Pentru a formula alte asemenea probleme, trebuie s
a g
asim trei numere nenule
1
n
1
. Evident c
a p, q 2,
p, q, n astfel nct p si q s
a divid
a n + 1, iar + =
p
q
n+1
n
1 1
1 1
5
m
acar unul cu inegalitate strict
a; atunci
= + + = , deci n 5.
n+1
p q
2 3
6
Considernd cazurile posibile, obtinem c
a (n, p, q) {(5, 2, 3) ; (3, 2, 4)}, binenteles
n ipoteza p q. Afar
a de problema dat
a, mai putem formula nc
a una.
Generalizare (Petru Asaftei, Iasi). Dac
a nteleptul are r c
amile, r 1 putem
formula o infinitate de probleme. Mai precis, pentru p, q 2 numere
naturale
fixate,

1 1

p + q 6= 4, vom determina n, r N , p - n, q - n, astfel nct


+
(n + r) = n.
p q
59

1 1
n
pq p q
1 1
+
(n + r) = n + =

=
Dac
a (p, q) = 1, atunci
p q
p q
n+r
pq
n
. Deoarece (pq p q, pq) = 1, avem r = k (pq p q), n + r = pqk n =
n+r
a (p, q) = d > 1, atunci
k (p + q), cu k N , k nefiind multipli de p sau q. Dac
p1 q1 d2 p1 d q1 d
p1 q1 d p1 q1
pq p q
=
, cu (p1 , q1 ) = 1. Deoarece
=
pq
p1 q1 d2
p1 q1 d
(p1 q1 d p1 q1 , p1 q1 d) = 1, atunci r = k (p1 q1 d p1 q1 ), n + r = p1 q1 dk
n = k (p1 + q1 ), cu k N , k nefiind multiplu de p sau q. Conditia p + q 6= 4 exclude
cazul p = q = 2, care ar conduce la n + r = n r = 0.
G58. Sa se rezolve n N2 ecuatia 2x + 1 = 5y .
Irina Mustata
a, si Valentina Blendea, Iasi
, elev
Solutie. Dac
a x, y sunt ambele pare, x = 2p si y = 2q, p, q N, atunci
2x + 1 = 4p + 1 = (3 + 1)p + 1 = M3 + 2, iar 5y = 25q = (M3 + 1)q = M3 + 1, deci
a x, y sunt ambele impare x = 2p + 1, y = 2q + 1, p, q N, atunci
2x + 1 6= 5y . Dac
2x + 1 = 2 6= 5y pentru p = 0 si 2x + 1 = 2 4p + 1 = M8 + 1, 5y = 5 25q = M8 + 5,
a x = 2p + 1, y = 2q, p, q N, atunci
deci 2x + 1 6= 5y si pentru p 1. Dac
2x +1 = 24p +1 = 2 (M3 + 1)+1 = M3 iar 5y = M3+1, deci 2x +1 6= 5y . n sfrsit,
pentru x = 2p, y = 2q + 1, p, q N, avem n cazul p 2 c
a 2x + 1 = 4p + 1 = M8 + 1,
y
x
y
iar 5 = M8 + 5, adic
a 2 + 1 6= 5 . Dac
a p 1, prin verific
ari obtinem unica solutie
a ecuatiei (2, 1).
G59. Fie A = {n N | s (2000n) + s (2002n) = 2s (2001n)}, unde prin s (x) am
notat suma cifrelor lui x. Demonstrati ca orice numar natural nenul are un multiplu
ce apartine lui A.
Gabriel Dospinescu, student, Bucuresti
Solutie. Not
am N (k, p) = 11
.
.
.
. . . 0} ; atunci pentru orice p > 3 si orice
| {z 1}00
| {z
p

. . . 0}, 2002 N (k.p) = 22244


. . . 4}22200
. . . 0},
k N avem c
a 2000 N (k, p) = 22
. . . 2}00
| {z
| {z
| {z
| {z
p

p3

k+3

. . . 0}, deci
2001 N (k, p) = 22233
. . . 3}11100
| {z
| {z
p3

s (2000 N (k, p))+s (2002 N (k, p)) = 2p+6+4 (p 3)+6 = 6p = 2s (2001 N (k, p)) ,
adic
a N (k, p) A. Fie acum m N oarecare; considernd numerele 1, 11, 111, . . . ,
conform principiului cutiei rezult
a c
a putem g
asi i < j astfel nct m s
a divid
a
diferenta 11
.
.
.
1

11
.
.
.
1
,
deci
m
|
N
(j

i,
i)
ceea
ce
ncheie
solu
t
ia.
| {z } | {z }
j

G60. Sa se demonstreze ca pentru orice a, b, c (0, ) are loc


ab
2

(a + b)

bc
2

(b + c)

ca
2

(c + a)

4abc
1
+
.
4 (a + b) (b + c) (c + a)

Gabriel Dospinescu, student, Bucuresti


60

Solutie. S
a observ
am c
a
4abc
2 [(a+b) (b+c) (c+a) ab (a+b) bc (b+c) ca (c+a)]
=
=
(a + b) (b + c) (c + a)
(a + b) (b + c) (c + a)

2
a + bc (b + c) + b + ac (a + c) + c + ab (a + b)
=
=2
(a + b) (b + c) (c + a)

c
a
a
b
b
b
a
c
c
c
b
a
+
+
+
+
+
=
=2
a+b b+c a+b a+c b+c a+b b+c a+c c+a b+c c+a a+b
"
#

bc
ac
b
c
c
a
ab
a
b
= 2
.

+
+
+
+

a+b b+c c+a


a+b b+c c+a
(a+b)2 (b+c)2 (a+c)2
Aplic
am inegalitatea mediilor numerelor

a
b
c
b
c
a
+
+
si
+
+
,
a+b b+c c+a a+b b+c c+a

a c
aror sum
a este 3; obtinem c
a

b
c
c
a
9
a
b
+
+
+
+
.
a+b b+c c+a
a+b b+c c+a
4

nlocuind n identitatea demonstrat


a, obtinem concluzia. Egalitatea este atins
a
atunci cnd dou
a dintre numerele a, b, c sunt egale.
G61. Sa se demonstreze ca pentru orice a, b, c (0, ) are loc
p

(a2 +b2 )(b2 +c2 )(c2 +a2 )


a+b b+c c+a
(a+b)(b+c)(c+a)
+
+
27
.
54 2
c
a
b
abc
abc
Marian Tetiva, Brlad
Solutie. Prima inegalitate se scrie succesiv
p

[ab (a+b) + bc (b+c) + ca (c+a)]3 54 2 a2 b2 c2 (a2 +b2 ) (b2 +c2 ) (c2 +a2 )
p

3
2 2
54 2 a2 b2 c2 (a2 +b2 ) (b2 +c2 ) (c2 +a2 ).
a b +c + b a2 +c2 + c a2 +b2
Pentru a demonstra
aceast
vom intercala ntre cele dou
a cantit
ati pe

a inegalitate,

27abc a2 + b2 b2 + c2 c2 + a2 . Faptul c
a

2
27abc a2 + b2 b2 + c2 c2 + a2
a b + c2 + b a2 + c2 + c a2 + b2

rezult
a din inegalitatea mediilor aplicat
a numerelor a b2 +c2 , b a2 +c2 si c a2 +b2 .
Apoi, avem
p

27abc a2 + b2 b2 + c2 c2 + a2 54 2 a2 b2 c2 (a2 + b2 ) (b2 + c2 ) (c2 + a2 )

2
a + b2 b2 + c2 c2 + a2 8a2 b2 c2 ,

fapt care rezult


a prin nmultirea membru cu membru a inegalit
atilor a2 + b2 2ab,
2
2
2
2
b + c 2bc, c + a 2ac.
n ce priveste a douap
inegalitate, ce rezult
a prin nmultirea membru
cu membru a
p
p
inegalit
atilor cunoscute 2 (a2 + b2 ) a+b, 2 (b2 + c2 ) b+c, 2 (c2 + a2 ) 2ac.
Egalitatea se atinge pentru a = b = c.
G62. Fie ABCD un patrulater convex n care se poate nscrie patratul MNP Q
de centru O (M (AB), N (BC), P (CD), Q (AD)). Sa se arate ca
AB + BC + CD + DA 2 (AO + BO + CO + DO). Cnd are loc egalitatea?
Lucian Tutescu, Craiova si Ioan S
erdean, Or
astie
61

Solutie. Conform inegalit


atii lui Ptolomeu
a n patrulaterul AMOQ, avem
aplicat
a 2
(AQ + AM ) a AO, unde a este
AQ MO + AM OQ AO MQ, adic
a
2

latura p
atratului M N P Q; prin urmare, AQ + AM 2 AO. Scriind celelalte
trei inegalit
ati analoage si adunndu-le, urmeaz
a concluzia. Egalitatea are loc dac
a
cele patru patrulatere n care s-a aplicat Ptolomeu sunt inscriptibile, fapt care se
ntmpl
a dac
a ABCD este dreptunghi. ns
a n ABCD trebuie s
a se poat
a nscrie
p
atratul MNP Q, deci ABCD este el nsusi p
atrat.
Q
A
Justificare: ABCD - dreptunghi, M N P Q - p
atrat
D
ABCD - p
atrat.
)
\
\
m(M
N B) + m(CN
P ) = 90
M
\
\
BM
N CN
P
\
\ = 90
m(MN
B) + m(NMB)
)
P
\ ) + m(AMQ)
\ = 90
m(BMN
\
\

BM
N

AQM
\
\ ) = 90
m(AM
Q) + m(AQM
Consider
am 4N CP , 4MBN si 4QAM
b = m(B)
b = m(C)
b = 90 )
(m(A)

[N P ] [M N ] [MQ]
IU
4N CP 4M BN 4QAM
\
\
\
CN
P BM
N AQM

[BN ] [AM]

[BC] [AB] ABCD - p


atrat.
[N C] [MB]
b = 10 si m(B)
b = 100 construim M (AB) si
G63. n 4ABC cu m(A)

\
\
\
N ).
N (AC) astfel ca m(MCB) = 40 si m(N BC) = 75 . Sa se afle m(AM
Octavian Bondoc, Pitesti
\
Solutie. Fie P (AC) astfel ncat m(P
BC) = 40 . Obtinem atunci 4BP C isoscel, deci [BP ] [BC], apoi 4MBC isoscel cu [BM ] [BC], de unde [BP ] [BM].
\ ) = 60 .
\
ns
a m(M
BP ) = 60 , deci 4MBP este echilateral si [M P ] [MB], iar m(BMP
Pe de alt
a parte, tot din congruente de unghiuri, 4N P B este isoscel cu [N P ] [P B],
\ ) = 65 . n final,
\
asim m(NMP
prin urmare [N P ] [P M ] si cum m(N
P M) = 50 , g
\
m(AM
N ) = 180 65 60 = 55 .
G64. Prin punctul P al laturii (AC) a 4ABC
A
se duc paralele la medianele AA0 si CC 0 , care intersecteaza laturile (BC) si (AB) n E, respectiv F .
F
Fie {M} = EF AA0 , {N } = EF CC 0 , iar L si Q
L
mijloacele segmentelor [F P ], respectiv [P E]. Sa se C
0 0
M
arate ca dreptele ML, N Q si A C sunt concurente.
P
G N
Andrei Nedelcu, Iasi
Q
S
Solutie. Fie G centrul de greutate al 4ABC
T
si {T } = P E CC 0 . Deoarece T N k P F , avem
C
E
B
A
EN
ET
ET
GA0
1
=
. ns
a
=
= , de unde 2EN =
NF
TP
TP
GA
2
= N F . Analog se arat
a c
a 2MF = ME, prin urmare F M = MN = N E. Fie acum
62

PC
PE
FS
C 0F
=
=
=
, deci F S = P E, adic
a
0
0
AA
CA
AC
AA0
2
F P ES este paralelogram. n 4F SP , M se afl
a pe mediana din F la de vrf, deci
3
M este centrul de greutate al 4F SP si atunci SM este median
a si va trece prin L.
Analog, Q SN , de unde concluzia.
F S k AA0 , S A0 C 0 ; atunci

G65. Fie SABCD o piramida cu baza ABCD dreptunghi, M proiectia lui D


pe SB si N proiectia lui C pe SA, iar {P } = AM N B. Stiind

ca M (SB) si
N (SA), sa se arate ca N P SA M B = SM AN P B.
Daniel S
tefan Ninu, elev, Iasi
Solutie. Fie {O} = AC BD si a = OA = OB = OC = OD. Avem
c
a N O = MO = a, ce mediane corespunz
atoare ipotenuzelor n triunghiuri dreptunghice. Prin urmare, punctele A, B, M , N apartin sferei de centru O si raz
a a.
ns
a cele patru puncte sunt coplanare, iar un plan taie o sfer
a dup
a un cerc, deci patrulaterul ABMN este inscriptibil. Atunci 4N P A 4MP B si 4N P M 4AP B,
NP
NA
PM
MN
NP P M
NM NA
de unde
=
, respectiv
=
, prin urmare

=
,
MP
MB
PB
AB
MP P B
MB AB
MB NP
MN
SM
MN
=
. Pe de alt
a parte, 4SM N 4SAB, deci
=
.
adic
a
AB
PB NA
AB
SA
Comparnd ultimele dou
a relatii, rezult
a concluzia.

B. Nivel liceal
L56. Fie ABCD patrulater convex si {P } = AB CD, {Q} = AD BC.
Consideram J (AQ), L (BQ), K (DP ), N (AP ) astfel nct QJ = AD,
QL = CB, P K = DC si P N = AB. Sa se arate ca JL k N K.
Carmen Nejneru, Iasi
Solutie. Fie M Int ABCD, avem:
Q
AD d (M, AD) BC d (M, BC)
+
=
2
2
QJ d (M, QJ) QL d (M, QL)
=
+
=
J
2
2
D
= SMJQ + SMQL = SQJL + SM JL

SMAD + SM BC =

L
d

si cum SQJL este constant


a, locul geometric al
C
punctelor M pentru care SMAD + SMBC = k este o
K
portiune dintr-o dreapt
a d paralel
a cu JL. Analog se
M
arat
a c
a locul geometric al punctelor M pentru care
BN
P
SMAB + SMCD = k0 este o portiune dintr-o dreapt
a A
1
0
0
d paralel
a cu N K. Lund k = k = SABCD cele dou
a locuri geometrice coincid,
2
prin urmare JL k N K.

Not
a. Solutie corect
a au dat urm
atorii elevi: Andrei-Codrut Onofrei, Lucian
Rotaru, Cosmin-Alexandru Spnu.
L57. Fie 4ABC nscris n cercul C si punctele D (CB, D0 (BC astfel nct
\ Se mai considera cercul C1 tangent la AD, BD si la
\ ABC,
\ BAD
\0 ACB.
CAD
63

C, cercul C2 tangent la AD0 , CD0 si la C, iar {E} = C1 [BD], {F } = C2 [D0 C]. Sa


se arate ca cercul circumscris 4AEF si cercul nscris n 4ABC sunt concentrice.
Neculai Roman, Mircesti (Iasi)
Solutie. Fie {N } = AD C1 , {M1 A} = AD C.
\ ABC
\ AMC,
\ deci [AC] [M C].
Avem c
a CAD
A
Aplic
am teorema lui Casey cercurilor cele A, C, M (deC1
generate) si C1 , tangente interior cercului C; obtinem
succesiv:
C
2

AC M N + AN M C = AM CE
AC MN + AN AC = AM CE
AC (M N + AN ) = AM CE AC = CE.

E D

Rezult
a c
a 4ACE este isoscel, deci mediatoarea lui [AE]
\ Analog se arat
M
este bisectoarea unghiului ACB.
a c
a
\
mediatoarea lui [AF ] este bisectoarea lui ABC, de unde concluzia.

D F C
C

L58. Pe muchiile (Ox, (Oy si (Oz ale unui triedru oarecare se considera punctele
A, L (Ox, B, M (Oy si C, N (Oz astfel nct OA = OB = OC = a si OL =
\
\
\
= OM = ON = b (a < b). Notam = m(Oy,
Oz), = m(Oz,
Ox), = m(Ox,
Oy)
si {P } = (AMN ) (BN L) (CLM), {Q} = (LBC) (M CA) (N AB). Sa se
calculeze distanta P Q n functie de a, b, , , .
Temistocle Brsan, Iasi
Solutie.
Existenta punctului P (ca si
a lui Q) se arat
a usor;
ntr-adev
ar,
z
(AMN ) (BN L) = N X, unde {X} = AM BL,
N
iar N X (CLM) = {P }.

Not
am OA = rA etc. n planul (AM N ) putem
C
scrie

P
AP = AM + AN
B
M
O
y
rP rA = (rM rA ) + (rN rA )
A
X
rP = (1 ) rA + rM + rN . (1)
L

Procednd similar n planele (BN L) si (CLM) x


obtinem si relatiile

rP = 1 0 0 rB + 0 rN + 0 rL , rP = 1 00 00 rC + 00 rL + 00 rM . (2)

Notnd cu u1 , u2 , u3 versorii pe (Ox, (Oy si (Oz, relatiile (1) si (2) se pot scrie

rP = (1 ) au1 + bu2 + bu3 = 1 0 0 au2 + 0 bu3 + 0 bu1 =

= 1 00 00 au3 + 00 bu1 + 00 bu2 .


(3)

Motive de simetrie ne sugereaz


a s
a consider
am = 0 = 00 si = 0 = 00
si s
a c
aut
am , pentru care au loc egalit
atile precedente. Egalnd coordonatele
corespunz
atoare versorilor u1 , u2 , u3 , obtinem sistemul
(1 ) a = b = b,

b = (1 ) a = b,
64

b = b = (1 ) a

a
. Ca urmare, relatiile (3) devine
2a + b

ab
rP = OP =
(u1 + u2 + u3 ) .
2a + b

care are solutia unic


a==

(4)

n mod analog, n privinta punctului Q g


asim

rQ = OQ =

ab
(u1 + u2 + u3 ) .
a + 2b

(5)

n sfrsit, avem urm


atoarele

ab
ab

(u1 + u2 + u3 ) ,
PQ =
a + 2b 2a + b

2

ab
ab
2
PQ = PQ PQ =
(u1 + u2 + u3 ) (u1 + u2 + u3 ) =

a + 2b 2a + b
=
deci

a2 b2 (a b)2
(3 + 2 cos + 2 cos + 2 cos ) ,
(a + 2b) (2a + b)

p
ab (a b)
PQ = p
3 + 2 (cos + cos + cos ).
(a + 2b) (2a + b)

L59. Care este probabilitatea ca latura si diagonalele unui romb, luat la ntmplare, sa fie laturile unui triunghi?
Petru Minut, Iasi
Solutie. Proprietatea de a putea construi un triunghi cu laturile congruente
cu latura si diagonalele unui romb este adev
arat
a sau nu pentru toate romburile
din plan dintr-o clas
a de romburi asemenea. Este suficient s
a rezolv
am problema
pentru romburile a c
aror laturi au lungimea egal
a cu 1. Un asemenea romb este
determinat de unghiul pe care l formeaz
a diagonala cea mai lung
a cu una dintre
laturile rombului;
not
a
m
m
a
sura
acestui
unghi
cu
x.
Mul
t
imea
cazurilor
posibile
i
este D = 0, .
D
C
4
Din triunghiul BCD, conform teoremei cosinusu2x
lui, avem
d21 = BD2 = BC 2 + DC 2 2 BC CD cos 2x =
= 2 2 cos 2x = 4 sin2 x d1 = 2 sin x.

180D 2 x

x
Analog, din 4ABC, se obtine c
a d2 = AC = 2 cos x.

A
B
rezult
a c
a d1 = 2 sin x 2 si
Din 0 < x
4

d2 = 2 cos x 2. Dintre numerele l = 1, d1 = 2 sin x, d2 = 2 sin x cel mai mare


a si numai
este d2 . Putem construi un triunghi cu laturile avnd aceste lungimi, dac
x
dac
a l + d1 > d2 1 + 2 sin x > 2 cos x. Notnd tg = t, avem
2
#

72
4t
1 t2
2
1+
,
.
>
3t + 4t 1 > 0 x 2 arctg
1 + t2
1 + t2
3
4
65

72
Domeniul valorilor posibile este D0 = 2 arctg
,
. Proprietatea cerut
a este
3
4

72
72

8 arctg
2 arctg
mes D0
4
3
3
=1
=
.
p=

mes D

4
L60. Fie A1 A2 . . . An si B1 B2 . . . Bn ( n > 2) doua poligoane nscrise n acelasi
cerc de centru O si avnd centrele de greutate tot n O. Sa se arate ca putem renumerota vrfurile poligonului A1 A2 . . . An pentru a obtine un nou poligon Ai1 Ai2 . . . Ain
n care Aij 6= Bj pentru j {1, 2, . . . , n}.
Gabriel Dospinescu, student, Bucuresti
Solutie. Trebuie s
a ar
at
am c
a exist
a o permutare Sn astfel nct A(1) 6= Bi
pentru i M = {1, 2, . . . , n}. S
a definim matricea T = (tij )i,jM , tij = Ai Bj2 .
Vom ar
ata c
a sumele elementelor de pe orice linie si coloan
a n T sunt egale. Putem
presupune c
a poligoanele sunt nscrise n cercul unitate (O fiind originea planului
complex) si fie ai , bi afixele punctelor Ai , Bi . Suma elementelor de pe linia i n
matricea T este
n
n
n
n

X
X
X
X
|ai | + |bj |2 aibj a
|ai bj |2 =
i bj = 2n ai
bj a
i
bj = 2n
j=1

j=1

j=1

j=1

(am folosit faptul c


a centrul de greutate al poligonului B1 B2 . . . Bn este O, deci
n
P
bj = 0).
j=1

Analog, suma elementelor de pe coloana j a matricei T este 2n. S


a presupunem c
a
pentru orice permutare Sn exist
a i astfel nct ti(i) = 0. Vom spune: "coloana
a elementul de la intersectia liniei si coloanei este nenul n
Ri place linia Sj " dac
matricea T . Rezult
a c
a nu putem asocia cte o linie distinct
a fiec
arei coloane astfel
nct coloanele respective s
a plac
a liniile asociate lor. Deci, din lema mariajelor,
rezult
a c
a exist
a k > 0 si k coloane ce plac cel mult k 1 linii. Prin permut
ari de
linii si coloane, putem presupune c
a aceste linii si coloane sunt primele din matricea
T . S
a facem suma elementelor dreptunghiului determinat de aceste k 1 linii si
k coloane. n dreptunghiul determinat de primele k coloane si ultimele n k + 1
linii avem numai zerouri (c
aci cele k coloane nu plac nici una dintre liniile k, k + 1,
. . . , n), deci suma elementelor din dreptunghi este egal
a cu suma elementelor de pe
primele k coloane, adic
a 2nk. Pe de alt
a parte, evident, suma este cel mult ct suma
elementelor de pe primele k 1 linii, adic
a 2n (k 1). Deducem c
a 2n (k 1) 2nk,
contradictie.
L61. Fie n 3. Sa se determine maximul expresiei
E = x31 x22 + x32 x23 + + x3n x21 + (n 1)2(n1) x31 x32 x3n ,
cnd numerele nenegative x1 , x2 , . . . , xn au suma 1.
Gabriel Dospinescu, student, Bucuresti
3
2
Solutie. Pentru x1 = , x2 = , xk = 0, k 3, n, obtinem pentru E valoarea
5
5
22 33
108
108
= 5 , deci maximul cerut este cel putin
. S
a demonstr
am c
a maximul
3125
5
3125
66

108
. Fie x1 , x2 , . . . , xn 0 cu suma 1, fixate. Permutnd ciclic
lui E este cel mult
3125
atii mediilor, avem
variabilele, putem presupune c
a x1 = max xk . Conform inegalit
s
2
3
5 x (x2 + + xn )
x1 x1 x1 x2 + + xn x2 + + xn
1
+
+
+
+
5
,
1=
3
3
3
2
2
33 22
deci este suficient s
a demonstr
am c
a
ns
a

x31 (x2 + + xn )2 x31 x22 + + x3n x21 + (n 1)2(n1) x31 x32 x3n .

(1)

x31 (x2 + + xn )2 2 x31 x2 x3 + + x31 xn1 xn + x31 x22 + x31 x2n

x31 x2 x3 + + x31 xn1 xn + x32 x23 + + x3n1 x2n + x31 x22 + x31 x2n

x31 x2 x3 + x31 x22 + x32 x23 + + x3n1 x2n + x31 x2n

x31 x2 x3 + x31 x22 + x32 x23 + + x3n1 x2n + x3n x21 ,

unde de fiecare dat


a am folosit faptul c
a x1 = max xk . n aceste conditii, pentru a
demonstra (1) este suficient s
a ar
at
am c
a
x31 x2 x3 (n 1)2(n1) x31 x32 x3n .
Pentru n = 3, aceasta se scrie x2 x3

1
si rezult
a din x2 x3
4

pentru n > 3, (2) se scrie sub forma


x22 x23 x34 x3n
si rezult
a din
x22 x23 x34 x3n (x2 x3 xn )2

(2)

x2 + x3
2

1
, iar
4

1
(n 1)2(n1)

x2 + x3 + + xn
n1

Rezolvarea este astfel ncheiat


a.

2(n1)

1
(n 1)2(n1)

L62. Rezolvati ecuatia 2x2 = y (y + 1); x, y N.

Mircea Brsan, Iasi


Solutia I. Cazul y = 2h, h N. Ecuatia dat
a devine x2 = h (2h + 1) si, ntruct
(h, 2h + 1) = 1, urmeaz
a c
a h, 2h + 1 sunt p
atrate perfecte, adic
a m, n N a..
h = n2 si 2h + 1 = m2 . De aici rezult
a c
a m si n trebuie s
a satisfac
a ecuatia Pell
m2 2n2 = 1,

(1)

iar solutiile ecuatiei initiale sunt date de

y = 2n2 .

x = mn,

(2)

Observ
am c
a (3, 2) este cea mai mic
a solutie nebanal
a a ecuatiei (1) si, dup
a cum
este cunoscut, solutiile ecuatiei (1) n N sunt perechile (mk , nk )kN cu

k
k
k
k
1
1
3+2 2 32 2
mk =
3+2 2 + 32 2
, nk =
.
2
2 2
(3)
67

n conformitate cu (2), solutiile ecuatiei date, n cazul y par, sunt perechile (xk , yk )kN ;
unde

2k
2k
2k
2k
1
1
xk =
3+2 2 32 2
, yk =
3+2 2 + 32 2 2 .
4
4 2
(4)
Cazul y = 2l 1, l N . Urm
am calea din cazul precedent. Ecuatia din enunt se
2,
scrie x2 = l (2l 1). Deoarece (l, 2l 1) = 1, aceste numere sunt de forma l = m
2
2l1 = n
. Rezult
a c
am
si n
verific
a urm
atoarea ecuatie Pell conjugat
a ecuatiei (1)
2m
2 n
2 = 1,

iar pentru ecuatia dat


a avem

(5)

y = 2m
2 1.

x=m
n
,

(6)

Cum (1, 1) este cea mai mic


a solutie nebanal
a a ecuatiei (5), solutiile acestei ecuatii
sunt perechile (m
k, n
k )kN cu
m
k = mk + nk ,

n
k = mk + 2nk ,

unde (mk , nk )kN sunt solutiile ecuatiei (1) date de relatiile (3) (T. Andreescu,
D. Andrica - Asupra rezolvarii n numere naturale a ecuatiei ax2 by 2 = 1, GM 4/1980, p. 146-148). Tinnd

seama de (6), ecuatia din enunt are, n cazul y impar,


solutiile (
xk , yk )kN cu
x
k = (mk + nk ) (mk + 2nk ) ,

yk = 2 (mk + nk )2 1,

(7)

unde mk , nk sunt date de (3).


xk , yk )kN cu xk ,
n concluzie, multimea solutiilor este format
a din (xk , yk )kN si (
yk , x
k , yk date de (4), (7) si (3).
Solutia II. Observ
am c
a ecuatia dat
a admite solutiile banale x = y = 0 si
x = y = 1. C
aut
am solutiile (x, y) cu x, y
/ {0, 1}.
Cazul y = 2h, h N . Ca mai sus, m, n N astfel nct h = n2 si 2h + 1 = m2 ;
deci m, n satisfac relatia 2n2 + 1 m2 . Rezult
a c
a m este impar si n este par,
a, obtinem
adic
a m = 2k + 1, n = 2l cu k, l N . nlocuind n relatia precedent
2l2 = k (k + 1). Asadar, dac
a (x, y) este o solutie nenul
a a ecuatiei date cu y par,
atunci exist
a o alt
a solutie nenul
a (l, k) astfel nct
x = 2l (2k + 1) ,

y = 8l2 ,

si x > l, y > k.

(1)

Cazul y = 2h + 1, h N . Procednd asem


an
ator, dar dup
a calcule putin mai
complicate, ajungem la concluzia c
a pentru orice solutie nebanal
a (x, y) a ecuatiei
date, cu y impar, exist
a o solutie nenul
a (u, v) a acesteia astfel nct
x = (4u + 2v + 1) (2u + 2v + 1) ,

y = (4u + 2v + 1)2

si x > u, y > v.

(2)

Rezult
a c
a oricare ar fi o solutie (x, y) a ecuatiei date diferit
a de (0, 0) si (1, 1),
dup
a un num
ar finit de pasi n care se g
asesc, recursiv, solutii mai mici determinate
prin relatii de tipul (1) sau (2), vom obtine solutia (1, 1) . Cu alte cuvinte, multimea

S
S a solutiilor este dat
a de S = {(0, 0)}
Sn , unde S0 = {(1, 1)} si Sn+1 =
n=0
n

2i (2j + 1) , 8i2 , (4i + 2j + 1) (2i + 2j + 1) , (4i + 2j + 1)2 | (i, j) Sn , n N.


Mai observ
am c
a Sm si Sn sunt disjuncte pentru m 6= n si card (Sn ) = 2n .
68

L63. Fie G Mn (R) un grup netrivial n raport cu produsul uzual al matricelor.


Presupunem ca exista X G astfel nct pe fiecare linie, respectiv coloana a sa sa
existe cel mult un element nenul si acesta egal cu 1. Sa se demonstreze ca exista
k {1, 2, . . . n} astfel nct G este izomorf cu un subgrup al lui GLk (R) (s-a notat
GLn (R) = {A Mn (R) | det A 6= 0}).
Ovidiu Munteanu, Brasov
Solutie. Vom spune c
a o matrice A are proprietatea (P ), dac
a pe fiecare linie,
respectiv coloan
a a sa exist
a cel mult un element nenul, si acesta egal cu 1. Cum
produsul a dou
a matrici cu proprietatea (P ) are proprietatea (P ) rezult
a c
a si X 2 ,
3
4
X , X , . . . au proprietatea (P ). Dar, cum exist
ar finit
de matrici de
a un num
a, deci
ordin n cu elemente 0 sau 1 rezult
a c
a multimea X, X 2 , X 3 , . . . este finit
k, h 1, k > h, astfel nct X k = X h . Acum, simplificnd n G, vom obtine
a E are proprietatea
X kh = E, unde E este elementul neutru n G. Avem deci c
(P ) si E 2 = E. Fie E = (eij )i,j=1,n ; s
a presupunem c
a i, j astfel ca eij = 1; rezult
a
P
c
a 1 =
eik ekj , deci s astfel ca eis = esj = 1. Dar, pe linia i a lui E nu se
k=1,n

afl
a dou
a elemente egale cu 1, deci s = j. Totodat
a, pe coloana j nu se afl
a dou
a
elemente egale cu 1, deci s = i. Rezult
a i = j, adic
a eventualele elemente nenule ale
lui E sunt pe diagonala principal
a. Evident E 6= On deoarece G este netrivial. Vom
presupune acum c
a elementele nenule ale lui E sunt ei1 j1 = 1, . . . , eir jr = 1. (1)
Fie A G oarecare; A = (aij )i,j=1,n . Din A = AE = EA rezult
a c
a
P
P
aij =
aik ekj =
eik akj . Folosind (1) va rezulta c
a singurele elemente nenule
k=1,n

k=1,n

a matricei
ale lui A pot fi din multimea {Ais jt }s,t=1,r . Fie aplicatia care asociaz
e Mr (R), A
e = (ais jt )
e = Ir si : G Mr (R)
A matricea A
.
Evident,
E
s,t=1,r
este morfism injectiv si unitar de monoizi. ntruct (G, ) este grup si este unitar,
rezult
a c
a (G) GLr (R). Prin urmare, G este izomorf cu (G), care este subgrup
al lui GLr (R).
[xn+1 , xn ]
L64. Fie sirul (xn )n1 definit prin: x1 , x2 N , xn+2 =
, n 1.
xn+1
Daca x2003 = 2004, demonstrati ca sirul nu este convergent.
Iuliana Georgescu si Paul Georgescu, Iasi
xn+1 xn

Solutie. Se observ
a c
a xn N , n 1. n plus, xn+2
= xn , deci
xn+1
(x2n )n1 si (x2n1 )n1 sunt monoton descresc
atoare si, cum sunt siruri de numere
naturale, sunt constante de la un loc ncolo, egale cu a, respectiv cu b. Astfel, pentru
[a, b]
, deci (a, b) = 1. S
a presupunem prin absurd c
a
n suficient de mare, b =
a
(xn )n1 este convergent; atunci a = b = 1. Fie i0 indicele primului termen din sir
care este egal cu 1; avem c
a i0 > 1 deoarece x1 x2003 > 1 si fie xi0 1 = a > 1.
[1, a]
[a, 1]
Avem c
a xi0 +1 =
= a, xi0 +2 =
= 1 si, prin inductie, xi0 +2k+1 = a,
1
a
a n contradictie cu convergenta sirului, fapt ce
xi0 +2k = 1, k 0. Acest fapt intr
ncheie rezolvarea.
L65. Fie n N si functiile f, g : R R, unde f (x) = x2n cos (1/x), x < 0,
f (0) = 0, f (x) = x2n sin (1/x), x > 0, iar g (x) = x2n+1 sin (1/x), x < 0,
69

g (0) = 0 si g (x) = x2n+1 cos (1/x), x > 0. Sa se afle cel mai nalt ordin de
derivabiliate al acestor functii si sa se studieze problema continuitatii acestor derivate
n origine.
Gheorghe Costovici, Iasi
Solutie. Se arat
a c
a f si g sunt derivabile de ordin n, f (n) este discontinu
a
a n origine. Aceste afirmatii decurg din urm
atorul
n origine si g (n) este continu
rezultat cunoscut (American Mathematical Monthly; 54(1947), p. 224 si 55(1948),
1
p. 97): functia h : R R data de h (0) = 0 si h (x) = xk sin , x 6= 0, este
x
derivabila de m ori daca k = 2m sau k = 2m + 1 si h(m) este discontinua sau
continua n x = 0 dupa cum k = 2m sau k = 2m + 1. Pentru demonstratia acestuia
se stabileste prin inductie complet
a c
a
1
1
h(r) (x) = Pr (x) sin + Qr (x) cos , x 6= 0, 0 r m,
x
x
atile:
unde Pr , Qr sunt functii polinomiale cu propriet
1) sunt de grad k 2r,
2) f (r) (0) = 0, 0 r m,
3) dac
a k = 2m, atunci sau Pm sau Qm are termen liber nenul,
4) dac
a k = 2m + 1, atunci Pm si Qm au termenii liberi nuli.

Truelul (r
aspuns la ntrebarea pus
a la p. 20)
S
a examin
am posibilit
atile lui X. Prima ar fi ca X s
a trag
a asupra lui Y . Dac
a
Y este ucis, atunci urm
atoarea lovitur
a revine lui Z. Dar Z nu mai are dect un
singur adversar si, cum Z este tr
ag
ator prefect, X va fi un om mort.
O optiune mai bun
a este s
a tinteasc
a asupra lui Z. Dac
a l doboar
a, atunci
urm
atoarea lovitur
a va reveni lui Y . Cum Y nimereste tinta de dou
a ori din trei,
exist
a o sans
a ca X s
a trag
a la rndul lui asupra lui Y si eventual s
a cstige truelul.
Exista ns
a o a treia optiune pe care o poate adopta X, mai bun
a dect precedentele: X poate s
a trag
a n aer. Va urma Y , care va tinti asupra lui Z, c
aci acesta
este adversarul cel mai periculos. Dac
a Z supravietueste, el va trage n Y , fiindc
a
acesta este adversarul mai periculos. Ca urmare, tr
agnd n aer, X i permite lui Y
s
a-l elimine pe Z si invers.
Aceasta este cea mai bun
a strategie a lui X: n cele din urm
a Y sau Z va muri si
X va trage asupra supravietuitorului, oricare ar fi el. X a manevrat astfel nct, n
loc de prima lovitur
a ntr-un truel, s
a aib
a prima lovitur
a ntr-un duel.

70

Probleme propuse1
Clasele primare
P.84. Aflati num
arul m stiind c
a 47 este mai mare dect m 14 cu 28.
(Clasa I )
nv. Maria Racu, Iasi
P.85. ntr-un cos sunt 6 mere, iar n altul sunt 5 pere. Cum pot primi 5 copii
mere si pere astfel nct nici un cos s
a nu r
amn
a gol?
(Clasa I )
Veronica Corbu, elev
a, Iasi
P.86. n urm
a cu 4 ani, cnd tat
al avea 29 de ani, s-a n
ascut fiul. Sora acestuia
avea atunci 2 ani, iar acum este de trei ori mai mare. Mama este de patru ori mai
mare dect aceasta. Cti ani are acum fiul?
(Clasa a II-a)
nv. Oana-Maria Lupu, Iasi
P.87. Un acrobat cade pe o plas
a elastic
a de la o anumit
a n
altime si se ridic
a
dup
a ce atinge plasa la jum
atatea distantei dintre plas
a si locul de unde a c
azut
anterior. S
tiind c
a atinge de 3 ori plasa si c
a ultima oar
a s-a ridicat la n
altimea de
2 m, iar plasa este montat
a la 2 m deasupra solului, s
a se afle distanta de la locul de
unde a c
azut prima dat
a pn
a la sol.
(Clasa a II-a)
Andrei Stativ
a, elev, Iasi
P.88. Tr
aiau odat
a o bab
a si un mosneag; mosul avea 100 ani, iar baba 90,
amndoi erau albi ca iarna si tristi ca vremea cea rea pentru c
a erau singuri. Se
spune c
a ar fi avut un copil pe cnd vrsta babei era jum
atate din jum
atatea de
acum a vrstei mosneagului si c
a acesta ar fi plecat n lume cnd vrsta mosneagului
era de dou
a ori ct vrsta aceea a babei. Fiul nu s-a mai ntors. Ce vrst
a avea fiul
cnd a plecat n lume?
(Clasa a III-a)
nv. Ileana Roscan, Iasi
P.89. La un concurs de biciclete, triciclete si masinute (cu patru roti), tat
al lui
Bogdan num
ar
a rotile vehiculelor si observ
a c
a sunt 34. Cte vehicule au fost de
fiecare fel? G
asiti toate posibilit
atile, stiind c
a num
arul vehiculelor de fiecare fel nu
dep
aseste 5.
(Clasa a III-a)
nv. Doinita Spnu, Iasi
P.90. Lungimea laturii unui p
atrat este de 17 m. O persoan
a pleac
a dintr-un
vrf al p
atratului si, mergnd n acelasi sens pe laturile acestuia, parcurge o distanta
de 637 m. Din punctul n care a ajuns se ntoarce si parcurge 773 m. Aflati la ce
distanta se va situa n final persoana fata de punctul de plecare.
(Clasa a III-a)
Oxana Pascal, elev
a, Iasi
P.91. Se mpart dou
a numere naturale. Dac
a mp
artitorul, ctul si restul sunt
trei numere consecutive cu suma 30, s
a se afle demp
artitul.
(Clasa a IV-a)
Vasile Solcanu, Bogd
anesti, Suceava
P.92. Observ
a regula si completeaz
a, apoi verific
a rezultatele g
asite: 2+4 = 3+3;
2 + 4 + 6 = 4 + 4 + 4; 2 + 4 + 6 + 8 = 5 + 5 + 5 + 5; 2 + 4 + 6 + + 12 = ;
2 + 4 + 6 + + 14 = ; . . . ; 2 + 4 + 6 + + (a + a) = .
(Clasa a IV-a)
Valeria Gheorghita
a, Iasi
, elev
1

Se primesc solutii pn
a la data de 31 decembrie 2005.

71

P.93. O foaie de hrtie dreptunghiular


a se ndoaie de-a lungul de 6 ori, formnduse 7 benzi egale si suprapuse. Dreptunghiul obtinut se ndoaie de-a latul de 9 ori,
rezultnd n final un p
atrat cu perimetrul de 12 cm. S
a se afle perimetrul dreptunghiului initial.
(Clasa a IV-a)
Petru Asaftei, Iasi

Clasa a V-a
V.56. Se consider
a num
arul A = 5 + 52 + 53 + + 52005 .
a) S
a se arate c
a A nu este p
atrat perfect.
b) S
a se g
aseasc
a 5 divizori mai mici dect 100 ai lui A.
Andrei Tofan, elev, Iasi
. . . 9} .
V.57. Aflati restul mp
artirii prin 47 a num
arului N = 1268 99
| {z
2005 cifre

Alexandru Negrescu, elev, Botosani


V.58. Aflati numerele naturale x, y, z cu proprietatea c
a
24x+1 + 23y+1 + 22z+1 = 9248.

Cristian - C
at
alin Budeanu, Iasi
2

a se arate c
a
V.59. Dac
a a1 a2 . . . an b1 b2 . . . bn = c1 c2 . . . cn 2 , s
2

a1 a2 . . . an a1 a2 . . . an 2 b1 b2 . . . bn b1 b2 . . . bn = c1 c2 . . . cn c1 c2 . . . cn 2 .

Petru Asaftei, Iasi


9

99

V.60. S
a se determine ultimele dou
a cifre ale num
arului 7 .
Artur B
al
auc
a, Botosani

Clasa a VI-a
VI.56. Determinati, n functie de num
arul ntreg x, cel mai mare divizor comun
al numerelor 2005x + 2 si 2006x + 3.
Tamara Culac, Iasi
VI.57. Un vnz
ator de autoturisme scade procentul beneficiului s
au de la 25% la
20% din valoarea vnz
arilor. Datorit
a sc
aderii preturilor, creste valoarea vnz
arilor.
Aflati procentul cu care a crescut valoarea vnz
arilor, stiind c
a beneficiul a crescut
cu 10%.
Marius Farcas, Iasi
VI.58. Se aseaz
a cifrele 2, 3, 4, 5, 6, 8, 9 ntr-o ordine oarecare obtinnd un
num
ar A. Se aseaz
a apoi aceleasi cifre n alt
a ordine, obtinnd num
arul B 6= A. S
a
se arate c
a A nu se divide cu B.
Cristian - C
at
alin Budeanu, Iasi
b = 120 . Dac
VI.59. Fie 4ABC cu m(B)
a mediana [BM] este perpendicular
a
pe BC, ar
atati c
a AB = 2BC.
Bogdan Posa, elev, Motru (Gorj)
VI.60. Fie 4ABC si punctele E (AB), F (AC) si M (BC) astfel nct
AE = EB, iar ntre 4AEF si 4EF M s
a existe o congruenta. S
a se arate c
a:
a) F este mijlocul lui [AC];
b) [AM] este median
a sau n
altime.
Ioan S
ac
aleanu, Hrl
au
72

Clasa a VII-a
VII.56. Fie x, y R cu x2 2y = y 2 + xy = 4. S
a se arate c
a x2 2x = y 2 .
Gigel Buth, Satu Mare
VII.57. Fie x (0, 1), iar n N \ {0, 1}. Ar
atati c
a nx2 + 2n > n + (1 + x)n .
Ion Visan, Craiova
VII.58. Fie a, b R+ astfel nct mediile aritmetic
a, geometric
a si armonic
a ale
lor s
a fie laturi ale unui triunghi dreptunghic. Aflati sinusul celui mai mic dintre
unghiurile triunghiului.
Romanta Ghita
si Ioan Ghita
, Blaj
VII.59. Fie 4ABC si A0 mijlocul lui [BC]. Dac
a D (AC), BD AA0 = {F }
1
1
1
1
si paralela prin F la BC taie AC n E, s
a se arate c
a
=
+
+
.
DE
AD CE
AC
Claudiu - S
tefan Popa, Iasi
VII.60. n cercul C se consider
a coardele [AM] si [AN ]astfel nct AM < AN .
1) S
a se determine multimea punctelor X C ce ndeplinesc conditia AM
AX AN .
2) n ce caz multimea g
asit
a este un arc de cerc?
Temistocle Brsan, Iasi

Clasa a VIII-a
VIII.56. S
a se determine x, y, z R pentru care x3 y 3 + z 3 = 8, x y + z = 2.
Andrei - Sorin Cozma, elev, Iasi
VIII.57. Fie a, b, c > 0 astfel nct a + b + c = 1. S
a se arate c
a
1
1
9
1
p
+p
+p
.
2
(1 a) (1 b)
(1 b) (1 c)
(1 c) (1 a)

Cristian S
avescu, elev, Focsani

VIII.58. Fie x, y, z (0, ) cu x + y + z 3. S


a se arate c
a xn + y n + z n 3,
n N.
Romeo Ilie, Brasov
4
VIII.59. Determinati x, y, z R, stiind c
a x+y+z = 1, iar xy+(x + y)(z + 1) = .
3
Gheorghe Molea, Curtea de Arges

4 3
0 0 0
VIII.60. Se consider
a prisma triunghiular
a regulat
a ABCA B C cu AB =
3


0
si AA = 3 3. S
a se arate c
a pentru fiecare num
ar a 0, 3 3 , exist
a exact dou
a
puncte Ma0 , Ma00 pe dreapta CC 0 astfel nct d (B 0 , (Ma0 AB)) = d (B 0 , (Ma00 AB)) = a.
Mirela Marin, Iasi

Clasa a IX-a
IX.56. Determinati numerele reale pozitive x, y, z, t pentru care x+y+z+t = 20,
iar xy + xz + xt + yz + yt + zt + 475 = xyzt.
Lucian Tutescu si Liviu Smarandache, Craiova
IX.57. S
a se determine toate functiile f : R R cu proprietatea
73

f (f (f (x) + y) f (x f (y))) = x2 y 2 ,

x, y R.

Adrian Zahariuc, elev, Bac


au

IX.58. Fie n N, n 2. Determinati a1 , a2 , . . . , an1 , c R pentru care


[x] + [x + a1 ] + + [x + an1 ] = [cx] ,

x R.

(n legatura cu G.42 din RecMat 1/2003.)


Iuliana Georgescu si Paul Georgescu, Iasi
IX.59. Fie C (I, r) cercul nscris n 4ABC. S
a se arate c
a
IA IB + IB IC + IC IA 12r2 .

D. M. B
atinetu - Giurgiu, Bucuresti

IX.60. Fie ABC un triunghi ascutitunghic cu a < b < c. Cercul C (I, r) nscris
triunghiului este tangent dreptelor BC, CA si AB n punctele D, E si respectiv F .
a AB n E 0 si BC
Dreapta ID intersecteaz
a CA n D0 si AB n D00 , IE intersecteaz
00
0
00
0 00
a BC n F si CA n F . Ar
atati c
a E E = D0 D00 + F 0 F 00 .
n E , iar IF intersecteaz
Temistocle Brsan, Iasi

Clasa a X-a
X.56. Fie tetraedrul ABCD si M un punct n spatiu. Dac
a G, GA , GB , GC ,
GD sunt centrele de greutate ale tetraedrelor ABCD, M BCD, M ACD, MABD,

respectiv MABC, s
a se arate c
a AGA + BGB + CGC + DGD = 0 dac
a si numai
dac
a M G.
Marius Olteanu, Rmnicu Vlcea
X.57. Dac
a x, y, a (1, ), s
a se arate c
a

x+y
(x + y + loga x) xy + loga x
+ (x + y + loga y) xy + loga y x+y

(x + xy + y) loga xy.

Mihail Bencze, Brasov

X.58. Let n N, n 1. Prove that

n 2
X
k2 + n2
n
2n 2
ln
<
.
k
n1
n2
k=1

Jos Luis Daz - Barrero, Barcelona, Spain

X.59. Fie f R [X] un polinom de grad n 3 ce admite n r


ad
acini reale,
pozitive si subunitare. Dac
a |f (0)| = f (1), s
a se arate c
a produsul r
ad
acinilor este
1
cel mult egal cu n .
2
Ioan S
erdean, Or
astie
X.60. Fie k N fixat. Alegem n {1, 2, . . . , k} si a1 , a2 , . . . , an numere prime
a se divid
a cu 24 este
mai mari dect 3. Dac
a probabilitatea ca a21 + a22 + + a2n s
1
cel putin egal
a cu
, s
a se arate c
a 24 divide k.
24
Cristian S
avescu, elev, Focsani

Clasa a XI-a
XI.56. Let n be a positive integer. For each positive integer k, let Fk be the kth
74

Fibonacci number (F1 = F2 = 1, Fk+2 = Fk+1 + Fk for all k 1). Prove that

2
2

Fn2
Fn+1
2Fn Fn+1 Fn+2

2
2
= 0.

Fn+2
2Fn+1 Fn+2 Fn2
Fn+1

2
2
2

2Fn Fn+2 Fn+1


Fn+2
Fn

Jos Luis Daz - Barrero, Barcelona, Spain

XI.57. Fie p
atratul ABCD circumscris cercului C. n p
atrat se nscrie octogonul EF GHIJKL, circumscris cercului C, astfel nct E, F (AB), E (AF ),
G, H (BC), G (BH), I, J (CD), I (CJ), K, L (DA), K (DL). Fie
{M} = EL AC, {N } = F G BD, {P } = HI AC, {Q} = JK BD. S
a se arate
c
a suma
AE BF BG CH CI DJ DK AL AM BN CP DQ
S=
+
+
+
+
+
+
+
+
+
+
+
EB F A GC HC ID JC KA LD MC N D P A QB

nu depinde de alegerea vrfurilor octogonului pe laturile p


atratului.
C
at
alin Calistru, Iasi
XI.58. Dac
a n este un num
ar natural iar p un num
ar prim, atunci sirul
a exponentul cu care
(xn+1 (p) xn (p))n0 este divergent, unde xn (p) reprezint
apare num
arul p n descompunerea lui n!.
Sorin Puspan
a, Craiova
XI.59. Fie sirul de numere supraunitare (an )n1 , astfel ca lim an = 1. S
a se
n

studieze continuitatea functiei f : R R, f (x) = lim {xan }, unde {x} reprezint


a
n
partea fractionar
a a num
arului real x.
Dan Popescu, Suceava
2
XI.60. Fie a (0, 1); s
a se demonstreze c
a pentru orice x >
este valabil
a
ln
a
1

1
inegalitatea (1 ax ) x+1 < 1 ax+1 x .
Angela Tig
aeru, Suceava

Clasa a XII-a

XII.56. Fie Sn multimea permut


arilor de ordin n, iar Sn . Se consider
a
functia m
arginit
a f : R R. S
a se calculeze:

1
1
1
f ( (1)) + f ( (2)) + + f ( (n)) .
lim
n n
2
n
(O generalizare a problemei 24131, G. M. 5-6/1999.)
Marius Olteanu, Rmnicu Vlcea

1 1
XII.57. Consider
am matricea A =
si multimea
1 1

1
G = Xa | Xa = I2 + aA, a ,
.
2

Ar
atati c
a (G, ) este grup izomorf cu (R, +). Calculati X 12 X 32 X 2n1 , n N .
2
Gheorghe Iurea, Iasi
XII.58. Let f : R [1, 1] be a continuous function. Prove that
75

Z
3

Z
q
1 (f (x))2 dx +

f (x) dx 4.

Zdravko Starc, Vrac, Serbia and Montenegro


XII.59. Fie f : R R, f derivabil
a si neconstant
a pe nici un interval al lui R.
Dac
a
f 0 (x)
f 0 (sin x) cos x + f 0 (cos x) sin x, x R,
1 + f 2 (x)
demonstrati c
a nu exist
a lim f (x).
x
Paul Georgescu si Gabriel Popa, Iasi
a
XII.60. Fie n > 1 si a1 , a2 , . . . , an (0, 1) astfel nct a1 + a2 + + an = 1. S
f (x)
= 1 si f (x) = f (a1 x) + f (a2 x) +
se determine functiile f : R R, dac
a lim
x0 x
+ f (an x) pentru orice x R.
Gabriel Dospinescu, Paris

IMPORTANT
n scopul unei leg
aturi rapide cu redactia revistei, pot fi utilizate urm
atoarele adrese e-mail: tbi@math.tuiasi.ro, vpgeo@lycos.co.uk,
profgpopa@yahoo.co.uk . Pe aceast
a cale colaboratorii pot purta cu
redactia un dialog privitor la materialele trimise acesteia, procurarea
numerelor revistei etc.
La problemele de tip L se primesc solutii de la orice iubitor de matematici
elementare (indiferent de preocupare profesionala sau vrsta ). Fiecare
dintre solutiile acestor probleme - ce sunt publicate n revist
a dup
a un
an - va fi urmat
a de numele tuturor celor care au rezolvat-o.
Lucr
arile originale ale elevilor vor fi publicate ntr-o rubric
a special
a
dedicat
a acestora: NOTA ELEVULUI . Anual, se vor acorda elevilor
- autori dou
a premii n bani, pentru cele mai bune note publicate n
revist
a.
Adres
am cu insistenta
amintea ca materialele trimise re rug
vistei s
a nu fie (s
a nu fi fost) trimise si altor publicatii.

76

Probleme pentru preg


atirea concursurilor1
A. Nivel gimnazial

2
x y = u2
y2 z = v2 .
G76. Rezolvati n multimea numerelor naturale sistemul
2
z x = t2
Adrian Zanoschi, Iasi
2
2
a
b
G77. i) Fie a, b, c R cu a > b > c; atunci
+
> a + 2b + c.
ab bc
a2 b2 c2 b2 a2 c2
+
+
3a4b+c.
ii) Fie a, b, c R cu a b c > 0; atunci
c
a
b
Ioan S
erdean, Or
astie

G78. Dac
a a, b, c, d (0, ), s
a se demonstreze inegalitatea

b (a + c) c (b + d) d (a + c) a (b + d)
+
+
+
4.
c (a + b) d (b + c) a (d + c) b (a + d)
Artur B
al
auc
a, Botosani

G79. Dac
a x, y, z (0, ) sunt astfel nct x + y + z = xyz, atunci
p
p
p
xy + yz + zx 3 + x2 + 1 + y 2 + 1 + z 2 + 1.

Florina Crlan si Marian Tetiva, Brlad

G80. Fie A multimea tuturor sumelor de tipul 12 32 52 (2n + 1)2 ,


n N, unde semnele pot fi alese n orice combinatie posibil
a. S
a se arate c
a A = Z.
(n legatura cu teorema Erds-Surnyi.)
Petru Asaftei, Iasi
G81. Fie n N si k {0, 1, . . . , 2n 1}. S
a se arate c
a exist
a o multime A R
cu n elemente care are exact k submultimi cu suma elementelor strict pozitiv
a.
Adrian Zahariuc, elev, Bac
au
G82. Un cal se afl
a pe tabla de sah n cmpul A1 si dorim s
a-l ducem pe pozitia
H8 ntr-un num
ar minim de s
arituri. Aflati care este acest num
ar minim, precum si
cte trasee de lungime minim
a exist
a.
Gheorghe Cr
aciun, Plopeni si Gabriel Popa, Iasi
G83. Fie ABCD patrulater convex si punctele M, N (AB), P, R (CD) astfel
BM N A P R CD
nct AD BC M R N P 6= . S
a se arate c
a

= 1.
M N DP RC AB
Andrei-Sorin Cozma, elev, Iasi
G84. Fie ABCD un trapez cu ABkCD, AB < CD. Se consider
a punctele
CF
AE
=
. Dreapta EF intersectez
a BD si AC
E (AD) si F (BC) astfel nct
ED
FB
DC AB
MN
=
.
n M, respectiv N . S
a se arate c
a
EF
DC + AB
Andrei Nedelcu, Iasi
1

Se primesc solutii pn
a la data de 31 decembrie 2005.

77

G85. Fie A0 , B 0 , C 0 picioarele bisectoarelor unghiurilor 4ABC. Pe latura (BC)


consider
am punctele D si E astfel nct D (BE) si cevienele AD si AE sunt
izogonale. S
a se demonstreze c
a DB 0 si EC 0 se intersectez
a pe AA0 . (n legatura cu
Propozitia 1, p. 99, RecMat - 2/2004.)
Titu Zvonaru, Com
anesti

B. Nivel liceal
L76. Fie cercurile C1 si C2 tangente interior unui cerc C n punctele distincte M ,
respectiv N . Cercurile C1 si C2 sunt secante sau tangente exterior iar axa radical
aa
cercurilor C1 si C2 taie cercul C n A si B. Dreptele AM si AN taie din nou cercurile
C1 si C2 n K, respectiv L. Ar
atati c
a AB 2KL. n ce caz avem egalitate?
Neculai Roman, Mircesti (Iasi)
L77. Fie punctele P1 , P2 , . . . , P13 n plan astfel nct oricare trei sunt necoliniare
si toate au coordonate ntregi. S
a se arate c
a exist
a cel putin un triunghi Pi Pj Pk
astfel nct centrul s
au de greutate s
a aib
a coordonate ntregi.
Vasile Prav
at si Titu Zvonaru, Com
anesti (Bac
au)
L78. Consider
am sirul de puncte (Pn )nN pe cercul trigonometric astfel nct
5
m(Pn\
pentru orice n N, Pn\
OPn+1 ) = arctg
OPn+1 fiind considerat ca unghi
12
orientat. S
a se arate c
aj N
a pentruorice punct P pe cercul trigonometric exist
1
astfel nct Pj Int C P,
.
2005
Lucian - Georges L
adunc
a si Andrei Nedelcu, Iasi
L79. Fie a1 , a2 , . . . , an R n asa fel nct a1 + a2 + + an = 0 si
1 hni hn + 1i
a a21 +a22 + +a2n
max {|ai aj | ; 1 i < j n} 1. Demonstrati c
n 2
2
si precizati n ce caz are loc egalitate.
Marius Pachitariu, elev, Iasi
L80. Fie un alfabet cu 4 litere a, b, c, d. n acest alfabet se pot forma cuvinte
dup
a urm
atoarele reguli: dup
a a nu poate urma b, dup
a b nu poate urma c, dup
ac
nu poate urma d si dup
a d nu poate urma a. Cte cuvinte palindromice de lungime
n, n 2, se pot forma conform acestor reguli? (Prin cuvnt palindromic ntelegem
un cuvnt n care litera de pe pozitia k coincide cu litera de pe pozitia n k + 1,
pentru orice k {1, 2, . . . , n}.)
Irina Mustata
a, Iasi
, elev
L81. Fie n 1 un num
ar natural fixat. O tabl
a infinit
a de sah este colorat
a
n alb si negru n maniera obisnuit
a. O multime C de c
asute ale tablei se numeste
conexa dac
a putem ajunge din fiecare c
asuta a lui C n fiecare alt
a c
asuta a lui C
printr-o succesiune de deplas
ari n C dintr-o c
asuta ntr-o c
asuta vecin
a (cu o latur
a
comun
a). Fie S o multime conex
a cu 4n c
asute. Numim raportul cromatic al multimii
S raportul dintre num
arul de c
asute albe si num
arul de c
asute negre din S. S
a se
afle cea mai mic
a si cea mai mare valoare posibil
a a raportului cromatic.
Adrian Zahariuc, elev, Bac
au
L82. Determinati P, Q R [X] pentru care f : R R, f (x) = {p (x) + sin q (x)}
este periodic
a, unde p, q : R R sunt functiile polinomiale asociate lui P , respec78

tiv Q.
Paul Georgescu si Gabriel Popa, Iasi
L83. S
a se calculeze
"
#
1
2
n

1 2
1 3
1 n+1
lim
1+
+ 1+
+ + 1 +
n .
n
n
n
n
Marius Olteanu, Rmnicu Vlcea
L84. Fie n N, n 3 si
n

n
A = x > 0; x = a0 + a1 n n + + an1 nn1 ;
Determinati inf A.

o
a0 , a1 , . . . , an Z; n 1 | a0 + a1 + + an .

Paul Georgescu si Gabriel Popa, Iasi


L85. Fie f : R R o functie pentru care multimea punctelor n care f are limit
a
finit
a la stnga este dens
a n R. S
a se arate c
a multimea punctelor n care f este
continu
a este de asemenea dens
a n R. (O multime D R se numeste densa n R
dac
a orice interval deschis al axei reale contine m
acar un element din D.)
Gabriel Dospinescu, Paris, si Marian Tetiva, Brlad

Training problems for mathematical contests


A. Junior high school
level

x2 y = u2
y2 z = v 2 in the set of natural numbers.
G76. Solve the system
2
z x = t2
Adrian Zanoschi, Iasi
2
2
a
b
G77. i) Prove that
+
> a + 2b + c for any a, b, c R, a > b > c.
ab bc
a2 b2
c2 b2
a2 c2
ii) Prove that
+
+
3a 4b + c for any a, b, c R,
c
a
b
a b c > 0.
Ioan S
erdean, Or
astie
G78. Prove that
b (a + c) c (b + d) d (a + c) a (b + d)
+
+
+
4
c (a + b) d (b + c) a (d + c) b (a + d)
for any a, b, c, d (0, ).
Artur B
al
auc
a, Botosani
G79. Prove that
p
p
p
xy + yz + zx 3 + x2 + 1 + y2 + 1 + z 2 + 1
for any x, y, z (0, ) such that x + y + z = xyz.
Florina Crlan and Marian Tetiva, Brlad
79

G80. Let A be the set of all sums of type 12 32 52 (2n + 1)2 , n N,


for any combination of the signs. Prove that A = Z. (Regarding Erds-Surnyi
theorem.)
Petru Asaftei, Iasi
G81. Let n N and k {0, 1, . . . , 2n 1}. Prove that there exists a set A R
with n elements which has precisely k subsets whose sum of elements are strictly
positive.
Adrian Zahariuc, high school student, Bac
au
G82. Find minimal number of moves required to transfer a knight on a chessboard
from the square A1 to the square H8. For this minimal number, find the number of
distinct paths of minimal length.
Gheorghe Cr
aciun, Plopeni, and Gabriel Popa, Iasi
G83. Let ABCD be a convex quadrilateral and let M, N (AB), P, R (CD)
BM N A P R CD
such that AD BC MR N P 6= . Prove that

= 1.
MN DP RC AB
Andrei-Sorin Cozma, junior high school student, Iasi
G84. Let ABCD be a trapezoid with ABkCD, AB < CD. Consider E (AD)
AE
CF
and F (BC) such that
=
. The line EF meets BD and AC in M ,
ED
FB
DC AB
MN
=
.
respectively in N . Prove that
EF
DC + AB
Andrei Nedelcu, Iasi
G85. Let ABC be a given triangle and let A0 , B 0 , C 0 be the legs of its bisectors.
Let D, E be points on the side (BC) such that D (BE) and the cevians AD and
AE are isogonal. Prove that DB 0 and EC 0 intersect each other in a point situated
on AA0 . (Regarding Proposition 1, p. 99, RecMat - 2/2004.)
Titu Zvonaru, Com
anesti

B. High school level


L76. Let C1 , C2 be circles which are internally tangent to a given circle C in M ,
respectively in N, M 6= N . Suppose that C1 and C2 are secant or externally tangent
and that the radical axis of C1 and C2 meets C in A and B. Let us denote by K,
respectively by L, the points in which the lines AM, AN meet again C1 , respectively
C2 . Prove that AB 2KL and characterize the case of equality.
Neculai Roman, Mircesti (Iasi)
L77. Let P1 , P2 , . . . , P13 points with integer coordinates in a plane such that
any given three are not collinear. Prove that there exists at least a triangle Pi Pj Pk
such that its centroid has integer coordinates.
Vasile Prav
at and Titu Zvonaru, Com
anesti (Bac
au)
Consider (Pn )nN a sequence of points on the unit circle such that
5
m(Pn\
for all n N, Pn\
OPn+1 ) = arctg
OPn+1 being considered as an oriented
12
angle. Prove that for any point P on the unit circle there exists j N such that
L78.

80


Pj Int C P,

1
.
2005

Lucian L
adunc
a and Andrei Nedelcu, Iasi
L79. Let a1 , a2 , . . . , an R be such that a1 + a2 + + an = 0 and
1 hni hn + 1i
max {|ai aj | ; 1 i < j n} 1. Prove that a21 + a22 + + a2n
n 2
2
and characterize the case of equality.
Marius Pachitariu, high school student, Iasi
L80. Consider an alphabet with four letters a, b, c, d. Using this alphabet, one
can construct words according to the following rules: b cannot succeed a, c cannot
succeed b, d cannot succeed c and a cannot succeed d. How many palyndromes of
length n, n 2, can one construct in this manner? (By palyndrome we mean a word
in which the k-th letter coincides with the n k + 1-th letter for all k {1, 2, . . . , n}.)
Irina Mustata
, high school student, Iasi
L81. Let n 1 be a fixed natural number. An infinite chessboard is colored in
black and white in the usual manner. A set C of squares is then called connected
set if one can reach any square in C starting from any given square in C through
a succession of moves in C from a square to a neighboring square (with a common
edge).
Let S be a connected set with 4n squares. One calls the chromatic index of S
the quotient between the number of white squares and the number of black squares.
Find the maximal and the minimal value of the chromatic index.
Adrian Zahariuc, high school student, Bac
au
L82. Find P, Q R [X] such that f : R R, f (x) = {p (x) + sin q (x)} is
periodic where p, q : R R are the polynomial functions associated to P , respectively
to Q.
Paul Georgescu and Gabriel Popa, Iasi
L83. Find
"
#
1
2
n

1 2
1 3
1 n+1
1+
+ 1+
+ + 1 +
n .
lim
n
n
n
n
Marius Olteanu, Rmnicu Vlcea
L84. Let n N, n 3 and
n

n
A = x > 0; x = a0 + a1 n n + + an1 nn1 ;
o
a0 , a1 , . . . , an Z; n 1 | a0 + a1 + + an .

Find inf A.

Paul Georgescu and Gabriel Popa, Iasi


L85. Let f : R R a function such that the set of points in which f has finite
left-sided limit is dense in R. Prove that the set of continuity points of f is also dense
in R. (A subset D of R is called dense in R if any open interval of R contains at least
an element of D.)
Gabriel Dospinescu, Paris, and Marian Tetiva, Brlad
81

Pagina rezolvitorilor
BRA
SOV
Dan: VIII(52-54), IX(52,53);
Liceul "M. Titulescu". Clasa a IX-a. BARDA
BOISTEANU Claudia: VIII(52-54), IX(53,54); BUTNARIU Anda: VIII(52-54),
IX(52-55); CHIRA Roxana: VIII(52-54), IX(52-55); COSTA Larisa: VIII(52-54),
SOAE Emanuela: VIII(52-54), IX(53,54); GIRIGAN Andreea: VIII
IX(52,53); ENA
AU
Cristi(52-54), IX(53,54); MZGACIU Alexandra: VIII(52-54), IX(53,54); NAN

na: VIII(52-54), IX(53,54); NEGOESCU Anamaria: VIII(52,53), IX(52-54), NILA


Iulian: VIII(52-54), IX(52,53,55); NUTU
Cosmin: VIII(52-54), IX(52-55); OBANCEA Dragos: VIII(52-54), IX(52,55); POTEC Monica: VIII(52-54), IX(53,54);
SZCS Daniel: VIII(52-54), IX(52-54); S
CHIOPU Iulian: VIII(52-54), IX(52,53);
ZBARCEA Adrian: VIII(52-54), IX(52,53).
CRAIOVA
Scoala

nr. 22 "M. Eliade". Clasa a V-a. STANCIU Ioan: P(74-76,78-80,82,83),


V(51,52).

HRLAU
Scoala

"P. Rares". Clasa a II-a (nv. BUDACEA Maria). NEICU Mara: P(6467,72,74-79).
Liceul "Stefan

cel Mare". Clasa a VII-a. CIOFU Alexandra: V(46-49), VI.46;


MANOLIE Ioan: V(46-49), VI.46; SAVA Cristina: V(46-49), VI.48; SCRIPCARU
Gabriela: V(46-49), VII.48; SPIRIDON Florin: V(46-49), VII.48; ZAMFIR CarmenAdela: V(46-49), VI.48. Clasa a IX-a. ONOFREI Andrei-Codrut: VII(46-48),
VIII.47, IX(46,47,49), L.56; ROTARU Lucian: VII(48-50), VIII.46, IX(46-49), L.56;
SPNU Cosmin-Alexandru: VII(46-48), VIII.49, IX.47, L.56.
IA
SI
Scoala

nr. 4 "I. Teodoreanu". Clasa a III-a (nv. BUJOR Lorena). LUPAN


Dana: P(74-80,82,83).
Scoala

nr. 7 "N. Tonitza". Clasa a III-a (nv. TUDOSE Elena). ANTON Iuliana: P(74-79); LEONTE Anca: P(74-79); NECHITA Paula: P(74-79); POS Simona-Alexandra: P(74-79); SAVIN R
TICA
azvan: P(74-79). Clasa a III-a

(nv. MELINTE Rodica). BOTOSANU Bianca-Mihaela: P(74-76,78,79); CEUCA


D
anut-Vasilic
a: P(74-76,78, 79); CONSTANTINESCU Diana-Gabriela: P(74-76,78,
79); GUSOVATE Diana-Stefana: P(74-76,78,79); LEOGAN Larisa-Diana: P(7476,78,79); SUCIUC Raluca: P(74-76,78,79).
Scoala

nr. 22 "B. P. Hasdeu". Clasa a III-a (nv. DOHOTARU Liliana). TURCU


Andrei-Daniel: P(74-79); CRUCEANU Evangeline Tamara: P(74-76,78,79). Clasa
ALI
TEI
a III-a (nv. TRZIORU Iuliana). ADASC

Victor: P(74-79,82,83); APOSTOL Ana-Maria: P(74-78); ESANU Georgiana: P(74-78); GREIEROSU Claudiu:
ATIC

P(74-78); GNDU Alexandra-Livia: P(74-76,78,79); LAM


Ioana: P(72-79);
REBEGEA Andrada Elena: P(74-79); UNGUREANU Teofana: P(74-76,78,79).
Laura). ANDRONICIUC Ana-Miruna: P(74-76,78,
Clasa a III-a (nv. CHIRILA
Sebastian-Andrei: P(74-76,78,79);
79); BOARU Adrian: P(74-76;78,79); BURUIANA
82

BUHU Vlad: P(74-76,78,79); CEOBANU Andrei-Nicolae: P(74-76,78,80); CHICHI Alexandra-Elena: P(74-76,78,79); COSTACHESCU

RAU
Ivona: P(74-76,78,79); DIACONESCU Matei: P(74-76,78,79); GHERAN Ana-Maria: P(74-76,78,79); GELIP
Ioana: P(74-76,78,79); HATESCU Iustina: P(74-76,78,79,83); HORBOVANU BiancaAlexandra: P(74-76,78,79).
Scoala

nr. 26 "G. Cosbuc". Clasa a III-a (nv. BUCATARIU Rica). DUMITRU


Ambra-Georgiana: P(64-68,74-76); IACOB Robert-Ionut: P(68,69,74-76,78,79);
IVANCIUC Dumitru-Florin: P(68,71,75,76,78); HRISCU Alexandru: P(64,68,76 Alexandru: P(64,65,68,
79); MOISA Adrian-Bogdan: P(68-70,76-79); MUSTEAT
A
74-79); SANDU Ioana-Luiza: P(68,74-78); SCUTARU Ionela-Cristina: P(68,70,74 Georgiana: P(64,66,
76,78,79); STOICA Diana-Maria: P(64,65,67,68,76); ZALINCA
69,74,78,79). Clasa a IV-a (nv. RACU Maria). BULGARU Ionela-Alexandra:

P(74-79); BURLACU S
tefan-Claudiu: P(74-79); CALIN
Andreea-Georgiana: P(74

79,82,83); IFROSA Adriana: P(74-78); IOJA Petru-Alexandru: P(74-78); MOISA

Bogdan-Gabriel: P(74-79,82,83); PINTILIE R


azvan-Florin: P(74-79); RAZLOG
Ionut: P(74-78). Clasa a IV-a (nv. GALIA Paraschiva). ALUPEI Andra-M
ad
alina:
Oana-C
Marius-C
P(74-79); CIOABA
at
alina: P(74-79); GHERCA
at
alin: P(74-79);
HOMEA Liviu: P(74-79); HUIDES Gina: P(74-79); MANOLIU M
ad
alina: P(74-79);

Alexandru: P(74-79); SCUTARU ConMIHAILESCU


Laura: P(74-79); PISICA
stantin: P(74-79); SEGNEANU Elena: P(74-79); ZDREVIT
Maria: P(74-79).
Liceul "M. Eminescu". Clasa a V-a. DUCA M
ad
alina: P(71-73,82,83), V(46,51);
IACOB Ioana: P(71,73,81-83), V(46,51); POPA Alexandra: P(71,73,81-83), V(46,51).
Clasa a VI-a. COHAL C
alin: V(51,53-55), VI.51.
Liceul "G. Ibraileanu". Clasa a VIII-a. UNGUREANU Dragos: VI(52,55),
VII(54,55), VIII.52.
Colegiul National "C. Negruzzi". Clasa a VI-a. OLARIU Tudor: P(71-73),
V(46-50,52-55), VI(46,47), G.66; TIBA Marius: V.48, VI(46-48,50), VIII(46,48).
Colegiul National "E. Racovita". Clasa a V-a. TUDORACHE AlexandruGabriel: P(80-83), V(51-55).
Colegiul National Iasi. Clasa a V-a. BACUSCA Alberto: P(71,73,81-83), V.46;
A
TN
Roxana-Maria: P(71-73,80-83), V(46-49); CEUCA
R
CAP

A
azvan: P(71,72,
Ivona:
82,83), V(46,49); CHELARU Adrian: P(71, 72,80-82); V(46,47); CURCA
P(71,72,81), V(46,47); MOCANU Dan: P(71-73,80-83), V(46,47,49,51); NEDELCU
Adina-Ioana: P(71-73,80-83), V(46,47,49,50); OROIAN Bianca: P(71,73,80), V(46,
47,49). Clasa a VI-a. CADAR Alexandra: V(46-48,51-53), VI(51,55). Clasa a
X-a. TIMOFTE Diana: VIII(46,47), IX(46,47,49,50), X(46,47).

83

Premii acordate rezolvitorilor


ASOCIATIA

"RECREATII

MATEMATICE" n colaborare cu redactia


revistei RECREATII
MATEMATICE acord
a cte o diplom
a si un premiu n
c
arti pentru trei aparitii la rubrica "Pagina rezolvitorilor" elevilor urm
atori:
Liceul "M. Eminescu"
AVRAM Mircea (cl. a IX-a): 1/2002 (7pb), 1/2003 (10pb), 2/2004 (5pb).
Scoala

nr. 7 "N. Tonitza"


SAVIN R
azvan (cl. a III-a): 1/2004 (5pb), 2/2004 (7pb), 1/2005 (6pb).
Scoala

nr. 22 "B. P. Hasdeu"


ANDRONICIUC Ana-Miruna (cl. a III-a): 1/2004 (6pb), 2/2004 (5pb), 1/2005
(5pb);
APOSTOL Ana-Maria (cl. a III-a): 1/2004 (6pb), 2/2004 (7pb), 1/2005 (5pb);
Sebastian-Andrei (cl. a III-a): 1/2004 (6pb), 2/2004 (6pb), 1/2005
BURUIANA
(5pb);
CEOBANU Andrei-Nicolae (cl. a III-a): 1/2004 (5pb), 2/2004 (5pb), 1/2005
(5pb);

COSTACHESCU
Ivona (cl. a III-a): 1/2004 (5pb), 2/2004 (6pb), 1/2005 (5pb);
GNDU Alexandra-Livia (cl. a III-a): 1/2004 (6pb), 2/2004 (6pb), 1/2005
(5pb);
GHERAN Ana-Maria (cl. a III-a): 1/2004 (5pb), 2/2004 (5pb), 1/2005 (5pb);
HORBOVANU Bianca-Alexandra (cl. a III-a): 1/2004 (5pb), 2/2004 (6pb),
1/2005 (5pb);
TURCU Andrei-Daniel (cl. a III-a): 1/2004 (6pb), 2/2004 (5pb), 1/2005 (6pb);
UNGUREANU Teofana (cl. a III-a): 1/2004 (6pb), 2/2004 (5pb), 1/2005 (5pb).
Scoala

nr. 26 "G. Cosbuc"


MANOLIU M
ad
alina (cl. a IV-a): 1/2004 (5pb), 2/2004 (5pb), 1/2005 (6pb).

84

Revista semestrial RECREAII MATEMATICE este editat de


ASOCIAIA RECREAII MATEMATICE. Apare la datele de 1 martie
i 1 septembrie i se adreseaz elevilor, profesorilor, studenilor i tuturor celor
pasionai de matematica elementar.
n atenia tuturor colaboratorilor
Materialele trimise redaciei spre publicare (note i articole, chestiuni de
metodic, probleme propuse etc.) trebuie prezentate ngrijit, clar i concis; ele
trebuie s prezinte interes pentru un cerc ct mai larg de cititori. Se recomand
ca textele s nu depeasc patru pagini. Evident, ele trebuie s fie originale
i s nu fi aprut sau s fi fost trimise spre publicare altor reviste.
Problemele destinate rubricilor: Probleme propuse i Probleme pentru
pregtirea concursurilor vor fi redactate pe foi separate cu enun i
demonstraie/rezolvare (cte una pe fiecare foaie) i vor fi nsoite de numele
autorului, coala i localitatea unde lucreaz/nva.
Redacia va decide asupra oportunitii publicrii materialelor primite.
n atenia elevilor
Numele elevilor ce vor trimite redaciei soluii corecte la problemele din
rubricile de Probleme propuse i Probleme pentru pregatirea concursurilor
vor fi menionate n Pagina rezolvitorilor. Se va ine seama de regulile:
1. Pot trimite soluii la minimum cinci probleme propuse n numrul
prezent i cel anterior al revistei; pe o foaie va fi redactat soluia unei
singure probleme.
2. Elevii din clasele VI-XII au dreptul s trimit soluii la problemele
propuse pentru clasa lor, pentru orice clas mai mare, din dou clase mai mici i
imediat anterioare. Elevii din clasa a V-a pot trimite soluii la problemele
propuse pentru clasele a IV-a, a V-a i orice clas mai mare, iar elevii claselor
I-IV pot trimite soluii la problemele propuse pentru oricare din clasele primare
i orice clas mai mare. Orice elev poate trimite soluii la problemele de
concurs (de tip G i L).
3. Vor fi menionate urmtoarele date personale: numele i prenumele,
clasa, coala i localitatea.
4. Plicul cu probleme rezolvate se va trimite prin pot (sau va fi adus
direct) la adresa Redaciei:
Prof. dr. Temistocle Brsan
Str. Aurora, nr. 3, sc. D, ap. 6,
700 474, Iai
Jud. IAI
E-mail: tbi@math.tuiasi.ro

CUPRINS
200 de ani de la naterea lui DIRICHLET............................................................ 1

ARTICOLE I NOTE
t. FRUNZ, I. FRUNZ Fractali (II)..................................................................... 3
D. MIHALACHE, M. TETIVA Asupra monotoniei unor iruri .............................. 6
G. DOSPINESCU O teorem uitat - inegalitatea lui Surnyi.............................. 11
T. ZVONARU, B. IONI Rapoarte determinate de o cevian
i o secant ntr-un triunghi................................................. 15
Gh. MOLEA Asupra unor ecuaii diofantice ptratice.......................................... 18
Fl. POPOVICI O caracterizare a funciilor convexe cu ajutorul
derivatelor pariale .................................................................. 21

NOTA ELEVULUI
A. ZAHARIUC Asupra problemei G67.................................................................. 22
I. MUSTA Matematic i algoritmi ................................................................... 24
A. NEGRESCU Asupra unei probleme de concurs .............................................. 27

CHESTIUNI METODICE
C. APOSTOL Din nou asupra unei probleme de concurs .................................... 29
P. GEORGESCU, G. POPA Dou funcii cu aceeai derivat
pe un interval nu difer neaprat printr-o constant......... 31

CORESPONDENE
V. GUU Probleme selectate de la Olimpiadele de Matematic
ale Republicii Moldova......................................................... 34
H. STEPHAN Probleme pentru clasa a VIII-a ...................................................... 38

CONCURSURI I EXAMENE
Concursul Recreaii Matematice ed. a IV-a, Muncel - Iai, 2004........................ 41
Concursul Octav Onicescu, ed. a VIII-a, Botoani, 2004 .................................... 43

PROBLEME I SOLUII
Soluiile problemelor propuse n nr. 1/2004............................................................... 44
Soluiile problemelor pentru pregtirea concursurilor din nr. 1/2004 ...................... 59
Probleme propuse.......................................................................................................... 71
Probleme pentru pregtirea concursurilor ................................................................... 77
Training problems for mathematical contests ............................................................ 79
Pagina rezolvitorilor ................................................................................................... 82

Anul VIII, Nr. 1

Ianuarie Iunie 2006

RECREAII
MATEMATICE
REVIST DE MATEMATIC PENTRU ELEVI I PROFESORI

e i = 1

Editura Recreaii Matematice


IAI - 2006

Semnificaia formulei de pe copert:


i
ntr-o form concis, formula e = 1 leag cele patru ramuri fundamentale
ale matematicii:
ARITMETICA
GEOMETRIA
ALGEBRA
ANALIZA MATEMATIC

reprezentat
reprezentat
reprezentat
reprezentat

de
de
de
de

i
e

Redacia revistei :
Petru ASAFTEI, Dumitru BTINEU-GIURGIU (Bucureti), Temistocle BRSAN, Dan
BRNZEI, Ctlin - Cristian BUDEANU, Constantin CHIRIL, Eugenia COHAL, Adrian
CORDUNEANU, Mihai CRCIUN (Pacani), Gabriel DOSPINESCU (student, Paris),
Marius FARCA, Paraschiva GALIA, Paul GEORGESCU, Mihai HAIVAS, Gheorghe
IUREA, Lucian Georges LDUNC, Mircea LUPAN, Dan tefan MARINESCU
(Hunedoara), Gabriel MRANU, Andrei NEDELCU, Gabriel POPA, Dan POPESCU
(Suceava), Florin POPOVICI (Braov), Maria RACU, Ioan SCLEANU (Hrlu), Ioan
ERDEAN (Ortie), Dan TIBA (Bucureti), Adrian ZAHARIUC (Bacu), Adrian
ZANOSCHI.

Adresa redaciei:
Catedra de Matematic Universitatea Tehnic Gh. Asachi Iai
Bd. Carol I, nr.11, 700506, Iai
Tel. 032 213737 / int. 123
E-mail: recreatii.matematice@gmail.com
http://www.recreatiimatematice.uv.ro
COPYRIGHT 2006, ASOCIAIA RECREAII MATEMATICE
Toate drepturile aparin Asociaiei Recraii Matematice. Reproducerea integral sau
parial a textului sau a ilustraiilor din aceast revist este posibil numai cu acordul prealabil
scris al acesteia.
TIPRIT LA SL&F IMPEX IAI
Bd. Carol I, nr. 3-5
Tel. 0788 498933
E-mail: simonaslf@yahoo.com

Anul VIII, Nr. 1

Ianuarie Iunie 2006

RECREAII
MATEMATICE
REVIST DE MATEMATIC PENTRU ELEVI I PROFESORI

e i = 1
Revist cu apariie semestrial
publicat de

ASOCIAIA RECREAII MATEMATICE

IAI - 2006

Elogiu adus revistei "Gazeta Matematic


a"
la 110 ani de aparitie nentrerupt
a
Cum podul de la Cernavod
a si ntinde bratele peste apele Dun
arii, asa si Gazeta
Matematic
a si ntinde existenta, care are nceputul n secolul al XIX-lea, peste
ntreg secolul al XX-lea si continu
a s
a-si aduc
a aportul la dezvoltarea nv
atamntului
si stiintelor matematice din tara noastr
a si n acest nou secol, al XXI-lea.
Gazeta Matematic
a, prin cele 110 tomuri durate n timp unul dup
a altul, este
piramida Keops a publicisticii periodice romnesti, punctul de maxim absolut al
acesteia. Este a doua publicatie de matematic
a din lume ce se adreseaz
a tineretului
si prima de acest fel n privinta aparitiei nentrerupte si longevit
atii.
A avut o existenta zbuciumat
a si cu multe momente dramatice, existenta strns
legat
a de soarta nv
atamntului din scolile romnesti - cel matematic mai ales - dar si
de cea a poporului romn. A trecut prin dou
a r
azboaie mondiale, reforme neinspirate
ale nv
atamntului public, regimuri adverse poporului romn etc. S
a amintim doar
un singur episod din existenta Gazetei. n primul r
azboi mondial, n urma ocup
arii
capitalei, Iasul devine centrul politic si administrativ al tarii; tot aici se refugiaz
a si
Gazeta Matematic
a, care, datorit
a devotamentului si str
adaniilor lui Traian Lalescu,
Vasile Teodoreanu si altor membri din redactie, continu
a s
a fie tip
arit
a si expediat
a,
ajungnd chiar si n minele abonatilor aflati n primele linii ale frontului.
Sub deviza Entuziasm, armonie, munca dezinteresata, sacrificii continue au fost
dep
asite toate greut
atile materiale si vicisitudinile vremurilor; oameni minunati strnsi
n jurul si p
atrunsi de spiritul Gazetei Matematice, au asigurat prin munca si b
arb
atia lor mersul ei nainte. n prima jum
atate de veac, la crma destinelor revistei
au fost "cei 4 stlpi ai Gazetei Matematice": Ion Ionescu, Gheorghe Ti
teica, Andrei
Ioachimescu si Vasile Cristescu. Merit
a elogiat
a, de asemenea, contributia lui Nicolae
Teodorescu, care a condus cu mult
a competenta, n ultimul sfert al secolului trecut,
activit
atile la Gazet
a si cele legate de ea, devenite ntre timp mult mai diverse.
Tinerii talentati din generatii succesive au "frecventat" scoala Gazetei Matematice, unde si-au format deprinderile si tehnicile de lucru, si-au slefuit rationamentul
matematic sau si-au v
azut publicate primele ncerc
ari originale n paginile acesteia.
Nume de viitori ilustri matematicieni romni se g
asesc mentionate n paginile
Gazetei Matematice printre rezolvitorii si propun
atorii de probleme, premiantii
unor concursuri sau ca autori de note originale interesante, iar unii dintre ei asumndu-si responsabilit
ati redactionale. Lista lor fiind prea lung
a, amintim doar cteva nume: Gh. Ti
teica, C. Popovici, T. Lalescu, N. Abramescu, O. Mayer, Al. Pantazi, D.
Barbilian, Fl. Vasilescu, N. Cioranescu, T. Popoviciu, Gr. Moisil, N. Teodorescu etc.
n acesti 110 ani de existenta, Gazeta Matematic
a a devenit o cariatid
a a culturii romnesti, simbol al permanentei si continuit
atii, component
a a nv
atamntului
matematic din tara noastr
a, pepinier
a de talente matematice.
Ca o recunoastere a meritelor sale, Presedintele Romniei, prin decretul 988 din 7
octombrie 2005, confer
a Gazetei Matematice Ordinul "Meritul Cultural" n gradul
de Ofiter, categoria H "cercetare stiintifica", pentru contributia deosebit
a la promovarea nv
atamntului si cercet
arii aprofundate a stiintelor matematice.

Prof. dr. Temistocle BRSAN


1

100 de ani de la nasterea matematicianului


Grigore C. Moisil
Multe capitole ale matematicii mi-au fost
dragi. Matematica e una (Gr. C. Moisil)
n Istoria matematicii n Romnia, George
St.
Andonie l prezint
a pe Grigore C. Moisil ca "o
fericita ntruchipare a dominantelor matematicii noastre: dinamism, varietate, tendinta spre universalitate".
A fost unul dintre cei mai mari si talentati matematicieni romni si, indiscutabil, cel mai prolific. Atacnd cu succes aproape toate domeniile matematicii
pure si aplicate s-a dovedit un creator n continu
a nnoire. Jovial si optimist, cu un umor plin de sev
a, era
un povestitor fermec
ator.
Opera lui Grigore C. Moisil nu este doar creatia
individual
a a unui om deosebit de nzestrat. Ea se
bazeaz
a pe traditia "dinastiei" Moisilestilor, pe sprijinul permanent al familiei care a stimulat si sprijinit
inteligenta sa sclipitoare. S-a mndrit ntotdeauna c
a
se trage dintr-o familie de gr
aniceri n
as
audeni. Familia sa, originar
a din Maramures,
a descins n comuna S
ant, n imediata vecin
atate a N
as
audului. Str
abunicul matematicianului, care purta numele de Grigore, a fost primul c
arturar care s-a ridicat din
comuna S
ant. A fost preot, profesor si primul director al celui de al patrulea liceu
romnesc nfiintat n Austria, la N
as
aud. Fiul acestuia, Constantin, a obtinut titlul
de doctor n stiinte filologice la Universitatea din Viena si a functionat 32 de ani ca
profesor la N
as
aud. Unul dintre fii acestuia, numit tot Constantin, tat
al matematicianului, a urmat, cu sprijinul lui Al. Odobescu, scoala Normal
a Superioar
a din
Bucuresti; a functionat ca profesor la Focsani, Tulcea si Bucuresti. Ulterior, a p
ar
asit
profesoratul consacrndu-se arheologiei si numismaticii, devenind un reputat specialist n acest domeniu si membru al Academiei Romne. La Tulcea s-a c
as
atorit cu
institutoarea Elena Nicolescu, care a devenit ulterior directoarea S
colii "En
achita
V
ac
arescu" din Bucuresti. La Tulcea s-au n
ascut primii trei copii ai familiei; Grigore
(10 ianuarie 1906), Florica (cercet
atoare la Biblioteca Academiei; c
as
atorit
a cu acad.
Emil Condurache) (1909) si Ioan (1910); ultimul copil, Gheorghe, s-a n
ascut n 1917
la Vaslui (n timpul refugiului). Ambii frati au fost ingineri, profesori universitari.
Dotat
a cu o inteligenta vie si un umor s
an
atos, mama, Elena, a avut un rol decisiv
n formarea lui Grigore, care i-a urmat cu sfintenie sfaturile, n ntreaga sa viata. De
la ea a mostenit deviza: "Nu crede tot ce ti se spune, judeca tu singur".
Grigore C. Moisil a urmat scoala primar
a n Bucuresti si liceul la "Mihail Kog
alniceanu" din Vaslui (1916 1918) si "Spiru Haret" din Bucuresti (1918 1923). S-a
nscris apoi la sectia de matematic
a de la Facultatea de S
tiinte a Universit
atii din
Bucuresti, unde a a vut ca profesori pe D. Pompeiu, Gh. Titeica,

A. Davidoglu,
Tr. Lalescu. Primul i-a fost mentor nu numai n matematic
a, ci si n anumite reguli
de viata. ntr-un articol ("Viata studenteasc
a", nr. 11, 1967) m
arturiseste: "For2

marea generatiei matematice din care fac parte coincide cu nceputurile matematicii
abstracte romnesti. Generatia mea a pasit cu dreptul. Ea a profitat de faptul de a
fi avut ca profesori oameni de stiinta si ai caror profesori si ei oameni de stiinta".
Ca student a participat si la cursuri de istorie (N. Iorga), filozofie, sociologie (Mihail Dragomirescu), istoria artelor. A urmat n paralel si sectia de constructii de la
Institutul Politehnic Bucuresti la care a renuntat (n anul al IV-lea) cnd a obtinut
doctoratul si a plecat n str
ain
atate.
A luat doctoratul la 4 iunie 1929 n fata unei comisii prezidat
a de Gh. Ti
teica si din
care f
aceau parte Dimitrie Pompeiu si Anton Davidoglu. n teza de doctorat ntitulat
a
Mecanica analitica a sistemelor continue a studiat analitic mecanica sistemelor cu
un num
ar infinit de grade de libertate folosind metoda functional
a (notiunea de
functional
a fusese introdus
a cu putin timp n urm
a de Vito Volterra). n 1930 pleac
a,
cu o burs
a a ministerului la Paris, unde ia contact cu Jaques Hadamard, Paul Levy,
Henri Villat, Paul Montel si Elie Cartan, care apreciaz
a elogios contributiile originale
din teza de doctorat.
La 1 iulie 1931 si trece docenta n specialitatea analiz
a matematic
a, la Universitatea din Bucuresti. Se ntoarce la Paris unde urmeaz
a cursul lui Vito Volterra.
n toamna anului 1932 se stabileste la Iasi fiind numit conferentiar la Universitatea
"Al. I. Cuza". Matematicianul Ion Creanga, fost profesor si rector al Universit
atii si
aminteste: n acel timp eram student n anul al III-lea al sectiei de matematica de
la Universitatea din Iasi; n curnd am aflat ca la sectia noastra a nceput un curs
de factura moderna predat de un tnar matematician, deja cu renume format si care
revolutioneaza conceptia noastra despre algebra. Am fost atras de acest curs, am nceput sa-l audiez si n curnd am fost furat de noutatile att de atractive cuprinse n
lectiile cursului. Prelegerile lui Moisil ne-au deschis portile spre fermecatoarea lume
a structurilor algebrice, a laticelor, a mpletirii strnse dintre procesele de logica si
abstractizarea teoriei multimilor. Perioada de 10 ani petrecuti n Iasi a fost de mare
importanta pentru creatia sa stiintific
a si pentru des
avrsirea personalit
atii sale. n
vrst
a de 26 de ani, a g
asit la Iasi o atmosfer
a de nalt
a cultur
a. Peste ani si amintea:
La Iasi era o extraordinara densitate de oameni destepti pe metrul patrat. Aici a g
asit
matematicieni de mare valoare stiintific
a si spiritual
a si a r
amas toat
a viata prieten cu
cei care l primiser
a cu simpatie la sosirea n Iasi: Alexandru si Vera Myller, Simion
Sanielovici, Octav Mayer, Mendel Haimovici, Ilie Popa, Adolf Haimovici si asistentul
s
au din acea perioad
a Ion Creanga. n Biblioteca Seminarului Matematic din Iasi a
g
asit c
artile care aveau s
a fac
a din el un matematician modern. Proasp
at titularizat
ca profesor n 1935, n introducerea primului curs de algebr
a abstract
a modern
a tinut
n Romnia, afirm
a: La Iasi am citit multe carti de algebra, dar cartea directoare
a fost cea a lui B. L. Van der Warden "Moderne Algebra". Era acolo un nou mod
de a concepe matematica si anume algebra, dar nu numai algebra; matematica era
conceputa nu ca o stiinta a cantitatii, ci ca o stiinta a structurii. Peste ctva ani au
aparut alte doua carti care evidentiau acelasi mod de a privi matematica: "Topologia" lui Kuratowski si cartea lui St. Banach asupra spatiilor care i poarta numele. Se
putea, cu aceste volume si punnd n fruntea lor "Teoria numerelor transfinite" a lui
W. Sierpinski, organiza un curs de matematici n ntelesul de studiu al structurilor.
ntelegeam ncet, ncet ca matematica se schimbase. Se schimba. Se va schimba.
3

n Iasi s-a simtit n largul s


au, a legat numeroase prietenii, participnd cu exuberanta specific
a tineretii la viata acestui oras pentru care a p
astrat permanent o afectiune nedesmintit
a. S-au creat legende n leg
atur
a cu viata boem
a a tn
arului "r
asf
atat" al Iasului. La restaurantul de lng
a vechea cl
adire a Academiei Mih
ailene s-a
p
astrat ntr-un colt discret, pn
a la demolarea localului, o mas
a cunoscut
a sub numele de "masa lui Moisil". Se spune c
a la restaurantul "Corso" din centrul orasului,
i pl
acea s
a asculte orchestra interpretnd un vals a c
arui melodie si versuri erau
compuse chiar de Moisil.
Tot la Iasi s-a petrecut un eveniment care i-a marcat ntreaga viata si creatie.
O cunoaste pe Viorica Constante cu care se va c
as
atori. Viorica Moisil i-a stat
al
aturi n permanenta, l-a sprijinit si stimulat, i-a asigurat calmul si confortul necesare
creatiei. Dup
a moartea savantului, pe baza scrisorilor si altor documente de familie,
i-a dedicat o carte minunat
a scris
a cu talent, dragoste si discretie "Un om ca oricare
altul. Grigore C. Moisil", ap
arut
a n 1979 n editura Albatros.
n anul 1942 s-a creat la Facultatea de Matematic
a a Universit
atii din Bucuresti
catedra de analiz
a superioar
a si logic
a la care este ncadrat Grigore C. Moisil. Dup
a
perioada 1946-1948 cnd a fost ambasador al Romniei n Turcia, revine la Universitatea Bucuresti unde a predat cursuri de elasticitate, algebr
a si masini de calcul.
n 1948 devine mebru activ al Academiei Romne si seful sectiei de algebr
a de la
Institutul de Matematic
a al Academiei, nou nfiintat. n 1948 este ales presedinte al
Societ
atii Romne de Matematic
a, post pe care l va ocupa toat
a viata.
Dup
a ce Grigore C. Moisil formeaz
a la Bucuresti o veritabil
a scoal
a de mecanica
solidelor deformabile, ncepnd din 1949 ia nastere n jurul s
au Scoala

de teorie algebrica a mecanismelor automate. Al


aturi de rusii V. I. Sestacov

si M. Gavrilov si americanul Shannon este fondatorul acestei teorii, care are la baz
a utilizarea algebrelor
Boole n studiul automatelor. n aceast
a directie public
a dou
a tratate: Teoria algebrica a mecanismelor ordonate si Teoria algebrica a schemelor cu contacte si relee.
ncepnd din 1955 c
al
atoreste foarte mult, fiind invitat la congrese, sesiuni de
comunic
ari, cursuri sau conferinte. Devine membru al Academiei din Bologna si al
Institutului International de Filozofie din Paris.
Grigore C. Moisil are lucr
ari importante n analiza functional
a, mecanica teoretic
a, geometrie diferential
a si algebr
a. Partea cea mai original
a din creatia sa o
constituie preocup
arile de logic
a matematic
a (ncepute n perioada de la Iasi, unde a
tinut si primele cursuri de logic
a matematic
a din Romnia), care l-au condus la consideratii filozofice asupra matematicii si la teoria algebric
a a mecanismelor automate.
Aceste preocup
ari i-au asigurat un loc cu totul aparte n matematica romneasc
a.
S-a stins din viata la 21 mai 1973, la Ottawa, n Canada, n timpul unei vizite n
care a pus jaloanele colabor
arii ntre informaticienii candieni si cei romni. La un an
de la disparitia sa, fostul s
au elev, Mircea Malita l caracteriza: Moisil a fost mai mult
dect un savant, a fost mai multi savanti ntruniti n sesiune permanenta sau lundusi locul unul altuia n cicluri succesive mari, reprezentate de temele fundamentale pe
care le-a abordat. A fost pna n ultimele zile deschizator de drumuri, inovator. n
aceasta aventura spirituala nu a admis dilentatismul superficial.

Prof. dr. Petru MINUT

Asupra problemei 809 din Gazeta Matematic


a,
volumul VIII (19021903)
D. M. B
ATINETU
- GIURGIU 1
Cu ocazia aniversarii a 110 ani de aparitie
nentrerupta a Gazetei Matematice
n istoria matematicii din tara noastr
a Traian Lalescu reprezint
a un creator
de diversitate rar
a, un mare animator al generatiei sale de matematicieni, un om
dotat cu o mare putere de munc
a si inteligenta scnteietoare, un profesor nzestrat
cu deosebit talent pedagogic.
Traian Lalescu s-a n
ascut la 12/24 iulie 1882, n Bucuresti. Studiile primare le-a
f
acut la Bucuresti, primele dou
a clase de gimnaziu la Craiova (1892-1894), iar clasele
a III-a si a IV-a la Roman (1894-1896). Clasele a V-a si a VI-a le-a f
acut la Liceul
Internat din Iasi (actualul Colegiu National "C. Negruzzi") n perioada 1896-1900.
n liceu, ca si n gimnaziu, Lalescu a fost premiantul I al clasei si a primit premiul
de onoare al scolii (Lalescu se afl
a trecut pe tabela de onoare a Liceului Internat din
Iasi).
Chiar din clasa a VI-a a liceului (februarie 1898), Lalescu ajunge corespondent la
Gazeta Matematic
a. Profesorul s
au de mai trziu, inginerul Ion Ionescu, relateaz
a
despre Traian Lalescu c
a: "Intrarea lui n rndul corespondentilor "Gazetei Matematice" nu s-a f
acut ca de obicei, n mod timid, lent, progresiv, ci deodat
a, intens, la
maximum posibil. A fost un caz unic de aparitiune la "Gazeta Matematic
a" de
activitate prodigioas
a a unui tn
ar licean!".
n v. VIII (1902-1903), la pagina 244, Traian Lalescu a propus Problema 809, cu
urm
atorul enunt:
1

ch
d2n+1
1
x
2n
Sa se arate ca:
x sh
= 2n+2 .
dx2n+1
x
x
La pag. 283 din Gazeta Matematic
a, v. IX (1903-1904), este publicat
a solutia
dat
a de Traian Lalescu acestei probleme, urmat
a de o not
a:
"Se stie c
a:
sh x =

x
x3
x2n+1
+
+ +
+
1!
3!
(2n + 1)!

Vom avea deci:


sh

si

ch x = 1 +

x2
x2n
+ +
+ .
2!
(2n)!

1
1
1
1
+ ...
+
= +
x
x 3!x3
(2n + 1)!x2n+1

si, prin urmare:


1
1
1
1
x2n sh = P (x) +
+ +
+ ,
+
x
(2n + 1)!x (2n + 3)!x3
(2n + 2p + 1)!x2p+1
P (x) fiind un polinom ntreg n x de gradul 2n.
1

Profesor, Colegiul National "Matei Basarab", Bucuresti

Seria din membrul al II-lea, uniform convergent


a n tot planul exceptnd originea,
e derivabil
a termen cu termen si rezultatele g
asite sunt serii convergente pe aceeasi
ntindere, ale c
aror sume sunt date de derivatele de acelasi ordin ale membrului I.
Observnd acum c
a:

2n+1
d
d2n+1
1
(2p + 1) (2p + 2) (2p + 2n + 1)
(P
(x))
=
0,
=
dx2n+1
dx2n+1 x2p+1
x2n+2p+2
si c
a, prin urmare

d2n+1
1
1
1
2n+2 ,
=
2n+1
2p+1
2p
dx
(2p + 2n + 1)!x
(2p)!x
x
obtinem
1

ch
d2n+1
1
1
1
1
2n
x
+ +
+ = 2n+2 .
x sh
= 2n+2 1 +
dx2n+1
x
x
2!x2
(2p)!x2p
x
(T.L.)
Not
a. Aceast
a problem
a a fost rezolvat
a de D-nii: N. Abramescu, Gr. Or
asanu,
G. Constantinescu, M. Radu, C. Gheorghiu si I. G. Niculescu.
n acelasi mod se pot demonstra si formulele:
1

cos
1/x
d2n+1
1
dn n1 1/x
n+1
n e
2n
x;
sin
e
etc. "
x
x
=
(1)
=
(1)
dx2n+1
x
x2n+2
dxn
xn+1

Ca un omagiu adus marelui matematician romn Traian Lalescu, vom da acestei


probleme o nou
a solutie, accesibil
a elevilor actualului liceu.
1
S
a consider
am functiile fn : R R, fn (x) = x2n sh , unde n N. Se conx
a, oricare ar fi n N. Ne propunem s
a
stat
a imediat c
a fn este indefinit derivabil
demonstr
am c
a
1
1
fn(2n+1) (x) = 2n+2 ch , n N
(1)
x
x
prin metoda inductiei matematice, folosind formula lui Leibniz de derivare a produsului a dou
a functii indefinit derivabile, adic
a
n
X
n
k (nk) (k)
Cn u
v , n N .
(2)
(uv) =
Avem f00 (x) =

sh

1
x

k=0

1
1
ch , deci pentru n = 0 formula (1) se verific
a.
2
x
x

De asemenea avem:
1
1
1
f1 (x) = x2 sh , deci f10 (x) = 2x sh ch ;
x
x
x
1
1
2
1
1
f100 (x) = 2 sh ch + 2 ch ;
x x
x x
x
2
1
1
1
1
1
2
2
1
1
f1000 (x) = 2 ch + 2 ch + 3 sh 4 ch = 4 ch ,
x
x x
x x
x x
x
x
x
deci si pentru n = 1 formula (1) se verific
a.
a are loc relatia
Presupunem c
a formula (1) este adev
arat
a pentru n N adic
6

1
ch ,
x
si demonstr
am c
a ea este adev
arat
a si pentru n + 1, adic
a avem
1
1
(2n+3)
fn+1 (x) = 2n+4 ch .
x
x
S
a observ
am c
a
fn+1 (x) = x2 fn (x) , n N,
si atunci, cu ajutorul formulei (2), avem:
fn(2n+1) (x) =

(2n+3)

fn+1

x2n+2

(3)

(4)
(5)

(2n+3) 2n+3
(k)
k
(x) = x2 fn+1 (x)
=
C2n+3
fn(2n+3k) (x) x2
=
k=0

0
1
2
fn(2n+3) (x) x2 + C2n+3
fn(2n+2) (x) 2x + C2n+3
f (2n+1) (x) 2 =
= C2n+3

= x2 fn(2n+3) (x) + 2(2n + 3) xfn(2n+2) (x) + (2n + 3)(2n + 2) fn(2n+1) (x), n N, (6)

Conform presupunerii relatia (3)


arat
a, rezult
a c
a:
fiind adev

0
1
1
2n
+
2
1
1
1
fn(2n+2) (x) = fn(2n+1) (x) = 2n+2 ch
= 2n+3 ch + 2n+4 sh , n N,
x
x
x
x x
x
(7)
si atunci

0
2n + 2
1
1
1
ch
sh
=
fn(2n+3) (x) = fn(2n+2) (x) =
+
x2n+3
x x2n+4
x
1 2n + 2
1 2n + 4
1
1
(2n + 2) (2n + 3)
1
ch 2n+5 sh 2n+5 sh 2n+6 ch =
=
x2n+4
x
x
x
x
x x
x
1 4n + 6
1
(2n + 2) (2n + 3) x2 + 1
ch 2n+5 sh .
(8)
=
2n+6
x
x
x
x
Dac
a tinem seama de relatiile (3), (7) si (8), relatia (6) devine
(2n+2)(2n+3) x2 + 1 1
4n+6 1 2(2n+2)(2n+3) 1
(2n+3)
ch 2n+5 sh +
ch +
fn+1 (x) =
x2n+4
x x
x
x2n+2
x
1 (2n + 2) (2n + 3)
1
2 (2n + 3)
sh
ch =
+
x2n+3
x
x2n+2
x

1
1
1
= 2n+5 (2n+2)(2n+3) x2 + 1 ch + 2x2 (2n+2) (2n+3) ch
x
x
x

1
1
1
2
(2n + 2) (2n + 3) x ch
= 2n+4 ch ,
x
x
x
ceea ce demonstreaz
a c
a relatia (4) este adev
arat
a.
Conform principiului inductiei matematice, rezult
a c
a
1
1
fn(2n+1) (x) = 2n+2 ch , n N.
x
x
Bibliografie

1. G. S
t. Andonie - Istoria matematicii n Romnia, v. 1, Ed. S
t., Buc., 1965.
2. M. D. B
atinetu-Giurgiu, M. B
atinetu-Giurgiu, I. Brchi-Damian, A. Semenescu - Analiza matematica. Probleme pentru clasa a XI-a, Ed. Matrix Rom, Buc.,
2003.
3. Colectia "Gazeta Matematica", 1895-2005.
7

Cteva propriet
ati ale subgrupurilor finite
din GLn (Z)
Gabriel DOSPINESCU 1
Cu ocazia aniversarii a 110 ani de aparitie
nentrerupta a Gazetei Matematice
1. Introducere: lema lui Serre. Ceea ce veti citi n continuare este o ncercare timid
a de a expune o colectie de rezultate referitoare la subgrupurile finite din
GLn (Z). Se prea poate ca demonstratiile care urmeaz
a s
a fie cunoscute; autorul le-a
g
asit "aproape" singur si crede c
a merit
a s
a fie prezentate. Articole (mai serioase)
despre propriet
atile acestor subgrupuri s-au scris multe si, cu siguranta, se vor mai
scrie, c
aci problemele referitoare la ele sunt dificile si multe dintre ele si asteapt
a de
ani buni rezolv
arile. l invit
am pe cititorul interesat de rezultate mai profunde s
a
citeasc
a articolele din bibliografie, mult mai tehnice si mai specializate. Se pare c
a n
[3] ar fi o descriere superb
a a acelorasi (sau chiar a mai multor) rezultate, ns
a, din
p
acate, nu am avut acces la acest articol, asa c
a nu putem dect s
a-l recomand
am
"orbeste" cititorilor interesati de asemenea aspecte.
Iat
a, mai nti, ce rezultate vom demonstra (sau doar aminti). Vom deduce forma
simpl
a a teoremei Jordan-Zassenhaus (cu ajutorul lemei lui Serre, de care am luat
cunostinta din [7]) relativ la finitudinea claselor de izomorfism ale subgrupurilor finite
ale lui GLn (Z), apoi vom demonstra c
a orice subgrup finit din GLn (Z) are cel mult
(2n)! elemente si c
a exist
a 9 clase de izomorfism pentru subgrupurile lui GL2 (Z).
Vom ncepe cu lema lui Serre, un rezultat de o frumusete deosebit
a, care permite
o prim
a majorare a ordinului subgrupurilor finite din GLn (Z); utilitatea acesteia ne
permite s
a o numim "teorem
a". Toate grupurile despre care va fi vorba n continuare
au cel putin dou
a elemente.
Teorema 1 (Lema lui Serre). Fie G GLn (Z) un grup finit si p > 2 un numar
prim. Consideram aplicatia : GLn (Z) GLn (Zp ) care asociaza fiecarei matrici
A matricea claselor de resturi modulo p ale elementelor din A. Atunci restrictia
acestei aplicatii la G este injectiva.
Demonstratie. Desigur, este bine definit
a si este un morfism ntre grupurile
GLn (Z) si GLn (Zp ) (asa cum se verific
a imediat). S
a presupunem c
a restrictia aplicatiei la G nu este injectiv
a, deci exist
a A G, A 6= In astfel nct (A) = (In ).
Asta nseamn
a c
a putem scrie A = In + pB, unde B Mn (Z). Fie 1 , 2 , . . . , n valorile proprii ale matricii B; se stie atunci c
a A are valorile proprii 1 + pi , 1 i n.
Acum s
a privim cu atentie sumele Sk = k1 + k2 + + kn (pentru k num
ar natural):
toate vor fi numere ntregi (cel mai simplu argument este teorema fundamental
aa
polinoamelor simetrice, c
aci toate aceste sume sunt polinoame cu coeficienti ntregi
n sumele simetrice fundamentale ale numerelor 1 , 2 , . . . , n , iar aceste sume simetrice sunt - modulo un semn plus sau minus - coeficientii polinomului caracteristic al
matricii B Mn (Z), deci ntregi). ns
a, G fiind finit, putem scrie A|G| = In , deci
n
trebuie s
a avem (1 + pi ) = 1, pentru fiecare 1 i n, iar de aici obtinem imediat
c
a |i | < 1, 1 i n. Or, aceasta nseamn
a c
a sirul de numere ntregi (Sk )k1 tinde
1

Student, cole Normale Suprieure, Paris

la zero, deci trebuie ca toti termenii s


ai s
a fie nuli (de la un rang ncolo). O simpl
a
aplicare a formulelor lui Newton ne va duce la concluzia c
a e necesar, pentru asta, ca
toti i s
a fie egali cu 0; dar atunci toate valorile proprii ale matricii A sunt egale cu 1,
deci (teorema Cayley-Hamilton) ea este "r
ad
acin
a" a polinomului (X 1)n . Cum am
v
azut, mai este r
ad
acin
a si pentru X |G| 1, deci va fi r
ad
acin
a pentru cel mai mare
divizor comun al acestor polinoame, care este X 1: adic
a A = In (alt argument ar
fi c
a identitatea este singura matrice unipotent
a diagonalizabil
a, iar matricea A are
aceste dou
a propriet
ati: este unipotent
a - c
aci tocmai am ar
atat c
a toate valorile sale
proprii sunt egale cu 1 - si diagonalizabil
a, deoarece polinomul s
au minimal nu are
dect r
ad
acini simple, fiind un divizor al lui X |G| 1) si teorema 1 este demonstrat
a.
S
a examin
am putin consecintele acestei teoreme; obtinem imediat c
a (G) (imaginea lui G prin morfismul ) este un subgrup cu |G| elemente din GLn (Zp ). ns
a
GLn (Zp ) are exact (pn 1)(pn p) (pn pn1 ) elemente (l
as
am cititorului ca
exercitiu demonstratia acestui rezultat clasic). Rezult
a atunci, din teorema lui Lagrange, c
a |G| divide pe (pn 1)(pn p) (pn pn1 ), pentru orice subgrup finit
G GLn (Z) si orice p > 2 prim. n particular, exist
a un num
ar finit de ordine
posibile ale matricilor din GLn (Z) (participantii la olimpiade - si nu numai ei - trebuie s
a-si fi amintit celebra problem
a: orice matrice din GL2 (Z) are ordinul 1, 2,
3, 4, sau 6; ncercati s
a demonstrati aceasta pentru n = 3!; mai mult, curajosii se
pot gndi la o variant
a mult mai general
a: multimile ordinelor posibile ale matricilor
din GL2k (Z) si GL2k+1 (Z) coincid, pentru orice k 1 natural). De asemenea, mai
rezult
a (tot ca un caz particular) c
a ordinul oric
arei matrici din GLn (Z) divide pe
(3n 1)(3n 3) (3n 3n1 ) (aceast
a problem
a a fost propus
a de autor n RecMat, pe vremea cnd nu cunostea lema lui Serre; de altfel, am reusit s
a demonstr
am
c
a ordinul oric
arei matrici din GLn (Z) este mai mic dect A n ln n , unde A este o
constant
a pozitiv
a ce nu depinde de n, dar nu despre asta ne-am propus s
a vorbim
aici). Tot din lema lui Serre mai putem deduce si varianta simpl
a a teoremei lui
Jordan-Zassenhaus, c
aci am obtinut c
a orice subgrup finit al lui GLn (Z) are cel mult
(3n 1)(3n 3) (3n 3n1 ) elemente, deci, cu siguranta, exist
a un num
ar finit
de clase de izomorfism n GLn (Z). Desigur, de aici si pn
a la demonstrarea teoremei
lui Jordan-Zassenhaus (care afirm
a finitudinea num
arului claselor de conjugare ale
subgrupurilor finite ale lui GLn (Z)) mai e mult de munc
a, si, oricum, nu vom face
asta aici; recomand
am excelentul articol [7].
2. Major
ari pentru ordinele subgrupurilor finite ale lui GLn (Z). S
i iat
a
c
a ne apropiem de un punct sensibil al acestei note, anume de obtinerea unei major
ari
bune pentru ordinul oric
arui subgrup finit din GLn (Z); am obtinut deja c
a cel mai
mare divizor comun al numerelor
(pn 1)(pn p) (pn pn1 ), p > 2, p prim
este un astfel de majorant. Minkowski a demonstrat si un rezultat asem
an
ator pentru
2
p = 2, anume c
a ordinul oric
arui subgrup finit din GLn (Z) divide pe 2n (2n 1)(2n
2) (2n 2n1 ). Din p
acate aceast
a majorare este oricum, dar nu usoar
a si este
departe de a fi cea mai bun
a. Vom ncerca s
a d
am un rezultat mai "simplu" (n sensul
c
a formula e mai simpl
a) care este, si el, departe de valoarea optimal
a conjecturat
a.
Teorema 2. Orice subgrup din GLn (Z) are cel mult (2n)! elemente; de fapt,
9

ordinul oricarui subgrup din GLn (Z) divide pe (2n)!.


Mention
am c
a o majorare bun
a pentru ordinul maxim al subgrupurilor din GLn (Z)
este, dup
a cte stim noi, o problem
a deschis
a si foarte dificil
a. Cititorul va fi observat
o minorare aproape evident
a: exist
a subgrupuri cu 2n n! elemente (gnditi-v
a, de
exemplu, la matricile ce au exact un 1 sau 1 pe fiecare linie si pe fiecare coloan
a,
n rest zerouri!). Cel mai bun rezultat obtinut pn
a n prezent pare s
a fie o majorare
de forma C n (n!)1+ , unde C este o constant
a care depinde de , nu si de n, ns
a
aceasta necesit
a un efort considerabil, pe care nu-l vom face aici. Invit
am cititorul s
a
g
aseasc
a mai multe detalii n [5], unde exist
a chiar si o mentiune referitoare la faptul
c
a 2n n! este valoarea maxim
a a ordinului unui subgrup finit din GLn (Z) pentru
toti n 6 {2, 4, 6, 7, 8, 9, 10} (afirmatie atribuit
a acolo lui W. Feit).
S
a revenim acum la Teorema 2, a c
arei origine nu o stim - stim doar c
a a ap
arut n
[7] f
ar
a mentiuni suplimentare si f
ar
a. . . demonstratie. Demonstratia (cel putin cea
pe care am g
asit-o noi) cere r
abdare din partea cititorului, precum si niste rezultate
ajut
atoare, pe care le vom numi tot teoreme, datorit
a frumusetii si utilit
atii lor.
Teorema 3. Fie G GLn (Z) un subgrup finit. Atunci, pentru orice k N ,
|G| este un divizor al numarului
X
(tr(g))k .
gG

Demonstratie. nainte de toate, s


a spunem c
a nici m
acar nu e nevoie s
a presupunem c
a elementele matricilor sunt numere complexe; acestea pot fi dintr-un corp
comutativ oarecare a c
arui caracteristic
a este num
ar prim cu |G|. Demonstr
am mai
nti afirmatia pentru k = 1. S
a consider
am matricea
1 X
M=
g
|G|
gG

pentru care, clar, avem

1 XX
gh = M,
M2 =
|G|2
gG hG

deoarece, pentru fiecare g G, avem (G fiind grup) {gh | h G} = G. Egalitatea


a faptul c
a toate valorile proprii ale matricii M sunt 0 sau 1, deci
M 2 = M implic
tr(M ) (care este urma matricii M , deci suma valorilor proprii) este un num
ar ntreg;
or, folosind propriet
atile urmei, avem
1 X
tr(M ) =
tr(g),
|G|
gG

deci demonstratia pentru k = 1 este ncheiat


a am rezolvat si o problem
a
P a (totodat
mai veche de la concursul Putnam: dac
a
tr(g) = 0, G fiind un grup finit de matrici
gG
P
P
g = 0; ntr-adev
ar, egalitatea
tr(g) = 0 implic
a faptul c
a
p
atratice, atunci
gG

gG

suma valorilor proprii ale matricii M - definit


a ca mai sus - este 0, deci toate valorile
proprii sunt 0; atunci M este idempotent
a si nilpotent
a, deci este matricea nul
a).
Acelasi argument nu functioneaz
a ns
a pentru k 2 (din p
acate); si totusi. . . O
clip
a de gratie n algebra liniar
a a permis introducerea notiunii de produs tensorial
a dou
a matrici. Astfel, dac
a A Mn (K) si B Mp (K), produsul lor tensorial este
definit prin
10


a11 B . . . a1n B

..
..
A B = ...
Mnp (K).
.
.
an1 B . . . ann B
O proprietate fundamental
a a produsului tensorial (usor de verificat) este c
a

(A B) (C D) = (AC) (BD),

A, C Mn (K),

B, D Mp (K);

aceast
a egalitate ne permite s
a definim un subgrup G0 GLn2 (Z) prin G0 = {g g |
g G} (relatia de mai sus, precum si faptul c
a det(A B) = (det A)p (det B)n ,
pentru A, B ca mai sus, folosesc ca s
a ar
at
am c
a G0 este subgrup al lui GLn2 (Z)).
Acest subgrup are, evident, tot |G| elemente, deci i putem aplica rezultatul deja
demonstrat pentru a deduce c
a
X
X
tr(g g) =
(tr(g))2
|G||
gG

gG

(dac
a mai folosim si formula foarte simpl
a tr(A B) = tr(A) tr(B)). Cititorul a
nteles acum modul n care va demonstra afirmatia pentru orice k N (vom mai
spune doar c
a pentru k = 3 trebuie considerat G00 = {(g g) g | g G}).

Acum putem ncepe s


a demonstr
am Teorema 2. S
a not
am x1 > x2 > > xq
elementele multimii {tr(g) | g G} si s
a observ
am c
a avem q 2 si x1 = n. Nefiind
evidente (dar interesante si n sine) vom demonstra aceste propriet
ati. n primul
rnd, am v
azut c
a, dac
a A G, atunci A|G| = In , deci valorile proprii ale lui A
sunt r
ad
acini ale unit
atii, n particular ele au modulul 1. E clar atunci c
a avem
| tr(A)| n, pentru orice A G; cum In G, se cheam
a c
a x1 = n. Dar, s
a mai
observ
am, dac
a A G {In } (si existenta unei asemenea matrici e asigurat
a de
presupunerea f
acut
a nc
a de la nceput), nu putem avea tr(A) = n, c
aci atunci toate
valorile proprii ale matricii A ar fi egale cu 1, ceea ce este imposibil (cititorul nu a
uitat argumentul final din demonstratia teoremei 1); deci q 2. n plus, dac
a not
am
cu a1 , a2 , . . . , aq num
arul aparitiilor numerelor x1 , x2 , . . . , xq respectiv n multimea
urmelor matricilor din G, teorema 3 afirm
a c
a
|G||a1 xk1 + a2 xk2 + + aq xkq ,

k 1.

Desigur, mai avem si |G| = a1 + a2 + + aq , precum si a1 = 1 (este suficient


s
a fi nteles argumentele din acest paragraf pentru a ne convinge si de acest lucru,
precum si de faptul c
a, dac
a xq = n, atunci si aq = 1; toate aceste observatii se vor
dovedi esentiale n studiul subgrupurilor finite ale lui GL2 (Z)). Iar avem nevoie de
un rezultat ajut
ator.
Teorema 4. Fie a1 , a2 , . . . , aq , x1 , x2 , . . . , xq si m numere ntregi astfel nct
Atunci avem si

m | a1 xk1 + a2 xk2 + + aq xkq ,

k N .

m | a1 (x1 x2 ) (x1 xq ).
Demonstratie. S
a consider
am seria formal
a
a1
a2
aq
f (z) =
+
+ +
1 x1 z 1 x2 z
1 xq z

si s
a observ
am c
a

11

f (z) =

q
X
i=1

ai +

q
X

ai xi

i=1

z+

q
X

ai x2i

i=1

z2 + ,

deci, folosind
rezult
a existenta unor numere ntregi b0 , b1 , b2 , . . . astfel nct
Pipoteza,
f (z) = m
bj z j . Pe de alt
a parte, putem scrie si
j0
P
a1 (1 x2 z) (1 xq z)
f (z) =
.
(1 x1 z)(1 x2 z) (1 xq z)
Asta ne arat
a c
a seria formal
a (de fapt, polinomul) de la num
ar
ator poate fi scris n
formaX
X
a1 (1 x2 z) (1 xq z) = m(1 x1 z)(1 x2 z) (1 xq z)
bj z j ,
deci are toti coeficientii divizibili cu m, de unde obtinem c
am|

q
P

j0
(i)
ai St ,

i=1
x1 , . . . , xi1 , xi+1 , . . . , xq ,

(i)

unde St

este a t-a sum


a simetric
a fundamental
a n
ceea ce implic
a
si
q
q
q
X
X
X
(i)
(i)
q2
q1
a

x
a
S
+

+
(1)
ai Sq1
m|xq1
i
i
1
1
1
i=1

sau

m|

q
X
i=1

i=1

i=1

(i)

(i)

ai (xq1
xq2
S1 + + (1)q1 Sq1 ).
1
1

a
Cum, pentru i > 1, avem (x1 x1 ) (x1 xi1 )(x1 xi+1 ) (x1 xq ) = 0, adic
(i)

(i)

xq2
S1 + + (1)q1 Sq1 = 0,
xq1
1
1

ne r
amne doar c
a
(1)
(1)
q1
m | a1 (x1 xq2
S1 + + (1)q1 Sq1 ) = a1 (x1 x2 ) (x1 xq ),
1
ceea ce trebuia demonstrat.
Iar asta ncheie si demonstratia teoremei 2: din teoremele 3 si 4 si faptul c
a a1 = 1,
rezult
a c
a |G| divide (x1 x2 ) (x1 xq ), care este produsul a q 1 numere naturale
diferite si cel mult egale cu 2n (deoarece urma oric
arei matrici din G este un num
ar
ntreg cuprins ntre n si n), deci divide si pe (2n)!.
Bibliografie
1. G. P. Dresden - There are only nine finite groups of fractional linear transforms
with integer coeicients, Mathematics Magazine, June 2004, 211-218.
2. R. A. Horn, Ch. R. Johnson - Analiza matriciala, Fundatia Theta, Bucuresti,
2001.
3. J. Kuzmanovich, A. Pavlichenkov - Finite groups of matrices whose entries are
integers, American Mathematical Monthly, February 2002.
4. T. J. Laey - Lectures in integer matrices.
5. D. N. Rockmore, Ki-Seng Tan - A note on the order of finite subgroups of
GLn (Z), Commutative Algebra, 2/1999.
6. Ken-Ichi Tahara - On the finite subgroups of GL3 (Z), Nagoya Math. Journal.
7. Nicolas Tossel - Reseaux et thormes de finitude, Revue des mathmatiques speciales, 1-2/2005.
12

Ceviene si triunghiuri triomologice


Temistocle BRSAN 1
Cu ocazia aniversarii a 110 ani de aparitie
nentrerupta a Gazetei Matematice
n aceast
a Not
a, pornind de la un triunghi oarecare, punem n evidenta o configuratie de triunghiuri triomologice cu acelasi centru de greutate ca si triunghiul initial.
Dou
a triunghiuri, 4ABC si 4XY Z, se numesc omologice dac
a dreptele AX, BY ,
CZ sunt concurente; punctul de concurenta se numeste centru de omologie al triunghiurilor. Triunghiurile date sunt triomologice dac
a admit trei centre de omologie.
1. Fie ABC un triunghi oarecare si numerele , , R \ {1} cu = 1. Pe
A B
dreapta BC consider
am punctele A , A , A determinate de rapoartele
= ,
A C
A B
A B
= si respectiv
= (utiliz
am segmentele orientate pentru ca punctele
A C
A C
A , A si A s
a poat
a fi situate n orice pozitie pe BC, exceptnd vrfurile B si C
ale 4ABC). Punctele B , B , B CA si C , C , C AB se determin
a n mod
similar. Conditia = 1 asigur
a existenta punctelor X , Y etc. definite prin
{X } = AA BB CC ,
{X } = AA BB CC ,
{X } = AA BB CC ,
{Y } = AA CC BB ,
(1)
{Y } = AA CC BB ,
{Y } = AA CC BB .
Att pe figur
a ct si schematic din
A B C
A B C
X ( )
( ) Y
(2)
X ( )
( ) Y
X ( )
( ) Y
se poate urm
ari formarea acestor puncte si a triunghiurilor X X X si Y Y Y .

C
C

1
3
1
=
2
= 6
=

B
X
Y
G

C
B
1

X
A

Y
A

Prof. dr., Catedra de matematic


a, Univ. Tehnic
a "Gh. Asachi", Iasi

13

Se observ
a c
a 4ABC si 4X X X sunt invers orientate, pe cnd 4ABC si
4Y Y Y sunt la fel orientate.

Propozitia 1. Triunghiurile X X X si Y Y Y sunt triomologice, centrele lor


de omologie fiind vrfurile triunghiului ABC.
Demonstratie. Vom ar
ata urm
atoarele:
(i) 4X X X si 4Y Y Y sunt omologice cu centrul A;
(ii) 4X X X si 4Y Y Y sunt omologice cu centrul C;
(iii) 4X X X si 4Y Y Y sunt omologice cu centrul B.
Aceste trei afirmatii decurg din (1). Astfel, afirmatia (1) revine la a vedea c
a
dreptele X Y , X Y si X Y sunt concurente n A. Cum din prima si a patra
egalitate din (1) rezult
a c
a X , Y AA , vom avea c
a A X Y . La fel obtinem
relatiile A X Y si A X Y . Asadar (i) este adev
arat
a. Pe aceeasi cale se
dovedesc (ii) si (iii). Q.e.d.
Observatie. n consecinta, configuratia contine si perechile de triunghiuri triomologice: 4ABC si 4X X X , 4ABC si 4Y Y Y ; pentru prima pereche avem:
4ABC, 4X X X ; Y ,
4ABC, 4X X X ; Y ,
4ABC, 4X X X ; Y ,
iar pentru a doua avem:
4ABC, 4Y Y Y ; X ,
4ABC, 4Y Y Y ; X ,
4ABC, 4Y Y Y ; X ,
(pe un rnd sunt scrise dou
a triunghiuri, pe baza schemei (2), si centrul lor de
omologie).
2. n aceast
a sectiune vom stabili o alt
a proprietate a configuratiei: cele trei triunghiuri au acelasi centru de greutate. Pentru aceasta, vom utiliza metoda vectorial
a.
Avem nevoie de urm
atoarea
Lem
a. Fie ABC un triunghi oarecare si punctele A0 BC, B 0 CA. Daca
0
AB
B0C
= 0 , = 0 si + 1 6= 1, atunci cevienele AA0 si BB 0 au un punct de
AC
BA
intersectie X si avem
1

r B
r C) .
(3)
(
r A+
rX =
+ 1

a vectorul de pozitie al punctului X fata de o origine arbitrar


a.)
(
r X noteaz
Demonstratie. Cu teorema lui Thales se arat
a usor c
a + 1 = 0
AA0 k BB 0 .
A0 B
B0C
A
a c
a
Din = 0 si = 0 urmeaz
AC
BA
B
1

r A0 =
rB
r C,
X
1
1
1

rC
r A.
r B0 =
1
1

14

Tinnd

cont de aceste relatii, ecuatiile vectoriale ale cevienelor: (AA0 )


r =
rA+

u ( r A0 r A ), (BB 0 ) r = r B + v ( r B 0 r B ) se scriu sub forma


u
u

(AA0 )
r = (1 u)
rA+
(4)
rB
r C,
1
1
v
v

rC
r A.
r = (1 v)
rB+
(5)
(BB 0 )
1
1

Vectorul
r X asociat punctului X de intersectie se obtine din (4) sau (5) pentru u
sau v luat dintr-o solutie (u, v) a sistemului liniar de ecuatii
v
u
u
v
1u=
,
= 1 v,
=
.
(6)
1
1
1
1
G
asim, cu usurinta, ca solutie a sistemului (6) perechea (u, v) cu
1

u=
, v=
.
(7)
+ 1
+ 1

Dup
a nlocuirea lui u sau v din (7) n (4) sau (5), obtinem pentru
r X reprezentarea
(3), q.e.d.
Propozitia 2. Triunghiurile ABC, X X X si Y Y Y au acelasi centru de
greutate.
Demonstratie. Vom ar
ata c
a 4X X X si 4ABC au acelasi centru de greutate
(la fel se procedeaz
a cu perechea format
a din 4Y Y Y si 4ABC). Este suficient s
a
stabilim c
a

r X +
r X +
r X =
r A+
rB +
r C.
(8)
ntr-adev
ar, utiliznd Lema relativ la 4ABC si cevienele AA si BB , obtinem
1

r X =
r B
r C) ;
(9)
(
r A+
+ 1
similar obtinem si relatiile:
1

r X =
r B +
r C)
(4CAB si CC , AA ),
(10)
(
r A
+ 1
1

r X =
r B +
r C)
(4BCA si BB , CC ). (11)
(
r A +
+ 1

Tinnd

seama de (9), (10) si (11), avem

r X +
r X +
r X =
( + 1 )
rA
+ 1

+ (1 + )
r B + ( + + 1)
rC

=
r A+
r B +
r C.
adic
a are loc (8), q.e.d.

3. S
a presupunem c
a triunghiul ABC este echilateral. Se constat
a usor, pe cale
elementar
a si ca o consecinta a relatiilor (1), c
a triunghiurile X X X si Y Y Y
sunt, la rndul lor, echilaterale. Conform Propozitiei 2, aceste triunghiuri au acelasi
centru ca si triunghiul ABC. Este evident
a, n acest caz particular, nrudirea cu un
rezultat remarcabil, teorema lui Barbilian: doua triunghiuri echilaterale cu acelasi
centru sunt triomologice.
15

Constructii aproximative cu rigla si compasul ale


num
arului
Alexandru MOSCALIUC 1
Notatia pentru raportul dintre lungimea unui cerc si diametrul s
au s-a ncet
atenit
n matematic
a datorit
a lui L. Euler, care a utilizat-o n tratatul s
au Introductio in
analysis infinitorum (1748). Valori aproximative ale lui au fost utilizate nc
a din
antichitatea timpurie de multe popoare. Amintim doar c
a Arhimede, n tratatul
10
1
Asupra masurarii cercului, a g
asit c
a3
< < 3 prin asa-numita acum metoda a
71
7
perimetrelor (cea cu poligoanele regulate nscrise si circumscrise).
n strns
a leg
atur
a cu identitatea num
arului este problema cuadraturii cercului
constructia cu rigla si compasul a unui p
atrat de arie egal
a cu aria unui cerc dat;
problema revine la rectificarea cercului constructia cu aceleasi instrumente a unui
segment de lungime egal
a cu lungimea unui cerc dat ce se reduce la rndu-i la
constructia cu rigla si compasul a unui segment de lungime .
Aceast
a problem
a celebr
a formulat
a de grecii antici si-a g
asit rezolvarea n anul
1882, cnd F. Lindemann a dovedit c
a este transcendent (adic
a nu-i num
ar
algebric). Gratie acestui rezultat si faptului c
a numerele ce se pot construi cu rigla si
compasul formeaz
a o parte a multimii numerelor algebrice, rezult
a c
a este imposibil
a
cuadratura cercului.
Putem aproxima, ns
a, num
arul cu numere constructibile cu rigla si compasul.
Scopul acestei lucr
ari este de a da o astfel de aproximare a lui si cteva aplicatii
ilustrative, ntr-o prezentare accesibil
a elevilor de cl. a IX-a.
Propozitie. Are loc urmatoarea inegalitate:

2 + 3 0, 01 < < 2 + 3,
(1)

i.e. 2 + 3 aproximeaza numarul prin adaos cu o eroare mai mica de 0,01 .


Solutie. Fie l, L lungimile laturilor poligoanelor reguA
l B
late cu n laturi nscris si respectiv circumscris unui cerc
de raz
a egal
a cu 1. ntre perimetrele acestor poligoane si
lungimea cercului avem relatia
1
nl < 2 < nL.
(2)
O

360
180
0 OB 0 ) =
\ = m(A\
Deoarece m(AOB)
si l = 2 sin
,
n
n

180
1
L = 2 tg
, relatia (2) se scrie
n
180
180
B
L A
n sin
< < n tg
.
(3)
n
n
Lund n (3) n = 60, obtinem
(4)
60 sin 3 < < 60 tg 3 .

Tinnd

seama c
a 3 = 18 15 , vom avea
tg 18 tg 15
sin 3 = sin 18 cos 15 sin 15 cos 18 si tg 3 =
.
(5)
1 + tg 18 tg 15
1

Profesor, S
coala general
a nr. 6, Botosani

16

r
q

1
1
2

sin 18 =
5 1 , cos 18 =
10 + 2 5, tg 18 = 1
5,
4
4
5

6+ 2
1
6 2 , cos 15 =
sin 15 =
, tg 15 = 2 3,
4
4
q
inegalit
atile (4) se scriu:

1 25 5 2 3

60
q
51
6 + 2 10 + 2 5 6 2 < < 60
.

16
1+ 1 2 5 2 3
Cum

(6)
Printr-un calcul de rutin
a
anevoios

s
i
nepl
a
cut
se
verific
a
faptul
c
a
membrul
stng

din (6) este mai mare ca 2 + 3 0, 01, pe cnd cel drept este mai mic ca 2 + 3.
n concluzie, inegalit
atile (1) sunt adev
arate.

Observatie. Constructia cu rigla si compasul a unui segment de lungime 2+ 3
(n prezenta unui segment unitate) este elementar
a. Ca urmare, Propozitia ofer
ao
modalitate de a construi aproximativ num
arul cu rigla si compasul.
n aplicatiile urm
atoare ale Propozitiei se face cuadratura/rectificarea unui cerc cu
rigla si compasul n mod aproximativ, adic
a se construieste cu aceste instrumente
un p
atrat/segment avnd aria/lungimea aproximativ aria/lungimea cercului dat.

Aplicatia 1. Fie ABC un triunghi isoscel cu AB = AC = 3, BC = 2 2


si C (I, r) cercul nscris acestuia. Atunci lungimea cercului C (I, r) este aproximativ
egala cu BC, iar aria lui este aproximativ egala cu aria 4BIC; n ambele situatii
eroarea aproximarii fiind mai mica ca 0, 01.
Solutie. Avem: AD2 = AB 2 BD2 =
A
1, deci AD = 1 si

2
S
AD BC
.
r= =
=
I
p
AB + BC + AC
2+ 3

Tinnd

cont de faptul c
a ' 2 + 3,
pentru cercul C (I, r) obtinem:

C
B
D

2
= 2 2, adic
2+ 3
a L ' BC;
L = 2r ' 2
2+ 3

!2


2
2
2

, adic

2+ 3
=
a A ' ABIC
A = r '
2+ 3
2+ 3

2
1
1
2
=
).
(ntr-adev
ar, ABIC = BC ID = 2 2
2
2
2+ 3
2+ 3
S
a dovedim c
a, n formulele g
asite, L si A suntaproximate
cu o eroare mai mic
a

2
+
3

<
0,
01
(adev
a
rat
a
dect 0, 01. ntr-adev
ar, nmultind inegalitatea

2 + 3 L < 0, 01 2r sau
conform Propozitiei!) cu 2r, obtinem 2
2+ 3

2 2
< 1, urmeaz
BC L < 0, 01 2r. Cum 2r =
a c
a BC L < 0, 01. La fel,
2+ 3
2
dar nmultind aceeasi inegalitate cu r , obtinem ABIC A < 0, 01.
17

Aplicatia 2. Fie cercul C (O, 1) si punctele A, B,


C si D ca n figura de mai jos: BC = 1, BD = AC.

D
C

Aratati ca lungimea semicercului AB (aria semicercului) este aproximativ egala cu lungimea segmentului
1
[AD] (respectiv aria triunghiului ABD), eroarea fiind
A
B
mai mica dect 0, 01.
O 1
Solu
t
ie.
Deoarece
AB
=
2

s
i
BC
=
1,
rezult
a
c
a

AC = 3; la fel, din
BC = 1 si BD = AC = 3,
deducem c
a CD = 2. Atunci, AD = AC + CD =

1
1
1
2 + 3 ' si AABD = AD BC =
2+ 3 ' =
12 etc.
2
2
2
2
Aplicatia 3. Dat un patrat de latura 1, construiti numai cu compasul un cerc
de lungime aproximativ egala cu perimetrul patratului.
Solutie. Mai nti, s
a observ
am c
a un cerc de lungime egal
a cu perimetrul p
atra

2
2
2
=2 3 2 .
tului dat are raza . Dar, tinnd cont de Propozitie, '

2
+ 3
Asadar, urmeaz
a s
a construim cu compasul un cerc de raz
a 2 3 2 .
Etapele unei posibile constructii sunt:
D
C
1. Construim simetricul E al punctului B fata de A:
{E} = C (A, 1) C (D, DB).
2. Construim punctul F astfel nct 4BEF s
a fie
1
E
A
B
echilateral, iar F si D s
a fie de o parte si de alta a
dreptei BE: {F } = C (B, BE)

C
(E,
EB);
evident,
A,

D, F sunt coliniare si AF = 3 (n
altime n 4BEF de
latur
a 2).
3. Construim punctul G de partea dreptei BE n care
G
se afl
a F prin {G}
=
C
(A,
AC)

C
(B,
AF
).
Deoarece

F
AB = 1, AG = 2 si BG = 3, rezult
a c
a 4AGB este
H
dreptunghic n A si, ca urmare,
punctele
A, F , G sunt
coliniare, iar F G = AF AG = 3 2.
4. Construm simetricul H al lui G fata de F (constructia,
P
numai cu compasul, a simetricului M 0 al punctului M fata
de un punct
arit
a pe figura al
aturat
a); evident
fi urm
O poate
M O
M
GH = 2 3 2 .
5. Construim
C(H, HG),
care
va
fi
cercul
c
a
utat:
lun

gimea lui este 4 3 2 ' 4 2 + 3


3 2 = 4,
Q
cu o eroare
de

h
i

4
3 2 4=4
3 2
3+ 2 <4
3 2 0, 01,

1
conform cu (1). Cum 3 2 < , vom avea
2


1
3 2 4 < 4 0, 01 = 0, 02 ,
4
2
adic
a eroarea cu care lungimea cercului construit este aproximat
a de perimetrul p
atratului este mai mic
a dect 0, 02.
18

Inegalit
ati generatoare de noi inegalit
ati
I. V. MAFTEI 1
Pornind de la anumite inegalit
ati cunoscute ne propunem s
a obtinem noi inegalit
ati.
Propozitia 1. Sa se demonstreze ca

+ xn1
+ + xn1
+ xn+k1
+ + xn+k1
,
xn+k1
x1 x2 xk xn1
1
2
1
2
k
k
x1 , x2 , . . . , xk

R+ ,

n, k N,

(1)

n, k 2.

Demonstratie. Utiliznd relatia dintre mediile aritmetic


a si geometric
a, aplicat
a
numerelor a1 , a2 , . . . , ak R+ , n, k N, n, k 2, obtinem succesiv:
p n
na1 + a2 + + ak
n+k1
a1 a2 ak
,
n+k1
p n
a1 + na2 + + ak
n+k1
a1 a2 ak
,
(2)
n+k1
.......................................
p
a1 + a2 + + nak
n+k1
a1 a2 ank
.
n+k1
Sumnd inegalit
atile (2), rezult
a c
a
p n
p n
p
n+k1
n+k1
a1 a2 ak +
a1 a2 ak + + n+k1 a1 a2 ank a1 + a2 + + ak .

Dac
a not
am n+k1 ai = xi , i = 1, k, obtinem
+ xn+k1
+ + xn+k1
,
xn1 x2 xk + x1 xn2 xk + + x1 x2 xnk xn+k1
1
2
k

care este tocmai inegalitatea (1).

Pentru k = 2 si n = 2h, h N , inegalitatea (1) devine

x2h+1
+ x2h+1
x1 x2 x2h1
+ x2h1
.
1
2
1
2

(3)

Propozitia 2. Fie a, b, c R+ si k N. Atunci, are loc inegalitatea


(ab)k1
k1

a2k+1 + b2k+1 + (ab)

(bc)k1
k1

(ac)k1

b2k+1 + c2k+1 + (bc)


a2k+1 + c2k+1 + (ac)
1
1
1

+
+
.
ab (a + b) + 1 bc (b + c) + 1 ac (a + c) + 1

k1

(4)

Demonstratie. Aplicnd inegalitatea (3) de k ori, obtinem


+ x2k+1
(x1 x2 )k (x1 + x2 ) ,
x2k+1
1
2

x1 , x2 R+ ,

k N.

(5)

Tinnd

seama de (5), putem scrie


de unde

a2k+1 + b2k+1 (ab)k (a + b) ,


a2k+1 + b2k+1 + (ab)

k1

k1

(ab)

Profesor, Colegiul National "Sf. Sava", Bucuresti

19

[ab (a + b) + 1]

(6)

sau

k1

1
.
ab (a + b) + 1
+
+ (ab)
Sumnd aceast
a inegalitate cu analoagele ei, obtinem (4).
Observatie. Dac
a n (4) lu
am k = 2 si consider
am abc = 1, suntem condusi la
inegalitatea
bc
ca
ab
+
+
1,
(7)
a5 + b5 + ab b5 + c5 + bc c5 + a5 + ca
care a fost discutat
a la O. I. M. din anul 1996, India.
(ab)

a2k+1

b2k+1

k1

Propozitia 3. Fie numerele a, b, c R+ . Sa se demonstreze ca

a2n+1 b2n+1 c2n+1


+ n +
an b + bn c + cn a, n N.
(8)
bn
c
an
Demonstratie. nmultind inegalitatea (6), considerat
a pentru k = n, cu an cn ,
n n
n n
iar analoagele ei cu b a si respectiv b c , vom obtine relatiile
a3n+1 cn + b2n+1 an cn a2n+1 bn cn + a2n bn+1 cn ,
b3n+1 an + c2n+1 bn cn b2n+1 an cn + b2n cn+1 an ,
c3n+1 bn + a2n+1 cn bn c2n+1 bn an + c2n an+1 bn ,

din care, prin adunare, deducem c


a
adic
a (8) .

a3n+1 cn + b3n+1 an + c3n+1 bn an bn cn (an b + bn c + cn a) ,

Procednd ca n Propozitia 3 se obtine


Propozitia 4. Pentru n N si a, b, c R+ avem
a

c2n+2
a2n+2
b2n+2
+
b
+
c
abn+2 + bcn+2 + can+2 .
cn
an
bn

(9)

Propozitia 5. S
a se arate c
a n N avem:
q
n (n + 1) n
(n!)2k ,
(10)
a) 12k+1 + 22k+1 + + n2k+1
2
n (n + 1)
b) 12n+1 + 22n+1 + + n2n+1
(11)
(n!)2 .
2
Demonstratie. Pentru n = 1 avem egalitate. Consider
am n 2 si nlocuim n
inegalitatea (6) succesiv a = 1 si b = n, a = 2 si b = n 1, . . . , a = n si b = 1.
Sumnd inegalit
atile rezultate, vom obtine
i
h
2k+1

2 1
+ 22k+1 + + n2k+1 (n + 1) 1k nk + 2k (n 1)k + + nk 1k .
q
q
2
n
n
Cum paranteza p
atrat
a este n (1k 2k nk ) = n (n!)2k , avem
q

n
2k
2 12k+1 + 22k+1 + + n2k+1 n (n + 1) (n!) ,
adic
a (10). Lund n (10) k = n, obtinem inegalitatea (11).
20

Asupra unei probleme dat


a la ONM, Bistrita, 2005
Claudiu-Stefan

POPA1
Cele ce urmeaz
a au ca punct de plecare o problem
a dat
a la ONM, Bistrita, 2005
[1] apartinnd autorului acestei note si pe care o vom nota n continuare cu (P ):
(P ) Fie ABCD un trapez cu bazele AB si CD, avnd diagonalele perpendiculare
n O. Pe semidreptele (OA si (OB se considera punctele M si respectiv N astfel
\
\
nct unghiurile AN
C si BM
D sa fie drepte. Notam cu E mijlocul segmentului
M N . Sa se arate ca:
a) triunghiurile OM N si OBA sunt asemenea;
b) dreapta OE este perpendiculara pe dreapta AB.
Rezolvarea acestei probleme poate fi g
asit
a de asemenea n [1].
Consider
am configuratia geometric
a pus
a n valoare de (P ) ndeajuns de generoas
a
pentru a prezenta alte cteva rezultate legate de ea. D
am nti o caracterizare a
trapezului ortodiagonal, interesant
a si n sine.
Propozitia 1. Fie ABCD un patrulater convex si AB k CD. Daca punctul O
este intersectia diagonalelor sale, patrulaterul este ortodiagonal daca si numai daca
AB CD = AO CO + BO DO.
Demonstratie. AB k CD 4AOB 4COD
AO
BO DO
AB CD
AO CO + BO DO
BO
AB
AO CO
=
=
=
.
=
=
=
2
2
2
CO
DO
CD
CO
DO
CD
CO2 + DO2
Acum AO CO + BO DO = AB CD CD2 = CO2 + DO2 AC BD, q.e.d.
Ad
aug
am la ipoteza problemei (P ): punctele K, L sunt mijloacele bazelor [AB],
respectiv [CD] iar punctul D0 este simetricul punctului D fata de punctul O. n
aceste conditii, pentru cele ce urmeaz
a presupunem cunoscute urm
atoarele: punctele
K, O si L sunt coliniare, AAOD = ABOC , A2AOD = AAOB ACOD ([2], p. 243).
Propozitia
2. n ipoteza problemei (P ), au loc urmatoarele:
i) M N = AB CD si M N < KL;
ii) M N KL;
iii) AOM N = AAOB ACOD ;
iv) AN k M D0 ;
Demonstratie. i) 4AN C si 4BM D sunt dreptunghice n N , respectiv M si
N O AC, M O BD. Aplicnd teorema n
altimii obtinem N O2 = AO CO si
2

\
M O = BO DO. Cum m(M ON ) = 90 , avem M O2 + N O2 = M N 2 si obtinem

AB
CD
AB + CD
M N = AB CD. Deoarece KL = KO + LO =
+
=
si
2
2
2

AB + CD
AB CD <
(ABCD trapez, deci AB 6= CD), rezult
a c
a M N < KL.
2
ii) Fie R (OL astfel nct L (OR) si (OL) (RL). Cum (DL) (CL),
urmeaz
a c
a OCRD este paralelogram. Dar CO DO, deci OCRD este dreptunghi si
1

Profesor, S
coala "Alecu Russo", Iasi

21

\ CRO.
\ Din (P ), punctul a) avem CDO
\N
\
\N
\
avem CDO
M O; deci CRO
M O.
Aceasta si M O CR conduc la N M RO N M KL.
ON OM
iii)
C
D
AMON =
=

2
AO CO BO DO
=
=
O
2

AO CO BO DO
N
.
=
D
2
P
Cum 4AOB 4COD, avem AO DO =
E
AO DO
A
BO CO, deci AMON =
= AAOD .
B
K
2
2
M
Dar
A
=
A
A
,
deci
A
=
AOB
COD
OMN
AOD
= AAOB ACOD .
ON
OD
ON
OD0
iv) La fel ca la iii), OM ON = OA OD. Deci
=
sau
=
,
OA
OM
OA
OM
0
adic
a AN k M D , q.e.d.

Observa
tie. Dac
a M Np LK = {P }, propunem cititorului s
a demonstreze c
a

OE = LO KO si OP = dist (O; AB) dist (O; CD).


Bibliografie

1. G.M. seria B, nr. 7/2005, p.298 si p. 301.


2. D. Mihalca, I. Chitescu, M. Chirita
- Geometria patrulaterului, Ed. Teora, Bucuresti, 1998.

ERATA
Mai multti colaboratori aduc la cunostinta Redactiei revistei urm
atoarea greseal
a
n scrierea numelui marelui matematician Leonhard Euler : n loc de Leonhard s-a
scris Leonard att n titlul materialului din nr. 2/2004, p. 129, ct si n cel din nr.
2/2005, p. 119 (prin preluarea primului pe calculator).
COMENTARIU
D-l D. Plaesu din Iasi semnaleaz
a Redactiei faptul c
a Problema L.62, autor M.
Brsan, publicat
a n nr. 1/2004 este cunoscut
a apare n cartea lui W. Sierpinski
intitulat
a Ce stim si ce nu stim despre numerele prime (n l. rom. la Editura S
tiintific
a, Bucuresti, 1966) la p. 104. Cele dou
a solutii date acestei probleme n nr.
1/2005, pp. 67-68, difer
a de solutia prezentat
a n cartea mentionat
a.

Vizitati pe Internet revista "Recreatii Matematice" la adresa

http://www.recreatiimatematice.uv.ro
22

Asupra criteriului de congruenta


LLU
Marius TIBA1
Asa cum se arat
a n [1], urm
atorul criteriu de congruenta a triunghiurilor ntinde
numeroase capcane prin aplicarea sa incorect
a. Red
am aici rezultatul din care decurge acest criteriu.
Propozitie. Daca doua laturi si unghiul opus uneia dintre ele ale unui triunghi
sunt respectiv congruente cu doua laturi si unghiul opus uneia dintre ele ale altui
b noteaza unghiurile opuse celorlalte laturi congruente,
triunghi, iar
b si
b sunt sau congruiente sau suplementare;
a) atunci
b si
b sunt de acelasi tip (adica ambele sunt
b) (Criteriu LLU) daca n plus
b si
ascutite, obtuze sau drepte), atunci cele doua triunghiuri sunt congruente.
\ se iau
Problema 1. Pe laturile (OX si (OY ale unui unghi ascutit XOY
punctele A si B astfel nct [OA] [OB]. n interiorul unghiului se ia un punct M
\
\
supus conditiei OM
A OM
B. Gasiti locul geometric descris de M .
Rezolvare. Vom ar
ata c
a locul c
autat este format din
O
\ si arcul cercului cirbisectoarea interioar
a a unghiului XOY
cumscris 4AOB cuprins n interiorul unghiului, pe care-l
not
am AB. Fie M un punct ce satisface conditiile din enunt.
\ si OBM
\
Ca urmare 4OAM si 4OBM au unghiurile OAM
sau congruente sau suplimentare (conform punctului a) al
\ OBM
\ , punctul M se
Propozitiei). n cazul n care OAM
b
afl
a pe bisectoarea unghiului O. Dac
a aceste unghiuri sunt
suplimentare, atunci patrulaterul OAM B este inscriptibil si,
ca urmare, M AB. Reciproca rezult
a imediat.

A
X

B
M

Mention
am c
a la faza judetean
a a O. M. din Vaslui, 2005, cl. a VI-a, s-a cerut
\.
s
a se arate c
a locul geometric este doar bisectoarea interioar
a a unghiului XOY
Nerespectarea Propozitiei conduce la erori ca aceasta (prezent
a si n barem) sau cea
din problema T12, [2, pag. 10].
Problema 1 ne sugereaz
a
\ OBM
\,
Problema 2. Gasiti locul geometric al punctelor M pentru care OAM
unde notatiile sunt aceleasi ca n Problema 1.
Rezolvare. Judecnd analog ca la Problema 1, obtinem locul c
autat format din
\ si segmentul [AB] (f
bisectoarea interioar
a a unghiului XOY
ar
a capete).
Ca o extindere a Problemei 1, d
am urm
atoarea
Problema 3. Fie ABCD un trapez isoscel si fie O mijlocul bazei mici [CD].
Gasiti locul geometric al punctelor M situate n interiorul liniei frnte formate din
\
\
semidreptele (DA, (CB si segmentul [CD], astfel nct OM
A OM
B.
1

Elev, cl. a VII-a, Colegiul National "C. Negruzzi", Iasi

23

Rezolvare. Aceast
a problem
a se reduce la Problema 1,
deoarece [OA] [OB] (4OBC 4OAD (LUL)). Astfel,
locul geometric cerut este format din punctele mediatoarei
segmentului CD si ale arcului cercului circumscris 4OAB,
aflate n interiorul liniei frnte date.
Problema 3 ne sugereaz
a urm
atoarele dou
a probleme, pe
care le propunem cititorului spre rezolvare:
Problema 4. Se modifica Problema 3 lund punctul O
la intersectia laturilor neparalele ale trapezului.
Problema 5. Modificam Problema 3, lund conditia
\
\
AM
D BM
C.

B
M

Bibliografie
1. D. Mihet - Criteriul de congruenta LLU, RMT an II (seria a 4-a), nr. 2, 1997,
pag. 3-7.
2. I. P
atrascu - Probleme de geometrie plana, Editura Cardinal, Craiova, 1996.

IMPORTANT
n scopul unei leg
aturi rapide cu redactia revistei, pot fi utilizate urm
atoarele
adrese e-mail: tbirsan@math.tuiasi.ro, profgpopa@yahoo.co.uk . Pe
aceast
a cale colaboratorii pot purta cu redactia un dialog privitor la materialele trimise acesteia, procurarea numerelor revistei etc. Suger
am colaboratorilor care trimit probleme originale pentru publicare s
a le numeroteze si
s
a-si retin
a o copie xerox a lor pentru a putea purta cu usurinta o discutie
prin e-mail asupra accept
arii/neaccept
arii acestora de c
atre redactia revistei.
La problemele de tip L se primesc solutii de la orice iubitor de matematici
elementare (indiferent de preocupare profesionala sau vrsta ). Fiecare dintre
solutiile acestor probleme - ce sunt publicate n revist
a dup
a un an - va fi
urmat
a de numele tuturor celor care au rezolvat-o.
Adres
am cu insistenta
amintea ca materialele trimise revistei
rug
s
a nu fie (s
a nu fi fost) trimise si altor publicatii.
Rug
am ca materialele tehnoredactate s
a fie trimise pe adresa redactiei nsotite de fisierele lor (de preferinta n LATEX).
24

O generalizare a identit
atii Botez - Catalan
Ioana OLAN 1
n 1872, N. S
t. Botez public
a o lucrare
original
a n care apare identitatea

1
1
1
1
1
1
1
+
++
=1
+
+ +
, n N ,
n+1 n+2
2n + 1
2 13 25
n (2n + 1)
care, dac
a tinem seama de formula de descompunere
1
1
1
=

, k N ,
2k (2k + 1)
2k 2k + 1
se aduce la forma
1
1
1
1 1 1
1
1
+
+ +
= 1 + + +

, n N , (1)
n+1 n+2
2n
2 3 4
2n 1 2n
numit
a identitatea Botez - Catalan. Ne propunem s
a-i d
am o generalizare.
Amintim o demonstratie a formulei (1), generalizarea obtinndu-se n acelasi fel:

1
1
1
1 1
1
1
1 1 1

= 1
+

++

=
1 + ++
2 3 4
2 3 4
2n

2n
1 2n
1 2n
1
1
1 1
1
1
1
1
= 1+ 2
+
+ 2
+ +
+
2
=
2
2
3 4
4
2n 1 2n
2n

1 1 1
1
1
1 1
1
= 1 + + + + +
+
2
+ + +
=
2 3 4
2n 1 2n
2 4
2n

1 1
1
1
1
1
1
1
1
= 1 + + + +
+
1 + + +
=
+
+ +
.
2 3
2n1 2n
2
n
n+1 n+2
2n
Propozitie. Pentru n N si m N, are loc egalitatea
2m 1
1
2m 1
1
2m 1
1
1
1
+ m
+ +
m
m =
m + +
m . (2)
m
m
2
3
4
(2n 1)
(2n)
(n + 1)
(2n)
(Pentru m = 1 se obtine identitatea (1).)
Demonstratie. ntr-adev
ar, avem:
2m 1
1
2m 1
1
2m1
1
+ m
+ +
m
m =
m
m
3
4
(2n
(2n)

1)

1
1
1
1
1
1
1
1
m
= 1+ m 2m m + m + m 2m m + +
+
2
=
2
2
3
4
4
(2n 1)m (2n)m
(2n)m

1
1
1
1
1
1
1
1
m
= 1 + m + m + m + +
+

2
+
+

+
=
2
3
4
2m 4m
(2n 1)m (2n)m
(2n)m

1
1
1
1
1
1
= 1 + m + m + +
=
+

1
+
+

+
m
m
2
3
2m
nm
(2n 1)
(2n)
1
1
1
, q.e.d.
=
m +
m + +
(n + 1)
(n + 2)
(2n)m
Cazuri particulare. Pentru m = 2 si m = n, formula (2) devine:
3
1
3
1
3
1
1
1
+

+ +

=
+
+ +
,
22 32 42
(2n1)2 (2n)2
(n+1)2 (n+2)2
(2n)2
2n 1 1 2n 1
1
2n 1
1
1
1
1 n + n n + +
n
n =
n+
n + +
n.
2
3
4
(2n1)
(2n)
(n+1)
(n+2)
(2n)
1

Elev
a, cl. a VIII-a, Colegiul National "C. Negruzzi", Iasi

25

Acoperiri ale planului laticial cu figuri


1
Marius PACHITARIU

Exist
a numeroase probleme de concurs care implic
a acoperiri ale unor figuri laticiale, de exemplu dreptunghiuri, cu un num
ar de copii ale unei alte figuri date (dominouri, trominouri) sau cu alte copii scalate ale nsusi dreptunghiului.
Observnd metodele si tehnicile acestor tipuri de probleme, putem analiza si
acoperiri ale ntregului plan cu diferite figuri. Vom lucra n continuare doar n planul
laticial. Pentru aceasta, s
a definim planul laticial si s
a d
am coordonate p
atr
atelelor
care l alc
atuiesc. Introducem n continuare o serie de notiuni.
Consider
am dreptele de ecuatie x = r, r Z, y = q, q Z. P
atratele de latur
a1
determinate de punctele lor de intersectie vor constitui elementele planului laticial.
Vom atribui coordonate acestor p
atrate n urm
atorul mod: dac
a un p
atrat se afl
a la
intersectia benzilor determinate de x = r, x = r + 1 si respectiv, y = q, y = q + 1,
atunci vom spune c
a p
atratul are coordonate r, q sau mai simplu vom numi p
atratul
[r, q]. Vom numi vector ntre dou
a p
atrate vectorul ai + bj, cu i, j versorii axelor si
a, b num
arul de p
atr
atele orizontale, respectiv verticale care separ
a cele dou
a p
atrate
considerate, cu semnul asociat corespunz
ator.
Numim plantatie orice colectie de p
atrate ale planului laticial, conex
a n sensul
c
a din orice p
atrat putem ajunge n oricare altul printr-o succesiune finit
a de deplas
ari unitare (translatii dup
a una din cele 4 directii), astfel nct dup
a fiecare pas
(translatie) ne afl
am nc
a n unul dintre p
atratele colectiei. Numim figura multimea
maximal
a de plantatii cu proprietatea c
a fiecare plantatie poate fi obtinut
a din oricare alta a figurii prin translatii, rotatii si simetrii fata de drepte paralele cu axele.
Cu alte cuvinte, figura reprezint
a o clas
a de echivalenta.
Fie un set X de figuri. Numim acoperire a planului orice set Y de plantatii cu
elemente apartinnd figurilor din X, astfel nct fiecare punct al planului apartine
la cel putin o plantatie din Y . Numim masura unei acoperiri sup n[i, j], unde n[i, j]
este num
arul de plantatii c
arora i apartine patratul [i, j]. Numim n-acoperire o
acoperire n care fiecare p
atrat al planului apartine aceluiasi num
ar de plantatii
modulo n. Pentru 2-acoperiri vom considera acoperire impar
a cea n care fiecare
p
atrat e acoperit de un num
ar impar de ori. 2-acoperirile pare nu ne intereseaz
a,
ntruct considernd acceasi plantatie de dou
a ori si considernd oricte astfel de
perechi, vom obtine ntotdeauna o 2-acoperire par
a.
Observatia 1. Reuniunea a dou
a acoperiri este o acoperire si m
asura reuniunii
a dou
a acoperiri este cel mult suma m
asurilor celor 2 acoperiri.
Putem acum s
a ne punem o serie de ntreb
ari:
ntrebarea 1. Care sunt seturile X de cardinal 1 pentru care exist
a acoperiri cu
m
asura 1?
Un exemplu netrivial de figuri n spatiu cu aceast
a proprietate l ofer
a urm
atoarea
Problem
a. Lipim cte un cub unitate pe fiecare fata a unui cub unitate. Ar
atati
c
a putem umple spatiul folosind copii ale solidului rezultat. (Austrian-Polish 2000)
1

Elev, Colegiul National, Iasi

26

ntrebarea 2. Care sunt seturile X de cardinal 2 pentru care exist


a acoperiri cu
m
asura 1?
ntrebarea 20 . Dati exemple de doua seturi Y si Z de cardinal 1 pentru care nu
exista acoperire cu m
asura 1 cu setul Y si nu exist
a acoperire
asura 1 cu setul
S cu m
Z, dar pentru care exist
a acoperire cu m
asura 1 cu setul Y Z.
Vezi la pagina 74 exemplul 1. Demonstrati c
a exemplul este ntr-adev
ar bun!
ntrebarea 200 . Dati exemple de dou
a seturi de figuri de cardinal 1: Y , pentru
care nu exist
a acoperire cu m
asura 1 si S
Z, pentru care exist
a o astfel de acoperire si
pentru care exist
a acoperire cu setul Y Z cu m
asura 1.
Vezi la pagina 74 exemplul 2. Demonstrati c
a exemplul este ntr-adev
ar bun!
ntrebarea 3. Care sunt seturile X de figuri de cardinal n, n N, pentru care
exist
a acoperiri cu m
asura 1?
0
ntrebarea 3 . Dati exemple de seturi de n seturi Xi , i = 1, n de figuri, de
cardinale 1, cu proprietatea c
a nici unul dintre ele nu poate genera o acoperire cu
m
asura 1, dar reuniunea lor da.
Vezi la pagina 74 exemplul 3. Demonstrati c
a exemplul este ntr-adev
ar bun!
ntrebarea 4. Orice figur
a poate genera o 2-acoperire impar
a a planului cu
m
asura finit
a? Dac
a vom considera minimul m
asurii peste toate acoperirile posibile,
putem g
asi un maxim pentru acesta n functie de figura folosit
a?
Nu vom r
aspunde aici ntreb
arilor 1, 2, 3, fiind prea generale. Desigur, seturile
X care constituie r
aspunsurile primelor 2 sunt particulariz
ari ale seturilor din a treia
ntrebare. Caracteriz
ari ale primului tip de seturi din anumite puncte de vedere sunt
cu siguranta posibile, pe cnd o caracterizare n cazul general pare imposibil
a (vezi
exemplele ntreb
arilor 20 , 200 , 30 ).
Vom r
aspunde n schimb ultimei ntreb
ari. R
aspunsurile sunt DA si DA. Acoperirile modulo 2 ne dau ntr-adev
ar de ajuns
a libertate. Pentru demonstratiile urm
atoare
vom renunta la conditia de conexitate din definitia plantatiilor.
Demonstratia 1. Vom numi cardinal al unei plantatii (figuri) num
arul de
p
atr
atele pe care le contine. Pentru o figur
a de cardinal impar putem lua urm
atoarea
acoperire: Fie o plantatie oarecare si toate translatiile ei care se p
astreaz
a pe latice.
Atunci plantatiile rezultate si cu cea initial
a reprezint
a o 2-acoperire. ntr-adev
ar,
fiecare p
atrat al planului laticial este acoperit de exact (cardinalul plantatiei)-ori,
deci pentru cardinalul impar avem o 2-acoperire. Mai mult, avem m
asura acoperirii
egal
a cu cardinalul figurii.
Din p
acate aceast
a cale nu pare s
a furnizeze solutie pentru figurile de cardinal
par.
Demonstratia 2. Figura F fiind finit
a, o putem include ntr-un p
atrat cu laturile
pe latice, deci o putem include ntr-un p
atrat cu latura putere a lui 2. Fie 2k latura
celui mai mic astfel de p
atrat. S
a consider
am acoperirea de m
asur
a 1 cu astfel de
p
atrate de latur
a 2k (chiar o omotetie a planului laticial). Plas
am n fiecare p
atrat de
latur
a 2k al acestei acoperiri figura F corespunz
atoare. Asociem p
atr
atelelor unitate
ale p
atratului de latur
a 2k valoarea 1 dac
a p
atr
atelul este n F si 0 altfel. Cu alte
cuvinte am f
acut o prim
a acoperire a planului cu figuri F . Vom face o serie de pasi.
27

Primul pas: Consider


am acoperirea obtinut
a ca mai sus, dar nlocuind figura F
cu simetrica ei fata de (Oy, astfel nct aceast
a acoperire s
a suprapun
a seturile de
p
atrate de latura 2k peste cele considerate anterior. Fie F 0 noua figur
a obtinut
a din
suprapunerea celor dou
a acoperiri (reuniunea) si considerarea modulo 2 (un patr
atel
are asociat 1 dac
a este acoperit de un num
ar impar de plantatii si 0 altfel). Proced
am
analog cu F 0 dar fata de axa (Ox. Obtinem o nou
a figur
a F 00 , care este simetric
a si
fata de orizontal
a si fata de vertical
a, ncadrat
a n p
atratul de latur
a 2k . S
a observ
am
c
a fiecare p
atratel a fost acoperit de cel mult 4 ori.
Realiz
am acoperirile determinate de translatii ale acoperirii de mai sus cu figura
F 00 , de vectori 2k1 i, 2k1 j, 2k1 i + 2k1 j.
S
a consider
am reuniunea celor 4 acoperiri. Privind mai atent, observ
am c
a am
obtinut astfel o nou
a acoperire cu p
atrate de latur
a 2k1 , cu o figur
a F (2) n fiecare
p
atrat, simetric
a orizontal si vertical. Acest proces reprezint
a de fapt echivalentul
primei p
arti, ntruct acoperim figura din p
atratul de latur
a 2k1 cu simetrica ei
fata de axa vertical
a si apoi cu simetricele fata de axa orizontal
a a celor dou
a figuri
obtinute. Fiecare p
atrat a fost acoperit de cel mult 4 + 4 + 4 + 4 = 4 4 ori.
Putem continua acum cu urm
atorii pasi analogi cu partea a doua a pasului 1
(partea cu translatiile) p
an
a ajungem la acoperirea cu p
atratul de latura 1, care are
n interiorul lui figura F (k + 1), care poate fi ori multimea vid
a ori nsusi p
atr
atelul,
caz n care am obtinut o 2-acoperire impar
a a planului cu F (si o m
asur
a de cel mult
4 4 4 . . . 4 = 4k ).
Pentru a determina num
arul la care ajungem (0 sau 1) n F (k + 1), vom considera
aceeasi serie de pasi ca mai sus, urm
arind n acelasi timp cum evolueaz
a suma tuturor
valorilor asociate patr
atelelor interiorului p
atratului de latur
a 2k .
S
a presupunem c
a am considerat p
atratul de vrfuri [1, 1], [1, 2k ], [2k , 1], [2k , 2k ]
(ne referim aici la cele 4 p
atrate unitare) si fie ai,j = 1 dac
a p
atratul [i, j] este n
figura F si 0 altfel.
Vom da acum un exemplu de "pasi" f
acuti ca mai sus:
Exemplul 4.
1
1
1
0

1
0
0
1

0
1
0
0

1
0
0
0

0
1
1
0

1
1
0
1

1
1
0
1

0
1
1
0

0
0
0
0

0
1
1
0

0
1
1
0

0
0
0
0

1 1
1 1

S
a not
am bi,j , i = 1, 2k1 , j = 1, 2k1 valorile asociate acoperirii rezultate din
suma celor 4 acoperiri (f
ar
a a le reduce modulo 2; adun
am ai,j -urile corespunz
atoare
pentru a proba dac
a [i, j] este sau nu n F (2)). Acum este usor s
a observ
am c
a
bi,j = ai,j + ai,2k +1j + a2k +1i,j + a2k +1i,2k +1j . Dar aceasta nseamn
a c
a:
X
X
X

bi,j =
ai,j
ai,j + ai,2k +1j + a2k +1i,j + a2k +1i,2k +1j =
i2k1
j2k1

i2k1
j2k1

i2k
j2k

Am demonstrat prin aceasta c


a suma valorilor asociate r
amne aceeasi. Cu alte
cuvinte, valoarea din ultimul p
atrat dup
a ultimul pas este egal
a cu suma valorilor
initiale ale figurii F . Deci, dac
a figura F are un num
ar impar de p
atr
atele, atunci
28

figura F (k + 1) este constituit


a dintr-un p
atr
atel, adic
a avem o acoperire a planului.
Dar nu am obtinut acelasi lucru n demonstratia 1 si mult mai usor?
Ba da, dar demonstratia 1 nu ne perminte o rafinare pentru a o face s
a functioneze
si pe figuri de cardinal par, pe cnd aceasta da.
Dac
a la sfrsit nu obtinem un p
atr
atel, ci multimea vid
a, rezult
a c
a la un moment
dat, dup
a o simetrizare, figura s-a anulat pe sine ns
asi. Dar aceasta nu se ntmpl
a
dect n cazul n care figura era deja simetric
a fata de axa fata de care s-a f
acut simetria. Dar suprimnd aceast
a etap
a din pasul corespunz
ator putem continua operatiile
de simetrizare, ntruct figura e deja simetric
a fata de acea ax
a. Suprimnd toate
etapele n care am face o astfel de simetrie nedorit
a, ne p
astr
am pe linia rationamentului anterior si, mai mult, putem fi siguri c
a ajungem la o figur
a unitar
a nevid
a. S
a
consider
am, de exemplu, ultimul pas, plecnd de la cele 16 posibilit
ati si suprimnd
acele simetrii care nu sunt favorabile.
Exemplul 5.
1 1
1 1

STOP

Exemplul 6.

1 1
1 0

0 0
1 1

1 1
1 1

STOP

Exemplul 7.

1 1
0 0

1 1
0 0

1 1
1 1

STOP

Exemplul 8.

1 0
1 0

1 1
1 1

STOP

1 0
1 1
1 1
STOP
0 0
0 0
1 1
Nu este greu de observat c
a f
ar
a a suprima nici o simetrizare avem exact 4k
plantatii peste fiecare p
atr
atel. Acest num
ar reprezint
a toate p
atr
atelele p
atratului
2k 2k , deci ne furnizeaz
a o m
asur
a mai mare dect cea furnizat
a de demonstratia
1 pentru figuri de cardinal impar. Pentru cardinal par n schimb, putem considera
aceast
a m
asur
a ca un bun majorant pentru valoarea minim
a posibil
a a m
asurii. Chiar
mai mult, tinnd cont c
a am suprimat cel putin o etap
a, nseamn
a c
a nu am mai
dublat n momentul acela m
asura, deci obtinem cel mult 2 4k1 care pentru figuri de
ajuns de compacte si de bine ncadrate n p
atratul de latur
a 2k corespunz
ator poate
fi chiar mai mic dect cardinalul plantatiei.

Exemplul 9.

Problem
a. Color
am planul n alb si negru ca tabla de sah. Fie o plantatie
pentru care num
arul de p
atr
atele albe pe care le contine, A, este impar si num
arul
de p
atr
atele negre pe care le contine, B, este tot impar. S
a se gaseasc
a o acoperire a
planului cu figura reprezentat
a de aceast
a plantatie cu m
asura egal
a cu max(A, B).
Conjenctur
a. Putem acoperi modulo 2 planul cu translatii ale unei aceleeasi
plantatii, oricare ar fi aceasta. Mai mult, m
asura obtinut
a poate fi mai mic
a dect
cea dat
a de demonstratia 2 si chiar dect cardinalul plantatiei.
Un nceput de demonstratie pentru conjenctur
a este sugerat de problema anterioar
a.
29

Metoda norm
arii
Marian TETIVA1
Introducere. n aceast
a not
a vrem s
a d
am cteva exemple de utilizare a metodei
normarii, pe care am preluat-o, cu tot cu acest nume, din excelenta carte [3]; am
pornit de la faptul c
a acolo nu exist
a prea multe aplicatii si, la nceput mai mult
n glum
a, am demonstrat pe aceast
a cale cteva inegalit
ati (nu tocmai usoare). Cu
timpul s-au adunat din ce n ce mai multe asemenea inegalit
ati (si din ce n ce mai
grele). Metoda s-a dovedit extrem de eficient
a pentru demonstrarea inegalit
atilor
omogene dar si pentru obtinerea unor identit
ati altfel greu de g
asit, iat
a de ce vrem s
a
o prezent
am aici; totusi, trebuie s-o spunem, metoda norm
arii nu este recomandat
a
celor care au "alergie" la calcule: este pentru cei r
abd
atori si st
apni pe tehnica
matematic
a (ne referim la calculul elementar). De asemenea, se aplic
a inegalit
atilor
n care variabilele implicate sunt numere reale pozitive (sau nenegative).
S
a ncepem cu o inegalitate foarte cunoscut
a (a se vedea si [3], capitolul 6):
a2 + b2 + c2 ab + ac + bc;

se stie c
a aceasta e valabil
a pentru orice numere reale a, b, c, dar noi o vom demonstra
(complicat, veti spune, dar e numai pentru a da un exemplu) doar pentru a, b, c 0
(iat
a un dezavantaj; nu unul mare, pentru c
a majoritatea inegalit
atilor care vor
urma prezint
a interes pentru cazul variabilelor nenegative si, uneori, se pot extinde,
plecnd de la acesta, la orice valori reale ale variabilelor). Datorit
a simetriei, putem
presupune f
ar
a a particulariza, c
a c = min {a, b, c}. Pentru c = 0 inegalitatea este
evident
a:
2
a2 + b2 ab (a b) + a2 + b2 0.
a
b
Fie c > 0 si s
a not
am = 1 + x, = 1 + y. Conform presupunerilor f
acute, avem
c
c
x 0 si y 0, iar inegalitatea de demonstrat devine (dup
a mp
artirea cu c2 si cu
noile notatii)
(1 + x)2 + (1 + y)2 + 1 (1 + x) (1 + y) + 1 + x + 1 + y x2 + y 2 xy,

evident
a (ca mai sus) chiar pentru orice x, y R, nu doar pentru x, y 0. Se vede
usor c
a egalitatea are loc doar pentru x = y = 0, deci numai dac
a a = b = c. S
i mai
mult, plecnd de la

(1 + x)2 + 1 + y 2 + 1 (1 + x) (1 + y) 1 x 1 y = x2 + y 2 xy
ac
bc
,y=
) g
asim identitatea
c
c
2
2
a2 + b2 + c2 ab ac bc = (a c) + (b c) (a c) (b c) ,

si revenind la a, b, c (cu x =

care permite demonstrarea inegalit


atii pentru orice a, b, c R si chiar obtinerea unei
rafin
ari:

1
2
2
a2 + b2 + c2 ab ac bc
(a c) + (b c) , a, b, c R !
2
1

Profesor, Colegiul National "Gheorghe Rosca Codreanu", Brlad

30

Desigur, toate acestea se puteau face si altfel si sunt cunoscute, dar . . . nu e tocmai
demonstratia obisnuit
a a acestei inegalit
ati, nu-i asa?
Mai departe s
a consider
am inegalitatea
a4 + b4 + c4 a3 b + b3 c + c3 a,

a, b, c 0,

c
areia i aplic
am acelasi tratament. Simetria (de data asta, doar circular
a) ne permite
s
a presupunem, f
ar
a a restrnge generalitatea, c
a c = min {a, b, c}; c = 0 ne las
a
inegalitatea n forma a4 + b4 a3 b, pentru orice a, b 0 (exercitiul 1: demonstrati
acest caz particular!). Mai departe fie c > 0 si s
a facem aceleasi notatii ca mai sus.
mp
artim cu c4 si inegalitatea devine
(1 + x)4 + (1 + y)4 + 1 (1 + x)3 (1 + y) + (1 + y)3 + 1 + x,
de demonstrat pentru x, y 0. Calcule simple (exercitiul 2: verificati-le!) o transform
a n

3 x2 + y 2 xy + 3 x3 + y 3 x2 y + x4 + y 4 x3 y 0;
aceasta este adev
arat
a, ba chiar se poate nt
ari, tinnd cont de

1 2
x + y2 ,
x2 + y 2 xy
2
de
2
x3 + y 3 x2 y xy 2 ( (x y) (x + y) 0)
si de

x4 + y 4 x3 y xy 3 ( (x y)2 x2 + xy + y 2 0).
Astfel am obtinut de fapt
(1 + x)4 + (1 + y)4 + 1 (1 + x)3 (1 + y) (1 + y)3 1 x

3 2
x + y 2 + 2xy 2 + xy 3 , x, y 0;

2
atati
aici s
a revenim la variabilele initiale a, b, c si s
a nmultim cu c4 . Exercitiul 3: ar
c
a ajungem la inegalitatea

a4 + b4 + c4 a3 b + b3 c + c3 a

3
2
2
2
3
c2 (a c) + (b c) + 3c (a c) (b c) + (a c) (b c) ,
2
pentru orice numere nenegative a, b, c, c fiind cel mai mic dintre ele (este important
acest lucru?). Iar exercitiul 4 v
a cere s
a demonstrati identitatea

3
2
2
2
a4 + b4 + c4 a3 b + b3 c + c3 a =
(a b) + (a c) + (b c) +
2
2
2
+3c (a b) (a + b 2c) + 3c (a c) (b c) +

2
2
2
3
+ (a b) (a c) + (b c) + (a c) (b c) + (a c) (b c)

(mai conteaz
a cum sunt numerele a, b, c?) si s
a obtineti si alte nt
ariri ale inegalit
atii
considerate.
Acum, c
a ati cam nteles n ce const
a metoda norm
arii si, n plus, ati c
ap
atat
antrenament la calcule de acest tip, puteti exersa chiar singuri si ceva mai serios:
31

Exercitiul 5. Ar
atati c
a, pentru orice numere reale nenegative a, b, c, d, cu
d = min {a, b, c, d} are loc inegalitatea
a4 + b4 + c4 + d4 + 2abcd a2 b2 a2 c2 a2 d2 b2 c2 b2 d2 c2 d2
2

d2 (a d) + d2 (b d) + d2 (c d) + 2d (a d) (b d) (c d) .
Deduceti inegalitatea lui Turkevici
a4 + b4 + c4 + d4 + 2abcd a2 b2 + a2 c2 + a2 d2 + b2 c2 + b2 d2 + c2 d2
(pentru orice a, b, c, d 0), identitate a c
arei consecinta este si, eventual, alte rafin
ari
ale ei.

Metoda norm
arii si o demonstratie a inegalit
atii mediilor. Dup
a cum
s-a v
azut, metoda descris
a n aceast
a not
a se aplic
a n cazul inegalit
atilor simetrice
si omogene n n + 1 variabile, s
a le spunem a1 , a2 , . . . , an+1 . Simetria ne permite
s
a consider
am, nerestrictiv, c
a, de exemplu, an+1 este cel mai mic dintre toate
numerele a1 , a2 , . . . , an+1 (chiar si simetria circular
a ne permite o asemenea presupunere). Dup
a ce verific
am inegalitatea pentru an+1 = 0 (dac
a e cazul) folosim
substitutiile
a2
an
a1
= 1 + xi ,
= 1 + x2 , . . . ,
= 1 + xn ,
an+1
an+1
an+1
a n noua form
aa
unde, desigur, x1 , x2 , . . . , xn sunt nenegative (ceea ce, de obicei, ajut
inegalit
atii). Se poate alege si an+1 = max {a1 , a2 , . . . , an+1 }, dar e doar o chestiune
de gust, nu schimb
a esential calculele. Apoi, folosind transform
arile inverse
a1 an+1
a2 an+1
an an+1
, x2 =
, . . . , xn =
,
x1 =
an+1
an+1
an+1
ne putem ntoarce la inegalitatea noastr
a pentru a obtine chiar nt
ariri ale ei (c
aci,
de obicei, r
amn n diferenta dintre cei doi membri ai inegalit
atii transformate multi
termeni nenegativi care pot fi utilizati n acest scop), sau identit
ati interesante (care
o implic
a). Cine a citit cu atentie descrierea metodei n [3] a observat deja c
a noi am
considerat o form
a particular
a a acesteia. Asta pentru c
a asa am lucrat noi si am
obtinut destule rezultate interesante (chiar mai multe dect cele expuse aici). Dar,
cititi [3] si veti afla si mai multe (chiar dac
a n capitolul dedicat norm
arii sunt putine
exemple)!
n aceast
a sectiune am ales pentru exemplificare demonstratia inegalit
atii
n
n
n
a1 + a2 + + an na1 a2 an , a1 , a2 , . . . , an 0,
adic
a a inegalitatii mediilor, o demonstratie care s-ar putea s
a par
a complicat
a (si
chiar este!) fata de multe alte demonstratii cunoscute (c
aci, nu-i asa? se cunosc
foarte multe demonstratii ale inegalit
atii mediilor); totusi s
a o facem.
Cum verificarea n cazurile n {1, 2} (sau chiar n = 3) nu mai constituie o problem
a trecem direct la pasul de inductie; pe care o facem dup
a schema: presupunem
c
a am demonstrat c
a, pentru fiecare k n inegalitatea
ak1 + ak2 + + akk ka1 a2 ak
are loc (pentru orice a1 , a2 , . . . , ak 0) si o dovedim pentru k = n+1. n inegalitatea
de demonstrat
an+1
+ an+1
+ + an+1
+ an+1
a1 , a2 , . . . an , an+1 0,
n
1
2
n+1 (n + 1) a1 a2 an an+1 ,
32

putem presupune, cum am spus, an+1 = min {a1 , a2 , . . . , an+1 }. Cum an+1 = 0 duce
la o inegalitate absolut banal
a, putem considera an+1 > 0, s
a mp
artim cu an+1
si
n+1
s
a facem substitutiile de mai sus; vom avea de demonstrat c
a
(1 + x1 )n+1 +(1 + x2 )n+1 + +(1 + xn )n+1 +1 (n + 1) (1 + x1 ) (1 + x2 ) (1 + xn ) ,
pentru orice x1 , x2 , . . . , xn 0.

Exercitiul 6. Ar
atati c
a, dup
a cteva calcule, ne r
amne inegalitatea

n
n
n
X
X
X
X
k
Cn+1
xkj (n + 1)
x1 x2 xk +
xn+1
0.
j
j=1

k=2

j=1

Cnk

Prin
x1 x2 xk ntelegem suma tuturor celor
produse de cte k factori (cu indici distincti) alesi dintre x1 , x2 , . . . , xn (pentru k = n suma contine doar un termen,
produsul x1 x2 xn ).
Pentru a demonstra aceast
a inegalitate, s
a observ
am nti c
a, dac
a folosim ipoteza
de inductie, avem (pentru 2 k n)
xk1 + xk2 + + xkk kx1 x2 xk

si chiar putem scrie Cnk inegalit


ati de acest tip (cte una pentru fiecare grup de k
dintre numerele x1 , x2 , . . . , xn ); adun
am toate aceste inegalit
ati si avem
X
X

k
k
k
x1 + x2 + + xk k
x1 x2 xk .

k1
Cum n membrul stng fiecare xj , 1 j n, apare de Cn1
ori, de fapt am obtinut
k1
Cn1

n
X
j=1

xkj k

x1 x2 xk

n
X
n + 1 k1 X k
xj (n + 1)
x1 x2 xk ,
Cn1
n
j=1

pentru fiecare k {2, . . . , n}. Prin urmare, membrul stng al inegalit


atii de demonstrat se minoreaz
a astfel

n
n
n
X
X
X
X
k
Cn+1
xkj (n + 1)
x1 x2 xk +
xn+1

j
j=1

k=2

j=1

X
n
n
n
X
X
n
+
1
k1
k
Cn+1

xkj +
xn+1
.
Cn1
j
k
j=1
j=1
k=2

Evident, mai avem s


a demonstr
am c
a expresia din membrul drept este 0. Un
calcul simplu ne arat
a c
a
n + 1 k2 k 1 k
n + 1 k1 n + 1 k1
k1
k

Cn+1
=
Cn1 =
Cn Cn1
Cn1 =
Cn+1 ,
n
n
k
n
deci, de fapt, ne-a mai r
amas

n
n
n
X k1
X
X
k

xkj +
xn+1
0,
Cn+1
j
n
j=1
j=1
k=2

care este evident


a, datorit
a faptului c
a x1 , x2 , . . . , xn sunt nenegative; demonstratia
prin inductie este ncheiat
a.
33

Exercitiul 7. Demonstrati identitatea


an1 + an2 + + ann na1 a2 an =
n1

n1
n1
X
X
X
X
k
n
k
= ank
(a

a
)
n
(a

a
)
(a

a
)

(a

a
)
+ (aj an )
C
j
n
1
n
2
n
k
n
n
n
k=1

j=1

j=1

pentru orice numere (chiar numere complexe) a1 , a2 , . . . , an . (Cea de-a doua sum
a
k
din paranteza mare cuprinde toate cele Cn1
produse de k factori alesi dintre a1 an ,
a2 an , . . . , an1 an .) De exemplu,

2
2
2
a4 +b4 + c4 + d4 4abcd = 2d2 3 (a d) + 3 (b d) + 3 (c d)

3
2 (a d) (b d) 2 (a d) (c d) 2 (b d) (c d) + 4d (a d) +

+ (b d)3 + (c d)3 (a d) (b d) (c d) + (a d)4 + (b d)4 + (c d)4 ,


pentru orice numere (complexe) a, b, c, d. S
i atunci nu v
a va fi greu cu

Exercitiul 8. Ar
atati c
a pentru orice numere reale nenegative a, b, c, d, d fiind
cel mai mic dintre ele, are loc inegalitatea

2
2
2
a4 + b4 + c4 + d4 4abcd 2d2 (a b) + (a c) + (a d) +

+ (b c)2 + (b d)2 + (c d)2 .


(Puteti rezolva asta prin metoda norm
arii?)
n ncheiere v
a mai propunem cteva exercitii (grele!). Primul dintre ele este
totusi mai simplu si vine n completarea celei de-a doua inegalit
ati din introducere.
Exercitiul 9. Demonstrati cea de a doua inegalitate din
a4 + b4 + c4 a3 b + b3 c + c3 a abc (a + b + c) , a, b, c 0.
Nu uitati s
a c
autati mbun
at
atiri ale acestei inegalit
ati, precum si identitatea a c
arei
consecinta este!
Exercitiul 10. Demonstrati, folosind metoda norm
arii, inegalitatea lui Surnyi [2]
n
n
n
(n 1) (a1 + a2 + + an ) + na1 a2 an

(a1 + a2 + + an ) an1
+ an1
+ + an1
n
1
2
si obtineti nt
ariri ale ei.
S
i, n sfrsit, pentru ultimul exercitiu aveti nevoie si de ceva inspiratie pentru a
demonstra inegalitatea la care se ajunge dup
a normare (nu ntotdeauna ea va rezulta
f
ar
a probleme!).
Exercitiul 11. Demonstrati c
a inegalitatea
2
a (a b) (a 2b) + b2 (b c) (b 2c) + c2 (c a) (c 2a) 0
are loc pentru orice numere reale a, b, c. Aceast
a inegalitate a fost publicat
a de
Vasile Crtoaje n Gazeta Matematica, n urm
a cu mai multi ani [1].
Bibliografie
1. V. Crtoaje - Problema 22694, Gazeta Matematic
a, seria B, 7-8/1992.
2. G. Dospinescu - O teorema uitata - inegalitatea lui Surnyi, RecMat - 1/2005.
3. M. Onucu Drimbe - Inegalitati - idei si metode, Editura Gil, Zal
au, 2003.
34

Asupra unei recurente de ordin doi


Gheorghe IUREA1
n [1], la pag. 59, apare urm
atoarea problem
a (autor Vasile Berinde):
Fie sirul de numere reale (xn )nN definit prin relatiile: x0 R, xn+1 = 2x2n +
2xn 1, n N.
a) Aflati numerele M R cu
ca, daca x0 M , atunci xn M , n N .
proprietatea

3 1
b) Aratati ca pentru x0 ,
sirul (xn )nN converge si calculati lim xn .
n
2 2
c) Determinati expresia lui xn n functie de x0 si n.
ntruct n rezolvarea acestei probleme dat
a n [1] sunt unele sc
ap
ari, ne propunem
s
a ne ocup
am mai am
anuntit cu acest sir interesant.
n privinta punctului a), fie M R cu proprietatea cerut
a; fie x0 = M t M ,
t 0 si trebuie s
a avem x1 M . Dar x1 = 2t2 2t (1 + 2M ) + 2M 2 + 2M 1 si cum
lim x1 = , deducem c
a pentru t suficient de mare avem x1 > M . Prin urmare, nu
t
exist
a M cu proprietatea cerut
a.

2
1
1
S
a observ
am c
a recurenta dat
a este echivalent
a cu xn+1 + = 2 xn +
1,
2
2
1
1
n N . Notnd yn = xn + , obtinem yn+1 = 2yn2 1, y0 = x0 + . Evident,
2
2
studiul sirului (xn )nN se reduce la studiul sirului (yn )nN .
Faptul c
a cerinta de la punctul b) este gresit
a rezult
a din urm
atoarea
Propozitie. Fie (yn )nN un sir de numere reale, definit prin
y0 R, yn+1 = 2yn2 1, n N.
a) Pentru y0 (, 1) (1, ), (yn )nN este divergent si lim yn = .
n

b) Pentru y0 [1, 1], (yn )nN este convergent daca si numai daca exista n0 N
astfel nct sirul (yn )nn0 este constant.
n
n
e2 t0 + e2 t0
n
=
cos
(2
t
),
t
=
arccos
y
pentru
y

[1,
1]
s

i
y
=
,
c) y
n
0 0
0
0
n
p
2
2
t0 = ln |y0 | + y0 1 , pentru y0 (, 1) (1, ).

d) Daca M = y0 R; (yn )nN este convergent atunci:


1. M este nevida, finita.
2. M = A B, unde

2
2k
n0
n00
3 + 2k
; k = 0, 1, . . . , 2 1 ,
A = cos n0 ; k = 0, 1, . . . , 2 1 , B = cos
0
2
2n0
cu n0 , n00 numere naturale fixate.
e) Sirul

(y )
este convergent daca si numai daca y0 A0 B 0 , unde
v n nN
v
s
s
r
r
u
u

u
u
1
1
1
1
1 1 1
1
1
1
1
1
t
t
0
0
,B =
,


+
A=
2 2 2
2 2 2
2 2 2
2 2 4
|
|
{z
}
{z
}
n00 radicali

n0 radicali

Profesor, Liceul Teoretic "Dimitrie Cantemir", Iasi

35

semnele fiind alese n toate modurile posibile.


Demonstratie. a) Dac
a y0 (, 1) (1, ) , atunci y02 > 1 si inductiv rezult
a c
a yn > 1 pentru orice n N. Cum yn+1 yn = 2yn2 1 yn =
(2yn + 1) (yn 1) > 0, rezult
a c
a (yn ) este strict cresc
a
ator. Prin urmare, exist
1
l = lim yn ; dac
a l R, rezult
a l = 2l2 1, deci l 1, , imposibil, ntruct
n
2
ator. Conchidem c
a lim yn = .
yn > 1, n N si (yn ) este strict cresc
n

1
2
b) Fie (yn ) convergent. Dac
a lim yn = l, rezult
a l = 2l 1, deci l 1, .
n
2
Dac
a l = 1, din relatia de recurenta rezult
a yn+1 1 = 2 (yn 1) (yn + 1),
deci |yn+1 1| = 2 |yn 1| |yn + 1|. Presupunnd c
a yn 6= 1 pentru orice n N,
|yn+1 1|
a
avem c
a lim
= lim 2 |yn + 1| = 4 si din criteriul raportului rezult
n |yn 1|
n
lim |yn 1| = , absurd. Asadar, exist
a n0 N cu yn0 = 1 si atunci yn = 1, n n0 .
n
1
n cazul l = scriem relatia de recurenta sub forma 2yn+1 +1 = (2yn +1)(2yn 1)
2
si continu
am ca mai sus.
Presupunem acum c
a exist
a n0 N astfel nct (yn )nn0 este constant. Din

1
a yn = 1, n n0 ,
relatia de recurenta deducem yn0 1, . Cum yn0 = 1 implic
2
1
a cu 1. La fel, dac
a yn0 =
rezult
a c
a (yn )nN este convergent cu limita egal
2
1
1
deducem yn = , n n0 si lim yn = .
n
2
2
c) Se demonstreaz
a prin inductie matematic
a.
a existenta
d) 1. Cum 1 M , rezult
a c
a M 6= . Folosind a) si b), y0 M implic
1
unui n0 astfel nct (yn )nn0 este constant (egal cu 1 sau ). Dac
a yn = 1, n n0 ,
2
2
2
2
din sistemul yn0 = 2yn0 1 1, yn0 1 = 2yn0 2 1, . . . , y1 = 2y0 1, din aproape
n aproape, g
asim un num
ar finit de valori pentru y0 . La fel se analizeaz
a cazul
1
yn = , n n0 .
2
1
2. y0 M implic
a faptul c
a yn = 1, n n0 sau yn = , n n00 (n0 , n00 numere
2
naturale arbitrare, dar fixate).
i y0 M [1, 1], deducem cos (2n arccos y0 ) = 1, de unde y0
Folosind c) s
2k
cos n0 ; k Z . Folosind periodicitatea functiei cosinus rezult
a c
a y0 A. La fel,
2
1
a y0 B. n concluzie, M = A B.
din yn = , n n00 , deducem c
r
2
x
1 1
e) Folosind formula cos =
+ cos x, x R, deducem c
a A A0 si cum
2
2 2
multimile au fiecare cte 2n0 elemente, rezult
a c
a A = A0 . La fel, B = B 0 .
Bibliografie
1. Gh. Eckstein et al. - Olimpiadele si concursurile de matematica IX XII, Editura Brchi, 2005.
36

Olimpiada International
a de Matematic
a
"B. O. Zhautykov"
Editia I, Alma-Ata, 2005
Enunturi si Solutii juniori
Prima zi 13 ianuarie 2005
1. Pe o tabl
a 9 9 sunt marcate 40 celule. O linie orizontal
a sau vertical
a
format
a din 9 celule se spune c
a este buna, dac
a ea are mai multe celule marcate
dect nemarcate. Care este cel mai mare num
ar de linii bune (orizontale si verticale)
pe care-l poate avea tabla?
2. Ar
atati c
a num
arul 22n+2 + 2m+2 + 1, unde m, n Z si 0 m 2n, este
p
atrat perfect dac
a si numai dac
a m = n.
3. Fie A o multime format
a din 2n puncte dintr-un plan astfel nct oricare
trei dintre acestea nu sunt coliniare. Ar
atati c
a pentru orice dou
a puncte distincte
a, b A exist
a o dreapt
a ce mparte A n dou
a submultimi continnd n puncte fiecare
si astfel nct a si b se afl
a de p
arti diferite n raport cu aceast
a dreapt
a.
A doua zi 14 ianuarie 2005
4. Pentru orice numere a, b, c reale si pozitive, ar
atati inegalitatea
c
a
b
+
+
1.
a + 2b b + 2c c + 2a
5. Cercul nscris triunghiului ABC este tangent laturii AB n punctul D, iar
M este mijlocul acestei laturi. Ar
atati c
a M , centrul cercului nscris si mijlocul
segmentului [CD] sunt coliniare.
6. Determinati numerele prime p, q mai mici ca 2005 si astfel nct p2 + 4 se
divide cu q, iar q 2 + 4 se divide cu p.
* *
*
1. Deoarece sunt marcate 40 celule si o linie are cel
putin 5 celule marcate, rezult
a c
a putem avea cel mult 8
linii orizontale bune si cel mult 8 linii verticale bune. n
total, putem avea cel mult 16 linii bune. Al
aturat este dat
un exemplu de tabl
a cu 16 linii bune. Deci 16 este num
arul
maxim de linii bune.

2
2. Dac
a m = n, atunci 22n+2 + 2m+2 + 1 = 2n+1 + 1 .

2
Dac
a m < n, atunci avem 2n+1 < 22n+2 + 2m+2 + 1 <
n+1
2
2
+ 1 si, deci, num
arul examinat nu poate fi un p
atrat perfect.
Fie acum n < m 2n. S
a presupunem c
a ar exista x natural astfel nct x2 =
22n+2 + 2m+2 + 1. Observ
am c
a x este impar si x > 1. Scriem relatia precedent
a n
forma

(x 1) (x + 1) = 2m+2 22nm + 1 .
()
37

Deoarece (x 1, x + 1) = 2, num
arul 2m+1 divide una dintre parantezele din membrul stng, iar cealalt
a nu va fi mai mic
a dect 2m+1 2. Cum m 2n m + 2 si
22nm+1 2, urmeaz
a c
a

2m+1 2 22nm+3 2 = 4 22nm+1 2 > 22nm+1 + 2.

n consecinta, (x 1) (x + 1) 2m+1 2m+1 2 > 2m+2 22nm + 1 , ceea ce contrazice (). Asadar, nici n acest caz num
arul dat nu-i p
atrat perfect.

3. Fie dab dreapta determinat


a de punctele a si b. Pe segmentul de extremit
ati
a si b alegem un punct O astfel nct orice dreapt
a ce trece prin O si nu coincide cu
dab contine cel putin un punct din A; pentru c
a A este multime finit
a, o astfel de
dreapt
a exist
a.
Not
am cu d dreapta obtinut
a rotind dab cu unghiul (n sens contrar acelor de
ceasornic, de exemplu); avem d0 = d = dab . Dac
a dab mparte A \ {a, b} n dou
a
submultimi cu n 1 elemente fiecare, atunci rotind-o cu un unghi suficient de mic
obtinem dreapt
a c
autat
a.
Presupunem c
a ntr-un semiplan determinat de d mpreun
a cu a se afl
a m puncte
din A, iar n cel
alalt mpreun
a cu b se afl
a 2nm (m 6= n). Dac
a unghiul de rotatie va
fi suficient de aproape de , atunci situatia se inverseaz
a: ntr-un semiplan mpreun
a
cu a se afl
a 2n m puncte din A, n cel
alalt mpreun
a cu b sunt m puncte.
Deoarece trecerea de la perechea (m, 2n m) la (2n m, m) prin rotatie n jurul
lui O se face printr-o compunere de transform
ari de tipul (x, y) (x 1, y 1), va
exista o valoare 0 pentru care corespunde perechea (n, n); d0 este dreapta c
autat
a.
4. Fie a + 2b = x, b + 2c = y, c + 2a = z. Rezolvnd n raport cu a, b, c, obtinem:
4
1
2
4
1
2
4
1
2
a = z + x y, b = x + y z, c = y + z x. Inegalitatea dat
a se rescrie
9
9
9
9
9
9
9
9
9

4 y
z
x
1 z
x y
2
+ +
+
+ +
3 1.
9 x y
z
9 x y
z
9

Cum sumele din paranteze sunt 3 (inegalitatea mediilor), deducem c


a ultima inegalitate este adev
arat
a.

5. Dac
a M coincide cu D, atunci se arat
a usor c
a BC = AC, adic
a triunghiul
este isoscel. n acest caz [CD este bisectoarea unghiului C si coliniaritatea celor trei
puncte este evident
a.
Dac
a M 6= D, fie E punctul diametral opus lui D pe cercul nscris si {F } =
CE AB. Se stie c
a F este punctul de tangenta cu latura AB a cercului exnscris corespunz
ator acestei laturi si c
a M este mijlocul segmentului [F D] (eventual,
demonstrati!). Atunci M I este linie mijlocie n 4DEF (I noteaz
a centrul cercului
nscris). Rezult
a c
a M I k CF si, n final, M I trece prin mijlocul segmentului [CD].
6. Dac
a p = q, atunci aceste numere divid 4 si, deci, p = q = 2. n acest caz,
obtinem solutia (p, q) = (2, 2).
S
a determin
am solutiile (p, q) cu p 6= q. Vom spune c
a perechea (x, y) de numere
naturale este admisa, dac
a ndeplineste conditiile: (A) x, y sunt relativ prime si
x y; (B) x2 + 4 se divide cu y si y 2 + 4 se divide cu x. Observ
am c
a o pereche
admis
a este format
a din numere impare.
38



Ar
at
am mai nti c
a, dac
a (x, y) este o pereche admis
a, atunci perechea y, y 2 +4 /x

este de asemenea admis


a. n acest scop, fie z = y 2 + 4 /x. Deoarece xy y 2 <
y 2 + 4, rezult
a c
a y < z. Apoi, dac
a d divide y si z, atunci d divide y si y 2 + 4, deci
d divide 4, ceea ce conduce la d = 1. Asadar, perechea (y, z) ndepline
tia
ste condi
2
2
2
+
8
+
4
x
+
4
y
y
(A). Evident, z divide y 2 + 4; pe de alt
a parte, z 2 + 4 =
,
x2
unde num
ar
atorul se divide cu y, care este relativ prim cu x. n concluzie, perechea
(y, z) este admis
a.

S
a consider
am sirul (ai )i0 definit de a0 = a1 = 1 si ai+2 = a2i+1 + 4 /ai , i 0.
Din ceea ce s-a stabilit mai sus, rezult
a c
a perechile (ai , ai+1 ), i N, sunt admise.
S
a ar
at
am acum c
a orice pereche admis
a este de forma (ai , ai+1 ) pentru un anumit
i 0. S
a presupunem c
a exist
a perechi admise ce nu-s de aceast
a form
a si fie (x, y)
perechea de acest fel cu suma x+y minim
a. Cum x2 +4 = ay si y 2 +b= bxsi a, x sunt

x2 x2 + 8 + 4 a2 + 4
relativ prime (se arat
a ca mai sus!), obtinem y 2 + 4 =
=
a2
bx si a2 + 4 se divide cu x. Dac
a a x, atunci (a, x) este pereche admis
a si,
datorit
a minimalit
atii, avem (a, x) = (ai , ai+1 ). Rezult
a c
a (x, y) = (ai+1 , ai+2 ), n
contradictie cu presupunerea f
acut
a. Dac
a a > x, atunci a x + 2 si cum din y > x
2
avem si y x + 2, putem scrie x2 + 4 = ay (x + 2) = x2 + 4x + 4, din nou o
contradictie.
n sfrsit, s
a scriem termenii sirului (ai )i0 ce nu dep
asesc 2005; acestia sunt:
a0 = a1 = 1, a2 = 5, a3 = 29, a4 = 169 si a5 = 985. Sunt numere prime numai 5 si
29. Solutiile problemei sunt perechile (p, q) {(2, 2) , (5, 29) , (29, 5)}.
Enunturi si Solutii seniori
Prima zi 13 ianuarie 2005
1. Ar
atati c
a ecuatia x5 + 31 = y 2 nu are solutii ntregi.
2. Fie r un num
ar real astfel nct pentru orice sir (an )n1 de numere reale
pozitive are loc inegalitatea

a1 + a2 + + am+1 ram ,

oricare ar fi m N . Ar
atati c
a r 4.

3. Fie SABC o piramid


a triunghiular
a regulat
a, i.e. SA = SB = SC si AB =
BC = AC. Determinati multimea punctelor D (D 6= S) din spatiu ce satisfac
conditia
|cos A 2 cos B 2 cos C | = 3,
unde X = ]XSD pentru X {A, B, C}.
A doua zi 14 ianuarie 2005
4. Pentru orice numere a, b, c, d reale si pozitive, ar
atati inegalitatea
d
a
b
4
c
+
+
+
.
a + 2b b + 2c c + 2d d + 2a
3
5. Se spune c
a punctul X interior unui patrulater (convex) este observabil din
latura Y Z dac
a piciorul perpendicularei din X pe dreapta Y Z apartine segmentului
39

[Y Z]. Un punct interior patrulaterului se spune c


a este k-punct dac
a este observabil
din exact k laturi ale patrulaterului (de exemplu, orice punct din interiorul unui
p
atrat este 4-punct). Ar
atati c
a, dac
a n interiorul unui patrulater exist
a un 1-punct,
atunci exist
a si un k-punct pentru k {2, 3, 4}.
6. Determinati numerele prime p, q mai mici ca 2005 si astfel nct p2 + 8 se
divide cu q, iar q 2 + 8 se divide cu p.
* *
*
1. Dac
a x este par, atunci x5 + 31 3 (mod 4) si nu poate fi p
atrat perfect.
Urmeaz
a c
a x este impar si, deci, y este par. Mai mult, x5 1 (mod 4) implic
a
x 1 (mod 4). S
a scriem ecuatia dat
a n forma
x5 + 25 = y 2 + 1.

Partea stng
a se divide cu x + 2 si x + 2 3 (mod 4) va avea un divizor prim de
tipul 4l + 3. Dar, conform lemei de mai jos, num
arul impar y 2 + 1 are divizori primi
numai de tipul 4m + 1. n concluzie, n ipoteza c
a ecuatia dat
a ar avea solutii ntregi,
ajungem la o contradictie.
Lem
a. Daca y 2 +1 admite un divizor prim impar p, atunci p este de tipul 4m+1.
ntr-adev
ar, avem y 2 1 (mod p). n conformitate cu mica teorem
a a lui
Fermat, avem si y p1 1 (mod p). Atunci
(p1)/2
(1)(p1)/2 1 (mod p) .
y p1 y 2
Ultima congruenta spune c
a (p 1) /2 este par, adic
a p 1 (mod 4).

2. Not
am bm = a1 + a2 + + am . Atunci, sirul (bn )n1 este strict cresc
ator si

),
m

N
.
Pentru
c
=
b
/
(b
r)
aceast
a
verific
a relatia bm+1 r (bm b
m1
m
m
m1
relatie devine cm+1 cm + 1 cm r.Deci, pentru orice m N avem

1
cm+1 +
r.
cm
1
Utiliznd inegalitatea c + 2 (c > 0), obtinem
c

1
1
1
+ cn +
+ + c2 +

n r cn+1 +
cn
cn1
c1
1
2 (n 1) , n N .
cn+1 + 2 (n 1) +
c1

2
n1
n1
, n N , si rezult
a c
a r 4 (num
arul
Deci r 4
poate fi orict de
n
n
aproape dorim de 1).

3. Fie eX versorul vectorului SX, X {A, B, C, D}. Atunci, tinnd seama c


a
cos = (eD , eX ) (produs scalar), X {A, B, C}, conditia din enunt se scrie

(eD , eA ) 2 (eD , eB ) 2 (eD , eC ) = eD , 1 eA 2 eB 2 eC = 1.


3

3
3
3
3
3
40

2
2
1
aci
eB + eC eA . Vectorul f este unitar, c
3
3
3

1 2
|f |2 = (f, f ) =
4eB + 4e2C + e2A + 8 (eB , eC ) 4 (eA , eB ) 4 (eA , eC ) =
9
1
= (9 + 8 cos 4 cos 4 cos ) = 1,
9
a cu faptul c
a
unde = ]ASB. Atunci, conditia |(eD , f )| = 1 este echivalent
vectorii eD si f sunt coliniari.
Fie SH n
altimea piramidei si F simetricul punctului A n raport cu H. Calcu
lalele urm
atoare arat
a c
a vectorii SF si f sunt coliniari:

1

AB = SB SA, AC = SC SA, AH =
AB + AC ;
3
2

2
AB + AC =
SB + SC 2 SA ;
AF = 2 AH =
3
3
1

SF = SA + AF =
2 SB + 2 SC SA .
3
Ca urmare, locul geometric c
autat este dreapta dSF din care se exclude punctul S.
Not
am f =

4. Not
am
c
d
a
b
+
+
+
,
a + 2b b + 2c c + 2d d + 2a
b + 2c c + 2d d + 2a a + 2b
B=
+
+
+
,
a + 2b b + 2c
c + 2d d + 2a
a+c
b+d
a+c
b+d
C=
+
+
+
.
a + 2b b + 2c c + 2d d + 2a
Constat
am usor c
a 2B + C = 5A + 4. Conform inegalit
atii mediilor, avem B 4.
1
4
1
(u, v > 0), obtinem
Tinnd

seama de inegalitatea +
u v
u+v
4
8
4
+ (b + d)

C (a + c)
a + 2b + c + 2d
b + 2c + d + 2a
3
x
y
2
(ultima inegalitate se obtine lund x = a + c si y = b + d n
+
,
x + 2y
y + 2x
3
iar aceast
a inegalitate se stabileste astfel:
3xy
3xy
x
y
2x2 + 2xy + 2y 2
2
=1 2
1
+
= 2
= ).
x + 2y y + 2x
2x + 5xy + 2y 2
2x + 5xy + 2y 2
9xy
3
8
4
Asadar, 5A + 4 8 + , deci A , q.e.d.
3
3
5. S
a numim zona de observatie a laturii XY a patrulaterului convex semibanda,
arginit
a de segmentul [XY ], perpendicularele pe aceasta
notat
a ZXY , care este m
duse n extremit
ati si care este situat
a n acelasi semiplan cu patrulaterul. Evident,
un punct interior patrulaterului este observabil din latura XY dac
a si numai dac
a
apartine ZXY .
S
a examin
am diferitele cazuri ce apar:
a) Dac
a patrulaterul nu are unghiuri obtuze, atunci el este dreptunghi si are
numai 4-puncte.
A=

41

b
b) Presupunem c
a patrulaterul ABCD are un singur unghi obtuz, anume A.
Atunci zonele de observare ale laturilor BC si CD acoper
a patrulaterul si, deci,
1-puncte nu exist
a.
b si C.
b n acest
c) Fie ABCD cu exact dou
a unghiuri obtuze si vecine, anume B
caz patrulaterul este situat n ntregime n ZAD , dar si n ZAB ZBC ZCD . Ca
urmare, nu exist
a 1-puncte.
b si D.
b Atunci
d) Fie ABCD cu exact dou
a unghiuri obtuze si opuse, anume B
patrulaterul este situat att n ZAB ZBC ct si n ZAD ZCD si nu va avea 1-puncte.
b unghiul s
e) Fie ABCD cu trei unghiuri obtuze si fie A
au ascutit. Atunci intersectia ZBC ZCD este situat
a n patrulater si formeaz
a paralelogramul LM N C (ca si
n ABCD, vrfurile sunt notate n sensul acelor de ceasornic). Fie {E} = BM AD
si {F } = DM AB. Se constat
a usor c
a 4ABE ZAB si 4AF D ZAD . Atunci,
M este 4-punct, punctele segmentului deschis (M F ) sunt 3-puncte, iar cele interioare
patrulaterului AF M E sunt 2-puncte.
6. Proced
am ca si n cazul problemei J6 (problema 6 de la juniori, prezentat
a
mai sus).
Dac
a p = q, atunci numerele p si q divid 8 si, deci, p = q = 2, adic
a (2, 2) este
solutie a problemei.
S
a determin
am solutiile (p, q) cu p 6= q. O pereche (x, y) de numere naturale se
numeste admisa dac
a: (A) x, y sunt relativ prime si x y; (B) x2 + 8 se divide cu
2
y, iar y + 8 se divide cu x. Mai nti, observ
am c
a o pereche admis
a este format
a
din numere impare. Apoi, ca si n problema
J
se
demonstreaz
a
c
a
,
dac
a
(x,
y)
este
6


pereche admis
a, atunci si perechea y, y 2 + 8 /x este admis
a. Acest rezultat are
urm
atoarele consecinte:

1) Dac
a (ai )i0 este sirul dat de a0 = a1 = 1 si ai+2 = a2i+1 + 8 /ai (i 0),
atunci orice pereche (ai , ai+1 ) este admis
a;

2) Dac
a (bi )i0 este sirul dat de b0 = 1, b1 = 3 si bi+2 = b2i+1 + 8 /bi (i 0),
atunci orice pereche (bi , bi+1 ) este admis
a.
S
a ar
at
am acum c
a orice pereche admis
a are forma (ai , ai+1 ) sau (bi , bi+1 ) pentru
un anumit indice i 0. Presupunem c
a ar fi adev
arat
a situatia contrar
a si fie (x, y)
perechea minimal
a (n raport cu suma x + y) care nu-i de nici una dintre formele
2
precedente. Cum
y2 + 8 = bx si a, x sunt relativ prime, obtinem
ay,
2 x + 8 =
2
2
+
16
+
8
a
+
8
x
x
y2 + 8 =
= bx si a2 + 8 se divide cu x. Dac
a a x,
a2
atunci (a, x) este pereche admis
a si, datorit
a minimalit
atii, avem (a, x) = (ai , ai+1 )
sau (a, x) = (bi , bi+1 ); rezult
a c
a (x, y) = (ai+1 , ai+2 ) sau (x, y) = (bi+1 , bi+2 ), n
contradictie cu presupunerea f
acut
a. Dac
a a > x, atunci x2 + 8 = ay (x + 2)2 =
2
a + 4x + 4, de unde x = 1, a = y = 3, din nou contradictie.
S
a scriem acum termenii sirurilor (ai )i0 si (bi )i0 ce nu dep
asesc 2005: a0 =
a1 = 1, a2 = 9, a3 = 89, a4 = 881; b0 = 1, b1 = 3, b2 = 17, b3 = 99, b4 = 577. Dintre
aceste numere sunt prime numai 3, 17, 89, 881 si 577. Ca urmare, solutiile problemei
sunt perechile (p, q) {(2, 2) , (3, 17) , (17, 3) , (89, 881) , (881, 89)}.

42

Solutiile problemelor propuse n nr. 1 / 2005


Clasele primare
P.84. Aflati numarul m stiind ca 47 este mai mare dect m 14 cu 28.
(Clasa I )
nv. Maria Racu, Iasi
Solutie. m 14 = 47 28; m 14 = 19; m = 14 + 19; m = 33.
P.85. ntr-un cos sunt 6 mere, iar n altul sunt 5 pere. Cum pot primi 5 copii
mere si pere astfel nct nici un cos sa nu ramna gol?
(Clasa I )
Veronica Corbu, elev
a, Iasi
Solutie. Fiecare copil primeste cte o par
a, unul dintre ei va primi para n cos.
Merele pot fi mp
artite astfel: fiecare copil primeste cte un m
ar si un m
ar r
amne
n cos sau unul dintre copii primeste dou
a mere care trebuie s
a se afle n cos.
P.86. n urma cu 4 ani, cnd tatal avea 29 de ani, s-a nascut fiul. Sora acestuia
avea atunci 2 ani, iar acum este de trei ori mai mare. Mama este de patru ori mai
mare dect aceasta. Cti ani are acum fiul?
(Clasa a II-a)
nv. Oana-Maria Lupu, Iasi
Solutie. Deoarece fiul s-a n
ascut n urm
a cu 4 ani, acesta are acum vrsta de 4
ani.
P.87. Un acrobat cade pe o plasa elastica de la o anumita naltime si se ridica
dupa ce atinge plasa la jumatatea distantei dintre plasa si locul de unde a cazut
anterior. Stiind

ca atinge de 3 ori plasa si ca ultima oara s-a ridicat la naltimea de


2 m, iar plasa este montata la 2 m deasupra solului, sa se afle distanta de la locul
de unde a cazut prima data pna la sol.
(Clasa a II-a)
Andrei Stativ
a, elev, Iasi
Solutie. A treia oar
a cade de la 2m + 2m = 4m fata de plas
a. A doua oar
a cade
de la 4m + 4m = 8m fata de plas
a. Prima oar
a a c
azut de la 8m + 8m = 16m fata
de plas
a. Distanta de la locul de unde a c
azut pn
a la sol este de 16m + 2m = 18m.
P.88. Traiau odata o baba si un mosneag; mosul avea 100 ani, iar baba 90,
amndoi erau albi ca iarna si tristi ca vremea cea rea pentru ca erau singuri. Se
spune ca ar fi avut un copil pe cnd vrsta babei era jumatate din jumatatea de acum
a vrstei mosneagului si ca acesta ar fi plecat n lume cnd vrsta mosneagului era
de doua ori ct vrsta aceea a babei. Fiul nu s-a mai ntors. Ce vrsta avea fiul cnd
a plecat n lume?
(Clasa a III-a)
nv. Ileana Roscan, Iasi
Solutie. La nasterea fiului, baba avea: 100 : 2 : 2 = 25 (ani). La nasterea fiului,
mosul avea: 25 + 10 = 35 (ani). Fiul a plecat n lume la vrsta de 50 35 = 15 (ani).
P.89. La un concurs de biciclete, triciclete si masinute (cu patru roti), tatal lui
Bogdan numara rotile vehiculelor si observa ca sunt 34. Cte vehicule au fost de
fiecare fel? Gasiti toate posibilitatile, stiind ca numarul vehiculelor de fiecare fel nu
depaseste 5.
(Clasa a III-a)
nv. Doinita Spnu, Iasi
Solutie. Nu putem avea un num
ar nepereche de triciclete deoarece num
arul total
de roti este num
ar pereche. Putem avea dou
a sau patru triciclete. Dac
a avem dou
a
43

triciclete, nseamn
a c
a num
arul rotilor bicicletelor si masinutelor este 34 2 3 =
34 6 = 28. n acest caz avem 4 biciclete si 5 masinute. Dac
a avem 4 triciclete,
atunci num
arul rotilor bicicletelor si masinutilor este 34 4 3 = 34 12 = 22. n
acest caz putem avea: o biciclet
a si 5 masinute sau 3 biciclete si 4 masinute sau 5
biciclete si 3 masinute.
P.90. Lungimea laturii unui patrat este de 17 m. O persoana pleaca dintr-un
vrf al patratului si, mergnd n acelasi sens pe laturile acestuia, parcurge o distanta
de 637 m. Din punctul n care a ajuns se ntoarce si parcurge 773 m. Aflati la ce
distanta se va situa n final persoana fata de punctul de plecare.
(Clasa a III-a)
Oxana Pascal, elev
a, Iasi
Solutie. Perimetrul p
atratului este 4 17m = 68m.
A
17 m
D
Consider
am primul sens de parcurs de la A c
atre B. Avem
10 68m = 680m = 637m + 43m. nseamn
a c
a persoana
a parcurs conturul n ntregime de 9 ori, iar din al zecelea contur a parcurs 68m 43m = 25m, oprindu-se n
punctul M . n punctul M se ntoarce si parcurge 773m.
Avem 773m = 680m + 68m + 25m, ceea ce nseamn
a c
a
a parcurs conturul n ntregime de 11 ori, iar din al doisprezecelea contur a parcurs 25m, adic
a M B + BA. n
8m
final c
al
atorul ajunge din nou n punctul de unde a plecat
B
M
C
initial. Distanta ceruta este de 0m.
P.91. Se mpart doua numere naturale. Daca mpartitorul, ctul si restul sunt
trei numere consecutive cu suma 30, sa se afle dempartitul.
(Clasa a IV-a)
Vasile Solcanu, Bogd
anesti, Suceava
Solutie. Cele trei numere consecutive sunt 9, 10 si 11. Formula mp
artirii cu
rest este D = I C + R, R < I. Se observ
a c
a 11 nu poate fi mp
artitor. Avem
D = 10 11 + 9 = 119 sau D = 9 11 + 10 = 109.
P.92. Observa regula si completeaza, apoi verifica rezultatele gasite: 2+4 = 3+3;
2 + 4 + 6 = 4 + 4 + 4; 2 + 4 + 6 + 8 = 5 + 5 + 5 + 5; 2 + 4 + 6 + + 12 = ;
2 + 4 + 6 + + 14 = ; . . . ; 2 + 4 + 6 + + (a + a) = .
(Clasa a IV-a)
Valeria Gheorghita
a, Iasi
, elev
Solutie. 2 + 6 = 3 + 3; 2 + 4 + 6 = (2 + 6) + 4 = 8 + 4 = 4 + 4 + 4; 2 + 4 +
6 + 8 = (2 + 8) + (4 + 6) = 10 + 10 = 5 + 5 + 5 + 5. Se observ
a c
a termenul care
se repet
a n membrul drept este jum
atatea sumei dintre primul termen si ultimul
termen din membrul stng. Acest termen se repet
a de un num
ar de ori egal cu
num
arul termenilor din membrul stng. n suma 2 + 4 + 6 + + 12 avem 6 termeni,
iar 2+12 = 7+7. Obtinem: 2+4+6+ +12 = |7 + 7 +
{z + 7}; 2+4+6+ +14 =
6 ori

8 + 8 + + 8; 2 + 4 + 6 + + (a + a) = (a + 1) + (a + 1) + + (a + 1).
|
{z
}
{z
}
|
7 ori

a ori

P.93. O foaie de hrtie dreptunghiulara se ndoaie de-a lungul de 6 ori, formnduse 7 benzi egale si suprapuse. Dreptunghiul obtinut se ndoaie de-a latul de 9 ori,
rezultnd n final un patrat cu perimetrul de 12 cm. Sa se afle perimetrul dreptun44

ghiului initial.
(Clasa a IV-a)
Petru Asaftei, Iasi
Solutie. Dup
a prima serie de ndoiri se obtine un dreptunghi care are lungimea
egal
a cu lungimea dreptunghiului initial, iar l
atimea este egal
a cu latura p
atratului
obtinut n final, a c
arui latur
a are lungimea 12m : 4 = 3m. Deoarece dreptunghiul
initial a fost ndoit de-a lungul de 6 ori, nseamn
a c
a l
atimea lui este 7 3m = 21m.
Ultimele 9 ndoiri sunt echivalente cu ndoirile de-a latul ale dreptunghiului initial,
de unde rezult
a c
a lungimea dreptunghiului initial este 10 3m = 30m. Perimetrul
dreptunghiului initial este 2 (30m + 21m) = 2 51m = 102m.

Clasa a V-a
V.56. Se considera numarul A = 5 + 52 + 53 + + 52005 .
a) Sa se arate ca A nu este patrat perfect.
b) Sa se gaseasca 5 divizori mai mici dect 100 ai lui A.
Andrei Tofan, elev, Iasi
..
.. 2
Solutie. a) Evident c
a A . 5, ns
a A /. 5 , prin urmare A nu poate fi p
atrat perfect.
b) Suma A are 2005 termeni, pe care i vom grupa cte 5:

A = 5 + 52 + 53 + 54 + 55 + + 52001 + 52002 + 52003 + 52004 + 52005 =

= 5 + 52 + 53 + 54 + 55 1 + 55 + + 52000 = 3905 1 + 55 + + 52000 .

Cum 3905 = 5 11 71, num


arul A admite ca divizori pe 1, 5, 11, 55, 71.
V.57. Aflati restul mpartirii prin 47 a numarului N = 1268 99
. . . 9} .
| {z
2005 cifre

Alexandru Negrescu, elev, Botosani


Solutie. Observ
am c
a N + 1 = 1269 00
. . . 0} = 1269 102005 = 47 27 102005 , deci
| {z
2005 cifre

N + 1 se divide cu 47. Atunci restul mp


artirii lui N prin 47 este 47 1 = 46.
V.58. Aflati numerele naturale x, y, z cu proprietatea ca
24x+1 + 23y+1 + 22z+1 = 9248.

Cristian - C
at
alin Budeanu, Iasi
Solutie. Deoarece 924810 = 100100001000002 , egalitatea din enunt revine, datorit
a unicit
atii scrierii unui num
ar n baza 2, la faptul c
a (4x + 1, 3y + 1, 2z + 1)
{(13, 10, 5) , (13, 5, 10) , (10, 13, 5) , (10, 5, 13) , (5, 10, 13) , (5, 13, 10)}. Cercetnd fiecare caz n parte, obtinem solutii numai n prima si penultima situatie, anume (x, y, z)
{(1, 3, 6) , (3, 3, 2)}.
2

V.59. Daca a1 a2 . . . an 2 b1 b2 . . . bn = c1 c2 . . . cn 2 , sa se arate ca


2

a1 a2 . . . an a1 a2 . . . an 2 b1 b2 . . . bn b1 b2 . . . bn = c1 c2 . . . cn c1 c2 . . . cn 2 .

Petru Asaftei, Iasi


Solutie. Not
am A = a1 a2 . . . an , B = b1 b2 . . . bn , C = c1 c2 . . . cn . Observ
am c
a
2

a1 a2 . . . an a1 a2 . . . an 2 = (A 10n + A) = A2 (10n + 1)

si nc
a dou
a relatii similare. Astfel, egalitatea de demonstrat revine la
A2 (10n + 1)2 B 2 (10n + 1)2 = C 2 (10n + 1)2 ,
45

care se obtine nmultind ambii membri ai ipotezei cu (10n + 1) .


99

V.60. Sa se determine ultimele doua cifre ale numarului 79 .


Artur B
al
auc
a, Botosani
n
Solutie. Urm
arind ultimele
dou
a
cifre
ale
num
a
rului
7
(pe
care
le vom nota

U2 (7n )), observ


am c
a U2 74k = 01, U2 74k+1 = 07, U2 74k+2 = 49, U2 74k+3 =
9
43. Pe de alt
a parte, cum 9 = M4 + 1, atunci orice putere a lui 9 (deci si 99 ) este
99

de forma M4 + 1. Din aceste dou


a observatii, rezult
a c
a U2 79

= 07.

Clasa a VI-a

VI.56. Determinati, n functie de numarul ntreg x, cel mai mare divizor comun
al numerelor 2005x + 2 si 2006x + 3.
Tamara Culac, Iasi
Solutie. Not
am d = (2005x + 2, 2006x + 3). Vom aplica algoritmul lui Euclid:
dac
a a = bq + r, a, b, q Z, r < b, atunci (a, b) = (b, r). Observ
am c
a
2006x + 3 = (2005x + 2) 1 + (x + 1) ;
2005x + 2 = (x + 1) 2005 + (2003) ,

prin urmare (2005x + 2, 2006x + 3) = (2005x + 2, x + 1) = (x + 1, 2003). ns


a 2003
este num
ar prim, deci:
.
a) Dac
a x + 1 .. 2003, atunci d = 2003;
.
b) Dac
a x + 1 /.. 2003, atunci d = 1.
VI.57. Un vnzator de autoturisme scade procentul beneficiului sau de la 25% la
20% din valoarea vnzarilor. Datorita scaderii preturilor, creste valoarea vnzarilor.
Aflati procentul cu care a crescut valoarea vnzarilor, stiind ca beneficiul a crescut
cu 10%.
Marius Farcas, Iasi
Solutie. Not
am cu x valoarea initial
a a vnz
arilor si cu p procentul de crestere
a valorii vnz
arilor. Initial, beneficiul global era 25% din x, iar n final acesta devine
20% din ((100 + p) % din x). Conform ipotezei, putem scrie ecuatia:
20 100 + p
110 25

x=

x p = 37, 5 .
100
100
100 100
Prin urmare, vnz
arile au crescut cu 37, 5% .
VI.58. Se aseaza cifrele 2, 3, 4, 5, 6, 8, 9 ntr-o ordine oarecare obtinnd un
numar A. Se aseaza apoi aceleasi cifre n alta ordine, obtinnd numarul B 6= A. Sa
se arate ca A nu se divide cu B.
Cristian - C
at
alin Budeanu, Iasi
Solutie. S
tim c
a un num
ar natural si num
arul dat de suma cifrelor sale dau
acelasi rest la mp
artirea prin 9. Cum suma cifrelor lui A, respectiv B, este 2 + 3 +
4 + 5 + 6 + 8 + 9 = 37, atunci A = 9a + 1, B = 9b + 1, cu a, b N.
.
Presupunem prin absurd c
a A .. B, atunci A > B si A = B c 9a + 1 = (9b + 1) c,

cu 1 < c < 5 (deoarece A 9865432, iar B 2345689). Obtinem astfel c


a
9 (a bc) = c 1, relatie imposibil
a deoarece c {2, 3, 4} si n membrul drept
nu putem avea multiplu de 9. n concluzie A nu se poate divide cu B.
46

b = 120 . Daca mediana [BM ] este perpendiculara


VI.59. Fie 4ABC cu m(B)
pe BC, aratati ca AB = 2BC.
A
Bogdan Posa, elev, Motru (Gorj)
Solutie. Fie T mijlocul lui [AB]. Cum [M T ] este linie
1
mijlocie n 4ABC, rezult
a c
a M T = BC si M T k BC.
2
ns
a M B BC, deci 4BM T va fi dreptunghic n M , cu
1
\
m(M
BT ) = 120 90 = 30 . Prin urmare, M T = BT ,
2
adic
a BC = BT , de unde concluzia.

M
T

VI.60. Fie 4ABC si punctele E (AB), F (AC) si M (BC) astfel nct


AE = EB, iar ntre 4AEF si 4EF M sa existe o congruenta. Sa se arate ca:
a) F este mijlocul lui [AC];
b) [AM ] este mediana sau naltime.
Ioan S
ac
aleanu, Hrl
au
Solutie. a) Fie {O} = AM EF , iar T , R proiectiA
ile pe EF ale punctelor A, respectiv M . Cum ntre
4AEF si 4EF M exist
a o congruenta, cele dou
a triunghiuri vor fi echivalente, deci
1
1
O R F
SAEF = SMEF EF AT = EF M R AT = M R. E
T
2
2
ns
a AT k M R si rezult
a c
a AT M R este paralelogram.
De aici, O este mijlocul lui [AM ], adic
a [EO] este linie
mijlocie n 4ABM si atunci EO k BC. Cu reciproca
teoremei liniei mijlocii n 4ABC, obtinem c
a F este
mijlocul lui [AC].

b) Deosebim trei situatii:


[ EF
\
i) AEF
M ; atunci AE k F M si, cu reciproca teoremei liniei mijlocii, rezult
a
c
a M este mijlocul lui [BC], adic
a [AM ] este median
a.
[ F\
ii) AEF
EM ; atunci 4AET 4M ER (C.U.), deci ET = ER, prin urmare
T = R. Rezult
a n acest caz c
a [AM ] este n
altime.
[ EM
\
[
iii) AEF
F ; atunci, dac
a AF
E F\
EM vom obtine ca la i) c
a [AM ] este
[
\
median
a, iar dac
a AF E EF M , [AM ] va fi n
altime.

Clasa a VII-a
VII.56. Fie x, y R cu x2 2y = y 2 + xy = 4. Sa se arate ca x2 2x = y 2 .
Gigel Buth, Satu Mare
Solutie. Prin sc
aderea relatiilor din enunt obtinem c
a x2 y 2 = y (x + 2) si,

1 2
asim c
a
nlocuind y =
x 4 , g
2

1 3
2
2
(1)
x y =
x + 2x2 4x 8 .
2
Pe de alt
a parte, eliminnd direct pe y ntre relatiile date, obtinem
x3 + 2x2 4x 8 = 4x.
47

(2)

1
4x, adic
a x2 2x = y 2 .
2
VII.57. Fie x (0, 1), iar n N \ {0, 1}. Aratati ca nx2 + 2n > n + (1 + x)n .
Ion Visan, Craiova
Solutie. Avem succesiv:
h
i
n
n2
n1
2n (1 + x) = (2 1 x) 2n1 + 2n2 (1 + x) + + 2 (1 + x)
+ (1 + x)
>
h
i
> (1 x) (1 + x)n1 + (1 + x)n2 (1 + x) + + (1 + x)(1 + x)n2 + (1 + x)n1 =

= (1 x) n (1 + x)n1 (1 x) n (1 + x) = n 1 x2 ,

Din (1) si (2) rezult


a c
a x2 y 2 =

de unde concluzia problemei.


VII.58. Fie a, b R+ astfel nct mediile aritmetica, geometrica si armonica
ale lor sa fie laturi ale unui triunghi dreptunghic. Aflati sinusul celui mai mic dintre
unghiurile triunghiului.
Romanta Ghita
si Ioan Ghita
, Blaj
Solutie. Cum ma mg mh , ipotenuza va fi ma , iar cel mai mic dintre
unghiuri va fi cel ce se opune lui mh . Cum ma , mg , mh sunt laturi ale unui triunghi
dreptunghic, n ipoteza nerestrictiv
a a > b avem:

2
2
2
2
a+b
2ab
(a + b)
4a2 b2
=
ab +

m2a = m2g + m2h


ab =
2
a+b
4
(a + b)2


(a b)2
4a2 b2
2
2
2
2

b
=
4ab

4ab

b
=
0

a
=
b
2

5 .
=
2
4
(a + b)

Evident, convine doar a = b 2 + 5 . Sinusul unghiului dorit este


4b2 2 + 5
4ab
mh
51
=
=
sin =
.
2 =
2
ma
2
(a + b)
b2 3 + 5

VII.59. Fie 4ABC si A0 mijlocul lui [BC]. Daca D (AC), BD AA0 = {F }


1
1
1
1
si paralela prin F la BC taie AC n E, sa se arate ca
=
+
+
.
DE
AD CE
AC
Claudiu - S
tefan Popa, Iasi
Solutia 1. Fie {E 0 } = EF AB si D0 [BF ] astfel
A
nct DF = D0 F . Cum [AA0 ] median
a n 4ABC si
EE 0 k BC, atunci EF = E 0 F . Rezult
a c
a DED0 E 0
0 0
este paralelogram, deci [D E ] este paralel si congruent
D
BE 0
0 0
a
cu [DE]. De aici, 4BE D 4BAD, adic
=
F
E
E
BA
0 0
0
0 0
BE
CE
ED
CE
ED
. ns
a
=
, deci
=
, de unde
AD
BA
CA
AD
CA
D
1
E 0 D0
ED
=
=
. Obtinem succesiv:
AC
AD CE
AD CE
B
A
C
1
AD + DE + CE
1
AC
=

AD CE
ED
AD CE
ED
1
1
DE
1
1
1
1
1
=
+
+

=
+
+
.
ED
CE
AD CE
AD
DE
CE
AC
AD
48

AF
= k, k (0, 1). Din teorema lui Menelaus
AC
BA0 CD AF
= 1, de unde

n 4AA0 C cu transversala B F D, obtinem


BC DA F A0
2k
AE
2k
2k
AF
=
. Deoarece 4AF E 4AA0 C, atunci
=
si AE =
b.
F A0
1k
AC
1+k
1+k
1k
k (1 k)
b si CE = AC AE =
b. Relatia
Prin urmare, DE = AE AD =
1+k
1+k
1+k
1 1+k
din enunt este echivalent
a cu
= +
+ 1, care se verific
a imediat.
k (1 k)
k 1k
Solutia 2. Fie AC = b si

VII.60. n cercul C se considera coardele [AM ] si [AN ] astfel nct AM < AN .


1) Sa se determine multimea punctelor X C ce ndeplinesc conditia AM
AX AN .
2) n ce caz multimea gasita este un arc de cerc?
Temistocle Brsan, Iasi
Solutie. 1) Consider
am cercurile de centru A si raze [AM ] si [AN ]. Acestea
intersecteaz
a a doua oar
a cercul C n punctele M 0 si respectiv N 0 . Punctele X ce
satisfac conditia AM AX AN se afl
a n coroana determinat
a de aceste cercuri.
Punctele X C ce satisfac aceast
a conditie se afl
a pe arcele cercului C interceptate
de coroan
a.

N
M
O

A
M

\
Dac
a O, centrul cercului C, este exterior unghiului M
AN , atunci multimea c
au0 0
\
tat
a este M N M N . Dac
a O este interior unghiului M
AN , atunci aceast
a multime

este M N 0 M 0 N (acest caz se reduce la precedentul prin nlocuirea coardei [AN ] cu


[AN 0 ]).
2) n ambele cazuri, obtinem un arc de cerc atunci cnd [AN ] este diametru.

Clasa a VIII-a
VIII.56. Sa se determine x, y, z R pentru care x3 y 3 + z 3 = 8, x y + z = 2.
Andrei - Sorin Cozma, elev, Iasi
Solutie. Este cunoscut
a identitatea
3

(a + b + c) = a3 + b3 + c3 + 3 (a + b) (b + c) (c + a) ,

a, b, c R.

Pentru a = x, b = y, c = z, folosind si ipoteza, obtinem c


a 8 = 8+3 (x y) (z y)
(x + z), deci x y = 0, z y = 0 sau x + z = 0. nlocuind pe rnd n a doua ecuatie,
g
asim solutiile sistemului: {(2, , ) , (, 2, ) , (, , 2) | , , R }.
49

VIII.57. Fie a, b, c > 0 astfel nct a + b + c = 1. Sa se arate ca


1
1
9
1
p
+p
+p
.
2
(1 a) (1 b)
(1 b) (1 c)
(1 c) (1 a)

Cristian S
avescu, elev, Focsani
Solutie. Evident c
a a, b, c sunt subunitare, deci au sens radicalii. Aplicnd
inegalitatea mediilor M G M A, obtinem
1
1
2
1
p

=
=
1

a
+
1

b
2

(1

c)
1
+
c
(1 a) (1 b)
2
2
si nc
a dou
a relatii analoage. Folosind acum inegalitatea mediilor M H M A sub
n 1
n
P
P
forma
n2
xi , deducem c
a
i=1 xi
i=1

X
1
1
1
9
9
1
p
2
+
+
2
= .
1+c 1+a 1+b
3 + (a + b + c)
2
(1 a) (1 b)
Not
a. Mai general, dac
a a1 , a2 , . . . , an > 0, a.. a1 + a2 + + an = k, atunci
1
1
n2
p
+ + p

.
(n 1) k
(k a1 ) (k a2 )
(k an ) (k a1 )

VIII.58. Fie x, y, z (0, ) cu x + y + z 3. Sa se arate ca xn + y n + z n 3,


n N.
Romeo Ilie, Brasov
Solutiile. Pentru n = 0, concluzia este evident
a; fie deci n 1. Conform
inegalit
atii mediilor M A M G, are loc:

n
n
xn + (n 1) = xn + |1 + 1 +
(1)
{z + 1} n x 1 1 . . . 1 = nx,
n1

pentru x (0, ) si n 2; pentru n = 1, inegalitatea (1) se transform


a n egalitate.
Aplicnd de trei ori (1), obtinem c
a
xn + y n + z n + 3 (n 1) nx + ny + nz = n (x + y + z) 3n,

de unde rezult
a concluzia.

4
VIII.59. Determinati x, y, z R, stiind ca x+y+z = 1, iar xy+(x + y)(z + 1) = .
3
Gheorghe Molea, Curtea de Arges
Solutia 1. Avem:
2

(x + y + z) = 1 x2 + y 2 + z 2 + 2 (xy + xz + yz) = 1

4
5
2
2
2
x +y +z +2
x y = 1 x2 + y 2 + z 2 2x 2y =
3
3
1
2
2
()
(x 1) + (y 1) + z 2 = .
3
1
Pe de alt
a parte, din binecunoscuta inegaliate 2 + 2 + 2 ( + + )2 (care
3
se reduce, dup
a calcule, la ( )2 + ( )2 + ( )2 0), pentru = x 1,
50

= y 1, = z, obtinem
(x 1)2 + (y 1)2 + z 2

1
1
1
[(x 1) + (y 1) + z]2 = (1)2 = ,
3
3
3

cu egalitate pentru x 1 = y 1 = z. Relatia () arat


a c
a se atinge egalitate, deci
avem, ntr-adev
ar, x1 = y1 = z x = z +1, y = z +1. nlocuind n x+y +z = 1,
2
2
1
obtinem c
ax= ,y= ,z= .
3
3
3
Solutia 2. Eliminnd pe z ntre cele dou
a relatii, obtinem xy+(x + y) (2 x y) =
4
2
2
a relatie ca ecuatie
sau nc
a 3x + 3y + 3xy 6x 6y + 4 = 0. Considernd aceast
3
2
2
de gradul doi cu necunoescuta x, avem: 3x + x (3y 6) + 3y 6y + 4 = 0. Cum
2
2
2
a y = . Pentru y =
x R, atunci x 0; iar x = 3 (3y 2) 0 implic
3
3
2
1
g
asim x = si z = .
3
3

4 3
0 0 0
VIII.60. Se considera prisma triunghiulara regulata ABCA B C cu AB =
3


si AA0 = 3 3. Sa se arate ca pentru fiecare numar a 0, 3 3 , exista exact doua
puncte Ma0 , Ma00 pe dreapta CC 0 astfel nct d (B 0 , (Ma0 AB)) = d (B 0 , (Ma00 AB)) = a.
Mirela Marin, Iasi
Solutie. Fie M CC 0 si vom nota cu x lungimea

C
segmentului CM , considernd x 0 dac
a M [CC 0 si A

N
0
0
0 0
x < 0 dac
a M CC \[CC . Deoarece A B k AB si AB
B
(M AB), atunci A0 B 0 k (M AB), deci d (B 0 , (M AB)) =
M
0
0
0 0
d (N , (M AB)), unde N este mijlocul lui [A B ]. Fie N
0
mijlocul lui [AB], iar P proiectia lui N pe M N . Cum
(N CC 0 ) (ABN ), atunci vom avea c
a N 0 P (ABM ),
0
0
P
adic
a d (B , (M AB)) = N P .
C
A
0
Din asem
anarea imediat
a 4N N P 4N M C,
N
NC NN0
NN0
N 0P
=
N 0P =
=
obtinem c
a
B
NC
MN
MN

23 3

. (Am folosit egalit


atile evidente N C = 2, N N 0 = 3 3, M N =
2
4+x
q

N C 2 + M C 2 = 4 + |x|2 = 4 + x2 .)


6 3
= a are exact
Pentru fiecare valoare a 0, 3 3 , ecuatia N 0 P = a
4 + x2

2 27 a2
dou
a solutii, anume x =
, de unde concluzia problemei.
a

Clasa a IX-a
IX.56. Determinati numerele reale pozitive x, y, z, t pentru care x+y+z+t = 20,
iar xy + xz + xt + yz + yt + zt + 475 = xyzt.
Lucian Tutescu si Liviu Smarandache, Craiova
51

Solutie. Conform inegalit


atii mediilor,

4
x+y+z+t
xyzt xyzt 625.
4

(1)

Tot conform inegalit


atii mediilor, obtinem

xyzt = xy + xz + xt + yz + yt + zt + 475 6 6 xy xz xt yz yt zt + 475 =

= 6 xyzt + 475.

a devine
Cu notatia a = xyzt, inegalitatea precedent
a2 6a + 475 (a 25) (a + 19) 0 a (, 19] [25, +).

ns
a evident c
a a > 0, prin urmare a [25, +), i.e. xyzt 625. Rezult
a c
a se
atinge egalitatea n (1), fapt care are loc atunci cnd x = y = z = t = 5.
IX.57. Sa se determine toate functiile f : R R cu proprietatea
f (f (f (x) + y) f (x f (y))) = x2 y 2 ,

x, y R.

Adrian Zahariuc, elev, Bac


au
Solutie. Pentru x = y = 0, obtinem c
a f (f (f (0)) f (f (0))) = 0; not
am
a = f (f (0)) f (f (0)), deci f (a) = 0. Pentru x = y = a, g
asim c
a
f (f (f (a) + a) f (a f (a))) = 0,

adic
a f (f (a) f (a)) = 0, prin urmare f (0) = 0.
Lu
am acum y oarecare si x = f (y) si obtinem f 2 (y) y 2 = 0, deci f (x) = x,
x R. Consider
am multimile A = {x R | f (x) = x} si B = {x R | f (x) = x}.

Dac
a A, B 6= R , exist
a x A si y B cu x 6= y si, ntorcndu-ne n ecuatia initial
a,
x2 y 2 = f (f (f (x) + y) f (x f (y))) = (x + y)2 x y = (x + y) ,

ceea ce contrazice x 6= y.
a f (x) = x, x R sau f (x) = x, x R.
Rezult
a c
a A = R sau B = R , adic
Aceste dou
a functii verific
a ecuatia, deci sunt solutii.
IX.58. Fie n N, n 2. Determinati a1 , a2 , . . . , an1 , c R pentru care
[x] + [x + a1 ] + + [x + an1 ] = [cx] ,

x R.

[x] + [x + b1 ] + + [x + bn1 ] = [cx] ,

x R.

(n leg
atur
a cu G.42 din RecMat 1/2003.)
Iuliana Georgescu si Paul Georgescu, Iasi
Solutie. Not
am ki = [ai ], bi = {ai }, i = 0, n 1. Cum bi [0, 1) pentru orice i,
lund x = 0 n relatia din ipotez
a obtinem k1 + k2 + + kn1 = 0, de unde
(1)

S
a presupunem, pentru fixarea ideilor, c
a b1 b2 bn1 ; n orice alt caz se
rationeaz
a analog. Lund, pe rnd, x = 1, apoi x = 1 n (1), obtinem c
a [c] = n,
respectiv [c] = n, prin urmare c = n.
1
Pentru x = b1 n (1), obtinem c
a 1 = [nb1 ], deci b1 . Apoi, pentru
n
2
i
a 2 = [nb2 ], deci b2
si, n continuare, bi ,
x = b2 n (1), obtinem c
n
n
i = 1, n 1.
52

1
n1
1
n (1) si g
asim c
a
+ bn1 = 1, deci bn1
; n
n
n
n
n1
2
n1
. Lund succesiv x = , . . . , x =
n (1), obtinem c
a
concluzie, bn1 =
n
n
n
i
bi = , i = 1, n 1.
n

1
2
n concluzie, {a1 , a2 , . . . , an1 } este o multime de forma k1 + , k2 + , . . . ,
n
n

n1
kn1 +
, cu k1 + k2 + + kn1 = 0, k1 , k2 , . . . , kn1 Z, iar c = n.
n
IX.59. Fie C (I, r) cercul nscris n 4ABC. Sa se arate ca
IA IB + IB IC + IC IA 12r2 .
D. M. B
atinetu - Giurgiu, Bucuresti
Solutia 1. Evident c
a IA + r AIA ha , unde IA = PrBC I, deci:
IA
b+c
IA+r ha a IA+ar aha = 2S = ar+br+cr a IA (b + c) r

r
a
si nc
a dou
a inegalit
ati similare. Prin urmare,

IA IB
b+c c+a
4c ab
4c

=
r2
a
b
ab
ab
si analoagele. Prin urmare, obtinem
s
X a
abc
IA IB
IB IC
IC IA
4 3 3 = 12
+
+
4
r2
r2
r2
bc
ab bc ca
Lu
am acum x =

IA IB + IB IC + IC IA 12r2 ,
cu egalitate n cazul triunghiului echilateral.
r
Solutia 2 (Mihai Haivas). Cum IA =
si analoagele, trebuie s
a demonsin A2
P
1
12. Conform inegalit
atii mediilor si binecunoscutei
str
am c
a
A
sin 2 sin B2
A
B
C
8 sin sin sin 1, avem:
s
2
2
2
X

1
1
3
2
33
2 A
2 B
2 C 3 8 = 12.
A
B
sin 2 sin 2
sin 2 sin 2 sin 2

IX.60. Fie ABC un triunghi ascutitunghic cu a < b < c. Cercul C (I, r) nscris
triunghiului este tangent dreptelor BC, CA si AB n punctele D, E si respectiv
F . Dreapta ID intersecteaza CA n D0 si AB n D00 , IE intersecteaza AB n E 0
si BC n E 00 , iar IF intersecteaza BC n F 0 si CA n F 00 . Aratati ca E 0 E 00 =
D0 D00 + F 0 F 00 .
Temistocle Brsan, Iasi
Solutie. Not
am cu Ha , Hb , Hc picioarele n
altimilor din vrfurile A, B si respectiv C. Conditia a < b < c impune ordinile urm
atoare: B DHa C, C Hb E A
si A F Hc B. De aici si din faptul c
a ID k AHa , deducem c
a punctele D0 si
00
D au pozitiile indicate pe figur
a; la fel se procedeaz
a pentru precizarea pozitiilor
punctelor E 0 , E 00 si F 0 , F 00 .
Vom utiliza urm
atoarele relatii cunoscute: AE = AF = p a, BD = BF = p b,
53

CD = CE = p c (2p = a + b + c). Cu aceste


preg
atiri, avem:
D0 D00 = DD0 DD00 = (p c) tg C (p b) tg B,
E 0 E 00 = EE 00 EE 0 = (p c) tg C (p a) tg A,
F 0 F 00 = F F 0 F F 00 = (p b) tg B (p a) tg A

si, ca urmare, stabilirea egalit


atii E 0 E 00 = D0 D00 +
F
0 00
E
F F este imediat
a.
I
E
Not
a. Prof. Titu Zvonaru observ
a c
a, dac
a nF
locuim cercul nscris cu cercul A-exnscris, concluzia
B
D C F
problemei r
amne adev
arat
a. S
i pentru celelalte cer- E
curi exnscrise are loc concluzia, n sensul c
a cel mai mare dintre segmente este suma
celorlaltor dou
a.

Clasa a X-a
X.56. Fie tetraedrul ABCD si M un punct n spatiu. Daca G, GA , GB , GC ,
GD sunt centrele de greutate ale tetraedrelor ABCD, M BCD, M ACD, M ABD,

respectiv M ABC, sa se arate ca AGA + BGB + CGC + DGD = 0 daca si numai


daca M G.
Marius Olteanu, Rmnicu Vlcea
Solutie (Alexandru Negrescu, elev, Botosani). Avem:
X
X
X X

AG + GGA = 0
AGA = 0
AG +
GGA = 0
X
1 X

GM + GB + GC + GD = 0
GGA = 0
4
3 X

GM +
GA = 0 GM = 0 G M.
4
X.57. Daca x, y, a (1, ), sa se arate ca

(x + y + loga x) xy + loga xx+y + (x + y + loga y) xy + loga y x+y

(x + xy + y)2 loga xy.


Mihail Bencze, Brasov
Solutie. Observ
am c
a


1
x z
z
1 1
x y
y
(x + y + z)
+ +
=3+ + +
+
+
+

x y z
y
x
z
y
x z
r
r 2

r
r
x
y
x
y
x y
.
+2
= 1+
+
3+ + +2
y
x
y
x
y
x
Considernd, pe rnd, z = loga x, apoi z = loga y, obtinem:

2
(x + y + loga x) xy + loga xx+y (x + xy + y) loga x;

2
(x + y + loga y) xy + loga y x+y (x + xy + y) loga y.

Adun
am membru cu membru cele dou
a inegalit
ati si rezult
a concluzia.
X.58. Let n N, n 1. Prove that

n 2
X
k2 + n2
2n 2
n
ln
<
.
n1
k
n2
k=1
Jos Luis Daz - Barrero, Barcelona, Spain
54

Solutie. Este cunoscut


a inegalitatea ln (1 + x) < x, x > 0, de unde se obtine
k 2 + n2
k2
c
a ln
< 2 , k, 1 k n. Atunci
n2
n

2
n
n 2
n 2
n
2
2
X n
X
X
k n
k + n2 X n k2
2n 2
n1
ln
<
=
=
=
.
n1
k
k
k1
n2
n2
n k
k=1

k=1

k=1

k=1

X.59. Fie f R [X] un polinom de grad n 3 ce admite n radacini reale,


pozitive si subunitare. Daca |f (0)| = f (1), sa se arate ca produsul radacinilor este
1
cel mult egal cu n .
2
Ioan S
erdean, Or
astie
Solutie. Putem presupune, f
ar
a a restrnge generalitatea c
a f are coeficientul
ad
acinile xi (0, 1),
dominant 1; fie f (X) = X n + an1 X n1 + + a1 X + a0 , cu r
i = 1, n. Evident c
a x1 x2 xn (0, 1). Folosind ipoteza, ultima relatie Vite si
descompunerea polinomului n factori, avem:
x1 x2 xn = |f (0)| = f (1) = (1 x1 ) (1 x2 ) (1 xn )

1
,
4n
1
1
conform inegalit
atilor 0 xi (1 xi ) , i = 1, n. Rezult
a x1 x2 xn n .
4
2
X.60. Fie k N fixat. Alegem n {1, 2, . . . , k} si a1 , a2 , . . . , an numere prime
mai mari dect 3. Daca probabilitatea ca a21 + a22 + + a2n sa se divida cu 24 este
1
cel putin egala cu
, sa se arate ca 24 divide k.
24
Cristian S
avescu, elev, Focsani
2

(x1 x2 xn ) = [x1 (1 x1 )] [x2 (1 x2 )] . . . [xn (1 xn )]

Solu
Cum ai sunt prime, atunci ai 1 (mod 3), deci a2i 1 (mod 3),
Ptie.
2
adic
a
ai n (mod 3). Analog,
8); n ambele
P 2ai 1 (mod 8) sau ai 3P(mod
cazuri, a2i 1 (mod 8), deci
ai Pn (mod 8). Rezult
a c
a
a2i n (mod 24)
si atunci probabilitatea
c
a 24
a2i este aceeasi cu probabilitatea ca n s
a se
divide
k 1
k 1
1
divid
a cu 24, care este

=
. Cum se atinge egalitatea conform
24 k
24
24 k
k
k
ipotezei, obtinem c
a
=
, adic
a 24 divide k.
24
24

Clasa a XI-a
XI.56. Let n be a positive integer. For each positive integer k, let Fk be the kth
Fibonacci number ( F1 = F2 = 1, Fk+2 = Fk+1 + Fk for all k 1). Prove that

2
2

Fn2
Fn+1
2Fn Fn+1 Fn+2

2
2
Fn+2
2Fn+1 Fn+2 Fn2
Fn+1

= 0.

2
2
2
2Fn Fn+2 Fn+1

Fn+2
Fn

Jos Luis Daz - Barrero, Barcelona, Spain


Solutie. Not
am cu n determinantul din problem
a; adunnd prima si a doua
coloan
a la cea de-a treia, obtinem c
a
55

Fn2

2
n =
Fn+2

2F F
2
n n+2 Fn+1

2
Fn+1

2Fn+1 Fn+2 Fn2


2
Fn+2

2
(Fn+1 + Fn )2 Fn+2

(Fn+2 Fn+1 )2 Fn2 .

2
(Fn+2 Fn )2 Fn+1

Folosind relatia de recurenta, observ


am c
a toti termenii de pe a treia coloan
a sunt
nuli, prin urmare n = 0.
XI.57. Fie patratul ABCD circumscris cercului C. n patrat se nscrie octogonul EF GHIJKL, circumscris cercului C, astfel nct E, F (AB), E (AF ),
G, H (BC), G (BH), I, J (CD), I (CJ), K, L (DA), K (DL). Fie
{M } = EL AC, {N } = F G BD, {P } = HI AC, {Q} = JK BD. Sa se arate
ca suma
AE BF BG CH CI DJ DK AL AM BN CP DQ
S=
+
+
+
+
+
+
+
+
+
+
+
EB F A GC HB ID JC KA LD M C N D P A QB
nu depinde de alegerea vrfurilor octogonului pe laturile patratului.
C
at
alin Calistru, Iasi
Solutie. Vom demonstra c
a
DK
DQ DJ
+
+
= 1.
(1)
KA
QB
JC
Odat
a justificat
a relatia (1), scriind nc
a trei egalit
ati similare corespunz
atoare vrfurilor A, B, C si adunndu-le, obtinem concluzia.
Vom demonstra (1) analitic: fix
am un reper cu originea n centrul p
atratului, astfel nct A (1, 1), B (1, 1), C (1,
1),
D
(1,
1).
Ecua
t
ia
cercului
este

2
2
C : x + y = 1 si fie T (cos t, sin t), t 0,
, punctul de contact cu cercul al
2

1 sin t
1 cos t
; K
,1 ;
dreptei KJ : x cos t + y sin t = 1; obtinem c
a J 1,
sin t
cos t

1
1
DJ
sin t + cos t 1 DQ
sin t + cos t 1
Q
,
. Atunci
=
;
=
;
sin t + cos t sin t + cos t
JC
sin t cos t + 1 QB
sin t + cos t + 1
DK
sin t + cos t 1
=
si, dup
a calcule de rutin
a n care nu intervine dect formula
KA
cos t sin t + 1
2
2
a c
a (1) este adev
arat
a.
fundamental
a sin t + cos t = 1, rezult
Not
a. Dl C
at
alin Calistru, autorul problemei, face cunoscut redactiei revistei
urm
atoarea posibil
a generalizare a relatiei (1):
a tangent
a cercului nscris poligonuFie A1 A2 . . . An un poligon regulat, d o dreapt
lui si P1 , P2 , . . . , Pn1 punctele determinate prin {Pi } = d A1 Ai+1 , i = 1, n 1.
Ar
atati c
a
A1 P
A1 P2
A1 Pn1
+
+ +
= 1.
P1 A2 P2 A3
Pn1 An
XI.58. Daca n este un numar natural iar p un numar prim, atunci sirul
(xn+1 (p) xn (p))n0 este divergent, unde xn (p) reprezinta exponentul cu care apare
numarul p n descompunerea lui n!.
Sorin Puspan
a, Craiova
Solutia 1.
Not
a
m
y
=
x
(p)

x
(p),
n

N.
Conform
teoremei
lui
Legendre
n
n n+1
n
pn 1
n
n
xn (p) =
a xpn (p) = xpn +1 (p) =
+ 2 + 3 + . . . . Obtinem c
, deci
p
p
p
p1
56

pn 1
pn 1
; xpn +p =
+ 1, deci ypn +p1 = 1. Astfel,
p1
p1
sunt subsiruri cu limite diferite ale sirului (yn ), prin urmare

ypn = 0 si xpn +p1 (p) =


(ypn )nN si (ypn +p1 )nN
(yn ) este divergent.

Solutia 2. Cazul p = 2 se trateaz


a ca mai sus. S
a presupunem c
a exist
ap3
a fie convergent. Atunci, conform lemei Cesaroastfel nct (xn+1 (p) xn (p))n0 s

xn (p)
este convergent si ultimele dou
a siruri au aceeasi limit
a.
Stolz, sirul
n
n1
1
1
xn (p)
Cum lim
=
, obtinem lim (xn+1 (p) xn (p)) =
, contradictie:
n
n
n
p1
p1
1
pentru p 3,

/ N, iar xn+1 (p) xn (p) N.


p1
XI.59. Fie sirul de numere supraunitare (an )n1 , astfel ca lim an = 1. Sa se
n

studieze continuitatea functiei f : R R, f (x) = lim {xan }, unde {x} reprezinta


n
partea fractionara a numarului real x.
Dan Popescu, Suceava
Solutie. Se stabileste cu usurinta c
a:
a) pentru x0
/ Z, lim [x] = [x0 ];
xx0

b) pentru x0 Z, lim [x] = x0 1, lim [x] = x0 .


x%x0

x&x0

Cum f (x) = lim (an x [an x]) = x lim [an x], obtinem:
n

1. pentru x
/ Z, f (x) = x [x] = {x};
2. pentru x Z, f (x) = x x = 0 = {x}, x > 0 (deoarece an x x si an x > x),
f (0) = 0, iar f (x) = x x + 1 = 1, x < 0 (deoarece an x x si an x < x).
1, x Z, x < 0
Prin urmare, f (x) =
. Astfel, f este continu
a pe R \ Z.
{x} , n rest
2
XI.60. Fie a (0, 1); sa se demonstreze ca pentru orice x >
este valabila
ln a
1

1
inegalitatea (1 ax ) x+1 < 1 ax+1 x .
Angela Tig
aeru, Suceava

x+1
x x
Solutie. Inegalitatea dat
a este echivalent
a cu (1 a ) < 1 ax+1
, x>

2
x

. Logaritmnd, aceasta este echivalent


a cu x ln (1 a ) < (x + 1) ln 1 ax+1 .
ln a
2
Ar
at
am c
a functia f (x) = x ln (1 ax ), x >
este strict cresc
atoare.
ln a
ax ln a
(1 ax ) ln (1 ax ) ax ln ax
=
.
f 0 (x) = ln (1 ax ) x
1 ax
1 ax
2
Cum 1 ax > 0 pentru x >
, a (0, 1), este suficient s
a demonstr
am c
a
ln a
2
2
(1 ax ) ln (1 ax )ax ln ax > 0 pentru x >
. Not
am t = ax si cum x >
,
ln a
ln a

2
a (0, 1), atunci t 0, 1/e . Prin urmare,
a demonstr
am c
a g (t) =
trebuie s
(1 t) ln (1 t) t ln t > 0 pentru t 0, 1/e2 . Avem g 0 (t) = ln (1 t) ln t 2.

1
a c
a ln ((1 t) t) < 2 sau nc
a
Cum, pentru t 0, 1/e2 , (1 t) t < 2 , rezult
e
57

0
2
ln t ln (1 t) 2 >
0,
deci
g
(t)
>
0
pentru
t

0,
1/e
, adic
a g 2este

strict
2
cresc
atoare pe 0, 1/e . Cum lim g (t) = 0, deducem g (t) > 0, t 0, 1/e , ceea ce
t&0

trebuia demonstrat.

Clasa a XII-a
XII.56. Fie Sn multimea permutarilor de ordin n, iar Sn . Se considera
functia marginita f : R
R. Sa se calculeze:

1
1
1
lim
f ( (1)) + f ( (2)) + + f ( (n)) .
n n
2
n
(O generalizare a problemei 24131, G. M. 5-6/1999.)
Marius Olteanu, Rmnicu
Vlcea

1
1
1
Solutie. Fie an =
f ( (1)) + f ( (2)) + + f ( (n)) . Deoarece f
n
2
n
este m
arginit
a exist
a m, M R astfel nct m f (x) M , x R. Prin urmare,

m
M
1
1
1
1
1 + + +
an
1 + + +
n
2
n
n
2
n

1
1
1
1 + + +
= 0, deducem lim an = 0.
si cum lim
n n
n
2
n

1 1
XII.57. Consideram matricea A =
si multimea
1 1

1
.
G = Xa | Xa = I2 + aA, a ,
2
Aratati ca (G, ) este grup izomorf cu (R, +). Calculati X 12 X 32 X 2n1 , n N .
2
Gheorghe Iurea, Iasi
usor c
a Xa Xb = Xa+b+2ab si cum a + b +
Solu
Deoarece
A2 = 2A,
tie.

deducem

1
2ab 2 , pentru a, b 12 , , nmultirea matricelor este lege de compozitie
pe G. (G, ) este grup cu elementul neutru I2 , iar inversul fiec
arui element Xa G
a
este X 1+2a
G. Functia f : G R, f (Xa ) = ln (2a + 1) realizeaz
a izomorfismul
cerut. Fie Xt = X 12 X 32 . . . X 2n1 , atunci:
2

f (Xt ) = f X 12 X 32 . . . X 2n1 = f X 12 + f X 32 + + f X 2n1


2

ln (1 + 2t) = ln 2 + ln 4 + + ln 2n.
2n n! 1
Deducem c
a 1 + 2t = 2 4 . . . 2n = 2n n!, deci t =
.
2
XII.58. Let f : R [1, 1] be a continuous function. Prove that
Z 1
Z 1q
3
1 (f (x))2 dx +
f (x) dx 4.
1

Zdravko Starc,
p Vrac, Serbia and Montenegro
Solutie. Pentru orice a [1, 1] avem: 3 (1 a2 ) + a 2, cu egalitate pentru
p
1
a relatie
a = . Prin urmare 3 (1 f 2 (x)) + f (x) 2, x R; integrnd aceast
2
ntre 1 si 1 obtinem inegalitatea cerut
a. Egalitate avem pentru f : R [1, 1],
1
f (x) = .
2
58

XII.59. Fie f : R R, f derivabila si neconstanta pe nici un interval al lui R.


Daca
f 0 (x)
f 0 (sin x) cos x + f 0 (cos x) sin x, x R,
1 + f 2 (x)
demonstrati ca nu exista lim f (x).
x
Paul Georgescu si Gabriel Popa, Iasi
Solutie. Pentru orice x1 , x2 R, x2 > x1 , integrnd membru cu membru inegalitatea din enunt avem
arctg f (x2 ) arctg f (x1 ) f (sin x2 ) f (sin x1 ) f (cos x2 ) + f (cos x1 ) ,
prin urmare g (x2 ) g (x1 ), unde g (x) = arctg f (x) f (sin x) + f (cos x), x R,
deci g este monoton cresc
atoare pe R. Cum g este si m
arginit
a, deducem c
a exist
a
lim g (x) R. Presupunem c
a exist
a lim f (x); atunci exist
a

lim (f (sin x) f (cos x)) = lim (arctg f (x) g (x)) R.

Deoarece h (x) = f (sin x) f (cos x) este periodic


a si are limit
a la , h este con
stant
a; exist
a deci c R astfel nct f (sin x) f (cos x) = c. Pentru x = 0 si x =
2
g
asim c = f (0) f (1) = f (1) f (0), deci c = 0, adic
a f (sin x) f (cos x) = 0,
x R.
Prin urmare g (x) = arctg f (x) si cum g este monoton
a, rezult
a c
a f este monoton
a;
dar f (0) = f (1) si atunci f (x) = f (0), x [0, 1], contradictie.
XII.60. Fie n > 1 si a1 , a2 , . . . , an (0, 1) astfel nct a1 + a2 + + an = 1. Sa
f (x)
se determine functiile f : R R, daca lim
= 1 si f (x) = f (a1 x) + f (a2 x) +
x0 x
+ f (an x) pentru orice x R.
Gabriel Dospinescu, Paris
Solutie. Vom demonstra prin inductie c
a

X
p!
f (x) =
()
f ak11 ak22 aknn x
k1 !k2 ! kn !
k1 ,...,kn 0
k1 +k2 ++kn =p

pentru orice x si orice p.


Pentru p = 1 este chiar relatia din enunt. Presupunem c
a relatia este adev
arat
a
pentru p si o demonstr
am pentru p + 1.nlocuim n relatia () pe rnd pe x cu a1 x,
. . . , an x si nsum
am relatiile obtinute, tinnd cont de relatia din enunt; rezult
a:

X
p!
f (x) =
f ak11 +1 aknn x + +
k1 ! kn !
k1 ,...,kn 0
k1 +k2 ++kn =p

k1 ,...,kn 0
k1 +k2 ++kn =p

k1 ,...,kn 0
k1 +k2 ++kn =p

p!
f ak11 aknn +1 x =
k1 ! kn !

p! (k1 + 1)
f ak11 +1 aknn x + +
(k1 + 1)! kn !
59

p! (kn + 1)
f ak11 aknn +1 x .
k1 ! kn1 ! (kn + 1)!

k1 ,...,kn 0
k1 +k2 ++kn =p
= k10 , . . . , kn +

Notnd k1 + 1

1 = kn0 , obtinem:

0
p!k10
k1
kn

a
x
+ +
f
a
n
1
k10 ! kn !

f (x) =

k10 ++kn =p+1

0 =p+1
k1 ++kn

0
p!kn0
kn
k1

a
x
.
f
a
n
1
k1 ! kn0 !

Renotnd indicii de sumare obtinem:

X
(k1 + k2 + + kn ) p! k1 k2
f a1 a2 aknn x =
f (x) =
k1 !k2 ! kn !
k1 ,...,kn 0
k1 +k2 ++kn =p+1

k1 ,...,kn 0
k1 +k2 ++kn =p+1

(p + 1)!
f ak11 ak22 aknn x ,
k1 !k2 ! kn !

ceea ce trebuia demonstrat.


Fie acum un > 0, arbitrar si un a > 0 fixat. Cum lim

x0

f (x)
= 1, exist
a
x

f (x)
< 1 + . Deoarece
x
k1 k2
kn
k1 + k2 + + kn = p, atunci a1 a2 an < (max (a1 , a2 , . . . , an ))p ; dar cum
p
max (a1 , a2 , . . . , an ) < 1, avem lim (max (a1 , a2 , . . . , an )) = 0, deci exist
a p0 astfel

> 0 astfel nct dac


a x (0, ), atunci 1 <
p

.
a
Deci pentru orice x (0, a) si orice k1 , . . . , kn cu suma p0 , avem

f ak11 aknn x
< 1 + .
1<
ak11 aknn x
Atunci

X
X
p0 !
p0 !
(1 )
xak11 aknn <
f ak11 aknn x <
k1 ! kn !
k1 ! kn !

nct (max (a1 , a2 , . . . , an ))

p0

<

k1 ++kn =p0

k1 ++kn =p0

< (1 + )

k1 ++kn =p0

p0 !
xak1 aknn .
k1 ! kn ! 1

p0 !
ak11 aknn = (a1 + + an )p0 = 1 si atunci (1 ) x <
k
!

k
!
1
n
k1 ++kn =p0
f (x) < (1 + ) x pentru orice x (0, a) si orice > 0, ceea ce atrage f (x) = x
pentru x (0, a).
Cum a a fost ales arbitrar rezult
a f (x) = x pentru x 0. S
a observ
am c
a
acelasi rationament functioneaz
a si pe (, 0), cu mentiunea c
a n acest caz toate
inegalit
atile si schimb
a sensul, prin urmare f (x) = x pentru x < 0. Prin urmare
f (x) = x pentru x R.
ns
a

60

Solutiile problemelor pentru preg


atirea concursurilor
din nr. 1 / 2005
A. Nivel gimnazial

2
x y = u2
y2 z = v2 .
G76. Rezolvati n multimea numerelor naturale sistemul
2
z x = t2
Adrian Zanoschi, Iasi
Solutia 1. Dac
a x = 0, atunci y = u2 si apoi y = 0, u = 0, z = 0, v = 0,
t = 0. Obtinem astfel solutia (x, y, z, u, v, t) = (0, 0, 0, 0, 0, 0). Dac
a x 6= 0 atunci
y 6= 0 si z 6= 0. Cum x2 y este p
atrat perfect mai mic dect x2 , rezult
a c
a
x2 y (x 1)2 y 2x 1. La fel z 2y 1 si x 2z 1. Adunnd ultimele
trei relatii, g
asim x + y + z 3 si cum x, y, z N , rezult
a x = y = z = 1. G
asim
solutia (x, y, z, u, v, t) = (1, 1, 1, 0, 0, 0).
Solutia 2. Fie x 6= 0 (deci y 6= 0 si z 6= 0); atunci
2

x u2 y 2 v 2 z 2 t2 = xyz,

()

deci

(x u) (y v) (z t) (x + u) (y + v) (z + t) = xyz.
Prin urmare (x u) (y v) (z t) este num
ar natural nenul, deci (x u) (y v)
(z t) 1 si cum (x + u) (y + v) (z + t) xyz, egalitate n () obtinem pentru
u = v = t = 0 si x u = y v = z t = 1, deci x = y = z = 1. Avem solutiile
(x, y, z, u, v, t) {(0, 0, 0, 0, 0, 0) , (1, 1, 1, 0, 0, 0)}.

b2
a2
+
> a + 2b + c.
ab bc
2
2
a b
c2 b2
a2 c2
ii) Fie a, b, c R cu a b c > 0; atunci
+
+

c
a
b
3a 4b + c.
Ioan S
erdean, Or
astie
a2 b2
b2
b2 c2
a2

= a + b, iar

= b+c
Solutie. i) Avem evident c
a
ab
ab
bc
bc
(egalitate avem dac
a b = 0, respectiv c = 0), de unde, prin adunare, obtinem c
a
b2
a2
+
> a+2b+c. Inegalitatea este strict
a deoarece nu putem avea b = c = 0.
ab bc
ii) Major
am pe rnd cei trei termeni ai membrului stng:
G77. i) Fie a, b, c R cu a > b > c; atunci

(a b) (a + b)
c+c
a2 b2
=

(a b) = 2 (a b) ;
c
c
c
c2 b2
(c b) (c + b)
a+a
=

(c b) = 2 (c b) ;
a
a
a
(am folosit faptul c
a c b 0, schimbnd sensul inegalit
atii!)

2
2
a c
(a c) (a + c)
a c
=
=
+
(a c) (1 + 0) (a c) = a c.
b
b
b
b
Adunnd membru cu membru relatiile (1), (2) si (3), obtinem concluzia.
61

(1)
(2)

(3)

G78. Daca a, b, c, d (0, ), sa se demonstreze inegalitatea


b (a + c) c (b + d) d (a + c) a (b + d)
+
+
+
4.
c (a + b) d (b + c) a (d + c) b (a + d)
Artur B
al
auc
a, Botosani
Solutia 1. Prelucr
am fiecare raport din stnga astfel:
a+c
1 1
a+c
+
b (a + c)
c
ac
a
c.
=
=
=
1 1
a+b
a+b
c (a + b)
+
a b
b
ab
1
1
1
1
n aceste conditii, cu substitutiile x = , y = , z = , d = , inegalitatea de
a
b
c
t
demonstrat se scrie succesiv:
y+t
z+x t+y
x+z
+
+
+
4
x+y y+z
z+t
t+x

1
1
1
1
+
+ (y + t)
+
4
(x + z)
x+y z+t
y+z
t+x
(x + z) (x + y + z + t) (y + t) (x + y + z + t)
+
4.
()
(x + y) (z + t)
(y + z) (t + x)
ns
a, conform inegalit
atii mediilor,
2

1
4
(x + y + z + t)

4
(x + y) (z + t)
(x + y + z + t)2
(x + z) (x + y + z + t)
4 (x + z)
4 (x + z) (x + y + z + t)
=

.
2
(x + y) (z + t)
x+y+z+t
(x + y + z + t)

(x + y) (z + t)

Scriind nc
a o inegalitate similar
a si adunndu-le, obtinem ().
1
1
1
1
Solutia 2. Cu substitutiile x = ; y = ; z = ; d =
inegalitatea este
a
b
c
t
echivalent
a cu:
y+t
z+x t+y
x+z
+
+
+
4, x, y, z, t (0, ) .
x+y y+z
z+t
t+x
Dar
a2
(a1 + a2 + + an )2
a21 a22
+
+ + n
, ai , bi (0, )
b1
b2
bn
b1 + b2 + + bn
si atunci
x+z
y+t
z+x t+y
+
+
+
=
x+y y+z
z+t
t+x
2

(x + z)
(y + t)
(z + x)
(t + y)
+
+
+

(x + y) (x + z) (y + t) (y + z) (z + t) (z + x) (t + y) (t + x)

(z + x + y + t + z + x + t + y)2
= 4.
(x + y) (x + z) + (y + t) (y + z) + (z + t) (z + x) + (t + y) (t + x)
G79. Daca x, y, z (0, ) sunt astfel nct x + y + z = xyz, atunci
p
p
p
xy + yz + zx 3 + x2 + 1 + y 2 + 1 + z 2 + 1.

Florina Crlan si Marian Tetiva, Brlad


62

Solutia 1 (a autorilor). Plecnd de la cunoscuta a2 + b2 + c2 ab + bc + ac,


avem:
1
1
1
1
1
1
+
+

+
+
= 1 x2 y 2 + x2 z 2 + y 2 z 2 x2 y 2 z 2
x2 y 2 z 2
xy xz yz
2

(xy + xz + yz) 2xyz (x + y + z) + x2 y 2 z 2 = 3 (x + y + z) .


Mai departe,
2

si

(xy + xz + yz 3) = (xy + xz + yz) 6 (xy + xz + yz) + 9

2
3 (x + y + z) 6 (xy + xz + yz) + 9 = 3 x2 + y 2 + z 2 + 9

(xy + xz + yz) (x + y + z) 9xyz xy + xz + yz 9 > 3,


p
prin urmare xy + xz + yz 3 + 3 (x2 + y 2 + z 2 ) + 9.
Se arat
a ns
a usor c
a
p
p
p
p
3 (x2 + y 2 + z 2 ) + 9 x2 + 1 + y 2 + 1 + z 2 + 1,
de unde concluzia rezult
a prin intercalare.

Solutia 2 (a autorilor). Avem


2

xyz = x + y + z 2 xy + z z ( xy) 2 xy z 0.

1 + 1 + z2
2
, obtinem de aici c
a
R
ad
acina pozitiv
a a trinomului zt 2t z fiind
z

p
1 + 1 + z2

xy
z xy 1 + 1 + z 2 .
z
Scriind nc
a dou
a inegalit
ati analoage, prin sumare g
asim:
p
p
p

xy + xz + yz x yz + y xz + z xy 3 + x2 + 1 + y 2 + 1 + z 2 + 1.

Solutia 3 (Gheorghe Iurea, Iasi). Din ipotez


a xy 1 =
xy > 1. Analog yz > 1, zx > 1. De asemeni,

x+y
> 0, deci
z

(xy 1) (yz 1) = xy 2 z xy yz + 1 = y (x + y + z) xy xz + 1 = y 2 + 1 ()
si nc
a dou
a relatii analoage.
n inegalitateaa2 + b2 + c2
arat
a pentru orice a, b, c numere
ab + ac + bc,adev
reale, punem a = xy 1, b = yz 1, c = zx 1 si folosind relatiile () obtinem
inegalitatea cerut
a.
Egalitate pentru a = b =
a x = y = z; folosind
c, deci xy = yz = zx sau nc
ipoteza, g
asim x = y = z = 3.
Solutia 4 (Paul Georgescu, Iasi). Deoarece x, y, z (0, ) exist
a a, b, c
(0, /2) astfel nct x = tg a, y = tg b, z = tg c. Folosind relatia din ipotez
a g
asim
tg (a + b + c) = 0 si apoi a + b + c = .
Inegalitatea de demonstrat este echivalent
a cu:
1
1
1
tg a tg b + tg b tg c + tg c tg a 3 +
+
+
.
()
cos a cos b cos c
63

cos (a + b)
cos c
=
, () este echivalent
a cu
coa cos b
cos a cos b
cos b
cos a
1
1
1
cos c
+
+

+
+
cos a cos b cos a cos c cos b cos c
cos a cos b cos c

Cum tg a tg b 1 =

sau nc
a:
cos2 a + cos2 b + cos2 c cos a cos b + cos b cos c + cos a cos c,

adev
arat
a pe baza inegalit
atii x2 + y 2 + z 2 xy + yz + zx, x, y, z R.

Egalitate dac
a cos a = cos b = cos c, deci a = b = c = si apoi x = y = z = 3.
3
2
G80. Fie A multimea tuturor sumelor de tipul 12 32 52 (2n + 1) ,
n N, unde semnele pot fi alese n orice combinatie posibila. Sa se arate ca
A = Z. (n leg
atur
a cu teorema Erds-Surnyi.)
Petru Asaftei, Iasi
2
2
2
2
Solutie. S
a observ
am c
a (2k + 1) (2k + 3) (2k + 5) + (2k + 7) = 16,
2
k N. Prin urmare, dac
a un num
ar ntreg n se scrie sub forma n = 1 32
2
(2k 1) pentru o anumit
a alegere a semnelor +, , num
arul n + 16 admite si
el o sciere de aceeasi form
a:
2

n + 16 = 12 32 (2k 1) + (2k + 1) (2k + 3) (2k + 5) + (2k + 7) .

De asemeni, dac
a n are o exprimare de forma dat
a, atunci si n au o exprimare
de aceeasi form
a (obtinut
a din exprimarea pe care o are n, prin schimbarea tuturor
semnelor).
Not
am cu P (n), n N, afirmatia: "n are o exprimare de forma n = 12
32 (2k 1)2 , k N ". Cum P (n) P (n + 16), conform unei variante de
inductie matematic
a este suficient s
a verific
am P (0), P (1), . . . , P (15):

0 = 12 32 52 + 72 92 112 132 + 152 ,


1 = 12 ,

2 = 12 + 32 + 52 72 + 92 112 132 + 152 ,


3 = 12 + 32 + 52 + 72 92 ,
X
4 = 12 + 32 + 52 + 72

k{4,8,...,20}

5 = 1 + 3 + 5 + 7 + 92

h
i
2
2
2
2
(2k + 1) (2k + 3) (2k + 5) + (2k + 7) ,

h
i
(2k + 1)2 (2k + 3)2 (2k + 5)2 + (2k + 7)2 ,

k{5,9,...,41}

6 = 1 3 + 5 7 92 + 112 ,
i
X h
2
2
2
2
7 = 12 + 32 + 52 + + 132
(2k + 1) (2k + 3) (2k + 5) + (2k + 7) ,
k{8,12,...,116}

8 = 1 3 + 5 7 9 + 112 ,

9 = 7 + 16 = 12 32 132 +
h
i
X
(2k + 1)2 (2k + 3)2 (2k + 5)2 + (2k + 7)2 +
+
k{8,12,...,116}

64

X h

k=120

(2k + 1) (2k + 3) (2k + 5) + (2k + 7)

La fel proced
am si cu numerele 10, 11, . . . , 15.
Prin urmare, orice num
ar natural are o exprimare de forma cerut
a si atunci orice
num
ar ntreg are o exprimare de forma cerut
a. Rezult
a A = Z.
G81. Fie n N si k {0, 1, . . . , 2n 1}. Sa se arate ca exista o multime A R
cu n elemente care are exact k submultimi cu suma elementelor strict pozitiva.
Adrian Zahariuc, elev, Bac
au
Solutie. Fie k = an an1 . . . a1 , ai {0, 1}, i = 1, n, scrierea n baza 2 a lui
k (admitem ca primele cifre s
a poat
a fi 0, astfel nct scrierea s
a se fac
a folosind n
cifre). Pentru fiecare i {0, 1, . . . , n
1},
fie

=
1
dac
a
a
=
0

s
i
i = 1 dac
a
i

i+1
a ea
ai+1 = 1; construim multimea A = i 2i | 0 i n 1 si vom demonstra c
verific
a proprietatea dorit
a.
Pentru fiecare submultime B A, numim indicele dominant al lui B, cel mai mare
i {0, 1, . . . , n 1} pentru care i 2i B. Dac
a i = 1, atunci suma elementelor lui
B este cel putin 2i 2i1 20 = 1 > 0. Dac
a i = 1, atunci suma elementelor
lui B este cel mult 2i + 2i1 + + 20 = 1 < 0. n concluzie, semnul sumei
elementelor lui B depinde numai de indicele dominat. Cum exist
a 2i submultimi cu
indicele dominant i, num
arul submultimilor cu suma elementelor strict pozitive este
20 a1 + 21 a2 + + 2n1 an = k si astfel problema este rezolvat
a.
G82. Un cal se afla pe tabla de sah n cmpul A1 si dorim sa-l ducem pe pozitia
H8 ntr-un numar minim de sarituri. Aflati care este acest numar minim, precum
si cte trasee de lungime minima exista.
Gheorghe Cr
aciun, Plopeni si Gabriel Popa, Iasi
Solutie (Gheorghe Iurea, Iasi). Pentru a ajunge din A1 n H8, calul trebuie
s
a cstige 7 coloane si 7 linii, n total 14 pozitii. La fiecare s
aritur
a, el cstig
a maxim
3 pozitii (dou
a linii si o coloan
a sau o linie si dou
a coloane); rezult
a c
a sunt necesare
cel putin 5 s
arituri. ns
a calul nu poate ajunge n cinci s
arituri din A1 n H8: cmpul
de pe care pleac
a si cel pe care soseste au culori diferite, ceea ce implic
a necesitatea
efectu
arii unui num
ar par de s
arituri (la o s
aritur
a, calul merge de pe alb pe negru
sau de pe negru pe alb).
Exist
a trasee de 6 s
arituri (de exemplu, A1 B3 C5 E6 G5 F 7 H8),
prin urmare num
arul minim de s
arituri necesare este 6.

8
7
6
5
4
3
2
1

94
1086
8
44
23
13 9 4 545
7
43 144 33
545 44
6
3
5 12 33 12 2 184 33 9 4
2
12 43 2 23 144 13 94
4
2
3
3 12 11
43 1 4
23
11 12 33
2
12
12
1
A B C D E F G H

A B C D E F G H
65

Numim drum un traseu format din 6 s


arituri care pleac
a din A1 si ajunge n
H8. Dup
a prima s
aritur
a putem ajunge n B3 sau n C2, iar la ultima drumul
trebuie s
a ajung
a n F 7 sau G6. La a dou
a s
aritur
a drumul ajunge n unul dintre
p
atr
atelele hasurate pe figura 1, iar la a cincea drumul trebuie s
a treac
a prin unul
dintre p
atr
atelele marcate cu . S
ariturile 3 si 4 ne duc dintr-un p
atr
atel hasurat n
unul marcat cu .
n figura 2, am notat num
arul s
ariturii cu cifr
a mic
a din dreapta sus. n centrul
c
asutelor am notat num
arul de drumuri partiale care duc din A1 pn
a n acel p
atrat.
Se observ
a c
a num
arul de drumuri pn
a ntr-un p
atrat la s
aritura k este egal cu suma
numerelor p
atratelor de la s
aritura k 1, aflate pe un p
atrat de latur
a 5 cu centrul
n acel p
atrat. Recurent, se obtin 108 drumuri de lungime minim
a.
Not
a. Faptul c
a num
arul minim de s
arituri este 6 este un rezultat clasic; vezi,
de exemplu, L. Panaitopol, D. S
erb
anescu - Probleme de teoria numerelor si combinatorica pentru juniori, GIL, Zal
au, 2003. Problema num
arului traseelor minimale
nu ne este ns
a cunoscut
a.
G83. Fie ABCD patrulater convex si punctele M, N (AB), P, R (CD)
BM N A P R CD
astfel nct AD BC M R N P = {O}. Sa se arate ca

= 1.
M N DP RC AB
Andrei-Sorin Cozma, elev, Iasi
Solutie. Se demonstreaz
a cu usurinta urm
atoarele dou
a propriet
ati:
BD
BD
SABC
SABD
si
.
1. Fiind dat triunghiul ABC si D (BC), atunci
=
=
DC
SACD
BC
SABC
SADE
2. Fiind dat triunghiul ABC si D (AB), E (AC), avem c
a
=
SABC
AD AE
O
.
AB AC
Folosind aceste rezultate avem
BM N A P R CD
C

=
P R
M N DP RC AB
D
BM N A P R CD
=

=
M N AB RC DP
SOMB SOAN SOP R SOCD
=

=
SON M SOAB SORC SODP
B
A
M
N
=

OM OB OA ON OP OR OC OD
SOMB SOAN SOP R SOCD

= 1.
SORC SODP SON M SOAB
OR OC OD OP ON OM OA OB

G84. Fie ABCD un trapez cu ABkCD, AB < CD. Se considera punctele


CF
AE
=
. Dreapta EF intersecteza BD si
E (AD) si F (BC) astfel nct
ED
FB
MN
DC AB
AC n M , respectiv N . Sa se arate ca
=
.
EF
DC + AB
Andrei Nedelcu, Iasi
AB
AO
BD
Solutie. Not
am k =
=
=
, k < 1, iar cu P , T intersectiile
CD
OC
OD
66

paralelei prin E la AC cu CD, respectiv BD.


A
B
AE
CP
AE
Cum
=
(teorema lui Thales) si
=
O
ED
PD
ED
F
CP
CF
CF
N
, urmeaz
a c
a
=
, adic
a PF k
M
FB
PD
FB
BD. Aplicnd Thales n 4EP F cu T M k P F , E
ET
EM
ET
AO
T
obtinem c
a
=
; ns
a
=
= k,
TP
MF
TP
OC
P
D
C
EM
k
deci
= k, i.e. EM =
EF . Analog se
MF
k+1
k
arat
a c
a NF =
EF . Astfel,
k+1
2k
MN
1k
CD AB
M N = EF (EM + N F ) = EF
EF
=
=
.
k+1
EF
1+k
CD + AB
G85. Fie A0 , B 0 , C 0 picioarele bisectoarelor unghiurilor 4ABC. Pe latura (BC)
consideram punctele D si E astfel nct D (BE) si cevienele AD si AE sunt
izogonale. Sa se demonstreze ca DB 0 si EC 0 se intersecteza pe AA0 . (n leg
atur
a
cu Propozitia 1, p. 99, RecMat - 2/2004.)
Titu Zvonaru, Com
anesti
0
0
\
[
\
\
Solutie. Not
am m(BAD) = m(EAC) = si m(DAA ) = m(EAA ) = . Fie
{X} = C 0 E AD si {Y } = DB 0 AE. Folosind teorema lui Menelaus n 4ABD cu
transversala C 0 XE obtinem:
ED BC 0 AX

= 1.
EB C 0 A XD
Dar
AD AE sin 2
ED
SDEA
AD sin 2
=
=
=
EB
SBEA
AB AE sin (2 + )
AB sin (2 + )
BC 0
BC
si 0 =
(din teorema bisectoarei).
CA
AC
Prin urmare:
AX
AB AC sin (2 + )
=
.
(1)
XD
AD BC sin 2
La fel
AB AC sin (2 + )
AY
=
.
(2)
YE
AE BC sin 2
Din teorema bisectoarei n triunghiul ADE,
AD
DA0
=
.
(3)
A0 E
AE
AX DA0 Y E

= 1 si din teorema lui Ceva rezult


a
Folosind (1), (2), (3) deducem
XD A0 E AY
c
a dreptele EX, DY , AA0 sunt concurente.
Not
a. Solutia autorului, n esenta aceeasi, foloseste explicit relatia lui Steiner.
Se poate ar
ata n aceeasi manier
a c
a rezultatul r
amne valabil si dac
a D (CE).

B. Nivel liceal
L76. Fie cercurile C1 si C2 tangente interior unui cerc C n punctele distincte M ,
respectiv N . Cercurile C1 si C2 sunt secante sau tangente exterior iar axa radicala a
67

cercurilor C1 si C2 taie cercul C n A si B. Dreptele AM si AN taie din nou cercurile


C1 si C2 n K, respectiv L. Aratati ca AB 2KL. n ce caz avem egalitate?
Neculai Roman, Mircesti (Iasi)
Solutie. Fie C1 = C (O1 , r1 ), C2 = C (O2 , r2 ), C = C (O, r) si {P, Q} = C1 C2
(fig. 1). Dac
a cercurile C1 si C2 sunt tangente exterior (fig. 2) atunci P = Q.

K
O

K
P

L
O2

O1
Q

O
O1

B
Fig. 1

O2

B
Fig. 2

Din AB ax
a radical
a a cercurilor C1 si C2 rezult
a c
a AK AM = AL AN , deci
\ AN
\
\ = m(AN
\
4AKL 4AN M , de unde obtinem AKL
M , adic
a m(AKL)
M) =
1
m(AM ).
2
Pe de alt
a parte, din
1
1

\
\
\
m(O\
1 KM ) = m(O1 M K) = m(OM A) = 90 m(M OA) = 90 m(AM )
2
2

\
obtinem m(O
a la C1 . Din O1 K k OA k O2 L
1 KL) = 90 , deci KL este tangent
rezult
a KL O2 L, de unde KL este o tangent
a comun
a exterioar
a cercurilor C1
si C2 . Aplic
am teorema lui Casey pentru cercurile A, C1 , B, C2 (A, B degenerate)
tangente interior cercului C si obtinem:
dAO1 dBO2 + dAO2 dBO1 = dAB dO1 O2

(am notat dO1 O2 lungimea tangentei comune exterioare cercurilor C1 si C2 )


p
p
p
p
AP AQ BP BQ + AP AQ BP BQ = AB KL
p

AP + BP AQ + BQ
AB KL = 2 AP BP AQ BQ AB KL 2

2
2
AB 2
AB 2KL.
AB KL
2
Dac
a cercurile C1 si C2 sunt tangente exterior (fig. 2), atunci avem:

2
AP + BP
AB 2
ABKL
ABKL = 2AP BP ABKL 2
AB 2KL
2
2
(egalitate dac
a AP = BP ).
Egaliate avem dac
a cercurile C1 si C2 sunt tangente exterior n mijlocul segmentului [AB].

L77. Fie punctele P1 , P2 , . . . , P13 n plan astfel nct oricare trei sunt necoliniare
68

si toate au coordonate ntregi. Sa se arate ca exista cel putin un triunghi Pi Pj Pk astfel


nct centrul sau de greutate sa aiba coordonate ntregi.
Vasile Prav
at si Titu Zvonaru, Com
anesti (Bac
au)
Solutie (Daniel V
acaru). Exist
a trei resturi modulo 3, asa c
a cel putin 5 abscise
xi dau acelasi rest la mp
artirea prin 3 (principiul cutiei!), ceea ce impune c
a abscisa
centrului de greutate este num
ar ntreg, oricum am alege trei indici din multimea
A a celor 5 determinati mai sus. Consider
am M multimea resturilor modulo 3 ale
numerelor {yi | i A}. Dac
a M are trei elemente, alegem i, j, k A astfel nct
yi + yj + yk 0 + 1 + 2 (mod 3) si problema este rezolvat
a. Dac
a M are dou
a
elemente, cel putin 3 ordonate dau acelasi rest la mp
artirea prin 3 si le alegem pe
ele. n sfrsit, dac
a M are un singur element, concluzia este imediat
a.
Not
a. Principial aceeasi solutie a dat Vlad Emanuel, elev, Sibiu.
L78. Consideram sirul de puncte (Pn )nN pe cercul trigonometric astfel nct
5
OPn+1 ) = arctg
OPn+1 fiind considerat ca unghi
m(Pn\
pentru orice n N, Pn\
12
orientat. Sa se arate c
a pentruorice punct P pe cercul trigonometric exista j N
1
astfel nct Pj Int C P,
.
2005
Lucian - Georges L
adunc
a si Andrei Nedelcu, Iasi
Solutie. Pentru ca multimea {Pn , n N} s
a fie dens
a pe cercul trigonometric,
de unde va rezulta imediat concluzia problemei folosind lema lui Kronecker, trebuie
5
arctg 12
5
5
demonstrat c
a
R \ Q. Observ
am mai nti c
a arctg
= arcsin .

12
13
5
p
Presupunem prin reducere la absurd c
a arcsin
= , (p, q) = 1, p, q N , q 2.
13
q
5
p
q
Fie = arcsin . Atunci sin q = 0 si cos q = (1) . Cum (cos + i sin ) =
13
p
(1) , obtinem c
a
sinq Cq2 sinq2 cos2 + Cq4 sinq4 cos4 = 0.

Dac
a q este par, obtinem c
a polinomul cu coeficienti ntregi xq Cq2 xq2 1 x2 +
q
[ q ]
5
2[ q ]
ad
acina rational
a sin =
, ceea ce este absurd,
. . . (1)[ 2 ] Cq 2 1 x2 2 are r
13
q
deoarece coeficientul termenului dominant este 2q1 , iar termenul liber este (1)[ 2 ] ,
1
deci polinomul respectiv poate avea numai r
ad
acini de forma n , n N . Cazul q
2
impar se trateaz
a analog.
arccos a
Nota. Vlad Emanuel demonstreaz
a c
a, dac
a a [1, 1] Q si
Q,

1
atunci a 0, 1, .
2
L79. Fie a1 , a2 , . . . , an R n asa fel nct a1 + a2 + + an = 0 si
1 hni hn + 1i
max {|ai aj | ; 1 i < j n} 1. Demonstrati ca a21 +a22 + +a2n
n 2
2
si precizati n ce caz are loc egalitate.
Marius Pachitariu, elev, Iasi
Solutie. Deoarece inegalitatea este simetric
a n a1 , a2 , . . . , an , putem presupune
69

c
a a1 a2 an . Conform identit
atii lui Lagrange, obtinem tinnd seama de
ipotezele problemei c
a
n
X
1 X
1 X
2
a2i =
(aj ai )
(aj ai ) ,
n
n
i=1
1i<jn

1i<jn

amne deci s
a
deoarece aj ai este pozitiv subunitar pentru toti 1 i < j n. R
hni hn + 1i
P
demonstr
am c
a
(aj ai )
.
2
2
1i<jn
Dac
a n este par, n = 2k, k N , atunci
X
(aj ai ) = (2k 1) (a2k a1 ) + (2k 3) (a2k1 a2 ) + + (ak+1 ak )

1i<jn

(2k 1 + 2k 3 + + 1) (a2k a1 ) = k 2 (a2k a1 ) .


hni hn + 1i
P
(aj ai ) k 2 =
n concluzie,
.
2
2
1i<jn
Dac
a n este impar, n = 2k + 1, k N , atunci
X
(aj ai ) = 2k (a2k+1 a1 ) + (2k 2) (a2k a2 ) + + 2 (an+2 an )
1i<jn

k (k + 1) (a2k+1 a1 ) .

hni hn + 1i
.
2
2
1i<jn
Dac
a n este par, n = 2k, k N , egalitatea din enunt are loc pentru a1 = a2 =
= ak = < 0, ak+1 = ak+2 = = a2k = > 0 si = 1, + = 0, deci
1
1
= ,= .
2
2
Dac
a n este impar, n = 2k + 1, k N , egalitatea din enunt are loc pentru
a1 = a2 = = ak+1 = < 0, ak+2 = ak+3 = = a2k+1 = > 0 sau
a1 = a2 = = ak = < 0, ak+1 = ak+2 = = a2k+1 = > 0 si = 1,
(k + 1) + k = 0, respectiv k + (k + 1) = 0. Rezult
a c
a a1 = a2 = = ak+1 =
k+1
k+1
k

, ak+2 = = a2k+1 =
, respectiv a1 = a2 = = ak =
,
2k + 1
2k + 1
2k + 1
k
ak+1 = ak+2 = = a2k+1 =
.
2k + 1
Not
a. Solutie corect
a s-a primit de la Vlad Emanuel, elev, Sibiu.
L80. Fie un alfabet cu 4 litere a, b, c, d. n acest alfabet se pot forma cuvinte
dupa urmatoarele reguli: dupa a nu poate urma b, dupa b nu poate urma c, dupa c
nu poate urma d si dupa d nu poate urma a. Cte cuvinte palindromice de lungime
n, n 2, se pot forma conform acestor reguli? (Prin cuvnt palindromic ntelegem
un cuvnt n care litera de pe pozitia k coincide cu litera de pe pozitia n k + 1,
pentru orice k {1, 2, . . . , n}.)
Irina Mustata
a, Iasi
, elev
Solutie. S
a observ
am c
a pentru a construi un cuvnt palindromic
(palindrom)
hni
de lungime n este suficient s
a construim un cuvnt C de lungime
, pe care l
2
invers
am si l ad
aug
am la sfrsitul lui C, ntre acestea insernd nc
a o liter
a dac
an
este impar.
n concluzie,

(aj ai ) k (k + 1) =

70

Tinem

seama c
a ntr-un palindrom, odat
a cu gruparea de dou
a litere consecutive
apare si inversa sa. Atunci grup
arile interzise n C sunt ab, bc, cd, da, ad, dc, cb, ba.
Not
am cu an , bn , cn , dn num
arul cuvintelor corecte de lungime n care se termin
a
n a, respectiv n b, c, d. Deoarece orice cuvnt corect de lungime n care se termin
a
n a este format dintr-un cuvnt corect de lungime n 1 care se termin
a n c sau a, la
care se adaug
a un a final, deducem c
a an = an1 + cn1 si analog bn = bn1 + dn1 ,
cn = cn1 + an1 , dn = dn1 + bn1 . Notnd cu Sn num
arul cuvintelor corecte
de lungime n, avem din cele de mai sus c
a Sn = 2Sn1 , iar cum S1 = 4 rezult
a c
a
Sn = 2n+1 . Not
am acum cu Pn num
arul palindroamelor de lungime n. Pentru n din
enunt, distingem dou
a cazuri.
n
1. n este par. Atunci Pn = S[ n ] = 2[ 2 ]+1 .
2
hni
2. n este impar. ntre cuvntul de lungime
si inversul s
au se mai insereaz
ao
2
n
+2
[
]
liter
a, care poate fi aleas
a n dou
a moduri. Atunci Pn = 2S[ n ] = 2 2 .
2

Not
a. Solutie corect
a s-a primit de la Vlad Emanuel, elev, Sibiu.
L81. Fie n 1 un numar natural fixat. O tabla infinita de sah este colorata
n alb si negru n maniera obisnuita. O multime C de casute ale tablei se numeste
conex
a daca putem ajunge din fiecare casuta a lui C n fiecare alta casuta a lui
C printr-o succesiune de deplasari n C dintr-o casuta ntr-o casuta vecina (cu o
latura comuna). Fie S o multime conexa cu 4n casute. Numim raportul cromatic
al multimii S raportul dintre numarul de casute albe si numarul de casute negre din
S. Sa se afle cea mai mica si cea mai mare valoare posibila a raportului cromatic.
Adrian Zahariuc, elev, Bac
au
Solutie. Fie w num
arul de c
asute albe, b num
arul de c
asute negre, iar N num
arul
de perechi (ordonate) de c
asute din S. Cum fiecare c
asuta alb
a are cel mult 4 vecini
negri n S, rezult
a c
a fiecare c
asuta alb
a apare n cel mult 4 perechi, deci N 4w.
Analog, N 4b.
Demonstr
am acum c
a N 4n 1. n acest scop, multimii S i se asociaz
a un graf
n felul urm
ator: fiec
arei c
asute i corespunde un nod, iar nodurile acestui graf sunt
unite prin muchii dac
a acele c
asute care le corespund sunt adiacente. Graful astfel
obtinut este si el conex, N reprezint
a num
arul s
au de muchii, iar 4n num
arul s
au de
vrfuri.
Observ
am c
a un graf conex cu v vrfuri are cel putin v 1 muchii, lucru care
se poate demonstra astfel: elimin
am muchiile care apartin unor cicluri pn
a cnd
obtinem un arbore, despre care stim c
a are exact v 1 muchii. Atunci conform celor
de mai sus, N 4n 1.
De aici, 4w 4n 1, deci w n si analog b n. Cum b + w = 4n, obtinem c
a
1
w
b 3n si w 3n. De aici,
3. R
amne acum s
a construim exemple n care
3
b
raportul cromatic atinge valorile extreme.
Numim o figur
a format
a dintr-o c
asuta si vecinii s
ai de sus, stnga si dreapta
un T -dreptunghi; acesta se va numi T -dreptunghi alb sau negru dup
a cum c
asuta
1
a
din centru este alb
a sau neagr
a. Pentru a atinge valoarea , construim o figur
3
format
a din n T -dreptunghiuri negre suprapuse, n timp ce pentru a atinge valoarea
71

3 construim o figur
a format
a din n T -dreptunghiuri albe suprapuse.
Not
a. Solutie corect
a s-a primit de la Vlad Emanuel, elev, Sibiu.
L82. Determinati P, Q R [X] pentru care f : R R, f (x) = {p (x) + sin q (x)}
este periodica, unde p, q : R R sunt functiile polinomiale asociate lui P , respectiv Q.
Paul Georgescu si Gabriel Popa, Iasi
Solutie. Fie T R+ o perioad
a a lui f . Atunci {p (x + T ) + sin q (x + T )} =
{p (x) + sin q (x)}, x R, deci p (x + T ) + sin q (x + T ) p (x) sin q (x) Z,
x R. Deoarece g : R R, g (x) = p (x + T ) + sin q (x + T ) p (x) sin q (x) este
continu
a, r
amne c
a ea este identic constant
a, deci exist
a k Z astfel ca p (x + T ) +
sin q (x + T ) p (x) sin q (x) = k, x R.
De aici, p (x + T ) p (x) k = sin q (x) sin q (x + T ), x R, de unde deducem
c
a p (x + nT ) p (x) nk = sin q (x) sin q (x + nT ), x R si n N , ceea ce
pentru x = 0 conduce la |p (nT ) p (0) nk| 2, n N . De aici rezult
a c
aP
este un polinom de grad maxim 1, deci P = aX + b, iar k = aT .
n concluzie, a (x + T ) + b + sin q (x + T ) ax b sin q (x) = aT , x R, deci
sin q (x + T ) sin q (x) = 0, x R. Aceasta implic
a faptul c
a
2 sin

q (x + T ) q (x)
q (x + T ) + q (x)
cos
= 0,
2
2

x R.

q (x + T ) q (x) q (x + T ) + q (x)
si
nu sunt identic constante, atunci membrul
Dac
a
2
2
stng poate avea cel mult o multime num
arabil
a de zerouri, contradictie. Rezult
a de
aici c
a q (x + T ) q (x) = 2k1 , x R, cu k1 Z, sau q (x + T ) + q (x) = 2k2 ,
x R, cu k2 Z.
n primul caz obtinem c
a Q este un polinom de grad cel mult 1, Q = a1 X + b1 ,
a1 , b1 R, cu a1 T = 2k1 . n al doilea caz rezult
a imediat c
a Q c, caz oricum
inclus n primul.
n concluzie, P = aX + b, Q = a1 X + b1 , unde a si a1 au proprietatea c
a exist
a
a1 T

T R+ astfel ca aT,
Z. Se observ
a imediat c
a n acest caz functia este
2
periodic
a.
Not
a. Solutie corect
a s-a primit de la Vlad Emanuel, elev, Sibiu.
L83. Sa se calculeze
"
#

1
2
n
1 2
1 3
1 n+1
1+
lim
+ 1+
+ + 1 +
n .
n
n
n
n
Marius Olteanu, Rmnicu Vlcea
Solutie. Mai nti, se observ
a c
a

k
n
n
X
1 n+1
1 k+1
nn 1+
n.
(1)
1+
n
n
k=1

De asemenea,
72

1
1

k
1+
1+
1 k+1
n
n
=
1+
,
1
1
1
n
1 k+1
1
+

1+
k+1 n
n
conform inegalit
atii lui Bernoulli. Rezult
a de aici c
a
k
k+1
X
n
n
n
X
X
1
(n + 1) (k + 1)
k
n
1+
1 =
=
n
n (k + 1) + 1
n (k + 1) + 1
k=1

k=1

k=1

n
n
k+1+
X
1 X
1
n
=
1+
.
n (k + 1) + 1
n
n (k + 1) + 1
k=1

n concluzie,

k=1

n
n
X
1 1X 1
1 k+1
1 1+
1+
.
n
n n
k+1
k

k=1

Avem n plus c
a

(2)

k=1

11
n
1 + n1 1+ n 1
1 n+1
lim n 1 +
n = lim
= 1,
1
n
n
n
n

limita din urm


a putndu-se calcula cu ajutorul regulii lui lHospital, iar
n
1X 1
lim
=0
n n
k+1
k=1

conform lemei Cesar-Stolz. Din (1) si (2) rezult


a conform criteriului clestelui c
a
limita din enunt este egal
a cu 1.
Not
a. Solutie corect
a s-a primit de la Vlad Emanuel, elev, Sibiu.
L84. Fie n N, n 3 si
n

n
A = x > 0; x = a0 + a1 n n + + an1 nn1 ;
o
a0 , a1 , . . . , an Z; n 1 | a0 + a1 + + an .

Determinati inf A.

Paul Georgescu si Gabriel Popa, Iasi

k
Solutie. Fie (xk )k1 definit de xk = ( n n 1) ; evident, x1 A.

n
Fie k 1. Presupunem c
a xk = Ak0 + Ak1 n n + + Akn1 nn1 A si demonstr
am c
a xk+1 A. Avem c
a

n
xk+1 = n n1 xk = nAkn1 Ak0 + Ak0 Ak1 n n + + Akn2 Akn1 nn1 =

n
n
n1
+ Ak+1
n + + Ak+1
= Ak+1
0
1
n1 n

..
k
si Ak+1
+ Ak+1
+ + Ak+1
0
1
n1 = (n 1) An1 . n 1 pentru k 1. Cum xk 0
pentru k obtinem c
a inf A = 0.
Not
a. Solutie corect
a s-a primit de la Daniel V
acaru.
L85. Fie f : R R o functie pentru care multimea punctelor n care f are limita
finita la stnga este densa n R. Sa se arate ca multimea punctelor n care f este
73

continua este de asemenea densa n R. (O multime D R se numeste dens


a n R
daca orice interval deschis al axei reale contine macar un element din D.)
Gabriel Dospinescu, Paris, si Marian Tetiva, Brlad
Solutie. Vom folosi urm
atoarea lem
a, demonstrabil
a usor cu ajutorul caracteriz
arii cu a limitei (finite) a unei functii ntr-un punct t0 R:
Daca functia h : R R are limita finita la stnga n t0 R, atunci pentru orice
> 0 exista un interval compact nedegenerat J, situat la stnga lui t0 , astfel nct
|h (x) h (y)| < pentru orice x, y J.
Fie acum I un interval deschis; trebuie s
a ar
at
am c
a acesta contine cel putin un
punct n care f este continu
a. Conform ipotezei, exist
a x0 I n care f are limit
a
finit
a la stnga si, aplicnd lema de mai sus, exist
a I0 interval compact nedegenerat
situat la stnga lui x0 astfel ca |f (x) f (y)| < 1 pentru orice x, y I0 ; I0 poate fi
presupus suficient de mic, astfel ca I0 I.
Fie I0 = [a0 , b0 ]; exist
a atunci x1 (a0 , b0 ) n care f are limit
a finit
a la stnga
n x1 si de asemenea exist
a un interval compact nedegenerat I1 (a0 , b0 ) situat la
1
stnga lui x1 astfel ca |f (x) f (y)| < pentru orice x, y I1 .
2
Continund iterativ, obtinem sirul de intervale compacte (I )) I0 ) I1 ) I2 . . .
1
astfel ca |f (x) f (y)| < i , x, y Ii . Conform lemei intervalelor nchise incluse,
2

T
1
exist
aa
In ( I). Atunci |f (a) f (x)| < i , x Ii , i N, de unde rezult
a
2
n=0
c
a f este continu
a n a I, ceea ce trebuia demonstrat.

Exemplele anuntate la pag. 27.

Ex. 1

Ex. 2

Ex. 3

Premiu special
"POIANA"
acordat de FUNDA
TIA CULTURALA
Fundatia Cultural
a "Poiana" (director d-l Dan Tiba) acord
a elevului VLAD
EMANUEL, cl. a XI-a, Colegiul National "Gh. Lazar", Sibiu, un premiu n valoare
de 100 lei (1 000 000 lei vechi).
Premiul este oferit la recomandarea Redactiei revistei pentru abilitatea si ingeniozitatea dovedite n rezolvarea unui num
ar mare de probleme din rubrica "Probleme
pentru preg
atirea concursurilor": L(67-69,77-83).
74

Probleme propuse1
Clasele primare
P.104. Suma dintre predecesorul unui num
ar si succesorul num
arului urm
ator
lui este 29. Care este acest num
ar?
(Clasa I )
Irina Luca, elev
a, Iasi

+2

-3

+13

-13

32

+1

P.105. Al
aturat se afl
a robotelul "MATE".
a) Completati casetele goale;
b) Aflati suma numerelor pe care le tine n mini;
c) Aflati diferenta numerelor scrise n t
alpile picioarelor.
(Clasa I )
Andrei Stativ
a, elev, Iasi

67
P.106. Pentru desemnarea campioanei, echipele de hochei pe
gheata A si B disput
a un num
ar de partide pn
a ce una dintre ele cstig
a de 4
ori. Care este num
arul maxim de partide care se pot juca, stiind c
a nu au avut loc
rezultate de egalitate?
(Clasa a II-a)
nv. Constanta Cristea si Inst. Iulian Cristea, Iasi
P107. Un grup de turisti a consumat 17 pr
ajituri si 31 nghetate. S
tiind c
a7
turisti au consumat cte o nghetat
a si cte o pr
ajitur
a, 5 turisti au consumat numai
cte dou
a nghetate, iar 4 turisti nu au consumat nimic, s
a se afle cti turisti sunt n
grup.
(Clasa a II-a)
Aliona Loghin, elev
a, Iasi
P108. Prin mp
artirea a dou
a numere naturale rezult
a ctul 3 si restul 6. S
tiind
c
a mp
artitorul este un num
ar mai mic dect 10, aflati cele dou
a numere.
(Clasa a III-a)
nv. Rica Buc
atariu, Iasi
F
P.109. Figura al
aturat
a este format
a din betisoare.
G
E
a) ndep
arteaz
a un singur betisor pentru a obtine tot attea triH
unghiuri ca si p
atrate;
D
I
b) Mut
a dou
a betisoare pentru a obtine de dou
a ori mai multe
C
J
dreptunghiuri dect p
atrate.
(Clasa a III-a)
Adina Voinescu, elev
a, Iasi
A B
P.110. Ce or
a indic
a primul ceas, stiind c
a acesta respect
a regula indicat
a de
celelalte trei?

12

12
3

9
6

12
3

9
6

12
3

9
6

9
6

(Clasa a III-a)
Veronica Corbu, elev
a, Iasi
P.111. Fie num
arul N = abc + acb + bac + bca + cab + cba.
a) Care este cea mai mic
a si cea mai mare valoare a lui N ?
b) Cte valori diferite poate avea num
arul N ?
(Clasa a IV-a)
Oxana Pascal, elev
a, Iasi
P.112. n urma desf
asur
arii unui joc didactic matematic, nv
atatorul a oferit
ca recompens
a 44 baloane. Cte 4 baloane au primit un num
ar de participanti ce
1

Se primesc solutii pn
a la data de 31 decembrie 2006.

75

reprezint
a a sasea parte din totalul lor, cte dou
a au primit a treia parte, iar restul
participantilor au primit cte un balon. Aflati num
arul participantilor la joc (solutie
aritmetic
a!).
(Clasa a IV-a)
Alexandra Nistor, elev
a, Iasi
P.113. Dan si-a pus timbrele n clasor, cte 10 pe unele pagini, cte 30 pe alte
pagini si au r
amas de 4 ori mai multe pagini goale dect folosite. Dac
a ar pune cte 5
timbre pe fiecare pagin
a, toate paginile ar fi folosite. Cte pagini poate avea clasorul,
stiind c
a nu dep
aseste 60 (solutie aritmetic
a!)?
(Clasa a IV-a)
Petru Asaftei, Iasi

Clasa a V-a
V.66. S
a se arate c
a, oricare ar fi cifra nenul
a a, num
arul x = 2131a +32a13 +43a31
se divide cu 10.
Otilia Nemes, Ocna Mures (Alba)
V.67. a) S
a se arate c
a, sc
aznd din suma a 2006 numere pare consecutive suma
numerelor situate ntre acestea, nu se poate obtine rezultatul 20062 .
b) S
a se afle 2006 numere pare consecutive astfel nct, sc
aznd din suma lor suma
numerelor situate ntre ele, s
a se obtin
a 20052 .
Marian Pantiruc, Iasi
a fie p
atrat perfect.
V.68. Ar
atati c
a nu exist
a n N pentru care An = 5n + 89 s
Iulia Plesca, elev
a, Iasi
V.69. S
a se rezolve n N2 ecuatia 8n + 15m = 6 + 62 + + 62006 .
Alexandru Gabriel Tudorache, elev, Iasi
V.70. Determinati a N pentru care numerele a, a + 2, a + 6, a + 12, a + 18,
a + 20, a + 26, a + 30, a + 32, a + 36, a + 60 sunt simultan prime.
Lucian Tutescu, Craiova

Clasa a VI-a

VI.66. Al
aturat este desenat
a o gr
adin
a avnd forma
unui poligon cu 7 laturi. n fiecare vrf se afl
a cte o
poart
a mobil
a astfel nct, n oricare dou
a vrfuri vecine,
portile s
a nchid
a perfect latura pe care acestea o determin
a. S
a se afle lungimile portilor.
Roxana C
ap
atn
a, elev
a, Iasi

9m

G
11 m

12 m

B
6m

C
7m

D
6m

F 10 m E
VI.67. n patrulaterul ABCD construim AP BD, CQ BD, P, Q BD
si fie M mijlocul lui (AC). Dac
a punctele M , P , Q sunt distincte dou
a cte dou
a,
demonstrati c
a 4M P Q este isoscel.
Marius Farcas, Iasi

\
VI.68. Fie punctele A, C, M cu m(AM C) 6= 90 si AC = 2 AM . S
a se arate c
a
\ ) = m(M
\
M este mijlocul lui [AC] dac
a si numai dac
a 2m(ACM
AC).
Ioan S
ac
aleanu, Hrl
au
VI.69. S
a se arate c
a pentru orice alegere a semnelor n expresia 12 22
2
2006 , rezultatul nu se divide cu 2006.
Mihail Bencze, Brasov
76

VI.70. Determinati m, n Z pentru care a =

3m + 1
n+2
+
Z.
2m + 1 3n + 5
Gheorghe Iurea, Iasi

Clasa a VII-a
VII.66. S
a se rezolve n R4 ecuatia
p
p

30 x y + 901 + 25 y z + 626 + 20 z x + 401 + 9 t x + 78 = 2006.

Ioana Olan, elev


a, Iasi

.
VII.67. Aflati a, b N dac
a a + b = 18 si 10a+1 9b + 71 .. 81.
Andrei-Sorin Cozma, elev, Iasi
VII.68. Fie 4ABC dreptunghic, cu ipotenuza de lungime
a, catetele b si c, iar

2 x2 + y 2
a2
=
dac
a si numai dac
a b si
aria S. Dac
a x, y (0, ), s
a se arate c
a
S
xy
c sunt direct sau invers proportionale cu x si y.
Veronica Pl
aesu si Dan Pl
aesu, Iasi
\
a punctele S, T, M (N S),
VII.69. Fie 4M N P cu m(N
M P ) = 90 ; se consider
M (P T ), astfel nct N S = 3 M S, P T = 3 M T . Dac
a {Q} = P S N T , atunci:
a) QM = N P ;
b) QN 2 + QP 2 = 5 N P 2 .
Dorel Luchian, Iasi
VII.70. Triunghiul al
aturat este considerat fix. n cte moduri putem aseza numerele 1, 2, 3, 4, 5, 6 n cerculete, astfel nct
suma numerelor de pe fiecare latur
a a triunghiului s
a fie aceeasi?
Petru Asaftei, Iasi

Clasa a VIII-a
VIII.66. S
a se demonstreze c
a
1
1
n1
1
+
+ + 4
<
.
24 + 22 + 1 34 + 32 + 1
n + n2 + 1
3n
Carmen Daniela Tamas, Brlad
VIII.67. Fie 0 < a < b < c < d < e si propozitiile:

2ac
b+d
2ce
; p2 : c =
; p3 : c = ae; p4 : d =
.
p1 : b =
a+c
2
c+e
S
a se arate c
a dac
a oricare trei dintre propozitii sunt adev
arate, atunci este adev
arat
a
si cea de-a patra.
Claudiu-
Stefan Popa, Iasi
VIII.68. Fie An = 2006n + 2005n 1992n 1991n , n N. S
a se determine n
..
pentru care An . 28.
Ionel Nechifor, Iasi
VIII.69. Fie x1 , x2 , x3 R astfel nct x21 + x22 + x23 = 1. Determinati cea mai
mic
a si cea mai mare valoare a expresiei
E (x1 , x2 , x3 ) = x1 + x2 + x3 + x1 x2 + x2 x3 + x3 x1 .
Ion Visan si Lucian Tutescu, Craiova
77

VIII.70. Se consider
a cubul ABCDA0 B 0 C 0 D0 si fie M , N mijloacele muchiilor
[AB], respectiv [BC], iar {S} = AN CD, {T } = DM BC. S
a se afle m
asura
unghiului format de D0 N si ST .
Gabriel Popa, Iasi

Clasa a IX-a
IX.66. Pentru x, y R, fie a = y + xy x, b = x2 + x xy.
a) Dac
a a, b (, 0), s
a se compare numerele x si y.
b) Ar
atati c
a exist
a o infinitate de numere rationale x, y pentru care a, b (, 0).
Ionut Onofrei, elev, Hrl
au
IX.67. Fie n 2 si a1 , a2 , . . . , an 0 astfel nct
2

(a2 a3 an ) + (a1 a3 a4 an ) + + (a1 a2 an1 ) = 1.


S
a se arate c
a
a1 + a2 + + an + a2 a3 an + a1 a3 an + + a1 a2 an1 n.

Adrian Zahariuc, elev, Bac


au
IX.68. n 4ABC se consider
a cevienele [AM ], [BN ], [CP ] concurente n T . S
a
TA
TB
TC
se arate c
a
=
=
dac
a si numai dac
a T este centrul de greutate al
TM
TN
TP
4ABC.
Ovidiu Pop, Satu Mare
IX.69. Fie 4ABC nedreptunghic. Paralela prin B la AC si simetrica dreptei
a
AC n raport cu BC se intersecteaz
a n A1 ; analog se obtin punctele B1 si C1 . Dac
AA1 , BB1 , CC1 sunt concurente, s
a se arate c
a 4ABC este echilateral.
Temistocle Brsan, Iasi
IX.70. S
a se arate c
a tg 15 + tg 25 + tg 35 + tg 85 > 4.
D. M. B
atinetu-Giurgiu, Bucuresti

Clasa a X-a
X.66. Not
am cu D multimea punctelor P (x, y) din planul xOy situate n interiorul sau pe laturile 4ABC. Fie a, b R, a2 + b2 6= 0; definim functia f : D R,
f (P ) = ax + by + c. S
a se arate c
a pentru orice P D, avem
min {f (A) , f (B) , f (C)} f (P ) max {f (A) , f (B) , f (C)} .

Adrian Corduneanu, Iasi


a se determine functiile cresc
atoare
X.67. Fie Q 2 = a + b 2 | a, b Q . S


f : Q 2 (0, ) pentru care f (x + y) = f (x) f (y), x, y Q 2 .
Dan-
Stefan Marinescu si Viorel Cornea, Hunedoara
X.68. Pe cercul trigonometric se consider
a punctele A, B, C de afixe 1, , 2 ,
2
2
unde = cos
+ i sin . Fie M (z) un punct al cercului situat pe arcul BC ce nu
3
3

z2 + z + 1
contine A. S
a se arate c
a z 2 + z + 1 =
.
z
Marian Tetiva, Brlad
X.69. Dac
a a, b, c > 1, s
a se demonstreze inegalitatea
2

(a + b + c)
3 log b+ 3 log c
3
3
3
3
a
a
+ b logb a+ logb c + c logc a+ logc b
.
a
3
Titu Zvonaru, Com
anesti
78

X.70. Fie p
atratul ABCD. S
a se determine multimea

= P Int ABCD | P A2 , 2P B P D, P C 2 sunt laturile unui triunghi .

C
at
alin Calistru, Iasi

Clasa a XI-a
XI.66. Fie xn , n N , cel mai mic num
ar natural cu proprietatea c
a exist
aM =

an an1 . . . a1 a0 (10) cu toate cifrele nenule, astfel nct M = (n + 1) n+1 an an1 a0 +


xn
a se calculeze lim
.
9xn . S
n 10n
Valeriu Brasoveanu, Brlad

XI.67. Fie sirul (xn )n1 definit prin x1 , 0 , xn+1 = 2xn tg xn , n 1.
4
S
a se studieze existenta limitelor lim xn si lim n xn .
n
n
Dan Popescu, Suceava
XI.68. Fie f : I R, I R interval, o functie de dou
a ori derivabil
a cu
f 00 (x) f 0 (x), x I. S
a se arate c
a f (x) f (a) (exa 1) f 0 (a), x, a I.
Pentru f (x) = ex , (, 0] [1, +), s
a se deduc
a inegalitatea lui Bernoulli.
Dumitru Mihalache, Brlad
XI.69. Fie A, B M3 (R) astfel nct det (AX + B) 0, X M3 (R). S
a se
arate c
a exist
a C M3 (R) pentru care A = BC.
Gheorghe Iurea, Iasi
XI.70. Fie a, b, c laturile unui triunghi ale c
arui unghiuri au m
asurile n radiani
A, B, C si care are raza cercului nscris r. S
a se arate c
a distanta
de la punctul
3
M (A, B, C) la planul P : ax + by + cz + r = 0 este mai mare dect
.
3
Sorin Puspan
a, Craiova

Clasa a XII-a
XII.66. Fie a, b R cu 0 a < b si fie f : [a, b] R o functie de dou
a ori
derivabil
a pe [a, b], cu f 00 continu
a. Dac
a
Z b
a2
b2
f (x) dx = f 0 (a) f 0 (b) + bf (b) af (a) ,
2
2
a

s
a se arate c
a exist
a (a, b) astfel nct f 00 () = 0.

Mihai Haivas, Iasi

XII.67. Fie f : [0, 1] R o functie cu proprietatea c


a exist
a L 0 astfel nct
|f (x) f (y)| L |x y|, x, y [0, 1]. S
a se arate c
a pentru orice primitiv
aF a
lui f si pentru orice x1 , x2 , . . . xn [0, 1], n N, n 2, are loc

F x1 + x2 + . . . xn F (x1 ) + F (x2 ) + + F (xn ) L


(xi yj )2 .

2
n
n
2n
1i<jn

Dan-
Stefan Marinescu, Hunedoara

P (x)

, g (x) = eQ(x) , unde P , Q sunt polinoame


XII.68. Fie f, g : R R, f (x) = e
de grad m 1, avnd coeficien
tia,
respectiv
tiiRdominan
R n b, a, b (0, ).
n
a) S
a se calculeze lim f (n) 0 g (x) dx
g (n) 0 f (x) dx .
n

79

b) S
a se studieze buna definire a sirurilor (an )n1 si (bn )n1 , unde f (an ) =
1 Rn
an
1 Rn
f (x) dx, g (bn ) =
g (x) dx si apoi s
a se calculeze lim
.
0
0
n bn
n
n
Marius Apetrii, Iasi
ad
acinile
XII.69. Fie f R [X] polinom reciproc de grad 4n + 2, n N , avnd r
distincte, complexe si nereale. S
a se arate c
a f are cel putin o r
ad
acin
a de modul 1.
C
at
alin Tig
aeru, Suceava
XII.70. Fie G un grup de ordin n 4 cu proprietatea c
a exist
a m N, 1 <
m1
m < n, astfel nct G contine exact Cn1
subgrupuri de ordin m. Ar
atati c
a G este
abelian.
Marius T
arn
auceanu, Iasi

Probleme pentru preg


atirea concursurilor
A. Nivel gimnazial
G96. Fie a = x12m + x12n , unde m, n N . S
a se arate c
a num
arul a este
divizibil cu 13, dac
a si numai dac
a x este divizibil cu 13.
Artur B
al
auc
a, Botosani
. . . }b,
G97. Determinati a, b {0, 1, 2, . . . , 9}, a 6= 0, astfel nct num
arul A = abb
| {z
n ori

n > 2, s
a fie p
atrat perfect.

Gheorghe Iurea, Iasi

m n+1
G98. S
a se determine m, n N astfel nct
N .
+
n
m2
Gabriel Dospinescu, student, Paris

G99. Fie m, n dou


a numere naturale nenule astfel nct m divide n 1. Toate
numerele naturale ntre 1 si n se aseaz
a la ntmplare pe un cerc. Se calculeaz
a suma
oric
arui grup de m numere vecine. S
a se demonstreze c
a printre aceste sume exist
a
dou
a pentru care diferenta dintre ele este strict mai mare dect m 1.
Titu Zvonaru, Com
anesti
G100. n cte moduri putem colora cu 5 culori un p
atrat 3 3, astfel nct n
fiecare p
atrat 2 2 s
a existe patru culori diferite?
Gabriel Popa, Iasi
G101. S
a se demonstreze inegalitatea

!
1
1
1
16
4
1+
,
2 +
2 +
2
(1
+
bc)
(1
+
ca) (1 + ab)
a (1 + bc)
b (1 + ca)
c (1 + ab)
a, b, c (0, ) n conditia abc = 1. Cnd are loc egalitatea?
Gabriel Mrsanu si Andrei Nedelcu, Iasi
G102. S
a se determine valoarea maxim
a a parametrului m R+ astfel nct
p
b2 + c2 c2 + a2
a2 + b2
+
+
m 3 (a2 + b2 + c2 ), a, b, c R+ .
a
b
c
Dorel B
aitan si I. V. Maftei, Bucuresti
80

G103. Pentru a, b, c (0, 1) cu a + b + c = 2, s


a se arate c
a
abc 8 (1 a) (1 b) (1 c) .

Alexandru Negrescu, elev, Botosani

\ = 120 . Fie O (BC) astfel nct [AO


G104. Triunghiul ABC are m(BAC)
\ Pe [AO se ia punctul D astfel nct [BC este
este bisectoarea unghiului BAC.
\ S
bisectoarea interioar
a a unghiului ABD.
a se arate c
a AD + BD = AB + AC si
AB + AC 4 AO.
Petru R
aducanu, Iasi
G105. Se consider
a trapezul ABCD cu bazele AB, CD (AB > CD) si fie O
intersectia diagonalelor trapezului. Se duce linia mijlocie M N a trapezului si paralela
P Q prin O la bazele trapezului (M, P (AB), N, Q (BC) ). S
a se demonstreze
c
a trapezele ABM N si P QCD au diagonalele respectiv paralele.
Claudiu-
Stefan Popa, Iasi

B. Nivel liceal
L96. Fie cercurile C1 , C2 , C astfel nct C1 si C2 sunt tangente exterior n D
si fiecare dintre ele este tangent interior lui C n B, respectiv C. Tangenta comun
a
interioar
a cercurilor C1 si C2 taie cercul C n A si A1 . Dreapta AB taie cercul C1 n K,
iar dreapta AC taie cercul C2 n L. Din punctul M de pe cercul C se duc tangentele
M T1 si M T2 la cercurile C1 , respectiv C2 (T1 C1 , T2 C2 ). Dac
a M A < M A1 ,
A1 M
AM
ar
atati c
a M T1 + M T2 =
KL si |M T1 M T2 | =
KL.
A1 D
AD
Neculai Roman, Mircesti (Iasi)
L97. S
a se demonstreze c
a n orice triunghi are loc inegalitatea
1
1
1
1
+
2.
+ 2
2
2
2
2
ma (mb + mc ma ) mb (mc + ma mb )
S
mc (ma + mb mc )
I. V. Maftei si Dorel B
aitan, Bucuresti
L98. Se consider
a un triunghi oarecare ABC. Demonstrati c
a
27 r 3
;
1) sin4 A + sin4 B + sin4 C
2 R
4
3 r
ra
rb
rc
2) cos4 A + cos4 B + cos4 C
5
5
5
,
8 R
r
r
r
unde R este raza cercului circumscris, r este raza cercului nscris, iar ra , rb , rc sunt
razele cercurilor exnscrise.
Oleg Faynshteyn, Leipzig, Germania
L99. a) Care este num
arul minim de puncte din plan de coordonate ntregi astfel
nct, oricum ar fi alese, s
a existe trei puncte cu centrul de greutate de coordonate
ntregi.
b) S
a se arate c
a ntr-un spatiu n-dimensional exist
a 2n+1 puncte de coordonate
ntregi astfel nct oricare trei dintre acestea au centrul de greutate cu cel putin o
coodonat
a care nu este un ntreg.
Irina Mustata
a, Bremen, Germania
, student

1 2

L100. Fie x (0, 1); ar


atati c
a exist
a n N astfel nct {nx}
,
.
3 3
Ciprian Baghiu si Gheorghe Iurea, Iasi
81

an ak
Z.
k=0 n k
Adrian Zahariuc, elev, Bac
au

L101. Fie a, n 2 dou


a numere ntregi. S
a se arate c
a

n1
Q

L102. Fie p = 2k + 1 un num


ar prim. Atunci
2k
k
X
X

i
i
Cp+i1
2p 2 mod p2 , S2 =
Cp+i1
2 2p mod p2 .
S1 =
i=1

i=k+1

Marius Pachitariu, elev, Iasi

2
L103. Fie a, b, c, d reale astfel nct 1 + a
1 + b2 1 + c2 1 + d2 = 16.
Ar
atati c
a
3 ab + bc + cd + da + ac + bd abcd 5.
Mai mult, avem egalitate n cel putin una din inegalit
atile de mai sus dac
a si numai
dac
a a + b + c + d = abc + bcd + cda + dab.
Gabriel Dospinescu, student, Paris
n
L104. Fie x0 > 0 si xn = x[ n ] + x[ n ] + , pentru orice n > 0.
2
3
6
x
n
este convergent la 1.
a) S
a se arate c
a sirul
n n1
5
xn n
b) S
a se arate c
a dac
a > log3 , atunci lim
= 0.
n
2
n
Gabriel Dospinescu, student, Paris
L105. S
a se determine toate functiile continue f : (0, ) R, care verific
a
ecuatia functional
a
nxn1 f (xn ) = (x + 1) f (x) ,

unde n N , n fixat.

x (0, ) ,

Marian Tetiva si Dumitru Mihalache, Brlad

Training problems for mathematical contests


A. Junior highschool level
G96. Let a = x12m + x12n , with m, n N . Prove that a is divisible by 13 if and
only if x is divisible by 13.
Artur B
al
auc
a, Botosani
G97. Find a, b {0, 1, 2, . . . , 9}, a 6= 0, such that A = abb
. . . }b, n > 2 is a perfect
| {z
n times
square.
Gheorghe Iurea, Iasi
m
n
+
1
G98. Find m, n N such that
N .
+
n
m2
Gabriel Dospinescu, student, Paris
G99. Let m, n two positive integer such that m divides n 1. All positive
integers between 1 and n are put on a circle in an arbitrary way. One compute the
sum of any set of m neighbors numbers. Prove that among all these sums, there are
two of them for which their dierence is strictly grater than m 1.
Titu Zvonaru, Com
anesti
82

G100. In how many ways could one colour a square 3 3 such that in any little
square 2 2 to be four dierent colours?
Gabriel Popa, Iasi
G101. Prove the following inequality

!
1
1
1
16
4
1+
,
2 +
2 +
2
(1
+
bc)
(1
+
ca) (1 + ab)
a (1 + bc)
b (1 + ca)
c (1 + ab)
a, b, c (0, ) under the condition abc = 1. When does the equality holds?
Gabriel Mrsanu and Andrei Nedelcu, Iasi
G102. Find the maximal value of the parameter m R+ such that
p
b2 + c2 c2 + a2
a2 + b2
+
+
m 3 (a2 + b2 + c2 ), a, b, c R+ .
a
b
c
Dorel B
aitan and I. V. Maftei, Bucuresti
G103. For a, b, c (0, 1) and a + b + c = 2, show that
abc 8 (1 a) (1 b) (1 c) .

Alexandru Negrescu, highschool student, Botosani


\ = 120 . Let O (BC) such that [AO is
G104. The triangle ABC has m(BAC)
\ Let D be a point on [AO such that [BC to be interior
the angle bisector of BAC.
\
angle bisector of ABD. Prove that AD + BD = AB + AC and AB + AC 4 AO.
Petru R
aducanu, Iasi
G105. Let ABCD be a trapezium with AB, CD (AB > CD) as the bases and
consider that the diagonals of the trapezium intersect in O. We construct M N to be
the mean line of the trapezium and the parallel P Q, through O, to the bases of the
trapezium (M, P (AB), N, Q (BC) ). Prove that the trapeziums ABM N and
P QCD have the diagonal respectively parallel.
Claudiu-
Stefan Popa, Iasi

B. Highschool level
L96. Let C1 , C2 , C such that C1 and C2 touch each other externally in D and
each of them is interior tangent to C in B and C, respectively. The common interior
tangent to the circles C1 and C2 cuts the circle C in A and A1 . The line AB cuts the
circle C1 in K and the line AC cuts the circle C2 in L. From the point M laying on
the circle C we construct the tangent lines M T1 and M T2 to the circles C1 and C2 ,
A1 M
respectively (T1 C1 , T2 C2 ). If M A < M A1 prove that M T1 + M T2 =
KL
A1 D
AM
and |M T1 M T2 | =
KL.
AD
Neculai Roman, Mircesti (Iasi)
L97. Prove that, in any triangle, the following inequality holds
1
1
1
1
+
2.
+
m2a (mb + mc ma ) m2b (mc + ma mb )2 m2c (ma + mb mc )2
S

I. V. Maftei and Dorel B


aitan, Bucuresti

L98. Let ABC be an arbitrary triangle. Prove that


83

27 r 3
;
2 R
4
3 r
ra
rb
rc
5
5
5
,
2) cos4 A + cos4 B + cos4 C
8 R
r
r
r
where R is the radius of the circumcircle circle, r is the radius of the inscribed circle
and ra , rb , rc are the radius of the exinscribed circles.
Oleg Faynshteyn, Leipzig, Germany
L99. a) Which is the minimal number of points of integer coordinates contained
in a plane, such that no mater how they are chosen, there exist three of them with
their centre of gravity expressed by integer coordinates.
b) Prove that, in an n-dimensional space exist 2n+1 points of integer coordinates
such that any 3 of them have the center of gravity with at least one coordinate not
an integer.
Irina Mustata
, student, Bremen, Germany

1 2

L100. Let x (0, 1); prove that there exist n N such that {nx}
,
.
3 3
(By {} we denoted the fractional part.)
Ciprian Baghiu and Gheorghe Iurea, Iasi
n1
Q an ak
L101. Let a, n 2 two integers. Prove that
Z.
k=0 n k
Adrian Zahariuc, highschool student, Bac
au
L102. Let p = 2k + 1 be a prime number. Then
1) sin4 A + sin4 B + sin4 C

S1 =

2k
X

i=k+1

i
Cp+i1
2p 2 mod p2 ,

S2 =

k
X
i=1

i
Cp+i1
2 2p mod p2 .

Marius Pachitariu, highschool student, Iasi


L103. Let a, b, c, d some real numbers such that

1 + a2 1 + b2 1 + c2 1 + d2 = 16.

Prove that

3 ab + bc + cd + da + ac + bd abcd 5.
Moreover, at least one from the above inequalities becomes an equality if and only if
a + b + c + d = abc + bcd + cda + dab.
Gabriel Dospinescu, student, Paris
n
L104. Let x0 > 0 and xn = x[ n ] + x[ n ] + , for any n > 0.
3
6
x 2
n
a) Prove that the sequence
is convergent to 1.
n n1
5
xn n
= 0.
b) Prove that if > log3 , then lim
n
2
n
Gabriel Dospinescu, student, Paris
L105. Find all continuous functions f : (0, ) R, which verify the functional
equation
nxn1 f (xn ) = (x + 1) f (x) , x (0, ) ,

where n N , n is fixed.
Marian Tetiva and Dumitru Mihalache, Brlad
84

Pagina rezolvitorilor
BRA
SOV
Liceul "N. Titulescu". Clasa a X-a. CHIRA Roxana: VIII(56,58,61-63), IX(56,
61); COSTA Larisa: VIII(57,58,61-63), IX(56,61); LUCACI Adina: VIII(61-63),
IX(61,63); MOCANU Vlad: VIII(61-63), IX(61,63,65); NEGOESCU Anamaria: VIII
Iulian: VIII (61-63), IX(61,63,65); NUTU
(56,57,61,62), IX(56,61,63,65); NILA
Cosmin: VIII(61-63), IX(61,63,65); OBANCEA Dragos: VIII(61-63), IX(61,63,65);
S
CHIOPU Iulian: VIII(56,61-63), IX(56,61); ZBARCEA Adrian: VIII(56,57,61-63),

IX(56,61). Clasa a XI-a. BOTH Alexandru: IX(61,63,65), X.62, XI.61; BRAGARU


Andreea: IX(61,63,65), X.62, XI.61; CAIA Claudiu: IX(61,63,65), X.62, XI.61;
DELAST-VOINEA Alexandru: IX(61,63,65), X(58,62), XI.61; DIACONESCU Adrian: IX(61,63,65), X.62, XI.61; DUCA R
azvan: IX(61,63, 65), X.62, XI.61; DUMITRU Silviu: IX(61,63,65), X.62, XI.61; ENACHE Florin: IX(61,63,65), X.62,
XI.61; FLOREA Luminita: IX(57,61,63,65), X.62, XI.61; GIURGIU Beniamin: IX(61,
63,65), X.62, XI.61; GONTEA

Paul: IX(61,63,65), X.62, XI.61; GULIC Mihai:


IX(61,63,65), X.62, XI.61; IVASCU Andreea: IX(61,63,65), X.62, XI.61; LUKACS
Raluca: IX(61,63,65), X.62, XI.61; MARDALE Mihai: IX(61,63,65), X.62, XI.61;
MATIS Gheorghe: IX(61,63,65), X.62, XI.61; MRZESCU Andreea: IX(61,63,65),
X.62, XI.61; NAFRADI Jen: IX(61,63,65), X.62, XI.61; NAN Maria: IX(61,63,65),
X.62, XI.61; PANAIT Mihail: IX(61,63,65), X.62, XI.61; POPA C
at
alina: IX(61,63,

65), X.62, XI.61; RACEAN


Bogdan: IX(61,63,65), X.62, XI.61; SAVA Cosmin:

Florin: IX(61,63,65), X.62, XI.61; TRESCA


IX(61,63,65), X.62, XI.61; TABU
SCA
Ana-Maria: IX(61,63,65), X.62, XI.61.
CRAIOVA
Colegiul National "Carol I". Clasa a VI-a. STANCIU Ioan: V(61-65), VI(6163), VIII.61, G.90.

HRLAU
Scoala

"Petru Rares". Clasa a III-a (nv. BUDACEA Maria). NEICU Mara:


P(89,95-97,99). Clasa a IV-a. (nv. CRETU
Maria). BOUTIUC M
ad
alina: P(94,96 Andreea: P(94-101); PINTILII Alina: P(94-101).
101); BUZILA
Liceul "Stefan

cel Mare". Clasa a VII-a. APACHITEI

Ana-Maria: V(56-58),
VI(59,60); ATRGOVITOAE

Anca-Elena: V(56-58), VI(59,60); COJOCARU Iulia:


Ioana: V(56-58), VI(59,60); LENTER
V(56-58), VI(59,60); CURCA

Sonia: V(5658), VI(59,60); PINTILII Anda: V(56-58), VI(59,60).


IA
SI
Scoala

nr. 3 "Al. Vlahuta". Clasa a III-a (inst. MAXIM Gabriela). CELMARE Raluca-Iuliana: P(94-96,98,99); NEAGU Ramona-Mihaela: P(94-96,98,99);
POPOVICI Ionut: P(94-96,98,99); RUSU Ioana-Andreea: P(94-96,98,99); RUSU

M
ad
alina: P(94-96,98,99); SAVA Vlad: P(94-96,98,99). Clasa a III-a (nv. MARI Valentina). CULEA Alina: P(94-96,98,99); POPA Iulian: P(94-96,98,99);
UT
A
PROCA Ancuta-Ioana: P(94-96,98,99). Clasa a VII-a. DODU Corina: V(6163), VI.61, VII.63; IRIMIA Andreea: V(61-63), VI.61, VII.63; RUSU Laura-Elena:
V(61-63), VI.61, VII.63; UNGURU Claudiu: V(61-63), VI.61, VII.63.
85

Scoala

nr. 4 "I. Teodoreanu". Clasa a IV-a (nv. BUJOR Lorena). LUPAN


Dana: P(98-100,102,103).
Scoala

nr. 13 "Alexandru cel Bun". Clasa II-a (inst. COJOCARIU Ana). AGA Andra: P(94-97,99); CALIN

FITEI

Elena-Roxana: P(94-97,99); CARAMALAU


Andreea-Claudia: P(94-97,99); COJOCARIU Andreea: P(94-97,99); DUDUMAN
Luisa-Stefania: P(94-97,99); LELEU Alexandrina-Stefana: P(94-97,99); LUPASCU
Diana-Maria: P(94-97,99); MANOLACHE M
ad
alina-Andreea: P(94-97,99); MI A
Narcisa-Lorena: P(94-97,99); PASCU Gabriela: P(94-97,99); PADURARU

HAIL

Tiberiu-Stefan: P(94-97,99); RADUCEA


Marin-Andrei: P(94-97,99); SAVIN Cristina-Simona: P(94-97,99); S
TEFAN Bogdan-Vasile: P(94-97,99); S
TIUBEI CosminIonut: P(94-97,99). Clasa a III-a (nv. OBREJA Rodica): APETRII Alexandru:
P(94-97,99); ESANU Mihai: P(94-97,99); VATAVU Iulian: P(94-97,99).
Laura). BUHU
Scoala

nr. 22 "B. P. Hasdeu". Clasa a IV-a (nv. CHIRILA

Vlad: P(94-100,102,103); CHICHIRAU Alexandra-Elena: P(94-100). Clasa a IVa (nv. TRZIORU Iuliana). APOSTOL Ana-Maria: P(94-97,99,100); BALAN
Andrei: P(94-99); GNDU Alexandra: P(94-100); GRIEROSU Claudiu: P(94-100);
ATIC

LAM
Ioana: P(94-102); REBEGEA Andrada Elena: P(94-100); UNGUREANU Teofana: P(94-100).
Scoala

nr. 26 "G. Cosbuc". Clasa a IV-a (nv. BUCATARIU Rica). AELENEI


George-Ciprian: P(94-97,100); DUMITRU Ambra-Georgiana: P(94-97,99,100); GOLDAN Alexandru-Iulian: P(94-100); HRISCU Alexandra: P(94-100); IACOB Robert Alexandra:
Ionut: P(94-100); MOISA Adrian-Bogdan: P(94-100); MUSTEAT
A
M
P(94-98); SCUTARU Ionela-Cristina: P(94-100); TUDOSA
ad
alina: P(94-99);

Georgiana: P(94-98).
ZALINC
A
Albert: V(56-60); BARON BogColegiul National. Clasa a VI-a. BACUSCA
R
dan: V(56-60); BOTU
Alexandru: P.101, V(61-63), VI.61; CEUCA
azvan: V(5660); MOCANU Dan Mihai: P.103, V(61,63,65), VI.61; PRISTOPAN Codrin: V(5660); TIBA Bianca M
ad
alina: V(59-63,65), VI.63; Clasa a VII-a. CADAR Alexandra: V(61-65), VI(61,62); HUMELNICU Roxana: V(61-64), VI(61-65).
Colegiul National "Emil Racovita". Clasa a VI-a. TUDORACHE AlexandruGabriel: P(101-103), V(61-65), VI(61,63,64).
Colegiul National "C. Negruzzi". Clasa a VII-a. TIBA Marius: VII(62,64,65),
VIII(61,62), G(89,95); Clasa a VIII-a. OLAN Ioana: VI(61,64), VII(63,65), G(89,
95).
SIBIU
Colegiul National "Gh. Lazar". Clasa a XI-a. VLAD Emanuel: L(77-83).
SUCEAVA
Scoala

nr. 3. Clasa a II-a (inst. NECHITA Daniela). FECHET Mircea: P(84,85,


87-89,94-97,99).

86

Premii acordate rezolvitorilor


ASOCIATIA

"RECREATII

MATEMATICE" n colaborare cu redactia


revistei RECREATII
MATEMATICE acord
a cte o diplom
a si un premiu n
c
arti pentru trei aparitii la rubrica "Pagina rezolvitorilor" elevilor urm
atori:
Liceul "N. Titulescu", Botosani
Iulian (cl. a X-a): 1/2005(6pb); 2/2005(8pb); 1/2006(6pb);
NILA
NUTU
Cosmin (cl. a X-a): 1/2005(7pb); 2/2005(6pb); 1/2006(6pb);
OBANCEA Dragos (cl. a X-a): 1/2005(5pb); 2/2005(8pb); 1/2006(6pb).
Colegiul Naional "Carol I", Craiova
STANCIU Ioan (cl. a VI-a): 1/2005(10pb); 2/2005(7pb); 1/2006(10pb).
Scoala

"P. Rares", Hrl


au
NEICU Mara (cl. a III-a): 1/2005(11pb); 2/2005(5pb); 1/2006(5pb).
Scoala

nr. 3 "Al. Vlahuta", Iasi


DODU Corina (cl. a VII-a): 2/2004(5pb); 2/2005(6pb); 1/2006(5pb);
IRIMIA Andreea (cl. a VII-a): 2/2004(5pb); 2/2005(5pb); 1/2006(5pb);
UNGURU Claudiu (cl. a VII-a): 2/2004(5pb); 2/2005(5pb); 1/2006(5pb).
Scoala

nr. 4 "I. Teodoreanu", Iasi


LUPAN Dana (cl. a IV-a): 1/2005(9pb); 2/2005(6pb); 1/2006(5pb).
Scoala

nr. 22 "B. P. Hasdeu", Iasi


BUHU Vlad (cl. a IV-a): 1/2005(5pb); 2/2005(7pb); 1/2006(9pb);
Alexandra-Elena (cl. a IV-a): 1/2005(5pb); 2/2005(5pb);
CHICHIRAU
1/2006(7pb);

LAMATIC
Ioana (cl. a IV-a): 1/2005(8pb); 2/2005(10pb); 1/2006(9pb);
REBEGEA Andrada Elena (cl. a IV-a): 1/2005(6pb); 2/2005(7pb); 1/2006(7pb);
Scoala

nr. 26 "G. Cosbuc", Iasi


DUMITRU Ambra-Georgiana (cl. a IV-a): 1/2005(8pb); 2/2005(5pb);
1/2006(6pb);
HRISCU Alexandra (cl. a IV-a): 1/2005(6pb); 2/2005(7pb); 1/2006(7pb);
Alexandra (cl. a IV-a): 1/2005(6pb); 2/2005(6pb); 1/2006(5pb);
MUSTEAT
A

Georgiana (cl. a IV-a): 1/2005(6pb); 2/2005(5pb); 1/2006(5pb).


ZALINC
A
Colegiul National, Iasi
CADAR Alexandra (cl. a VII-a): 1/2005(8pb); 2/2005(9pb); 1/2006(7pb).
Colegiul National "Emil Racovita", Iasi
TUDORACHE Alexandru-Gabriel (cl. a VI-a): 1/2005(9pb); 2/2005(8pb);
1/2006(12pb).
87

ASOCIA
TIA RECREA
TII MATEMATICE
La data de 14.02.2005 a luat fiinta ASOCIATIA

RECREATII

MATEMATICE, cu sediul n Iasi (str. Aurora, nr. 3, sc. D, ap. 6), avnd ca scop sprijinirea activitatilor de matematica specifice nvatamntului preuniversitar, organizarea si desfasurarea de activitati care sa contribuie la dezvoltarea gustului pentru
matematica n rndurile elevilor, profesorilor si iubitorilor de matematica si stimularea preocuparilor si cercetatorilor originale.
Obiectivele majore pentru atingerea scopului propus sunt:
1. editarea unei reviste destinat
a elevilor si profesorilor revista "Recreatii
Matematice";
2. fondarea unei biblioteci de matematic
a elementar
a biblioteca "Recreatii
Matematice";
3. alc
atuirea unei colectii de c
arti de matematic
a elementar
a, c
arti de referinta
si aflate la prima aparitie Colectia "Recreatii Matematice".
Poate deveni membru al Asociatiei, printr-o simpla completare a unei cerei tip,
orice perosana care adera la obiectivele acesteia si sprijina realizarea lor.

Membri de onoare, academicienii:

Constantin Corduneanu
Radu Miron

Continuarea listei membrilor din RecMat - 2/2005:


21
22
23
24
25
26
27
28
29
30
31
32
33
34
35
36
37
38
39

Numele si prenumele
Dumitru Neagu
Tudorache Rodica
Tiba I. Dan
Popovici Mihaela
Popovici Florin
Negrescu Alexandru
S
erdean Ioan
Marinescu Dan-Stefan
Popa Vasile
Marin Mirela
Maxim Gabriela
Bejan Cornelia - Livia
Brnzei Dan
Teodoru Georgeta
Roman Neculai
Bors Dan Mircea
Haivas Mihai
Popa Claudiu - S
tefan
Minut Petru

Locul de munca
Data nscrierii
Lic. "G. Ibr
aileanu", Iasi
10.03.2005
Univ. Tehn. "Gh. Asachi", Iasi
14.03.2005
Inst. de Mat. al Acad. Rom., Bucuresti 20.03.2005
S. C. Selgros Cash&Carry, Brasov
20.03.2005
Lic. "N. Titulescu", Brasov
20.03.2005
C. N. "A.T. Laurian", Botosani (elev) 25.03.2005
C. N. "Aurel Vlaicu", Or
astie
28.03.2005
C. N. "Iancu de Hunedoara", Huned. 28.03.2005
C. N. "V. Alecsandri", Galati
29.03.2005
S
c. "Al. Vlahuta", Iasi
12.04.2005
S
c. "Al. Vlahuta", Iasi
14.04.2005
Univ. Tehn. "Gh. Asachi", Iasi
25.04.2005
Univ. "Al. I. Cuza", Iasi
29.04.2005
Univ. Tehn. "Gh. Asachi", Iasi
09.05.2005
S
c. "V. Alecsandri", Mircesti (Iasi)
11.05.2005
Univ. Tehn. "Gh. Asachi", Iasi
13.05.2005
Inst. Cerc. Ec. "Gh. Zane", Iasi
18.05.2005
S
c. "Alecu Russo", Iasi
27.05.2005
Univ. "D. Cantemir", Tg. Mures
30.06.2005
88

Revista semestrial RECREAII MATEMATICE


este editat de
ASOCIAIA RECREAII MATEMATICE. Apare la datele de 1 martie i
1 septembrie i se adreseaz elevilor, profesorilor, studenilor i tuturor celor
pasionai de matematica elementar.
n atenia tuturor colaboratorilor
Materialele trimise redaciei spre publicare (note i articole, chestiuni de
metodic, probleme propuse etc.) trebuie prezentate ngrijit, clar i concis; ele
trebuie s prezinte interes pentru un cerc ct mai larg de cititori. Se recomand ca
textele s nu depeasc patru pagini. Evident, ele trebuie s fie originale i s
nu fi aprut sau s fi fost trimise spre publicare altor reviste. Rugm ca materialele tehnoredactate s fie nsoite de fiierele lor.
Problemele destinate rubricilor: Probleme propuse i Probleme pentru
pregtirea concursurilor vor fi redactate pe foi separate cu enun i demonstraie/rezolvare (cte una pe fiecare foaie) i vor fi nsoite de numele autorului, coala i localitatea unde lucreaz/nva.
Redacia va decide asupra oportunitii publicrii materialelor primite.
n atenia elevilor
Numele elevilor ce vor trimite redaciei soluii corecte la problemele din
rubricile de Probleme propuse i Probleme pentru pregatirea concursurilor
vor fi menionate n Pagina rezolvitorilor. Se va ine seama de regulile:
1. Pot trimite soluii la minimum cinci probleme propuse n numrul
prezent i cel anterior al revistei; pe o foaie va fi redactat soluia unei singure
probleme.
2. Elevii din clasele VI-XII au dreptul s trimit soluii la problemele
propuse pentru clasa lor, pentru orice clas mai mare, din dou clase mai mici i
imediat anterioare. Elevii din clasa a V-a pot trimite soluii la problemele propuse
pentru clasele a IV-a, a V-a i orice clas mai mare, iar elevii claselor I-IV pot
trimite soluii la problemele propuse pentru oricare din clasele primare i orice clas mai mare. Orice elev poate trimite soluii la problemele de concurs (tip G i L).
3. Vor fi menionate urmtoarele date personale: numele i prenumele,
clasa, coala i localitatea.
4. Plicul cu probleme rezolvate se va trimite prin pot (sau va fi adus
direct) la adresa Redaciei:
Prof. dr. Temistocle Brsan
Str. Aurora, nr. 3, sc. D, ap. 6,
700 474, Iai
Jud. IAI
E-mail: tbirsan@math.tuiasi.ro

CUPRINS
Elogiu adus revistei Gazeta Matematic la 110 ani de apariie nentrerupt .................. 1
100 de ani de la naterea matematicianului Grigore C. Moisil ............................................. 2

ARTICOLE I NOTE
D. M. BTINEU-GIURGIU Asupra problemei 809 din Gazeta Matematic,
volumul VIII (19021903)................................................................. 5
G. DOSPINESCU Cteva proprieti ale subgrupurilor finite din GLn ( \ ) ................... 8
T. BRSAN Ceviene i triunghiuri triomologice............................................................... 13
A. MOSCALIUC Construcii aproximative cu rigla i compasul ale numrului ...... 16
I. V. MAFTEI Inegaliti generatoare de noi inegaliti .................................................. 19
C.-t. POPA Asupra unei probleme dat la ONM, Bistria, 2005................................. 21

NOTA ELEVULUI
M. TIBA Asupra criteriului de congruen LLU.............................................................. 23
I. OLAN O generalizare a identitii Botez - Catalan ...................................................... 25
M. PACHIARIU Acoperiri ale planului laticial cu figuri............................................. 26

CHESTIUNI METODICE
M. TETIVA Metoda normrii ............................................................................................. 30
Gh. IUREA Asupra unei recurene de ordin doi ............................................................. 35

CORESPONDENE
Olimpiada Internaional de Matematic B. O. Zhautykov ............................................... 37

PROBLEME I SOLUII
Soluiile problemelor propuse n nr. 1/2005.......................................................................... 43
Soluiile problemelor pentru pregtirea concursurilor din nr. 1/2005 ................................. 61
Probleme propuse..................................................................................................................... 75
Probleme pentru pregtirea concursurilor .............................................................................. 80
Training problems for mathematical contests ....................................................................... 82
Pagina rezolvitorilor .............................................................................................................. 85
ASOCIAIA RECREAII MATEMATICE ...................................................................... 88
5 lei

Anul IX, Nr. 1

Ianuarie Iunie 2007

RECREAII
MATEMATICE
REVIST DE MATEMATIC PENTRU ELEVI I PROFESORI

e i = 1

Asociaia Recreaii Matematice


IAI - 2007

Semnificaia formulei de pe copert:


i
ntr-o form concis, formula e = 1 leag cele patru ramuri fundamentale
ale matematicii:
ARITMETICA
GEOMETRIA
ALGEBRA
ANALIZA MATEMATIC

reprezentat
reprezentat
reprezentat
reprezentat

de
de
de
de

i
e

Redacia revistei :
Petru ASAFTEI, Dumitru BTINEU-GIURGIU (Bucureti), Temistocle BRSAN, Dan
BRNZEI, Ctlin - Cristian BUDEANU, Constantin CHIRIL, Eugenia COHAL, Adrian
CORDUNEANU, Mihai CRCIUN (Pacani), Gabriel DOSPINESCU (student, Paris),
Paraschiva GALIA, Paul GEORGESCU, Mihai HAIVAS, Gheorghe IUREA, Lucian Georges LDUNC, Mircea LUPAN, Gabriel MRANU, Andrei NEDELCU, Gabriel
POPA, Dan POPESCU (Suceava), Florin POPOVICI (Braov), Maria RACU, Neculai
ROMAN (Mirceti), Ioan SCLEANU (Hrlu), Ioan ERDEAN (Ortie), Dan TIBA
(Bucureti), Marian TETIVA (Brlad), Lucian TUESCU (Craiova), Adrian ZAHARIUC
(Bacu), Adrian ZANOSCHI, Titu ZVONARU (Comneti).

Adresa redaciei:
Catedra de Matematic Universitatea Tehnic Gh. Asachi Iai
Bd. Carol I, nr.11, 700506, Iai
Tel. 032 213737 / int. 123
E-mail: recreatii.matematice@gmail.com
http://www.recreatiimatematice.uv.ro
COPYRIGHT 2007, ASOCIAIA RECREAII MATEMATICE
Toate drepturile aparin Asociaiei Recraii Matematice. Reproducerea integral sau
parial a textului sau a ilustraiilor din aceast revist este posibil numai cu acordul prealabil
scris al acesteia.
TIPRIT LA SL&F IMPEX IAI
Bd. Carol I, nr. 3-5
Tel. 0788 498933
E-mail: simonaslf@yahoo.com

Anul IX, Nr. 1

Ianuarie Iunie 2007

RECREAII
MATEMATICE
REVIST DE MATEMATIC PENTRU ELEVI I PROFESORI

e i = 1
Revist cu apariie semestrial
publicat de

ASOCIAIA RECREAII MATEMATICE

IAI - 2007

300 de ani
de la nasterea lui
Leonhard Euler (1707 1783)
Cititi pe Euler! Cititi pe Euler, el
este Maestrul nostru, al tuturor.
P. S. Laplace
El este geniul care a pus n valoare
geniile succesorilor sai.
J. Bertrand
Matematician, astronom, fizician si inginer,
Leonhard Euler a fost fara ndoiala cel mai
prolific si unul dintre cei mai mari matematicieni ai tuturor timpurilor, dominnd cu autoritate matematica secolului XVIII (apropiindu-se
ca valoare de Lagrange).
Euler, ca savant, a reunit o intuitie
formidabil
a cu o imaginatie creatoare exceptional
a, o memorie inegalabil
a, abilit
ati de calcul extraordinare si o putere de munca fantastica.
A debutat n activitatea de cercetare la
vrsta de 16 ani cu o expunere privind filozofia
lui Newton si Descartes. n pofida faptului c
a
la 28 de ani pierde vederea la ochiul drept si
la 54 de ani ramne complet orb, a continuat cu
asiduitate aceast
a munc
a pn
a la vrsta de 77 de
ani, cnd, asa cum afirm
a Condorcet "nceteaz
a
de odata cu a calcula si de a trai". A redactat anual, n medie, n jur de 800 pagini
text stiintific publicnd peste 900 de articole si 90 de volume (din care 62 de matematic
a, mecanic
a si astronomie). Dup
a ce a orbit complet, a dictat secretarilor s
ai 320
de memorii (dintre care multe sunt volume ntregi).
Leonhard Euler s-a nascut la 15 aprilie 1707 la Basel, n Elvetia, n familia
unui preot sarac. Tatal sau, Paul Euler, era pasionat de matematica si studiase n
tinerete cu Jean si Iacob Bernoulli. Paul si initiaz
a fiul n matematic
a si dorind
ca acesta s
a-i continue cariera de preot l trimite s
a studieze filozofia si teologia la
Universitatea din Basel. Aici are ca profesor pe Jean Bernoulli care remarca talentul
sau matematic de exceptie. Leonhard Euler devine prieten si colaborator cu fiii
profesorului, matematicienii Nicolas (16871759) si Daniel Bernoulli (17001782).
La propunerea acestora n 1727 devine membru al Academiei de Stiin
te din Sankt
Petersburg, nfiintata de tarina Ecaterina I-a a Rusiei. n 1730 obtine catedra de
matematica la aceasta academie. n acelasi an se casatoreste cu fiica unui pictor rus
cu care a avut 13 copii, dintre care numai 5 i-au supravietuit. n 1740 regele Frederic
al II-lea (cel mare) al Prusiei reorganizeaz
a Academia din Berlin, unde Euler este
numit director al sectiei de matematica. Postul de presedinte al acestei academii a
1

fost atribuit lui Voltaire, ceea ce l nemultumeste pe Euler. n 1866, la solicitarea


tarinei Ecaterina a II-a si n urma unui conflict cu regele Frederic, se ntoarce la
Sankt Petersburg mpreuna cu fiii sai Johan Albrecht (17341800), matematician,
Carol (17401790), medic si Cristoph (17431812), ofiter n armata prusac
a, si care
apoi a murit ca general n armata rus
a. Euler este numit director al Academiei de
Stiin
te din Sankt Petersburg si i se ofera o locuinta confortabila. Din nefericire, la
scurt timp orbeste complet, iar casa i este distrusa de un incendiu n care viata lui
Euler a fost salvat
a cu dificultate. Cu aceast
a ocazie si-a pierdut si o mare parte
dintre manuscrise. La vrsta de 64 de ani i moare sotia si se rec
as
atoreste cu o sor
a
vitrega a primei sale sotii. La 18 septembrie 1783, dupa ce rezolvase o problema
dificila privind miscarea baloanelor (inventate de curnd de Mongolfier ) si bea un
ceai mpreun
a cu nepotii s
ai le spune acestora "eu mor". Acestea au fost ultimele
cuvinte ale acestui savant ntre savanti, supranumit de contemporanii s
ai Principele
matematicii sau Analiza ncarnata. Apropiatii sai au fost unanimi n al aprecia ca
un om blnd si binevoitor, de o modestie coplesitoare.
Extraordinara vitalitate a lui L. Euler, pn
a n ultima clip
a a vietii, se explic
a
prin constitutia sa robust
a si s
an
atoas
a, viata organizat
a, munca sa continu
a, pasiunea pentru matematica, precum si prin credinta sa. A fost de o religiozitate
profund
a (o justificare a existentei lui Dumnezeu, bazat
a pe notiunea de limit
a, s-a
p
astrat n unele manuale de teologie pn
a n zilele noastre).
Opera lui L. Euler a impulsionat profund ntreaga dezvoltare a matematicii. El a
asimilat n mod critic matematica marilor sai predecesori (Fermat, Pascal, Descartes,
Newton, Leibniz s.a.), a mbog
atit continutul matematicii, a introdus metode noi de
cercetare, a introdus simboluri adecvate, a promovat utilizarea aparatului matematic
n domenii nematematice si a insistat pentru expunerea clar
a, logic
a si ntr-un limbaj
accesibil unui cerc ct mai larg de cititori a problemelor de matematica. Este extrem
de dificil s
a se prezinte n cteva rnduri uriasa contributie a lui Euler n matematic
a.
n teoria numerelor L. Euler este primul care a studiat sistematic problema distributiei numerelor prime introducnd pentru prima oar
a asa numita functie zeta a
lui Riemann (n cazul argumentului real), a dat noi demonstratii teoremei lui Euclid
(care afirm
a c
a exist
a o infinitate de numere prime), a ar
atat necesitatea conditiei
suficiente date de Euclid pentru ca un num
ar natural par s
a fie perfect. A introdus
functia indicatoare care i poart
a numele si cu ajutorul c
areia a generalizat si demonstrat mica teorema a lui Fermat, a dat primul enunt al teoremei lui Dirichlet privind
numerele prime din progresii aritmetice, a generalizat algoritmul lui Euclid (introducnd parantezele lui Euler ), a perfectionat aparatul fractiilor continue, a creat
teoria resturilor patratice si a demonstrat teorema privind reprezentarea numerelor
ntregi prin forme patratice. Abilitatile sale de calcul l-au ajutat sa gaseasca exemple
care ne uimesc si ast
azi: a dat exemple de numere perfecte mergnd pn
a la ordinul
lui 1010 , a dat 65 de exemple de perechi de numere prietene (amiabile), exemple de
numere prime gemene si a construit contraexemple care dovedeau c
a unele "teoreme"
obtinute de predecesori ai sai, prin inductie incompleta, nu sunt adevarate.
n Geometrie, L. Euler a studiat transform
arile de coordonate n spatiul euclidian, a stabilit reprezent
arile analitice ale unor figuri din acest spatiu (cilindri, conuri,
suprafete de rotatie), a facut un studiu analitic al conicelor, a studiat curbele din
2

spatiul euclidian, a introdus notiunile de linie geodezica si curbura normala pentru


suprafete, a studiat suprafetele de arie minima care includ o curba nchisa neplana.
n geometria elementara numele sau este legat de cercul celor noua puncte, dreapta
pe care se afl
a centrul de greutate, ortocentrul si centrul cercului circumscris unui
triunghi (dreapta lui Euler ) etc.
El este initiatorul cercetarilor de topologie algebrica, stabilind n 1752 relatia
v m + f = 2 (v = numarul vrfurilor, m = numarul muchiilor si f numarul
fetelor unui poliedru convex), rezolvnd problema celor sapte poduri din Knigsberg
etc.
n Algebra a ncercat sa demonstreze teorema fundamentala a algebrei, a enuntat
teorema functiilor simetrice si a dezvoltat teoria determinantilor.
A obtinut rezultate numeroase si fundamentale n diverse domenii: calculul diferential si integral (a publicat dou
a tratate renumite de analiz
a matematic
a), teoria
ecuatiilor diferentiale ordinare si cu derivate partiale, teoria integralelor eliptice etc.
A introdus integrala multipla, functiile gamma si beta, a definit numarul e si
logaritmul natural, a definit functiile trigonometrice ca functii circulare, a ar
atat c
a
ei = 1 si, mai general, eix = cos x + i sin x etc.
De la el mostenim multe din notatiile folosite astazi: f (x), e, , ex , ln x,
(constanta lui Euler ) s.a.
Euler a adus contributii n mecanica punctului si mecanica corpului solid, definind
notiuni ca: centru de masa, centru de inertie, moment de inertie. n 1755 a publicat
un tratat de hidrodinamica cu numeroase aplicatii practice. A fost preocupat si de
optica si tehnica (constructia navelor).
Se spune c
a n ultimii ani de viata L. Euler avea n memorie fiecare am
anunt din
creatia sa uriasa. Dar Euler nu a fost numai un mare matematician. El era un om de
vasta cultura generala avnd cunostinte bogate de teologie, filozofie, botanica, chimie,
istorie, medicin
a, literatur
a si muzic
a (a scris si o teorie matematic
a a muzicii). Recita
din memorie n ntregime "Eneida" si stia cu ce vers ncepe si se termin
a fiecare pagin
a
din cartea lui Virgiliu.
Asa cum afirma J. Bertrand, nici un alt mare savant nu a lucrat cu mai mult
zel, mai multa rvna si cu mult folos pentru progresul unei stiinte, ca Euler pentru
matematica.

Prof. dr. Petru Minu


t

Profesorul Dumitru Ion Mangeron (1906 1991)


In Memoriam
Printre profesorii de seam
a, n
ascuti n urm
a cu un
secol, se numara si regretatul D. I. Mangeron. Cunoscuta personalitate a vietii universitare si academice
iesene, D. I. Mangeron s-a n
ascut la 15/28 noiembrie
1906 n orasul Chisin
au. Dup
a absolvirea liceului n
orasul sau natal s-a nscris n anul 1926 la sectia de
matematica a Facultatii de stiinte din Iasi, n care pe
atunci existau sectii pentru toate stiintele exacte.
A absolvit n mod str
alucit studiile universitare n
anul 1930, avnd profesori de mare valoare, daca nu ar fi
sa-l amintim dect pe Alexandru Myller, care luase doctoratul la vestitul matematician David Hilbert. Odat
a
cu nfiintarea n 1910 de c
atre A. Myller a Seminarului
matematic, care astazi i poarta numele, la Iasi ncepuse
cu adevarat o activitate propriu-zisa de cercetare, la care
si-a adus aportul si fostul student D. I. Mangeron. Timp
de doi ani (1930 - 1932), preg
ateste sub ndrumarea renumitului profesor Mauro
Picone teza de doctorat, intitulata Sopra un problema al contorno per unequazione
dierenziale alle derivati parziali di quatro ordine con le caratteristiche reali doppie.
ntors n tar
a, a functionat la Universitatea iesean
a mai nti ca asistent, fiind numit
n 1936 conferentiar la disciplina de analiz
a matematic
a. ncepnd cu anul universitar 1938/1939, a fost numit conferentiar la disciplina de matematici generale la nou
nfiintata Scoala
Politehnica din Iasi, unde, n anul 1941, devine profesor la catedra

de mecanic
a. ncepnd cu acest an, a predat cursuri de matematic
a si de mecanic
a
teoretic
a la facult
atile de mecanic
a si de electrotehnic
a. Preocup
arile sale stiintifice au fost strns legate si de cerintele unui nvatamnt tehnic superior de nalta
tinuta, publicnd aum, pe lnga lucrarile din teoria ecuatiilor cu derivate partiale,
si numeroase lucr
ari de mecanica analitica, teoria mecanismelor, teoria optimizarii,
teoria acceleratiilor reduse si a acceleratiilor de ordin superior. A publicat si lucrari de istoria matematicii. A fost membru activ la mai multe societati stiintifice
de matematica, mecanica teoretica si aplicata, astronomie, om de stiinta emerit al
Romniei, redactor responsabil la Buletinul Institutului Politehnic din Iasi, membru n consiliul de conducere al revistelor Revue Roumaine des Sciences Tchniques
(Srie de Mcanique Applique) si Studii si Cercetari de Mecanica Aplicata, conducator de doctorate n mecanica tehnica (30 de ingineri au obtinut titlul de doctor sub
ndrumarea sa).
Studiul ecuatiilor cu derivate partiale a devenit principalul s
au domeniu de cercetare n care, singur sau cu unii dintre elevii si colaboratorii sai, a publicat sute de
lucrari n diverse reviste de specialitate din tara noastra sau din tari cu mare prestigiu
stiintific, ceea ce face ca acum n matematic
a s
a se vorbeasc
a despre ecuatii polivalente
Mangeron, operatori interpolanti Mangeron, teoreme Mangeron, ecuatiile Mangeron.
Profesorul D. I. Mangeron a fost un om plin de entuziasm, energie, spirit de
4

initiativ
a, colegialitate des
avrsit
a, un om care ncuraja tineretul studios si care era
capabil de prietenia cea mai sincera. Dovada stau scrisorile pline de atentie si delicatete catre bunul sau prieten de o viata, Al. C. Climescu, trimise de la Universitatea
din Edmonton (Canada), unde n mai multe rnduri a fost visiting profesor, dup
a
iesirea sa la pensie. Cu alt
a ocazie, despre Al. C. Climescu a scris: A fost tocmai
atunci (la nceputul razboiului) mobilizat n arma antiaeriana si astfel au trecut ani
de zile, pna n 1945. ntre timp, am reusit sa se formeze comisia de concurs pentru
ncadrarea sa ca profesor. Comunicndu-i ca el se va ntoarce pe noua sa pozitie de
profesor titular, am primit cu emotie telegrama n care El afirma ca nu va uita niciodata acest rezultat al eforturilor facute n lipsa lui. Al. C. Climescu a avut ocazia sa
se revanseze atunci cnd, asa cum scrie Profesorul, n acele vremuri s-a facut tabula
rasa din tot ce am lucrat si realizat eu. n 1954, Al. C. Climescu descoper
a n revista Memorial des Sciences Mathmatiques un articol intitulat Calcul Symbolique,
n care exista fraza: On leur doit Mauro Picone et D. Mangeron deux progrs
principaux. Aceasta recunoastere internationala a facut ca, n cele din urma, sicanele
la care era supus Profesorul s
a nceteze, s
a i se recunoasc
a meritele de cercet
ator de
valoare si, partea oarecum comic
a, s
a i se acorde o nou
a diplom
a de doctor n stiinte
fizico-matematice pentru ca, chipurile, cea din Italia nu ar fi fost buna. n legatura
cu aceasta, Profesorul a spus: Am trait astfel, n cadrul greutatilor pe care le-am avut
ani de zile, o reusita faptica si morala, gratie atentiei si preocuparilor de pasionat
cercetator al iubitului nostru Prieten disparut.
Un aspect foarte important al activitatii Profesorului D. I. Mangeron este legat
de nfiintarea revistei Buletinul Institutului Politehnic din Iasi (1946), pentru care
a trebuit s
a nving
a greut
atile ivite dup
a terminarea r
azboiului si obtuzitatea noilor
factori de decizie. A contribuit din plin la afirmarea pe plan international si nu
numai national a acestei reviste, mpreuna cu alti profesori ai Politehnicii si oameni
de stiinta din alte tari, printre care si ctiva laureati ai premiului Nobel.
Profesorul D. I. Mangeron a r
amas activ si optimist pn
a la inevitabilul sfrsit,
desi a locuit ntr-un apartament modest, lipsit de confort. Cnd, n cele din urm
a, dl.
rector al Politehnicii, devenita acum Universitatea Tehnica "Gh. Asachi", a reusit sa
obtin
a pentru Profesor o repartitie ntr-o nou
a locuinta, acesta a spus: "Dragul meu,
este prea trziu". Prea trziu, a sosit si stirea alegerii sale ca membru corespondent
al Academiei Romne. Se afla n spital, unde a decedat la 26 februarie 1991.
Ne-am despartit de stimatul nostru Profesor n ziua de 1 martie 1991, dupa slujba
la care au asistat multi dintre colaboratorii si elevii s
ai, colegi, fosti doctoranzi,
ingineri si profesori, tinut
a la biserica Sfntul Nicolae din dealul Copoului. Pe drumul
c
atre cimitirul Podgoria, o ninsoare linistit
a a adus mp
acarea n sufletele noastre si
ne-am gndit ca, cel care a iubit att de mult scoala, oamenii si natura a plecat dintre
noi la fel de frumos cum a tr
ait, l
asnd o amintire si o oper
a pe m
asur
a, demn
a de
spiritele cu adev
arat alese.
Ast
azi, la mormntul Profesorului D. I. Mangeron se afl
a un monument, ridicat
de fostii sai admiratori, iar Bulevardul pe care se afla facultatile si celelalte cladiri
ale Universit
atii Tehnice poart
a numele "Bulevardul D. Mangeron".

Prof. dr. Adrian Corduneanu


5

Grigore Moisil concentrat n 1939, Sinesti, cu grad de sublocotenent (r)


(persoana culcat
a)
Fotografia-document (recto-verso) a fost oferit
a pentru publicare redactiei
de catre conf. dr. Gh. Costovici.
6

Similitudini n plan si puncte Torricelli asociate


1

C
at
alin TIG
AERU

Subiectul lucrarii l reprezinta operatia de compunere a similitudinilor aplicata unei


configuratii geometrice: un triunghi ABC si doua puncte arbitrare M, N
/ {A, B, C},
la care vom atasa punctul P , care este centrul similitudinii S3 = S2 S1 , unde S1 este
similitudinea centrat
a n C, care l transport
a pe M n A, iar S2 este centrat
a n B,
transportndu-l pe A n N . Doua aspecte vom lamuri, legate de subiectul n cauza.
n primul rnd, vom arata cum rolul pe care l joaca punctul P poate fi preluat si de
punctele M si N . Mai mult, vom demonstra c
a putem inversa rolurile triunghiurilor
ABC si M N P . Al doilea aspect al lucr
arii se refer
a la identificarea a dou
a puncte
Torricelli generalizate, pe care le numim asociate compunerii celor doua similitudini.
n final vom lamuri si o situatie interesanta, credem, cu caracter de noutate, legata de
coincidenta acestor puncte. n expunere se foloseste formalismul complex, deoarece
permite atacarea unor probleme grele, pentru care solutia sintetic
a se dovedeste a fi,
n prima faza, greu de vazut. Interpretarile geometrice nsotesc, n limita spatiului,
rezultatele teoretice. Mai precizam ca unele rezultate sunt demonstrate n [3] si [5].
1. Compunerea similitudinilor si teorema fundamental
a. Multime punctelor planului P se identific
a, prin fixarea unui reper, cu multimea numerelor complexe.
Definitie. Similitudinea de centru M0 P , de raport k [0, ) si de unghi
(, ] este functia SM0 (k, ) : P P , definita astfel: daca M P si M 0 =
SM0 (k, ) (M ), avem
|M0 M 0 |
(a) dac
a M 6= M0 , atunci k =
, = m(M\
M0 M 0 );
|M0 M |
(b) daca M = M0 , atunci M 0 = M0 .
1

Reamintim c
a similitudinile sunt bijectii, anume (SM0 (k, )) = SM0 (1/k, ).
Dac
a z0 este afixul lui M0 si z afixul lui M , atunci expresia analitic
a a similitudinii
este descrisa de functia
(1)
sz0 (k, ) (z) = z0 + (z z0 ) kei .

Sa notam ca, daca z 0 este afixul lui M 0 , atunci


z 0 z0
kei =
.
(10 )
z z0
Propozitia 1. Consideram similitudinile SM1 (k1 , 1 ), SM2 (k2 , 2 ), afixele punctelor M1 si M2 fiind respectiv z1 si z2 . Daca k1 k2 ei(1 +2 ) 6= 1, atunci exista punctul
X, de afix x, astfel nct SM2 (k2 , 2 ) SM1 (k1 , 1 ) = SX (k1 k2 , 1 + 2 ), unde
afixul lui X este determinat de relatia
1 k2 ei2
1 k2 ei2
=
z
+
(z

z
)
.
(2)
x = z1 + (z2 z1 )
1
2
1
1 k1 ei1 k2 ei2
1 k1 k2 ei(1 +2 )
Demonstratia acestui rezultat clasic se g
aseste n [4]. S
a not
am c
a relatia
a cu relatiile k1 k2 = 1 si 1 + 2 = 0 (mod 2).
k1 k2 ei(1 +2 ) = 1 este echivalent
1

Lect. dr., Univ."Stefan cel Mare", Suceava

n cele ce urmeaz
a, fix
am cadrul n care se va desf
asura analiza noastr
a; anume,
se considera triunghiul ABC, cu afixele respectiv a, b, c si fie M , N doua puncte din
plan, fixate, diferite de A, B si C, de afixe m si n. Luam n considerare similitudinile
de centre C si B, care transfer
a pe M n A, respectiv pe A n N si vom determina

punctul P , care va fi centrul compunerii celor dou


a similitudini. Tinnd
cont de (10 )
a

c
n

b
si de (2), dac
a punem k1 ei1 =
, k2 ei2 =
, atunci punctul P satisface
mc
ab
au, notat cu p, fiind descris de
SB (k2 , 2 )SC (k1 , 1 ) = SP (k1 k2 , 1 + 2 ), afixul s
p = c + (b c)

na
ba
ac
nb
mc ab

(3)

Pentru nceput cteva observatii, legate de aspectele geometrice foarte particulare


ale formulei de mai sus, pe care cititorul le poate verifica prin calcul direct:
cazul particular k1 k2 6= 1 si 1 = 2 = 0 (mod ) este echivalent cu teorema
lui Menelaos;
dac
a 1 + 2 = 0 (mod ), atunci punctele M , N , P sunt coliniare, pentru
k1 k2 = 1 si 1 + 2 = , punctul P fiind mijlocul segmentului M N ;
o analiz
a a formulei arat
a c
a, dac
a M, N
/ {A, B, C}, atunci si P
/ {B, C};
exista situatii pentru care P A, cum se verifica n: a = 0, b = 1, c = i, m = 1,
n = i; conform (3), p = a = 0.
Urmatorul rezultat lamureste prima problema asociata tripletelor{A, B, C} si
{P, M, N }, descrisa n introducere.
Teorema 1. Daca P are afixul p, determinat de formula (3), atunci
cb
ma
ba
pc
(a) daca punem k3 ei3 =
, k4 ei4 =
, k5 ei5 =
, k6 ei6 =
,
pb
ca
na
bc
va rezulta SA (k4 , 4 ) SB (k3 , 3 ) = SN (k3 k4 , 3 + 4 ), SC (k6 , 6 ) SA (k5 , 5 ) =
SM (k5 k6 , 5 + 6 );
nm
bp
pn
cm
(b) daca punem q1 ei1 =
, q2 ei2 =
, q3 ei3 =
, q4 ei4 =
,
am
np
bn
pm
mp
an
q5 ei5 =
, q6 ei6 =
, atunci SP (q2 , 2 )SM (q1 , 1 ) = SC (q1 q2 , 1 + 2 ),
cp
mn
SM (q4 , 4 )SN (q3 , 3 ) = SA (q3 q4 , 3 + 4 ), SN (q6 , 6 )SP (q5 , 5 ) = SB (q5 q6 , 5 + 6 ).
Demonstratie. ncepem prin a demonstra o forma echivalenta a formulei (3).
Lem
a. Afixul punctului P , notat cu p, care este centrul compunerii similitudinilor SB (k2 , 2 ) SC (k1 , 1 ), verifica relatia
np
ac nb
=

(#)
mp
mc ab
si reciproc, daca afixul p verifica (#), atunci verifica si (3).
Demonstratie. Din ipoteza rezulta ca SP (k1 k2 , 1 + 2 ) (M ) = N , de unde,
conform formulei (10 ), rezult
a (#). S
a presupunem c
a afixul punctului P verific
a (#).
ac nb
i(1 +2 )
=
Daca notam cu w = k1 k2 e

, atunci putem scrie (1 w) p =


mc ab
ac
(n b) =
n wm, de unde (p c) (1 w) = n wm (1 w) c = n c
ab
8

na
bn ac + ab + cn
= (b c)
, de unde (3).
ab
ba

Revenim la demonstratia teoremei. Pentru punctul (a), trebuie demonstrat ca,


dac
a p satisface (3), atunci m si n verific
a formulele analoge, ceea ce, n virtutea
lemei, revine la demonstrarea formulelor
mn
ma cb
si
=

(#0 )
pn
ca pb
pm
ba pc
=

.
(#00 )
nm
na bc
nb ma

pb
mn
1w
Din (3) se obtine
= ab mc si din (#) deducem
=
. Mai
cb
1w
pn
w
nb ma mc ab
ma
pb mn

=
, de unde rezult
a
departe avem
cb pn
ab mc ca nb
ca
0
00
(# ). Analog se demonstreaza si (# ). Pentru punctul (b), trebuie demonstrat ca,
dac
a p satisface (3), atunci a, b, si c satisfac relatiile a = n + (m n)

cp
mp
pn cm
bn pm

1
ca
pn cm
si celelalte, ceea ce este echivalent cu a demonstra relatiile
si
=

ba
bn pm
0
00
celelalte, care nu sunt altceva dect rescrieri ale relatiilor (#), (# ) si (# ). Q. e. d.
2. Punctele Torricelli asociate compunerii a dou
a similitudini. n
continuare, proced
am dup
a cum urmeaz
a: consider
am punctele A0 , B 0 si C 0 , de
afixe a0 , b0 si respectiv c0 , definite de A0 = SC (k1 , 1 ) (P ), B 0 = SA (k5 , 5 ) (M ),
C 0 = SB (k3 , 3 ) (N ).
Din Teorema 1 deducem si ca A0 = SB (1/k2 , 2 ) (P ), B 0 = SC (1/k6 , 6 ) (M ),
0
C = SA (1/k4 , 4 ) (P ). Trecnd la nivelul afixelor, relatiile de mai sus se traduc

n formulele
ac
ab

a0 = c + (p c)
= b + (p b)
,

mc
nb

ba
bc
b0 = a + (m a)
= c + (m c)
,
(4)
n

a
p
c

cb
ca

c0 = b + (n b)
= a + (n a)
.
pb
ma
Dac
a invers
am rolurile tripletelor {A, B, C} si {P, M, N } (Teorema 1 ne permite
acest lucru), putem considera analogele punctelor A0 , B 0 si C 0 , anume P 0 , M 0 si
N 0 , de afixe p0 , m0 si respectiv n0 , unde P 0 = SN (q3 , 3 ) (A), N 0 = SM (q1 , 1 ) (C),
M 0 = SP (q5 , 5 ) (B), analogele formulelor (4) fiind

pn
pm

= m + (a m)
,
p0 = n + (a n)

n
cm

p
m

n
= n + (b n)
,
m0 = p + (b p)
(5)
cp
an

m
n

n0 = m + (c m)
= p + (c p)
.
am
bp
Propozitia 2. P P 0 AA0 , M M 0 BB 0 , N N 0 CC 0 sunt paralelograme, eventual degenerate.
Demonstratie. Demonstram relatiile importante:
9

a a0 = (p p0 ) ,

b b0 = (m m0 ) ,

c c0 = (n n0 ) .

(6)
0

Rationamentul se urmareste usor n cele ce urmeaza: din (5) se obtine p p =


(b a) (n p)
(n p) (b a)
si din (4) se obtine a0 a =
; de aici rezulta ca
bn
nb
0
0
a
a a = (p p ); celelalte relatii se deduc n acelasi fel. Conchidem imediat c
perechile de segmente {[AA0 ] , [P P 0 ]}, {[BB 0 ] , [M M 0 ]} si {[CC 0 ] , [N N 0 ]} sunt respectiv congruente, paralele sau confundate, ceea ce ncheie demonstratia.
Propozitia 3. Sunt adevarate urmatoarele relatii:
c0 b
cb
c b0
,
(7)
= 0
=
ab
a b
a b0
pn
p n0
p0 n
= 0
=
;
(8)
mn
m n0
m n
c a0
m n0
b c0
p m0
a b0
n p0
=
,
=
,
=
.
(9)
0
0
0
0
0
mp
ba
pn
ac
nm
c b0
Demonstratie. Relatiile (4) se mai scriu si
c0 a
na
b a a0 b
pb
c b b0 c
mc
ac
=
= 0
;
=
= 0
;
=
= 0
. (40 )
ca
ma
b a a b
nb
c b b c
pc
a c
c0 b
cb
Din a doua relatie se obtine
= 0
. Analog se obtine si egalitatea cu celalalt
ab
a b
raport din (7). Inversnd rolurile triunghiurilor ABC si M N P , rezulta si (8). Remarcam si relatiile
p0 n
an
m n m0 p
bp
n p n0 m
cm
pm
=
= 0
,
=
= 0
,
=
= 0
, (50 )
pn
bn
m n m p
cp
n p n m
am
p m
care sunt echivalentele relatiilor (40 ). Relatiile (9) se deduc astfel: din (4) si din
(p c) (a c) (n b)
p0 n
a0 c
=
, respectiv 0
=
(5), coroborat cu (#), obtinem 0
a b
(p b) (a b) (m c)
p m
(p c) (n b)
(a c) (a n)
an
a c
a
. nmultind (#), (#0 ) si (#00 ), rezult
=
,
(a b) (a m)
(p b) (m c)
am
ceea ce, dup
a nlocuire, ncheie demonstratia.
Teorema 2. Consideram punctele A0 , B 0 , C 0 , de afixe a0 , b0 si respectiv c0 ,
definite de relatiile (4) si punctele P 0 , M 0 , N 0 , de afixe p0 , m0 si respectiv n0 , care
satisfac relatiile (5).
b a ma
(a) Daca

R, atunci tripletele {AA0 , BB 0 , CC 0 } si {M M 0 , N N 0 , P P 0 }


na ca
sunt formate din drepte paralele.
b a ma
(b) Daca

/ R, atunci tripletele {AA0 , BB 0 , CC 0 } si {M M 0 , N N 0 , P P 0 }


na ca
sunt concurente.
Demonstratie. n virtutea Teoremei 1, sunt suficiente demonstratiile afirmatiilor referitoare la tripletul {AA0 , BB 0 , CC 0 }. Demonstram ca AA0 k BB 0 k CC 0 . Din
c a0
c0 a
na ca
=
(4) se obtine
=

R, adica C 0 AB. Din (7) deducem


ba
b a ma
b a0
0
c a
ca
a parte, din (40 ) rezult
a
=
R, adic
a A0 BC si B 0 CA. Pe de alt
b0 a
ba
10

b b0
ba
b b0
a b0
c c0
=
=
R, deci
=
, ceea ce nseamn
a c
a dreptele
0
0
0
cc
ca
ac
bb
c c0
a BB 0 CC 0 , atunci {B 0 } =
BB 0 si CC 0 sau sunt paralele sau confundate. Dac
AC CC 0 = {C}, ceea ce, n virtutea lui (4), ar conduce la M C sau B C, ceea
ce este fals. Deci BB 0 k CC 0 , la fel demonstrdu-se si celalalt paralelism.
c
a

B
A

M
N

T
T
C

B
P

A
Figura 1
ba ma
Demonstr
am punctul (b); dac
a

C\R, rezult
a c
a dreptele AA0 , BB 0
na ca
si CC 0 se intersecteaz
a cel putin dou
a cte dou
a. Fie {T } = BB 0 CC 0 si fie t afixul
s
au. Va rezulta c
a exist
a , R astfel nct b b0 = (b0 t), c c0 = (c t) si,
0
b0 a b0 t
b a
b b0

tinnd cont si de
= 0
, dedusa din (40 ), ajungem la
:
=
ca
c c
ca ct

a c
a patrulaterul B 0 CT A este inscriptibil.
R; cum {C 0 , A, B} nu sunt coliniare, rezult
a0 c
ca
Analog dovedim ca BC 0 AT este inscriptibil. Din (7) rezulta ca 0
= 0
;
a b
b a
b0 a c t
a c
a
deoarece B 0 CT A este inscriptibil si deoarece T BB 0 , rezult

R si
c a b0 t
0
0
a c tc
b t
R, deci 0
:
R, adica si BA0 CT este inscriptibil. Demonstram ca
tb
a b tb
t c ba0
T AA0 ; conditiile de inscriptibilitate ale patrulaterelor se pot scrie si

ta0 b c
11

t c b0 a
b0 c
bc
=
0
R si tinnd cont de
, dedus
a din (7), rezult
a c
a
t a b c
ba0
b0 a
0
0
ta
ta b c b a t c

R. Rezulta ca AA0 , BB 0 si CC 0 sunt

R, adica
t c ba0 b0 c t a
ta0
concurente. Inversnd rolurile tripletelor {AA0 , BB 0 , CC 0 } si {M M 0 , N N 0 , P P 0 },
obtinem si concurenta dreptelor P P 0 , M M 0 si CC 0 . Demonstratia este ncheiat
a.
R,

Observatia 1. La punctul (a) nu obtinem AA0 k BB 0 k CC 0 k P P 0 k M M 0 k N N 0 ,


cum s-ar parea ca rezulta din Propozitia 2, deoarece n exemplul a = 0, b = 1, c = i,
1
1
m = 1, n = i, p = 0, unde a0 = (1 + i), b0 = n = i, c0 = m = 1, p0 = (1 + i),
2
2
avem AA0 P P 0 , BB 0 M M 0 , CC 0 N N 0 .
ba ma

R, care se refer
a numai la
na ca
\
\ ) {0, }, adic
pozitia punctelor din ipotez
a, se traduce prin m(N
AB) + m(CAM
a
\
\

unghiurile N AB si CAM sunt sau opuse ca orientare si egale n valoare absoluta sau
suplementare.
Observatia 2. Geometric, conditia

Observatia 3. Cele spuse se urm


aresc usor pe figura 1, corespunz
atoare cazului
1 > 0, 2 > 0. Se remarca paralelogramele din Propozitia 2 si urmatoarele siruri de
triunghiuri direct asemenea, n ordinea n care sunt scrise, care se deduc din interpret
arile geometrice ale formulelor (7), (8) si (9): 4C 0 BC 4N BP 4ABA0
4N AP 0 , 4A0 CA 4P CM 4BCB 0 4P 0 AM , 4B 0 AB 4M AN 4CAC 0
4M CN 0 si 4C 0 BA 4CBA0 4CB 0 A 4N 0 M P 4N M P 0 4N M 0 P .
Tot de aici se deduce faptul c
a, dac
a, de exemplu, M este un punct important n
4ACB 0 , atunci si N si P sunt acelasi tip de punct n triunghiurile analoge.
Observatia 4. Punctele T si T 0 , obtinute la punctul (b), sunt puncte Torricelli
generalizate asociate tripletelor {A, B, C} si {P, M, N }. n adevar, daca punem, de
exemplu, = |b c|, = |c a0 |, = |a0 b| (sau orice numere proportionale
cu ele), atunci punctul T este punctul n care suma |XA| + |XB| + |XC| si
atinge minimul, unde X este un punct oarecare din plan. Analog, punctul T 0 este
punctul n care suma |XP | + |XM | + |XN | si atinge minimul. Pentru detalii,
recomand
am cititorului paragraful 1.49 din [3], unde demonstratiile sunt prezentate
pe larg si unde sunt expuse si alte propriet
ati ale punctelor Torricelli.
n continuare, ne plasam n conditiile punctului (b) din Teorema 2.
Teorema 3. T T 0 daca si numai daca punctele A, B, C, M , N , P sunt
conciclice.
Demonstratie. Daca T T 0 , atunci relatia (6) asigura coincidenta dreptelor
AA0 P P 0 , respectiv BB 0 M M 0 si CC 0 N N 0 , adic
a {C 0 , N, T, C, N 0 },
0
0
0
0
{P , A, T, P, A }, {M , B, T, M, B } sunt puncte coliniare, deci exist
a , R astfel
ca c0 t = (c0 n), a t = (a p).
a t c0 b
Deoarece patrulaterul AC 0 BT este inscriptibil, rezulta ca

R, de
a b c0 t
0
ap c b

unde, dup
a nlocuire, se obtine

R. Tinnd
cont de relatia a treia
a b c0 n
12

P
C
N

A
M

T
B
M

Figura 2
c0 b
cb
ap cb
din (4), putem scrie 0
=
, care conduce la

R, deci patruc n
cp
ab cp
laterul ABP C este inscriptibil. Analog se demonstreaza si ca M , N apartin cercului
circumscris triunghiului ABC. Reciproc, s
a presupunem c
a punctele A, B, C, M ,
a punctul P apartine cercului cirN , sunt conciclice si demonstr
am c
a T T 0 si c
ab
b0 b
=
; dup
a nlocuirea
cumscris triunghiului ABC. Din (4) deducem c
a
nm
na
ab mn
cu

R, asigurat
a de patrulaterul inscriptibil AN BM , rezult
a c
a
na mb
0
0
b b
b b mn

R, deci
R, adica M BB 0 ; folosind celalalt patrulater innm mb
mb
a
scriptibil, se arat
a si c
a N CC 0 ; de aici BB 0 M M 0 , CC 0 N N 0 , de unde rezult
a0 a
a0 a p m
0
0
a c
a
c
a T T . Deoarece P AA , rezult
R, adic
a

R, ceea
pa
pm pa
a0 a
ca
ca pm
ce, coroborat cu
=
, dedus
a din (4), conduce la

R,
pm
cm
cm pa
deci patrulaterul AM CP este inscriptibil. Q. e. d.
De interes credem c
a este o interpretare fizic
a a teoremelor 2 si 3 (v. [3], pag. 142).
Bibliografie
1. C.Ionescu-Bujor - Elemente de transformari geometrice, vol. I-IV Bibl. Soc. S
t.
Matematice a R.S.R., Ed. Tehnica, Bucuresti, 1958.
2. N. Mih
aileanu - Utilizarea numerelor complexe n geometrie, Bibl. Soc. S
t. Matematice a R.S.R., Ed. Tehnic
a, Bucuresti, 1968.
3. L. Nicolaescu, V. Boskoff - Probleme practice de geometrie, Seria "Culegeri de
matematic
a si fizic
a", Ed. Tehnic
a, Bucuresti, 1990.
4. D. Smaranda, N. Soare - Transformari geometrice, Bibl. profesorului de matematica, Ed. Acad. R.S.R., 1988.
aeru - Asupra unei clase de transformari geometrice, Matematica n scoala
5. C. Tig
suceveana, nr. 8/1990, 1-7.
13

Ordinul elementelor grupului GLn (Z)


Adrian REISNER1
n RecMat 2/2005, Gabriel Dospinescu a propus problema
L94. Fie
A Mn (C) o matrice cu elemente ntregi, inversabila si astfel nct
multimea Ak | k N este finita. Sa se demonstreze ca aceasta multime are cel mult
2
3n elemente. Ramne rezultatul adevarat daca suprimam conditia ca elementele
matricei sa fie ntregi?
Solutia autorului a fost publicat
a n num
arul 2/2006 al revistei. Propunem n
continuare o abordare oarecum diferit
a a problemei, care permite o mai bun
a privire
asupra laticei subgrupurilor lui GLn (Z).
Not
am cu GLn (Z) multimea matricelor inversabile cu elemente ntregi, pentru
care inversa are tot elemente ntregi. Evident ca M GLn (Z) daca si numai daca
M Mn (Z) si det M = 1. Daca G este un subgrup finit al grupului GLn (Z), iar
p 3 este prim, consider
am aplicatia : G GLn (Z), care asociaz
a unei matrice
M G acea matrice care are ca elemente redusele modulo p ale elementelor lui M .
Propozitie. Aplicatia este un monomorfism de grupuri.
Demonstratie. Se observa usor ca este bine definita, n sensul ca (M ) este
matrice inversabila n Mn (Zp ), precum si faptul ca este morfism de grupuri. Vom
ar
ata c
a este injectiv
a demonstrnd c
a nucleul s
au Ker este {In }. Fie M Ker ,
deci M In (mod p); exist
a atunci o matrice N Mn (Z) astfel nct M = In +pN .
Grupul G fiind finit, rezulta ca M este de ordin finit m 1: M m = In . Din aceasta
egalitate deducem ca polinomul P = (1 + pX)m 1 este polinom anulator pentru
matricea N . Dac
a 0 , 1 , . . . , m1 sunt r
ad
acinile de ordin m ale unit
atii, r
ad
acinile

1
lui P vor fi j =
1 , j = 0, m 1 si acestea sunt numere distincte; rezulta
p j
c
a N este diagonalizabil
a. n plus, cum p 3, avem c
a |j | < 1, prin urmare
a k suficient de mare
lim N k = On . Matricea N fiind cu elemente ntregi, exist

pentru care N k = On , deci N este nilpotenta. O matrice diagonalizabila si nilpotenta


este nul
a si deducem c
a M = In + pOn = In , ceea ce ncheie demonstratia.
Consecinta 1. Ordinul oricarui subgrup finit al grupului GLn (Z) este majorat
2
de 3n , iar GLn (Z) contine un numar finit de subgrupuri finite neizomorfe.
Demonstratie. Cardinalul unui subgrup G este majorat, conform propozitiei
2
precedente, de cardinalul lui GLn (Z3 ), care este 3n (deoarece aplicatia este injectiva). A doua afirmatie rezulta imediat, deoarece GLn (Z3 ) are un numar finit de
subgrupuri neizomorfe, fiind el nsusi grup finit.
Consecinta 2. Ordinul oricarui element al grupului GLn (Z) este sau infinit,
2
sau majorat de 3n .
1

Cercet
ator, Centrul de Calcul E.N.S.T., Paris

14

Demonstratie. Concluzia rezult


a din faptul c
a ordinul oric
arui element al subgrupului divide ordinul grupului.
Din Consecinta 2 urmeaza imediat prima parte a problemei L94.
Proprietatea nu mai are loc daca A nu are elemente ntregi. De exemplu, un
subgrup G GL2 (R) care este de torsiune (x G, q N cu xq = I2 ) poate s
a
fie infinit; acest fapt aduce o mbun
at
atire solutiei problemei L94 dat
a n nr. 2/2006,
unde se considera un contraexemplu bazat pe o matrice cu elemente complexe nereale.
Fie G subgrupul lui GL2 (R) format din matricele de forma

cos sin
R () =
, {2r | r Q} .
sin cos
p
Daca = , cu p, q Z, q 1, evident ca
q

cos 2p sin 2p
q
= I2 ,
[R ()] =
sin 2p cos 2p
deci G este grup de torsiune. Pe de alta parte, G este de ordin infinit, izomorf cu
grupul (Q/Z, +); izomorfismul asociaz
a matricei R (), cu = 2r, clasa din Q/Z a
p
artii fractionare a lui r (verific
arile se fac usor).

Puncte coliniare
Ora de matematic
a la o clas
a cu profil sportiv. Un elev de cl. a IX-a are de
rezolvat la tabla o problema simpla de coliniaritate, dar . . .
Ce sunt punctele coliniare? intervine profesorul, decis sa-l ajute.
!?!??
Bine! S-o lu
am altfel . . . Tu esti fotbalist. Ce nseamn
a coechiper?
Din aceeasi echipa, de ce rdeti de mine d-le profesor?
Atunci, ce-ar putea sa nsemne coliniar?
Din echipa advers
a!. . .

Vreti s
a v
a ghicesc num
arul ales?
Alegeti un numar de doua cifre pe care-l doriti. nmultiti prima cifra cu 5, adunati
3, dublati rezultatul, adaugati a doua cifra si spuneti-mi rezultatul. Va voi indica pe
loc num
arul ales.
(Explicatii gasiti la pagina 28.)
15

Variatiuni pe tema dreptei lui Euler


si cercului celor nou
a puncte
Temistocle BRSAN 1
Dou
a dintre cele mai cunoscute "vedete" ale geometrie triunghiului sunt dreapta
lui Euler si cercul celor noua puncte (cercul lui Euler ) (fig. 1). Vom adopta notatiile
uzuale. Fie 4ABC, dreapta lui Euler (determinata de H si O) si E cercul celor
nou
a puncte (determinat de mijloacele laturilor A0 , B 0 , C 0 ). Sunt binecunoscute
urm
atoarele propriet
ati ale acestei configuratii:

1 G, O9 (centrul cercului E) sunt pe 4;


2 E contine picioarele n
altimilor D, E, F si mijloacele segmentelor [AH], [BH], [CH], adica punctele
A00 , B 00 , C 00 ;

C
B
F
GO
H O9

3 HO9 = OO9 , HG = 2 OG;


4 punctele A0 si A00 sunt diametrul opuse n E si
A0 O = AA00 = A00 H;

5 AO k A0 A00 si AO = A0 A00 ;
1/2

1/2

Fig. 1

6 cercurile C (circumscris 4ABC) si E sunt omotetice prin omotetiile hH si hG

[1]; daca C are raza R, atunci E are raza

R
.
2

Urmatorul rezultat, sugerat de configuratia de mai sus, reprezinta o generalizare


am un punct oarecare):
a propriet
atii 4 (n loc de H consider
Propozitia 1. Fie 4ABC si P un punct oarecare. Aratati ca dreptele ce unesc
mijloacele A00 , B 00 si C 00 ale segmentelor ceviene [AP ], [BP ] si respectiv [CP ] cu
mijloacele A0 , B 0 , C 0 ale laturilor opuse sunt concurente (fig. 2).
Demonstratie. Patrulaterul B 0 C 0 B 00 C 00 este paralelogram, c
aci B 0 C 0 si B 00 C 00 sunt paralele cu BC si egale
A
BC
0 00
0 00
cu
. Ca urmare, [B B ] si [C C ] se intersecteaza n
2
A
0
P aflat la jum
atatea fiec
aruia. La fel se arat
a c
a A0 A00

C
B
si una dintre B 0 B 00 , C 0 C 00 se intersecteaz
a n P 0 .
P

Observatii. 1) Pentru a obtine punctul P 0 este suficient s


a lu
am o singur
a cevian
a; dac
a aceasta este AP ,
atunci P 0 este mijlocul segmentului [A0 A00 ].
2) Distingem trei cercuri cu centrul n punctul P 0 si
de raze P 0 A0 , P 0 B 0 , P 0 C 0 ce apar n locul cercului celor
nou
a puncte.
1

Prof. dr., Catedra de matematic


a, Univ. Tehnic
a "Gh. Asachi", Iasi

16

C
A
Fig. 2

Propozitia 2. Daca P este pe , atunci P 0 , obtinut din P ca n Propozitia 1,


este de asemenea pe (fig. 3).
Demonstratie. Fie A00 mijlocul segmentului eulerian [AH] si A000 mijlocul segmentului cevian [AP ]; deci
A00 A000 k . Not
am cu Q intersectia paralelei prin A0 la
ceviana AP . Din acest fapt si din proprietatile A0 O k
AA00 si A0 O = AA00 , rezulta ca 4A0 OQ, 4AA00 A000 sunt
congruente (ULU), deci A0 Q = AA000 = A000 P . Patrulaterul P A0 QA000 este paralelogram si P 0 este mijlocul
segmentului [P Q]. Cum P, Q , urmeaza ca P 0 .
Revenind la Propozitia 1, vom impune punctului P
conditii suplimentare, care s
a-l apropie de H.

A
A

P Q
H P O
B

Fig. 3

Propozitia 3. Fie P un punct n planul 4ABC. Punctele B 0 , C 0 , B 00 , C 00


(fig.2) sunt conciclice daca si numai daca P AH.

Demonstratie. Am observat deja ca B 0 C 0 B 00 C 00 este paralelogram. Daca B 0 ,


C , B 00 , C 00 sunt conciclice, atunci B 0 C 0 B 00 C 00 va fi dreptunghi, deci B 00 C 0 BC.
Dar B 00 C 0 k AP (n 4P AB). Deci AP BC, adic
a P AH. Implicatia reciproc
a
se dovedeste pe cale invers
a.
0

Propozitia 4. Fie P n planul 4ABC. Daca punctele B 0 , C 0 , B 00 , C 00 si A0


(sau A00 ) (fig.2) sunt conciclice, atunci P coincide cu H.
Demonstratie. Cercul pe care se afl
a punctele are centrul n P 0 . Ambele
0
00
0 0
punctele A si A vor fi pe cerc, c
aci P A = P 0 A00 si unul din ele, prin ipotez
a,
este pe cerc. Conform Propozitiei 3, aplicat
a de trei ori, avem P AH, P BH si
P CH, adica P si H coincid.

Propozitia 5. Daca punctul P verifica conditia AP BC si punctele A0 , B 0 ,


C , A00 (fig.2) sunt conciclice, atunci P este ortocentrul H al 4ABC.
0

altimii
Demonstratie. Punctul A1 definit prin {A1 } = AP BC este piciorul n
duse din A. Ca urmare, A1 E si 4A1 A0 A00 dreptunghic n A1 este nscris n E; deci,
A0 A00 este un diametru n E. n consecinta, mijlocul lui A0 A00 , care este este P 0 , va fi
centrul cercului E. Din faptul c
a B 0 , C 0 E, rezult
a c
a punctele diametral opuse lor
00
vor fi pe acest cerc, adic
a B , C 00 E. Se poate aplica Propozitia 4, conform c
areia
P este punctul H.
Propozitia 6. Daca punctul P verifica conditia AP BC si punctele A00 , B 00 ,
C , A0 (fig.2) sunt conciclice, atunci P este ortocentrul H.
Demonstratie. Simetricele punctelor A00 , B 00 , C 00 si A0 fata de P 0 sunt de
asemenea conciclice; asadar, A0 , B 0 , C 0 si A00 sunt conciclice. Ipotezele Propozitiei 5
fiind ndeplinite, rezulta ca P coincide cu H.
00

Observatii. 1) Propozitiile 4, 5 si 6 pot fi privite ca reciproce ale binecunoscutei


afirmatii: daca H este ortocentrul unui triunghi, atunci mijloacele laturilor sale,
mijloacele segmentelor euleriene si picioarele naltimilor sunt conciclice (cercul lui
Euler).
17

2) Remarc
am c
a n enuntul Propozitiei 4 este absent
a conditia AP BC. Faptul
este doar aparent, caci n ipotezele acesteia, rezulta ca mentionata conditie are loc
(conform Propozitiei 3).
Revenind din nou la Propozitia 1, s
a examin
am rezultatul acesteia din alt punct
de vedere. Notam cu transformarea geometrica (a planului 4ABC) care pune
punctul P n corespondenta cu P 0 , punct construit ca n Propozitia 1 (a se vedea si

Observatia ce-i urmeaza, punct 1) ); deci P P 0 sau (P ) = P 0 .


Vom indica cteva propriet
ati ale transformatei si o vom compara cu omotetiile
1/2
1/2
hH si hG .
Iata cteva proprietati ale lui , care decurg direct din definitiile lui sau au fost
stabilite mai sus:

1 G G, H O9 ;

2 (A) = mijlocul medianei [AA0 ] (analog, (B), (C));


3 (4) = 4, adic
a dreapta lui Euler este transformat
a n ea ns
asi (conform
Propozitiei 2).
Propozitia 7. Sunt adevarate afirmatiile:
a) (O) = U , unde U este mijlocul segmentului [O9 O];
b) (O9 ) = V , unde V este mijlocul segmentului [O9 U ], adica O9 V =
Demonstratie. a) Afirmatia rezulta din faptul ca
patrulaterul OA0 O9 L (fig.4) este paralelogram (O9 L ca
linie mijlocie n 4AHO este paralel
a cu AH si egal
a cu
AH
a cum am amintit la nceput, are de
, iar OA0 , dup
2
asemenea aceste dou
a prpriet
ati).
b) Argument similar: O9 A0 U K este paralelogram
OA
(A0 O9 si U K sunt paralele cu OA si egale cu
).
2

1
O9 O.
4

AK L
V
H O9 U O
B

Fig. 4
1/4
Propozitia 8. Are loc egalitatea = hG , adica este omotetia de centru G si
1
raport .
4
Demonstratie. Faptul c
a imaginea P 0 a punctului P se obtine construind mai
nti mijlocul A00 al segmentului [AP ] si apoi P 0 ca mijlocal segmentului
[A0 A00 ] se

1/2
1/2
1/2
1/2
1/2
1/2
1/2
scrie: = hA0 hA (ntr-adevar, hA0 hA (P ) = hA0 hA (P ) = hA0 (A00 ) =
a omotetii, va fi tot o omotetie, cu centrul coliniar cu
P 0 ). Ca produs de dou
1 1
centrele omotetiilor factor si de raport ([1, p.81], [4, p.85]). Sa notam cu T
2 2
centrul omotetiei , T AA0 . Pozitia punctului T pe AA0 poate fi aflata cu ajutorul
unor formule prezente n locurile citate mai nainte; prefer
am s
a o determin
am direct.
Avem:
1/2
1/2
1/2
1/2
(T ) = T hA0 hA (T ) = T hA0 (S) = T , unde S = hA (T )

0

1
1
AT =
A0 S si AS = AT AT = 2AS = 2(AA0 + A0 S) =
2
2
18

= 2(AA0 + 2A0 T ) = 2(AT + A0 T ) AT = 2A0 T T coincide cu G,


ceea ce ncheie demonstratia.
Speculnd faptul c
a omotetiile transform
a dreptele ce trec prin centru n ele nsele
si cercurile n cercuri ([1], [4]), putem completa lista proprietatilor lui de mai sus:
4 dreptele suport ale medianelor 4ABC sunt invariante la ;

5 dreapta lui Nagel IN (N este punctul lui Nagel ) este invariant


a, c
aci G IN ;
avem (I) = I 0 , cu I 0 IN , 4 GI 0 = GI si (N ) = S, unde S este punctul lui
Spiecker centrul cercului nscris n triunghiul median 4A0 B 0 C 0 [2, pp.90 si 233].
Observatie. Propozitia 2, c
areia i-am dat o demonstratie direct
a, decurge si din
faptul ca 4 trece prin G si este omotetie.
ncheiem cu o problem
a care poate p
area dificil
a, dar care este usor de rezolvat
n contextul nostru.
Problem
a. Cercul determinat de mijloacele medianelor 4ABC are centrul pe
R
dreapta lui Euler n mijlocul segmentului [OO9 ] si raza .
4
Solutie. Observ
am c
a acest cerc este imaginea prin a cercului circumscris
4ABC (proprietatea 2) ) si apoi utiliz
am Propozitia 7, a).
Bibliografie
1. D. Brnzei, S. Anita, C. Cocea - Planul si spatiul euclidian, Biblioteca profesorului de matematica, Ed. Academiei, Bucuresti, 1986.
2. D. Brnzei, S. Anita, M. Chirciu - Geometrie. Clasa a IX-a, Colectia Mate-2000,
ed. a III-a, Paralela 45, Pitesti, 1998.
3. T. Lalescu - Geometria triunghiului, Ed. Tineretului, Bucuresti, 1958.
4. D. Smaranda, N. Soare - Transformari geometrice, Biblioteca profesional
a de
matematic
a, Ed. Academiei, Bucuresti, 1988.

Vizitati pe Internet revista "Recreatii Matematice" la adresa

http://www.recreatiimatematice.uv.ro

19

New Proof for an Old Inequality


Marian TETIVA1
The inequality

a2 + b2 + c2 8R2 + 4r2
is well-known to be the sharpest from all the inequalities of the form (see [2])
a2 + b2 + c2 kR2 + hr2 ;
also its a known fact (and it is easy to obtain) that it can be stated in the equivalent
form
(1 cos A)(1 cos B)(1 cos C) cos A cos B cos C.
Many beautiful proofs for this remarkable inequality are available ([1], [2]). Our
intention is to present in the sequel a new proof of this trigonometric form of the
inequality ("new" as far as we know it) and also to derive some related inequalities.
Of course, the case of the right-angled or of the obtuse-angled triangle is immediately obtained, as long as the left-hand side is always positive and the right-hand
side is less than (or equal to) zero in those cases. Therefore we may assume, without mistaking, the triangle to be acute-angled, hence the cosines of its angles to be
positive; then we can also put the inequality as

1
1
1
1
1
1 1.
cos A
cos B
cos C
Let us denote x = cos A, y = cos B, z = cos C and the above parentheses with u, v, w:
1
1
1
1
1
1
, y=
, z=
.
u = 1, v = 1, w = 1 x =
x
y
z
u+1
v+1
w+1
Now observe that
cos A = cos(B + C) = cos B cos C + sin B sin C
p
x + yz = (1 y 2 )(1 z 2 ),

whence squaring yields

x2 + y 2 + z 2 + 2xyz = 1,
or, with the u, v, w - notations
1
1
2
1
+
+
+
= 1.
(u + 1)2
(v + 1)2
(w + 1)2 (u + 1)(v + 1)(w + 1)
Thus we are left with the algebraic inequality uvw 1, for positive u, v, w satisfying
the previous condition, or
Y
Y
X
(u + 1)2
(u + 1)2 (v + 1)2 + 2 (u + 1) =
(the sum and the products are cyclic). Further we introduce the notations
S = u + v + w,

Q = uv + uw + vw,

P = uvw;

some simple (but boring) calculations transform the above condition in


S 2 + 4S + 4 = P 2 + 2P Q + 4P S + 6P
and we want to get from here the inequality P 1.
1

Professor, National College "Gheorghe Rosca Codreanu", Brlad

20

()

Indeed, suppose that it is not so, hence P < 1; one gets


S 2 + 4S + 4 < 1 + 2Q + 4S + 6 S 2 < 2Q + 3.

But Q S 2 /3 is a classic inequality which, together with the previous one, implies
S < 3 and this is not possible, because using () and the inequalities P S 3 /27 and
Q S 2 /3, we infer
t6 + 6t5 + 12t4 + 6t3 9t2 12t 4 0,

for t = S/3. This last inequality may be rewritten as

(t 1)(t5 + 7t4 + 19t3 + 25t2 + 16t + 4) 0,

and it shows that t 1 S 3. Thus the inequality S < 3 obtained from our
supposition that P < 1 is contradictory, hence P 1 and the proof is done.
How to
obtain new inequalities?
For example, with the AM-GM inequality we

3
have S 3 3 P and Q 3 P 2 ; by P 1 these yield S 3 and Q 3, therefore
S 2 + 4S + 4 = P 2 + 2P Q + 4P S + 6P P 2 + 6P + 12P + 6P = P 2 + 24P.

Again by P 1 one gets

S 2 + 4S + 4 25P (S + 2)2 25P S 5 P 2;

now remember who S and P are and transform this into


r

X 1
Y
1
5
1 + 1,
cos A
cos A
which we think that is not an easy to get inequality.

A slight modification of the above calculations leads us to another inequality


between S and P , namely
Y 1

X 1
S P +2
1
1 + 2;
cos A
cos A
a short computation transforms this into another (known, we believe) inequality
involving the cosines of an acute-angled triangle:
X
Y
cos A 1 + 4
cos A.

We suggest to the reader to proceed for improving this last inequality: one can
get S 2P + 1 and S 3P , inequalities that can also be put in the form
X
X
Y
2
cos A 2 +
cos A cos B + 2
cos A,

respectively

cos A 3 + 2

cos A cos B.

Or, and we are very sure of it, the reader may find his/hers own way to get other
(beautiful and hard enough, we think) inequalities with this method.
Bibliografie
1. D. Grinberg, M. Lascu, M. Pachitariu, M. Tetiva - Din nou despre inegalitati
geometrice, G. M. 6/2006.
2. L. Panaitopol - O inegalitate geometrica, G. M. 4/1982.
21

Asupra calcul
arii unor limite de siruri
1

D. M. BATINE
TU-GIURGIU

Oricarui sir (an )n1 de numere reale strict pozitive i vom asocia sirurile (an !)n1

si n an ! n1 , unde a1 ! = a1 , 1 a1 ! = a1 , an+1 ! = an ! an+1 , n N . Pentru orice


sir (xn )n1 de numere reale, vom nota xn = xn+1 xn , n N .
Vom considera multimile de siruri:
o
n
S R+ = (xn )n1 | xn R+ , n N ,
o
n

D R+ = (xn )n1 S R+ | lim xn = x R+ ,
n
o
n

xn
B R+ = (xn )n1 S R+ | lim
= x R+ ,
n n
pentru care evidentiem cteva propriet
ati.


P1 . Oricare ar fi (xn )n1 D R+ , atunci (xn )n1 B R+ .

Demonstratie. Fie (xn )n1 D R+ , deci lim xn = x R+ . Ca urmare,
n


xn
xn+1 xn
lim
= lim
= x. Deci D R+ B R+ .
n n
n (n + 1) n
(1)n
Aceasta incluziune este stricta: sirul (xn )n1 dat de xn = n +
este n
3


B R+ , dar nu si n D R+ .

xn+1
P2 . Oricare ar fi (xn )n1 B R+ , atunci lim
= 1.
n xn

xn+1 n n + 1
1
xn+1
= lim

= x = 1.
Demonstratie. ntr-adevar, lim
n xn
n n + 1 xn
n
x

n
xn ! n1 B R+ .
P3 . Oricare ar fi (xn )n1 B R+ , atunci
Demonstratie. Conform criteriului Cauchy-d Alembert, avem:

!
r

n
xn !
nn
xn+1 !
n xn !
= lim
lim
= lim
=
n+1 x !
n
n
n
n
nn
n
(n + 1)

n
xn+1
x
n
= R+ .
= lim
n n + 1
n+1
e



P4 . Oricare ar fi (xn )n1 B R+ , atunci n xn ! n1 D R+ .

n

xn ! x
Demonstratie. n P3 am aratat ca, daca (xn )n1 B R+ , atunci lim
= ;
n n
e
ca urmare,
p
!
p
n+1
n+1
xn+1 !
xn+1 !
n+1
n
x e

lim un = lim
= lim


= 1 = 1,
n
n
n
n
n
n+1
n
e x
xn !
xn !

Profesor, Colegiul National "Matei Basarab", Bucuresti

22

lim

lim (un ) = lim

n
!n
p
n+1
xn+1 !

n
xn !

un 1
= 1,
ln un

xn+1
= lim

n
n+1

n+1
p
n+1
xn+1 !

=x

e
= e.
x

Rezult
a atunci c
a

p
n
p
xn ! un 1
x
n
n
n
n+1
xn+1 ! xn ! = lim
ln (un )
= ,
lim xn ! = lim

n
n
n
n
ln un
e

q.e.d.

Aplicatii (ale proprietatii P4 ).



xn
1. Daca xn = n, n N , atunci (xn )n1 B R+ , cu lim
= 1. Conform
n n

D R+ si
cu P4 , rezulta ca n n!
n1

1
n
lim n+1 (n + 1)! n! = lim Ln = ,
n
n
e
unde (Ln )ns>1 este sirul lui Lalescu (Problema 579, G.M., vol VI (1900-1901), p.148).

xn
2. Dac
a xn = 2n1, n N , atunci (xn )n1 B R+ cu lim
= 2. Rezult
a
n
n

D R+ si
ca n (2n 1)!!
n1
2
p
p
lim n+1 (2n + 1)!! n (2n 1)!! = ;
n
e
am obtinut limita Problemei C:904, G.M. - 5/1989, p.187.


Propozitia 1. Fie (xn )n1 B R+ ; atunci (xn )n1 D R+ daca si numai

n
xn+1
= a R+ .
daca exista lim
n
xn
xn
xn+1
Demonstratie. Conform ipotezei lim
= 1.
= x R+ si deci (din P2 ) lim
n n
n xn
Avem relatia urm
atoare:
n "
xn # xn xn

xn xn n
xn+1
=
1+
, n N .
xn
xn

n
xn+1
R+ , atunci trecnd la limit
a n aceast
a relatie pentru
Dac
a a = lim
n
xn
1/x lim xn

n , obtinem a = e n
, de unde lim xn = x ln a.
n
Reciproc, dac
a b = lim xn R+ , trecnd la limit
a n aceeasi relatie rezult
a
n

n
xn+1
c
a lim
= eb/x , adic
a a = eb/x .
n
xn

Propozitia 2. Daca sirurile (xn )n1 , (yn )n1 , (zn )n1 D R+ , atunci: a)

xn yn
xy
(un )n1 D R+ , unde un =
, n N , si b) lim un =
.
n
zn
z
23

Demonstratie. Conform enuntului, lim xn = x, lim yn = y si lim zn = z


n
n
n
cu x, y, z R+ . Cum, pentru orice n N , avem
xn+1 yn+1 xn yn
xn+1 yn+1 xn yn+1 xn yn+1 xn yn
un =

=
zn+1
zn
zn+1
zn+1
zn+1
zn
xn
yn+1
xn +
(yn+1 zn yn zn+1 ) =
=
zn+1
zn zn+1
xn
yn+1
xn +
(yn+1 zn yn zn + yn zn yn zn+1 ) =
=
zn+1
zn zn+1
xn
xn yn
yn+1
xn +
yn
zn ,
=
zn+1
zn+1
zn zn+1
y
xy
x
xy
si
trecnd la limit
a cu n rezult
a c
a lim un = x + y 2 z =
n
z
z
z
z
demonstratia se ncheie.
Aplicatii (ale propozitiei 2).
!

n2
(n + 1)2
n

p
1. Daca xn = yn = n, zn = n!, n N , atunci lim n+1

=e
n
n
(n + 1)!
n!
(Problema C:890, G.M. - 4/1988).
p

2. Dac
a xn = n n!, yn = n (2n 1)!!, zn = n, atunci
p
!
p
n+1
n
(n + 1)! (2n + 1)!!
n! (2n 1)!!
2
lim

= 2,
n
n+1
n
e

iar dac
a xn = yn = n n!, zn = n, n N , atunci
p
2 p
2
n+1
n
(2n
+
1)!!
(2n

1)!!

4
p

lim
= ,
n
n+1
n
e
n
(n + 1)!

etc.
3. Tot ca aplicatie a Propozitiei 2 se obtin limitele problemelor urm
atoare:
C:2869, C:2878, C:2987, C:3010 din G.M., L.192 din Revista matematica din Constanta si PP.2312, PP.2759, PP.3680, PP.5219, PP.5220, PP.5224, PP.5227 din
Octogon Mathematical Magazine.
Bibliografie

1. D. M. B
atinetu - Siruri,
Editura Albatros, Bucuresti, 1979.

2. D. M. B
atinetu-Giurgiu - O abordare a unor limite, G.M. - 5/2006, 227-229.

3. M. Tena
- O alta solutie a problemei 579 (G.M.), Revista "Lic
ariri" a Liceului
"Nicolae Balcescu", Craiova, 1978, 13-14.

24

O generalizare a teoremei lui Van Aubel


Silviu BOGA1
Considernd o cevian
a oarecare n locul bisectoarei unui
A
triunghi se obtine urmatoarea generalizare a teoremei bisectoarei :
\
MB
AB sin BAM
(1)
=

MC
AC sin CAM
\
(se stabileste aplicnd teorema sinusurilor n 4ABM si B
M
C
4ACM pentru a exprima M B si M C).
Acest rezultat cunoscut a fost utilizat ca instrument de lucru n [1], [3] s.a.
La rndul ei, relatia (1) poate fi generalizata la cazul n care ceviana AM este
nlocuit
a cu o transversal
a B1 C1 M ce nu-i paralel
a cu AB, AC (cteva pozitii ale
acesteia sunt prezente n figurile de mai jos):
A

C1

B1

A
B1

B1

C1

C1

c1
BC1 sin C
MB

;
(2)
=
c1
MC
CB1 sin B
evident, pentru A B1 C1 relatia (2) devine (1).
Pentru a dovedi aceast
a relatie, proced
am ca si n cazul relatiei (1): cu teorema
BC1
sinusurilor aplicat
a n 4C1 BM si 4B1 CM obtinem M B =
sin C1 si M C =
c
sin M
CB1
sin B1 , care, prin mpartire, dau (2).
c
sin M
n unele aplicatii este utila o consecinta directa a rezultatului dat de (2), consecinta prin care sunt eliminate n fapt unghiurile.
C1
Fie trei drepte a, b, c concurente dou
a cte dou
a si
0
transversalele t si t . Adopt
am notatiile prezente pe figura
A
alaturata si convenim ca (a; b) sa nsemne masura unuia
dintre unghiurile determinate de dreptele a si b.
B1

Propozitie. n conditiile specificate mai nainte, are


B
M C
loc formula
t
B
M 0 B 0 C 0 B1
M B CB1
M
= 0 0 0 .
(3)

C
a
c
b
M C BC1
M C B C1
Demonstratie. ntr-adevar, conform cu (2), aplicata triunghiurilor 4ABC si
4AB 0 C 0 si transversalei B1 C1 , avem:
B 0 C1 sin (a; c)
BC1 sin (a; c)
M 0B0
MB

=
,
.
MC
CB1 sin (b; c)
M 0C 0
C 0 B1 sin (b; c)
1

Profesor, Colegiul National, Iasi

25

De aici, rezult
a c
a

M B CB1
M 0 B 0 C 0 B1
sin (a; c)
= 0 0 0
=

, q.e.d.
M C BC1
M C B C1
sin (b; c)

Observatii 1. Egalitatea (3) si demonstratia ei nu sufer


a modific
ari pentru alte
pozitii ale dreptelor a, b, c (concurente dou
a cte dou
a) si transversalelor t si t0 .
2. Daca dreptele a, b, c sunt concurente n A, adica B1 si C1 coincid cu A, atunci
(3) se scrie n forma
M B AC
M 0 B 0 AC 0
.
(30 )

= 0 0
0
M
C
AB
M
C
AB
Mentionam ca (30 ) poate fi stabilita usor cu ajutorul relatiei (1).
Teorema 1. (generalizarea teoremei Van Aubel ). Fie 4ABC si punctele B 0
(AC), C 0 (AB) cu BB 0 CC 0 = {O}. Daca prin O trece o dreapta care taie (BC)
n A1 , (BA n C1 si (CA n B1 , atunci are loc relatia
B 0 A OA1
C 0 A OA1
+
= 1.
(4)

C 0 B OB1 B 0 C OC1
C1
Demonstratie. Aplic
am (3) mai nti la dreptele CA,
CB, CC 0 si transversalele AB si A1 B1 si apoi la dreptele
A
B1
BA, BC, BB 0 si transversalele AC si A1 C1 :
B
C
B 0 A BC
C 0 A CB
OB1 CA1
OC1 BA1

=
O
C 0 B CA
OA1 CB1
B 0 C BA
OA1 BC1
D A1
C
De aici obtinem
B
0
0
B A OA1
CA CA1
BA BA1
C A OA1
+
=
+
.
(5)

C 0 B OB1 B 0 C OC1
CB CB1 BC BC1
CD
BD
CA1
BA1
si
Construim AD k B1 A1 si observam ca
=
=
. Atunci, pentru
CB1
CA
BC1
BA
membrul drept al relatiei (5) revine la
CA CD BA BD

=1
CB CA
BC BA
si, n consecinta, (4) este dovedita.
Observatii. 1. Dac
a A B1 C1 , se obtine relatia Van Aubel.
2. Particulariznd O G (centrul de greutate al triunghiului), relatia (4) se scrie
1
1
1
+
=
(S. Boga, [1, p.43]).
GB1 GC1
GA1
3. Particulariznd O I (centrul cercului nscris triunghiului), obtinem
AB
BC
AC
+
=
.
IB1 IC1
IA1
Bibliografie
1. S. Boga - O proprietate remarcabila de fascicul. Matematica n scoala suceveana,
6/1989, 3-8.
2. D. Brnzei, S. Anita, M. Chirchiu - Geometrie. Clasa a IX-a (Colectia "Mate
2000"), ed. a III-a, Editura Paralela 45, Pitesti, 1998.
3. C. Artenie, C. Constanda - Generalizarea problemei bisectoarei glisate. Recreatii
Matematice, 1/2001, 32-33.
26

O propozitie echivalent
a cu conjectura lui Goldbach
Bogdan CIACOI 1
Punctul de plecare al acestei Note este celebra si aparent simpla conjectur
a a
lui Goldbach. Aceasta problema, care a impulsionat considerabil dezvoltarea teoriei
numerelor, se afla pe lista problemelor de matematica nerezolvate nca. Ea si are
originea n corespondenta dintre Christian Goldbach si Leonhard Euler anul 1742
si se formuleaz
a astfel:
(CG) Orice numar natural par, mai mare ca 2, este suma a doua numere prime.
O form
a mai tare a acesteia este:
0
(CG ) Orice numar natural par, mai mare ca 6, este suma a doua numere prime
diferite.
Dupa cum se poate vedea n [2], (CG0 ) este echivalenta cu afirmatia urmatoare:
Orice numar natural mai mare ca 17 este suma a trei numere prime diferite.
Sa notam cu P[n] multimea tuturor perechilor de numere prime egal departate de
0
n si cu P[n]
multimea tuturor perechilor de numere prime diferite si egal departate
de n. Exemple:
0
P[2]
= ,
P[2] = {(2, 2)},
0
P[3] = {(3, 3)},
P[3] = ,
0
P[4] = {(3, 5)},
P[4]
= {(3, 5)},
0
P[5] = {(3, 7)},
P[5]
= {(3, 7)},
0
P[6] = {(5, 7)},
P[6] = {(5, 7)},
0
P[7] = {(3, 11), (7, 7)},
P[7]
= {(3, 11)},
0
P[8] = {(3, 13), (5, 11)},
P[8]
= {(3, 13), (5, 11)},
0
P[9] = {(5, 13), (7, 11)},
P[9] = {(5, 13), (7, 11)},
0
P[10] = {(3, 17), (7, 13)},
P[9]
= {(5, 13), (7, 11)},
0
P[11] = {(3, 19), (5, 17), (11, 11)} etc.
P[11]
= {(3, 19), (5, 17)} etc.
0
0
Evident, au loc: P[n]
P[n] , n 2 si P[n]
= P[n] daca n nu este numar prim.

Observatii. 1. Postulatul lui Bertrand (pentru orice n 2 exist


a cel putin
un numar prim ntre n si 2n) da o sansa existentei unei perechi de numere prime
echidistante fata de n, dar nu o garanteaza.
2. Daca n 3, atunci, din considerente de paritate, se constata ca numarul prim
0
.
2 nu poate intra n perechile din P[n] si P[n]
0
rezulta ca avem:
3. Din modul cum au fost introduse multimile P[n] si P[n]
(i) (p, q) P[n] , n 2 k {0, 1, . . . , n 2} astfel nct numerele p = n k si
q = n + k sunt prime.
0
(ii) (p, q) P[n]
, n 4 k {1, 2, . . . , n 3} astfel nct numerele p = n k si
q = n + k sunt prime.
1

Elev, cl. a XII-a, Liceul Teoretic Ana Ip


atescu, Gherla

27

4. Dac
a fix
am n + k ca si num
ar prim ntre n si 2n, se poate ar
ata usor prin
calcul ca n k este de forma 6m 1, forma necesara a unui numar prim; binenteles
aceasta nu garanteaza ca n k este prim.
Formul
am urm
atoarele dou
a conjecturi:
(CCE) Pentru orice numar natural n 2 exista cel putin o pereche de numere
prime egal departate de el.
(CCE 0 ) Pentru orice numar natural n 4 exista cel putin o pereche de numere
prime diferite si egal departate de el.
Aceste pregatiri permit sa enuntam urmatoarea
Propozitie. Sunt adevarate urmatoarele afirmatii:
a) (CCE) este echivalenta cu (CG);
b) (CCE 0 ) este echivalenta cu (CG0 ).
Demonstratie. ntruct a) si b) se dovedesc n mod similar, vom demonstra
numai punctul b).
n
n
(CCE 0 ) (CG0 ) Fie n par si mai mare ca 6. Atunci N,
4 si, conform
2
2
n
n
ipotezei, k {1, 2, . . . , n 3} astfel nct
k si
+ k sunt prime. Atunci,
2
2
n n
n
n
n = + = ( k) + ( + k), adic
a n este suma a dou
a numere prime distincte.
2 0 2
2 0
2
(CG ) (CCE ) Fie n N, n 4. Atunci 2n este mai mare ca 6, si (CG0 )
fiind presupus
a adevarat
a, exist
a dou
a numere prime distincte p si q astfel nct
2n = p + q. Dac
a p < q (analog proced
am n cazul q < p), din egalitatea precedent
a
rezulta ca n p = q n N ; notam k = n p = q n si constatam ca avem
k {1, 2, . . . , n 3}. Cum (p, q) (n k, n + k) este o pereche de numere prime
echidistante de n, implicatia este dovedit
a.
Este posibil ca aceast
a form
a echivalent
a cu conjectura lui Goldbach s
a prezinte
un anumit interes si n privinta distributiei numerelor prime.
Bibliografie
1. I. Creang
a, C. Cazacu, P. Minut, Gh. Opait, C. Reicher - Introducere n
teoria numerelor, Editura Didactica si Pedagogica, Bucuresti, 1965.
2. P. Minut - Asupra ipotezei lui Goldbach, Recreatii Matematice, 4(2002), nr.1, 5-6.
3. W. Sierpi
nski - Ce stim si ce nu stim despre numerele prime, Editura S
tiintific
a,
Bucuresti, 1966.

Solutie (procedeul g
asirii num
arului ales - v. pag. 15). Scad 6 din num
arul ce
mi l-ati comunicat si obtin tocmai num
arul ales. Justificarea este dat
a de egalitatea
(a 5 + 3) 2 + b = ab + 6.
Observatie. Dar daca numarul ales ar avea trei cifre? Am proceda similar, pe
baza egalitatii
(a 5 + 3) 20 + (b 5 + 3) 2 + c = abc + 66.
28

Cum se poate obtine o inegalitate


1
Lucian TUTESCU

Prezentam n cele ce urmeaza modul n care s-a obtinut inegalitatea (1) de mai
jos, pe care o consideram a fi interesanta si susceptibila de generalizari. Totodata,
socotim c
a maniera de lucru urmat
a poate fi pentru elevi un model n obtinerea de
rezultate proprii sugerate de probleme ce apar n paginile diverselor reviste.
Daca x, y sunt numere reale cu acelasi semn, atunci

n
x2n + y 2n xn y n x2 + y 2 xy , n N.

(1)

Desi inegalitatea (1) este una algebric


a, povestea ei ncepe odat
a cu o problem
a
de geometrie propus
a la Concursul de matematica de la Kazanlk, Bulgaria, martie
2003.
\ = 2m(ACB).
\ Aratati ca:
Fie G centrul de greutate al 4ABC avnd m(AGB)
4
4
4
2
2

\
b) m(ACB) 60 .
a) AB = AC + BC AC BC ;
\ Folosind teorema cosinusului, teorema mediSolutie. a) Not
am = m(ACB).
anei si relatii uzuale privind aria triunghiului, obtinem:
AG2 + BG2 AB 2
a2 + b2 5c2

ctg 2 =
2 AG BG
9 4SABG
2 cos2 1
a2 + b2 5c2
a2 + b2 5c2
1

=
= 2 cos

2 sin cos
12SABC
3ab
cos
a2 + b2 + c2
a2 + b2 c2
2ab
ab
a2 + b2 5c2

= 2
= 2
2
2
3ab
ab
a +b c
3ab
a + b2 c2
2
2
a + b2 c4 = 3a2 b2 c4 = a4 + b4 a2 b2 .
cos 2 =

1
b) Este suficient sa aratam ca cos , adica c2 = a2 +b2 2ab cos a2 +b2 ab,
2

2
astfel spus c4 a2 + b2 ab . Apare deci firesc s
a dovedim inegalitatea

2
a4 + b4 a2 b2 a2 + b2 ab ,
(2)

care este tocmai (1) pentru n = 2. Relatia (2) se justific


a usor:

2
(2) a4 + b4 a2 b2 a2 + b2 + a2 b2 2ab a2 + b2

2
2ab a2 + b2 4a2 b2 2ab (a b) 0,

cu egalitate pentru a = b.
Un prim pas n trecerea de la (2) la (1) este dat de:

Dac
a x, y sunt numere reale nenule de acelasi semn, atunci

2k
k+1
k+1
k
k
x2
+ y2
x2 y 2 x2 + y 2 xy
, k N.
Profesor, Colegiul National "Fratii Buzesti", Craiova

29

(3)

Justificarea acestui rezultat se obtine cu usurinta din (2), prin inductie matematica. Tot inductia ne ajuta sa facem trecerea la cazul general (1); prezentam n
continuare calculele complete.
Verificarea lui (1) pentru n = 0, 1, 2 este imediat
a. Presupunem c
a (1) este
adev
arat
a pentru n = k si vom demonstra c
a este adev
arat
a pentru n = k + 1. Este
suficient sa aratam ca

x2k+2 + y 2k+2 xk+1 y k+1 x2 + y 2 xy x2k + y 2k xk y k ,


()
de unde va urma usor concluzia inductiv
a. Inegalitatea () devine succesiv:

x2k+2 + y 2k+2 xk+1 y k+1 x2k+2 + x2 y 2k xk+2 y k + y 2 x2k + y 2k+2


xk y k+2 x2k+1 y xy 2k+1 + xk+1 y k+1

xk+2 y k xk y k+2 + x2k+1 y + xy 2k+1 x2 y 2k y 2 x2k 2xk+1 y k+1 0

xy xk y k + x2 y k xk y k xk y 2 xk y k 0

xy xk y k xk y k + xy k1 yxk1 0

xy xk y k (x y) xk1 + y k1 0

xy (x y)2 xk1 + xk2 y + + y k1 xk1 + y k1 0,

relatie evident adev


arat
a. Egaliate se obtine cnd x = y.

S
a not
am c
a (1) nu este, n general, adev
arat
a atunci cnd x si y au semne
contrare. De exemplu, pentru n = 4, x = 1, y = 1 obtinem contradictia 1 34 .
Dou
a generaliz
ari naturale ale lui (1) ar putea fi inegalit
atile:
4
n
4n
3n n
2n 2n
n 3n
4n
3
2 2
x x y + x y x y + y x x y + x y xy 3 + y 4 ;

n
x2n + y 2n + z 2n xn y n y n z n z n xn x2 + y 2 + y 2 xy xz yz ,

(4)
(5)

unde n N, iar x, y, z sunt numere reale de acelasi semn.


Nu cunoastem pn
a la data public
arii acestui articol demonstratii pentru (4) si
(5) n cazul general, nsa am justificat cele doua inegalitati n cazul n = 2. Prezentam
calculele numai pentru (4):

2
x8 x6 y 2 + x4 y 4 x2 y 6 + y 8 x4 x3 y + x2 y 2 xy 3 + y 4
x8 x6 y 2 + x4 y 4 x2 y 6 + y 8 x8 + x6 y 2 + x4 y 4 + x2 y 6 + y 8 2x7 y + 2x6 y 2
2x5 y 3 + 2x4 y 4 2x5 y 3 + 2x4 y 4 2x3 y 5 2x3 y 5 + 2x2 y 6 2xy 7
4x2 y 6 4x6 y 2 4x4 y 4 + 2x7 y + 4x5 y 3 + 4x3 y 5 + 2xy 7 0
h

i
2xy x3 y 3 + 2xy 2 x3 y 3 2x2 y x3 y 3 0

2xy x3 y 3 (x y) x2 + xy + y 2 2xy 0
2
2
2xy (x y) x2 xy + y 2 0,

ceea ce este adevarat.


L
as
am deschis
a problema demonstr
arii inegalit
atilor (4) si (5).

30

Solutiile problemelor propuse n nr. 1 / 2006


Clasele primare
P.104. Suma dintre predecesorul unui numar si succesorul numarului urmator
lui este 29. Care este acest numar?
(Clasa I )
Irina Luca, elev
a, Iasi
Solutie. Suma dintre numarul cautat si succesorul lui este 29 = 14 + (14 + 1).
Deducem c
a num
arul c
autat este 14.
32

+1

+2

-3

+13

-13

P.105. Alaturat se afla robotelul "MATE".


a) Completati casetele goale;
b) Aflati suma numerelor pe care le tine n mini;
c) Aflati diferenta numerelor scrise n talpile picioarelor.
(Clasa I )
Andrei Stativ
a, elev, Iasi
Solutie. a) Pe umeri se afl
a num
arul a astfel nct a 13 = 32,
67
de unde a = 32 + 13 = 45.
b) n mna stnga are 45 3 = 42, iar n dreapta are numarul cu 13 mai mic
dect 45, adic
a 32. Suma numerelor pe care le tine n mini este 42 + 32 = 74.
c) La baza bustului are num
arul 67 + 2 = 69, iar pe talpa dreapt
a tine num
arul
69 + 1 = 70. Diferenta este 70 67 = 3.
P.106. Pentru desemnarea campioanei, echipele de hochei pe gheata A si B
disputa un numar de partide pna ce una dintre ele cstiga de 4 ori. Care este
numarul maxim de partide care se pot juca, stiind ca nu au fost rezultate de egalitate?
(Clasa a II-a)
nv. Constanta Cristea si Inst. Iulian Cristea, Iasi
Solutie. Numarul maxim de partide care se pot juca este 7. ntr-adevar, este
posibil sa se realizeze scorul 4 3.
P107. Un grup de turisti a consumat 17 prajituri si 31 nghetate. Stiind
ca 7

turisti au consumat cte o nghetata si cte o prajitura, 5 turisti au consumat numai


cte doua nghetate, iar 4 turisti nu au consumat nimic, iar restul cte un singur
produs (nghetata sau prajitura), sa se afle cti turisti sunt n grup.
(Clasa a II-a)
Aliona Loghin, elev
a, Iasi
Solutie. Num
arul turistilor care au consumat cte o pr
ajitur
a este 17 7 = 10.
Num
arul turistilor care au consumat cte o nghetat
a este 31 7 5 5 = 14. n
cofetarie au intrat 7 + 10 + 5 + 14 + 4 = 40 turisti.
P108. Prin mpartirea a doua numere naturale rezulta ctul 3 si restul 6. Stiind

ca mpartitorul este un numar mai mic dect 10, aflati cele doua numere.
(Clasa a III-a)
nv. Rica Buc
atariu, Iasi
Solutie. Punnd conditia r < , putem avea: 7 3 + 6 = 27, 8 3 + 6 = 30;
9 3 + 6 = 33.
P.109. Figura alaturata este formata din betisoare.
a) ndeparteaza un singur betisor pentru a obtine tot attea triunghiuri ca si patrate;
31

F
b) Muta doua betisoare pentru a obtine de doua ori mai multe drepG
E
tunghiuri dect patrate.
(Clasa a III-a)
Adina Voinescu, elev
a, Iasi H
D
I
Solutie. a) ndep
artnd unul din betisoarele DE sau EG sau GI
C
J
obtinem dou
a triunghiuri si dou
a p
atrate.
b) Mutam betisoarele ID si JC astfel nct sa formam un drepA B
tunghi de latime AB si lungime BF . n acest caz vom avea un patrat si doua
dreptunghiuri.
P.110. Ce ora indica primul ceas, stiind ca acesta respecta regula indicata de
celelalte trei?
12

12
3

9
6

12
3

12

9
6

(Clasa a III-a)
Veronica Corbu, elev
a, Iasi
Solutie. De la al doilea ceas la al treilea ceas avem o crestere de 1h 200 , iar de
la al treilea ceas la al patrulea ceas avem o crestere de 2h 400 . Regula const
a n
dublarea cresterii. Cum 1h 200 este dublul lui 400 , primul ceas arata 6h 200 .
P.111. Fie numarul N = abc + acb + bac + bca + cab + cba.
a) Care este cea mai mica si cea mai mare valoare a lui N ?
b) Cte valori diferite poate avea numarul N ?
(Clasa a IV-a)
Oxana Pascal, elev
a, Iasi
Solutie. Avem N = 222 (a + b + c).
a) Cea mai mic
a valoare a ui N este 222 3 = 666.
Cea mai mare valoare a lui N este 222 27 = 5994.
b) Valorile sumei a + b + c sunt de la 3 la 27. Numarul N poate lua 27 3 + 1 = 25
valori diferite.
P.112. n urma desfasurarii unui joc didactic matematic, nvatatorul a oferit
ca recompensa 44 baloane. Cte 4 baloane au primit un numar de participanti ce
reprezinta a sasea parte din totalul lor, cte doua au primit a treia parte, iar restul
participantilor au primit cte un balon. Aflati numarul participantilor la joc (solutie
aritmetica!).
(Clasa a IV-a)
Alexandra Nistor, elev
a, Iasi
Solu
t
ie.
Presupunem
c
a
mai
ad
a
ug
a
m

niste
4
elevi,
n
mod
convenabil,
astfel
nct
fiecare
elev s
a
6
4
1
44 primeasca cte un singur balon din cele 44. Astfel,
6
6
3
4 1
4
6
se transform
a n , se transform
a n , iar restul
6 3
6
3
1
se transforma n Sa figuram noua situatie.
2
6
O sesime din numarul elevilor participanti la concurs primeste 44 : 11 = 4 baloane.
Numarul elevilor participanti la concurs este 4 6 = 24.
P.113. Dan si-a pus timbrele n clasor, cte 10 pe unele pagini, cte 30 pe alte
pagini si au ramas de 4 ori mai multe pagini goale dect folosite. Daca ar pune
cte 5 timbre pe fiecare pagina, toate paginile ar fi folosite. Cte pagini poate avea
32

clasorul, stiind ca nu depaseste 60 (solutie aritmetica!)?


(Clasa a IV-a)
Petru Asaftei, Iasi
Solutie. Notam cu a numarul de pagini cu cte 10 timbre si cu b numarul
de pagini cu cte 30 timbre. Din prima informatie deducem c
a num
arul paginilor
clasorului este 5 (a + b). Din a doua informatie rezulta c
a num
arul paginilor clasorului
este 2a + 6b. Sa figuram aceasta situatie.

b
Constatam ca b = 3a.

Distingem cazurile:
a = 1 b = 3 5 (1 + 3) = 5 4 = 20 (pagini);
a = 2 b = 3 2 b = 6 5 (2 + 6) = 5 8 = 40 (pagini);
a = 3 b = 3 3 b = 9 5 (3 + 9) = 60 (pagini).

Clasa a V-a

V.66. Sa se arate ca, oricare ar fi cifra nenula a, numarul x = 2131a + 32a13 +


43
se divide cu 10.
Otilia Nemes, Ocna Mures (Alba)
Solutie. Deoarece a13 = M 4 + 1, iar a31 = M 4 + 3, atunci U (2131a ) = 1,
.
U (32a13 ) = 2, iar U (43a31 ) = 7. Deducem c
a U (x) = 0, deci x .. 10.
a31

V.67. a) Sa se arate ca, scaznd din suma a 2006 numere pare consecutive suma
numerelor situate ntre acestea, nu se poate obtine rezultatul 20062 .
b) Sa se afle 2006 numere pare consecutive astfel nct, scaznd din suma lor
suma numerelor situate ntre ele, sa se obtina 20052 .
Marian Pantiruc, Iasi
Solutie. a) ntre 2006 numere pare consecutive se afl
a 2005 numere impare, cu
suma num
ar impar; deducem c
a si diferenta este tot impar
a.
b) Fie a num
arul cel mai mic; atunci
[a + (a + 2) + + (a + 4010)] [(a + 1) + (a + 3) + + (a + 4009)] = 20052

a + 2005 = 20052 a = 2004 2005.


V.68. Aratati ca nu exista n N pentru care An = 5n + 89 sa fie patrat perfect.
Iulia Plesca, elev
a, Iasi
Solutie. Avem c
a A0 = 90, A1 = 94 nu sunt p
atrate perfecte. Pentru n 2,
.
ultimele dou
a cifre ale lui 5n sunt 25, deci An se termin
a n 14. Deducem c
a An .. 2,
.
dar An ../4, prin urmare An nu poate fi patrat perfect.
V.69. Sa se rezolve n N2 ecuatia 8n + 15m = 6 + 62 + + 62006 .
Alexandru Gabriel Tudorache, elev, Iasi
Solutie (Cezara Maria Enea, elev
a, Iasi). Din 15m - num
ar impar si 6 +
2
2006
6 + + 6
- num
ar par, rezult
a c
a 8n este num
ar impar, deci n = 0. Atunci
1 + 15n 6= M 3, iar 6 + 62 + + 62006 = M 3, contradictie, deci ecuatia nu are solutii.
V.70. Determinati a N pentru care numerele a, a + 2, a + 6, a + 12, a + 18,
a + 20, a + 26, a + 30, a + 32, a + 36, a + 60 sunt simultan prime.
Lucian Tutescu, Craiova
33

Solutie. Cum numerele 0, 2, 6, 12, 18, 20, 26, 30, 32, 36, 60 genereaz
a toate
resturile posibile la mpartirea prin 11, acelasi lucru se ntmpla si pentru cele 11
numere date. Deducem ca cel putin unul din ele se divide cu 11 si, cum numerele
sunt prime, m
acar unul este egal cu 11. Avem de studiat trei cazuri:
i) Dac
a a = 11, numere sunt 11, 13, 17, 23, 29, 31, 37, 41, 43, 47, 71, toate prime.
ii) Daca a + 2 = 11, atunci a = 9 nu este prim.
iii) Daca a + 6 = 11, atunci a = 5, nsa a + 20 = 25 nu este prim.
Valoarea c
autat
a a lui a este 11.

Clasa a VI-a

A 12 m B
VI.66. Alaturat este desenata o gradina avnd forma 9 m
6m
unui poligon cu 7 laturi. n fiecare vrf se afla cte o
C
poarta mobila astfel nct, n oricare doua vrfuri vecine, G
7m
portile sa nchida perfect latura pe care acestea o deter11
m
mina. Sa se afle lungimile portilor.
D
6m
Roxana C
ap
atn
a, elev
a, Iasi
F 10 m E
Solutie. Fie x lungimea portii din A. Atunci portile
din B, C, D, E, F , G au respectiv lungimele:12x; 6(12 x) = x6; 7(x 6) =
13 x; 6 (13 x) = x 7, 10 (x 7) = 17 x; 11 (17 x) = x 6. Punem
conditia de nchidere a portilor pe latura [AG] : x+(x 6) = 9 x = 7, 5. Lungimile
celor 7 porti vor fi 7, 5m; 4, 5m; 1, 5m; 5, 5m; 0, 5m; 9, 5m, respectiv 1, 5m.
VI.67. n patrulaterul ABCD construim AP BD, CQ BD, P, Q BD
si fie M mijlocullui (AC). Daca punctele M , P , Q sunt distincte doua cte doua,
demonstrati ca 4M P Q este isoscel.
Marius Farcas, Iasi
Solutia 1 (a autorului). Fie {R} = P M CQ.
C
\
\ (alterne interne), AM
\
Avem c
a P
AM RCM
P D
\
CM
R (opuse la vrf) si AM = M C, prin urmare
P M
R
4AM P 4CM R (U.L.U). Deducem c
a P M = M R si
O
Q
atunci QM este median
a n 4P QR dreptunghic n Q,
S
1
A
de unde rezulta ca QM = P R = P M , adica 4M P Q
2
B
este isoscel.
Solutia 2 (Gabriel Popa). Fie S mijlocul lui [AQ], iar {O} = M SBD. Atunci
M S este linie mijlocie n 4ACQ, prin urmare M S k CQ. Rezult
a c
a OS k AP si,
cum S este mijlocul lui [AQ], avem ca OS este linie mijlocie n 4OAP . Astfel, O
este mijlocul lui [P Q] si M O P Q, deci 4M P Q este isoscel.
Solutia 3 (Alexandra Cadar, eleva, Iasi). Aplicam teorema medianei si teorema lui Pitagora:

2 P A2 + P C 2 AC 2
2 P A2 + P Q2 + QC 2 AC 2
2
PM =
=
=
4
4

2 AQ2 + QC 2 AC 2
= M Q2 .
=
4
\
VI.68. Fie punctele A, C, M cu m(AM
C) 6= 90 si AC = 2 AM . Sa se arate
34

\ ) = m(M
\
ca M este mijlocul lui [AC] daca si numai daca 2m(ACM
AC).
Ioan S
ac
aleanu, Hrl
au
Solutie. Daca M este mijlocul lui [AC], atunci
M
\ ) = m(M
\
\) =
m(ACM
AC) = 0 , de unde 2m(ACM
T
\
\) =
m(M
AC). Reciproc, s
a presupunem c
a 2m(ACM
\
m(M
AC). Dac
a A, M , C nu ar fi coliniare, fie N mijlocul
\
C
N
lui [AC] si [AT bisectoarea lui M
AC, T M C. Atunci A
4T AC este isoscel si T N este mediana, prin urmare T N AC. Pe de alta parte,
\
\
4M AT 4N AT (L.U.L.), deci m(AM
C) = m(AN
T ) = 90 , ceea ce contravine
ipotezei. Ramne ca punctele A, M , C sunt coliniare. n plus, nu putem avea punc\
\ ).
tul M pe prelungirile segmentului [AC], altfel m(M
AC) = 180 6= 0 = 2m(ACM
Deducem c
a M este mijlocul lui [AC], ceea ce ncheie rezolvarea.
VI.69. Sa se arate ca pentru orice alegere a semnelor n expresia 12 22
20062 , rezultatul nu se divide cu 2006.
Mihail Bencze, Brasov
Solutie. Dac
a ar exista o alegere a semnelor pentru care rezultatul s
a se divid
a
cu 2006, n mod necesar ar trebui ca 1 2 2006 si 2006 sa aiba aceeasi
paritate (o putere are aceeasi paritate cu baza sa). Cum suma si diferenta au aceeasi
paritate, ar rezulta c
a 1 + 2 + + 2006 este num
ar par, deci 1003 2007 este par,
absurd. Astfel, r
amne adev
arat
a concluzia.
3m + 1
n+2
VI.70. Determinati m, n Z pentru care a =
+
Z.
2m + 1 3n + 5
Gheorghe Iurea, Iasi
Solutie. Cele doua fractii din expresia lui a sunt ireductibile. ntr-adevar:
d | 3m + 1, d | 2m + 1 d | 3 (2m + 1) 2 (3m + 1) d | 1

si analog pentru a doua. Cum (3m + 1, 2m + 1) = 1, (3n + 5, n + 2) = 1, obtinem:


3m + 1
(3n + 5) a = (3n + 5)
+ n + 2 Z 2m + 1 | 3n + 5;
2m + 1
n+2
Z 3n + 5 | 2m + 1,
(2m + 1) a = 3m + 1 + (2m + 1)
3n + 5
prin urmare |3n + 5| = |2m + 1|.
i) Daca 3n + 5 = 2m + 1, atunci 3n = 2 (m 2), deci m 2 = 3k, n = 2k,
11k + 9
k Z, iar a =
Z. Obtinem c
a 6k + 5 | 11k + 9, de unde 6k + 5 |
6k + 5
6 (11k + 9) 11 (6k + 5), adic
a 6k + 5 | 1. Deducem c
a 6k + 5 {1, 1}, deci
k = 1 si astfel (m, n) = (1, 2).
ii) Daca 3n+5 = 2m1, atunci 3n = 2 (m + 3), de unde m+3 = 3k, n = 2k,
11k 10
Z. Ca mai sus, g
asim solutiile (m, n) {(3, 0) ; (0, 2)}.
k Z, iar a =
6k 5

Clasa a VII-a

VII.66. Sa se rezolve n R4 ecuatia


p
p

30 x y + 901 + 25 y z + 626 + 20 z x + 401 + 9 t x + 78 = 2006.

Ioana Olan, elev


a, Iasi

35

Solutia 1. n conditiile de existenta a radicalilor, folosind inegalitatea mediilor,


avem:
p
p

30 x y + 901 + 25 y z + 626 + 20 z t + 401 + 9 t x + 78 =


p
p
p
p
= 900 (xy+901) + 625 (yz+626) + 400 (zt+401) + 81 (tx+78)
900+xy+901 625+yz+626 400+zt+401 81+tx+78

+
+
+
= 2006.
2
2
2
2
Cum se atinge egalitatea, n mod necesar vom avea c
a x y + 901, y z + 626 = 625,
z t + 401 = 400 si t x + 78 = 81. Sistemul obtinut este nedeterminat, cu solutiile
{(, + 1, + 2, + 3) | R}.
Solutia 2. Egalitatea din ipotez
a se poate scrie sub forma

2
2
p
p
30 x y + 901 + 25 y z + 626 +

2
2

+ 20 z t + 401 + 9 t x + 78 = 0.

Fiecare termen al sumei trebuie s


a se anuleze etc.

.
VII.67. Aflati a, b N daca a + b = 18 si 10a+1 9b + 71 .. 81.
Andrei-Sorin Cozma, elev, Iasi
Solutie. Avem:

10a+1 9b + 71 = 10a+1 1 9 (b + 1) + 81 = 99
. . . 9} 9 (b + 1) + 81 =
| {z
a+1 cifre

= 9 11
.
.
.
1

1
+
81
=
9
11
.
.
.
10

b
+ 81,
| {z }
| {z }
a+1 cifre

a de 1

.
.
. . 10} b .. 9. Restul mpartirii lui 11
. . 10} prin 9 este
deci 10a+1 9b + 71 .. 81 11
| .{z
| .{z
a de 1

a de 1

.
.
. . 10}b .. 9 ab .. 9.
acelasi cu restul mp
artirii lui |1 + 1 + {z
+ 1 + 0} prin 9, deci 11
| .{z
a de 1

a de 1

Cum a + b = 18, obtinem solutiile (a, b) {(0, 18) ; (9, 9) ; (18, 0)}.

VII.68. Fie 4ABC dreptunghic, cu ipotenuza de


a, catetele b si c, iar
lungime
2 x2 + y 2
a2
aria S. Daca x, y (0, ), sa se arate ca
=
daca si numai daca b
S
xy
si c sunt direct sau invers proportionale cu x si y.
Veronica Pl
aesu si Dan Pl
aesu, Iasi
Solutie. Avem succesiv:

2 x2 + y 2
2 b2 + c2
2 x2 + y 2
a2
=

=
xy b2 + c2 = bc x2 + y 2
S
xy
bc
xy
bx (by cx) cy (by cx) = 0 (by cx) (bx cy) = 0
c
c
b
b
by cx = 0 sau bx cy = 0 = sau 1 = 1 .
x
y
x
y
36

\
VII.69. Fie 4M N P cu m(N
M P ) = 90 ; se considera punctele S, T, M
(N S), M (P T ), astfel nct N S = 3 M S, P T = 3 M T . Daca {Q} = P S N T ,
atunci:
a) QM = N P ;
b) QN 2 + QP 2 = 5 N P 2 .
Dorel Luchian, Iasi
MS
MT
Solutie. a) Fie {R} = QM N P . Cum
=
= Q
MN
MP
1
, din reciproca teoremei lui Thales rezult
a c
a T S k NP .
2
TS
1
Atunci 4M T S 4M P N , de unde
= , iar 4QT S
NP
2
QS
QT
TS
1
T
S
4QN P , prin urmare
=
=
= . Astfel, N S
QP
QN
NP
2
M
si P T sunt mediane n 4QN P , deci M va fi centrul de greutate al 4QN P . Rezult
a c
a R este mijlocul lui [N P ], iar
QM = 2M R. Pe de alta parte, M R este mediana n 4M N P
dreptunghic, deci N P = 2M R. Deducem c
a QM = N P .
N
R
P
b) Aplic
am n mod repetat teorema lui Pitagora:

2
2
QN 2 + QP 2 = (2T N ) + (2SP ) = 4 M T 2 + M N 2 + 4 M S 2 + M P 2 =

= 4 M N 2 + M P 2 + 4 M T 2 + M S 2 = 4N P 2 + 4T S 2 = 5N P 2 .

VII.70. Triunghiul alaturat este considerat fix. n cte moduri putem aseza numerele 1, 2, 3, 4, 5, 6 n cerculete, astfel nct
suma numerelor de pe fiecare latura a triunghiului sa fie aceeasi?
Petru Asaftei, Iasi
Solutie. Fie i, j, k cele trei numere din vrfuri; cum fiecare
vrf apartine la cte dou
a laturi, suma numerelor de pe fiecare latur
a va fi:
1
i+j+k
S = [(1 + 2 + + 6) + (i + j + k)] = 7 +
.
3
3
..
Acest numar trebuie sa fie natural, deci i + j + k . 3. Valoarea minima pentru i + j + k
este 1 + 2 + 3 = 6, iar cea maxim
a este 4 + 5 + 6 = 15, prin urmare i + j + k
{6, 9, 12, 15}. Deducem c
a
{i, j, k} {{1, 2, 3} ; {1, 2, 6} ; {1, 3, 5} ; {2, 3, 4} ; {1, 5, 6} ; {2, 4, 6} ; {3, 4, 5} ; {4, 5, 6}} .
Odat
a fixat
a multimea {i, j, k}, cele trei numere pot fi permutate pe cele trei cerculete
din vrfuri n 6 moduri, iar apoi numerele din mijloacele laturilor, dac
a exist
a, sunt
bine determinate. Spre exemplu, daca {i, j, k} = {1, 2, 3}, avem ca S = 9 si obtinem
completarile:
1
6
2

1
5

5
3

2
6

6
2

2
4

4
3

5
1

3
4

4
2

Se constata usor ca nu avem posibilitatea completarii triunghiului daca {i, j, k}


{{1, 2, 6} ; {2, 3, 4} ; {1, 5, 6} , {3, 4, 5}}. Obtinem astfel c
a num
arul de complet
ari
posibile este 4 6 = 24.

Clasa a VIII-a

VIII.66. Sa se demonstreze ca
37

1
1
n1
1
+ 4
+ + 4
<
.
2
2
2
+2 +1 3 +3 +1
n +n +1
3n
Carmen Daniela Tamas, Brlad
1
1
, k N , deoarece aceasta
Solutie. Are loc inegalitatea 4

2+1
2
k
+
k
3k
2
2
revine la k 1 0. Egalitatea se atinge numai pentru k = 1. Atunci
1
1
1
+
+ + 4
<
24 + 22 + 1 34 + 32 + 1
n + n2 + 1

1
1
1
1
1
1
1
+
+ +
<
+
+ +
=
<
3 22 3 32
3 n2
3 12 23
(n 1) n

1
1
1 1
1
1
1
1
n1
=
1
+

+ +

=
1
=
.
3
2
2 3
n1 n
3
n
3n
Not
a. Elevul Florian P
alita
a inegalitatea prin inductie
, Petrosani, demonstreaz
matematica.
VIII.67. Fie 0 < a < b < c < d < e si propozitiile:

2ac
b+d
2ce
p1 : b =
; p2 : c =
; p3 : c = ae; p4 : d =
.
a+c
2
c+e
Sa se arate ca daca oricare trei dintre propozitii sunt adevarate, atunci este adevarata
si cea de-a patra.
Claudiu-
Stefan Popa, Iasi
Solutie. Consideram nti ca p1 , p4 sunt adevarate si sa aratam ca p2 p3 .
Avem:
b+d
2ac
2ce
a
e
c=
2c =
+
1=
+

2
a+c c+e
a+c c+e

a (c + e) + e (a + c) = (a + c) (c + e) c2 = ae c = ae.
a
c
a
c
Demonstr
am acum c
a p1 p2 p3 p4 . Din p3 obtinem c
a = , deci
=
.
c
e
a+c
c+e
Atunci, folosind p2 si apoi p1 , avem:

2ac
a
2c2
2ae
2ce
d = 2c b = 2c
= 2c 1
=
=
=
,
a+c
a+c
a+c
a+c
c+e
deci p4 este adevarata. La fel se arata ca p2 p3 p4 p1 .
VIII.68. Fie An = 2006n + 2005n 1992n 1991n , n N. Sa se determine n
.
pentru care An .. 28.
Ionel Nechifor, Iasi
.
.
Solutie. Folosind faptul c
a an bn .. a b, n N, deducem c
a 2006n 1992n .. 7,
.
.
.
.
.
2005n 1991n .. 7, deci An .. 7. Sa vedem cnd An .. 4. Avem ca A0 = 0 .. 4, A1 = 28 .. 4.
.
Daca n 2, atunci 2006n = (M 4 + 2)n = M 4 + 2n .. 4 si este evident ca 1992n =
.
.
.
.
(M 4)n .. 4. Astfel, An .. 4 2005n 1991n .. 4 (M 4 + 1)n (M 4 1)n .. 4
.
n .
n
M 4 + 1 (1) .. 1 (1) = 0 n par. n concluzie, A .. 28 pentru n
24

{1} {2k | k N}.


VIII.69. Fie x1 , x2 , x3 R astfel nct x21 + x22 + x23 = 1. Determinati cea mai
mica si cea mai mare valoare a expresiei
38

E (x1 , x2 , x3 ) = x1 + x2 + x3 + x1 x2 + x2 x3 + x3 x1 .
Ion Visan si Lucian Tutescu, Craiova
(x1 + x2 + x3 + 1)2 2
Solutie. Expresia se scrie sub forma E =
. Atunci
2
x
=
x
=
0,
x3 = 1.
Emin = 1, iar aceasta valoare se atinge, de exemplu, pentru
1
2
r
2
2
2
x1 + x2 + x3
x1 + x2 + x3
1
Din inegalitatea M A M P , avem c
a

= , cu
3
3
3

3 1/ 3 + 1 2
1
= 1 + 3,
egalitate cnd x1 = x2 = x3 = . Astfel, Emax =
2
3
1
iar aceasta valoare se atinge pentru x1 = x2 = x3 = .
3
VIII.70. Se considera cubul ABCDA0 B 0 C 0 D0 si fie M , N mijloacele muchiilor
[AB], respectiv [BC], iar {S} = AN CD, {T } = DM BC. Sa se afle masura
unghiului format de D0 N si ST .
Gabriel Popa, Iasi
Solutie.
Fie {P } = AN DM .
Din
D
C
\P
\
4AM D 4BN A (C.C) rezulta ca ADP
AM . A
B
\
\) = 90 ,deci m(ADP
\) +
nsa m(P
AM ) + m(DAP
\) = 90 si atunci m(AP
\
m(DAP
a
D) = 90 , adic
S
D
C
AN DM . Deducem c
a SP si T C sunt n
altimi
P
n 4DST , prin urmare N va fi ortocentrul acesN
B
tui triunghi, iar DN este tot n
altime: DN T S. A M
T
Din DD0 (ABC) urmeaz
a c
a DD0 T S, deci
0
0
T S (DD N ), de unde D N ST .

Clasa a IX-a
IX.66. Pentru x, y R, fie a = y + xy x, b = x2 + x xy.
a) Daca a, b (, 0), sa se compare numerele x si y.
b) Aratati ca exista o infinitate de numere rationale x, y pentru care a, b (, 0).
Ionut Onofrei, elev, Hrl
au
Solutie. a) Din a + b < 0, obtinem c
a x2 + y < 0, deci y < x2 < 0. S
a
presupunem prin absurd c
a x 0; din b = x(x + 1 y) < 0 rezult
a c
a x + 1 y < 0,
deci x < 1 + y. Dar 1 + y < 0 si am ajuns la o contradictie. Cum x, y < 0,
deducem ca xy > 0, prin urmare y x = a xy < 0. n concluzie, y < x.
1
n+1
b) De exemplu, putem considera x = , y =
, cu n N ; atunci
n
n
2
n2 + n + 1
si b = sunt ambele negative.
a=
n2
n
IX.67. Fie n 2 si a1 , a2 , . . . , an 0 astfel nct
2

(a2 a3 an ) + (a1 a3 a4 an ) + + (a1 a2 an1 ) = 1.


Sa se arate ca a1 + a2 + + an + a2 a3 an + a1 a3 an + + a1 a2 an1 n.
Adrian Zahariuc, elev, Bac
au
n
Q
Q
Solutie. n dezvoltarea produsului
aj apar toti termenii de tipul
ai +
i=1

39

j6=i

2
n
n Q
Q
Q P
, i = 1, n, deci
aj
aj = 1. Din inegalitatea mediai +
i=1 j6=i
j6=i
j6=i
s i=1
n
n
Q
P
Q
Q
aj
aj , de unde concluzia problemei.
ai +
ai +
ilor avem c
a nn
aj

i=1

j6=i

i=1

j6=i

Egalitatea se atinge cnd n 1 numere sunt egale cu 1, iar cel r


amas este 0.
IX.68. n 4ABC se considera cevienele [AM ], [BN ], [CP ] concurente n T .
TA
TB
TC
Sa se arate ca
=
=
daca si numai daca T este centrul de greutate al
TM
TN
TP
4ABC.
Ovidiu Pop, Satu Mare
TC
TB
[
\
si BT
=
Solutie. Cum
C N
T P (opuse
A
TN
TP
[
la vrf), rezult
a c
a 4BT C 4N T P , de unde T BC
\
T
N P , prin urmare P N k BC. Analog se arata ca
PB
AP
P
N
; atunci
= 1 k,
M N k AB, P M k AC. Fie k =
T
AB
AB
AP
BM
AN
BM
BP
AN
=
= k,
=
= k,
=
= 1 k.
AC
AB
BC
AC
BC
AB
1
Din ultimele doua relatii obtinem k = , deci P este mij- B
M
C
2
locul lui [AB], apoi M , N sunt mijloace pentru [BC], respectiv [AC]. n concluzie,
T este centrul de greutate al 4ABC. Reciproca este imediata.
IX.69. Fie 4ABC nedreptunghic. Paralela prin B la AC si simetrica dreptei
AC n raport cu BC se intersecteaza n A1 ; analog se obtin punctele B1 si C1 . Daca
AA1 , BB1 , CC1 sunt concurente, sa se arate ca 4ABC este echilateral.
Temistocle Brsan, Iasi
A
Solutie. Pentru nceput, fie 4ABC ascutitunghic.
Se vede usor c
a 4BCA1 este isoscel, deci A1 se afl
a pe
mediatoarea lui [BC]. Fie {X} = AA1 BC; evident ca
X (BC). Din 4BXA1 4CXA obtinem
BA1
XB
a
XB
A
=

=
B
C
XC
CA
XC
2b cos C
X
0
BA
(caci n 4A0 BA1 dreptunghic avem cos C =
=
BA1
A1
YC
b
a
). Analog caracterizam Y (CA) si Z (AB) prin
=
,
2BA1
YA
2c cos A
ZA
c
=
. Concurenta dreptelor AA1 , BB1 , CC1 conduce la
ZB
2a cos B
a
b
c
1

= 1 cos A cos B cos C = .


2b cos C 2c cos A 2a cos B
8
1
n binecunoscuta inegalitate cos A cos B cos C , egalitatea se atinge pentru
8
A = B = C, deci 4ABC este echilateral.
Dac
a 4ABC ar fi obtuzunghic, dou
a dintre punctele X, Y , Z sunt situate pe
laturi, iar al treilea pe prelungirea laturii corespunz
atoare. n acest caz, dreptele
AA1 , BB1 , CC1 nu vor putea fi concurente.
40

IX.70. Sa se arate ca tg 15 + tg 25 + tg 35 + tg 85 > 4.


D. M. B
atinetu-Giurgiu, Bucuresti
Not
a. Cu ajutorul unui tabel care da valorile tangentei, se constata ca tg 85 '
11, 43, valoare deja mai mare dect 4.

Clasa a X-a
X.66. Notam cu D multimea punctelor P (x, y) din planul xOy situate n interiorul sau pe laturile 4ABC. Fie a, b R, a2 + b2 6= 0; definim functia f : D R,
f (P ) = ax + by + c. Sa se arate ca pentru orice P D, avem
min {f (A) , f (B) , f (C)} f (P ) max {f (A) , f (B) , f (C)} .

Adrian Corduneanu, Iasi


at
am c
a valorile minim
a si maxim
a ale lui
Solutie. Fie M (1 , 1 ), N (1 , 1 ); ar
f (P ), cnd P parcurge [M N ], se ating n capetele segmentului. Daca P [M N ],
exista t [0, 1] astfel nct xP = 1 + t (2 1 ), yP = 1 + t ( 2 1 ). Atunci
f (P ) = (a1 +b 1 +c) + t [a (2 1 ) + b ( 2 1 )] = f (M ) + t [f (N )f (M )] .

Daca f (N ) f (M ) > 0, atunci f (M ) f (P ) f (N ), P [M N ]. Daca


f (N ) f (M ) < 0, atunci f (N ) f (P ) f (M ), P [M N ]. n sfrsit, daca
f (N ) f (M ) = 0, atunci f (P ) = f (M ), P [M N ].
Revenim la problema initial
a. F
ar
a a micsora generalitatea, presupunem c
a
f (A) f (B) f (C). Fie P D oarecare, iar {Q} = AP BC. Deoarece
f (B) f (Q) f (C), avem f (A) f (P ) f (C) si demonstratia este ncheiata.

X.67. Fie Q 2 = a + b 2 | a, b Q . Sa se determine functiile crescatoare




f : Q 2 (0, ) pentru care f (x + y) = f (x) f (y), x, y Q 2 .
Dan-

Stefan Marinescu si Viorel Cornea, Hunedoara


Solutie. Fie g : Q 2 R, g (x) = lg f (x); atunci g (x + y) = g (x) + g (y),

a de rezolvare a ecuatiilor
x, y Q 2 . Fie k = g (1); cu metoda obisnuit


functionale de tip Cauchy, se arat
a c
a g (x) = kx, x Q si g x 2 = g 2 x,
x Q. Pentru n N arbitrar, avem:
h i
h i

h i

h i

2 g n 2 +1
n 2 n 2 < n 2 + 1 g n 2 ng
h i
h i
n2 +1
kh i
k n 2 n g 2 n 2 +1 k
n 2 g 2
k,
n
n


deci g 2 k 2 , > 0. Astfel, rezulta ca g 2 = k 2, de unde

g m + n 2 = g (m) + g n 2 = km + kn 2 = k m + n 2 ,

adica g (x) = kx, x Q 2 , unde k (0, +). Notnd a = 10k 1, obtinem ca

x
f (x) = 10g(x) = 10k = ax , cu a 1. Pentru orice a [1, +), functia f (x) = ax
verifica ipoteza problemei, deci functiile cautate sunt cele exponentiale cu baza 1.
X.68. Pe cercul trigonometric se considera punctele A, B, C de afixe 1, , 2 ,
2
2
unde = cos
+ i sin . Fie M (z) un punct al cercului situat pe arcul BC ce nu
3
3

z2 + z + 1
contine A. Sa se arate ca z 2 + z + 1 =
.
z
Marian Tetiva, Brlad
41

Solutie. Fie z = cos + i sin , cu [2/3, 4/3]. Avem:

2
z + z + 12 = (cos 2 + cos + 1)2 + (sin 2 + sin )2 =

= cos2 2 + sin2 2 + cos2 + sin2 + 1+


+2 (cos 2 cos + sin 2 sin ) + 2 cos 2 + 2 cos =

2
= 3 + 2 cos + 2 2 cos2 1 + 2 cos = 4 cos2 + 4 cos + 1 = (2 cos + 1) .

Cum [2/3, 4/3], atunci 2 cos + 1 0, deci z 2 + z + 1 = |2 cos + 1| =


z2 + z + 1
2 cos 1. Evident ca
= 2 cos + 1, de unde concluzia.
z
X.69. Daca a, b, c > 1, sa se demonstreze inegalitatea

(a + b + c)2
3
3
3
3
3
3
.
a loga b+ loga c + b logb a+ logb c + c logc a+ logc b
3
Titu Zvonaru, Com
anesti
Solutie. Deoarece a, b, c > 1, atunci loga b > 0, loga c > 0 etc. Folosind inegalitatea mediilor, obtinem:
p
p
p
p
3
loga b + 3 loga c = 3 1 1 loga b + 3 1 1 loga c

1
1
1
3
(1 + 1 + loga b) + (1 + 1 + loga c) =
loga a2 b + loga a2 c = loga a abc.
3
3
3
Cum a > 1, b > 1, c > 1, avem:

3
3
3
3
3
3
a loga b+ loga c + b logb a+ logb c + c logc a+ logc b

logb b

3
abc

logc c

3
abc

aloga a abc + b
+c
=

(a + b + c)2
3
3
3
3
= a abc + b abc + c abc = (a + b + c) abc
.
3
Egalitatea se obtine pentru a = b = c.
X.70. Fie patratul ABCD. Sa se determine multimea

= P Int ABCD | P A2 , 2P B P D, P C 2 sunt laturile unui triunghi .

C
at
alin Calistru, Iasi

Solutie. Not
am P A = x, P B = y, P C = z,
P D = t; atunci

4 = P Int ABCD | x2 + 2yt > z 2 ,

x2 + z 2 > 2yt, z 2 + 2yt > x2 .

A
x
P

D
x
x

y
z
Aplicnd teorema medianei n 4P AC si 4P BD,
t
cum AC = BD, obtinem c
a x2 +z 2 = y 2 +t2 . Astfel, x2 +z 2 > 2yt y 2 + t2 > 2yt (y t)2 > 0 C
C
D=B
y 6= t P
/ AC. Celelalte dou
a inegalit
ati care intervin n definitia lui sunt
satisf
acute de c
atre orice punct P Int ABCD, prin urmare = Int ABCD \ [AC].
S
a demonstr
am c
a x2 + 2yt > x2 , P Int ABCD, pentru prima inegalitate

procedndu-se analog. Aplic


am p
atratului o rotatie de unghi
n jurul lui A; se
2

42

conserv
a astfel
x, y, z, t,deci 4AP P este dreptunghic isoscel
lungimile segmentelor
0
0
cu P P = x 2. Din 4P P B, obtinem:

y + t > x 2 y 2 + t2 + 2yt > 2x2 x2 + z 2 + 2yt > 2x2 z 2 + 2yt > x2 .

Clasa a XI-a
XI.66. Fie xn , n N , cel mai mic numar natural cu proprietatea ca exista M =

an an1 . . . a1 a0 (10) cu toate cifrele nenule, astfel nct M = (n+1) n+1 an an1 a0 +
xn
.
9xn . Sa se calculeze lim
n 10n
Valeriu Brasoveanu, Brlad
1
1

Solutie. xn = M (n + 1) n+1 an ani a0 [M (an + an1 + + a0 )],


9
9
cu egalitate pentru an = an1 = an1 = = a0 = k. Conditia de minimalitate
impusa lui xn conduce la

i
1
kh
xn =
kk
.
.
.
k

k
+
k
+

+
k
11
.
.
.
1

(n
+
1)
.
=
{z
}
9 | {z } |
9
n+1 cifre

n+1 termeni

10n+1 9n10
Din aceeasi conditie de minimalitate deducem k = 1, deci xn =
.
81
xn
10
=
Atunci lim
.
n 10n
81

XI.67. Fie sirul (xn )n1 definit prin x1 , 0 , xn+1 = 2xn tg xn , n 1.
4
Sa se studieze existenta limitelor lim xn si lim n xn .
n
n
Dan Popescu, Suceava

Solutie. Daca presupunem ca xn , 0 , atunci xn+1 = f (xn ), unde f :
4
h i

, 0 R, f (t) = 2t tg t este functie strict cresc
atoare. Obtinem c
af
=
4
4

+ 1 < xn+1 = f (xn ) < f (0) = 0 si deoarece + 1, 0 , 0 , rezulta ca


2
2
4

sirul (xn )n>1 este corect definit, strict cresc
ator si m
arginit. Pentru x ,
,
2 2
avem c
a tg x = x x = 0; folosind acest fapt, prin trecere la limit
a n relatia de
recurenta rezult
a c
a lim xn = 0.
n
1
ln(xn )
xn+1
n
Apoi, (xn ) = e n , n 1, iar ln (xn+1 ) ln (xn ) = ln
=
xn
2 tg xn xn
2 tg xn xn
ln
, n 1. Cum lim
= 1, cu criteriul lui Stolz-Cesro se
n
xn
xn

deduce c
a lim n xn = e0 = 1.
n

XI.68. Fie f : I R, I R interval, o functie de doua ori derivabila cu


f 00 (x) f 0 (x), x I. Sa se arate ca f (x) f (a) (exa 1) f 0 (a), x, a I.
Pentru f (x) = ex , (, 0] [1, +), sa se deduca inegalitatea lui Bernoulli.
Dumitru Mihalache, Brlad
xa
Solutie. Consideram functiag : I R, g (x)
=
1) f 0 (a).
f (x) f (a) (e
0
0
f (x) f (a)
f 0 (x)
Avem ca g(a) = 0, iar g 0 (x) = ex
a
. Functia h : I R, h (x) =
x
e
e
ex
00
x
0
x
00
0
f (x) e f (x) e
f x f (x)
are derivata pozitiv
a pe I: h0 (x) =
=
0,
e2x
ex
43

x I, deci h este monoton cresc


atoare. Deducem c
a g 0 (x) 0 pentru x > a si
g 0 (x) 0 pentru x < a, adica a este punct de minim pentru g. Cum g (a) = 0,
obtinem ca g (x) 0, x I, de unde inegalitatea dorita.
Pentru f (x) = ex , conditia f 00 (x) f 0 (x) revine la 2 , deci este satisf
acut
a
pentru (, 0][1, ). Deci are loc concluzia problemei, ce se mai scrie sub forma

ex e(1)x e(1)a (1 ) ea .
Trecem x ln x, a ln a si obtinem

x x1 a1 (1 ) a x > (1 ) a + xa1 ,

pentru orice x, a > 0 si orice (, 0] [1, ). Pentru a = 1, rezulta ca


x 1 + (x 1), (, 0] [1, ), ceea ce constituie o usoar
a mbun
at
atire
a inegalit
atii lui Bernoulli.
XI.69. Fie A, B M3 (R) astfel nct det (AX + B) 0, X M3 (R). Sa se
arate ca exista C M3 (R) pentru care A = BC.
Gheorghe Iurea, Iasi
Solutie. Pentru X = O3 , obtinem c
a det B 0. Dac
a det B 6= 0, atunci
exist
a B 1 ; fie D = B 1 A. Avem c
a det B (DX + I3 ) 0, X M
3 (R) si cum

a b c
det B > 0, deducem c
a det (DX + I3 ) 0, X M3 (R). Dac
a D = ,

u v w
x 0 0
ax + 1 0 0
1 0, cu x R oarecare.
pentru X = 0 0 0 obtinem c
a DX +I3 = x
0 0 0
ux
0 1
Atunci
det
(DX
+
I
)
=
ax
+
1

0,
x

R,
de
unde
a
= 0. Considernd acum
3

0 0 0
X = x 0 0, x R, gasim ca bx + 1 0, x R, deci b = 0 etc. Rezulta ca
0 0 0
D = O3 , adica B 1 A = O3 , de unde A = O3 . Luand C = O3 , are loc concluzia.
Daca detB = 0, sa presupunem prin absurd ca exista X0 cu det(AX0 +B) > 0.
Cum det (AX + (AX0 + B)) = det (A (X + X0 ) + B) 0, X M3 (R), folosind
cele demonstrate anterior rezult
a c
a A = O3 si atunci det B > 0, fals. R
amne c
a
a
P
,
Q
det (AX + B) = 0, X M3 (R). Pe de alta parte, din det B = 0 rezult

1 0 0
inversabile astfel nct B = P SQ, unde S este una dintre matricele 0 1 0,

0 0 0
1 0 0
0 0 0
0 0 0 sau 0 0 0. Obtinem
0 0 0
0 0 0
det (AX + P SQ) = 0, X M3 (R)

det P P 1 AX + S Q = 0, X M3 (R)

det (A1 X + S) = 0, X M3 (R) ,

1 0 0
0
n cazul n care S = 0 1 0, lund X = 0
0 0 0
0
44

unde A1 = P 1 A.

0 1
0 0 0
0 0, apoi X = 0 0 1
0 0
0 0 0

a b c
a b c
gasim ca A1 = x y z = SM , unde M = x y z . Atunci A = P A1 =
0 0 0
0 0 0
1
1
P
SM
=
P
SQQ
M
=
BC,
cu
C
=
Q
M
.
La
fel se procedeaza cnd S =

1 0 0
0 0 0
0 0 0 sau S = 0 0 0.
0 0 0
0 0 0
XI.70. Fie a, b, c laturile unui triunghi ale carui unghiuri au masurile n radiani
A, B, C si care are raza cercului nscris r. Sa se arate ca distanta
de la punctul
3
.
M (A, B, C) la planul P : ax + by + cz + r = 0 este mai mare dect
3
Sorin Puspan
a, Craiova
Solutie. S
a ar
at
am nti c
a n 4ABC are loc inegalitatea
A
B
C
A
2
2
S > (p a) + (p b) + .
()
2
2
2
Pentru demonstratie, observ
am c
a exist
a trei cercuri tangente
C
doua cte doua, cu centrele n vrfurile triunghiului; razele lor
B
sunt p a, p b, respectiv p c. Evalund aria zonei hasurate,
obtinem imediat ().
Cum A + B + C = si S = rp, relatia () se scrie echivalent

p2 2 (aA + bB + cC + r) p + a2 A + b2 B + c2 C < 0.
Daca 0, expresia de gradul II n p din stnga pastreaza semn constant, iar acesta
este +. Rezult
a c
a > 0, deci
p
aA + bB + cC + r > (a2 A + b2 B + c2 C)

s
2
2
2
aA + bB + cC + r

> a A + b B + c C 3,

a2 + b2 + c2
3
a2 + b2 + c2
la ultima inegalitate folosind inegalitatea lui Cebsev.

Clasa a XII-a

XII.66. Fie a, b R cu 0 a < b si fie f : [a, b] R o functie de doua ori


derivabila pe [a, b], cu f 00 continua. Daca
Z b
a2
b2
f (x) dx = f 0 (a) f 0 (b) + bf (b) af (a) ,
2
2
a
sa se arate ca exista (a, b) astfel nct f 00 () = 0.
Mihai Haivas, Iasi
Solutie. Avem:
Z b
b Z b

f (x) dx = xf (x)
xf 0 (x) dx =
a
a
a
"
#
b Z b x2
x2 0

00
= bf (b) af (a)
f (x)
f (x) dx =
2
a
a 2
Z
a2
b2
1 b 2 00
= bf (b) af (a) + f 0 (a) f 0 (b) +
x f (x) dx.
2
2
2 a
45

Rb
Din ipotez
a rezult
a c
a a x2 f 00 (x) dx = 0. Conform teoremei de medie, exist
a
R b 2 00
2 00
00
(a, b) astfel ncat a x f (x) dx = f () (b a) si cum 6= 0, atunci f () = 0.
XII.67. Fie f : [0, 1] R o functie cu proprietatea ca exista L 0 astfel nct
|f (x) f (y)| L |x y|, x, y [0, 1]. Sa se arate ca pentru orice primitiva F a
lui f si pentru orice x1 , x2 , . . . xn [0, 1], n N, n 2, are loc

F x1 + x2 + + xn F (x1 ) + F (x2 ) + + F (xn ) L


(xi yj )2 .

2
n
n
2n
1i<jn

Dan-
Stefan Marinescu, Hunedoara
Solutie. Cum f este lipschitzian
a, ea este continu
a si n consecinta admite primitive. Fie x, y [0, 1], x > y; atunci L (x y) f (x) f (y) L (x y), deci
functia g : [0, 1] R, g (x) = f (x)Lx este descresc
atoare, iar functia h : [0, 1] R,
h (x) = f (x) + Lx este cresc
atoare. Deducem c
a functia G : [0, 1] R, G (x) =
L
L
F (x) x2 este concav
a, iar H : [0, 1] R, H (x) = F (x) + x2 este convex
a.
2
2
Aplicnd inegalitatea lui Jensen, obtinem:

2
F (x1 ) + + F (xn ) L x21 + + x2n
x1 + + xn
L x1 + + xn

;
n
2
n
n
2
n

2
F (x1 ) + + F (xn ) L x21 + + x2n
x1 + + xn
L x1 + + xn

F
+
+
.
n
2
n
n
2
n
Folosind aceste relatii si identitatea evident
a

2
2
2
X
1
x1 + + xn
x1 + + xn
= 2
(xi xj )2 ,

n
n
n
1i<jn

obtinem concluzia.
XII.68. Fie f, g : R R, f (x) = eP (x) , g (x) = eQ(x) , unde P , Q sunt polinoame
de grad m 1, avnd coeficien
ti
a,respectiv
R n b, a, b
(0, ).
tiiRdominan
n
a) Sa se calculeze lim f (n) 0 g (x) dx
g (n) 0 f (x) dx .
n

b) Sa se studieze buna definire a sirurilor (an )n1 si (bn )n1 , unde f (an ) =
1 Rn
an
1 Rn
f (x) dx, g (bn ) =
g (x) dx si apoi sa se calculeze lim
.
0
0
n bn
n
n
Marius Apetrii, Iasi
Solutie. a) Evident ca f si g sunt strict crescatoare. n ipoteza nerestrictiva

a b, folosind regula lui lHospital pentru nedeterminari de tipul


, obtinem:

R
Rx
x
f (x) 0 g (t) dt
eP (x)Q(x) 0 eQ(t) dt
Rx
Rx
= lim
=
lim
x g (x)
f (t) dt x
eP (t) dt
0
0
Rx
Rx
[P 0 (x) Q0 (x)] 0 eQ(t) dt
P 0 (x) Q0 (x) xm1 0 eQ(t) dt
= 1 + lim

=
= 1 + lim
x
x
xm1
eQ(x)
eQ(x)
eQ(x)
=
= 1 + m (a b) lim
x (1 m) xm eQ(x) + x1m Q0 (x) eQ(x)
1
a
= 1 + m (a b)
= .
mb
b
46

b) Deoarece f si g sunt strict monotone, din teorema de medie rezult


a exis1 Rx
f (t) dt, g ( (x)) =
tenta si unicitatea functiilor si definite prin f ( (x)) =
x 0

R
R
1 x
1 Rx
1 x
1
1
g
(t)
dt,
deci

(x)
=
f
f
(t)
dt
,

(x)
=
g
g
(t)
dt
. Din
x 0
x 0
x 0
teorema de derivare a inversei, functiile si sunt derivabile, iar lim (x) =
x

lim (x) = +. Avem:

1 Rx
f (x)
2 0 f (t) dt
(x)
0 (x)
g 0 ( (x))
x
lim
=
= lim 0
= lim 0
x
x (x)
x (x)
x f ( (x))
1 Rx
g (x)
2 0 g (t) dt
x
x

1 Rx
1
f
(t)
dt
1

Q0 ( (x)) g ( (x)) f (x)


f (x) x 0
=

= lim 0

x P ( (x)) f ( (x)) g (x)


1
1 Rx
1
g
(t)
dt
g (x) x 0
Rx

m1

m1
f (x) 0 g (t) dt
(x)
ba
(x)
b
R
,
=
lim
lim
=
x
a x (x)
a b x (x)
g (x) 0 f (t) dt

(x)
= 1. Am folosit pe parcurs faptul c
a
x (x)
1 Rx
Rx
f (t) dt
f (t) dt
0
f (x)
1
x
lim
= lim 0
= lim
= lim
= 0.
x
x xf (x)
x f (x)+xP 0 (x) f (x)
x 1+xP 0 (x)
f (x)
de unde lim

XII.69. Fie f R [X] polinom reciproc de grad 4n + 2, n N , avnd radacinile


distincte, complexe si nereale. Sa se arate ca f are cel putin o radacina de modul 1.
aeru, Suceava
C
at
alin Tig
Solutie. Cum f are coeficienti reali si este reciproc, daca a C \ R este radacina
1
1
a lui f cu modulul diferit de 1, atunci a, , a,
sunt patru r
ad
acini distincte, de
a
a
module diferite de 1, ale lui f . Cum polinomul are 4n + 2 r
ad
acini, concluzia este
imediat
a.
XII.70. Fie G un grup de ordin n 4 cu proprietatea ca exista m N, 1 < m < n,
m1
subgrupuri de ordin m. Aratati ca G este abelian.
astfel nct G contine exact Cn1
Marius T
arn
auceanu, Iasi
Solutie. Consider
am submultimile lui G care contin elementul neutru e al lui G
m1
si nca m1 elemente din G\{e}. Numarul acestor submultimi este Cn1
, deci toate
aceste submultimi sunt subgrupuri ale lui G. Daca m > 2, alegem x, y G \ {e} cu
x 6= y. Cum n3 m2 1, putem alege m2 elemente din G\{e, x, y}, fie acestea
a1 , a2 , . . . , am2 . Not
am H1 = {e, x, a1 , . . . , am2 }, H2 = {e, y, a1 , . . . , am2 }. Cum
H1 si H2 sunt subgrupuri ale lui G, xa1 H1 (deoarece H1 subgrup), xa1 6= e (altfel
x = a1
H2 ), xa1 6= x (n caz contrar a1 = e) si xa1 6= ai , i = 1, m 2 (altfel
1
x = ai a1
tia la care am ajuns arat
a c
a m = 2 si a2 = e, a G,
1 H2 ). Contradic
deci G este grup abelian.
47

Solutiile problemelor pentru preg


atirea concursurilor
din nr. 1 / 2006
A. Nivel gimnazial
G96. Fie a = x12m + x12n , unde m, n N . Sa se arate ca numarul a este
divizibil cu 13, daca si numai daca x este divizibil cu 13.
Artur B
al
auc
a, Botosani
Solutie. Daca 13 divide x atunci 13 divide x12m + x12n = a. Reciproc, sa zicem
ca 13 divide a. Presupunem ca n m, adica m n = p, p N. Daca 13 nu divide
x, din faptul c
a 13 divide a = x12m + x12n = x12n x12p + 1 rezult
a c
a 13 divide
num
arul natural x12p + 1. Conform teoremei lui Fermat, orice x N care nu se
divide cu 13 are proprietatea ca x12 da restul 1 la mpartirea cu 13 si atunci x12p + 1
da la mpartirea cu 13 restul 2, ceea ce contrazice faptul ca 13 divide a. Deci, x se
divide cu 13.
G97. Determinati a, b {0, 1, 2, . . . , 9}, a 6= 0, astfel nct numarul A =
abb
. . . }b, n 2, sa fie patrat perfect.
| {z
Gheorghe Iurea, Iasi
n ori
Solutie. Dac
a b = 0, atunci A este p
atrat perfect dac
a si numai dac
a a {1, 4, 9},
iar n este par. Fie acum b 6= 0; cum b este ultima cifra a unui patrat perfect, avem
c
a b {1, 4, 5, 6, 9}. Pentru b {1, 9}, rezult
a c
a A = M4 + 3, care nu este p
atrat
perfect. Dac
a b = 5, atunci A = . . . 55 = M25 + 5, care iar
asi nu este p
atrat perfect.
Cnd b = 6, obtinem situatia nefavorabil
a A = M4 + 2.
Ramne de studiat cazul b = 4. Avem:
A = a44 . . . 4 = a 10n + 4 11 . . . 1 = 4t2 , cu t2 = a 10n2 25 + 11 . . . 1.

Pentru n 3, numarul t2 este impar, deci t = 2k + 1; deducem ca a 10n2 25 +


11
. . . 1} 0 = 4k (k + 1). Nu putem avea n 4, pentru ca ar rezulta ca 11 . . . 10 este
| {z
n1 de 1

multiplu de 4. Dac
a n = 3, atunci
2 2 2 2 2 2 2 2 2 2 2 2 2 2
A = a444 32 , 38 , 42 , 48 , 52 , 58 , 62 , 68 , 72 , 78 , 82 , 88 , 92 , 98 .

Dup
a calcule, retinem A = 382 = 1444. n sfrsit, pentru n = 2, avem

A = a44 122 , 182 , 222 , 282

si retinem A = 122 = 144.

m n+1
N .
+
n
m2
Gabriel Dospinescu, student, Paris
Solutie. Dac
a m, n verific
a enuntul, atunci exist
a p N astfel nct

G98. Sa se determine m, n N astfel nct

m3 + n2 + n = pm2 n.

(1)

Este evident c
a n dividem , adic
a exist
a k N astfel nct m = kn. Din (1)
3
deducem ca n +1+k = p k 2 n2 (k + n + 1)3 = p3 n2 k 2 . Este evident ca (n, k) = 1
si deoarece n k este cub perfect rezult
a c
a n si k sunt cuburi. Fie u, v N cu
3
3
3
u = k, v = n. Atunci m = uv si u + u3 + 1 = pu2 v 2 . Dac
a u = v, atunci
u = v = 1, deci m = n = 1. Fie acum u > v. Este evident ca (u, v) = 1 si
48

u2 v 2 pu2 v 2 = u3 + v 3 + 1 < 2u3 + 1. Deci, u

v2
. Pe de alta parte avem u2
2

v4
. De aici rezulta ca v 4. Dar v 3 + 1
4
se divide cu u2 , deci nu este liber de p
atrate. Rezult
a c
a v = 2 si u = 3. Datorit
a
simetriei rezulta si solutia v = 3, u = 2. Corespunzator acestor doua solutii obtinem
(m, n) {(1; 1) , (6; 8) , (6; 27)}.
divide v 3 + 1 si atunci v 3 + 1 u2

G99. Fie m, n doua numere naturale nenule astfel nct m divide n 1. Toate
numerele naturale ntre 1 si n se aseaza la ntmplare pe un cerc. Se calculeaza suma
oricarui grup de m numere vecine. Sa se demonstreze ca printre aceste sume exista
doua pentru care diferenta dintre ele este strict mai mare dect m 1.
Titu Zvonaru, Com
anesti
Solutie. Deoarece n 1 se divide cu m rezult
a c
a n = km + 1. Dac
a elimn
P
(n 1) (n + 2)
in
am num
arul 1, suma numerelor r
amase este
i=
. Considernd
2
i=2
sumele din cele k grupuri de cte m numere vecine, rezulta ca media acestora este
(n + 2) m
(n + 2) km
si deci cel putin una din sumele calculate, s
=
a zicem c
a
2k
2
(n + 2) m
aceasta este a, este mai mare sau egala ca media lor, adica
a. Eliminnd
2
n1
P
(n 1) n
i=
. Rationnd ca
acum numarul n obtinem suma numerelor ramase
2
i=1
nm
mai sus rezulta existenta unei sume, sa zicem b, astfel nct
b. Avem ca
2
(n 2) m nm
ab

= m, deci exist
a dou
a sume (de exemplu a si b) pentru care
2
2
diferenta lor este strict mai mare dect m 1.
G100. n cte moduri putem colora cu 5 culori un patrat 3 3, astfel nct n
fiecare patrat 2 2 sa existe patru culori diferite?
Gabriel Popa, Iasi
Solutie. Patratul 2 2 din stnga poate fi colorat n 5 4 3 2
moduri; fie A, B, C, D culorile folosite ntr-un anumit caz, ca n figur
a. A B
Dac
a n p
atr
atelul (1, 3) (adic
a linia 1, coloana 3, i.e. p
atr
atelul din C D
dreapta sus) folosim culoarea C, n patratelul (2, 3) poate fi folosita una
dintre culorile A sau E. Daca n patratelul (3, 1) folosim culoarea din (2, 3), atunci
att (3, 2) ct si (3, 3) pot fi colorate n cte dou
a moduri; dac
a nu, n (3, 1) putem
folosi 2 culori, n (3, 2) culoarea este fixat
a, iar n (3, 3) putem folosi 2 culori. Obtinem
1 2 (1 2 2 + 2 1 2) = 24 modalitati de colorare ale conturului exterior.
Daca n (1, 3) nu folosim culoarea C, putem folosi una dintre culorile A sau E
(neavnd importanta, s
a zicem c
a aceasta este A).
dac
a n (2, 3) folosim C, n (3, 1) putem utiliza A, B sau E, pentru (3, 2) r
amn
2 culori si la fel pentru (3, 3).
daca n (2, 3) nu folosim C, atunci acest patratel va fi colorat cu E. Daca n
(3, 1) utiliz
am E, avem cte dou
a modalit
ati de colorare pentru (3, 2) si (3, 3). Dac
a
n (3, 1) folosim A sau B, culoarea din (3, 2) este fixat
a, iar pentru (3, 3) avem dou
a
posibilitati.
49

Obtinem astfel 2 (1 3 2 2 + 1 1 2 2 + 1 2 1 2) = 23 5 modalit


ati de colorare ale conturului exterior.

n concluzie, patratul 33 se poate colora n 5432 24 + 23 5 = 26 357 = 6720


moduri.
G101. Sa se demonstreze inegalitatea

!
1
1
1
16
4
1+
,
2 +
2 +
2
(1 + bc) (1 + ca) (1 + ab)
a (1 + bc)
b (1 + ca)
c (1 + ab)
a, b, c (0, ) astfel nct abc = 1.

Gabriel Mrsanu si Andrei Nedelcu, Iasi


1
a
si analoagele. De asemeSolutie. Avem:

2 =
2 =
1
a (1 + bc)
(1 + a)2
a 1+ a
nea,
1
1
1

=
=
1
1
1
(1 + bc) (1 + ca) (1 + ab)
(1 + a) (1 + b) (1 + c)
1+ a 1+ b 1+ c
si atunci inegalitatea enuntului devine
!

b
c
16
a
1+
4
.
(1)
2 +
2 +
2
(1 + a) (1 + b) (1 + c)
(1 + a)
(1 + b)
(1 + c)
!

1
1
1
Adunnd n ambii termeni ai relatiei (1) pe 4
2 +
2 +
2
(1 + a)
(1 + b)
(1 + c)
obtinem inegalitatea echivalenta
!

1
1
1
1
1
16
1
4
+
+
1+
+4
2 +
2 +
2 .
1+a 1+b 1+c
(1+a) (1+b) (1+c)
(1+a)
(1+b)
(1+c)
(2)
2
2
2
Dac
a vom considera x =
,y=
,z=
inegalitatea (2) devine:
1+a
1+b
1+c
(3)
2 (x + y + z) 1 + 2xyz + x2 + y 2 + z 2 .
Conditia abc = 1 devine:

2
2
2
1
1
1 = 1 8+2 (xy + yz + zx)4 (x + y + z) = 2xyz. (4)
x
y
z
Conform cu (4), inegalitatea (3) devine:
2 (x + y + z) 1 + 8 + 2 (xy + yz + zx) 4 (x + y + z) + x2 + y 2 + z 2
1

(x + y + z)2 6 (x + y + z) + 9 0 (x + y + z 3)2 0,
ceea ce este evident.
G102. Sa se determine valoarea maxima a parametrului m R+ astfel nct
p
a2 + b2
b2 + c2 c2 + a2
+
+
m 3 (a2 + b2 + c2 ), a, b, c R+ .
a
b
c
Dorel B
aitan si I. V. Maftei, Bucuresti
pP
P b2 + c2
Solutie. Vom demonstra c
a
a2 , egalitatea fiind atins
a
2 3
a
2
2
Pb +c
P bc
2 P 2 2
a vom
pentru a = b = c. Este evident c
a
2
=
b c . Dac
a
a
abc
50

demonstra c
a
1 P 2 2 pP 2
a , atunci problema este rezolvat
a. Avem:
b c 3
abc
qX
X
2
X
X

b2 c2 3abc
a2
a2
b2 c2 3a2 b2 c2
X
X
X
X
X

b4 c4 + 2a2 b2 c2
a2 3a2 b2 c2
a2
a2 . (1)
b4 c4 a2 b2 c2
P
P 2
xy, x, y, z R nlocuim x = b2 c2 , y = c2 a2 ,
Dac
a n inegalitatea
x
2 2
z = a b , obtinem inegalitatea (1). R
aspunsul la cerinta problemei este deci m = 2.
G103. Pentru a, b, c (0, 1) cu a + b + c = 2, sa se arate ca
abc 8 (1 a) (1 b) (1 c) .

Alexandru Negrescu, elev, Botosani


Solutia 1 (a autorului). Este evident ca a < 1 a + b + c < 1 + b + c 2 <
1+b+c 1 < b+c si deci a < 1 < b+c, adic
a a < b+c si analoagele. Rezult
a c
a a, b,
c pot fi lungimile laturilor unui triunghi ABC de semiperimetru p = 1. Inegalitatea
enuntului este echivalenta cu:
abc 8 (p a) (p b) (p c) abcp 8p (p a) (p b) (p c)
4RSp 8S 2 Rp 2S Rp 2rp R 2r

ceea ce este evident.


Solutia 2 (Florian P
alita
, elev, Petro
psani). Cum 1 a > 0, 1 b > 0 si
1 c > 0, se aplica inegalitatea mediilor 2 (1 a) (1 b) 2 a b = c si nca
doua analoage. Prin nmultire membru cu membru, rezulta concluzia.
Solutia 3 (Marius Tiba, elev, Iasi). Notnd x = 1 a, y = 1 b, z = 1 c
obtinem x + y + z = 1. nlocuind, inegalitatea devine (1 x) (1 y) (1 z) 8xyz,
apoi
1 xyz + xy + xz + yz x y z 8xyz 1 + xy + xz + yz 1 9xyz

1 1 1
1
1 1
+ + 9 (x + y + z)
+ +
9,
x y z
x y z
inegalitate binecunoscuta.
\ = 120 . Fie O (BC) astfel nct [AO
G104. Triunghiul ABC are m(BAC)
\ Pe [AO se ia punctul D astfel nct [BC este
este bisectoarea unghiului BAC.
\ Sa se arate ca AD + BD = AB + AC si
bisectoarea interioara a unghiului ABD.
AB + AC 4 AO.
Petru R
aducanu, Iasi
Solutie. Fie E AD astfel nct BE este paralel
a cu
B
[ AEB
\ si m(EAC)
[ m(EAB)
\ =
AC. Deoarece EAC
60 rezulta ca triunghiul ABE este echilateral, adica AB = A
O
BE = AE.
Prelungim [AE] cu EF = AC. Deoarece (BE) (AB),
D
\) = m(BAC)
\ = 120 rezult
(EF ) (AC) si m(BEF
a c
a
E
C
tringhiurile BEF si BAC sunt congruente de unde obtinem
F
\ BF
\
\ ABC,
\ deci CBD
\ F
\
ca BCA
E si EBF
BE si
51

\ CBE.
\ Prin
BF = BC. Din paralelismul dreptelor AC si BE rezult
a ACB
\
\
\
\
\
\ =
constructie CBD ABC si atunci CBD EBF , deci m(CBD) + m(DBE)
\
\ = m(BCA)
\ = m(BF
\
m(EF
B) + m(DBE)
E). Prin urmare triunghiul BDF este
isoscel cu BD = DF . Rezulta ca AD + BD = AD + DF = AF = AE + EF =
AB + AC.
\) = 60 si din BF = BC rezult
Observ
am c
a m(CBF
a c
a triunghiul BCF este

\
a
echilateral si deci m(BCF ) = 60 . Triunghiurile AOB si ACF fiind asemenea rezult
AO
AB
AB AC
ca
=
, deci AO =
, de unde
AC
AF
AB + AC
AB + AC
2AB AC

,
(1)
2 AO =
AB + AC
2
adica 4AO AB + AC. n (1) avem egalitate ntre media aritmetica si media
armonic
a dac
a si numai dac
a AB = AC.
G105. Se considera trapezul ABCD cu bazele AB, CD ( AB > CD) si fie O
intersectia diagonalelor trapezului. Se duce linia mijlocie M N a trapezului si paralela
P Q prin O la bazele trapezului ( M, P (AB), N, Q (BC) ). Sa se demonstreze
ca trapezele ABM N si P QCD au diagonalele respectiv paralele.
Claudiu-
Stefan Popa, Iasi
Solutia 1 (a autorului). Fie E interD
C
sectia dreptelor DQ si AB. Din asem
an
ari
CD
CQ CQ
Q
imediate obtinem c
a
=
,
=
P
BE
QB QB
O
M
N
CD
CD
CD
DO OD
,
=
. Rezulta ca
=
,
OB OB
AB
BE
AB
de unde deducem c
a BE = AB.
Deoarece M este mijlocul segmentului
B
E
[AD] iar B este mijlocul segmentului [AE], A
rezult
a c
a [M B] este linie mijlocie n triunghiul ADE, atunci M B este paralel
a cu
DQ. Analog demonstr
am c
a AN este paralel
a cu P C.
2xy
,
Solutia 2 (Marius Tiba, elev, Iasi). Fie x = AB, y = CD; atunci P Q =
x+y
x+y
DP
2DP
2DP
2DO
2y
MN =
, iar
=
=
=
=
. Obtinem
2
MA
AD DP + P
DO + OB
x+y
A
DC
CQ
PQ
2y
DP
si, cum congruenta unghiurilor este
=
=
=
=
ca
MA
MN
NB
AB
x+y
\
imediata, patrulaterele DCQP si M N BA vor fi asemenea. Urmeaza ca P
DQ
\
\
\
AM B si P CQ AN B, deci DQ k M B si P C k AN .

B. Nivel liceal
L96. Fie cercurile C1 , C2 , C astfel nct C1 si C2 sunt tangente exterior n D si
fiecare dintre ele este tangent interior lui C n B, respectiv C. Tangenta comuna
interioara cercurilor C1 si C2 taie cercul C n A si A1 . Dreapta AB taie cercul C1
n K, iar dreapta AC taie cercul C2 n L. Din punctul M de pe cercul C se duc
tangentele M T1 si M T2 la cercurile C1 , respectiv C2 ( T1 C1 , T2 C2 ). Daca
52

A1 M
AM
KL si |M T1 M T2 | =
KL.
A1 D
AD
Neculai Roman, Mircesti (Iasi)
Solutie. Domnul Titu Zvonaru, Com
anesti, remarc
a faptul c
a aceast
a problem
a este ndeaproape nrudit
a cu problema L76, publicat
a de acelasi autor n RecMat 1/2005. Consideram totusi utila elevilor includerea unei solutii detaliate.
n rezolvarea acestei probleme vom folosi teorema lui Casey, pe care o prezent
am
n continuare (fara demonstratie).
Teorema lui Casey. Daca cercurile C1 , C2 , C3 , C4 sunt tangente (toate interior sau toate exterior) la cercul C, ordinea punctelor de tangenta fiind data de
numerotarea acestor cercuri, atunci are loc relatia
M BAC, aratati ca M T1 + M T2 =

d12 d34 + d23 d41 = d13 d24 ,

unde dij este lungimea tangentei comune exterioare a cercurilor Ci si Cj ( dij se


numeste distanta tangentiala a celor doua cercuri). Rezultatul ramne valabil si
daca cercurile Ci (toate sau o parte dintre ele) degenereaza n puncte sau daca cercul
C devine o dreapta.
Sa revenim acum la problema considerata.
Fie {E} = BC C1 , {F } = BC C2 , d tangenta comun
a a cercurilor C, C1 si T d
\
[ ) = m(CBT
[) =
[
astfel nct m(T BC) = m(BA1 C)/2. Deoarece m(EKB) = m(EBT
\ rezulta ca AC k KE. Analog, obtinem AB k LF .
m(BAC),
Cum AD este axa radical
a a cercurilor C1 si C2 , rezult
a c
a AK AB = AL AC,
\ ALK.
\ De
ceea ce nseamna ca patrulaterul BCLK este inscriptibil, deci ABC
\
\
\
\
aici, tinnd seama de relatiile ALK LKE (pentru c
a AC k KE) si ABC KBE,
\ KBE,
\ deci LK este tangenta cercului C1 . Analog demonstram
rezulta ca LKE
c
a LK este tangent
a si cercului C2 .
Aplic
am teorema lui Casey pentru cercurile M , C1 (O1 ), A1 , C2 (O2 ) (M si A1
degenerate) tangente interior cercului C si obtinem
dMO1 dO2 A1 + dMO2 dA1 O1 = dM A1 dO1 O2

A1 M
KL.
A1 D
Pentru a demonstra a doua relatie, aplic
a teorema lui Casey cercurilor M , A, C2 (O2 )
si C1 (O1 ):
M T1 A1 D + M T2 A1 D = A1 M KL M T1 + M T2 =

dMA dO1 O2 + dM O1 dAO2 = dMO2 dAO1 M A KL + M T1 AD = M T2 AD,

de unde concluzia.
L97. Sa se demonstreze ca n orice triunghi are loc inegalitatea
1
1
1
1
+
2.
+
m2a (mb + mc ma ) m2b (mc + ma mb )2 m2c (ma + mb mc )2
S

I. V. Maftei si Dorel B
aitan, Bucuresti
Not
a. Concomitent cu publicarea n revista noastra, problema a aparut cu
numarul 25449 n G.M. nr. 12/2005. Solutia sa poate fi gasita n G.M. 6/2006.
P
S2
9
n fapt, inegalitatea se reduce la
2 4.
2
a (p a)
53

Vlad Emanuel, elev, Sibiu, noteaz


a x = p a, y = p b, z = p c, iar
inegalitatea precedenta (demonstrata trigonometric de catre autorii problemei) revine
P (x + y + z) xyz
9
. Daca m = xy, n = xz, p = yz, avem de aratat ca
la
2
2
4
x (y + z)
P mn + mp + np
9
, m, n, p > 0, care este chiar inegalitatea 114 din excelenta
2
4
(m + n)
Old and New Inequalities, autori T. Andreescu, G. Dospinescu, V. Crtoaje, M. Lascu,
ap
arut
a n 2004 la Editura GIL.
L98. Se considera un triunghi oarecare ABC. Demonstrati ca
27 r 3
;
1) sin4 A + sin4 B + sin4 C
2 R
4
3 r
ra
rb
rc
2) cos4 A + cos4 B + cos4 C
5
5
5
,
8 R
r
r
r
unde R este raza cercului circumscris, r este raza cercului nscris, iar ra , rb , rc sunt
razele cercurilor exnscrise.
Oleg Faynshteyn, Leipzig, Germania
Solutie. 1) Avnd n vedere inegalitatea x2 + y 2 + z 2 xy + yz + zx (x, y, z R)

a
b
z
abc
S
si relatiile
=
=
= 2R, R =
, r = si p 3 3r, deducem ca
sin A
sin B
sin C
4S
p
sin4 A + sin4 B + sin4 C (sin A sin B)2 + (sin B sin C)2 + (sin C sin A)2
abc a + b + c

=
sin A sin B sin C (sin A + sin B + sin C) =
8R3
2R

p
r
27 r 3
rp2
S

.
3 3r =
=
=
2
3
3
2R R
2R
2R
2 R
2) Utiliznd iarasi inegalitatea x2 + y 2 + z 2 xy + yz + zx, obtinem
cos4 A + cos4 B + cos4 C cos A cos B cos C (cos A + cos B + cos C) .

(1)

Vom exprima acum termenul din dreapta al inegalit


atii (1) n functie de R, r, ra ,
rb , rc . n acest scop utiliz
am faptul c
a n orice triunghi are loc relatia cos A+cos B +
R+r
2R + r ra
2R + r rb
cos C =
, precum si cos A =
, cos B =
, cos C =
R
2R
2R
2R + r ra
ra r
A
2R + r rc
(ntr-adev
ar, cos A =
1 cos A =
2 sin2
=
2R
2R
2
2R

S
S
(p b) (p c)
aS
1

2
=
p (p a) (p b) (p c) =
2R p a
p
bc
2Rp (p a)
abc
S S 2 = S S).
4R

Tinnd
seama de acestea si de inegalitatea R 2r, rezult
a c
a:
R + r 2R + r ra 2R + r rb 2R + r rc

R
2R
2R
2R

3r 5r ra 5r rb 5r rc
ra
3 r 4
rb
rc

=
5
5
,
R
2R
2R
2R
8 R
r
r
r
adic
a tocmai ceea ce trebuia demonstrat.
cos4 A + cos4 B + cos4 C

Not
a. Am primit de la Neculai Roman, Mircesti (Iasi), o interesanta rafinare
54

a inegalit
atii de la b), anume
cos4 A + cos4 B + cos4 C

3
ra
3 r 4
rb
rc
5

5
5
.
16
8 R
r
r
r

9
Pentru demonstratia primei parti, folosim cunoscuta sin2 A + sin2 B + sin2 C
4
si inegalitatea CBS; obtinem
2
1 2
cos4 A + cos4 B + cos4 C
cos A + cos2 B + cos2 C =
3

2
2 1
2
3
9
1
2
2

=
3 sin A + sin B + sin C
3
.
=
3
3
4
16
Pentru partea a doua, folosim ra + rb + rc = 4R + r; ra rb + ra rc + rb rc = p2 ;
ra rb rc = p2 r; a2 + b2 + c2 = 2p2 2r2 8Rr. Avem ca a2 + b2 + c2 9R2 , de unde
2p2 2r2 + 8Rr + 9R2 , apoi
ra
3 r 4
rb
rc 3 r 4 3 ra + rb + rc 2
5
[5
5
5
=
5 +
8 R
r
r
r
8 R
r

ra rb rc
3 r 4
R
ra rb + ra rc + rb rc
p2

]
=
100

100

+
+
4
r2
r3
8 R
r
r2

r 4 63 r 3 27 r 2

3 r 4
R
2

+
.
100 100 + 2 2r2 + 8Rr + 9R2 = 39
8 R
r
r
R
2 R
4 R

63
27
r
1
Daca x =
0, , functia f (x) = 39x4 x3 + x2 are un maxim egal cu
R
2
2
4
3
1
, adica pentru x = , de unde concluzia anuntata.
16
2
L99. a) Care este numarul minim de puncte din plan de coordonate ntregi astfel
nct, oricum ar fi alese, sa existe trei puncte cu centrul de greutate de coordonate
ntregi.
b) Sa se arate ca ntr-un spatiu n-dimensional exista 2n+1 puncte de coordonate
ntregi astfel nct oricare trei dintre acestea au centrul de greutate cu cel putin o
coodonata care nu este un ntreg.
Irina Mustata
a, Bremen, Germania
, student
Solutie (Eugenia Rosu, eleva, si Adrian Zanoschi, profesor, Iasi). a) Centrul
de
greutate al triunghiului cuvrfurile n (x1 , y1 ), (x2 , y2 ), (x3 , y3 ) are coordonatele
x1 + x2 + x3 y1 + y2 + y3
,
; aceste coordonate sunt ntregi daca si numai daca
3
3
x1 + x2 + x3 si y1 + y2 + y3 sunt multipli de 3. Pentru a simplifica rezolvarea, consideram, mai departe, n locul coordonatelor punctelor, resturile acestora modulo 3.
Pentru 8 numere, rezultatul nu este valabil. ntr-adevar daca alegem sistemul
x
y

1
0

1
0

1
1

1
1

0
0

0
0

0
1

0
1

observam ca suma x1 + x2 + x3 este multiplu de 3 daca si numai daca cele trei abscise
sunt egale. Dar, n acest caz, cea de-a doua sum
a y1 + y2 + y3 este 1 sau 2, ceea
ce nseamn
a c
a, pentru orice trei dintre aceste puncte, centrul de greutate nu are
ambele coordonate ntregi.
55

S
a analiz
am, n continuare, cazul a 9 numere. Reamintim c
a dou
a coordonate
congruente modulo 3 sunt considerate egale.
Daca exista 5 punte care au una dintre coordonate identica, atunci pentru cea
de-a doua coordonat
a avem unul din urm
atoarele cazuri: apar toate cele trei resturi
posibile la mp
artirea cu 3 sau, conform principiului cutiei, unul dintre resturi apare
de trei ori. n ambele situatii putem gasi trei coordonate cu suma multiplu de 3, deci
exista trei puncte cu centrul de greutate de coordonate ntregi.
Presupumen acum c
a nu este ndeplinit
a conditia precedent
a. Atunci, nseamn
a
c
a fiecare din numerele 0, 1, 2 apare cel putin o dat
a si cel mult de patru ori att
printre abscise ct si printre ordonate. Daca doua dintre aceste numere ar aparea de
cel mult doua ori, atunci cel putin cinci coordonate ar fi egale cu al treilea numar,
ceea ce contrazice presupunerea f
acut
a. Deci, dou
a dintre numerele 0, 1, 2 apar de
cel putin 3 ori la o coordonat
a. Not
am aceste numere cu m si n.
Consideram, pentru fiecare dintre cele doua grupuri cu prima coordonata m,
respectiv n, ca cea de-a doua coordonata ia cel putin doua valori distincte (astfel
exist
a trei puncte identice si centrul lor de greutate are coordonatele ntregi). Dac
a
avem toate cele trei valori (0, 1, 2) la ordonate, atunci suma lor este 3 si centrul de
greutate respectiv are coordonatele ntregi. Cazul cel mai nefavorabil ar fi, asadar,
cel n care n ambele grupuri, ordonatele iau dou
a valori a si b pentru primul grup si
c si d pentru cel
alalt grup (a 6= b, c 6= d).
ntruct a, b, c, d {0, 1, 2}, conform principiului cutiei, rezult
a c
a exist
a dou
a
numere egale. Cum a 6= b si c 6= d, nseamna ca unul dintre numerele a, b este egal
cu c sau d. Putem presupune, f
ar
a a restrnge generalitatea, c
a a = c.
S
a ar
at
am c
a sumele a + c, a + d, b + c, b + d iau cele trei valori posibile 0, 1, 2.
Avem relatiile a 6= b a + c 6= b + c, c 6= d b + c 6= b + d, b 6= a b + d 6= a + d.
Astfel, daca cele patru sume ar lua doar doua valori din cele trei, atunci a + c = b + d
si b + c = a + d, ceea ce implic
a c = d. Contradictia la care am ajuns dovedeste c
a
fiecare dintre numerele 0, 1, 2 se afl
a printre cele patru sume.
Acum, s
a revenim la cel de-al treilea grup - al punctelor care au ca prim
a coordonata un numar diferit de m si n (exista cel putin un punct de acest fel). Ordonata
unui astfel de punct adunat
a cu una din sumele a + c, a + d, b + c, b + d va da un
multiplu de 3, pentru c
a printre aceste sume se afl
a toate numerele 0, 1, 2.
Evident, punctele din primele dou
a grupuri cu ordonatele din suma g
asit
a mai
sus si cu punctul ales din grupa a treia formeaza un triplet a carui centru de greutate
are coordonate ntregi. Cu aceasta, demonstratia este ncheiat
a.
b) Vom demonstra propozitia prin inductie dup
a n. n cazul n = 2, am g
asit la
punctul a) un exemplu de 23 puncte care satisfac conditia din enunt.
Presupunem propozitia adevarata pentru un numar natural n 2. Sa aratam
c
a propozitia este adev
arat
a pentru n + 1. Conform ipotezei de inductie, exist
a
2n+1 puncte n spatiul n - dimensional astfel nct centrul de greutate al oric
aror
trei dintre ele sa nu aiba toate coordonatele ntregi. Celor 2n+1 puncte de mai
sus le mai adaugam la sfrsit nca o coordonata egala cu 0 si apoi, din nou tuturor
punctelor, nc
a o coordonat
a egal
a cu 1. Astfel obtinem 2n+2 puncte din spatiul n+1
dimensional (jum
atate dintre ele se termin
a cu 0, iar cealalt
a jum
atate se termin
a
cu 1). Se observa usor ca, oricum am alege trei dintre aceste 2n+2 puncte, centrul
56

lor de greutate nu are toate coordonatele ntregi.


Not
a. Solutie corecta s-a primit de la Vlad Emanuel, elev, Sibiu.

1 2
L100. Fie x (0, 1); aratati ca exista n N astfel nct {nx}
,
.
3 3
Ciprian

Baghiu si Gheorghe Iurea, Iasi


1
2
1
1
Solutia 1(a autorilor). Daca x 0,
, cum

=
> 1, exista
3x
3x 3x

3
1 2
1 2
1 2
n N cu n
,
, deci nx
,
, adica {nx}
,
. Pentru
3x
3x
3
3
3 3

1 2
2
x
,
putem alege n = 1, iar pentru x = lu
am n = 2.
3 3
3

2
2
2 3
3 4
R
amne de analizat situatia x
, 1 . Cum
,1 =
,
,

3
3
3 4
4 5

k
k+1
k
k+1
,
. . . , exist
a k 2 astfel nct x
,
. Cnd k = 3p,
k+1 k+2
k + 1 k + 2

3p
3p + 1
2p
2p + 1
ax
p N , obtinem c
,
, deci (p + 1) x p +
,p +
,
3p + 1 3p
+2
3p + 1
3p + 2

2p
2p + 1
1 2
adic
a {(p + 1) x}
,

,
. Cnd k = 3p + 1, p N , vom avea
3p
+
1
3p
+
2
3
3

2p + 1
2
2p + 1 2
1 2
ca (p + 1) x p +
, p+
, deci {(p + 1) x}
,

,
. n
3p + 2
3
3p + 2 3
3 3

2p + 1
2p + 2
sfrsit, daca k = 3p + 2, p N, atunci (p + 2) x p + 1 +
,p + 1 +
,
3p + 3
3p + 4

2p + 1 2p + 2
1 2
deci {(p + 2) x}
,

,
.
3p + 3 3p + 4
3 3
Solutia 2 (Vlad Emanuel, elev, Sibiu). Daca x R \Q, multimea {{nx} | n N}
este densa n [0, 1] (Lema lui Kronecker) si atunci concluzia este imediata. Daca
p
x Q , fie x = , cu p, q N , (p, q) = 1; observam ca ip, i = 1, q, parcurge toate
q


ip
ri
ip
= , deci expresia
,
resturile modulo q. Not
am ri = ip (mod q); atunci
q
q
q

1 2
qq
i = 1, q, va lua toate valorile din multimea 0, , , . . . ,
. Pentru a rezolva
q q
q
m
2
1

< ,
problema, ar fi suficient s
a g
asim m {1, 2, . . . , q 1} astfel nct
3
q
3
q
2q
adic
a m<
. Cum x (0, 1), atunci q 2. Pentru q {2, 3}, lu
am m = 1.
3
3

q
q
2q
2q
> 1 si astfel exist
a cel putin un ntreg n
,
. Acel ntreg
Pentru q 4,
3
3
3 3
2q
2q
; dac
a q {4, 5}, atunci

/ N, iar dac
a q 6, exist
a cel putin doi
nu este chiar
3
3
ntregi n intervalul respectiv.
n1
Q an ak
Z.
L101. Fie a, n 2 doua numere ntregi. Sa se arate ca
k=0 n k
Adrian Zahariuc, elev, Bac
au
57

Solutie. S
a facem mai nti cteva notatii. Fie A =
n
Q

k=1

n1
Q
k=0

an ak si E =

a 1 . n acest caz, avem A = a a2 . . . an1 E. Daca p este un divizor prim


k

al num
arului m N , atunci not
am cu expp m exponentul lui p din descompunerea
n factori primi a lui m.
n1
Q an ak
A
Pentru a demonstra ca
=
Z este suficient sa aratam ca, pentru
n! k=0 n k
orice num
ar prim p, p n, are loc inegalitatea
expp n! expp A.

(1)


X
n
, oricare ar fi num
arul prim p.
expp n! =
pk

(2)

Se verific
a usor c
a

k=1

Fie p un num
ar prim mai mic sau egal ca n.
n(n1)
I. Daca p | a, atunci p2 | a2 , p3 | a3 , pn | an si p 2 | a a2 . . . an1 E = A.
Notnd cu l cel mai mare num
ar natural cu proprietatea p2 n, avem

l
X
n
n (n 1)
1
n
expp n!
=
1 l
expp A,
l
p
p1
p
2
k=1

deci, relatia (1) este adevarata.

k
II. Presupunem, n continuare, ca (a, p) = 1. Deoarece a(p ) 1 mod pk

k
k1
1, deci pk |
(teorema lui Euler) si pk = pk pk1 , rezulta ca pk | ap p
m(pk pk1 )
a
1, oricare ar fi m N .
S
a demonstr
am c
a are loc inegalitatea

X
expp E
k=1

n
.
pk pk1

(3)

am multimile Ak = m pk pk1 | m N
Pentru fiecare k N , consider
si Bk = Ak {1, 2, . . . , n}. Se observ
a c
a B1 B2 Bk si |Bk | =
k
k1
n
. Dup
a cum am v
azut, pk | am(p p ) 1, deci fiecare element al lui
pk pk1
Bk contribuie cu cel putin k la expp E. Fie Ck = Bk Bk+1 ntruct multimile Ck
sunt disjuncte doua cte doua, putem scrie:


X
X
X
X

k |Ck | =
k (|Bk | |Bk+1 |) =
|Bk | =
expp E
pk pk1 ,
k=1

k=1

k=1

k=1

ceea ce nseamn
a c
a inegalitatea (3) este adev
arat
a.
n sfrsit, din relatiile (2) si (3), rezult
a c
a
X



X
n
n

expp E expp A,
expp n! =
pk
pk pk1
k=1

k=1

deci inegalitatea (1) este adevarata.

58

Not
a. Aceeasi solutie a dat Vlad Emanuel, elev, Sibiu.
L102. Fie p = 2k + 1 un numar prim. Atunci
2k
k
X
X

i
i
Cp+i1
2p 2 mod p2 , S2 =
Cp+i1
2 2p mod p2 .
S1 =
i=1

i=k+1

Marius Pachitariu, elev, Iasi

a i {1, 2, . . . , p 1}, atunci este


Solutie. Consider
am inelul Zp2 , +, . Dac
p
un element inversabil al inelului. Notam cu reprezentantul canonic al clasei p 1 .
i
p
p
Cu acest sens dat scrierii , avem N. Astfel, putem scrie
i
i
i1

p (i 1)!
Mp2 + (1)
p (p 1) . . . (p i + 1)
=
=
i!
i!

Mp2
p
p
+ (1)i1 (1)i1
mod p2 ,
i!
i
i

p
i
unde i {1, 2, . . . , p 1}. Analog, obtinem relatia Cp+i1

mod p2 , pentru
i
orice i {1, 2, . . . , 2k}. Avem

p1
X
1 1
1
Cpi p 1 +

2 3
p1
i=1

1
2 2
2
1 1

p 1 + + + +
2 3
p1 2 4
p1

2k
X

1
1
1
i
Cp+i1
mod p2 .
+
+ +

p
k+1 k+2
2k
Cpi =

i=k+1

Deoarece

p1
P
i=1

Cpi = 2p 2, rezulta ca

2k
P

i=k+1

i
Cp+i1
2p 2 mod p2 .

1 1
1
Pentru a demonstra a doua cerinta, observam ca 1+ + + +
0 (mod p),
2 3
p1
ntruct termenii sumei dau resturi distincte la mpartirea cu p si suma acestor resturi
p (p 1)
0 (mod p). Prin urmare
este 1 + 2 + + (p 1) =
2

p1
X
1 1
1
2 2
2
Cpi = 2p 2 p 1 + + + +

2 3
p1 2 4
p1
i=1

k
X

1
1
i
Cp+i1
mod p2 ,
p 1
=
2
k
i=1

p
2
ceea ce nseamn
a c
a S2 2 2 mod p .

Not
a. Solutie asemanatoare a dat Vlad Emanuel, elev, Sibiu.

L103. Fie a, b, c, d reale astfel nct 1 + a2 1 + b2 1 + c2 1 + d2 = 16.


Aratati ca
3 ab + bc + cd + da + ac + bd abcd 5.
59

Mai mult, avem egalitate n cel putin una din inegalitatile de mai sus daca si numai
daca a + b + c + d = abc + bcd + cda + dab.
Gabriel Dospinescu, student, Paris
Solutie. Egalitatea din ipotez
a este echivalent
a cu
Y
Y
(i + a)
(i a) = 16

X
X
X
X
X
X
1i
a
ab + i
abc + abcd 1 + i
a
ab i
abc + abcd = 16.
Deoarece ultima egalitate se poate scrie n forma
2 X

2
X
X
1
ab + abcd +
a
abc = 16,
()
P
rezult
a c
a |1 ab + abcd| 4, de unde obtinem 3 ab + bc + cd + da + ac +
bd + abcd 5. Se
Pobserva ca, n relatia precedenta, putem avea o egalitate daca si
numai
dac
a
|1

ab + abcd| = 4, ceea ce, n virtutea identitatii (), este echivalent

P
P
cu
a = abc.
Not
a. Vlad Emanuel, elev, Sibiu, remarc
a faptul c
a problema apare si n Old
and New Inequalities (citat
a la solutia problemei L97), semnat
a de acelasi autor.
n
L104. Fie x0 > 0 si xn = x[ n ] + x[ n ] + , pentru orice n > 0.
3
6
x 2
n
este convergent la 1.
a) Sa se arate ca sirul
n n1
5
xn n
= 0.
b) Sa se arate ca daca > log3 , atunci lim
n
2
n
Gabriel Dospinescu, student, Paris
n
x2 o
Solutie. a) Fie M = max 1, x1 ,
. Vom demonstra prin inductie ca xn < nM ,
2

a modului n care a fost ales num


arul M , rezult
a c
a
oricare ar fi n N . Datorit
propozitia este adevarata pentru n = 1 si n = 2. hDac
a inegalitateah este
adevarata
i
i
n
n
pentru orice k n 1 (n 3), atunci din x[ n ]
M si x[ n ]
M , obtinem
2
3
2
3
hni
hni

n n n
n
xn
M+
M+
+ +
M = nM . Drept urmare, avem xn < nM ,
2
3
6
2
3
6
x
n
a c
a sirul
este m
arginit. Egalitatea
oricare ar fi n N , ceea ce nseamn
n n1
din enunt poate fi scris
a n forma
n
n
x[ n ]
x[ n ]
1
x
2
3
2

= n
+ n 3 + .
n
n
n
6
2
3
n
n
1
1
2

De aici, tinnd seama de relatiile lim


= , lim 3 = si lim (an + bn )
n n
2 n n
3 n
lim an + lim bn , deducem ca
n

xn
xn
lim
lim
xn
1
n n
n n
lim

+
+ ,
n n
2
3
6
xn
1. Analog, demonstram ca lim xn 1. Din relatiile
de unde obtinem lim
n n
n
xn
xn
xn
xn
si lim
lim
lim
1 lim
rezulta ca cele doua limite extreme
n n
n n
n n
n n
60

sunt egale cu 1, deci lim

xn
= 1.
n

k
n
x2 o
5
si ak = 1 + (M 1)
, k 0.
b) Consideram, din nou, M = max 1, x1 ,
2
6
S
a demonstr
am, prin inductie dup
a k, inegalitatea xn nak , oricare ar fi n 2 3k .
Pentru k = 0, am dovedit c
a propozitia este adev
arat
a la punctul
a). Presupunem
c
a
hni
hni
k+1
k
k
relatia este adev
arat
a pentru k. Fie n > 2 3 . Atunci,
2 3 si
23 .
2
3
Conform ipotezei de inductie, rezult
a c
a
hni
hni
n
5ak + 1
xn
ak +
ak + n
= nak+1 ,
2
3
6
6
ceea ce ncheie induc
htia. n i
Fie n > 1 si k = log3
. Cum
2
log3 n2 1
k
5
5
xn
1 + (M 1)
=
1 + (M 1)
n
6
6
log3 n
6 (M 1) 5
6 (M 1) n log3 56
2
=1+
,
=1+
5
6
5
2
nseamn
a c
a exist
a o constant
a a 0 astfel nct
5
xn
(1)
1 + anlog3 6 , oricare ar fi n > 1.
n
k+1
5
si dk = 2k+1 2. Vom ar
Consider
am sirurile ck = 1
ata, prin inductie,
6
n
c
a xn > nck dk , oricare ar fi n 2 3k . Pentru k = 0, inegalitatea devine xn > ,
6
n 2, care este adev
arat
a. Presupunem propozitia adev
arat
a pentru k. Alegem un
num
ar n 2 3k+1 si atunci, conform ipotezei de inductie, avem

hni
hni
n
n
5n
ck dk +
ck dk + >
2 ck 2dk + =
xn >
2
3
6
6
6
5ck + 1
n 2ck 2dk > ck+1 n 2 2dk = ck+1 n dk+1 .
=
6
Cu aceasta, demonstratia prin
tie s-a terminat.
h induc
ni
Acum, luam din nou k = log3
. Deoarece
2
k+1
log3 n2
n
dk
2 2k
2 2log3 2
5
2
5
xn

> ck
=1
+ >1
,
n
n
6
n
n
6
n
rezulta ca
log3 56
log3 56

x
6
12
1
2 n log3 2 log3 6
1
n
5 =

n
21log3 2 nlog3 15 .
1 nlog3 5 >
n
2
n 2
2
x

6
n
arginit,
De aici, avnd n vedere si relatia (1), deducem c
a sirul
1 nlog3 5 este m
n

xn n
xn n
este m
arginit. Prin urmare, lim
= 0.
deci sirul
5
log
n
3
nx
2
n
n1
61

L105. Sa se determine toate functiile continue f : (0, ) R, care verifica


ecuatia functionala
nxn1 f (xn ) = (x + 1) f (x) ,

unde n N , n fixat.

x (0, ) ,

Marian Tetiva si Dumitru Mihalache, Brlad


Solutie. Pentru n = 1, ecuatia considerata devine f (x) = (x + 1) f (x), x > 0.
Solutia ei, n acest caz, este f (x) = 0, x > 0.
Presupunem, n continuare, ca n 2. Ecuatia data este echivalenta cu
x+1
f (xn ) =
f (x) , x > 0,
n1
nx

de unde, nlocuind succesiv pe x cu n x, obtinem:

n
n2

x + 1
x + 1 n
2
n
n
f (x) =
x , f x =
x ,...,
n1 f
n1 f
nx n
nx n2

nk
x + 1 n
n1
k
f n
x =
x .
n1 f
nx nk
De aici, deducem c
a

2
k

( n x + 1) n x + 1 n x + 1 n
k
n
f
x , k N .
f (x ) =
n1
n1
n1
nk x n + n2 ++ nk

n1
n1
n1
1
1
1
n1
+ +
=
Deoarece
+
1 k : 1
= 1 k,
2
k
n
n
n
n
n
n
n
rezulta ca

2
k

( n x + 1) n x + 1 n x + 1 n
k
f
x , k N .
(1)
f (x) =
1
1
nk x nk

1
k
k
Avem lim x1 nk = x si lim n x = f lim x1/n = f (1). Sa calculam acum
k
k
k
n

2
k
n
( x + 1)
x + 1 n x + 1
.
L = lim
k
nk

2
k
a) Daca n = 2, atunci putem restrnge produsul P = ( x +1) 2 x +1 2 x +1 :

k
2
k
k
x + 1 2 x+ 1 2 x + 1 2 x 1 =
P 2 x1 =

2
k1
k1
=
x + 1 2 x+ 1 2
x+1 2
x 1 = =

=
x+1
x 1 = x 1,
x1
, pentru x 6= 1. Drept urmare,
deci P = 2
k
x1

2
k
( x + 1) 2 x + 1 . . . 2 x + 1
x1
x1
L = lim
= lim 1/2k
=
, x 6= 1.
1
k
k x
2k
ln x
k
1/2

n acest caz, trecnd la limit


a n relatia (1), pentru k , obtinem:
x1
f (1) , x 6= 1.
f (x) =
x ln x
62

Notam pe f (1) cu c (c R). Cum lim f (x) = lim


x1

x1

solutia ecuatiei considerate este:

x1
c = c, nseamna ca
x ln x

( x1
c, x 6= 1
f : (0, ) R, f (x) =
, unde c R .
x ln x
c,
u=
6 1

b) S
a vedem ce se ntmpl
a dac
a n > 2. Exist
a l N , astfel nct n > 2l , deci
1
1
< li , i = 1, k. Daca x > 1, atunci
ni
2



2
k
l
2l
lk
n
x + 1 n x + 1 n x + 1 < 2 x + 1 2 x + 1 2 x + 1 <

2
3
lk
< 2 x + 1 2 x + 1 2 x + 1 2 x + 1 .
Astfel, avem

2
k
( n x + 1) n x + 1 n x + 1
0
<
nk
2

2
3
lk
x + 1 2 x+ 1 2 x + 1
( x + 1)
<
=
nk
1
x1
x1
k = 1/2kl k .
= 2kl

x
1 n
x1 n
kl
l
1/2

n k
x1/2 1
= ln x si lim
= , deducem egalitatea
kl
k
k 2l
1/2
L = 0, unde x > 1, deci f (x) = 0, x > 1.
1
Dac
a x (0, 1), not
am pe cu y si atunci y > 1. Se observ
a c
a
x

1
1
1
2
k
n
x+ 1 n x+ 1 n x + 1 =
+
1
+
1

+
1
=

n y
n2 y
nk y

n y + 1
n2 y + 1 nk y + 1
n y + 1
n2 y + 1 nk y + 1
=
=
=
1
1
1
1 11/nk
y n y n2 y nk
y n 11/n

n y + 1
n2 y + 1 nk y + 1
=
.
11/nk
y n1
Asadar, avem

n y + 1
n2 y + 1 nk y + 1
1
= 0 1 = 0,
L = lim
11/nk
k
n1
y
nk y n1
kl

De aici, tinnd cont c


a lim

de unde, rezulta ca f (x) = 0, oricare ar fi x (0, 1).


Deoarece f este continu
a n x = 1, conchidem c
a solutia problemei, n acest caz,
este
f : (0, ) R, f (x) = 0.

63

Probleme propuse1
Clasele primare
P.124. Schimba locul unui singur betisor pentru a obtine o egalitate.
(Clasa I )
Mariana Nastasia, elev
a, Iasi
P.125. ntr-o clasa cu 24 elevi sunt 3 perechi de gemeni. La sedinta cu parintii
este prezent cte un singur p
arinte din fiecare familie. Cti p
arinti particip
a la
sedinta?
(Clasa I )
Mihaela Glc
a, elev
a, Iasi
P.126. Pentru a fierbe un ou sunt necesare 4 minute. Mama vrea s
a fiarb
a 20
oua n trei transe. Cte minute sunt necesare?
(Clasa a II-a)
Ionela B
ar
agan, elev
a Iasi
P.127. Mircea are cu 35 timbre mai mult dect fratele sau, Marius. Ct devine
diferenta, dac
a Mircea ar mai primi 10 timbre, iar Marius ar da unui prieten 5 timbre?
(Clasa a II-a)
Inst. Maria Racu, Iasi
P.128. Cte numere de forma RM AT ndeplinesc conditia RAM = M AT ?
(Clasa a III-a)
Dragos Covrig, elev, Iasi

P.129. Scrieti toate adunarile de forma

MARI +
ARI
RI
.
I
7676

(Clasa a III-a)

Dana Brsan, elev


a, Iasi

P.130. Dac
a a, b, c sunt cifre, cte egalit
ati de tipul a c = b : c se pot scrie?
Justificati raspunsul.
(Clasa a III-a)
Adina Voinescu, elev
a, Iasi
P.131. Verificati daca afirmatia "A se mparte exact la 5, unde A = 2000 +
2 (1 + 2 + 3 + + 1999) + 1999 + 1997" este adev
arat
a sau fals
a.
(Clasa a IV-a)
Prof. Nicolae Iv
aschescu, Craiova
P.132. Mama Oanei a mplinit 17532 zile pe data de 1 ianuarie 2007. n ce an,
lun
a si zi a avut o vrst
a de 3 ori mai mic
a?
(Clasa a IV-a)
nv. Geta Cretu, Vaslui
P.133. Doi elevi spun pe rnd cte un numar natural, cel putin egal cu 1 si cel
mult egal cu 7. Fiecare nou numar spus se aduna la celelalte. Sa se arate ca primul
elev poate s
a indice n asa fel numerele nct s
a ajung
a primul la suma 99.
(Clasa a IV-a)
Prof. Petru Asaftei, Iasi

Clasa a V-a
V.76. Dac
a a, b, x sunt cifre n baza 10, s
a se rezolve ecuatia cu necunoscuta x:
bxa + baa + xb + ab = abb + aab.
Marius Farcas, Iasi
1

Se primesc solutii pn
a la data de 31 decembrie 2007.

64

a se determine cte numere de trei cifre distincte abc au proprietatea c


a
V.77. S
abc cba : 11 este patrat perfect.
Otilia Nemes, Ocna Mures (Alba)
V.78. Ar
atati c
a nu exist
a trei numere prime a, b, c astfel nct a (b + c) = bc.
Nicolae Iv
aschescu, Craiova

V.79. Ar
atati c
a num
arul 131000 91000 se divide cu 1000.
Damian Marinescu, Trgoviste
V.80. Dac
a restul mp
artirii unui num
ar natural la 10 este mai mare dect 5,
spunem ca acel numar este favorabil. Aflati numerele favorabile ab cu proprietatea
ca nici ab, nici ba nu pot fi scrise ca suma de doua numere favorabile.
Ioan S
ac
aleanu, Hrl
au

Clasa a VI-a
VI.76. Determinati a, b, c Z dac
a

a
b
3c + 5
= =
.
3
4
2c + 1

Gheorghe Iurea, Iasi

VI.77. S
a se arate c
a ntre oricare dou
a puteri naturale consecutive ale lui 3 se
afl
a cel putin o putere a lui 2. Exist
a dou
a puteri consecutive ale lui 3 ntre care s
a
putem gasi trei puteri ale lui 2?
Marius Damian, Br
aila
VI. 78. Fie a, b Z astfel nct multimile {a + b, a + 2b, . . . , a + 2007b} si
{1, 2, . . . , 2007} coincid. S
a se arate c
a exist
a k N pentru care a + kb = k.
Dan Nedeianu, Drobeta-Tr. Severin
VI.79. Se consider
a 4ABC ascutitunghic, iar M un punct n planul s
au. Paralela prin M la AB taie AC si BC n P , respectiv N . Demonstrati ca daca doua
dintre urmatoarele afirmatii sunt adevarate, atunci este adevarata si a treia:
\ (ii) M C M B; (iii) [N P ] linie mijlocie n
(i) BM bisectoare pentru ABC;
4ABC.
Carmen-Daniela Tamas, Brlad
VI.80. Sa se demonstreze ca portiunea hasurata din figura
al
aturat
a poate fi scris
a ca reuniune de segmente nchise, dou
a
cte dou
a disjuncte.
Marius Tiba, elev, Iasi

Clasa a VII-a
VII.76. Aflati numerele naturale a, b, c pentru care 11 (a b 9) > c (c 20),
11 (b c 9) > a (a 20) si 11 (c a 9) > b (b 20).
Veronica Pl
aesu si Dan Pl
aesu, Iasi
VII.77. Fie x, y numere reale pozitive, ambele subunitare sau ambele suprau1
1
1
+ 2 x + + y + . Dac
a unul dintre numere este
nitare; s
a se arate c
a xy +
xy
x
y
mai mic, iar cel
alalt mai mare ca 1, inegalitatea si schimb
a sensul.
Marian Tetiva, Brlad
VII.78. S
a se rezolve n numere naturale ecuatia 6a 5b = 1.
Tudor P
adurariu, elev, Onesti
65

VII.79. Fie ABCD un patrulater convex, iar E si F intersectiile bisectoarelor


b respectiv B,
b cu diagonala [AC]. Sa se arate ca punctele E si F coincid
unghiurilor D,
daca si numai daca AB CD = AD BC.
Claudiu-
Stfean Popa, Iasi
VII.80. Fie ABCDEF un hexagon regulat nscris ntr-un cerc, iar P un punct
pe arcul mic BC. S
a se arate c
a P E + P F = P A + P B + P C + P D.
Dan Radu, Bucuresti

Clasa a VIII-a
VIII.76. Fie ABCD un trapez cu AB k CD, M (AD) si N (BC) cu
M N k AB, iar E (AB), F (CD) oarecare. Fie {O} = EF M N , [OP ] si
[N T ] perpendiculare de aceeasi parte pe planul trapezului, G centrul de greutate al
4P EF , {Q} = M G (T BC). S
a se arate c
a M N este linie mijlocie n trapez dac
a
si numai dac
a Q TN.
Bogdan Raita
, elev, Iasi

VIII.77. Pentru x, y R+ , sa se demonstreze inegalitatea


2


x6 + y 6 x2 y 2 x2 + y 2 x3 + y 3 xy (x + y) .
Lucian Tutescu, Craiova si Gheorghe Nedelea, Pitesti
VIII.78. Pentru a, b, c R, s
a se demonstreze inegalitatea
p
p
p
a2 + b2 ab + b2 + c2 bc + c2 + a2 ca a + b + c.

Claudiu-
Stfean Popa, Iasi
VIII.79. S
a se rezolve n numere naturale ecuatia x (x + 1) = y 2007 .
Alexandru Negrescu, elev, Botosani
VIII.80. S
tiind ca 1 ianuarie 2007 este ntr-o zi de luni, sa se arate ca pna n
anul 2100 exist
a trei ani bisecti n care luna februarie are trei duminici care cad n
zile impare.
Petru Asaftei, Iasi

Clasa a IX-a
3
2
n
n
n
IX.76. Fie d1 , d2 , . . . , dk divizorii
num
3narului 5 7 , iar Sn = d1 + d2 + + dk ,
4n
5 +1 7 +1
a se arate c
a S2n =
Sn , n N . Generalizare.
n N . S
(5n + 1) (7n + 1)
Petru Asaftei, Iasi
p
3
3
2
2
IX.77. Sa se arate ca a + b ab 2 (a + b ), a, b 0.
Ovidiu Pop, Satu Mare
IX.78. Fie a, b, c laturile 4ABC, iar G centrul s
au de greutate. Not
am cu D,
E, F punctele de contact ale cercului nscris cu laturile BC, CA, respectiv AB. S
a

se arate ca aGD + bGE + cGF = 0 daca si numai daca 4ABC este echilateral.
Marian Urs
arescu, Roman
IX.79. Fie 4ABC echilateral si P un punct n interiorul sau. Consideram
A1 AB, B1 BC, C1 CA astfel nct P A = P A1 , P B = P B1 si P C = P C1 .
S
a se arate c
a P este centrul de greutate al 4A1 B1 C1 .
Iulia Plesca, elev
a, Iasi

66

IX.80. Fie , , , patru numere pozitive cu suma . S


a se afle maximul sumei
S = sin sin + sin sin si sa se determine situatia n care acest maxim este atins.
Adrian Corduneanu, Iasi

Clasa a X-a
X.76. Fie C = {z C | |z| = 1}. S
a se rezolve n C 2 ecuatia z1 z2 = z1 + z2 + 3.
Gabriel Popa si Paul Georgescu, Iasi
X.77. Fie a, b C si z1 , z2 solutiile ecuatiei z 2 az + b = 0. S
a se arate c
a
urmatoarele afirmatii sunt echivalente:

(ii) |a|2 + a2 4b < 2 |b|2 + 1 < 4.


(i) |z1 | < 1 si |z2 | < 1;
Marian Tetiva, Brlad
X.78. Determinati triunghiurile n care tangentele unghiurilor se exprim
a prin
numere naturale, exact doua dintre ele avnd aceeasi paritate.
C
at
alin Calistru, Iasi

X.79. Sa se arate ca n orice triunghi are loc inegalitatea (p r 2R) (p ra )


(p rb ) (p rc ) 0. Cnd se atinge egalitatea?
I. V. Maftei si Dorel B
aitan, Bucuresti
X.80. Aratati ca exista o infinitate de valori n N pentru care numerele 2n + 1
si 3n + 1 sunt p
atrate perfecte.
Gheorghe Iurea, Iasi

Clasa a XI-a
XI.76. Dac
a A M4 (R), s
a se arate c
a det (A + t A i) = det (A t A i).
Generalizare.
Dan Popescu, Suceava
XI.77. Fie f : [a, b] R o functie de dou
a ori derivabil
a pe [a, b], cu ambele derivate strict pozitive. Pentru [a, b], consideram punctele A (a, f (a)),
B (b, f (b)), C (, yC ) Gf si D (, yD ) AB. Demonstrati ca exista si este unic
0 a+b
2 , b astfel nct f (b) yD = yC f (a).
aeru, Suceava
C
at
alin Tig
XI.78. Pentru x R+ , sa se demonstreze inegalitatile:
1
1
a) ln x + a (1 + ln a), unde a > 0;
x
a
k
b) ax > (1 + x) , unde a > ek , k N , iar {1}.
Gheorghe Costovici, Iasi

XI.79. Fie (xn )n1 un sir convergent, a c


arui limit
a o not
am L (xn ). Demonstrati

n
xn
n
= a R+ dac
a si numai dac
a exist
a lim (1 + xn L (xn )) =
c
a exist
a lim
n L (xn )
n
b R+ . Ce legatura este ntre a si b?
D. M. B
atinetu-Giurgiu, Bucuresti
XI.80. Fie f : [0, 1] R o functie continua pe [0, 1], derivabila pe
cu
(0, 1), x
< M xe ,
f (0) = 0. Presupunem c
a exist
a M > 0 astfel nct f 0 (x) 1x
f
(x)
x
x (0, 1). S
a se arate c
a f este derivabil
a n origine.
Mihai Cr
aciun, Pascani
67

Clasa a XII-a
XII.76. Fie functia f : [a, b] R si n N . Aratati ca exista c (a, b) astfel
Z b
nct
f (c) + f (a) + f (b) (b a)n
(f (x) + f (a) + f (b)) dx =

.
n1
n
(c a)
a
Dumitru Mihalache, Brlad
R 1/nk

am sirurile an = 1/(n+1)k arcsin nk x dx,


XII.77. Fie k N fixat. Consider

R 1/nk
an
n 1 si bn = 1/(n+1)k arctg nk x dx, n 1. S
a se calculeze lim
.
n bn
Liviu Smarandache si Lucian Tutescu, Craiova
XII.78. Daca f : R R este o functie continua si fara puncte fixe, sa se arate
ca nici functiile f f f , n N , nu au puncte fixe.
|
{z
}
n ori
Dorin M
arghidanu, Corabia
XII.79. Fie V spatiu vectorial de dimensiune n peste corpul K, iar u un endormorfism nilpotent al lui V (i.e., exist
a p N astfel nct up = u
| u {z u} = 0).
n ori

S
a se arate c
a un = 0.

Adrian Reisner, Paris

4
4
XII.80. Fie A un inel n care x y = (x y) (x + y) x2 + y 2 , x, y A.
a) Dac
a inelul are unitate, s
a se arate c
a A este comutativ.
b) Ramne valabil rezultatul de la a) daca inelul A nu este unitar?
Gabriel Dospinescu, Paris si Marian Tetiva, Brlad

Probleme pentru preg


atirea concursurilor
A. Nivel gimnazial
G116. Aflati toate numerele naturale N de patru cifre nenule distincte cu proprietatea ca diferenta dintre cel mai mare numar obtinut prin permutarea cifrelor lui
N si cel mai mic asemenea num
ar este tocmai N .
Maria Mihet, Timisoara
G117. Fie multimea A = {1, 2, 3, . . . , 98}. Aratati ca oricum am alege 50 de
elemente ale lui A, exist
a dou
a printre ele avnd suma cub perfect.
Titu Zvonaru, Com
anesti

G118. n interiorul unui paralelogram avnd unghiul ascutit de 30 si lungimile
laturilor 17 cm si 59 cm, se considera 2007 puncte. Sa se arate ca putem alege trei
dintre aceste puncte astfel nct aria triunghiului determinat de ele s
a fie cel mult
egal
a cu 14 cm2 .
Mihai Haivas, Iasi

0
n

G119.
Fie
n

2
+

2
|

,
.
.
.
,

{1}
, iar
s
i
A
=

0
n
0
1
n

n+1
B = m | m 2 Z + 1, |m| 2
1 . Sa se arate ca A = B.
Dorel Mihet, Timisoara
2005
2007
G120. Rezolvati n N ecuatia x! (y!)
= (z!)
.
Anca S
tefania Tutescu, elev
a, Craiova
68

G121. Dac
a a, b (0, 3/2), s
a se demonstreze inegalitatea
r
1
1
1
1
1 1
+ .
+ +
+
a b a+b+3
a
a+ b
b
Andrei Laurentiu Ciupan, elev, Bucuresti
G122. Fie G centrul de greutate al 4ABC si G0 proiec
a
tia sa pe dreapta BC. S
se arate c
a G0
/ [BC] dac
a si numai dac
a 3a2 < b2 c2 .
Temistocle Brsan, Iasi

G123. Fie ABC un triunghi echilateral. Sa se arate ca orice punct M din plan
cu proprietatea ca M B = M A + M C poate fi determinat folosind doar echerul. (Un
echer poate fi folosit pentru a trasa drepte si unghiuri drepte.)
Nicolae Iv
aschescu, Craiova
G124. Fie 4ABC, A0 mijlocul lui [BC], iar P si Q proiectiile lui A0 pe AB,
respectiv AC. Sa se arate ca 4P Q AB + BC + CA.
Adrian Zahariuc, elev, Bac
au
G125. Fie ABCD un patrat, M (AB), {O} = AC BD, {S} = CM DA,
iar {E} = SO M D. Consider
am AA0 (ABC), AA0 = AB, I mijlocul lui [A0 D],
0
iar {H} = M I A E. S
a se arate c
a:

2
0
V
AB
A ADH
0
a) M D (A AE);
b)
=
.
VMADH
AM
Petru R
aducanu, Iasi

B. Nivel liceal
L116. Cercul nscris n 4ABC este tangent laturii BC n punctul D1 , iar cercul
A-exnscris este tangent aceleeasi laturi n punctul D2 . Dreapta AD2 intersecteaza
cercul nscris n punctele S si T . S
a se arate c
a 4ST D1 este dreptunghic.
Titu Zvonaru, Com
anesti
L117. Fie 4ABC, D (BC), iar C1 , C2 cercurile exnscrise triunghiurilor ADB,
si ADC, tangente la BC. Aratati ca o tangenta comuna interioara cercurilor C1 si
C2 trece prin punctul de contact cu BC al cercului A-exnscris.
Neculai Roman, Mircesti, Iasi
L118. Fie M un punct al elipsei E, de focare F si F 0 . Dreptele M F si M F 0
intersecteaz
a elipsa n A, respectiv A0 . S
a se arate c
a, atunci cnd M parcurege E,
dreapta AA0 este mereu tangenta unei curbe fixe, care se cere a fi determinata.
Adrian Reisner, Paris
L119. Fie n N si a.b, c R+ cu ab + bc + ca = 3. Sa se arate ca an+3 + bn+3 +
c
+ 2abc (an + bn + cn ) 9.
Titu Zvonaru, Com
anesti si Bogdan Ionita
, Bucuresti
n+3

L120. Pentru a1 , a2 , . . . , an reale pozitive, s


a se demonstreze inegalitatea
(n1)2
(n1)2 (n1)2
a2
an
+ a2 a2n1
+ + an a2n1
a1
a1 a2n1
2
3
1
+
+ +

.
a2
a3
a1
a21 a22 a2n

Marian Tetiva, Brlad

69

1
L121. Fie n N dat. Sa se arate ca exista ctiva termeni ai sirului
m3 mn+1
1
a c
aror sum
a este mai mare dect
.
(n + 1) (2n + 1)
Dumitru Mihalache si Marian Tetiva, Brlad
L122. La un campionat de fotbal particip
a 2n echipe, astfel nct dintre oricare dou
a se poate dinainte indica echipa mai bun
a. n prima etap
a, echipele se
mpart aleator n perechi si disputa cte un meci, echipa mai buna trecnd n etapa
urmatoare. Procedeul se repeta pna la finala.
a) Care este probabilitatea ca a doua echip
a ca valoare s
a ias
a vicecampioan
a?
b) Dac
a se disput
a si o final
a mic
a, ce probabilitate este ca, n plus, cea de-a treia
echipa ca valoare sa se claseze pe locul 3?
Irina Mustata
a, Bremen
, student
L123. Pe o tabl
a 8 9 se aseaz
a dreptunghiuri 3 1 si "figuri"
de forma unui dreptunghi 1 3 c
aruia i lipseste p
atratul median (ca
n desenul alaturat). "Figurile" si dreptunghiurile nu se pot roti si nu
au puncte interioare comune. Sa se arate ca exista o multime S de 18
p
atrate 1 1 astfel nct, dac
a pe tabl
a r
amn 2 p
atrate neacoperite de
dreptunghiuri sau "figuri", atunci cele dou
a p
atrate sunt obligatoriu din S.
Gabriel Dospinescu, student, Paris

L124. Fie n N fixat.Determinati matricele A Mn (C) pentru care t A A =


In , iar A2007 + A + In = On (cu am notat operatia de conjugare).
Vlad Emanuel, elev, Sibiu
L125. Fie f : R R o functie periodic
a si lipschitzian
a (exist
a L > 0 pentru
care |f (x) f (y)| L |x y|, x, y R), iae (xn )n1 un sir strict cresc
ator, cu
lim xn = + si lim (xn+1 xn ) = 0. Sa se arate ca multimea punctelor limita
n
n
ale sirului (f (xn ))n1 coincide cu Im f .
Paul Georgescu si Gabriel Popa, Iasi

Training problems for mathematical contests


A. Junior highschool level
G116. Find all the natural numbers N of four distinct nonzero digits with the
property that the diferrence between the largest number obtained by permuting the
four digitis of N and the smallest number obtained in the same manner equals just N .
Maria Mihet, Timisoara
G117. Let us consider the set A = {1, 2, 3, . . . , 98}. Show that two elements exist
among any 50 elements arbitrarily chosen from A such that their sum is a perfect
cube.
Titu Zvonaru, Com
anesti
G118. 2007 points are considered in the interior of a parallelogram with its acute
angle equal to 30 and the lengths of its sides of 17 cm and 59 cm. Show that three
among these points can be selected so that the area of the triangle determined by
70

them be at most equal to

1
4

cm2 .

Mihai Haivas, Iasi


G119. Let n N and A = {0 20 + + n 2n | 0 , 1 , . . . , n {1, 1}}, and
B = {m | m 2Z + 1, |m| 2n+1 1}. Show that A = B.
Dorel Mihet, Timisoara

G120. Solve in N the equation x!(y!)2005 = (z!)2007 .


Anca S
tefania Tutescu, high-school student, Craiova
G121. For 0 < a, b < 3/2, prove that the inequality
r
1 1
1
1
1
1
+ .
+ +
+
a b a+b+3
a
a+ b
b
holds.
Andrei Laurentiu Ciupan, high-school student, Bucuresti
0
G122. Let G be the gravity center of ABC
G its projection on the line
2 and
0
2
2
BC. Show that G
/ [BC] if and only if 3a < b c .
Temistocle Brsan, Iasi
G123. Let ABC be an equilateral triangle. Show that any point M in the
plane with the property that M B = M A + M C can be determined using a tracing
square only. (A tracing square can be used for tracing straight lines and right angles).
Nicolae Iv
aschescu, Craiova
G124. Let ABC be a triangle with the midpoint of [BC] denoted A0 and P , Q
the projections of A0 on AB and respectively AC. Prove that 4P Q AB +BC +CA.
Adrian Zahariuc, high-school student, Bac
au
G125. Let ABCD be a square, M (AB), {O} = AC BD, {S} = CM DA,
and {E} = SO M D. We consider AA0 (ABC), AA0 = AB, I the midpoint of
[A0 D] and {H} = M I A0 E. Show that:

2
AB
VA0 ADH
0
a) M D (A AE );
b)
=
.
VMADH
AM
Petru R
aducanu, Iasi

B. Highschool level
L116. The circle inscribed in ABC is tangent to the side BC at the point D1 ,
and the A exinscribed circle is tangent to the same side at point D2 . The straight
line AD2 intersects the inscribed circle at the points S and T . Show that ST D1 is
a right-angled triangle.
Titu Zvonaru, Com
anesti
L117. Let us consider ABC, D (BC), and C1 , C2 the exinscribed circles to
the triangles ADB and ADC that are tangent to BC. Show that a common tangent
that passes between the circles C1 and C2 also passes through the contact point with
BC of the A exinscribed circle to ABC.
Neculai Roman, Mircesti, Iasi
L118. Let M be a point of the ellipse E whose foci are F , F 0 . The straight lines
M F and M F 0 intersect the ellipse at points A, respectively A0 . Show that, when
the point M runs over E, the line AA0 is ceaselessly tangent to a fixed curve that is
required to be determined.
Adrian Reisner, Paris
71

L119. Let n N and a, b, c R+ with ab + bc + ca = 3. Show that


an+3 + bn+3 + cn+3 + 2abc(an + bn + cn ) 9.

Titu Zvonaru, Com


anesti and Bogdan Ionita
, Bucuresti
L120. Let a1 , a2 , . . . , an be positive real numbers. Prove the inequality
(n1)2
(n1)2 (n1)2
a2
an
a1 a22 n1 + a2 a23 n1 + + an a21 n1
a1
+
+ +

.
a2
a3
a1
a21 a22 a2n
Marian Tetiva, Brlad

Let n N be given. Show that a couple of terms of the sequence


L121.

1
1
exist whose sum is greater than
.
3
m mn1
(n + 1)(2 n + 1)
Dumitru Mihalache and Marian Tetiva, Brlad
L122. A football championship is attended by 2n teams such that the best team
among each pair of teams can be predicted. In the first stage, the teams are randomly
grouped in pairs and play by one match each pair so that the better team passes to
the next stage. The procedure is repeated until the final match has to be played.
a) Which is the probability for the second team as to its value to be classified as
vice-champion?
b) If a small final match is also disputed, which is the probability for the third
team as to its value to get classified on the third position?
Irina Mustata
, student, Bremen
L123. On a table of size 8 9, rectangles of size 3 1 are placed
together with "figures" with the shape of a 1 3 rectangle with the
middle square missing (as in the drawing aside). The "figures" and the
rectagles cannot be rotated and they have no common interior points.
Show that there exists a set S of 18 squares 1 1 such that, if only two
squares remain on the table that are not covered by rectangles or "figures", then the
two squares are necessarily in S.
Gabriel Dospinescu, student, Paris

L124. Let n N be fixed. Determine the matrices A Mn (C) such that t (A)
A = In , and A2007 + A + In = On ( denotes the conjugate of the element under the
bar).
Vlad Emanuel, high-school student, Sibiu
L125. Let f : R R be a periodical and lipschitzian function (i.e., there exists
L > 0 such that |f (x) f (y)| L |x y|, x, y R), and let (xn )n1 be a strictly
increasing sequence with lim xn = and lim (xn+1 xn ) = 0. Show that the set
n

of limit points of the sequence (f (xn ))n1 is Im f .


Paul Georgescu and Gabriel Popa, Iasi

72

Pagina rezolvitorilor
BRA
SOV
Colegiul National de Informatica "Gr. Moisil". Clasa a IX-a. ALDEA Adrian:
VII(71,73), VIII(71-74); ANGHEL C
at
alin: VII(71,73), VIII(71-74); ARMEANU
Sebastian: VII(71,72), VIII(72-74), IX.73; BAUM Bianca: VII(71-73), VIII(71-74);
BERCEA Laura: VII(71-73), VIII(71-74); BONTAS Marcel: VII(71-73), VIII(7274); COROESCU Alexandra: VII(71,73), VIII(72-74); DIEA Ionut: VII(71-73), VIII
(71-74); DINU Cristian: VII(71-73), VIII(73,74); DRAGOMIR Dragos: VII(71,73),
VIII(72-74); DRAGU Bianca: VII(71,73), VIII(72-74), IX.73; FARSCH Cristina:

VII(71-73), VIII(71-74); FOTIN Alexandra: VII(71,73), VIII(71,73-75); GHIMBAS


AN Andreea: VII(71,73), VIII(72-74); HERMENEANU Horia: VII(71-73), VIII(7275); HNCU Ramona: VII(71-73), VIII(71-74); IONESCU Marius: VII(71,73), VIII

(72-74), IX.73; IVAN Iulia: VII(71,73), VIII(71-74); LEVITCHI


Alexandra: VII(71,
73), VIII(71-73); MAN Andrada: VII(71,73), VIII(71-74); MARIN Ionut: VII(71,73),
VIII(72-74); MATHE Emil: VII(71,73), VIII(72-74); MIEREA Ciprian: VII(71,72),
VIII(71-74); MOLDOVAI Adelina: VII(71-73), VIII(71-75), IX(72,73); MUNTEANU
Bogdan: VII(71,73), VIII(72-74); OJOG
Oana: VII(71,73), VIII(72-74); NEGUT
Alexandru: VII(71,73), VIII(71-74); PAGU Roxana: VII(71,73), VIII(72-74);
PETRUSCA Alin: VII(71,73), VIII(72-74); POPA Alexandra: VII(71,73), VIII(7274); POPESCU D
anut: VII(71,73), VIII(71-74); POTEC Ionela: VII(71-73), VIII(71
75); PUZDREA George: VII(71,73), VIII(72-74); RADUCU
Gabriela: VII(71-73),
VIII(72,75), IX.73; ROMAN Radu: VII(71,73), VIII(71-74), IX.72; SATNOIANU

Livia: VII(71,73), VIII(71-74); SCORTEA


Liliana: VII(71-73), VIII(71-74);
S
ERBAN C
at
alina: VII(71-73), VIII(72-75); TAUS Alina: VII(71-73), VIII(73-75);

TENEA
Codruta: VII(71,73), VIII(71-74); TICA
Doris: VII(71-73), VIII(72-75),

IX.73; VASII
Cristian: VII(71,73), VIII(72-74); VLADU Ioana: VII(71,73), VIII(7274); ZSILINSZKY Laura: VII(71-73), VIII(72-75).
CRAIOVA
Colegiul National "Fratii Buzesti". Clasa a VII-a. DAN Ana-Veronica: V(71
75). Clasa a IX-a. TUTESCU
Anca: IX(72,73), IX(71-73).
Colegiul National "Carol I". Clasa a VII-a. STANCIU Ioan: VI(71-75), VII(71,
73,75), G(106-108,112).

HRLAU
ALEANU

Scoala
"Petru Rares". Clasa a III-a (nv. NICULESCU Carmen). SAC

Olga). NEICU
Emilian: P(104-106,108-110). Clasa a IV-a (nv. PRIALA
Maria). PINTILII
Mara: P(111,112,118,119,123). Clasa a IV-a (nv. CRETU
Alina: P(104-106,108,110,112,113).
Andreea: P(104-106,
Liceul Teoretic "Stefan
cel Mare". Clasa a V-a. BUZILA

A Andreea: P(111,112), V(66,67,70); MATEI


108,110-113), V(66,67,69); IVANUT
Madalina: P.113, V(66-70). Clasa a XI-a. BURICAN Bogdan Alexandru: IX(6668,70), X.68.
IA
SI

Valentina). BRA
Scoala
nr. 3 "Al. Vlahuta". Clasa a IV-a (nv. MARIU
T

73


LADEANU
Andrei: P(114,118,119,121,123); CULEA Alina: P(114,118,119,121,123);
Theodor: P(104,114,118,119,121,123); HADARAG Ana-Maria: P(114,
DIACONITA

118,119,121,123); NASTASE
Cosmin: P(114,118,119,121,123); POPA Iulian: P(114,
118,119,121,123); PROCA Ancuta-Ioana: P(114,118,119,121,123); TNCU Alexandra-Ioana: P(114,118,119,121,123). Clasa a IV-a (inst. MAXIM Gabriela). BACIU
Ionela-Lavinia: P(104,105,108,114,118,121); CELMARE Raluca-Iuliana: P(104,105,
108,114,118,121); GHEMU Laura: P(104,105,108,114,118,121); HURCHE Robert:
P(104,105,108,114,118,121); NEAGU Ramona-Mihaela: P(104,105,108,114,118,121);
POPOVICI Ionut: P(104,105,108,114,118,121); RUSU Ioana-Andreea: P(104,105,
108,114,118,121); RUSU Madalina-Andreea: P(104,105,108,114,118,121); SAVA Vlad:
P(104,105,108,114,118,121); VECHIU Madalina: P(104,105,108,114,118,121); ZANCHI Georgiana-Alexandra: P(104,105,108,114,118,121). Clasa a VI-a. MARDARI
ALI
TA
Georgiana: V(71,73,74), VI(71,
Magdalena: V(71,73,74), VI(71,74); DASC
74); TEODORESCU Oana: V(71,73,74), VI(71,74).
Scoala
nr. 13 "Alexandru cel Bun". Clasa a III-a (inst. COJOCARIU Ana).

Andra: P(114-117,119);

AGAFITEI
Elena-Roxana: P(114-117,119); CARAMALAU

CALIN Andreea-Claudia: P(114-117,119); COJOCARIU Andreea: P(114-117,119);


DUDUMAN Luisa-Stefania: P(114-117,119); LELEU Alexandrina-Stefana: P(114117,119); LUPASCU Diana-Maria: P(114-117,119); MANOLACHE Madalina-Andre A
Narcisa-Lorena: P(114-117,119); PASCU Gabriela:
ea: P(114-117,119); MIHAIL

P(114-117,119); PADURARU Tiberiu-Stefan: P(114-117,119); RADUCEA


MarinAndrei: P(114-117,119); SAVIN Cristina-Simona: P(114-117,119); S
TEFAN BogdanVasile: P(114-117,119); S
TIUBEI Cosmin-Ionut: P(114-117,119).
Scoala
nr.
26
"Gh.
Co
sbuc". Clasa a II-a (inst. RACU Maria). Apachitei Aura
Georgiana: P(104,114-117); BURA Emma-Andreea: P(104,114-117); CIORNEI

Alexandra: P(104,114-117); CRACIUN


Ioana-Daniela: P(104,114-117); FILIP Ingrid
S
tefania: P(104,114-117); GRADINARIU
Georgiana: P(104,114-117); HRISCU
Ovidiu Constantin: P(104,114-117); HUZA Madalina: P(104,114-117); MARICIUC

Dragos-Claudiu: P(104,114-117); MAXIM Alexandra-Camelia: P(104,114-117); TU Cosmin: P(104,114-117); VASILE Bogdan-Andrei: P(104,114-117).
CA
Scoala
Normala "V. Lupu". Clasa a VI-a. NASTASIA Mariana: P(112,121),

V(66-75).
Lieul Teoretic "M. Eminescu". Clasa a V-a. BRNOSCHI Letitia: P(121
123), V.71, VI.71; ETCU
Magda: P.123, V(71,73,74), VI.71; FILIMON Lucian-Jan:
P(121-123), V(71-73).

Colegiul National "Emil Racovita", locatia "Gh. Asachi". Clasa a III-a (inst. CA
LINESCU Rodica). BAJENARU
Br
adita: P(114-120); CHIVULESCU Alexandru:
P(114,116-120); PETREA Madalina: P(114-120); UNGUREANU George: P(114120). Clasa a VII-a. TUDORACHE Alexandru Gabriel: VI(71-75), VII(73-74),
G(106,108,110).
ALOI

Colegiul National "C. Negruzzi". Clasa a VI-a. PAV


Alexandru: P.111,
V(66,67,69,70). Clasa a VIII-a. TIBA Marius: G(107,108,110-112).
Colegiul National. Clasa a VII-a. MOCANU Dan Mihai: V(71,72,75), VI(71,72),
VII.75; OROIAN Bianca: V(66,67,69,70), VI.69, VII.70; PETRESCU Emmanuel:
74

V(71,73,75), VI.71, VII.71.


ONE
STI (Bac
au)

Scoala
"Ghita Mocanu". Clasa a VII-a. PADURARIU
Tudor: VII (67-70),

VIII(66,67,69), IX(68,70), G(96-98,101-105).


SIBIU
Colegiul National "Gh. Lazar". Clasa a XII-a. VLAD Emanuel: L(97,99-103).
SUCEAVA
Scoala
generala nr. 3. Clasa a II-a (nv. TABARCEA Silvestru). FECHET

S
tefan: P(104,105,107-109). Clasa a III-a (inst. NECHITA Daniela). FECHET
Mircea: P(104-106,108-110).

Premii acordate rezolvitorilor

ASOCIA
TIA "RECREATII
MATEMATICE" n colaborare cu redactia
MATEMATICE acorda cte o diplom
revistei RECREATII
a si un premiu n
c
arti, pentru trei aparitii la rubrica Pagina rezolvitorilor, elevilor urmatori:
Scoala
"Petru Rares", Hrl
au

PINTILII Alina (cl. a IV-a): 2/2005(8pb), 1/2006(8pb), 1/2007(7pb).


Liceul Teoretic "Stefan
cel Mare", Hrl
au

Andreea (cl. a V-a): 2/2005(8pb), 1/2006(8pb), 1/2007(11pb);


BUZILA
BURICAN Bogdan Alexandru (cl. a XI-a): 1/2003(5pb), 1/2004(7pb), 1/2007(5pb).
Scoala
nr. 3 "Al. Vlahuta", Iasi

CELMARE Raluca-Iuliana (cl. a IV-a): 1/2006(5pb), 2/2006(8pb), 1/2007(6pb);


CULEA Alina (cl. a IV-a): 1/2006(5pb), 2/2006(6pb), 1/2007(5pb);
NEAGU Ramona-Mihaela (cl. a IV-a): 1/2006(5pb), 2/2006(8pb), 1/2007(6pb);
POPOVICI Ionut (cl. a IV-a): 1/2006(5pb), 2/2006(8pb), 1/2007(6pb);
POPA Iulian (cl. a IV-a): 1/2006(5pb), 2/2006(6pb), 1/2007(5pb);
PROCA Ancuta-Ioana (cl. a IV-a): 1/2006(5pb), 2/2006(6pb), 1/2007(5pb);
RUSU Ioana-Andreea (cl. a IV-a): 1/2006(5pb), 2/2006(8pb), 1/2007(6pb);
VECHIU M
ad
alina (cl. a IV-a): 1/2006(6pb), 2/2006(8pb), 1/2007(6pb).
Scoala
nr. 13 "Alexandru cel Bun", Iasi

A
Narcisa-Lorena (cl. a III-a): 1/2006(5pb), 2/2006(5pb), 1/2007(5pb).
MIHAIL
Colegiul National "C. Negruzzi", Iasi
TIBA Marius (cl. a VIII-a): 1/2006(7pb), 2/2006(8pb), 1/2007(5pb).
Colegiul National, Iasi
MOCANU Dan Mihai (cl. a VII-a): 1/2005(11pb), 1/2006(5pb), 1/2007(6pb).

75

IMPORTANT
n scopul unei leg
aturi rapide cu redactia revistei, pot fi utilizate urm
atoarele
adrese e-mail: tbirsan@math.tuiasi.ro sau t_birsan@yahoo.com si
profgpopa@yahoo.co.uk . Pe aceasta cale colaboratorii pot purta cu redactia un dialog privitor la materialele trimise acesteia, procurarea numerelor
revistei etc. Suger
am colaboratorilor care trimit probleme originale pentru
publicare sa le numeroteze si sa-si retina o copie xerox a lor pentru a putea
purta cu usurinta o discutie prin e-mail asupra acceptarii/neacceptarii acestora de c
atre redactia revistei.
La problemele de tip L se primesc solutii de la orice iubitor de matematici
elementare (indiferent de preocupare profesionala sau vrsta ). Fiecare dintre
solutiile acestor probleme - ce sunt publicate n revista dupa un an - va fi
urmat
a de numele tuturor celor care au rezolvat-o.
Adres
am cu insistenta
amintea ca materialele trimise revistei
rug
s
a nu fie (s
a nu fi fost) trimise si altor publicatii.
Rug
am ca materialele tehnoredactate s
a fie trimise pe adresa redactiei nsotite de fisierele lor (de preferinta n LATEX).

ASOCIA
TIA RECREA
TII MATEMATICE

La data de 14.02.2005 a luat fiinta ASOCIATIA


RECREATII
MATEMATICE, cu sediul n Iasi (str. Aurora, nr. 3, sc. D, ap. 6), avnd ca scop sprijinirea activitatilor de matematica specifice nvatamntului preuniversitar, organizarea si desfasurarea de activitati care sa contribuie la dezvoltarea gustului pentru
matematica n rndurile elevilor, profesorilor si iubitorilor de matematica si stimularea preocuparilor si cercetarilor originale.
Obiectivele majore pentru atingerea scopului propus sunt:
1. editarea unei reviste destinata elevilor si profesorilor revista "Recreatii
Matematice";
2. fondarea unei biblioteci de matematic
a elementar
a biblioteca "Recreatii
Matematice";
3. alcatuirea unei colectii de carti de matematica elementara, carti de referinta
si aflate la prima aparitie Colectia "Recreatii Matematice".
Poate deveni membru al Asociatiei, printr-o simpla completare a unei cerei tip,
orice perosana care adera la obiectivele acesteia si sprijina realizarea lor.
Membri de onoare ai Asociatiei, academicienii:
76

Constantin Corduneanu
Radu Miron

Revista semestrial RECREAII MATEMATICE


este editat de
ASOCIAIA RECREAII MATEMATICE. Apare la datele de 1 martie i
1 septembrie i se adreseaz elevilor, profesorilor, studenilor i tuturor celor
pasionai de matematica elementar.
n atenia tuturor colaboratorilor
Materialele trimise redaciei spre publicare (note i articole, chestiuni de
metodic, probleme propuse etc.) trebuie prezentate ngrijit, clar i concis; ele
trebuie s prezinte interes pentru un cerc ct mai larg de cititori. Se recomand ca
textele s nu depeasc patru pagini. Evident, ele trebuie s fie originale i s
nu fi aprut sau s fi fost trimise spre publicare altor reviste. Rugm ca materialele tehnoredactate s fie nsoite de fiierele lor.
Problemele destinate rubricilor: Probleme propuse i Probleme pentru
pregtirea concursurilor vor fi redactate pe foi separate cu enun i demonstraie/rezolvare (cte una pe fiecare foaie) i vor fi nsoite de numele autorului, coala i localitatea unde lucreaz/nva.
Redacia va decide asupra oportunitii publicrii materialelor primite.
n atenia elevilor
Numele elevilor ce vor trimite redaciei soluii corecte la problemele din
rubricile de Probleme propuse i Probleme pentru pregatirea concursurilor
vor fi menionate n Pagina rezolvitorilor. Se va ine seama de regulile:
1. Pot trimite soluii la minimum cinci probleme propuse n numrul
prezent i cel anterior al revistei; pe o foaie va fi redactat soluia unei singure
probleme.
2. Elevii din clasele VI-XII au dreptul s trimit soluii la problemele
propuse pentru clasa lor, pentru orice clas mai mare, din dou clase mai mici i
imediat anterioare. Elevii din clasa a V-a pot trimite soluii la problemele propuse
pentru clasele a IV-a, a V-a i orice clas mai mare, iar elevii claselor I-IV pot
trimite soluii la problemele propuse pentru oricare din clasele primare i orice clas mai mare. Orice elev poate trimite soluii la problemele de concurs (tip G i L).
3. Vor fi menionate urmtoarele date personale: numele i prenumele,
clasa, coala i localitatea.
4. Plicul cu probleme rezolvate se va trimite prin pot (sau va fi adus
direct) la adresa Redaciei:
Prof. dr. Temistocle Brsan
Str. Aurora, nr. 3, sc. D, ap. 6,
700 474, Iai
Jud. IAI
E-mail: tbirsan@math.tuiasi.ro sau t_birsan@yahoo.com

CUPRINS
300 de ani de la naterea lui Leonhard Euler (1707 1783)......................................... 1
Profesorul Dumitru Ion Mangeron (1906 1991) In Memoriam ............................... 4
Grigore Moisil fotografie-document inedit .................................................................... 6

ARTICOLE I NOTE
C. IGERU Similitudini n plan i puncte Torricelli asociate...................................... 7
A. REISNER Ordinul elementelor grupului GLn ( Z ) ...................................................... 14
T. BRSAN Variaiuni pe tema dreptei lui Euler i cercului celor nou puncte......... 16
M. TETIVA New Proof for an Old Inequality ................................................................ 20
D. M. BTINEU-GIURGIU Asupra calculrii unor limite de iruri............................ 22
S. BOGA O generalizare a teoremei lui Van Aubel....................................................... 25

NOTA ELEVULUI
B. CIACOI O propoziie echivalent cu conjectura lui Goldbach ................................. 27

CHESTIUNI METODICE
L. TUESCU Cum se poate obine o inegalitate ............................................................ 29

PROBLEME I SOLUII
Soluiile problemelor propuse n nr. 1/2006.......................................................................... 31
Soluiile problemelor pentru pregtirea concursurilor din nr. 1/2006 ................................. 48
Probleme propuse..................................................................................................................... 64
Probleme pentru pregtirea concursurilor .............................................................................. 68
Training problems for mathematical contests ....................................................................... 70
Pagina rezolvitorilor .............................................................................................................. 73

5 lei

Anul X, Nr. 1

Ianuarie Iunie 2008

RECREAII
MATEMATICE
REVIST DE MATEMATIC PENTRU ELEVI I PROFESORI

125 de ani de la apariia


revistei Recreaii tiinifice
(1883 1888)

e i = 1
Asociaia Recreaii Matematice
IAI - 2008

Semnificaia formulei de pe copert:


i
ntr-o form concis, formula e = 1 leag cele patru ramuri fundamentale
ale matematicii:
ARITMETICA
GEOMETRIA
ALGEBRA
ANALIZA MATEMATIC

reprezentat
reprezentat
reprezentat
reprezentat

de
de
de
de

i
e

Redacia revistei :
Petru ASAFTEI, Dumitru BTINEU-GIURGIU (Bucureti), Temistocle BRSAN, Dan
BRNZEI, Ctlin - Cristian BUDEANU, Constantin CHIRIL, Eugenia COHAL, Adrian
CORDUNEANU, Mihai CRCIUN (Pacani), Paraschiva GALIA, Paul GEORGESCU,
Mihai HAIVAS, Gheorghe IUREA, Lucian -Georges LDUNC, Mircea LUPAN, Gabriel
MRANU, Andrei NEDELCU, Alexandru NEGRESCU (student, Iai), Gabriel POPA,
Dan POPESCU (Suceava), Florin POPOVICI (Braov), Maria RACU, Neculai ROMAN
(Mirceti), Ioan SCLEANU (Hrlu), Ioan ERDEAN (Ortie), Dan TIBA (Bucureti),
Marian TETIVA (Brlad), Lucian TUESCU (Craiova), Adrian ZANOSCHI, Titu
ZVONARU (Comneti).
Adresa redaciei:
Catedra de Matematic Universitatea Tehnic Gh. Asachi Iai
Bd. Carol I, nr.11, 700506, Iai
Tel. 032 213737 / int. 123
E-mail: recreatii.matematice@gmail.com
http://www.recreatiimatematice.uv.ro
COPYRIGHT 2008, ASOCIAIA RECREAII MATEMATICE
Toate drepturile aparin Asociaiei Recreaii Matematice. Reproducerea integral sau
parial a textului sau a ilustraiilor din aceast revist este posibil numai cu acordul prealabil
scris al acesteia.
TIPRIT LA SL&F IMPEX IAI
Bd. Carol I, nr. 3-5
Tel. 0788 498933
E-mail: simonaslf@yahoo.com
ISSN 1582 - 1765

Anul X, Nr. 1

Ianuarie Iunie 2008

RECREAII
MATEMATICE
REVIST DE MATEMATIC PENTRU ELEVI I PROFESORI

e i = 1
Revist cu apariie semestrial

EDITURA RECREAII MATEMATICE

IAI - 2008

Recreatii S
tiintifice 125 de ani de la aparitie
La 15 ianuarie 1883, la Iasi (n tipo-litografia "Buciumului Romn"), aparea
primul numar al revistei Recreatii Stiin
tifice, publicatie stiintifica adresata unei largi
audiente, o premier
a n peisajul cultural romnesc de la acea dat
a. Colectivul de entuziasti care au pus piatra de temelie a acestei ntreprinderi cu profunde reverberatii
n viata academica romna era format din N. Culianu, C. Climescu, I. Melik (de
la Facultatea de S
tiinte din Iasi), G.I. Lucescu, V. Paladi, G.I. Rosiu, I.D. Rallet,
G. Zarifopol, I.V. Praja si I.M. Dospinescu (profesori la diferite licee si scoli din
Iasi). Colaboratori regulati au fost si M. Tzony, V.C. Butureanu, A. Scriban de la
Universitatea din Iasi.
Noua publicatie a avut ca model reviste de prestigiu ce apareau n tarile cu traditie
ndelungat
a din Europa, din care s-au preluat, n timp, articole si probleme: Analele
lui Gergonne (Franta), Mathesis (Gand, Belgia - a nu se confunda cu societatea
italiana cu acelasi nume fondata la Torino n 1895), Journal de mathmatiques lmentaires (Paris), Revue Scientifique etc.
Revista a ap
arut lunar, f
ar
a ntrerupere, timp de sase ani. Initial, fiecare num
ar
avea 32 de pagini, cuprinznd articole cu subiecte variate: aritmetica, algebra, geometrie, geometrie analitica, trigonometrie, calcul diferential si integral, istoria matematicii, mecanica, topografie, cosmografie, astronomie, chimie, geografie sau diverse.
Ulterior, s-au publicat si numere de 24 de pagini sau numere comasate (7 si 8, numere de vacanta) de 48 sau 40 de pagini.
Revista a cunoscut o buna raspndire n Regatul Romniei (n ntinderea sa de
atunci), primind colaborari sub forma de note, scrisori, etc. din Iasi, Bucuresti,
Bac
au, Botosani, dar si din N
as
aud, Paris si a constituit un model pentru ntreprinderi similare ulterioare, cum ar fi cunoscuta Gazeta Matematic
a.
Ca "profesie de credinta" a inimosului colectiv de editori, apare scrisa o scurta
adresare Catra Cetitori n prima pagina a nr.1 din 1883, n care se afirma dorinta ca
noua publicatie s
a remedieze din lacunele existente n nv
atamntul nostru, s
a ofere
ospitalitate profesorilor sau institutorilor care "trateaza o chestiune de stiinte dupa o
metoada proprie lui", sa ncurajeze tinerimea studioasa. Cu modestie si obiectivitate
se adauga:
"Nu pretindem ca vom produce lucrari originale. n starea n care se afla tara
noastra, lucrari originale, pe terenul stiintific, snt foarte greu de intreprins. Nimine
nu este vinovat pentru aceasta; trebue sa ne facem stagiul cuvenit."
ntr-adevar, cteva date sunt edificatoare n privinta situatiei existente atunci n
tara noastr
a. Imediat dup
a Unirea Principatelor s-a nl
aturat, oficial, scrierea cu al
fabetul chirilic (n anul 1860 n Tara
Romneasc
a si 1863 n Moldova), iar n scrierea
cu caractere latine s-a adoptat sistemul etimologic cu utilizarea semnelor diacritice.
n perioada de pna la 1880, limba romna literara (moderna) si-a ncheiat procesul
de unificare si de stabilizare n forma pe care o are ast
azi. nv
atamntul trecea prin
mari prefaceri si fr
amnt
ari: nfiintarea universit
atilor din Iasi si Bucuresti, legea
nvatamntului din 1864, numeroasele regulamente menite sa organizeze reteaua de
1

scoli si sa stabileasca programele acestora, lipsa de manuale si cursuri n limba romna


etc. Aceste dificult
ati, greut
atile materiale, ct si lipsa unui public format pentru
receptarea unei reviste stiintifice de acest gen fac ca actul public
arii Recreatiilor
S
tiintifice sa fie unul temerar si pornit dintr-o nalta responsabilitate, pentru ndeplinirea caruia au fost necesare multa daruire si multe sacrificii. In vol.VI, nr.1,
se subliniaz
a conceptia umanist
a a colectivului de redactie, abordarea constient
aa
actului stiintific si a procesului de nv
atamnt de la noi:
"Credem ca noi am tras cea ntai brazda care conduce catra lucrari originale.
Brazdai mica si ngusta, dar exista ! [. . . ] Noi de la nceput am avut n videre
ca o sa trebuiasca sa facem sacrificiu de timp si bani. Am facut si vom face acest
sacrificiu."
In numarul 7 din anul IV este amintit rolul deosebit jucat de Ioan Pop (Popp),
care "venit din Transilvania n anul 1858 a predat cel ntaiu n mod sistematic si
complect stiintele matematice n cursul superior al Liceului din Iasi. [. . . ] a fost un
excelent profesor. A stiut sa inspire gustul stiintilor matematice la o multime din
elevii lui [. . . ]."; printre acestia sunt amintiti Gh. Rosiu, M. Tzony, C. Climescu,
V. Palade s.a.
R
asfoind numerele Recreatiilor S
tiintifice, cititorul de azi va aprecia frumoasa
limb
a folosit
a acum peste o sut
a de ani, calitatea nalt
a a materialelor incluse, dar
va constata si existenta multor lacune n terminologia de specialitate, mai cu seama
n matematic
a.
Aproape n fiecare num
ar erau prezente rubricile de Probleme Propuse si Probleme
Rezolvate (solutii primite de la cititori). Terminologia vremii era Probleme Rezolvite,
iar rezolvitorii erau de la diferite licee si scoli din Iasi (n primele luni) ca, mai apoi, sa
apar
a rezolvitori din toat
a Moldova (Bac
au, Brlad, Dorohoi, Focsani, Galati etc.),
de la Bucuresti (Liceul Sf. Sava, S
coala de Poduri si S
osele, S
coala Militar
a, S
coala
Normal
a Superioar
a), Craiova, Alexandria sau chiar de la Paris, unde si continuau
studiile multi tineri romni (Institutul Duvignau, Institutul Jauret, Collge SainteBarbe, Liceul St. Louis). Cel mai activ rezolvitor a fost Vasile Cristescu, nti elev
la Liceul din Iasi, apoi student la S
coala de Poduri si S
osele din Bucuresti, calitate n

care a publicat si cteva Note; ulterior, a fost membru fondator al renumitei Gazeta
Matematica - nfiintata n anul 1895, cu aparitie nentrerupta pna astazi - si unul din
cei patru "stlpi" ai acesteia. n ultimul an de aparitie a Recreatiilor S
tiintifice,1888,
printre rezolvitori este de remarcat prezenta lui Dimitrie Pompeiu (1873-1954),
marele matematician de mai trziu, pe atunci n vrsta de 15 ani, elev n cl. a III-a
la gimnaziul din Dorohoi. Sunt numerosi rezovitorii care au devenit apoi nume de
prestigiu: Ermil Pangrati (profesor de geometrie descriptiv
a la universit
atile din Iasi
si Bucuresti, rector al celei din urm
a, organizatorul S
colii de arhitectur
a, deputat,
ministru, senator), Anastasie Obregia (profesor de chimie organica, Universitatea din
Iasi), Grigore G. Stratilescu, Petru N. Culianu, Vasile Teodoreanu s. a.
Unele materiale publicate sunt menite s
a suplineasc
a lipsa de c
arti si cursuri la
ndemna studentilor si profesorilor. Constantin Climescu expune, ntr-un ciclu de
2

noua articole intitulat Cteva curbe celebre si importante (opt aparute n vol.II-1884
si unul n nr.1 din 1885), principalele curbe plane clasice: cisoida lui Diocles, concoide, cicloide, spirale etc. ncepnd cu nr.4 din 1885 si continund num
ar de num
ar
(cu putine exceptii) pna la disparitia revistei, Miltiade Tzony publica Un curs de
probleme, ce cuprinde 98 de probleme de mecanic
a rational
a (anume, static
a), care
sunt complet rezolvate si nsotite de figuri (v. [8], pp. 138-140, pentru un studiu am
anuntit). O expunere a determinantilor si utiliz
arii lor a fost f
acut
a de Ion D. Rallet
n nr.11 si 12 din 1883 si nr.1 din 1884; o alta prezentare a acestora (dupa O. Hesse),
ce apare n vol.V (pp. 150 si 190), este semnat
a cu pseudonimul Candide. n [1],
notele de subsol de la pp. 239 si 256, se aduc argumente pentru identificarea acestui
pseudonim cu Victor Costin, pe atunci student, mai trziu profesor la universitatea
din Iasi. Un membru fondator si constant colaborator, publicnd chestiuni variate
de matematic
a elementar
a, este I.V. Praja.
Din bogatul continut al Recreatiilor S
tiintifice, punem n evidenta alte cteva
directii dezvoltate n paginile sale. G.I. Lucescu public
a un studiu amplu si documentat despre calendar. Vasile C. Butureanu semneaza doua lungi cicluri de articole
n domeniul mineralogiei. August Scriban public
a o serie de articole de geografia
Asiei centrale. Iacob Solomon trateaz
a, ntr-o serie de sase note din vol.VI, chestiuni
de istoria matematicii n antichitate utiliznd surse la zi (de exemplu, Geschichte der
Mathematik (Istoria matematicii) a lui Moritz Cantor, 1880).
Revista Recreatii Stiin
azduit si traduceri (partiale) ale unor c
arti de
tifice a g
referinta. G.I. Rosiu traduce n romneste (dup
a o editie italian
a a lui E. Betti si
F. Brioschi, Florenta,1868) si public
a n vol. II si III ale revistei prima carte din
Elementele lui Euclid. (Precizam ca traducerea completa a Elementelor a fost facuta
mult mai trziu de Victor Marian si publicat
a n Biblioteca Gazetei Matematice n
trei volume, 1939-1941.) Sub acelasi pseudonim Candide sunt prezentate n vol. IV
(nr.4-6 si 8-11) si vol. V (nr. 2 si 3) traduceri din Geometrie der Lage (Geometria de
pozitie) a lui Staudt.
Rubrica Diverse, cu scop de informare, are un continut bogat si variat acoperind
toate domeniile stiintei. n nr.1 din 15 ianuarie 1883, la p.21, se preia articolul "Fotografia Miscarii" (ce anticipeaza cinematografia) dupa Revue Scientifique din 23
dec. 1882 - o adevarata dovada de promptitudine si de capacitate de selectie. n nr.2
din 1883, la p.47, apare o scurt
a notita despre nfiintarea la Stockholm, n 1882, a
faimoasei Acta Mathematica, sub auspiciile regelui Suedo-Norvegiei si sub conducerea
lui G. Mittag-Lefler; printre colaboratori se numarau si Appell, Goursat, Poincar
(din Franta) care, mai trziu (n 1905), vor alcatui comisia care a examinat celebra
tez
a de doctorat a lui Dimitrie Pompeiu, la Sorbona [5], [6]. In nr.9 din 1884, un
articol ntreg este dedicat eruptiei vulcanului Krakatoa, catastrof
a care a afectat tot
globul (relatare oculara). n vol.III (1885), nr.5 si 6, doua articole sunt dedicate
marelui matematician belgian Eugne-Charles Catalan (1814 -1894), mentionnduse inclusiv celebra sa conjectur
a din 1844 (numerele 8 si 9 sunt singurele numere
naturale consecutive care sunt puteri exacte) rezolvat
a mult mai trziu, n anul 2002,
3

de matematicianul romn Preda Mihailescu [12]. Vol. IV din 1885, p.208 si urmatoarele, prezint
a fenomenul natural numit mascaret sau pororoca, iar la pp.235-237
cititorul este informat asupra unor date tehnice privind proiectul turnului Eiel ct
si asupra unor controverse contemporane generate de aceasta constructie.
Sunt cultivate dezbateri n jurul unor probleme din actualitatea stiintifica sau
din realitatea nv
atamntului romnesc. Astfel, articolul "Sf. Gheorghe si Pastile"
din nr. 5, anul VI, semnat de Paul Tanco (din N
as
aud), primul romn doctor n
matematici, este comentat de Constantin Gogu, ilustru profesor de la Universitatea
din Bucuresti, care mai public
a, apoi, cinci scrisori asupra regulelor ntrebuintate
pentru g
asirea zilei Pastilor. Mention
am polemicile sustinute cu publicisti de la
Contemporanul pe teme de chimie sau astronomie (p. 139, p. 142, p. 180 n nr. 5 si
nr. 6 din 1883 etc.). Cu deosebita putere de patrundere, G. Zarifopol scrie: "teorii,
adeseori ipoteze gratuite, emise de un nvatat strain sunt luate de redactia ziarului
Contemporanul ca ultimele adevaruri ale stiintei". Este interesant
a analiza atent
a
si riguroas
a f
acut
a de G. Lucescu n vol.VI unui manual manuscris de Aritmetic
a
destinata cl. I si a II-a primara si urmata de avizul negativ dat acestuia. Semnificativa
pentru corectitudinea redactiei este faptul c
a sunt publicate al
aturat att critica
adus
a de un referent lucr
arii lui I.V. Praja "Curs de aritmetic
a rationat
a", Iasi,
1885, 359 pagini, ct si replica acestui autor (vol.VI, pp.238-247).
Sa remarcam si faptul ca, dintre membrii fondatori, numai N. Culianu si V. Paladi
nu apar n paginile revistei cu nici un fel de contributie, iar I.M. Zamfirescu public
a
doar un rezumat al unui articol din Revue Scientifique asupra filoxerei.
Alte tr
as
aturi foarte interesante sunt rigurozitatea deosebit
a a activit
atii publicistice, bazata pe standarde care sunt valabile si astazi ct si conectarea excelenta la
viata stiintific
a mondial
a. Se manifest
a o bun
a ntelegere a fenomenului cercet
arii
stiintifice si a evenimentelor care fr
amntau lumea n penultimul deceniu al secolului
al XIX-lea.
Redactia acorda maxima importanta relatiilor directe cu cititorii, dovada fiind
publicarea de scrisori, note, a unor solutii diferite pentru aceeasi problem
a si a unor
statistici am
anuntite referitoare la rezolvitori.
Pentru toate articolele si notele publicate se indic
a sursele folosite. Cu totul extraordinar este aparatul bibliografic folosit de catre autori, la toate disciplinele ce fac
obiectul publicatiei. Se citeaz
a c
arti si reviste de specialitate str
aine contemporane
(an de aparitie chiar si 1888), dar si "Aritmetica" lui Amfilohie Hotiniul din 1795,
Iasi, sau Memoriile Academiei din Paris, ncepnd cu 1699.
Semnalam cititorilor si prezenta unor erori specifice epocii de pionerat (de paginatie, de numerotare a problemelor, de tip
arire etc.), corectate n parte n diverse
erate de c
atre redactia revistei si care nu afecteaz
a n mod esential lectura.
Fara nici o ndoiala, revista Recreatii Stiin
tifice a fost dedicata n primul rnd
chestiunilor de matematica. O statistica care ia n considerare numarul de pagini
arat
a c
a n aproximativ 90% din spatiul revistei sunt tratate subiecte de matematic
a, mecanic
a si astronomie. Num
arul total al problemelor propuse, publicate
4

n cei sase ani de aparitie, este de 298 (cu 284 fiind numerotate doua probleme). n
dictionarele de periodice romnesti revista este mentionat
a ca o publicatie cu o contributie important
a la educatia matematic
a a tineretului [3, 7].
n istoria matematicii romnesti, perioada 1860-1898 este marcata de eforturile
f
acute n scopul organiz
arii si moderniz
arii nv
atamntului si punerii bazelor cercet
arii
stiintifice originale. Contributia Recreatiilor Stiin
tifice la realizarea acestui program
a fost recunoscut
a si apreciat
a de generatiile care i-au urmat. n Introducerea din nr.1,
an I (1895), redactorii Gazetei Matematice spun: "Mai multi dintre noi datoresc acest
gust [pentru matematic
a - n. n.] revistei "Recreatii Stiin
tifice" ce a aparut n timp
de 6 ani la Iasi si pe care noi ncercam a o continua" [17]. Mai trziu, n 1935, cnd
aceast
a revist
a s
arb
atorea 40 ani de existenta, I. Ionescu, Gh. Ti
teica si multi altii
aminteau rolul avut de Recreatiile Stiin
teica spunea: "Cea dinti
tifice [9, 16]. Gh. Ti
ncercare de a iesi din acest impas, de a rupe cu inertia, de a determina un curent de
preocupare stiintifica si de a crea astfel un nceput de atmosfera prielnica dezvoltarii
stiintei matematice, a fost facuta la Iasi prin publicarea "Recreatiilor Stiin
tifice"
" [16, p. 69]. Alte aprecieri ale unor distinsi matematicieni romni pot fi gasite
n [18]. I. Popa face n 1955 [14], un studiu aprofundat asupra continutului si
aportului Recreatiilor S
tiintifice - numind-o precursoare a Gazetei Matematice - cu
prilejul s
arb
atoririi a 60 de ani de aparitie a Gazetei Matematice; studiul este reluat
si n volumul omagial dedicat centenarului Universitatii din Iasi [15]. Monografiile
[1], [11], dar si articolele [2], [4], [13] dau cititorului noi surse de informare.
Constantin Climescu a fost, prin bog
atia si varietatea subiectelor publicate si
sacrificiile materiale f
acute, sustin
atorul principal si sufletul Recreatiilor Stiin
tifice.
Pe coperta interioara a revistei din al VI-lea an de aparitie este scris: "Redactia si
Administratia la Dl. C. Climescu, Profesor la Facultatea de Stiin
te, Strada Butu 22."
Aceeasi adres
a apare si n casetele ce urmeaz
a titlul n fiecare num
ar din acest ultim
an.
Aparitia revistei Recreatii Stiin
tifice, nvingnd dificultati de tot felul, reprezinta
un act de curaj, d
aruire, ntelepciune si clarviziune. Revista a reusit ca, n cei sase
ani de existenta, s
a contribuie la ridicarea nivelului nv
atamntului din tara noastr
a,
n special al celui matematic.

Bibliografie
1. G. S
t. Andonie - Istoria matematicii n Romnia, vol. I, Ed. stiintifica, Bucuresti,
1965 (pp. 236-240).
2. Gh. Bantas - O pagina din istoria matematicii romnesti: centenarul revistei
"Recreatii Stiin
tifice", Probleme de istoria si filozofia stiintei, vol. X, 1984, Filiala
Iasi a Academiei Romne, 15-30.
3. S
t. Brs
anescu, F. Brs
anescu - Educatia, nvatamntul, gndirea pedagogica din
Romnia. Dictionar cronologic, Ed. stiintifica si enciclopedica, Bucuresti, 1978.
5

4. T. Brsan - Recreatii Stiin


tifice - "cea ntai brazda", Recreatii Matematice, 5(2003),
nr. 1, 1-5.
5. T. Brsan, D. Tiba - O suta de ani de la publicarea tezei de doctorat a lui Dimitrie
Pompeiu, Recreatii Matematice VII(2005), nr.2, 85-89.
6. T. Brsan, D. Tiba - One hundred years since the introduction of the set distance
by Dimitrie Pompeiu, in IFIP, vol.199, "System modeling and optimization", F. Ceragioli, A. Dontchev, H. Furuta, K. Marti, L. Pandolfi eds., Springer, Boston, 2006,
35-39.
7. M. Bordeianu, P. Vladcovschi - nvatamntul romnesc n date, Junimea, Iasi,
1979.
8. Gh. Gheorghiev, D. Iesan - Miltiade Tzony primul profesor de mecanica la
Universitatea din Iasi, Probleme de istoria si filozofia stiintei, vol. X, 1984, Filiala
Iasi a Academiei Romne, 125-146. Ap
arut
a, ntr-o form
a prescurtat
a, si n l. englez
a:
Miltiade Tzony the first professor in Mechanics at the University of Iasi, Noesis,
Travaux du Comit Roumain dHistoire et de Philosophie des Sciences, VI(1978),
55-60.
9. I. Ionescu - Constituirea, administrarea si redactarea "Gazetei Matematice", ap
arut
n volumul jubiliar Gazeta Matematica, 1895-1935. Istoric-nvataminte, Biblioteca
"Gazetei Matematice", vol. XI, Bucuresti, 1935 .
10. G. Iv
anescu - Istoria limbii romne, Junimea, Iasi, 2000.
11. N. Mih
aileanu - Reviste de matematici elementare din Romnia (pna la 1948),
Ed. Gil, Zalau, 1995.
12. P. Mih
ailescu - Primary cyclotomic units and a proof of Catalans conjecture,
J.Reine Angew. Math. 572 (2004), 167-196.
13. R. Miron - Centenarul revistei "Recreatii Stiin
tifice", Probleme de istoria si filozofia stiintei, vol. X, 1984, Filiala Iasi a Academiei Romne, 17-19.
14. I. Popa - "Recreatii Stiin
tifice"- precursoare a "Gazetei Matematice", Gazeta
Matematica si Fizica, seria A, nr. 9, 1955, 492-493.
15. I. Popa - Dezvoltatrea matematicii, ap
arut n Contributii la istoria dezvoltarii
Universitatii din Iasi, vol. II, pp. 7-39, Bucuresti, 1960.
teica - Rolul "Gazetei Matematice" n dezvoltarea stiintei matematice n
16. G. Ti
Romnia, aparut n volumul jubiliar Gazeta Matematica, 1895-1935. Istoric-nvataminte, Biblioteca "Gazetei Matematice", vol. XI, Bucuresti, 1935, 67-75.
17. *** - Introducere, Gazeta Matematic
a, an I, nr. 1, septembrie 1895.
18. *** - "Recreatii Stiin
tifice"- prezenta n constiinta posteritatii, Recreatii Matematice, 5(2003), nr.1, p.5.

Temistocle BRSAN, Iasi

Dan TIBA, Bucuresti


6

Proiect de reeditare
n anul 1999, la Iasi, a aparut revista Recreatii Matematice, iar ulterior, n 2005,
a fost nfiintata o asociatie cu acelasi nume, Asociatia "Recreatii Matematice", cu
membri n toat
a tara. Continuarea traditiei revistei Recreatii Stiin
tifice prima
revist
a stiintific
a (predominant matematic
a) adresat
a tineretului este un obiectiv
important al noii asociatii: nca mai sunt valabile unele din observatiile continute n
cuvntul "Catra Cetitori" din ianuarie 1883.
Dar peisajul publicatiilor matematice romnesti adresate tineretului studios este
mult mai bogat acum ca n epoca de nceput: Gazeta Matematica apare n dou
a
serii, A si B, apar Revista Matematica din Timisoara, Arhimede (Bucuresti), Octogon
(Brasov), Foaie Matematica (Chisinau) si multe alte publicatii locale. Se sustin numeroase concursuri de matematic
a, la nivel local, regional, national si international.
Chiar si ast
azi, n aceste conditii de exigenta sporit
a, vechea revist
a Recreatii
Stiin
tifice prezinta interes, prospetime si utilitate.
Asociatia "Recreatii Matematice" a considerat o datorie de onoare reeditarea integral
a a colectiei revistei Recreatii Stiin
a, nemodificat
a, prin
tifice n forma originar
utilizarea tehnicilor moderne de reproducere a textelor si si-a fixat ca termen limit
a
de realizare a acestui proiect data de 15 ianuarie 2008.
La ora actual
a, exist
a doar putine exemplare complete ale colectiei revistei Recreatii Stiin
a ntr-o stare destul de precar
a. La Seminarul Matematic
tifice si care se afl
"Al. Myller" din Iasi exist
a dou
a colectii complete ale Recreatiilor Stiin
tifice: una
donata acestei biblioteci de C. Climescu, fondator si principalul animator al revistei
si o a doua donat
a de V. I. Praja, redactor fondator si colaborator al ei. Biblioteca
Academiei Romne are, de asemenea, colectia complet
a a Recreatiilor S
tiintifice.
Redarea n circuitul public a acestui monument de nceput al culturii stiintifice
romnesti, este de un real folos tinerimii studioase si unor cercuri largi de cititori.
Realizarea acestui proiect n-ar fi fost posibil
a f
ar
a sprijinul entuziast si neconditionat al doamnei Marinela Ghigea si al firmelor Kepler Systmes dInformation si
Dazoot din Bucuresti. Exprimam pe aceasta cale multumirile noastre cele mai sincere
pentru ntelegerea si efortul depus pe parcursul a aproape doi ani de munca.
Reeditarea cuprinde cele sase volume originale ale revistei si o brosur
a care include
o introducere, o not
a asupra editiei, erat
a, index de autori, de rezolvitori, de subiecte
si de surse (partial) si o galerie de portrete. n varianta electronica, aceste instrumente
sunt interactive.
Republicarea actual
a, inclusiv n format electronic, are ca scop prentmpinarea
riscului disparitiei acestei opere si, pe de alt
a parte, s
a o fac
a accesibil
a gratuit ca
CD (sau on-line la adresa http://www.recreatiistiintifice.ro , de unde se va
putea prelua gratuit). Speram ca aceasta actiune, dedicat
a anivers
arii a 125 de
ani de la aparitia revistei, va stimula specialistii s
a reanalizeze fenomenul stiintific
n fascinantul secol XIX, veac de ctitorie n stiinta romneasc
a modern
a.
Asociatia "Recreatii Matematice"
7

Polinoame Fibonacci, polinoame ciclotomice


Loredana STRUGARIU, Ciprian STRUGARIU 1
Deoarece sirul lui Fibonacci este cunoscut elevilor nca din cl. a IX-a, iar radacinile
de ordinul n ale unit
atii si polinoamele ciclotomice sunt n materia prev
azut
a la
cl. a X-a pentru olimpiada de matematic
a, consider
am c
a abordarea unui asemenea
subiect este util
a att elevilor ct si profesorilor. Vom prezenta cteva rezultate
privind polinoamele Fibonacci si cele ciclotomice si legatura dintre ele.
1. Polinoame Fibonacci - definire, leg
atura cu triunghiul lui Pascal.
Polinoamele Fibonacci sunt definite prin relatia de recurenta
cu F1 (x) = 1 si F2 (x) = x

Fn+1 (x) = xFn (x) + Fn1 (x) ,

(1)

sau prin urmatoarea formula explicita


[(n1)/2]

n j 1 n2j1
,
(2)
x
j
j=0

nj1
j
. Se convine ca
unde [x] este partea ntreag
a a lui x, iar
Cnj1
j
F0 = 0.
Observatie. Polinoamele Fibonacci pot fi definite prin relatia de recurenta
Fn (x) =

cu F0 (x) = 0 si F1 (x) = 1.

Fn (x) = xFn1 (x) + Fn2 (x) ,

(10 )

Exemple: F1 (x) = 1, F2 (x) = x, F3 (x) = x2 +1, F4 (x) = x3 +2x, F5 (x) = x4 +3x2 +1


etc. Lund x = 1 n (2), obtinem Fn (1) = Fn , unde Fn este sirul lui Fibonacci.
Lema 1 (proprietatea de divizibilitate). Daca m este divizor al lui n, atunci
Fm este divizor al lui Fn . Daca p este un numar prim, atunci Fp (x) este polinom
ireductibil.
Teorema 1. Fie F0 , F1 , F2 , . . . polinoamele Fibonacci peste cmpul Galois de
dimensiune 2. Atunci, avem:
1) F2n+1 sunt singurii termeni de grad par si nu sunt divizibili cu x; F2n sunt
singurii termeni de grad impar, n 0;
2) Fnt + Fn+t = xFn Ft , pentru 0 t n;
3) F2n = xFn2 , pentru n 0;
2
4) F2n+1 = Fn2 + Fn+1
, pentru n 0;
5) Fmn (x) = Fm (x) Fn (xFm (x)), pentru pentru m, n 0;
6) F2mnp = xFmn Fmnp + Fp , pentru 0 p mn;
7) F2mn+p = xFmn Fmn+p + Fp .

k
Radacinile polinomului Fn (x) sunt de forma xk = 2i cos
, pentru k =
n
1, . . . , n 1. Pentru p num
ar prim, aceste r
ad
acini sunt de 2i ori partea real
a a
1

Profesori, Colegiul National "Eudoxiu Hurmuzachi", R


ad
auti (Suceava)

r
ad
acinilor polinomului ciclotomic de ordinul p. Analiznd coeficientii primelor polinoame Fibonacci se observa legatura dintre triunghiul lui Pascal si aceste polinoame:
F1 (x) = 1x0
F2 (x) = 1x1
F3 (x) = 1x2 + 1x0
F4 (x) = 1x3 + 2x
F5 (x) = 1x4 + 3x2 + 1x0
F6 (x) = 1x5 + 4x3 + 3x1
F7 (x) = 1x6 + 5x4 + 6x2 + 1x0
F8 (x) = 1x7 + 6x5 + 10x3 + 4x1
F9 (x) = 1x8 + 7x6 + 15x4 + 10x2 + 1x0
F10 (x) = 1x9 + 8x7 + 21x5 + 20x2 + 5x1 .
A. N. Philippou si asociatii sai [4] au studiat polinoamele Fibonacci de ordinul
k, k 2, pe care le-au definite astfel:
(k)

(k)

F0 (x) = 0, F1 (x) = 1
n
P
(k)
(k)
Fn (x) =
xkj Fnj (x) ,
(k)
Fn

(x) =

j=1
k
P
j=1

kj

(k)
Fnj

(x) ,

n = 2, 3, . . . , k;

(3)

n = k + 1, k + 2, . . .

Observatie. Pentru k = 2 acestea se reduc la Fn (x), iar pentru k = 1 se reduc


(k)
la sirul lui Fibonacci Fn de ordinul k.
2. Polinoame ciclotomice - definire, propriet
ati. Pentru fiecare num
ar
natural n 1 radacinile complexe ale ecuatiei xn = 1 se numesc radacinile de ordin
n ale unitatii. Acestea sunt numere complexe de forma
2k
2k
xk = cos
+ i sin
, k = 1, 2, . . . , n 1.
(4)
n
n
Multimea acestor radacini se noteaza cu
Un = {x C | xn = 1} .

Teorema 2. Multimea Un este un grup ciclic fata de nmultirea numerelor complexe, numit grupul radacinilor de ordinul n ale unitatii.

2k
Propozitie. Fie Un = xk = cos 2k
n + i sin n ; k = 1, 2, . . . , n 1. . Atunci
hxk i = Un (k, n) = 1.

(5)

Grupul ciclic Un are (n) generatori, unde (n) este numarul numerelor naturale
mai mici ca n, relativ prime cu n (indicatorul lui Euler ). Cele (n) radacini de
ordinul n ale unit
atii care genereaz
a grupul Un , adic
a numerele complexe de forma
2k
2k
xk = cos
+ i sin
, k = 1, 2, . . . , n 1, (k, n) = 1.
(6)
n
n
se numesc radacinile primitive de ordinul n ale unitatii. Not
am cu Pn multimea
r
ad
acinilor primitive de ordinul n ale unit
atii si cu o r
ad
acin
a primitiv
a (oarecare)
de ordinul n a unitatii.
9

Teorema 3.

n
o
Pn = k | 0 k n 1, (k, n) = 1 .

(7)

Observatie. Dac
a n = p = num
ar prim, atunci Pp = Up \ {1}.
S
Pd = Un ,
Teorema 4. 1)
d|n
T
2) Pd1 Pd2 = , d1 , d2 divizori naturali ai lui n, d1 6= d2 .

Polinomul monic ale c


arui r
ad
acini sunt r
ad
acinile primitive de ordinul n ale
unit
atii, se numeste al n-lea polinom ciclotomic si are forma
Y
n (X) =
(X ) , n N .
(8)
Pn

Gradul polinomului ciclotomic n (X) este egal cu cardinalul multimii Pn , adic


a
(n).
Teorema 5 (relatia lui Dedekind ).
Xn 1 =

d (X)

(9)

d|n

unde produsul se face dupa toti divizorii naturali ai lui n.


Demonstratie. Folosind descompunerea n factori a polinomului X n 1 si Teorema 4, avem

Y
Y
Y Y
Y

Xn 1 =
(X ) =
(X ) =
(X ) =
d (X) .
Un

Pd

d|n

Pd

d|n

d|n

Teorema 6. Pentru p > 0, numar prim, avem:


p (X) = X p1 + X p2 + + X + 1.
k1
Observatie. pk (X) = p X p
ar prim, p > 0.
, k N , p num
Exemple. Primele 10 polinoame ciclotomice:
1 (X) = X 1
2 (X) = X + 1
3 (X) = X 2 + X + 1
4 (X) = X 2 + 1
5 (X) = X 4 + X 3 + X 2 + X + 1
6 (X) = X 2 X + 1
7 (X) = X 6 + X 5 + X 4 + X 3 + X 2 + X + 1
8 (X) = X 4 + 1
9 (X) = X 6 + X 3 + 1
10 (X) = X 4 X 3 + X 2 X + 1.

(10)

Teorema 7 (relatia lui Mbius-Dedekind ). Pentru orice n N exista relatia:


Y
( n )
n (X) =
Xd 1 d ,
(11)
d|n

10

unde : N {1, 0, 1} este functia lui Mbius, dat


a prin

a n = 1,
1, dac
k
(n) =
(1) , dac
a n = p1 p2 pk (p1 ,p2 ,. . . ,pk prime distincte),

0, daca n se divide prin patratul unui numar prim.


Alte propriet
ati ale polinoamelor ciclotomice:
1) n (X) Z [X], n N ,
2) n (X) este ireductibil n inelul Z [X], n N ,
3) n (X) este un polinom reciproc, n 2,
4) Pentru n N si p > 0 un numar prim avem:
i) daca p divide n, np (X) = n (X p ),
n (X p )
ii) daca p nu divide n, np (X) =
,
n (X)
5) 2n (X) = n (X), pentru n > 1, num
ar natural impar.

3. Leg
atura dintre polinoamele Fibonacci si polinoamele ciclotomice.
Aceast
a leg
atur
a a fost expus
a de K. Kuwano n The Design of Mathematic Workshop (n japoneza), Scientist, 2004 si apoi preluata de K. Motose n [3].
Considernd doua variabile x si y si notnd cu X = x + y si Y = xy, vom defini
polinoamele simetrice Fn (X, Y ) prin
xn y n
,
(12)
xy
numite polinoame Fibonacci de doua variabile.
Exemple. F0 = 0, F1 = 1, F2 = x + y = X, F3 = x2 + y 2 + xy = X 2 Y etc.
Fn (X, Y ) =

Lema 2. i) Fm+n = Fm Fn+1 Y Fm1 Fn , pentru n 0 si m 1; n particular,


Fn+2 = XFn+1 Y Fn .

(13)

ii) Daca m este divizor al lui n, atunci Fm este divizor al lui Fn .


Demonstratie. i) direct prin nlocuire n formula (12).
ii) Deoarece xm y m este divizor al lui xn y n , rezult
a afirmatia.

Observatie. Dac
a consider
am n formula (13) X = 1 si Y = 1, obtinem
sirul lui Fibonacci, fapt pentru care polinoamele Fn (X, Y ) au fost numite polinoame
Fibonacci. O alt
a form
a a polinoamelor Fibonacci de dou
a variabile este:

[(n1)/2]
X
n j 1 n2j1 j
Fn (x, y) =
y , n 1.
(14)
x
j
j=0
n mod inductiv, vom defini polinoamele ciclotomice de doua variabile
Y
1 (x, y) = x y, xn y n =
d (x, y) .
d|n

Lema 3. i) n (x, y) =

( nd )
Q d
.
x yd

d|n

ii) n (x) = n (x, 1).


iii) n (x, y) = n (y, x), pentru n 2.

11

(15)

Demonstratie. i) rezult
a din formula de inversiune aP
lui Mbius,
iar ii) din i) si

definitia lui n (x). iii) decurge din punctul i) si formula
nd = 0 pentru n 2.
d|n

Deoarece polinoamele n (x, y) sunt simetrice pentru n 2, putem defini polinoamele Pn (X, Y ) unde X = x + y, Y = xy astfel nct n (x, y) = Pn (X, Y ). De
exemplu, P6 = x2 + y 2 xy = X 2 3Y .
Teorema 8. Vom conveni ca P1 = 1. Atunci, avem:
1) Pn este
Qireductibil n Z [X, Y ];
2) Fn =
Pd ;
d|n

3) Pn =

d|n

( n
d)

Fd

4) (Fm , Fn ) = F(m,n) , unde ( , ) reprezinta c.m.m.d.c.; n particular (Fn , Fn+1 ) = 1.


Demonstratie. 1) Pn Z [X, Y ] din definitie. Daca Pn = ST , cu S, T Q[X, Y ]
polinoame neconstante, atunci (x) = (x, 1) = P (x+1, x) = S (x+1, x) T (x+1, x)
pentru polinoamele neconstante S (x + 1, x) , T (x + 1, x) Q [x], contrar ireductibilitatii peste Q.
2) Rezulta din urmatoarea ecuatie:
Y
Y
xn y n
Fn (X, Y ) =
d (x, y) =
Pd (x, y) .
=
xy
1d|n

d|n

3) Rezult
a din formula de inversiune a lui Mbius.
4) Dac
a mai nti consider
am Pd = Pd0 atunci avem:
d (x) = d (x, 1) = Pd (x + 1, x) = Pd0 (x + 1, x) = d0 (x)
si astfel d = d0 . Din Lema 2, ii), stim ca F(m,n) este divizor comun al lui Fm si Fn .
Daca Pd este divizor comun al lui Fm si Fn , atunci d este divizor comun al lui m
si n si deci d este divizor al lui (m, n). Astfel Pd este divizor al lui F(m,n) . Deci, dac
a
D este divizor comun al lui Fm si Fn , atunci D este divizor al lui F(m,n) , deoarece D
este un produs al polinoamelor ireductibile distincte Pd , ceea ce implica afirmatia.
Bibliografie
1. T. M. Apostol - Resultants of Cyclotomic Polynomials. Proc. Amer. Math. Soc.
24(1970), 457-462.
2. T. Koshy - Fibonacci and Lucas Numbers with Applications. New York: Wiley,
2001.
3. K. Motose - On values of cyclotomic polynomials. VII, Math J. Okayama Univ.,
2004.
4. A. N. Philippou, C. Georghiou, G. N. Philippou - Fibonacci polynomials of
order k, multinomial expansions and probability, Internat. J. Math. Math. Sci.,
6(1983), 545-550.

5. M. Tena
- Radacinile unitatii, Soc. S
t. Mat., Bucuresti, 2005.
6. W. A. Webb, E. A. Parberry - Divizibility Properties of Fibonacci Polynomials,
Fibonacci Quarterly 7.5 (1969), 457-463.
7. http://mathworld.wolfram.com
12

Submultimi ale unei multimi finite si matrici binare


Adrian REISNER1
Fie data o multime X de cardinal |X| finit. Consideram familia F de submultimi
ale lui X avnd anumite propriet
ati. Utiliznd matricele binare (adic
a acelea ce au
ca elemente 0 sau 1), vom demonstra cteva rezultate privind familia F.

I Familii cu proprietatea P. Fie F o familie de submultimi X1 , X2 , . . . , Xn


ale multimii {1, 2, . . . , n}. Spunem c
a familia F verific
a proprietatea P, dac
a ea
ndeplineste conditiile urm
atoare:
a) |Xi | = + , unde , N , i {1, 2, . . . , n};
b) |Xi Xj | = , i, j {1, 2, . . . , n}, i 6= j.
Spunem c
a familia F verific
a proprietatea duala P 0 a propriet
atii P dac
a ndeplineste conditiile:
a0 ) j {1, 2, . . . , n} apartine la exact + submultimi Xi ale lui F;
b0 ) i, j {1, 2, . . . , n} si distincte apartin la exact submultimi ale lui F.
Numim matrice asociata familiei F, matricea binara A = (aij ) Mn (R) definita
prin aij = 1 dac
a i Xj si aij = 0 n caz contrar. De asemenea, dat
a o matrice
binar
a A, se poate face trecerea "invers
a" la o familie de submultimi ale lui X ntr-un
mod evident.
Pentru familiile cu proprietatea P, ne propunem s
a g
asim o relatie ntre n, ,
si s
a demonstr
am proprietatea dual
a P 0.

Propozitia 1. Sunt adevarate afirmatiile:


1. F are proprietatea P t AA = I + J (1);
2. F are proprietatea P 0 A t A = I + J (10 ),
unde I este matricea unitate si J este matricea cu toate elementele 1.
Demonstratie. Calculnd produsul t AA, tinnd seama de conditiile a) si b),
obtinem matricea avnd elementele de pe diagonala principal
a egale cu + si
celelalte egale cu , adica matricea I + J; formula (1) este astfel stabilita.
n
n
P
P
Invers, dac
a (1) are loc, avem |Xi | =
aki =
aki aki = elemetul de indice i
(1)

k=1

k=1

a proprietatea a). La fel, |Xi Xj | =


de pe diagonala matricei A t A = + , adic
n
P
aki akj = , adica proprietatea b).

k=1

Afirmatia 10 se dovedeste cu argumente similare.

p
Propozitia 2. Daca F are proprietatea P, atunci A GLn (R) si (+n) n1
Zn .
Demonstratie. Sa dovedim ca A este inversabila, i.e. A GLn (R). Avem:

1
+
1
...
1

...

+ ...


+ ...

=
= ( + n)
det (I + J) =

...
...
. . .
...
...
...
. . .
...

. . . +

. . . +
= ( + n) n1 .

Cercet
ator, Centrul de calcul E.N.S.T., Paris

13

(ultima egalitate se obtine n urma sc


aderii primei coloane din
p celelalte). Deci,
2
t
n1
si detA = (+n) n1 6= 0,
(detA) = det ( AA) = det (I +J) = (+n)
unde = 1. Demonstratia se ncheie, deoarece pentru orice matrice binar
a A avem
det A Z.

Propozitia 3. Daca familia F are proprietatea P, atunci + n = ( + )2 .


Demonstratie. Prin calcul direct, obtinem c
a JA = ( + ) J (3) sau
1
1
1
a) sau JA =
J = ( + ) JA (A fiind inversabil
J (4).
+
Din (1) deducem c
a J t AA = J + J 2 = ( + n) J, de unde, tinnd seama de
+ n
+ n
(4), J tA = ( + n) JA1 =
J. Ca urmare, AJ = t (J tA) =
J (5)
+
+
+ n
+ n 2
si JAJ =
J =n
J. Pe de alta parte, datorita relatiei (3), JAJ =
+
+
+ n
( + ) J 2 = n ( + ) J. n consecinta, n
= n ( + ) si rezult
a c
a (3) este
+
adev
arat
a.
Propozitia 4. F avnd proprietatea P, au loc relatiile:
1 AJ = JA,
2 t AA = A t A (i.e. A este matrice normala).
(5) + n
(2)
(3)
Demonstratie. 1 AJ =
J = ( + ) J = JA.
+

(1)
1
2 t AA = I + J = I + t A1 t A J = I + t A1 (t AJ) =

(1)
= I + t A1 J t A = t A1 (I + J) t A = t A1 t AA t A = A t A.

Propozitia 5. Daca familia de submultimi F are proprietatea P, atunci ea are


si proprietatea duala P 0 .
Demonstratie. Conform Propozitiilor 1 si 4, avem t AA = I + J si t AA =
t
A A. Deci A t A = I + J si, conform punctului 10 al Propozitiei 1, deducem c
aF
are proprietatea P 0 .

Caz particular si exemplu. Pentru = 1, relatiile (1) si (2) se scriu t AA =


I + J si n = 2 + + 1 = ( + 1) + 1 (deci n este impar!). Atunci det A =
p
(+1)
( + n) n1 = ( + 1) 2 Z . De exemplu, matricea binar
a

1 1 1 0 0 0 0
1 0 0 1 1 0 0

1 0 0 0 0 1 1

A=
0 1 0 1 0 1 0
0 1 0 0 1 0 1

0 0 1 1 0 0 1
0 0 1 0 1 1 0
verific
a t AA = A t A = 2I + J. Avem = 2 si det A = 24.

II Familii cu proprietatea R. Fie X o multime de cardinal n 3 si


{X1 , X2 , . . . , Xm } o familie de submultimi strict incluse n X. Spunem ca aceasta
14

familie verific
a proprietatea R dac
a pentru orice pereche (i, j) X 2 exist
a si este
unica o multime Xk din familie astfel nct {i, j} Xk .
Asociem matricea binara B = (bij ) Mn,m (R) lund bij = 1 daca i Xj si

seama de proprietatea R si utiliznd notatii evidente


bij = 0 n caz contrar. Tinnd
pentru elementele unei matrice relativ la matricea B t B Mn (R) avem:
(i) (B t B)ij = 1 daca i 6= j, caci elementul (B t B)ij corespunde la numarul
submultimilor continnd {i, j};
(ii) (B t B)ii = di , unde di este num
arul submultimilor ce contin elementul i.
Asadar,

d1
1 ... 1
1
d2 . . . 1

B tB =
. . . . . . . . . . . . .
1
1 . . . dn
Fie Xk o submultime ce contine elementul i. Deoarece Xk 6= X, exist
a j 6= i cu
j
/ Xk . Ca urmare, exist
a cel putin o submultime Xl diferit
a de Xk continnd pe i.
Rezulta ca di > 1, adica di = 1 + ai , iar ai > 0.
Propozitia 6. Daca familia {X1 , X2 , . . . , Xm } are proprietatea R, atunci matricea B t B Mn (R) este inversabila, i.e. B t B GLn (R).
Demonstratie. Consider
am

x + a1
x
...
x

x
x + a2 . . .
x
f (x) =
, x R.
...
...
. . .
...
x
x
. . . x + an

Observ
am c
a f este o functie polinomial
a si f (1) = det (B t B).
o sum
a de n determinanti avnd o coloan
a (coloana derivat
a!) cu
egale cu 1:

x + a1
1
x
...
x
x

1
x
+
a
x + a2
.
.
.
x
2
0

+ +
f (x) =
...
...
. . .
...
...
. . .
x
1
x
. . . x + an
x

Derivata f 0 este
toate elementele
...
...
...
...

1
1
.
. . .
1

Derivata f 00 va fi o sum
a de determinanti avnd dou
a coloane cu elementele egale cu
1; deci f 00 (x) = 0, x R.
Ca urmare, f (x) = ax + b si, din f (0) = b si f 0 (0) = a, gasim
n
Y
Y
Y
ai , a =
ai + +
ai .
b=
i=1

i6=1

i6=n

n sfrsit,
n
Y
Y
Y

ai +
ai + +
ai 6= 0,
det B t B = f (1) = a + b =
i=1

adica B t B GLn (R).

i6=1

i6=n

Observatie. n ipotezele Propozitiei 6, avem rang (B t B) = n m (deoarece


rang (B t B) = n rang B m).
15

O generalizare a teoremelor Stolz-Cesaro


1
Sorin PU
SPANA
1. Rezultate clasice. Vom prezenta n aceast
a prim
a parte bine-cunoscutele
teoreme Stolz-Cesaro si o reciproca a lor, omitnd demonstratiile (pentru care pot fi
consultate [2] si [3]).
Teorema 1. Daca (an )n1 si (bn )n1 sunt doua siruri de numere reale astfel
an+1 an
nct: i) sirul (bn )n1 este strict crescator si nemarginit, ii) lim
=
n bn+1 bn
an
= l.
l R, atunci lim
n bn
Teorema 2. Daca (an )n1 si (bn )n1 sunt doua siruri de numere reale astfel
an+1 an
nct: i) lim an = lim bn = 0, ii) lim
= l R, iii) sirul (bn )n1
n
n
n bn+1 bn
an
este strict descrescator, atunci lim
= l.
n bn
Teorema 3. Daca (an )n1 si (bn )n1 sunt doua siruri de numere reale astfel
bn+1
an
an+1 an
nct: i) lim
R\{1}, ii) lim
= l R, atunci lim
= l.
n bn
n bn
n bn+1 bn

2. Generaliz
ari. Teorema 1 admite urm
atoarea generalizarea (cu pierderea
cazului limitei infinite):
Teorema 4. Daca (an )n1 si (bn )n1 sunt doua siruri de numere reale ast

n1
P
1
|bi+1 bi |
este marginit, iii)
fel nct: i) lim |bn | = , ii) sirul |bn |
n

i=1

n1

an+1 an
an
= l R, atunci lim
= l.
lim
n bn+1 bn
n bn
Demonstratie. Daca > 0 si M este un majorant pentru sirul de la ii), atunci
exist
a m N astfel nct n m s
a avem

an+1 an

bn+1 bn l < 2M |an+1 an l (bn+1 bn )| < 2M |bn+1 bn |


|an am l (bn bm )| <

Obtinem astfel

Dar

n1
X

|bi+1 bi | |bn | ,
2M i=m
2

|bn |
an am
<

l
2 |bn bm | ,
bn bm

n m.

n > m.

am lbm
an
an am
bn bm

<
+
l
bn l =
bn
bn bm
bn

|bn bm |
|am lbm | an am
|am lbm |

<
l
+
<
+ < + = ,
|bn |
bn bm
|bn |
|bn |
2
2 2
1

Profesor, Craiova

16

n m.

Corolarul 1. Daca (an )n1 si (bn )n1 sunt doua siruri de numere reale astfel

|bn+1 bn |
nct: i) sirul (|bn |)n1 este strict crescator si nemarginit, ii) sirul
|bn+1 ||bn | n1
an+1 an
an
este marginit, iii) lim
= l R, atunci lim
= l.
n bn+1 bn
n bn
ntr-adevar, ipotezele i) si ii) implica primele doua ipoteze ale Teoremei 4.

Teorema 2 admite urmatoarea generalizare (cu pierderea cazului limitei infinite):


Teorema 5. Daca (an )n1 si (bn )n1 sunt doua siruri de numere reale astfel

P
1 n1
|bi+1 bi |
este marginit,
nct: i) lim an = lim bn = 0, ii) sirul
n
n
|bn | i=1
n1
an+1 an
an
= l R, atunci lim
= l.
iii) lim
n bn+1 bn
n bn
Demonstratie. Daca > 0 si M este un majorant pentru sirul de la ii), atunci
exist
a m N astfel nct
n m s
a avem

an+1 an

bn+1 bn l < M |an+1 an l (bn+1 bn )| < M |bn+1 bn |


|an+p an l (bn+p bn )| <

n+p1
X
|bi+1 bi | < |bn+p | ,
M i=1

n m, p 1.

ns
a, pentru orice n N, exist
a un sir cresc
ator de numere naturale (p (k))k1 astfel

a |bn+p | |bn |, n m,
nct bn+p(k) |bn |, k 1, deci putem presupune c
p 1. Obtinem astfel |an+p an l (bn+p bn )| < |bn |, n m, p 1. Trecnd
an

la limita, dupa p , obtinem |an lbn | < |bn |


l < , n m, adica
bn
an
lim
= l.
n bn
Corolarul 2. Daca (an )n1 si (bn )n1 sunt siruri de numere reale astfel nct:
i) lim an = lim bn = 0, ii) sirul (|bn |)n1 este strict descrescator, iii) sirul
n
n
|bn+1 bn |
an+1 an
an
este marginit, iv) lim
= l R, atunci lim
= l.
n
n
|bn+1 ||bn | n1
bn+1 bn
bn
Observatii. i) Daca n cele doua teoreme si corolare nlocuim (an )n1 si (bn )n1
n
n
cu ((1) an )n1 si, respectiv, ((1) bn )n1 obtinem c
a, pe lng
a celelalte ipoteze,

P
|bn+1 + bn |
1 n1
si din
|bi+1 + bi |
respectiv
din marginirea sirului
|bn | i=1
|bn+1 | |bn | n1
n1
an+1 + an
an
= l R avem lim
= l.
lim
n bn+1 + bn
n bn
i) ca sirul

ii) Teoremareciproca 3 poate fi mbunatatita cernd n locul ipotezei


an+1 an

|bn+1 | + |bn |
sa fie marginit. ntr-adevar pentru > 0 avem
l <
|bn+1 bn | n1
bn+1 bn
|bn+1 | + |bn |

, n m.
|bn+1 bn |
17

iii) Din demonstratiile date, si din enunturile teoremelor si corolarelor, este evident ca ele ramn valabile si n ipoteza ca (an )n1 si (bn )n1 sunt siruri de numere
complexe. De fapt rezultatele pot fi extinse si n cadru mult mai larg al algebrelor
normate cu unitate; enunturile rezultatelor de mai sus se adapteaz
a cu usurinta, iar
demonstratiile se fac cu aceleasi argumente.
3. Aplicatii.
Problema 1. Fie (xn )n1 si (un )n1 doua siruri de numere reale astfel nct
lim un = u (1, ). Atunci sirul (xn )n1 are limita daca si numai daca sirul
n
1
(un xn+1 xn )n1 are limita, caz n care avem lim xn =
lim (un xn+1 xn ).
n
u 1 n
Indicatie. Se aplic
a Teoremele 1 si 3 sirurilor an = u1 u2 un1 xn si bn =
u1 u2 un1 .
Folosind Corolarul 1 putem extinde acest rezultat si pentru valori negative ale lui
u, pierznd nsa cazul limitelor infinite, ca mai jos:
Problema 2. Fie (xn )n1 si (un )n1 doua siruri de numere reale astfel nct
lim un = u, |u| > 1. Atunci sirul (xn )n1 este convergnt daca si numai daca sirul
n
1
(un xn+1 xn )n1 este convergent, caz n care avem lim xn =
lim (un xn+1 xn ).
n
u1 n
Observatie. Rezultatul ramne valabil dac
a sirurile ce intervin n enunt sunt
siruri de numere complexe. Urm
atoarea problem
a ne prezint
a n ce conditii rezultatul
anterior ramne valabil n cazul |u| < 1.
Problema 3. Fie (xn )n1 si (un )n1 doua siruri de numere reale astfel nct
(xn )n1 sa fie marginit iar lim un = u, |u| < 1. Atunci sirul (xn )n1 este convern
gent daca si numai daca sirul (un xn+1 xn )n1 este convergent, caz n care avem
1
lim xn =
lim (un xn+1 xn ).
n
u 1 n
Indicatie. Se aplic
a Corolarul 2 sirurilor an = u1 u2 un1 xn si bn = u1 u2 un1 .
Din nou facem observatia ca rezultatul ramne valabil si pentru siruri de numere
complexe. Este evident c
a tinnd seama de observatia i) rezultatele anterioare r
amn
valabile dac
a nlocuim un xn+1 xn cu un xn+1 + xn si u 1 cu u + 1.
Toate acestea pot fi restrnse n
Problema 4. Fie (xn )n1 un sir marginit de numere complexe, iar (un )n1

si (vn )n1 doua siruri convergente de numere complexe astfel nct lim un 6=
n

Atunci sirul (xn )n1 este convergent daca si numai daca sirul
lim vn .
n

(un xn+1 + vn xn )n1 este convergent, caz n care avem

lim (un xn+1 + vn xn )

lim xn =

lim un + lim vn

Problema urmatoare este demonstrata n [4] si generalizeaza Problema 2.


18

Problema 5. Fie a0 , a1 , ..., ak R, a0 6= 0 si (xn )nk un sir de numere reale cu


proprietatea ca sirul (yn )nk , yn = a0 xn + a1 xn1 + + ak xnk , este convergent.
Daca polinomul a0 xk + a1 xk1 + + ak are toate radacinile de modul subunitar,
atunci sirul (xn )nk este convergent.
Indicatie. Demonstratia se face prin inductie dupa k, primul pas al inductiei
reducndu-se la: || < 1 si (xn+1 xn )nk convergent (xn )nk convergent,
adic
a un caz particular al Problemei 2.
Am v
azut ns
a c
a dac
a cerem m
arginirea lui (xn )nk , atunci afirmatia anterioar
a
r
amne adev
arat
a pentru || 6= 1 si, prin urmare, teorema r
amne valabil
a dac
a
radacinile polinomului sunt n modul diferite de unitate. Obtinem astfel urmatoarea
generalizare:
(0)
(1)
(k)
Problema 6. Fie an n1 , an n1 ,. . . , an n1 , k + 1 siruri de numere
complexe, convergente respectiv catre a(0) , a(1) ,. . . , a(k) , si astfel nct radacinile
polinomului a(0) xk + a(1) xk1 + + a(k) sa fie n modul diferite de unitate. Atunci,
un sir marginit (xn )n1 este convergent daca si numai daca sirul

(0)
(1)
(k)
este convergent, caz n care avem
an xn + an xn1 + + an xnk
nk+1

(0)
(1)
(k)
lim an xn + an xn1 + + an xnk
.
lim xn = n
n
a(0) + a(1) + + a(k)
(0)

(1)

(k)

Demonstratie. Dac
a yn = an xn + an xn1 + + an xnk si zn = a(0) xn +
(k)
a xn1 + +a xnk atunci, cum (xn )n1 este m
arginit, rezult
a c
a lim (yn zn ) = 0.
(1)

Cum nsa (yn )n1 este convergent rezulta ca (zn )n1 este convergent si conform problemei precedente (care este valabil
a si pentru siruri de numere complexe) si observatiei f
acute, rezult
a c
a sirul (xn )n1 este convergent iar relatia final
a este evident
a.
P
Problema 7. (Jensen) Fie sirurile (xn )n1 si (yn )n1 astfel nct: seria
yn

n1
|y1 | + + |yn |
xn
este convergenta, sirul
este marginit si lim
= l. Atunci
n yn
|y1 + + yn | n1
x1 + + xn
lim
= l.
n y1 + + yn
Bibliografie

1. R. Cristescu - Analiza functionala, Ed. Didactic


a si Pedagogic
a, Bucuresti, 1979.
2. D.M. Batinetu - Siruri,
Ed. Albatros, Bucuresti, 1979.

3. D.M. B
atinetu, I.V. Maftei, I.M. Stancu-Minasian - Exercitii si probleme de
analiza matematica, Ed. Didactic
a si Pedagogic
a, Bucuresti, 1981.
4. O. Mayer - Teoria functiilor de o variabila complexa, Ed. Academiei, 1981.
5. D. Mihet - Despre calculul unor limite de siruri, R.M.T. nr. 2/1990.
6. D.-
St. Marinescu , V. Cornea - In legatura cu o problema din G.M.A., G.M.-A
nr. 3/2005.
7. D. Mihet, M. Piticari - O problema de convergenta, G.M.-A nr. 1/1990.
19

O problem
a de combinatoric
a destul de grea
Marian TETIVA1
n aceast
a not
a ne ocup
am de urm
atoarea
Problem
a. Din oricare 2n + 1 numere ntregi (distincte) ale caror module nu
depasesc pe 2n 1, se pot alege trei (tot distincte) care au suma zero.

Este o problema de olimpiada (cu regret, nu stiu unde am ntlnit-o) mai grea
dect pare la prima vedere. Pentru rezolvare, ns
a, nu avem nevoie dect de asanumitul "principiu al cutiei" ("principiul lui Dirichlet", mai este numit n literatura
matematica romneasca; "the pigeonhole principle", n cea engleza si americana):
daca vrem sa repartizam n + 1 obiecte n n cutii, atunci trebuie ca ntr-o cutie sa fie
mai mult de un obiect. De asemenea, nu e nevoie, n esenta, de alt tip de rationament
dect cel din problema lui Erds [1] (una din rarele probleme "foarte simple" propuse
de Erds). Aceasta problema cere
sa se arate ca pentru k > [(n + 1)/2] si a1 , . . . , ak numere ntregi astfel nct
1 a1 < < ak n exista doua printre ele a caror suma este tot unul din aceste
numere.
Solutia [2] este destul de simpl
a: s
a observ
am c
a a1 , . . . , ak pe de o parte, si a2 a1 ,
. . . , ak a1 pe de alta, sunt 2k 1 numere din multimea {1, . . . , n}. Cum 2k 1 > n
doua dintre acestea sunt egale (conform principiului cutiei!) si, pentru ca numerele
din prima grup
a sunt diferite dou
a cte dou
a (ceea ce e valabil si pentru numerele
din a doua grup
a), ajungem la o egalitate de forma ai = aj a1 ai + a1 = aj care
rezolva problema. Mai mult, pentru k = [(n + 1)/2], se pot gasi k numere printre
primele n numere ntregi pozitive astfel nct suma oricaror doua nu este egala cu un
al treilea dintre ele; de exemplu, numerele [n/2] + 1, [n/2] + 2, . . . , n (tot din solutia
citat
a).
n cele ce urmeaz
a vom ar
ata cum se rezolv
a problema de la care am pornit folosind aceeasi idee (de mai multe ori vom utiliza acest tip de rationament),
considernd-o folositoare pentru cei care se preg
atesc pentru olimpiade, sau, n general, pentru o activitate matematic
a sustinut
a.
Solutia problemei. Avem de considerat cteva cazuri.
Primul (si cel mai simplu) dintre ele este acela n care 0 este unul dintre numerele
a1 , . . . , a2n+1 . ntr-adevar, 2n numere din cele 2n + 1 sunt nenule, deci modulele lor
apartin multimii {1, 2, . . . , 2n 1}; atunci (conform principiului cutiei) dou
a dintre
module trebuie s
a fie egale, iar o asemenea egalitate furnizeaz
a dou
a numere care au
suma 0. Aceste doua numere si 0 sunt, desigur, cele trei numere cautate (care au
suma 0).
Consider
am acum c
a 0 nu este printre cele 2n + 1 numere, pe care le not
am cu
(2n 1) a1 < < ai < 0 < ai+1 < < a2n+1 2n 1.

Se poate usor observa ca exista cel putin doua numere negative si cel putin doua
numere pozitive.
1

Profesor, Colegiul National "Gheorghe Rosca Codreanu", Brlad

20

Al doilea caz este acela n care |ai | 6= ai+1 ; s


a presupunem, de exemplu, c
a
|ai | < ai+1 . Avem
1 ai + ai+1 < < ai + a2n+1 2n 1

si

1 ai1 < < a1 2n 1,


adic
a din nou dou
a grupe continnd un total de 2n numere din multimea {1, . . . , 2n1},
n fiecare grup
a numerele fiind distincte; prin urmare exist
a un j {i + 1, . . . , 2n + 1}
si un k {1, . . . , i 1} astfel nct ai + aj = ak ai + aj + ak = 0, ceea ce era
de demonstrat (evident, i, j, k sunt diferite doua cte doua). Cazul |ai | > ai+1 este
absolut analog, suntem deci siguri c
a nu va reprezenta o problem
a pentru cititorul
interesat.
Cazul al treilea este si cel mai greu. Ne aflam acum n situatia n care |ai | = ai+1
si observ
am (asa cum cititorul trebuie s
a fi observat deja) c
a demonstratia de mai
a creasc
a num
arul "cutiilor" n
sus nu mai este valabil
a pentru c
a ai + ai+1 = 0 face s
care trebuie asezate obiectele (facnd principiul inaplicabil!). Din fericire, obstacolul
nu este insurmontabil. Observam mai nti ca acum avem, de fapt,
1 ai + ai+2 < < ai + a2n+1 2n 2

si, pentru a1 > (2n 1),

1 ai1 < < a1 2n 2;

astfel ca demonstratia din al doilea caz se poate utiliza si acum. Similar se rezolva
cazul a2n+1 < 2n 1, deci putem presupune n continuare ca a1 = (2n 1) si
a2n+1 = 2n 1. S
i iar consider
am dou
a posibilit
ati: i n sau i n + 1.
Dac
a i n avem numerele ai+1 , . . . , a2n si 2n 1 ai+1 , . . . , 2n 1 a2n , care
sunt toate din multimea {1, . . . , 2n 1} si sunt n numar de 2(2n i) 2n; rezulta
ca exista j, k {i + 1, . . . , 2n} asa nct aj = 2n 1 ak . Avem j 6= k deoarece
2n 1 este impar si egalitatea se mai scrie a1 + aj + ak = 0.
n cazul i n + 1 proced
am la fel cu numerele a2 , . . . , ai si 2n 1 + a2 , . . . ,
2n 1 + ai , ceea ce ncheie demonstratia.
Observatii. 1) Cititorul atent trebuie s
a fi observat deja c
a 0, 1, . . . , 2n 1
sunt 2n numere din intervalul [(2n 1), 2n 1] printre care nu exist
a trei cu
suma 0; sau, analog, se pot alege 0, 1, . . . , (2n 1) cu aceeasi proprietate, deci
2n + 1 este cel mai mic numar k astfel nct oricum am alege k numere din multimea
{(2n 1), . . . , 2n 1} exist
a printre ele trei cu suma 0.
2) O ntrebare se pune n mod natural n leg
atur
a cu aceast
a problem
a si cu
observatia anterioara: care sunt toate posibilitatile de a alege 2n numere din multimea
{(2n 1), . . . , 2n 1} astfel nct printre ele s
a nu existe trei a c
aror sum
a s
a fie
0? L
as
am n seama cititorului rezolvarea acestei probleme.
Bibliografie
1. P. Erds - Problema E736, The American Mathematical Monthly, 53(1946), 462.
2. L. Moser - Solutia problemei E736, The American Mathematical Monthly, 54(1947),
229-230.
21

Tehnici de stabilire a unor inegalit


ati geometrice
I. M. MAFTEI 1 , Mihai HAIVAS 2
Fie a, b, c lungimile laturilor unui triunghi T . n diverse moduri putem trece de
la T la un triunghi derivat T 0 , lungimile laturilor acestuia din urma fiind expresii
n a, b, c (rezultate ntr-un mod indicat). Dac
a aplic
am triunghiului T 0 inegalit
ati

3 3
R,
geometrice valabile n orice triunghi (de exemplu, R 2r, 3 3r p
2

3
a2 + b2 + c2 4 3S, 1 < cos A + cos B + cos C etc.), vom obtine, cu oarecare
2
sansa, noi inegalitati interesante. Ne propunem sa ilustram mai jos acest procedeu.
Cteva triunghiuri derivate ale lui T sunt date de urm
atoarea
Lem
a. Daca a, b, c sunt lungimile laturilor unui triunghi, atunci
1) pentru orice x 0, numerele l1 = ax + b, l2 = bx + c, l3 = cx + a sunt, de
asemenea, lungimile laturilor unui triunghi;
2) pentru orice [0, 1], numerele L1 = a , L2 = b , L3 = c sunt lungimile
laturilor unui triunghi;
c) pentru orice x 0 si [0, 1], (ax + b) , (bx + c) si (cx + a) sunt lungimile
laturilor unui trinughi.
Demonstratie. 1) S
a ar
at
am, de exemplu, c
a l1 + l2 > l3 . Avem (ax + b) +
(bx + c) > (cx + a) (a + b c) x + (b + c a) > 0, ceea ce are loc x 0.
2) Considernd c = max {a, b, c}, avem de aratat ca L1 + L2 > L3 , adica a +
a b
+
este descresc
atoare; deci
b > c . Functia f : [0, 1] R, f () =
c
a+b c
f () f (1), [0, 1]. Dar f (1) =
> 1. Deci f () > 1, [0, 1], adica
c

a + b > c , [0, 1].


3) Rezulta combinnd punctele precedente.
Propozitie. Fie a, b, c lungimile laturilor unui triunghi. Sa se arate x 0
avem urmatoarele inegalitati:
(a + b + c)3 (x + 1)3 27 [(ab) (1x) + c (1+x)] [(bc) (1x) + a (1+x)]
[(c a) (1 x) + b (1 + x)] ,
(1)

(a+bc) x + b+ca (b+ca) x + c+ab (c+ab) x + a+bc


p
+ p
+ p
3,
(ax + b) (bx + c)
(bx + c) (cx + a)
(cx + a) (ax + b)
(b + c) x + a + c (c + a) x + b + a (a + b) x + c + b

+
+
cx + a
ax + b
bx + c

2
ax + b + bx + c + cx + a ,

(2)

(3)

Demonstratie. 1) Fie T1 triunghiul cu lungimile laturilor l1 = ax+b, l2 = bx+c


si l3 = cx + a (x 0). Consideram subntelese notatiile: p1 , r1 , R1 , S1 relativ la T1 .
1
2

Profesor, Colegiul National "Sf. Sava", Bucuresti


Cercet
ator, Academia Romn
a, Inst. Cerc. Economice "Gh. Zane", Iasi

22


Inegalitatea (1) se obtine aplicnd triunghiului T1 inegalitatea p 3 3r. ntr1
1
adevar, p1 = (l1 + l2 + l3 ) = (a + b + c) (x + 1), iar, cu formula lui Heron pentru
2
2
arie,
S2
(a + b + c) (x + 1)
r12 = 21 =
2
2
p1
4 (a + b + c) (x + 1)
[(b + c a) x + c + a b] [(c + a b) x + a + b c] [(a + b c) x + b + c a] =

[(a b) (1 x) + c (1 + x)] [(b c) (1 x) + a (1 + x)] [(c a) (1 x) + b (1 + x)]


.
4 (a + b + c) (x + 1)
Introducnd n p21 27r12 , vom obtine inegalitatea (1).

2) Consideram triunghiul T2 avnd lungimile laturilor u = ax + b, v = bx + c,

1
w = cx + a, x 0 (Lema, punctul 3) cu = ). Atunci
2
v 2 + w2 u2
(b + c a) x + c + a b
p
,
cos A2 =
=
2vw
2 (bx + c) (cx + a)
(c + a b) x + a + b c
(a + b c) x + b + c a
p
p
cos B2 =
, cos C2 =
.
2 (cx + a) (ax + b)
2 (ax + b) (bx + c)
Cum cos A2 + cos B2 + cos C2 3, obtinem imediat (2).
3) Pentru medianele mu , mv , mw ale triunghiului T2 avem:

2 v 2 + w2 u2
(2b + 2c a) x + 2c + 2a b
m2u =
=
,
4
4
(2c + 2a b) x + 2a + 2b c
(2a + 2b c) x + 2b + 2c a
, m2w =
.
m2v =
4
4
2
P ma
3

(a + b + c) n
Tinnd
seama de acestea si utiliznd cunoscuta inegaliate
a
4
triunghiul T2 , obtinem (3).
Observatie. Avem egalitate n (1), (2), (3) doar pentru triunghiul echilateral.
Cazuri particulare. Daca luam x = 0 sau x = 1 n (1), (2), (3), obtinem
inegalit
atile (mereu n ipoteza c
a a, b, c sunt lungimile laturilor unui triunghi):

(a + b + c) 27 (a b + c) (b c + a) (c a + b) ,
3

(10 )

(11 )
(a + b + c) 27abc,

a (b + c a) + b (c + a b) + c (a + b c) 3 abc,
(20 )
p

3
(a + b) (b + c) (c + a),
(21 )
a b+c+b c+a+c a+b
2


b+c c+a a+b
+ + 2
a+ b+ c ,
(30 )
a
c
b

c
b
1
a

+
a+b+ b+c+ c+a .
(31 )
+

2
c+a
b+c
a+b
Observatie. Se pot obtine cu acest procedeu si alte inegalit
ati interesante.
Suge

ram cititorului sa aplice procedeul triunghiului cu laturile ax2 + bx + c ,


2
2

bx + cx + a , cx + ax + b , x 0 si [0, 1].
23

Asupra unei probleme de concurs


1

Angela TIG
AERU

n nota de fata ne propunem determinarea unei conditii suficiente care sa asigure


convergenta integralelor de forma
Z
f (x)
dx, a > 0.
x
a
Vom demonstra c
a uniforma m
arginire a expresiei xf (x) este suficient
a pentru convergenta acestui tip de integrale, extinznd astfel rezultatul cuprins n Problema 1,
cl. a XII-a, data la O.N.M. din anul 2000, propusa de Mihai Piticari si Sorin R
adulescu, conditia folosit
a acolo fiind doar existenta si finitudinea limitei lim xf (x)
x

(problema citat
a, cu rezolvarea autorilor, poate fi consultat
a n [1]).
Pentru nceput, vom prezenta cteva definitii si rezultate de baza. Fie f : [a, )
R o functie integrabila Riemann pe orice interval compact inclus n [a, ).
Definitie. Spunem ca functia f este integrabila pe intervalul [a, ) daca exista
Rx
si este finita limita lim f (t) dt. Daca f este integrabla pe [a, ), vom nota
x a

f (t) dt = lim

f (t) dt.

(1)

Daca conditia (1) este ndeplinita, se mai foloseste formularea: integrala

f (t) dt

este covergenta. O conditie necesar


a, nu si suficient
a, ndeplinit
a de o functie integrabila pe intervalul [a, ) este ca lim f (x) = 0.
x

Un rezultat teoretic fundamental, pe care l folosim n demonstratie este


Criteriul lui Cauchy. Functia f : [a, ) R este integrabila pe [a, ) daca si
numai daca, pentru orice > 0, exista numarul real A () > 0, astfel nct, pentru
orice x > A () si pentru orice y > 0, este valabila relatia
Z x+y

f (t) dt < .
(2)

Exist
a o literatur
a vast
a care are ca subiect determinarea unor criterii eficiente
de convergenta a integralelor de tipul (1) sau de calcul efectiv, cititorul interesat de
mai multe detalii putnd consulta [3].
Rezultatul notei de fata este
Propozitia 1. Se considera functia f : [a, ) R, a > 0, integrabila pe orice
compact inclus n [a, ). Daca exista M > 0 astfel nct
atunci

R f (x)
a

|xf (x)| M,

x [a, ) ,

dx este convergenta.

Profesor, Colegiul National "Petru Rares", Suceava

24

(3)

Demonstratie. n adevar, pentru un > 0, consideram A () =


x > A (), atunci, pentru orice y > 0, avem
Z x+y
Z x+y
Z

1
f (t)

|tf (t)| 2 dt M
dt

t
t

M
. Daca

x+y

M y
1
dt =
.
2
t
x x+y
x
x
x
M
M y
y
< 1 si deoarece x > A () =
, rezult
a c
a
< , deci
Cum
x
+
y

x x+y

x+y

R f (t)
R f (t)
dt < , adic
a, urmare a criteriului Cauchy, integrala
dt este con
x

t
t
a
vergenta.

O consecinta a rezultatului demonstrat mai sus este


Propozitia 2. Se considera functia f : [1, ) R, care satisface conditiile
Propozitiei 1. Atunci, pentru orice a > 1, este valabila relatia
Z a
Z
f (t)
lim x
f (tx ) dt =
dt.
(4)
x
t
1
1
Demonstratie. Cu schimbarea de variabil
a u = tx se obtine egalitatea
x
Z a
Z a
f (u) x1
x
f (tx ) dt =
u du.
u
1
1
Avem succesiv:
Z x
Z x

Z ax
Zax
1
a f (u) 1

a
x 1
1
u
1
f
(u)

2
x
x
|uf (u)|
du M
2 du =
u
u du
du

u
u
u2
u
1
1
1

x 1x
1 + ax 1 ;
=M
a
1x
x

x
aR f (u)
a
R f (u)
1

u x du
du = 0,
trecnd la limit
a si tinnd cont de a > 1, deducem lim
x 1

u
u
1
deci lim

a
Rx

x 1

a
Rx f (u)
R f (t)
f (u) 1
x
u du = lim
du =
dt, q.e.d.
x 1
u
u
t
1

Urmatorul exemplu poate forma o imagine asupra ariei de aplicabilitate a rezultatelor de mai sus.
Ra {xn }
dx = 1c, unde {x} reprezinta partea
Exemplul 1. Sa se arate ca lim n
n 1 xn
fractionar
a a lui x, a > 1 si c este constanta lui Euler.
{x}
Rezolvare. Consider
am functia f : [1, ) R, f (x) =
, care satisface
x
n
x
a
a
R {x }
R {t }
R {x}
dx
=
lim
x
dt
=
dx.
conditiile din propozitia 2, deci lim n
2
n 1 xn
x 1 tx
1 x
Pentru calculul ultimei integrale proced
am astfel:
n
k+1
k+1

Z
Z
n1
n1
X Z {x}
X Z xk
{x}
{x}
dx = lim
dx = lim
dx = lim
dx =
n
n
n
x2
x2
x2
x2
1

k=1 k

25

k=1 k

n1
X

n1
k+1
X 1
1
k k+1

= lim
+ ln x
= lim
k

+ (ln (k + 1) ln k) =

n
n
x k
k+1 k
k
k=1
k=1

n1
X
1
1
1

= lim
+ (ln (k + 1) ln k) = lim 1 1 + + ln n = 1c,
n
n
k+1
2
n
k=1
n
Ra {x }
dx = 1 c.
deci lim n
n 1 xn
Propozitia 1 st
a si la baza urm
atorului rezultat
Propozitia 3. Se considera functia f : [a, ) R, a > 0, continua pe [a, ),
R F (x)
care satisface conditia (3). Atunci
dx este convergenta, unde F : [a, ) R
x
a
este o primitiva a functiei f si, mai mult, este adevarata relatia
Z
Z
F (a)
F (x)
f (x)
dt =
+
dx.
(5)
x2
a
x
a

Demonstratie. Pentru x > a avem


Z x
Z x
Z x 0
Z x
F (x) F (a)
F (t) x
f (t)
F (t)
F (t)
F (t)
+
dt
=
dt. ()
dt =
dt =

2
t
t
t
t
x
a
t2
a
a
a
a
a
Tot din conditia (3), se obtine ca lim f (x) = 0, de unde, conform Lemei 1 din
x

F (x)
[2], se deduce c
a lim
= 0, demonstratia nefiind imediat
a, deoarece conditia
x
x
(3) nu asigura aplicarea directa a lemei lui lHospital. Trecnd la limita n relatia
(), se obtine (5) si demonstratia se ncheie.
|sin x|
Exemplul 2. Se considera f : [0, ) R, f (x) =
, prelungita prin
x
R F (x)
dx este
continuitate n x = 0. Dac
a F este o primitiv
a a lui f , atunci
x2
a
R F (x)
F (a) R |sin x|
dt =
dx.
convergenta pentru orice a > 0 si
+
2
x
a
x2
a
a
ncheiem, propunndu-i cititorului exercitiul urmator:
Sa se arate ca

Za
32 52 (2n 1)2
1
d (xn )
lim n
dx = lim ln
0,
,
n
n
xn
42n n! (n 1)!
4
1

unde d (x) = inf {x n | n N}.


Bibliografie
1. Gazeta Matematica, 7-8/2000, p.265 si p.275.
2. D. T. Onofrei - Asupra comportarii la limita a unor primitive, Recreatii matematice,
II(2000), nr. 2, 28-29.
3. A. Precupanu - Bazele analizei matematice, Editura Polirom, Iasi, 1998.
26

O clas
a de inegalit
ati
1
Mihai DICU, Lucian TUTESCU

Propozitia 1. Fie numerele k, m N .


1a) Daca x, y, z R, atunci are loc inegalitatea

x2m+2k + y 2m+2k + z 2m+2k xk y k z k (x2mk + y 2mk + z 2mk );

1b) Daca x, y, z R+ , atunci are loc inegalitatea

x2m+2k+1 + y 2m+2k+1 + z 2m+2k+1 xk y k z k (x2mk+1 + y 2mk+1 + z 2mk+1 );

1c) Daca x, y, z R+ , atunci are loc inegalitatea

x
+ y 2m+2k + z 2m+2k xk y k z k (x2mk1 yz + y 2mk1 xz + z 2mk1 xy).
2m+2k

Demonstratie. 1a) Avem

(x2m y 2m )(x2k y 2k ) 0,

(y 2m z 2m )(y 2k z 2k ) 0,

(z 2m x2m )(z 2k x2k ) 0

si apoi

x2m+2k + y 2m+2k x2k y 2m + x2m y 2k


y

2m+2k

2m+2k

+z

2m+2k
2m+2k

+x

Adunnd (1), (2), (3), punem sub forma

2k 2m

y z

2k 2m

x z

+y

(1a)

(1)

2m 2k

(2)

2m 2k

(3)

+x

2(x2m+2k+y 2m+2k+z 2m+2k ) (x2my 2k+x2mz 2k )+(x2ky 2m+y 2mz 2k )+(x2kz 2m+y 2kz 2m )

2 x2m y k z k + 2 xk y 2m z k + 2 xk y k z 2m 2xk y k z k (x2mk + y 2mk + z 2mk ),


dup
a aplicarea inegalit
atii mediilor pentru fiecare parantez
a n parte.
1b) La fel ca la punctul a) pentru x, y, z 0 se pleac
a de la inegalit
atile:
(x2m+1 y 2m+1 )(x2k y 2k ) 0, (y 2m+1 z 2m+1 )(y 2k z 2k ) 0,
(z 2m+1 x2m+1 )(z 2k x2k ) 0.

(1b)

Dup
a adunare, grup
am sub forma

2(x2m+2k+1 + y 2m+2k+1 + z 2m+2k+1 )

(x2m+1 y 2k + x2m+1 z 2k ) + (x2k y 2m+1 + y 2m+1 z 2k ) + (x2k z 2m+1 + y 2k z 2m+1 ).

Dup
a aplicarea inegalit
atii mediilor, pentru fiecare parantez
a, se g
aseste inegalitatea 1b).
1c) Pentru x, y, z 0 se folosesc inegalitatile:
(x2m1 y 2m1 )(x2k+1 y 2k+1 ) 0, (y 2m1 z 2m1 )(y 2k+1 z 2k+1 ) 0,
(1c)
(z 2m1 x2m1 )(z 2k+1 x2k+1 ) 0

si se parcurg aceleasi etape ca la punctul a) si b), grupnd corespunz


ator.
Observatia 1. Se vede usor c
a fiecare dintre inegalit
atile (1a), (1b), (1c) este
mai tare dect inegalitatea mediilor.
1

Profesori, C.N. "Fratii Buzesti" Craiova

27

Cazuri particulare
1a1 ) Daca luam k N si notam 2mk = n N, atunci cu k +n par si x, y, z R
avem
x3k+n + y 3k+n + z 3k+n xk y k z k (xn + y n + z n ).
1a2 ) Dac
a n aceasta particulariz
am k = 1, se obtine
xn+3 + y n+3 + z n+3 xyz(xn + y n + z n ),

cu n numar natural impar si x, y, z numere reale oarecare.


Concursul "Gheorghe Dumitrescu", 2006
a remarc
am cteva cazuri particulare ale ultimei inegalit
ati:
1a3 ) S
1a31 )
x4 + y 4 + z 4 xyz(x + y + z), x, y, z R;
1a32 )
x6 + y 6 + z 6 xyz(x3 + y 3 + z 3 ), x, y, z R.
Concursul revistei "Arhimede", 2005
1b1 ) Daca luam k N si notam 2m k + 1 = n N, atunci cu k + n impar si
x, y, z R+ avem
x3k+n + y 3k+n + z 3k+n xk y k z k (xn + y n + z n ).

a n aceasta particulariz
am k = 1, se obtine
1b2 ) Dac

xn+3 + y n+3 + z n+3 xyz(xn + y n + z n ),

cu n num
ar natural par si x, y, z numere reale pozitive oarecare.
Concursul "Gheorghe Dumitrescu", 2006
1b21 ) pentru n = 0 si x, y, z numere reale pozitive oarecare se obtine
x3 + y 3 + z 3 3xyz,

adic
a inegalitatea mediilor pentru trei numere.
1b22 ) pentru n = 2 si x, y, z numere reale pozitive, se obtine
x5 + y 5 + z 5 xyz(x2 + y 2 + z 2 ).

1c1 ) Daca luam k N si notam 2m k 1 = n N, atunci, cu k + n impar si


x, y, z R+ , avem

x3k+n+1 + y 3k+n+1 + z 3k+n+1 xk y k z k (xn yz + y n xz + z n xy).


Propozitia 2. Fie numerele k, m N .
2a) Daca x, y, z R, atunci are loc inegalitatea

x2m+2k + y 2m+2k + z 2m+2k xk y k z k (x2mk + y m z mk + z m y mk );

2b) Daca x, y, z R+ , atunci are loc inegalitatea


+y 2m+2k+1 +z 2m+2k+1 xk y k z k x2mk+1 + y m z mk yz + y mk z m yz ;
x
2m+2k+1

2c) Daca x, y, z R+ , atunci are loc inegalitatea


r

r
x
x
2m+2k
2m+2k
2m+2k
k k k
2mk1
m mk
mk m
x
+y
+z
x y z x
yz + y z
z
+y
.
z
y
Demonstratie. 2a) Adun
am (1), (2), (3) si grup
am termenii din membrul drept
sub forma

2(x2m+2k +y 2m+2k +z 2m+2k ) (x2my 2k +x2mz 2k )+(x2ky 2m +y 2kz 2m )+(x2kz 2m +y 2mz 2k ).


28

Aplicnd inegalitatea mediilor pentru fiecare parantez


a, obtinem:
k m+k m

2m k k
2m+2k
2m+2k
2m+2k

2(x
+y
+z
)2 x y z +2 x y
z + 2 xk y m z m+k

2 x2m y k z k + 2 xk y m+k z m + 2 xk y m z m+k


2xk y k z k (x2mk + y m z mk + z m y mk ).
2b) Plecnd de la (1b), dupa adunare, grupam astfel:
2(x2m+2k+1 + y 2m+2k+1 + z 2m+2k+1 )

(x2m+1 y 2k + x2m+1 z 2k ) + (x2k y 2m+1 + y 2k z 2m+1 ) + (x2k z 2m+1 + z 2k y 2m+1 )

2(x2m+1 y k z k + xk y m+k z m yz + xk y m z m+k yz) =

= 2xk y k z k (x2mk+1 + y m z mk yz + y mk z m yz).


2c) Folosind (1c), dupa adunare, termenii se grupeaza astfel:
2(x2m+2k + y 2m+2k + z 2m+2k )

(x2m1y 2k+1+x2m1z 2k+1 )+(x2k+1y 2m1+y 2k+1z 2m1 )+(x2k+1z 2m1+y 2m1z 2k+1 )
r

r
x
x
2m1 k k
k m+k m
2 x
y z yz + x y
z
+ xk y m z m+k
=
z
y
r

r
x
x

+ y mk z m
.
= 2xk y k z k x2mk1 yz + y m z mk
z
y
Observatia 2. Se vede usor c
a fiecare dintre inegalit
atile 2a), 2b), 2c) este mai
tare dect inegalitatea mediilor.
Observatia 3. n cazul n care x, y, z R+ inegalitatea 1a) este mai tare dect
2a) pentru m > k, respectiv, mai slab
a n caz contrar, pentru c
a
2mk
2mk
2mk
2mk
m mk
m mk
m
m
x
+y
+z
x
+y z
+z y
(y z )(y mk z mk ) 0.
Cazuri particulare
2a1 ) Pentru m = 2k si x, y, z R, avem
x6k + y 6k + z 6k xk y k z k (x3k + y 2k z k + z k y 2k ).
2a2 ) Pentru k = 2m si x, y, z R, avem

y m z m
6m
6m
6m
2m 2m 2m
x +y +z x y z
+
1+
.
z
y
2b1 ) Pentru m = 2k si x, y, z R+ , avem

x6k+1 + y 6k+1 + z 6k+1 xk y k z k (x3k+1 + y 2k z k yz + y k z k yz).


2b2 ) Pentru k = 2m si x, y, z R+ , avem

x6m+1 + y 6m+1 + z 6m+1 x2m y 2m z 2m (x + y m z m yz + y m z m yz).


2c1 ) Pentru m = 2k si x, y, z R+ , avem
r

r
x
x
6k
6k
6k
k k k
3k1
2k k
k 2k
x +y +z x y z x
yz + y z
+y z
.
z
y
2c2 ) Pentru k = 2m si x, y, z R+ , avem
r

r
x
x

x6m + y 6m + z 6m x2m y 2m z 2m x1 yz + y m z m
+ y m z m
.
z
y
29

Inegalit
ati omogene si putin
a analiz
a...
Titu ZVONARU 1
Vom stabili printr-o metoda unitara cteva inegalitati n trei variabile a, b, c care
ndeplinesc conditia a + b + c = 1. Sa remarcam ca, pentru inegalitati omogene,
aceast
a conditie poate fi oricnd ad
augat
a, f
ar
a pierderea generalit
atii.
b
c
3
a
+
+
, a, b, c R+ .
Exemplul 1 (Inegalitea lui Nesbitt).
b+c c+a a+b
2
Solutie. Inegalitatea fiind omogena, putem presupune a + b + c = 1. Avem
x
9x 1

, x (0, 1) (inegalitate echivalenta cu (3x 1)2 0). Ca urmare


1x
4
a
b
c
a
b
c
9a 1 9b 1 9c 1
3
+
+
=
+
+

+
+
= .
b+c c+a a+b
1a 1b 1c
4
4
4
2
(egalit
atile care apar sunt justificate de faptul c
a a + b + c = 1) si inegalitatea este
demonstrat
a.
Aceasta solutie este de tip "iepurasul scos din joben" si cititorul se ntreaba de
unde a ap
arut inegalitatea ajut
atoare. R
aspundem acestei ntreb
ari, ar
atnd totodat
a si ideea c
al
auzitoare a notei noastre.
x
Consider
am functia f : (0, 1) R dat
a prin f (x) =
. Tangenta la graficul
1x
1
ei n punctul de abscisa are ecuatia
3
1
1
1
9x 1
yf
= f0
x
y=
.
3
3
3
4
Deoarece functia este convexa, pe tot domeniul sau de definitie, graficul ei este
situat "deasupra" tangentei, fapt care se traduce prin inegalitatea utilizat
a mai
1
este, desigur, motivat
a de cazul de egalitate
sus. Alegerea punctului de abscis
a
3
1
a=b=c= .
3
3
Exemplul 2 (Polonia, 1996 ). Pentru orice a, b, c cu a + b + c = 1 are loc
4
a
b
c
9
+
+

.
a2 + 1 b2 + 1 c2 + 1
10
x
; tangenta la graficul functiei n punctul
Solutie. Fie f : R R, f (x) = 2
x +1
36x + 3
1
are ecuatia y =
, iar acest punct este situat ntr-un interval
de abscisa
3
50
x
36x + 3
de concavitate a functiei f . Inegalitatea 2

(de altfel echivalenta


x +1
50
3
cu (3x 1)2 (4x + 3) 0), este valabil
a pentru orice x si se poate aplica
4
numerelor a, b, c. Inegalitatea cerut
a rezult
a prin sumarea inegalit
atilor de acest tip
scrise pentru a, b, c si din faptul c
a a + b + c = 1:
a
b
c
36a + 3 36b + 3 36c + 3
9
+
+

+
+
=
.
a2 + 1 b2 + 1 c2 + 1
50
50
50
10
1

Com
anesti, e-mail: tzvonaru@hotmail.com

30

(Pn
a la urm
a, metoda aceasta este o metod
a de spargere a inegalit
atilor.)
Exemplul 3 (USAMO, 2003 ). Pentru orice a, b, c pozitive are loc
(2b + c + a)2
(2c + a + b)2
(2a + b + c)2
+ 2
+ 2
8.
2
2
2
2a + (b + c)
2b + (c + a)
2c + (a + b)2
Indicatie. Putem presupune f
ar
a a restrnge generalitatea c
a a + b + c = 1.
4x + 1
12x + 3
Are loc inegalitatea

, x > 0, (care este echivalent


a cu
3x2 2x + 1
2
(4x + 1)(3x 1)2 0) si apoi o folosim ca n exemplele anterioare.
Exemplul 4 (OIM, 1995 si C:1952 din GM 7-8/1997 ). Pentru a, b, c > 0 avem
a2
b2
c2
a+b+c
+
+

.
b+c c+a a+b
2
5x 1
x2

,
Indicatie. Inegalitatea care trebuie folosita n acest exemplu este
1x
4
x (0, 1).
Exemplul 5 (E:10888 din GM 2/1995 ). Fie a, b, c lungimile laturilor unui
triunghi. Atunci
b
c
a
+
+
3.
b+ca c+ab a+bc
1
x
Indicatie. Are loc inegalitatea
9x 2, pentru orice x 0, .
1 2x
2
Exemplul 6 (Japonia, 1997 ). Pentru orice a, b, c > 0 are loc inegalitatea
(c + a b)2
(a + b c)2
3
(b + c a)2
+ 2
+ 2
.
2
2
2
a + (b + c)
b + (c + a)
c + (a + b)2
5
54x + 27
1

, x > 0.
Indicatie. Se va utiliza
2
2x 2x + 1
25
a
b
c
3
Exemplul 7.
+
+

, a, b, c R+ , m 1
ma+b+c a+mb+c a+b+mc m+2
(o generalizare a problemei C:1079, GM 1/1991 ).
9x + m 1
x
! ncercati si cu sub
Indicatie. Folositi inegalitatea
1 + (m 1)x
(m + 2)2
stitutiile ma + b + c = u, a + mb + c = v, a + b + mc = w !
Exemplul 8 (USAMO, SummerProgram, 2002 ). Pentru orice a, b, c > 0 are loc
2
23
23

2b
2c
2a 3
+
+
3.
b+c
c+a
a+b
2x 23
3x (3x 1)2 (3x 4) 0 pentru orice x (0, 1).
Indicatie. Avem
1x
Propunem cititorului urm
atoarele exercitii:
Exercitiul 1 (Gigel Buth si Liviu Vlaicu, RMT 2/1998 ). Pentru a, b, c > 0 si
n m > 0 are loc
a
b
c
3
+
+

.
ma + nb + nc mb + nc + na mc + na + nb
m + 2n
31

Exercitiul 2 (Ungaria, 1996 ). Pentru a si b mai mari ca 1 cu a + b = 1, avem


b2
1
a2
+
.
a+1 b+1
3

2n
1
si a1 , . . . , an 2
astfel nct
n2
n 1
a1 + + an = 1. Sa se demonstreze inegalitatea
a1
an
n2
+

.
a21 +
a2n +
n2 + 1
Exercitiul 3. Fie n N, n 2, >

Exercitiul 4 (India,1995 ). Pentru orice a1 , . . . , an pozitive si cu suma 1, are loc


inegalitatea
a1
an
n
+ +

.
1 a1
1 an
n1
Exercitiul 5. Pentru a, b, c > 0 cu a + b + c = 1 avem
b
c
27
a
+ 3
+ 3

.
3
a +1 b +1 c +1
28
Exercitiul 6 (Sefket Arslanagic, 2787, Crux Mathematicorum). Fie a, b, c > 0
astfel nct a + b + c = 1. Atunci
1
1
1
27
.
c+a 2
a+b 2 +
b+c 2 +
8
1 2
1 2
1 2

Exercitiul 7 (Sefket Arslanagic, 2739, Crux Mathematicorum). Pentru orice


a, b, c > 0 are loc inegalitatea

9+3 3
a+b+c+ a
a+b+c+ b
a+b+c+ c
+
+

.
b+c
c+a
a+b
2 a+b+c
Exercitiul 8. Pentru a, b, c > 0 si avnd suma 1, este adevarata inegalitatea
2
2
2

1
1
100
1
+ b+
+ c+

.
a+
a
b
c
9
Exercitiul 9 (Sefket Arslanagic, 2738, Crux Mathematicorum). Fie a, b, c > 0
astfel nct a2 + b2 + c2 = 1. Atunci avem

b
c
3 3
a
+
+

.
1 a2 1 b2 1 c2
2
Exercitiul 10 (Panos E. Tsaoussoglou, 2946, Crux Mathematicorum).
a, b, c > 0 si a2 + b2 + c2 = 1. Avem

1 1 1
1 1 1
+ + a b c 2 3 si
+ + + a + b + c 4 3.
a b
c
a b
c
Exercitiul 11. Pentru a, b, c 1 cu a + b + c = 1 are loc
1
1
1
27
+
+

.
1 + a2 1 + b2
1 + c2
10
32

Fie

Despre numerele reale algebrice


Silviu BOGA1
n cele ce urmeaz
a, cadrul de studiu este inelul polinoamelor cu coeficienti rationali
n care s-au definit conceptul de divizibilitate, cel mai mare divizor comun a dou
a
polinoame, num
ar real algebric si polinom minimal asociat unui num
ar real algebric.
Rezultata imediat din algoritmul lui Euclid si deosebit de utila n rationamentele ce
vor urma este proprietatea: f, g Q[X] u, v Q[X] nct u f + v g = (f ; g).
Propozitia 1. Daca doua polinoame f, g Q[X] au o radacina comuna C,
atunci (f ; g) 6= 1.
Demonstratie. Presupunnd prin absurd c
a (f ; g) = 1, conform propriet
atii
anterior enuntat
a, u, v Q[X] nct u f + v g = 1. n acest caz, cum f () =
g() = 0, vom obtine ca (u f + v g) () = 1, deci 0 = 1, contradictie.
Propozitia 2. Daca doua polinoame f, g Q[X] au o radacina comuna C
si h = (f ; g), atunci h() = 0.
Demonstratie. Fie f = f h, g = g h si atunci, cum (f; g) = 1, u, v Q[X]
nct u f+v g = 1 si astfel (u f+v g)h = h uf +v g = h (uf +v g) () =
h() h() = 0.
Propozitia 3. Fie f Q[X], f = X p + a1 X p1 + a2 X p2 + + ap1 X + ap ,
pentru care
(i) f este de grad impar;
(ii) f este ireductibil n Q[X];
(iii) f are o radacina R de semn contrar cu termenul liber ap 6= 0;
(iv) a21 + a22 + + a2p1 6= 0.
n conditiile (i) (iv), pentru orice n N , n este numar real irational.
Demonstratie. Vom presupune, f
ar
a a restrnge generalitatea, c
a termenul liber
este pozitiv. Din cele mentionate, evident ca R \ Q. n acest caz, daca pentru
un anume n N ar avea loc n = a Q, polinomul g = X n a Q[X] avnd
o r
ad
acin
a comun
a cu polinomul f nu va fi prim cu f si, cum f este ireductibil, ar
..
rezulta g . f . Dar r
ad
acinile x1 , x2 , . . . , xn C ale polinomului g au |x1 | = |x2 | =
p
.
n
= |xn | =
|a| si astfel, din g .. f , arprezulta ca si radacinile polinomului f sunt

toate de acelasi modul, implicit || = p |ap | si, din conditia (iii), = p ap . n

aceast
a situatie, din exprimarea f = (X p + ap )+X a1 X p2 + a2 X p3 + + ap1
se va deduce ca polinoamele v = X p + ap si w = a1 X p2 + a2 X p3 + + ap1 au
radacina comuna . Dar w = a1 X p2 + a2 X p3 + + ap1 este neconstant conform
cu (i) si (iv) si, datorit
a r
ad
acinii comune, v si w nu ar fi prime ntre ele, ceea ce ar
face ca f s
a nu fie ireductibil n Q[X], n contradictie cu (ii). R
amne c
a n R \ Q.
a se
Observatie. Chestiuni de genul: Dac
a x R verific
a x3 + 2x + 2 = 0, s
2006
arate c
ax
R \ Q, imediat justificate de Propozitia 3, apar n mai multe rnduri
printre subiectele de bacalaureat, sesiunile 2006 si 2007 (a se vedea [1]).
Urmnd pas cu pas demonstratia Propozitiei 3 (adapt
arile necesare sunt evidente!)
vom obtine imediat
1

Profesor, Liceul "V. Alecsandri", Iasi

33

Propozitia 4. Fie f Q[X], f = X p + a1 X p1 + a2 X p2 + + ap1 X + ap ,


pentru care
(i) f este de grad par;
(ii) f este ireductibil n Q[X];
(iii) f are o radacina R si ap < 0;
(iv) a21 + a22 + + a2p1 6= 0.
n conditiile (i) (iv), pentru orice n N , n este numar real irational.
Observatie. Analog cu prima observatie, prin Propozitia 4 se demonstreaz
a
imediat afirmatii de genul: Dac
a x R verific
a x4 + x3 + x2 + x = 2007, s
a se arate
ca xn R \ Q, n N .
Propozitia 5. Daca R este un numar real algebric si exista n N nct
.
n Q atunci n .. p, unde p = grad f , f fiind polinomul minimal asociat lui
(adic
a polinomul din Q[X] de grad minim si care admite ca r
ad
acin
a ).
Demonstratie. Fie n = a Q; atunci g = X n a are r
ad
acin
a comun
a cu f ,
.
deci (g; f ) = h 6= 1. Dar f fiind polinom minimal, este ireductibil si astfel g .. f ,
.
de unde n .. grad f .

Propozitia 6. Daca R este un numar real algebric si exista n N nct


n
Q, atunci polinomul minimal asociat lui este de forma f = X p p , unde
p = min {n | n Q}.
Demonstratie. n cazul Q se observ
a c
a f = X . n cazul R \ Q,
evident {n | n Q} 6= si fie f = X p + a1 X p1 + a2 X p2 + + ap1 X + ap
.
polinomul minimal al lui . Cum n Q, conform propozitiei anterioare n .. p si

totodat
a , 2 , . . . , p1 R\Q, deoarece n caz contrar f nu ar mai fi minimal.
p Dar
n acest caz n p si analog cu rationamentul din propozitiile anterioare || = p |ap |,
deci ||p = |ap | Q. Astfel p Q si considernd g = a1 X p1 + a2 X p2 + +
ap1 X + (ap + p ), deducem ca g() = f () = 0 si cum f este minimal, atunci
g = 0, prin urmare f = X p p .
Observatie. Conform ultimei propozitii, orice r
ad
acin
a real
a irational
a a unui
polinom f Q[X] ireductibil si care nu este de forma f = X p a, are proprietatea
n R \ Q, n N .
Concluzie. Oricare ar fi R\Q radacina a unui polinom ireductibil f Q[X],
f = X p + a1 X p1 + a2 X p2 + + ap1 X + ap , are loc exact una din situatiile:
a) a21 + a22 + + a2p1 6= 0 si n acest caz n R \ Q, n N ;
b) a21 + a22 + + a2p1 = 0 si n acest caz n R \ Q n N este nedivizibil
prin p.
Bibliografie
1. http://www.subiecte2007.edu.ro bacalaureat, subiecte M11, variantele 47 si 84.
2. C. N
ast
asescu, C. Nita
a,
- Teoria calitativa a ecuatiilor algebrice, Ed. Tehnic
Bucuresti, 1982.

34

Asupra unei note din revista "Recreatii matematice"


1

Maria BATINE
TU-GIURGIU
,

D. M. BATINETU-GIURGIU

n "Recreatii Matematice", an. V, nr. 2/2003 este publicat


a o Not
a matematic
a
interesant
a [5], pentru care eleva Oana Crj
a a primit premiul Fundatiei "Poiana"
pe anul 2003. Ne propunem sa ntarim si sa extindem rezultatele acestei Note.
Fie (xn )n1 un sir de numere reale strict pozitive astfel nct lim (xn+1 xn ) =
n
= x R+ . Atunci, exist
a si limitele:

xn+1
xn+1 n n + 1
= lim

= 1.
n
xn
n + 1 xn
n
xn
Daca, nsa, (xn )n1 are proprietatea ca exista lim
= t R+ , nu rezulta ca
n n
n
exist
a lim (xn+1 xn ), dup
a cum arat
a sirul xn = n + (1) , n N .
lim

xn
xn+1 xn
= lim
= x si
n
x
(n + 1) n

lim

Teorema 1. Fie sirul de numere reale strict pozitive (xn )n1 astfel nct

n
xn
xn+1
lim
si
= x R+ . Atunci, lim (xn1 xn ) lim
n x
n
n
x

n n
xn+1
lim (xn+1 xn ) = x ln lim
.
(1)
n
n
xn
xn+1
Demonstratia I (D. M. B
atinetu-Giurgiu). Notnd un =
, n N ,
xn
avem
xn un 1
ln (un )n , n N .
(2)
xn+1 xn = xn (un 1) =

n
ln un
un 1
= 1. Dac
a y = lim (xn+1 xn ), trecnd la
Dar lim un = 1 si, deci, lim
n
n ln un
n

n
n
limit
a n (2), obtinem: y = xln lim (un ) , de unde lim (un ) = ey/x . Reciproc,
n
n

n
xn+1
dac
a z = lim
, utiliznd din nou (2), obtinem: lim (xn+1 xn ) =
n
n
xn
x ln z.

Demonstratia a II-a (M. Tena).


Se procedeaza la fel, n locul relatiei (2)
folosind formula
n "
xn # xn (xn+1 xn )

xn+1
xn+1 xn xn+1 xn n
=
1+
, n N .
(3)
xn
xn

Fie sirurile de numere reale strict pozitive (an )n1 , (bn )n1 , (xn )n1 si (yn )n1
an+1
bn+1
an
cu propriet
atile: lim
= lim
= a R+ , lim
= b R+ si exist
a
n an
n bn
n bn
xn
yn+1
lim
= x R+ , lim
= y R+ . Vom numi sir Lalescu definit de (xn )n1
n n
n n
1
2

Prof., dr., Academia Tehnic


a Militar
a, Bucuresti
Prof., Colegiul National "Matei Basarab", Bucuresti

35

si ponderat cu (an )n1 , (bn )n1 , (yn )n1 sirul (Ln )n2 dat de
p

Ln = xn+1 n+1 an+1 yn+1 xn n bn yn , n 2.

(4)

Teorema 2. Sirul
Lalescu (Ln )n2 definit prin (4) este convergent daca si numai

n
xn+1
daca sirul
este convergent.
xn
n1

xn+1 n+1 an+1 yn+1

Demonstratie. Notnd vn =

, n 2, vom avea:
n
xn
bn yn
p
xn p
vn 1
n
bn yn
ln (vn )n , n 2.
(5)
Ln = xn n bn yn (vn 1) =
n
ln vn

yn+1

= 1,
n ipotezele noastre, rezult
a c
a lim n an = lim n bn = a. Cum lim
n
n
n yn

n
yn = 1. Ca urmare, lim vn = 1. Din faptul ca
avem si lim
n
n

n
xn+1
an+1 an yn+1
1
, n 2,
(vn )n =

xn
an bn yn n+1 an+1 n+1 yn+1

n
xn+1
n
.
rezulta ca lim (vn ) = b lim
n
n
xn

n
xn+1
Daca z = lim
, trecnd la limita n (5), obtinem lim Ln = ax ln (bz).
n
n
xn

n
Reciproc, dac
a c = lim Ln , din (5) rezult
a c
a avem c = ax ln lim (vn ) , adic
a
n
n

n
1 c
xn+1
c
lim (vn )n = e ax si, n final, lim
= e ax .
n
n
xn
b

1
n!
n
Aplicatii ale Teoremei 1. 1) Luam xn = n!, n 2 .Cum lim
=
n
n
e

n
1
(n + 1)!
n+1
xn+1
p
p
si lim
= lim n+1
= e, rezult
a,
= lim
n+1
n
n
n
xn
n!
(n + 1)!
(n + 1)!
p

1
conform Teoremei 1, ca lim n+1 (n + 1)! n n! =
(limita sirului lui Traian
n
e
Lalescu).

2) Lund xn = n n n, n 2, avem

n
xn
1
(n + 1)n+1
xn+1

= lim
n+1
= e.
lim
= 1 si
lim
n n
n
n
xn
nn+1
n+1


Deci lim (n + 1) n+1 n + 1 n n n = 1 (limita sirului lui Romeo T. Ianculescu).
n
p
xn
3) Daca xn = n (2n 1)!!, n 2, atunci, dupa calcule de rutina, avem lim
=
n
n

p
p
2
xn+1
2
si lim
= e. Ca urmare, lim n+1 (2n + 1)!! n (2n 1)!! =
n
n
e
xn
e
(D.M. Batinetu-Giurgiu,
C:905, G.M.-5/1989).
p
4) Fie xn = n d1 d3 d2n1 , n 2, unde dn R+ si lim (dn+1 dn ) = d
n

R+ . Atunci
36

d1 d3 d2n1
d1 d2n+1
nn
=
lim
=
n (n + 1)n+1 d1 d2n1
nn

n
2d
n
d2n+1 2n + 1
= lim
=
,
n 2n + 1 n + 1
n+1
e

n
d1 d3 d2n+1
1
xn+1
p
= lim
=
lim
n
n d1 d3 d2n1 n+1 d d d
xn
1 3
2n+1
e
n+1
d2n+1
p
= lim
= 2d
= e.
n n + 1 n+1 d d d
2d
1 3
2n+1
xn
lim
= lim
n n
n

Ca urmare, lim (xn+1 xn ) =


n

- 2-3/1992).

2d
2d
ln e =
(D.M. Batinetu-Giurgiu, C:1024, G.M.
e
e

Aplicatii ale Teoremei 2. 1) Date cn R+ , n 1, sa notam c1 ! = c1 ,

am yn = 1, n 1 si xn = n cn !,
cn+1 ! = cn+1 cn !, n 1. n Teorema 2 consider
c
xn
si
n 2, unde lim (cn+1 cn ) = c R+ . Ca mai sus, avem lim
=
n n
e

n n
p

xn+1
ac
ln (be).
= e. n consecinta, lim n+1 cn+1 !an+1 n cn !bn =
lim
n
n
xn
e
(Teorema 3 din [6]).
2) Fie cn R+ , n 1, astfel nct lim (cn+1 cn ) = c R+ . Considen

a lim xn+1 n+1 an+1 xn+1 xn n bn xn =


rnd xn = yn = cn , n 1, obtinem c
n

= ac ln (be) (Teorema 4 din [6]).

nn
3) Pentru n N , sa consideram xn = n, yn = 1, an = bn =
. Avem
n! n
xn+1
an+1
bn+1
an
xn
lim
= lim
= e, lim
= 1, lim
= e
= 1, lim
n an
n bn
n bn
n n
n
xn
!

n2
(n + 1)2
p

si, deci, obtinem lim

= e (D.M. Batinetu-Giurgiu, C:890,


n
n+1
n
(n + 1)!
n!
G.M.-4/1989).
Bibliografie
D. M. B
atinetu - Siruri,
Editura Albatros, 1979.

D. M. B
atinetu-Giurgiu - Siruri
Lalescu, R.M.T., 1-2/1989, 33-36.

D. M. B
atinetu-Giurgiu - Ponderarea unor siruri, G.M., 2-3/1992, 46-49.
D. M. B
atinetu-Giurgiu - Asupra calcularii unor limite de siruri, RecMat-1/2007,
22-24.
5. O. Crj
a - Un procedeu de calcul al limitelor unor sirui de forma (an+1 an )n1 ,
RecMat-2/2003, 23-24.
6. A. Stroe - Asupra unei clase de siruri, G.M., seria A, 3/2007, 217-227.

7. M. Tena
- O alta solutie a Problemei 579 din G.M., Revista "Lic
ariri" a Liceului
"N. B
alcescu" din Craiova, 1978, 13-14.
1.
2.
3.
4.

37

Limita unor siruri de numere reale


Gheorghe COSTOVICI 1
Prezent
am mai jos o generalizare direct
a a Propozitiei din [1] si cteva aplicatii.
Propozitie. Fie f : (0, ) R continua, descrescatoare si cu lim f (x) = 0, F o
n

primitiva a lui f pe (0, ) si numerele p, q N , p < q. Daca lim [F (qn) F (pn)] =


n

= l R, atunci avem si
lim [f (pn + 1) + f (pn + 2) + + f (qn)] = l.

(1)

Demonstratie. Sa consideram sirul (xn )n1 dat de xn = f (1) + f (2) + +


+f (n) F (n). Procednd ca n [1], aratam ca (xn )n1 este descrescator si marginit.
ntr-adevar, cu teorema de medie a lui Lagrange, avem
xn+1 xn = f (n + 1)[F (n + 1) F (n)] = f (n + 1)F 0 (cn ) = f (n + 1)f (cn ) 0
(unde n < cn < n + 1). Deci, xn+1 xn , n 1.
Pe de alta parte, cu aceeasi teorema de medie, avem F (k + 1) F (k) = f (ck )
f (k) (k < ck < k + 1) si, sumnd pentru k = 1, n, obtinem
F (n + 1)F (1) f (1)+f (2)+ +f (n) , n 2 xn+1 f (n + 1)F (1) , n 2.
De aici si din faptul c
a sirul (f (n))n1 este m
arginit (consecinta a conditiei lim f (x) =
x

= 0), rezulta ca (xn )n1 este marginit inferior. Asadar, sirul (xn )n1 este convergent.
Se constat
a usor c
a
f (pn + 1) + f (pn + 2) + + f (qn) = xqn xpn + [F (qn) F (pn)] , n 1,
care, pentru n , conduce la (1).
Observatie. Propozitia din [1] se obtine pentru p = 1 si q = 2.

Calculul limitelor urmatoare devine simplu prin aplicarea acestei Propozitii (lasam
n seama cititorului verificarea conditiilor de aplicare).

1
1
1
q
1
Exemple. 1) lim
+
+ +
= ln
(f (x) = ).
n pn + 1
pn + 2
qn
p
x
(qp)n
P
1
1
= 0 (f (x) =
).
2) lim
2
n k=1 1 + (pn + k)
1 + x2
(qp)n
P
1

= .
3) lim
n k=1
pn + k + 1
(qp)n
P
q
4)
arctg (pn + k) = ln
( f (x) = arctg x).
p
k=1
(qp)n
P
1
1
q
= 0 (f (x) =
).
5)
2
x x2 + 1
k=1 (pn + k)
(pn + k) + 1
Bibliografie

1. C. Chiser - O metoda eleganta de calcul al unor limite de siruri, R.M.T.-3/2007,


9-10.
1

Conf. dr., Catedra de matematic


a, Univ. Tehnic
a "Gh. Asachi"

38

O rafinare a inegalit
atii dintre media aritmetic
a
si cea logaritmic
a
Mihail BENCZE 1
Fie 0 < a < b. Vom nota cu
a+b
ba
si
A (a, b) =
L (a, b) =
2
ln b ln a
media aritmetica si, respectiv, media logaritmica. Se stie [1] ca
L (a, b) < A (a, b) .

(1)

(2)

n aceast
a not
a vom da o rafinare a acestei inegalit
ati.
Teorem
a. Daca 0 < a < b atunci avem inegalitatile
a + b h i2
a, b
< A (a, b) .
(3)
L (a, b) < L
, ab < A
2
Demonstratie. Inegalitatea din dreapta rezult
a imediat:

h i2

2
a+b
a + b 2
a, b

a b > 0.
A (a, b) > A
>
2
2
h i2
a + b
Pe de alt
a parte, A
a, b
= A
, ab si aplicnd (2) obtinem
2
a + b
a + b
A
a inegalitatea din mijloc.
, ab > L
, ab , adic
2
2
b
Mai r
amne de demonstrat inegalitatea din stnga, care, cu notatia t = > 1,
a
se scrie

t+1

t+1

t1
t+1
t1
2 t

t+1
>
t
ln
t
>
t
+
1
ln

.
>
ln t
ln t
2
2 ln t+1
ln t
ln 2 ln t
2
Pentru a demonstra aceasta noua inegalitate folosim notatiile t = x2 si f (x) =
x2 + 1
2x ln x (x + 1) ln
. Atunci
2
x2 + 1 2 (x 1)
2 (x 1)2 (x + 1)
si f 00 (x) =
> 0,
f 0 (x) = 2 ln x
2
2
x +1
x (x2 + 1)2

deci f 0 (x) > f 0 (1) = 0 si f (x) > f (1) = 0 si astfel am demonstrat inegalitatea
a + b

t+1

t ln t >
t + 1 ln
, adic
a L (a, b) < L
, ab .
2
2
Bibliografie

1. F. Burk - The Geometric, Logaritmic and Aritmetic Mean Inequality, Amer. Math.
Monthly 94(1987), 527-528.
2. M. Bencze - New Means, new Inequalities and Refinement, Octogon Mathematical
Magazine, Vol. 9, Nr. 1/2001, 46-105.
1

Profesor, Brasov

39

Estim
ari de sume
1

Tudor PADURARIU

n ultimii ani, la diverse concursuri, nationale sau internationale, au fost propuse


probleme n care se cere s
a se g
aseasc
a sau s
a se demonstreze o anumit
a aproximare
pentru o suma (numerica, vectoriala etc). Metodele de rezolvare a unor asemenea
probleme sunt variate si presupun rationamente care folosesc reducerea la absurd,
principiul cutiei, principiul extremal, inductia matematic
a s. a.
Prezent
am n continuare cteva exemple; cititorul interesat poate exersa tehnicile dobndite rezolvnd problemele propuse n finalul notei, precum si selectia de
probleme din articolul lui Gabriel Carroll - Estimating Sums, care poate fi gasit
pe internet la adresa [1] si care a constituit punctul de plecare al demersului nostru.
Problema 1. Fie x1 , x2 ,. . . , xn numere reale cu proprietatea ca x21 + x22 + +
x2n = 1. Demonstrati ca pentru orice k N, k 2, exista ntregii e1 , e2 ,. . . , en
, nu
(k 1) n
.
toti nuli, cu |ei | < k 1, i = 1, k, pentru care |e1 x1 + + en xn |
kn 1
(I.M.O., 1987)
Solu
Cauchy-Schwarz, obtinem ca |x1 | + |x2 | + +
ptie. Folosind inegalitatea

|xn | n(x21 + + x2n ) = n. n total exist


a k n sume de tipul e1 x1 + + en xn ,
cu 0 xi k 1, iar toate aceste sume se afl
a n interiorul unui interval de lungime
(k 1) n. Aceste interval poate fi acoperit cu k n 1 subintervale de lungime
(k 1) n
(concluzia problemei sugereaza acest lucru!). Din principiul cutiei, exista
kn 1
dou
a sume care se g
asesc n acelasi subinterval, iar diferenta lor satisface cerintele.
n+1
, i = 1, n,
Problema 2. Fie x1 , x2 ,. . . , xn numere reale cu |xi |
2
iar |x1 + x2 + + xn | = 1. Demonstrati ca exista o permutare y1 , y2 ,. . . , yn a
n+1
numerelor x1 , x2 ,. . . , xn astfel nct |y1 + 2y2 + + nyn |
. (I.M.O., 1997)
2
Solutie. Presupunem prin absurd c
a pentru orice permutare y1 , y2 ,. . . , yn a
n+1
numerelor x1 , x2 ,. . . , xn am avea |y1 + 2y2 + + nyn | >
. Observ
am c
a
2
|(x1 + + nxn ) + (xn + 2xn1 + + nx1 )| = (n + 1) |x1 + x2 + + xn | = n + 1
si, daca numerele S1 = x1 + 2x2 + + nxn si S2 = xn + 2xn1 + + nx1 ar avea
acelasi semn, modulul sumei ar fi egal cu suma modulelor, contradictie. Ramne ca
S1 si S2 au semne contrare; s
a zicem c
a S1 > 0, S2 < 0.
Putem realiza trecerea de la S1 la S2 schimbnd n mod repetat ntre ele cte
doua numere consecutive xk si xk+1 . La un anumit pas, de exemplu cnd trecem
de la suma S3 = z1 + + jzj + (j + 1) zj+1 + + nzn la suma S4 = z1 + +
jzj+1 + (j + 1) zj + + nzn , semnul sumei S se va schimba: S3 > 0, iar S4 < 0.
n+1
n+1
n+1
n+1
Cum |S3 | >
, |S4 | >
, vom avea ca S3 >
, iar S4 <
, deci
2
2
2
2
S3 S4 > n + 1. Pe de alta parte, S3 S4 = j (zj zj+1 ) + (j + 1) (zj+1 zj ) =
1

Elev, Colegiul National "Gr. Moisil", Onesti

40

n+1
zj+1 zj si S3 S4 = |S3 S4 | = |zj+1 zj | |zj+1 | + |zj | 2
= n + 1.
2
Contradictia la care am ajuns arat
a c
a presupunerea initial
a este fals
a.
n3 + 5n
Problema 3. Fie n 2 un numar ntreg. Coloram fiecare dintre
6
numere ntregi consecutive n rosu sau albastru. Demonstrati ca exista o submultime
monocolora {a1 , a2 , . . . , an } astfel nct 1 a2 a1 a3 a2 an an1 .
(Concursul "Al. Myller", 2007)
n3 + 5n
Solutie. Putem considera ca numerele sunt 1, 2,. . . ,
, culorile sunt A si R
6
n (n 1)
si fie dn =
+ 1. Vom demonstra prin inductie c
a pentru orice n 2, exist
a
2
submultimea monocolor
a {a1 , a2 , . . . , an } pentru care 1 a2 a1 an an1
dn . Proprietatea este imediat
a pentru n = 2; o presupunem adev
arat
a pentru n si
n3 + 5n
sa o verificam pentru n + 1. Conform ipotezei inductive, printre primele
6
numere exista submultimea monocolora {a1 , a2 , . . . , an }, colorata cu A, astfel nct
1 a2 a1 an an1 dn . Consider
am numerele an + dn , an + dn + 1,. . . ,
an + dn + n, n num
ar de n + 1. Dac
a toate au culoarea R, proprietatea este verificat
a
pentru n+1. Daca unul dintre ele, fie acesta an+1 , are culoarea A, atunci an+1 an
3
n3 + 5n
(n + 1) + 5 (n + 1)
dn + n = dn+1 , iar an+1
+ dn + n =
, deci multimea
6
6
{a1 , . . . , an , an+1 } ndeplineste conditia dorita.
n2
Problema 4. Fiind date n numere reale, demonstrati ca exista cel mult
4
perechi (ai , aj ) cu proprietatea ca 1 < |ai aj | < 2.
Solutie. Cerinta problemei sugereaza metoda de rezolvare: un graf cu n vrfuri,
n2
ale c
arui muchii nu formeaz
a triunghiuri, are cel mult
muchii (rezultatul lui
4
Mantel, caz particular al teoremei lui Turan). Construim un graf astfel: vrfurile sunt
numerele date, iar dou
a vrfuri ai , aj se unesc printr-o muchie cnd 1 < |ai aj | <
2. Dac
a, prin absurd, ar exista trei vrfuri unite prin muchii, am avea simultan
1 < |ai aj | < 2, 1 < |aj ak | < 2 si 1 < |ai ak | < 2. Putem presupune ca
ai > aj > ak si relatiile precedente devin 1 + aj < ai < 2 + aj si 1 + ak < ai < 1 + ak ,
de unde 1 + aj < 2 + ak , adic
a aj ak < 1, imposibil. Astfel, am demonstrat c
a
graful nu are triunghiuri si aplicarea rezultatului mentionat initial ncheie rezolvarea.
ncheiem prin a propune spre rezolvare celor interesati cteva probleme:

Problema 5. Fie O un punct pe o dreapta d, iar OP1 , OP2 ,. . . , OPn versori


cu extremit
atile ntr-un acelasi semiplan
determinat de d. Daca n este impar, sa se


arate ca OP1 + OP2 + + OPn 1 (I.M.O., 1973).
Problema 6. ntr-o secventa de numere reale, suma oricaror 7 termeni consecutivi este negativa, iar suma oricaror 11 termeni consecutivi este pozitiva. Sa se
determine numarul maxim de termeni ai unei asemenea secvente. (I.M.O., 1977)

Bibliografie
1. http://web.mit.edu/rwbarton/public/mop
2. http://www.mathlinks.ro
41

Acuratetea limbajului matematic n combinatoric


a
Lauren
tiu MODAN 1
Combinatorica, ramura distincta a Matematicii, a aparut ca o consecinta a ncerc
arii de rezolvare a problemelor de num
arare. Mai putin analitice dect n alte discipline matematice, rationamentele sale, f
acute aproape n mod primordial, o aproprie
de aceea ce astazi se numeste metalogica.
Cum aratam si n [2], o gndire matematica riguroasa si educata ar trebui sa
nceap
a cu nv
atarea Combinatoricii enumerative si s
a continue cu Probabilitatile
combinatoriale, strict legate de cotidian, asa cum actualmente se ntmpl
a la un
numar important de natiuni cultivate stiintific. Nu trebuie sa uitam ca astazi Combinatorica, prin noile sale ramuri: teoria grafurilor, teoria matroizilor, teoria codurilor
etc., este cel mai dinamic domeniu al matematicii, avnd num
arul cel mai important
de conjecturi enuntate si rezolvate anual.
n Romnia zilelor noastre, Matematica discreta, prin urmare si Combinatorica,
sunt efectiv neglijate! Iar educatia precara a tinerilor, n acest domeniu, se reduce
doar la mnuirea unor simple relatii algebrice, deseori numite "formule", f
ar
a ns
a ca
ei s
a poat
a discerne cnd trebuie s
a foloseasc
a permutari, aranjamente sau combinari,
simple sau cu repetitie, respectiv cnd trebuie sa foloseasca numarul functiilor ce
actioneaza de la o multime de cardinal n, la o alta de cardinal m.
Vom reaminti (v. [3]) c
a, pentru multimea M = {x1 , x2 , . . . , xn } de cardinal n,
i) permutarile sale difer
a unele de altele doar prin ordinea elementelor si sunt n
numar de n!;
ii) aranjamentele grupelor de cte m elemente difera prin ordinea si tipul obiectelor
si sunt Am
n;
iii) combinarile canum
ar al submultimilor cu m elemente difer
a doar prin tipul
n
obiectelor si sunt
= Cnm .
m
Numarul functiilor f : M {y1 , y2 , . . . , ym }, egal cu mn , a fost detaliat n [2].
Mention
am c
a este regretabil faptul c
a tinerii elevi si studenti nu sunt educati s
a
nteleaga rolul esential al cuvintelor si exprimarilor folosite n Combinatorica. Astfel,
constat
am cu surprindere, c
a n culegerea Exercitii si probleme de algebra [1], ap
arut
a
n numeroase reedit
ari, timp de peste un sfert de veac, apar erori perpetue privind
elementele de Combinatoric
a. n acest sens, la Problema 25 din cap. X, constat
am
aceeasi eroare ca si n editia din 1981, tiparita de Editura Didactica si Pedagogica:
(1) Pentru un joc, 3 baieti si 5 fete trebuie sa formeze doua echipe de cte 4. n
cte moduri se pot forma echipele? Dar daca n fiecare echipa trebuie sa fie neaparat
un singur baiat?
Observ
am c
a alc
atuirea celor dou
a echipe, de cte 4 persoane, revine la g
asirea
submultimilor de cte 4 elemente, prin folosirea combinarilor. Prima echipa se va
1

Prof. dr., Departamentul de matematic


a, Facultatea de informatic
a, A.S.E., Bucuresti

42

forma n C84 moduri, n timp ce a doua se realizeaz


a din cele 4 persoane r
amase, n
C44 moduri. Prin urmare, alcatuirea celor doua echipe se face n C84 C44 = 70 moduri.
Vom constata apoi, ca partea a doua a problemei este fara sens! ntr-adevar, daca
n fiecare echip
a s-ar afla un singur b
aiat, atunci pentru prima ar trebui s
a avem 1
b
aiat si 3 fete, iar pentru a doua ar mai trebui s
a existe o fat
a n plus.
ntr-un limbaj combinatoric corect, partea a doua a problemei (1) ar trebui reformulata n maniera urmatoare:
(2) n cte moduri se pot alcatui cele doua echipe de cte 4 persoane, daca n
fiecare intra cel putin un baiat?
Pentru rezolvare, prima echip
a, constituit
a dintr-un b
aiat si 3 fete, poate fi aleas
a
n C31 C53 = 3C52 = 30 moduri, n timp ce a doua echipa, formata din 2 baieti si 2
fete, se alege n C22 C22 = 1 mod. Cumulnd situatiile anterioare, decidem ca echipele
de 4 persoane, ce contin cel putin un b
aiat fiecare, se constituie n 30 1 = 30 moduri.
Problema anterioar
a admite urm
atoarea generalizare:
(3) n cte moduri 2n 1 baieti si 2n + 1 fete pot forma doua echipe de cte 2n
persoane? Dar daca n fiecare din cele doua grupe trebuie sa intre cel putin un baiat?
Lasam cititorului placerea de a constata ca, n cazul general, cele doua echipe
(4n)!
se constituie n
moduri, n timp ce pentru situatia suplimentar
a, acestea se
[(2n)!]2
formeaza n 4n3 n + 1 moduri. Cnd n = 2, regasim situatia discutata la (2), ca
punct de plecare al prezentei note.
Pentru exersarea notiunilor din Combinatoric
a, propunem cititorului interesat,
rezolvarea urmatoarei probleme:

(4) Fie A = 3n | n = 0, 33 si B submultimea numerelor din A divizibile cu 6.


i) n cte moduri se pot scrie pe 17 cartoane de culori diferite elementele lui B?
ii) Cte numere de 4 cifre se pot forma cu elementele de cte 2 cifre, continute n B?
iii) Cte numere de 4 cifre, ordonate crescator, se pot forma cu elementele de
cte doua cifre, continute n B?
Ca mijloc de verificare pentru cititor, d
am r
aspunsurile la (4), ce au ca punct de
plecare multimea B = {0, 6, 12, 18, 24, 30, 36, 42, 48, 54, 60, 66, 72, 78, 84, 90, 96}:
i) 17! moduri de scriere;
ii) 15+210 = 225 numere de 4 cifre, formate cu elementele de cte doua cifre din B;
iii) 105 numere de 4 cifre, ordonate cresc
ator si formate cu elementele de cte
dou
a cifre, din B.
Bibliografie
1. M. Brandiburu, D. Joita, C. N
ast
asescu, I. Nita
- Exercitii si probleme de
Algebra, Ed. Rotech Pro, 2004, Bucuresti.
2. L. Modan - Some Remarks Counting Functions, Octogon Mathematical Magazin,
Brasov, 15(2007), no.1, 262-265.
3. E. Rogai - Tabele si formule matematice, Ed. Tehnic
a, 1984, Bucuresti.

43

Asupra unei identit


ati clasice privind partea ntreag
a
Florin POPOVICI 1
Prezent
am cteva consideratiuni de ordin metodic asupra unei identit
ati clasice
privind partea ntreag
a a numerelor reale, pentru care d
am dou
a demostratii relativ
cunoscute precum si o a treia, simpla si eleganta, despre care credem ca este noua.
Problema 1 ([1], p. 20). S
a se arate c
a
h i
[x]
x
=
, x R, n N .
n
n

(1)

Metoda II ([1], pb. 11, p. 21 si 206). Evident, avem


h i
x
[x]

.
n
n

(4)

Solutie. Metoda I ([2], p. 122). Conform teoremei mpartirii cu rest


h xn
i Z, existra
q, r Z, unice, astfel nct [x] = nq + r, 0 r n 1. Urmeaza ca
=q+ .
n
n
Rezult
a c
a

[x]
= q.
(2)
n
x
[x] + {x}
r + {x}
r + {x}
n1+1
si 0
Deoarece =
=q+
<
= 1 rezulta ca
n
n
n
n
n
hxi
= q.
(3)
n
Din (2) si (3) rezulta ca are loc (1).

Din definitia partii ntregi a unui numar real rezulta ca [x] + [y] [x + y], x, y
R. Prin inductie deducem c
a n [x] [nx], x R , n N . Renotnd variabilele,
i
h
x
[x]
a c
a
obtinem

, x R, n N . Urmeaz
n
n
h i
[x]
x

.
(5)
n
n
Din (4) si (5) rezult
a (1).

am functia f : R Z definit
a prin f (x) =
Metoda
h iIII. Fie n N . Consider
[x]
x

, x R. Avem
n
n





hxi
[x + n]
x+n
[x] + n
x+n
[x]
f (x+n) =

=
+1
1 = f (x),
n
n
n
n
n
n

[x]
<1
deci functia f este periodic
a de perioad
a n. Pentru orice x [0, n] avem 0
n
x
si 0 < 1. Urmeaza ca f (x) = 0, x [0, n]. Rezulta ca f (x) = 0, x R, deci
n
are loc (1).
1

Prof. dr., Colegiul National "Gr. Moisil", Brasov

44

Observatia 1. Metoda III este inspirat


a de una dintre metodele clasice [2] pentru
demonstrarea celebrei identitati a lui Hermite:

1
n1
[x] + x +
+ + x +
= [nx] , x R, n N , n 2.
n
n
Bibliografie
1. L. Niculescu, I. P
atrascu, A. Secl
aman, M. G
al
ateanu - Exercitii si probleme
de matematica. Clasa a IX-a, Editura Cardinal, Craiova, 2004.
2. I. M. Vinogradov - Bazele teoriei numerelor, Editura Academiei, Bucuresti, 1954.

Profesorul (c
atre elevul de la tabl
a): Pretul unui produs se mareste cu 10%,
dupa care se micsoreaza cu 10%. Compara pretul initial cu cel final!
Elevul:
xy + 10% 10% = xy.
Profesorul: Bine, greseala cu 10% din "nu se stie ce" e veche, dar de ce xy?
Elevul: P
ai, nu ati spus produs?
(Bogdan Enescu)

Scaderea de mai jos scrisa cu cifre romane este corecta:

XI X = I.

Rotiti pagina cu 180 si veti constata ca relatia ramne adevarata (verificati!). Mai
gasiti si o alta scadere de acest fel cu aceeasi proprietate!
(Titu Zvonaru)
R
aspuns. Deoarece pot fi folosite doar numerele
I, II, III, IX, X, XI, XIX, XX, XXX,
prin ncerc
ari se obtin toate sc
aderile de acest fel:
X IX = I
XI X = I
XI IX = II.

45

O abordare analitic
a a unor probleme de geometrie
2
Gabriel POPA1 , Ioan SERDEAN

Cu prilejul elabor
arii lucr
arii [1], am constatat c
a o serie de probleme de geometrie propuse juniorilor la O.B.M.J. admit rezolv
ari analitico-trigonometrice ceva mai
simple dect cele "oficiale". ntruct o parte dintre elevii din cl. a IX-a sunt nca
eligibili pentru lotul juniorilor, iar elevii buni si pasionati de matematica parcurg
materia n avans, consider
am util
a prezentarea n aceast
a manier
a a ctorva solutii
ale unor probleme care, abordate sintetic (vezi [1]), sunt dificile.
b +
Problema 1. Fie ABCD un trapez cu AB k CD, AB > CD si m(A)

b = 90 . Sa se arate ca distanta dintre mijloacele laturilor paralele este egala cu


m(B)
semidiferenta bazelor.
(Problema 132, Lista scurt
a O.B.M.J., 2007)
Solutie. Raport
am planul la un reper cartezian
cu originea n A, ca n figura. Fie D0 , C 0 proiectiile
\
punctelor D, respectiv C pe AB; not
am t = m(DAB),
0
0 0
0
\
a = AD , b = D C , c = C B. Avem ca m(CBA) =
90 t si atunci DD0 = AD0 tg t = a tg t, iar CC 0 =
c
c
C 0 B tg (90 t) =
. Deci, a tg t =
, prin urmare
tg t
tg t
trapezului
c = a tg2 t. Vrfurile

vor avea coordonatele


A (0, 0); B a 1 + tg2 t + b, 0 ; C (a+ b, a tg t); D (a, a tg t), iar mijloacele bazelor
a 1 + tg2 t + b
2a + b

[AB] si [CD] au coordonatele M


, 0 , respectiv N
, a tg t .
2
2
Lungimea segmentului M N este
s
s
2
2

2
a tg t 1
a2 tg2 t + 1
a tg2 t + 1
2
2
+ a tg t =
=
.
MN =
4
4
2

2
a tg t + 1
AB CD
a+c
Pe de alta parte,
=
=
, de unde concluzia.
2
2
2
Problema 2. Fie ABCD patrat, E mijlocul lui [CD], iar M un punct interior
\
\
\
patratului astfel nct m(M
AB) = m(M
BC) = m(BM
E) = x. Sa se afle x.
(Problema 203, Baraj O.B.M.J., 2003)
Solutie. Raport
am planul la un reper cu originea
n A, ca n figura; consideram unitatea egala cu latura
patratului si atunci A (0, 0); B (1, 0); C (1, 1); D (0, 1);
E (1/2, 1). Not
am m = tg x, m (0, 1) (1, );
panta dreptei AM este m, iar panta dreptei BM este
1
1
tg (90 + x) =
= . Astfel, AM : y = mx si
tg x
m
1
BM : y = (x 1), iar prin intersectarea celor doua
m
1
2

Profesor, Colegiul National, Iasi


Profesor, Liceul Teoretic "Aurel Vlaicu", Or
astie

46

drepte obtinem coordonatele lui M : xM =

1
m
, yM =
. Panta dreptei
2
1+m
1 + m2

yE yM
2m2 2m + 2
=
, prin urmare
xE xM
m2 1

mBM mME 2m3 m2 + 2m 1 2m 1


=
=
.
\
tg BM
E =
1 + mBM mME m3 2m2 + m 2 m 2

2m 1
= m, cu solutiile m
\
Cum tg BM
E = tg x = m, obtinem ecuatia
m2

1, 2 3, 2 + 3 . Am vazut ca m 6= 1 si atunci ramne ca m 2 3, 2 + 3 ,


\
\
adic
a x {15 , 75 }. Prima solutie nu convine: dac
a m(M
AB) = m(M
BC) = 15 ,

\
atunci BM
E este unghi obtuz. n concluzie, x = 75 .
M E este

Problema 3. Se considera triunghiul ABC cu AB = AC. Un semicerc de


diametru [EF ], cu E, F [BC], este tangent laturilor AB si AC n M , respectiv
N , iar AE retaie semicercul n P . Sa se arate ca dreapta P F trece prin mijlocul
coardei [M N ].
(Problema 94, Lista scurta O.B.M.J., 2003)
Solutie. Raportam planul la un reper cartezian cu
originea n mijlocul O al segmentului [BC], avnd dreapta
BC drept ax
a a absciselor si n
altimea din A drept ax
a
a ordonatelor. Consideram ca F (1, 0), E (1, 0), C (b, 0),
\ ); atunci N (cos t, sin t),
B (b, 0) si fie t = m(CON
[ = 90 t, avem c
M ( cos t, sin t). Cum m(ACO)
a
1
AO
[ = ctg t, deci AO = b ctg t =
= tg(ACO)
, c
aci
OC
sin t
b cos t = ON = 1 (din triunghiul dreptunghic ON C) si

1
1
astfel A 0,
. Ecuatia dreptei AE va fi y =
(x + 1) si, intersectnd aceast
a
sin t
sin t
2
2
dreapt
a cu cercul x + y = 1, obtinem ecuatia n x:

1 + sin2 t x2 + 2x + 1 sin2 t = 0 (x + 1) 1 + sin2 t x sin2 t 1 = 0.

sin2 t 1
, carora le corespund punctele E, respectiv P .
sin2 t + 1
2

sin t 1 2 sin t
1
2 sin t
Cum y2 =
, avem P
,
.
(x2 + 1) =
sin t
sin2 t + 1
sin2 t + 1 sin2 t + 1
Daca R este mijlocul segmentului [M N ], atunci R (0, sin t); scriem imediat ecuatia
dreptei RF : y = (1 x) sin t. Coordonatele (x2 , y2 ) ale punctului P verifica aceasta
ecuatie si de aici rezult
a concluzia problemei.
Ca urmare, x1 = 1 si x2 =

Problema 4. Un semicerc avnd diametrul [EF ]


inclus n latura [BC] a triunghiului ABC este tangent laturilor AB si AC n Q, repsectiv P . Notam
{K} = EP F Q. Sa se arate ca AK este naltime
n triunghiul ABC.
(Problema 15, O.B.M.J., 2000)
Solutie. Raport
am planul la un reper cartezian
cu originea n O mijlocul segmentului [EF ], avnd
47

pe BC ca ax
a Ox si perpendiculara n O pe BC ca ax
a Oy. Consider
am c
a F (1, 0),
E (1, 0), P (cos a, sin a), Q (cos b, sin b). Panta lui OP este m = tg a si atunci panta
1
lui AC va fi = ctg a; obtinem ecuatia lui AC : x cos a + y sin a = 1. Analog,
m
AB : x cos b + y sin b = 1 si, intersectnd cele doua drepte, vom obtine pentru abscisa
punctului A
ab
cos a+b
2 cos a+b
sin a sin b
2 sin 2
2
=
.
=
xA =
ab
ab
sin (a b)
2 sin ab
cos
cos
2
2
2
cos a+b
2
ax=
.
n
altimea din A fiind paralel
a cu Oy, va avea ecuatia x = xA , adic
cos ab
2
Pentru a afla coordonatele punctului K, vom intersecta dreptele EP :
x+1
y
x1
y
si F Q :
=
=
. Eliminnd pe y, g
asim c
a
cos a + 1
sin a
cos b 1
sin b
sin a cos b + cos a sin b sin a + sin b
sin (a + b) (sin a sin b)
xK =
=
=
sin b cos a sin a cos b + sin a + sin b
sin (a b) + (sin a + sin b)

ab
sin a+b
2 cos a+b
cos a+b
2
2 sin 2
2

=
.
=
a+b
ab
ab
2 cos ab

sin
cos
sin
2
2
2
2
Rezult
a astfel c
a punctul K apartine n
altimii din A, de unde concluzia problemei.

Rezolvnd problemele de geometrie din [1], am remarcat ca un procent semnificativ dintre ele (aproape 20%) admit solutii calculatorii, n maniera celor prezentate n
aceast
a not
a. ncheiem prin a propune ca tem
a trei astfel de probleme.
Problema 5. Fie ABC un triunghi echilateral de centru O, iar M (BC). Fie
K, L proiectiile lui M pe AB, respectiv AC. Sa se arate ca OM trece prin mijlocul
segmentului [KL].
(Problema 135, Lista scurt
a O.B.M.J., 2006)
Problema 6. Punctele M si N se gasesc pe laturile (AD) si (BC) ale rombului ABCD. Dreapta M C intersecteaza segmentul [BD] n T , iar dreapta M N
intersecteaza [BD] n U . Dreapta CU intersecteaza dreapta AB n Q, iar QT intersecteaza latura [CD] n P . Aratati ca triunghiurile QCP si M CN au aceeasi arie.
(Problema 232, Baraj O.B.M.J., 2005)
Problema 7. Fie ABC un triunghi dreptunghic n C si punctele D, E pe
BD
AE
laturile [BC], respectiv [CA], astfel nct
=
= k. Dreptele BE si AD se
AC
CD

\ = 60 daca si numai daca k = 3.


intersecteaza n O. Sa se arate ca m(BOD)
(Problema 246, Baraj O.B.M.J., 2006)
Bibliografie
1. D. Brnzei, D. S
erb
anescu, G. Popa, I. S
erdean - 10 ani de Olimpiade Balcanice ale Juniorilor, Paralela 45, Pitesti, 2007.

48

A study of a new geometric inequality


Chang Jian ZHAO 1
Abstract. In this paper, we find a new geometric inequality. Then we extend
and strengthen the inequality and get several quite wider results.

1. Introduction. Recently, we have found a new inequality in a triangle. The


result can be stated as follows
Proposition. In the triangle ABC, point P is on side AC or its extension line
and Q is on the side AB or its extension line. Points D and E are both on the side
BC and BD = DE = EC. Line AD intersects line P Q at F and AE intersects
line P Q at G. Then
1
4AF G 4AQP
(1.1)
3
and equality holds if and only if the line P Q and BC are parallel to each other.
In this paper, we shall generalize and strengthen this inequality. To simplify the
notation, let 4AF G denote the directed area of the triangle AF G, etc.
2. Extension of the inequality. Our main result is given by the following
theorem.
Theorem 2.1. In the triangle ABC, point Q is on the side AB or its extension
line and P is on the side AC or its extension line. Points Di are on the side BC
and lines ADi intersect line QP at Ei , where i = 1, 2, . . . , n 1, and BD1 = m1 ,
D1 D2 = m2 , . . . , Dn2 Dn1 = mn1 , Dn1 C = mn . Then
n
X
i=1

mi

n1
X
j=2

mj 4AQP n1 m1 mn nn 4AE1 En1

n1
Y
i=2

4AEi1 Ei

and equality holds if and only if QE1 = E1 E2 = = En2 En1 = En1 P .


Proof. Let B = D0 , C = Dn , Q = E0 and
P = En , then
n
X
4AEi1 Ei
4AQP
=
=
4AD0 D1
4AD0 D1
i=1
=

n
n
X
X
mi 4AEi1 Ei
mi AEi1 AEi
=
m
4AD
D
m
1
i1
i
1 ADi1 ADi
i=1
i=1
v
uQ
n AE
Q
i1 AEi
u n
mi
u
n
t
i=1 ADi1 ADi
n i=1
(2.2)
mn1

Department of Mathematics, Binzhou Teachers College, Shandong 256604, China

49

(2.1)

and equality holds if and only if all 4AEi1 Ei (i = 1, 2, . . . , n) are equal. Now we
suppose that
n
Y
AEi1 AEi
M=
.
AD
i1 ADi
i=1

Then

M=

=
On the other hand,

n1
AE0 AEn AE1 AEn1 Y AEi1 AEi

=
AD0 ADn AD1 ADn1 i=2 ADi1 ADi
n1
4AQP 4AE1 En1 Y 4AEi1 Ei

4ABC 4AD1 Dn1 i=2 4ADi1 Di

(2.3)

m1
4AQP
4AQP
,
= Pn
4ABC
4AD
m
0 D1
i
i=1

(2.4)

m1
4AE1 En1
4AE1 En1
= Pn1
,
4AD1 Dn1
4AD0 D1
j=2 mj

and

(2.5)

Qn1
n1
Y 4AEi1 Ei
4AEi1 Ei
mn2
1
i=2 4AEi1 Ei
=
= Qn1
.
mi
4AD
D
4AD0 D1n2
i1 i
4AD0 D1
i=2 mi
i=2
i=2
m1
Therefore, by (2.3), (2.4), (2.5), and (2.6) we have
n1
Y

M = Pn

i=1

mi

mn1
Pn1
j=2

mj

Qn1
i=2

n1
4AQP 4AE1 En1 Y
4AEi1 Ei .
4AD0 D1n
mi
i=2

(2.6)

(2.7)

Moreover, (2.2) and (2.7) yield that


v
u
u m1 mn 4AQP 4AE1 En1 n1
Y
4AQP
n
nt
4AEi1 Ei .
Pn
Pn1
n
4AD0 D1
i=1 mi
j=2 mj 4AD0 D1 i=2

Consequently,

q.e.d.

n1
4AQP 4AE1 En1 Y
4AQP n
m1 mn nn

4AEi1 Ei ,
Pn
Pn1
4AD0 D1n
4AD0 D1n
i=1 mi
j=2 mj
i=2

According to Theorem 1, we shall get the following


Corollary 2.1. If the hypotheses of Theorem 1 are fulfilled, then
P Qn1

n
X
i=1

mi

n1
X
j=2

mj m1 mn nn E1 En1

n1
Y

Ei1 Ei

i=2

and equality holds if and only if the QE1 = E1 E2 = = En2 En1 = En1 P .
Moreover, if we take m1 = m2 = = mn in Theorem 1, then we have
50

Corollary 2.2. Under the hypotheses of Theorem 1 and m1 = m2 = = mn ,


we have
n1
Y
(n 2) 4AQP n1 nn1 4AE1 En1
4AEi1 Ei
(2.8)
i=2

and equality holds if and only if the line P Q and BC are parallel to each other.
Similarly, we can find easily also the following
Corollary 2.3. If the hypotheses of Corollary 2.2 are fulfilled, then
(n 2) P Qn1 nn1 E1 En1

n1
Y

Ei1 Ei

i=2

and equality holds if and only if the line P Q and BC are parallel to each other.
Corollary 2.4. Under the hypotheses of Theorem 1, if the point Q and B are
coincident, then
n
X
i=1

mi

n1
X
j=2

mi 4ABP n1 m1 mn nn 4AE1 En1

n1
Y
i=2

4AEi1 Ei .

3. Strengthener of the inequality. Our first result is the following


Theorem 3.1. If the hypotheses of Theorem 1 are fulfilled, then
n1 X

n
n1
n1
n
X
X
X
3
3
mi
mj R2(n1)
mi
mj 4AQP n1
4
i=1
j=2
i=1
j=2
m1 mn nn 4AE1 En1

n1
Y
i=2

4AEi1 Ei

(3.1)

and the first inequality in (3.1) becomes an equality if and only if AQP is a regular
triangle.

3
In fact, it is well known that 4
(abc)2/3 where a, b, c are the sides of an
4
arbitrary triangle and 4 is the area. Hence the left side hand side of (3.1) is easily
proved.
Finally, we can again give a very well result.
Theorem 3.2. Under the hypotheses of Theorem 1 and letting 4AEi1 Ei = Si
( i = 1, 2, . . . , n), then
4AQP
where
Hi =
and
X

HS +

n2
X

(Hi1 + Hi + Hi+1 ) Si ,

(3.2)

i=3

mi (mi1 + mi + mi+1 )
,
27mi1 mi+1

i = 2, 3, . . . , n 1,

HS = (H2 + 1) S1 + (Hn1 + 1) Sn + (H2 + H3 ) S2 + (Hn2 + Hn1 ) Sn1 ,


51

and equality holds if and only if QE1 = E1 E2 = = En2 En1 = En1 P .


Proof. If we take n = 3 in Theorem 2.1, then the inequality (2.1) reduces to the
following inequality
s
m2 (m1 + m2 + m3 )
4AE1 E2
4AE0 E3 ,
(3.3)
27m1 m3
and equality holds if and only if the line P Q and BC are parallel to each other, i.e.
4AE1 E2 H2 (4AE0 E1 + 4AE1 E2 + 4AE2 E3 ) ,

that is, S2 H2 (S1 + S2 + S3 ). Consequently, we have Si Hi (Si1 + Si + Si+1 ),


where i = 2, 3, . . . , n 1. Hence, by summation
n1
X
i=2

On the other hand,


n1
X

Si

Hi (Si1 + Si + Si+1 ) =

i=2

n1
X

n2
X

Hi (Si1 + Si + Si+1 ) .

(3.4)

i=2

(Hi1 + Hi + Hi+1 ) Si +

i=3

HS S1 Sn . (3.5)

Therefore, by (3.4) and (3.5), we get the inequality (3.2), q.e.d.


Acknowledgment. I wish to express my gratitude to professor Li Wenrong for
his valuable help in writing this paper.
References
1. D. S. Mitrinovic - Analytic inequalities, Springer-Verlag, 1970.
2. B. G. Pachpate - On Some New Inequality Similar to Hilberts Inequality, J. Math.
Anal. Appl. 226(1998), 166-179.
3. Ch. J. Zhao - The further research of Yang Le inequality, Journal of Binzhou
Teachers College, 12(1996), 32-34.
4. L. Carlitz, F. Leuenberger - Problem E1454, Amer. Math. Monthly 68(1961),
177 and 68 (1961), 805-806.
5. P. Finsler and H. Hadwiger - Einige Relationen in Dreieck, Comment. Math. Helv.
10(1937/38), 316-326.

52

Concursul "Recreatii Matematice"


Editia a V-a, Muncel (Iasi), 31 august 2007
Clasa a V-a
1. Fie a, b N. S
a se arate c
a dac
a ultima cifr
a a num
arului a2 + b2 este 9, atunci
2
ultima cifra a numarului (a + b) este tot 9. Reciproca este adevarata?
2. Sa se determine numerele naturale nenule n, a, b, c, d, e, f stiind ca b = 2c si
1 2 3 n = 2a 3b 5c 7d 11e 13f .
3. S
a se determine numerele naturale distincte m, n si p astfel nct s
a aib
a loc
relatia 653 < 5m + 5n + 5p < 809.

Clasa a VI-a
1. Fie A = 3m 5n , unde m, n N. Not
am cu a, b, c num
arul divizorilor numerelor
A, 3A si, respectiv, 5A. S
tiind c
a numerele a si b sunt direct proportionale cu 3 si 4,
iar numerele b si c sunt invers proportionale cu 15 si 16, sa se determine numarul A.
2. Sa se afle numerele prime p, q si r stiind ca numarul pq +pr este patrat perfect.
3. Fie paralelogramul ABCD n care AB = 2AD, iar punctele M si N sunt
mijloacele laturilor [AB] si, respectiv, [CD]. Dac
a [EM ] [AB], A [EM ] si
DE N B = {F }, s
a se arate c
a dreptele AN si DM se intersecteaz
a n centrul de
greutate al triunghiului F BE.

Clasa a VII-a
1. S
a se rezolve n multimea numerelor naturale ecuatia 6a 5b = 1.
\ = m(ABM
\) =
2. Aflati aria triunghiului ABC stiind c
a AC = 4 cm si m(ACB)
45 , unde punctul M este mijlocul laturii (AC).
b = 30 . Se considera DE
b = 105 si M (B)
3. Fie triunghiul ABC cu m(A)
\
mediatoarea segmentului [BC], D BC, E AB, [CF bisectoarea unghiului BCE,
F AB, iar {I} = CF DE, {G} = CE AI. S
a se arate c
a
a) triunghiul DF G este echilateral;
b) AS SB, unde {S} = BI EC;
1
c) d (S, AB) = AB.
4

Vizitati pe Internet revista "Recreatii Matematice" la adresa

http://www.recreatiimatematice.uv.ro
53

South Eastern European Mathematical Olympiad


for University Students (SEEMOUS)
Agros, Cyprus, 7-12 March, 2007
Problems
1. Given a (0, 1) Q let a = 0, a1 a2 a3 . . . be its decimal representation. Define

X
fa (x) =
an xn , x (0, 1) .
n=1

P (x)
Prove that fa is a rational function of the form fa (x) =
, where P and Q are
Q (x)
polynomials with integer coecients.

P
an xn for
Conversely, if ak {0, 1, 2, . . . , 9} for all k N, and fa (x) =
n=1
P (x)
x (0, 1) is a rational function of the form fa (x) =
, where P and Q are polyQ (x)
nomials with integer coecients, prove that the number a = 0, a1 a2 a3 . . . is rational.
2. Let f (x) = max |xi | for x = (x1 , x2 , . . . , xn )T Rn and let A be an n n
i

matrix such that f (Ax) = f (x) for all x Rn . Prove that exists a positive integer
m such that Am is the identity matrix In .

3. Let F be a field and let P : F F F be a function such that for every


x0 F the function P (x0 , y) is a polynomial in y and for every y0 F the function
P (x, y0 ) is a polynomial in x.
It is true that P is necessarily a polynomial in x and y, when
a) F = Q, the field of rational numbers?
b) F is a finite field?
Prove your claims.
4. For x R, y 0 and n Z denote by wn (x, y) [0, ) the angle in radians
with which the segment joining the point (n, 0) to the point (n + y, 0) is seen from
the point (x, 1) R2 .

P
a) Show that for every x R and y 0, the series
wn (x, y) converges. If
n=

P
wn (x, y), show that w (x, y) ([y] + 1) . ( [y] is the
we now set w (x, y) =
n=

integer part of y.)


b) Prove that for every > 0 there exists > 0 such that for every y with
0 < y < and every x R we have w (x, y) < .
c) Prove that the function w : R [0, +) [0, +) defined in a) is continuous.

Solutions
1. As the expansion of a is periodic, there is a n0 such that an = an+p for n > n0 .
Now, for n = n0 + mp, we have
a1 x + a2 x2 + + an xn + =

= a1 x+a2 x2 + +an0 xn0 +xn0 +1 R(x)+xn0 +p+1R(x)+ +xn0 +(m1)p+1R(x)+. . . ,


54

where R (x) = an0 +1 + an0 +2 x + + an0 +p xp1 . Hence this series is a sum of the
polynomial S (x) = a0 + a1 x + a2 x2 + + an xn = a0 + a1 x + a2 x2 + + an0 xn0 and
an infinite geometric series with first element xn0 +1 R (x) and a factor xp . Therefore
xn0 +1 R (x)
the above series is equal to S (x) +
. So the conclusion easily follows.
1 xp
1
For the converse, evaluate the expression for x =
.
10
T

2. Let A = (aij ). Taking x = (1, 0, . . . , 0) we get max |ai1 | = 1. Similarly taking


i

x = (0, 1, . . . , 0)T , . . . , x = (0, 0, . . . , 1)T we get max |aij | = 1 for j = 1, 2, . . . , n.


i

Take x = (ai1 , ai2 , . . . , ain ) ; note that f (x) 1. We have Ax = (y1 , . . . , yn ) ,


n
P
where yi =
a2ij 1.
i=1

Calculating S =

n
n P
P

i=1 j=1

a2ij by columns and by rows leads us to inequality n


n
P

a2ij equals to n. This means thaat in each

column of A there exists exactly one nonzero element ai(i) = 1, and here is
a permutation of {1, 2, . . . , n}. Therefore we have An! = diag (b1 , b2 , . . . , bn ) is a
diagonal matrix with |b1 | = |b2 | = = |bn | = 1. Finally, A2n! = In .
S n. Hence S = n and each sum

j=1

3. a) If F = Q or any other countable field, the assertion is false.


Let (qn ) be an enumeration of F . For consider the function P : F F F given
as follows: if y Q, so that y = qn for some n N, set
n
X
P (x, y) =
(x q1 ) (x q2 ) (x qk ) (qn q1 ) (qn q2 ) (qn qk ) ;
k=1

similarly, for x Q, x = qm for some m N, set


m
X
P (qm , y) =
(qm q1 ) (qm q2 ) (qm qk ) (y q1 ) (y q2 ) (y qk ) .
k=1

The two expression give the same values for x = qm , y = qn .


It is clear that it cannot be a polynomial in x and y. For instance, if it were of
degree N in x then the coecient of xN +1 is zero, yet the above has coecient whose
values at y = qN +2 is nonzero number (qN +2 q1 ) (qN+2 q2 ) (qN+2 qN +1 ).
b) If F is a finite field then thee answer is yes, because every function P : F
F F is a polynomial. Namely a linear combination of polynomials of the type
Q x ak Q y ak
which take the value 1 for x = am , y = an and 0 for all
k6=m am ak k6=n an ak
other cases.
4. a) If y = 0 the result is clear. If 0 < y < 1 the angle are disjoint so their sum is
less than the angle with which R is seen from (x, 1), that is . This is geometrically
clear, but can also be seen analytically by observing that
Z nx+y
Z
ds
ds
wn (x, y) = arctan (n x + y)arctan (n x) =

= .
2
2
1+s
nx
1 + s
55

In general, y = [y] + r with 0 r < 1 and

[y]1

wn (x, [y] + r) =

n=

X X

wn+j (x, 1) +

j=0 n=

= [y] +

n=

wn+[y] (x, r) =

n=

wn (x [y] , r) < ([y] + 1) .

b) Let x R be fixed and let positive < 1 be given. Define A to be a (large)


positive integer such that the segment [A, A] is seen from (x, 1) by an angle > ,
and set S = Z\ {A, A + 1, . . . , A 1, A}. For 0 < y < 1 we have
X
wn (x, y) < .
(1)
nS

For n in {A, A + 1, . . . , A 1, A} (a finite set), using the fact that


Z nx+y
ds
lim wn (x, y) = lim
= 0,
y0+
y0+ nx
1 + s2
P
it follows that there is a > 0 such that
wn (x, y) < for every y with 0 < y < .
nZ\S

Combining with (1), the required result follows.


To show that the limit is uniform note that, by the first part of b),

X
X
X
y0+
wn (x, y)
wn ([x] + 1, y)
wn (0, y) 0.
n[x]+1

n=0

n[x]+1

Also (for 0 < y < 1), again by the first part of b),

X
X
X
y0+
wn (x, y) =
wn (0, y)
wn (0, y) 0.
n=0

n1

n[x]1

Hence 0 w (x, y) 2

n=0

wn (0, y) + w0 (0, y), which does not depend on x R,

and converges to 0 as y 0+. The conclusion now follows.


c) Let x1 , x2 R and y1 , y2 (0, ).
Z

Z nx2 +y2
X
nx1 +y1
ds
ds

|w (x1 , y1 ) w (x2 , y2 )| =

2
n= nx1
1 + s2
1
+
s
nx2

Z nx2
Z nx1 +y1
X
X

ds
ds

1 + s2 n= nx2 +y2 1 + s2
n= nx1

X
X
|x2 x1 |
|x2 x1 | + |t2 t1 |
+
2
1
+
s
1 + u2n
n
n=
n=

where sn is between n x1 and n x2 , and un is between n x1 + y1 and n x2 + y2 .


Since the two series are convergent, the result follows on R [0, +). Moreover,
the uniform convergence of part b) completes the proof.

56

Solutiile problemelor propuse n nr. 1/2007


Clasele primare
P.124. Schimba locul unui singur betisor pentru a obtine o egalitate.
(Clasa I )
Mariana Nastasia, elev
a, Iasi
Solutie.

P.125. ntr-o clasa cu 24 elevi sunt 3 perechi de gemeni. La sedinta cu parintii


este prezent cte un singur parinte din fiecare familie. Cti parinti participa la
sedinta?
(Clasa I )
Mihaela Glc
a, elev
a, Iasi
Solutie. La sedinta participa 24 3 = 21 parinti.

P.126. Pentru a fierbe un ou sunt necesare 4 minute. Mama vrea sa fiarba 20


oua n trei transe. Cte minute sunt necesare?
(Clasa a II-a)
Ionela B
ar
agan, elev
a Iasi
Solutie. 4 min +4 min +4 min = 12 min.
P.127. Mircea are cu 35 timbre mai mult dect fratele sau, Marius. Ct devine
diferenta, daca Mircea ar mai primi 10 timbre, iar Marius ar da unui prieten 5
timbre?
(Clasa a II-a)
Inst. Maria Racu, Iasi
Solutie. Diferenta se mareste atunci cnd descazutul se mareste sau scazatorul
se micsoreaz
a. Diferenta devine 35 + 10 + 5 = 50.
P.128. Cte numere de forma RM AT ndeplinesc conditia RAM = M AT ?
(Clasa a III-a)
Dragos Covrig, elev, Iasi
Solutie. R = M = T (9 valori); A (10 valori); 9 10 = 90 (numere).
P.129. Scrieti toate adunarile de forma

MARI +
ARI
RI
.
I
7676

(Clasa a III-a)

Solutie.

Dana Brsan, elev


a, Iasi
7324 +
324
24
4
7676

7289 +
289
89
9
7676

6789 +
789
89
9
7676

P.130. Daca a, b, c sunt cifre, cte egalitati de tipul a c = b : c se pot scrie?


Justificati raspunsul.
(Clasa a III-a)
Adina Voinescu, elev
a, Iasi
57


c = 1 a 1 = b : 1 a = b = 0, 1, . . . , 9
(10 cazuri)
a = 1 si b = 4, a = 2 si b = 8 (2 cazuri)

Solutie. c = 2

c=3
a = 1 si b = 9
(1 caz)
10 + 2 + 1 = 13 egalitati.
P.131. Verificati daca afirmatia "A se mparte exact la 5, unde A = 2000 +
2 (1 + 2 + 3 + + 1999) + 1999 + 1997" este adevarata sau falsa.
(Clasa a IV-a)
Prof. Nicolae Iv
aschescu, Craiova
Solutie. A = 2000 + 2000 1999 + 3996. Deoarece 3996 nu se mparte exact la 5,
aceeasi proprietate o are si A. Deci, afirmatia din enunt este fals
a.
P.132. Mama Oanei a mplinit 17532 zile pe data de 1 ianuarie 2007. n ce an,
luna si zi a avut o vrsta de 3 ori mai mica?
(Clasa a IV-a)
nv. Geta Cretu, Vaslui
Solutie. Din 17532 = 48 365 + 12 se deduce ca anul nasterii este 2007 48 =
= 1959, pe 1 ianuarie. Restul 12 justific
a c
a de la 1.01.1959 pn
a la 1.01.2007 sunt
12 ani bisecti. Din 17532 = 2 5844 si 5844 = 16 365 + 4, rezult
a c
a vrsta de 3
ori mai mica a avut-o pe 1 ianuarie 1975.
P.133. Doi elevi spun pe rnd cte un numar natural, cel putin egal cu 1 si cel
mult egal cu 7. Fiecare nou numar spus se aduna la celelalte. Sa se arate ca primul
elev poate sa indice n asa fel numerele nct sa ajunga primul la suma 99.
(Clasa a IV-a)
Prof. Petru Asaftei, Iasi
Solutie. Avem 99 = 8 12 + 3. Primul elev va spune, prima dat
a, num
arul 3.
Al doilea elev trebuie sa spuna un numar n, 1 n 7. n continuare, de fiecare
data cnd i vine rndul, primul elev va spune un numar de forma 8 n. n acest
fel primul elev va completa o sum
a de tipul 8 k + 3, k fiind num
ar natural nenul.
Pentru k = 12 se obtine suma 99.

Clasa a V-a
V.76. Daca a, b, x sunt cifre n baza 10, sa se rezolve ecuatia cu necunoscuta
x: bxa + baa + xb + ab = abb + aab.
Marius Farcas, Iasi
Solutie. Efectund descompunerea n baza 10, dupa reduceri de termeni, obtinem
.
egalitatea 20x + 190b = 188a. Atunci 188a .. 10 si, cum a 6= 0, trebuie s
a avem a = 5.

Dupa nlocuire, gasim ca 2x + 19b = 94, deci b va fi o cifra para. ncercam pe rnd
b {2, 4, 6, 8} si obtinem valoare naturala pentru x doar cnd b = 4. n concluzie,
ecuatia nu are solutie dac
a (a, b) 6= (5, 4), iar n cazul n care (a, b) = (5, 4) avem
solutia x = 9.
Observatie. ncerc
arile pot fi evitate observnd c
a 76 19b 94, de unde
b = 4.
V.77. Sa se determine cte numere de trei cifre distincte abc au proprietatea ca
abc cba : 11 este patrat perfect.
Otilia Nemes, Ocna Mures (Alba)
Solutie. Din ipoteza deducem ca 9 (a c) este patrat perfect si, cum a, c sunt
cifre distincte, atunci a c {1, 4}. Dac
a a c = 1, cifrele a, c fiind nenule, obtinem
c
a (a, c) {(9, 8) ; (8, 7) ; . . . ; (2, 1)}. n fiecare caz, exist
a cte 8 valori ale lui b
diferite de a si c si astfel gasim 64 de numere abc. Daca a c = 4, vom obtine 5
58

perechi (a, c), deci 5 8 = 40 de numere abc. n total exist


a 64 + 40 = 104 numere cu
proprietatile dorite.
V.78. Aratati ca nu exista trei numere prime a, b, c astfel nct a (b + c) = bc.
Nicolae Iv
aschescu, Craiova
Solutie. Sa presupunem prin absurd ca ar exista trei numere prime cu proprietatea dat
a. Distingem trei cazuri:
i) b, c impare; atunci a (b + c) este par, bc este impar, imposibil.
ii) b, c pare; atunci b = c = 2, deci a (2 + 2) = 2 2, adica a = 1, fals.
iii) b par, c impar (sau invers); atunci b = 2, deci a (2 + c) = 2c, cu 2 + c impar.
Deducem c
a a este par, adic
a a = 2 si se ajunge la contradictia 2 + c = c.
V.79. Aratati ca numarul 131000 91000 se divide cu 1000.
Damian Marinescu, Trgoviste
Solutie. Deoarece 134 = 28561, iar 94 = 6561, avem:
250 4 250
250
250
13100 91000 = 134
9
= (M1000 + 561) (M1000 + 561)
= M1000 .

V.80. Daca restul mpartirii unui numar natural la 10 este mai mare dect 5,
spunem ca acel numar este favorabil. Aflati numerele favorabile ab cu proprietatea
ca nici ab, nici ba nu pot fi scrise ca suma de doua numere favorabile.
Ioan S
ac
aleanu, Hrl
au
Solutie. Numerele favorabile sunt cele care se temin
a n 6, 7, 8 sau 9. Se observ
a
c
a un num
ar natural se scrie ca sum
a de dou
a numere favorabile dac
a el se termin
a
n 2, 3, 4, 5, 6, 7 sau 8, iar aceast
a scriere nu este posibil
a atunci cnd el se termin
a
n 1 sau 9. Cum ab si ba sunt ambele favorabile, obtinem ca a = b = 9.

Clasa a VI-a
VI.76. Determinati a, b, c Z daca

a
b
3c + 5
= =
.
3
4
2c + 1

Gheorghe Iurea, Iasi


..
b
3c + 5
Solutie. Cum a = 3 si a, b Z, atunci b . 4. Rezult
a c
a
Z, deci
4
2c + 1
2c + 1 | 2 (3c + 5) 3 (2c + 1), adic
a 2c + 1 | 7. Obtinem c
a c {4, 1, 0, 3}, iar
solutiile problemei vor fi: (3, 4, 4), (6, 8, 1), (15, 20, 0), (6, 8, 3).
VI.77. Sa se arate ca ntre oricare doua puteri naturale consecutive ale lui 3 se
afla cel putin o putere a lui 2. Exista doua puteri consecutive ale lui 3 ntre care sa
putem gasi trei puteri ale lui 2?
Marius Damian, Br
aila
Not
a. Anterior public
arii n revista noastr
a, problema a ap
arut n R.M.T.
2/2006, cu numarul O.VI.146, semnata de acelasi autor. Solutia poate fi gasita
R.M.T. 3/2006.
VI. 78. Fie a, b Z astfel nct multimile {a + b, a + 2b, . . . , a + 2007b} si
{1, 2, . . . , 2007} coincid. Sa se arate ca exista k N pentru care a + kb = k.
Dan Nedeianu, Drobeta-Tr. Severin
Solutie (M
ad
alina-Vasilica Solcanu). Din ipotez
a deducem c
a
a + b + a + 2b + + a + 2007b = 1 + 2 + + 2007,
prin urmare 2007a + 2007 1004b = 2007 1004, deci a + 1004b = 1004. Astfel, exist
a
k = 1004 N pentru care a + kb = k.
59

VI.79. Se considera 4ABC ascutitunghic, iar M un punct n planul sau. Paralela prin M la AB taie AC si BC n P , respectiv N . Demonstrati ca daca doua
dintre urmatoarele afirmatii sunt adevarate, atunci este adevarata si a treia:
\ (ii) M C M B; (iii) [N P ] linie mijlocie n
(i) BM bisectoare pentru ABC;
4ABC.
Carmen-Daniela Tamas, Brlad
\
\
Solutie. (i) + (ii) (iii) Cum N
BM M
BA
\
BM
N , rezult
a c
a 4N M B este isoscel cu N B = N M .
\
\
Apoi, N M C N
CM , ambele avnd acelasi complement, deci 4N M C este isoscel cu N M = N C. Deducem c
a N este mijlocul lui [BC] si cum P N k AB,
atunci [N P ] va fi linie mijlocie n 4ABC.
(ii) + (iii) (i) Mediana [M N ] din 4M BC dreptunghic este egal
a cu [BN ], deci 4N BM este isos\
\
\
\
cel, cu N
BM N
M B. nsa N
MB M
BA (alterne interne), prin urmare
\
\
ABM M BC.
(iii) + (i) (ii) Ca la prima implicatie obtinem ca BN = N M . Cum N este
mijlocul lui [BC], [M N ] va fi median
a n 4M BC, egal
a cu jum
atate din latura BC.
\
Deducem ca 4BM C este dreptunghic, cu m(BM
C) = 90 .
VI.80. Sa se demonstreze ca portiunea hasurata din figura
alaturata poate fi scrisa ca reuniune de segmente nchise, doua
cte doua disjuncte.
Marius Tiba, elev, Iasi
Solutie. Not
am cu C1 conturul exterior si cu C2 pe cel interior. Dac
a O este un
punct fixatn interiorul lui C2 , M este un punct care parcurge C1 , iar {N } = OM C2 ,
atunci multimea tuturor segmentelor de forma [M N ], M C1 , verifica cerintele
problemei.

Clasa a VII-a
VII.76. Aflati numerele naturale a, b, c pentru care 11 (a b 9) > c (c 20),
11 (b c 9) > a (a 20) si 11 (c a 9) > b (b 20).
Veronica Pl
aesu si Dan Pl
aesu, Iasi
Solutie. Dac
a m, n Z si m < n, atunci m + 1 n. Astfel,
c (c 20) < 11 (a b 9) c2 20c 11a 11b 100.
Analog obtinem nc
a dou
a relatii si, prin adunare, deducem c
a
a2 + b2 + c2 20 (a + b + c) 300 (a 10)2 + (b 10)2 + (c 10)2 0,
prin urmare a = b = c = 10.
VII.77. Fie x, y numere reale pozitive, ambele subunitare sau ambele suprau1
1
1
nitare; sa se arate ca xy +
+ 2 x + + y + . Daca unul dintre numere este
xy
x
y
mai mic, iar celalalt mai mare ca 1, inegalitatea si schimba sensul.
Marian Tetiva, Brlad
Solutie. Avem voie s
a elimin
am numitorii si obtinem succesiv
x2 y 2 + 1 + 2xy x2 y + y + xy 2 + x (xy + 1)2 (x + y) (xy + 1) 0,
60

adic
a (xy + 1) (x 1) (y 1) 0, evident adev
arat. Cu aceleasi calcule se demonstreaza si partea a doua. Egalitatea se realizeza daca si numai daca macar unul dintre
numere este 1.
VII.78. Sa se rezolve n numere naturale ecuatia 6a 5b = 1.
Tudor P
adurariu, elev, Onesti
Solutie. Daca a, b 2, atunci 5b se termina n 25, iar 6a se termina n x6, cu
x cifr
a impar
a; evident c
a diferenta lor nu va fi niciodat
a 1. Cercetnd situatiile
r
amase, obtinem c
a (1, 1) este unica solutie a ecuatiei.
VII.79. Fie ABCD un patrulater convex, iar E si F intersectiile bisectoarelor
b respectiv B,
b cu diagonala [AC]. Sa se arate ca punctele E si F
unghiurilor D,
coincid daca si numai daca AB CD = AD BC.
Claudiu-
Stefan Popa, Iasi
Solutie.
Din teorema bisectoarei obtinem ca
AE
AB
AF
AD AC
AD
si
=
=
, de unde AE =
,
CD
CE
BC
CF
AD + CD
AB AC
. Cum punctele E si F sunt interioare
iar AF =
AB + BC
patrulaterului, atunci
AD AC
AB AC
E = F AE = AF
=

AD + CD
AB + BC
AD (AB + BC) = AB (AD + CD) ADBC = ABCD.
VII.80. Fie ABCDEF un hexagon regulat nscris ntr-un cerc, iar P un punct

pe arcul mic BC. Sa se arate ca P E + P F = P A + P B + P C + P D.


Dan Radu, Bucuresti
Solutie. Teorema lui Ptolemeu aplicata n patrulaterele inscriptibile P AEC si
P BF D d
a P A CE + P C AE = AC P E si P B F D + P D BF = P F BD. Cum
CE = AE = AC si F D = BF = BD, rezult
a c
a P A + P C = P E si P B + P D = P F
() de unde, prin sumare obtinem concluzia.
Observatii. 1) Aplicnd teorema lui Van Schooten (care poate fi stabilit
a f
ar
a
teorema lui Ptolemeu!) punctului P si 4ACE, 4BDF echilaterale, obtinem egalitatile () si continuam ca mai sus.
2) Alexandru Tudorache, elev, Iasi, propune o solutie trigonometrica bazata
pe teorema sinusurilor. Daca
= m(P C) si R este raza cercului
circumscris,

obtinem: P E = 2R sin 60 +
, P F = 2R cos , P A = 2R sin 60
, PB =
2
2
2

2R sin 30
, P C = 2R sin si P D = 2R sin 30 +
. Relatia este verificat
a
2
2
2
astfel cu usurinta.

Clasa a VIII-a
VIII.76. Fie ABCD un trapez cu AB k CD, M (AD) si N (BC) cu
M N k AB, iar E (AB), F (CD) oarecare. Fie {O} = EF M N , [OP ] si [N T ]
perpendiculare de aceeasi parte pe planul trapezului, G centrul de greutate al 4P EF ,
{Q} = M G (T BC). Sa se arate ca M N este linie mijlocie n trapez daca si numai
daca Q T N .
Bogdan Raita
, elev, Iasi
61

Solutie. Avem: Q T N QN (ABC)


(M QN ) (ABC) (M GO) (ABC)
GO (ABC) G P O P O este mediana
EO
AM
n 4P EF
=1
= 1 M N este
OF
MD
linie mijlocie a trapezului.
VIII.77. Pentru x, y R+ , sa se demonstreze inegalitatea
2


x6 +y 6 x2 y 2 x2 + y 2 x3 + y 3 xy (x + y) .

Lucian Tutescu, Craiova, si Gheorghe Nedelea, Pitesti


Solutie. Inegalitatea devine succesiv:

2
x6 + y 6 x4 y 2 x2 y 4 x3 + y 3 2xy (x + y) x3 + y 3 + x2 y 2 (x + y)

x4 y xy 4 2x3 y 3 2xy x4 + xy 3 + x3 y + y 4 + x4 y 2 + 2x3 y 3 + x2 y 4

2
4x3 y 3 + 2x5 y + 2xy 5 0 2xy x2 y 2 0,

evident adevarat. Egalitatea se atinge cnd x = y.


VIII.78. Pentru a, b, c R, sa se demonstreze inegalitatea
p
p
p
a2 + b2 ab + b2 + c2 bc + c2 + a2 ca a + b + c.

Claudiu-
Stefan Popa, Iasi

a+b
2
2
Solutie. Are loc inegalitatea a + b ab
; acest fapt este clar daca
2
a + b < 0, iar pentru a + b 0 ea revine, dupa calcule, la (a b)2 0. Scriind nca
dou
a relatii analoage si sumndu-le, obtinem concluzia. Egalitatea se atinge pentru
a = b = c 0.
VIII.79. Sa se rezolve n numere naturale ecuatia x (x + 1) = y 2007 .
Alexandru Negrescu, elev, Botosani
Solutie. Pentru x = 0, obtinem y = 0. Fie acum x 6= 0; cum (x, x + 1) = 1,
numerele x si x + 1 trebuie sa fie puteri de exponent 2007: x = a2007 , x + 1 = b2007 ,
cu b > a > 0. Atunci
x + 1 = b2007 (a + 1)

2007

> (a + 1) a2006 = a2007 + a2006 a2007 + 1 = x + 1,

contradictie. Rezulta ca unica solutie a ecuatiei este (0, 0).


VIII.80. Stiind
ca 1 ianuarie 2007 este ntr-o zi de luni, sa se arate ca pna n

anul 2100 exista trei ani bisecti n care luna februarie are trei duminici care cad n
zile impare.
Petru Asaftei, Iasi
Solutie. Trei duminici ale unei luni februarie dintr-un an bisect cad n zile impare
atunci cnd 1 februarie este ntr-o duminica. Considernd o axa a timpului n care
ziua nti este 1 ianuarie 2007, fiecare duminic
a va avea ca num
ar de ordine un
multiplu de 7. Ziua de 31 ianuarie 2008 are num
arul de ordine 396, iar un ciclu de
patru ani de tipul 1 februarie 2008 31 ianuarie 2012 are 1461 de zile. Pentru ca o
.
zi de 1 februarie a unui an bisect s
a cad
a duminic
a, trebuie ca 396 + 1461k + 1 .. 7,
deci 1461k = M7 + 2. Deducem ca (7 208 + 5) k = M7 + 2, adica 5k = M7 + 2. Cele
62

mai mici valori ale lui k avnd aceast


a proprietate sunt 6, 13, 20, iar lor le corespund
anii bisecti 2032, 2060, 2088.

Clasa a IX-a
IX.76. Fie d1 , d2 , . . . , dk divizorii
arului 53 72 , iar Sn = dn1 +dn2 + +dnk ,
num
3n
4n
5 +1 7 +1
n N . Sa se arate ca S2n =
Sn , n N . Generalizare.
(5n + 1) (7n + 1)
Petru Asaftei, Iasi
Solutie. Num
arul 53 72 are 4 3 = 12 divizori, anume 1, 5, 52 , 53 , 7 1, 7 5,
7 52 , 7 53 , 72 1, 72 5, 72 52 , 72 53 . Se constata usor ca

54n 1 73n 1
Sn = 1 + 5n + 52n + 53n 1 + 7n + 72n = n

;
5 1 7n 1

58n 1 76n 1
S2n = 1 + 52n + 54n + 56n 1 + 72n + 74n = 2n

=
5 1 72n 1
4n
3n
3n

3n

4n
4n
5 1 5 +1
7 1 7 +1
5 +1 7 +1
=

=
Sn .
(5n 1) (5n + 1)
(7n 1) (7n + 1)
(5n + 1) (7n + 1)

Generalizarea este imediat


a: dac
a p1 , p2 , . . . , pm sunt prime, iar d1 , d2 , . . . , dk
m
1 2
sunt divizorii lui p
1 p2 pm , atunci

( +1)n
( +1)n
+ 1 pm m
+1
p1 1
S2n =
Sn , n N .
(pn1 + 1) (pnm + 1)
p
IX.77. Sa se arate ca a3 + b3 ab 2 (a2 + b2 ), a, b 0.
Ovidiu Pop, Satu Mare
Solutie. Dac
a a = 0 sau b = 0, inegalitatea este evident
a. Fie a, b > 0; cu
a
b
notatiile x = + , inegalitatea se scrie succesiv
b
a

b3
a
a3
b
+

a6 + 2a3 b3 + b6 2a2 b2 a2 + b2 3 + 2 + 3 2
b
a
b
a

x3 5x + 2 0 (x 2) x2 + 2x 1 0.
2

Cum x 2, iar x2 + 2x 1 = (x + 1) 2, aceast


a din urm
a inegalitate este
adevarata si de aici rezulta cerinta problemei. Egalitatea se atinge cnd x = 2, deci
pentru a = b.

IX.78. Fie a, b, c laturile 4ABC, iar G centrul sau de greutate. Notam cu D,


E, F punctele de contact ale cercului nscris cu laturile BC, CA, respectiv AB. Sa

se arate ca aGD + bGE + cGF = 0 daca si numai daca 4ABC este echilateral.
Marian Urs
arescu, Roman

Solutie. Implicatia invers


a este imediat
a. Fie deci aGD + bGE + cGF = 0 si s
a
DB
pb
aratam ca a = b = c. Pentru k =
=
, obtinem ca
DC
pc

GB + k GC
(p c) GB + (p b) GC
GD =
=
,
1+k
a
63

deci aGD = (p c) GB + (p b) GC. Scriem nc


a dou
a relatii similare si le sum
am;

rezulta ca aGA + bGB + cGC = 0 . nsa GA + GB + GC = 0 , deci



aGA+bGB c GA + GB = 0 (a c) GA+(b c) GB = 0 ac = bc = 0.
n o
(am folosit faptul ca GA, GB constituie o baza). Deducem ca a = b = c.

IX.79. Fie 4ABC echilateral si P un punct n interiorul sau. Consideram


A1 AB, B1 BC, C1 CA astfel nct P A = P A1 , P B = P B1 si P C = P C1 .
Sa se arate ca P este centrul de greutate al 4A1 B1 C1 .
Iulia Plesca, elev
a, Iasi
Solutie. Fie P1 , P2 , P3 proiectiile lui P pe AB, BC,
respectiv CA. Daca O este centrul 4ABC, atunci
1 1
P P1 + P P2 + P P3 =
P A + P A1 +
P B + P B1 +
2
2

1
+
P C + P C1 =
2
3 1
P A1 + P B1 + P C1 .
= PO +
2
2
3
Pe de alta parte, P P1 + P P2 + P P3 = P O () si astfel
2

deducem ca P A1 + P B1 + P C1 = 0 , prin urmare P este


centrul de greutate al 4A1 B1 C1 .
Relatia () este relativ uzual
a si se justific
a astfel: ducem prin P segmentele
U V k AB, XY k BC, ST k AC, cu X, S AB, U, T BC, V, Y AC. Triunghi
1
urile P U T , P Y V si P XS sunt echilaterale si obtinem ca P P1 =
PX + PS ,
2
1 1
P U + P T , P P3 =
P V + P Y . Din regula paralelogramului,
P P2 =
2 2

P X + P U = P B, P T + P Y = P C si P S + P V = P A. Atunci
1 3
P P1 + P P2 + P P3 =
P A + P B + P C = P O.
2
2

IX.80. Fie , , , patru numere pozitive cu suma . Sa se afle maximul


sumei S = sin sin + sin sin si sa se determine situatia n care acest maxim este
atins.
Adrian Corduneanu, Iasi
Solutie. Consider
am un cerc de centru O si raz
a R, n care nscriem patrulaterul
\ = 2, m(BOC)
\ = 2, m(COD)
\ = 2 si m(DOA)
\ = 2. Atunci
ABCD cu m(ADB)
AB = 2R sin , BC = 2R sin , CD = 2R sin , AD = 2R sin , iar AC 2R,

BD 2R. Tinnd
seama de teorema lui Ptolemeu,
AB CD + BC AD = AC BD 4R2 sin sin + 4R2 sin sin 4R2 ,

deci S 1. Aceast
a valoare maxim
a este atins
a cnd AC = BD = 2R, deci cnd

ABCD este dreptunghi; rezulta ca S este maxima cnd = , = = , cu


2

0<< .
2
64

Clasa a X-a
X.76. Fie C = {z C | |z| = 1}. Sa se rezolve n C 2 ecuatia z1 z2 = z1 + z2 + 3.
Gabriel Popa si Paul Georgescu, Iasi
Solutia 1 (a autorilor). Dac
a z1 , z2 C, atunci
z1 z2 C, iar z1 + z2 + 3 este un num
ar complex
cu imaginea pe discul D de centru B (3, 0) si raza 2.
Evident ca C D = {A}, cu A (1, 0), prin urmare
z1 z2 = z1 + z2 + 3 = 1. Atunci z2 = z1 2 si obtinem
c
a z1 (z1 2) = 1 z12 + 2z1 + 1 = 0 z1 = 1,
apoi z2 = 1.
Solutia 2 (Mihai Haivas). Fie z1 = cos + i sin , z2 = cos + i sin ; ecuatia
data devine
cos ( + ) + i sin ( + ) = (cos + cos + 3) + i (sin + sin ) .
+
+
+

Egalnd p
artile imaginare, g
asim c
a sin
cos
= sin
cos
. Dac
a
2
2
2
2
+
+ = 2k (deci sin
= 0), obtinem c
a 1 = cos ( + ) = cos + cos + 3,
2
de unde cos + cos = 2, deci cos = cos = 1 si atunci z1 = z2 = 1. Dac
a
+

+ 6= 2k, deducem ca cos


= cos
, de unde
=
+ 2k.
2
2
2
2
Ambele situatii conduc imediat la contradictii.
X.77. Fie a, b C si z1 , z2 solutiile ecuatiei z 2 az + b = 0. Sa se arate ca
urmatoarele afirmatii sunt echivalente:

(i) |z1 | < 1 si |z2 | < 1;


(ii) |a|2 + a2 4b < 2 |b|2 + 1 < 4.
Marian Tetiva, Brlad
Solutie. (i) (ii) Fie r1 = |z1 |, r2 = |z2 |; conditia r1 < 1, r2 < 1 este
echivalent
a cu (r1 1) + (r2 1) < 0 si (r1 1) (r2 1) > 0, deci cu r1 + r2 < 2 si
r1 + r2 < r1 r2 + 1. Din a doua relatie, prin ridicare la p
atrat, obtinem

2
2
2
2
2 2
r1 + r2 < r1 r2 + 1 |z1 + z2 | + |z1 z2 | < 2 |z1 z2 |2 + 1 .

Cum z1 + z2 = a si z1 z2 = b, avem prima conditie din (ii). n plus, |b| = r1 r2 < 1,


de unde a doua parte a lui (ii).
(ii) (i) Facem un rationament invers celui de mai sus.
X.78. Determinati triunghiurile n care tangentele unghiurilor se exprima prin
numere naturale, exact doua dintre ele avnd aceeasi paritate.
C
at
alin Calistru, Iasi
Solutie. Se stie c
a tg A + tg B + tg C = tg A tg B tg C. Distingem situatiile:
a) tg A = 2p, tg B = 2q, tg C = 2r+1, cu p, q N , r N. Atunci 2p+2q+2r+1 =
4pq (2r + 1), egalitate imposibil
a ntruct membrul stng este impar, iar cel drept este
par.
b) tg A = 2p + 1, tg B = 2q + 1, tg C = 2r, cu p, q N, r N ; avem
2 (p + q + 1) + 2r = (2p + 1) (2q + 1) 2r p + q + 1 = (4pq + 2p + 2q) r

si dac
a p + q > 1, atunci r (4pq + 2p + 2q) 4pq + 2p + 2q > p + q + 1. R
amne
ca p + q 1 si, cum nu putem avea p = q = 0, deducem ca p + q = 1 si apoi
65


r = 1. Triunghiurile care satisfac cerinta sunt asemenea, avnd unghiurile , arctg 2
4
si arctg 3.
X.79. Sa se arate ca n orice triunghi are loc inegalitatea
(p r 2R) (p ra ) (p rb ) (p rc ) 0.
Cnd se atinge egalitatea?
I. V. Maftei si Dorel
B
aitan, Bucuresti
Q
Q
A
Solutie. Are loc identitatea
(p ra ) = p3
1 tg
, prin urmare pro2
Q
a cum 4ABC este ascutitunghic,
dusul (p ra ) este pozitiv, nul sau negativ dup
dreptunghic sau obtuzunghic. Pe de alta parte, avem:
1 X

Y
X

1
1 2
cos A =
a + b2 + c2 8R2 =
1
cos2 A =
sin2 A 2 =
2
2
2
8R
i

1 2
1 h 2
2
2
2
=

4Rr

8R

(2R
+
r)
p

2
p
=
8R2
4R2
Y
4R2
p r 2R =

cos A.
p + r + 2R
Deducem de aici c
a p r 2R este pozitiv, nul sau negativ, dup
a cum 4ABC este
ascutitunghic, dreptunghic sau obtuzunghic, ceea ce ncheie justificarea inegalit
atii.
Egalitatea se atinge n cazul triunghiului dreptunghic.
X.80. Aratati ca exista o infinitate de valori n N pentru care numerele 2n + 1
si 3n + 1 sunt patrate perfecte.
Gheorghe Iurea, Iasi
Solutie. Observam ca

2k+1i i
X
2k+1 2k+1

i
3+ 2
=
C2k+1
3
2 = Ak 3 + Bk 2, cu Ak , Bk N,
i=0

2k+1

3 2
= Ak 3 Bk 2.
deoarece 2k + 1 i si i au paritati diferite, iar
Rezulta ca


i2k+1

h
3+ 2
3 2
= 1.
3A2k 2Bk2 = Ak 3 + Bk 2 Ak 3 Bk 2 =

Considernd nk = Bk2 A2k obtinem ca 2nk + 1 = A2k , iar 3nk + 1 = Bk2 , prin
urmare numerele 2n + 1 si 3n + 1 sunt patrate perfecte pentru o infinitate de valori
ale lui n.

Clasa a XI-a
XI.76. Daca A M4 (R), sa se arate ca det (A + t A i) = det (A t A i).
Generalizare.
Dan Popescu, Suceava
Solutie. Dac
a A Mn (R), atunci

1
n
n
t
t
det A A i = (i) det A A = (i) det t A + i A =
i

= (i)n det t t A + i A = (i)n det A + t A i ,

prin urmare det (A + t A i) = det (A t A i) dac


a si numai dac
a n este par. n particular, egalitatea are loc pentru n = 4.
66

XI.77. Fie f : [a, b] R o functie de doua ori derivabila pe [a, b], cu ambele derivate strict pozitive. Pentru [a, b], consideram punctele A (a, f (a)),
B (b, f (b)), C (, yC ) Gf si D (, yD ) AB. Demonstrati ca exista si este unic
0 a+b
2 , b astfel nct f (b) yD = yC f (a).
aeru, Suceava
C
at
alin Tig
b
a
Solutie. Obtinem imediat ca yC = f (), iar yD =
f (a) +
f (b).
ba
ba
Consider
am g : [a, b] R,
b
a
g () = f (b) g (b) yD (yC f (a)) =
f (b) +
f (a) f () .
ba
ba
f (b) f (a)

Functia g este de dou


a ori derivabil
a pe [a, b]. Cum g 0 () =
ba
0
a c
a g este strict descresc
atoare; n plus, g(a) g(b) =
f () < 0, [a, b], rezult
(f (b) f (a))2 < 0. Urmeaza ca exista si este unic 0 (a, b) pentru care g (0 ) =
a + b
a + b
1
0. Faptul ca 0
, b rezulta din aceea ca g
= [f (a) + f (b)]
2
2
2
a + b
f
> 0, ntruct f este strict convexa, iar g (b) = f (a) f (b) < 0, ntruct f
2
este strict cresc
atoare.
XI.78. Pentru x R+ , sa se demonstreze inegalitatile:
1
1
a) ln x + a (1 + ln a), unde a > 0;
x
a
k
b) ax > (1 + x) , unde a > ek , k N , iar {1}.
Gheorghe Costovici, Iasi
1
1
Solutie. a) Fie f : (0, ) R, f (x) = ln x + a (1 + ln a); atunci f 0 (x) =
x
a

1
1
a
a+1 = a+1 (xa a). Avem f 0 (x) < 0 pentru x 0, a1/a si f 0 (x) > 0
x
x

x
pentru x a1/a , + , prin urmare f are un minim n x0 = a1/a , egal cu f (x0 ) = 0.
Deducem ca f (x) 0, cu egalitate cnd x = a1/a .
k
b) Fie g : (0, ) R, g (x) = ax (1 + x) . Dezvolt
am functia g n serie Taylor
ntr-o vecin
atate a originii:
x
x2
xk (k)
xk+1 (k+1)
g (x) = g (0) + g 0 (0) + g 00 (0) + +
(xk ) ,
g (0) +
g
1!
2!
k!
(k + 1)!
k+1

unde 0 < k < 1. Avem g (0) = 0, g (k+1) (xk ) = axk (ln a)


> 0, c
aci ln a > k,
iar g (p) (0) = (ln a)p p k (k 1) (k 2) (k (k p + 1)), p = 1, k. Cum a > ek ,
obtinem ca (ln a)p > k p , prin urmare g (p) (0) > 0, p = 1, k si astfel se obtine g (x) > 0,
x (0, ).
XI.79. Fie (xn )n1 un sir convergent, a carui limita o notam L(xn ). Demonstrati
x n
n
n
= a R+ daca si numai daca exista lim (1 + xn L (xn )) =
ca exista lim
n L (xn )
n
b R+ . Ce legatura este ntre a si b?
D. M. B
atinetu-Giurgiu, Bucuresti
Solutie. Sunt imediate egalit
atile:
x n
1
lim n(xn L(xn ))
lim n(xn L(xn ))
n
lim
= e L(xn ) n
;
lim (1 + xn L(xn ))n = en
.
n L (xn )
n
67

x n
n
n
lim n (xn L (xn )) lim (1 + xn L (xn )) .
n L (xn )
n
n
Se deduce usor ca b = aL(xn ) .
Atunci exist
a lim

XI.80. Fie f : [0, 1] R o functie continua pe [0, 1], derivabila pe


cu
(0, 1),x
< M xe ,
f (0) = 0. Presupunem ca exista M > 0 astfel nct f 0 (x) 1x
f
(x)
x
x (0, 1). Sa se arate ca f este derivabila n origine.
Mihai Cr
aciun, Pascani
Solutie. Conditia din enunt se scrie echivalent
x

xe f (x) + xex f 0 (x) ex f (x)

< M |h0 (x)| < M, x (0, 1) ,

x2

ex f (x)
. Din teorema lui Lagrange,
x
obtinem c
a |h (x) h (y)| < M |x y|, x, y (0, 1), x 6= y. Fie (xn )n0 (0, 1) un
sir convergent la zero; avem c
a |h (xn ) h (xm )| M |xn xm |, m, n N, deci sirul
(h (xn ))n0 este sir Cauchy. Deducem ca (h (xn ))n0 este convergent si fie l limita sa.
Dac
a (yn )n0 este un alt sir convergent la zero, cum |h (xn ) h (yn )| M |xn yn |,
n N, rezult
a c
a lim h (yn ) = lim h (xn ) = l, prin urmare exist
a lim h (x) = l.
unde h : (0, 1) R este definit
a prin h (x) =

x0

f (x)
Urmarind definitia lui h, obtinem existenta lui f (0) = lim
= l, deci f este
x0 x
derivabil
a n origine.
0

Clasa a XII-a
XII.76. Fie functia continua f : [a, b] R si n N . Aratati ca exista c (a, b)
astfel nct
Z b
f (c) + f (a) + f (b) (b a)n
(f (x) + f (a) + f (b)) dx =

.
n1
n
(c a)
a

Dumitru Mihalache, Brlad


Solutie. Vom considera functia : [a, b] R,
Z x
(x a)n
(f (t) + f (a) + f (b)) dt k
(x) =
,
n
a

unde k este o constanta ce urmeaza a fi determinata din conditia (a) = (b).


Rb
n
Cum (a) = 0 rezulta (b) = 0, adica k = a (f (t) + f (a) + f (b)) dt
n.
(b a)
Aplicnd teorema lui Rolle functiei , exist
a c (a, b) astfel nct 0 (c) = 0
f (c) + f (a) + f (b)
n1
= 0. Obtinem k =
. Egalnd
f (c) + f (a) + f (b) k (c a)
n1
(c a)
cele doua expresii ale lui k, rezulta concluzia .
Rb
Not
a. Pentru n = 1 obtinem c
a a f (x) dx = f (c) (b a), prin urmare problema
constituie o generalizare pentru teorema de medie.

R 1/nk
XII.77. Fie k N , fixat. Consideram sirurile an = 1/(n+1)k arcsin nk x dx,
R 1/nk

an
.
n 1 si bn = 1/(n+1)k arctg nk x dx, n 1. Sa se calculeze lim
n bn
Liviu Smarandache si Lucian Tutescu, Craiova
68

Solutie. Conform teoremei de medie, pentru orice n 1, exist


a cn , dn
1
1
,
astfel nct
(n + 1)k nk
1
1

1
1
si bn =
an =

arcsin nk cn
arctg nk dn .
k
k
k
k
n
n
(n + 1)
(n + 1)
k

arcsin n cn
an
= lim
Astfel, lim
. Cu teorema clestelui obtinem c
a lim nk cn =
k
n bn
n arctg (n dn )
n
k
arcsin 1
an
=
= 2.
lim n dn = 1. Prin urmare, lim
n
n bn
arctg 1
XII.78. Daca f : R R este o functie continua si fara puncte fixe, sa se arate
ca nici functiile f f f , n N , nu au puncte fixe.
{z
}
|
n ori
Dorin M
arghidanu, Corabia
Solutie. Consider
am functia g : R R, g (x) = f (x) x, care este continua si nu se anuleaza pe R; deducem ca g pastreaza semn constant pe R. Notnd
f n = f f f , rezult
a c
a pentru orice x R, numerele g (x), g (f (x)), . . . ,
{z
}
|

n ori
g f n1 (n) sunt simultan sau pozitive, sau negative. Cum

f n (x) x = g f n1 (x) + g f n2 (x) + + g (f (x)) + g (x) , x R,


urmeaz
a afirmatia problemei.
XII.79. Fie V spatiu vectorial de dimensiune n peste corpul K, iar u un endomorfism nilpotent al lui V (i.e., exista p N astfel nct up = u
| u {z u} = 0).

n ori

Sa se arate ca un = 0.

Adrian Reisner, Paris


Solutie. Presupunem prin absurd c
a un 6= 0; atunci polinomul minimal al endomorfismului u este X p , cu p > n. Acest fapt este imposibil, ntruct polinomul
minimal al lui u divide polinomul caracteristic al lui u (teorema Hamilton-Cayley),
iar polinomul caracteristic este de grad n.
Not
a. n fapt, autorul a propus spre publicare urm
atorul rezultat, mai general,
a c
arui demonstrare dep
aseste ns
a nivelul unui absolvent de liceu:
Fie V spatiu vectorial de dimensiune n peste corpul K, iar u1 , u2 , . . . , un
endomorfisme nilpotente, care comuta doua cte doua; atunci u1 u2 un = 0.

XII.80. Fie A un inel n care x4 y 4 = (x y) (x + y) x2 + y 2 , x, y A.


a) Daca inelul are unitate, sa se arate ca A este comutativ.
b) Ramne valabil rezultatul de la a) daca inelul A nu este unitar?
Gabriel Dospinescu, Paris si Marian Tetiva, Brlad
Solutie. Conform ipotezei, avem c
a

f (x, y) = (x y) (x + y) x2 + y 2 x4 y 4 =
= x2 y 2 + xyx2 + xy 3 yx3 yxy 2 y 2 x2 = 0, x, y A.
Dupa calcule simple, obtinem apoi:
g (x, y) = f (x + 1, y) f (x, y) = 2xy 2 + 2xyx + xy 2y 2 x 2yx2 yx;
f (x + 2, y) 2f (x + 1, y) + f (x, y) = g (x + 1, y) g (x, y) = 2 (xy yx) ,
69

pentru orice x, y A (unde 2 = 1 + 1, iar 1 este unitatea inelului). Cum expresiile


calculate sunt identic nule datorita ipotezei, obtinem ca g (x, y) = 0 si 2 (xy yx) =
0, x, y A. Atunci

g (x, y) = 2 xy 2 y 2 x + 2 (xy yx) x + xy yx = 0, x, y A


0 + 0 + xy yx = 0, x, y A xy = yx, x, y A.

Absenta elementului unitate anuleaz


avalabilitatea
a cum arat
a
concluziei, dup
0 a b
exemplul inelului matricelor de forma 0 0 c; a, b, c R. Cum produsul
0 0 0
oricaror trei matrice de acest tip este matricea nula, e clar ca egalitatea din enunt
este valabil
a n acest inel, ns
a inelul nu este comutativ.

IMPORTANT
n scopul unei leg
aturi rapide cu redactia revistei, pot fi utilizate urm
atoarele
adrese e-mail: t_birsan@yahoo.com si profgpopa@yahoo.co.uk . Pe
aceasta cale colaboratorii pot purta cu redactia un dialog privitor la materialele trimise acesteia, procurarea numerelor revistei etc. Sugeram colaboratorilor care trimit probleme originale pentru publicare s
a le numeroteze si
s
a-si retin
a o copie xerox a lor pentru a putea purta cu usurinta o discutie
prin e-mail asupra acceptarii/neacceptarii acestora de catre redactia revistei.
La problemele de tip L se primesc solutii de la orice iubitor de matematici
elementare (indiferent de preocupare profesionala sau vrsta ). Fiecare dintre
solutiile acestor probleme - ce sunt publicate n revist
a dup
a un an - va fi
urmata de numele tuturor celor care au rezolvat-o.
Adres
am cu insistenta
amintea ca materialele trimise revistei
rug
s
a nu fie (s
a nu fi fost) trimise si altor publicatii.
Rug
am ca materialele tehnoredactate s
a fie trimise pe adresa redactiei nsotite de fisierele lor (de preferinta n LATEX).
Pentru a facilita comunicarea redactiei cu colaboratorii ei, autorii materialelor sunt rugati s
a indice adresa e-mail.

70

Solutiile problemelor pentru preg


atirea concursurilor
propuse n nr. 1/2007
A. Nivel gimnazial
G116. Aflati toate numerele naturale N de patru cifre nenule distincte cu proprietatea ca diferenta dintre cel mai mare numar obtinut prin permutarea cifrelor lui
N si cel mai mic asemenea numar este tocmai N .
Maria Mihet, Timisoara
Not
a. A se vedea articolul d-lui Titu Zvonaru O problema cu cifrele unui
numar, ap
arut n RecMat 1/2007.
G117. Fie multimea A = {1, 2, 3, . . . , 98}. Aratati ca oricum am alege 50 de
elemente ale lui A, exista doua printre ele avnd suma cub perfect.
Titu Zvonaru, Com
anesti
Solutie. Consideram multimile Ai = {i, 27 i}, i = 1, 2, . . . , 13, Aj = {j + 13,
112 j}, j = 14, 15, . . . , 49, cu proprietatea c
a suma elementelor din fiecare multime
este cub perfect, iar A = A1 A2 A49 . La alegerea a 50 de elemente, vor exista
dou
a din aceeasi multime Ai , conform principiului cutiei, si de aici rezult
a concluzia.
G118. n interiorul unui paralelogram avnd unghiul ascutit de 30 si lungimile
laturilor 17 cm si 59 cm, se considera 2007 puncte. Sa se arate ca putem alege trei
dintre aceste puncte astfel nct aria triunghiului determinat de ele sa fie cel mult
egala cu 14 cm 2 .
Mihai Haivas, Iasi
Solutie. Descompunem paralelogramul n 17 59 = 1003 romburi de latura
1
1, aria fiecaruia fiind 1 1 sin 30 =
2
a
cm2 . Conform principiului cutiei, exist
un romb care sa contina (n interior sau
pe laturi) m
acar 3 dintre cele 2007 puncte. Dac
a E, F , G sunt aceste puncte,
apartinnd rombului M N P Q, fie E 0 si F 0 intersectiile lui EF cu dou
a din laturile
rombului, iar {G} = E 0 G N P (notatiile sunt cele din figur
a). Evident atunci c
a
1
1
2
AEF G AE 0 F 0 G0 AM NP Q = cm .
2
4

s
i
A
=
0 20 + + n 2n | 0 , 1 , . . . , n {1} , iar
G119.
Fie
n

B = m | m 2 Z + 1, |m| 2n+1 1 . Sa se arate ca A = B.


Dorel Mihet, Timisoara
Solutie. Multimea B are 2 2n = 2n+1 elemente. Sa remarcam ca toate numerele
din A sunt impare, A are cel mult 2n+1 elemente (sunt 2n+1 posibilitati de alegere
n
n+1
ale numerelor i ), cel mai mare
1, iar
element din A este 1 + 2 + + 2 = 2
n+1
cel mai mic este 2
1 . Dac
a ar
at
am c
a la combinatii diferite de i corespund
numere diferite, va rezulta ca A are exact 2n+1 elemente, deci A = B.
Fie 0 20 + +n 2n = 0 20 + + n 2n , cu i , i {1, 1}; ramne sa aratam
c
a i = i , i = 1, n. Observ
am c
a semnul lui 0 20 + + n 2n este dat de semnul
lui n (se demonstreaz
a imediat, tinnd seama de faptul c
a 2n > 2n1 + + 1). n
0
n
0
aceste conditii, din 0 2 + + n 2 = 0 2 + + n 2n , rezulta ca n = n .
71

Reducnd termenii cu 2n , obtinem o egalitate similar


a cu n1 si n1 n loc de n
si n , deci n1 = n1 etc.
O alta solutie se poate da folosind unicitatea scrierii unui numar n baza 2.
G120. Rezolvati n N ecuatia x! (y!)2005 = (z!)2007 .
Anca S
tefania Tutescu, elev
a, Craiova
Solutie. Vom ar
ata c
a, dac
a (x, y, z) este solutie, atunci x, y, z {0, 1}. Presupunem prin absurd c
a z 2 si fie p cel mai mare num
ar prim z. Atunci
p2007 | (z!)2007 , de unde p2007 | x! (y!)2005 . Daca p2 nu divide x!, rezulta ca
2005
p2006 | (y!)
, de unde p2 | y!; prin urmare, cel putin unul dintre numerele x!
sau y! se divide cu p2 si, s
a zicem c
a x! are aceast
a proprietate. Deducem c
a n produsul x! apar factorii p si 2p si este cunoscut faptul ca ntre p si 2p mai putem gasi
macar un numar prim q (Postulatul lui Bertrand). Evident ca q!z! si deci p < q z,
ceea ce contrazice maximalitatea lui p.
Ecuatia admite 8 solutii (x, y, z), cu x, y, z {0, 1}.
G121. Daca a, b (0, 3/2), sa se demonstreze inegalitatea
r
1
1
1
1
1 1
+ .
+ +
+
a b a+b+3
a
a+ b
b
Andrei Laurentiu Ciupan, elev, Bucuresti

3
3
Solutie. Din a, b 0,
, rezult
a c
a ab si atunci a + b + 3 a + b +
2
2

a + b . Este cunoscut
a (sau se poate demonstra usor) identitatea
2 ab =

2
1
1
1
1
1
1
+
+
=
; obtinem:
+
x2 y 2 (x + y)2
x y x+y
2

1
1
1 1
1
1 1
1
1
+ +
+ +
2 = a + b a + b .

a b a+b+3
a b
a+ b
De aici, inegalitatea din enunt este imediat
a.
0
G122. Fie G centrul de greutate al 4ABC si G
tia sa pe dreapta BC.
2 proiec
0
2
Sa se arate ca G
/ [BC] daca si numai daca 3a < b c2 .
Temistocle Brsan, Iasi
Solutie. Conditia G0
/ [BC] este echivalenta cu faptul ca 4GBC este ob\ sau GCB,
\ dup
tuzunghic, avnd unghiul obtuz GBC
a cum c < b sau b < c. Avem:

2
2
2
\ obtuz cos GBC
\ < 0 BC +GB GC < 0
GBC
2 BC GB

2
2
2
2
a2 + mb mc < 0 9a2 + 4m2b 4m2c < 0
3
3
2

2
2
2
9a + 2 a + c b 2 a2 + b2 + c2 < 0 3a2 < b2 c2 .
\ este obtuz 3a2 < c2 b2 si urmeaza concluzia.
Analog, GCB
G123. Fie ABC un triunghi echilateral. Sa se arate ca orice punct M din plan
cu proprietatea ca M B = M A + M C poate fi determinat folosind doar echerul. (Un

72

echer poate fi folosit pentru a trasa drepte si unghiuri drepte.)


Nicolae Iv
aschescu, Craiova
Solutie. Folosind teorema lui Ptolemeu, se arata ca
punctele M cu proprietatea din enunt sunt cele de pe arcul
mic AC al cercului circumscris triunghiului (a se vedea si
solutia problemei V II.80). Mijlocul acestui arc se afl
a intersectnd perpendiculara n A pe AB cu perpendiculara
n C pe BC. Celelalte puncte ale arcului se construiesc
proiectnd mijlocul arcului AC pe semidrepte cu originea
\
n B, interioare unghiului ABC.
G124. Fie 4ABC, A0 mijlocul lui [BC], iar P si Q proiectiile lui A0 pe AB,
respectiv AC. Sa se arate ca 4P Q AB + BC + CA.
Adrian Zahariuc, elev, Bac
au
Solutie. Notam cu B 0 si C 0 mijloacele laturilor [AC] si
[AB], iar cu T si S mijloacele segmentelor [A0 C 0 ], respectiv
[A0 B 0 ]. Atunci
P Q P T + T S + SQ =

A0 C 0
B0C 0
A0 B 0
AC
BC
AB
+
+
=
+
+
,
2
2
2
4
4
4
de unde 4P Q AB + BC + CA.
=

Nota autorului. Dup


a calcule laborioase, se poate demonstra c
a problema este
R
ma
.
echivalent
a cu inegalitatea
ha
2r
G125. Fie ABCD un patrat, M (AB), {O} = AC BD, {S} = CM DA,
iar {E} = SO M D. Consideram AA0 (ABC), AA0 = AB, I mijlocul lui [A0 D],
iar {H} = M I A0 E. Sa se arate ca:

2
VA0 ADH
AB
a) M D (A0 AE);
b)
=
.
VMADH
AM
Petru R
aducanu, Iasi
Solutie. a) Fie {L} = SO AB. Aplicnd teorema lui
Menelaus n 4AM D cu transversala E L S si n 4AM C
cu transversala O L S, obtinem
EM SD LA
LA SM OC

= 1;

= 1.
ED SA LM
LM SC OA
SC
SD
CD
EM
si deducem c
ns
a
=
=
a
=
SM
SA
MA
ED
2
AM
, prin urmare AE M D. Avem si c
a AA0
AD2
M D si astfel M D (A0 AE).
b) De la a) obtinem ca A0 E M D. n plus, se
observ
a c
a M D = M A0 si, cum M I este median
a n
0
a c
a H este
4M DA isoscel, atunci M I A0 D. Rezult
ortocentru n 4M DA0 , deci DH A0 M . Avem si
73

A0 M DA (deoarece DA (AM A0 )), prin urmare A0 M (ADH). Deducem c


a

2
2
1
AADH A0 T
AB
A0 A
A0 T
VA0 ADH
= 13
=
.
=
=
VMADH
M
T
AM
AM
A

M
T
3 ADH

B. Nivel liceal
L116. Cercul nscris n 4ABC este tangent laturii BC n punctul D1 , iar cercul
A-exnscris este tangent aceleeasi laturi n punctul D2 . Dreapta AD2 intersecteaza
cercul nscris n punctele S si T . Sa se arate ca 4ST D1 este dreptunghic.
Titu Zvonaru, Com
anesti
Solutie. Fie E1 , E2 punctele de tangenta ale cercului nscris, respectiv A-exnscris cu latura AC. Cu
notatiile uzuale, avem: BD2 = p c, CD2 = p b,
AE1 = p a, AE2 = p, D1 D2 = |b c|. Dac
a
= AD2 , cu relatia lui Stewart obtinem:
a2 = c2 (p b) + b2 (p c) a (p b) (p c) =
2

= p (b c) + ap (p a) .

Mai notam x = AS, y = T D2 , z = ST . Folosind


puterea punctului fata de cerc avem AS AT = AE12 si
D2 S D2 T = D1 D22 , prin urmare
2

x2 + xz = (p a) ,

y 2 + yz = (b c) ,

x + y + z = .

nlocuim z = x y n primele dou


a ecuatii:
x xy = (p a)2 ,

y xy = (p c)2 .

(b c)
si prima ecuatie devine
x
h
i
(b c)2 x
2
2
2
x +
= (pa) ax2 x (p a) (b c) + a (b + c) + a (p a) = 0.
x
(p a)
p (p a)
si x2 =
si corespunzator
Solutiile acestei ecuatii sunt x1 =
p

2
p (b c)
a
obtinem y1 =
, y2 =
. Observ
am c
a
a
p
Acum substituim y =

x2 + y2 > p2 (p ) + a2 > 2 p p2 (p a) > 2 (p a)

ap2 > p (b c)2 + ap (p a) (b c)2 < a (p p + ) (b c)2 < a2 ,


adevarat. Atunci valorile x2 , y2 nu convin si ramne ca

AS
pa
=
.
AD2
p

Dac
a I, Ia sunt centrele cercurilor nscris, respectiv A-exnscris, atunci

AI
=
AIa

pa
AS
AI
AE1
=
=
, de unde IS k Ia D2 . ns
a Ia D2 BC
, prin urmare
AE2
p
AIa
AD2
si deducem c
a c
a punctele S, I, D1 sunt
a IS BC. Avem c
a ID1 BC; rezult
coliniare, adica [SD1 ] este diametru si astfel 4ST D1 este dreptunghic.
74

L117. Fie 4ABC, D (BC), iar C1 , C2 cercurile exnscrise triunghiurilor


ADB si ADC, tangente la BC. Aratati ca o tangenta comuna interioara cercurilor
C1 si C2 trece prin punctul de contact cu BC al cercului A-exnscris.
Neculai Roman, Mircesti, Iasi
Solutie. Dac
a D este punctul de contact cu BC al
cercului A-exnscris 4ABC, atunci AD este tangenta
comuna interioara cautata. Mai mult, n acest caz se
poate ar
ata c
a C1 si C2 sunt tangente exterior.
Consider
am c
a D nu este punctul de contact cu
BC al cercului A-exnscris 4ABC si notam {M1 } =
C1 BC, {M2 } = C2 BC, {N1 } = AD C1 ,
{N2 } = AD C2 , P1 si P2 sunt punctele de contact cu C1 , respectiv C2 , ale tangentei comune interioare cercurilor C1 si C2 (diferita de AD), iar {T } =
1
a DM1 = (AB + BD AD),
P1 P2 BC. Avem c
2
1
DM2 = (AC + CD AD). Distingem dou
a cazuri:
2
I. T (CD); avem c
a T B = BD + T D, T D = T M1 DM1 , T M1 = T P1 =
T P2 + P1 P2 = T M2 + N1 N2 = T M2 + DN2 DN1 = T M2 + DM2 DM1 . Pe de
alta parte, T M1 = DM1 + DM2 T M2 , prin urmare DM2 = T M1 . Apoi
AC AB + DC DB

T D = T M1 DM1 = DM2 DM1 =


2
AC AB + DC DB
AC + BC AB
T B = BD +
=
,
2
2
ceea ce trebuie demonstrat.
II. T (BD); se efecteaz
a un calcul asem
an
ator.

Not
a. Vlad Emanuel, student, Bucuresti, a solutionat problema cu ajutorul
unor calcule elaborate, etalnd tehnici diverse si de mare finete.
L118. Fie M un punct al elipsei E, de focare F si F 0 . Dreptele M F si M F 0
intersecteaza elipsa n A, respectiv A0 . Sa se arate ca, atunci cnd M parcurge E,
dreapta AA0 este mereu tangenta unei curbe fixe, care se cere a fi determinata.
Adrian Reisner, Paris
Solutie. Consideram elipsa
a la axele sale de simetrie; ecuatiile para
E raportat
a 1 t2
2bt
,y=
, t R. Fie M corespunzator valometrice ale elipsei vor fi x =
1 + t2
1 + t2
0
rii t0 a parametrului; vom calcula valorile t1 si t1 ce corespund punctelor A, respectiv
x
y
A0 . Ecuatia dreptei M A este (1 t0 t1 ) + (t0 + t1 ) (1 + t0 t1 ) = 0 si, exprimnd
a
b
c
faptul ca M A trece prin punctul F (c, 0), obtinem ca (1 t0 t1 ) (1 + t0 t1 ) = 0, de
a
ca
c+a
unde se deduce c
a t1 =
. Analog g
asim t01 =
, deci ecuatia dreptei
(c + a) t0
(c a) t0
0
AA este

2 a2 + c2 t0

2
0 x
y t20 + 1 = 0.
t 1
AA :
a 0
b3
75

x
x

2 a2 + c2 y
Scriem aceast
a ecuatie sub forma
t0
1 t20
+ 1 = 0 si
3
a
b
a
impunem ca ea sa admita solutie dubla n t0 , adica = 0. Dupa calcule, obtinem

2
2
a + c2 y 2
x2
elipsa E 0 : 2 +
1 = 0 si aceasta va fi curba fixa cautata; ea poarta
a
b6
numele de curba nfasuratoare a familiei de drepte AA0 .
L119. Fie n N si a.b, c R+ cu ab + bc + ca = 3. Sa se arate ca an+3 + bn+3 +
n+3
c
+ 2abc (an + bn + cn ) 9.
Titu Zvonaru, Com
anesti si Bogdan Ionita
, Bucuresti
Solutie. Inegalitatea lui Schur
an+1 (a b) (a c) + bn+1 (b c) (b a) + cn+1 (c a) (c b) 0
devine, tinnd seama si de ipotez
a,

n+3
n+3
n+3
n
+b
+c
+ 2abc (a + bn + cn ) (ab + bc + ca) an+1 + bn+1 + cn+1
a

an+3 + bn+3 + cn+3 + 2abc (an + bn + cn ) 3 an+1 + bn+1 + cn+1 .

Problema este rezolvata daca dovedim ca, n ipoteza ab + bc + ca = 3, avem ap + bp +


cp 3, p N. Cu inegalitatea M A M G, obtinem

ap + bp + 1 + + 1 pab, bp + cp + 1 + + 1 pbc, cp + ap + 1 + + 1 pac.


| {z }
| {z }
| {z }
p2

p2

p2

Prin adunare, rezulta ca 2 (a + b + c ) + 3p 6 p (ab + bc + ca), adica ap + bp +


cp 3.
L120. Pentru a1 , a2 , . . . , an reale pozitive, sa se demonstreze inegalitatea
(n1)2
(n1)2 (n1)2
+ a2 a2n1
+ + an a2n1
a2
an
a1 a2n1
a1
2
3
1
+
+ +

.
2
2
a2
a3
a1
a1 a2 a2n
Marian Tetiva, Brlad
Solutie. Folosind inegalitatea M A M G pentru (n 1)2 numere, obtinem
(n1)2
(n1)2

(n1)2
(n1)2
a1
a2
an2
an1
(2n3)
+ (2n5)
+ + 3
+

a2
a3
an1
an
v
u"
2n5
#(n1)2
2n3
u
2
an1
a1
a2
a2n1 an
2 (n1)t
(n 1)

= (n 1)2 2 1 2
.
a2
a3
an
a1 a2 a2n

Scriem nc
a n 1 inegalit
ati de acelasi tip (obtinute prin permut
ari circulare), le
sum
am si rezult
a inegalitatea dorit
a.

1
L121. Fie n N dat. Sa se arate ca exista ctiva termeni ai sirului
m3 mn+1
1
a caror suma este mai mare dect
.
(n + 1) (2n + 1)
Dumitru Mihalache si Marian Tetiva, Brlad
1
1
a a seriei armonice de exponent
Solutie. Not
am Sn = 1+ 3 + + 3 suma partial
2
n
3, deci supraunitar. Rezult
a c
a sirul (Sn )n1 este convergent; fie S limita sa. Pentru
1
a rezolva problema, ar trebui sa dovedim ca lim (Sn+k Sn ) >
,
k
(n + 1) (2n + 1)
76

1
, n 1. Evident, pentru asta ar fi suficient s
a
(n + 1) (2n + 1)
1
dovedim ca sirul (xn )n1 , xn = Sn +
, n 1 (a carui limita este
(n + 1) (2n + 1)
tot S) este strict cresc
ator, fapt care rezult
a dup
a un calcul simplu:
deci c
a S > Sn +

xn+1 xn =
=

1
(n + 1)

1
1

=
(n + 2) (2n + 3) (n + 1) (2n + 1)

3n2 + 5n + 1
3

(n + 1) (n + 2) (2n + 1) (2n + 3)

n N .

Not
a. Principial, aceeasi solutie a dat Vlad Emanuel, student, Bucuresti.
L122. La un campionat de fotbal participa 2n echipe, astfel nct dintre oricare
doua se poate dinainte indica echipa mai buna. n prima etapa, echipele se mpart
aleator n perechi si disputa cte un meci, echipa mai buna trecnd n etapa urmatoare.
Procedeul se repeta pna la finala.
a) Care este probabilitatea ca a doua echipa ca valoare sa iasa vicecampioana?
b) Daca se disputa si o finala mica, ce probabilitate este ca, n plus, cea de-a treia
echipa ca valoare sa se claseze pe locul 3?
Irina Mustata
a, Bremen
, student
Solutie. Numerot
am echipele de la 1 la 2n , 1 fiind cea mai bun
a. Fie P2n
num
arul modalit
atilor distincte (ignornd permut
arile) de a mp
arti 2n echipe n n
2
perechi. Din 2n numere, o pereche se poate alege n C2n
moduri, urm
atoarea pereche
2
n C2n2 moduri, etc. gnornd permutarile implicate, obtinem ca
P2n =

1 2 2
(2n)!
C22 = n = (2n 1)!!
C C
n! 2n 2n2
2 n!

a) n prima etap
a, num
arul cazurilor posibile este P2n . Cazurile nefavorabile
sunt cele n care este selectat
a perechea (1, 2), n num
ar de P2n 2 ; num
arul cazurilor
favorabile va fi P2n P2n 2 . Continund rationamentul, probabilitatea evenimentului
cerut este
n
n

Q
Q
(P2i P2i 2 )
2i 3 !! 2 2i1 1
2n1
i=2
P = i=2 Q
=
=
.
n
n
Q
2n 1
P2i
(2i 1)!!
i=1

i=2

b) n semifinale avem permise doar mperecherile (1, 3), (2, 4) sau (1, 4), (2, 3),
iar ntr-o etap
a anterioar
a sunt interzise meciurile (1, 2), (1, 3) si (2, 3). Num
arul de
repartiz
ari convenabile ntr-o etap
a cu 2i echipe este P2i 3P2i 2 si atunci probabilitatea ceruta va fi
n
n

Q
Q
2
2
(P2i 3P2i 2 )
2i 3 !! 4 2i2 1
22n3
i=3
=
P 0 = i=3 Q
=
.
n
n
Q
(2n 1) (2n1 1)
P2i
(2i 1)!!
i=2

i=2

Not
a. Principial, aceeasi solutie a dat Vlad Emanuel, student, Bucuresti.
77

L123. Pe o tabla 8 9 se aseaza dreptunghiuri 3 1 si "figuri"


de forma unui dreptunghi 1 3 caruia i lipseste patratul median (ca
n desenul alaturat). "Figurile" si dreptunghiurile nu se pot roti si nu
au puncte interioare comune. Sa se arate ca exista o multime S de 18
patrate 1 1 astfel nct, daca pe tabla ramn 2 patrate neacoperite de
dreptunghiuri sau "figuri", atunci cele doua patrate sunt obligatoriu din S.
Gabriel Dospinescu, student, Paris
Solutie. Numerot
am p
atratele tablei cu (1, 1), (1, 2), . . . , (8, 9) si punem n
p
atratul (k, j) num
arul k ij , unde si i sunt r
ad
acinile primitive de ordin 3, respectiv
4 ale unitatii. Cum 2 + + 1 = 0 si i2 = 1, suma numerelor din orice
P dreptunghi
sau "figura" va fi 0. Suma tuturor numerelor de pe tabla este
k ij =
P
8

k=1

P
8

j=1

1k8,1j9

= i. Sa presupunem ca au ramas patratele (a1 , b1 ) si (a2 , b2 );

scriind n doua moduri suma numerelor de pe tabla, obtinem ca a1 ib1 + a2 ib2 = i.


Not
am z1 = a1 ib1 1 , z2 = a2 ib2 1 si vom avea c
a |z1 | = |z2 | = 1, iar z1 + z2 = 1.
1
1
+
= 1 si de aici z13 = z23 = 1. Astfel, i3(b1 1) = i3(b2 1) = 1,
Deducem c
a
z1
z2
adic
a b1 1 (mod 4) si b2 1 (mod 4). Cum a1 + a2 = 1, rezult
a c
a a1 si a2 dau
resturile 1 si 2 la mpartirea cu 3. Vom considera S ca multimea patratelor de la
intersectiile liniilor 1, 2, 4, 5, 7, 8 cu coloanele 1, 5, 9 si rezolvarea este ncheiata.
Fie n N fixat. Determinati matricele A Mn (C) pentru care

L124.
t
A A = In , iar A2007 + A + In = On (cu am notat operatia de conjugare).
Vlad Emanuel, elev, Sibiu
Solutie. Demonstram nti doua rezultate ajutatoare:

Lema 1. Daca t A A = In , atunci toate valorile proprii ale matricei A sunt
de modul 1.
Demonstratie. Pentru X = (xi )n1 , Y = (yi )n1 definim un produs scalar
n
P
xi yi (proprietatile produsului scalar se verifica imediat).
complex prin hX, Y i =
i=1

Daca A Mn (C), avem


n X
n
n
n
n

X
X
X
X


hAX, Y i =
aij xj yi =
aij xj yi =
xj
aij yi = X, t A Y .
i=1

j=1

i,j=1

j=1

i=1

Fie valoare proprie a lui A, deci AX = X, cu X 6= On,1 . Atunci

hAX, AXi = X, t A AX = hX, Xi hX, Xi = hX, Xi


hX, Xi = hX, Xi = 1 || = 1.

.
Lema 2. Fie x C cu |x| = 1 si xn + x + 1 = 0; atunci n 2 .. 3.
Demonstratie. Fie x = a + bi, cu a, b R, a2 + b2 = 1. Avem:

1
|1 + x| = |xn | = |xn | = 1 (a + 1)2 + b2 = 1 a2 = (a + 1)2 a =
2

3
si apoi g
. n ambele cazuri x3 = 1, iar 1 + x + x2 = 0, prin urmare
asim c
ab=
2
78

.
xn x2 = 0, de unde xn2 = 1, adic
a n 2 .. 3.
Revenim la problem
a. Fie o valoare proprie pentru A; atunci matricea
A2007 +A

2007
2007
va avea ca valoare proprie pe
+. nsa det A
+ A (1) In = det On = 0,
deci 1 este valoare proprie pentru A2007 + A, prin urmare exist
a C astfel nct
2007 + = 1. Din Lema 1 deducem c
a || = 1, iar din Lema 2 rezult
a apoi c
a
..
2007 2 = 2005 . 3, fals. n concluzie, nu exista matrice A cu proprietatile dorite.

L125. Fie f : R R o functie periodica si lipschitziana (exista L > 0 pentru


care |f (x) f (y)| L |x y|, x, y R), iar (xn )n1 un sir strict crescator, cu
lim xn = + si lim (xn+1 xn ) = 0. Sa se arate ca multimea punctelor limita
n

ale sirului (f (xn ))n1 coincide cu Im f .

Paul Georgescu si Gabriel Popa, Iasi


Solutie. Fie Im f ; din periodicitatea functiei, exista > 0 astfel nct
= f (). Fie T > 0 o perioad
a a lui f . Not
am n = xn xn1 , n 2; sirul
(n )n2 are limita 0. Pentru k N oarecare, not
am cu nk primul indice n pentru
care xn + kT ; existenta si unicitatea lui nk sunt asigurate de faptul ca (xn )n1
este strict crescator, cu limita +. Cum nk este primul indice cu proprietatea ceruta,
atunci xnk + kT + nk . Astfel, xnk Ik , unde Ik = [ + kT, + kT + nk ] este
interval de lungime nk .
Din lipschitzianitatea functiei, deducem ca f (xnk ) apartine unui interval Jk =
[uk , vk ]: f este continua, deci transporta compactul Ik n compactul Jk , unde uk =
inf {f (x) | x Ik }, iar vk = sup {f (x) | x Ik }. n plus, lungimea lui Jk nu dep
aseste L nk , iar Jk contine = f (). Pentru k , aplicnd criteriul clestelui,
obtinem ca f (xnk ) , ceea ce trebuia demonstrat.
Not
a. Solutie corect
a a dat Vlad Emanuel, student, Bucuresti.

Diplom
a de excelenta

acordat
a de c
atre Ministerul Educatiei, Cercet
arii si Tineretului elevului

MARIUS TIBA
Pentru rezultatele deosebite obtinute la olimpiadele internationale desfasurate n
anul scolar 2006 - 2007, elevul Marius Tiba, acum n cl. a IX-a, C. N. "C. Negruzzi"
din Iasi, a fost distins cu o diplom
a de excelen
ta
atre M. E. C. T.
de c
Principalele rezultate obtinute n perioada mentionata sunt: medalia de argint
la Balcaniada de Matematica - juniori (Sofia, iunie 2007), medalia de aur la
Olimpiada Nationala de Matematica (aprilie 2007), medalia de aur la testul de
tip O.I.M. (Vlcea, aprilie 2007), diploma "Marele merit" pentru punctaj maxim
obtinut la Concursul "Cristian Calude" (Galati, noiembrie 2006), premiul I la Concursul "Fl. T. Cmpan", etapa judeteana (Iasi, februarie 2007), diploma pentru merite
deosebite n colaborarea cu revista "Recreatii Matematice".
S
a mai spunem c
a, la sfrsitul anului scolar 2006-2007, ca elev n cl. a VIII-a, a
primit premiul I pentru rezultate la nvatatura si purtare.
Redactia revistei "Recreatii Matematice" felicita calduros pe talentatul elev
Marius Tiba si i adreseaza pe acesta cale urari de sanatate deplina si de succese,
n egala masura, n anii ce urmeaza.
79

Probleme propuse1
Clasele primare
P.144. Elevii clasei I intra n clasa n rnd cte unul. Cti elevi sunt n clasa, daca
Matei este al 12-lea cnd se numara ncepnd din fata si al 16-lea cnd se numara
ncepnd din spate.
(Clasa I-a)
nv. Eleva Porfir, Iasi
P.145. Un fluture zboar
a din floarea 1 n floarea 3,
apoi din aceasta n floarea 5 si asa mai departe (figura 1).
Dup
a cte zboruri ajunge n floarea de pe care a plecat?
(Clasa I-a)
Evelina Zaporojanu, elev
a, Iasi
P.146. Dup
a ce fratele meu mi-a dat un sfert din
merele sale, le-am amestecat cu cele 6 ale mele si pe acestea le-am asezat pe doua farfurii cu cte 5 mere fiecare.
Cte mere are fratele meu?
(Clasa a II-a)
Inst. Elena Nuta
, Iasi

Figura 1

P.147. Dac
a numerele ar fi puse corect n cele trei
cercuri, atunci am avea aceeasi suma a celor aflate n
fiecare dintre cercuri (figura 2). n cte moduri pot fi
asezate corect aceste numere?
(Clasa a II-a)
C
at
alina Istrate, elev
a, Iasi
Figura 2
P.148. Aflati valoarea a stiind c
a 10099 : 9998 : 9897 : 97 a : a = 11.
(Clasa a III-a)
Mariana Nastasia, elev
a, Iasi
P.149. Irina i spune Mioarei:
D
a-mi 2 lei ca s
a am si eu ct tine!
Mioara i r
aspunde:
Da-mi tu 2 lei ca eu sa am o suma de 2 ori mai mare dect suma ce-ti ramne
tie!
Ce sum
a a avut la nceput fiecare fat
a?
(Clasa a III-a)

Inst. Maria Racu, Iasi

P.150. Romanul Harry Potter are 7 volume. S


tiind c
a fiecare volum, ncepnd cu
al doilea, are cu 144 pagini mai putin dect dublul numarului de pagini al volumului
precedent, iar al treilea volum are 176 de pagini, aflati cte pagini are ntregul roman.
(Clasa a III-a)
Robert Vicol, elev, Iasi
P.151. Descoperiti regula de formare a sirului 1, 3, 6, 10, 15, 21, . . . si scrieti
numarul de pe locul 2008.
(Clasa a IV-a)
Petru Asaftei, Iasi
P.152. n sirul de patrate egale, fiecare patrat este mp
artit n p
atrate mai mici, dup
ao
anumit
a regul
a.
a) Aratati ca nu exista n acest sir un patrat
mpartit n 23 patrate mai mici;
1

Se primesc solutii pn
a la data de 31 decembrie 2008.

80

b) Ar
atati c
a exist
a n sir un p
atrat mp
artit n 2008 p
atrate mai mici!
(Clasa a IV-a)
Ana T
ab
acaru, elev
a, Iasi
P.153. O veverita transport
a niste alune la scorbura sa n 6 ore, iar alta face
acelasi lucru n 3 ore. n cte ore cele dou
a veverite ar transporta alunele mpreun
a?
(Clasa a IV-a)
Alexandru-Dumitru Chiriac, elev, Iasi

Clasa a V-a
V.88. O secventa de numere este formata din multipli consecutivi ai lui 4, astfel
nct suma dintre primul si ultimul num
ar este 280, iar suma ultimelor dou
a numere
este 508. Ar
atati c
a media aritmetic
a a tuturor numerelor este termen al secventei
considerate.
Mirela Marin, Iasi
V.89. Determinati cifrele x, y, z pentru care xy 2 + xz 2 = 168x.
Ioan S
ac
aleanu, Hrl
au
V.90. Fie E (n) = 3n + 5n , n N. Aflati ultimele dou
a cifre ale numerelor E (10)
si E (2008).
Mihaela Buc
ataru, Iasi
V.91. S
a se arate c
a 61n , n N , se poate scrie att ca sum
a, ct si ca diferenta
de doua patrate perfecte nenule.
Alexandru Negrescu, student, Iasi
2
2
2
2
3
4
172
V.92. Demonstrati ca
+
+
+ +
> 171.
1
2
3
16
Petru Asaftei, Iasi
V.93. Fie A = {2, 3, 4, . . . , 50, 52, 53, 100}. Folosind fiecare element al lui A cte
o singura data, fie ca numarator, fie ca numitor, se scriu 49 de fractii. Demonstrati
ca macar una dintre aceste fractii este reductibila.
Gabriel Popa, Iasi
V.94. Fie A multimea acelor numere naturale cel mult egale cu 2008, care se divid
cu 2, dar nu se divid cu 6. Dac
a scriem elementele lui A n ordine descresc
atoare,
care este al 322-lea num
ar?
Enache P
atrascu, Focsani

Clasa a VI-a
VI.88. Fie a, b, c, d numere rationale pozitive astfel nct
c+1
a+x
c+x
. S
a se arate c
a
=
, x Q+ .
d+1
b+x
d+x

a
c
a+1
si
=
=
b
d
b+1

Claudiu-
Stefan Popa, Iasi
2007

VI.89. Ar
atati c
a num
arul N = 1

2007

+2

+ + 20082007 se divide cu 2009.


C
at
alin Calistru, Iasi

VI.90. Sa se determine numerele naturale cu proprietatea ca att ele ct si


r
asturnatele lor se scriu ca produs de doi factori primi, fiecare factor avnd dou
a
cifre si fiind r
asturnatul celuilalt.
Temistocle Brsan, Iasi
VI.91. Consider
am numerele scrise n baza 8: a1 = 0, 0 (4)(8) ; a2 = 0, 0 (04)(8) ;
81

. . . ; an = 0, 0(00
. . . 0} 4)(8) . S
a se arate c
a num
arul N =
| {z
n1

divizibil cu 14(10) .

1
1
1
+
+ +
este
a1
a2
an
Vasile Chiriac, Bac
au

VI.92. De o parte si de alta a unei drepte AB se considera punctele M si N


\
\
astfel nct 4ABM 4ABN , m(M
AN ) + m(M
BN ) = 180 , iar [AB] [M N ] = .
S
a se arate c
a B este ortocentrul 4AM N .
Petru Asaftei, Iasi
b
b
VI.93. Fie ABCD un patrulater convex cu m(A) = m(B) = 80 si AB = CD =
\) = 20 .
DA si astfel nct exista F (BC) pentru care m(BAF
a) Demonstrati c
a 4AF D este echilateral.
b si D.
b
b) Determinati m
asurile unghiurilor C
Cristian Laz
ar, Iasi

VI.94. Un joc are trei beculete. Primul se aprinde la fiecare dou


a secunde. Al
doilea se aprinde prima dat
a la o secund
a dup
a aprinderea primului, apoi la fiecare
trei secunde. Al treilea se aprinde prima dat
a la a dou
a aprindere a primului, apoi la
fiecare 5 secunde. n primele 10 minute de functionare, de cte ori cele trei beculete
sunt aprinse simultan?
Gabriel Popa, Iasi

Clasa a VII-a
VII.88. Fie x, y, z numere reale distincte, iar a = (xy) (yz), b = (yz) (zx),
c = (zx) (xy). Sa se arate ca exact doua dintre numerele a, b, c sunt negative,
iar al treilea este pozitiv.
Ovidiu Pop, Satu Mare
p
VII.89. Determinati cifrele x, y, z pentru care 14xyzx5 Q.
Damian Marinescu, Trgoviste
VII.90. Rezolvati n numere ntregi ecuatia 4x = 5y + 4.

Ion Visan, Craiova


1
1
1
VII.91. Fie a N , a 98, iar n =
+
+ +
.
a (a + 1) (a + 1) (a + 2)
98 99
Demonstrati ca n nu poate fi patratul unui numar rational.
Gheorghe Iurea, Iasi
VII.92. n trapezul ABCD cu baza mare [CD], diagonala BD este bisectoarea
\ Perpendiculara n B pe diagonala BD intersecteaz
unghiului ABC.
a dreapta AD n
E. S
a se arate c
a dreapta CE trece prin mijlocul laturii [AB].
Dan Nedeianu, Dr. Tr. Severin
VII.93. Pe latura [AB] a triunghiului ABC se considera punctul M si notam
m = AM , n = BM . Paralela prin M la AC taie BC n N , iar paralela prin N la AB
m
taie AC n P . Fie S1 = ABMN , S2 = ACN P , S = AABC , iar x = . Sa se exprime
n
S1 + S2
raportul
n functie de x si s
a se afle x pentru care acest raport este minim.
S
Adrian Corduneanu, Iasi
VII.94. Determinati poligoanele regulate care au proprietatea ca oricare trei
82

vrfuri ale lor determin


a un triunghi isoscel.
Gheorghe Iurea, Iasi

Clasa a VIII-a
VIII.88. Fie A = {1, 3, 5, . . . , 2n 1}, iar S1 si S2 reprezinta suma elementelor
lui A, respectiv suma patratelor elementelor lui A. Sa se determine n N pentru
care S2 3 |A| S1 .
Laurentiu Modan, Bucuresti

4n2 + 2n + 1
VIII.89. Demonstrati ca n2 + 1 + n2 + 2 + + n2 + 2n <
,
2
n N .
Lucian Tutescu, Craiova

VIII.90. Demonstrati ca multimea A = x | x = 276n+2 + 312n+5 + 1, n N


nu contine numere prime.
Damian Marinescu, Trgoviste
VIII.91. Se considera functia f : {1, 2, . . . , 2008} N care asociaza unui element
n al domeniului, numarul divizorilor sai naturali.
a) Determinati n pentru care f (n) = 7.
b) Aflati valoarea maxim
a a functiei.
c) Daca f (n) + f (m) + f (p) = 33, aratati ca macar unul dintre numerele n, m
sau p este patrat perfect.
Monica Nedelcu, Iasi
VIII.92. S
a se arate c
a pentru orice num
ar ntreg impar n, exist
a numerele
naturale a si b astfel nct a (a + 2n) = b (b + 2n).
Constantin Apostol, Rm. S
arat
VIII.93. Fie ABCDA0 B 0 C 0 D0 un paralelipiped oarecare si O, O0 punctele de
intersectie a diagonalelor bazelor. Se noteaz
a cu GA si GA0 centrele de greutate ale
4BCD si, respectiv, 4B 0 C 0 D0 si cu A1 mijlocul segmentului [GA GA0 ]. Notatiile
GB , GB 0 si B1 etc. se introduc n mod similar. Aratati ca dreptele AA1 , BB1 , CC1 ,
DD1 sunt concurente ntr-un punct P situat pe OO0 si precizati pozitia lui P pe
[OO0 ].
Temistocle Brsan, Iasi
a; not
am cu P poligonul A1 A2 . . . An
VIII.94.
oa regulat
n Fie V A1 A2 . . . An o piramid
si fie U = m(V M, \
a max U < 2 min U .
(A1 A2 A3 )) | M P . Demonstrati c
Claudiu-
Stefan Popa, Iasi

Clasa a IX-a
IX.86. Fie O mijlocul ipotenuzei [BC] a triunghiului dreptunghic ABC, r raza
cercului nscris, iar R1 si R2 razele cercurilor cirscumscise triunghiurilor AOB, re
a2
.
spectiv AOC. S
a se demonstreze c
a R1 R2
2a + 4r
D. M. B
atinetu-Giurgiu, Bucuresti
IX.87. Demonstrati c
a ntr-un triunghi ascutitunghic, cu notatiile uzuale, are
loc inegalitatea
83

b
c
3
a
+ 4
+ 4
<
.
4
4
4
+c
c +a
a +b
4Rrp
Gheorghe Molea, Curtea de Arges
.
IX.88. Demonstrati ca 15 25n + 32 n2 + 120n 15 .. 128, n N.
Lucian Tutescu, Craiova

IX.89. S
a se arate c
a pentru orice n N are loc inegalitatea
!

1
1
1
1
1
1
1
1+
+ 2 + +
>
+
+ +
.
2
2
5
n+1 n+2
3n + 1
2 2 2
(3n 1)
b4

Titu Zvonaru, Com


anesti
a
IX.90. S
a se determine toate sirurile de numere reale (an )n0 cu proprietatea c
an+m + anm = a3n + n, n, m N.
I. V. Maftei, Bucuresti si Mihai Haivas, Iasi

Clasa a X-a

x
x 3
= 3 lg lg .
y
3 y
A. V. Mihai, Bucuresti
X.87. Fie A N si f : A A o functie injectiva; notam fn = f f . . . f .
{z
}
|

X.86. Aflati x, y R+ pentru care lg2

n funct ii

Determinati f , stiind c
a exist
a p, q N numere prime ntre ele astfel nct fp (x) +
fq (x) = 2x, x A.
Romeo Ilie, Brasov
X.88. Fie ABCD un paralelogram, iar M si N mijloacele laturilor (BC), respectiv (CD). Daca AM = BN si AM BN , aratati ca ABCD este patrat.
Gheorghe Iurea, Iasi
X.89. n planul complex se consider
a punctele A (3i), B (4), iar M este un punct
variabil de modul 1.
a) Determinati locul geometric al punctului N cu proprietatea ca triunghiurile
AOB si AM N sunt asemenea si la fel orientate.
b) G
asiti punctele N1 , N2 ale locului ce se plaseaz
a pe segmentele [BA], respectiv
[BO], precum si punctelor M1 , M2 din care provin.
Dan Brnzei, Iasi
X.90. Fie X1 , X2 , . . . , Xn variabile aleatoare independente, fiecare dintre ele
lund valorile 1 si 1 cu probabilit
atile p, respectiv q. Consider
am Y = X1 + X2 +
+ Xn .
a) Sa se calculeze media si dispersia lui Y .
b) S
a se precizeze care este valoarea luat
a de Y cu probabilitate maxim
a.
Petru Minut, Iasi

Clasa a XI-a

XI.86. Fie n 2N si A Mm (R); ar


atati c
a numerele det An+1 Im si
det (A Im ) au acelasi semn.
Romanta Ghita
si Ioan Ghita
, Blaj
84

XI.87. Studiati convergenta sirurilor (an )n1 si (bn )n1 , unde

2008 + cos n
2009 + cos n

, bn =
, n N .
an =
2008 + cos n + 1
2008 + cos n + 1
Liviu Smarandache, Craiova

XI.88. Fie sirul (xn )n1 definit prin: x1 0,
, xn+1 = 2 tg xn+1 xn ,
2
n N . Studiati existenta limitelor lim xn si lim nxn .
n
n
Dan Popescu, Suceava

1
1
1
+

+
.
XI.89. Calculati lim
+
n 12 sin 1
22 sin 12
n2 sin n1
Silviu Boga, Iasi
XI.90. Exista functii polinomiale p : R R care sa aiba exact n puncte fixe
distincte a1 , a2 , . . . , an R si astfel nct pentru fiecare 1 j n, ecuatia p (x) = aj
s
a aib
a solutie real
a unic
a?
Marian Tetiva, Brlad

Clasa a XII-a
XII.86. Fie c R , iar f, g : [a, b] R+ functii continue astfel nct f (a+bx) =
g (x), x [a, b]. S
a se determine y [a, b] pentru care
Z a+by
Z b
g(x)
f (x)
[f (x)]
[g (x)]
dx
=
c
dx.
g(x)
a [f (x)]
y
+ [g (x)]f (x)
[f (x)]g(x) + [g (x)]f (x)
D. M. B
atinetu-Giurgiu, Bucuresti
XII.87. a) Fie f : [0, a] R derivabila cu derivata continua, astfel nct f (a) =
a
2
R
a3 Ra 0
2
0. S
a se arate c
a
f (t) dt

[f (t)] dt. Pentru ce functii se realizeaz


a
3 0
0
egalitatea?
b) Fie f : [0, a] R de dou
a ori derivabil
a cu f 00 continu
a, astfel nct f (a) =
a
2
5 Ra
R
a
2
f 0 (a) = 0. S
a se arate c
a
f (t) dt

(f 00 (t)) dt. Pentru ce functii se


20 0
0
realizeaz
a egalitatea?
Adrian Corduneanu, Iasi
XII.88. S
a se arate c
a sirul (an )n1 este convergent si s
a se afle limita sa, unde
n k 2 n2
1 P
k+n
ln
an =
, n N .
n k=1 n2
k
Laurentiu Modan, Bucuresti

XII.89. Sa se arate ca polinomul f = 200X 5 + 196X 4 49X 3 + 35X 2 + 14X + 62


este reductibil peste Q.
Mihai Haivas, Iasi
XII.90. Fie H corpul cuaternionilor, iar i, j, k unit
atile cuaternionice (i2 = j 2 =
k = 1, ij = ji = k, jk = kj = i, ki = ik = j). Definim
(
)

b+c 3
b 3 c + 2d 3
3b c 3 2d
K= a+
i+
j+
k | a, b, c, d R ,
2
4
4
2

85

pe care consider
am operatiile uzuale ntre cuaternioni. S
a se arate c
a n acest mod
obtinem un corp necomutativ, izomorf cu corpul cuaternionilor.
Dumitru Mihalache, Brlad

Probleme pentru preg


atirea concursurilor
A. Nivel gimnazial
G136. Determinati numerele reale x, y, z, pentru care
2x + 3 2y + 2z = 2x + 3 2y+2 + 2z+2 = 9.

Andrei Nedelcu, Iasi

2 a+ b c

G137. Fie a, b, c Q+ si =
. Sa se exprime n functie de a, b,
a+ b+ c

a b+ c

c si numarul real =
.
2 a+ b+ c
I. V. Maftei, Bucuresti si Mihai Haivas, Iasi
G138. a) Numerele reale pozitive a, b, c sunt astfel nct 4abc = a + b + c + 1.
b2 + c2 c2 + a2 b2 + a2
Sa se arate ca
+
+
2 (ab + bc + ca).
a
b
c
b2 + c2 c2 + a2 b2 + a2
+
+

b) Numerele reale pozitive a, b, c sunt astfel nct


a
b
c
2 (ab + bc + ca). Sa se arate ca a + b + c + 1 4abc.
Andrei Laurentiu Ciupan, elev, Bucuresti
G139. Denisa sccrie pe tabla numerele 1, 2, 3, . . . , 2008. Ea alege doua numere,
le sterge de pe tabl
a si scrie n loc modulul diferentei lor, repetnd aceast
a operatie
pn
a cnd pe tabl
a r
amne un singur num
ar. Poate proceda Denisa n asa fel nct
numarul ramas sa fie 2007? Dar 2008?
Julieta Grigoras, Iasi
G140. Un poligon cu n laturi este mpartit n n2 triunghiuri cu ajutorul a n3
diagonale ale sale care nu se intersecteaz
a n puncte interioare (o astfel de mp
artire
se numeste triangulatie a poligonului). Not
am cu T0 num
arul triunghiurilor ale c
aror
laturi sunt toate diagonale ale poligonului si cu T2 numarul triunghiurilor care au
cte doua laturi care sunt laturi si pentru poligon, iar a treia latura diagonala a
poligonului. S
a se arate c
a T2 = T0 + 2.
Marian Tetiva, Brlad
G141. Se consider
a o retea de drepte care formeaz
a prin intersectii p
atrate
congruente. Marcam 2n + 1 vrfuri ale unor astfel de patrate, n 2, astfel nct
orice dreapta din retea sa contina cel mult un punct marcat. Sa se arate ca exista
m
acar dou
a puncte marcate care sunt separate att pe orizontal
a, ct si pe vertical
a,
de cte un num
ar impar de drepte ale retelei.
Petru Asaftei, Iasi
G142. Spunem c
a vrful A al triunghiului ABC are proprietatea (P ) dac
a AX <
BC, X (BC). Sa se arate ca daca fiecare vrf al 4ABC are proprietatea (P ),
86

atunci triunghiul este echilateral.


Doru Buzac, Iasi
G143. Consider
am triunghiul ABC, iar D, D0 sunt puncte pe dreapta BC astfel
\
\
\0 ACB.
\ Bisectoarele interioare ale unghiurilor BAD
\
nct CAD ABC, iar BAD
\0 taie dreapta BC n E, respectiv F . S
si CAD
a se arate c
a cercul circumscris 4AEF
si cercul nscris n 4ABC sunt concentrice.
Neculai Roman, Mircesti (Iasi)
\ +
G144. Fie ABCD un patrulater cu AB = BC. S
a se arate c
a m(BAD)

2
2
2
2
2
2
\

m(BCD) = 90 daca si numai daca AB CD + AD BC = AC BD .


Ioan S
ac
aleanu, Hrl
au
G145. Se considera triunghiul isoscel ABC cu AB = AC, iar pe arcul deschis
BC care nu-l contine pe A al cercului circumscris triunghiului se ia un punct M . S
a
se arate ca

AB AC
.
MB MC < MA < MB MC +
MB MC
Gheorghe Costovici, Iasi

B. Nivel liceal
L136. Fie A, B, C trei puncte pe sfera S de centru O, iar M1 si M2 doua
puncte exterioare sferei astfel nct OM1 si OM2 sa intersecteze planul (ABC) n
dou
a puncte interioare 4ABC. Dac
a M1 A M2 A, M1 B M2 B si M1 C M2 C,
s
a se arate c
a M1 O M2 O.
aeru, Suceava
C
at
alin Tig
L137. Consider
am 4ABC nscris n cercul C si fie C1 cercul de centru O1 , tangent
la AB, BC si la cercul C n M , K, respectiv L. Paralela prin B la M K intersecteaza
\
dreptele LM si LK n R, respectiv S. S
a se arate c
a unghiul RO
tit.
1 S este ascu
Neculai Roman, Mircesti (Iasi)
b < 90 , unghiul A
b fiind cel mai
L138. Fie ABC un triunghi cu AB 6= AC, m(A)
mare al triunghiului. Not
am cu M mijlocul lui [BC] si T punctul de intersectie al
simedianei din A cu mediatoarea lui [BC]. S
a se arate c
a 2AM < AT .
Titu Zvonaru, Com
anesti si Cristian Prav
at, Iasi
L139. Fie A1 A2 An un poligon regulat, iar M un punct variabil n interiorul
sau pe laturile poligonului. S
a se determine maximul produsului f (M ) = M A1
M A2 M An , precum si punctele M care realizeaza acest maxim, n fiecare din
cazurile: a) n = 3; b) n = 6.
Dumitru Mihalache si Marian Tetiva, Brlad
L140. Fie a, b, c R+ astfel nct (a+b)2+(b+c)2+(c+a)2+(a+b)(b+c)(c+a) = 4.
a
b
c
a+b b+c c+a
Sa se arate ca
+
+

+
+
.
bc ca ab
c
a
b
Andrei Vr
ajitoarea, elev, Craiova
L141. Daca x, y, z sunt numere reale pozitive cu x3 + y 3 + z 3 = 3, atunci
87

y+2
z+2
x+2
+ 2
+ 2
3.
2
2x + 1 2y + 1 2z + 1
Titu Zvonaru, Com
anesti si Nela Ciceu, Bac
au
L142. Consideram n N , numerele reale strict pozitive a1 < a2 < < an
si A multimea tuturor sumelor a1 a2 an , unde semnele se aleg n toate
n2 + n + 2
si determinati numerele an pentru
modurile posibile. Ar
atati c
a |A| >
2
care se atinge egalitatea.
Gheorghe Iurea, Iasi
L143. Sa se arate ca pentru
p
numar natural
prim si m, n {0, 1, . . . , p 1},

2p + m
p+m
m
m > n, avem
2

mod p2 .
2p + n
p+n
n
Marian Tetiva, Brlad
p

L144. Fie p N, p 2; definim sirurile (xn )n1 si (yn )n1 prin: x1 = p (p 1),
p

a partea
xn+1 = p (p 1) + xn , yn = 2n pn1 xn , n N , unde {} desemneaz
fractionar
a. S
a se arate c
a sirul (yn ) este strict monoton.
Sorin Puspan
a, Craiova
L145. Fie 0 < < ; definim sirurile (xn )n0 , (yn )n0 prin x0 = , y0 = 0,
y
x
Rn 2
Rn 2
e t2 dt, yn+1 =
e t2 dt, n N. Ar
atati c
a cele dou
a siruri sunt
xn+1 =
xn

yn

convergente si aflati limitele lor.

Marius Apetrei, Iasi

Training problems for mathematical contests


A. Junior highschool level
G136. Determine the real numbers x, y, z satisfying the equation
2x + 3 2y + 2z = 2x + 3 2y+2 + 2z+2 = 9.
Andrei Nedelcu, Iasi

2 a+ b c

G137. Let us consider a, b, c Q+ and =


. It is required to
a + b + c

a b+ c

express, in terms of a, b, c and , the real number =


.
2 a+ b+ c
I. V. Maftei, Bucuresti and Mihai Haivas, Iasi
G138. a) The positive real numbers a, b, c satisfy the equation 4abc = a+b+c+1.
b2 + c2 c2 + a2 b2 + a2
Prove that
+
+
2 (ab + bc + ca).
a
b
c
c2 + a2
b2 + c2
+
+
b) The positive real numbers a, b, c satisfy the inequality
a
b
2
2
b +a
2 (ab + bc + ca). Show that a + b + c + 1 4 abc.
c
Andrei Laurentiu Ciupan, highschool student, Bucuresti
88

G139. Denise writes down on the blackboard the numbers 1, 2, 3, . . . , 2008. She
chooses two numbers, deletes them from the blackboard and replaces them by the
modulus of their dierence, repeating this operation until a single number remains
on the blackboard. Can Denise proceed in such a way that the remaining number be
2007? What about 2008?
Julieta Grigoras, Iasi
G140. A polygon with n sides is divided into n 2 triangles by means of n 3 of
its diagonals that do not cut each other(s) at interior points (such a partition being
called a triangulation of the polygon). Let us denote by T0 the number of triangles
whose sides are all diagonals of the polygon, and by T2 the number of triangles with
two of their sides being sides of the polygon as well, the third side being a diagonal
of the polygon. Prove that T2 = T0 + 2.
Marian Tetiva, Brlad
G141. It is considered a network of straight lines that form congruent squares
by intersecting themselves. We mark 2 n + 1 corners of such squares, n 2, so that
any line in the network pass through at most one marked vertex point. Show that
at least two marked points exist such that they are separated, both along horizontal
and vertical directions, by an odd number of lines in the network.
Petru Asaftei, Iasi
G142. We say that the vertex A of the triangle ABC has the property (P ) if
AX < BC, X (BC). Show that if each vertex of 4ABC enjoys the property (P )
then the triangle is equilateral.
Doru Buzac, Iasi
0
G143. We consider the triangle ABC with D, D two points on the line BC such
\ ABC,
\ iar BAD
\0 ACB.
\ The interior bisectrices of the angles BAD
\
that CAD
\0 cut the line BC at E, respectively F . Show that the circle circumscribed
and CAD
to 4AEF and the circle inscribed in 4ABC are concentric.
Neculai Roman, Mircesti (Iasi)
\ +
G144. Let ABCD be a quadrilateral with AB = BC. Show that m(BAD)

2
2
2
2
2
2
\
m(BCD) = 90 if and only if AB CD + AD BC = AC BD .
Ioan S
ac
aleanu, Hrl
au
G145. It is considered the isosceles triangle ABC with AB = AC and a point
M is taken on the open arc BC, which does not contain the corner A, of the circle
circumscribed to the triangle. Show that

AB AC
.
MB MC < MA < MB MC +
MB MC
Gheorghe Costovici, Iasi

B. Highschool level
L136. Let A, B, C be three points on the sphere S of center O, while M1 and
M2 are two exterior points with respect to the sphere S such that OM1 and OM2
intersect the plane (ABC) at two points that are interior to 4ABC. If M1 A M2 A,
M1 B M2 B and M1 C M2 C, show that M1 O M2 O.
aeru, Suceava
C
at
alin Tig
89

L137. We consider 4ABC inscribed in the circle C and let C1 be the circle of
center O1 , which is tangent to AB, BC and to the circle C at M , K, and respectively
L. The parallel line through B to M K intersects the lines LM and LK at R,
\
respectively S. Show that the angle RO
1 S is acute.
Neculai Roman, Mircesti (Iasi)
b being
b < 90 , the angle A
L138. Let ABC be a triangle with AB 6= AC, m(A)
the largest angle of this triangle. We denote by M the midpoint of [BC] and by T
the intersection point of the simedian from A with the mid-perpendicular of [BC].
Show that 2 AM < AT .
Titu Zvonaru, Com
anesti and Cristian Prav
at, Iasi
L139. Fie A1 A2 An be a regular polygon, and M a variable point inside
the polygon or on its sides. Determine the highest value of the product f (M ) =
M A1 M A2 M An , as well as the points M at which this maximum is reached,
in each of the cases: a) n = 3; b) n = 6.
Dumitru Mihalache and Marian Tetiva, Brlad

L140. Let a, b, c R+ be numbers such that (a + b)2 + (b + c)2 + (c + a)2 +


a
b
c
a+b b+c c+a
(a + b) (b + c) (c + a) = 4. Prove that
+
+

+
+
.
bc ca ab
c
a
b
Andrei Vr
ajitoarea, highschool student, Craiova
L141. If x, y, z are real reale positive numbers with x3 + y 3 + z 3 = 3, then
x+2
y+2
z+2
+ 2
+ 2
3.
2
2x + 1 2y + 1 2z + 1
Titu Zvonaru, Com
anesti and Nela Ciceu, Bac
au

L142. We consider n N , the strictly positive real numbers a1 < a2 < < an
and A = the set of all the sums a1 a2 an , where the signs are chosen in all
n2 + n + 2
possible ways. Show that |A| >
and determine the numbers an for which
2
the equality is achieved.
Gheorghe Iurea, Iasi
L143. Showthat, for a prime
p and m, n {0, 1, . . . , p 1},
natural
number

2p + m
p+m
m
m > n, we have
2

mod p2 .
2p + n
p+n
n
Marian Tetiva, Brlad
L144. Let p N, p 2; we define the sequences (xn )n1 si (yn )n1 by: x1 =
p
p

p (p 1), xn+1 = p (p 1) + xn , yn = 2n pn1 xn , n N , where {} denotes


the decimal (or fractional) part. Show that the sequence (yn ) is strictly monotone.
Sorin Puspan
a, Craiova
L145. Let 0 < < ; we define the sequences (xn )n0 , (yn )n0 by x0 = ,
y
x
Rn 2
Rn 2
y0 = 0, xn+1 = e t2 dt, yn+1 = e t2 dt, n N. Show that the two sequences
xn

yn

are convergent and find their limits.

Marius Apetrei, Iasi

90

Pagina rezolvitorilor
CRAIOVA
Colegiul National "Carol I". Clasa a VII-a (prof. STANCA Monica). STANCIU
Ioan: VII(76-80), VIII(77-80), G(116-123).
Not
a. n nr. 2/2007 a fost omisa localitatea Craiova la care figura elevul
STANCIU Ioan cu 11 probleme rezolvate corect. Ne cerem scuze pentru acest incident.

HRLAU
ALEANU

Liceul "Stefan
cel Mare". Clasa a V-a (prof. SAC
Ioan). BRNZA

Carla: P(131,132), V(78,80), VI.78; CALINESCU


Ana Maria: P(131,132), V(76-78),

VI.78; HUTANU
M
ad
alina-Georgiana: P(131,132), V(76,78,80), VI.78; LOGHIN
Bianca: P.131, V(76,78,80), VI.78; MITITELU Melisa: P(131,132), V(77,78), VI.78;
NEICU Mara: P.131, V(76-78), VI.78; SAVA Diana Alexandra: P(131,132), V(76 ALEANU

Larisa-Ionela: P.131,
78), VI.78. Clasa a VI-a (prof. SAC
Ioan). BARAU
Andreea: P.131, V(76,77,80), VI.76; CEUCA
R
V(76-78), VI.76; BUZILA
azvan:

Andreea Simona: P.131, V(76,78), VI(76,78);


A
P.131, V(76,77), VI(76,78); IVANU
T
JITARIU Adina-Diana: P.131, V(76,77), VI(76,78); PINTILII Alina: P.131, V(76,77),
VI(76,78); PLETAN Denisa Elena: P.131, V(76-78), VI.78.
IA
SI

Scoala
nr. 11 "Otilia Cazimir". Clasa a III-a (inst. HUZUM Lina). ANICAI

Laura: P(134,136-139); HUZUM Andrei: P(135-139); MARIU


TA S
imina: P(134,136139); STOIAN Ioana: P(135-139).
Scoala
nr. 13 "Alexandru cel Bun". Clasa a IV-a (inst. COJOCARIU Ana).

Andra: P(134-136,

AGAFITEI
Elena-Roxana: P(134-136,138,139); CARAMALAU

138,139); CALIN
Andreea-Claudia: P(134-136,138,139); COJOCARIU Andreea:
P(134-136,138,139); DUDUMAN Luisa-Stefania: P(134-136,138,139); LELEU Alexandrina-Stefana: P(134-136,138,139); LUPASCU Diana-Maria: P(134-136,138,139);
A
Narcisa-Lorena:
MANOLACHE Madalina-Andreea: P(134-136,138,139); MIHAIL

P(134-136,138,139); PASCU Gabriela: P(134-136,138,139); PADURARU


Tiberiu
S
tefan: P(134-136,138,139); RADUCEA
Marin-Andrei: P(134-136,138,139); SAVIN
Cristina-Simona: P(134-136,138,139); S
TEFAN Bogdan-Vasile: P(134-136,138,139);
S
TIUBEI Cosmin-Ionut: P(134-136,138,139).
Elena). BACIU TuA
Scoala
nr. 14 "Gh. Mrzescu". Clasa a III-a (inst. NUT

A George-Stefan: P(134-139); POSTUDOR Georgianador: P(134-139); CHIRILUT


Madalina: P(134-139); STOICA Adriana: P(134-139).

Scoala
nr. 26 "G. Cosbuc". Clasa a III-a (inst. RACU Maria). APACHITEI

Aura Georgiana: P(134-139); BURA Emma-Andreea: P(134-139); CIORNEI Ale


xandra: P(134-139); CRACIUN
Ioana-Daniela: P(134-139); FILIP Ingrid-Stefania:
Narcis-Eugen: P(134-139); HRISCU Ovidiu-Constantin:
A
P(134-139); GHEORGHIT
P(134-139); HUZA Madalina: P(134-139); MARICIUC Dragos-Claudiu: P(134139); MAXIM Alexandra-Camelia: P(134-139); TUDOSE Ema-Alina: P(134-139);
Cosmin: P(134-139); VASILE Bogdan-Andrei: P(134-139). Clasa a III-a

TUC
A
Andrei-Alexandru: P(134-139); DUMBRAVA

(nv. HRIMIUC Valeria). BRUMA


91

Bianca: P(134-139); HARAPCIUC Eduard-Gabriel: P(134-139); MANTALEA Alex Alexandru: P(134-139); OLARU Alexandra: P(134Adrian: P(134-139); NEAMTU
139); SANDU Beatrice-Gabriela: P(134-139); VORNICU Sorin: P(134-139).

Colegiul National - locatia Scoala


"Gh. Asachi". Clasa a IV-a (inst. CALINESCU

Rodica). BAJENARU
Br
adita: P(134-141,143); BERECHET Alexandru: P(134141,143); CHIVULESCU Alexandru: P(134-141,143); PETREA M
ad
alina: P(134136,138,139); UNGUREANU George: P(134-141,143).
Colegiul National. Clasa a V-a (prof. POPA Gabriel). AMARANDI Marta:
P.141, V(81-84); CERNAT Radu: P.143, V(81-84); MANGALAGIU Ioan: P.141,
V(81-84); PURICE Ioana: P(141,142), V(81,82,84); STOLERU Georgian Ingrid:
Iulia M
P(141-143), V(81-84); S
TREANGA
ad
alina: P(141,143), V(81-84). Clasa a
Norbert-Traian: V(81-86), VI(82-87), VII.87.
A
VII-a (prof. POPA Gabriel). IONIT
Colegiul National "C. Negruzzi". Clasa a VII-a (prof. IONESEI Silviana).
ALOI

PAV
Alexandru: VI(76,79), VII(76,78,79); Clasa a X-a (prof. ZANOSCHI
Adrian). VLAD Ilinca: VIII(76,77,80), IX(77,78,80), X(76,77).
Leon). TUDOColegiul National "E. Racovita". Clasa a VIII-a (prof. PITU
RACHE Alexandru: VIII(81-84,86,87), G131.
SFNTU GHEORGHE
Oana:
Scoala
"Avram Iancu". Clasa a VII-a (prof. LZR Emese). FURTUNA

V(76-78,80), VI(77-80), VII(76,78,79).


SIBIU
Colegiul National "Gh. Lazar". Clasa a XII-a. VLAD Emanuel: XII(78,79),
L(116-122,125).
SUCEAVA
Scoala
cu clasele I-VIII, nr. 3. Clasa a III-a (nv. TABARCEA Silvestru).

FECHET S
tefan: P(127-130,132); Clasa a IV-a (inst. NECHITA Daniela). FECHET Mircea: P(125-131).

TRGU NEAMT
Colegiul National "Stefan
cel Mare". Clasa a III-a (nv. ). CEBERE Tudor:

P(125-127,129,134-136,139).

Premii acordate rezolvitorilor


Scoala
nr. 13 "Alexandru cel Bun", Iasi

AGAFITEI
Elena-Roxana (cl. a IV-a): 1/2007(5pb), 2/2007(5pb), 1/2008(5pb);
Andra (cl. a IV-a): 1/2007(5pb), 2/2007(5pb), 1/2008(5pb);
CARAMALAU

CALIN
Andreea-Claudia (cl. a IV-a): 1/2007(5pb), 2/2007(5pb), 1/2008(5pb);
COJOCARIU Andreea (cl. a IV-a): 1/2007(5pb), 2/2007(5pb), 1/2008(5pb);
92

DUDUMAN Luisa-
Stefania (cl. a IV-a): 1/2007(5pb), 2/2007(5pb), 1/2008(5pb);
LELEU Alexandrina-
Stefana (cl. a IV-a): 1/2007(5pb), 2/2007(5pb), 1/2008(5pb);
LUPA
SCU Diana-Maria (cl. a IV-a): 1/2007(5pb), 2/2007(5pb), 1/2008(5pb);
MANOLACHE M
ad
alina-Andreea (cl. a IV-a): 1/2007(5pb), 2/2007(5pb),
1/2008(5pb);
PASCU Gabriela (cl. a IV-a): 1/2007(5pb), 2/2007(5pb), 1/2008(5pb);

PADURARU
Tiberiu-
Stefan (cl. a IV-a): 1/2007(5pb), 2/2007(5pb), 1/2008(5pb);

RADUCEA
Marin-Andrei (cl. a IV-a): 1/2007(5pb), 2/2007(5pb), 1/2008(5pb);
SAVIN Cristina-Simona (cl. a IV-a): 1/2007(5pb), 2/2007(5pb), 1/2008(5pb);
S
TEFAN Bogdan-Vasile (cl. a IV-a): 1/2007(5pb), 2/2007(5pb), 1/2008(5pb);
S
TIUBEI Cosmin-Ionut (cl. a IV-a): 1/2007(5pb), 2/2007(5pb), 1/2008(5pb);
Colegiul National - locatia Scoala
"Gh. Asachi", Iasi

BAJENARU
Br
adita (cl. a IV-a): 1/2007(7pb), 2/2007(5pb), 1/2008(9pb);
CHIVULESCU Alexandru (cl. a IV-a): 1/2007(6pb), 2/2007(5pb), 1/2008(9pb);
Scoala
nr. 26 "G. Cosbuc", Iasi

CIORNEI Alexandra (cl. a III-a): 1/2007(5pb), 2/2007(5pb), 1/2008(6pb);

CRACIUN
Ioana-Daniela (cl. a III-a): 1/2007(5pb), 2/2007(5pb), 1/2008(6pb);
MARICIUC Dragos-Claudiu (cl. a III-a): 1/2007(5pb), 2/2007(5pb), 1/2008(6pb);
Cosmin (cl. a III-a): 1/2007(5pb), 2/2007(5pb), 1/2008(6pb);

TUC
A
Colegiul National "Carol I", Craiova
STANCIU Ioan (cl. a VII-a): 2/2006(13pb), 1/2007(12pb), 2/2007(14pb);

Liceul "Stefan
cel Mare", HRLAU

NEICU Mara (cl. a V-a): 2/2006(5pb), 1/2007(5pb), 1/2008(5pb);

Andreea Simona (cl. a VI-a): 1/2007(5pb), 2/2007(5pb), 1/2008(5pb);


IVANU
TA

93

Revista semestrial RECREAII MATEMATICE


este editat de
ASOCIAIA RECREAII MATEMATICE. Apare la datele de 1 martie i
1 septembrie i se adreseaz elevilor, profesorilor, studenilor i tuturor celor
pasionai de matematica elementar.
n atenia tuturor colaboratorilor
Materialele trimise redaciei spre publicare (note i articole, chestiuni de
metodic, probleme propuse etc.) trebuie prezentate ngrijit, clar i concis; ele
trebuie s prezinte interes pentru un cerc ct mai larg de cititori. Se recomand ca
textele s nu depeasc patru pagini. Evident, ele trebuie s fie originale i s
nu fi aprut sau s fi fost trimise spre publicare altor reviste. Rugm ca materialele tehnoredactate s fie nsoite de fiierele lor.
Problemele destinate rubricilor: Probleme propuse i Probleme pentru
pregtirea concursurilor vor fi redactate pe foi separate cu enun i demonstraie/rezolvare (cte una pe fiecare foaie) i vor fi nsoite de numele autorului, coala i localitatea unde lucreaz/nva.
Redacia va decide asupra oportunitii publicrii materialelor primite.
n atenia elevilor
Numele elevilor ce vor trimite redaciei soluii corecte la problemele din
rubricile de Probleme propuse i Probleme pentru pregatirea concursurilor
vor fi menionate n Pagina rezolvitorilor. Se va ine seama de regulile:
1. Pot trimite soluii la minimum cinci probleme propuse n numrul
prezent i cel anterior al revistei; pe o foaie va fi redactat soluia unei singure
probleme.
2. Elevii din clasele VI-XII au dreptul s trimit soluii la problemele
propuse pentru clasa lor, pentru orice clas mai mare, din dou clase mai mici i
imediat anterioare. Elevii din clasa a V-a pot trimite soluii la problemele propuse
pentru clasele a IV-a, a V-a i orice clas mai mare, iar elevii claselor I-IV pot
trimite soluii la problemele propuse pentru oricare din clasele primare i orice clas mai mare. Orice elev poate trimite soluii la problemele de concurs (tip G i L).
3. Vor fi menionate urmtoarele date personale: numele i prenumele,
clasa, coala i localitatea.
4. Plicul cu probleme rezolvate se va trimite prin pot (sau va fi adus
direct) la adresa Redaciei:
Prof. dr. Temistocle Brsan
Str. Aurora, nr. 3, sc. D, ap. 6,
700 474, Iai
Jud. IAI
E-mail: t_birsan@yahoo.com

CUPRINS
Recreaii tiinifice 125 de ani de la apariie .................................................................................. 1
Proiect de reeditare ................................................................................................................................. 7

AR TIC O LE I N OTE
L. STRUGARIU, C. STRUGARIU Polinoame Fibonacci, polinoame ciclotomice ............................ 8
A. REISNER Submulimi ale unei mulimi finite i matrici binare ............................................... 13
S. PUPAN O generalizare a teoremelor Stolz-Cesaro ................................................................. 16
M. TETIVA O problem de combinatoric destul de grea ............................................................. 20
I. M. MAFTEI, M. HAIVAS Tehnici de stabilire a unor inegaliti geometrice ........................... 22
A. IGERU Asupra unei probleme de concurs.............................................................................. 24
M. DICU, L. TUESCU O clas de inegaliti ................................................................................. 27
T. ZVONARU Inegaliti omogene i puin analiz... ................................................................ 30
S. BOGA Despre numerele reale algebrice........................................................................................ 33
M. BTINEU-GIURGIU, D. M. BTINEU-GIURGIU Asupra unei note
din revista "Recreaii matematice"............................................... 35
Gh. COSTOVICI Limita unor iruri de numere reale...................................................................... 38
M. BENCZE O rafinare a inegalitii dintre media aritmetic i cea logaritmic ........................ 39

NO TA ELEVU LUI
T. PDURARIU Estimri de sume .................................................................................................... 40

CHESTIUNI M ETOD IC E
L. MODAN Acurateea limbajului matematic n combinatoric ...................................................... 42
F. POPOVICI Asupra unei identiti clasice privind partea ntreag .............................................. 44
G. POPA, I. ERDEAN O abordare analitic a unor probleme de geometrie .............................. 46

COR E SPOND EN E
C. J. ZHAO A study of a new geometric inequality ....................................................................... 49

CONCUR SUR I I EX AMEN E


Concursul "Recreaii Matematice", Ediia a V-a, Muncel (Iai), ............................................................ 53
South Eastern European Mathematical Olympiad for University Students, Cyprus, 2007 ............... 54

PR O BLEME I S O LU I I
Soluiile problemelor propuse n nr. 1/2007 .......................................................................................... 57
Soluiile problemelor pentru pregtirea concursurilor din nr. 1/2007 .................................................. 71
Probleme propuse ..................................................................................................................................... 80
Probleme pentru pregtirea concursurilor............................................................................................... 86
Training problems for mathematical contests ........................................................................................ 88
Pagina rezolvitorilor............................................................................................................................... 91

ISSN 1582 1765

6 lei

Prof. Constantin Corduneanu la a 75-a aniversare


La 26 iulie a. c., prof. C. Corduneanu mplineste vrsta de 75 ani. N
ascut la
26 iulie 1928 la Iasi, domnia sa a urmat scoala primar
a n satul Potngeni, com.
Movileni, jud. Iasi, unde parintii sai erau nvatatori. Studiile liceale le-a facut la
Liceul Militar (Iasi si Predeal) n perioada 19401947. Citam dintr-un interviu al
d-sale, acordat ziarului "Curierul Romnesc": "Am avut sansa de a studia n acea
scoala de elita, unde profesorul de istorie era doctor la Oxford, cel de chimie era de
asemenea doctor n chimie si inginer chimist, cel de geografie era simultan si asistent
universitar, cel de franceza era doctor n filologie, cu stagiu de specializare n Franta,
devenind apoi seful catedrei de limba franceza la Universitatea din Iasi. Profesorul
de matematica fusese si cadru universitar (conferentiar suplinitor). Cnd am pasit
pragul universitatii iesene, optiunile mele de viitor erau deja clare." n clasa a VII-a
de liceu (1947), a luat premiul I pe tara la concursul anual, pe care l organiza atunci
Gazeta matematica.
Dup
a trecerea bacalaureatului n 1947, se nscrie la sectia de matematic
a a Fac.
de S
tiinte din Iasi, promovnd primii doi ani de studii ntr-un singur an, astfel ca n
1950 este declarat absolvent. Fiind un student eminent, n ultimul an de facultate a
fost numit preparator, regulamentul n vigoare la acea vreme permitnd acest lucru.
Despre perioada studentiei si despre profesorii s
ai, precum si despre activitatea
din cadrul Seminarului Matematic "Al. Myller", spunea: "A fost o alta sansa a
vietii mele sa am posibilitatea sa ma formez ca specialist ntr-o atmosfera de nalta
tinuta stiintifica si sa fiu sprijinit, n momente critice ale vietii, de catre profesorii
mei" (citat din acelasi interviu). Dintr-o scrisoare redactat
a n urma plec
arii sale
din tara, citam: "Ramnem si noi, cei dinafara, datori fostilor profesori, care ne-au
pregatit att de solid pentru viata. Toata activitatea noastra este patrunsa de simtul
respectabilitatii fata de bunul nume pe care matematica romneasca l-a dobndit si
continua, credem, sa-l pastreze. Este tributul pe care-l datoram memoriei profesorilor
nostri, singurii cu adevarat fauritori ai viitorului nostru, n conditii care nu au fost
tocmai prielnice".
A urcat cu rapiditate treptele ierarhiei universitare, n 1965 fiind profesor titular.
ntre anii 19541968, a fost, n paralel, cercet
ator principal si apoi sef de sector la
Institutul de Matematica al Academiei, filiala Iasi. n 1961 este distins cu premiul
Ministerului Educatiei si nvatamntului, iar n 1964 primeste premiul "Gh. Lazar"
al Academiei.
A fost rector al Institutului Pedagogic din Suceava (19641967), decan al Fac. de
Matematica din Iasi (19681972) si prorector al Univ. "Al. I. Cuza" (19721977). n
1974, a fost ales membru corespondent al Academiei Romne (sectia matematica).
Desfiintarea Institutului de Matematic
a si deteriorarea continu
a a climatului politic, social si cultural din tara noastr
a l-au determinat s
a emigreze n str
ain
atate
(1977). Dupa un an petrecut la Univ. din Kingston (Rhode Island) si altul la Univ.
din Knoxville (Tennessee), se stabileste definitiv la Univ. of Texas at Arlington, unde
a functionat pn
a la pensionare. i place s
a spun
a c
a a avut 4000 studenti n Romnia
si aproximativ 3000 n America, perioada activit
atii sale universitare fiind de 47 ani.
Activitatea stiintifica a prof. C. Corduneanu s-a desfasurat fara ntrerupere,
1

fiind continuat
a si n prezent. Este autorul a peste 150 de articole, publicate n
reviste de mare prestigiu din tara si din strainatate. La Iasi, a nfiintat Seminarul
special de teoria calitativa a ecuatiilor diferentiale si integrale, d-sa fiind initiatorul
acestei directii de cercetare la Iasi. Multi tineri s-au format n ambianta acestui
Seminar, fiindu-i recunosc
atori profesorului lor.
Domeniul de cercetare a prof. C. Corduneanu este teoria ecuatiilor diferentiale,
integrale si integro-diferentiale, n care a abordat probleme de existenta, comportare
asimptotic
a, periodicitate, aproape-periodicitate si stabilitate a solutiilor unor astfel de ecuatii. Merit
a subliniat
a introducerea "metodei comparatiei" n teoria stabilitatii, studiul sistematic al teoriei admisibilitatii operatorilor integrali, metoda
frecventiala n studiul stabilitatii ecuatiilor integrale Volterra etc. Pentru a ne face
o idee despre opera matematic
a a d-sale, amintim doar monografiile pe care le-a
publicat pn
a n prezent:
1. Functii aproape periodice, Ed. Academiei, Bucuresti, 1961;
2. Almost Periodic Functions, Interscience Publ., New-York, 1968; Chelsea Publ.
Company, New-York, 1989;
3. Principles of Dierential Equations and Integral Equations, Allyn and Bacon,
Boston, 1971; Chelsea Publ. Company, New-York, 1977; 1988;
4. Integral Equations and Stability of Feedback Systems, Academic Press, NewYork, 1973;
5. Integral Equations and Applications, Cambridge Univ. Press, 1991;
6. Functional Equations with Causal Operators, Taylor and Francis, London 2002.
Monografia "Functii aproape periodice" a fost prima pe plan mondial consacrat
a
acestui important subiect. Cele dou
a editii n limba englez
a ale ei au fost completate
si dezvoltate cu noi capitole; la cea de a doua editie englez
a si-au adus contributia
si renumitii profesori ieseni N. Gheorghiu si V. Barbu. Se cuvine a adauga ca prof.
C. Corduneanu a fost primul matematician din Iasi, ale c
arui c
arti au fost publicate
la edituri din str
ain
atate dac
a avem n vedere perioada postbelic
a. Multi matematicieni din tar
a si de peste hotare citeaz
a si folosesc contributiile originale ale d-sale n
lucrarile lor de specialitate. Dintre cei din ultima categorie amintim pe L. Cesari,
R. Conti, J. Massera, J. J. Scher, N. Rouche, W. Coppel si Ph. Hartman.
Prof. C. Corduneanu a fost si este membru n comitetul de redactie al unor
importante reviste de matematic
a din tara noastr
a si din alte tari. Este editor al
revistei Libertas Mathematica fondata n anul 1981 si ajunsa anul acesta la al XXIIIlea num
ar. Este membru al Academiei Romno-Americane de S
tiinte si Arte (SUA),
fiind chiar presedintele acestei institutii, n perioada 19951998. Este Doctor Honoris
Causa al universitatilor din Iasi, Constanta si Brasov. Are o prezenta activ
a la multe
manifestari stiintifice din tara si din America. Este invitat de numeroase universitati
pentru a tine conferinte si a prezenta comunic
ari.
A ncurajat revista Recreatii matematice, sustinnd necesitatea unei astfel de
publicatii, ori de cte ori venea la Iasi.
La a 75-a aniversare, revista Recreatii matematice ureaza acad. prof. C. Corduneanu "La multi ani si multe succese n activitatea sa viitoare!".
Redactia revistei
2

A. N. Kolmogorov - 100 de ani de la nastere


Andrei Nicolaevici Kolmogorov este unul dintre cei mai mari matematicieni
ai secolului al XX-lea si unul dintre cei mai fertili matematicieni ai tuturor timpurilor. n peste 300 de lucrari stiintifice, manuale si monografii, Kolmogorov acopera
aproape toate domeniile matematicii, exceptnd teoria numerelor. n lucrarile sale
atac
a numai subiecte fundamentale si care deschid noi domenii de investigatie. O list
a
neexhaustiv
a a directiilor cercet
arilor sale cuprinde: teoria seriilor trigonometrice,
teoria multimilor, teoria masurii, teoria integrarii, logica constructiva (intuitionista),
topologia, teoria aproximarii, teoria probabilitatilor, teoria proceselor stochastice, teoria informatiei, statistica matematica, sisteme dinamice, teoria automatelor, teoria
algoritmilor, lingvistica matematica, mecanica cereasca, teoria turbulentelor, ecuatii
diferentiale, problema a 13-a a lui Hilbert, balistica, aplicatii ale matematicii n biologie, geologie si cristalizarea metalelor.
S-a n
ascut la 25 aprilie 1903 n orasul Tambov (Rusia). R
amne orfan de mam
a
nc
a de la nastere. Era de origine aristocrat
a prin mama sa; tat
al s
au, inginer
agronom, avea o vasta cultura generala. n 1920 a intrat la Universitatea din Moscova,
urmnd si absolvind cursurile facultatii de matematica - mecanica. n formarea lui
A. N. Kolmogorov un rol important a avut seminarul condus de V. V. Stepanov
consacrat seriilor trigonometrice si scoala de analiz
a matematic
a creat
a la Moscova
de N. N. Luzin. n 1931 devine profesor al acestei universitati. S-a stins din viata
la Moscova la 20 octombie 1982 dupa o bogata activitate stiintifica si didactica de
aproape sapte decenii.
n lucr
arile sale, A. N. Kolmogorov prezint
a ntr-o unitate surprinz
atoare
chestiuni din domenii aparent deosebite ale matematicii. nca din studentie a obtinut
rezultatele care au produs o puternica impresie n lumea matematicienilor. n 1922 a
construit un exemplu de serie Fourier - Lebesgue divergent
a aproape peste tot si un
altul de serie divergent
a n fiecare punct. Sub influenta lucr
arilor lui M. S. Suslin si
N. N. Luzin, n acelasi an, face un studiu asupra operatiilor cu multimi introducnd
o clasa foarte larga de operatii.
ncepnd din 1924 interesul lui A. N. Kolmogorov se ndreapt
a spre teoria
probabilit
atilor, domeniu n care va deveni o autoritate de necontestat. Folosind
metode noi, n particular asa numita inegalitate a lui Kolmogorov, stabileste conditii
necesare si suficiente pentru legea numerelor mari si demonstreaza legea logaritmului iterat. nc
a din 1909, E. Borel a nteles importanta teoriei m
asurii pentru
constructia fundamentelor teoriei probabilit
atilor. Ideile sale au fost dezvoltate de
A. Lomnicki ntr-un articol din 1923 si au devenit obiectul cercetarilor lui Kolmogorov n 1929 care, n lucrarea "Teoria generala a masurii si calculul probabilitatilor", propune primul sistem axiomatic al teoriei probabilit
atilor fundamentat
pe teoria m
asurii si teoria functiilor de variabil
a real
a. Aceast
a axiomatizare cap
at
a
forma finala, acceptata astazi unanim, n monografia "Notiunile fundamentale ale
teoriei probabilitatilor" aparuta n 1933, editura Springer. Referindu-se la lucrarea
"Metode analitice n teoria probabilititalor" (1931), unul dintre elevii str
aluciti ai lui
Kolmogorov, B. V. Gnedenko, afirm
a: "Teoria actuala a proceselor aleatoare Markov
sau, asa cum le numste Kolmogorov, proceselor fara postactiune, a fost fundamentata
3

de Kolmogorov n aceasta lucrare. A aparut un domeniu al matematicii cu aplicatii n


fizica, biologie, chimie, activitatea inginereasca. Gndirea lui Kolmogorov a actionat
n diverse domenii n care a pus probleme noi si a dat raspunsuri unor chestiuni
principiale".
Pentru preocup
arile sale de teoria informatiei unii matematicieni l-au numit pe
A. N. Kolmogorov "Newton al secolului XX". n teoria algoritmica a informatiei
a introdus notiunea de -entropie a unei multimi dintr-un spatiu metric, care are
aplicatii n probleme privind superpozitiile (compunerile) de functii si i-a permis s
a
revin
a asupra problemei a 13-a a lui Hilbert: "O functie continua de trei variabile
poate fi reprezentata ca o superpozitie de functii continue de doua variabile?". n
1956 Kolmogorov a demonstrat ca orice functie continua de mai multe variabile se
reprezint
a ca o superpozitie de functii continue de trei variabile. n 1957, V. I. Arnold,
probabil cel mai renumit dintre elevii lui Kolmogorov, a demonstrat c
a orice functie
continua de trei variabile se reprezinta ca o superpozitie de funtii continue de doua
variabile. n acelasi an, Kolmogorov arata ca o functie continua de un numar oarecare
de variabile se reprezint
a ca o superpozitie de functii continue de o variabil
a real
a la
care se adaug
a operatia de adunare (functie continu
a de dou
a variabile). Creator al
teoriei algoritmice a informatiei, Kolmogorov a introdus notiunea centrala a acestei
teorii, aceea de complexitate a unui obiect matematic (numit
a ast
azi complexitate
Kolmogorov ).
n 1925 A. N. Kolmogorov a publicat un articol despre legea tertului exclus.
Acest articol a intrat n fondul de aur al logicii matematice fiind prima lucrare pe
plan mondial a logicii matematice. n 1932 dezvolt
a semantica logicii intuitioniste a
lui A. Heyting dnd acesteia aspectul de logica constructiva. n 1952 a dat definitiile
cele mai generale ale notiunilor de obiect constructiv si algoritm. n anii 60 a reusit
sa atraga un numar mare de cercetatorii n studiul limbii si literaturii prin metode
matematice.
A. N. Kolmogorov a creat puternice scoli de cercetare matematic
a din care
s-au ridicat un num
ar impresionant de matematicieni de mare valoare, dar n acelasi
timp a acordat o atentie deosebita si nvatamntului preuniversitar. Din initiativa
sa a fost nfiintat
a la Moscova scoala internat num
arul 18 (cunoscut
a sub numele
de "scoala Kolmogorov"), un liceu de matematic
a unde erau selectati copiii talentati
la matematic
a, care se remarcau la olimpiade, de pe teritoriul ntregii foste Uniuni
Sovietice.
A fost membru al Academiei de Stiin
te din tara sa (1939), al academiilor din
Olanda, Anglia, Franta, SUA, Germania, Polonia, India, Romnia (1956) s. a.,
doctor honoris causa al multor universit
ati din ntreaga lume, membru de onoare al
unor societati prestigioase. A fost laureat al premiului de stat si de sapte ori laureat
al premiului Lenin, cel mai prestigios premiu din fosta Uniune Sovietic
a.
n 1963 a primit cea mai nalt
a distinctie international
a care se acord
a matematicienilor, premiul Bolzano.

Prof. dr. Petru MINUT

Inegalit
ati pentru mediane, bimediane, bisectoare
Dan Stefan
MARINESCU si Viorel CORNEA1

Vom demonstra pentru nceput urmatoarea


Lem
a. Fie ABC un triunghi si M (BC) astfel nct
BM
= k (0, 1). Atunci AM < k AC + (1 k) AB.
BC
Demonstratie. Fie N (AB) astfel nct M N k AC
(fig. 1). Din teorema fundamentala a asemanarii obtinem
M N = k AC si AN = (1 k) AB. Aplicnd n triunghiul
AM N inegalitatea triunghiului avem AM < M N + AN
sau, conform celor de mai sus, AM < k AC + (1 k) AB.

M
Fig. 1

Propozitia 1. Fie ABCD un tetraedru, M int (BCD), N (CM BD si


BP
BN
= u si
= v, atunci
P (DM BC. Daca
ND
PC
v
u
1
AB +
AC +
AD.
AM <
u+v+1
u+v+1
u+v+1
Demonstratie. Fie Q (BM CD (fig. 2). n baza teoremei lui van Aubel,
BM
BN
BP
BM
u+v
=
+
= u + v, de unde
=
.
(1)
MQ
ND PC
BQ
u+v+1

Tinnd
seama de (1) si de Lem
a, n 4ABQ avem
1
u+v
AM <
AB +
AQ.
u+v+1
u+v+1
Din teorema lui Ceva, aplicata n 4BCD, obtinem
CQ
u
CQ
u
= , adica
=
. Aplicnd iarasi Lema
QD
v
CD
u+v
n 4ACD vom avea
v
u
AQ <
AC +
AD.
(3)
u+v
u+v
Relatiile (2) si (3) conduc la
1
v
u
AM <
AB +
AC +
AD, B
u+v+1
u+v+1
u+v+1
ceea ce ncheie demonstratia acestei propozitii.
Cteva cazuri particulare ale acestui rezultat prezint
a interes n sine.

(2)
A

N
M
P

D
Q

C
Fig. 2

Corolarul 1 (Inegalitatea medianei). Fie ABCD un tetraedru si GA centrul de


1
greutate al fetei BCD; atunci AGA < (AB + AC + AD).
3
1

Profesori, Liceul Teoretic "Iancu de Hunedoara", Hunedoara

Demonstratie. Evident, n acest caz u = v = 1 si aplicnd Propozitia 1 obtinem


inegalitatea ceruta.
Corolarul 2. Medianele unui tetraedru pot fi lungimile laturilor unui patrulater.
Demonstratie. Fie GA , GB , GC , GD centrele de greutate ale fetelor BCD,
ACD, ABD, ABC n tetraedrul ABCD si G centrul s
au de greutate. Aplicnd n
1
tetraedrul GBCD inegalitatea din Corolarul 1, avem GGA < (GB + GC + GD).
3
Conform unui rezultat cunoscut
3
3
3
BG = BGB , CG = CGC , DG = DGD .
4
4
4
1
1
Ca urmare, GGA < (BGB + CGC + DGD ). Cum GGA = AGA , gasim ca
4
4
AGA < BGB + CGC + DGD .
Procednd la fel g
asim si inegalit
ati similare, ceea ce asigur
a faptul c
a medianele
tetraedrului pot fi laturile unui patrulater.
Corolarul 3 (Inegalitatea bisectoarei). Fie ABCD un tetraedru, AE bisectoarea
triedrului cu vrful n A, E int (BCD); atunci
SC
SD
SB
AB +
AC +
AD.
AE <
SB + SC + SD
SB + SC + SD
SB + SC + SD
Demonstratie. Din teorema planului bisector (AE este intersectia planelor bisectoare ale diedrelor ce compun triedrul cu vrful n A), avem (fig. 2, cu M E):
BN
BP
SD
SC
si v =
u=
. Aplicnd Propozitia 1, vom obtine:
=
=
ND
SB
PC
SB
SB
SC
SD
AE <
AB +
AC +
AD,
SB + SC + SD
SB + SC + SD
SB + SC + SD
adic
a inegalitatea din enunt.
Corolarul 4. Fie ABCD un tetraedru si IA centrul cercului nscris n triunghiul
BCD; atunci:
CD
BD
BC
AIA <
AB +
AC +
AD.
BC + CD + BD
BC + CD + BD
BC + CD + BD
Demonstratie. n fig. 2 consideram M IA ; din teorema bisectoarei, avem
BN
BC
BP
BD
si v =
u=
=
=
. Se aplic
a Propozitia 1.
ND
CD
PC
CD
Observatie. Propozitia 1 se poate demonstra si cu ajutorul relatiei lui Stewart,
care permite determinarea lui AM n functie de AB, AC, AD si u, v.
AM
= u si
Propozitia 2. Fie ABCD un tetraedru, M (AB) astfel nct
AB
CN
N (CD) astfel nct
= 1 u; atunci au loc
CD
|u BC (1 u) AD| < M N < u BC + (1 u) AD,
|u BD (1 u) AC| < M N < u BD + (1 u) AC.
Demonstratie. Fie P (AC) astfel nct M P k BC (fig. 3). Din teorema
CN
PC
= 1 u. Cum
= 1 u,
fundamentala a asemanarii avem M P = u BC si
AC
CD
6

vom obtine c
a P N k AD si n consecinta, n baza teoremei pomenite mai sus, vom avea P N = (1 u) AD.
Deoarece punctele M , P , N nu pot fi coliniare, din
inegalit
atile triunghiului obtinem
|M P P N | < M N < M P + P N

sau, dup
a nlocuiri, avem

|u BC (1 u) AD| < M N < u BC + (1 u) AD.

Procednd la fel obtinem si al doilea grup de inegalit


ati.

D
N
C
Fig. 3

Corolarul 5 (Inegalit
atile bimedianei). Fie ABCD un tetraedru, M mijlocul lui
[AB] si N mijlocul lui [CD]; atunci
|BC AD| < 2M N < BC + AD, |BD AC| < 2M N < BD + AC.
1
Demonstratie. Lund u = n Propozitia 2, gasim inegalitatile enuntate.
2
Pentru inegalitatile care urmeaza avem nevoie de urmatoarea
Lem
a. Fie ABCD un tetraedru. Cu conventiile din Propozitia 1 (fig. 2), are loc
egalitatea:
1
v
u
AB 2 +
AC 2 +
AD2
AM 2 =
1+u+v
1+u+v
1+u+v
u
uv
v
2
2
2

2 BC
2 BD
2 CD .
(1 + u + v)
(1 + u + v)
(1 + u + v)
BM
u+v
CQ
u
si
Demonstratie. Ca n Propozitia 1, obtinem
=
=
.
BQ
1+u+v
CD
u+v
Aplicnd relatia lui Stewart n triunghiul ABQ, avem:
1
u+v
u+v
2
(1)
AB 2 +
AQ2
AM 2 =
2 BQ .
1+u+v
1+u+v
(1 + u + v)
Aceeasi relatie aplicat
a n triunghiurile BCD si ACD conduce la
v
u
uv
2
BQ =
CD2 ,
BC 2 +
BD2
u+v
u+v
(u + v)2
v
u
uv
AQ2 =
CD2 .
AC 2 +
AD2
u+v
u+v
(u + v)2

(2)
(3)

Din (1), (2) si (3) obtinem egalitatea din enunt.


Propozitia 3. Fie ABCD un tetraedru. Are loc inegalitatea

1
v
u
1
2
2
2
AM
AB +
AC +
AD ,
2R 1 + u + v
1+u+v
1+u+v

unde R este raza sferei circumscrise tetraedrului.


Demonstratie. Fie A0 punctul n care (AM intersecteaz
a a doua oar
a sfera
circumscrisa tetraedrului. Avem evident
2R AM AA0 AM = (AM + A0 M ) AM = AM 2 + A0 M AM.
7

(1)

Puterea punctului M fata de sfer


a este egal
a cu
A0 M AM = R2 OM 2 ,
(2)
unde O este centrul sferei. Din lema, vom avea:
1
v
u
AM 2 =
AB 2 +
AC 2 +
AD2
1+u+v
1+u+v
1+u+v
u
uv
v
2
2
CD2 ,
(3)

2 BC
2 BD
(1 + u + v)
(1 + u + v)
(1 + u + v)2
1
v
u
OM 2 =
OB 2 +
OC 2 +
OD2
1+u+v
1+u+v
1+u+v
u
uv
v
2
2
CD2 .

2 BC
2 BD
(1 + u + v)
(1 + u + v)
(1 + u + v)2
Cum OB = OC = OD = R, ultima egalitate devine:
v
u
uv
2
2
2
OM 2 = R2
(4)
2 BC
2 BD
2 CD .
(1 + u + v)
(1 + u + v)
(1 + u + v)
Din (1), (2), (3) si (4) obtinem inegalitatea din enunt.
Corolarul 6. Fie ABCD un tetraedru si GA centrul de greutate al fetei BCD;
atunci

1
AB 2 + AC 2 + AD2 .
AGA
6R
Demonstratie. n Propozitia 3 consider
am M GA (u = v = 1).
n cele ce urmeaza, daca ABCD este un tetraedru, vom nota cu mX , hX , lX
mediana, n
altimea, bisectoarea din vrful X (X {A, B, C, D}).
Corolarul 7. n orice tetraedru ABCD avem
3R (mA + mB + mC + mD ) a2 + b2 + c2 + l2 + m2 + n2 ,
unde a = BC, b = CA, c = AB, l = DA, m = DB, n = DC, iar mA = AGA etc.
Demonstratie. Din Corolarul 6 avem 6R mA c2 +b2 +l2 , 6R mB c2 +a2 +m2 ,
6R mC a2 + b2 + n2 , 6R mD l2 + m2 + n2 , de unde, prin sumare, obtinem
inegalitatea ceruta.
Corolarul 8. Fie ABCD un tetraedru si IA punctul n care bisectoarea din A a
tetraedrului intersecteaza fata BCD; atunci
SC
SD
SB
AB 2 +
AC 2 +
AD2 .
2R AIA
SB + SC + SD
SB + SC + SD
SB + SC + SD
SC
SD
si v =
.
Demonstratie. n Propozitia 3 se pune u =
SB
SB
Observatie. Ca si n Corolarul 7 se pot obtine inegalit
ati pentru suma bisectoarelor si suma naltimilor.
Bibliografie
1. D. Brnzei, S. Anita, C. Cocea - Planul si spatiul euclidian, Editura Academiei,
Bucuresti, 1986.
2. D. S
t. Marinescu - Inegalitati pentru ceviene, R.M.C. 12, 19951996, 57.
3. N. Pavelescu, M. Lascu - Inegalitati n triunghiuri si tetraedre, G.M. 10/1989,
362366.
8

Propriet
ati de coliniaritate n patrulatere
Temistocle BRSAN 1
n scopul stabilirii proprietatilor de mai jos vom utiliza urmatoarele trei leme.
A
Lema 1. Fie ABC un triunghi, X (BC), Y (CA)
si {Z} = AX BY . Are loc relatia
Y
BX AC
BZ
=

.
(1)
ZY
XC AY
Z
Demonstratie. Conform Teoremei lui Menelaus,
aplicata triunghiului BCY si transversalei XZ, avem
B
C
X
BX AC ZY

= 1, de unde deducem (1).


Fig. 1
XC AY BZ
Lema 2 [1, p. 65]. Fie ABCD un patrulater convex si {I} = AC BD. ConEA
si
sideram punctele E (AB) si F (CD) cu pozitiile date de rapoartele p =
EB
FD
respectiv q =
. Atunci
FC
I EF IA ID = pq IB IC.
(2)
Demonstratie. Presupunem c
a AB si CD
C
F
se intersecteaza n S. Notnd {I 0 } = EF AC
D
S
si {I 00 } = EF BD, obtinem relatiile:
I
0
I'
ES I A F C
I''
0
= 1 (4SAC si transversala EF ),
EA I C F S
A
ES I 00 B F D

= 1 (4SBD si transversala EF ),
E
EB I 00 D F S
B
de unde, prin egalare,
Fig.
2
()
I 0 A I 00 D = pq I 00 B I 0 C.
()

Atunci, avem I EF I 0 = I 00 = I IA ID = pq IB IC. Reciproc,


I 0 A I 00 D
() IA ID
IA ID = pq IB IC

= 0 00 I 0 = I 00 = I
IC IB
IC I B
(ntr-adev
ar, dac
a I 0 6= I, atunci si I 00 6= I si am avea, n pozitia din figur
a pentru
I 0 A ID
I 00 D
IA
< 0 ,
< 00 ).
EF , c
a
IC
I C IB
I B
Mention
am c
a egalitatea () se obtine prin asem
anare de triunghiuri, dac
a AB k
CD, restul demonstratiei ramnnd acelasi.
Lema 3. n conditiile Lemei 2 au loc relatiile
q + 1 IB
1 q + 1 IA
IE
=p

.
(3)
IF
p + 1 ID
q p + 1 IC
Demonstratie. n 4ECD scriem relatia lui van
Aubel:
IE
JE
KE
=
+
,
IF
JD KC
1

I
J
A

Prof. dr., Catedra de Matematic


a , Univ. Tehnic
a "Gh. Asachi", Iasi

K
E

B
Fig. 3

unde {J} = ED AC si {K} = EC BD. Conform Lemei 1, avem


ID AB
CK
IC AB
DJ
=

(4ABD) si
=

(4BCA) .
JE
IB AE
KE
IA BE
Ca urmare, obtinem:
IB AE
IA BE
p
IB
1
IA
IE
=
+
=

=
IF
ID AB
IC AB
p + 1 ID p + 1 IC
1
p IB IC + IA ID
=

.
p+1
IC ID
p (q + 1) IB
IE
=

. A doua
Deoarece IA ID = pq IB IC (Lema 2), rezult
a c
a
IF
p+1
ID
egalitate rezult
a din prima si relatia IA ID = pq IB IC.

V
Fie ABCD un patrulater convex si
{I} = AC BD. Consider
am un punct M D
L T H
(AI) si adopt
am notatiile:{X} = BM AD,
P
H'

Q
{Y } = DM AB, apoi {N } = CY BD,
U

T'
I
{Q} = CX BD si, n sfrsit {U } = AN BC,
K'

S' M
{V } = AQ CD (fig. 4).
L'
X
N

G'
Consideram pozitia punctului M pe diagoS K
G
nala AC data de raportul
B
Y
A
AM
.
(4)
=
Fig. 4
MI
Cu usurinta putem preciza pozitiile punctelor X, Y , N , Q, U si V functie de
si elementele patrulaterului. ntr-adev
ar, aplic
am Lema 1 de dou
a ori n triunghiul
AX BD
AM
AY DB
AM
si
=

, adica
4ABD si obtinem relatiile:
MI
XD BI
MI
Y B DI
IB
AY
ID
AX
si
=
=
.
(5)
XD
BD
YB
BD
Pentru determinarea pozitiilor punctelor N si Q aplicam Lema 1 n 4CAB si 4CAD;
BN
BY CA
DQ
DX CA
si
si, tinnd seama de (5), vom avea
obtinem
=

NI
Y A CI
QI
XA CI
BN
1 AC BD
DQ
1 AC BD
=
,
=
.
(6)
NI
IC ID
QI
IB IC
BU AC
BN
=

,
Pentru punctele U si V aplic
am Lema 1 n 4ABC si 4ADC:
NI
U C AI
DV AC
DQ
=

. Din aceste relatii si (6) rezult


a c
a
QI
V C AI
1 BD IA
DV
1 BD IA
BU
=
,
=
.
(7)
UC
IC ID
VC
IB IC
Punem acum n evidenta cteva proprietati de coliniaritate ale configuratiei (fig. 4).

Propozitia 1. Punctul P definit de {P } = BV DU se afla pe diagonala AC.


Demonstratie. A arata ca P AC revine la a arata ca n 4BCD cevianele
DU , BV si CI sunt concurente. Datorita relatiilor (7), avem
IB IC ID
BU CV DI
1 BD IA

=1
U C V D IB
IC ID
BD IA IB
10

si aplic
am reciproca teoremei lui Ceva.
Observatie. Octogonul stelat AY BU CV DX, generat pornind de la punctul
M , este nscris n patrulaterul convex dat avnd patru din vrfurile sale tocmai
vrfurile patrulaterului, iar celelalte patru vrfuri, ce alterneaza cu acestea, fiind
situate pe laturile patrulaterului. Propozitia precedenta si cele care vor urma pun n
evidenta faptul c
a punctul I (de intersectie a diagonalelor patrulaterului) joac
a un
rol important pentru acest octogon stelat.
Propozitia 2. Punctele din fiecare dintre tripletele X, I, U si Y , I, V sunt
coliniare.
Demonstratie. Conform Lemei 2, pentru ca punctele X, I, U sa fie coliniare
este suficient ca egalitatea urmatoare sa fie ndeplinita:
XD U C
ID IC =

IA IB.
XA U B
Dar, tinnd sema de (5) si (7), egalitatea devine
IC ID
1 BD

IA IB 1 = 1.
ID IC =
IB
BD IA
La fel se arat
a c
a Y , I, V sunt coliniare.
Observatie. Triunghiurile XBC si U DA sunt omologice si punctul I este centrul
lor de omologie. Conform teoremei lui Desargue, perechile de drepte (BC, DA),
(XB, U D) si (XC, U A) au puncte de intersectie coliniare. Observatie similara relativ
la triunghiurile Y CD si V AB.
Introducerea punctelor G, G0 , H, H 0 , K, K 0 , L, L0 , S, S 0 , T , T 0 rezulta din fig. 4.
Propozitia 3. Punctele G, I, H sunt coliniare. Aceeasi proprietate o au si
punctele G0 , I, H 0 .
Demonstratie. Conform Lemei 2, aplicat
a patrulaterului Y BV D si punctelor
G si H, pentru coliniaritatea punctelor G, I, H este suficient s
a ar
at
am c
a
GD HV
ID IV =

IY IB.
(8)
GY HB
Cu Lema 3, aplicat
a relativ la patrulaterul ABCD si punctele Y si V , avem
q + 1 IB
AY
DV
IY
si q =
=p

, unde p =
.
IV
p + 1 ID
YB
VC
1 BD IA
ID
si, dup
,q=
a calcule, urmeaz
a c
a
Din (5) si (7), avem c
ap=
BD
IB IC
IB IC + IA BD
IY
=
.
(9)
IV
IC ( ID + BD)
Pe de alt
a parte, utiliznd Lema 1 n 4ADB si 4CBD, obtinem
N D AB
HB
QB CD
GD
si
=

.
(10)
GY
N B AY
HV
QD CV
Din nou apelnd la relatiile (5) si (7), g
asim
AB
ID + BD
CD
IB IC + IA BD
=
,
=
.
(11)
AY
ID
CV
IB IC
11

N D QB
si
prin elementele patrulaterului dat, le scriem
N B QD
ND
NI
ID QB
QI
IB
si apoi folosim relatiile
mai nti sub forma
=
+
,
=
+
NB
NB
N B QD
QD QD
(6); se obtine
ND
ID (IC + AC)
QB
IB (IC + AC)
si
=
=
.
(12)
NB
IB AC
QD
ID AC
Pentru a exprima rapoartele

nlocuind factorii prezenti n (8) cu expresiile lor date de (9) (12) se ajunge la
egalitatea 1 = 1. n concluzie, punctele G, I, H sunt coliniare. Similar se obtine ca
si punctele G0 , I, H 0 sunt coliniare.
n aceeasi maniera, adica folosind lemele si formulele (4) (7), se dovedesc si
rezultatele urm
atoare.
Propozitia 4. Punctele K, I, L sunt coliniare. Sunt coliniare de asemenea si
punctele K 0 , I, L0 .
Propozitia 5. Punctele S, I, T si punctele S 0 , I, T 0 sunt triplete de puncte
coliniare.
Observatie. ntruct propriet
atile precedente sunt de natur
a proiectiv
a nu este
lipsit
a de interes stabilirea acestora cu mijloacele geometriei proiective.
Pentru configuratia n discutie pot fi puse n evidenta si propriet
ati de alt
a natur
a.
Propozitia 6. Daca patrulaterul ABCD dat este paralelogram, atunci patrulaterul XY U V este un paralelogram cu laturile paralele cu diagonalele celui dat.
Afirmatia reciproca este de asemenea adevarata (fig. 4).
Demonstratie. Faptul c
a ABCD este paralelogram este echivalent cu
IA = IC si IB = ID.
(13)
AX
AY
Din (13) si (5) deducem c
a
=
, adic
a XY k BD; din (13) si (7) rezult
a
XD
YB
BU
DV
AX
CV
ca
=
, adica U V k BD. La fel deducem ca
=
((13), (5) si (7))
UC
VC
XD
VD
AY
CU
si
=
((13), (5) si (7)); ca urmare, XV k AC si Y U k AC. n concluzie,
YB
UB
XY U V este paralelogram. Afirmatia reciproca se dovedeste pe cale inversa.
Corolar. Dndu-se un paralelogram ABCD si un punct M (AD), sa se
construiasca numai cu rigla (negradata) un paralelogram nscris n acesta si avnd
punctul X ca unul dintre vrfurile sale.
Solutie. Cu rigla construim punctul M ca intersectie a dreptelor BX si AC.
Pornind de la M construim cu rigla punctele Y , U , V asa cum s-a procedat la
nceputul acestei note. Conform Propozitiei 6, XY U V ndeplineste conditiile cerute.
Bibliografie
1. D. Mihalca, I. Chitescu, M. Chirita
- Geometria patrulaterului, Teora, 1998.

12

O constructie geometric
a a unor medii
Claudiu Stefan
POPA1

Se stie ca n orice trapez, lungimea liniei mijlocii este media aritmetica a lungimilor
bazelor, iar lungimea segmentului care se sprijina pe laturile neparalele, trece prin
intersectia diagonalelor si este paralel cu bazele, este media armonic
a a lungimilor
bazelor. De asemenea, o paralel
a la baze ce mparte o latur
a neparalel
a n raportul
m

a n interiorul trapezului un segment a c


arui lungime este
, m, n N , determin
n
media ponderat
a a bazelor cu ponderile m si n (toate aceste rezultate pot fi g
asite,
spre exemplu, n [2]).
n lucrarea [1] se demonstreaza ca un segment paralel cu bazele si avnd ca
lungime media geometrica a acestora este situat "ntre" linia mijlocie si segmentul
paralel cu bazele ce trece prin intersectia diagonalelor. Ne propunem n continuare
s
a d
am o constructie efectiv
a a segmentului paralel cu bazele, de lungime media geometrica a acestora, bazndu-ne pe un rezultat interesant n sine si pe care autorul
nu l-a mai ntlnit n literatura de specialitate.
Propozitie. Fie ABCD un trapez, AB k CD, {O} = AC BD si fie S (AB),
R (CD) astfel nct RS k AD. Atunci O RS daca si numai daca AS este media
geometrica a lungimilor segmentelor [CR] si [BS].
Demonstratie. S
a presupunem c
a O RS si fie
D
R C
P (AD), Q (BC) astfel ca P Q k AB, O (P Q).
Se stie ca P O = OQ si atunci OQRD si OQSA sunt
O
paralelograme (OQ k DR k AS, OQ = DR = AS).
Q
P
Urmeaz
a c
a RQ k BD, SQ k AC. Aplicnd teorema lui Thales n 4CDB si 4BAC, obtinem c
a
BQ
BS
DR
si, cum DR = SA, rezult
=
=
a c
a
RC
QC
SA
SA2 = CR BS.

Reciproc, presupunem c
a RD = SA = SB RC.
S
A
B
Not
am SB
= a, RC = b si atunci AB = a + ab,
Fig. 1
CD = b+ ab. Deoarece lungimea segmentului paralelel cu bazele ce trece prin intersectia diagonalelor este media armonic
a a lungimilor
bazelor, avem c
a



2

a
+
ab
b
+
ab
a
b
a
+
b

AB CD

PQ = 2
=2
= 2 ab,
=2

AB + CD
a + b + 2 ab
a+ b
prin urmare P O = OQ = AS = DR, adica P ORD si P OSA sunt paralelograme.
De aici, OR k AD, OS k AD, deci O RS.

Drept consecinta a acestui rezultat obtinem un procedeu de constructie a mediei


geometrice si a mediei patratice ale bazelor unui trapez, ca segmente ce se sprijina pe
laturile neparalele si sunt paralele cu bazele.
1

Profesor, S
coala "Alecu Russo", Iasi

13

Fie, ca n figur
a, [P Q] si [M N ] segmentele ce
reprezinta media armonica, respectiv media aritmetica ale bazelor trapezului ABCD. Obtinem
un segment avnd ca lungime media geometric
aa
bazelor, folosind unul dintre procedeele cunoscute
([2]). Asezam acest segment n prelungirea uneia
dintre bazele trapezului; punctul D0 astfel obtinut
l unim cu B si fie {E} = BD0 AD. Paralela
prin E la baze determin
a segmentul [EF ] c
autat.

D'

D
P
E

M'

C
O

M
K
X

Fie XY simetrica dreptei P Q fata de M N , cu


X AD, Y BC. Atunci
2 AB CD
XY = 2 M N P Q = AB + CD
=
A
AB + CD
2
2
2
Fig. 2
(AB + CD) 2 AB CD
AB + CD
=
=

AB + CD
AB + CD
s
r
AB 2 + CD2
AB 2 + CD2 AB + CD

= XY M N .
2
AB + CD
2

Q
F
N
L
Y
B

Prin urmare, constructia cu rigorile impuse initial a segmentului de lungime media


patratica a bazelor trapezului ABCD se reduce la constructia segmentului [KL]
avnd ca lungime media geometrica a bazelor trapezului XY M N (fig. 2).
Propunem spre rezolvare urmatoarele probleme:
1. Fie ABCD trapez cu
[AB] baza mare si A0 (AB), D0 (DC astfel nct
0
0
0
a se demonstreze c
a:
C (DD ) si AA = DD = AB CD. S
i) A0 C, BD0 si AD sunt drepte concurente;
ii)
daca E este punctul de concurenta de la i) si EF k AB, F BC, atunci
EF = AB CD.
2. Fie ABCD un trapez, AB k CD si punctele E (AD) si F BC astfel
nct EF k AB. Trapezele
ABF E si EF CD au diagonalele respectiv paralele dac
a

si numai dac
a EF = AB CD.
3. Fie trapezul ABCD cu baza mare AB, AC BD = {O}, {M, E, P } (AD),
{N, F, Q} (BC) astfel nct [M N ] este linia mijlocie a trapezului, EF k AB si
AABY X
, atunci
EF 2 = AB CD, iar P Q k AB, O P Q. Daca notam RXY =
ACDXY
2
REF = RMN RP Q .
Bibliografie.
1. L. Constantinescu - O interpretare geometrica a inegalitatii mediilor, R. M. T. nr.
1/1982.
2. J. Hadamard - Geometrie plana, Ed. Tehnica, Bucuresti, 1960.

14

Irelevanta primitivabilit
atii pentru ecuatii
functionale de forma f (x + y) = g (f (x) , f (y))
Dan Stefan
MARINESCU si Viorel CORNEA1

n [1] este demonstrat urm


atorul rezultat:
"Daca f : R R admite primitive si f (x + y) = f (x) + f (y) pentru orice
x, y R, atunci f este continua".
De remarcat c
a demonstratia se bazeaz
a pregnant pe primitivabilitatea functiei
f.
n cele ce urmeaz
a vom dovedi urm
atoarea
Propozitie. Fie g : R R o functie arbitrara si f : R R o functie care
admite primitive. Daca
f (x + y) = g (f (x) , f (x)) ,

x, y R,

atunci f este continua.


Demonstratie. Daca f este injectiva, mpreuna cu faptul ca f are proprietatea
lui Darboux, suntem condusi la monotonia functiei f pe R. Se deduce imediat continuitatea functiei f , ceea ce ncheie demonstratia.
Dac
a f nu este injectiv
a, vom dovedi c
a f este constant
a pe R si, n consecinta,
f este continua. Din faptul ca f nu este injectiva exista a, b R cu a < b si
f (a) = f (b). Ar
at
am c
a
> 0, t1 , t2 R cu 0 < t2 t1 < si f (t1 ) = f (t2 ) .
(1)
b

a
Pentru aceasta fie n N nct
< 1 si functia
n

ba
f (x) , x R.
h : R R, h (x) = f x +
n
0
Evident,
h are
proprietatea lui Darboux: h = G , unde G : R R, G (x) =

ba
F x+
F (x), x R, cu F o primitiva a lui f .
n
Dac
a h nu se anuleaz
a pe R, cum h are proprietatea lui Darboux, avem h (x) < 0,
sau h (x) > 0, x R. Fie h (x) > 0, x R; atunci

ba
ba
ba
> 0, h a + 2
> 0, . . . , h a + (n 1)
> 0,
h (a) > 0, h a +
n
n
n
de unde

ba
f a+
f (a) > 0,
n

ba
ba
> 0,
f a+2
f a+
n
n
......................................................
1

Profesori, Liceul Teoretic "Iancu de Hunedoara", Hunedoara

15

ba
ba
> 0,
a+n
f a + (n 1)
n
n
relatii care adunate conduc la f (b) > f (a), ceea ce este fals.

ba
n consecinta, exista c R astfel ca h (0) = 0, adica f c +
= f (c); lund
n
ba
ba
t1 = c, t2 = c +
avem f (t1 ) = f (t2 ) cu 0 < t2 t1 =
< , adica (1) este
n
n
dovedit
a.
Aratam n continuare ca t2 t1 este perioada a functiei f . n adevar,
f

f (x + t2 t1 ) = f (x t1 + t2 ) = g (f (x t1 ) , f (t2 )) =
= g (f (x t1 ) , f (t1 )) = f (x t1 + t1 ) = f (x) .

Din aceasta si (1) deducem c


a exist
a un sir (an )nN de numere reale cu an > 0,
an 0 si
(2)
f (x + an ) = f (x) , x R, n N.
Fie F o primitiva a functiei f si n N. Definim functia
F (x + an ) F (x)
, x R.
Fn : R R, Fn (x) =
an
Evident Fn este derivabila si Fn0 (x) = 0 (n conformitate cu (2)), adica Fn este
constanta pe R; deci Fn (x) = Fn (0), x R, adica
F (an ) F (0)
F (x + an ) F (x)
=
, x R,
an
an
de unde, prin trecere la limita pentru n tinznd la +, gasim f (x) = f (0), x R.
Asadar, f este constant
a pe R. Cu aceasta demonstratia este ncheiat
a.
Corolar. Fie g : RR R o functie, P (R) = {f : R R | f admite primitive},
C (R) = {f : R R |f continua} si ecuatia functionala
f (x + y) = g (f (x) , f (y)) ,

x, y R.

(3)

Atunci ecuatia (3) are solutie n P (R) daca si numai daca are solutie n C (R).
Demonstratie. Dac
a (3) are solutie n C (R), atunci, evident, are solutie n
P (R). Reciproc, daca (3) are solutie n P (R), atunci conform Propozitiei are solutie
n C (R).
Bibliografie
1. S. R
adulescu, P. Alexandrescu, M. Chirita
- Olimpiada locala, Bucuresti, 1996.

16

Asupra problemei XII.32 din RecMat - 2/2002


Marian TETIVA1
n num
arul 2/2002 al Recreatiilor Matematice a fost publicat
a problema
XII.32. Fie (G, ) grup, iar a G\{e} fixat. Aratati ca numarul morfismelor
2 x = a este egal cu
surjective de la G la (Z3 , +) cu proprietatea ca f (x) = b
numarul subgrupurilor H ale lui G care nu-l contin pe a si care au proprietatea ca
x3 H, x G.
Dana Stan, elev
a, Iasi
Ideea autoarei era stabilirea unei corespondente bijective ntre multimile
o
n
2x=a ;
A = f : G Z3 | f morfism surjectiv cu f (x) = b

B= HG|a
/ H si x3 H, x G .

0). Avem c
a
Pentru aceasta, se defineste functia F : A B prin F (f ) = f 1 (b
0, deci
f 1 (b
0) = Ker f este subgrup n G, a
/ f 1 (b
0), iar f x3 = 3f (x) = b
x3 f 1 (b
0), x G; prin urmare, F este bine definita. Daca F (f ) = F (g), atunci
f 1 (b
0) = g 1 (b
0). n plus, f 1 (b
2) = g 1 (b
2), iar f 1 (b
1) = g 1 (b
1) din surjectivitatea
functiilor f si g; n concluzie, F este injectiv
a.
Ramne sa dovedim ca F este surjectiva. Fie H B; este normal sa consideram
fH : G Z3 data prin fH (x) = b
0, x H, fH (a) = b
2, iar fH (x) = b
1, x
G \ (H {a}). Avem c
a F (fH ) = H, fH este surjectiv
a, iar fH (x) = b
2 x = a.
Este ns
a aplicatia fH astfel definit
a morfism de grupuri? Dovedirea acestui fapt
impune studierea modului n care conditia din ipoteza influenteaza structura grupului
G. Acest studiu va releva ca ipoteza poate fi slabita, concluzia poate fi mbunatatita
si va permite obtinerea unei generaliz
ari interesante a problemei n discutie.
Fie deci f : G Z3 morfism de grupuri astfel nct exist
a a G \ {e} cu
proprietatea
f (x) = b
2x=a
()
3
2
b
b
b
b
b
Avem atunci f (a) = 2, apoi f a = 2 + 2 = 1 si f a = 0; prin urmare, conditia

2 si din ()
de surjectivitate impus
a lui f este superflu
a. n continuare, f a4 = b
obtinem c
a a4 = a, adic
a a3 = e. Pe de alt
a parte, fie b G astfel nct f (b) = b
0;
avem succesiv:
f (ab) = f (a) + f (b) = b
2+b
0=b
2 ab = a b = e.

Iar dac
a se consider
a c G cu f (b
c) = b
1, atunci
2
5
2
5
b
f c = f c = 2 c = c = a c2 = a si c3 = e ac = e c = a2 .

a un singur morfism
n concluzie, G = e, a, a2 , cu a element de ordin 3, deci exist
2
b
b
b
ca n enunt (f (e) = 0, f (a) = 2, f a = 1) si un singur subgrup H cu proprietatile
cerute, H = {e} (oricum, si morfisme si subgrupuri sunt cte doua de toate!).
Iata acum generalizarea anuntata:

Profesor, Colegiul National "Gh. Rosca Codreanu", Brlad

17

Propozitie. Fie (C, ) un grup ciclic si g C un generator al sau. Fie (G, ) un


alt grup (notam la fel operatiile celor doua grupuri, pentru simplitate). Presupunem
ca exista un morfism f : G C si un element a G, a 6= e, astfel nct f (x) = g
x = a. Atunci G este izomorf cu C.

Demonstratie. n primul rnd, f (a) = g f ak = g k , k Z. Acum,
fie x G un element oarecare. Trebuie sa avem f (x) = g k , pentru un anume

1k
k Z. Rezult
a (f fiind morfism) c
a f a1k x = f (a)
f (x) = g 1k g k = g.
Dar singurul element din G pentru care f ia valoarea g este a, deci n mod necesar
avema1k x = a x = ak . Asadar orice element al lui G este o putere a lui a:
G ak | k Z .

Din f ak = g k , k Z, rezult
a c
a f este
o aplica
surjectiv
a. n cazul n care C
tie
n+1
n
n+1

=g
este finit si are n elemente avem g = e si f a
= g an+1 = a an = e
k
(e fiind elementul neutru din C), deci G a | k = 0, 1, . . . , n 1 si cum exist
a
o surjectie de la G la C, obligatoriu G are tot n elemente si, evident, este izomorf
cu C.
Daca C este infinit nu putem avea ak =am pentru exponentii ntregi distincti k si
m, deoarece asta ar nsemna si g k = f ak = f (am ) = g m , ceea ce este imposibil, g
fiind acum de ordin infinit. Deci si n acest caz G este izomorf cu C, fiind acum grup
ciclic infinit (generat tot de a, dar acesta nu mai are acum ordin finit). Demonstratia
este ncheiata.

Recreaii matematice
1. Patru oameni a, b, c si d vor sa traverseze ntr-o noapte un pod, venind toti
din aceeasi directie. Pentru a ajunge toti de cealalt
a parte, au la dispozitie 17 minute
si doar o lantern
a. Podul sustine cel mult 2 oameni odat
a si orice echip
a care trece
podul, de unul sau doi oameni, trebuie sa aiba lanterna cu ei. Lanterna trebuie
sa fie transportata nainte si napoi, deci, ea nu poate fi aruncata etc. Se stie ca
a traverseaz
a podul ntr-un minut, b n dou
a minute, c n cinci minute, d n zece
minute, iar o echip
a traverseaz
a podul cu viteza celui mai lent dintre componentii ei.
Cum procedeaza cei patru pentru a trece podul n timpul stabilit?

2. Dac
a v
a puteti imagina c
a desenul al
aturat reprezint
a un taur care se uit
a spre est, schimbati pozitia a
dou
a segmente astfel nct acesta s
a se uite spre vest.

Not
a. Solutiile problemelor 1 si 2 se pot g
asi la pagina 43.
18

Combinatoric
a . . . algebric
a
Gabriel DOSPINESCU 1
Expansiunea combinatoricii n concursurile de matematic
a de orice nivel impune
cunoasterea unor procedee si metode ct mai variate de abordare a problemelor de
acest fel. Scopul acestei note este prezentarea, pe un numar de exemple, a modului
cum pot fi utilizate unele mijloace algebrice: numere complexe, polinoame etc. n
rezolvarea problemelor de combinatoric
a.
Cardinalul unei multimi A va fi notat |A|. Daca A este o multime de numere
naturale, atunci suma elementelor acesteia se noteaza m (A) si se numeste masura
lui A (prin conventie, m () = 0). Multimea A se numeste par
a / impar
a dac
a m (A)
este num
ar par / impar ( este par
a).
Vom utiliza n mod frecvent urmatoarea
2
2
Lem
a. Fie = cos
+ i sin . Are loc egalitatea
n
n
a0 + a1 + . . . + an1 n1 = 0,

a0 , a1 , . . . , an1 R,

daca si numai daca a0 = a1 = . . . = an1 .


Demonstratie. Definim polinoamele f, g R [X] prin f = a0 + a1 X + . . . +
+an1 X n1 si g = 1 + X + . . . + X n1 . Daca f , g au radacini comune, atunci (f, g)
divide g. Deoarece g este ireductibil n R [X], rezulta (f, g) = g, adica g | f , si, deci,
g = kf , k R. n consecinta, avem a0 = a1 = . . . = an1 . Reciproca este evident
a.
1. Cte numere de n cifre 2, 3, 7 sau 9 se divid cu 3? (Concursul "Traian
Lalescu", 2003)
Solutie. Fie xn , yn si zn numarul numerelor cu n cifre 2, 3, 7 sau 9 congruente
2
2
cu 0, 1 si respectiv 2 modulo 3. Se cere sa se afle xn . Fie = cos
+ i sin
.
3
3
X
n
2
a1 +...+an
Evident, xn + yn + zn = 4 si xn + yn + zn =

a1 ,...,an {2,3,7,9}

2
3
7
9 n
2
= 1. Deci xn 1 + yn + zn = 0, de unde rezult
a c
a
= + + +
1 n
n
xn 1 = yn = zn = k. Atunci 3k = xn +yn +zn 1 = 4 1, de unde k = (4 1).
3
1
Ca urmare, xn = (4n + 2).
3

2. Fie Sn = {1, 2, . . . , 2n} si An (Bn ) familia submultimilor pare (impare) ale


multimii Sn , avnd n elemente. Sa se determine |An | |Bn |.X
(Polonia, 2001)
(1)m(A) . Dar
Solutie. Ideea esentiala este ca avem |An | |Bn | =
ultima expresie este coeficientul lui X n n dezvoltarea

ASn , |A|=n
2n
Yh
i

i=1
1

Elev, Liceul "Dimitrie Cantemir", Onesti

19

i
1 + (1) X . Cum acest

n
n
produs este egal cu (1 + X) (1 X) = 1 X 2 , deducem c
a |An ||Bn | estecoe-

n
n
2 n
, adica este 0 pentru n impar si (1)n/2
ficientul lui X n dezvoltarea 1 X
n/2
pentru n par.
3. Fie p un numar prim impar, numerele naturale m si n divizibile cu p, iar n
impar. Pentru fiecare m -upla (c1 , . . . , cm ), unde ci {1, 2, . . . , n}, cu proprietatea
m
X
ci , consideram produsul c1 . . . cm . Sa se demonstreze ca suma acestor
ca p |
i=1
m
n
. (Gabriel Dospinescu)
produse este divizibila cu
p
X
Solutie. Pentru orice k {0, 1, . . . , p 1}, fie xk =
c1 . . . cm , suma
m
X
lundu-se dupa toate m-uplele (c1 , . . . , cm ) pentru care
ci k (mod p). Lund
i=1

2
2
= cos
+ i sin , avem
p
p

+ 22 + . . . + nn
=

c1 ,...,cm {1,2,...,n}

c1 . . . cm c1 +...+cm =

p1
X

xk k .

k=0

Cum, printr-un calcul simplu, se obtine


+ 22 + . . . + nn =

nn+2 (n + 1) n+1 +
2

( 1)

rezulta ca

n
,
1

p1

X
nm
xk k .
m =
( 1)

(1)

k=0

Pe de alt
a parte, din p1 + . . . + + 1 = 0 deducem c
a

1
1
= p2 + 2p3 + . . . + (p 2) + p 1
1
p
si obtinem
m

p2
m
nm
n

=
+ 2p3 + . . . + (p 2) + p 1 .

m
p
( 1)
Scriind
p2
m
X
+ 2X p3 + . . . + (p 2) X + (p 1) = b0 + b1 X + . . . + bm(p2) X m(p2) ,
deducem ca

unde yj =

nm
=
( 1)m
X
bk .

y0 + y1 + . . . + yp1 p1 ,
p

kj(mod p)

Din (1) si (2), obtinem relatia


x0 ry0 + (x1 ry1 ) + (x2 ry2 ) 2 + . . . + (xp1 ryp1 ) p1 = 0,
20

(2)

m
n
. Din aceasta rezulta ca x0 ry0 = . . . = xp1 ryp1 = k.
p
Ramne sa aratam ca r | x0 . Este suficient sa aratam ca r | k. Dar

unde r =

pk = x0 + . . . + xp1 r (y0 + . . . + yp1 ) =

m
= (1 + 2 + . . . + n) r b0 + . . . + bm(p2) =
m
m
= (1 + 2 + . . . + n) r (1 + 2 + . . . + (p 1)) .

m
m

n (n + 1)
p (p 1)
si, cum membrul drept se divide cu pr,
Deci pk =
r
2
2
rezulta ca r | k.
4. Fie n, a1 , a2 , . . . , am N si f (k) numarul m-uplelor (c1 , . . . , cm ) pentru care
m
X
1 ci ai si
ci k (mod n). Sa se arate ca f (0) = f (1) = . . . = f (n 1)
i=1

daca si numai daca exista un indice i astfel nct n | ai . (Rookie Contest, 1999)
Solutie. Sa observam ca au loc relatiile
m
X
Y

X + X 2 + . . . X ai =
X c1 +c2 +...+cm si
i=1

1ci ai

f (0) + f (1) + . . . + f (n 1) n1 =

c1 +...+cm =

1ci ai

m
Y

i=1

+ 2 + . . . + ai ,

2
2
+ i sin
. Atunci f (0) = f (1) = . . . =
n
n
= f (n 1) este echivalent cu f (0) + f (1) + . . . + f (n 1) n1 = 0, deci cu
m
Y

+ 2 + . . . + ai = 0. Ultima realtie are loc dac


a si numai dac
a exist
a un

unde am notat n mod firesc = cos

i=1

indice i astfel nct + 2 + . . . + ai = 0 si, ca urmare, ai = 1, adic


a n | ai .

5. Fie p un numar prim si A = {1, 2, . . . , 2p}. Sa se determine numarul multimilor B A cu p elemente si avnd proprietatea ca p | m (B). (OIM - 1995,
Polonia)
2
2
+ i sin
Solutie. Cazul p = 2 fiind banal, vom considera p 3. Fie = cos
p
p
si xj numarul multimilor B A cu p elemente si pentru care m (B) j (mod p).
Atunci
p1
X
X
X
xj j =
m(B) =
c1 +...+cp ,
j=0

1c1 ...cp 2p

BA, |B|=p
p

2p .
ultima sum
a fiind coeficientul lui X n dezvoltarea
(X + ) X + 2 . . . X +

Cum X p 1 = (X 1) (X ) . . . X p1 , deducem c
a (X + ) X + 2 . . .
p1
X

2
xj j = 2 sau
X + 2p = (X p + 1) si, deci, coeficientul lui X p este 2. Asadar,
j=0

21

x0 2 + x1 + . . . + xp1 p1 = 0, deunde
a c
a x0 2 = x1 = . .
. = xp1
k.
=
rezult
1
2p
2p
Urmeaz
a c
a pk = x0 +. . .+xp1 2 =
2 .
2. n concluzie, x0 = 2+
p
p
p
6. Fie A o multime de numere naturale ce con
tine cel putin doua numere impare.
X
(1)m(B) mk (B) = 0 ( B poate
Sa se arate ca pentru k {0, 1, 2} are loc relatia
BA

fi sau A). (Gabriel Dospinescu)


Solutie. Pornim de la observatia c
a
Y
X
(1 + X a ) =
X m(B) .
aA

n (1) facem x = 1 si deducem ca

BA

(1)m(B) =

BA

(1)
(1 + (1)a ) = 0. Derivam

aA

relatia (1) si obtinem (notnd A = {a1 , ..., ap })


X
X
m(B)X m(B) .
a1 X a1 (1 + X a2 ) . . . (1 + X ap ) =

(2)

BA

n (2) facem x = 1 si deducem


a din ipotez
a fiecare termen al sumei
X(tinnd cont c
m(B)
din membrul stng este 0) ca
m(B)(1)
= 0. Derivam apoi (2) si obtinem

analog ca

BA

(1)m(B) m2 (B) = 0.

BA

n final propunem spre rezolvare pe aceeasi cale problemele urm


atoare. Pentru
a realiza ca aceasta metoda merita sa fie cunoscuta, sugeram ca att problemele
anterioare ct si cele urmatoare sa fie rezolvate si "clasic".
7. Fie an numarul submultimilor B {1, 2, ..., 6n} pentruY
care m(B) 5 (mod 6)
si bn numarul submultimilor C {1, 2, ..., 7n} pentru care
x 5 (mod 7). Sa se
xC

an
. (Polonia)
bn
8. Sa se calculeze suma elementelor submultimilor lui {1, 2, ..., 3n} care au masura
multiplu de 3. (Gabriel Dospinescu)
9. Fie A = {1, 2, .., n}. a) Sa se arate ca familiile submultimilor pare si impare
ale lui A au acelasi numar de elemente si aceeasi suma a masurilor elementelor.
b) Sa se afle suma masurilor submultimilor pare ale lui A. (Test de selectie,
1994)
determine

22

Un procedeu de calcul al limitelor unor siruri


de forma (an+1 an )n1
1
Oana CRJA

1. Scopul acestei note este prezentarea unei scheme de calcul al limitelor unor
siruri de forma (an+1 an )n1 , schem
a desprins
a din solutia dat
a de M. Tena

relativ
la sirul lui Lalescu [1, p. 443].
Baza teoretica a acestei scheme este data de urmatoarea
Propozitie. Fie (an )n1 un sir de numere strict pozitive ce verifica conditiile:

n
an+1
an
an+1
= 1; (ii) lim
= R.
(i) lim
= (0, ) ; (iii) lim
n an
n n
n
an
Atunci lim (an+1 an ) = ln .
n
Demonstratie. Avem:

an+1
an

n (an+1 an )
an
an

an+1 an an+1 an

, n 1,
= 1+

an

de unde, prin logaritmare, obtinem:

an

an+1 an an+1 an
an
an+1
(an+1 an ) ln 1 +
=
, n 1.
ln
an
n
an
an+1 an
Din (i) rezulta imediat ca lim
= 0 si, deci,
n
an
an

an+1 an an+1 an
= 1.
lim ln 1 +
n
an

(1)

(2)

Relatiile (1), (2), (ii) si (iii) arat


a c
a exist
a lim (an+1 an ), finit
a sau nu, si, prin
n

trecere la limita n (1), obtinem ca aceasta limita este ln , q.e.d.

Observatie. Mention
am faptul c
a pentru calculul limitei de la punctul (iii)
(unde apare o nedeterminare de tipul 1 ) nu putem utiliza limita fundamentala
1/x
= e, ci trebuie procedat n alt mod.
corespunz
atoare, anume lim (1 + x)
k

2. Aplicatie la siruri remarcabile.


Avem
n vedere urm
atoarele trei siruri:
p
lui T.Lalescu Ln = n+1 (n + 1)! n n!, n 2.
1 Sirul

Notam an = n n!, n 2 si obtinem:

n
an
n!
1
lim
= lim
= ;
n n
n n
p e
p
n+1
(n + 1)!
1
an+1
n n+1 (n + 1)! n + 1

= lim
= lim
= e 1 = 1;
lim
n
n an
n
n n n!
n+1
n
e
n!
1

Elev
a, cl. a XI-a, Colegiul National "C. Negruzzi", Iasi

23

!n
p

n/(n+1)
(n + 1)!
n+1

lim
= lim
=
lim
= e.
n
n
n
n
n
n!
n!
1
1
Conform Propozitiei, lim Ln = ln e = .
n
e
e

lui R. T. Ianculescu In = (n + 1) n+1 n + 1 n n n, n 2 [1, p. 521].


2 Sirul

an
an+1
Fie an = n n n, n 2. Se obtine imediat ca lim
= 1. Apoi,
= 1 si lim
n n
n an

n
n

1
n+1
an+1
(n + 1) n+1 n + 1
n+1

lim
=e
= lim
= lim
n+1

n
n
n
an
n
n
nnn
n+1
si, deci, lim In = 1 ln e = 1.

an+1
an

n+1

nn1
(n + 1)n

n2 , n 2 [1, p. 520].
n1
n
(n 1)
Luam an = nn1 / (n 1)n2 , n 2. Prin calcul, gasim = = e (cu notatiile din
Propozitie). Ca urmare, lim Gn = e.
lui M. Ghermanescu Gn =
3 Sirul

3. Probleme de concurs. Cu Propozitia de mai pot fi calculate multe dintre


limitele ce sunt date la concursurile scolare.

1
1
1
1
a, 2003)
1 Calculati lim e1+ 2 +...+ n+1 e1+ 2 +...+ n . (Olimpiada local
n
1
an+1
1
1+ 12 +...+ n
si constatam ca lim
Luam an = e
= lim e n+1 = 1,
n an
n

n
n
a
an+1
1
1
n
1+
lim
= lim e n+1 = e si lim
= lim e 2 +...+ n ln n = eC , unde
n
n
n n
n
an

1
1
C = lim 1 + + . . . + ln n este constanta lui Euler. Deci, limita este eC .
n
2
n

2 Fie (xn )n1 si (yn )n1 doua siruri de numere reale strict pozitive astfel nct
xn+1
yn

si lim (yn+1 yn ) = c R. Demonstrati


, lim
= a R+
= b R+
lim
n xn
n n
n

atinetu-Giurgiu, M. S
omodi,
ca lim yn+1 n+1 xn+1 yn n xn = ac. (D. M. B
n
C:844, G. M.-11/1988)
yn+1 yn
yn+1
Observam ca lim yn = +, lim
= 0, lim
= 1 si
n
n
n yn
y
n

n
yn+1 yn
yn+1
= lim 1 +
= . . . = ec/b (calcul de rutin
a).
lim
n
n
yn
yn

a
an+1
yn+1 n+1 xn+1

= lim
= 1
Fie an = yn n xn ; obtinem: lim
= 1,

n x
n an
n yn
a

n n
an
yn
an+1
yn+1
xn+1
1
n x
= lim
=
lim
= lim

n+1

n = ba, lim
n n
n n
n
n
an
yn
xn
xn+1
= ec/b . Conform Propozitiei, limita ceruta este egala cu (ba) ln ec/b , adica este ac.
Propunem spre rezolvare urm
atoarele probleme selectate din G. M.: C:2062
(7-8/1998), 24628 (1/2002) si 24708 (5-6/2002).
Bibliografie
Ed. "Albatros", Bucuresti, 1979.
1. D. M. B
atinetu - Siruri,

24

Asupra unei probleme propuse


la Concursul Florica T. Cmpan, martie 2003
Horia-Nicolai TEODORESCU 1
La etapa judeteana a concursului amintit, s-a dat si urmatoarea problema (reprodusa aici sumar):
Pe un biliard dreptunghiular ABCD, o bila porneste din M catre AB si loveste o
alta bila n N . Unde loveste prima bila latura AB? Se cunosc coordonatele punctelor
M si N . (Autor necunoscut)
Baremul afisat pe usa scolii unde s-a desfasurat concursul preciza drept solutie
unic
a un punct de coordonate corespunz
atoare datelor problemei, (0, 2). Aceasta este
ns
a numai o solutie particular
a (este drept, singura care putea fi determinat
a precis
din datele problemei). Din figura alaturata se poate constata ca exista o infinitate
de solutii, obtinute prin ciocniri multiple fie si numai dupa laturile AB si respectiv
CD (semi-biliardul biliardul semi-infinit creat de dreptele AB si CD, f
ar
a laturile
AD si BC).

D
M

Deoarece la ciocnire traiectoria face unghiuri egale cu normala la suprafata de


ciocnire, din egalit
atile de unghiuri, obtinem, cu notatiile M (xM , yM ), N (xN , yN ):
xN xM = yM tg + yN tg + 2pa tg ,

unde p este num


arul de ciocniri cu latura CD, iar a este distanta dintre laturile AB
si CD. Deci,
xN xM = (yM + yN + 2pa) tg ,
de unde
xN xM
tg =
, = (p) , p = 0, 1, . . .
yM + yN + 2pa
In textul problemei, nu se da valoarea a, dar aceasta nu face sa dispara infinitatea
de solutii posibile dupa relatia de mai sus. Corespunzator fiecarei valori de unghi, se
determin
a punctele de lovire a dreptei AB, iar pentru p = 0, se obtine solutia indicat
a
n timpul concursului. Desigur, exist
a si alte familii de solutii, prin ciocniri numai
1

Prof. dr., Fac. de Electronic


a si Telecomunicatii, Univ. Tehnic
a "Gh. Asachi", Iasi

25

cu peretii AD si BC (strict similare cu cele discutate) sau obtinute prin ciocniri cu


toti cei patru pereti solutii pe care nu le analizam aici.
O anumita ambiguitate a textului se pare ca a produs ncurcaturi unor candidati.
Printre exprim
arile mai putin fericite se num
ar
a se ndreapt
a spre AB. In ce sens
ar trebui interpretat
a aceast
a afirmatie? Orice traiectorie n jos se ndreapt
a spre
AB, caci se apropie de AB distanta dintre punctul de pe traiectorie si AB scade.
Ar fi trebuit spus traiectoria punctului este initial dupa o dreapta, care intersecteaza
segmentul AB. Sau, mai simplu, bila se ciocneste nti de AB. Dac
a se mai f
acea
si precizarea se ciocneste o singur
a dat
a cu AB si nici o dat
a de alte margini ale
biliardului, nainte de a lovi bila din N , atunci problema ar fi fost nu numai mult
mai clara, dar ar fi avut ca unica solutie chiar solutia indicata participantilor la
concurs.
Unii candidati au fost nclinati s
a considere c
a bila ce porneste din M se loveste
nti de bila din N si abia apoi de marginea AB. Interpretarea nu era gresita, caci
textul problemei nu contrazice aceasta interpretare. Din nefericire, aceasta complica
problema, c
aci dou
a puncte materiale se misc
a dup
a ciocnire pe dreapta dup
a care
s-au miscat initial, dar n sens opus, deci bila din M urma s
a se ntoarc
a nti spre
CD (sau AD, CD functie de dimensiunile biliardului) si apoi spre AB! Daca
bilele se presupun nepunctuale, problema este nedeterminat
a (este necesar s
a se mai
precizeze unghiul de contact al bilelor la prima ciocnire); cu precizarea unghiului de
ciocnire, problema devine rezolvabil
a, dar dep
aseste nivelul de cunostinte de fizic
a la
clasa a VIII-a.
In concluzie, problema discutat
a avea prea multe neclarit
ati si imprecizii pentru
a fi inclus
a ca atare ntr-un concurs de nivel judetean, iar acordarea de note mari la
aceast
a problem
a probabil a l
asat pe unii elevi cu falsa impresie c
a au nteles si chiar
rezolvat corect (si complet) problema daca au dat solutia indicata n ziua concursului.
In final, cred c
a este meritoriu pentru Comisia concursului amintit c
a a dat atentie
unei probleme de tip biliard, dat fiind c
a domeniul biliardelor formale (n particular,
teoria biliardelor hiperbolice) este dintre cele mai profunde si fertile ast
azi n teoria
sistemelor ergodice si a sistemelor cu dinamica neliniara [1], [2]. Biliardele poligonale si eliptice n plan si cele cubice si tetraedrice sunt de altfel probleme clasice
n geometrie (vezi problema lui Alhazen, datnd din antichitate si intens studiat
a n
evul mediu, sau porismul lui Poncelet), cu multiple si importante aplicatii n fizic
a,
prea putin reflectate n literatura de specialitate pentru elevi de la noi din tara. Sa
astept
am deci si alte probleme despre biliarde n culegeri si n concursuri scolare.
Bibliografie
1. Lai-Sang Young - Developments in Chaotic Dynamics, Notices of AMS, 1998, vol.
45, nr. 10, pp. 13181328.
2. M. Hasewinkel - Encyclopedia of Mathematics, Vol. 1, pp. 406411 (Pessin Theory),
Kluwer Academic, 1997.

26

O problem
a . . . si opt solutii
Adrian ZANOSCHI 1
La prima editie a Concursului de matematic
a "Alexandru Myller", care a
avut loc la Iasi, n perioada 4 - 6 aprilie 2003, a fost propus
a elevilor de clasa a X-a
urm
atoarea problem
a [1]:
b=D
b = 90 si AB = AD.
Fie ABC si ADE doua triunghiuri dreptunghice cu B
Fie F proiectia lui B pe AC si G proiectia lui D pe AE. Sa se arate ca punctele
B, F , E sunt coliniare daca si numai daca punctele D, G, C sunt coliniare.
Problema, rezolvat
a corect de aproximativ 60% dintre concurenti, a prilejuit acestora etalarea unor variate tehnici si metode de geometrie sintetic
a, analitic
a si vectorial
a. V
a prezent
am n continuare opt dintre rezolv
arile care au fost date n concurs
si n afara lui, lasndu-va pe dumneavoastra sa decideti care este cea mai frumoasa.

Solutia I (sintetic
a). Aplicnd teorema
catetei n triunghiurile ABC si ADE, obtinem:
AF AC = AB 2 = AD2 = AG AE,
de unde rezulta ca AF AC = AG AE, adica
punctele F , C, E si G sunt conciclice. De aici,
\
deducem c
a CF
E = 90 dac
a si numai dac
a
\ = 90 , ceea ce nseamna ca punctele B, F ,
EGC
E sunt coliniare dac
a si numai dac
a punctele D,
G, C sunt coliniare. (Andrei S
tef
anescu, elev,
Colegiul National de informatica "Tudor Vianu",
Bucuresti)

D
G

C
F

Fig. 1

Solutia a II-a (sintetic


a). Vom demonstra mai nti urmatoarea
Lem
a. Fie un triunghi ABC si un punct M (BC). n aceste conditii, AM
este perpendiculara pe BC daca si numai daca:
AB 2 BM 2 = AC 2 CM 2 .

(1)

Demonstratie. I Dac
a AM BC, atunci din egalit
atile AB 2 BM 2 = AM 2 si AC 2 CM 2 = AM 2 ,
rezulta relatia cautata.
II Sa presupunem ca are loc relatia (1). Notnd cu
\
unghiul AM
B, avem

AB 2 = AM 2 + BM 2 2 AM BM cos ,
AC 2 = AM 2 + CM 2 + 2 AM CM cos ,

de unde, folosind egalitatea (1), deducem AM


BM cos = AM CM cos sau AM BC cos = 0,
deci = 90 .
1

Profesor, Colegiul National "C. Negruzzi", Iasi

27

M
Fig. 2

Revenind la problema noastr


a, s
a presupunem c
a B, F , E sunt coliniare. Aplicnd
Lema n triunghiurile ABC, ADE si ACE, obtinem
AB 2 AF 2 = CB 2 CF 2 ,

(2)

(3)

(4)

AD AG = DE EG ,

CE CF = EA F A .

Cum AB = AD, din (2) si (3) rezulta ca:

CB 2 CF 2 + AF 2 = DE 2 EG2 + AG2 ,

de unde, avnd n vedere relatia (4), gasim:


sau

CB 2 + EA2 CE 2 = DE 2 EG2 + AG2 ,

CB 2 + EA2 DE 2 AG2 = CE 2 EG2 ,

CB 2 + AD2 AG2 = CE 2 EG2 ,


CA2 AG2 = CE 2 EG2 ,

ceea ce nseamna, conform Lemei, ca CG AE. Prin urmare, punctele D, G, C


sunt coliniare. Reciproca se demonstreaz
a n mod analog. (Adrian Zanoschi)
Solutia a III-a (sintetic
a). Presupunem ca punctele B, F , E sunt coliniare.
Fie G0 proiectia punctului C pe AE. Din faptul ca 4AG0 C 4AF E rezulta ca
AG0
AC AF
AC
AB 2
=
sau AG0 =
=
(teorema catetei n 4ABC). Ca urmare,
AF
AE
AE
AE
AD2
= AG (teorema catetei n 4ADE), adic
a G0 coincide cu G si, deci,
AG0 =
AE
punctele D, G, C sunt coliniare. Implicatia invers
a se demonstreaz
a la fel. (Petru
Asaftei, prof., Scoala
Normala "V. Lupu", Iasi )

Solutia a IV-a (sintetic


a). Vom utiliza rezultatul urm
ator: Un patrulater
M N P Q (convex sau concav) este ortodiagonal daca si numai daca este ndeplinita
relatia M N 2 + P Q2 = M Q2 + N P 2 .
Daca punctele B, F , E sunt coliniare, atunci patrulaterul ABCE este ortodiagonal si are loc relatia AB 2 + CE 2 = AE 2 + BC 2 . Deoarece AB = AD,
AE 2 = AD2 +ED2 , BC 2 = AC 2 AB 2 = AC 2 AD2 , relatia se scrie AD2 +CE 2 =
= AC 2 + ED2 , adica patrulaterul ADEC este ortodiagonal. Deci D, G, C sunt coliniare. Mentionam faptul ca n conditiile E BF si E
/ [BF ) patrulaterul ABCE
este concav si c
a E [BF ] nu poate avea loc. Se arat
a la fel implicatia invers
a.
(Temistocle Brsan)
Solutia a V-a (vectorial
a). S
a presupunem c
a punctele B, F si E sunt coliniare.

n acest caz, avem BE AC, deci BE CA = 0. Sa aratam ca CG AE:



CG AE = CB + BA + AG AB + BE =





= CB AB + CB BE + BA AB + BA BE + AG AB + AG BE =

= CB BE AB 2 + BA BE + AG AB + AG BE =
28



= CB + BA BE AB 2 + AG AB + BE =


= CA BE AB 2 + AG AE = AB 2 + AG AE = AD2 + AG AE = 0.

De aici, avnd n vedere c


a DG AE, rezult
a c
a punctele D, G si C sunt coliniare. Cealalt
a implicatie se demonstreaz
a n mod analog. (Cristina Gutue, elev
a,
Colegiul National "I. C. Bratianu", Pitesti si Bianca Milatinovici, eleva, Liceul de
Informatica "Gr. C. Moisil ", Iasi )
Solutia a VI-a (vectorial
a). Fie xOy un sistem ortogonal de coordonate astfel

nct Ox s
a fie mediatoarea lui BD. Vom nota cu
r M vectorul de pozitie al unui
punct oarecare M (fata de polul O). Punctele B, F si E sunt coliniare dac
a si numai

r B ) (
r A ) = 0 sau
daca BE AC, adica (
r E
r C

r
r
r
(1)
r
r +
r =
r +
r .
B

r D ) (
r A ) = 0,
r E
Punctele D, G si C sunt coliniare dac
a si numai dac
a (
r C
sau

r E +
rA=
r C +
rD .
r D
rC
r E
rA
(2)

b
b
r A)
r D
Deoarece B = D = 90 , rezulta ca ( r B r A ) ( r C r B ) = 0 si (

r D ) = 0, deci
(
r E

r
r
r2
(3)
r
r +
r =
r +
B

si

r A
r E
r 2D .
(4)
r D
r E +
rD=
r A+

Datorita modului de alegere a sistemului de coordonate avem


r 2B =
r 2D si

r A
r A
rB =
r D si astfel, din (3) si (4), obtinem

r E
r D
r C
(5)
r C +
rA=
r E +
rA .
r B

Prin urmare, avnd n vedere relatia (5) si egalitatea r r = r r , rezulta


A

c
a relatiile (1) si (2) sunt echivalente, adic
a ceea ce trebuia demonstrat. (Adrian
Zanoschi)

Solutia a VII-a (cu numere complexe). Deoarece relatiile F BE si G DC


sunt echivalente cu BE AC si respectiv cu DC AE, problema noastra revine la
a ar
ata echivalenta BE AC DC AE.
Consider
am un reper ortogonal n plan cu originea n A si not
am cu b, c, d si e
afixele punctelor B, C, D si E. Cum AB = AD, rezult
a c
a |b| = |d|. Dac
a not
am
cu x y (x, y C) produsul real al numerelor complexe x si y, avem:

BE AC (e b) c = 0 (e b) c + e b c = 0 bc + bc = ce + ce,
(1)

DC AE (c d) e = 0 (d c) e + d c e = 0 de + de = ce + ce, (2)

BC AB (c b) b = 0 (c b) b + c b b = 0 bc + bc = 2bb,
(3)

DE AD (e d) d = 0 (e d) d + e d d = 0 de + de = 2dd.
(4)

Cum bb = |b|2 = |d|2 = dd, din relatiile (3) si (4) rezulta ca bc + bc = de + de, de unde
deducem c
a relatiile (1) si (2) sunt echivalente, deci BE AC, dac
a si numai dac
a
DC AE. (Cezar Chiril
a, elev, Colegiul National "M. Eminescu", Botosani )
29

Solutia a VIII-a (analitic


a). Fie
xOy un sistem ortogonal de coordonate cu
originea n A si (Ox = (AC. n acest
caz, punctele din problem
a vor avea coordonatele: A (0, 0), B (b, b0 ), C (c, 0), D (d, d0 ),
E (e, e0 ), F (b, 0), G (g, g 0 ). Sa presupunem
ca B, F , E sunt coliniare si sa aratam ca
D, G, C sunt coliniare (reciproca se poate
demonstra n mod analog, alegnd sistemul
de coordonate astfel nct triunghiul ADE
sa joace rolul triunghiului ABC). n acest
caz e = b.
b=D
b = 90 , rezult
Din B
a c
a
sau

B(b,b')

O=A(0,0)
F(b,0)

C(c,0)

G(g,g')
E(e,e')
D(d,d')
Fig. 3

mAB mBC = mAD mDE = 1

b0
b0
d0 e0 d0

= 1,
b bc
d ed
2
2
de unde obtinem b2 + (b0 ) = bc si ed + e0 d0 = d2 + (d0 ) . De aici, avnd n vedere
2
2
0 2
0 2
a:
c
a AB = AD, deci b + (b ) = d + (d ) , deducem c
ed + e0 d0 = bc.
Cum DG AE, rezulta ca mDG mAE = 1, sau
g 0 d0
e
= 0.
gd
e

(1)

(2)

n sfrsit, tinnd cont de (1) si (2), putem scrie:


g 0 d0
e
d0
d0 (2)
=

0 =
mDG = mDC
gd
dc
e
dc
(1)

ed + e0 d0 = ec bc = ec b = e,

care este adev


arat
a, conform ipotezei. (Evelina Sl
atineanu, elev
a, Liceul
"D. Cantemir", Iasi)
Bibliografie
1. Concursul de matematic
a "Al. Myller", editia I, 2003, Problema X.3 (n Recreatii matematice, suplimentul nr. 1 (2003) sau n prezentul numar, p. 33).
2. Liviu Nicolescu, Vladimir Boskoff - Probleme practice de geometrie, Ed. Tehnica,
Bucuresti, 1990.

30

Asupra unor sume cu radicali


Dan POPESCU 1
Scopul acestei note este prezentarea unor propriet
ati ale radicalilor accesibile elevului de gimnaziu si utile n abordarea unitara a unor tipuri de probleme.

Propozitia 1. Daca n N este astfel nct n Q, atunci n N.

p
p2
Demonstratie. Fie n = , cu p, q N, q 6= 0, (p, q) = 1. Atunci n = 2 , deci
q
q
q 2 | p2 , de unde q | p, adic
a n N.


Propozitia 2. Fie a, b Q+ astfel nct a, b R \ Q. Atunci ab Q daca
r
a
si numai daca
Q.
b
r
a
1
ab.
=
Demonstratie. Imediat, din faptul c
a
b
b

Propozitia 3. Fie a, b Q+ si , Q astfel nct a + b Q ; atunci

a Q si b Q.

2 a 2 b
Q si atunci
Demonstratie. Observ
am c
a a b =
a+ b

i

1 h
a=
a + b + a b Q; analog pentru b.
2

Propozi
tia 4. Fie a Q , iar b Q astfel nct b R \ Q; atunci

a b R \ Q si a b R \ Q.


Demonstratie. Daca, prin absurd, a + b Q, atunci b = a + b a Q,

1
a b Q, fals.
contradictie. La fel, daca a b Q, atunci b =
a
ar irational este nenul, iar inversul oric
arui num
ar
Consecinta
. Deoarece un num
a
1
Q.
a R\Q,
irational este tot num
ar irational, n conditiile P4 avem si c
b
a b

Propozitia 5. Fie a, b Q+ astfel nct a, b, ab R \ Q, iar m, n Q.

Atunci m a + n b Q daca si numai daca m2 +n2 = 0.


Demonstratie.
a m = n = 0, atunci m a + n b = 0 Q. Invers, sa
Dac
presupunem
c
a
m
a
+
n
b Q. Patratul acestui numar este tot rational si atunci

mn
ab

Q.
ns
a
ab

R \ Q, deci mn = 0. n cazul n care m = 0, avem c


a

n b Q, adic
a n = 0. Analog, pentru n = 0 obtinem c
a si m = 0. n concluzie,
m2 + n2 = 0.

seama de P2 , ipoteza ab R \ Q poate fi nlocuita cu


r Observatie. Tinnd
a
R \ Q.
b
Prezentam n continuare cteva aplicatii ale acestor rezultate, iar n ncheiere sunt
propuse alte exercitii care pot fi rezolvate asemanator.
1

Profesor, Colegiul National "Stefan cel Mare", Suceava

31

Problema 1. Sa se rezolve n N N ecuatiile

3
2
a+
b = 5;
b) a + b = 18.
a)
3
4


Solutie. a) Aplicnd P3 , obtinem ca a, b Q si atunci a, b N, conform
. .
a a..3, b..4 si cercetnd posibilit
atile
P1 . n plus, deoarece (3, 4) = 1, avem c
existente, g
asim unica solutie (9,16).
b) Ecua
tia se scrie echivalent 2a+ 2b = 6 si aplicnd din nou P3 si P1 , obtinem

ca 2a, 2b N, deci a = 2x2 , b = 2y 2 cu x, y N. nlocuind, gasim ca x + y = 3,


de unde (a, b) {(0, 18) , (18, 0) , (2, 8) , (8, 2)}.

Problema 2. Sa se arate ca n + n + 1 R \ Q, n N .
Solutie. S
a observ
am mai nti c
a n si n + 1 nu pot fi simultan p
atrate perfecte
pentru n N . Dac
a exact unul dintre ele este p
atrat perfect, concluzia urmeaz
a din
P4p
. Daca nici unul nu este patrat perfect, atunci se poate aplica P5 , dat fiind faptul
ca n (n + 1) R \ Q (numarul n (n + 1) este cuprins strict ntre patratele perfecte
consecutive n2 si (n + 1)2 ).
Problema 3. Sa se rezolve n N3 ecuatia
2

p
p
m 3m + 2
n2 + n + p2 4
n2 + 5n + 6 = 0.

Solutie. Singura valoare a lui n N pentru care unul dintre radicali dispare
este n = 0; n acest caz, ecuatia devine p2 4 = 0, iar multimea solutiilor este
{(a, 0, 2) | a N}. Dac
a n 6= 0, suntem n ipotezele P5 :
2

n2 < n2 + n < (n + 1) ; (n + 2) < n2 + 5n + 6 < (n + 3) ;


2
2


2
n + 3n < n2 + n n2 + 5n + 6 = n2 + 3n n2 + 3n + 2 < n2 + 3n + 1 ,

prin urmare m2 3m + 2 = 0 si p2 4 = 0, deci multimea solutiilor ecuatiei este


S = {(1, a, 2) | a N } {(2, b, 2) | b N }.
Probleme propuse.
1. S
a se rezolve n N N ecua

tiile:

a) 2 x + 3 y = 12;
b) m + n = 10.

2. Sa se arate ca ecuatia a b = p, unde p N este prim, are o infinitate


de solutii n N N.

3. Sa se arate ca n2 + n + n2 + 3n + 2 R \ Q, n N.
4. Sa se arate ca:
1
1
R \ Q, n N;

R \ Q, n N.
a)
b)
5n + 2 + 5n + 1
5n + 7 11

5. Sa se rezolve n Q ecuatia 2x2 + 5x + 3 11 2x2 + 3x 21 = 0.


n
o

6. Fie A =
a + b | a, b N , a, b 100 . S
a se afle cardinalul multimii
A Q.

32

Concursul "Alexandru Myller"


Editia I, Iasi, 4 - 6 aprilie 2003
Alexandru Myller (1879 - 1965) s-a n
ascut n Bucuresti dintr-o familie de
intelectuali. A absolvit Facultatea de S
tiinte sectia matematica din Bucuresti. ntre
anii 1902 - 1906 se afla n Germania pentru a continua studiile. n 1906, la Gtingen,
sustine teza de doctorat sub conducerea lui David Hilbert. Tot aici l cunoaste pe
Felix Klein, fondatorul faimosului "Program de la Erlangen". Rentors n tar
a, este
titularizat n 1910 ca profesor de geometrie analitica la Univ. "Al. I. Cuza" din Iasi.
n 1910 pune bazele unei biblioteci, Seminarul Matematic, care se va dezvolta
treptat, si a unei scoli de geometrie ce va fi renumit
a n ntreaga lume. Este initiatorul unei colectii de modele geometrice si se preocup
a de problemele nv
atamntului
secundar. A adus contributii de valoare n geometria diferentiala, ecuatii diferentiale
si integrale si istoria matematicii.
A fost rector al universitatii iesene n perioada 1945 - 46. Din 1949 este membru
al Academiei Romne. n 1960 universitatea Humbold din Berlin i-a conferit titlul
de doctor honoris causa.
Not
a. Concursul "Al. Myller" se adreseaz
a elevilor din clasele VII - XII care au obtinut
premii si mentiuni la fazele superioare ale olimpiadelor scolare din anul n curs sau cel
anterior. Prima editie a acestui concurs a fost sprijinit
a de c
atre Univ. "Al. I. Cuza" si
Filiala din Iasi a SSMR. Sarcina organiz
arii si desf
asur
arii acesteia au avut-o I. S. J. Iasi si
urm
atoarele licee: Colegiul National si Liceul de Informatica "Gr. Moisil". Aceast
a editie a
Concursului "Al. Myller" a fost un succes deplin sub aspect calitativ si organizatoric.

Clasa a VII-a
1. Determinati numerele ntregi a, b, c, d care verific
a relatiile a2 + b2 = 2 (c + d)
2
2
si c + d = 2 (a + b).
Gheorghe Iurea, Iasi
2. Fie ABCD un p
atrat fix si punctele variabile M (BC), N (CD) astfel
nct M N = BM + DN . Demonstrati ca masura unghiului ]N AM este constanta.
Gheorghe Iurea, Iasi
3. Se considera un triunghi ABC, un punct M (AC) si punctul N BC astfel
nct M N BC. Perpendiculara din C pe AN si perpendiculara dusa n B pe BC
se intersecteaz
a n P , iar dreptele M P si AN se intersecteaz
a n Q. Demonstrati c
a
AP CQ dac
a si numai dac
a AB AC.
Petru R
aducanu, Iasi
4. Fie a, b, c numere reale astfel nct 0 a 1, 0 b 1, 0 c 1 si
ab + bc + ca = 1. Demonstrati ca a2 + b2 + c2 2.
Mircea Becheanu, Bucuresti

Clasa a VIII-a
1. Determinati numerele x, y, z care verifica relatiile x+y 2z si x2 +y 2 2z 2 = 8.
Adrian Zanoschi, Iasi
2. Un tetraedru regulat cu muchiile de lungime 1 se proiecteaza pe un plan.
Demonstrati ca aria figurii obtinute este cel mult 1/2.
***
33

3. Fie tetraedrul ABCD n care AB = CD = a, AC = BD = b, AD = BC = c si


GA , GB , GC , GD centrele de greutate ale fetelor BCD, ACD, ABD respectiv ABC.
Determinati lungimea minima a unui drum care este situat pe fetele tetraedrului si
trece prin GA , GB , GC , GD .
***
4. Fie n 3 un numar ntreg. Demonstrati ca este posibil ca, eliminnd cel mult
dou
a dintre elementele multimii {1, 2, . . . , n}, s
a obtinem o multime care are suma
elementelor p
atrat perfect.
Mihai B
alun
a, Bucuresti

Clasa a IX-a
1. Fie ABCD un patrulater convex si O un punct n interiorul acestuia.
Not
am cu a, b, c, d, e, f respectiv ariile triunghiurilor formate de O cu
AB, BC, CD, DA, AC si BD. S
a se arate c
a |ac bd| = ef .
Alexandru Myller
2. a) Aratati ca exista functii f : R R, f (x) = ax2 + bx + c, a 6= 0, astfel nct
f (f (k)) = k, k {1, 2, 3}.
b) Ar
atati c
a, dac
a f este o functie ca la a), atunci numerele a, b, c nu sunt toate
ntregi.
Gheorghe Iurea, Iasi
1
1
1
3. Fie a, b, c numere reale pozitive astfel nct
+
+
= 1. S
a se arate c
a
ab bc ca
ab 1 bc 1 ca 1
3

+
+
< 2.
2
ab + 1 bc + 1 ca + 1
Mircea Becheanu, Bucuresti

S = (x, y) R | x, y > 0 primul cadran si T : S S, T (x, y) =


4. Fie
1 1
o transformare a lui S. Numim "S - dreapt
a" intersectia dintre S si o
=
,
x y
dreapta din plan.
a) Ar
atati c
a orice S - dreapt
a fix
a a lui T contine punctele fixe ale lui T .
b) Determinati S - dreptele fixe ale lui T .
(M S se numeste submultime fix
a a lui T dac
a T (M ) = M ).
Gabriel Popa, Iasi

Clasa a X-a
1. Fie A1 A2 . . . An un poligon regulat nscris n cercul C (O, R) si M un punct n
planul acestuia. S
a se arate c
a nR M A1 + M A2 + + M An n (R + OM ).
Gheorghe Iurea, Iasi
Fie polinomul f (X) = X n + 2X n1 + 3X n2 + + nX + n + 1 si

2
2
n
= cos
+ i sin
. S
a se arate c
a f () f 2 . . . f n+1 = (n + 2) .
n+2
n+2
Mihai Piticari, C-lung Moldovenesc
2.

3. Fie ABC si ADE doua triunghiuri dreptunghice cu m (]B) = m (]D) = 90


si AB = AD. Fie F proiectia lui B pe AC si G proiectia lui D pe AE. S
a se arate c
a
punctele B, F , E sunt coliniare dac
a si numai dac
a punctele D, G, C sunt coliniare.
***
34

4. La un concurs se dau cinci probe, cu rezultatul admis - respins. Care este


numarul minim de participanti la concurs pentru care, orice rezultate ar obtine acestia, sa existe doi concurenti A si B astfel nct A sa fie admis la toate probele la care
a fost admis si B?
***

Clasa a XI-a

1. Consider
am An = A M2 (R) | An+1 = 2003n A , unde n este un num
ar
natural nenul fixat.
a) Aratati ca An contine o infinitate de elemente.
b) Determinati A3 A2003 .
Gheorghe Iurea, Iasi
2. Fie A, B M3 (R) doua matrice cu proprietatea ca, oricare ar fi o matrice
X M3,1 (R) pentru care AX = O3,1 , avem c
a BX = O3,1 . S
a se arate c
a exist
ao
matrice C M3 (R) astfel nct B = CA.
Mircea Becheanu, Bucuresti
3. S
a se arate c
a pentru orice num
ar natural n 0 exist
a numerele rationale
strict pozitive a0 < a1 < < an cu urm
atoarele propriet
ati:
a0 a1
an
1
a)
+
+ ... +
= ;
0!
1!
n!
n!
3
b) a0 + a1 + . . . + an < n .
2
Dorin Andrica, Cluj - Napoca
4. S
a se determine functiile derivabile f : [0, ) R cu propriet
atile:
a) f (0) = 0.

1 0 x 2 0 2x
0
+ f
, x [0, ).
b) f (x) = f
3
3
3
3
Mihai Piticari, C-lung Moldovenesc

Clasa a XII-a
1. Fie f si g dou
a polinoame cu coeficienti rationali, ireductibile n Q [X] si
, C astfel nct f () = g () = 0. Sa se arate ca, daca + Q, atunci f si g
au acelasi grad.
Bogdan Enescu, Buz
au
Z 2
2. S
a se calculeze
cos t cos2 (2t) cos3 (3t) . . . cos2002 (2002t) dt.
0

Dorin Andrica, Cluj - Napoca


3. Fie (A, +, ) un inel necomutativ, unitar si a, b, c A astfel nct ab + c = 1.
Dac
a exist
a x A astfel nct a + cx este inversabil, atunci exist
a y A astfel nct
b + yc este inversabil.
Andrei Nedelcu si Lucian L
adunc
a, Iasi
Z t
sin x
4. a) S
a se arate c
a exist
a lim
dx si aceasta este finit
a, unde n N
t 1
xn
fixat.
Z t
sin x
b) Daca se noteaza cu ln = lim
dx, sa se calculeze lim ln .
t 1
n
xn
Mihai Piticari, C-lung Moldovenesc
35

Concursul Florica T. Cmpan, editia a III-a1


Faza judetean
a, 1 martie 2003
Clasa a IV-a
1. Care este cel mai mare num
ar care mp
artit la 10 d
a ctul 9?
2. Sa se ordoneze numerele din sirul urmator n ordinea crescatoare a sumei
cifrelor lor: 132, 456, 199, 897, 1124,9191.
3. Cte triunghiuri sunt n figur
a?

4. S
a se taie 7 cifre din sirul 123123123123, astfel nct num
arul r
amas s
a fie cel
mai mare posibil. Care este numarul?
5. Pe o farfurie sunt 19 fructe: prune, caise, piersici. Num
arul piersicilor este de
9 ori mai mare dect cel al prunelor. Cte caise sunt?
6. O coloan
a de militari, lung
a de 100 metri, trece pe un pod lung de 100 metri
cu viteza de 100 metri pe minut. Ct timp dureaza pna ce coloana parcurge podul?
7. Cnd tu veneai pe lume, eu aveam cu 1 an mai mult dect de 4 ori vrsta ta
de acum. As putea s
a ajung la 99 ani dac
a voi mai tr
ai cu 2 ani mai mult dect ai
trait tu pna acum. Cti ani am eu acum?

Clasa a V-a
1. Suma cifrelor unui num
ar natural este 23, iar ctul mp
artirii sale prin 9 este
96. S
a se afle num
arul.
2. n cte zerouri se termina numarul
56
34

N = 12

61
45

23

12
56

34

23
61

45

56

34
12

45
23

61

Monica Nedelcu
3. Numerele 1, 2, 3, 4, 5, 6, 7, 8, 9 sunt asezate ntr-un tablou triunghiular astfel
a
b

c
d

h
g

Daca suma numerelor de pe fiecare latura a triunghiului este 20, sa se arate ca


num
arul 5 este unul dintre vrfuri.
Andrei Nedelcu
4. Un colier este format din bile pe care sunt nscrise numere naturale nenule
astfel nct pe bilele vecine uneia este nscris un divizor sau multiplu al numarului
nscris pe acea bil
a, f
ar
a ca un acelasi num
ar s
a apar
a pe mai multe bile. Care este
1

Not
a . Toate subiectele sunt obligatorii. Timp de lucru: 1.5 ore - cl. IV si 2 ore - cl. V-VIII.

36

cel mai lung colier care poate fi format cu numerele naturale mai mici sau egale cu
100? Descrieti toate solutiile cu numar maxim de bile!
Mihaela Cianga

Clasa a VI-a
1. Fie S = 2 + 22 + 23 + 24 + . . . + 22003 . Calculati S + 2.
2. Determinati toate numerele de forma abcd stiind ca
a+b+c
b+c+d
c+d+a
=
=
.
5
2
3
3. n jurul unui punct O consider
am unghiurile cu m
asurile din figur
a. Dac
a
x = 9 , calculati n .
4
3
2
1
x
n

4. ntr-un triunghi laturile sunt numere naturale pare. O latur


a este egal
a cu 2.
Ar
atati c
a triunghiul este isoscel.

Clasa a VII-a
1. Fiecare celula a unui tabel 2003 2003 este colorata la ntmplare cu una din
2002 culori. La un pas se permite recolorarea cu o aceeasi culoare a unei linii sau
a unei coloane, dac
a pe aceast
a linie (coloan
a) se afl
a m
acar dou
a celule de aceast
a
culoare. Prezentati un algoritm cu un numar minim de pasi care permite ca orice
tabel sa devina monocolor.
a se
2. Fie r1 , r2 , r3 , . . . , r2003 o rearanjare a numerelor ntregi 1,2,3,. . . ,2003. S
demonstreze c
a produsul P = (r1 1) (r2 2) . . . (r2003 2003) este num
ar par.
3. Fie 4ABC si M (BC). Notam cu M 0 , C 0 , A0 , B 0 simetricele punctelor A,
0
M , C 0 , A0 respectiv fata de M , C, A, B. S
a se arate c
a punctele M 0 si B 0 coincid
dac
a si numai dac
a M este mijlocul lui (BC).
Gabriel Popa
b = 80 , m(C)
b = 60 si AC = 1. S
4. Se consider
a 4ABC cu m(A)
a se demonstreze c
a BC este medie proportional
a (geometric
a) ntre AC si (AB + 1).

Clasa a VIII-a

1. Fie n N . Sa se gaseasca partea ntreaga a numarului n2 + 3n.


2. Fie A = {1, 2, 3, . . . , 2003} si fie f : A A o functie liniar
a neconstant
a.
Ar
atati c
a f (1002) = 1002.
Gheorghe Iurea
3. Dou
a furnici merg cu vitez
a constant
a pe paralelipipedul dreptunghic
ABCDA0 B 0 C 0 D0 , cu AB = 10 cm, BC = 15 cm, AA0 = 12 cm. Prima furnic
a
pleaca din A si ajunge n A0 traversnd, n ordine, muchiile [BB 0 ], [CC 0 ] si [DD0 ]
37

(pe drumul cel mai scurt). A doua furnic


a pleac
a din B 0 si ajunge n B traversnd,
0
0
0
n ordine, muchiile [CC ], [DD ] si [AA ] (pe drumul cel mai scurt). Cunoscnd ca
furnicile pornesc n acelasi timp si ca ele se ntlnesc, aflati raportul vitezelor lor.
4. Fie o mas
a de biliard dreptunghiular
a ABCD la care s-au ales drept axe de
coordonate laturile AB si AD (AB e axa absciselor). Se dau 2 bile situate n punctele
M (5, 6) si N (1, 2). Bila din M pleaca liniar catre AB astfel nct sa loveasca bila
din N . S
a se g
aseasc
a punctul n care bila loveste latura AB.

Faza interjudetean
a, 24 mai 2003
Clasa a IV-a
1. Pe fiecare dintre cele cinci carti, numarul de jos este ntr-o aceeasi legatura
ascuns
a cu num
arul de sus. Care este al doilea num
ar scris pe a cincea carte?

93

84

55

62

51

2. Un elev trebuie s
a nvete pentru a doua zi la istorie, matematic
a si englez
a. n
cte moduri si poate stabili ordinea disciplinelor la care nvata? Precizati-le!
3. Dac
a un pahar si o sticl
a cnt
aresc ct o can
a, sticla respectiv
a cnt
areste
ct paharul si o farfurie, iar dou
a c
ani cnt
aresc ct trei farfurii, atunci cte pahare
cntaresc ct o sticla?
4. Am vizitat gr
adina zoologic
a. Am v
azut ursii, leii, lupii si maimutele, dar nu
n aceast
a ordine. n prima cusc
a animalele dormeau si erau ursi sau maimute. n a
doua cusca nu erau lupi si nici lei. n a treia cusca animalele se uitau n alta parte,
nu la mine. n a patra cusca nu erau maimute si nici ursi. Maimutele nu dormeau.
Lupii se uitau la mine. n ce ordine am vizitat animalele?

Clasa a V-a
1. Determinati cel mai mic num
ar scris n baza 10 numai cu cifrele 0 si 1, divizibil
cu 225.
2. Aratati ca numerele 1, 2, 3 . . . , 16 nu pot fi aranjate pe o circumferinta astfel
nct suma oric
aror dou
a numere vecine s
a fie p
atrat perfect. Este posibil
a o astfel
de aranjare pe o linie? Justificati.
56
3. Fie fractia 2003 .
2
a) Justificati ca fractia este zecimala finita;
b) Care sunt ultimele dou
a zecimale nenule? Dar ultimele trei?
4. Un grup de prieteni hot
ar
asc s
a fac
a o c
al
atorie la Viena. Fiecare dintre
ei prezinta la vama acelasi numar de bancnote, unele de 100 , altele de 100 $.
Organizatorul grupului detine un sfert din bancnotele de 100 si o sesime din cele
de 100 $. Cte persoane sunt n grup si care e minimul num
arului total de bancnote,
stiind c
a fiecare trebuie s
a aib
a cel putin 5 astfel de bancnote?
Mihaela Cianga

Clasa a VI-a
1. Pe sase recipiente avem scrise capacitatile lor: 8 l, 13 l, 15 l, 17 l, 19 l si
38

respectiv 31 l. Recipientele sunt umplute cu ulei sau otet. Un client cump


ar
a de
840000 lei otet si tot de 840000 lei ulei, golind cinci din cele sase recipiente si lasnd
unul singur neatins. Care recipient a fost neatins? Care este pretul unui litru de ulei,
stiind c
a pretul uleiului este de dou
a ori mai mare dect pretul otetului?
Nicu Miron
2. Fie numerele naturale nenule a1 < a2 < a3 < a4 < a5 < a6 . Spunem ca
multimea {a1 , a2 , . . . , a6 } are proprietatea P daca k {3, 4, 5, 6}, i, j {1, 2, 3, 4, 5, 6},
a se afle cte multimi cu proprietatea P sunt de
i 6= j astfel nct ak = ai + aj . S
forma {1, 2, a, b, c, d}.
Petru Asaftei
3. Se considera triunghiul ABC dreptunghic n A. Daca AB = 2AC, aratati ca
b este mai mic
m
asura unghiului C
a dect 67 300 .
Petru Asaftei
4. S
ase drepte se afla n acelasi plan. Aratati ca cel putin doua dintre aceste
drepte fac ntre ele un unghi cu m
asura mai mic
a dect 31 .

Clasa a VII-a

1
7
1
+
=
1. S
a se rezolve ecuatia
.
12
x2 + 4
x2 + 11
2. Sa se arate ca toate dreptele care mpart un dreptunghi n doua parti de arii
egale sunt concurente.
3. Un cerc este mp
artit n n p
arti egale. Plecnd din fiecare punct de diviziune,
se num
ar
a m puncte consecutive si se uneste punctul initial cu cel obtinut (punctele
unite nu sunt diametral opuse). Sa se demonstreze ca nu exista trei drepte care sa
fie concurente n interiorul cercului.
4. Alice si Bob au un s
aculet cu 2003 bile. Alice scoate ntre una si trei bile din
s
aculet dup
a care Bob are dreptul s
a scoat
a si el ntre una si trei bile. Procedeul se
repeta pna la extragerea tuturor bilelor din saculet. Aratati ca Alice poate proceda
n asa fel nct s
a extrag
a ea ultima bil
a, indiferent de felul n care actioneaz
a Bob.

Clasa a VIII-a

1. a) Demonstrati c
a (n + 3)2 (n + 2)2 (n + 1)2 + n2 = 4, n N.
b) Aratati ca putem alege semnele astfel nct n N sa aiba loc egalitatea
2

(n + 7) (n + 6) (n + 5) (n + 4) (n + 3) (n + 2) (n + 1) n2 = 0.
c) Fie A = {2003, 2004, . . . , 20042}. Ar
atati c
a exist
a multimile B si C disjuncte
astfel nct B C = A, suma elementelor lui B este egal
a cu suma elementelor lui C
si suma patratelor elementelor lui B este egala cu suma patratelor elementelor lui C.
2. La o balanta bratele n care se pun greutatile si marfa trebuie sa fie n echilibru.
Un cump
arator a sesizat faptul c
a balanta este defect
a, deoarece punnd marfa pe
un taler si greut
atile pe cel
alalt taler si apoi invers, balanta nu este n echilibru.
Cumparatorul, care vrea sa achizitioneze 2 kg, a cerut sa i se cntareasca 1 kg de
marfa ntr-un mod si 1 kg de marfa n celalalt mod. A iesit n pierdere sau n cstig?
3. Folosind dou
a butoaie cilindrice, unul de 50 l, altul de 60 l si avnd orict de
mult
a ap
a la dispozitie, prin mai multe m
asur
atori s
a se obtin
a 55 l de ap
a.
C
at
alin Budeanu
39

a triunghiular
a regulat
a cu toate muchiile egale.
4. Fie ABCA0 B 0 C 0 o prism
a) Determinati pozitia punctului M pe segmentul [BB 0 ] astfel nct A4AMC 0 sa
fie minima.
b) Dac
a M este mijlocul segmentului [BB 0 ], s
a se determine unghiul dintre planele
(ABC) si (AM C 0 ).

Not
a de cititor
Multi dintre nv
atatorii ieseni au fost probabil contrariati n primele momente, la
fel ca si mine, cnd au aflat de organizarea unui concurs de matematic
a, la clasele
IVIII avnd pe generic numele "Florica T. Cmpan".
Dupa ce surpriza a trecut, curiozitatea si-a facut loc printre gnduri de tot felul
si m-a ndemnat s
a aflu ce zvcnire de spirit se ascunde n spatele acestui nume, f
ar
a
chip nc
a pentru mine, dar care a determinat o mobilizare considerabil
a de forte.
Aveam s
a constat n scurt timp c
a bibliotecile aveau suficiente materiale care s
a
ma ajute sa gasesc raspunsuri convingatoare.
Din paginile c
artilor r
asfoite sau citite cu aviditate, se contura personalitatea unui
om de cultur
a, profesor de prestigiu si d
at
ator "de carti atractive si lamuritoare"
(D. Brnzei ) ale geometriei, ale sirurilor de numere si ale p
atratelor magice, ale
istoriei matematicii.
Nu mi-am propus n aceast
a not
a o incursiune n bibliografia acestei Doamne
a matematicii. Au f
acut-o altii cu mai mult
a rvn
a si pricepere naintea mea.
M-am gndit doar c
a sfrsitul toamnei poate constitui pentru ieseni (si nu numai)
un prilej de aducere aminte a faptului ca pe 26 noiembrie 1906 la Iasi, pe tarmul
matematicii o "aleas
a a Domnului" se ivea s
a-si mplineasc
a harul. C
aci pentru
profesor doctor-docent Florica T. Cmpan matematica nu este o simpl
a stiinta.
Ea reprezint
a, ca si credinta, o cale prin care poti s
a fii mai aproape de divinitate.
Rndurile mele se doresc a fi mai mult un prilej de a scrie despre ceva drag mie:
redescoperirea prin lectur
a a universului matematicii.
S
i poate atunci cnd iarna si va intra n drepturi, veti g
asi o clip
a de r
agaz s
a cititi
despre "Istoria numarului ", despre "Probleme celebre din istoria matematicii", sa
traiti "Aventura geometriilor neuclidiene", sa aflati cine sunt "Licuricii din adncuri"
si s
a simtiti c
a "Dumnezeu si matematica" au aceeasi esenta.
Supletea si persuasiunea discursului matematic, profunzimea discursului filozofic,
savoarea dialogului te fac sa uiti ariditatea "terenului" pe care te afli, ti aduc matematica macar mai aproape de suflet, daca nu de minte.
Chiar dac
a, personal, nu am excelat n domeniu si nici timp s-o fac nu mai am,
m-am aflat prin intermediul d-nei Florica T. Cmpan ntr-un dialog cu matematica
dincolo de catalog, dincolo de folosirea ei marunta si lenesa, la interferenta dintre real
si divin.
A venit apoi firesc ntrebarea: un nv
atator aproape nestiut poate aduce ceva nou
n lumea matematicii? R
aspunsul a venit prompt. Da, poate veni cu puterea lui
de patrundere, cu putina lumina n mintea copiilor, iar daca nu are nimic din toate
acestea, poate veni cu sufletul... Pentru ca ea, MATEMATICA, este pretentioasa:
VREA TOTUL!
nv. Luminita Murariu, S
coala "Elena Cuza", Iasi
40

Concursul "Adolf Haimovici", editia a VII-a1


pentru liceele economice, industriale si agricole

Faza judetean
a, 22 februarie 2003
Clasa a IX-a
1. Sa se demonstreze ca exista o singura functie f : R R, f (x) = ax2 + bx + c,
unde a, b, c R, a 6= 0, astfel nct a, , P si S s
a fie numere ntregi consecutive n
aceast
a ordine.
2. Sa se arate ca daca a, b, c [0, +) si a + b + c = 1, atunci
8
abc (a + b) (b + c) (c + a)
.
729
3. S
a se scrie ecuatiile laturilor unui triunghi ABC dac
a se cunosc A (1, 3) si
ecuatiile a doua mediane: x 2y + 1 = 0 si y 1 = 0.

Clasa a X-a

1. a) Se considera numerele reale strict pozitive a1 , a2 , a3 cu produsul p = a1 a2 a3


diferit de 1. Dac
a m = logp a1 , n = logp a2 , s
a se exprime n functie de m si n num
arul
logp aq3 , unde q R.
b) Daca m = log70 2, n = log70 5, calculati log70 49.
c) Rezolvati ecuatia 3|x+1| 2 |3x 1| = 3x + 2.
1
1
1
1
2. a) Aratati ca
2 < a a + 1 < a2 , a (0, ).
(a + 1)
1
1
1
1
+
+ +
+
< 102 .
b) Demonstrati c
a
2
2
2
101
102
2002
20032
3. Fie O mijlocul laturii BC a triunghiului ABC, M un punct pe perpendiculara
n O pe planul (ABC). Fie D proiectia pe BC a lui A, E proiectia pe M B a lui A,
1
F proiectia pe M C a lui A. Ar
atati c
a dac
a M O = BC, atunci (ADE) (ADF ).
2

Clasa a XI-a

x
I. 1. S
a se rezolve ecuatia n x: 2
x3
x

1
c
= 0, unde a, b, c C. Discutie.
c2
c3

2
x
3
x
2. Valorile parametrului real a pentru care matricea A = x 1
3 x+2 a+3
este inversabil
a
pentru
orice
x

R
sunt:

1
1
a)
, 2 ; b) ,
(2, ); c) (, 1) (2, ); d) ; e) R.
2
2

n
Q
1
1 k , n 1.
II. 1. Sa se studieze convergenta sirului an =
2
k=1
1 cos x cos 2x . . . cos nx
2. Fie an =
.
x2
1

1
a
a2
a3

1
b
b2
b3

Not
a . Toate subiectele sunt obligatorii. Timp de lucru: 2 ore.

41

a) S
a se determine lim a1 .
x0

b) Sa se demonstreze ca an = an1 cos nx +


c) Sa se calculeze lim an .
x0

1 cos nx
.
x2

(x + ex )1/x , x 6= 0
III. 1. Studiati continuitatea functiei f (x) =
n x0 = 0.
1, x = 0

2. Valorile a, b, c R asa nct lim x ax cx2 + bx 2 = 1 sunt:


x

a) a = c = 1, b = 0; b) a = 0, b = 1, c < 0; c) a > 0, c < 0, b = 0; d) ;


/
e) a = b = c = 0.

Clasa a XII-a
I. Fie multimea A = (0, ) {1}, a A, R . Definim pe A legea de
compozitie x y = x loga y . Not
am cu Ga, = (A, ). Demonstrati c
a Ga, este grup
abelian.
1 + z + z2
II. Ar
atati c
a dac
a z C \ R si
R, atunci |z| = 1.
1 z + z2
III.
1. Sa se determine
k R astfel nct functia f : R R,

ln x2 + 1 , x > 0
s
a admit
a primitive si s
a se g
aseasc
a o primitiv
a a sa.
f (x) =
xe2x + k, x 0
x2 + 1
, unde
2. Determinati primitivele functiei f : D R, f (x) = 2
x 2x2 cos 2 + 1
este oarecare din intervalul (0, /4), iar D R.

Faza interjudetean
a, Iasi, 9 - 11 mai, 2003
Clasa a IX-a

x2 + (m + 1) x 5
1. S
a se determine m R astfel nct 7 <
< 3, x R.
x2 x + 1

2. Fie a R fixat. S
a se rezolve ecuatia 3 x + 3 a3 x = a.
1
3. a) S
a se demonstreze identitatea sin 10 cos 20 cos 40 = .
8
DM
b) Se da paralelogramul ABCD. Fie M [DC] astfel nct
= k si N [AM ]
DC

astfel nct M N = k N A. Sa se arate ca D N B coliniare.

Clasa a X-a
a. Dac
a 3a24 + a26 + 6 (a4 + a6 + 2) = 0, aflati:
1. Fie (an )n1 o progresie aritmetic
a) a1 , r; b) a8 , S8 .
2. Fie numerele a, b, c (1, 2]. S
a se demonstreze egalitatea:
loga (3b 2) + logb (3c 2) + logc (3a 2) 6.

3. a) Daca a, b R , n dezvoltarea (a + b)n nu exista trei termeni consecutivi


egali;
n

b) S
a se arate c
a partea ntreag
a a num
arului 3 + 2 2 este un num
ar natural
impar, n N .
42

Clasa a XI-a

q
2
1. Fie a, b R distincte si f : R R, f (x) = 3 (x a) (x b).
a) Sa se determine multimea A = {x R | f are derivata n x};
b) S
a se arate c
a functia f are dou
a puncte de extrem local x1 , x2 , iar dac
aa<b
atunci x1 , x2 [a, b).

1p + 2p 2! + 3p 3 3! + + np n n!
.
2. S
a se calculeze lim
n
np+2

a
0 ...
0
b
0
0 ...
0
0

2
2

3. Fie A Mn (R), A =
. . . . . . . . . . . . . . ., 0 a + b < 1. Sa se
0

0 ...
0
0
b 0 . . .
0
a
arate c
a Am 6= On , m N , dar lim Am = On .
m

Clasa a XII-a
a se
1. Fie G = R+ \ {1} si operatia x y = xloga y , x, y G, a > 0, a 6= 1. S
arate ca (G, ) este grup abelian si (G, )
= (R , ).
Z
sin x
2. Sa se calculeze
dx.
3 sin x + 4 cos x

3. Fie G = a + b 5 | a, b Z, a2 5b2 = 1 .
a) S
a se arate c
a (G, ) este grup abelian;
b) S
a se arate c
a G are cel putin 2003 elemente.

Recreaii matematice
Solutiile problemelor enuntate la pagina 18.
1. Cei patru oameni procedeaza astfel:
a si b trec podul (2 minute);
a se ntoarce (1 minut);
c si d trec podul (10 minute);
b se ntoarce (2 minute);
a si b trec podul (2 minute).
n acea noapte, dup
a 17 minute, a, b, c si d trec astfel podul.

2. Modificarea necesar
a se vede pe figur
a.

43

Concursul Traian Lalescu, editia a IV-a1


mai 2003, Iasi
1. Sa se calculeze: 1 (6 + 12 : 3) : 10.
2. Suma a doua numere este 60. Daca unul dintre ele este de 3 ori mai mare
dect cel
alalt, s
a se afle diferenta lor.
3. S
a se afle x din egalitatea: 33 + 3 [(3 + 99 : x) 9 33] = 96.
4. S
a se afle restul mp
artiri num
arului a = 1903 + 1904 + 1905 + . . . + 2103 la
2003.
5. Sa se calculeze: (900 1 2 . . . 40) 80 80 80.
6. Sa se determine numerele naturale nenule a, b, c stiind ca: a[7 + 4 (3b + 2c)] =
= 35.
7. Sa se determine cte numere de trei cifre abc au proprietatea: abc = cba.
8. Elevii unei clase, n num
ar de 30, au participat la un concurs de rezolvat
probleme. S
tiind c
a 25 elevi au rezolvat bine prima problem
a, 24 pe a doua, 23 pe a
treia si 22 pe a patra, s
a se determine num
arul minim de elevi care au rezolvat bine
toate problemele.
9. ntr-o sala de spectacole scaunele sunt asezate cte 25 pe rnd. Daca Ioana
ocup
a locul 630 pe rndul din mijloc, ce loc ocup
a Cristina, care este pe ultimul rnd
n dreptul Ioanei?
10. Ana si Maria au mpreun
a 63 de ani. Ana are n prezent de dou
a ori mai multi
ani dect a avut Maria atunci cnd Ana avea ct are Maria acum. Sa se determine
ce vrsta are acum Ana si ce vrsta are Maria.
11. ntr-o clas
a fiecare b
aiat este prieten cu trei fete si fiecare fat
a este prieten
a
cu doi b
aieti. Dac
a n clas
a sunt 19 b
anci (de cte dou
a locuri) si 31 de elevi sunt
pasionati de matematic
a, cti elevi sunt n clas
a?
12. Elevii prezenti la Concursul de matematica "Traian Lalescu" au fost repartizati n mod egal n 18 sali de clasa, astfel nct n fiecare sala numarul elevilor sa
fie mai mare dect 11 si mai mic dect 17. Dac
a num
arul b
aietilor este de patru ori
mai mic dect num
arul fetelor, s
a se afle num
arul concurentilor.
13. n p
atratul al
aturat suma numerelor de pe fiecare linie,
b
a
8
de pe fiecare coloana si de pe fiecare din cele doua diagonale este
c
5
d
aceeasi. Sa se determine numerele a, b, c, d, e.
e

22

14. Fiecare num


ar nscris ntr-un p
atrat din figura al
aturat
a
este egal cu suma numerelor din p
atratele pe care se sprijin
a. S
a
se determine numerele a, b, c.

21
b
3

Not
a . Fiecare subiect va fi notat cu cinci puncte. Timp de lucru: 2 ore.

44

12

a
a

Olimpiada de matematic
a cl. a V-a si a VI-a
Etapa judetean
a, 10 mai 2003
Not
a. Toate subiectele sunt obligatorii. Timp de lucru: 2 ore.

Clasa a V-a
1. Suma a k numere naturale consecutive poate fi o putere a lui 2 (k > 1)?
2. Sa se gaseaca restul mpartirii numarului n = 10001000 la 27.
Aurel Benza
II. 1. Determinati cel mai mic num
ar k N astfel nct 93 divide 999
. . . 9} .
| {z

I.

k cifre

Petru Asaftei
2. Avem la dispozitie un num
ar nelimitat de jetoane pe care sunt scrise
numerele naturale 5, 7 sau 11. Spunem c
a "am obtinut num
arul n" dac
a putem g
asi
jetoane cu suma numerelor de pe ele egala cu n. Aratati ca 13 este cel mai mare
num
ar care nu poate fi obtinut.
Valerica Benta
III. La un stadion cu capacitatea de 10000 locuri, vin spectatorii. n primul
minut vine un spectator, n al doilea minut vin trei spectatori, n al treilea minut vin
cinci spectatori si asa mai departe. S
a se afle dup
a cte minute se umple stadionul.

Clasa a VI-a
I. Ar
atati c
a dac
a x + y, y + z, z + x sunt direct propertionale cu numerele a + 1,
x z
2
a + 2 si a + 3, a N , atunci + 4 .
y
x
3
II. Fie a1 , a2 , a3 , b1 , b2 , b3 , c1 , c2 , c3 Z . Demonstrati ca numerele: a1 b2 c3 ,
a2 b3 c1 , a3 b1 c2 , a1 b3 c2 , a2 b1 c3 , a3 b2 c1 nu pot fi simultan pozitive.
III. 1. n triunghiul ABC, bisectoarea unghiului B intersecteaz
a n
altimea AM ,
M (BC), n punctul O. Construim OP AB, P (AB).
a) Daca P este mijlocul lui [AB], demonstrati ca masura unghiului BAM este
30 .
b) Dac
a O este centrul de greutate al triunghiului, ar
atati c
a triunghiul ABC este
echilateral.
Marius Farcas
2. Se consider
a triunghiul ABC si punctele M (AB), N (AC) astfel nct
\
[BM ] [AN ]. Sa se calculeze masura unghiului CP
N , unde BN CM = {P }.

ERRATA
1. n finalul solutiei problemei XII.26 (RecMat 1/2003, p. 64) au fost omise

rndurile: ". . . n
cazul A = R. Daca A = Q, atunci
(M, ) = (Q, +), nsa grupurile

(Q, +) si Q+ , nu sunt izomorfe. n sfrsit Z+ , nu este grup.".

2. n enuntul problemei V.40 (RecMat 1/2003, p. 80) n loc de "2n + 3 submultimi " se va citi "4n + 3 submultimi ".

45

Olimpiada Balcanic
a de Matematic
a pentru Juniori
Editia a VII-a, Izmir (Turcia), 20 - 25 iunie 2003
A. Problemele de concurs - enunturi si solutii
1. Fie n N . Numarul A este format din 2n cifre de 4, iar numarul B este
format din n cifre de 8. S
a se arate c
a A + 2B + 4 este p
atrat perfect.
S. Grkovska, Macedonia
2. Fie n puncte n plan, oricare trei necoliniare, cu propietatea (P ): oricum
am numerota aceste puncte A1 , A2 , . . . , An , linia frnta A1 A2 . . . An nu se autointersecteaza. Gasiti valoarea maxima a lui n.
D. S
erb
anescu, Romnia
3. Pe cercul circumscris 4ABC, arcele AB, BC, CA se consider
a astfel nct

/ BC, B
/ CA si fie F , D, respectiv E mijloacele acestor arce. Notam cu
C
/ AB, A
G, H punctele de intersectie ale lui DE cu CB, respectiv CA si cu I, J punctele de
intersectie ale lui DF cu BC, respectiv BA. Fie M , N mijloacele lui [GH], respectiv
[IJ].
a) Gasiti unghiurile 4DM N functie de unghiurile 4ABC.
b) Dac
a O este centrul cercului circumscris 4DM N si {P } = AD EF , ar
atati
c
a O, P , M si N sunt conciclice.
Ch. Lozanov, Bulgaria
1 + x2
1 + y2
1 + z2
4. Fie x, y, z (1, ). Sa se arate ca
+
+
2.
1 + y + z 2 1 + z + x2 1 + x + y 2
L. Panaitopol, Romnia
1. Solutia redactiei. Se constata usor ca
102n 1
10n 1
.
.
.
1
+
16

11
.
.
.
1
+
4
=
4

+
16

+4=
A + 2B + 4 = 4 11
| {z }
| {z }
9
9
n
2n
2

4 102n + 16 10n + 16
2 10n + 4
= 66
. . 68} 2 .
=
=
| .{z
9
3
n

2. Exist
a multimi de 4 puncte cu proprietatea (P ), de exemplu multimea vrfurilor unui patrulater concav. Vom ar
ata c
a nu exist
a multimi cu n 5 puncte
satisf
acnd (P ). S
a observ
am c
a dac
a printre cele n puncte exist
a patru A, B, C,
D astfel ca ABCD sa fie patrulater convex, cu renotarea A1 = A, A2 = C, A3 = B,
A4 = D am avea [A1 A2 ] [A3 A4 ] 6= , deci (P ) nu ar avea loc. Ar
at
am c
a pentru
n 5, putem selecta 4 puncte care s
a fie vrfurile unui patrulater convex. Lu
am
arbitrar 5 puncte din multime si consider
am nchiderea lor convex
a. Dac
a aceasta nu
este triunghi, problema este rezolvata. Daca este triunghi, dreapta determinata de
cele dou
a puncte din interiorul lui taie exact dou
a laturi ale triunghiului; fie A vrful
comun al acestor dou
a laturi. Cele patru puncte r
amase determin
a un patrulater
convex, ceea ce ncheie solutia.
b = m(BD) = m(DC) = , m(B)
b = m(AE) = m(EC) = ,
3. a) Notam m(A)
1
b = ( + ) = 1 (180 ) = 90 .
b = m(AF ) = m(F B) = . Atunci m(D)
m(C)
2
2
2

\
\
\
Analog, m(DEF ) = 90 , iar m(DF E) = 90 . Pe de alt
a parte, m(EHA) =
2
2
46

i
i
A
1h
1h
m(AE) + m(CD) =
m(CE) + m(BD) =
E
2
2
L
P
K
\
\
\
= m(CGE), deci CHG CGH, adica 4CGH este F
isoscel. Cum CF este bisectoare, ea va fi mediana si
\
H
n
altime, prin urmare M CF si m(EM
F ) = 90 .
J

\
Analog se arat
a c
a m(F
N E) = 90 , deci patrulaterul
M
N
\
EM N F este inscriptibil si atunci m(DM
N) =
B
C
I
G
\ ) = 90 , iar m(DM
\
\
= m(DEF
N ) = m(DF
E) =
2

90 .
D
2
b) Fie {K} = AB EF , {L} = AC EF ; ca la punctul a) se arata ca AP KL,

\
\
\ ) = m(ALP
[ ) = 90 , deci
m(F
P N ) = m(EP
M ) = 90 . nsa m(AKP
2
2
\
\ = . Avem ca 4DM N este
AB k P N si AC k P M , de unde m(M
P N ) = m(BAC)

\
ascutitunghic (i. e. O este punct interior lui), iar m(M
DN ) = 90 si atunci
2
\
\
\
ON ) + m(M
P N ) = 180 , adica punctele
m(M
ON ) = 180 . Urmeaza ca m(M
O, P , M , N sunt conciclice.
1 + y2
1 + x2
4. Deoarece y
(cu egalitate pentru y = 1), avem ca

2
1 + y + z2

2 1 + x2
. Scriind analoagele si adunndu-le, cu notatiile a = 1 + x2 ,

2 (1 + z 2 ) + (1 + y 2 )
b = 1 + y 2 , c = 1 + z 2 , inegalitatea de demonstrat devine
a
b
c
+
+
1, a, b, c > 0.
()
2c + b 2a + c 2b + a
Pentru a demonstra (), folosind Cauchy-Schwarz si binecunoscuta a2 + b2 + c2
ab + bc + ac, avem
=

a
b
c
a2
b2
c2
(a + b + c)
+
+
=
+
+

1,
2c + b 2a + c 2b + a
2ac + ab 2ab + bc 2bc + ac
3 (ab + bc + ac)
cu egalitate daca si numai daca a = b = c. Egalitatea n inegalitatea initiala se atinge
pentru x = y = z = 1.
Altfel, () se poate demonstra renotnd numitorii A = 2c + b, B = 2a + c,
C = 2b + a; dup
a calcule, se obtine

B
C
A B
C
A
+ + +4
+ +
15, A, B, C > 0.
A B
C
A B C
r
C
A B
C A B
Este nsa clar ca
+ +
33

= 3 si analoaga.
A B
C
A B C

B. Probleme aflate n atentia juriului - enunturi


a b c
a c b
1. Fie a, b, c lungimile laturilor unui triunghi, p = + + , iar q = + + .
b c a
c b a
Sa se arate ca |p q| < 1.
5
2. Fie a, b, c R cu a2 +b2 +c2 = 1. S
a se arate c
a ab+bc+ca2 (a + b + c) > .
2
47

r
1
c3
1
1
3. Fie a, b, c Q cu
+
=
. Ar
atati c
a
Q.
a + bc b + ac
a+b
c+1
a, b, c N lungimile laturilor
unui triunghi neisoscel. Demonstrati c
a

24. Fie
ab + bc2 + ca2 a2 b b2 a c2 a 2.

5. Fie a, b, c (0, ) cu ab + bc + ca = 3. Aratati ca a + b + c abc + 2.


6. Demonstrati c
a exist
a multimi disjuncte A = {x, y, z} si B = {m, n, p} de
numere naturale mai mari ca 2003 astfel nct x + y + z = m + n + p si x2 + y 2 + z 2 =
= m2 + n2 + p2 .
7. Numerele 1, 2, 3, . . . , 2003 sunt renotate a1 , a2 , a3 , . . . , a2003 . Definim b1 = a1 ,
b2 = 2a2 , b3 = 3a3 , . . . , b2003 = 2003a2003 si fie B cel mai mare dintre bk , k = 1, 2003.
a) Dac
a a1 = 2003, a2 = 2002, . . . , a2003 = 1, g
asiti valoarea lui B.
b) Demonstrati ca B 10022 .
8. Fie M = {1, 2, 3, 4}. Fiecare punct al planului este colorat n rosu sau albastru.
Ar
atati c
a exist
a cel putin un triunghi echilateral de latur
a m M cu vrfurile de
aceeasi culoare.
9. Exista un patrulater convex pe care diagonalele sa-l mparta n patru triunghiuri cu ariile numere prime distincte?
10. Exist
a un triunghi cu aria 12 cm2 si perimetrul 12 cm?
11. Fie G centrul de greutate al 4ABC, iar A0 simetricul lui A fata de C. Sa se
arate c
a G, B, C, A0 sunt conciclice dac
a si numai dac
a GA GC.
12. Trei cercuri egale au n comun un punct M si se intersecteaz
a cte dou
a n
A, B, C. Demonstrati ca M este ortocentrul 4ABC 1 .
13. Fie 4ABC cu AB = AC. Un semicerc de diametru [EF ], cu E, F [BC],
este tangent laturilor AB si AC n M , respectiv N , iar AE retaie semicercul n P .
Demonstrati c
a dreapta P F trece prin mijlocul coardei [M N ].
14. Paralelele la laturile unui triunghi duse printr-un punct

interior mpart interiorul triunghiului n 6 parti cu ariile notate


c
b
a
b
c
3
ca n figur
a. Demonstrati c
a + + .

2
a
Echipa Romniei a fost condusa de prof. Dan Brnzei, asistat de prof. Dinu
S
erb
anescu. n clasamentul neoficial pe natiuni, Romnia a ocupat primul loc cu
205 puncte din 240 posibile, urmat
a de Bulgaria (182 puncte) si Turcia (124 puncte);
n continuare, s-au situat R. Moldova, Serbia, Macedonia, Grecia si Ciprul. Componentii echipei Romniei au obtinut urmatoarele punctaje si medalii (cu mentiunea
c
a primii doi sunt singurii elevi care au realizat punctajul maxim!):
Dragos Michnea (Baia Mare)
40 p Aur
Adrian Zahariuc (Bacau)
40 p Aur

Lucian Turea
(Bucuresti)
38 p Aur
Cristian T
al
au (Craiova)
37 p Aur
Sebastian Dumitrescu (Bucuresti) 29 p Argint
Beniamin Bogosel (Arad)
21 p Bronz.
1

teica.
Identificat
a drept problema piesei de 5 lei a lui Ti

48

Un nou concurs international de matematic


a
n prim
avara anului 2001, Romnia a lansat prin "Fundatia pentru Integrare
European
a Sigma", un nou concurs international de matematica. Competitia intitulata "MCM - Multiple Choice Contest in Mathematics", este prevazuta a se
desf
asura pe echipe de cte 4-6 elevi ce apartin la patru categorii de vrst
a: 11-12 ani,
13-14 ani, 15-16 ani si 17-18 ani. n fiecare echip
a pot intra cel mult dou
a persoane
din aceeasi grupa de vrsta si este posibila colaborarea ntre membrii ei. Fiecare participant primeste cte 20 probleme-grila gradate pe trei nivele de dificultate, timpul
de lucru fiind de 90 minute. Oricare dintre probleme are 5 variante de r
aspuns, una
singur
a fiind corect
a. Un r
aspuns bun aduce 4 puncte, unul incorect scade 2 puncte,
n timp ce cazurile netratate aduc diminuari de cte 1 punct. Clasamentul echipei
este dat de media aritmetica a punctelor obtinute de componentii ei.
Jocul-concurs "MCM" s-a dorit a fi faza final
a international
a a cstig
atorilor
nationali ai jocului-concurs "Cangurul", adus din Australia n Europa prin intermediul asociatiei pariziene "Kangourou sans Frontires" (n Romnia aceasta
competitie s-a introdus ncepnd cu anul 1994). Competitia a avut loc pe 18 iulie
2001 la Poiana Pinului (Buz
au) si au participat urm
atoarele tari: Austria, Belarus,
Bulgaria, Franta, Georgia, Italia, Lituania, R. Moldova, Romnia, Spania, Ucraina
si Ungaria. Pe primele trei locuri s-au situat: 1. Romnia, 2. Franta, 3. Bulgaria.
Echipa Romniei a fost formata din elevii: Eduard Dogaru (12 ani), Roxana
Leonte (12 ani), Gabriel Kreindler (14 ani), Bogdan Bucsa
(16 ani), Tiberiu
Pristavu (16 ani) si Bogdan Stan (18 ani).
Vom prezenta mai jos 12 probleme, cte 3 pentru fiecare categorie de vrsta.

Grupa 1 (11-12 ani)


1.

300

315

1730

12

12

12

A)

B)

12

C)

12

D)

12
3

E)

12

12
3

9
6

2. a, b, c {0, 1, . . . , 9} ab + bc + ca =?
A) 132 B) 48 C) 72 D) 51 E) 37

Laurentiu Modan

3. n =?
1

21

n-2

n-1

n-1

n-2

A) 28 B) 25 C) 27 D) 29 E) 31
49

Grupa 2 (13-14 ani)


11

4.
x

x =?
3

A) 15 B) 20 C) 18 D) 16 E) 25

1
1
1
+
+ ... +

1+ 2
2+ 3
2001 + 2002

A) E < 1 B) 2001 < E < 2002 C) 43 < E < 44 D) 33 < E < 34 E) E 2002
5. E =

6.

=?

A) 2 +

B)

1 C) + 1 D) 3 +
2
2
4

E)
V

1
3

Grupa 3 (15-16 ani)

G3

7. M1 A1 = M1 A2 , M2 A2 = M2 A3 , M3 A1 = M3 A3 ,
G1
G2
G2 M2
G3 M3
1 SG1 G2 G3
G1 M1
A1
A3
=?
=
=
= ,
M3
G1 V
G2 V
G3 V
2 SA1 A2 A3
M2
M1
1
1
2
4
1
A)
B)
C)
D)
E)
A2
3
9
4

9 3

8) a2 1 x4 + a2 3 x3 3a2 + 1 x2 + 5a2 + 3 x2 a2 1 = 0, a R\{1},


x1 = x2 = x3 = 1. a =?

A) a = 0 B) a 2
C) a D) a = 2 E) a = 1/2
Laurentiu Modan si Dinu Teodorescu
A

9.

b = 3 C,
b B
b =?
BC = 2 AB, A

b
b
b = 50
A) B = 60 B) B = 30 C) B
b = 40
E) B

b = 90
D) B

Grupa 4 (17-18 ani)

3n+2
+ x + 2 = x2 + 3x + 3 C (x) + R (x), n N, 0 grad R (x) < 2.
10. (3x + 1)
A) R (x) = x2 1 B) R (x) = 2x+3 C) R (x) = 0 D) R (x) = 1 E) R (x) = 12x
Traian Lalescu
0
1
2001
+ C2001
+ . . . + C2001
= a1 . . . an , ai {0, . . . , 9}, i = 1, n.
11. C2001
A) an = 2 B) an = 4 C) an = 8 D) an = 1 E) an = 6
Laurentiu Modan

1, x (0, 1)
0
12. f : R R, f (ln x) =
, f (0) = 1, s = f (1) + f (1)
x, x [1, )
A) s < 0 B) s = e C) s = 2 D) s = e 2 E) s = (e + 1) /2
Laurentiu Modan si Dinu Teodorescu
R
aspunsuri: 1 A; 2 A; 3 C; 4 A; 5 C; 6 B; 7 A; 8 C; 9 A; 10 C; 11 A; 12 B.

tiu Modan
Conf. dr. Lauren
Fac. de Cibernetic
a si Informatic
a Ecoomic
a
A. S. E., Bucuresti
50

Solutiile problemelor propuse n nr. 2 / 2002


Clasele primare
P.33. Care este cel mai mare numar pe care l spunem atunci cnd numaram
crescator din doi n doi, din trei n trei sau din cinci n cinci, pornind de la 1 si fara
sa depasim 100?
( Clasa I )
Raluca Popa, elev
a, Iasi
Solutie. Sunt spuse sirurile de numere (1, 3, 5, . . . , 97, 99), (1, 4, 7, . . . , 97, 100) si
(1, 6, 11, . . . , 91, 96). Num
arul 100 ndeplineste cerinta problemei.
P.34. Numarul merelor de pe o farfurie este cu 3 mai mare dect cel mai mare
numar natural scris cu o cifra. Numarul perelor de pe aceeasi farfurie nu depaseste
numarul merelor, dar este mai mare dect jumatate din numarul acestora. Cte pere
pot fi pe farfurie?
( Clasa I )
nv. Maria Racu, Iasi
Solutie. Num
arul merelor este 9 + 3 = 12. Jum
atatea lui 12 este 6. Pe farfurie
pot fi: 7, 8, 9, 10, 11 sau 12 pere.
P.35. Care dintre numerele 3132, 8182, 3435, 3932, 2021, 5960 este intrusul?
( Clasa a II-a)
Matei Luca, elev, Iasi
Solutie. Toate numerele, cu exceptia lui 3932, sunt formate cu cifrele a doua
numere consecutive. Intrusul este 3932.
P.36. Cum putem realiza egalitatile
4 4 4 4 = 28 si

4 4 4 4 4 4 = 28

nsernd ntre cifrele 4 de mai sus semnele grafice +, , , : , ( )?


( Clasa a II-a)
Alexandru-Gabriel Tudorache, elev, Iasi
Solutie. Pentru prima egalitate avem (4 + 4) 4 4 = 28. Pentru a doua
egalitate avem (4 + 4) 4 4 : (4 : 4) = 28 sau (4 + 4) 4 4 + 4 4 = 28.
P.37. Suma a doua numere naturale este 109. Daca l dublam pe primul si l
triplam pe al doilea, suma devine 267. Care sunt numerele?
( Clasa a II-a)
nv. Galia Paraschiva, Iasi
Solutie.

109

267

Dublul sumei numerelor a si b este 1092 = 218. Al doilea num


ar este 267218 = 49
iar primul numar este 109 49 = 60.
P.38. Cte nmultiri de tipul abc 9 = 8d1e sunt posibile?
( Clasa a III-a)

Sergiu Diaconu, elev, Iasi


Solutie. Avem abc = 8d1e : 9 = 8010 + d0e : 9 = 890 + d0e : 9. Distingem
cazurile: d = 1, e = 8; d = 2, e = 7; . . .; d = 8, e = 1; d = 9, e = 0, 9; n total, 10
cazuri. Ad
augnd si cazul d = 0, e = 0, 9, obtinem 12 nmultiri posibile.
P.39. Scrieti cel mai mic numar natural de sase cifre care ndeplineste, n acelasi
timp, conditiile: a) nu are cifre care se repeta; b) suma cifrelor sale este 30; c) este
51

mai mare dect 900000.


( Clasa a III-a)
nv. Maria Racu, Iasi
Solutie. Cel mai mare numar natural scris cu sase cifre distincte este 987654
care are suma cifrelor 39. Num
arul c
autat este 987510.
P.40. Emilia are de rezolvat un numar de probleme. A hotart sa rezolve cte 4
probleme pe zi. Ea lucreaza nsa mai mult cu 2 probleme pe zi si termina de rezolvat
cu 5 zile mai devreme. Cte probleme a avut de rezolvat si n cte zile le-a terminat?
( Clasa a III-a)
nv. Doinita Spnu, Iasi
Solutie. Emilia rezolva cte 4+2 = 6 probleme pe zi. n ultimele 5 zile trebuia sa
rezolve 5 4 = 20 probleme. Emilia a terminat de rezolvat problemele n 20 : 2 = 10
zile si a rezolvat 10 6 = 60 probleme.
ca data de 1 Decembrie din anul 2001 a fost ntr-o zi de smbata,
P.41. Stiind

sa se afle care va fi urmatorul an n care ziua de 1 Decembrie se va sarbatori ntr-o


zi de duminica.
( Clasa a IV-a)
nv. Rodica Rotaru, Brlad
Solutie. De la 1.12.2001 pn
a la 30.11.2002 sunt 365 zile. Deoarece 365 =
= 752+1, ziua de 30.11.2002 este ntr-o smb
at
a. Ziua de 1.12.2002 se va s
arb
atori
ntr-o zi de duminica.
P.42. Danila Prepeleac i-a propus dracului sa se ntreaca la trnta, dar pentru
a-l pune la ncercare i-a spus ca are un unchi, mos Ursila, batrn de 999 ani si 52
1
saptamni, si de-l va putea trnti pe dnsul, se vor ntrece apoi amndoi . Daca
4
5
din vrsta lui mos Ursila depaseste cu 220 ani
din vrsta nepotului, ce vrsta are
8
Danila?
( Clasa a IV-a)
nv. Valerica Beldiman, Iasi
Solutie. Vrsta lui mos Ursil
a este 999 ani +52 s
apt
amni = 1000 ani. O p
atrime
din vrsta lui mos Ursil
a este 1000 : 4 = 250 ani. Cinci optimi din vrsta nepotului
reprezinta 250 220 = 30 ani. Vrsta nepotului este 30 : 5 8 = 6 8 = 48 ani.
P.43. Primele douasprezece numere dintr-un sir de numere sunt: 1, 2, 0, 3, 4,
1, 5, 6, 2, 7, 8, 0.
a) Scrieti urmatoarele 6 numere din sir;
b) Calculati suma primelor 111 numere din sir.
( Clasa a IV-a)
Alina Stan, elev
a, Iasi
Solutie. a) Observam ca: (1 + 2) : 3 = 1 (rest 0); (3 + 4) : 3 = 2 (rest 1); (5 + 6) :
: 3 = 3 (rest 2). Dupa fiecare grupa de doua numere naturale consecutive a fost pus
restul mp
artirii sumei lor la 3. Urm
atoarele sase numere sunt: 9, 10, 1, 11, 12, 2.
b) Avem 111 : 3 = 37 grupe de cte trei numere n care intr
a primele 74 numere
naturale cu suma S1 = (1 + 74) 74 : 2 = 2775. n trei grupe consecutive avem
resturile 0, 1, 2 care au suma 3. Cum 37 = 3 12 + 1, nseamna ca suma resturilor
S2 = 12 3 = 36. Suma celor 111 numere este S = S1 + S2 = 2775 + 36 = 2811.

Clasa a V-a

V.31. Sa se arate ca

1
1
7
1
+
+ ... +
>
.
101 102
200
12
52

Petru Asaftei, Iasi

Solutie. Grupnd termenii din sum


a obtinem

1
1
1
1
1
1
1
+
>
+
+ ... +
=
+ ... +
+ ... +
101 102
200
101
150
151
200
50
50
1 1
7
>
+
= + =
.
150 200
3 4
12
V.32. Determinati numerele prime a, b, c pentru care 5a + 4b + 7c = 107.
Mihai Cr
aciun, Pascani
Solutie. Deoarece 5a + 7c trebuie s
a fie impar, a si c au parit
ati diferite; fiind
prime, unul dintre ele este egal cu 2. Dac
a a = 2, atunci 4b + 7c = 97, cu solutiile
b = 19, c = 3 si b = 5, c = 11. Daca c = 2, atunci 5a + 4b = 93, cu solutiile a = 5,
b = 17; a = 13, b = 7 si a = 17, b = 2.
V.33. Un numar natural scris n baza 10 are suma cifrelor 603. Este posibil ca
succesorul sau sa aiba suma cifrelor 1? Dar ca acesta sa aiba suma cifrelor 3?
Matei Luca, elev, Iasi
are
suma
cifrelor
603,
iar
succesorul
s
au este 10
. . . 0},
Solutie. Num
arul 99
.
.
.
9
| {z
| {z }
67 cifre

68 cifre

cu suma cifrelor 1. La a doua ntrebare, raspunsul este negativ, deoarece nu este


posibil ca att num
arul ct si succesorul s
au s
a fie multipli de 3.

V.34. Aflati numarul abc, stiind ca abc = 2n ab + 3n bc + 5n ca, unde n N.


Nicolae St
anic
a, Br
aila
Solutie. n mod evident, a, b, c 1. Pentru n = 0, obtinem c
a 89a = b + 10c, si
cum b + 10c 99, rezulta ca a = 1. De aici, b + 10c = 89 si deci b = 9, c = 8.
Pentru n = 1 obtinem ca 75a = 22b + 52c, de unde 25 | 25 (b + 2c) + (2c 3b),
si deci 25 | 2c 3b. Cum 2c 3b < 25 si 2c 3b 25, obtinem 2c 3b = 0 sau
2c 3b = 25. Dac
a 2c = 3b, atunci 3 | c. Cum 3 | 75a si 3 | 52c, rezult
a c
a 3 | 22b,

deci 3 | b si de fapt c este multiplu de 9. Obtinem deci c = 9, b = 6, a = 8. Daca


2c 3b = 25, atunci c = 1, b = 9 si atunci 75a = 250, deci a nu este ntreg.
Pentru n = 2 obtinem c
a 25a = 84b + 258c. Cum 25a 225 si 84b + 258c
342, ecuatia nu are solutie n acest caz.
Pentru n 3, 2n ab + 3n bc + 5n ca 11 (2n + 3n + 5n ) 1760, deci ecuatia nu are
solutie. n final, abc {198, 869}.

V.35. Se da numarul N = 77 . . . 7 cu 2002 cifre. Cercetati daca N se poate scrie


ca suma a doua sau trei patrate perfecte impare.
Tamara Culac, Iasi
Solutie. N = a 100+77, deci N = M4 +1. Dac
a N = x2 +y 2 +z 2 , cu x, y, z N
impare, atunci N = (2k + 1)2 + (2l + 1)2 + (2n + 1)2 = M4 + 3, contradictie. Daca
N = x2 +y 2 , cu x, y N impare, se obtine n mod analog ca N = M4 +2, contradictie.

Clasa a VI-a
VI.31. Fie S = a1 a1 . . . a1 + a2 a2 . . . a2 + . . . + an an . . . an , unde k1 , k2 , . . . ,
| {z } | {z }
| {z }
k1 cifre

k2 cifre

kn cifre

kn 2. Aratati ca S se divide cu 4 daca si numai daca a1 + a2 + . . . + an se divide


cu 4.
Dumitru Gherman, Pascani
53

. . . 1} = a1 11 . . . 100 + 12 1 = M4 a1
Solutie. Deoarece a1 a1 . . . a1 = a1 11
| {z
| {z }
k1 cifre

k1 cifre

si analoagele, obtinem ca S = M4 (a1 + a2 + . . . + an ), de unde concluzia.

VI.32. Aflati ab stiind ca ab = (a b)! ba 3 (unde 0! = 1, iar n! =


= 1 2 3 . . . n, n 1).
Nicolae St
anic
a, Br
aila
Solutie. Pentru a b = 0 sau a b = 1 obtinem ab = ba 3, deci 9a = 9b 3,
contradictie. Pentru ab = 2 obtinem ab = 2ba6, deci 8a = 19b6. Cum ab = 2,
rezulta, ca a = 4, b = 2. Pentru ab =3, avem ab = 6ba18,
adica 55b = 30, absurd.

Pentru a b 4 obtinem ca (a b)! ba 3 24 ba 3 24 (10b + b + 4 3),

deci (a b)! ba 3 264b 264, contradictie. n final ab = 42.

VI.33. ntr-o urna sunt bile albe, rosii, negre si albastre. Numarul bilelor albe
3
este
din numarul celorlaltor bile; bilele rosii reprezinta jumatate din celelalte bile,
5
iar bilele negre a treia parte din numarul celorlaltor bile. Daca extragem o bila,
calculati probabilitatea ca aceasta sa fie rosie sau albastra.
Marcel Rotaru, Brlad
Solutie. Not
am num
arul total de bile cu n. Cu ajutorul proportiilor derivate
3n
n
, iar al bilelor negre
obtinem c
a num
arul bilelor rosii este , al bilelor albe este
3
8
3n
n
este
. De aici rezult
a c
a num
arul bilelor albastre este
. Probabilitatea cerut
a
4
24
3
n/3 + n/24
= .
este atunci p =
n
8
AB
VI.34. n 4ABC, fie M mijlocul laturii [BC]. Daca d (M, AC) =
, aratati
2
b = 90 .
ca m(A)
N. N. Hrtan, Iasi
Solutie. Fie M N AC si BA0 AC, M, A0 AC. Atunci M N = d (M, AC) si
BA0
. Rezult
a de aici c
a BA0 = BA
M N este linie mijlocie n 4BA0 C, deci M N =
2
b = 90 .
si deoarece BA0 AC, rezulta ca m(A)
b = 45 . Bisectoarea [AD si naltimea
b = 60 si m(C)
VI.35. n 4ABC, m(A)
2
AM
[BE] se intersecteaza n M (cu D [BC], E [AC]). Sa se arate ca
= .
EC
3
Romeo Cernat, Iasi
\ = 180 90 45 = 45 , deci 4EBC este
Solutie. n 4ABC, m(EBC)
\ ) = 75 45 = 30 = m(BAM
\ ), 4ABM
isoscel cu EC = BE. Deoarece m(ABM

\
\
este isoscel cu AM = BM . n 4AEM , m(AEM ) = 90 si m(EAM ) = 30 , deci
AM
AM
AM
AM
AM
2
ME =
. De aici,
=
=
=
= .
2
EC
EB
BM + M E
AM + AM/2
3

Clasa a VII-a
VII.31. Sa se rezolve ecuatia 1 + a2x + b2x = ax + bx + ax bx , cu a, b R , a 6= b.
Dumitru Neagu, Iasi
Solutie. nmultind egalitatea cu 2 obtinem, dupa gruparea convenabila a terme54

nilor, (ax 1) (bx 1) + (ax bx ) = 0, de unde ax = bx = 1, deci x = 0.


VII.32. Fie a si b, c lungimile ipotenuzei
si respectiv
catetelor unui triunghi

2
1
1
1
dreptunghic. Sa se arate ca a + b2 + c2
+
+ 2 10.
a2 b2
c
Claudiu - S
tefan Popa, Iasi
Solutie. Deoarece a2 = b2 + c2 , obtinem ca

2
2
1
1
1
b2 + c2
2
a + b2 + c2
+
+
+
c
+

10

2
b
10
a2
b2 c2
b2 + c2
b2 c2
2
2

2
b + c2
4 b2 c2 0,

2
2
b c
ultima inegalitate fiind evidenta.
VII.33. Triunghiul ABC are unghiul A obtuz si semiperimetrul p. Cercurile de
diametre [AB] si [AC] delimiteaza o suprafata comuna S. Aflati valoarea de adevar
a propozitiei: Exista P S astfel nct d1 + d2 + d3 = p, unde di sunt distantele
de la P la laturile triunghiului ABC.
C
at
alin Calistru, Iasi
Solutie. Este cunoscut ca ntr-un triunghi dreptunghic mediana ce pleaca din
vrful unghiului drept este jumatate din ipotenuza, n timp ce ntr-un triunghi obtuzunghic mediana ce pleac
a din vrful unghiului obtuz este mai mic
a strict dect
jum
atate din latura ce se opune unghiului obtuz.
Fie P S si A1 , B1 , C1 mijloacele laturilor [BC], [CA], [AB]. Atunci d1 +d2 +d3
\
[
\
B) 90 , m(AP
C) 90 , m(BP
C) 90 , iar cel
P A1 +P B1 +P C1 . Cum m(AP
putin una din inegalit
ati este strict
a, prin aplicarea rezultatului mentionat anterior
c a b
obtinem c
a d1 + d2 + d3 < + + , deci d1 + d2 + d3 < p si propozitia din enunt
2 2 2
este falsa.
VII.34. Fie ABC un triunghi, iar I un punct interior lui. Daca cercurile
nscrise n triunghiurile AIB, BIC si CIA sunt congruente si tangente doua cte
doua, atunci 4ABC este echilateral.
Ioan S
ac
aleanu, Hrl
au
Solutie. Fie C1 (O1 , r), C2 (O2 , r), C3 (O3 , r) cercurile nscrise n 4BIC, 4CIA,
4AIB si not
am T1 = C2 C3 , T2 = C1 C3 , T3 = C1 C2 ; evident, T1 AI, T2 BI,
T3 CI. Atunci O1 O2 = O1 T3 + T3 O2 = 2r si analog O1 O3 = O2 O3 = 2r, deci
4O1 O2 O3 este echilateral. n patrulaterul IT1 O3 T2 ,

m(T\
1 IT2 ) = 360 90 90 60 = 120 ,
[ = 120 si analog demonstr
[ = m(CIA)
[ = 120 . n
adic
a m(AIB)
am c
a m(BIC)
4AIB,
b
b
b
[ m(IBA)
[ = 180 m(A) + m(B) = 90 + m(C) ,
[ = 180 m(IAB)
m(AIB)
2
2
b = m(B)
b = 60 , de unde obtinem c
b = 60 si analog m(A)
a 4ABC
de unde m(C)
este echilateral.
VII.35. Fie ABCD un paralelogram, O centrul cercului circumscris 4ABD, iar
H ortocentrul 4BCD. Sa se arate ca punctele A, O, H sunt coliniare.
Constantin Cocea si Dumitru Neagu, Iasi
55

\ = 90 ;
Solutie. Deoarece BH CD si AB k CD, BH AB si deci m(ABH)

\ = 90 . De aici rezult
analog obtinem c
a m(HDA)
a c
a ABHD este inscriptibil si cum

\
m(ABH) = 90 , [AH] este diametru pentru cercul circumscris triunghiului ABD,
deci l contine pe O.

Clasa a VIII-a

4n 1
VIII.31. Sa se determine multimea A = (m, n) N N |
N .
mn + 1
A. V. Mihai, Bucuresti
4n 1
N, rezult
a c
a 4n1 mn+1. De aici, n(4m)2 > 0
Solutie. Deoarece
mn+1
5
4n 1
si deci m < 4, adic
= 4
N,
a m {1, 2, 3}. Pentru m = 1 obtinem c
a
mn + 1
n+1
3
4n 1
= 2
N, deci n = 1.
ceea ce implica n = 4. Pentru m = 2, avem ca
mn + 1
2n + 1
4n 1
n2
Pentru m = 3 obtinem ca
= 1+
. Cum 3n + 1 > n 2, este necesar
mn + 1
3n + 1
ca n 2 = 0, deci n = 2. n final, A = {(1, 4) , (2, 1) , (3, 2)}.
VIII.32. Sa se rezolve ecuatia
1
2
2002
+ 2
+ ... + 2
= 2002.
2
x x+1 x x+2
x x + 2002
Mihaela Predescu, Pitesti
Solutie.
1
2
2002
+ 2
+ ... + 2
= 2002
2
x x+1 x x+2
x x + 2002

1
2
2002
1 +
1 + ... +
1 =0

x2 x + 1
x2 x + 2
x2 x + 2002

1
1
1
= 0.
x x2
+
+ ... + 2
x2 x + 1 x2 x + 2
x x + 2002

Cum x2 x + 1, x2 x + 2, . . . , x2 x + 2002 > 0, x R, rezult


a c
a x {0, 1}.
VIII.33. Determinati functiile f : R R pentru care

1 + f (x + y) f (x) + f (y) x + y + 2, x, y R.
Gheorghe Iurea, Iasi
Solutie. Pentru x = y = 0 obtinem 1 + f (0) 2f (0) 2, de unde f (0) = 1.
Fiindc
a f (x) + f (y) x + y + 2, pentru y = 0 g
asim c
a f (x) x + 1, x R.
Deoarece f (x) + f (y) 1 + f (x + y), pentru y = x obtinem ca f (x) 2 f (x).
Dar f (x) x + 1, de unde f (x) 1 + x, x R, ceea ce implica f (x) = 1 + x.

VIII.34. Fie AB dreapta solutiilor ecuatiei x y = 5 si CD dreapta solutiilor


ecuatiei x + y = 3, cu A, C Ox, B, D Oy. a) Aratati ca AB CD; b)
Calculati aria si perimetrul triunghiului BCD; c) Aratati ca AD BC.
Vasile Solcanu, Bogd
anesti (Suceava)
Solutie. a) Printr-un calcul imediat se deduce c
a A (5, 0), B (0, 5), C (3, 0),
D (0, 3). Fie AB CD = {E} si EF BD, F Oy. Atunci E (4, 1), F (0, 1).
56

Deoarece EF = BF = F D = 4 si EF BD, 4BEF si 4F ED sunt dreptunghice

\) = m(F
\
\
isoscele si m(BEF
ED)
= 45
, de unde m(BED) = 90 si deci AB CD.
b) SBCD = 12, PBCD = 34 + 3 2 + 8.
c) Deoarece AO si DE sunt n
altimi n 4BAD si AO DE = {C}, C este
ortocentrul 4BAD si deci BC este de asemenea naltime n 4BAD.
VIII.35. Fie ABCDA0 B 0 C 0 D0 un cub de muchie a. Determinati pozitiile punctelor M (BB 0 ) si N (CC 0 ) pentru care perimetrul patrulaterului strmb AM N D0
este minim; aflati aceasta valoare minima.
Mihaela Buc
ataru, Iasi
Solutie. PAMN D0 = AM + M N +
A'
B'
C'
D'
+N D0 + AD0 este minim AM + M N +
N
+N D0 este minim
a. Desf
asurnd suprafata
M
lateral
a a cubului ca n figur
a, S = AM +
0
+M N + N D0 este minima A, M, N,
D
0
sunt coliniare. Atunci Smin = AD = a 10,
0
A
B
C
D
iar valoarea
minim
a a PAMN D0 este AD +
+Smin = a 2 + a 10.

Clasa a IX-a

IX.31. Fie multimile


A = x2 + x |x Z , B = x3 + x | x Z , C = x4 +

+x3 + x2 + x | x Z , D = 2x4 | x Z . Determinati A C, B D, A D, A B.


Andrei Nedelcu, Iasi
Solutie. Fie u = n2 + n = m4 + m3 + m2 + m A C; m, n Z. Atunci
2
(2n + 1) = 4m4 + 4m3 + 4n2 + 4m + 1, si grupnd n dou
a moduri termenii din
membrul drept obtinem

(2n + 1)2 = 2m2 + m + 3m2 + 4m + 1 ,


(1)
respectiv

2
(2n + 1)2 = 2m2 + m + 1 + 2m m2 .
Folosind semnul functiei de gradul al doilea, obtinem

2
(1) (2n + 1)2 > 2m2 + m
m Z \ {1} ,

2
2
m Z \ {0, 1, 2} .
(2) (2n + 1) < 2m2 + m + 1
2

(2)

a ntre p
atratele a dou
a
Urmeaz
a c
a dac
a m Z \ {1, 0, 1, 2}, atunci (2n + 1) se afl
numere ntregi consecutive, ceea ce este imposibil.
Pentru m = 1, obtinem n = 0 sau n = 1 si u = 0. Pentru m = 0 obtinem din
nou n = 0 sau n = 1 si u = 0. Pentru m = 1 nu exist
a n cu proprietatea c
autat
a,
iar pentru m = 2 obtinem n = 5 sau n = 6 si u = 30. n concluzie, A C = {0, 30}.
Fie acum u = n3 +n = 2m4 BD; m, n Z. Atunci (n + 1)4 (n 1)4 = (2m)4 ,
ecuatie care nu are solutii nebanale, conform teoremei lui Fermat. Atunci n 1 = 0,
deci m = 1 sau m = 1, de unde u = 2, sau 2m = 0, deci n = 0, de unde u = 0. n
concluzie, B D = {0, 2}.
Analog se trateaza A D si A B.
IX.32. Fie fi , gi : R R, i {1, 2, . . . , n}, functii care pastreaza semnul
variabilei. Sa se rezolve sistemul:
57


x1 + f1 (x1 ) + g1 (x1 ) = x2

x2 + f2 (x2 ) + g2 (x1 + x2 ) = x3
...

xn + fn (xn ) + gn (x1 + x2 + . . . + xn ) = x1 .
Obtineti sisteme diverse prin particularizarea functiilor!
Iuliana Georgescu si Paul Georgescu, Iasi
Solutie. Pentru x1 > 0, deducem din prima ecuatie ca x2 > x1 > 0. n mod
asem
an
ator, 0 < x1 < x2 < . . . < xn < x1 , contradictie. Pentru x1 < 0, obtinem
analog c
a 0 > x1 > x2 > . . . > xn > x1 . De aici, x1 = 0 si analog x2 = . . . = xn = 0.
Exemple
de functii care pastreaz
semnul variabilei:
f (x) = x2n+1 , f (x) = arctag x,

a
2
2
f (x) = x 1 + x , f (x) = ln x + 1 + x etc.
hni
hni hni h n i h n i
+
+
= 11
.
IX.33. Rezolvati n N ecuatia n

2
3
12
18
36
Gheorghe
Iurea,
si
hni hni h n i h n i
h Ia
ni
+
+
.
Solutie. Fie f : N Z, f (n) = n

11
2
3
12
18
36
Avem ca f (n + 36) = f (n), n N, deci f periodica de perioada 36. De asemenea,
hni hni n n
hni
5n
f (0) = 0, iar pentru n = 1, 2, . . . , 35,
+
= 0,
+ =
< n, iar
2
3
2
3
6
36
deci 1, 2, . . . , 35 nu sunt solutii ale ecuatiei. n concluzie, multimea solutiilor ecuatiei
este S = {36k; k N}.
IX.34. Fie n N, n 2. Sa se determine x1 , x2 , . . . , xn R stiind ca

n+1

2
n
2
2
2
x1 + x2 + . . . + xn +
.
= 2 x1 sin
+ x2 sin
+ . . . + xn sin
2
n+1
n+1
n+1
Vladimir Martinusi, Iasi
Solutie. Egalitatea din enunt revine la
2 X
n
n
X
n+1
k
k
xk sin
=
sin2

.
n+1
n+1
2
k=1
k=1
na
(n + 1) a
sin
cos
n
P
1

cos
2a
2
2

si
cos ka =
,
Tinnd
seama de formulele sin2 a =
a
2
k=1
sin

2
n
2
P
k
k
xk sin
= 0, de unde xk = sin
, k = 1, n.
obtinem ca
n+1
n+1
k=1

x+y
, x, y 0,
.
IX.35. Aratati ca sin (cos x) + sin (cos y) < 2 cos
2
2
DanPopescu,
Suceava

, obtinem
Solutie. Tinnd
seama c
a sin x < x si 0 < cos x < 1, x 0,
2
x+y
xy
x+y
sin (cos x) + sin (cos y) < cos x + cos y = 2 cos
cos
2 cos
,
2
2
2
ceea ce trebuia demonstrat.

Clasa a X-a
X.31. Consideram sirurile (Fn )n0 , (Ln )n0 definite prin F0 = 0, F1 = 1,
L0 = 2,qL1 = 1, Ln+2 = Ln+1 + Ln , n 0.
Fn+2 = Fn+1 + Fn , n 0, respectiv
qP
pPn
Pn
n
2
2
2
L
F
Sa se arate ca
k=1 k
k=1 k1 +
k=1 Fk+1 .
Mihail Bencze, Brasov
58

Solutie. Se arat
a (prin inductie) c
a Ln = Fn1 + Fn+1 , n N . Conform
inegalitatii lui Minkowski, urmeaza concluzia.
X.32. Fie (an )n0 un sir de numere reale si pozitive. Pentru orice n N se
noteaza cu Pn graficul functiei fn (x) = an x2 + an+1 x + an+2 . Determinati legea de
definire a sirului stiind ca parabolele Pn au vrfurile pe axa Ox.
Temistocle Brsan, Iasi
Solutie. Conditia ca parabolele sa fie cu vrfurile pe Ox revine la n = a2n+1
4an an+2 = 0, n N. Prin logaritmare, obtinem 2 ln an+1 = ln an + ln an+2 + ln 4
n N, sau tn+1 = tn ln 4, n N, unde tn = ln an+1 ln an . Asadar, tn
este o progresie aritmetic
a cu ratia r = ln 4, si deci tn = t0 + nr, ceea ce ima1
t0 +nr
plic
a an+1 = e an , n N. Rezult
a c
a an = en ln a0 n(n1) ln 2 a0 , de unde
n
a0
a1
.
an = n(n1)
a0
2

X.33. Fie functiile f, g : R (0, ), f (x) = ax , g (x) = bx , unde a (1, ),


b (0, 1), ab 6= 1. Sa se arate ca exista o infinitate de paralelograme cu vrfurile pe
reuniunea graficelor celor doua functii.
Petru Asaftei, Iasi
Solutie. Fie x1 > 0 si B (x1 , g (x1 )). Deoarece f ((, 0)) = (0, 1), x2 < 0 astfel nct f (x2 ) = g (x1 ) si atunci si f (x2 ) = g (x1 ). Construim A (x2 , f (x2 )),
A0 (x2 , f (x2 )), B 0 (x1 , g (x1 )). Deoarece ab 6= 1 rezult
ac
a x2 6= x1 si
f (x2 )+f (x2 )
0 0
0
0
,
ABA B este nedegenerat. Segmentele AA si BB au mijloacele M 0,
2

g (x1 ) + g (x1 )
respectiv N 0,
, deci M N si ABA0 B 0 este paralelogram. Cum
2
x1 a fost arbitrar, problema este rezolvata.
X.34. Sa se arate ca ecuatiile 4 9x + (4x 45) 3x + 11 x = 0 si 4 18x
3 6x 128 3x + 2x + 32 = 0 sunt echivalente.
, Bucuresti
Marcel Chirita

1
1
x
x
(3 11 + x) = 0, deci 3x = ,
Solutie. Din prima ecuatie obtinem ca 3
4
4
a deoarece f1 : R
cu solutia x1 = 2 log3 2, sau 3x = 11 x, cu solutia x2 = 2, unic
(0, ), f1 (x) = 3x este strict crescatoare, iar f2 : R R, f2 (x) = 11x este strict
descrescatoare. Din cea de-a doua ecuatie obtinem ca (4 3x 1) (6x 2x 32) = 0,
1
32
a cu 3x = 1 + x .
deci 3x = , cu solutia x3 = 2 log3 2 sau 6x = 2x + 32, echivalent
4
2
32
Cum f3 : R (0, ), f3 (x) = 3x este strict cresc
atoare iar f4 : R R, f4 (x) = 1+ x
2
este strict descresc
atoare, ecuatia f3 (x) = f4 (x) are o singur
a solutie, anume x4 = 2.
X.35. Fie n N, n 3 si z1 , z2 , . . . , zn C de module egale astfel nct
zk
1 Q
zk + zj2 , j = 1, n. Aratati
C\ {1, i} 1 k 6= t n. Notam aj =
zt
zj k6=j
ca daca doua dintre numerele aj sunt reale, atunci toate sunt reale. n plus, daca
n
Q
zk = 1 (n legatura cu problema X.86 din R.M.T. nr. 3-4/2000).
n 6= 4, atunci
k=1
Daniel Jinga, Pitesti
59

Solutie. Fie P =

n
Q

k=1

zk si r = |zk |, k = 1, n. Atunci aj = zj2 +

P
, j = 1, n.
zj2

P
P2
2
z
,
b
=

z 2q . Atunci
p
q
zp2
zq2
ap = zp2 + zp 2 + bp si cum zp2 + zp 2 R, avem ca bp R. Analog obtinem ca bq R.
Din definitiile lui bp si bq obtinem ca
bp zp2 = bq zq2 = P r4 ,
(1)

Fie p, q 1, n astfel nct ap , aq R si notam bp =

de unde, prin trecere la module, deducem c


a |bp | = |bq |.
a zp2 = zq2 , de unde
Dac
a bp 6= 0, atunci bp = bq si conform (1) obtinem c
4
zp
zp
= 1. De aici,
{1, i}, contradictie. Atunci bp = 0 si din (1) deducem
zq
zq
|zj |4
ca P = r4 , de unde j = 1, n, aj = zj2 + 2 = zj2 + zj 2 R. Fie acum n 6= 4.
zj
4
a rn = r4 , deci r = 1 si atunci
Deoarece P = r , prin trecere la module obtinem c
P = 1.

Clasa a XI-a
XI.31. Fie A, B Mn (Q). Daca det A 6= det B, demonstrati ca det (A + B) 6=
6= det (B + A).
Paul Georgescu si Gabriel Popa, Iasi
Solutie. Fie P () = det (A + B) det (B + A). Evident, P Q []. Presupunem prin reducere la absurd ca det (A + B) = det (B + A). Atunci P () = 0
si cum nu este algebric, P 0. De aici, det (A + B) = det (B + A), R.
Pentru = 0 obtinem det A = det B, contradictie.
i
Pk
Pn h
p p
k
. Calculati
(1)
C
(k

p)
XI.32. Fie sn = k=1 k 2 + k + 1
k
p=0

p
lim n+1 1 + sn n 1 + sn1 .
n

S
tefan Alexe, Pitesti

Solutie. Suma k =

k
P

(1)

p=0

Ckp

(k p) reprezint
a num
arul functiilor surjec-

tive f : A A, unde A are k elemente. Cum orice asemenea functie este si injectiva
rezulta ca k = k! si deci
n
n h
n
n
i
X
X
X
X
2

2
(k + 1) k k! =
k + k + 1 k! =
(k + 1) (k + 1)!
k k! .
sn =
k=1

k=1

k=1

k=1

De aici, sn = (n + 1) (n + 1)! 1. Notam un = n+1 1 + sn n 1 + sn1 . Atunci


p

n
un = n+1 (n + 1) (n + 1)! n n! =

p
n n!

n n! n+1

n
n+1
n+1
n+1
n
(n + 1)
n+1
(n + 1)! n! +
n+1n n
n+1 .
=
n
n

n
p

1
1
n!
Este cunoscut ca lim n+1 (n + 1)! n n! = , lim
= si lim n n = 1.
n
n
n
n
e
e

De asemenea se poate demonstra usor c
a lim (n + 1) n+1 n + 1 n n n = 1 (de
n

exemplu, aplicnd teorema lui Lagrange functiei f : (0, ) (1, ), f (x) = x1+ x
60

pe intervalul [n, n + 1] si estimnd cantit


atile obtinute). De aici, limita din enunt
1
este egala cu .
e
XI.33. Fie (xn )n1 un sir de numere reale pozitive cu proprietatea ca exista
p N si numerele a1 , a2 , . . . , ap > 0 cu a1 + a2 + . . . + ap > 1 astfel nct xn =
= a1 xn+1 + a2 xn+2 + . . . + ap xn+p , n 1. Sa se arate ca inf {xn | n N } = 0.
Marian Tetiva, Brlad
a m = inf {xn | n N } si m 0. Din
Solutie. Deoarece xn 0, n N , exist
a xn a1 m + a2 m + . . . +
ipotez
a si din faptul c
a xn m, n N , deducem c

+ap m = am, n N , unde am notat a = a1 + a2 + . . . + ap > 1. n continuare,


xn a1 am + a2 am + . . . + ap am = a2 m, n N si prin inductie urmeaza imediat
c
a xn ak m, n, k N . Dac
a am presupune prin absurd c
a m > 0, am obtine c
a
xn lim mak = +, contradictie. R
amne c
a m = 0, ceea ce trebuia demonstrat.
k

XI.34. Fie a (0, 1) (1, ) si b un numar ntre 2 ln a si 2 ln a. Stabiliti


1
semnul functiei f : R R, f (x) = (ax + ax ) bx.
2
Gheorghe Costovici, Iasi
Solutie. Ar
at
am mai nti c
a
2

ax + ax
x2 (ln a)

+ 1.
(1)
2
2
ax + ax
x2 (ln a)2
si f2 : R R, f2 (x) =
+ 1 sunt
Deoarece f1 : R R, f1 (x) =
2
2
functii pare, este suficient s
a demonstr
am (1) doar pentru x [0, ). Atunci

2
2
(1) ax/2 ax/2 (x ln a) , x 0 ax/2 ax/2 x ln a, x 0.

ln a
ln a
+ ax/2

2
2
ln a 0, x 0, deci g este cresc
atoare si g (x) g (0) = 0, x 0.
2
i b
x2 h
x2 (ln a)2
2
2
2 (ln a) b +
+ 1 bx =
x 1 , deci
De asemenea,
2
4
2
Fie g : R R, g (x) = ax/2 ax/2 x ln a. Atunci g 0 (x) = ax/2

x2 (ln a)2
+ 1 bx > 0, x 0.
(2)
2
ax + ax
> bx, x R, deci f este strict pozitiv
a.
Din (1) si (2) obtinem c
a
2
n
n
XI.35. Fie f : [0, ) R o functie continua, iar (ak )k=1 , (bk )k=1 progresii geometrice astfel nct ak < bk , k = 1, n. Daca f (a1 a2 an ) < 1 si f (b1 b2 bn ) > 1,
aratati ca exista (ck )nk=1 progresie geometrica, ck (ak , bk ) astfel nct f (c1 c2 cn ) = 1.
Doru - Dumitru Buzac, Iasi
Solutie. Not
a
m
p
=
a
a
.
.
.
a
,
p
=
b
b
.
.
.
bn . Consider
am g : [0, 1] R,
a
1
2
n
b
1
2

; observam ca g (0) = f (pb ) > 1, iar g (1) = f (pa ) < 1. Deoarece


g (t) = f pta p1t
b
g este continua, t0 (0, 1) astfel ca g (t0 ) = 1.
n
n
0
Fie (ck )k=1 definit prin ck = atk0 b1t
. Se observ
a c
a (ck )k=1 este de asemenea
k
progresie geometric
a si ak < ck < bk , k = 1, n. n plus, deoarece g (t0 ) = 1, rezult
a
ca f (c1 c2 . . . cn ) = 1, ceea ce trebuia demonstrat.
61

Clasa a XII-a

XII.31. Fie M = A M2 (C) | A2 = I2 . Determinati toate subgrupurile lui


(M2 (C) , ) continute n M .
aeru si C
aeru, Suceava
Angela Tig
at
alin Tig
Solu
t
ie.
Prin
calcul
direct
observ
a
m
c
a
M
=
M
,
cu
M1 = {I2 , I2 } si

1
2

a b
2
| a + bc = 1, a, b, c C . n plus, M1 M2 = . Demonstr
am
M2 =
c a
acum urm
atorul rezultat auxiliar:
Lem
a. a) Dac
a A1 , A2 M astfel nct A1 A2 M , atunci A1 A2 = A2 A1 .
b) Daca A1 , A2 M2 astfel nct A1 A2 M , atunci A1 = A2 sau A1 = A2 .
Demonstratie. a) Fie A1 , A2 M astfel ca A1 A2 M . Atunci A21 = A22 = I2 si
de asemenea A1 A2 A1 A2 = I2 . nmultind ultima relatie la stnga cu A1 si la dreapta
cerut
cu A2 obtinem egalitatea
a.

a1 b1
a2 b2
, A2 =
b) Fie A1 =
M2 . Deoarece A1 A2 M , din
c1 a1
c2 a2
A1 A2 = A2 A1 , obtinem ca b1 c2 = b2 c1 , a1 b2 = a2 b1 , a1 c2 = a2 c1 , de unde A1 A2 =
= (a1 a2 + b1 c2 ) I2 . Cum A1 A2 M si (a1 a2 + b1 c2 ) I2
/ M2 , obtinem ca A1 A2 M1 ,
deci A1 A2 = I2 , de unde b).
Singurele subgrupuri H continute n M1 sunt H1 = I2 , H2 = {I2 , I2 }. Fie acum
H un subgrup al lui M , H M2 6= . Exista atunci A1 H M2 ; pentru o alta
matrice A2 H M2 obtinem ca A1 A2 H M deci, conform b), A1 = A2 sau
A1 = A2 . Obtinem de aici c
a, al
aturi de H1 si H2 , singurele subgrupuri ale lui
M2 (C) continute n M sunt cele de tipurile H3 = {I2 , A} si H4 = {I2 , I2 , A, A},
cu A M2 .

XII.32. Fie (G, ) grup, iar a G\{e} fixat. Aratati ca numarul morfismelor
surjective de la G la (Z3 , +) cu proprietatea ca f (x) = b
2 x = a este egal cu
numarul subgrupurilor H ale lui G care nu-l contin pe a si care au proprietatea ca
x3 H, x G.
Dana Stan, elev
a, Iasi
Solutie. A se vedea Nota de la p. 17 din acest num
ar al revistei.

XII.33. Fie f : R R o functie monotona al carei grafic are un centru de


simetrie ce nu apartine graficului. Aratati ca f nu admite primitive.
Oana Marangoci, elev
a, Pascani
Solutie. Deoarece A (x0 , y0 ) este centrul de simetrie al graficului lui f , are loc
egalitatea f (2x0 x) = 2y0 f (x), x R. Presupunem prin reducere la absurd
ca f este continua n x0 . Trecnd la limita n egalitatea de mai sus obtinem ca
lim f (2x0 x) = lim (2y0 f (x)), deci f (x0 ) = y0 , contradictie. De aici, f nu
xx0

xx0

este continua n x0 si, deoarece f este monotona, x0 este punct de discontinuitate de


speta nti. De aici rezulta ca f nu admite primitive.

XII.34. Fie f : [0, 1] (0, ) o functie care admite primitive si fie F o primitiva
c 2
a sa cu F (1) = 0. Aratati ca exista c (0, 1) astfel nct F (c) > ec f (c).
2
Rodica
Luca Tudorache, Iasi

x2
F (x). Se observa ca G este
Solutie. Fie G : [0, 1] R, G (x) = 1 e
continua pe [0, 1], derivabila pe (0, 1), iar G (0) = G (1) = 0. Conform teoremei lui
62

Rolle,
a c (0, 1) astfel nct G0 (c) = 0, ceea ce nseamn
a c
a 2cec F (c) +
exist
2
2
+ ec 1 f (c) = 0. Deoarece ex 1 > x2 , x (0, 1), obtinem imediat
inegalitatea din enunt.
Pn
XII.35. Fie f, g : R R, unde f este o functie injectiva si g (x) = k=0 a2k+1
2k+1
+ sin f (x), cu a1 [1, ) si a2k+1 (0, ), k = 1, n. Aratati ca f
[f (x)]
admite primitive daca si numai daca g admite primitive.
Lucian Georges L
adunc
a, Iasi
Solutie. Fie h : R R, h (x) = a1 x + a3 x3 + . . . + a2n+1 x2n+1 + sin x. Atunci
h0 (x) = a1 + 3a2 x2 + . . . + (2n + 1) a2n+1 x2n + cos x > 0, x R, prin urmare h este
strict cresc
atoare si deci injectiv
a. Cum lim h (x) = , lim = + si h este
x

continu
a, Im f = R, deci h este surjectiv
a si n final bijectiv
a.
Atunci g = h f , deci g este injectiv
a. Cum o functie injectiv
a admite primitive
daca si numai daca este continua, cu aceasta observatie cerinta problemei se rescrie
sub forma "f este continu
a g este continu
a". Dar acest lucru este imediat, tinnd
seama c
a h = g f , cu h continu
a si bijectiv
a.

IMPORTANT
n scopul unei legaturi rapide cu redactia revistei, pot fi utilizate urmatoarele adrese e-mail: tbi@math.tuiasi.ro, popagabriel@go.com .
Pe aceast
a cale colaboratorii pot purta cu redactia un dialog privitor la
materialele trimise acesteia, procurarea numerelor revistei etc.
La problemele de tip L se primesc solutii de la orice iubitor de matematici
elementare (indiferent de preocupare profesionala sau vrsta ). Fiecare
dintre solutiile acestor probleme - ce sunt publicate n revist
a dup
a un
an - va fi urmata de numele tuturor celor care au rezolvat-o.
Adres
am cu insistenta
amintea ca materialele trimise re rug
vistei s
a nu fie (s
a nu fi fost) trimise si altor publicatii.

63

Solutiile problemelor pentru preg


atirea concursurilor
din nr. 2 / 2002
A. Nivel gimnazial
n

G21. Sa se afle restul mpartirii prin 43 a numarului 62002 , n N.


C
at
alin - Cristian Budeanu, Iasi
n
n
Solutie. Avem c
a 2002n = (2001 + 1) = 3k + 1, cu k N . Atunci 62002 =
n
= 6 63k = 6 (5 43 + 1)k , de unde restul mpartirii lui 62002 la 43 este 6.

G22. Sa se arate ca ntre n si n! exista cel putin un numar prim, oricare ar fi


n N\ {0, 1, 2}.
***
Solutie. Fie p un divizor prim al num
arului n! 1 (care evident exist
a, eventual
fiind n! 1). Daca p n, atunci p | n!, si cum p | n! 1 rezulta ca p | 1, absurd. n
concluzie p > n.
G23. Sa se rezolve n Z2 ecuatia x2 + 10x + y! = 2002.

Adrian Zanoschi, Iasi


Solutia I (a autorului). Evident, trebuie impusa conditia y 0. Deoarece
x (x + 1) nu poate fi dect de forma 3k sau 3k + 2, k Z, de asemenea x2 + 10x =
= x (x + 1)+9x este de una din aceste forme. Dac
a y 3, atunci 3 | y! si x2 +10x+y!
este tot de forma 3k sau 3k + 2, dar 2002 = 3 667 + 1, absurd. n concluzie,
y {0, 1, 2}. Daca y = 0 sau y = 1, obtinem ca x2 + 10x = 2001, ecuatie care nu are
solutii n Z. Daca y = 2, atunci x2 + 10x = 2000, cu solutiile x1 = 40 si x2 = 50.
n concluzie, (x, y) {(40, 2) , (50, 2)}.

Solutia a II-a (dat


a de Alexandru
Bejinariu,
2 elev, Iasi). Pentru y 5,
2
u.
c.
(y!)
=
0.
n
plus,
u.
c.
x
+
10x
=
u.
c.
x {0, 1, 4, 5, 6, 9}, deci u. c.

2
x + 10x + y! nu ar fi 2. Ramne ca y < 5 si considernd pe rnd cele cinci cazuri,
obtinem solutiile de mai sus.

Solutia a III-a. Deoarece 7! = 5040 > 2002, avem c


a y < 7; problema se reduce
la rezolvarea a sapte ecuatii de grad II.

G24. Aflati cte numere de 4 cifre au proprietatea ca cifrele citite de la stnga


spre dreapta sunt invers proportionale cu cifrele citite de la dreapta spre stnga.
Aceeasi problema pentru numerele de 3, respectiv 5 cifre.
Gabriel Popa, Iasi
Solutie. Pentru a putea vorbi despre inversa proportionalitate, este necesar n
fiecare caz ca toate cifrele sa fie nenule.
a
b
(i) Dac
a abcd este un num
ar cu proprietatea din enunt, atunci
=
=
1/d
1/c
c
d
=
=
, deci ad = bc. Dac
a {a, d} = {b, c} iar a = d, obtinem 9 numere.
1/b
1/a
Daca {a, d} = {b, c} iar a 6= d, obtinem 4 36 = 144 numere (36 este numarul
perechilor de cifre nenule (a, d) cu a < d, iar corespunz
ator fiec
arei perechi se pot
ari). Dac
a {a, d} 6= {b, c}, atunci
obtine 4 numere abcd distincte prin diverse permut
{{a, d} , {b, c}} {{1, 4} , {2, 2}}, {{1, 6} , {2, 3}}, {{1, 8} , {4, 2}}, {{2, 6} , {3, 4}},
{{2, 8} , {4, 4}}, {{2, 9} , {3, 6}}, {{3, 8} , {4, 6}}, {{4, 9} , {6, 6}}. Obtinem 3 4+
+5 8 = 52 numere (pentru fiecare element subliniat obtinem 4 numere abcd, iar n
64

celelalte cazuri cte 8). n total obtinem 9 + 144 + 52 = 205 numere.


(ii) Daca abc este un numar cu proprietatea din enunt, deducem ca ac = b2 .
Daca a = c, obtinem 9 numere. Daca a 6= c, atunci b = 2, {a, c} = {1, 4}; b = 3,
{a, c} = {1, 9}; b = 4, {a, c} = {2, 8}; b = 6, {a, c} = {4, 9}. Obtinem 4 2 = 8
numere. n total obtinem 9 + 8 = 17 numere.
(iii) Daca abcde este un numar cu proprietatea din enunt, deducem ca ae = bd = c2 .
Daca a = b = c = d = e, obtinem 9 numere. n caz contrar,
c = 2,
c = 3,
c = 4,
c = 6,

{{a, e} , {b, d}} {{{1, 4} , {2, 2}} , {{1, 4} , {1, 4}}} ;


{{a, e} , {b, d}} {{{1, 9} , {3, 3}} , {{1, 9} , {1, 9}}} ;
{{a, e} , {b, d}} {{{2, 8} , {4, 4}} , {{2, 8} , {2, 8}}} ;
{{a, e} , {b, d}} {{{4, 9} , {6, 6}} , {{4, 9} , {4, 9}}} .

Obtinem 4 (4 + 4) = 32 numere. n total, obtinem 9 + 32 = 41 numere.


G25. Sa se arate ca pentru orice x, y, x R, exista a, b, c Z astfel nct
3
(x a)2 + (y b)2 + (z c)2 . Generalizare.
4
Vladimir Martinusi, Iasi

1
Solutie. Daca : R Z, (x) = x + , este functia de rotunjire, atunci
2
1
| (x) x| , x R. Pentru a = (x), b = (y), c = (z), obtinem concluzia
2
problemei. Cu acelasi rationament obtinem ca daca n N si x1 , x2 , . . . , xn R,
n
atunci exist
a a1 , a2 , . . . , an Z astfel ca (x1 a1 )2 +(x2 a2 )2 + +(xn an )2 .
4
G26. Daca suma, produsul si ctul a doua numere irationale sunt numere rationale, calculati suma cuburilor celor doua numere.
Claudiu
-S
tefan Popa, Iasi
a

a
Solutie. Daca + 1 6= 0, atunci a + b = b
+ 1 R \ Q, deoarece b R \ Q.
b
b
a
R
amne c
a + 1 = 0, deci a = b si a3 + b3 = 0.
b
Observatie. Ipoteza "ab Q" nu este necesar
a.
G27. Determinati n N minim pentru care 2n 1 este divizibil cu 125.
Gheorghe Iurea, Iasi
Solutie. Fie n = 4q + r, q, r N, 0 r < 4. Atunci 2n 1 = 2r 24q 1 =
= 2r 16q 1. Cum ultima cifr
a a lui 16q este 6 (q = 0 nu convine), ca 2n 1 s
a se
n
divida cu 5 e necesar r = 0, n caz contrar
ultima
cifr
a
a
lui
2

1
fiind
1,
3
sau
7.

Atunci 2n 1 = 24q 1 = 15 16q1 + 16q2 + + 1 si deci ca 2n 1 sa


se divid
a cu 125 este necesar si suficient ca 25 | 16q1 + 16q2 + + 1. Cum
p
p
16 = (15 + 1) = M5 + 1 pentru p 0, ca suma s
a se divid
a cu 5 este necesar si
suficient ca numarul termenilor sa se divida cu 5, deci q = 5k, k N . Atunci

2n 1 = 220k 1 = 10242k 1 = 1048576k 1 = 1048575 1048576k1 + + 1 .


Cum 1048575 este divizibil cu 25, dar nu si cu 125, este necesar si suficient ca 5 s
a
divida al doilea factor. Deoarece 1048576p = M5 + 1 pentru p 0, este necesar si
suficient ca numarul termenilor sa se divida cu 5, deci k = 5m, m N si n = 100m,
arul n minim este 100.
m N . Num
G28. Sa se arate ca ecuatia
65

x4 (a + b) x3 + (a + b + ab 2) x2 a2 + b2 a b x + (a 1) (b 1) = 0

are cel putin doua solutii reale pentru orice a, b R.


Marian Teler,
Costesti (Arge
s)

Solutie. Ecuatia data se descompune n x2 ax + b 1 x2 bx + a 1 = 0.


Se obtin ecuatiile x2 bx + a 1 = 0 si x2 ax + b 1 = 0. Atunci
1 + 2 = b2 4a + 4 + a2 4b + 4 = (a 2)2 + (b 2)2 0,

a ecuatii are solutii reale.


deci 1 0 sau 2 0 si cel putin una dintre cele dou
b < m(C).
b Pe bisectoarea interioara a unghiuG29. Fie triunghiul ABC cu m(A)
b luam un punct E astfel nct EAB
\ Se prelungeste latura [BC] cu
\ ACB.
lui B
segmentul [BD] [AB], B ntre C si D. Sa se arate ca mijlocul M al segmentului
[AC] se afla pe dreapta DE.
Constantin Chiril
a, Iasi
Solutia I (a autorului). Construim prin E o dreapt
a paralel
a cu AC care
\ si EAB
\
\ EBQ
intersecteaz
a AD si CD n P si Q. n 4EBA si EQB, EBA
\
\
EQB ( ACB). Cum [EB] [EB], 4AEB 4BEQ (ULU), deci [EA] [EQ].
\ obtinem c
\ Cum EAB
\ ACB,
\ BDA.
a
Deoarece 4ABD este isoscel, BAD
[
\ si deoarece CAD
\ EP
[ CAD,
A, 4EAP este isoscel, deci [EA] [EP ].
EAP
Urmeaz
a c
a [EP ] [EQ], adic
a [DE] este median
a n 4DP Q si atunci dreapta DE
njum
at
ateste segmentul [AC] paralel cu "baza" P Q.
Solutia a II-a. Fie {A0 } = BC AE
A
si {F } = AB DE. Deoarece 4ABC
c
a
b
4A0 BA, putem scrie: 0 = = 0 ,
E
M
AB
c
AA
2
F
bc
c
si A0 B = . Conform
de unde A0 A =
a
a
DB EA0
=
teoremei lui Menelaus, avem:

C A'
B
D
DA0 EA
c
DB
EA0
BA0
c
FB
0
si
=
(4ABA si DE). Cum
=
=
(teorema
=
FA
DA0
c + c2 /a
EA
BA
a
c
FB
bisectoarei), rezult
a c
a
=
. Pentru a dovedi c
a M DE, ar
at
am c
a
FA
a+c
punctele D, M , E sunt coliniare utiliznd reciproca teoremei lui Menelaus relativ la
c
a+c
DB M C F A

=
1
= 1.
4ABC; ntr-adev
ar, avem
DC M A F B
c+a
c
G30. Fie triunghiul AB0 B1 dreptunghic n B0 si triunghiurile ABi Bi+1 cu
ABi+1 ) = 30 , i N .
Bi Bi+1 ABi , i N , iar m(Bi\

a) Demonstrati ca punctele A, Bq si Br sunt coliniare, unde q = 2 n3 n + 1 ,


r = 32002 32000 + 31998 31996 + . . . + 32 30 .
b) Aflati aria triunghiului AB2001 B2002 functie de a = B0 B1 .
Romanta Ghita
si Ioan Ghita
, Blaj
Solutie. a) Observam mai nti ca daca i N, i = 12q1 + r1 , q1 , r1 N,
0 r1 < 12, atunci Br1 [AB
+ 1) (n 1) + 2, q = M12 + 2 si
i ]. Deoarece q = 2n (n
B2 [ABq ]. n plus, r = 8 32000 + 31996 + + 30 = M12 + 8 si B8 [ABr ]. Cum
B2 , A, B8 sunt coliniare, Br , A si Bq sunt de asemenea coliniare.
66

am c
a 4ABi Bi+1 4ABi+1 Bi+2 , i N, raportul de asem
anare fiind
b) Observ

22001
2 3
2 3
24001 2
. Atunci SAB2001 B2002 =
SAB0 B1 = 2001 a 3.
3
3
3

G31. Se considera un triunghi isoscel ABC cu baza BC = 2 2 cm. Fie punctele


variabile M (AB) si N (AC) astfel nct [AM ] [CN ]. Fie O mijlocul
segmentului [M N ] si P intersectia dreptelor AO si BC. Aflati perimetrul 4ABC
stiind ca aria minima a reuniunii suprafetelor triunghiulare [M BP ] si [N CP ] este

3 cm2 .
Adriana Maxiniuc, Botosani
\\
\
ACB,
ABC si cum ABC
Solutie. Fie N Q k AB, Q (BC). Atunci N QC \
4N QC este isoscel, deci [N C] [N Q]. Deoarece [N C] [AM ], rezult
a c
a [N Q] [AM ]
si cum N Q k AM , AN QM este paralelogram. Notam AQ M N = {O0 }. Atunci O0
njum
at
ateste [M N ], deci O0 O si atunci P Q, deci AM P N este paralelogram.
a
Deoarece SMBP +SN CP +SAMN P = SABC = constant, aria din enunt este minim
\ = AM (AC AM )
cnd SAMNP este maxima. Cum SAMN P = AM AN sin BAC
\ iar AM (AC AM ) este maxim cnd AM = AC , rezulta ca SAMNP
sin BAC,
2
este maxima atunci cnd M si N sunt mijloacele segmentelor [AB], respectiv [AC],
SABC
SABC
cu valoarea
. Atunci minimul ariei din enunt este de asemenea
, deci
2
2

lungimea naltimii din A este 6 cm. Rezulta AB = 2 2 cm, iar PABC = 6 2 cm.
b = m(D)
b = 90 , M un punct pe
G32. Fie patrulaterul convex ABCD cu m(A)
dreapta AD, iar N BC astfel nct M N BC. Aratati ca SCMB SAND .
Neculai Roman, Mircesti (Iasi)
Solutie (Constantin Tonu, elev, Iasi). Patrulaterul ABCD este dreptunghi
1
sau trapez dreptunghic. Fie N 0 proiectia lui N pe AD; atunci SCMB = M N BC,
2
1
SAN D = N N 0 AD. ns
a M N N N 0 , iar BC AD, de unde concluzia.
2
b = 2, fie D (BC) piciorul bisectoarei
G33. n triunghiul ABC cu m(A)
b iar M , N puncte pe (AB) respectiv (AC). Daca {P } = AD M N ,
unghiului A,
1
2 cos
1
demonstrati ca
+
=
(n legatura cu C:2402 din G.M. 5-6/2001).
AM
AN
AP
Mihaela Buc
ataru, Iasi
A
2 AM AN
cos , egalitate adev
arat
a, deoarece
Solutie. Relatia se scrie AP =
AM + AN
2
[AP ] este bisectoare n triunghiul AM N .
\ = 135 se nscrie patratul M N P Q cu
G34. n triunghiul ABC avnd m(BAC)
M, N (BC), P (CA)
si Q (AB). Aratati ca:
b
+
c
c AM
AM
2
BM
;
2
1
=
=
.
AN
CN
b AN
c+b 2
Temistocle Brsan, Iasi
\ = 135 , A este pe cercul circumscris patratului.
Solutie. Deoarece m(BAC)
\ ) = m(M
\
\
Observ
am de aici c
a m(BAM
AN ) = m(N
AC) = 45 . Not
am l = M N .
1 Deoarece AP M Q este inscriptibil, AM P Q = AP M Q + AQ M P , deci
67


AP
b
AQ
c
AP AQ
si
. Deoarece 4AQP 4AP C,
+ 2
=
= .
PQ
PQ
PQ
a
PQ
a

l
l
b + 2 c . Analog obtinem ca AN =
2 b + c , de unde rezulta
Atunci AM =
a
a
concluzia.
BM
AM
CN
si
=
=
2 Utiliznd teorema sinusurilor n 4ABM si 4ACN ,
sin 45
sin B sin 45
AN
BM
AM sin C
c AM
=
. Atunci
=

=
.
sin C
CN
AN sin B
b AN
G35. Fie [ABCD] un tetraedru. n planele (ABC), (ADC), (ADB) consideram
tangentele n A la cercurile circumscrise triunghiurilor ABC, ADC respectiv ADB,
MB
,
care intersecteaza dreptele BC, CD, DB n M , N respectiv P . Notam x =
MC
NC
PD
1
1
1
y=
,z=
. Sa se arate ca
+
+
1.
ND
PA
1+x 1+y 1+z
Marian Ionescu, Pitesti
M B AB M A
=
=
.
Solutie. n planul (ABC), observam ca 4AM B 4CM A, deci
M A AC M C
2

2
2
MB
AB
AC
MB MA
AB
si analog deducem ca y =
Atunci
, deci x =
,
=

=
2
MC
MA MC
AC
AC
AD2
AD2
1
1
z=
, prin urmare xyz = 1. Observam ca
+
= 1, deci inegalitatea
2
AB
1 + x 1 + yz
1
1
1
de demonstrat se reduce la
+

, care se obtine prin calcul direct.


1+y 1+z
1 + yz
AM = l

B. Nivel liceal
L21. Rezolvati n N2 ecuatia a2 + 3b2 = 2n , unde n N este fixat.
Gheorghe Iurea, Iasi
Solutie (dat
a de Andrei Nedelcu, Iasi). Dac
a a si b au parit
ati diferite,
atunci n = 0 si avem solutia a = 1, b = 0. Dac
a n > 0, atunci a, b au aceeasi

si atunci
paritate
fie a + b = 2x, x N si a b = 2y, y Z. nlocuind, obtinem

4 x2 xy + y 2 = 2n , de unde n 2. Fie d = (x, y); avem ca d = 2s , 2s n 2, iar


x = 2s x1 , y = 2s y1 , cu (x1 , y1 ) = 1. nlocuind din nou, g
asim x21 x1 y1 +y12 = 2n22s
si cum membrul stng este impar, n = 2 (s + 1) este numar par, iar x21 x1 y1 +y12 = 1.
Fiindca x1 , y1 N, cu x1 6= 0, obtinem ca x1 = y1 = 1 sau x1 = 1, y1 = 0, cu solutiile
n2
corespunzatoare a = 2n/2 , b = 0, respectiv a = b = 2 2 .
Cu aceast
a tehnic
a putem aborda rezolvarea ecuatiei n Q2 . S
a observ
am mai
nti c
a este suficient s
a g
asim solutiile (a, b) Q2+ , celelalte solutii fiind de forma
x
y
(a, b), (a, b), (a, b). Fie a = , b = , cu x, y, z, w N, (x, z) = (y, w) = 1.
z
w
Se observ
a usor c
a z = w si ecuatia devine x2 + 3y 2 = 2n z 2 (). Pentru n = 0,
putem scrie x2 y 2 = z 2 4y 2 , i. e. (x y) (x + y) = (z 2y) (z + 2y). Daca x = y,
1
xy
z + 2y
m
atunci z = 2y, deci a = b = . Dac
a x 6= y, atunci
=
= , de unde
2
x + y
n

z 2y

x = k m2 4mn + n2 , y = k m2 n2 , z = 2k m2 mn + n2 , cu k, m, n Z
astfel nct x, y, z 0.
si fie x + y = 2, x y = 2. Prin
Fie n 6= 0; atunci x, y au
aceeasi paritate

nlocuire n (), obtinem 4 2 + 2 = 2n z 2 . Daca n impar, aceasta ecuatie


68

nu are solutii, deoarece n membrul stng si n z 2 factorul 2 apare numai la puteripare. Daca n este par, fie (, ) = 2s , z = 2r z1 cu z1 impar si atunci 4
22s 21 1 1 + 21 = 2n 22r z12, deci 2s + 2 = n +2r, iar 21 1 1 + 21
= z12 ,
2
2
2
2
2
sau
cu solutia z1 = k m mn + n , iar (1 , 1 ) = k m n , k 2mn n
invers etc.
Not
a. Solutie corect
a s-a primit de la Alin Iacob, elev, Iasi.
L22. Fie n N un numar scris n baza 10. Acestui numar i adaugam la sfrsit
147, numarului obtinut i adaugam din nou la sfrsit 147 si asa mai departe. Aratati
ca printre numerele astfel obtinute exista numere compuse.
Adrian Zanoschi, Iasi
Solutie. Avem n1 = 1000n + 147 = 37 (27n + 4) + n 1, de unde obtinem
ca n1 n 1 (mod 37). Apoi n2 = 1000n1 + 147 = 37 (27n1 + 4) + n1 1,
deci n2 n1 1 (mod 37) n 2 (mod 37) si asa mai departe. Astfel obtinem c
a
numerele din sirul construit sunt congruente, modulo 37, cu n 1, n 2, . . . , n 37,
. . . , deci exista printre ele numere divizibile cu 37, adica numere compuse.
L23. Fie f : R R o functie periodica de perioada principala T astfel nct pe
[0, T ] f se anuleaza de un numar finit de ori si fie (xn )n1 un sir de numere reale
nenule. Aratati ca exista R astfel nct f (xn ) f ( + xn ) 6= 0 n N .
Paul Georgescu si Iuliana Georgescu, Iasi
Solutie. Fie {a1 , a2 , . . . , ak } multimea pe care f se anuleaz
a n [0, T ] si fie
!

! [
k
[
[ [ ai
T
+
(ai xn + T Z) .
A=
Z

xn xn
n=1 i=1
n=1 i=1
ai
T
ns
a
+
arabile, i 1, k, n N , deci si
Z si ai xn + T Z sunt multimi num
xn xn
A este num
arabil
a. Cum R este nenum
arabil
a, R \ A 6= .
ai
T
+
/ ai + T Z,
Z, i 1, k, n N , deci xn
Fie R \ A. Atunci
/
xn
xn
a c
a f (xn ) 6= 0, n N . Analog deducem c
a
i 1, k, n N , ceea ce nseamn

f ( + xn ) 6= 0, n N , de unde rezult
a concluzia problemei.
L24. Fie A, B Mn (R) matrice nesingulare cu det A + det B = 0. Exista > 0
astfel nct A2 B B 2 A = A?
C
at
alin Calistru, Iasi
Solutie. Presupunem prin reducere la absurd ca exista > 0 astfel ca A2 B
B 2 A = A. nmultind la dreapta cu A1 , obtinem ca A2 BA1 = B 2 + In . nsa

det B 2 + In = det B + i In det B i In = det B + i In ,

2
iar det A2 BA1 = det A det B = (det A) < 0, contradictie.
L25. Fie A Mn (R) o matrice cu proprietatea ca exista m N, m 3
si R, || 1, astfel nct Am+1 Am A + In = On . Sa se arate ca
|det A| = 1.
Lucian Georges L
adunc
a, Iasi
Solutie. Fie PA (X) = det (XI A) = (X x1 )k1 (X x2 )k2 . . . (X xl )kl
polinomul caracteristic al matricei A. Cum x1 , x2 , . . . , xl sunt r
ad
acini si ale lui
mA , polinomul minimal al matricei A peste C, iar mA divide X m+1 X m A + 1,
rezulta ca x1 , x2 , . . . , xl au modulele egale cu 1, deoarce ultimul polinom are toate
69

r
ad
acinile de modul 1. Dar PA (0) = det (A) = (1)
n

Pl

i=1 ki

xk11 xk22

. . . xkl l ,

Pl

i=1

ki

xk11 xk22 . . . xkl l . Atunci

det (A) = (1) det A = (1)


de unde |det A| = 1.
L26. Fie f : Sn Sn endomorfism astfel nct exista Sn pentru care
(f f ) () = 1 , Sn . Aratati ca f are un punct fix (i.e. Sn ,
f () = ).
Ovidiu Munteanu, Brasov
Solutie. Fie Sn cu proprietatea din enunt. Este usor de observat ca functia
: Sn Sn , () = 1 este bijectiv
a. Relatia din enunt se scrie sub forma
f f = , de unde rezult
a c
a f este bijectiv
a.

Pentru Sn , (f f f ) () = f ( ()) = f 1 = f ( ) f () f 1
1
si, de asemenea,
1 (f f f ) ()1= (f ()) = f () . Rezulta de aici ca
= f () , Sn si, deoarece
f ( ) f ()
f este surjectiv
a,

f
f ( ) f 1 = 1 , Sn . Deci 1 f ( ) = 1 f ( ) , Sn si cum
1 f ( ) comut
a cu toate elementele lui Sn , 1 f ( ) = e. Obtinem c
a f ( ) = .

L27. Fie (A, +, ) un inel cu unitate si n, k N , k impar, astfel nct xn+k = xn ,


x A. Sa se arate ca xk+1 = x, x A (n legatura cu C: 1896 din G.M. nr.1/1997).
Dragos Deliu si Marian Tetiva, Brlad
Solutie. Daca n = 1, nu avem nimic de demonstrat. Fie acum n 2. Deoarece
(1)n+k = (1)n , iar n si n + k au paritati diferite, avem ca 1 = 1, deci 1 + 1 = 0
si n concluzie 2a = a + a = 0, a A.
Demonstr
am acum c
a n inelul A este valabil
a implicatia x2 = 0 x = 0. ntr2
s
adevar, fie x A astfel ca x = 0. Atunci x = 0, s N, s 2 si aplicnd formula
binomiala rezulta ca (1 + x)p = 1 + px, p N. Cum (1 + x)n+k = (1 + x)n din
ipotez
a, obtinem c
a 1 + (n k) x = 1 + nx, de unde kx = 0. Deoarece k este impar
si 2x = 0, rezult
a imediat c
a x = 0, deci implicatia este dovedit
a.
Din ipoteza, xn+k = xn , x A, deci si xm+k = xm , x A, pentru m n.
Vom avea atunci
2

n+k1
x
xn1 = x2n+k2+k x2n+k2 x2n2+k x2n2 = 0,
ceea ce implic
a xn+k1 xn1 = 0. Am obtinut c
a xn+k1 = xn1 ; repetnd
n+k2
n2
k+1
rationamentul obtinem ca x
=x
, ... , x
= x, q.e.d.
0 0 0
L28. Fie ABC si A B C doua triunghiuri ascutitunghice. Daca
a 2 b 2 c 2
S

0 0,
a0
b0
c
S
aratati ca triunghiurile sunt asemenea.
Ioan S
ac
aleanu, Hrl
au
sin A
sin B
sin C
Solutie. Notam cu x =
, y =
, z =
. Folosind teorema
sin A0
sin B 0
sin C 0
2
2
sinusurilor si formula S = 2R sin A sin B sin C, obtinem c
a x y 2 z 2 xyz. De
aici, x y z si xy z. Atunci si xy y, de unde x 1 si deci z y x 1.
0 0 0
Deoarece
h i 4ABC si 4A B C sunt ascutitunghice, iar functia sinus este crescatoare
b m(A
c0 ), m(B)
b m(B
c0 ),
pe 0, , din x 1, y 1, z 1 obtinem ca m(A)
2
b m(C
c0 ). Cum m(A)
b + m(B)
b + m(C)
b = m(A
c0 ) + m(B
c0 ) + m(C
c0 ) = 180 ,
m(C)
b = m(A
c0 ), m(B)
b = m(B
c0 ), m(C)
b = m(C
c0 ), deci cele dou
rezult
a c
a m(A)
a triunghiuri
sunt asemenea.
70

Not
a. Solutie corect
a s-a primit de la Alin Iacob, elev, Iasi.
L29. Fie ABC un triunghi, iar C un cerc tangent laturilor [AB] si [AC] n F ,
respectiv E si care intersecteaza latura [BC] n M si N . Fie X un punct interior
triunghiului astfel nct exista un cerc C1 tangent laturilor [XB] si [XC] n Z, respectiv Y si care taie [BC] tot n M si N . Demonstrati ca patrulaterul EF ZY este
inscriptibil.
Neculai Roman, Mircesti, (Iasi)
Solutie. Dac
a [AB] [AC], punctul X va fi n mod necesar pe axa triunghiului
ABC (altfel C1 n-ar exista). Ca urmare, patrulaterul EF ZY este trapez isoscel, deci
este inscriptibil. Fie acum [AB] 6 [AC] si fie {K} = F E BC, {K 0 } = ZY BC
a deoarce triunghiul XBC nu va fi nici el isoscel). Aplicnd teorema lui
(K 0 exist
KB EC F A
Menelaus pentru 4ABC cu transversala KF E obtinem

= 1, deci
KC EA F B
FB
KB
=
. Aplicnd teorema lui Menelaus pentru 4XBC cu transversala KZY
KC
EC0
K 0B
ZB
K B Y C ZX

= 1, deci 0 =
. Dar F B = ZB si Y C = EC
obtinem 0
K C Y X ZB
KC
YC
KB
K 0B
(deoarece B si C apartin axei radicale a cercurilor C1 si C), prin urmare
= 0 ,
KC
KC
a pe axa radical
a a cercurilor C1 si C, avem
de unde K = K 0 . Deoarece K se afl
KE KF = KY KZ, deci punctele E, F , Y , Z sunt conciclice.
Not
a. Solutie corecta s-a primit de la Alin Iacob, elev, Iasi.
L30. Fie ABC un triunghi echilateral, iar P un punct n planul triunghiului. Notam cu A1 , B1 , C1 simetricele lui P fata de BC, CA si respectiv AB. Sa se arate
ca se poate forma un triunghi avnd lungimile laturilor egale cu AA1 , BB1 , CC1 .
Constantin Cocea, Iasi
Solutie. Fie A (a), B (b), C (c), P (p) si fie O (0) centrul cercului circumscris
4ABC. Putem presupune, f
ar
a a restrnge generalitatea, c
a a = 1, b = , c = 2 .
bc
bc
Avem BC =
=
= bc, de unde P A1 = bc, deoarece P A1 BC.
1/b 1/c
bc

Fie {A0 } = P A1 BC. Ecuatia dreptei BC este zb = bc z b si poate fi rescris


a
sub forma z + bcz = b + c. Ecuatia dreptei P A1 este z p = bc (z p). De aici
1
obtinem c
a afixul a0 al punctului A0 este dat de egalitatea a0 = (b + c + p bcp).
2
p + a1
Cum a0 =
a c
a a1 = b + c bcp si
, unde a1 este afixul punctului A1 , urmeaz
2
n mod similar, b1 = c + a cap, c1 = a + b abp.
De aici, (a a1 )+(b b1 )+(c c1 ) = 0 si se obtine ca |a a1 |+|b b1 | |c c1 |,
de unde AA1 +BB1 CC1 . Analog obtinem ca AA1 +CC1 BB1 si BB1 +CC1 AA1 ,
ceea ce trebuia demonstrat.
L31. Fie (an )n1 un sir de numere reale astfel nct a1 = a > 1, an+1 =
p
an
p
= an + ap1
1, n N , unde p N, p 2. Sa se calculeze lim p (n
n
n n
legatura cu C:1463 din G.M. nr.11/1993 ).
Viorel Cornea si Dan S
tefan Marinescu, Hunedoara
Solutie. Deoarece a > 1, putem demonstra
prin inductie c
a an > 1, n 1,
p
p
sirul (an )n1 este
de unde, tinnd seama c
a an+1 an = ap1

1,
ob
t
inem
c
a
n
strict crescator. Daca (an )n1 ar fi marginit superior, atunci ar avea o limita finita
71


p
l. Trecnd la limit
a n relatia de recurenta s-ar obtine c
a l = l + lp1 1, deci
l = 1, contradictie. n concluzie, lim an = +.
r
n
an+1
1
1
a
= 1+ p
,
mp
artind acum n relatia de recurenta prin a1 obtinem c
an
an an

p a
an+1
n
cu ajutorul lemei lui
de unde lim
= 1. Calcul
am acum limita lim
n an
n n
Cesaro - Stolz.Observam
ca

p a
p a
a
an
n+1
n
q n+1
lim
=
= lim q
p
p
n
n
p2
n+1n
p
p
ap1
an+1
an + . . . + ap1
n
n+1 +
r
1
1 p p1
1
an
= lim s
= .
p1 s
p2
n
p
an+1
an+1
p
+ p
+ ... + 1
an
an

p a
1
1
an
n
= , deci lim p = p .
De aici, lim
n n
n
p
n
p

xn+1
L32. Fie (xn )n1 un sir de numere naturale care satisface conditia
= xn ,
xn
xn
n 1, iar x1 = 2. Sa se arate ca exista > 1 pentru care lim 2n = 1.
n
Cristinel Mortici, Trgoviste
xn+1
2
< xn + 1, deci xn xn+1 < x2n + xn ; n particular,
Solutie. Avem c
a xn
xn
ln xn
n1
putem deduce prin inductie ca xn 22 , n 1. Notam yn =
. Atunci
2n

2
ln xn+1 /xn
ln (1 + 1/xn )
1
, deci 0 yn+1 yn <
.
<
yn+1 yn =
2n+1
2n+1
xn 2n+1
ator si, n plus,
Se obtine c
a (yn )n1 , este monoton cresc
n
n
X
X
1
(yk+1 yk ) y1 +
,
yn+1 = y1 +
k+1
2
k=1

k=1

de unde yn+1 < y1 + 1/2, adica (yn )n1 este marginit superior. Deducem ca (yn )n1
este convergent si fie ln limita sa, > 1. Deoarece
p1
p1
X
X
1
(yn+k+1 yn+k ) <
,
yn+p yn =
xn+k 2n+k+1
k=0

k=0

1
obtinem ca yn+p yn <
. Cum (yn )n1 este monoton crescator, pentru p
xn 2n
ln xn
1
1
obtinem ca 0 ln yn
. De aici, 0 ln n
, deci
n
xn 2
2
xn 2n
n
n
2
2
xn
1
0 ln
. Pentru n obtinem ca lim ln
= 0, deci lim 2n = 1.

n
n
xn
xn
xn
L33. Fie m N, m 2 fixat. Aratati ca solutiile continue ale ecuatiei
functionale
m !
m
!
X
X
X
f
xi +
f (xi xj ) = m
f (xi ) , xi R, i = 1, m,
i=1

i=1

1i<jm

72

sunt functiile de forma f (x) = cx2 , cu c R.

Adrian Corduneanu, Iasi


2
Solutie. Pentru x1 = x2 = . . . = xm = 0, obtinem ca f (0) + Cm
f (0) = m2 f (0),
deci f (0) = 0. Pentru x1 = x2 = . . . = xm1 = 0 si xm = x, obtinem c
a
f (x) + (m 1) f (x) = mf (x), x R, deci f (x) = f (x), x R si f este
functie para. Este deci suficient sa determinam f pe (0, ).
Notam f (1) = c. Pentru x1 = x2 = 1 si x3 = . . . = xm = 0, obtinem ca
f (2) + (m 2) f (1) = 2mf (1), deci f (2) = 4c = c 22 . Pentru x1 = n, x2 = 1 si
x3 = . . . = xm = 0, obtinem c
a f (n + 1) + (m 2) f (n) + f (n 1) + (m 2) f (1) =
= m (f (n) + f (1)) si deci f (n + 1) = 2f (n) f (n 1) + 2f (1). De aici se demonstreaza usor prin inductie ca f (n) = cn2 , n N.
Pentru x1 = x2 = . . . = xm = x R+ obtinem c
a f (mx) = m2 f (x). De aici,

n 2
x
n

f (x)
f (x)
x

=
=
=
c
f
,
de
unde
f
,
s
i,
prin
induc
t
ie,
f
m
m2
m2k
mk
mk
omk
n n
; n, k N . Din cele demonstrate mai sus, f (x) = cx2 ,
n N. Not
am M =
mk
x M . Demonstr
am c
a x > 0 si > 0, x M astfel ca |x x | < . Fie
1
a n N
k N astfel ca k < min {x, }. Conform axiomei lui Arhimede, exist
m

n
n
1
n
n + 1

,
deci
0

<
<
,
de
unde
astfel ca k x <
x
< .
m
mk
mk
mk
mk
1
n
Rezult
a c
a putem alege x = k . Pentru = , l N, obtinem c
a (xl )l1 M
m
l
1
astfel ca |x xl | < , l 1, deci xl x pentru l . Cum f (xl ) = cx2l , iar f
l
este continu
a, obtinem c
a f (x) = cx2 . n concluzie, f (x) = cx2 , x R.
L34. Se considera sirul de functii (fn )nN , fn : [2, ) R definit prin f0 (x) =
p
= x, fn (x) = 2 + fn1
Notam
(x), n 1.
4n

1 + lim fn (x)
, daca x [2, 2]
n
2
f (x) =
.
n

lim [3 fn (x)]4 , dac


a x (2, )
n

Aratati ca f (x) defineste o functie pe [2, ) si cercetati daca aceasta functie admite
primitive.
S
tefan Alexe, Pitesti
Solutie. Fie mai nti x [2, 2]. Se observ
a c
a functia g : [0, ] [1, 1],
x
g (x) = cos x este bijectiv
a. n concluzie, exist
a un unic t = t (x) [0, ], t = arccos ,

t2
x
astfel nct cos t = , deci f0 (x) = 2 cos t. Atunci f1 (x) = 2 + 2 cos t = 2 cos si
2
2
t
se demonstreaz
a prin inductie c
a fn (x) = 2 cos n .
2
Fie acum x (2, +). Se observ
a c
a functia h : (0, ) (1, ), h (x) =
ex + ex
=
= ch x este bijectiva. n concluzie, exista un unic t = t (x) (0, ),
!
2

x
x2 4
x
+
astfel nct ch t = . Atunci f1 (x) = 2 + 2 ch t =
t = ln
2
2
2

t
= 2 + et + et , deci f1 (x) = et/2 + et/2 = 2 ch si se demonstreaz
a prin in2
73

t
ductie ca fn (x) = 2 ch n .
2
G
asirea expresiei functiei f se reduce la calculul unor limite elementare.
Se obtine

2 x+

2 x

x2 4

2
a x [2, 2], respectiv f (x) = e ln
, dac
a
c
a f (x) = 1 + e 2 arccos 2 , dac
x (2, +). Cum lim f (x) = 2, iar lim f (x) = 1, x = 2 este punct de discontinui-

x2
x<2

x2
x>2

tate de speta nti pentru f , deci f nu admite primitive pe [2, +).


L35. Sa se arate ca exista si este unic > 1 astfel nct
Z /4
Z /4
2
ex
sin x
dx

dx
=
ex + sin x
ex + sin x
64
0
0

(se stie ca 0, 45 < e/4 < 0, 46).


Gabriel Popa si Paul Georgescu, Iasi
Z /4
Z /4
ex
sin x
Solutie. Notam I = I () =
dx,
J
=
J
()
=
dx;
x
x
e + sin x
e + sin x
0
0

am c
a
se deduce imediat c
a I + J = , > 0. Observ
4
2
1

2
I
I =
I= .
IJ =
64
4
64
8

R
amne s
a demonstr
am c
a exist
a un unic > 1 astfel ca I () = . Fie 0 < 1 <
8
< 2 < . Deoarece
Z /4
ex sin x
I (1 ) I (2 ) = (2 1 )
dx,
x
(e + 1 sin x) (ex + 2 sin x)
0

obtinem ca I (1 ) > I (2 ) si |I (1 ) I (2 )| (2 1 ), deci I () este o


4
functie continua strict descrescatoare pe (0, ).
h i
ex
Fie > 0 si f : 0,
R, f (x) = x
. Atunci
4
e + sin x
h
2ex sin (x /4)
0,
,
<
0,
x

f0 (x) =
2
4
(ex + sin x)
h i
deci f este strict descresc
atoare pe 0, . Aplicnd teorema de medie integralei
4

astfel ca
I (1), obtinem ca exista c 0,
4

ec
e/4

>
I (1) = c
>
.
(1)
4 e + sin c
4 e/4 + 2
7
2
h i
Fie acum 0, . Atunci
8
Z
Z /4

ex
ex
e
I () =
dx
+
dx +

.
x
x
e + sin x
4
e + sin
0 e + sin x

seama de (1), de continuiPentru obtinem ca lim I () < . Tinnd

8
tatea lui I () si de stricta sa monotonie, obtinem c
a exist
a un unic > 1 astfel ca

I () = , de unde obtinem concluzia problemei.


8
74

Probleme propuse
Clasele primare
P.54. Calculati a si b daca 46 a = 36 + a si b 3 = 17 b.
( Clasa I )
nv. Doinita Spnu, Iasi
P.55. n cte moduri pot fi aranjate n linie dreapta 9 mingi rosii si una galbena?
( Clasa I )
Georgiana Ciobanu, elev
a, Iasi
P56. Cu cinci ani n urma, suma vrstelor a trei copii era de 11 ani. Care va fi
suma vrstelor acelorasi copii peste 6 ani?
( Clasa a II-a)
nv. Rodica Rotaru, Brlad
P.57. n cte moduri pot fi mp
artiti 8 b
aieti n dou
a echipe de cte 4 juc
atori,
dac
a Petru vrea s
a fie n echip
a cu Mihai si Dan, dar nu vrea s
a fie cu Avram?
( Clasa a II-a)
Adina Dohotaru, elev
a, Iasi
P. 58. S
a se arate c
a suma 1 + 4 + 7 + . . . + 100 mp
artit
a la 3 d
a restul 1.
( Clasa a III-a)
Alexandru - Gabriel Tudorache, elev, Iasi
P.59. Fie a si b doua numere consecutive. Suma acestor numere mpreuna cu
numerele obtinute marind cu 12 fiecare dintre vecinii lor este 939. Care sunt cele
dou
a numere?
( Clasa a III-a)
nv. Maria Racu, Iasi
P.60. Din 16 bile, una este mai grea dect celelalte 15, care au mase egale. Care
este cel mai mic num
ar de cnt
ariri prin care se poate stabili bila mai grea?
( Clasa a III-a)
Carmen Ciolacu, elev
a, Iasi
P.61. Suma a doua numere este un numar de doua cifre al caror produs este 3.
Diferenta dintre cele doua numere este 7. Care sunt cele doua numere?
( Clasa a IV-a)
nv. Maria Racu, Iasi
P.62. Doua ceasuri au nceput sa functioneze la aceeasi ora. Se constata ca la
fiecare 30 minute (fata de ora exact
a) unul r
amne n urm
a cu un minut iar cel
alalt
avanseaz
a cu un minut. La un moment dat orele indicate de aceste ceasuri sunt: 18
h 36 min si 19 h 24 min. La ce ora au nceput sa functioneze?
( Clasa a IV-a)
Felicia Amih
aiesei, elev
a, Iasi
P.63. Alege un numar format din trei cifre. Scrie la dreapta lui un numar format
din dou
a cifre. Scoate din num
arul format de 99 ori num
arul format din trei cifre.
Din rezultat scoate diferenta dintre num
arul de trei cifre si num
arul de dou
a cifre si
scrie rezultatul. Eu ti ghicesc numarul format din doua cifre. Cum se explica acest
lucru?
( Clasa a IV-a)
Prof. Petru Asaftei, Iasi

Clasa a V-a
V.41. Fie a numar natural compus astfel nct daca p | a, cu p prim, atunci
p + 1 | a. S
a se arate c
a 12 | a si s
a se afle cel mai mare num
ar a de trei cifre.
Ciprian Baghiu, Iasi
a se
V.42. Se dau numerele xy, ab scrise n baza 10 astfel nct xy divide ab. S
arate c
a x = y dac
a si numai dac
a a = b.
Ioan S
ac
aleanu, Hrl
au
75

V.43. S
a se afle cifrele a si b stiind c
a a b = cd si ab = dc.
Romanta Ghita
si Ioan Ghita
, Blaj
V.44. S
a se afle x, y, z Q+ pentru care xn = yz, y n = xz, z n = xy, cu n N.
N. N. Hrtan, Iasi
V.45. Se dau sase urne, unele continnd bile. Fie operatia: se aleg trei urne si
se pune cte o bil
a n fiecare dintre ele.
a) Compozitia urnelor fiind 0, 0, 4, 6, 6, 8, s
a se indice o succesiune de operatii n
urma carora toate urnele sa contina acelasi numar de bile.
b) Compozitia urnelor fiind 0, 1, 2, 3, 4, 4, sa se arate ca nu exista o succesiune de
operatii n urma c
arora toate urnele s
a contin
a acelasi num
ar de bile.
Gheorghe Iurea, Iasi

Clasa a VI-a
VI.41. Pe opt cartonase sunt nscrise cte unul din numerele 1, 2, 22 , 23 , 3,
3 , 33 , 34 . Dac
a P (k) este probabilitatea ca, extr
agnd dou
a cartonase, numerele
obtinute sa aiba n total k divizori distincti, sa se rezolve inecuatia P (k) 1/7.
Dumitru Dominic Bucescu, Iasi
VI.42. Fie x, y, z N pentru care 84x + 91y + 98z = 2002. S
a se afle valoarea
maxima a sumei x + y + z.
Adrian Zanoschi, Iasi

VI.43. Fie {a1 , a2 , . . . , an } Z pentru care k {1, 2, . . . , n}, i, j {1, 2, . . . , n},


i 6= j, astfel nct ak = ai + aj . Sa se arate ca n 6.
Petru Asaftei, Iasi
VI.44. Fie ABCD un paralelogram si M AB, N AD triunghiuri echilaterale construite n exteriorul acestuia. Demonstrati c
a [M N ] [BD] dac
a si numai dac
a
N D k M B.
Ciprian Baghiu, Iasi
VI.45. Fie E, F picioarele n
altimilor din B si C ale triunghiului ascutitunghic
ABC. Dac
a P , N sunt mijloacele
laturilor [AB],
respectiv [AC], iar {Q} = P EF N ,

b . (n legatura cu Q1086 din Parabola,


\
s
a se arate c
a m(P
QF ) = 180 3 m(A)
nr. 3/2000 )
Titu Zvonaru, Bucuresti
2

Clasa a VII-a
a
b
+
= 1.
b+1 a+1
Alexandru Negrescu, elev, Botosani
2
2

2
VII.42. S
a se arate c
a a + 1 b + 1 c + 1 (|a| + |b|) (|b| + |c|) (|c| + |a|),
a, b, c R.
Dorin M
arghidanu, Corabia
VII.43. Pentru n N, not
am cu s (n) num
arul de reprezent
ari distincte ale
lui n ca sum
a de dou
a numere naturale (n = a + b si n = b + a constituie aceeasi
reprezentare). Sa se arate ca:
n
hni hn + 1i
X
1
mn
.
s (k) =

a) s (m + n) = s (m) + s (n) [1 + (1) ]; b)


2
2
2
k=0
Petru Minut, Iasi
VII.41. Rezolvati n N2 ecuatia

76

VII.44. Fie [AB] diametru al cercului C de centru O, N, M C astfel nct


\ ) = 36 , iar [OM este bisectoare pentru N
\
m(AON
OB. Dac
a T este simetricul lui
O fata de M N , s
a se arate c
a proiectia lui T pe AB este mijlocul lui [AO].
Valentina Blendea, Iasi
VII.45. Fie 4ABC echilateral, iar P (BC). Not
am cu D, E simetricele lui P
fata de AC, respectiv AB. S
a se arate c
a dreptele AP , BD si CE sunt concurente.
Constantin Cocea si Julieta Grigoras, Iasi

Clasa a VIII-a
VIII.41. Fie f1 , f2 , f3 functii liniare ale caror grafice sunt drepte concurente
doua cte doua. Cele trei drepte sunt concurente daca si numai daca exista unic
R si exist
a u 6= v R astfel ca
f1 (u)
f2 (u)
f3 (u)
=
=
, cu fi (v) 6= , i {1, 2, 3} .
f1 (v)
f2 (v)
f3 (v)
Claudiu S
tefan Popa, Iasi
VIII.42. Fie x, y, z (0, ). Sa se arate ca

yz + zx
xy + xz
p
p
+
+

yz + zx + x2 + y 2 + z 2 yz zx
xy + xz + x2 + y 2 + z 2 xy xz

xy + yz
p
2.
+

xy + yz + x2 + y 2 + z 2 xy yz

Lucian Tutescu, Craiova


VIII.43. Dac
a un triunghi dreptunghic are laturile numere naturale, iar suma
catetelor este p
atrat perfect, atunci suma cuburilor catetelor este sum
a de dou
a
patrate.
Andrei Nedelcu, Iasi
VIII.44. Pe laturile [AB], [CD], [BC], [AD], [AC] si [BD] ale tetraedruBP
AQ
lui ABCD se iau respectiv punctele M , N , P , Q, R, S astfel ca
=
,
BC
AD
DN AR
DS
AM
=
,
=
. Notam cu V1 , V2 , V3 , V4 , V respectiv volumele tetraedrelor
AB
DC AC
BD
AM RQ, BP M S, CP N R, DN QS si ABCD. S
a se arate c
a 212 V1 V2 V3 V4 V 4 .
Viorel Cornea si Dan S
tefan Marinescu, Hunedoara
VIII.45. Fie A1 , A2 , . . . , Ak puncte pe un cerc C. S
a se determine o conditie
necesar
a si suficient
a pentru a putea nscrie n C un poligon regulat cu n laturi, ce
admite punctele date ca vrfuri (nu neap
arat consecutive).
Irina Mustata
a, Iasi
, elev

Clasa a IX-a
IX.41. Pentru n N, n 10, notam cu u2 (n) numarul format din ultimele doua
cifre ale lui
a se arate c
a:
n. S
a) u2 a20k+p = u2 (ap ), p {4, 5, . . . , 23}, k N, a {2, 3, 8};
b) u2 a10k+p = u2 (ap ), p {2, 3, . . . , 11}, k N, a {4, 9};
n
c) u2 (5
) = 25, n N;
d) u2 65k+p = u2 (6p ), p {2, 3, . . . , 6}, k N;
77

e) u2 74k+p = u2 (7p ), p {2, 3, 4, 5}, k N.

Ovidiu Pop, Satu Mare


1
1
1
IX.42. Fie a, b, c R astfel nct a + b + c = + + .
a b
c
a) Daca |abc| > 1, sa se arate ca unul dintre numere este mai mare n modul ca
1, iar altul mai mic n modul ca 1.
b) S
a se afle numerele dac
a |abc| = 1.
Marius Pachitariu, elev, Iasi
IX.43. Fie functia f : R R, iar a (1, ). S
tiind c
a

1
1
f x2 + ax a f 2
+ 1, x (, 0) ,
f
x
x
s
a se arate c
a f nu este injectiv
a.
Titu Zvonaru, Bucuresti
IX.44. Dac
a 4ABC
este ascutitunghic,
a se g
aseasc
a maximul expresiei
s
E = sin A cos A + sin B cos B + sin C cos C.
Cezar Lupu, elev, si Tudorel Lupu, Constanta
IX.45. Demonstrati ca 4ABC n care are loc egalitatea
X
ha hb mc
= 1,
ma mb mc + ha hb mc + ma mb ic
suma fiind obtinut
a prin permut
ari circulare, iar notatiile fiind cele uzuale, este
echilateral.
Iuliana Georgescu si Paul Georgescu, Iasi

2
F2n
2n
, where the Fibonacci numbers Fn
X.41. Prove the inequality

n
Fn1 Fn
are defined by F0 = F1 = 1, Fn+1 = Fn + Fn1 , n 1.
Zdravko Starc, Vrac, Serbia and Muntenegro
X.42. S
a se rezolve ecuatia 2[x] + 6[x] + 7[x] = 3[x] + 4[x] + 8[x] .
Daniel Jinga, Pitesti
X.43. Fie f o functie reala nenula cu proprietatea ca

Clasa a X-a

f (x + y xy) = f (x + y) f (x) f (y) ,

2003
.
S
a se calculeze f
2002

x, y R.

Adrian Zanoschi, Iasi


X.44. Urnele U1 , U2 , . . . , Un contin fiecare cte a bile albe si b bile negre. Din
fiecare urn
a se extrage cte o bil
a care se depune ntr-o alt
a urn
a U . Din urna U se
scoate o bil
a si se constat
a c
a este alb
a. Care este compozitia cea mai probabil
aa
urnei U ?
Petru Minut, Iasi
X.45. Se considera triunghiul ascutitunghic ABC. Sa se arate ca exista un
0B0) =
\
triunghi A0 B 0 C 0 astfel nct A0 (BC), B 0 (AC), C 0 (AB), iar m(AC
0
0
0
0
\
\
= m(BA C ) = m(CB A ) = (0, 90]. Daca n plus 4ABC este echilateral, sa se
calculeze lungimile laturilor 4A0 B 0 C 0 n functie de a = BC si . (n legatura cu o
problema propusa la O. N. M., 2002 )
Dan Popescu, Suceava
78

Clasa a XI-a
XI.41. Fie A1 , A2 , . . . , Ak Mn (Z) astfel nct

A(1) A(2) . . . A(k) = In ,


.
unde Sk este multimea permut
arilor de ordin k. S
a se arate c
a n .. k!.
Vladimir Martinusi, Iasi
XI.42. Prin punctele M1 si M2 ale unei elipse se duc normalele la elipsa, care
intersecteaza una din axele de simetrie ale acesteia n M10 , respectiv M20 . Sa se
arate c
a mediatoarea segmentului [M1 M2 ] trece prin mijlocul lui [M10 M20 ]. R
amne
proprietatea adev
arat
a pentru hiperbol
a sau pentru parabol
a?
Gheorghe Costovici, Iasi
XI.43. Consideram sirul de functii fn : (0, ) R, fn (x) = nx + ln x (n 1)
si fie xn solutia unic
a a ecuatiei fn (x) = 0. S
a se calculeze limitele sirurilor (xn )n1
si ((xn )n )n1 .
aeru, Suceava
Angela Tig
XI.44.
S
a se determine
functiile continue f : (0, ) (0, ) pentru care f (x) =

2x2 2x + 1 , x > 0.
=f
Marian Urs
arescu, Roman
,
a
,
.
.
.
,
a
XI.45. Fie k N, k 2 si numerele
reale
pozitive
a
1 2
k , b1 , b2 , . . . , bk cu
p
a1 < a2 < < ak . Definim xn = n b1 an1 + b2 an2 + . . . + bk ank .
a) Sa se demonstreze ca lim xn = ak ;
n
b) Sa se arate ca lim n (xn ak ) = ak ln bk ;
n
n

ak
c) Dac
a bk = 1, are loc lim n
(xn ak ) = ak bk1 .
n
ak1
Marian Tetiva, Brlad

Clasa a XII-a
Z (1 + x) 1 + x2 . . . 1 + x2n1
dx, unde x [1, ),
XII.41. Sa se calculeze
x2n

nN .
Oana Marangoci, student
a, Iasi

Z 5
4

5
f (x) sin 2x dx = .
XII.42. Fie f : ,
R o functie continu
a pentru care
4
4

5
astfel nct f (c) (1, 2).
S
a se arate c
a exist
a c ,
4
Mihai Haivas, Iasi
XII.43. S
a se arate c
a

Z 1
ln a
arctg ln a
x2

1
, a > 1.
a
dx

3
ln a
0
Petru R
aducanu, Iasi
XII.44. S
a se afle num
arul r
ad
acinilor reale ale polinomului P Z [X] de grad
minim, care admite r
ad
acina 2 + , unde verific
a ecuatia x3 x + 1 = 0.
Laurentiu Modan, Bucuresti
XII.45. Fie S5 . Sa se arate ca 2 are puncte fixe daca si numai daca 3 are
puncte fixe.
Paul Georgescu si Gabriel Popa, Iasi
79

Sk

Probleme pentru preg


atirea concursurilor
A. Nivel gimnazial
G46. Determinati ultimele cinci cifre ale num
arului
A = 72000 + 72001 + 72002 + 72003 .
Viorel Cornea si Dan S
tefan Marinescu, Hunedoara
G47. Determinati valorile parametrilor a, b Z pentru care solutiile sistemului
y
x
x=a
; y=b
y+1
x+1
sunt n Z Z.
Temistocle Brsan, Iasi
2002
si avnd proprietatea ca, daca
G48. Fie A (0, ) o multime care contine
2003
a
+
1
a
a
A (a, b N ), atunci
A si
A. Sa se arate ca A Q+ .
b
b
2b
Gheorghe Iurea, Iasi
G49. Fie x1 , x2 , . . . , xn+1 R+ astfel nct x1 + x2 + . . . + xn+1 (n + 2) m, iar
n+4 2
a se arate c
a
M , unde m = min xi , M = max xi . S
x21 + x22 + . . . + x2n+1
4
exact n dintre numerele date sunt egale.
Eugen Jecan, Dej
G50. Fie a N, a 3. Sa se arate ca

an + 1 =
an + 2 = . . . =
an + a 1 , n N a {3, 4} .

Ovidiu Pop, Satu Mare

3
a se arate c
a
G51. Fie a, b, c
, cu a + b + c = 1. S
10

2
3
a a + bc + b b + ca + c c + ab < .
3
4
Gabriel Dospinescu, elev, Onesti
G52. Se consider
a o piramid
a format
a din p
atrate 1 1,
avnd n trepte, pe treapta k existnd 2k 1 p
atrate (n figur
a,
n = 4). Aflati numarul minim de dreptunghiuri, fiecare alcatuit
numai din casute ntregi, n care poate fi mpartita tabla.
Adrian Zahariuc, elev, Bac
au
G53. Fie ABCD un patrat de latura 70.
Sa se arate ca exista o multime de
patrate Pk = Ai Bi Ci Di | Ai Bi = i, i = 1, k care sa aiba suma ariilor egala cu aria
p
atratului dat. Putem acoperi p
atratul ABCD cu elementele multimii Pk ?
Petru Asaftei, Iasi
G54. Sa se arate ca nu putem alege nici un punct n interiorul triunghiului
echilateral ABC de latur
a l 10, care s
a aib
a distantele la vrfuri numere prime
distincte.
Doru Buzac, Iasi
G55. Printr-un punct situat n interiorul unui tetraedru se duc planele paralele cu
fetele tatraedrului. Daca V1 , V2 , V3 , V4 sunt volumele tetraedrelor unic determinate

80

de aceste plane, iar V este volumul tetraedrului dat, s


a se arate c
a
V 16 (V1 + V2 + V3 + V4 ) .
Neculai Roman, Mircesti (Iasi)

B. Nivel liceal

b si B
b se
L46. Fie ABCD un patrulater inscriptibil. Bisectoarele unghiurilor A
intersecteaza ntr-un punct situat pe latura [CD]. Sa se arate ca CD = AD + BC.
Mircea Becheanu, Bucuresti
L47. Dac
a un triunghi are p
atratele laturilor n progresie aritmetic
a, atunci
simetricul centrului de greutate fata de latura mijlocie se afl
a pe cercul circumscris
triunghiului.
Gabriel Popa si Paul Georgescu, Iasi
L48. Fie R, r, R1 raza cercului circumscris 4ABC, raza cercului nscris 4ABC,
respectiv raza cercului circumscris 4DEF determinat de picioarele bisectoarelor
interioare ale 4ABC. Sa se arate ca R/2 R1 r.
Marian Tetiva, Brlad
L49. ntr-un p
atrat 10 10 se nscriu numerele 1, 2, 3, . . . , 100 n asa fel nct
oricare dou
a numere consecutive s
a se afle n c
asute vecine. Demonstrati c
a exist
ao
linie sau o coloana ce contine macar doua patrate perfecte.
Adrian Zahariuc, elev, Bac
au
a avnd a1 = 5, r = 2002. Pentru un
L50. Fie (an )n1 o progresie aritmetic
element b al progresiei, s
a se arate c
a bm apartine progresiei dac
a si numai dac
a
60 | m 1.
Mihai Piticari, C-lung Moldovenesc

L51. Fie A, B M2 (R) dou


a matrice care comut
a si pentru care det A2 + B 2 <
2
< (det A + det B) . S
a se arate c
a xA + yB este matrice nesingular
a, x, y R .
C
at
alin Calistru, Iasi
L52. Fie Q C [X] un polinom de grad m avnd radacinile distincte. Sa se
determine cardinalul multimii
E = {P C [X] | A Mn (C) a. . Q (A) = On si P (X) = det (XIn A)} .
Ovidiu Munteanu, Brasov

L53. Fie n 2 si (A, +, ) un inel comutativ cu n2 elemente, care are cel mult
n 2 divizori ai lui zero. S
a se arate c
a A este corp.
Gabriel Dospinescu, elev, Onesti
L54. Fie f : R R o functie cu derivata continu
a pentru care f (x) 6= 0, x 6= 0.
Sa se determine functiile continue : R R care satisfac identitatea
Z y

Z x

1
1
f (x)
(t) dt (y) = f (y)
(t) dt (x) , x, y R,
a
a
0
0
unde a 6= 0 este o constant
a dat
a.
Adrian Corduneanu, Iasi
a1
a
; xn =

L55. Fie a (0, ) \ {1}. Definim sirul (xn )n1 prin x0 =


ln a
ln a
n
atati c
a sirul este convergent si calculati lim xn si lim nxn .

xn1 , n 1. Ar
n
n
ln a
Gheorghe Iurea, Iasi
81

Premii acordate de FUNDA


TIA POIANA
Fundatia Poiana, prin d-l Dan Tiba, pune la dispozitia revistei "Recreatii
matematice" suma de 100 care se constituie ca fond de premii acordate elevilor colaboratori care se disting prin calitatea articolelor, notelor si problemelor originale
aparute n paginile revistei.
Redactia revistei decide premierea cu cte 1 000 000 lei a urmatorilor elevi:
1. DOSPINESCU Gabriel (Liceul "D. Cantemir", Onesti )
Combinatorica . . . algebrica (RecMat 2/2003, 1922),
probleme propuse: G.51, L.53;

2. PACHITARIU
Marius (Colegiul National, Iasi )
Cteva aplicatii ale teoremei lui Casey (RecMat 1/2003, 3031),
probleme propuse: IX.37, IX.42;
OANA (Colegiul National "C. Negruzzi", Iasi )
3. CRJA
Aplicatii ale rotatiei planului complex (RecMat 2/2002, 2123),
Un procedeu de calcul al limitelor unor siruri de forma (an+1 an )n1
(RecMat 2/2003, 2324).
Premiile se pot ridica direct de la redactie sau pot fi trimise prin mandat postal.

Premii acordate rezolvitorilor


Pentru aparitia de trei ori la rubrica "Pagina rezolvitorilor" redactia revistei
"Recreatii matematice" acorda o diplom
a si un premiu n carti elevei
SOFICU Crina Maria (Scoala
nr.3 "Al. Vlahuta", clasa a IV-a, Iasi ):

RecMat 1/2002 (6 pb), 2/2002 (6 pb), 1/2003 (5 pb).


C
artile au fost oferite de c
atre

Editura PARALELA 45

82

Pagina rezolvitorilor
BOTO
SANI
Scoala
nr. 7 "Octav Bancila". Clasa a VIII-a. NEGRESCU Alexandru: VI(31,

37), VII(31,36,38), VIII(31,33,37), IX.31, G(23,26,27,33,38-41).


CRAIOVA
Colegiul National "Fratii Buzesti". Clasa a V-a. AL KHATIB Anne Marie:

P(51,53), V(36,37,39); BAZA-VERDE


Daniela: P(50-53), V.36; DECA Alexandra

Maria: P(51-53), V(37,40); PATRA


SCU Andrada: P(50-53), V.37. Clasa a VI-a.
POPESCU Mihnea: V.37, VI(37-40). Clasa a VII-a. VASILE Teodor: VI(37-40),
VII(39,40).
Scoala
nr. 22 "M. Eliade". Clasa a III-a (nv. STAICU Angela). STANCIU

Ioan: P(44-50,52,53).
IA
SI
Colegiul National. Clasa a VII-a. COSBUC Mircea: VI(36-40), VII(37-40),
VIII.37; IANUS Andrada: VI(36-39), VII.40; PRELIPCEAN Cristina: VI(37,38,40),
VII(36-40), G.40. Clasa a VIII-a. ANDRIES Delia: VI(36-39), VII.36, G.40;
APETROAEI Georgiana: VII(36,39,40), VIII(39,40); BALAN Doru: VI(36,39),

VII.38, VIII.40, G.40; BALANIUC Dragos: VI(36-38,40), VII.39; BATRNU


M
ad
alina: VI(36-40), VII(36,39), VIII.37; BELCESCU Cosmin Cezar: VI(37,38), VII(36 Ariel: VI(37,38), VII(36,38,39); CHIRUTA
Marta: VI(36,38),
38,40); CHELSAU
VII(36,38-40), VIII.37; CHITIC Ionut: VI(36,38,40); VII(36-39); VIII.37; CONSTANTIN Diana: VII.39, VIII(37-40); CROITORU Cosmina: VI(36-39), VII(36,
Tudor: VI(36,38,40), VII(38,40), VIII.39; DO38), VIII.36, G(37,40,42); DOBRILA

DEA Andreea: VI.40, VII(38,40), VIII.36, G.40; FALTICEANU


Paul: VI(36-40),

VII.40; FLORESCU Darian: VI(37-40), VII(36,38,40); GRADINARIU


Ioana Alexan
dra: VII(36,39,40), VIII(37,38); GRADINARU
Andrei: VI(36,37,39,40), VII(39,
40); ILIESCU Alca-Iolanda: VI(36,38,39), VII.39, VIII.37; LUCA Paul: VI(36-38),
VII(38-40); MACUC Andra: VI(36-39), VII.39; MATEI Silvia: VII(36-40), VIII.37;
MARARI Cezarina: VI(36-38), VII(36,37,39); MARTINUS Luciana: VII(36-39),
G.40; SAVENCU Ramona-Irina: VI(36-38), VII.39, VIII.37; TOMESCU Sebastian:

VI(36,38-40), VII.36; TUCALIUC Vlad: VI(36,38,40), VII(38-40); TURCANU


Roxana: V.37, VI(36,38), VII(38,40); VNTU Calin: VII(36-40), VIII.40; ZANOSCHI
Delia: VI(36,39, 40), VIII(36,38); ZANOSCHI Iulia: VI(36,38), VII.38, VIII(36,38).

Clasa a IX-a. PACHITARIU


Marius: VIII(36-40), IX(36-40), X(37-40), G(36,38,
Irina: VIII(36-38,40), IX(36-40), X(36A
41,42), L(36,39). Clasa a X-a. MUSTAT
38), XII.36.
Colegiul National "C.Negruzzi". Clasa a V-a. HARAGA Anca-Elena: P(5052), V(37,38), VI(37,38). Clasa a VII-a. DICU Ciprian-Dinu: VI(38,39), VII(3638). Clasa a X-a. BEJINARIU Alexandru: VIII(31-35), IX(32-35), X(31-33),
G(23,25,26,28,30,32,33).
Liceul "Garabet Ibraileanu". Clasa a IV-a (nv. LISNIC Sebastian). TIBA Marius P(47-53). Clasa a VI-a. BUDEANU S
tefana: P(52,53), V(37,39), VI.38; LUCA
Matei: P(52,53), V(37,39), VI.38; PLACINSCHI Oana: P(41,42,50), V(31,33). Clasa

a VIII-a. TANASE
Ioana: VI(37-40), VII.40, VIII.40.
83

Liceul Teoretic "M.Eminescu". Clasa a V-a. CIURARU Ionela: P(42,43,50-53),


V(31,33,37-39); MASTALERU Alexandra: P(50,52), V(36,37,39).
Scoala
nr. 17 "I. Creanga". Clasa a VII-a. IFTODE Andreea: VI(33-35),

VII.32, IX.31; PAULIN Ana Maria: VI(33-35), VII(31,32,34); SOFRONEA Gabriela:


VI(33-35), VII(31,32); TANANA Irina-Eliza: VI(33-35), VII(31,32).
Scoala
"G.Cosbuc". Clasa a II-a (nv. GALIA Paraschiva). ALUPEI Andra:

Oana-Catalina: P(37,44-47); GHERCA


Catalin: P(37,44P(37,44-47); CIOABA
47); HOMEA Liviu: P(37,44-47); HUIDES Gina: P(37,44-47); IGNAT Andrei:

Alexandru: P(37,44-47);
P(37,44-47); MIHAILESCU
Laura: P(37,44-47); PISICA
SCUTARU Constantin: P(37,44-47). Clasa a II-a (nv. RACU Maria). BARA Ioana: P(37, 44-47); BULGARU Ionela: P(37, 44-47); CALOIAN Andrei:
BULA

P(37, 44-47); CALIN


Georgiana: P(37, 44-47); CRACIUN
M
ad
alina: P(37, 44-47);

IFROSA Adriana: P(37, 44-47); IOJA Petru-Alexandru: P(34,44-47); LEAGAN


Crina-Alexandra: P(37, 44-47); MOISA Bogdan: P(37, 44-47); PINTILIE Razvan:
P(37, 44-47); RUSU Flavia: P(37, 44-47).
Scoala
"Alexandru cel Bun". Clasa a II-a (nv. SPNU Doinita). BURLACU

Ionut-Mihai: P(44-47,53); DAMIAN Daniel: P(44-47,53); FLOREA Roxana-Maria:


Marta: P(44-47,53); IFTENIE Ioana-Catalina: P(44-47,53);
P(44-47,53); FURTUNA
RUSU Alexandru: P(44-47,53); URSU Gina-Ioana: P(44-47,53).
Scoala
"N.Tonitza". Clasa I (nv. TUDOSE Elena). ANCHIDIN Alexandru:

P(33,34,44-46); CRNU Alina: P(33,34,44-46); DOBRIN Diana: P(33,34,44-46);


Simona: P(33,34,44-46); ROTARIU
LEONTE Anca: P(33,34,44-46); POSTICA
Larisa-Maria: P(33,34,44-46). Clasa I (nv. MELINTE Rodica). BACIU Ciprian:
P(33,34,44-46); BRZU Constantin: P(33,34,44-46); BOTOSANU Bianca-Mihaela:
D
P(33,34,44-46); BUZDUGAN Petru-C
at
alin: P(33,34,44-46); CEUCA
anut-Vasilic
a:
P(33,34,44-46); CONSTANTINESCU Diana-Gabriela: P(33,34,44-46); CUCUTEANU Paul-C
at
alin: P(33,34,44-46); GUSOVATE Diana-Stefana: P(33,34,44-46); LEOGAN Larisa-Diana: P(33,34,44-46); MIRON Vlad-Stefan: P(33,34,44-46); MOTAN
Geanina-Diana: P(33,34,44-46); ROTARIU Marian: P(33,34,44-46); SUCIUC Raluca:
P(33,34,44-46); TEIU-COSTIN Andra: P(33,34,44-46). Clasa a III-a (nv. MARCU
Alin: P(44-49).
A
Monica). BUTNARU Valentin: P(44-49); ONUT
ALI
TEI

Scoala
"B.P.Hasdeu". Clasa I (nv. TRZIORU Iuliana). ADASC
Vic
tor: P(33,34,44-46); BALAN Andrei: P(33,34,44-46); CUBERSCHI PAUL: P(3335,44-46); ESANU Georgiana: P(33,34,44-46); GREIEROSU Claudiu: P(33,34,44 ATIC

46); LAM
Ioana: P(33-35,44-46); REBEGEA Andrada: P(33,34,44-46). Cla Eduard-Andrei:
sa I (nv. TUTU Laura). BUHU Vlad: P(33,34,44-46); BUZA
Alexandra-Elena: P(33,34,44-46); GURAU
RalucaP(33,34,44-46); CHICHIRAU

Claudia: P(33,34,44-46); HATESCU Iustina: P(33,34,44-46); NASTASE


Andrei:
P(33,34,44-46); SIMIRAD Andrei: P(33,34,44-48). Clasa a IV-a (nv. S
TEFAN
Liviu). PINTILIE Liviu: P(44-53); PINTILIE Nicoleta: P(44-53); S
TERBULEAC
Daniel: P(44-53).
Beatrice). TUDORACHE
Scoala
"T.Maiorescu". Clasa a III-a (nv. CHIRILA

Alexandru-Gabriel: P(44-53).

84

Anul VI, Nr. 2

Iulie Decembrie 2004

RECREAII
MATEMATICE
REVIST DE MATEMATIC PENTRU ELEVI I PROFESORI

e i = 1

Editura Crengua Gldu


IAI, 2004

Semnificaia formulei de pe copert:


i
ntr-o form concis, formula e = 1 leag cele patru ramuri fundamentale ale matematicii:
ARITMETICA
GEOMETRIA
ALGEBRA
ANALIZA MATEMATIC

reprezentat
reprezentat
reprezentat
reprezentat

de
de
de
de

i
e

Redacia revistei :
Petru ASAFTEI , Temistocle BRSAN, Dan BRNZEI, Ctlin - Cristian BUDEANU,
Constantin CHIRIL, Eugenia COHAL, Adrian CORDUNEANU, Mihai CRCIUN
(Pacani), Paraschiva GALIA, Paul GEORGESCU, Dumitru GHERMAN (Pacani),
Gheorghe IUREA, Lucian Georges LDUNC, Mircea LUPAN, Dan tefan
MARINESCU (Hunedoara), Gabriel MRANU, Andrei NEDELCU, Gabriel POPA,
Dan POPESCU (Suceava), Florin POPOVICI (Braov), Maria RACU, Ioan ERDEAN
(Ortie), Dan TIBA (Bucureti), Lucian Tuescu (Craiova), Adrian ZANOSCHI.

Responsabili de numr :
Temistocle BRSAN, Gabriel POPA, Paul GEORGESCU, Gheorghe IUREA,
Lucian Georges LDUNC, Mircea LUPAN, Andrei NEDELCU

Adresa redaciei:
Catedra de Matematic Universitatea Tehnic Gh. Asachi Iai
Bd. Carol I, nr.11, 700506, Iai
Tel. 032 213737 / int. 123
E-mail: acord@math.tuiasi.ro

EDITURA CRENGUA GLDU


Toate drepturile rezervate
ISSN 1582 - 1765
Bd. N. Iorga, Bl. K2, ap. 4, IAI
Tel. / Fax: 032 - 230598

Anul VI, Nr. 2

Iulie Decembrie 2004

RECREAII
MATEMATICE
REVIST DE MATEMATIC PENTRU ELEVI I PROFESORI

e i = 1
Apare cu sprijinul
FILIALEI IAI a SOCIETII de TIINE MATEMATICE
i INSPECTORATULUI COLAR al JUDEULUI IAI

IAI, 2004

C
atre cititori
dup
a cinci ani de aparitie a revistei
Fugit irreparabile tempus
Revista "Recreatii matematice", cu doua aparitii pe an (exceptnd primul an, 1999,
n care a fost publicat un singur numar), a intrat n al saselea an al existentei sale.
Numele ei deriv
a din cel al revistei "Recreatii stiintifice", care a ap
arut la Iasi n
perioada 1883-1888 si care este prima publicatie din tara noastr
a destinat
a tineretului studios. Revista "Recreatii stiintifice" a publicat materiale din toate domeniile
stiintei, dar cu precadere articole, note si probleme de matematica. Este o cutezanta
faptul de a ne lega numele de aceast
a veche si prestigioas
a revist
a si este de o mare
r
aspundere ncercarea noastr
a de a o continua prin asumarea obiectivelor acesteia
(v. RecMat - 1/2003, pp. 1-5), ramase actuale si acum, la mai mult de o suta de ani.
Membrii fondatori ai revistei "Recreatii Matematice" sunt cei prezenti n redactia
primului num
ar: Temistocle Brsan, Catalin Calistru, Alexandru Carausu, Constantin Cocea, Adrian Corduneanu si Gheorghe Iurea. Redactia revistei s-a modificat si s-a largit cu timpul; actuala redactie (prezenta pe coperta, interior) este formata din profesori, nvatatori si elevi, care fac din entuziasm si cu pasiune o munca
necompensat
a de vreo r
asplat
a material
a. Nu-l vom uita pe Alin Spuma, exemplu
de abnegatie, pasiune si competenta.
n acesti primi ani de existenta, eforturile redactiei s-au dirijat spre cristalizarea
unei identitati a revistei si dobndirea unui impact favorabil al acesteia cu publicul
interesat de matematica elementar
a. ntr-adev
ar, revista are n prezent o form
a
grafic
a de prezentare definitivat
a si un continut structurat pe un num
ar de rubrici
bine conturat. Pe de alta parte, punctele de distributie a revistei sunt raspndite pe
o arie ntinsa, iar colaboratorii cu note si probleme originale provin din toata tara.
Aceste preocup
ari sunt n leg
atur
a strns
a cu atingerea obiectivelor revistei, care
constituie n fapt ratiunea aparitiei sale.
Revista se adreseaza elevilor de la cei mici, din clasele primare, pna la absolventii liceelor studentilor preocupati de viitoarea lor munca la catedra, profesorilor
si tuturor celor ce ndr
agesc matematica elementar
a.
Prima parte a fiec
arui num
ar ce ocup
a aproape jum
atate din spatiul ei este
destinata articolelor de informare, studiilor si notelor originale, chestiunilor metodice
si din istoria matematicii. A fost publicat n acesti cinci ani un numar de aproximativ
o sut
a de articole de acest fel, care a oferit cititorilor un material variat, pentru toate
nivelurile de preg
atire, n bun
a parte accesibil si elevilor. Credem c
a existenta acestui
spatiu larg de publicare a stimulat posibilitatile creative ale cititorilor nostri.
n scopul cultivarii gustului pentru matematica, au fost publicate un numar de
portrete de matematicieni ilustri (Fermat, Abel, Kolmogorov etc.) sau au fost
prezentate unele probleme celebre (postulatul V al lui Euclid, marea teorema a lui
Fermat, problema celor patru culori, trisectia unghiului etc.). Au fost evocate personalitati remarcabile ale matematicii romnesti (Spiru Haret, Gh. Vr
anceanu etc.),
reviste cu un aport major n cultura stiintific
a a tarii (Recreatii stiintifice, Revista stiintifica "V. Adamachi"), institutii cu un rol important n nv
atamntul si cercetarea
romneasca (Seminarul matematic "Al. Myller", Observatorul astronomic din Iasi ),
85

momente de vrf ale matematicii din tara noastr


a (Al V-leaCongres international al
matematicienilor romni ), ct si multe figuri de matematicieni ieseni, disparuti sau
n viata (I. Creang
a, Gh. Gheorghiev, R. Miron, C. Corduneanu etc) si figuri
de prestigiu ale nv
atamntului liceal (N. Colibaba).
Mention
am n mod special rubrica "Nota elevului", care a devenit permanent
a
ncepnd cu nr. 1/2001 al revistei si care a gazduit deja 11 articole. Elevii cei mai
buni au n aceasta rubrica un spatiu destinat ncercarilor lor pe un anume subiect sau
pe o idee propice. La recomandarea redactiei revistei, Fundatia culturala "Poiana"
(director d-l Dan Tiba) ofer
a anual premii n bani tinerilor autori ai celor mai bune
astfel de note. Pna n prezent au fost premiati un numar de cinci elevi.
Partea a doua cea mai dinamica parte a revistei este destinata concursurilor,
problemelor propuse si solutiilor acestora si se ncheie cu o list
a a rezolvitorilor.
Sunt publicate cu regularitate subiectele date la urm
atoarele concursuri initiate si
organizate de ieseni: Concursul "Al. Myller" (concurs national, cl. VII-XII), Concursul "F. T. Cmpan" (concurs interjudetean, cl. IV-VIII), Concursul "Recreatii
matematice" (concurs n cadrul Taberei nationale de matematic
a, cl. VII-XI), Concursul "A. Haimovici" (concurs interjudetean pentru liceele economice, industriale
si agricole, cl. IX-XII). Tot cu scopul informarii elevilor competitivi, sunt prezentate
si concursuri organizate n alte centre: Concursul "R. Miron" (Vaslui), Concursul
"Unirea" (Focsani), Concursul "O. Onicescu" (Botosani) toate fiind interjudetene.
Pentru elevii din gimnaziu talentati au fost publicate enunturile si solutiile problemelor date la OBM (juniori), ct si problemele aflate n atentia juriului acesteia.
Subiectele date la examenul de admitere n cteva facult
ati din universit
atile
iesene ofer
a o orientare candidatilor care opteaz
a pentru facult
atile respective.
Desigur, n centrul demersului nostru se afl
a rubricile "Probleme propuse" si
"Solutiile problemelor propuse" prin care se urmareste atragerea elevilor pentru
studiul matematicii si o bun
a preg
atire a celor care reusesc s
a devin
a rezolvitori
constanti si pasionati. n acesti cinci ani de aparitie, s-au publicat 73 probleme pentru clasele primare si cte 50 probleme pentru fiecare clas
a de gimnaziu sau de liceu.
Sunt rezolvate toate problemele propuse dupa un an de la publicarea lor, se dau mai
multe solutii (atunci cnd acestea apar) si se mentioneaz
a autorii acestora.
Rubrica "Probleme pentru pregatirea concursurilor", deschis
a n nr. 2/2001 al
revistei si mp
artit
a n dou
a nivele (gimnazial si liceal), pune la dispozitia elevilor cu
nclinatii speciale probleme cu un grad de dificultate sporit.
Revista se ncheie cu "Pagina rezolvitorilor", o list
a a elevilor rezolvitori de probleme menit
a s
a-i ncurajeze si s
a-i ambitioneze. Orice elev mentionat de trei ori n
aceast
a rubric
a primeste din partea redactiei o diplom
a si un premiu n c
arti.
Printre note si probleme au fost presarate si un numar de "Recreatii . . . matematice" pentru a dovedi c
a matematica poate lumina fetele celor pasionati nu numai
cu idei inspirate, ci si cu zmbete.
Dup
a acesti cinci ani de aparitie, desprindem o concluzie: s
a continu
am cu aceeasi
pasiune pentru a ndrepta ceea ce am gresit, a completa ceea ce am facut multumitor
si a tinde spre perfectiune n acele directii n care am reusit s
a facem ceva bun.
Redactia revistei "Recreatii matematice"
86

Alexandru Myller, ctitorul scolii matematice iesene


(3 decembrie 1879 - 4 iulie 1965)
La 3 decembrie a. c. se mplinesc cinci
sferturi de veac de la nasterea lui Alexandru Myller, savant de reputatie internationala si eminent profesor al Universitatii din Iasi.
S-a nascut n Bucuresti, unde a urmat
scoala primar
a, liceul (1896) si Facultatea
de S
tiinte (1900), avnd ca profesori pe
ilustrii matematicieni romni S. Haret, E.
Pangrati, N. Coculescu si D. Emmanuel.
Dup
a un scurt stagiu ca profesor la
Liceul "V. Alecsandri" din Galati, pleac
a,
n 1902, la studii la Gttingen, unde a avut
ca profesori pe celebrii matematicieni Felix
Klein si David Hilbert. Prelund creator
noua teorie a lui Hilbert asupra ecuatiilor
integrale, Myller publica un ciclu de lucrari
printre care si teza de doctorat (1906) elaborat
a sub ndrumarea lui Hilbert.
Lucr
arile lui Myller n domeniul analizei
matematice, elaborate la Gttingen si continuate la Bucuresti, au marcat, prin ideile,
metodele si rezultatele obtinute, un moment important n dezvoltarea si afirmarea
matematicii romnesti pe plan national si international. Acestea, precum si titlul de
elev al lui Hilbert, l-au consacrat ca matematician de prima marime. La Iasi, Myller
si schimba directia cercetarilor matematice, trecnd la geometrie, domeniu care l-a
fascinat nc
a din tinerete.
A obtinut numeroase rezultate n domeniul teoriei ecuatiilor diferentiale si
integrale prin: extinderea unor rezultate ale lui Hilbert la cazul unor ecuatii diferentiale de ordin arbitrar; ecuatii integrale cu nucleu antisimetric; probleme bilocale, la
limita si de periodicitate pentru ecuatii diferentiale ordinare si cu derivate partiale;
utilizarea metodelor functionale n rezolvarea unor probleme de fizica matematica.
n domeniul geometriei diferentiale contributiile sale se refera la: geometrie
algebrica si geometrie riglata; definirea notiunii de concurenta a vectorilor contravarianti ca o generalizare a paralelismului Levi Civita; paralelismul ntr-un sistem
de plane, care a condus la notiunea de configuratie Myller; dezvoltarea, mpreuna cu
O. Mayer a geometriei diferentiale centroafine. Aceste cercetari au fost declansate si
impulsionate de aparitia teoriei relativitati generale, unde se folosea n mod consistent teoria conexiunilor afine si alte notiuni geometrice legate de conexiuni. Aceste
preocupari au marcat momentul intrarii scolii de matematica de la Iasi pe arena
mondiala a cercetarii stiintifice.
87

n domeniul istoriei matematicii a reusit s


a repun
a n valoare contributiile
originale ale unor precursori precum D. Asachi, St.
Botez,
E. Bacaloglu etc.

Numit n 1910 profesor titular la catedra de geometrie analitica a Universitatii din


Iasi, Alexandru Myller pune bazele nvatamntului matematic modern la aceasta
institutie de nvatamnt prin: fondarea vestitei biblioteci de specialitate (18. X. 1910)
ca fundament al cercetarilor originale; ncadrarea unui corp profesoral de mare valoare; atragerea unor tineri cu care creeaza prima scoala romneasca de matematica,
cunoscuta sub numele de Seminarul Matematic din Ia
si; initierea n premiera
la Iasi a unor studii de istoria matematicii romnesti si universale; introducerea
unor cursuri libere de specialitate si a lucrarilor de licenta; aporturi originale n
geometria diferentiala, care au lansat scoala ieseana n competitie internationala.
Aceste evenimente s-au petrecut, prima oar
a n 1922 cnd au fost publicate lucr
arile
legate de paralelismul lui Levi Civita, a doua oar
a, n 1933, cnd a ap
arut memoriul
de geometrie diferentiala centroafina elaborat n colaborare cu O. Mayer.
Ca profesor, Al. Myller a fost un maestru n arta comunicarii cu studentii. A
aplicat, pentru prima oar
a n tara noastr
a, metoda euristic
a n predarea matematicii
la nivel universitar. Lectiile sale erau adev
arate momente de creatie n care profesorul
ghida pe studenti sa descopere, mpreuna, adevarurile stiintei predate. A mpartasit
cu dragoste si generozitate fiec
arei generatii de studenti tot ce a acumulat din punct
de vedere stiintific si metodic.
Sub ndrumarea sa, a fost obtinut n 1920 primul doctorat n matematici pure, la o
universitate romneasca, de catre Octav Mayer, iar n 1925 a fost obtinut doctoratul
n matematici de c
atre Silvia Creanga, care devine prima femeie doctor n matematici
la o universitate romneasc
a.
Dup
a r
azboi, a functionat ca rector n dificila perioad
a 1944-1945, reusind s
a
deschida cursurile universitare n mai 1945, dupa refugiul n Transilvania. A fost
preocupat de reconstructia cl
adirii, de revenirea la Iasi a laboratoarelor si bibliotecilor, precum si a profesorilor si studentilor.
n 1947 iese la pensie, dar r
amne legat de activitatea Seminarului Matematic,
unde continua cercetarile stiintifice, se preocupa de instruirea tinerilor doctoranzi
si de bunul mers al bibliotecii urm
arind cu asiduitate obtinerea unor publicatii mai
greu de procurat.
A mai desf
asurat variate activit
ati si la Institutul de Matematic
a de la Filiala Iasi
a Academiei Romne. A continuat sa lucreze pna la stingerea sa din viata survenita
la 4 iulie 1965.
Pentru meritele sale de exceptie, Academia Romn
a l-a ales membru titular n 1938. n 1959 Universitatea Humbold din Berlin i-a decernat titlul
de Doctor Honoris Causa pentru straduinti deosebite de a creea o matematica
romneasca de sine statatoare. A primit numeroase distinctii si decoratii din partea
statului romn.
La mplinirea a 125 ani de la nasterea savantului Alexandru Myller, elevii si
elevii elevilor sai l omagiaza n semn de adnca pretuire pentru opera sa nchinata
dezvolt
arii stiintei si nv
atamntului modern n tara noastr
a.
Prof. dr. Gheorghe BANTA
S
Prof. dr. Vasile OPROIU
88

Henri Poincar
la 150 de ani de la nasterea sa
La pense nest quun clair,
au milieu dune longue nuit,
mais cest cet clair qui est tout.
H. P.
Este anevoie pentru un om obisnuit
sa compare colosii; adncimi nebanuite tulbur
a vederea detaliilor si ratiunea
nu reuseste s
a completeze o imagine
de nteles. Exista consensul ca Henri
Poincar a fost ultimul matematician
universal si s-a dovedit genial n tot ce
a ntreprins. Acum, la 150 de ani de la
nasterea sa, zabovim un pic gndind la
el.
Henri Poincar s-a n
ascut la
Nancy, pe 29 aprilie 1854, ntr-o distins
a
familie de intelectuali. Ambidextru si
miop, a suferit de o slaba coordonare
muscular
a. Din scoala elementar
a a excelat n compozitii scrise. n 1862 este
nscris la Liceul din Nancy si timp de
11 ani s-a dovedit aici un elev stralucit la toate materiile. Profesorul s
au de
matematic
a l considera un monstru al
matematicii. La concursurile generale ale liceelor din Franta a cstigat premiile nti. n 1873 este admis la cole Polytechnique pe care o absolva n 1875, depasindusi clar colegii n domeniile matematicii. Citea mult si variat, realiznd conexiuni
neasteptate si dovedind o memorie vizual
a excelent
a. si completeaz
a cu mult interes
studiile la cole des Mines si profeseaza ca inginer n timp ce si elaboreaza lucrarea
de doctorat, sub conducerea stiintifica a lui Charles Hermite, n domeniul ecuatiilor
diferentiale. Devine doctor n matematic
a n 1879, convingnd dar si uimind comisia.
Pred
a analiz
a matematic
a 2 ani la Caen, din 1881 primeste catedr
a la Facultatea de S
tiinte din Paris, din 1886 trece la Sorbona pe Catedra de ecuatiile fizicii
matematice si probabilitati, preda si la cole Polytechnique. Lectiile sale acopera
domenii variate, mereu n schimbare, dar nu sunt usor de urm
arit de c
atre studenti
din cauza abundentei de idei. Este ales n Academia de Stiin
te n 1887; caz unic,
este ales n fiecare din cele cinci sectii ; n 1906 devine secretar permanent al acesteia.
n 1908 este ales n Academia Franceza al carei secretar permanent devine n anul
prematurului s
au deces, la 17 iulie 1912.
Activitatea sa de creatie stiintific
a se desf
asura cu precizie: ntre 10 si 12 dimineata,
ntre 5 si 7 dupa amiaza. Dupa ora 7 se informa. Pornea cte o lucrare fara un
89

plan prestabilit, f
ar
a calcule preliminare; dup
a primii pasi urmau natural urm
atorii.
Aborda si abandona subiecte din unghiuri de vedere diferite dar subconstientul sau
continua studiile, ntregind imaginea. Dificultatea de a opri cercetarea l determina
s
a nu ntreprind
a lucr
ari importante dup
a ora 7 seara, spre a nu-si tulbura somnul.
A dezvoltat studiul functiilor automorfe dup
a o idee ce sustine c
a i-a venit f
ar
a
nici o pregatire prealabila n momentul cnd urca ntr-un autobuz; folosea n context
complet diferit transformarile din geometria neeuclidiana. Prin lucrarea Analysis
situs, publicat
a n 1895, ntemeiaza domeniul topologiei algebrice n care clasica sa
conjectur
a este nc
a actual
a. Este considerat fondatorul teoriei functiilor analitice
de mai multe variabile complexe. A adus contributii esentiale n geometria algebrica, dezvoltnd metode ce au permis deduceri directe ale unor rezultate profunde
ce se bazau pe o idee de demonstratie gresit
a. n 1901 a rezolvat o problem
a crucial
a de teoria numerelor: c
autarea punctelor de coordonate rationale pe o curb
a
algebrica f (x, y, z) = 0 cu coeficienti rationali. Geometria hiperbolica, creata de
catre Lobacevski si Bolyai, a devenit de nteles pe modelul de univers imaginat de
catre Poincar. Este considerat al
aturi de Einstein (si Lorentz ) fondator al teoriei
relativitatii.
Retinem atentia cu cteva detalii referitoare la problema celor 3 corpuri. Se
presupun date la un moment initial 3 corpuri date prin trei puncte de mase invariabile
prin pozitia lor, viteze si acceleratii. Se presupune c
a asupra lor nu intervine nici
o forta exterioar
a, dar c
a ele evolueaz
a respectnd legea atractiei universale. Se
cere sa se evalueze comportarea lor n timp. Problema abordata de catre Poincar
n 1889 era ct pe ce s
a fie compromis
a de o eroare comis
a de editorul de la Acta
Mathematica. Un intens schimb de scrisori cu Mittag-Leler a l
amurit chestiunea
si un memoriu al lui Poincar a ap
arut n 1890, fiind considerat act de nastere a
teoriei haosului. Motivul este ca aceasta problema a celor 3 corpuri revine la un
sistem de ecuatii diferentiale (cu necunoscutele cele 9 functii ce dau coordonatele
punctelor) care nu este stabil. Aceasta nseamn
a c
a modific
ari infinitezimale ale
datelor problemei conduc la modific
ari substantiale ale traiectoriilor.
H. Poincar a adus contributii esentiale n numeroase domenii ale matematicilor aplicate: mecanica cereasca, cosmologie, mecanica cuantica, optica, electricitate, hidrodinamica, telegrafie, termodinamica, teoria elasticitatii, electromagnetism,
capilaritate.
n special, modul diferit de a concepe matematica la Poincar si Hilbert a condus
la filosofii distincte ale matematicii. n opozitie cu punctul de vedere logicist si
formalist, H. Poincar a sustinut un punct de vedere intuitionist: prin logica
demonstram, dar prin intuitie creem, logica ramne sterila fara a fi fertilizata de
intuitie.
H. Poincar este un maestru al populariz
arii stiintei; c
artile sale: Stiin
ta si
ipoteza (1902), Valoarea stiintei (1905), Stiin
ta si metoda (1908), Ultimele gnduri
(1908, postum) sunt disponibile si n limba romn
a.
La funeraliile sale s-a spus: ... a fost matematician, geometru, filosof,
scriitor, poet al infinitului, bard al stiintelor.
Prof. dr. Dan BRNZEI
90

Trecerea planetei Venus prin fata Soarelui


Planeta Venus, a doua planeta dupa Mercur ca departare de Soare, cunoscuta si
sub numele de Luceafarul de dimineata sau de seara, trece din cnd n cnd prin fata
Soarelui (se spune c
a n acel moment planeta se afl
a n conjunctie inferioara ). Acest
"din cnd n cnd" reprezint
a un interval de timp de cel putin 100 de ani. Pe data
de 8 iunie a acestui an fenomenul se va desfasura din nou, iar noi contemporanii
fenomenului vom avea prilejul sa-l vedem, daca cerul va fi favorabil observatiilor.
Trecerea lui Venus zeita frumusetii si dragostei la romani, numit
a la greci
Afrodita, ca fiica lui Zeus si Dionei a prezentat o importanta deosebit
a n istoria
astronomiei si de ce nu si n istoria civilizatiei. Omul, aceasta creatie extraordinara a
legilor lumii materiale, prin existenta si natura sa, s-a ntrebat de multa vreme cum a
ap
arut; s-a n
ascut aici pe P
amnt, a venit de undeva, a fost creat de cineva, care este
locul s
au n lumea Universului observabil? Iat
a ntreb
ari, devenite fascinante timp
de secole, la care astronomia, desi a raspuns doar partial, totusi a dat raspunsuri mai
mult sau mai putin convingatoare.
Pozitia P
amntului n Univers poate fi determinat
a dac
a se cunoaste distanta
Pamnt - Soare. Aceast
a distanta, m
asurat
a prin procedee astronomice, este de
149 597 870 km. Ei bine, aceasta distanta a fost ameliorata treptat prin observarea
timp de cteva secole si atunci cnd Venus trecea prin fata discului solar. Acum
exist
a metode ce permit determinarea distantei P
amnt - Soare cu precizie de cteva
zeci de kilometri, ns
a pentru acele vremuri metoda trecerii a reprezentat un succes
remarcabil de la trecere la trecere.
Nici acum observarea fenomenului nu este lipsita de interes daca observatiile se
fac cu suficient
a precizie (precizia n timp n determinarea fenomenului, trebuie s
a
fie de cel putin o sutime de secund
a; important este s
a se cunoasc
a la fel de precis
pozitia locului din care se fac observatiile).
Trecerea planetei Venus prin fata Soarelui are loc o data la 115 ani dupa care
urmeaza o alta trecere dupa 8 ani. Urmatoarea trecere urmnd sa aiba loc dupa 122
de ani, urmata de o alta dupa nca 8 ani. Asa se succed aceste treceri. Dac
a planul
traiectoriei lui Venus ar fi acelasi cu planul orbitei terestre, atunci trecerea lui Venus
ar avea loc dupa fiecare 584 de zile si 22 de ore. Acest interval de timp reprezinta
perioada sinodica a lui Venus (intervalul dintre dou
a pozitii asem
an
atoare ale planetei
n raport cu Soarele si P
amntul).
ntr-o viata de om, trairea acestui eveniment nu poate fi dect de doua ori, o
singura data sau deloc, cu exceptia celor ce au o longevitate mai mare de 115 ani
sau 122 de ani. Penultima trecere vizibila la noi a avut loc n anul 1874, fenomen
urm
arit de profesorul Neculai Culianu, fost decan al Facult
atii de S
tiinte si apoi
rector ntre anii 1888 - 1898 al prestigioasei universitati iesene. Ultima traversare a
putut fi observata dupa 8 ani (1882).
Fenomenul se ncadreaza n categoria eclipselor de Soare cnd Luna, Mercur sau
Venus si Pamntul sunt n linie dreapta. Spre deosebire de eclipsele de Soare, cnd
Luna interpunndu-se ntre Pamnt si Soare poate sa-l acopere complet, trecerea lui
91

Venus nu are aceast


a calitate. Venus fiind mult dep
artat
a de P
amnt (n acel moment
departarea va fi de 43 228 162 km) si mica n raport cu Soarele, va lasa o umbra ca
un mic disc ntunecat reprezentnd a 33-a parte din diametrul Soarelui.
Intrarea peste discul solar (primul contact exterior) va avea loc la ora 8 h 19 m
47 s ca dup
a 19 m si 27 s Venus s
a intre complet peste discul solar la ora 8 h 39 m
14 s (primul contact interior). Urmatoarele etape vor fi: ultimul contact interior la
ora 14 h 03 m 39 s, iar ultimul contact exterior va avea loc la ora 14 h 22 m 49 s.
Momentele caracteristice mentionate mai sus au fost calculate de c
atre Institutul
de Mecanica Cereasca si Calculul Efemeridelor din Franta, pentru Bucuresti. Pe
teritoriul tarii noastre aceste momente vor diferi de la localitate la localitate, cu nu
mai mult de cteva secunde.
NASA a f
acut calcule si pentru Iasi si iat
a momentele caracteristice: 8 h 19 m
34 s primul contact exterior, 8 h 39 m 11 s primul contact interior, 14 h 03 m
15 s ultimul contact interior si 14 h 22 m 37 s ultimul contact exterior.
nceputul intrarii lui Venus va avea loc prin partea stnga (partea estica) din
vecin
atatea marginii inferioare a discului solar asa cum se poate vedea n desenul al
aturat. Tot n desen se arat
a traiectoria umbrei si momentele caracteristice mentionate
mai sus. Durata fenomenului, de la primul contact interior la ultimul contact interior
92

va fi de 5 h 24 m 25 s.
Pentru observarea fenomenului trebuiesc luate masuri de protectie a ochilor. Recomandam ca atunci cnd se ndreapta privirea spre Soare sa se foloseasca o sticla
afumat
a sau o sticl
a ce se utilizeaz
a la ochelarii pentru sudur
a electric
a. Cel mai la
ndemn
a ar fi folosirea foliei metalizate utilizat
a drept ambalaj de c
atre vnz
atoarele
de flori. Aceasta ultima protectie trebuie confectionata din cel putin doua sau trei
folii suprapuse, functie de sensibilitatea ochilor celor ce vor sa observe fenomenul.
Observatii se pot face si cu o lunet
a dotat
a cu un ocular prin care s
a se proiecteze
Soarele pe un ecran situat la o distanta convenabil
a. Imaginea Soarelui ar putea
ajunge la dimensiunile unui cerc cu raza 10 - 15 cm si chiar mai mult. Evident, luneta
trebuie instalata pe un trepied pentru a-i asigura stablitate si urmarire comoda a
Soarelui. O urm
arire asem
an
atoare a fenomenului poate fi f
acut
a proiectnd imaginea
Soarelui ca mai sus, folosind un binoclu.
Amatorii care vor sa aduca contributii la observarea trecerii, avnd n vedere ca
unele observatoare specializate pot sa nu aiba vreme favorabila observatiilor, urmeaza
s
a tin
a seama de urm
atoarele: 1) s
a posede un ceas cu cronometru si pus la or
a dup
a
un post de radio national, 2) s
a urm
areasc
a momentul primului contact exterior si
sa-l cronometreze si la fel sa procedeze pentru celelalte trei momente caracteristice
(primul contact interior, al doilea contact interior si al doilea contact exterior). n
plus fata de aceste recomand
ari, trebuie s
a apeleze la o cunostinta ce posed
a un
aparat numit GPS cu ajutorul c
aruia s
a se determine coordonatele locului unde a
facut observatiile (longitudinea si latitudinea geografica). Daca masuratorile vor fi
f
acute cu o eroare n timp de plus sau minus 0,1 secunde, iar coordonatele locului vor
fi determinate cu o eroare de maximum 8 metri n jurul punctului unde s-au f
acut
m
asur
atorile, atunci observatiile pot fi luate n consideratie.
Observatiile pot fi transmise n scris la Observatorul Astronomic din Iasi, Aleea
Sadoveanu nr. 5, textul urmnd s
a contin
a: mijloacele cu care s-au f
acut observatiile,
momentele caracteristice m
asurate si coordonatele locului.
Operatia, pe plan mondial, este coordonat
a de c
atre Observatorul European de
Sud cu sediul n Garching (Germania) iar calculele centralizate vor fi facute la Institutul de Mecanica Cereasca si Calculul Efemeridelor cu sediul la Paris.
Urm
arirea fenomenului nu este lipsit
a de interes dac
a avem n vedere raritatea
lui, importanta stiintific
a, ncrederea si seriozitatea ce se poate acorda calculelor
astronomice, spre deosebire de ideile avansate de prezicatori si astrologi, care dau
interpret
ari apocaliptice unor astfel de fenomene. Din anul 3000 . H. si pn
a n
prezent, au avut loc 64 de treceri si alte evenimente astronomice deosebite, dar nu s-a
constatat, n afara unor fenomene naturale izolate, uneori devastatoare (cutremure,
inundatii, razboaie provocate de om etc.), sa se fi produs acel eveniment apocaliptic
mult proferat de prezic
atori.

Iulian BREAHNA
Membru al Uniunii Astronomice Internationale,
ex-director al Observatorului Astronomic din Iasi

93

Cteva noi aplicatii ale unei idei consacrate


Gabriel DOSPINESCU 1
Nu credem c
a exager
am spunnd c
a 99% din inegalit
atile care sunt propuse ca
probleme de concurs pot fi demonstrate direct, aplicnd alte inegalit
ati deja consacrate. Exist
a ns
a o categorie aparte de probleme foarte greu de rezolvat n acest
mod. Aceste probleme au solutii frumoase, bazate pe rationamente indirecte, dar
extrem de eficiente.
Vom prezenta n continuare ideea comuna aflata n spatele tuturor acestor solutii.
S
a presupunem c
a avem de demonstrat o inegalitate de forma g (x1 ) + g (x2 ) +
+ + g (xn ) 1 n ipoteza c
a variabilele x1 , x2 , . . . , xn verific
a o relatie de tipul
f (x1 ) + f (x2 ) + + f (xn ) = 1.
Presupunem prin reducere la absurd ca g (x1 ) + g (x2 ) + + g (xn ) < 1. Notnd
1
a S =
S = g (x1 ) + g (x2 ) + + g (xn ), obtinem c
pentru un anumit k > 1.
k
Punem kg(xi ) = ai , i = 1, n, rezolvam ecuatiile n xi si introducem aceste valori
asim c
a variabilele ai ,
ca functii de ai n relatia f (x1 ) + f (x2 ) + + f (xn ) = 1. G
i = 1, n satisfac de asemenea o relatie de forma h(x1 ) + h(x2 ) + + h(xn ) = 0
precum si a1 + a2 + + an = 1. Problema se reduce acum la a dovedi ca egalitatea
h(x1 ) + h(x2 ) + + h(xn ) = 0 este imposibila, lucru care se realizeaza deseori
demonstrnd c
a h(x1 ) + h(x2 ) + + h(xn ) > 0 (sau < 0) pentru orice numere a1 ,
a2 , . . . , an cu sum
a 1.
Credem ca noua problema este adesea mai usoara dect cea initiala si pentru a
exemplifica acest lucru vom rezolva cteva probleme de concurs utiliznd strategia
de mai sus.
Prima dintre ele, Problema 3, OM China 2003, este faimoasa datorita dificultatii
sale, fiind de asemenea nrudit
a cu o alt
a binecunoscut
a problem
a care va fi discutat
a
n cele ce urmeaz
a, Problema 2, OIM 2001.
Exemplul 1. Fie x1 , x2 , . . . , xn (0, /2) astfel ca tg x1 tg x2 . . . tg xn = 2n/2 .
Determinati cel mai mic kn pentru care inegalitatea cos x1 +cos x2 + +cos xn kn
este ntotdeauna adevarata.
Solutie. Substituind tg2 xi = 2ai , problema revine la a determina supremumul
1
1
1
+
+ +
pentru a1 , a2 , . . . , an > 0 si cu
expresiei
1 + 2a1
1 + 2a2
1 + 2an
produsul egal cu 1.
Nu vom discuta cazul n = 1, acesta
fiind trivial. Pentru n = 2, ramne de gasit
1
x
+
, x > 0. Studiind ceea ce se ntmpla pentru
valoarea maxima a lui
1 + 2x
x+2
2
x = 1, x si x 0, deducem c
a valoarea c
autat
a este . Pentru a dovedi acest
3

1
x
lucru, este suficient s
a se studieze monotonia functiei f (x) =
+
,
1 + 2x
x+2
1

Student, Facultatea de Matematic


a si Informatic
a, Bucuresti

94

x > 0, cu ajutorul derivatei.


Sa studiem ce se ntmpla pentru n > 2. Daca toti ai , i = 1, n, sunt egali cu 1,
n
valoarea expresiei de mai sus este . Acum, dac
a ncerc
am s
a "apropiem" ct mai
3
multi ai de 0, observam ca putem "apropia" de 0 cel mult n 1 dintre ei, caz n care
n
valoarea expresiei va tinde la n 1. Deoarece n 1 > , este clar c
a va trebui s
a
3
1
1
1
dovedim ca
+
+ +
n 1.
1 + 2a1
1 + 2a2
1 + 2an
Este usor de v
azut c
a rationnd ca n problema precedent
a ajungem repede ntrun impas. Observ
am ns
a c
a este suficient s
a demonstr
am inegalitatea de mai sus
doar pentru n = 3. De ce acest lucru? Daca n > 3 si a1 a2 an = 1, atunci putem
alege ai , aj , ak al caror produs este cel putin 1. Atunci
1
1
1
1
1
1

+
+ +
< n3+
+p
+
.
1 + 2a1
1 + 2a2
1 + 2an
1 + 2ai
1 + 2ak
1 + 2aj

Dar

1
1
1
1
1
1

+p
+

+p
+q
2,
1 + 2ai
1 + 2ak
1 + 2ai
1 + 2aj
1 + 2aj
1 + 2 ai1aj

presupunnd c
a inegalitatea este adev
arat
a pentru n = 3, iar din relatia precedent
a
rezult
a c
a ea este adev
arat
a si pentru n oarecare.
Sa dovedim deci inegalitatea pentru n = 3. Presupunem ca ea nu este adevarata
1
1
1
2
+
+
= , cu p < 1.
pentru o tripleta a1 , a2 , a3 . Atunci
p
1 + 2a1 1 + 2a2 1 + 2a
3
3
3
Q
Q
p
1
1
p2
1
, obtinem c
a1=

<
. Este
Notnd xi =
2
2
2
2
2 1 + 2ai
k=1 8xk
k=1 8xk
1
deci suficient sa demonstram ca pentru orice 0 < x, y, z < cu x + y + z = 1 avem
2

1
1
1
1
1
1

1, inegalitate echivalenta cu
ca
8x2
2
8y 2 2
8z 2 2
(1 2x)(1 2y)(1 2z)(1 + 2x)(1 + 2y)(1 + 2z) 83 x2 y 2 z 2 .

O alta schimbare de variabila este acum necesara. Desigur, aceasta este 12x = a,
1 2y = b, 1 2z = c. Ramne deci sa demonstram ca, pentru orice a, b, c > 0 cu
suma 1, avem
8(1 a)2 (1 b)2 (1 c)2 abc(2 a)(2 b)(2 c).
Avem ns
a c
a

2
8
8(1 a)2 (1 b)2 (1 c)2 = 8(b + c)2 (c + a)2 (a + b)2 8 (a + b + c)(ab + bc + ca) =
9
512
512
512
=
(ab + bc + ca)2
abc (a + b + c) =
abc.
81
27
27
512
, ceea ce este trivial,
Este suficient, deci, sa demonstram ca (2 a)(2 b)(2 c) <
27
deoarece (2 a)(2 b)(2 c) < 8, iar problema este acum rezolvata.
95

A fost mentionat
a anterior, n trecere, frumoasa Problema 2, OIM 2001, propus
a
de Hojoo Lee. Problema n cauza fiind att de mult discutata si popularizata,
s-ar putea crede ca nu mai este nimic nou de spus despre ea. Totusi, credem ca
urm
atoarea demonstratie a unei generaliz
ari a problemei este nou
a.
Exemplul 2. Demonstrati ca daca a1 , a2 , . . . , an sunt numere reale strict pozitive astfel nct a1 a2 an = 1, iar k > 1 este un numar natural, atunci
1
1
1
p
+ p
+ + p
1.
k
k
k
k
k
1 + (n 1) a1
1 + (n 1) a2
1 + (nk 1) an

Vasile Crtoaje
Solutie. Presupunem c
a
1
1
1
1
p
+ p
+ + p
= ,
k
k
k
k
k
k
p
1 + (n 1) a1
1 + (n 1) a2
1 + (n 1) an
p
pentru un anumit p > 1. Notam p
= xi , i = 1, n si obtinem ca
k
1 + (nk 1) ai

n
n
k
n
Q
Q
pk
1
n 1 =

1
>
1 , iar x1 + x2 + + xn = 1.
k
k
i=1 xi
i=1 xi

n
Q
1
Pentru a ajunge la o contradictie, vom dovedi ca

1
nk 1
k
i=1 xi
pentru orice x1 , x2 , . . . , xn > 0 astfel nct x1 +x2 + +xn = 1. Pentru demonstrarea
acestei inegalitati, sa observam ca
n
n
Q
Q

(x1 + + xi1 + xi+1 + + xn )


1 + xi + + xk1
n
i
Y
1
i=1
i=1

1
=
(1)
xki
(x1 x2 . . . xn )k
i=1

si, din inegalitatea mediilor,


n
Y
(x1 + x2 + + xi1 + xi+1 + + xn ) (n 1)n x1 x2 . . . xn .

(2)

Sumnd inegalitatile de mai sus, obtinem


n
Y


n
1 + xj + + xk1
1 + G + G2 + + Gk1 ,
j

(3)

i=1

Desigur, pentru minorarea celuilalt produs din formula (1), aplicarea direct
a a inegalit
atii mediilor nu mai este la fel de eficient
a. Conform inegalit
atii mediilor, avem
q
n
n
X
n (x1 x2 . . . xn )i
xij
qQ

, i = 0, k 1.
n
k1
n
1 + xj + + xk1
j
j=1
j=1 1 + xj + + xj

j=1

unde G =

n
x1 x2 . . . xn . Din (1), (2) si (3) deducem c
a
n

n
Y
(n 1)n Gn 1 + G + G2 + + Gk1
1
1
,
Gnk
xki
i=1
96

iar pentru a finaliza solutia problemei este suficient s


a demonstr
am c
a
1
1 + G + G2 + + Gk1
1 + n + + nk1 , ceea ce este trivial, deoarece G .
Gk1
n
Este acum momentul s
a discut
am o alt
a problem
a deosebit
a, propus
a la Concursul
anual al Gazetei Matematice de catre Vasile Crtoaje. Solutia autorului se bazeaza
pe aplicarea succesiva a ctorva identitati, fiind aproape imposibil de gasit n mod
independent. Credem c
a solutia ce urmeaz
a este mai natural
a din acest punct de
vedere.
Exemplul 3. Demonstrati ca pentru orice a, b, c, d > 0 astfel ca a2+b2+c2+d2 = 1
este satisfacuta inegalitatea (1 a)(1 b)(1 c)(1 d) abcd.
(1 a) (1 b) (1 c) (1 d)
Solutie. S
a presupunem c
a
= p4 , cu p < 1. Fie
abcd
P
P
1
1b
1c
1d
1
1a
>
,
= x,
= y,
= z,
= t. Atunci 1 =
2
pa
pb
pc
pd
(1 + px)
(1 + x)2
iar xyzt = 1. Problema se reduce, deci, la a demonstra ca
1
1
1
1
+
+
+
1
(1 + x)2 (1 + y)2
(1 + z)2
(1 + t)2
pentru orice x, y, z, t > 0 verificnd xyzt = 1.
Profitnd de faptul c
a enuntul problemei utilizeaz
a 4 numere, vom separa numerele n 2 grupe si demonstra c
a
1
1
1
1
1
1
si
2 +
2 1 + xy
2 +
2 1 + zt .
(1 + x)
(1 + y)
(1 + z)
(1 + t)
Prima inegalitate se reduce la xy (x y)2 + (1 xy)2 0, care este n mod
evident adevarata, pentru a doua rationndu-se n mod analog. Prin sumarea celor
dou
a inegalit
ati si tinnd seama de faptul c
a xyzt = 1, rezult
a concluzia.
n ncheierea acestui articol vom discuta alte doua probleme deosebite, carora li se
pot da solutii rapide utiliznd ideile de mai sus. Prima dintre ele a fost publicata n
revista "American Mathematical Monthly", fiind propus
a de c
atre Vasile Crtoaje.
1
1
Exemplul 4. Fie x1 , x2 , . . . , xn > 0 astfel nct x1 + x2 + + xn =
+ +
x1 x2
1
1
1
1
. Demonstrati ca
+
+ +
1.
+ +
xn
n 1 + x1
n 1 + x2
n 1 + xn
1
1
1
1
Solutie. Presupunem ca
+
+ +
= ,
n 1 + x1
n 1 + x2
n 1 + xn
p
p
1
, i = 1, n si deducem c
a ai <
p < 1. Not
am ai =
, i = 1, n, iar
n 1 + xi
n1
a1 + a2 + + an = 1. Conditia din enunt se transcrie sub forma
X
n
n
X
p
1
n+1 =
.
p
ak

n+1
ak
k=1

k=1

Deoarece p < 1, putem scrie


X
X
n
n
n
X
1
p
n+1 >
n+1 =
ak
ak
k=1

p
k=1 ak

k=1

97

X
1
>
n+1

1
k=1 ak

1
.
n+1

R
amne, deci, s
a demonstr
a pentru
oricena1 , a2 , . . . , an > 0 cu a1 +a2 + +an = 1
am c
n
P
P
1
1
are loc inegalitatea
n+1
, care este echivalent
a cu
1
k=1 ak
k=1 ak n + 1
n
n 1 (n 1) a
P
P
ak
k

.
1

(n

1)
a
a
k
k
k=1
k=1
O alta substitutie este acum necesara, anume 1 (n 1)ak = bk , k = 1, n. Se
observ
a c
a avem de asemenea b1 + b2 + + bn = 1 si r
amne deci s
a dovedim
n 1b
n
P
bk
k
2 P
c
a
(n 1)
, rezultat care se poate obtine prin sumarea a n
bk
k=1
k=1 1 bk
P bk
bk
1
bk
=P

.
inegalit
ati de forma
2
1 bk
b
(n 1) i6=k bi
i6=k i

n final vom discuta o alta frumoasa problema de concurs, propusa la Barajul de


selectie a lotului Romniei pentru OIM, 1999, de c
atre Gheorghe Eckstein.
Exemplul 5. Demonstrati ca daca x1 , x2 , . . . , xn > 0 satisfac x1 x2 . . . xn = 1,
1
1
1
+
+ +
1.
atunci
n 1 + x1 n 1 + x2
n 1 + xn
Solutie. Ca mai sus, problema se reduce la a demonstra ca pentru
2, . . . ,
orice x1 , x
n
Q
1
1
xn <
n + 1 1.
astfel nct x1 +x2 + +xn = 1 are loc inegalitatea
n1
i=1 xi
Cu substitutia bi = 1 (n 1) xi problema de demonstrat se reduce la
n

(1 b1 ) (1 b2 ) . . . (1 bn ) (n 1) b1 b2 . . . bn .

Conform inegalitatii mediilor,


n
n X
n h
Y
Y
Y
(1 bi ) =
bj
(n 1)
i=1

i=1

j6=i

i=1

sY

n1

j6=i

i
n
bj = (n 1) b1 b2 . . . bn ,

ceea ce trebuia demonstrat.


n ncheiere, tin s
a multumesc prof. Marian Tetiva pentru lectura acestui articol
si observatiile pretioase facute, care au contribuit la mbunatatirea formei finale.

ERRATA
n articolul "Combinatorica . . . algebrica" de Gabriel Dospinescu, publicat n
nr. 2/2003 al revistei, forma corecta a Lemei de la pag. 19 este:
2
2
Fie n numar natural prim si = cos
+ i sin . Are loc egalitatea
n
n
a0 + a1 + + an1 n1 = 0, a0 , a1 , . . . , an1 Q,

daca si numai daca a0 = a1 = = an1 .


Diferentele ce apar ne-au fost semnalate de prof. Sergiu Romascu, Vaslui; acestea nu afecteaz
a cu nimic restul articolului. De aceast
a neglijenta se face vinovat
a
redactia si nu autorul articolului.
98

Cteva propriet
ati ale medianelor
Temistocle BRSAN 1
1. Ne propunem s
a indic
am un num
ar de propriet
ati ale medianelor la nivelul
de ntelegere al unui elev bun de gimnaziu. Instrumentele principale de lucru vor fi
teorema lui Thales si teorema lui Menelaus.
Pentru o exprimare scurt
a, vom folosi notiunile de puncte izotomice si puncte
conjugate armonic. Fie A, B, X, Y patru puncte coliniare. Punctele X si Y sunt
izotomice n raport cu A si B daca sunt simetrice fata de mijlocul segmentului [AB]
(evident, X si Y sunt fie interioare, fie exterioare acestui segment); aceasta conditie
revine la egalitatea AX = BY . Punctele X si Y sunt conjugate armonic n raport
XA
YA
cu A si B daca mpart segmentul [AB] n rapoarte egale:
=
(evident, X
XB
YB
este interior segmentului si Y exterior sau invers).
Propozitia 1. Fie ABC un triunghi oarecare, A0 , B 0 , C 0 mijloacele laturilor
[BC], [CA], respectiv [AB] si M, N BC doua puncte izotomice n raport cu vrfurile B si C, cu M diferit de mijlocul segmentului [A0 C]. Atunci, sunt adevarate
afirmatiile:
1 dreptele B 0 M si C 0 N se intersecteaza ntr-un punct X AA0 ;
2 paralela prin M la AC si paralela prin N la AB se intersecteaza ntr-un punct
0
X AA0 ;
3 punctele X si X 0 sunt conjugate armonic n raport cu A si A0 .
Demonstratie. Avem patru situatii distincte ilustrate n figurile de mai jos;
demonstratia este nsa aceeasi.
X
A

A
C X

C
C M

M
N B

A
B

A M

Deoarece M nu-i mijlocul lui [A0 C] rezulta ca N nu-i mijlocul lui [A0 B] si,
ca urmare, dreptele B 0 M si C 0 N intersecteaz
a AA0 . Fie {X} = AA0 B 0 M si
{X1 } = AA0 C 0 N . Conform teoremei lui Menelaus, aplicat
a la 4AA0 C si transversala B 0 M si la 4AA0 B si C 0 N , avem
X1 A N A0 C 0 B
XA M A0 B 0 C

=
1
s
i

= 1,
XA0 M C B 0 A
X1 A0 N B C 0 A
1

Prof. dr., Catedra de matematic


a, Univ. Tehnic
a "Gh. Asachi", Iasi

99

XA
MC
NB
X1 A
si
=
=
. Punctele M , N fiind izotomice n raport cu
0
0
0
XA
MA
X1 A
N A0
X1 A
XA
B si C, avem M A0 = N A0 si M C = N B. Rezult
=
, deci punctul
a c
a
XA0
X1 A0

X1 coincide cu X. Afirmatia 1 este dovedita.


Fie X 0 , X10 intersectiile dreptei AA0 cu paralela prin M la AC, respectiv paralela
MC
X 0A
si
=
prin N la AB. Conform teoremei lui Thales, au loc relatiile:
0
0
XA
M A0
0
NB
X1 A
=
. Din acestea si din izotomia punctelor M si N fata de B si C,
X10 A0
N A0
deducem egalitatea rapoartelor din membrii din stnga. Ca urmare, X10 coincide cu
X 0 , deci 2 este demonstrata.
MC
MC
X 0A
XA
si 0 0 =
=
, deci X si X 0 mpart [AA0 ]
Mai sus s-a aratat ca
0
0
XA 0
MA
XA
M A0
n acelasi raport, adic
a X, X sunt conjugate armonic fata de A, A0 . Afirmatia 3 ,
deci si propozitia, este demonstrat
a.
de unde

Observatie. Sa urmarim deplasarea punctului X pe AA0 , atunci cnd M parcurge BC. Not
am cu G centrul de greutate al triunghiului ABC si cu A mijlocul
0
medianei [AA ]. Dac
a M este n C, atunci X coincide cu A. Dac
a M se ndep
arteaz
a
de C, la "dreapta" acestuia, atunci X (AA ). Daca M se apropie de B, din
"stnga" acestuia, atunci X (A G). Pentru M situat n B, X coincide cu G. Daca
M (BA0 ], atunci X (GA0 ]; punctul M n pozitia A0 coincide cu X. Pentru M
ntre A0 si mijlocul segmentului [A0 C], X parcurge semidreapta de origine A0 ce nu
contine A. n sfrsit, daca M este ntre mijlocul lui [A0 C] si C, atunci punctul X
este situat pe semidreapta de origine A ce nu contine A0 si se apropie de vrful A.
O pozitie particular
a interesant
a a punctului M este semnalat
a n urm
atorul
Corolar. Daca sunt ndeplinite conditiile din Propozitia 1 si n plus M C =
a
= N B = , atunci X este izotomicul punctului G n raport cu A si A0 , iar X 0 este
2
simetricul lui A0 fata de A.
XA
MC
a/2
1
=
=
Demonstratie. Avem X [AA0 ] si
= , adica
XA0
M A0
a/2 + a/2
2
1
1
a XA = GA0 si
XA0 = 2XA sau XA = AA0 . Cum avem si GA0 = AA0 , rezult
3
3
0
XA
XA
1
= ,
prima afirmatie este dovedita. Pe de alta parte, X 0 [AA0 ] si 0 0 =
0
XA
XA
2
deci 2X 0 A = X 0 A0 , de unde X 0 A = AA0 . Asadar X 0 si A0 sunt simetrice fata de A.
2. Aplicatii. Fie D, Da , Db , Dc punctele de tangenta a cercurilor nscris, Aexnscris, B-exnscris respectiv C-exnscris triunghiului ABC cu dreapta BC. Amintim c
a au loc egalit
atile [1], p.30:
DB = Da C = p b si Db C = Dc B = p a (2p = a + b + c);

a
primele spun c
a punctele D si Da sunt izotomice fata de B si C, iar ultimele c
aceast
a proprietate o au si punctele Db si Dc . Putem presupune c
a AB 6= AC
pentru a evita cazul trivial n care 4ABC ar fi isoscel cu vrful A.
100

Propozitia 2. Relativ la punctele Db , Dc , sunt adevarate afirmatiile:


1 B 0 Db si C 0 Dc se intersecteaza ntr-un punct X (AA );
2 B 0 Dc si C 0 Db se intersecteaza ntr-un punct Y (A G);
YA
XA
+
= 2;
3
0
XA
Y
A0

4 X este izotomicul lui G n raport cu A si A0 daca si numai daca 2a = b + c.


Demonstratie. 1 si 2 decurg direct din Propozitia 1 si observatia de mai sus.
De asemenea, avem
YA
Db C
Dc C
pa
p
2p a
XA
+
=
+
=
+
=
= 2,
XA0
Y A0
Db A0
Dc A0
(p a) + a/2 p a/2
p a/2

a
adic
a are loc 3 . n sfrsit, avnd n vedere Corolarul, X este izotomicul lui G dac
a
a
si numai daca Db C = , adica p a = sau 2a = b + c.
2
2
Observatie. Triunghiurile ce satisfac conditia
A
2a = b+c (o latur
a este media aritmetic
a a celorlaltor
doua) sunt speciale, cu multe proprietati cunoscute
X
(n [2], p. 242, sunt date opt propriet
ati). Afirmatia
B
C
Y

4 indic
a o nou
a proprietate caracteristic
a lor.
U
Un rezultat similar se obtine dac
a lu
am punctele
Dc B
D A Da C Db
izotomice D, Da n locul punctelor Db , Dc .
Propozitia 3. Daca 4ABC satisface conditia
a
|b c| 6= , atunci relativ la punctele D, Da avem:
2
1 B 0 D si C 0 Da se intersecteaza n U AA0 ;
V
2 B 0 Da si C 0 D se intersecteaza n V AA0 ;
U
A
V
A
3

= 2 (+ n cazul b > c si n cazul b < c).


U A0
V A0
Omitem demonstratia, ce urmeaza pe cea din Propozitia 2, dar mentionam ca
a
a fie mijlocul segmentului [A0 C].
prin conditia |b c| 6= se evit
a ca D sau Da s
2
Observatie. Punctele X 0 , Y 0 , U 0 , V 0 conjugatele armonic ale punctelor X, Y ,
U , V n raport cu A si A0 sunt alte patru puncte pe dreapta AA0 care pot fi puse n
conexiune cu punctele de tangenta D, Da , Db , Dc , asa cum se indica n Propozitia 1.
R
amne n seama cititorului examinarea lor.
Bibliografie
1. T. Lalescu - Geometria triunghiului, Ed. Tineretului, Bucuresti, 1958.
2. V. Gh. Vod
a - Vraja geometriei demodate, Ed. Albatros, Bucuresti, 1983.

101

O constructie geometric
a a mediilor (II)
Claudiu - Stefan
POPA1

n [2] am prezentat o constructie geometric


a a mediilor armonic
a, geometric
a,
aritmetica, patratica si ponderata a lungimilor bazelor unui trapez, ca segmente cu
capetele pe laturile neparalele ale trapezului si paralele cu bazele lui. Ne propunem
n continuare dezvoltarea acestor idei, fapt ce va conduce la o serie de consideratii
cu interesante aplicatii geometrice.
Propozitie. Fie ABCD un trapez cu AB k CD si punctele A0 , B 0 , C 0 , D0 astfel
nct A (A0 B), B (AB 0 ), C (C 0 D), D (CD0 ), iar AA0 = BB 0 = CC 0 = DD0 .
Notam {E} = A0 C BD0 , {F } = AC 0 B 0 D, {K} = EF AD, {L} = EF BC.
n aceste conditii, EF k AB, EF = AA0 , iar KE = F L.

Demonstratie. Deoarece AA0 k CC 0 si


C
D D
C
a c
a AC 0 CA0 este paraleloAA0 = CC 0 , rezult
gram, deci AC 0 k A0 C, AC 0 = A0 C. Analog,
K
BD0 k B 0 D, AD k A0 D0 si BC k B 0 C 0 , cu egaliK
L
E
F
t
atile de segmente corespunz
atoare. Unghiurile
0 A0 E
\ au laturile respectiv paralele si vor
\
si DAF
D
0 D 0 E ADF
\. Avem nc
fi congruente; la fel, A\
a
0 0
A D = AD, prin urmare 4A0 D0 E 4ADF , A A
B
B
deci A0 E = AF . nsa AE 0 k AF si atunci AF EA0
este paralelogram. Rezulta ca EF k AA0 si EF = AA0 , primele doua afirmatii ale
concluziei.
Fie {K 0 } = EF A0 D0 . Deoarece A0 C si BD0 sunt diagonale n trapezul A0 BCD0 ,
iar K 0 L este paralela la baze prin punctul de intersectie a diagonalelor trapezului,
urmeaza ca EL = EK 0 . nsa KK 0 = AA0 + EF , deci EL EF = EK 0 KK 0 , adica
F L = KE, ceea ce ncheie demonstratia.
Observatie. Concluzia se p
astreaz
a, cu demonstratie asem
an
atoare, n cazul n
care punctele A0 , B 0 , C 0 , D0 se afl
a pe semidreptele [AB, [BA, [CD, respectiv [DC.
n cele ce urmeaz
a vom folosi notatiile: AB = a, CD = b, AA0 = x. Studiem problema variatiei lungimii segmentului [KL] functie de x; vom gndi lungimea segmentului [AA0 ] ca fiind pozitiv
a n cazul n care A (A0 B) si negativ
a pentru A0 (AB.

2ab
Pentru x = 0, avem KL =
(= mh ), iar pentru x = ab (= mg ), avem
a+b
KL = mg , dupa cum s-a demonstrat n [2]. Desennd figurile pentru cteva valori
ale lui x > 0, observam ca segmentul [KL] "coboara" pe masura ce x creste, fara a
"atinge" ns
a linia mijlocie a trapezului. n momentul n care vom demonstra riguros
acest lucru, vom avea o (probabil) nou
a demonstratie pentru inegalitatea mediilor a
doua numere reale pozitive.
n trapezul A0 BCD0 , segmentul [K 0 L] are ca lungime media armonica a bazelor
2 (a + x) (b + x)
.
A0 B = a + x si CD0 = b + x, deci K 0 L =
a + b + 2x
1

Profesor, S
coala "Alecu Russo", Iasi

102

Atunci, avem
ma x + m2g
2(a + x)(b + x)
x(a + b) + 2ab
x + mh
= ma
.
x =
=
a + b + ax
2x + (a + b)
x + ma
x + ma
Suntem astfel condusi la studiul functiei
x + mh
y
.
f : R\ {ma } R, f (x) = ma
x + ma
Aceasta este o functie omografica, strict
ma
crescatoare pe (, ma ) si pe (ma , +),
mg
al c
arei grafic este o hiperbol
a de asimptote
mg
mh
ma
y = ma si x = ma . Prin calcul, stabilim c
a
mh O m
valorile x = mg sunt puncte fixe ale functiei.
x
g
Deoarece f ([0, )) = [mh , ma ), iar restrictia
mg
lui f la [0, ) este strict cresc
atoare, justificarea afirmatiilor anterioare este complet
a.
Consideratiile precedente conduc la urm
atoarele interpretari geometrice:
1. Faptul ca f (mh ) = 0 arata ca, daca ABCD este trapez cu AB k CD,
A0 [AB, D0 [DC sunt astfel nct AA0 = DD0 = mh , iar {E} = A0 C BD0 ,
atunci paralela prin E la bazele trapezului dat trece prin punctul de concurenta al
prelungirilor laturilor neparalele ale acestuia.
2. Faptul ca f (mg ) = mg se interpreteaza geometric astfel: daca ABCD este
un trapez cu AB k CD, iar A0 [AB, D0 [DC sunt astfel nct AA0 = DD0 = mg ,
atunci A0 C, BD0 si AD sunt concurente ntr-un punct E, iar paralela prin E la baze
intersecteaz
a BC n F astfel nct EF este media geometric
a a lungimilor bazelor.
a + kb
a kb
3. n sfrsit, sa observam ca pentru x =
, obtinem f (x) =
1+k
1k
(calculul se efectueaza cu usurinta). Prin urmare, considernd segmentul [AA0 ] de
lungime egal
a cu media ponderat
a a bazelor cu ponderile 1 si k, segmentul [KL] va
reprezenta media ponderat
a a bazelor, cu ponderile 1 si k. Acest fapt ne permite
sa construim cu rigla si compasul conjugatul armonic al unui punct; cititorul poate
dezvolta singur ideile.
KL = K 0 LKK 0 =

Bibliografie
1. L. Constantinescu - O interpretare geometrica a inegalitatii mediilor, R.M.T. 1/1982, 30.
2. C. - S
t. Popa - O constructie geometrica a unor medii, Rec. Mat. - 2/2003, 13-14.

103

O generalizare a teoremelor de baz


a
ale calculului diferential
Florin POPOVICI 1
Nota de fata si propune sa extinda teoremele de baza ale calculului diferential prin
impunerea conditiei de derivabilitate bilateral
a n locul conditiei clasice de derivabilitate. Rezultatele obtinute se exprim
a sub forma unor conditii de apartenenta,
care pot fi, nsa, interpretate geometric.
Teorema 1 (Teorema lui Fermat generalizata ). Fie f : [a, b] R o functie data.
Daca c (a, b) este un punct de extrem local al functiei f si aceasta este derivabila
bilateral (la stnga si la dreapta)
0 n punctul
c, atunci
0

0
0
0 min f
(c) , f+
(c) , max f
(c) , f+
(c) .
(1)
Demonstratie. Fie c punct de maxim local. Asadar, > 0 astfel nct avem:
f (x) f (c)
0
(c) 0,
x (c , c) [a, b] f (x) f (c)
0 f
xc
f (x) f (c)
0
x (c, c + ) [a, b] f (x) f (c)
(c) 0
0 f+
xc

0
0
si, ca urmare, 0 f+ (c) , f
(c) , adic
a are loc2 (1).
Exemplu. Functia f (x) = max x, 1 x , x 0, este
y

51
, pentru care
derivabil
a cu exceptia punctului x0 =
2

0
0
f (x0 ) = 1 5 si f+ (x0 ) = 1. Relatia (1) revine la 0

1 5, 1 , adic
a semitangentele la grafic n M0 (x0 , x0 ) sunt
x
x0
de parti diferite fata de paralela prin acest punct la axa Ox.
Teorema 2 (Teorema lui Rolle generalizata ). Daca f : [a, b] R este o functie
continua pe [a, b], derivabila bilateral pe (a, b) si f (a) = f (b), atunci exista un punct
c (a, b) astfel nct are loc relatia (1).
Demonstratie. Conform teoremei lui Weierstrass, functia f este m
arginit
a si si
atinge marginile, adica c1 , c2 [a, b] astfel nct f (c1 ) f (x) f (c2 ), x [a, b].
Daca {c1 , c2 } {a, b}, atunci rezulta ca f este functie constanta si (1) are loc
pentru orice c (a, b). Dac
a {c1 , c2 } 6 {a, b}, fie c {c1 , c2 } \ {a, b}; conform
Teoremei 1 pentru acest punct c are loc (1).
Teorema 3 (Teorema lui Lagrange generalizata ). Daca f : [a, b] R este
o functie continua pe [a, b] si derivabila bilateral pe (a, b), atunci exista un punct
c (a, b) astfel nct

0
f (b) f (a)
0
0
(c) , f+
(c) , max f
(c) , f+
(c) .
(2)
min f
ba
Demonstratie. Se aplica Teorema 2 functiei auxiliare g : [a, b] R definite prin
f (b) f (a)
g (x) = f (x)
x, x [a, b].
ba
Corolarul 1 (Teorema lui Lagrange pentru functii convexe). Fie f : I R ( I
interval deschis) o functie convexa. Atunci, pentru orice puncte a, b I, a < b,
exista c (a, b) astfel nct
1

Profesor, Liceul Teoretic "N. Titulescu", Brasov

104

f (b) f (a)
0
(c) .
f+
ba
Demonstratie. Deoarece f este convex
a pe I (deschis), rezult
a c
a f este continu
a
0
0
(x) f+
(x), x I. Prin aplicarea Teoremei 3
pe I, derivabil
a bilateral pe I si f
pe intervalul [a, b], obtinem rezultatul cerut.
Corolarul 2. Fie f : I R ( I interval deschis) o functie continua si derivabila bilateral pe I. Atunci functia f este crescatoare pe I daca si numai daca este
satisfacuta conditia
0

0
min f
(x) , f+
(x) 0, x I.
(3)
Demonstratie. Necesitatea conditiei este usor de dovedit. Pentru suficienta, fie
x1 , x2 I cu x1 < x2 . Conform Teoremei 3, aplicat
a restrictiei functiei f la [x1 , x2 ],
0

f (x2 ) f (x1 )
0
min f
(c) , f+
(c) .
c (x1 , x2 ) astfel nct
x2 x1
De aici si din (3), deducem c
a f (x1 ) f (x2 ); ca urmare, functia f este cresc
atoare.
Teorema 4 (Teorema lui Cauchy generalizata ). Daca f, g : [a, b] R sunt doua
functii continue pe [a, b], derivabile bilateral pe (a, b) si

0
0
0
/ min g
(x) , g+
(x) , max g
(x) , g+
(x) , x (a, b) ,
(4)
atunci g (a) 6= g (b) si exista un punct c (a, b) astfel nct

0
0
f (c) f+
f (c) f+
(c)
(c)
f (b) f (a)
min
,
,
max
,
.
(5)
0 (c) g 0 (c)
0 (c) g 0 (c)
g (b) g (a)
g
g
+
+
Demonstratie. Daca am avea g (a) =
2, ar
conform
Teoremei
exista
g0 (b), atunci,
0
0
0
(c) , g+
(c) , max g
(c) , g+
(c) , ceea
un punct c (a, b) astfel nct 0 min g
ce contrazice (4). Deci are loc g (a) 6= g (b).
Consider
am acum functia h : [a, b] R definit
a prin
f (b) f (a)
h (x) = f (x)
g (x) , x [a, b] .
g (b) g (a)
Observam ca h este continua pe [a, b], derivabila bilateral pe (a, b) si avem
f (a) g (b) f (b) g (a)
h (a) =
= h (b). Conform Teoremei 2, exista c (a, b) astfel
(b) g (a)

g
nct 0 min h0 (x) , h0+ (x) , max h0 (x) , h0+ (x) , adica avem
f (b) f (a) 0
f (b) f (a) 0
0
0
(c)
(c)
(6)
g (c) 0 f+
g (c)
f
g (b) g (a)
g (b) g (a) +
sau
f (b) f (a) 0
f (b) f (a) 0
0
0
(c)
(c)
(7)
g (c) 0 f
g (c) .
f+
g (b) g (a) +
g (b) g (a)
Presupunem ca are loc (6) (se procedeaza analog, daca ar avea loc (7)). Datorita
ipotezei (4), putem scrie

0
0
0
(c) , g+
(c) > 0 (8)
sau
max g
(c) , g+
(c) < 0 (9) .
min g
Dac
a are loc (8), atunci (6) ia forma
0
(c)
f
f 0 (c)
f (b) f (a)

+
0
0 (c) ,
g (c)
g (b) g (a)
g+
deci (5) este adevarata. Daca are loc (9), atunci (6) se scrie
0
f 0 (c)
(c)
f+
f (b) f (a)


0
0 (c) ,
g+ (c)
g (b) g (a)
g
deci (5) este adev
arat
a si n acest caz. Demonstratia este complet
a.
0
f
(c)

105

Asupra unei inegalit


ati
Alexandru NEGRESCU 1
La a V-a editie a Concursului interjudetean de matematica "Radu Miron", noiembrie 2003, elevilor clasei a IX-a li s-a propus urm
atoarea problem
a:
1
1
1
Fie x1 , x2 , . . . , xn (2, +) astfel nct
+
+ +
= 1.
x1 1
x2 1
xn 1
n
Demonstrati ca x1 x2 . . . xn (n + 1) .
Vom da 5 demonstratii acestei inegalit
ati.

Solutia I. Notam xi 1 = ai , i = 1, n; deci ai > 1. Aplicam inegalitatea lui


Huygens:

n
(1 + a1 ) (1 + a2 ) (1 + an ) (1 + n a1 a2 an ) , a1 , a2 , . . . , an 0.
Obtinem

n
p
(1)
x1 x2 . . . xn 1 + n (x1 1) (x2 1) (xn 1) .

Dar, conform cu inegalitatea dintre mediile armonica si geometrica, avem


p
n
n (x1 1) (x2 1) (xn 1)
1
1
1
+
+ +
x1 1 x2 1
xn 1
sau, tinnd seama de conditia din enunt,
p
n n (x1 1) (x2 1) (xn 1).
Din (1) si (2) rezulta ca x1 x2 . . . xn (1 + n)n , q.e.d.
Solutia II. Scriem

(2)

s
n
xi 1
xi 1 xi 1
xi 1
n+1
.
xi = 1 + (xi 1) = 1 +
+
+ +
(n + 1)
n
n
n
n
Ca urmare,
r
n
n
n
n n+1 (x1 1) (x2 1) (xn 1)
.
(3)
x1 x2 . . . xn (n + 1)
2
nn
n
Din (2), avem (x1 1) (x2 1) (xn 1) n , deci
r
n
n
n
n+1 (x1 1) (x2 1) (xn 1)
1.
(4)
nn2
Combinnd (3) si (4), obtinem inegalitatea ceruta.
1
Solutia III. Not
am
= yi , i = 1, n; deci yi (0, 1) si y1 + y2 + + yn = 1.
xi 1
yi + 1
Rezult
a c
a xi =
, 1, n si avem:
yi
y1 + 1 y2 + 1
yn + 1

...
=
x1 x2 . . . xn =
y1
y2
yn
1

Elev, cl. a IX-a, Colegiul National "A. T. Laurian", Botosani

106

2y1 + y2 + + yn y1 + 2y2 + + yn
y1 + y2 + + 2yn

...

y1
y2
yn
q
n
n+1
(n + 1) n+1 (y1 y2 yn )
n

= (n + 1) .
y1 y2 yn

1
1
Solutia IV. Apelam la metoda lui Sturm. Deoarece xi = 1 +
,
xi 1
xi 1
i = 1, n, inegalitatea de demonstrat se scrie
"n
# Y
n
Y
1
1
(5)
1+
(n + 1)n .
x

1
x

1
i
i
i=1
i=1
=

1+a 1+b
Sa analizam comportarea produsului

, cu a, b > 0 si a + b este cona


b
stant
a sububitar
a, atunci cnd a si b "se apropie". Presupunem a < b si nlocuim
numerele a si b cu a + t si respectiv b t, unde 0 < t < b a. Atunci
t (1 + a + b) (a b + t)
1+a+t 1+bt 1+a 1+b

=
< 0,
a+t
bt
a
b
ab (a + t) (b t)

1+a 1+b
ceea ce arat
a c
a apropiind numerele a si b produsul

descreste.
a
b
1
1
1
Daca printre numerele
,
, ... ,
exista doua inegale, atunci
x1 1 x2 1
xn 1
1
1
1
1
si celalalt este strict mai mare ca ; fie
unul este strict mai mic ca
<
n
n
x1 1
n
1
1
1
1
1
1
1
1
si

> . nlocuim
si
prin si respectiv
+
.
x2 1
n
x1 1
x2 1
n
x1 1 x2 1 n
1
1
1
1
1
1
Suma numerelor ,
+
,
, ... ,
r
amne aceeasi,
n x1 1
x2 1
n x3 1
xn 1
dar membrul stng n (5), obtinut prin aceast
a nlocuire, este mai mic. n noul set
1
de numere avem unul egal cu , iar, daca printre celelalte exista doua inegale, se
n
1
procedeaza la fel pna cnd se obtine un set de numere egale cu . n acest caz
n
membrul stng are valoare minim
a, anume,

1 .1
n
= (n + 1) .
1+
n
n
Solutia V. Vom dovedi mai nti rezultatul urmator:
Lem
a. Daca ai , bi > 0, i = 1, n, atunci
v
v
v
un
un
un
uY
uY
uY
n
n
n
t
t
(ai + bi )
ai + t
bi .
i=1

i=1

i=1

Demonstratie. Inegalitatea se poate scrie astfel:


r
r
an
bn
a1
b1
n
1
...
+ n
...
a1 + b1
an + bn
a1 + b1
an + bn
107

(6)

si rezult
a aplicnd inegalitatea mediilor:

b1
a1
1
an
bn
1=
+ +
+ +
+

n
a1 + b1
an + bn
a1 + b1
an + bn
r
r
an
bn
a1
b1
n
...
+ n
...
.
a1 + b1
an + bn
a1 + b1
an + bn
Lund n (6) ai = xi 1 si bi = 1, i = 1, n, obtinem
(2)
p

n
x1 x2 . . . xn n (x1 1) (x2 2) (xn 1) + 1 n + 1,
de unde rezult
a inegalitatea dorit
a.
Observatie. Aceasta problema poate fi usor generalizata astfel:
1
1
1
Fie x1 , x2 , . . . , xn (2, +) astfel nct
+
+ +
= k.
x2 1
xn 1

n x1 1
k+n
Demonstrati ca x1 x2 . . . xn
.
k
Bibliografie
1. Gh. Andrei si colab. - Exercitii si probleme de algebra pentru concursuri si olimpiade scolare, Partea I, Constanta, 1990.
2. M. Ganga - Manual pentru clasa a IX-a, Profil M1,M2, Ed. Mathpress, Ploiesti,
2003.
3. M. Ganga - Probleme elementare de matematica, v. II, Ed. Mathpress, Ploiesti,
2003.
4. D. S
t. Marinescu, V. Cornea - Doua inegalitati si unele aplicatii ale acestora,
Gazeta Matematic
a, CVI (2001), nr. 3, 102-104.
5. I. Nedelcu - Probleme de matematica pentru liceu, Ed. Mathpress, Ploiesti, 2003.
6. L. Panaitopol, M. Lascu, V. B
andil
a - Inegalitati, Ed. GIL, Zalau, 1996.

1 (Problema de cnt
arire a lui Bachet). Care este cel mai mic num
ar de
greutati care pot fi folosite pentru a cntari cu o balanta ori numar ntreg de kilograme
de la 1 la 40?
2. G
asiti dou
a numere care se scriu n baza 10 numai cu ajutorul cifrei 1 si care
au suma egala cu produsul lor.
3. Un urs pleac
a din brlogul s
au 1 km spre sud, se ntoarce si parcurge 1 km
spre est, apoi 1 km spre nord, revenind astfel n punctul de plecare. Ce culoare are
ursul?
Not
a. R
aspunsurile la aceste probleme se g
asesc la p. 110 si la p. 123

108

Asupra problemei VII.41 din RecMat - 2/2003


n nr. 2/2003 al revistei Recreatii matematice este publicata urmatoarea problema,
propus
a de elevul Alexandru Negrescu din Botosani:
a
b
+
= 1.
b+1 a+1
Aceast
a problem
a a fost apoi propus
a elevilor de cl. a VII-a n cadrul Concursului
"Recreatii matematice", editia a III-a, 2003, Iasi.
Aceste mprejurari, ct si accesibilitatea problemei, au facut ca aceasta sa se
bucure de atentia elevilor. Ca rezultat, au fost date mai multe solutii distincte
sau variante ale lor. Nota de fata colecteaz
a aceste solutii. Cititorul va observa
entuziasmul si pasiunea cu care elevii au atacat problema VII.41.
VII.41. Rezolvati n N2 ecuatia

Solutia I (Alexandra Ciofu, eleva, Hrlau). Daca perechea (a, b) este solutie
a
b
a ecuatiei, atunci
1 si
1, de unde a b + 1 si b a + 1.
b+1
a+1
I Daca a b, rezulta ca 0 a b 1, deci a b {0, 1}. n cazul a b = 0,
2a
= 1. De aici, obtinem a = 1
urmeaz
a a = b si, deoarece (a, b) este solutie,
a+1
si, deci, perechea (1, 1) va fi solutie a ecuatiei date. n cazul a b = 1, nlocuind n
b
= 1, deci b = 0. Asadar, n acest caz
ecuatia dat
a pe a cu b + 1 obtinem 1 +
b+2
obtinem solutia (1, 0).
II Dac
a a b, rolurile numerelor a si b se schimb
a si (a, b) va fi (1, 1) sau (0, 1).
Rezumnd, multimea solutiilor ecuatiei date este {(0, 1) , (1, 0) , (1, 1)}.
Solutia II (Maria Cr
aciun, eleva, Hunedoara). Ecuatia data este echivalenta
a
cu a (a + 1) + b (b + 1) = (a + 1) (b + 1), deci cu a2 ab + b2 = 1. Aceasta din urm
poate fi scris
a sub forma a (a b) b (a b) = 1 ab sau (a b)2 = 1 ab. Asadar,
numarul 1 ab este un patrat perfect cel mult egal cu 1, daca perechea (a, b) este
solutie a ecuatiei. Ca urmare, 1 ab {0, 1}, adic
a ab {0, 1}.
Dac
a ab = 1, atunci perechea (a, b) va fi (1, 1) si aceasta verific
a ecuatia.
Dac
a ab = 0, atunci a = 0 sau b = 0. Dac
a a = 0, nlocuind n ecuatie obtinem
b = 1; perechea (0, 1) verifica ecuatia din enunt. Daca b = 0, obtinem n mod
asem
an
ator solutia (1, 0). Dac
a a = 0 si b = 0, verific
am c
a perechea (0, 0) nu-i
solutie.
n concluzie, multimea solutiilor ecuatiei este {(0, 1) , (1, 0) , (1, 1)}.
Solutia III (Bogdan-Alexandru Burican, elev, Hrlau). Ecuatia este echiva2
lent
a cu a2 ab + b2 = 1 si apoi cu a2 + b2 + (a b) = 2. Rezult
a c
a a2 2, deci
a {0, 1}.
Daca a = 0, din ecuatia data obtinem b = 1.
Dac
a a = 1, din relatia a2 ab + b2 = 1 obtinem b2 b = 0, de unde b = 0 sau
b = 1.
Prin urmare, perechile (0, 1), (1, 0) si (1, 1) sunt solutiile ecuatiei date.
Solutia IV (Diana Prodan, eleva, Iasi). Ecuatia a2 ab + b2 = 1, echivalenta
109

cu ecuatia din enuntul problemei, se pune n forma 3a2 + (a 2b) = 4. De aici,


avem 3a2 4, deci a {0, 1}. Se continua ca n Solutia III.
Solutia V (Adrian Hamciuc, elev, Iasi). Ca mai sus, obtinem ecuatia
a2 ab + b2 = 1. Cum a2 + b2 2ab, rezulta ca ab 1, adica ab {0, 1}. Se
ncheie ca n Solutia II.
Solutia VI (Diana Timofte, eleva, Iasi). Fie (a, b) o solu
2tie a ecuatiei date, deci
2
si a ecuatiei a2 ab + b2 = 1. Ca urmare, ecua
t
ia
b

ab

a 1 = 0, considerat
a

n b, are solutii reale si atunci = a2 4 a2 1 0. Rezulta ca 3a2 4, deci


a {0, 1}. Se continu
a ca n Solutia III.
Solutia VII (Alexandru Negrescu, elev, Botosani). Conform inegalit
atii
Cauchy-Buniakovski-Schwarz, avem

a
b
[(b + 1) + (a + 1)]
a+ b ,
+

b+1 a+1

a + b (b + 1) + (a + 1), adica a + b + 2 ab a + b + 2 sau ab 1.


de unde
De aici ab {0, 1} etc.
Observatie. Solutiile prezentate mai sus au comun faptul ca n prima parte se
obtine, cu tehnici diverse de calcul, una dintre relatiile a {0, 1}, a b {0, 1},
ab {0, 1}. n partea a doua sunt utilizate aceste relatii (una dintre ele) pentru
determinarea multimii solutiilor ecuatiei.

Solutia Problemei de cntarire a lui Bachet (p. 108)


Dac
a greut
atile se pun pe un singur taler, sunt necesare sase greut
ati: 1 kg, 2 kg,
4 kg, 8 kg, 16 kg, 32 kg. ntr-adevar, orice greutate de la 1 la 40 poate fi atinsa
astfel:
1 = 1,
6 = 4 + 2,
2 = 2,
...............
3 = 2 + 1, . . . . . . . . . . . . . . .
4 = 4,
39 = 32 + 4 + 2 + 1,
5 = 4 + 1, 40 = 32 + 8.
Daca greutatile pot fi puse pe ambele talere, sunt necesare numai patru greutati:
1 kg, 3 kg, 9 kg, 27 kg. ntr-adevar, avem:
1 = 1,
6 = 9 3,
2 = 3 1,
...............
3 = 3,
...............
4 = 3 + 1,
39 = 27 + 9 + 3,
5 = 9 3 1, 40 = 27 + 9 + 3 + 1.
110

Asupra unei probleme de concurs


Dumitru MIHALACHE, Marian TETIVA1
n aceast
a not
a ne propunem s
a prezent
am dou
a modalit
ati de abordare a unei
probleme de geometrie si s
a obtinem o generalizare a sa (rostul ghilimelelor se va
vedea la vremea potrivita). Problema a fost propusa de C. Apostol la Concursul
National de Matematica Laurentiu Duican (ajuns, iata, la cea de a XI-a editie:
felicit
ari si succes n continuare!) si enuntul ei poate fi citit n [1]; iat
a acest enunt
(modificat pentru a retine doar esentialul):
b = 80 , m(B)
b = 120 ,
Problema 1. n patrulaterul convex ABCD avem m(A)

b
b
b
m(C) = 50 , m(D) = 110 . Sa se arate ca, daca (BD este bisectoarea unghiului B,
b
atunci (AC este bisectoarea unghiului A.

ntr-o prima faza, negasind nici o alta idee, am ncercat o rezolvare a problemei
bazata pe calcule trigonometrice. Rezultatul acestei cautari este
Metoda I de rezolvare a problemei. Din
C
\ = m(DBC)
\ =
D
ipotez
a rezult
a c
a m(DBA)
N
\ = 40 , iar
= 60 si, apoi, ca m(ADB)
\ = 70 . S
m(BDC)
a consider
am punctele
M AB (A (M B)) si N (BC) astfel nct triunghiurile M BD si BN D sa fie
P
\
echilaterale (prin urmare m(M
DA) = 20 si
\
m(N
DC) = 10 ). Fie a lungimea laturilor M
A
B
acestor triunghiuri. Cu teorema sinusurilor n
triunghiul M AD obtinem
MA
a
a sin 20
a sin 20
=

M
A
=
=
,
sin 20
sin 100
sin 100
sin 80
iar din triunghiul N CD,
a
a sin 10
NC
=

N
C
=
.
sin 10
sin 50
sin 50
Atunci
2 sin 30 cos 50
cos 50
sin 80 sin 20
=a
=a
AB = M B M A = a

sin 80
sin 80
sin 80
si
cos 20
sin 50 + sin 10
=a
,
BC = BN + N C = a

sin 50
sin 50
prin urmare
2 cos 20 sin 80
sin 40
BC
cos 20 sin 80

=
=
2
cos
20
=
=
AB
sin 50 cos 50
sin 100
sin 20
(este evident c
a am aranjat acest raport, stiind unde vrem s
a ajungem).
1

Profesori, Colegiul National "Gh. Rosca Codreanu", Brlad

111

\ deci m(BCA)
\ = 60 x; tot cu teorema sinusurilor
Acum, fie x = m(BAC),
(acum n triunghiul ABC) si tinnd seama de calculul anterior, avem
sin x
sin 40
sin x sin 20 = sin (60 x) sin 40 ;
=
sin (60 x)
sin 20
transformam produsele n sume si avem
deci

cos (x 20 ) cos (x + 20 ) = cos (20 x) cos (100 x) ,

cos (100 x) cos (x + 20 ) = 0 sin 60 sin (40 x) = 0.


Cum, evident, 0 < x < 80 , de aici rezulta x = 40 si problema este rezolvata.
Desigur, asemenea calcule nu sunt pentru cl. VII-a (iar problema a fost propusa
elevilor acestei clase); s
a vedem asadar si o solutie a problemei la acest nivel.
Metoda a II-a se bazeaz
a pe observatia urm
atoare: dac
a P (BC) este punctul

\
pentru care m(P DC) = 50 , atunci triunghiul P DC este isoscel cu P D = P C
\
(evident), iar triunghiul ADP este echilateral. ntr-adev
ar, unghiul BP
D, se vede
imediat, are masura de 100 , de aceea patrulaterul ABP D este inscriptibil (avnd
\
\ = 60 , iar, pe de
dou
a unghiuri opuse suplementare). Atunci m(AP
D) = m(DBA)

\) = m(ADC)
\ m(P
\
alt
a parte, m(ADP
DC) = 110 50 = 60 , deci triunghiul

ADP are dou


a unghiuri de m
asur
a 60 .
Rezult
a c
a avem AP = DP = P C si triunghiul P AC este tot isoscel; prin urmare
[
180 m(AP
C)
m(P[
AC) = m(P[
CA) =
= 20 ,
2
\ = 40 si demonstratia se ncheie.
de unde rezult
a imediat m(BAC)
F
ar
a ndoial
a, aceast
a a doua variant
a de solutionare a problemei este mult mai
simpl
a dect prima; totusi, ideea de a alege punctul P nu vine prea usor. n schimb,
rolul sau n problema este fundamental; de fapt, noi credem ca problema a fost
construita pornind de la cele doua triunghiuri isoscele lipite, ADP si CDP . Ca
triunghiul ADP este chiar echilateral nu este esential si de asta v
a puteti convinge
rezolvnd urm
atoarea generalizare a primei probleme.
b = + , m(B)
b = 2 + ,
Problema 10 . Fie ABCD un patrulater n care m(A)

b = , m(D)
b = + , m(DBC)
\ = , unde , , > 0 si 2 + + = 180 .
m(C)
Atunci:
a) 90 > > .
b
b) (AC este bisectoarea unghiului A.
b
\ avem 2+ > si 2 < 2++ = 180
Solutie. a) Deoarece m(B) > m(DBC),
\ = 2+ si m(ADB)
\ = 180 (2 + + + ) =
< 90 . Apoi, m(ABD)
= , deci > .
b) Alegem un punct P (BC) astfel nct
\
\
m(P
DC) = m(P
DB) = .

\ = + . Mai departe
Acest punct exist
a pe segmentul (BC), deoarece m(BDC)
demonstratia decurge ca mai sus: observati ca triunghiul P CD este isoscel, apoi ca
112

\
ABP D este patrulater inscriptibil si c
a triunghiul P AD este isoscel (cu m(P
AD) =
\
= m(P DA) = ). Rezult
a triunghiul P AC isoscel, de unde se va putea calcula
\ = + , ceea ce ncheie rezolvarea.
masura lui P[
AC si apoi se ajunge la m(BAC)
2
Nu e o generalizare efectiv
a, totul se bazeaz
a pe aceeasi idee (Problema 1 se
a de ce am pus cuvntul ntre
reg
aseste pentru = 50 , = 60 si = 20 ): iat
ghilimele.
Iar pentru a ncheia lasam ca tema o problema nrudita cu cele de mai sus
(eventual ncercati si o generalizare a ei).
b = 110 si m(C)
b = 50 . ConsideProblema 2. Fie ABC un triunghi cu m(B)
\
\
ram punctele M (AC) si N (AB) astfel nct m(M
BC) = 70 si m(N
CB) = 30 .

\
Sa se calculeze m(AM
N ). (R
aspuns: 60 .)
Bibliografie
1. F. Diac - A XI-a editie a Concursului National de Matematica Laurentiu Duican,
Brasov, 2003, G. M. 11/2003.

LISTA MEMBRILOR FILIALEI IA


SI a S. S. M.
continuare din nr. 1/2000, 1/2001, 1/2002, 1/2003 si 1/2004
126.
127.
128.
129.
130.
131.
132.
133.
134.
135.
136.
137.
138.
139.
140.
141.
142.
143.
144.
145.
146.
147.

DIMITRIU Gabriel

RADUCANU
Petru
IUREA Gheorghe
Cristian
LAZAR
PETCU Alina Emilia
POPA Gabriel
AM
ANU

Laura
A
VAT
T
NEDELCU Andrei
AU
SU Alexandru
CAR
ROMAN Neculai

CALIN
Ionela
S Angelica
GOLAE
BEJAN Tinuta
BUZAC Gabriela - Tamara

PADURARU
Adriana
LUCHIAN Dorel
COZLAC Magda
LUCA TUDORACHE Rodica
POPA Antoaneta
ARBONE Dorina
IONESCU Mihaela
SAVA Radu

I. M. F., Iasi
Liceul "D. Cantemir", Iasi
Liceul "D. Cantemir", Iasi
Colegiul National, Iasi
Liceul Energetic, Iasi
Colegiul National, Iasi
S
coala Waldorf, Iasi
Liceul "Gr. Moisil", Iasi
Univ. Tehnic
a "Gh. Asachi", Iasi
S
coala "V. Alecsandri", Mircesti (Iasi)
Liceul "D. Mangeron", Iasi
Gr. sc. ind. usoar
a "Victoria", Iasi
S
coala "Al. Vlahuta", Iasi
Liceul Economic nr. 1, Iasi
S
coala "B. P. Hasdeu", Iasi
Liceul "M. Costin", Iasi
Liceul ind. nr. 7, Iasi
Univ. Tehnica "Gh. Asachi", Iasi
S
coala Mnzatesti (Iasi)
S
coala "Mircea cel B
atrn", Iasi
S
coala "I. Ghica", Iasi
Colegiul "C. Negruzzi", Iasi
(continuare la p. 128)
113

Exponentul num
arului natural a n produsul n!
1

Mihai CRACIUN

n cele ce urmeaz
a, date fiind a, n N , vom indica o formul
a de calcul a exponentului lui a n n!, notat expn! (a), mpreun
a cu cteva aplicatii.

n
n
n
Teorema 1. Daca p este prim, atunci expn! (p) =
+ 2 + 3 + .
p
p
p
Demonstratie. Mai nti, se observ
a c
a suma din enunt este finit
a si se continu
a
pn
a cnd termenul curent are partea ntreag
a egal
a
cu
0.

n
Dintre factorii
produsului
n!,
un
num
a
r
de
vor fi multipli de p. Dintre ei,
un

p
n
n
2
numar de 2 vor fi multipli de p , iar dintre acestia din urma, un numar de 3
p
p
vor fi multipli ai lui p3 etc. Suma numerelor indicate va fi exponentul cerut, deoarece
fiecare factor al produsului n! care este multiplu al lui pm f
ar
a a fi si multiplu al lui
pm+1 se socoteste n modul indicat de m ori, ca multiplu al lui p, p2 , . . . , pm .

50
50
50
50
Exemplul 1. exp50! (3) =
+
+
+
= 16 + 5 + 1 + 0 = 22.
3
9
27
81

expn! (p)

Teorema 2. Daca p este prim iar N , atunci expn! (p ) =


.

Demonstratia este evident


a.


7
exp50! 3
22
=
= 3.
Exemplul 2. exp50! 3 =
7
7
Teorema 3 (Legendre). Daca a N este descompus n factor primi sub forma
k
k
1
2
1 2
a = p
1 p2 pk , atunci expn! (a) = min {expn! (p1 ) , expn! (p2 ) , . . . , expn! (pk )} .
Demonstratia este imediat
a.
Exemplul 3.



exp253! 2
exp253! 3
exp253! (108) = min exp253! 22 , exp253! 33 = min
,
.
2
3
Deoarece exp253! 2 = 246 si exp253! 3 = 125, urmeaza ca exp253! (108) = 41.
Problema 1. Determinati cu cte zerouri se termina 1958!
Solutie. Num
arul de zerouri este dat de exp1958! (10). Se observ
a c
a
exp1958! (10) = min {exp1958! 2, exp1958! 5} = exp1958! (5) .


1958
1958
1958
1958
1958
+
+
+
= 467,
+
exp1958! (5) =
5
52
53
54
55
deci 1958! se termin
a cu 467 zerouri.
Problema 2. Demonstrati ca n! nu se divide cu 2n , n N.
Solutie. Fie n N si fie k N astfel ca 2k n < 2k+1 . Atunci

hni h n i
h n i n n
n
1
expn! (2) =
+ 2 + + k+1 + 2 + + k+1 = n 1
<n
2
2
2
2 2
2
2k + 1

Atunci

Profesor, Liceul "M. Sadoveanu", Pascani

114

deci expn! (2) < n si n! nu se divide cu 2n .

5n 1
Problema 3. Determinati cu cte zerouri se termin
a
!, unde n N .
4
n
5 1
. Ca mai sus, numarul de zerouri este dat de
Solutie. Sa notam N =
4
expN ! 5. n plus
n

n
5 1
5 1
5 1
+ +
+ =
expN ! 5 =
+
45
4 52
4 5k

n
n
5 5k + 5k 1
5 5+51
5 52 + 52 1
+ +
+ =
=
+
45
4 52
4 5k
5n1 1 5n2 1
5nn 1
5n 4n 1
=
+
+ +
=
,
4
4
4
16
5k 1
5k 1
deoarece
[0, 1). De aici, N ! se termina cu
N pentru k N , iar
4
4 5k
5n 4n 1
zerouri.
16
Problema 4. Factorialul caror numere se termina exact n 1000 zerouri?
Solutie. Fie n N un numar al carui factorial
a exact
h n i hsen termin
i
h n ni 1000 zerouri
k
k+1
si fie k N astfel ca 5 n < 5 . Atunci
+ 2 + + k+1 = 1000, deci

5
5
5h i
n
n
n
n
1
n
+ 2 + + k =
1 k > 1000 si n > 1000. n plus,
< 1000, deci
5
5
5
4
5
5
n < 5005,
ceea
a pentru
n 4004, n! se termin
a n cel

a k = 5.
Se
observ
ac

ce implic
4004
4004
4004
4004
4004
mult
+
+
+
= 999 zerouri, n vreme ce pentru
+
4
5
5
52
53
5 5

4010
4010
4010
4010
4010
n 4010, n! se termin
a n cel putin
+
+
= 1001
+ 2 +
5
5
53
54
55
zerouri. Numerele cerute sunt 4005, 4006, 4007, 4008, 4009.
(2m)! (2n)!
Problema 5. Demonstrati ca
N, unde m, n N.
m!n! (m + n)!
Solutie. Mai nti, se poate demonstra inegalitatea
[x] + [y] + [x + y] [2x] + [2y] , x, y R.
(1)
Exponentul
unui factor prim p din descompunerea canonica a numaratorului este
X n m m + n
s1 =
+ k +
, n vreme ce exponentul lui p din descompk
p
pk
X 2m 2n
k
punerea canonic
a a num
ar
atorului este s2 =
+ k
. Folosind (1)
pk
p
k
(2m)! (2n)!
N.
obtinem c
a s1 s2 si deci
m!n! (m + n)!
Probleme propuse.
101001!
?
1. Cu cte zerouri se termin
a num
arul
2004!
2. Sa se demonstreze ca numerele A = 111! 222! 333! 444! si B = 555! 666!
777! 888! sunt divizibile cu 10268 , respectiv 10715 .
3. Factorialul c
aror numere se termin
a cu exact 2004
zerouri?
4. Aflati exponentul lui k n 1 + k + k 2 + + kn !, unde k, n N , k 3.
115

Concursul de matematic
a Al. Myller
Editia a II-a, Iasi, martie 2004
Not
a (pentru clasele IV-VI ). Toate subiectele sunt obligatorii. Timp efectiv de lucru
90 min. Se acord
a din oficiu 30 puncte, cte 6 puncte pentru problemele 1-5, cte 8 puncte
pentru problemele 6-10 si cte 10 puncte pentru problemele 11-15.

Clasa a IV-a
1. Un elev rezolva fiecare dintre primele 5 probleme ale acestui test n cte 3
minute, iar pe fiecare dintre urm
atoarele 5 n cte 5 minute. Cte minute i trebuie
pentru a rezolva una dintre ultimele 5 probleme, presupunnd c
a fiecare din ele i
solicita acelasi timp? (timpul total de lucru este 1h 30 min.)
2. Calculeaza (100 99) + (98 97) + (96 95) + + (2 1).
3. Calculeaz
a (5 + 55 + 555 + 5555 + 55555) : (1 + 1 + 111 + 1111 + 11111) : 5.
4. Cei 41 de elevi ai unei clase urc
a n sir pe munte. Mircea observ
a c
a n fata
lui sunt un sfert dintre colegii sai. Al ctalea n sir este Mircea?
5. Delia calculeaza suma cifrelor pe care le afiseaza ceasul ei digital (de exemplu
la ora 14:28 ea obtine 1 + 4 + 2 + 8 = 15). Care este suma maxim
a pe care o poate
obtine?
6. Care sunt ultimele trei cifre ale numarului 1 2 3 4 5 . . . 2004 + 12?
7. Aflati suma dintre dempartit, mpartitor, ct si rest, stiind ca restul este cu
18 mai mic dect ctul, ctul este 25, demp
artitul este impar, iar mp
artitorul are o
singur
a cifr
a.
8. Un numar se mparte la 3 si da restul 2. Ctul se mparte din nou la 3,
obtinnd restul 2. Noul ct se mparte iar la 3 si gasim ctul 2 si restul 2. Care a
fost num
arul initial?
9. La un magazin se aduc 301 kg de mere n l
azi de 25 kg si 21 kg. Cte l
azi se
folosesc n total?
10. 58 de elevi sunt asezati pe 4 rnduri, fiecare rnd avnd cu 3 elevi mai putin
dect rndul din fata sa. Cti elevi sunt pe ultimul rnd?
11. Dan vrea s
a cumpere mingi. Dac
a ar cump
ara 5, i-ar mai r
amne 100 000 lei,
iar daca ar dori sa cumpere 7, ar mai avea nevoie de 220 000 lei. Ct costa o minge?
12. Marinarii de pe un vapor au hrana pentru 60 de zile. Ei gasesc pe o insula
30 de naufragiati si astfel hrana le va ajunge tuturor doar 50 de zile. Cti marinari
erau pe vapor?
13. 12 baeti si 8 fete sunt membri ai cercului de matematica. n fiecare saptamna,
nca 2 fete si 1 baiat sunt acceptati ca membri ai cercului. Cti membri va avea cercul
de matematic
a atunci cnd num
arul b
aietilor va fi egal cu num
arul fetelor?
14. Cte numere naturale de patru cifre au ultima cifr
a 3?
15. Pe o insula locuiesc numai arici, serpi si vulpi. Fiecare animal mannca o
data pe zi, astfel nct orice arici mannca la micul dejun cte un sarpe, orice vulpe
m
annc
a la prnz cte un arici, iar orice sarpe m
annc
a la cin
a cte o vulpe. La
sfrsitul zilei de miercuri, pe insul
a a r
amas un singur animal. Cte animale existau
pe insula luni, nainte de micul dejun?
116

Clasa a V-a
1. Sa se calculeze suma 4 + 8 + 12 + + 2000.
2. Determinati numerele naturale x astfel nct multimile A = {2x; 6x + 4; 3x + 5}
si B = {2x 1; 2x + 1; 5x + 6} s
a aib
a un singur element comun.
3. Care sunt numerele prime a, b, c pentru care a + 10 b + 12 c = 82?
4. Aflati suma cifrelor numarului A = 102004 1.
5. Sa se determine perechile de numere naturale (x, y) pentru care fractia
15
este echiunitar
a.
(x + 1) (y 4)
6. Sa se determine x N astfel nct numarul a = 5x + 5x+1 + 5x+2 + 5x+3 sa
aiba exact 48 divizori.
1 1
1
1
1
1
7. Sa se calculeze suma S = + +
+
+
+ +
.
2 6 12 20 30
360
3a + 16
8. Aflati numerele a N pentru care
N.
2a 5
9. Care este suma ultimelor trei cifre ale produsului 1 2 3 . . . 25?
10. S
a se afle cel mai mic num
ar natural de 2 cifre pentru care suma dintre p
atratul
si cubul lui este patrat perfect.
11. Asezati n ordine cresc
atoare numerele a = 250 , b = 247 + 224 , c = 245 + 244 +
23
22
48
23
+2 + 2 , d = 2 + 2 .
12. n pester
a erau dragoni rosii si dragoni verzi. Fiecare dragon rosu avea 6
capete, 8 picioare si 2 cozi. Fiecare dragon verde avea 8 capete, 6 picioare si 4 cozi.
n total dragonii aveau 44 cozi. Sunt, de asemenea, cu 6 picioare verzi mai putin
dect capete rosii. Cti dragoni rosii sunt n pester
a?
13. Dan spal
a o masin
a n 40 de minute, iar Ionut spal
a o masin
a n 2 ore. n ct
timp vor spala mpreuna 3 masini?
14. Dintre cei 101 de dalmatieni, 56 au o pat
a neagr
a pe urechea stng
a, 25 au
pat
a neagr
a pe urechea dreapt
a, iar 29 au ambele urechi albe. Cti dalmatieni au
pete negre pe ambele urechi?
15. Fie a = 2214 + 3143 si b = 3143 . Care dintre numere este mai mare?

Clasa a VI-a
7a 2b
2
b
=
, atunci este ..............................
5a + 4b
15
a
2. Numarul triunghiurilor din figura este ..............................
3. Diferenta dintre masurile suplementului si complementului aceluiasi unghi este ..............................
1
1
1
1
4. Rezultatul calculului
+
+
+ +
este .........................
13 35 57
2003 2005
0, 2 (4)
5. Doua unghiuri complementare au raportul masurilor lor egal cu
; atunci
0, 4 (2)
m
asura unghiului mai mare este ..............................

6. Rezultatul calculului ab0 ba : 99 este ..............................


7. n cte moduri putem aseza patru persoane ntr-un rnd?
8. Ultima cifr
a a num
arului 22004 + 32004 + + 92004 este ..............................
9. Avem la dispozitie timbre de 4000 lei si de 9000 lei. Pentru a expedia o
scrisoare sunt necesare timbre n valoare de 35000 lei. Numarul total de timbre
1. Dac
a

117

este ..............................
10. Suplementul unui unghi si complementul sau au masurile invers proportionale
cu 2 si 5. Suma masurilor lor este ..............................
11. Num
arul maxim de unghiuri n jurul unui punct cu m
asuri numere naturale
diferite este ..............................
12. Un produs se scumpeste cu 10% si apoi cu 20%. Acelasi produs se scumpeste
cu 20% si apoi cu 10%. n ce caz pretul final este mai mare?
13. Solutia ecuatiei 1, (1x) + 2, (2x) + + 9, (9x) = 50 este ..............................
14. Cte cifre de 0 are la sfrsit num
arul 1 2 3 4 . . . 2004?
15. Pe o tabla de sah 4 4 se trage o linie dreapta. Cel mai mare numar de
patratele 1 1 care pot fi taiate n doua parti este ..............................

Not
a (pentru clasele VII-XII ). Toate subiectele sunt obligatorii. Timp efectiv de lucru
3 ore. Pentru fiecare subiect se acord
a 7 puncte.

Clasa a VII-a
1. a) Demonstrati c
a dac
a m si n sunt numere naturale, atunci 25n 7m este
divizibil cu 3.
b) Determinati cel mai mic num
ar de forma |25n 7m 3m |, unde m si n sunt
numere naturale nenule.
Marius Ghergu, Slatina
2. Fie a, b dou
a numere reale avnd modulele cel putin 2. Demonstrati c
a
(a2 + 1)(b2 + 1) (a + b)(ab + 1) + 5. Cnd are loc egalitatea?
Marius Durea, Iasi
3. n triunghiul ABC se consider
a n
altimea [CM ], M AB, iar N este simetricul punctului M fata de BC. Paralela prin punctul N la CM intersecteaz
a BC
n P si AC n Q.
a) Demonstrati ca M Q AP daca si numai daca [AB] [AC].
b) Ar
atati cum pot fi obtinute pozitiile punctelor A, B, C atunci cnd cunoastem
doar pozitiile punctelor M , N , P .
Petru R
aducanu, Iasi
4. Se consider
a un triunghi ABC, n puncte distincte A1 , A2 , . . . , An pe latura
(BC), n puncte distincte B1 , B2 , . . . , Bn pe latura (AC) si n puncte distincte C1 ,
C2 , . . . , Cn pe latura (AB). Fie M multimea punctelor care se obtin la intersectia
a cel putin doua din segmentele (AAi ), (BBj ), (CCk ). Determinati numarul minim
si num
arul maxim de elemente pe care le poate avea multimea M .
Dan Brnzei, Iasi

Clasa a VIII-a
1. Determinati numerele ntregi a, b pentru care, oricare ar fi x real,
(2a a2 )x4 + (2a + 2b 4)x3 + (3b 3)x2 + (2b2 2b)x + b + 2 0.
Gheorghe Iurea, Iasi
118

2. Un zar este un cub de latur


a 1, pe fetele c
aruia sunt imprimate puncte ca n figura, astfel nct suma numerelor de puncte
de pe fetele opuse sa fie 7. Din opt zaruri alcatuim un cub de
latur
a 2.
a) Ce valori poate avea num
arul punctelor care sunt vizibile
pe fetele cubului de latura 2?
b) Este posibil sa asezam zarurile astfel nct oricare doua fete care sunt n contact
s
a aib
a un num
ar egal de puncte?
***

3. Fie ABCD o piramida n care AC = BC = 1 si AB = AD = BD = CD = 2.


Determinati distanta de la punctul A la planul (BCD).
***
4. Fie cubul ABCDA0 B 0 C 0 D0 de latura a si puncte M (AB), N (CC 0 ),
P (D0 A0 ), astfel ca AM = CN = D0 P = x.
a) Calculati M P .
b) Aratati ca triunghiul M N P are centrul de greutate pe segmentul [B 0 D].
Dan Brnzei

Clasa a IX-a
1. Fie n un numar natural si numerele reale a, b, c astfel nct an = a + b;
b = b + c si cn = c + a. S
a se arate c
a a = b = c.
Gheorghe Iurea, Iasi
2. Pe laturile [AB], [BC], [CD], [DA] ale unui patrulater convex ABCD se
MA
NB
PD
QA
considera punctele M , N , P , respectiv Q astfel nct
=
=
=
= k,
MB
NC
PC
QD
unde k 6= 1. Sa se arate ca S[ABCD] = 2S[M N P Q] daca si numai daca S[ABD] =
= S[BCD] .
Petre Asaftei, Iasi
3. Fie ABC un triunghi dreptunghic n A si D un punct apartinnd laturii
[BC]. Bisectoarele unghiurilor ADB si ADC intersecteaz
a laturile AB si AC n

si M N are m
punctele
M
s
i
N
.
S
a
se
arate
c
a
unghiul
dintre
dreptele
BC
asura

1 b

b daca si numai daca D este piciorul perpendicularei din A.


m(B) m(C)
2
Bogdan
Enescu, Buz
au
p
4. S
a se determine numerele reale x, x > 1 pentru care n [xn ]este ntreg, oricare
ar fi n 2. (Se noteaz
a cu [a] partea ntreag
a a num
arului real a).
Mihai Piticari, C-lung Moldovenesc, Dan Popescu, Suceava
n

Clasa a X-a
1. Fie A = {1, 2, 3, 4, 5}. Sa se determine numarul functiilor f : A A cu
proprietatea c
a nu exist
a numere distincte a, b, c A astfel nct f (a) = f (b) = f (c).
Adrian Zanoschi, Iasi
2. Se consider
a tetraedrul ABCD n care medianele din A n triunghiurile ABC,
ABD si ACD sunt perpendiculare doua cte doua. Sa se arate ca muchiile din A
sunt egale.
Dinu S
erb
anescu, Bucuresti
3. Se considera numerele reale x, y, z cu proprietatea ca cos x+cos y +cos z = 0 si
119

cos 3x + cos 3y + cos 3z = 0. S


a se demonstreze inegalitatea cos 2x cos 2y cos 2z 0.
Bogdan
Enescu,
au

Buz
4. Fie a 2 un numar natural. Consideram multimea A = { a, 3 a, 4 a, 5 a, . . .}.
a) S
a se arate c
a A nu contine o progresie geometric
a infinit
a neconstant
a.
b) S
a se arate c
a pentru orice n 3, exist
a n elemente din A care sunt n progresie
geometrica.
Bogdan Enescu, Buz
au

Clasa a XI-a
1. a) Fie (xn ) un sir de numere reale, cu proprietatea |xn+1 xn |

1
, n N.
2n

Ar
atati c
a sirul (xn ) este convergent.
b) S
a se construiasc
a un sir de numere reale (yn ), care s
a aib
a simultan proprietatile:
(i) lim (yn+1 yn ) = 0;
(ii) (yn ) este marginit;
(iii) (yn ) este divergent.
n
Eugen Popa, Iasi
2. Se da paralelogramul ABCD, cu laturi inegale. Vrful B se proiecteaza
pe AC n punctul E. Perpendiculara n E pe BD intersecteaza dreptele BC si
AB n punctele F , respectiv G. S
a se arate c
a EF = EG dac
a si numai dac
a
\ = 90 .
m(ABC)
Mircea Becheanu, Bucuresti
3. Fie matricele A, B M2 (Z) astfel nct AB = BA si det B = 1. S
a se arate
c
a dac
a det(A3 + B 3 ) = 1, atunci A2 = O.
Mircea Becheanu, Bucuresti
4. Fie f : R R o functie cu proprietatea lui Darboux. S
a se arate c
a dac
af
este cresc
atoare pe R \ Q, atunci f este continu
a.
Mihai Piticari, C-lung Moldovenesc

Clasa a XII-a
1. Fie p un numar prim, p > 3. Sa se arate ca ecuatia (x + y)1 = x1 + y 1 are
solutii n corpul Zp dac
a si numai dac
a 3 divide p 1.
Mihai Piticari, C-lung Moldovenesc
2. Sa se calculeze limita

X ln 1 + i ln 1 + j
n
n
p
lim
.
n
n4 + i2 + j 2
1i<jn

Gabriel Mrsanu si Andrei Nedelcu, Iasi


3. Fie n 3 un num
ar impar si A un inel comutativ cu 3n elemente. S
a se arate
c
a num
arul elementelor nilpotente ale lui A este cel mult n. (Elementul a A se
numeste nilpotent daca an = 0, pentru n > 0 convenabil).
***
4. S
a se arate c
a, pentru orice num
ar natural p,
Z 1
p+1
lim n
enx ln (1 + xp ) dx = p!.
n

Gheorghe Iurea, Iasi

120

Concursul de matematic
a Florica T. Cmpan
Etapa interjudetean
a, 8 mai 2004
Not
a. Toate subiectele sunt obligatorii. Timp de lucru: cl. a IV-a 90 de minute,
cl. V-VIII 2 ore.

Clasa a IV-a
1. Precizeaz
a regula de formare a numerelor urm
atoare si identific
a cifrele care
lipsesc n ultimul numar: 2798, 5783, 3574, 7862, 54 .
2. ntr-o urn
a sunt bile albe, galbene si rosii. Dac
a mp
artim num
arul bilelor albe
la numarul bilelor galbene obtinem ctul 2 si restul 2. mpartind numarul bilelor rosii
la suma celorlalte obtinem ctul 3 si restul 3. Aflati cte bile sunt n urn
a, stiind c
a
diferenta dintre num
arul bilelor rosii si dublul sumei celorlalte este 17.
3. n timpul unui campionat de sah, doi participanti care jucaser
a acelasi num
ar
de partide s-au mboln
avit si s-au retras, iar ceilalti au continuat turneul pn
a la
sfrsit. Este adevarat ca cei doi participanti au ajuns sa joace ntre ei, daca se stie ca
n total s-au jucat 23 de partide? (Turneul s-a jucat n sistemul "fiecare cu fiecare"
cte o singur
a partid
a.) Justificati r
aspunsul.

Clasa a V-a
1. Aflati vrstele tatalui, fiului si nepotului, stiind ca sunt exprimate prin trei numere prime, iar peste cinci ani vrstele lor vor fi exprimate prin trei numere naturale
p
atrate perfecte.
2. Determinati toate numerele de forma abbc, cifrele a, b, c fiind distincte, stiind
c
a sunt ndeplinite conditiile:
a) ba se divide cu 13; b) ab este num
ar prim; c) c este p
atratul unui num
ar natural.
Cte solutii are problema?
3. Pe trei jetoane asezate cu fata n jos sunt scrise trei numere naturale nenule
si distincte a caror suma este 13. Jetoanele sunt asezate n ordine crescatoare de
la stnga la dreapta. Ana ridic
a, prima, jetonul din stnga, apoi Dan pe cel din
dreapta, iar ultimul, S
tefan, pe cel din mijloc, declarnd fiecare, n aceast
a ordine,
ca nu are suficiente informatii pentru a descoperi celelalte numere. Dar tu, acum,
poti spune ce numar a vazut S
tefan?
Mihaela Cianga, Iasi

Clasa a VI-a
1. Se consider
a multimea A = {1, 2, 3, . . . , 2004}. Determinati:
a) probabilitatea ca, alegnd la ntmplare un element din A, acesta sa fie divizibil
cu 167.
b) probabilitatea ca, alegnd la ntmplare o submultime B 6= a lui A, produsul
tuturor elementelor multimii B s
a fie egal cu produsul tuturor elementelor multimii
A \ B.
Ioan Lungu, Vaslui
\ 90. Fie D (BC)
2. Se consider
a triunghiul ABC cu [AB] [AC], m(BAC)

\
astfel nct m(ADC) = 60 si fie punctul E astfel nct D (AE) si [AE] [CD].
\
Sa se determine m(EBD).
Mihai Gavrilut, Roman
121

3. Un pilot de avion parcurge o anumit


a distanta cu avionul si efectueaz
a urm
atorul calcul: aduna numerele naturale nenule ce reprezinta distanta (n km) cu viteza
(n km/h) si cu timpul (n h) si obtine 6008. Determinati ct timp a durat zborul.
Cristian Laz
ar, Iasi

Clasa a VII-a
1. S
a se arate c
a oricum am alege 51 de numere naturale distincte de la 1 la 100,
printre ele exista doua numere naturale distincte a si b astfel nct a | b.
2. Un casier de banca distrat, platindu-i un cec lui Lucian Georges, a ncurcat
euro cu centii (1 Euro = 100 centi), dndu-i acestuia euro n loc de centi si centi n
loc de euro. Dup
a ce si-a cump
arat o acadea de 5 centi, Lucian Georges a descoperit
c
a i-a mai r
amas exact o sum
a reprezentnd dublul sumei initiale de pe cec. Care
este suma scrisa pe cec?
C
at
alin Budeanu, Iasi
3. 26 de pisici sunt nchise ntr-un labirint care
are forma din figura alaturata (fiecare latura are
lungimea de 1 m). Dac
a dou
a pisici se afl
a la distanta mai mic
a de 1, 5 m se vor zgria! Ar
atati c
a
oricum am aseza cele 26 de pisici n labirint, macar
doua se vor zgria.
Monica Nedelcu, Iasi
1m

1m

Clasa a VIII-a
1. Fie tetraedrul ABCD. Vrfului A i asociem numarul natural n, iar vrfurilor
B, C, D le asociem num
arul 0. Numim "mutare" alegerea a dou
a vrfuri oarecare si
m
arirea numerelor asociate lor cu cte o unitate. S
a se arate c
a dup
a un num
ar finit
de "mutari" putem face ca fiecarui vrf sa-i fie asociat acelasi numar, daca si numai
daca n este numar par.
Gheorghe Iurea, Iasi
2. Pentru a R consideram functia fa : R R, fa (x) = ax + 2 a, x R si
fie Ma simetricul originii fata de graficul functiei fa .
a) Ar
atati c
a graficele functiilor fa trec printr-un punct fix, oricare ar fi a R.
b) Sa se arate ca
, oricare ar fi a si b R, lungimea segmentului Ma Mb este mai
mica sau egala cu 2 5.
Gabriel Popa, Iasi
3. Un corp gol n forma de tetraedru regulat este asezat cu o fata pe pamnt si
sufer
a dou
a r
asturn
ari instantanee, consecutive, pe alte dou
a fete ale sale. O bil
a neelastic
a, aflat
a initial n vrful ce nu atinge p
amntul, se misc
a sub actiunea atractiei
pamntului naintea primei rasturnari si pna dupa ultima rasturnare, imediat dupa
fiecare rasturnare plecnd spre alta fata din punctul n care a ajuns.
a) Ar
atati c
a bila cade de fiecare dat
a (nu se rostogoleste pe fete).
b)
Calcula
t
i
lungimea
parcursului
bilei,
dac
a latura tetraedrului are lungimea de

9 6
m.
32
Claudiu-
Stefan Popa, Iasi

122

Concursul Traian Lalescu


Editia a V-a, Iasi, 2004
Not
a. Fiecare subiect va fi notat cu cinci puncte. Timp de lucru: 2 ore.

1. Calculati: [62 + 8 (24 24 : 4 3)] : 5.


2. Aflati valoarea lui x din egalitatea: 1500 [(409 307) 4 + 315 : 15 x] : 3 =
= 1322.
3. Determinati cel mai mic num
ar natural n astfel nct: n+3n+5n+7n+9n >
> 2004.
4. Cte numere de trei cifre au suma cifrelor mai mare sau egala cu 25?
5. Pe tabla s-a scris de douazeci si trei de ori numarul 13 si de treisprezece
ori num
arul 23. Cte numere trebuie sterse de pe tabl
a pentru ca suma numerelor
r
amase s
a fie 464?
6. Gasiti cel mai mic numar natural care are suma cifrelor 102.
7. Multimea numerelor naturale diferite de zero se mparte n grupe astfel: (1),
(2, 3), (4, 5, 6), (7, 8, 9, 10), . . . . Cu ce num
ar ncepe grupa cu num
arul 51?
8. Suma anilor de nastere ai Raluc
ai si Elenei este 3961. S
tiind c
a la data de 8
mai 1998 Raluca era de sase ori mai mare dect Elena, cti ani are astazi Raluca?
9. De-a lungul unui gard sunt opt tufe de zmeura. Numarul fructelor de pe doua
tufe vecine difer
a cu 1. Num
arul fructelor de pe toate tufele poate fi:
a) 101; b) 213; c) 225; d) 229; e) alt num
ar.
10. ntr-o clas
a, elevii politicosi sunt de dou
a ori mai multi dect cei nepoliticosi.
Numarul fetelor politicoase si al baietilor nepoliticosi este de doua ori mai mare dect
cel al baietilor politicosi si al fetelor nepoliticoase, iar numarul fetelor politicoase este
egal cu num
arul tuturor b
aietilor. Cti b
aieti sunt n clas
a, dac
a aceasta are ntre 20
si 30 de elevi?
11. mparatul Rosu lasa mostenire celor patru fii herghelia de cai astfel: cel mai
mare ia o treime din herghelie, al doilea ia trei optimi din rest, al treilea trei cincimi
din noul rest, iar mezinul a primit cei 10 cai r
amasi. Cti cai are herghelia?
12. Pe distanta de 124 de metri s-au instalat 18 conducte de ap
a. S
tiind c
a n
depozit se gaseau 15 conducte de 6 m si 10 conducte de 8 m, determinati cti metri
masoara mpreuna conductele care au mai ramas n depozit.

Solutiile problemelor enuntate la p. 108.


1. Numerele cautate sunt 11 si 1, 1, caci 11 + 1, 1 = 11 1, 1.
2. Ursul se afla la Polul Nord. El are culoarea alba.
123

Concursul Adolf Haimovici, editia a VIII-a


pentru liceele economice, industriale si agricole

Faza interjudetean
a, Iasi, 8 - 9 mai, 2004
Not
a. Toate subiectele sunt obligatorii. Timp de lucru: 2 ore.

Clasa a IX-a
1. a) Se d
a graficul functiei f : R R, f (x) = x2 + ax + a, din figura 1. Ct
este a?
b) n figura 2 sunt reprezentate graficele a trei trinoame de gradul al doilea. Pot
s
a fie acestea urm
atoarele trinoame: ax2 + bx + c, cx2 + ax + b, bx2 + cx + a? Aceeasi
ntrebare pentru figura 3.
y

Figura 1.

Figura 2.

Figura 3.

c) Se consider
a trinoamele de gradul al doilea de forma x2 + px + q, p, q Z si
p + q = 30. Care dintre ele au r
ad
acini ntregi?
2. Se d
a triunghiul ABC n care AB = 3, AC = 4 si BC = 5.
a) Sa se calculeze aria triunghiului ABC.
b) Sa se afle raza cercului nscris si raza cercului circumscris triunghiului.
c) S
a se calculeze sin B, cos B, sin 2B, cos 2B.
d) S
a se arate c
a cos nB Q, n N.
e) Argumentati ca cos nB 6= 0 si ca exista o infinitate de numere naturale n cu
proprietatea ca cos nB > 0.
3. a) Sa se arate ca (n + 1)2 (n + 2)2 (n + 3)2 + (n + 4)2 = 4, n N.
b) S
a se arate c
a pentru orice k Z, exist
a n N si o alegere convenabil
a a
semnelor + si astfel nct k = 12 22 ... n2 .

Clasa a X-a

1. Fie z1 , z2 , z3 numere complexe astfel nct |z1 | = |z2 | = |z3 | = 2004 si


|z1 + z2 + z3 | = 2004. Atunci (z1 + z2 )(z1 + z3 )(z2 + z3 ) = 0.
2. Codul Morse, care a fost mult utilizat n comunicatii telegrafice, a utilizat
punctele si liniile pentru a codifica anumite caractere. De exemplu E se codifica
printr-un punct, T printr-o linie, zero prin 5 linii etc. Cte caractere se pot codifica
prin cel mult 5 semnale Morse (puncte sau linii)?
3. Se consider
a polinoamele f = 8X 3 6X 1, g = 4X 3 3X.
a) S
a se arate c
a f nu are r
ad
acini rationale.
b) Sa se afle ctul si restul mpartirii lui f la g.
124

c) Ar
atati c
a f cos
= 0.
9

d) Este cos irational? Argumentati.


9

e) Daca h Q[X] si h cos


= 0, sa se arate ca h .. f .
9

Clasa a XI-a
1. Se d
a graficul functiei
f : R R,

f (x) = ax4 x2 + bx + c.

Sa se afle semnele numerelor a, b si c.


2. Fie dreptele d1 si d2 perpendiculare n A. Un punct F asezat n interiorul unuia
din unghiurile formate de cele dou
a drepte, este situat la distantele 1 si respectiv 8
fata de acestea. S
a se determine lungimea minim
a a unui segment [BC] (B d1 ,
C d2 ) care trece prin punctul F .

a b c

3. Fie multimile M = c a b | a, b, c N si

b c a

K = n N | n = a3 + b3 + c3 3abc; a, b, c N .
a) S
a se arate c
a dac
a A, B M, atunci AB M.

b) Pentru orice m, n K, avem mn K.


a b c
c) Exist
a o matrice E M cu proprietatea c
a c a b = aI3 + bE + cE 2 ,
b c a
a, b, c N?
d) Fie A M, nesingular
a, cu d = det A. S
a se arate c
a

22004 1
2004

si
det . . . (A ) . . .
= d2
. . . (A ) . . . = d 3 A.
{z
}
|
{z
}
|
2004

2004

Clasa a XII-a

1. Pe multimea G = {a, b, c, d, e} operatia defineste


o structura de grup. Completati tabla grupului G.

a
b
c
d
e

b
d

e
2. Fie polinoamele f = 8X 3 6X 1 si g = 4X 3 3X.
b
a) Sa se arate ca f nu are radacini rationale.
si restul mp
b) S
a se calculeze
artirii lui f la g.
ctul

c) Ar
atati c
a f cos
= 0.
9

d) Este irational num


arul cos ?
9

= 0, sa se arate ca h .. f .
e) Daca h Q[X] si h cos
9
3. a) Calculati aria subgraficului functiei f : [1, e] R, f (x) = ln x.
b) Dac
a a, b, c sunt numere reale oarecare din intervalul [1, e] astfel nct a b c,
atunci (b a) ln a + (c a) ln b + (c b) ln c < 2.

125

Olimpiada Balcanic
a de Matematic
a (juniori), 2004
Editia a VIII-a, Novi Sad (Serbia si Muntenegru)
A. Problemele de concurs - enunturi si solutii
1. Pentru x, y R, nu simultan nule, s
a se demonstreze inegalitatea

x+y
2 2
p
.
x2 xy + y 2
x2 + y 2

2. Fie 4ABC isoscel (AC = BC), M mijlocul segmentului [AC], iar [CH]
n
altimea din C. Cercul ce trece prin B, C si M intersecteaz
a a doua oar
a dreapta
CH n Q. Sa se afle raza cercului circumscris 4ABC functie de m = CQ.

3. Fie x, y N astfel nct 3x + 4y si 4x + 3y s


a fie ambele p
atrate perfecte. S
a
se arate c
a numerele x si y sunt multipli de 7.
4. Se consider
a un poligon convex cu n 4 vrfuri. Descompunem arbitrar
poligonul n triunghiuri ale c
aror vrfuri sunt printre cele ale poligonului, astfel nct
oricare doua triunghiuri sa nu aiba puncte interioare comune. Coloram cu negru
triunghiurile ce au doua laturi care sunt si laturi ale poligonului, cu rosu triunghiurile
ce au exact o latur
a care este si latur
a a poligonului, celelalte triunghiuri l
asndu-le
albe. S
a se demonstreze c
a num
arul triunghiurilor negre este cu 2 mai mare dect
numarul triunghiurilor albe.
1. Observam ca cei doi numitori sunt ntotdeauna pozitivi. Daca x + y < 0,
inegalitatea este evident
a (si strict
a). Dac
a x + y > 0, se demonstreaz
a imediat c
a
p
x2 + y 2
x + y 2 (x2 + y 2 ), x2 xy + y 2
,
2
prin urmare
p

2 (x2 + y 2 )
x+y
2 2
x+y
p

.
x2 xy + y 2
x2 xy + y 2
x2 + y 2
x2 + y 2
2
Egalitate se atinge pentru x = y > 0.
C
2. Fie P si O centrele cercurilor circumscrise triunghiurilor BCM , respectiv ABC, K mijlocul lui
[CM ], iar {L} = CH P K. Evident c
a O CH si
L
K
cum OM si P K sunt perpendiculare pe AC, rezulta
P
M
ca OM k P K. Urmeaza ca LK este linie mijlocie n
O
4CM O, deci CL = LO.
[ ) = 90
Deoarece OP BC, avem ca m(LOP
Q
\
[ ) = m(CLK)
\ = 90
m(OCB).
Apoi, m(OLP
A
H
B
[
\ OCA,
[ deoarece n
m(OCA).
ns
a OCB
altimea
CH este bisectoare n 4ABC isoscel, deci P[
OL P[
LO si P L = P O. Observam ca
4P CQ este isoscel cu P C = P Q, prin urmare va rezulta c
a CL = OQ.
2
2
n concluzie, CL = LO = OQ, deci R = OC = CQ = m.
3
3

126

.
3. Fie m2 = 3x + 4y, n2 = 4x + 3y; atunci m2 + n2 = 7 (x + y), deci m2 + n2 .. 7.
Un p
atrat perfect d
a la mp
artirea prin 7 unul din resturile 0, 1, 2, 4 si atunci, pentru
ca o suma de patrate perfecte sa se divida cu 7, trebuie ca fiecare n parte sa se divida
.
.
.
.
cu 7. ns
a, dac
a m2 .. 7, atunci m2 .. 72 si analog n2 .. 72 , adic
a 7 (x + y) = m2 + n2 .. 49,
.
.
.
deci x + y .. 7. Urmeaza ca x = (4x + 3y) 3 (x + y) .. 7, y = (3x + 4y) 3 (x + y) .. 7.

4. Notam cu x, y, z numarul triunghiurilor colorate n negru, rosu, respectiv alb;


atunci x + y + z = n 2, acesta fiind num
arul triunghiurilor din descompunerea
poligonului. Deoarece fiecare latur
a a poligonului este latur
a a exact unui triunghi
din descompunere, avem ca 2x + y = n. Combinnd cele doua relatii, obtinem ca
x z = 2, ceea ce trebuia sa demonstram.

B. Probleme aflate n atentia juriului - enunturi


1. Fie a, b, p, q N cu p, q 3, iar a, b relativ prime si de parit
ati diferite. S
a
se arate ca numarul N = 2ap b 2abq nu poate fi patrat perfect.

2. Gasiti toate numerele naturale A care se scriu cu patru cifre n baza 10, cu
2
1
asturnatul lui A).
proprietatea c
a A + 2000 = A. (Cu A am notat r
3
3
3. G
asiti toate numerele naturale n 3 cu proprietatea c
a n divide (n 2)!.
4. Pentru a, b, c [1, +), demonstrati inegalitatea

1 1 1
(1 + abc)
+ +
3 + a + b + c.
a b
c
5. Pentru x, y, z R, demonstrati inegalitatea

y2 z2
z 2 x2
x2 y 2
+
+
(x + y + z)2 .
2x2 + 1 2y 2 + 1 2z 2 + 1

a
< b < 2a, sa se arate ca
b

2
2ab a2
2ab b2
1 a b
+
1+
.

7ab 3b2 2a2


7ab 3a2 2b2
4 b a
7. Dou
a cercuri C1 si C2 sunt secante n A si B. Un cerc C cu centrul n A taie
C1 n M si P , iar pe C2 n N si Q, astfel nct N si Q sunt situate de o parte si de
\
\
alta a dreptei M P , iar AB > AM . S
a se demonstreze c
aM
BQ N
BP .
6. Daca 0 <

8. Fie E, F dou
a puncte distincte n interiorul paralelogramului ABCD. Determinati numarul maxim posibil de triunghiuri avnd aceeasi arie, cu vrfurile n trei
dintre punctele A, B, C, D, E, F .
9. Fie 4ABC nscris n cercul C. Cercurile C1 , C2 , C3 sunt tangente interior
cercului C n punctele A1 , B1 , respectiv C1 si tangente laturilor [BC], [CA], [AB] n
punctele A2 , B2 , respectiv C2 , astfel nct A, A1 sunt de o parte si de alta a lui BC
etc. Dreptele A1 A2 , B1 B2 si C1 C2 intersecteaz
a a doua oar
a cercul C n punctele A0 ,
0
0
0
0
\
B , respectiv C . Dac
a {M } = BB CC , demonstrati c
a m(M
AA0 ) = 90 .
b = 90 si punctele D [AC], E [BC]. Spre interiorul
10. Fie 4ABC cu m(C)
triunghiului construim semicercurile C1 , C2 , C3 , C4 de diametre [AC], [BC], [CD],
127

respectiv [CE] si fie {C, K} = C1 C2 , {C, M } = C3 C4 , {C, L} = C2 C3 , {C, N } =


C1 C4 . Aratati ca punctele K, L, M , N sunt conciclice.
11. Pe o tabl
a dreptunghiular
a cu m linii si n coloane, n fiecare c
asuta se afl
a
scris semnul "". Putem efectua urm
atoarele operatii:
(i) schimbarea tuturor semnelor de pe o linie (din "+" n "", iar din "" n "+");
(ii) schimbarea tuturor semnelor de pe o coloana.
a) Dac
a m = n = 100, ar
atati c
a nu putem obtine 2004 semne "+" utiliznd de
un num
ar finit de ori operatiile descrise;
b) Daca m = 100, gasiti cea mai mica valoare a lui n > 100 pentru care putem
obtine 2004 semne "+".
(continuarea tabelului din p. 113)
148.
149.
150.
151.
152.
153.
154.
155.
156.
157.
158.
159.
160.
161.
162.
163.
164.
165.
166.
167.
168.
169.
170.
171.
172.
173.
174.
175.
176.
177.

PRECUPANU Codrin
DUREA Magdalena
BURGHELEA Diana
FARCAS Marius
BAGHIU Ciprian
ION Elena
BUCESCU Dominic
PASA Narcisa

SUFITCHI
Viorica
POPA Gabriela
RUSU Virginia
ALESCU

DASC
Diana
GHERGHELAS Liliana
Olimpia
FERENT
LEONTIES Rodica
PANAINTE Ecaterina Bronia
POPA Dumitru

CARBUNE
Ioan
BRESUG Constantin

CAILEANU
Sorin
DOCA Laurentia
Cristina
BRNZILA
SU Dan
PLAE
Valentin
CONTU
OLENIUC Mariana
Amalia - Patricia
ROMILA

GRADINARU Daniela
ARBONE Ion
NECHITA Remus
BUJOR Lorena

Colegiul "C. Negruzzi", Iasi


S
coala "T. Maiorescu", Iasi
S
coala "T. Maiorescu", Iasi
S
coala "T. Maiorescu", Iasi
Liceul "D. Cantemir", Iasi
Liceul "G. Ibr
aileanu", Iasi
S
coala "I. Creang
a", Iasi
Liceul de arta, Iasi
S
coala gen. nr. 26, Iasi
S
coala gen. nr. 43, Iasi
Liceul "M. Costin", Iasi
Liceul "M. Eminescu", Iasi
Gr. sc. "Victoria", Iasi
Gr. sc. "Victoria", Iasi
Liceul "Al. I. Cuza", Iasi
S
coala "Gh. I. Br
atianu", Iasi
S
coala "St. Brsanescu", Iasi
Liceul "I. Neculce", Tg. Frumos
Liceul "I. Neculce", Tg. Frumos
Liceul "I. Neculce", Tg. Frumos
Liceul "I. Neculce", Tg. Frumos
Liceul "D. Cantemir", Iasi
S
coala normala "V. Lupu", Iasi
S
coala "I. Cantacuzino", Pascani
Gr. sc. "D. Mangeron", Iasi
S
coala normal
a "V. Lupu", Iasi
Gr. sc. "A. Saligny", Iasi
informatician
Liceul "M. Costin", Iasi
S
coala "I. Teodoreanu", Iasi
(continuare la p. 165)
128

Cercul de matematic
a Leonard Euler
organizat la Universitatea Humboldt, Berlin
Probleme pentru clasa a VIII-a
Holger STEPHAN 1

Not
a. Rubrica "Corespondente" are ca scop informarea elevilor si profesorilor din
tara noastr
a cu privire la activitatea de performanta din alte colturi ale lumii. Pentru
orice "corespondenta", solutiile problemelor vor fi publicate n num
arul urm
ator al revistei.
Materialul de fata a fost obtinut prin str
adania d-lui Dan Tiba, cercet
ator principal I,
Institutul de Matematic
a al Academiei Romne.

1. Patru numere adunate doua cte doua dau sumele 4, 7, 9, 14, 16, 19. Care
sunt cele patru numere?
2. Demonstrati c
a prin "rotirea c
atre dreapta" a unui num
ar de 8 cifre, divizibil
cu 73, se obtine tot un num
ar divizibil cu 73. (Se spune c
a un num
ar natural este
"rotit catre dreapta", daca ultima cifra este mutata n fata primei cifre; exemplu:
1234 4123.)
3. Aflati cifrele necunoscute x, y, z din egalitatea
20 058 473 11! = x00yz0055046400.
4. Un num
ar x format din cinci cifre diferite si nenule este divizibil cu 9. Ar
atati
c
a suma tuturor numerelor de cinci cifre distincte ce se pot forma cu aceste cinci cifre
(inclusiv x) este divizibila cu 2399976.
5. Gasiti toate perechile de numere ntregi x si y care sunt solutii ale ecuatiei
diofantice 2x2 + 7xy + 3y 2 = 228.
6. G
asiti toate perechile de numere ntregi x si y care sunt solutii ale ecuatiei
diofantice 2x2 + 3y 2 = 77.
7. Consider
am num
arul natural n, 1000 n < 5000. Form
am num
arul (de 12
sau 13 cifre) obtinut scriind n ordine cifrele lui 3n, 2n si respectiv n. Aratati ca
acest numar este divizibil cu 28 + 1.
8. S
ase numere prime 7 < p1 < p2 < p3 < p4 < p5 < p6 formeaz
a un "sextet
de numere prime", dac
a p2 , p3 si p4 , p5 sunt numere prime gemene (adic
a p3 p2 =
= p5 p4 = 2), iar p2 p1 = p4 p3 = p6 p5 = 4. Demonstrati c
a suma lor este
divizibila cu 630.
9. Este posibil ca suma a sapte patrate perfecte succesive sa fie un patrat perfect?

Cercet
ator dr., Institutul Weierstrass, Berlin (e-mail: stephan@wias-berlin.de)

129

Solutiile problemelor propuse n nr. 2 / 2003


Clasele primare
P.54. Calculati a si b daca 46 a = 36 + a si b 3 = 17 b.
( Clasa I )
nv. Doinita Spnu, Iasi
Solutie. 46 a = 36 + a se scrie 36 + 10 a = 36 + a, de unde 10 a = a si
a + a = 10, deci a = 5. Analog, b 3 = 17 b se scrie b = 3 + 17 b, de unde
b + b = 20, deci b = 10.
P.55. n cte moduri pot fi aranjate n linie dreapta 9 mingi rosii si una galbena?
( Clasa I )
Georgiana Ciobanu, elev
a, Iasi
Solutie. Toate mingile ocup
a 10 locuri. Mingea galben
a poate ocupa fiecare loc
din cele 10. Sunt 10 moduri.
P56. Cu cinci ani n urma, suma vrstelor a trei copii era de 11 ani. Care va fi
suma vrstelor acelorasi copii peste 6 ani?
( Clasa a II-a)
nv. Rodica Rotaru, Brlad
Solutie. In prezent suma vrstelor copiilor este 11 + 3 5 = 26 ani, iar peste 6
ani aceasta va fi 26 + 3 6 = 44 ani.
P.57. n cte moduri pot fi mpartiti 8 baieti n doua echipe de cte 4 jucatori,
daca Petru vrea sa fie n echipa cu Mihai si Dan, dar nu vrea sa fie cu Avram?
( Clasa a II-a)
Adina Dohotaru, elev
a, Iasi
Solutie. n gruparea Petru, Mihai, Dan mai trebuie un singur b
aiat. Acesta
poate fi luat din restul b
aietilor, cu exceptia lui Avram. Al patrulea elev din echip
a
poate fi ales n patru moduri.
P. 58. Sa se arate ca suma 1 + 4 + 7 + . . . + 100 mpartita la 3 da restul 1.
( Clasa a III-a)
Alexandru - Gabriel Tudorache, elev, Iasi
Solutie. Suma se scrie 1 + (3 1 + 1) + (3 2 + 1) + . . . + (3 33 + 1). Restul
mp
artirii la 3 este acelasi cu restul mp
artirii sumei |1 + 1 + 1{z+ . . . + 1} la 3, care
34 ori
este 1.
P.59. Fie a si b doua numere consecutive. Suma acestor numere mpreuna cu
numerele obtinute marind cu 12 fiecare dintre vecinii lor este 939. Care sunt cele
doua numere?
( Clasa a III-a)
nv. Maria Racu, Iasi
a
Solutie. Consider
am b = a + 1. Folosim metoda figua
11
rativ
a. Primul num
ar este
13
a
[939 (11 + 13 + 1 + 1 + 11 + 1 + 13)] : 6 = 888 : 6 = 148.

Al doilea numar este 148 + 1 = 149.

939
11
13

P.60. Din 16 bile, una este mai grea dect celelalte 15, care au mase egale. Care
este cel mai mic numar de cntariri prin care se poate stabili bila mai grea?
( Clasa a III-a)
Carmen Ciolacu, elev
a, Iasi
Solutie. Asez
am cte 8 bile pe fiecare taler. Dup
a prima cnt
arire se determin
a
grupul de 8 bile care contine bila mai grea. Din acestea asez
am cte 3 pe fiecare
taler. Daca balanta este n echilibru, atunci bila mai grea se afla n perechea ramasa.
130

Printr-o nou
a cnt
arire se afl
a bila mai grea. Dac
a balanta nu este n echilibru,
atunci bila mai grea se afla ntr-o grupare de 3 bile. Aseznd cte o bila pe fiecare
taler se determina bila mai grea. Numarul minim de cntariri este 3.
P.61. Suma a doua numere este un numar de doua cifre al caror produs este 3.
Diferenta dintre cele doua numere este 7. Care sunt cele doua numere?
( Clasa a IV-a)
nv. Maria Racu, Iasi
Solutie. Suma celor dou
a numere poate fi 13 sau 31. Cum diferenta numerelor
este 7, n primul caz numerele sunt 3 si 10, iar n al doilea numerele sunt 12 si 19.
P.62. Doua ceasuri au nceput sa functioneze la aceeasi ora. Se constata ca la
fiecare 30 minute (fata de ora exacta) unul ramne n urma cu un minut iar celalalt
avanseaza cu un minut. La un moment dat orele indicate de aceste ceasuri sunt: 18
h 36 min si 19 h 24 min. La ce ora au nceput sa functioneze?
( Clasa a IV-a)
Felicia Amih
aiesei, elev
a, Iasi
Solutie. Cele dou
a ceasuri se abat cu acelasi num
ar de minute fata de ora exact
a,
unul prin lipsa iar celalalt prin adaos. n momentul citirii abaterea este (19 h 24 min
18 h 36 min) : 2 = 48 min : 2 = 24 min. Ceasurile au fost citite la ora 19 h 24 min
24 min = 19 h. Num
arul de ore n care ceasurile au functionat este 24 : 2 = 12.
Ceasurile au nceput s
a functioneze la ora 19 12 = 7.
P.63. Alege un numar format din trei cifre. Scrie la dreapta lui un numar format
din doua cifre. Scoate din numarul format de 99 ori numarul format din trei cifre.
Din rezultat scoate diferenta dintre numarul de trei cifre si numarul de doua cifre si
scrie rezultatul. Eu ti ghicesc numarul format din doua cifre. Cum se explica acest
lucru?
( Clasa a IV-a)
Prof. Petru Asaftei, Iasi
Solutie. Fie abc numarul de trei cifre si xy numarul de doua cifre. Avem abcxy =
= abc00 + xy = 100 abc + xy, 100 abc + xy 99 abc = abc + xy. n continuare
folosim metoda figurativ
a.

n urma efectu
arii operatiilor indicate n problem
a se obtine dublul num
arului de
doua cifre.

Clasa a V-a
V.41. Fie a numar natural compus astfel nct daca p | a, cu p prim, atunci
p + 1 | a. Sa se arate ca 12 | a si sa se afle cel mai mare numar a de trei cifre.
Ciprian Baghiu, Iasi
Solutie. Fie p un divizor prim al lui a, conform enuntului. Atunci p + 1 | a, deci
p (p + 1) | a si, cum p (p + 1) este num
ar par, deducem c
a 2 | a. Din 2 | a rezult
a
3 | a si apoi 4 | a; prin urmare 12 | a.
Cel mai mare num
ar de trei cifre divizibil cu 12 este 996 = 22 3 83; cum 84 - 996,
deducem ca 996 nu este solutie. La fel 984 nu este solutie. Numarul 972 = 22 35
verific
a cerintele problemei.
V.42. Se dau numerele xy, ab scrise n baza 10 astfel nct xy divide ab. Sa se
131

arate ca x = y daca si numai daca a = b.


Ioan S
ac
aleanu, Hrl
au
Solutie. Fie x = y. Atunci din xx = 11x | ab rezulta ca 11 | ab, deci ab {11, 22,
33, . . . , 99}, adic
a a = b.
Fie a = b. Atunci xy | 11a si, cum a este cifr
a nenul
a, deducem c
a xy se divide la
11. Ca mai sus, avem x = y.
V.43. Sa se afle cifrele a si b stiind ca a b = cd si ab = dc.
Romanta Ghita
si Ioan Ghita
, Blaj
Solutie. Evident, a {2, 3, . . . , 9}. Daca a = 2, atunci b 5 si cum 2 5 = 10 si
25 = 32, 2 6 = 12 si 26 = 64, iar 27 > 100 rezult
a c
a nu avem solutii. Dac
a a = 3,
urmeaz
a b 4 si din 3 4 = 12 si 34 = 81, iar 35 > 100 deducem c
a nici n acest caz
nu avem solutii. Considernd si verificnd toate posibilitatile, obtinem solutia a = 9
si b = 2.
V.44. Sa se afle x, y, z Q+ pentru care xn = yz, y n = xz, z n = xy, cu n N.
N. N. Hrtan, Iasi
Solutie. Avem xn+1 = y n+1 = z n+1 = xyz, deci, n mod necesar, x = y = z.
Conditiile initiale se reduc la xn = x2 , prin urmare, x = y = z = 1 daca n N \ {2}
si x = y = z Q+ daca n = 2.
V.45. Se dau sase urne, unele continnd bile. Fie operatia: se aleg trei urne si
se pune cte o bila n fiecare dintre ele.
a) Compozitia urnelor fiind 0, 0, 4, 6, 6, 8, sa se indice o succesiune de operatii n
urma carora toate urnele sa contina acelasi numar de bile.
b) Compozitia urnelor fiind 0, 1, 2, 3, 4, 4, sa se arate ca nu exista o succesiune de
operatii n urma carora toate urnele sa contina acelasi numar de bile.
Gheorghe Iurea, Iasi
Solutie. a) O succesiune de operatii ce atinge scopul este (0, 0, 4, 6, 6, 8)
(1, 1, 5, 6, 6, 8) (2, 2, 6, 6, 6, 8) (3, 3, 7, 6, 6, 8) (4, 4, 8, 6, 6, 8) (5, 5, 8, 7, 6, 8)
(6, 6, 8, 8, 6, 8) (7, 7, 8, 8, 7, 8) (8, 8, 8, 8, 8, 8).
b) Presupunem c
a dup
a n operatii urnele contin acelasi num
ar m de bile, rezult
a
ca numarul total de bile din urne este 6m = 3n + 14, imposibil.

Clasa a VI-a
VI.41. Pe opt cartonase sunt nscrise cte unul din numerele 1, 2, 22 , 23 , 3,
3 , 33 , 34 . Daca P (k) este probabilitatea ca, extragnd doua cartonase, numerele
obtinute sa aiba n total k divizori distincti, sa se rezolve inecuatia P (k) 1/7.
Dumitru Dominic Bucescu, Iasi
Solutie. Considernd cele 28 de posibilitati de extragere a cartonaselor si numa2
rnd divizorii distincti ai numerelor extrase, obtinem: P (1) = 0; P (2) =
;
28
5
8
7
3
2
1
si P (k) = 0,
P (3) =
; P (4) = ; P (5) = ; P (6) = ; P (7) = ; P (8) =
28
28
28
28
28
28
1
k 9. Prin urmare, multimea solutiilor inecuatiei P (k) este S = {3, 4, 5}.
7
VI.42. Fie x, y, z N pentru care 84x + 91y + 98z = 2002. Sa se afle valoarea
maxima a sumei x + y + z.
Adrian Zanoschi, Iasi
2

132

Solutie. Relatia din enunt este echivalent


a cu 12x + 13y + 14z = 286 sau
12 (x + y + z) = 286 (y + 2z). De aici, rezulta ca x + y + z este maxim daca
y + 2z este minim si 12 divide pe 286 (y + 2z). Obtinem x + y + z maxim
pentru y + 2z = 10. Valoarea maxim
a a expresiei este 23 si se obtine pentru
(x, y, z) {(18, 0, 5) ; (17, 2, 4) ; (16, 4, 3) ; (15, 6, 2) ; (14, 8, 1) ; (13, 10, 0)}.
Not
a. D-l Titu Zvonaru, Comanesti (Bacau), stabileste valoarea minima a
sumei x + y + z. ntr-adev
ar, dac
a x + y + z 19, atunci y + 2z 57 si am avea
12 (x + y + z) + y + 2z 12 19 + 57 = 285; prin urmare, x + y + z 20. Dac
a
x + y + z = 20, atunci y + 2z = 46. Pentru y < 15, z < 15, avem y + 2z < 45. Pentru
16 y 20, avem ca z 4, deci y + 2z 28. Pentru 16 z 20, avem y 4, deci
y + 2z 44. Rezult
a c
a valoarea minim
a a sumei x + y + z este 21, care se obtine,
de exemplu, pentru x = 2, y = 4, z = 15.
VI.43. Fie {a1 , a2 , . . . , an } Z pentru care k {1, 2, . . . , n}, i, j {1, 2, . . . , n},
i 6= j, astfel nct ak = ai + aj . Sa se arate ca n 6.
Petru Asaftei, Iasi
Solutie. Putem presupune c
a a1 < a2 < < an . Considernd num
arul an ,
exista i, j {1, 2, . . . , n}, i 6= j, astfel ca an = ai + aj . Daca ai < 0, aj < 0, atunci
an < ai , imposibil. La fel, ai < 0, aj > 0 implica an < aj , imposibil. Deci ai si aj
sunt pozitive si atunci multimea contine cel putin trei numere pozitive. Considernd
num
arul a1 deducem c
a multimea contine cel putin trei numere negative, prin urmare
n 6.
Un exemplu de multime cu 6 elemente este {3, 2, 1, 1, 2, 3} iar una cu n 6
elemente este {3, 2, 1, 1, 2, 3, . . . , n 3}.
VI.44. Fie ABCD un paralelogram si M AB, N AD triunghiuri echilaterale construite n exteriorul acestuia. Demonstrati ca [M N ] [BD] daca si numai daca
N D k M B.
Ciprian Baghiu, Iasi
Solutie. Presupunem [M N ] [BD]. Din conD
C
gruenta triunghiurilor BAD si M AN deducem ca
360 120
\ = m(M
\
m(BAD)
AN ) =
= 120 , deci N
2
B
\ ) = 180 , adic
A
a punctele B, A, N sunt colim(BAN
\
\
niare. Cum M
BN DN
B, rezulta M B k N D.
\
Presupunem M B k N D. Ca urmare, m(ABD)+

\
\
+m(ADB) = 180 120 = 60 , deci m(BAD) =
\
= 120 ; totodata m(M
AN ) = 120 si atunci triM
unghiurile M AN si BAD sunt congruente, de unde rezult
a c
a [BD] [M N ].
VI.45. Fie E, F picioarele naltimilor din B si C ale triunghiului ascutitunghic ABC. Daca P , N sunt mijloacele laturilor [AB], respectiv
[AC], iar {Q} =


b
\
= P E F N , sa se arate ca m(P QF ) = 180 3 m(A). (n legatura cu Q1086
din Parabola, nr. 3/2000 )
Titu Zvonaru, Bucuresti
133

Solutie. Fie Q exterior triunghiului ABC. Din


A
4AEB dreptunghic n E, cu (EP ) mediana, deducem
Q
[ P[
E
c
a triunghiul AEP este isoscel cu AEP
AE; deci
P
b La fel, din triunghiul dreptunghic
\
m(QP
F ) = 2m(A).
N
F
\
\
AF C cu mediana (F N ) deducem N
FA F
AN . Din
b S
\
4P QF rezult
a c
a m(P
QF ) = 180 3m(A).
a remarC
B
b
c
am c
a punctul Q este exterior dac
a m(A) < 60 . Cazul
b > 60 , se
n care Q este interior triunghiului ABC, ce corespunde situatiei m(A)

b = 60 , dreptele P E si F N sunt
trateaz
a folosind aceleasi argumente. Dac
a m(A)

\
paralele si putem considera ca m(P QF ) = 0 .

Clasa a VII-a

a
b
+
= 1.
b+1 a+1
Alexandru Negrescu, elev, Botosani
Not
a. Mai multe solutii ale acestei probleme sunt date n articolul prezent n
acest num
ar la p. 109.

VII.42. Sa se arate ca a2 + 1 b2 + 1 c2 + 1 (|a| + |b|) (|b| + |c|) (|c| + |a|),


a, b, c R.
Dorin M
arghidanu, Corabia
Solutie.
Notnd
|a|
=
x,
|b|
=
y,
|c|
=
z,
x,
y,
z

R
,
inegalit
atea este echiva+

lent
a cu x2 + 1 y 2 + 1 z 2 + 1 (x + y) (y + z) (z + x), x, y, z R+ .Avem
p
2

2
x + 1 y 2 + 1 = x2 y 2 +x2 +y 2 +1 x2 +y 2 +2xy = (x + y) , deci (x2 + 1) (y 2 + 1)
p
p
x + y. La fel, au loc:
(x2 + 1) (z 2 + 1) x + z si (y 2 + 1) (z 2 + 1) y + z.
Prin nmultirea ultimelor trei relatii se obtine inegalitatea dorit
a.
Egalitate se obtine pentru xy = 1, yz = 1, zx = 1, deci pentru x = y = z = 1 sau
|a| = |b| = |c| = 1.
VII.41. Rezolvati n N2 ecuatia

VII.43. Pentru n N, notam cu s (n) numarul de reprezentari distincte ale


lui n ca suma de doua numere naturale (n = a + b si n = b + a constituie aceeasi
reprezentare). Sa se arate ca:
n
hni hn + 1i
X
1
a) s (m + n) = s (m) + s (n) [1 + (1)mn ]; b)
s (k) =

.
2
2
2
k=0
Petru Minut, Iasi
hni
Solutie. Ar
at
am c
a s (n) =
+ 1. ntr-adev
ar, pentru n par, n = 2k, avem:
2
hni
+ 1. Pentru
n = 0 + n = 1 + (n 1) = . . . = k + k, deci s (n) = k + 1 =
2
n impar, n = 2k
h n+i 1, avem: n = 0 + n = 1 + (n 1) = . . . = k + (k + 1), deci
s (n) = k + 1 =
+ 1.
2
a) Relatia se demonstreaza analiznd paritatea numerelor m si n.
b) Daca n = 2k + 1, avem: s (0) + s (1) + . . . + s (n) = 1 + 1 + 2 + 2 + . . . +
hni hn + 1i
+k + k = k (k + 1) =

. Dac
a n = 2k, avem s (0) + s (1) + . . . + s (n) =
2
2
hni hn + 1i
= 2 (1 + 2 + . . . + (k 1)) + k = k2 =

.
2
2
134

VII.44. Fie [AB] diametru al cercului C de centru O; N, M C astfel nct


\
\
m(AON ) = 36 , iar [OM este bisectoare pentru N
OB. Daca T este simetricul lui
O fata de M N , sa se arate ca proiectia lui T pe AB este mijlocul lui [AO].
Valentina Blendea, Iasi
\
Solutie. n triunghiul T N O isoscel, m(T
N O) = 180
T
1 \
\
2m(N
OT ) = 180 m(N
OB) = 108 . n triunghiul
2
M
1

\
isoscel OAN , m(AN O) =
(180 36 ) = 72 , deci
N
2

\
\
m(T
N O) + m(AN
O) = 180 , prin urmare T , N , A sunt
coliniare. Cum m(T[
AO) = m(T[
OA) = 72 , triunghiul T AO
O
B
A
este isoscel si concluzia este imediat
a.
VII.45. Fie 4ABC echilateral, iar P (BC). Notam cu D, E simetricele lui P
fata de AC, respectiv AB. Sa se arate ca dreptele AP , BD si CE sunt concurente.
Constantin Cocea si Julieta Grigoras, Iasi
Solutie. Fie {M } = BD AC, {N } = CE AB;
A
AB = a, BP = x, P C = a x. Din asem
anarea triCM
CD
ax
D
unghiurilor DM C si BM A g
asim
=
=
,
MA
AB
a
M
iar din asemanarea triunghiurilor BEN si ACN gasim E N
BN
x
BE
=
= .
AC
AN
a
B
P
C
BP CM AN
n triunghiul ABC avem

= 1; conform reciprocei teoremei lui


P C MA NB
Ceva, dreptele AP , BM , CN sunt concurente.

Clasa a VIII-a
VIII.41. Fie f1 , f2 , f3 functii liniare ale caror grafice sunt drepte concurente
doua cte doua. Cele trei drepte sunt concurente daca si numai daca exista unic
R si exista u 6= v R astfel ca
f1 (u)
f2 (u)
f3 (u)
=
=
,
f1 (v)
f2 (v)
f3 (v)

cu fi (v) 6= , i {1, 2, 3} .

Claudiu-
Stefan Popa, Iasi
Solutie. Fie fi : R R, fi (x) = ai x + bi , i = 1, 3. Graficele functiilor sunt
concurente doua cte doua daca si numai daca a1 , a2 , a3 sunt distincte.
b2
b1
=
=
Prin proportii derivate relatia din enunt este echivalent
a cu
a1
a2
b3
=
, cu conventia c
a, dac
a unul dintre numerele ai este zero, atunci si bi
a3
este zero.
Graficele functiilor fi sunt concurente dac
a si numai dac
a exist
a (, ), -unic,
astfel nct fi () = , i = 1, 3, adic
a a1 + b1 = a2 + b2 = a3 + b3 = sau nc
a
b2
b3
b1
=
=
, ceea ce ncheie rezolvarea problemei.
=
a1
a2
a3
VIII.42. Fie x, y, z (0, ). Sa se arate ca
135

yz + zx
xy + xz
p
p
+
+

yz + zx + x2 + y 2 + z 2 yz zx
xy + xz + x2 + y 2 + z 2 xy xz

xy + yz
p
+
2.

xy + yz + x2 + y 2 + z 2 xy yz

Lucian Tutescu, Craiova


Solutie. Avem x2 +
y 2 + z 2
xy yz zx 0 deci x2+ y 2 +z 2 yz zx xy
yz + zx
yz + zx
p

si atunci

. Sumnd

2
2
2
yz + zx + xy
yz + zx + x + y + z yz zx
aceasta cu nc
a dou
a inegalit
ati asem
an
atoare, obtinem concluzia problemei.

VIII.43. Daca un triunghi dreptunghic are laturile numere naturale, iar suma
catetelor este patrat perfect, atunci suma cuburilor catetelor este suma de doua patrate perfecte.
Andrei Nedelcu, Iasi
Solutie. Fie (a, b, c) un triplet de numere naturale nenule care satisfac relatia
a2 + b2 = c2 . Urmeaz
a c
a toate trei sunt pare sau numai unul este par; deci
cb+a c+ba

,
N .
2
2

2
2
1
1
Avem a2 + b2 ab =
(c b + a) +
(c + b a) . Deoarece a + b = p2 si
2
2
hp
i2 h p
i2
2

3
3
2
3
a +b = (a + b) a + b ab , rezulta ca a +b3 = (c b + a) + (c + b a) .
2
2
VIII.44. Pe laturile [AB], [CD], [BC], [AD], [AC] si [BD] ale tetraedruBP
AQ
lui ABCD se iau respectiv punctele M , N , P , Q, R, S astfel ca
=
,
BC
AD
DN AR
DS
AM
=
,
=
. Notam cu V1 , V2 , V3 , V4 , V respectiv volumele tetraedrelor
AB
DC AC
BD
AM RQ, BP M S, CP N R, DN QS si ABCD. Sa se arate ca 212 V1 V2 V3 V4 V 4 .
Viorel Cornea si Dan S
tefan Marinescu, Hunedoara
V1
V2
Solutie. Notnd cele trei rapoarte cu , , , avem:
= ,
= (1 )
V
V
V3
V4
(1 ),
= (1 ) (1 ) (1 ),
= (1 ) . Atunci
V
V
2 2 2
1
1
1
1
V1 V2 V3 V4
2 2
2 2
2
2
=

(1

(1

(1

= 12 ,
4
V
4
4
4
2

1
, x [0, 1].
4
VIII.45. Fie A1 , A2 , . . . , Ak puncte pe un cerc C. Sa se determine o conditie
necesara si suficienta pentru a putea nscrie n C un poligon regulat ce admite punctele
date ca vrfuri (nu neaparat consecutive).
Irina Mustata
a, Iasi
, elev
Solutie. Presupunem c
a P este un poligon regulat cu n laturi ce are punctele
A1 , A2 , . . . , Ak ca vrfuri. Dac
a ntre Ai si Ai+1 sunt mi laturi ale lui P , cum
2
unghiul la centru dintre dou
a vrfuri consecutive ale lui P este
, deducem c
a
n
unde am folosit inegalitatea x (1 x)

136

2
m(Ai\
OAi+1 )
m(Ai\
OAi+1 ) = mi
, deci
Q, i = 1, k (cu conventia Ak+1 = A1 ).
n
2
m(Ai\
OAi+1 ) mi
S
a ar
at
am c
a aceast
a conditie este si suficient
a. Fie
, (mi , ni ) = 1,
=
2
ni
a c
a
i = 1, k. Alegnd n multiplu comun al numerelor n1 , n2 , . . . , nk rezult
A1 B2 . . . Bn este poligon regulat care contine vrfurile A1 , A2 , . . . , Ak (deoarece
m1
nm1 2
nm1
si
2 =

N, deci A2 P etc.).
n aceast
a situatie m(A\
1 OA2 ) =
n1
n1
n
n1

Clasa a IX-a
IX.41. Pentru n N, n 10, notam cu u2 (n) numarul format din ultimele
doua cifre ale lui n. Sa se arate ca:
a) u2 a20k+p = u2 (ap ), p {4, 5, . . . , 23}, k N, a {2, 3, 8};
b) u2 a10k+p = u2 (ap ), p {2, 3, . . . , 11}, k N, a {4, 9};
n
c) u2 (5
) = 25, n N;
5k+p
d) u2 6
= u2 (6p ), p {2, 3, . . . , 6}, k N;

e) u2 74k+p = u2 (7p ), p {2, 3, 4, 5}, k N.


Ovidiu Pop, Satu Mare
Solutie. n rezolvarea problemei vom folosi faptul c
a u2 (a) = u2 (b) dac
a si
numai daca a b se divide cu 100.
2k
a) 220k+p = 210
2p = 10242k 2p = (1025 1)2k 2p = (M25 + 1) 2p =
p
= M100 + 2 ;
k
k
320k+p = 910k 3p = 815 3p = 01 3p = (M100 + 1) 3p = M100 + 3p ;
820k+p = 260k+3p = 10246k 23p = (M25 1)6k 8p = (M25 + 1)8p = M100+8p .
Egalit
atile b) e) se demonstreaz
a analog.
1 1 1
IX.42. Fie a, b, c R astfel nct a + b + c = + + . Sa se afle numerele
a
b
c
a, b, c, daca |abc| = 1. (enunt scurtat)
Marius Pachitariu, elev, Iasi
am c
a
Solutie. Ecuatia |abc| = 1 este echivalent
a cu a2 b2 c2 = 1. Observ
(1 ab) (1 bc) (1 ca) = 1 ab bc ca + abc (a + b + c) a2 b2 c2 = 1 a2 b2 c2

(s-a tinut seama de relatia din enunt). Ca urmare, suntem condusi la ecuatia
(1 ab) (1 bc) (1 ca) = 0. Dac
a ab = 1, g
asim c = 1 si, deci, tripletele
(x, 1/x, 1), x R , sunt solutii ale problemei. Celelalte solutii se obtin din acestea
prin permutari circulare.
IX.43. Fie functia f : R R, iar a (1, ). Stiind
ca


1
2
2 1
f x + ax a f x f x + 1, x (, 0) ,

sa se arate ca f nu este injectiva.

Titu Zvonaru, Bucuresti


1
Solutie. Ecuatia x + ax a = , x (, 0) este echivalent
a cu (x 1)
x

2
2
x + ax + x + 1 = 0 sau cu x + x (a + 1) + 1 = 0, x (, 0). Cum > 0,
S < 0, P > 0 ea are dou
a r
ad
acini reale, distincte,
negative

x1 ,x2 .
1
2
Pentru x = x1 , relatia din enunt devine f x1 f x11 f x11 + 1, deci
2

137



2

f x11 1 0, de unde deducem f x11 = 1. La fel f x12 = 1. Prin urmare


f x11 = f x12 , x1 6= x2 ; deci f nu este injectiv
a.

IX.44. Daca 4ABC


este ascutitunghic,
sa se gaseasca maximul expresiei
E = sin A cos A + sin B cos B + sin C cos C.
Cezar Lupu, elev, si Tudorel Lupu, Constan
ta

Solu
t
ie.
Putem
presupune
A

C,
deci
sin
A

sin
B

sin
C
s
i
cos
A

cos B cos C. Folosind inegalitatea lui Cebsev, vom avea

sin A + sin B + sin C


E
cos A + cos B + cos C .

3 3
Dar sin A+sin B+sin C
, cos A+ cos B+ cos C 3(cos A+cos B +cos C)
2
r

3
3 6
3 6
3
. Cum pentru A = B = C = 60 obtinem E =
,
3 = . Deci E
2
4
4
2
3 6
.
deducem c
a maximul lui E este
4
IX.45. Demonstrati ca 4ABC n care are loc egalitatea
X
ha hb mc
= 1,
ma mb mc + ha hb mc + ma mb ic
suma fiind obtinuta prin permutari circulare, iar notatiile fiind cele uzuale, este
echilateral.
Iuliana Georgescu si Paul Georgescu, Iasi
Solutie. Avem ha ia ma si analoagele; deci suma din enunt este mai mica
X
ha hb mc
ha
hb
. Notnd x =
, y =
,
sau egal
a cu S 0 =
ma mb mc + ha hb mc + ma mb hc
ma
mb
X
hc
xy
z=
, avem 0 x, y, z 1 si S 0 =
.
mc
1 + xy + z
X
xy
Cum 1 + xy x + y pentru x, y [0, 1] si analoagele, avem S 0

x+y+z
X
x
= 1; egalitate are loc daca si numai daca x = y = z = 1, deci

x+y+z
ha = ma , hb = mb , hc = mc , adica daca si numai daca triunghiul este echilateral.

Clasa a X-a


2
F2n
2n
X.41. Prove the inequality

, where the Fibonacci numbers Fn


n
Fn1 Fn
are defined by F0 = F1 = 1, Fn+1 = Fn + Fn1 , n 1.
Zdravko Starc, Vrac, Serbia and Montenegro
Solutie. Conform inegalit
atii Cauchy-Schwarz, avem
"
2 #

2

2
2

n
n 2
n
n
n
n
F0 + F12 + + Fn2
+
+ +
F1 + +
F
.
F0 +
1
n
1
n n
0
0
n
2

n
n
P
P 2
P
2n
n
n
=
,
Fk = Fn Fn+1 ,
Fk = F2n (v. RecMat Deoarece
n
k
k=0
k=0 k
k=0
2n
2
2/2002, p. 70), obtinem
Fn Fn+1 F2n
. Egalitate apare pentru n = 1.
n
138

X.42. Sa se rezolve ecuatia 2[x] + 6[x] + 7[x] = 3[x] + 4[x] + 8[x] .


Daniel Jinga, Pitesti
Solutie. Notam [x] = y, y Z. Avem de rezolvat n Z ecuatia 2y + 6y + 7y =
= 3y + 4y + 8y . Dac
a y < 0 avem 7y > 8y si 2y + 6y > 3y + 4y (deoarece aceasta este
echivalent
a cu (2y 1) (3y 2y ) > 0, y < 0). Prin urmare nu avem solutii cu y < 0.
Se verifica usor faptul ca y = 0, y = 1, y = 2 sunt solutii.
Pentru n 3, n N, demonstram ca 3n + 4n + 8n > 2n + 6n + 7n . Este suficient
s
a ar
at
am prin inductie c
a 8n + 4n > 6n + 7n , n 3, care este un exercitiu de rutin
a.
Prin urmare y {0, 1, 2} si atunci x [0, 1) [1, 2) [2, 3). Multimea solutiilor
ecuatiei este intervalul [0, 3).
X.43. Fie f o functie reala nenula cu proprietatea ca
f (x + y xy) = f (x + y) f (x) f (y) ,

2003
Sa se calculeze f
.
2002

x, y R.

Adrian Zanoschi, Iasi


Solutie. Rezolvam problema n mai multe etape.
1. Pentru x = y = 0, din relatia dat
a g
asim f (0) = 0.
2. Pentru y = x = 1, deducem f (1) + f (1) f (1) = 0, adic
a f (1) = 0, sau
f (1) = 1. Dac
a f (1) = 0, pentru y = 1 relatia dat
a implic
a f (x + 1) = 0,
x R, adica f = 0, contrar ipotezei.
3. Ne ocup
am de cazul f (1) = 1 si f (1) = a 6= 0. Pentru y = 1, g
asim
f (x + 1) = af (x) + a,

x R.

(1)

Pentru y = 1 si x x + 1, obtinem f (2x + 1) = f (x + 1) + f (x) sau nc


a
af (2x) + a = af (x) + a + f (x), deci
a+1
f (2x) =
f (x) , x R.
(2)
a
Din (1) si (2) pentru x = 1, deducem f (2) = a2 + a = a + 1, de unde a = 1.
Daca a = 1, relatia (2) implica f (2x) = 0, x R, fals. Prin urmare a = f (1) = 1.
4. Din (1), gasim f (x + 1) = f (x) + 1, x R. Prin inductie, gasim f (x + k) =
= f (x) + k, x R si k Z. Pentru x = 0 deducem c
a f (k) = k, k Z.

p
p
p
5. n relatia dat
a nlocuim x cu si y cu q (p, q Z, q 6= 0) si g
asim f
= ,
q
q
q

2003
2003
adic
a f (x) = x, x Q. Prin urmare, f
=
.
2002
2002
X.44. Urnele U1 , U2 , . . . , Un contin fiecare cte a bile albe si b bile negre. Din
fiecare urna se extrage cte o bila care se depune ntr-o alta urna U . Din urna U
se scoate o bila si se constata ca este alba. Care este compozitia cea mai probabila a
urnei U ?
Petru Minut, Iasi
Solutie. Notam cu Ek evenimentul constnd n faptul ca n U ar fi depuse k bile

k
nk
a
b
k
albe (si n k bile negre). Avem P (Ek ) = Cn
, k = 0, n. Fie
a+b
a+b
E evenimentul ca bila extrasa din U sa fie alba. Conform formulei lui Bayes
139

P (Ek ) PEk (E)


kCnk ak bnk
kC k ak bnk
PE (Ek ) = Pn
.
= Pn n i i ni =
na (a + b)n1
i=1 P (Ei ) PEi (E)
i=1 iCn a b
Aceasta probabilitate este maxima cnd f (k) = kCnk ak bnk , k = 0, n este maxima.
f (k + 1)
na
f (k + 1)
na
si
Observam ca
>1k <
<1k>
. Daca k0
f (k)
a+b
f (k)
a+b
este punctul de maxim pentru f (k), avem
f (k0 + 1) f (k0 ) si f (k0 ) f (k0 1),
na
na
na
na
din care deducem k0
,
+ 1 . Prin urmare, k0 =
, dac
a
N
a+b a+b
a+b
a+b

na
na
si k0 =
+ 1, dac
a

/ N.
a+b
a+b
X.45. Se considera triunghiul ascutitunghic ABC. Sa se arate ca exista un
0B0) =
\
triunghi A0 B 0 C 0 astfel nct A0 (BC), B 0 (AC), C 0 (AB), iar m(AC
0 C 0 ) = m(CB
0 A0 ) = (0, 90]. Daca
\
\
= m(BA
n plus 4ABC este echilateral, sa se
calculeze lungimile laturilor 4A0 B 0 C 0 n functie de a = BC si . (n legatura cu o
problema propusa la O. N. M., 2002)
Dan Popescu, Suceava
Solutie. Fie D proiectia punctului C pe AB, deci D (AB). Fie C 00 (DA) si
00 A) =
00 A00 B) = . De asemenea,
\
E (AC) nct m(EC
si A00 (BC) nct m(C\
00 F C ) = .
fie A1 si C1 , A (BA1 ), C (BC1 ) si F (A1 C1 ) cu A1 C1 k AC si m(A\
1
n aceste conditii, (C 00 E (A00 F = {B 00 }, (BB 00 (AC) = {B 0 }, iar omotetia de

a A00 n A0 (BC)
centru B si raport , raport definit de BB 0 = BB 00 , transform
00
0
0
0
0
si C n C (AB). Prin urmare A B C satisface enuntul.
n cazul n care triunghiul ABC este echilateral se deduce relativ usor ca 4A0 B 0 C 0
este echilateral si (AB 0 ) (CA0 ) (BC 0 ). Cu teorema sinusurilor n 4AB 0 C 0
a

deducem B 0 C 0 =
.
cos + 3 sin

Clasa a XI-a
XI.41. Fie A1 , A2 , . . . , Ak Mn (Z) astfel nct

A(1) A(2) . . . A(k) = In ,


.
unde Sk este multimea permutarilor de ordin k. Sa se arate ca n .. k!.
Vladimir Martinusi, Iasi
Solu
t
ie.
Deoarece
tr
(AB)
=
tr
(BA),
A,
B

M
ca
n (C), inductiv se arata

tr A(1) A(2) A(k) = tr (A1 A2 Ak ), Sk . Tinnd


cont c
a tr (A + B) =
= tr A + tr B, A, B Mn (C) deducem c
a

!
!

P
P
tr
A(1) A(k) = tr (In ) tr
A(1) A(k) = n
Sk

Sk

Sk

Sk

tr A(1) A(k) = n k! tr (A1 A2 Ak ) = n

si cum tr (A1 A2 Ak ) Z, concluzia problemei este imediata.

XI.42. Prin punctele M1 si M2 ale unei elipse se duc normalele la elipsa, care
intersecteaza una din axele de simetrie ale acesteia n M10 , respectiv M20 . Sa se
arate ca mediatoarea segmentului [M1 M2 ] trece prin mijlocul lui [M10 M20 ]. Ramne
proprietatea adevarata pentru hiperbola sau pentru parabola?
Gheorghe Costovici, Iasi
140

x2
y2
+ 2 = 1, = 1 si Mi (xi , yi ) pe curba,
2
a
b
a2 yi
(x xi ), i = 1, 2, si
i = 1, 2. Normala n Mi la curba are ecuatia y yi = 2
b xi

2
a b2
0
xi , 0 , i = 1, 2.
intersecteaza una din axele simetrie, de exemplu Ox, n Mi
a2
2

a b2
Mijlocul M 0 al segmentului [M10 , M20 ] are coordonatele
(x
+
x
)
,
0
. Me1
2
2a2

y1 + y2
x1 + x2
x2 x1
diatoarea segmentului M1 M2 are ecuatia y
x
=
.
2
y2 y1
2
Se verifica usor ca M 0 este situat pe mediatoarea segmentului [M1 M2 ].
Prin calcul se verifica ca proprietatea ramne valabila si pentru parabola.
XI.43. Consideram sirul de functii fn : (0, ) R, fn (x) = nx + ln x (n 1)
si fie xn solutia unica a ecuatiei fn (x) = 0. Sa se calculeze limitele sirurilor (xn )n1
si ((xn )n )n1 .
aeru, Suceava
Angela Tig
atoare si surjective, ceea ce asigur
a
Solutie. Evident, functiile fn sunt strict cresc
existenta si unicitatea solutiei xn .
x1
Folosim n continuare inegalitatile
ln x x 1, x (0, ). Avem:
x

n
1
n
1
n
=
+ ln

+
1 = 0 si
fn
n+1
n+1
n+1
n+1 n+1

n+1
n+1
2
n+1
n+2
2
fn
= ln
+

1
+
> 0, n N .
n+2
n+2 n+2
n+2
n+1 n+2

n+1
n
n+1
n
si atunci
fn (xn ) fn
xn
,
Prin urmare, fn
n+1
n+2
n+1
n+2
1
n 1. G
asim lim xn = 1 si lim xnn = .
n
n
e
XI.44.Sa se determine
func
t
iile
continue
f : (0, ) (0, ) pentru care

f (x) = f
2x2 2x + 1 , x > 0.
Marian Urs
arescu, Roman
p
Solutie. Fie x0 (0, 1] fixat si sirul (xn )n0 definit prin xn+1 = 2x2n 2xn + 1,
n N. Se verifica faptul ca xn (0, 1], n N, (xn )n0 este crescator si lim xn = 1.
p
n
Pentru x = xn , relatia din enunt devine f (xn ) = f
2x2n 2xn + 1 = f (xn+1 ),
n N, deci f (x0 ) = f (xn ). Ca urmare, f (x0 ) = lim f (xn ). Rezult
a c
a f (x) =
Solutie. Fie curba de ecuatie

= f (1), x (0, 1].

p
2x2n 1
, n N. Se arata ca
2
xn > 1, n N. (xn )n0 este descrescator si lim xn = 1. Cum relatia de ren
q
curenta se poate scrie si n forma xn = 2x2n+1 2xn+1 + 1, obtinem f (xn+1 ) =
q

=f
2x2n+1 2xn+1 + 1 = f (xn ), apoi f (x0 ) = f (xn ) si f (x) = f (1), x > 1.
Fie x0 > 1 si sirul definit prin xn+1 =

1+

n concluzie, f (x) = f (1), x (0, ), deci f este functie constanta.


141

XI.45. Fie k N, k 2 si numerele


p reale pozitive a1 , a2 , . . . , ak , b1 , b2 , . . . , bk
cu a1 < a2 < < ak . Definim xn = n b1 an1 + b2 an2 + . . . + bk ank .
a) Sa se demonstreze ca lim xn = ak ;
n
b) Sa se arate ca lim n (xn ak ) = ak ln bk ;
n

n
ak
c) Daca bk = 1, are loc lim n
(xn ak ) = ak bk1 .
n
ak1
p

Marian Tetiva, Brlad


Solutie. a) Avem ak n bk n an1 b1 + + ank bk ak n b1 + b2 + + bk , n N.
p
Deducem ca lim n b1 an1 + b2 an2 + + bk ank = ak .
n

b)

n
ln xn
xn
e ak 1 xn
xn
1 = ak lim n
ln
=
a
lim
ln
=
lim n (xn ak ) = ak lim n
k
x
n
n
n
n
n
ak
ln ak
ak
ak
n
n

a1
a2
ak1
= ak lim ln b1
+ b2
+ + bk1
+ bk = ak ln bk .
n
ak
ak
ak
c)

n
n ln xan
ak
ak
e k 1 xn
lim n
(xn ak ) = lim ak n
ln
=
n
n
ak1
ak1
ln xank
ak

n n
n

n
ak
a1
a2
ak1
ln b1
+ b2
+ + bk1
+1 =
= ak lim
n ak1
ak
ak
ak

n n
n

n
ak
a1
a2
ak1
= ak lim
+ b2
+ + bk1
b1
=
n ak1
ak
ak
ak

n

a1
a2
= ak lim b1
+ b2
+ + bk1 = ak bk1 .
n
ak1
ak1

Clasa a XII-a
XII.41. Sa se calculeze

nN .

Z (1 + x) 1 + x2 . . . 1 + x2n1
x2n

dx, unde x [1, ),

Oana Marangoci,
student
a, Iasi


2
2n1
Solutie. Pentru x (1, ), avem (1 + x) 1 + x 1 + x
=
2

1 x2
1 x2
1 x2 1 x2
n

.
.
.

=
=

= 1 + x + x2 + + x2 1 si atunci
n1
1 x 1 x2
1
x
1 x2

n1
(1 + x) 1 + x2 1 + x2
1
1
1
= 2n + 2n 1 + + ,
n
2
x
x
x
x
relatie care se verific
a si pentru x = 1. Primitivele functiei sunt

1
1
1
+
+ +
+ ln x + C.
(2n 1) x2n 1 (2n 2) x2n 2
x

Z 5
4

5
f (x) sin 2x dx = .
R o functie continua pentru care
XII.42. Fie f : ,
4
4

142

Sa se arate ca exista c

5
,
astfel nct f (c) (1, 2).
4

Mihai Haivas, Iasi


Solutie. Avem
5
Z 5
Z 5
4
4
4
sin 2x
(cos 2x)0
= .
dx
=

dx
=

arctg
(cos
2x)
2

1
2
1
+
cos
2x
4
1

sin
2x

5
5
Z
Z
Z 5
4
4
4
2
sin 2x
dx
sin 2x dx = 0 .
f (x) sin 2x dx =
f (x)
2
2
1
1

sin
2x
2

sin
x

5
Aplicnd teorema de medie exista c ,
astfel nct
4

2
sin 2c = 0 f (c) =
.
f (c)
2
4
2 sin 2c
2 sin2 2c

2
5
Dar

(1,
2)
pentru
c

,
, adica f (c) (1, 2).
4
2 sin2 2c
XII.43. Sa se arate ca

Z 1
arctg ln a
ln a
x2

, a > 1.
a
dx
1

3
ln a
0
Petru R
aducanu, Iasi
Solutie. Se arat
a c
a ax x ln a + 1, x R si a > 1. Obtinem astfel
2
1
1
x2 ln a + 1 ax = x2 2
, x R, a > 1.
x ln a + 1
a
Integrnd ntre 0 si 1, obtinem inegalitatile cerute.
XII.44. Sa se afle numarul radacinilor reale ale polinomului P Z [X] de grad
minim, care admite radacina 2 + , unde verifica ecuatia x3 x + 1 = 0.
Laurentiu Modan, Bucuresti
Solutie. Polinomul n y care se cere, apare prin eliminarea lui x ntre ecuatiile
x3 x + 1 = 0 si y = x2 + x. Deoarece x2 = y x, g
asim x (y x) x + 1 = 0 si
y

1
(y 1)2 y 1
asim x =
+
(evident y 6= 0). Deducem y =
nlocuind din nou x2 g
y
y2
y
3
2
si, n final, P (y) = y 2y + 3y 1. Folosind sirul lui Rolle, polinomul P (y) admite
o singur
a r
ad
acin
a real
a y (0, 1).
XII.45. Fie S5 . Sa se arate ca 2 are puncte fixe daca si numai daca 3
are puncte fixe.
Paul Georgescu si Gabriel Popa, Iasi
Solutie. Folosind descompunerea unei permutari n produs de cicli disjuncti,
se poate observa ca, daca k , 1 k 5, are puncte fixe, atunci contine n
descompunerea sa cicli de lungime l, cu l | k.
Presupunem c
a 2 are puncte fixe. Atunci contine n descompunerea sa cicli de
lungime 1 (puncte fixe) sau 2. Daca are puncte fixe, 3 are de asemenea puncte
fixe. Daca contine n descompunerea sa cicli de lungime 2, fara a avea puncte fixe,
atunci contine n descompunerea sa si un ciclu de lungime 3. Elementele acestui
ciclu vor fi toate puncte fixe pentru 3 . Implicatia reciproc
a se demonstreaz
a analog.
143

Solutiile problemelor pentru preg


atirea concursurilor
din nr. 2 / 2003
A. Nivel gimnazial
G46. Determinati ultimele cinci cifre ale numarului
A = 72000 + 72001 + 72002 + 72003 .
Viorel Cornea si Dan S
tefan Marinescu, Hunedoara
Solutia I. Scriem A = 72000 400. Se constant
a c
a 720 = M1000 + 1. Ca urmare,
2000
7
= M1000 + 1 si A = (M1000 + 1) 400 = M100000 + 400, deci ultimele cinci
cifre ale lui A sunt 00400.
Solutia II (Irina Mustata
, eleva, Iasi). Conform teoremei lui Euler, a(n)
1(mod n) pentru (a, n) = 1. Considernd a = 7, n = 1000, avem (1000) = 400, deci

5
7400 1(mod 1000), adic
a 72000 = 7400 1(mod 1000), de unde A = 72000 400
400(mod 100000), deci num
arul A se termin
a cu 00400.
G47. Determinati valorile parametrilor a, b Z pentru care solutiile sistemului
y
x
x=a
; y=b
y+1
x+1
sunt n Z Z.
Temistocle Brsan, Iasi
Solutie. Fie M multimea perechilor (a, b) Z Z pentru care este ndeplinit
a
cerinta problemei. Observ
am c
a (0, 0) Z Z este solutie a sistemului si c
a aceasta
este unica solutie daca a = 0 sau b = 0. Ca urmare, (a, 0) M , a Z si (0, b) M ,
b Z.
Pe (R \ {1}) (R \ {1}) sistemul dat, dac
a facem abstractie de solutia (0, 0),
este echivalent cu
(b + 1) x = ab 1, (a + 1) y = ab 1.
(1)
ab 1
ab 1
I. a 6= 1, b 6= 1. Obtinem x =
,y=
. Atunci (x, y) Z Z
b+1
a+1
b + 1 | ab 1 si a + 1 | ab 1. Cum ab 1 = (a + 1) (b + 1) (a + 1) (b + 1),
deducem c
a (x, y) Z Z b + 1 | a + 1 si a + 1 | b + 1 b + 1 = a + 1 sau
b + 1 = (a + 1). Prin urmare (a, a) M , a Z, a 6= 1 si (a, a 2) M ,
Z, a 6= 1.
II. a = b = 1. Cum rezult
a ab 1 = 0, sistemul (1) are o infinitate de solutii si
nu toate sunt n Z Z. Deci (1, 1)
/ M.
III. a = 1, b 6= 1 (analog a 6= 1, b = 1). Avem ab 1 6= 0, deci (1) nu
are solutii, iar sistemul dat are (0, 0) ca unic
a solutie. Asadar, (a, 1) M , a Z,
a 6= 1 si (1, b) Z, b Z, b 6= 1.
n concluzie:
M = {(a, 0) , a Z} {(0, b) , b Z} {(a, 1) , a Z} {(1, b) , b Z}
{(a, a) , a Z} {(a, a 2) , a Z} \ {(1, 1)} .
2002
G48. Fie A (0, ) o multime care contine
si avnd proprietatea ca,
2003
a
a+1
a
daca A (a, b N ), atunci
A si
A. Sa se arate ca A Q+ .
b
b
2b
Gheorghe Iurea, Iasi
144

Solutie. Consider
am transform
arile
a+1
a
A
A
b
b

(1)

si
a
a
A
A.
b
2b

(2)

Avem:
2002
(2) 1001
(1) 1002
(2) 501
(1) 502
(2) 251
(1)
(2) 1
A
A
A
A
A
A
A.
2003
2003
2003
2003
2003
2003
2003
2
1
1
(1)
(1)
(1) 2003
A
A
A, deci
A.
2003
2003
2003
1
1
a
b
1
a
a
Fie
Q+ . Cum A A A A, rezulta ca A, adica
b
1
b
b
b
b
Q+ A.

G49. Fie x1 , x2 , . . . , xn+1 R+ astfel nct x1 + x2 + . . . + xn+1 (n + 2) m,


n+4 2
iar x21 + x22 + . . . + x2n+1
M , unde m = min xi , M = max xi . Sa se arate
4
ca exact n dintre numerele date sunt egale.
Eugen Jecan, Dej
Solutie. Daca toate numerele ar fi egale, relatia a doua din enunt ar fi
n+4 2
(n + 1) M 2
M , imposibil. Exista cel putin un numar egal cu M , fie acesta
4
xn+1 . Deducem ca (n + 2) m x1 + + xn + M deci 2m M . De asemenea,
n+4 2
a M = 2m si
M x21 + + x2n + M 2 nm2 + M 2 sau M 2m. Rezult
4
prima relatie devine x1 + x2 + +xn +2m (n + 2) m, deci x1 +x2 + + xn nm
sau nca (x1 m)+ +(xn m) 0. Cum x1 m 0, x2 m 0, . . . , xn m 0,
deducem ca x1 = x2 = = xn = m, valori care verifica si a doua conditie.
n concluzie exact n numere sunt egale cu m si unul egal cu M = 2m.
G50. Fie a N, a 3. Sa se arate ca

an + 1 =
an + 2 = . . . =
an + a 1 , n N a {3, 4} .

Pop, Satu
Mare

Ovidiu
1 =
2 = =
a1 ,
Solutie. Pentru n = 0 relatia din enunt devine

a 1 = 1, echivalent cu 2 a < 5, si cum a N, a 3, avem a {3, 4}.


deci
Reciproc, arat
am ca, dac
a {3, 4},
a
au loc egalitatile din enunt. Fie a = 3;
trebuie ar
atat c
a
3n + 1 =
3n + 2 , n N.
Pentru un num
ar n N exist
a k N (k depinde de n) astfel nct k2 3n + 1 <
2
< (k + 1) . Considernd numarul 3n+2 avem 3n+2 < (k + 1)2 sau 3n+2 = (k + 1)2 .
2
Cum 3n + 2 nu este
atrat perfect,
deducem c
a k2 3n + 1 <
+ 2 < (k+ 1) , de
3n
p

unde rezulta k 3n + 1 < 3n + 2 < k + 1, deci


3n + 1 =
3n + 2 .

4n + 1 =
4n + 2 =
4n + 3 ,
Pentru a = 4, trebuie demonstrat c
a

2
n N. Considernd k =
4n + 1 avem k 2 4n + 1 < (k + 1) si cum 4n + 2,
2
4n+3 nu sunt p
atrate perfecte
pentru nici
un
n N, deducem
k 4n+1 < 4n+2 <
< 4n + 3 < (k + 1)2 si atunci
4n + 1 =
4n + 2 =
4n + 3 = k.
145

3
, cu a + b + c = 1. Sa se arate ca
10

3
2
a a + bc + b b + ca + c c + ab < .
3
4
Gabriel Dospinescu, elev, Onesti
p

a+b+a+c
1+a
si atunci
=
Solutia. Avem a + bc = (a + b) (a + c)
2
2

2
Xa +a
X
X
6
1
2
=
1+
a2 . Cum a = 1 (b + c) < 1
=
a a + bc
2
2
10
5

X 1
1
4
1 37
3
etc., rezulta ca
1+
a2 <
1+3
=
< .
2
2
25
2 25
4

2
Pentru inegalitatea din stnga avem: a + bc = (a + b) (a + c) a + bc si
X
X
X
X

a a + bc = 1 2
ab +
a bc. R
amne s
a ar
at
am
atunci
a a + bc >
2

X (a b)
X
X
X
2
1
a b . Este
c
a 2
ab+
a bc 0 sau, echivalent,

c
3
3
2

2
2
(a b)
a b , echivalenta cu
a + b 3c
suficient sa aratam ca
c
3

sau cu a + b + 2 ab 3 3a 3b.
Ultima
inegalitate se scrie 4a + 4b + 2 ab 3 si
3
este adevarata n conditia a, b
, .
10
G52. Se considera o piramida formata din patrate 1 1,
avnd n trepte, pe treapta k existnd 2k 1 patrate (n figura,
n = 4). Aflati numarul minim de dreptunghiuri, fiecare alcatuit
numai din casute ntregi, n care poate fi mpartita tabla.
Adrian Zahariuc, elev, Bac
au
Solutia I (a autorului). Color
am c
asutele tablei alternativ n alb si negru
(c
asuta din vrf este neagr
a). Cum modulul diferentei dintre num
arul de c
asute albe
si num
arul de c
asute negre dintr-un dreptunghi este cel mult 1 si cum sunt cu n
casute negre mai multe dect albe, trebuie sa avem cel putin n dreptunghiuri. Acest
num
ar poate fi obtinut t
aind pe nivele. Asadar, r
aspunsul este n.
Solutia II (Irina Mustata
a, Iasi). Unim, ca n figu, elev
ra, vrfurile din stnga sus ale patratelor marginale din stnga;
se observa ca deasupra acestei drepte nu mai exista vrfuri
ale piramidei. Oricare dou
a p
atrate marginale din stnga nu
pot apartine aceluiasi dreptunghi, deoarece ar nsemna c
a acel
dreptunghi va avea coltul din stnga sus deasupra dreptei considerate; prin urmare,
numarul minim de dreptunghiuri este n, minim atins pentru mpartirea pe trepte.
Solutia III (Marius Pachitariu, elev, Iasi). Prin inductie complet
a.
G53. FieABCD un patrat de latura 70.
S
a
se
arate
c
a
exist
a
o
multime de

patrate Pk = Ai Bi Ci Di | Ai Bi = i, i = 1, k care sa aiba suma ariilor egala cu aria


patratului dat. Putem acoperi patratul ABCD cu elementele multimii Pk ?
Petru Asaftei, Iasi
k(k+1)(2k+1)
Solutie. Trebuie s
a avem 12 +22 + +k2 = 702 , echivalent cu
=
6
G51. Fie a, b, c

146

= 702 , de unde k = 24. Solutia este unic


a deoarece k > 24 implic
a 12 +22 + +k2 >
> 702 iar k < 24, 12 + 22 + + k 2 < 702 .
Aratam ca multimea Pk nu poate acoperi patratul ABCD. Pentru aceasta, sa
observ
am c
a p
atratele Ai Bi Ci Di , pentru a acoperi Pk , nu au puncte comune, exceptie
f
acnd laturile (altfel aria acoperit
a de acestea este mai mic
a dect 12 +22 + +242 =
= 702 ); de asemenea, patratele Ai Bi Ci Di nu lasa "spatii goale" ntre ele (altfel aria
acoperita de ele plus ariile "spatiilor goale" este mai mare ca 12 +22 + +242 = 702 ).
Analiznd pozitia p
atratului de latur
a 1 n p
atratul ABCD, observ
am c
a r
amne
o suprafata ce nu mai poate fi acoperit
a (r
amne o suprafata dreptunghiular
a de
laturi 1 si l 1 n care nu "ncape" nici unul din patratele ramase, care au laturile
mai mari sau egale cu 2).
G54. Sa se arate ca nu putem alege nici un punct n interiorul triunghiului
echilateral ABC de latura l 10, care sa aiba distantele la vrfuri numere prime
distincte.
Doru Buzac, Iasi
Solutie.
Presupunem, prin absurd, c
a exist
a
A
M Int (ABC) astfel nct M A, M B, M C s
a fie numere prime.
Cum M A, M B, M C < l si M A, M B, M C pot fi
7
laturile unui triunghi (teorema lui Pompeiu), deducem
c
a AM, BM, CM {3, 5, 7}. Fie AM = 7, BM = 3,
M
3
5
CM = 5.

\
Fie N astfel nct BN = 3 si m(M
BN ) = 60 . Evi- B
C
dent, triunghiul BM N este echilateral. Din congruenta
3
7
triunghiurilor AM B si CN B deducem N C = 7. Atunci
N
M N 2 +M C 2 N C 2
1
\
\
\
C) = 180 ,
cos N
MC =
= , deci m(N
M C) = 120 si m(BM
2 MN MC
2
fals. Prin urmare, nu exista M Int (ABC) cu proprietatea ceruta.
G55. Printr-un punct situat n interiorul unui tetraedru se duc planele paralele cu
fetele tetraedrului. Daca V1 , V2 , V3 , V4 sunt volumele tetraedrelor unic determinate
de aceste plane, iar V este volumul tetraedrului dat, sa se arate ca
27V 16 (V1 + V2 + V3 + V4 ) .
Neculai Roman, Mircesti (Iasi)
Solutie. Not
am cu xi distanta de la punctul considerat la fata tetraedrului
A1 A2 A3 A4 opusa vrfului Ai si cu hi naltimea corespunzatoare acestei fete. Fie
4
p
P
hi xi
xi
ai = 3 Vi /V , i = 1, 4. Avem ai =
= 1 , i = 1, 4, deci
ai =
hi
hi
i=1
4 x
4 S x
4
P
P
1 P
i
i i
=4
=4
Vi = 4 1 = 3.
=4
V i=1
i=1 hi
i=1 Si hi

Relatia de demonstrat se scrie 27 16 a31 + a32 + a33 + a34 si decurge din faptul

3
a3 + a32 + a33 + a34
33
27
a1 + a2 + a3 + a4
c
a 1
= 3 =

.
4
4
4
64

B. Nivel liceal

b si B
b se
L46. Fie ABCD un patrulater inscriptibil. Bisectoarele unghiurilor A
147

intersecteaza ntr-un punct situat pe latura [CD]. Sa se arate ca CD = AD + BC.


Mircea Becheanu, Bucuresti
b = 2,
Solutia I (Irina Mustata
,
elev
a
,
Ia
s
i).
Fie
m(
A)

D
E P
b
m(B) = 2, iar P (CD) astfel nct AD = DP ; avem ca

C
1
b = . Deosebim dou
\
a cazuri,
180 m(D)
m(AP
D) =
2
dup
a cum E (DP ) sau E (P C); ne plas
am n prima
[
\ patrulaterul ABP E este
situatie. Deoarece EP
A EBA,
B
\
\) = 180 A
inscriptibil, deci m(CP
B) = . Atunci m(CBP
b = , deci 4CP B este isoscel cu CP = CB, de unde concluzia.
m(C)
Solutia II. Folosind teorema sinusurilor n triunghiurile ADE, BEC si AEB,
obtinem:
B
A
sin
sin
2
2
AB, CE =
AB,
DE =
B
A
A+B
A+B
2 cos sin
2 cos sin
2
2
2
2

A
B
sin B
sin A
2
2
AD =
AB, BC =
AB,
B
A
A+B
A+B
2 cos sin
2 cos sin
2
2
2
2
din care deducem
sin A + sin B
CD = DE + CE = AD + BC =
AB.
A
B
A+B
4 cos cos sin
2
2
2
Not
a. S-a mai primit solutie corecta de la Marius Pachitariu, elev, Iasi.
L47. Daca un triunghi are patratele laturilor n progresie aritmetica, atunci
simetricul centrului de greutate fata de latura mijlocie se afla pe cercul circumscris
triunghiului.
Gabriel Popa si Paul Georgescu, Iasi
Solutie. Fie a, b, c lungimile laturilor triunghiului ABC, c < a < b cu 2a2 = b2 +c2 .
Simetricul cercului circumscris triunghiului ABC fata de BC este cercul circumscris triunghiului BHC si atunci simetricul lui G fata de BC este pe cercul circumscris daca si numai daca BHGC este patrulater inscriptibil, echivalent cu faptul ca
b
\ = m(BHC)
\ = m(A).
m(BGC)
2
\ = a (se foloseste relatia 2a2 = b2 +c2
n triunghiul BGC se determina cos BGC
2bc
a2
\ = cos A, deci
si formula medianei). Cum cos A =
, deducem c
a avem cos BGC
2bc
b = , ceea ce ncheie solutia.
\ + m(A)
m(BGC)
Not
a. Solutii corecte au dat Irina Mustata
si Marius Pachitariu, elevi, Iasi.
L48. Fie R, r, R1 raza cercului circumscris 4ABC, raza cercului nscris 4ABC,
respectiv raza cercului circumscris 4DEF determinat de picioarele bisectoarelor interioare ale 4ABC. Sa se arate ca R/2 R1 r.
Marian Tetiva, Brlad
Solutie (Titu Zvonaru, Com
anesti (Bac
au)). Inegalitatea r R1 este adevarata oricare ar fi punctele D (BC), E (CA), F (AB) (Liliana Niculescu 148

O metoda de demonstrare a unor inegalitati geometrice, GM - 2-3/1993; Teorema 1,


p. 51). ntr-adevar, fie Q centrul cercului circumscris 4DEF si d1 , d2 , d3 distantele
de la Q la laturile BC, CA, AB. Cum fiecare dintre dreptele BC, CA si AB este
secant
a sau tangent
a cercului circumscris 4DEF , avem d1 R1 , d2 R1 , d3 R1 ,
deci
ad1 bd2 cd3 aR1 bR1 cR1
pr = A[ABC] = A[QBC] +A[QCA] +A[QAB] =
+
+

+
+
= pR1 ,
2
2
2
2
2
2
adica r R1 .
Inegalitatea R1 R/2 este adev
arat
a, dac
a punctele D, E, F apartin segmentelor
determinate de picioarele n
altimilor si mijloacele laturilor respective (asa cum se
ntmpl
a cu picioarele bisectoarelor). n acest caz aceste puncte sunt n interiorul
cercului lui Euler al 4ABC, a carui raza este R/2, si rezulta R1 R/2.

L49. ntr-un patrat 10 10 se nscriu numerele 1, 2, 3, . . . , 100 n asa fel nct


oricare doua numere consecutive sa se afle n casute vecine. Demonstrati ca exista
o linie sau o coloana ce contine macar doua patrate perfecte.
Adrian Zahariuc, elev, Bac
au
Solutie. Observam ca avem 10 patrate perfecte dintre care 5 sunt pare. Presupunem ca patratele perfecte sunt situate pe linii si coloane diferite. Coloram tabla
ca pe o tabl
a de sah n alb si negru. Numerele pare vor fi situate pe c
asute de aceasi
culoare, la fel numerele impare.
Fie (x1 , y1 ), (x2 , y2 ), . . . , (x10 , y10 ) coordonatele casutelor n care sunt situate
p
atratele perfecte. Datorit
a presupunerii c
a p
atratele perfecte sunt pe linii si coloane
diferite, numerele xi , i = 1, 10 snt diferite dou
a cte dou
a si {x1 , x2 , . . . , x10 } =
= {1, 2, . . . , 10}; la fel pentru numerele yi , i = 1, 10. Obtinem (x1 + y1 ) + (x2 + y2 ) +
+ + (x10 + y10 ) = 2 (1 + 2 + + 10) = numar par si rezulta ca un numar par de
perechi are suma par
a. Cum perechile cu suma combinatiilor par
a au aceasi culoare,
deducem c
a exist
a un num
ar par de p
atrate perfecte pare, absurd.
Not
a. Solutie corect
a a dat Marius Pachitariu, elev, Iasi.

L50. Fie (an )n1 o progresie aritmetica avnd a1 = 5, r = 2002. Pentru un


element b al progresiei, sa se arate ca bm apartine progresiei daca si numai daca
60 | m 1.
Mihai Piticari, C-lung Moldovenesc
Solutie. S
a demonstr
am mai nti urm
atoarea
Lem
a. Daca a, m, n sunt numere naturale nenule astfel nct n | am 1 si m
este cel mai mic numar cu aceasta proprietate, atunci n | ak 1 daca si numai daca
m | k.
ntr-adev
ar, dac
a m | k, atunci k = sm si din n | am 1 | asm 1 rezult
a
k
n | a 1. Presupunem n | ak 1. Conform teoremei mp
artirii cu rest, exist
a
c si r numere naturale astfel nct k = mc + r, r < m. Din n | ak 1, rezult
a
n | amc+r 1 = ar (amc 1) + ar 1 si deducem n | ar 1. Cum r < m, pe baza
minimalit
atii lui m rezult
a r = 0, deci m | k.

Revenim la problema data. Evident, bm apartine progresiei daca si numai daca


2002 | bm b. Cum b = 5+2002p, rezult
a c
a 2002 este prim cu b si atunci bm apartine
m1
progresiei daca si numai daca 2002 | b
1. Din bm1 1 = (5 + 2002p)m1
149

5m1 + 5m1 1, urmeaz


a c
a 2002 | bm1 1 dac
a si numai dac
a 2002 | 5m1 1.
Avem 2002 = 2 7 11 13 si 6, 10, 4 sunt minime cu proprietatile 7 | 56 1,
11 | 510 1, 13 | 54 1. Deci, pe baza lemei, 2002 | 5m1 1 daca si numai daca
6 | m 1, 10 | m 1, 4 | m 1, adic
a dac
a si numai dac
a 60 | m 1.
Not
a. Solutie corect
a a dat Marius Pachitariu, elev, Iasi.

L51. Fie A, B M2 (R) doua matrice care comuta si pentru care det A2 + B 2 <
< (det A + det B)2 . Sa se arate ca xA + yB este matrice nesingulara, x, y R .
C
at
alin Calistru, Iasi
Solutie. Se arat
a cu usurinta c
a
det (xA + yB) = x2 det A + y 2 det B + xy [det (A + B) det A det B] , x, y C,

prin urmare

det (A + iB) det (A iB) = (det A det B)2 + [det (A + B) det A det B]2 .

Cum AB = BA, avem (A + iB) (A iB) = A2 + B 2 si relatia anterioar


a devine

2
2
2
det A + B = (det A det B) + (det (A + B) det B det A)2 .

Conditia din enunt se rezum


a la a ar
ata c
a x, y R , avem det (xA + yB) 6= 0, deci
2
2
ecuatia x det A + y det B + xy [det (A + B) det A det B] = 0 nu admite solutii
reale nebanale. ntr-adev
ar, discriminantul acestei ecuatii este

= [det (A + B) det A det B]2 4 det A det B = det A2 + B 2

2
2
(det A det B) 4 det A det B = det A2 + B 2 (det A + det B) < 0.

L52. Fie Q C [X] un polinom de grad m avnd radacinile distincte. Sa se


determine cardinalul multimii
E = {P C [X] | A Mn (C) a. . Q (A) = On si P (X) = det (XIn A)} .

Ovidiu Munteanu, Brasov


Solutie. Fie P E. Exist
a A Mn (C), Q (A) = On si P (x) = det (xIn A).
Fie C o radacina a lui P . Rezulta ca sistemul AX = X are si o solutie nebanala,
notat
a X0 . Este usor de v
azut c
a Q (A) X0 = Q () X0 si prin urmare Q () = 0, deci
ad
acinile
r
ad
acinile lui P sunt n numere din multimea {1 , 2 . . . m }, unde i sunt r
lui Q. Prin urmare, P este determinat de n numere, nu neap
arat distincte, 1 , 2 ,
. . . , n din multimea {1 , 2 , . . . , m }.
Invers, dnd un polinom P care are ca r
ad
acini n numere 1 , 2 , . . . , n ca
mai sus, fie A matricea care are pe diagonala principal
a 1 , 2 , . . . , n si 0 n rest.
Evident, det (xIn A) = (x 1 ) (x n ) = P (x), iar Q (A) este o matrice
care are pe diagonala principala Q (i ) = 0 si 0 n rest, deci Q (A) = On . Rezulta ca
P E. Prin urmare, num
arul de elemente al lui E este egal cu num
arul de posibilit
ati
de a alege n numere oarecare dintr-o multime cu m elemente, f
ar
a a conta ordinea,
n
adic
a Cm
.
L53. Fie n 2 si (A, +, ) un inel comutativ cu n2 elemente, care are cel mult
n 2 divizori ai lui zero. Sa se arate ca A este corp.
Gabriel Dospinescu, elev, Onesti
Solutie. Presupunem c
a A nu este corp. Fie T multimea divizorilor lui zero;
urmeaza ca T 6= . Presupunem ca T are k elemente, 1 k n 2. Consideram
150

x T si a A \T . Exist
a d 6= 0 nct xd = dx = 0. Atunci (ax) d = d (ax) = 0, deci
ax = 0 sau ax T . Daca ax = 0, avem a T , fals. Deci ax T si prin urmare putem
defini f : A \T T , f (a) = ax. Cum A \T are n2 k1 elemente, T are k elemente
n2 k 1
si
> n, exist
a elementele diferite a1 , a2 , . . . , an+1 A \ T astfel nct
k
f (a1 ) = f (a2 ) = = f (an+1 ). Deducem ca (an+1 ai ) x = x (an+1 ai ) = 0,
i = 1, n. Deci an+1 ai T pentru i = 1, n. Asadar T are cel putin n elemente,
contradictie.
L54. Fie f : R R o functie cu derivata continua pentru care f (x) 6= 0, x 6= 0.
Sa se determine functiile continue : R R care satisfac identitatea
Z y

Z x

1
1
f (x)
(t) dt (y) = f (y)
(t) dt (x) , x, y R,
a
a
0
0
unde a 6= 0 este o constanta data.
Adrian Corduneanu, Iasi
Solutie. Relatia dat
a pote fi scris
a sub forma
Rx
Ry
1
1
(t) dt (y)
(t) dt (x)
0
0
a
a
=
, x, y6=0.
f (y)
f (x)
Rx
1
Z x
(t) dt (x)
0
1
a
(t) dt (x) = cf (x),
Prin urmare,
= c, c R, deci
f (x)
a
0
egalitatea avnd loc si pentru x = 0, n baza continuitatii. Pentru x = 0 rezulta
(0) = acf (0).
1
Prin derivare obtinem ecuatia (x) 0 (x) = cf 0 (x), ce are solutia
a

Z x
eat (acf 0 (t)) dt , unde k = (0) = acf (0). Asadar,
(x) = eax k +

0
Z x
at 0
ax
(x) = ace
e f (t) dt , c R.
f (0) +
0

a1
a
L55. Fie a (0, ) \ {1}. Definim sirul (xn )n1 prin x0 =
; xn =

ln a
ln a
n

xn1 , n 1. Aratati ca sirul este convergent si calculati lim xn si lim nxn .


n
n
ln a
Gheorghe
Iurea,
Iasi
Z 1
x n
Solutie. Prin inductie matematic
a se arat
a c
a xn =
a x dx, n N ,
0

a1
. Pentru a (0, 1) avem xn a xn ax xn , x [0, 1]; integrnd,
ln a
Z 1
a
1
obtinem
ax xn dx

. Deci lim xn = 0. Pentru a (1, ), din


n
n+1
n+1
0
1
a
xn
, n N. Ca urmare,
xn xn ax xn a, x [0, 1], deducem
n+1
n+1
lim xn = 0.
x0 =

n concluzie, pentru orice a (0, ) \ {1}, lim xn = 0. Din relatia de recurenta


n
rezult
a lim nxn1 = a, de unde deducem c
a lim nxn = a.
n

151

Probleme propuse1
Clasele primare
P.74. Descopera regula de formare, apoi completeaza sirurile urmatoare:
a) 1,2,3; 2,3,5; 3,,; 5,,.
b) 11,10,12; 13,12,14; 15,,; 17,,.
c) 2,6,4; 3,7,5; 4,8,6; 5,,; 6,,.
( Clasa I )
nv. Maria Racu, Iasi
P.75. R
aspundeti la urm
atoarele ntreb
ari:
a) De cte suprafete este m
arginit cubul?
b) Ce forma au fetele cuboidului?
c) Ce forma are un obiect care se aseamana cu sfera?
( Clasa I )
Aliona Loghin, elev
a, Iasi
P.76. Completati casetele din expresia 654321 cu semnele grafice "+"
sau "" pentru a obtine cel mai mic rezultat posibil.
( Clasa a II-a)
nv. Gheorghe Toma, Muncelu de Sus (Iasi)
P.77. Un corp este format din trei cuburi a, b, c ca n
4
5
2
figura al
aturat
a. Fiecare cub are fetele numerotate de la
6
1 la 6, iar suma numerelor de pe oricare dou
a fete opuse
3
2
4
ale sale este 7. S
tiind c
a pe fetele lipite ale cuburilor a si
b este scris acelasi numar si ca aceeasi proprietate o au si
a
b
c
cuburile b si c, s
a se afle suma tuturor numerelor scrise pe fetele corpului care nu se
v
ad.
( Clasa a II-a)
Oxana Pascal, elev
a, Iasi

P.78. a) Verific
a egalit
atile: 1 + 3 + 5 + 7 = 4 4, 1 + 3 + 5 + 7 + 9 + 11 = 6 6;
b) Scrie rezultatul la fel ca la punctul a) pentru 1 + 3 + 5 + 7 + + 19.
( Clasa a III-a)
Andreea Surugiu, student
a, Iasi
P.79. 7 elevi m
anc
a 7 inghetate n 6 minute. Cti elevi vor mnca 24 nghetate
n 36 minute?
( Clasa a III-a)
Alexandru Tudorache, elev, Iasi
P.80. Dou
a orase sunt legate printr-o linie de cale ferat
a. La fiecare or
a pleac
a
un tren din fiecare oras c
atre cel
alalt. Toate trenurile merg cu aceeasi vitez
a si fiecare
calatorie de la un oras la altul dureaza 6 ore. De cte ori fiecare tren, care parcurge
distanta dintre orase, se ntlneste cu trenuri care merg n sens opus?
( Clasa a IV-a)
Alexandru Tudorache, elev, Iasi
P.81. Sa se arate ca din fetele unui cub confectionat din carton putem construi,
f
ar
a resturi, fetele a sase cuburi.
( Clasa a IV-a)
Petru Asaftei, Iasi
P.82. Sa se afle cel mai mare numar natural de forma abcd cu proprietatile:
a 6= d, b + c = 5 (a + d).
( Clasa a IV-a)
Adrian Andronic, elev, Iasi
P.83. Mircea mpreuna cu fratele sau au un numar de bomboane mai mic dect
30. Mircea are de 3 ori mai multe dect fratele s
au. Aflati cte bomboane trebuie s
a
1

Se primesc solutii pn
a la data de 1 iunie 2005.

152

i dea Mircea fratelui s


au pentru a r
amne cu un num
ar de dou
a ori mai mare dect
al fratelui. Cte bomboane avea Mircea la nceput si cu cte a ramas?
( Clasa a IV-a)
Inst. Tudor Tudorache, Craiova

Clasa a V-a
V.51. ntre oricare doua numere naturale definim operatia a b = ab + a.
a) Sa se rezolve ecuatia 2 (x + 1) = 34.
b) Este operatia dat
a comutativ
a?
Vasile Solcanu, Bogd
anesti (Suceava)
V.52. Un dreptunghi se poate descompune n 1344 patrate de arie 25 cm2 . Aflati
perimetrul dreptunghiului daca acesta este: a) maxim posibil; b) minim posibil.
Romanta Ghita
si Ioan Ghita
, Blaj
V.53. Determinati n N pentru care

3
1
1
1 + + + n
2 + 6 + + 98
2
3
3
=

.
1
1
5
1 + 3 + + 17
1 + + + n
4
5
5
Viorel Cornea, Hunedoara
V.54. S
a se arate c
a nu exist
a numere rationale pozitive a, b, c astfel nct
b+c
c+a
a+b
= 22003 ,
= 22004 si
= 22005 .
ab
bc
ca
Andrei - Sorin Cozma, elev, Iasi
2
3
2004
1
+
+
+ +
, unde ai N ,
V.55. Fie num
arul rational N =
a1
a2
a3
a2004
1
i = 1, 2004. Sa se arate ca exista a1 , a2 , . . . , a2004 astfel nct N =
. Gene2005
ralizare.
Petru Asaftei, Iasi

Clasa a VI-a
VI.51. Pentru efectuarea unei lucr
ari, trei muncitori au fost retribuiti cu sume
de bani direct proportionale cu numerele 16, 14, 17. Unul dintre muncitori constata
ca daca sumele primite ar fi fost invers proportionale cu numerele 3, 4, 5, el ar fi
primit mai putin cu 1000000 lei. Aflati ce sum
a de bani a primit fiecare muncitor.
Ion Visan, Craiova
1 2 3
VI.52. Determinati numerele ntregi n care pot fi scrise sub forma n = + + ,
a
b c
cu a, b, c Z .
Gheorghe Iurea, Iasi
d
VI.53. Se da unghiul ascutit xOy si punctele A, B (Ox, C, D (Oy astfel
nct A (OB), C (OD), AB 6= CD si t OA + s AB = t OC + s CD, cu
d sunt
s, t R . Atunci mediatoarele segmentelor [AB] si [CD] si bisectoarea lui xOy
trei drepte concurente daca si numai daca t = 2s.
Ioan S
ac
aleanu, Hrl
au
VI.54. Fie 4ABC isoscel (AB = AC), N mijlocul lui [AC], iar D un punct pe
prelungirea lui [BC] astfel nct CD < BC. S
a se arate c
a ntre triunghiurile ABN
si N CD nu exist
a nici o congruenta.
Romanta Ghita
si Ioan Ghita
, Blaj
153

VI.55. Fie punctele O, A1 , A2 , A3 , . . . astfel nct OA1 = OA2 = OA3 = =

\
\
= 1 cm, iar m(A\
1 OA2 ) = 1 , m(A2 OA3 ) = 2 , m(A3 OA4 ) = 3 etc. (toate unghiurile se consider
a n sens orar). S
a se arate c
a exist
a k 6= l astfel nct Ak = Al .
Cristian Laz
ar, Iasi

Clasa a VII-a
VII.51. Fie a, b N astfel nct n2004 a se divide cu n b, pentru orice n N,
n 6= b. Sa se arate ca a = b2004 .
Alexandru Negrescu, elev, Botosani
VII.52. Fie a, b, c R cu a + b + c = 0; s
a se arate c
a
3

3
3
3
a b3 + b3 c3 + c3 a3 = 3(a b)(b c)(c a) a2 bc b2 ac c2 ab .
Anca Tutescu, elev
a, Craiova
VII.53. Determinati m, n, p Z astfel nct solutia inecuatiei |mx 1| n s
a
fie [p, p + m + 1].
Ciprian Baghiu, Iasi
d de masura 10 si un segment [M N ] de lungime
VII.54. Se considera unghiul xOy
a. S
a se construiasc
a, folosind numai rigla si compasul, un triunghi dreptunghic
OAB, A (Ox, B (Oy, avnd o catet
a de lungime a.
Florin As
avoaie, elev, Iasi
VII.55. Fie ABCD patrulater convex, iar {O} = AC BD. Bisectoarele in\ BOC,
\ COD,
\ DOA
\ taie laturile (AB), (BC), (CD),
terioare ale unghiurilor AOB,
respectiv (DA) n M , N , P , respectiv Q. Sa se arate ca dreptele M Q, N P si BD
sunt concurente sau paralele.
Constantin Cocea si Dumitru Neagu, Iasi

Clasa a VIII-a
VIII.51. Se considera functiile f, g, h : R R definite prin f (x) = x, g (x) =
x
= , h (x) = 3. Not
am {A} = Gg Gh , {B} = Gf Gh , iar C si D sunt punctele de
3
intersectie ale dreptei x = 2 cu Gf , respectiv Gg . Determinati masurile unghiurilor,
perimetrul si aria patrulaterului ABCD.
Dumitru - Dominic Bucescu, Iasi
VIII.52. Fie E (x, y) = 2004 2x2 5y 2 + 2xy + 6y, cu x, y N. Determinati
valoarea maxima a lui E.
Gheorghe Iurea, Iasi
VIII.53. S
a se arate c
a pentru orice a, b, c R, are loc inegalitatea

a4 + b4 + c4 + 3a2 b2 + 3a2 c2 + 3b2 c2 2 a3 b + ab3 + a3 c + ac3 + b3 c + bc3 .


Marian Tetiva, Brlad
VIII.54. Pentru a, b, c (0, ), s
a se demonstreze inegalitatea

p
p
p
2a b2 + c2
2
2
2
2
2
a+ b +c a +b +c <
< a + b2 + c2 .
a2 + b2 + c2
Radu Frunz
a si Mircea Cosbuc, elevi, Iasi
VIII.55. O piramida triunghiulara regulata este tetraedru regulat daca si numai
dac
a unghiurile f
acute de o fata lateral
a cu planul bazei, respectiv cu o alt
a fata
lateral
a, sunt congruente.
Claudiu - S
tefan Popa, Iasi
154

Clasa a IX-a
IX.51. Fie sirul (an )n1 definit prin: a1 = 1 + 2 3; a2 = a1 + 4 + 5 + 6 7 8;
a3 = a2 + 9 + 10 + 11 + 12 13 14 15 etc.
a) S
a se determine semnele cu care apar 100 n a100 , respectiv 91 n a91 .
b) S
a se afle formula termenului general al sirului.
Lidia Nicola, Craiova
IX.52. S
a se determine functiile f, g : Z R cu propriet
atile: f (0) = 2004,

f este par
a
,
g
este
impar
a
s
i
exist
a
a,
b

N
astfel
ncat
f
(x)
=
f
x
+
x
+
a
,
2

g (x) = g x + x + b , x Z.
D. M. B
atinetu-Giurgiu, Bucuresti
IX.53. Exista functii f : R R pentru care
|f (x + y + z + t) + cos x + cos y + cos z + cos t| < 4,

x, y, z, t R?

Lucian Tutescu, Craiova


[ ) = .
IX.54. Fie ABCD un patrat de latura a, iar T (AD) astfel nct m(ABT
Notam {S} = AC BT si fie R punctul n care perpendiculara n S pe BT intersecteaz
a AB.
a) S
a se arate c
a 4RST este isoscel.
b) Sa se exprime RS functie de a si .
Gheorghe Costovici, Iasi
IX.55. Fie ABC un triunghi cu c < b. Not
am cu M si N mijloacele laturilor
[AB], respectiv [AC] si cu D si E punctele de tangenta a cercurilor nscris si respectiv
A- exnscris triunghiului cu latura [BC]. Aratati ca
(i) M E si N D se intersecteaza pe mediana din vrful A;
(ii) M D k N E a = 2 (b c);
(iii) dac
a a 6= 2 (b c), atunci M D si N E se intersecteaz
a pe prelungirea medianei
din A.
Temistocle Brsan, Iasi

Clasa a X-a
X.51. Fie OABC un tetraedru cu OA OB OC, circumscris
unei sfere de
3+1
R

.
raz
a r. Dac
a R este raza cercului circumscris 4ABC, atunci
r
2
Cezar Lupu, elev, Constanta
X.52. Fie polinomul
X
6n+1 12n+2

=
ak X k , n N.
P (X) = 1 + X + X 2

S
a se arate c
a

2n
P

k=0

a6k =

2n
P

k=0

a6k+2 .

k=0

C
at
alin Calistru, Iasi
X.53. Fie a, b, c (1, ) astfel nct a + b + c = 9. S
a se arate c
a
3

3
3
3
loga 2b + c + logb 2c + a + logc 2a + b 12.
aeru, Suceava
Angela Tig
X.54. Definim multimile Ak , k 1, prin
n n
o

A1 =
| n = 1, 2, . . . , 10000 \ {1} ; Ak = (Ak1 \ {a, b}) a2 lg b ,
2003
155

cu a, b Ak1 arbitrare, k 2. S
a se determine A9999 .
Marius Pachitariu, elev, Iasi
2
X.55. Fie a, b Z cu a 4b < 0, iar o solutie a ecuatiei x2 + ax + b = 0.
Definim functia f : Z Z Z, f (x, y) = x2 axy + by 2 . Pentru orice pereche
1
(x, y) f 1 (1), sa se arate ca (x + y)card f (1) = 1.
Andrei Nedelcu, Iasi

Clasa a XI-a
XI.51. S
a se calculeze determinantul unei matrice p
atratice de ordinul patru care
are toti minorii de ordin trei egali.
Lucian - Georges L
adunc
a, Iasi

XI.52. Fie functia f : M2 (R) [0, ), f (A) = det A2 + I2 , A M2 (R).


2
2
a) S
a se arate c
a f (A) = (det A 1) + (tr A) , A M2 (R).
b) Sa se demonstreze ca f este surjectiva, dar nu este injectiva.
Ovidiu Pop, Satu Mare
XI.53. Fie R; pentru n 3, definim

cos + 2k cos
sin
+ 2k

sin

n
n

k =
, k N .
2(k+1)
cos sin + 2(k+1)
sin
cos +
n
n

S
a se calculeze limita sirului (an )n3 , an =

n2
P
k=1

|k |.

Gheorghe Croitoru si Gabriel Popa, Iasi


XI.54. Fie k N ; sa se arate ca ecuatia xn+k xn xn1 x 1 = 0 are
o singura solutie pozitiva, pe care o notam xn . Sa se arate apoi ca sirul (xn )n1 este
convergent; ce se poate spune despre limita sa?
Dumitru Mihalache si Marian Tetiva, Brlad
XI.55. Determinati toate functiile f : R R pentru care

f x2n+1 + x x f 2n+1 (x) + f (x) , x R,

unde n N. (n leg
atur
a cu problema 2811 din Crux Mathematicorum, nr. 1/2003)
Titu Zvonaru, Com
anesti

Clasa a XII-a

sin x
dx, p < 2, este convergent.
p
1/n x
Rodica Luca Tudorache, Iasi
XII.52. Fie f : [0, 1] R derivabil
a, cu derivata continu
a, astfel nct
f (x) + f 0 (x) = 0, x [0, 1]. S
a se arate c
a
Z
Z 1
(2e 5) f (1) 4 2 xex f (x 1)
f (x) dx
dx.
+
5
e 1 (x2 + 1)2
0
Mihail Bencze, Brasov
XII.53. Prove that
Z x t
2 (ex 1)
e

dt x, x 0.
t+1
ex + 1
e
x
Zdravko Starc, Vrsac, Serbia and Montenegro
XII.51. S
a se arate c
a sirul (an )n1 , an =

156

XII.54. S
a se afle functiile continue u = u (t), solutii ale ecuatiei
Z t
Z a
u (t) = +
b (s) u (s) ds +
b (s) u (s) ds, 0 t a,
0

unde este constant


a, iar b = b (t) este continu
a pe [0, a].
Adrian Corduneanu, Iasi
XII.55. S
a se arate c
a pentru orice n N , exist
a monoizi care nu sunt grupuri
si care contin exact n elemente inversabile.
Paul Georgescu si Gabriel Popa, Iasi

Probleme pentru preg


atirea concursurilor
A. Nivel gimnazial
G66. Se considera multimea A = {1, n + 1, 2n + 1, . . . , mn + 1}, m, n N ,
m > n. S
a se afle cte valori distincte poate lua suma a1 + a2 + + an , unde
a1 , a2 , . . . , an A.
Petru Asaftei, Iasi
G67. Fie b N, b 2. Spunem ca un numar natural este decompozabil daca se
poate scrie ca suma a dou
a numere cu aceesi sum
a a cifrelor n baza b. S
a se arate
c
a exist
a o infinitate de numere care nu sunt decompozabile.
Adrian Zahariuc, elev, Bac
au
G68. Fie N N ; s
a se arate c
a exist
a n N astfel nct factorialul niciunui
num
ar natural s
a nu se termine cu n, n + 1, . . . , n + N zerouri.
Iuliana Georgescu, Iasi
G69. Fie E (x) = ax2 + bx + c, a, b, c Q, x R. Dac
a a + b + c Z, ar
atati c
a
exist
a o infinitate de numere ntregi n astfel nct E (n) s
a fie num
ar ntreg.
Gheorghe Iurea, Iasi
G70. S
a se arate c
a ecuatia x2 + y 2 + 3x + y 707 = 0 nu are solutii n Q2 .
Dan Popescu, Suceava
a se demonstreze inegalitatea
G71. Fie (m, n) N2 \ {(0, 0)}. S
a
b
c
+
+

2
2
2
2
2
2
2
(m + n) a + mb + nc
(m + n) b + mc + na
(m + n) c + ma2 + nb2

1 1 1
1
+ +
, a, b, c (0, ) .

2 (m + n) a b
c
Titu Zvonaru, Com
anesti
G72. Fie 4ABC circumscris cercului de centru I. Cercul de diametru [AI]
b si C
b n M , respectiv N . Sa se arate ca M si
intersecteaza bisectoarele unghiurilor B
N se afl
a pe dreapta suport a liniei mijlocii paralele cu BC.
Doru Buzac, Iasi
G73. Fie ABCD un dreptunghi de centru O. Consider
am N (AO), M mijlocul
lui [AD], {P } = M N CD, {E} = OP BC. S
a se arate c
a N E BC.
Andrei Nedelcu, Iasi
157

G74. Fie n puncte n spatiu astfel nct oricare patru s


a formeze tetraedre de
volum cel mult 1. Sa se arate ca exista un tetraedru de volum cel mult 27 care sa
contina n interior toate cele n puncte.
Tudor Chiril
a, elev, Iasi
G75. Fie A1 A2 . . . An un poligon regulat de latur
a 1, n 4. Pe latura [A1 A2 ]
se considera punctul P1 cu P1 A1 = a (0, 1). Din punctul P1 se propaga o raza de
lumina care se reflecta de laturile [A2 A3 ], [A3 A4 ], . . . , genernd pe laturi punctele
de incidenta P2 , P3 , . . . (presupunnd c
a raza de lumin
niciodat
a ntr-un
a nu ajunge

2
vrf al poligonului) astfel nct m(A\
,
. S
a se afle valoarea
2 P1 P2 ) =
n n
minima a lui l pentru care Pl si Pl+1 nu apartin la doua laturi consecutive.
Irina Mustata
a, Iasi
, elev

B. Nivel liceal
L66. Fie ABC un triunghi, D si Da punctele n care cercurile nscris si Aexnscris sunt tangente la BC si Eb , Fc punctele n care cercurile B-exnscris si Cexnscris sunt tangente la AC si respectiv AB. Sa se arate ca punctele D, Da , Eb ,
b = 90 .
Fc sunt conciclice dac
a si numai dac
a AB = AC sau m(A)
Temistocle Brsan, Iasi

L67. Dreptele paralele t1 si t2 sunt tangente cercului C de centru O. Cercul C1


de centru O1 este tangent la t1 si C, iar cercul C2 de centru O2 este tangent la t2 , C
si C1 ; cele trei cercuri sunt exterioare unul celuilalt. Sa se arate ca unghiul O\
1 OO2
este ascutit si s
a se afle valoarea maxim
a a m
asurii acestuia.
Neculai Roman, Mircesti (Iasi)
L68. a) Pentru x, y, z (0, ), sa se demonstreze inegalitatea
s
r

r
1
1 1
x
y
(x + y + z)
+ +
1+
+
.
x y z
y
x
b) Folosind eventual a), s
a se arate c
a n orice triunghi, cu notatiile uzuale, are
loc inegalitatea
s
r
r
pa
pb
R
1+4 1+
+
.
r
pb
pa
Marian Tetiva, Brlad
L69. Pentru ce numere naturale n 3, exista n plan n puncte albastre si n
puncte rosii, oricare trei necoliniare, astfel nct n interiorul oric
arui triunghi cu
vrfurile albastre s
a existe cel putin un punct rosu, iar n interiorul oric
arui triunghi
cu vrfurile rosii s
a existe cel putin un punct albastru?
Adrian Zahariuc, elev, Bac
au
L70. Fie k, p N si un dreptunghi de dimensiuni 82k 2p, acoperit complet si
fara suprapuneri cu dreptunghiuri 7 5 si 6 4. Sa se arate ca numarul patratelelor
(x, y), x par, y impar, ale dreptunghiului mare, care sunt colturi n dreptunghiuri
75, este egal cu num
arul de dreptunghiuri 75. (Prin dreptunghiuri 75 ntelegem
dreptunghiuri cu lungimea egal
a cu 7 si l
atimea egal
a cu 5.)
Marius Pachitariu, elev, Iasi
158

L71. Fie n N, n 2 fixat. S


a se determine cea mai tare inegalitate de forma
n
n
q
X
X
a2k + n2 1 m
ak + M,
k=1

k=1

unde m, M nu depind de a1 , a2 , . . . , an , valabil


a pentru orice numere a1 , a2 , . . . ,
an pozitive si cu produsul 1.
Gabriel Dospinescu, Bucuresti
L72. Fie a, b numere rationale, pozitive, distincte, astfel nct an bn Z pentru
o infinitate de numere naturale n. Sa se arate ca a si b sunt ntregi.
Gabriel Dospinescu, Bucuresti

L73. Fie k N , k 3. S
a se determine n N \ {0, 1} pentru care
r
q

a1 + a2 + + ak n a1 a2 . . . ak , a1 , a2 , . . . , ak [0, ).

Gabriel Popa si Paul Georgescu, Iasi


a f : [a, b] R este continu
a si
L74. Fie n N , n 2 si a, b R, a < b. Dac
Rb k
x
f
(x)
dx
=
0
pentru
0

n,
atunci
f
are
cel
pu
t
in
n
+
1
zerouri
distincte
n
a
(a, b).
Andrei Nedelcu, Iasi
L75. Sa se determine n N pentru care este adevarata inegalitatea
i
1
cos < p
,

0,
.
8
2
1 + n sin4

C
at
alin Calistru, Iasi

Training problems for mathematical contests


Junior high school level
G66. Considering the set A = {1, n+1, 2n+1, . . . , mn+1}, m, n N , m > n, find
the number of distinct values taken by the sum a1+a2+. . .+an , when a1 , a2 , . . . , an A.
Petru Asaftei, Iasi
G67. Let b N, b 2. It is said that G N is decomposable if we can write G as
a sum of two numbers such that their expansions in the basis b have the same sum of
digits. Prove that there exist infinitely many numbers which are not decomposable.
Adrian Zahariuc, high school student, Bac
au

G68. Let N N . Prove that there is n N such that no factorial ends in n,


n + 1, . . . , n + N zeros.
Iuliana Georgescu, Iasi
2
G69. Let E (x) = ax + bx + c, a, b, c Q, x R. If a + b + c is an integer, prove
that there exist infinitely many integers n such that E (n) is also an integer.
Gheorghe Iurea, Iasi
G70. Prove that the equation x2 + y 2 + 3x + y 707 = 0 has no solutions in Q2 .
Dan Popescu, Iasi
159

G71. Let (m, n) N2 \ {(0, 0)}. Prove that


(m +

n) a2

b
c
a
+
+

2
2
2
2
2
2
+ mb + nc
(m + n) b + mc + na
(m + n) c + ma2 + nb2

1 1 1
1
+ +
, a, b, c (0, ) .

2 (m + n) a b
c
Titu Zvonaru, Com
anesti

G72. Let I be the incenter of a triangle ABC. The circle with diameter [AI]
b and C
b in M , respectively N . Prove that M and N lie on
meets the bisectors of B
the line joining the midpoints of [AB] and [AC].
Doru Buzac, Iasi
G73. Let ABCD be a rectangle with center O. Let N (AO), let M be the
midpoint of [AD] and let {P } = M N CD, {E} = OP BC. Prove that N E BC.
Andrei Nedelcu, Iasi
G74. Let us consider n points such that any given four are the vertices of a
tethraedron with volume at most 1. Prove that there is a tethraedron with volume
at most 27 which contains all n points in its interior.
Tudor Chiril
a, high school student, Iasi
G75. Let A1 A2 . . . An be a regular n-gon with side 1, n 4. We consider P1
on the side [A1 A2 ] such that P1 A1 = a (0, 1). A ray of light is emitted from P1
towards and is reflected by the sides [A2 A3 ], [A3 A4 ], . . . , generating the incidence
points P2 , . . . , Pn (supposingthat the
ray never meets the vertices A1 , A2 , . . . , An )
2
\
such that m(A2 P1 P2 ) =
,
. Find the minimal value of l such that Pl and
n n
Pl+1 do not belong to adjacent sides.
Irina Mustata
, high school student, Iasi

High school level


L66. Let ABC be a given triangle and let D, Da be the points in which the
incircle, respectively the A-escribed circle are tangent to the side BC. Let also Eb ,
Fc be the points in which the B-escribed and C-escribed circles are tangent to the
side AC, respectively to the side AB. Prove that D, Da , Eb , Fc are concyclic if and
b = 90 .
only if AB = AC or m(A)
Temistocle Brsan, Iasi

L67. The parallel lines t1 and t2 are tangent to the circle C with center O, the
circle C1 with center O1 is tangent to t1 and C and the circle C2 with center O2 is
tangent to t2 , C and C1 ; C, C1 and C2 being exterior to each other. Prove that the
angle O\
1 OO2 is acute and find the minimum value of its measure.
Neculai Roman, Mircesti (Iasi)
L68. a) Prove that, for x, y, z (0, ),
s
r

r
x
y
1
1 1
(x + y + z)
+ +
1+
+
.
x y z
y
x
160

b) Using a), prove that


r

R
1+
1+4
r

pa
+
pb

for any given triangle with the usual notations.

pb
pa

Marian Tetiva, Brlad


L69. Find n N, n 3, such that there are n blue points and n red points in
the same plane, no three points being collinear, such that the interior of any triangle
with blue vertices contains at least one red point and the interior of any triangle with
red vertices contains at least one blue point.
Adrian Zahariuc, high school student, Bac
au
L70. Let k, p N and let a 82k 2p rectangle which is completely covered with
7 5 and 6 4 rectangles with no superpositions. Prove that the number of squares
(x, y) with side 1, x even, y odd, which are vertices of 7 5 rectangles equals the
total number of 7 5 rectangles. (By a 7 5 rectangle we mean a rectangle with
length 7 and height 5).
Marius Pachitariu, high school student, Iasi
L71. Let n N, n 2. Find the best constants m, M such that
n q
n
X
X
a2k + n2 1 m
ak + M
k=1

k=1

for any a1 , a2 , . . . , an > 0 satisfying a1 a2 . . . an = 1.


Gabriel Dospinescu, Bucuresti
n
L72. Let a, b Q, a, b > 0, a 6= b such that a bn Z for infinitely many n N.
Prove that a, b Z.
Gabriel Dospinescu, Bucuresti

L73. Let k N , k 3. Find n N \ {0, 1} such that


r
q

a1 + a2 + + ak n a1 a2 . . . ak , a1 , a2 , . . . , ak [0, ).
Gabriel Popa and Paul Georgescu, Iasi
L74. Let n N , n 2 and a, b R, a < b. If f : [a, b] R is a continuous
Rb
function such that a xk f (x) dx = 0 for any k N, 0 k n, then f has at least
n + 1 distinct zeros in (a, b).
Andrei Nedelcu, Iasi
L75. Find n N such that
i
1
cos < p
,

0,
.
8
2
1 + n sin4
C
at
alin Calistru, Iasi

161

Pagina rezolvitorilor
BOTO
SANI
Scoala
nr. 7 "O. Bancila". Clasa a IV-a. IFTODE Cozmin: P(54-59,61,63,64,

67,68,70).
Colegiul National "A. T. Laurian". Clasa a IX-a. NEGRESCU Alexandru:
VII(43,47,48), VIII(47,49), IX(42,46,48), X.42, G46.
CRAIOVA
Angela). STANCIU
Scoala
nr. 22 "M. Eliade". Clasa a IV-a (inst. VANTU

Ioan: P (64-73)
IA
SI
Scoala
nr. 3 "Al. Vlahuta". Clasa a V-a. COJOCARU Ioana: P.71, V(47
49),VI.46; DODU Corina: P.71, V(47-49),VI.46; IRIMIA Andreea: P.71, V(4749),VI.46; S
TIRBAN Ioana: P.71, V(47-49),VI.46; UNGURU George Claudiu: P.71,
V(47-49),VI.46.
Scoala
nr. 7 "N. Tonitza". Clasa a II-a (nv. TUDOSE Elena). DOBRIN Diana
Simona - AlexanMaria: P(64-68,70,72); LEONTE Anca: P(64-68,70,72); POSTICA
dra: P(64-68,70,72); ROTARU Larisa-Maria: P(64-68,70,72); SAVIN Razvan: P(6468,70,72). Clasa a II-a (nv. MELINTE Rodica). BACIU Ciprian: P(64-69,71,72);
BRZU Constantin: P(64-69,71,72); BOTOSANU Bianca-Mihaela: P(64-69,71,72);
D
BUZDUGAN Petru-C
at
alin: P(64-69,71,72); CEUCA
anut-Vasilic
a: P(64-69,71,
72); CONSTANTINESCU Diana-Gabriela: P(64-69,71,72); CUCUTEANU PaulC
at
alin: P(64-69,71,72); GUSOVATE Diana-Stefana: P(64-69,71,72); LEOGAN
Larisa-Diana: P(64-69,71,72); MIRON Vlad-Stefan: P(64-69,71,72); MOTAN Geanina-Diana: P(64-69,71,72); ROTARIU Marian: P(64-69,71,72); SUCIUC Raluca:
P(64-69,71,72); TEIU-COSTIN Andra-Mihaela: P(64-69,71,72). Clasa a III-a
AROIU

(nv. PASANIUC Maria). ATASIEI Vl


adut: P(64,65,67,68,72); PAS
Bogdan:
P(66-68,70,71); SOLOMON Ana-Maria: (65,68-71); TINCU Andrei: P(64,65,68
70,72); ZAMFIR Loredana-Cristiana: P(64,65,69-71). Clasa a III-a (nv. GEAMAN
ALI
TEI

P(65,68-71); GHIARASIM Olivia: P(64,65,67,


Gabriela). ADASC
IONUT:
68,72); MACOVEI Alina: P(64,65,68-70,72); NEGRESCU Vlad-Petru: P(64,65,6871); TROCIN Monica-Andreea: P(66,68-71).
Scoala
nr. 13 "Alexandru cel Bun". Clasa a IV-a (inst. COJOCARIU Ana). CO
C
A
BILIT
at
alina-Elena: P(64-68,70); CURMEI Renata-Maria: P(64-68,70); DALAS
Radu: P(64-68,70); ILIE Laura: P(64-68,70); MAGDICI Magda-Otilia: P(64-68,70);
MARDARI Claudiu: P(64-68,70); LUCAN Mihaela-Alexandra; P(64-68,70); NECHIFOR S
tefan-Marian: P(64-68,70); NICULAE-GRIGORESCU Andrei: P(64-68,70);
PRISACARU Carmen-Georgiana: P(64-68,70); PETREA Silvia: P(64-68,70); S
TE
FAN Teodora-Ioana: P(64-68,70); TRLAGEANU
Ingrid-Maria: P(64-68,70).
Scoala
nr. 17 "I. Creanga". Clasa a VIII-a. DUMITRIU Vlad: VI(46-48),VII.48,

VIII.46.

CoScoala
nr. 22 "B. P. Hasdeu". Clasa I (nv. S
TEFAN Liviu). DANIL
A

drin P(55,64,65,67,71); IGNAT Andreea: P(54-56,64,65); NICOLA Delia-Corina:


P(55,64,65,67,71); PURICE Dumitru-Ciprian: P(64-67,71). Clasa a II-a (nv. DO162

HOTARU Liliana). TURCU Andrei-Daniel: P(64-67,71); Clasa a II-a (nv. TRZI ALI
TEI

ORU Iuliana). ADASC


Victor: P(64-69,71,72); APOSTOL Ana-Maria:
Catalina: P(64-68,72);
P(64-68,71,72); BALAN Andrei: P(64-68,71,72); BURUIANA
CUBERSCHI Paul: P(64-67,70-72); ESANU Georgiana: P(64-68,71); GREIEROSU
ATIC

Claudiu: P(64-68,72); GNDU Alexandra-Livia: P(64-66,68,71,72); LAM


Ioana: P(61,64-68,71); MOGA Alexandru: P(64-68,71,72); REBEGEA Andrada: P(6468,71); UNGUREANU Teofana: P(64-68,71,72). Clasa a II-a (nv. TUTU Laura).
AILENEI-OPREA Adriana: P(64-68); ANDRONICIUC Ana-Miruna: P(64-68); BR
LADEANU
Claus-Alex: P(64-66,68,71); BOARU Adrian: P(64-66,68,71); BURUIA Sebastian-Andrei: P(64-68,71): BUHU Vlad: P(64-68,71); CEOBANU AndreiNA
Alexandra-Elena: P(64-68,71); COSTACHESCU

Nicolae: P(64-67,71); CHICHIRAU

Alexandru: P(64-66,68,71); DIACONESCU Matei:


Ivona: P(64-68,71); DANIL
A
P(64-68,71); DOROHOI Ovidiu: P(64-68,71); GHERAN Ana-Maria: P(64-66,68,71);
Raluca-Claudia: P(64-68,71); HATESGRIGORE Georgiana: P(64-68,71); GURAU

CU Iustina: P(64-68,71); HORBOVANU Bianca-Alexandra: P(64-68,71); NASTASE


Andrei-Ionut: P(64-68,71); ONOFREI Liviana Ana-Maria: P(64-68,71); RADU
Andrei: P(64-66,68,71); SIMIRAD Andrei: P(54,55,57-59,61,62,64-72); SMEREA
Alexandra-Arina: P(64-66,68,71). Clasa a V-a. PINTILIE Mina-Liviu: P(61,62,7173); PINTILIE Nicoleta-Livia: P(61,62,71-73); S
TERBULEAC Daniel: P(61-63,7173),V(42,43).
Beatrice). TUDOScoala
nr. 23 "T. Maiorescu". Clasa a IV-a (nv. CHIRILA

RACHE Alexandru-Gabriel: P(64-73).


Scoala
nr. 26 "G. Cosbuc". Clasa a II-a (nv. BUCATARIU Rica). IACOB

Robert-Ionut: P(54-57,64-67); IVANCIUC Dumitru-Florin: P(55,64-67); MOISA


Adrian-Bogdan: P(54-57,64-67); SANDU Ioana-Luiza: P(54-57,64-67); SCUTARU
Ionela-Cristina: P(54-57,64-67). Clasa a III-a (nv. RACU Maria). BULGARU

Ionela-Alexandra: P(64-68,70-72); BURLACU S


tefan-Claudiu: P(64-68,70,72); CA

LIN Andreea-Georgiana: P(64-68,70-72); IFROSA Adriana: P(64,66-72); IOJA Petru-Alexandru: P(64-70,72); MOISA Bogdan: P(64-68,70-72); PINTILIE R
azvan
Florin: P(64-68,70-72); RAZLOG
Ionut: P(64-68,70-72). Clasa a III-a (nv. GALIA
Oana-Catalina:
Paraschiva). ALUPEI Andra-Madalina: P(64-67,68,70,71); CIOABA

P(64-68,70,71); GHERCA Marius-Catalin: P(64-68,70,71); HOMEA Liviu: P(6468,70,71); HUIDES Gina: P(64-68,70,71); MANOLIU M
ad
alina: P(64-68,70,71);

Alexandru: P(64-68,70,71); POPA


MIHAILESCU
Laura; P(64-68,70,71); PISICA
Florin: P(64-68,70,71); SCUTARU Constantin: P(64-68,70,71,73).
Colegiul National "C. Negruzzi". Clasa a V-a. ANDRIESCU Gabriela: P(56,58,
59,61,63),V.42; TIBA Marius: P(71-73),V(46,47,49,50),VI.46.
Colegiul National Iasi. Clasa a V-a. VLCU Maria Caterina: V(40,41,43-45).
Liceul "M. Eminescu". Clasa a VI-a. CIURARU Ionela: V(46,48), VI(47,48,50);
IPATE Cristina: V(46,48),VI(47,48,50). Clasa a IX-a. AVRAM Mircea: VII(41,43,
44,46,48). Clasa a X-a. COMAN S
tefan: VIII(46,49), IX(46,47), X.47.

163

Premii acordate rezolvitorilor


Pentru aparitia de trei ori la rubrica "Pagina rezolvitorilor" redactia revistei
"Recreatii matematice" acorda o diplom
a si un premiu n carti elevilor urmatori:
Scoala
nr. 22 "M. Eliade", Craiova

STANCIU Ioan (cl. a IV-a): 2/2003 (9pb), 1/2004 (10pb), 2/2004 (10pb).
Scoala
nr. 7 "N. Tonitza"

BACIU Ciprian (cl. a II-a): 2/2003 (5pb), 1/2004 (5pb), 2/2004 (8pb);
BRZU Constantin (cl. a II-a): 2/2003 (5pb), 1/2004 (5pb), 2/2004 (7pb);
BOTO
SANU Bianca-Mihaela (cl. a II-a): 2/2003 (5pb), 1/2004 (5pb), 2/2004
(7pb);
BUZDUGAN Petru-C
at
alin (cl. a II-a): 2/2003 (5pb), 1/2004 (5pb), 2/2004
(7pb);
D
CEUCA
anut-Vasilic
a (cl. a II-a): 2/2003 (5pb), 1/2004 (5pb), 2/2004 (8pb)
CONSTANTINESCU Diana-Gabriela (cl. a II-a): 2/2003 (5pb), 1/2004 (5pb),
2/2004 (8pb);
CUCUTEANU Paul-C
at
alin (cl. a II-a): 2/2003 (5pb), 1/2004 (5pb), 2/2004
(8pb);
DOBRIN Diana-Maria (cl. a II-a): 2/2003 (5pb), 1/2004 (5pb), 2/2004 (7pb);
GU
SOVATE Diana-
Stefana (cl. a II-a): 2/2003 (5pb), 1/2004 (5pb), 2/2004
(8pb);
LEOGAN Larisa-Diana (cl. a II-a): 2/2003 (5pb), 1/2004 (5pb), 2/2004 (8pb);
LEONTE Anca (cl. a II-a): 2/2003 (5pb), 1/2004 (5pb), 2/2004 (7pb);
MIRON Vlad-
Stefan (cl. a II-a): 2/2003 (5pb), 1/2004 (5pb), 2/2004 (8pb);
MOTAN Geanina-Diana (cl. a II-a): 2/2003 (5pb), 1/2004 (5pb), 2/2004 (8pb);
Simona-Alexandra (cl. a II-a): 2/2003 (5pb), 1/2004 (5pb), 2/2004
POSTICA
(7pb);
ROTARIU Larisa-Maria (cl. a II-a): 2/2003 (5pb), 1/2004 (5pb), 2/2004 (7pb);
ROTARIU Marian (cl. a II-a): 2/2003 (5pb), 1/2004 (5pb), 2/2004 (8pb);
SUCIUC Raluca (cl. a II-a): 2/2003 (5pb), 1/2004 (5pb), 2/2004 (8pb);
TEIU-COSTIN Andrada-Mihaela (cl. a II-a): 2/2003 (5pb), 1/2004 (5pb),
2/2004 (8pb).
Scoala
nr. 22 "B. P. Hasdeu"

TEI

ADASC
ALI
Victor (cl. a II-a): 2/2003 (5pb), 1/2004, (5pb), 2/2004 (9pb);
BALAN Andrei (cl. a II-a): 2/2003 (5pb), 1/2004 (5pb), 2/2004 (8pb);
BUHU Vlad (cl. a II-a): 2/2003 (5pb), 1/2004 (5pb), 2/2004 (6pb);
Alexandra-Elena (cl. a II-a): 2/2003 (5pb), 1/2004 (6pb), 2/2004
CHICHIRAU
(6pb);
CUBERSCHI Paul (cl. a II-a): 2/2003 (5pb), 1/2004 (6pb), 2/2004 (7pb);
E
SANU Georgiana (cl. a II-a): 2/2003 (5pb), 1/2004 (5pb), 2/2004 (5pb);
GREIEROSU Claudiu (cl. a II-a): 2/2003 (5pb), 1/2004 (6pb), 2/2004 (6pb);
Raluca-Claudia (cl. a II-a): 2/2003 (5pb), 1/2004 (5pb), 2/2004 (6pb);
GURAU
164


NASTASE
Andrei-Ionut (cl. a II-a): 2/2003 (5pb), 1/2004 (6pb), 2/2004 (6pb);
HATESCU Iustina (cl. a II-a): 2/2003 (5pb), 1/2004 (5pb), 2/2004 (6pb);
ATIC

LAM
Ioana (cl. a II-a): 2/2003 (5pb), 1/2004 (6pb), 2/2004 (7pb);
REBEGEA Andrada (cl. a II-a): 2/2003 (5pb), 1/2004 (5pb), 2/2004 (6pb).
Scoala
nr. 23 "Titu Maiorescu"

TUDORACHE Alexandru-Gabriel (cl. a IV-a): 2/2003 (10pb), 1/2004 (10pb),


2/2004 (10pb).
Scoala
nr. 26 "G. Cosbuc"

ALUPEI Andra-M
ad
alina (cl. a III-a): 2/2003 (5pb), 1/2004 (5pb), 2/2004 (7pb);
BULGARU Ionela-Alexandra (cl. a III-a): 2/2003 (5pb), 1/2004 (5pb), 2/2004
(8 pb);
BURLACU S
tefan-Claudiu (cl. a III-a): 1/2003 (5pb), 1/2004 (5pb), 2/2004 (8pb);
Marius-C
GHERCA
at
alin (cl. a III-a): 2/2003 (5pb), 1/2004 (5pb), 2/2004 (7pb);
HOMEA Liviu (cl. a III-a): 2/2003 (5pb), 1/2004 (5pb), 2/2004 (7pb);
HUIDE
S Gina (cl. a III-a): 2/2003 (5pb), 1/2004 (5pb), 2/2004 (7pb);
Adriana (cl. a III-a): 2/2003 (5pb), 1/2004 (5pb), 2/2004 (8pb);
IFROSA
Petru-Alexandru (cl. a III-a): 2/2003 (5pb), 1/2004 (5pb), 2/2004 (8pb);
IOJA
Alexandru (cl. a III-a): 2/2003 (5pb), 1/2004 (5pb), 2/2004 (7pb).
PISICA
Colegiul National "C. Negruzzi"
TIBA Marius (cl. V-a): 1/2003 (6pb), 2/2003 (7pb), 1/2004 (7pb).
Liceul "M. Eminescu"
CIURARU Ionela (cl. VI-a): 2/2003 (11pb), 1/2004 (5pb), 2/2004 (5pb).

(continuarea tabelului din p. 128)


178.
179.
180.
181.
182.
183.

PIFTOR Rositta
SPIRIDON Doina
MARTINUSI Vladimir
TIMOHE TUMAC Gabriel
ANTON Florina Cristiana
PRICOP Vasile

Liceul economic nr. 2, Iasi


Liceul de arta, Iasi
Colegiul National "E. Racovita", Iasi
Liceul "G. Ibraaileanu", Iasi
Colegiul National "E. Racovita", Iasi
Pascani

165

Premiile pe anul 2004 acordate


"POIANA"
de FUNDA
TIA CULTURALA
Fundatia Cultural
a "Poiana" (director d-l Dan Tiba) acord
a anual premii
elevilor - colaboratori ai revistei "Recreatii matematice" care se disting prin calitatea
articolelor, notelor si problemelor originale publicate n paginile acesteia.
Redactia revistei decide ca pentru anul 2004 premiile oferite, n valoare de cte
1 000 000 lei, s
a fie atribuite urm
atorilor elevi:
1. LUPU Cezar (Colegiul National "Mircea cel Batrn", Constanta)
Asupra unei inegalit
ati conditionate (RecMat 1/2004, 27-28),
probleme propuse: IX.36 (1/2003), IX.44 (2/2003), IX.48, XI.46 (1/2004),
X.51 (2/2004).
2. NEGRESCU Alexandru (Colegiul National "A. T. Laurian", Botosani )
Asupra unei inegalitati (RecMat 2/2004, 106-108),
probleme propuse: VI.38 (1/2003), VII.41 (2/2003), VIII.46 (1/2004),
VII.51 (2/2204).
Premiile se pot ridica direct de la redactie sau pot fi trimise prin mandat postal
la adresa elevului premiat.

IMPORTANT
n scopul unei leg
aturi rapide cu redactia revistei, pot fi utilizate urm
atoarele adrese e-mail: tbi@math.tuiasi.ro, popagabriel@go.com .
Pe aceast
a cale colaboratorii pot purta cu redactia un dialog privitor la
materialele trimise acesteia, procurarea numerelor revistei etc.
La problemele de tip L se primesc solutii de la orice iubitor de matematici
elementare (indiferent de preocupare profesionala sau vrsta ). Fiecare
dintre solutiile acestor probleme - ce sunt publicate n revist
a dup
a un
an - va fi urmat
a de numele tuturor celor care au rezolvat-o.
Adres
am cu insistenta
amintea ca materialele trimise re rug
vistei s
a nu fie (s
a nu fi fost) trimise si altor publicatii.

166

Revista RECREAII MATEMATICE apare de dou ori pe an (la


datele de 1 martie i 1 septembrie) i se adreseaz elevilor, profesorilor,
studenilor i tuturor celor pasionai de matematicile elementare.
n atenia tuturor colaboratorilor
Materialele trimise redaciei spre publicare (note i articole, chestiuni de
metodic, probleme propuse etc.) trebuie prezentate ngrijit, clar i concis; ele
trebuie s prezinte interes pentru un cerc ct mai larg de cititori. Se recomand
ca textele s nu depeasc patru pagini. Evident, ele trebuie s fie originale
i s nu fi aprut sau s fi fost trimise spre publicare altor reviste.
Problemele originale destinate rubricii Probleme propuse vor fi
redactate pe foi separate cte una pe fiecare foaie, cu enun i
demonstraie/rezolvare, fiind nsoite de numele autorului, coala i localitatea
unde lucreaz/nva.
Redacia revistei va decide asupra oportunitii publicrii materialelor
primite.
n atenia elevilor
Numele elevilor care vor trimite redaciei soluii corecte la exerciiile i
problemele din rubrica Probleme propuse vor fi menionate n Pagina
rezolvitorilor. Elevii vor ine seama de urmtoarele reguli:
1. Pot trimite soluii la minimum cinci probleme propuse n numrul
prezent i cel anterior al revistei; pe o foaie va fi redactat soluia unei
singure probleme.
2. Elevii din clasele VI-XII au dreptul s trimit soluii la problemele
propuse pentru clasa lor, pentru orice clas mai mare, din dou clase mai mici i
imediat anterioare. Elevii din clasa a V-a pot trimite soluii la problemele
propuse pentru clasele a IV-a, a V-a i orice clas mai mare, iar elevii claselor
I-IV pot trimite soluii la problemele propuse pentru oricare din clasele primare
i orice clas mai mare. Orice elev poate trimite soluii la problemele de
concurs (de tip G i L).
3. Vor fi menionate urmtoarele date personale: numele i prenumele,
clasa, coala i localitatea.
4. Plicul cu probleme rezolvate se va trimite prin pot (sau va fi adus
direct) la adresa Redaciei:
Prof. dr. Temistocle Brsan
Catedra de Matematic
Universitatea Tehnic Gh. Asachi Iai
Bulevardul Carol I nr. 11, 700506, Iai
E-mail: tbi@math.tuiasi.ro

CUPRINS
Ctre cititori .................................................................................................................. 85
ALEXANDRU MYLLER, ctitorul colii matematice ieene.................................. 87
HENRI POINCAR la 150 de ani de la naterea sa ........................................ 89
Trecerea planetei Venus prin faa Soarelui ............................................................... 91

ARTICOLE I NOTE
G. DOSPINESCU Cteva noi aplicaii ale unei idei consacrate ........................ 94
T. BRSAN Cteva proprieti ale medianelor....................................................... 99
C. - t. POPA O construcie geometric a mediilor (II)..................................... 102
F. POPOVICI O generalizare a teoremelor de baz ale calculului diferenial ........ 104

NOTA ELEVULUI
A. NEGRESCU Asupra unei inegaliti................................................................ 106
***
Asupra problemei VII.41 din RecMat 2/2003 ....................... 109

CHESTIUNI METODICE
D. MIHALACHE, M. TETIVA Asupra unei probleme de concurs.................... 111

CHESTIUNI COMPLEMENTARE MANUALELOR


M. CRCIUN Exponentul numrului natural a n produsul n!......................... 114

CONCURSURI I EXAMENE
Concursul
Concursul
Concursul
Concursul
Olimpiada

Alexandru Myller ed. a II-a, Iai, 2004 ............................................. 116


Florica T. Cmpan, ed. a IV-a, 2004 ................................................. 121
Traian Lalescu, ed. a V-a, Iai, 2004 ................................................ 123
Adolf Haimovici, ed. a VIII-a, 2004 .................................................. 124
Balcanic de Matematic (juniori), ed. a VIII-a, 2004 ....................... 126

CORESPONDENE
H. STEPHAN Probleme pentru clasa a VIII-a ..................................................... 129

PROBLEME I SOLUII
Soluiile problemelor propuse n nr. 2/2003............................................................. 130
Soluiile problemelor pentru pregtirea concursurilor din nr. 2/2003 .................... 144
Probleme propuse........................................................................................................ 152
Probleme pentru pregtirea concursurilor ................................................................. 157
Training problems for mathematical contests .......................................................... 159
Pagina rezolvitorilor.................................................................................................... 162

Anul VIII, Nr. 2

Iulie Decembrie 2006

RECREAII
MATEMATICE
REVIST DE MATEMATIC PENTRU ELEVI I PROFESORI

e i = 1

Editura Recreaii Matematice


IAI - 2006

Semnificaia formulei de pe copert:


i
ntr-o form concis, formula e = 1 leag cele patru ramuri fundamentale
ale matematicii:
ARITMETICA
GEOMETRIA
ALGEBRA
ANALIZA MATEMATIC

reprezentat
reprezentat
reprezentat
reprezentat

de
de
de
de

i
e

Redacia revistei :
Petru ASAFTEI, Dumitru BTINEU-GIURGIU (Bucureti), Cornelia - Livia BEJAN,
Temistocle BRSAN, Dan BRNZEI, Ctlin - Cristian BUDEANU, Alexandru
CRUU, Constantin CHIRIL, Eugenia COHAL, Adrian CORDUNEANU, Mihai
CRCIUN (Pacani), Gabriel DOSPINESCU (student, Paris), Marius FARCA, Paraschiva
GALIA, Paul GEORGESCU, Mihai HAIVAS, Gheorghe IUREA, Lucian - Georges
LDUNC, Mircea LUPAN, Gabriel MRANU, Andrei NEDELCU, Gabriel POPA, Dan
POPESCU (Suceava), Florin POPOVICI (Braov), Maria RACU, Ioan SCLEANU
(Hrlu), Ioan ERDEAN (Ortie), Dan TIBA (Bucureti), Adrian ZAHARIUC (Bacu),
Adrian ZANOSCHI.

Adresa redaciei:
Catedra de Matematic Universitatea Tehnic Gh. Asachi Iai
Bd. Carol I, nr.11, 700506, Iai
Tel. 032 213737 / int. 123
E-mail: recreatii.matematice@gmail.com
http://www.recreatiimatematice.uv.ro
COPYRIGHT 2006, ASOCIAIA RECREAII MATEMATICE
Toate drepturile aparin Asociaiei Recraii Matematice. Reproducerea integral sau
parial a textului sau a ilustraiilor din aceast revist este posibil numai cu acordul prealabil
scris al acesteia.
TIPRIT LA SL&F IMPEX IAI
Bd. Carol I, nr. 3-5
Tel. 0788 498933
E-mail: simonaslf@yahoo.com

Anul VIII, Nr. 2

Iulie Decembrie 2006

RECREAII
MATEMATICE
REVIST DE MATEMATIC PENTRU ELEVI I PROFESORI

e i = 1
Revist cu apariie semestrial
publicat de

ASOCIAIA RECREAII MATEMATICE

IAI - 2006

Mendel Haimovici si S
coala matematic
a din Iasi
S
coala matematic
a din Iasi, fondat
a n 1910
n cadrul Seminarului Matematic de c
atre savantul
Al. Myller, s-a dezvoltat si consolidat prin opera unor
mari personalit
ati, ca Octav Mayer, Mendel Haimovici
si prin contributia unor str
aluciti continuatori ai lor.
Departamentul de mecanic
a al acestei S
coli a fost
creat de profesorul Mendel Haimovici, membru al
Academiei Romne. Calit
atile sale de Om, Savant si
Profesor, lupt
ator neobosit pentru dreptate, au trezit
admiratia opiniei publice si a colegilor, colaboratorilor
si studentilor s
ai. Contributia sa la dezvoltarea ideilor
matematice moderne este cu totul remarcabil
a. ntreaga sa activitate poart
a marca personalit
atii sale,
caracterizat
a prin inteligenta, curiozitate stiintific
a,
creativitate, vast
a cultur
a si etic
a profesional
a.
M. Haimovici s-a n
ascut la Iasi, n 30 noiembrie 1906. A f
acut studiile liceale la
Colegiul (Liceul) National. S
i-a continuat preg
atirea la Facultate de S
tiinte (sectia
Matematic
a) pe care a absolvit-o n 1930. A fost imediat numit asistent si a devenit
colaborator al fostilor s
ai profesori. A f
acut stagii de specializare la Roma, Paris si
Londra. n iulie 1933 si-a luat doctoratul n Italia, cu o valoroas
a tez
a de mecanica
fluidelor, sub ndrumarea lui Tullio Levi-Civita. ntors n tar
a, si-a reluat munca de
asistent. n 1940, dup
a promulgarea legilor rasiale, a fost ndep
artat ctiva ani din
Universitate. La ncetarea r
azboiului a fost numit profesor si a devenit seful catedrei
de mecanic
a, pe care a condus-o pn
a la sfrsitul vietii, adic
a timp de 28 de ani.
n 1948, M. Haimovici a fost ales membru corespondent, iar ctiva ani mai trziu
a devenit membru titular al Academiei Romne. Cnd s-a nfiintat Filiala Iasi a Academiei, a fost numit director al Institutului de Matematic
a, pe care l-a condus pn
a
la sfrsitul vietii. A fost invitat s
a ia parte la numeroase conferinte internationale si
a organizat la Iasi manifest
ari stiintifice de mare prestigiu. A ncetat din viata la 31
martie 1973, l
asnd o oper
a valoroas
a si o amintire luminoas
a de remarcabil savant.
Cercet
arile lui M. Haimovici se raporteaz
a la trei domenii principale ale matematicii: geometrie, teoria ecuatiilor cu derivate patiale si mecanic
a. Cu toat
a marea
influenta exercitat
a de Myller si Mayer n domeniul geometriilor cu grup fundamental, a abordat un nou subiect din geometria acelui timp. Dup
a vizita lui Elie Cartan
la Iasi, n 1931, atras de ideile acestuia a eleborat importante lucr
ari asupra unor
probleme dificile. Astfel, a descoperit clase remarcabile de spatii Finsler si ecuatiile fundamentale ale hipersuprafetelor din aceste spatii. Mai trziu, matematicianul
japonez Makoto Matsumo a inclus lucr
arile lui M. Haimovici n celebra sa monografie
asupra fundamentelor geometriei Finsler. M. Haimovici a fost pionier si n domeniul
geometriei integrale, descoperind o formul
a care generalizeaz
a teorema cunoscut
aa
lui Crofton. A descoperit conceptul de quasigrup diferentiabil si a construit o teorie
complet
a si unitar
a; a demonstrat c
a aceste obiecte se comport
a fata de grupurile
Lie n aceeasi manier
a n care variet
atile plate se comport
a n raport cu cele curbate.
Actualmente, notiunea de quasigrup este studiat
a din punct de vedere algebric si
89

topologic n scoli renumite de matematic


a din SUA si Rusia.
O lung
a perioad
a de timp, M. Haimovici s-a ocupat de geometria variet
atilor
neolonome. Importanta lor const
a n faptul c
a ele conduc la cel mai adecvat model
care descrie miscarea sistemelor mecanice neolonome. Se poate spune c
a geometria
intrinsec
a a variet
atilor neolonome n spatii Riemann este creatia lui M. Haimovici.
n 1998, a ap
arut volumul I al operelor sale complete, iar volumul al doilea va ap
area
cu prilejul Centenarului nasterii savantului.
Prin contributiile sale, M. Haimovici a scos la lumin
a noi domenii de cercetare,
n care au lucrat si lucreaz
a continuatorii operei sale, printre care si semnatarul
acestei Note, care este totodat
a si primul cercet
ator care a obtinut doctoratul sub
ndrumarea lui M. Haimovici.
Profesorul M. Haimovici a avut vocatie de geometru. Pentru cei care nu sunt
familiarizati cu personalitatea sa, poate p
area curios cum a putut el deveni si un
specialist n mecanic
a. Pentru a l
amuri acest aspect, se cuvine s
a facem un scurt
istoric al pred
arii mecanicii la Universitatea din Iasi. La nfiintarea acesteia, exista si
o catedr
a de mecanic
a. Primul curs "Mecanic
a elementar
a si rational
a" a fost tinut de
Nicolae Culianu. Ulterior, aceast
a catedr
a a fost ilustrat
a de Ioan Melik si Miltiade
Tzony (absolvent de la Sorbona), care a publicat acum 125 de ani o culegere de
probleme de mecanic
a n revista "Recreatii S
tiintifice". Manuscrisul cursului tinut
de Tzony se p
astreaz
a si ast
azi la Biblioteca Seminarului Matematic. Tot lui i se
datoreaz
a si nfiintarea Laboratorului. Tzony r
amne unul dintre cei mai importanti
oameni care au pus bazele unui nv
atamnt de nalt
a valoare n tara noastr
a. A
condus catedra de mecanic
a pn
a n anul 1898. n 1907, Dimitrie Pompeiu, doctor
n matematic
a de la Sorbona, a preluat aceast
a catedr
a. El era un excelent analist,
dar a fost si primul n Iasi, care a publicat lucr
ari originale de mecanic
a. Dup
a
ctiva ani (cinci) a fost chemat la Bucuresti, pentru a lua postul r
amas vacant prin
retragerea lui Spiru Haret. Ulterior, catedra de mecanic
a a fost ilustrat
a succesiv de
Victor Vlcovici, Simion Sanielevici si Ioan Placinteanu.
n 1945, M. Haimovici a devenit seful catedrei de mecanic
a. Avnd vaste cunostinte de geometrie, ecuatii diferentiale si cu derivate partiale, domenii n care a adus
contributii de valoare, a g
asit acum oportunitatea de a materializa n mecanic
a o
sintez
a a experientei predecesorilor si a sa personal
a. A tinut cursuri de mecanic
a,
hidrodinamic
a si mecanica mediilor continue. A publicat un curs de elasticitate
si a organizat seminarii de specialitate si conferinte nationale. A condus grupul
de mecanicieni de la Institutul de Matematic
a. A condus teze de doctorat, avnd
subiecte din teoria mediilor continue, hidrodinamic
a, magneto-hidrodinamic
a, elasticitate isotropic
a si neisotropic
a n medii omogene si neomogene, termoelasticitate,
plasticitate, medii continue cu microstructur
a si mecanic
a analitic
a.
Aceast
a activitate intens
a care a dus la crearea unei S
coli originale de mecanic
a
este, f
ar
a ndoial
a, rezultatul muncii de o viata a profesorului M. Haimovici. Din
nefericire, activitatea nentrerupt
a, plin
a de abnegatie si permanentul s
au efort de a
satisface toate exigentele au contribuit la sfrsitul s
au prematur.
D
aruirea sa pentru stiinta este concretizat
a n cele dou
a mari realiz
ari ale sale:
Scoala

de Mecanica din Iasi si Institutul de Matematica (care a ren


ascut dup
a 1989).

Acad. Radu MIRON


90

Florica T. Cmpan
Gndim c
a, n luna noiembrie a acestui an 2006,
multe clopote de aur si argint vor suna s
a aminteasc
a
100 de ani de la nasterea Floric
ai T. Cmpan. Poate
nu ntr-o zi anume, c
aci socotitorii de vreme au schimbat
stil vechi pe altul nou, ncurcnd un 13 cu un 26. Poate
nu n clopotnite oficiind ntre cer si p
amnt, nici n sobre aule academice, ci doar n inimi ce adun
a dragostea
cu ratiunea. Poate nu cu frecvente si intensit
ati care
s
a zguduie timpane, ci profund, grav, linistit, care s
a
mngie si s
a ndemne suflete. Gndim c
a str
adania
Recreatiilor Matematice de a marca momentul este justificat
a si va reverbera ecouri.
Multi oameni cred despre ei nsisi c
a stiu s
a nteleag
a,
s
a pretuiasc
a si s
a iubeasc
a tot ce i nconjoar
a: natur
a,
oameni, obiceiuri, gnduri, deveniri, c
arti. De fapt sunt putini; o larg
a majoritate
si concentreaz
a ntelegerea spre agonisiri de folos personal. (Citez ns
a un prieten
matematician din Focsani: Dumnezeu si arata dispretul fata de avere prin cei carora
le-o distribuie). Dintre cei putini care pot s
a nteleag
a, s
a pretuiasc
a si s
a iubeasc
a
sunt tare putini care dovedesc generozitate, capacitate si profunzime comparabile cu
ale Floric
ai.
Florica a nteles si iubit fizica si mai ales matematica, dobndind licente n 1928 si
1929. Din matematic
a a nteles profund si a iubit cu ntelepciune mai ales Geometria,
Fundamentele si Istoria (matematicii). n geometrie si-a mplinit si doctoratul (n
1942, sub bagheta renumitului f
auritor de scoal
a Alexandru Myller ), cu o tez
a despre
suprafete paralele si asemenea, ilustrat
a cu elegante interpret
ari geometrice si fizice.
Nu putem uita aici inspirata carte Aventura geometriilor neeuclidiene. Gnduri
adnci si originale asupra Fundamentelor matematicii s-au nchegat n remarcabila
carte Licuricii din adncuri. Generoasa aplecare spre Istoria matematicii a fost
valorificat
a n numeroase lucr
ari si comunic
ari stiintifice dar si n c
arti ca Istoria
num
arului , Probleme celebre din istoria matematicii.
Florica a nteles si iubit cartea. Semnatarul acestor rnduri nu poate estompa
unele imagini din amintire. Era deja pensionar
a cnd ajungea la usa bibliotecii
Seminarului matematic cu mult naintea harnicei bibliotecare. Se scuza c
a nu a
retinut prea bine niste gnduri din anume c
arti. Auzeam apoi (cu tres
ariri de team
a)
un scrtit de scar
a lung
a, dar subtire (spre a fi usoar
a), spre rafturi nalte. Urca
ner
abd
atoare cu o minuscul
a batist
a feminin
a n mn
a. Dup
a ce scotea o carte, nti
o stergea grijuliu de un b
anuit colb, apoi o mngia cu ochii si cu mna si abia trziu
cobora de pe scar
a (linistindu-mi temeri). Nu accepta s
a fie ajutat
a; era un drept
inalienabil al ei, instaurat nc
a de Alexandru Myller. S
tergea cu aceeasi batist
a (sau
poate cu alta asem
an
atore) si masa pe care aseza cartea. O deschidea ncet, pagin
a
cu pagin
a, si ad
asta ndelung notnd ades ntr-un caiet de elev. Pn
a spre sear
a,
pe mas
a se aduna un teanc de c
arti si de reviste care aproape o ascundeau. La or
a
de nchidere cerea ng
aduinta s
a p
astreze pe acea mas
a si c
arti care s
a le cerceteze
91

a doua zi. Cnd socotea c


a dialogul nu are pe cine tulbura, mp
art
asea si altora
bucuriile ei: iata ce idee grozava a avut X, sau iata ce frumos o spune Y. O sup
arau
stng
aciile: cum poate Z sa scrie despre asta fara a-l cita pe...? Pentru contrast, am
v
azut frecvent multi care se ncruntau cnd descopereau idei prin reviste: cum nu
mi-a venit mie ideea asta?
Mai cu seam
a, Florica a nteles si iubit adolescentii n str
adaniile lor de a se
apropia de matematic
a. Din dragoste fata de ei a fost o excelent
a profesoar
a nti
la Liceul "Oltea Doamna", unde fusese si eleva primei femei din Romnia doctor n
matematic
a, Silvia Creanga. Mai apoi a fost conferentiar si profesor la Facultatea de
matematic
a din Iasi. Din aceeasi dragoste a scris si cunoscutele si mult-cititile sale
c
arti. Altii au conceput c
arti asem
an
atoare nu din dragoste fata de cititor, ci spre a
zgnd
ari glorii; s
a ne mir
am c
a nu au avut ecou? Parc
a spunea cineva c
a dragostea
este ca o minge pe care cu ct o arunci mai departe de tine, revine mai repede napoi.
Un mic dar gingas, semn c
a iesenii nu au uitat-o pe Florica, sunt cele cinci editii
(judetene si interjudetene) ale Concursului de matematica "Florica T. Cmpan". S
i
rndurile de mai sus doresc a semnala adnc
a si nentinat
a pretuire.

Prof. dr. Dan BRNZEI

92

Eclipsele de Soare
Se spune c
a un corp ceresc f
ar
a lumin
a proprie este eclipsat atunci cnd acesta
intr
a n conul de umbr
a al unei planete, fiind astfel lipsit de lumina Soarelui si nu
poate fi v
azut. Asa se produc eclipsele de Lun
a sau eclipsele satelitilor planetelor.
Fenomenul de disparitie a unui astru din cmpul nostru de vedere, cnd ntre acel astru si observator se interpune un alt astru, se numeste ocultatie. Astfel, n miscarea
sa n jurul P
amntului, Luna poate s
a acopere partial sau total Soarele. Fenomenul
produs este impropriu spus eclipsa de Soare, cnd de fapt este vorba de ocultatia
Soarelui de c
atre Lun
a. Trecnd peste aceste mici nentelegeri lingvistice s
a urm
arim cum se produce o eclips
a de Soare.
Disparitia treptat
a a Soarelui din cmpul de vizibilitate are loc atunci cnd
Soarele, Luna si P
amntul sunt aproximativ n vecin
atatea aceleiasi drepte, cnd
au loc eclipse partiale, totale sau inelare. ntruct Luna se roteste n jurul P
amntului de aproximativ 12,368 ori pe an, ar rezulta c
a n fiecare lun
a s-ar produce o
eclips
a de Soare si una de Lun
a. Totusi, nu este asa si iat
a de ce: P
amntul se
roteste n jurul Soarelui pe o elips
a, Soarele aflndu-se ntr-unul din focare (legea a
treia a lui Kepler ). Elipsa, curba plan
a descris
a de P
amnt, este foarte aproape de
un cerc; distanta minim
a a P
amntului fata de Soare (P
amntul la periheliu) este de
147 099 000 km, iar cea maxim
a (P
amntul la afeliu) este de 152 097 090 km. Pe de
alt
a parte, Luna se roteste n jurul P
amntului tot pe o elips
a, planul orbitei Lunii
(planul elipsei) face, ns
a, cu planul orbitei terestre un unghi de 5 grade, 8 minute si
48 secunde, asa c
a, nu la fiecare rotatie cele trei obiecte pot fi coliniare.

n fig.1, SA reprezint
a raza Soarelui, egal
a cu 696 260 km, LB raza Lunii, egal
a
cu 1738 km, iar V un punct situat n vrful conului de umbr
a. Din calcule, rezult
a
c
a n
altimea conului variaz
a n limitele 363 200 km si 375 900 km, n timp ce distanta
de la P
amnt la Lun
a r
amne cuprins
a ntre 363 100 km (perigeu) si 405 500 km
(apogeu). Rezult
a de aici c
a, n cazul n care Luna se afl
a la distanta mai mare de
375 900 km, vrful conului de umbr
a nu poate atinge nici un punct de pe suprafata
P
amntului si, n consecinta, nu poate avea loc nici o eclips
a total
a. Din contra,
93

atunci cnd Luna se afl


a ntre cele dou
a limite mentionate pot avea loc toate tipurile
de eclipse (totale, partiale si inelare), vizibile n mod diferit n functie de locul punctului de observatie.

n fig. 2 se arat
a cazul unei eclipse totale de Soare de o anumit
a durat
a, durat
a ce
depinde de lungimea conului de umbr
a. Durata maxim
a pentru un loc dat nu poate
dep
asi 8 minute, dar poate fi si instantanee.

n fig. 3 este exemplificat


a o eclips
a inelar
a cnd de pe P
amnt Soarele se vede
la un moment dat sub forma unui inel luminos.
Asa cum se poate constata din figurile de mai sus, Luna las
a pe suprafata P
amntului o umbr
a (multimea punctelor de intersectie a conului de umbr
a cu sfera terestr
a)
al c
arui contur este o curb
a ce numai ntr-un singur caz poate fi cerc: atunci cnd
centrele Soarelui, Lunii si P
amntului sunt colineare iar Luna se afl
a la zenit. Diversitatea de manifestare a fenomenului depinde nemijlocit de raza Soarelui si Lunii,
precum si de cele dou
a distante, SL si LV .
n desf
asurarea unei eclipse totale de Soare se deosebesc cinci momente importante. Astfel, momentul n care discul Lunii este tangent discului Soarelui dinspre
partea vestic
a sau, altfel spus, momentul la care ncepe eclipsa partial
a se numeste
94

momentul primului contact exterior. Urmeaz


a acoperirea treptat
a a Soarelui pn
a
cnd acesta nu se mai vede, momentul primului contact interior, dup
a care Soarele
st
a n totalitate un timp, care nu poate fi mai mare de 8 minute. n continuare
discul Lunii devine tangent exterior, moment numit ultimul contact interior, dup
a
care Soarele iese din totalitate. Dup
a momentul cnd cele dou
a discuri devin din
nou tangente exterior, ultimul contact exterior, eclipsa ia sfrsit. Ansamblul tuturor punctelor de pe suprafata P
amntului pe unde trece pata de umbr
a se numeste
banda de totalitate, a c
arei l
atime maxim
a poate ajunge la 260 km, cnd Luna este la
perigeu. n afara benzii de totalitate, unde eclipsa este partial
a, cele dou
a momente
interioare lipsesc.
Determinarea momentelor caracteristice, la care se adaug
a un al cincilea, numit
momentul fazei maxime, ofer
a informatii importante pentru mecanica cereasc
a.
Acum circa 3000 de ani, caldeenii au notat datele la care observau fiecare eclips
a
si au constatat c
a eclipsele de Soare si Lun
a se reproduc n conditii identice ntr-un
interval de timp de 18 ani, 11 zile si 8 ore. Aceast
a perioad
a de repetare a eclipselor
a primit numele de ciclu Saros (saros n limba arab
a nseamn
a repetitie). ntr-un
ciclu Saros au loc 71 de eclipse din care 43 de Soare si 28 de Lun
a. Ulterior, dup
a
mbun
at
atirea mijloacelor de observare si dezvoltarea mecanicii ceresti, s-a constatat
c
a perioada ciclului Saros nu a fost determinat
a cu suficinent
a precizie ceea ce a
condus la abateri mari n calcularea datei celei mai mari s
arb
atori a crestin
at
atii,
s
arb
atoarea Pastelui. Ca exemplu, n anul acesta crestinii catolici au s
arb
atorit
Pastele la data de 16 aprilie, iar ortodocsii la 23 aprilie. Diferentele dintre cele dou
a
date pot dep
asi uneori si o perioad
a sinodic
a a Lunii (perioada sinodica reprezint
a
timpul necesar Lunii ca s
a treac
a prin dou
a faze succesive de acelasi fel si are o durat
a
de 29 zile, 12 ore, 44 minute si 3 secunde).
n ziua de 29 martie a acestui an a avut loc o eclips
a total
a de Soare, vizibil
a
la Iasi ca eclipsa partiala, pentru care momentele caracteristice calculate (pentru Iasi)
sunt: nceputul eclipsei partiale - 12h 50m 53s, faza maxim
a - 14h 02m 02s, sfrsitul
eclipsei partiale 15h 12m 01s, iar acoperirea maxim
a a discului solar a fost de 73,3%.
Chiar si acum observarea eclipselor prezint
a interes stiintific pentru o cunoastere
mai bun
a a drumului parcurs de P
amnt n jurul Soarelui precum si pentru studiul
unor fenomene solare ce nu pot fi observate dect n timpul eclipselor. Toate observatiile f
acute cu mare rigurozitate sunt binevenite.

Iulian BREAHN
A
Membru al Uniunii Astronomice Interna
tionale
Ex-director al Observatorului Astronomic din Ia
si

95

OLIMPIADA 57
Cel care ns
ail
a aceste rnduri socoteste (de multisor) c
a olimpiadele de matematic
a sunt evenimente majore, pline de semnificatii pentru mult
a lume: pentru tinerii
eroi care si ncearc
a puterile n munci mai relevante dect cele ale lui Hercules, pentru
(vesnic tinerii) care slefuiesc cu migal
a subiecte sau evalueaz
a redact
ari, pentru inimosii care asigur
a detalii organizatorice, pentru sponsorii care au avut ntelepciune s
a
plaseze bine b
anuti trebuitori. Lista de mai sus a fost nevoit
a s
a sar
a peste forte care
preg
atesc bune prestatii ale elevilor n scoal
a si acas
a: profesori (n general) care respect
a eforturi, profesori de matematic
a aprinz
atori de scntei, p
arinti care vegheaz
a
iubitori, editori care adun
a str
adanii trecute spre a fi de folos celor care urmeaz
a.
S
a mai spunem c
a apelativul de olimpiad
a s-a conturat prin anii 50, mai ales prin
str
adanii ale neasemuitului Grigore C. Moisil. (n 2006 s-a evocat n lumea larg
a
centenarul nasterii sale.) Exista buna experienta a concursurilor gazetei matematice cu suflet, inim
a si minte donate de Gheorghe Ti
teica. Dar a existat si vnt
potrivnic dinspre r
as
arit care ar fi preferat un nume de spartakiad
a. Institutia care
a asigurat geneza acestor olimpiade a fost Societatea de Stiin
te Matematice si Fizice.
Este benefic s
a ne minun
am de vigoarea cu care au crescut si s-au maturizat
olimpiadele. Erau 50, apoi o sut
a, apoi dou
a de finalisti; acum ar veni (eventual pe
cheltuial
a proprie) cam 10 mii de finalisti; spre a preveni auto-sufoc
ari s-a conturat
prin anul 2000 algoritm prin care s
a nu se treac
a de 600 de participanti.
Buna experienta a olimpiadelor nationale a ndrept
atit Romnia s
a initieze Olimpiade Internationale, apoi Olimpiade Balcanice. Acum finala national
a nu mai apare ca
ultim scop; urmeaz
a ndat
a prime baraje, calific
ari la loturi (de seniori si de juniori),
alte baraje, calific
ari n echipe nationale, variate ntreceri internationale.
Dulcele Trg al Iesilor se ntlnise cu Olimpiada National
a de Matematic
a n 1975.
(A fost atunci creditat la treab
a si semnatarul acestor rnduri.) Au mai existat cereri
de rentlnire; ce sustineau unii, boicotau altii. Iasul se afirmase prin anii 90, n special prin trio-ul de elevi Turinici Gheba Brsan, iar prin 99 prin multiplu campion
Marius Beceanu. Dintre vechi olimpici (dar nu numai) fuseser
a invitati la Comisii
Nationale distinsi profesori ieseni. Exist
a o familie olimpic
a iesean
a; cit
am aici doar
pe ultimul: campionul Pachi. Loturi nationale se preg
atiser
a n Bucium. Intraser
a
n calendar national si Tabere organizate de ieseni. Se aprobase si se desf
asurase cu
succes Concursul National "Al. Myller". Factori diriguitori sustinuser
a Olimpiad
a
National
a de Matematic
a la Iasi n 2005. A fost aprobare, dar s-a acordat prioritate
Olimpiadei Balcanice; se stia c
a Olimpiada National
a din 2006 va fi la Iasi.
A fost si a iesit bine. Climatul intelectual al Iasului a constituit un fond favorabil.
Profesori dotati si inimosi au contribuit cu capacitate si d
aruire. Colegiile Negruzzi
si National au colaborat excelent ca gazde. Facultatea de matematic
a a asigurat gir
academic; presedinte de onoare a fost academicianul Radu Miron. Au fost creditati
ca propun
atori sau evaluatori cam 15 profesori ieseni. A functionat impecabil un
secretariat ce a nsumat cam 30 de profesori destoinici. Elevii ieseni au materializat
ceva din avantajul terenului, cifra pentru olimpiada national
a crescnd de la 16 la 20.
nc
a nu putem face un bilant al comport
arilor internationale, dar suntem optimisti.

Prof. dr. Dan BRNZEI


96

Pseudoinvers
a si invers
a generalizat
a
ale unei aplicatii liniare
Adrian REISNER1
1. Pseudoinvers
a a unui endomorfism ntr-un spatiu vectorial de dimensiune finit
a. Fie S un R-spatiu vectorial de dimensiune finit
a. Are loc
Teorema 1. Fiind date doua endomorfisme u, v ale spatiului S, daca doua din
cele trei conditii urmatoare:
a) uvu = u,

b) vuv = v,

c) rang u = rang v

sunt verificate, atunci a treia este de asemenea verificata.


Demonstratie. a) si b) c) Avem: rang u = rang(uvu) rang(uv) rang v
din conditia a), rang v = rang(vuv) rang(vu) rang u din conditia b). Deducem
conditia c).
a) si c) b) Conditia a) conduce, nmultind la dreapta cu v, la egalitatea
uv = (uvu)v = u(vuv).

()

Tinnd

seama de a), avem, pe de alt


a parte, c
a rang u rang(uv) si, fiindc
a
rang(uv) rang u, deducem egalitatea rang(uv) = rang u. Dar rangul endomorfismului uv este rangul restrictiei endomorfismului u la subspatiul Im v. Aceast
a restrictie u/ Im v este deci injectiv
a si egalitatea () poate fi simplificat
a cu u la stnga,
ceea ce nchide demonstratia.
b) si c) a) rezult
a imediat, endomorfismele u si v avnd roluri simetrice n relatiile a), b), c). Teorema este demonstrat
a.
Corolar. Fiind dat un endomorfism u al spatiului vectorial S, exista un endomorfism v (nu unic) verificnd conditiile a), b) si c).
Demonstratie. Tinnd

seama de teorem
a, ne propunem s
a construim endomorfismul v verificnd conditiile b) si c). Fie v un astfel de endomorfism. Avem
pentru y Im v : vu(y) = y. Deducem c
a, dac
a x Im v Ker u, x = 0 si deci
Im v Ker u = {0}. Suma acestor dou
a spatii vectoriale este deci direct
a . Folosind
conditia rang u = rang v, deducem atunci c
a Im v Ker u = S.
Fiind dat endomorfismul u, fie F un suplementar al subspatiului vectorial Ker u
si G un suplementar al spatiului Im u. Aplicatia u0 : F Im u, x 7 u(x) este un
izomorfism. Definim aplicatia v prin restrictiile sale la cele dou
a subspatii suplementare Im u si G prin:
v/ Im u = (u0 )1 ,
v/G = 0.
Aceast
a aplicatie v este evident liniar
a si verific
a:
Im v = Im(u0 )1 = F , deci rang v = dim F = rang u si rezult
a c
a v verific
a c);
x S, v(x) F , deci vuv(x) = v {u0 [v(x)]} = (vu0 )[v(x)] = v(x), i.e. v
verific
a vuv = v, adic
a b).
Corolarul este astfel stabilit.
1

Cercet
ator, Centrul de Calcul E.N.S.T., Paris

97

Observatie. Endomorfismul v nu este unic: el depinde de alegerea subspatiilor


F si G din demonstratia corolarului.
Definitii. Fiind dat endomorfismul u, orice endomorfism verificnd conditia a) se
numeste inversa generalizata a lui u. Fiind dat endomorfismul u, orice endomorfism
verificnd conditiile a), b) se numeste pseudoinversa a lui u.
Cu aceleasi notatii ca n corolarul precedent, o invers
a generalizat
a v a endomorfismului u este definit
a prin restrictiile sale la subspatiile suplementare Im u si G
prin:
v/ Im u = (u0 )1 ,
v/G morfism oarecare apartinnd spatiului L(G, S).
O invers
a generalizat
a a endomorfismului u depinde deci de alegerea subspatiilor F ,
G si de alegerea morfismului v/G . Vom numi Ing(u) = {v L(S) |uvu = u}.
Observatii. 1) Inversa generalizat
a a unui automorfism u este unic
a si coincide

cu inversa u1 a lui u : Ing(u) = u1 .


2) Fie v Ing(u). Avem:
endomorfismul w = uv este un proiector (w2 = w) de imagine Im u:
w2 = (uv)(uv) = (uvu)v = uv = w,
endomorfismul w0 = vu este un proiector (w02 = w0 ) de nucleu Ker u:
w02 = (vu)(vu) = v(uvu) = vu = w0 .
Aplicatie. Vom nota cu aceeasi liter
a un endomorfism al spatiului vectorial
Rn si matricea acestui endomorfism n baza canonica a lui Rn . Fiind date trei
endomorfisme A, B, C ale spatiului vectorial Rn , fie A Ing(A) si C Ing(C). Are
loc
Teorema 2. Ecuatia matriceala AXC = B admite o solutie X daca si numai
CC
= B.
daca avem AAB
CC
= B, deducem c
C
Demonstratie. Presupunnd c
a AAB
a matricea X = AB

verific
a AXC = B. Vom nota X0 = AB C.

=
Invers, dac
a exist
a X L(Rn ) verificnd AXC = B, atunci avem (AAA)X(C
CC)

B si, n consecinta, B = AA(AXC)CC = AAB CC.


A si C fiind dou
a matrice oarecare apartinnd respectiv la Ing(A) si Ing(C),
not
am
n
o

K(A, C) = Y AAY
C C | Y Mn (R) .

C +
Teorema 3. Solutia generala a ecuatiei matriceale AXC = B este X = AB
Z, unde Z K(A, C).
Demonstratie. Solutia general
a a ecuatiei matriceale AXC = B este X =
C (v. demonstratia Teoremei 2) si Z este solutia general
X0 + Z, unde X0 = AB
aa
ecuatiei AZC = O. Ne propunem s
a justific
am echivalenta urm
atoare:
a) AZC = O

a) b) Z = Z O = Z A(AZC)
C;
98

b) Z K(A, C).


b) a) Y Mn (R), matricea Z = Y AAY
C C verific
a

= AY C (AAA)Y

=O
AZC = AY C AAAY
C CC
(C CC)

si Teorema 3 este demonstrat


a.

Exemplu. Fie ecuatia matriceal


a


0 1 0
1 0 0
k
1 0 0 X 0 0 0 = 0
0 0 0
0 0 0
0

S
a g
asim solutia
general
a
0 1
Avem: A = 1 0
0 0

0 0
0 0 , unde k R.
0 0

(1)

X
M3 (R) a
acestei ecua
tii.

0
0 1 0
1 0 0
0 si A = 1 0 0 Ing(A); C = 0 0 0
0
0 0 0
0 0 0

k 0 0
CC,
ecuatia (1) admite solutii,
si C = C Ing(C). Cum B = 0 0 0 = AAB
0 0 0

0 0 0
C = k 0 0. Tinnd
o solutie fiind matricea X = AB

seama de Teorema 3,
0 0 0
solutia general
a a ecuatiei (1) este

0 0 0
C + Y AAY

X = AB
C C = k 0 0 + Y AAY
C C,
0 0 0
unde Y este o matrice oarecare apartinnd spatiului de matrice M3 (R).
a, b, c, d, e, f , g, h, i numere reale oarecare, obtinem,
Considernd

a b c
0
d e f , c
a solutia general
a a ecuatiei matriceale (1) este X = k
g h i
g
unde b, c, e, f , g, h, i sunt numere reale oarecare.

cu
b
e
h

Y
=
c
f ,
i

2. Pseudoinversa unui endomorfism ntr-un spatiu euclidian. E fiind


un spatiu euclidian, produsul scalar va fi notat h, i. [Aceleasi definitii si rezultate
sunt valabile dac
a spatiul E este un spatiu hermitian, adic
a un C - spatiu vectorial
nzestrat cu un produs scalar, form
a hermitian
a pozitiv definit
a pe E.] Amintim
definitia si teorema urm
atoare:
Definitie si teorem
a. Fiind dat un endomorfism al spatiului euclidian E, exista
un singur endomorfism u al spatiului E verificnd
x, y E,

hu(x), yi = hx, u (y)i .

Acest endomorfism se numeste adjunctul endomorfismului u.


Avem Ker u = (Im u) si Im u = (Ker u) .
Demonstr
am c
a Ker u = (Im u) ; ntr-adev
ar,
y (Im u) x E, hu(x), yi = 0 x E, hx, u (y)i = 0 y Ker u .

Asem
an
ator proced
am pentru egalitatea Im u = (Ker u) .
99

Fie u un endomorfism al spatiului euclidian E . Not


am K = (Ker u) ortogonalul
nucleului lui u si I = (Im u) ortogonalul imaginii endomorfismului u.
Not
am P proiectia ortogonal
a de imagine K (de nucleu Ker u), P 0 proiectia
ortogonal
a de imagine Im u (de nucleu I = (Im u) ). Avem: P 0 u = u = uP ;
P 0 (y) Im u, y E; u(1) P 0 (y) (imaginea reciproc
a a elementului P 0 (y)) este
o clas
a modulo Ker u a spatiului euclidian E (atentie: u(1) nu este o aplicatie!).
Imaginea acestei clase prin proiectia P este un element unic al spatiului K.
Definitie. Aplicatia liniar
a u+ = P u(1) P 0 se numeste pseudoinversa a endomorfismului u.
Teorema 4. Aplicatia pseudoinversa u+ a endomorfismului u verifica relatiile:
a) uu+ u = u,
b) u+ uu+ = u+ ,
c) uu+ si u+ u sunt endomorfisme ortogonale, i.e. (uu+ ) = uu+ , (u+ u) = u+ u.
Mai mult, u+ este singurul endomorfism al spatiului E verificnd asertiunile a), b)
si c).
Demonstratie. P si P 0 fiind proiectori, avem P u+ = u+ P 0 = u+ . Atunci:
u+ u = P u(1) P 0 u = P u(1) u = P, proiector ortogonal,
uu+ = uP u(1) P 0 = uu(1) P 0 = P 0 , proiector ortogonal,
si, n consecinta, uu+ u = uP = u; u+ uu+ = u+ P 0 = u+ . n plus,
Im u+ = Im P = K = (Ker u) = Im u ,

Ker u+ = Ker P 0 = I = (Im u) = Ker u .

Demonstr
am acum unicitatea endomorfismului u+ . Fie u0 un alt endomorfism
verificnd conditiile a), b) si c). Avem:
u0 = u0 uu0 = u0 u0 u = u0 u0 (u u+ u ) = u0 (uu0 )(uu+ ) = u0 uu+ ;
u0 uu+ = u u0 u+ = (u u+ u )u0 u+ = (u+ u)(u0 u)u+ = u+ uu+ = u+ .
Deci u0 = u+ , c.c.t.d.
Observatie. Am v
azut n prima parte c
a o pseudoinvers
a a lui u depinde n
particular de alegerea subspatiilor suplementare F si G ale spatiilor Ker u si respectiv
Im u. n cazul n care E este un spatiu euclidian F si G sunt unic definite prin
F = (Ker u) si G = (Im u) , de unde unicitatea pseudoinversei.
Dac
a y este un vector oarecare al spatiului euclidian E, not
am x0 = u+ (y). Avem
Teorema 5. Vectorul x0 = u+ (y) verifica:
a) kux0 ykeste minimum, i.e. kux0 yk kux yk, x E;
b) kx0 k este minim din toate elementele
kux yk = kux0 yk.
+ x verificnd

0
Demonstratie. a) kux0 yk = uu y y = kP y yk,
2

kux yk2 = ku(x x0 ) + ux0 yk2 = ku(x x0 ) + P 0 y yk =


2

= ku(x x0 )k2 + kP 0 y yk kux0 yk2 .

b) Avem kux yk = kux0 yk numai cnd u(x x0 ) = 0. n cazul acesta,


2
2
2
2
x = x0 + z, unde x0 K = (Ker u) si z Ker u. Deci kxk = kx0 k + kzk kx0 k
si deducem b).
100

Observatie. Din teorema precedent


a deducem c
a x0 = u+ (y) este cea mai bun
a
solutie apropiat
a n norm
a a ecuatiei y = ux.
Pseudoinversa unei matrice. Dac
a alegem n spatiul euclidian E o baz
a
ortonormat
a B, obtinem U -matricea endomorfismului u n aceast
a baz
a: M at(u ,B) =
T
U = U T .[n cazul n care E este hermitian avem M at(u , B) = U = U .]
Traducnd cele de mai sus n limbaj matriceal, obtinem
Fiind data o matrice U Mn (R), exista o singura matrice V verificnd conditiile
urmatoare:
U V U = U,

V U V = V,

(U V ) = U V,

(V U ) = V U.

Aceasta unica matrice V se numeste pseudoinversa matricei U si se noteaza U + .


X0 = U + Y este cea mai bun
a aproximatie cuadratic
a a ecuatiei U X = Y .
Exemplu de calcul al pseudoinversei unei matrice. Fie matricea

1 0 1
U = 0 1 0 M3 (R).
1 0 1
S
a calcul
am matricea U + .
Matricea U fiind simetric
a, avem Ker u = (Im u) .
Fie B = {e1 , e2 , e3 } baza canonic
a a spatiului R3 nzestrat cu produsul scalar
canonic. Pentru endomorfismul u asociat matricei U n aceast
a baz
a, avem:
Im u = Vect {e2 , e1 + e3 } ,

Ker u = R(e1 e3 ).

Im v = (Ker u) = Im u.

Endomorfismul v de matrice U

n baza canonic
a B verific
a atunci

Ker v = (Im u) = Ker u;

Pe de alt
a parte:
uvu(e2 ) = u(e2 ) uv(e2 ) = u(e2 ) v(e2 ) = e2 + (e1 e3 ); v(e2 ) Im v =
Im u = 0, deci v(e2 ) = e2 ;
e1 + e3
e1 + e3
uvu(e1 ) = u(e1 ) uv(e1 + e3 ) = e1 + e3 = u(
) v(e1 + e3 ) =
+
2
2
e1 + e3
(e1 e3 ); v(e1 + e3 ) Im u = 0, deci v(e1 + e3 ) =
. Dat fiind c
a
2
e1 + e3
v(e1 e3 ) = 0 [Ker v = Ker u], deducem c
a v(e1 ) = v(e3 ) =
.
4
Finalmente matricea U + este

1 0 1
1
U + = 0 4 0 .
4
1 0 1

101

O solutie partial
a a unei probleme a lui N. Papacu
Cat
alin TIG

AERU 1
Profesorul Nicolae Papacu analizeaz
a n [1] recurenta p
atratic
a, subliniind faptul c
a natura sirului este dictat
a de alegerea termenului initial si las
a ca problem
a
deschis
a cazul (III.3.d). Ne propunem s
a d
am un r
aspuns partial acestei probleme.
I. O problem
a deschis
a si cteva rezultate generale. Recurenta
xn+1 = ax2n + bxn + c, x0 R, a 6= 0,
b
se reduce, cu substitutia yn = axn + , la recurenta yn+1 = yn2 + , unde =
2
1
a ca problem
a deschis
a, notat
a (III.3.d)
(2b ), cu = b2 4ac. Situatia propus
2
n articolul citat, se refer
a la cazul n care < 0 si y0 (L2 , L2 ) \ {L1 }, unde

1
a punctele fixe ale recurentei. Dac
a punem =
L1,2 =
1 1 4 reprezint
2
2
amne de studiat recurenta
, atunci ne r

y0 [L2 , L2 ] , yn+1 = f (yn ) , n N, unde f : R 2 , , f (x) = x2 2 , (1)


este functia atasat
a recurentei, care este surjectiv
a, strict descresc
atoare pe ( , 0] ,
strict cresc
atoare pe [0, ) si care are punctele fixe

p
p
1
1
(2)
L1 =
1 1 + 4 2 , L2 =
1 + 1 + 4 2 .
2
2
Introducem notatia: dac
a g : M M , atunci g g . . . g = g [n] , unde
compunerea s-a efectuat de n ori. Astfel, constat
am c
a recurenta (1) se mai scrie si
[n]
y0 [L2 , L2 ] , yn+1 = f (y0 ) , n N.
(10 )
Analiza care urmeaz
a va stabili urm
atoarele tipuri distincte de comportamente ale
sirurilor studiate: (i) siruri convergente, care nu sunt constante de la un loc ncolo;
(ii) siruri convergente, constante de la un loc ncolo; (iii) siruri m
arginite, f
ar
a limit
a,
cu multimea valorilor finit
a; (iv) siruri m
arginite, f
ar
a limit
a, cu multimea valorilor
infinit
a; (v) siruri nem
arginite, cu limita egal
a cu .
O conditie suficient
a, care s
a asigure m
arginirea sirului (yn )nN , este dat
a de

Propozitia 1. Daca 0 < 2, atunci sirul (yn )nN este marginit.

Demonstratie. Pornim cu observatia c


a f ([L2 , L2 ]) = 2 , L2 ; dac
a

L2 2 (), atunci 2 , L2 [L2 , L2 ], de unde

f 2 , L2 = f 2 , 0 [0, L2 ] = f ([0,L2 ]) = 2 , L2 .

ar,
Demonstr
am, prin inductie, c
a f [n] ([L2 , L2 ]) = 2 , L2 , n N . n adev

dac
a presupunem c
a f [n1]([L2 , L2 ]) = 2, L2 , atunci

f [n] ([L2 , L2 ]) = f f [n1] ([L2 , L2 ]) = f 2 , L2 = 2 , L2 .

Deducem c
a f [n] ([L2 , L2 ]) = 2 , L2 , n N , adic
a, dac
a este ndeplinit
a
conditia (), atunci sirul (yn )nN este m
arginit. Rezolvnd inegalitatea si tinnd cont

a.
de > 0, obtinem 0 < 2 si demonstratia este ncheiat
1

Lect. dr., Universitatea S


tefan cel Mare, Suceava

102

Tinnd

cont
artim analiza noastr
a n dou
a cazuri mari:
de acest rezultat, mp
2 si > 2. Teoremele materialului prezent aduc l
amuriri doar n primul caz,
n al doilea limitndu-ne la cteva comentarii si la formularea unei conjecturi.
Lema 1. Daca (yn )nN este sirul definit de (1), atunci yn > 0, n N, daca si
numai daca y0 = L2 .
Demonstratie. S
a presupunem prin reducere la absurd c
a exist
a (yn )nN , definit
de (1) astfel nct yn > 0, n N, cu y0 (0, L2 ). Demonstr
am c
a, n aceast
a
ipotez
a, sirul ar fi strict descresc
ator: n adev
ar, dac
a y0 (0, L2 ), atunci, deoarece
f (x) < x, x (0, L2 ), ar rezulta c
a y1 < y0 , de unde, tinnd cont de faptul c
af
este strict cresc
atoare pe (0, ), s-ar deduce, prin inductie, c
a yn+1 < yn , n N.
Mai departe, ar rezulta c
a sirul este covergent, deci limita ar fi unul din punctele
fixe L1 sau L2 . Cum sirul este strict descresc
ator, s-ar deduce c
a lim yn = L1 < 0,
n
ceea ce contrazice faptul c
a yn > 0, n N. Constatarea faptului c
a, dac
a y0 = L2 ,
atunci yn = L2 , n N, ncheie demonstratia.
Lema 2. Fie (yn )nN sirul definit de (1); daca > 1, atunci yn < 0, n N,
daca si numai daca y0 = L1 .
Demonstratie. Consider
am un invers la dreapta al functiei f :
p
2

h : , L2 [L2 , 0] , h (x) = x + 2 ,
(3)
2

unde > 1; evident h satisface (f h) (x) = x, x , L2 . Deoarece > 1,


deducem c
a 2 < < L1 < 0, de unde rezult
a c
a h ([, 0]) [, 0]. Aceast
a
constatare permite construirea sirului (n )nN , definit astfel:
0 = 0,

n+1 = h (n ) , n N.

Tinnd

cont de faptul c
a h este strict descresc
atoare si de faptul c
a L1 este unicul
punct fix al functiei h, se demonstreaz
a prin inductie c
a
= 1 < 3 < < 2n1 < < L1 < < 2n < < 2 < 0 = 0

a cum urmeaz
a:
si c
a lim n = L1 . Definim sirul de intervale dup
n

I0 = (1 , 3 ), In = (2n1 , 2n+1 ), n N si J0 = (2 , 0 ), Jn = (2n+2 , 2n ), n N.


Constatnd c
a h (In ) = Jn , n N , h (Jn ) = In+1 , n N, obtinem (h h)[n] (I0 ) =
[n]
In , (h h) (J0 ) = Jn , ceea ce conduce la
(f [2n1] ) (In ) = J0 , (f [2n] ) (Jn ) = J0 ,

n N .

(4)

Dac
a y0 [L2 , 0), atunci avem de analizat situatiile:
(a) y0 [L2 , ); atunci y1 = f (y0 ) > 0;
[m1]
(b) y0 {n | n N}; atunci
exist
a un
(0), de
m N astfel ca y0 = m = h
[m1]
[m1]
unde obtinem c
a ym = f
(0) = 0; mai departe ajungem la ym+2 > 0.
h
(c) Dac
a y0 [, 0] \ {{n | n N} {L1 }},
a y0 < L1 , exist
a
atunci: (c.1) dac
un m N astfel nct y0 Im , deci y2m1 = f [2m1] (y0 ) J0 , de unde y2m

f (J0 ) = 2 , , ceea ce conduce la y2m+1 f 2 , (0, L2 ); (c.2) dac


a
[2m]
y0 > L1 , exist
a un m N astfel nct y0 Jm , deci y2m = f
(y0 ) J0 , de unde
y2m+2 (0, L2 ).
103

S-a demonstrat c
a, dac
a y0 [L2 , 0] \ {L1 }, atunci m N astfel nct y0 > 0.
Deoarece, dac
a y0 = L1 , atunci yn = L1 < 0, n N, deducem c
a yn < 0, n N,
dac
a si numai dac
a y0 = L1 .
Urm
atorul rezultat marcheaz
a locul aparitiei sirurilor periodice.
Propozitia 2. Daca 1 , atunci:
a) ecuatia f (x) = h (x) are o unica solutie pozitiva a [0, );
b) daca y0 = a, atunci sirul (yn )nN , definit de (1), este periodic, cu y2n = a > 0,
y2n+1 = f (a) < 0.
p
2
2
Demonstratie. a) Fie functia g (x) = f (x) h (x) = x2 + x +
p , ce
2
2
este strict cresc
atoare pe [0, ). Deoarece g (0) = 0, g () = + > 0
(egalitatea apare pentru = 1), deducem c
a ecuatia g (x) = 0 are o solutie unic
a
a [0, ) [0, L2 ).

b) se deduce din (f h) (x) = x, x 2 , L2 . n adev


ar, dac
a y0 = a, atunci
y2 = f (f (a)) = f (h (a)) = a, de unde, prin inductie obtinem y2n = a, n N.
Analog vom obtine y2n+1 = f (a), n N . q.e.d.
II. Analiza cazurilor corespunz
atoare lui 0 < 1.
1. Cazul 0 < < 1. Comportamentul sirului (1)este descris de
L1 ; y0 (L2 , L2 ) ,
Teorema 1. Daca 0 < < 1, atunci lim yn =
.
L2 ; y0 {L2 , L2 } .
n
Demonstratie. n primul rnd,
demonstr
am c
a, dac
a y0 (L2 , L2 ), atunci

exist
a n0 2 astfel nct yn 2 , 0 , n n0 . n adev
ar, dac
a y0 [, ],
2
2
atunci y2 , 0 ; demonstr
am prin inductie c
a yn , 0 , n 2: dac
a

2
2
yn1 2 , 0 , atunci yn = yn1
2 4 2 = 2 2 1 < 0; cum yn = yn1

2 2 , obtinem yn 2 , 0 , n 2. S-a demonstrat c


a, dac
a y0 [, ],
2
atunci yn , 0 , n 2.
Dac
a yn (L2 , ) (, L2 ), din Lema 1 deducem c
a n0 astfel ca yn0 < 0.
Deoarece y1 > 0,rezult
a c
a n0 2. Pentru c
a yn0 = f (yn0 1
), cu n0 1 1,
deducem c
a yn0 2 , 0 , de unde, ca mai sus, rezult
a c
a yn 2 , 0 , n n0 .
a, dac
a y0 (L2 , L2 ), atunci exist
a n0 2 astfel nct yn
Am
demonstrat c
2 , 0 , n n0 . Mai departe, dac
a y0 (L2 , L2 ) \ {L1 }, atunci, deoarece f este

strict descresc
atoare pe 2 , 0 , obtinem, prin inductie, urm
atoarele concluzii:
2

(a) n cazul yn0 , L1 , avem m N


yn0 < yn0 +2 < < yn0 +2m < < L1 < < yn0 +2m+1 < < yn0 +3 < yn0 +1 ;
(b) n cazul yn0 (L1 , 0), avem m N

yn0 +1 < yn0 +3 < < yn0 +2m+1 < < L1 < < yn0 +2m < < yn0 +2 < yn0 ;

a y0 = L1 , atunci yn = L1 , n N,
n ambele cazuri se deduce lim yn = L1 . Dac
n

deci, din nou lim yn = L1 . Tinnd

cont c
a, dac
a y0 {L2 , L2 }, atunci yn =
n
L1 , n 2, deci lim yn = L2 , demonstratia se ncheie.
n

2. Cazul = 1. Conform Propozitiilor 1 si 2, vor apare siruri m


arginite, f
ar
a
limit
a. Pentru a enunta rezultatul, construim multimea M0 astfel: pentru fiecare n
104

S
N , consider
am multimile M0n = x [L2 , L2 ] | f [n] (x) = 0 si fie M0 =
M0n .
n=1

2n
S
a not
am faptul c
a, pentru fiecare n N , avem
M02n1
0 M0 , {1}
. Mai
[n]
mult, dac
a M0 , atunci avem si incluziunea x [L2 , L2 ] | f (x) = M0 .

Teorema 2. Daca = 1, atunci sirul (yn )nN este convergent daca si numai
L1 ; y0 (L2 , L2 ) \ M0 ,
daca y0 [L2 , L2 ] \ M0 si lim yn =
L2 ; y0 {L2 , L2 } .
n
Daca y0 M0 , atunci (yn )nN nu are limita si exista n1 N astfel nct
1
yn = ((1)n 1), n n1 .
2
Demonstratie. Se demonstreaz
a prin inductie c
a, dac
a y0 (1, 1), atunci yn
(1, 0), n 1. Mai departe, judecnd ca n Teorema 1, deducem c
a lim yn = L1 .
n

S
a presupunem c
a y0 (L2 , 1) (1, L2 ) \ M0 ; conform Lemei 1, exist
a n0 2
astfel ca yn0 < 0. Deoarece y0
/ M0 , rezult
a c
a f [n0 ] (y0 ) 6= 0, deci yn0 +1 6= 0, de
unde 1 < yn0 < 0. La fel ca mai sus, obtinem lim yn = L1 .
n

Dac
a y0 M0 , atunci exist
a n0 N astfel nct yn0 +1 = f [n0 ] (y0 ) = 0; n
1
continuare obtinem yn = ((1)n 1), n n1 = n0 + 1 si demonstratia se ncheie.
2

3. Cazul 1 < 2. Pentru fiecare n N , consider


am multimile

n
o
[
[
n
[n]
n
ML1,2 = x [L2 , L2 ] | f (x) = L1,2 , ML1 =
ML1 , ML2 =
MLn2 .
n=1

n=1

a ML1 ML2 = .
S
a not
am deocamdat
a c
a L1 ML1 , L2 ML2 si c

Teorema 3. Daca 1 < 2, atunci sirul (yn )nN este convergent daca
si numai daca y0 ML1 ML2 si lim yn = L1 daca y0 ML1 si = L2 daca
n
y0 ML2 .
Daca y0 [L2 , L2 ] \ {ML1 ML2 }, atunci sirul (yn )nN este marginit, nu are
limita si contine o infinitate de termeni negativi si o infinitate de termeni pozitivi.
Mai mult, exista valori ale lui y0 pentru care multimea valorilor sirului este finita si
valori ale lui y0 pentru care multimea valorilor sirului este infinita.
Demonstratie. S
a presupunem c
a y0 [L2 , L2 ] \ {ML1 ML2 } si s
a presupunem prin absurd c
a exist
a n0 N astfel ca yn > 0, n n0 . Atunci putem
considera sirul (
yn )nN definit astfel
y0 = yn0 , yn+1 = f (
yn ) , n N .
()
Deoarece yn = yn+n0 > 0, n N, deducem, pe baza Lemei 1, c
a y0 = L2 ,
adic
a yn0 = L2 ; dar aceasta conduce la y0 ML2 , fals. Presupunem prin absurd
c
a exist
a n0 N astfel ca yn < 0, n n0 . Lund n considerare sirul (
yn )nN
definit de (), deducem c
a yn = yn+n0 < 0, n N, ceea ce, n virtutea Lemei 2,
conduce la y0 = L1 , adic
a yn0 = L1 , aceasta nsemnnd c
a y0 ML1 , fals din nou.
S-a demonstrat astfel c
a, dac
a y0 [L2 , L2 ] \ {ML1 ML2 }, atunci sirul (yn )nN
este m
arginit, nu are limit
a si contine o infinitate de termeni pozitivi si o infinitate
de termeni negativi. Dac
a y0 ML1 ML2 , deducem c
a lim yn {L1 , L2 }.
n
Propozitia 2 semnaleaz
a faptul c
a, n acest caz, exist
a siruri periodice sau, mai
general, siruri cu multimea valorilor finit
a. Demonstr
am n continuare c
a putem alege
105

y0 astfel nct multimea valorilor sirului s


a fie infinit
a. Pentru aceasta, consider
am
[X], unde R [X] este inelul polinoamelor cu coeficienti reali si
aplicatia A : N
= N N \ {(n, n) | n N}, definit
N
a prin A (n1 , n2 ) = f [n1 ] (X) f [n2 ] (X) R [X].
Consider
am multimea S care are ca elemente toate r
ad
acinile din intervalul [L2 , L2 ]
ale polinoamelor ce apartin imaginii functiei A. Cum S este num
arabil
a, alegem
y0 = b [L2 , L2 ] \ S; demonstr
am c
a functia Y : N [L2 , L2 ], Y (n) = yn este
injectiv
a: n adev
ar, dac
a ar exista n1 , n2 N, n1 6= n2 , astfel ca Y (n1 ) = Y (n2 ),

atunci am avea f [n1 ] (b) = f [n2 ] (b), ceea ce ar nsemna c


a polinomul g A(N),
g (X) = f [n1 ] (X) f [n2 ] (X), are r
ad
acina b S, ceea ce este fals. Q.e.d.

III. Complet
ari si comentarii finale. 1. Propozitia 2 asigur
a, pentru 1 ,
existenta sirurilor periodice, de perioad
a 2. F
ar
a a intra n detalii, preciz
am doar c
a
se pot construi siruri care au perioade mai mari dect 2.

2. Pentru = 2 se cunoaste forma analitic


a a termenului general, aceast
a
situatie f
acnd obiectul mai multor probleme de concurs (vezi [1], [3] ).
3. n cazul > 1, putem constata: (a) dac
a y0 ML1 ML2 , atunci lim yn = L1
n
dac
a y0 ML1 si = L2 dac
a y0 ML2 .
(b) Conform propozitiei 2, exist
a siruri periodice sau siruri periodice de la un loc
ncolo, deci siruri m
arginite, cu un num
ar finit de valori.
a aib
a
(c) Exist
a posibilitatea de a alege y0 [L2 , L2 ] astfel nct sirul (yn )nN s
limita egal
a cu ; de exemplu, dac
a este y0 = 0, atunci y1 = 2 , y2 = 4 2 > L2 ,
de unde lim yn = . Ceea ce nu putem preciza este dac
a exist
a posibilitatea de
n

a alege y0 [L2 , L2 ], astfel nct sirul s


a fie m
arginit si multimea valorilor sirului
infinit
a. n leg
atur
a cu acest subiect propunem
conjectura:

p
p
1
Sirul

(yn )nN , definit de recurenta y0 1 + 1 + 4 2 , 1 + 1 + 4 2 ,


2
2

yn+1 = yn2 2 , > 2, este marginit daca si numai daca este periodic de la un loc
ncolo.
4. Sintetiz
am concluziile articolului
Valorile lui
(0, 1) ;
= 1;

1, 2 ;
2, ;

n urm
atorul tabel:
Tipul de siruri
(i) , (ii) ;
(i) , (ii) , (iii) ;
(ii) , (iii) , (iv) ;
(ii) , (iii) , (v) , (iv?) .

5. Dl prof. Gheorghe Marchitan a citit cu atentie materialul, mbun


at
atindu-l
prin observatii. Acelasi efect benefic l-au avut si discutiile purtate cu dl prof. Marius
Marchitan. Amndurora, autorul le multumeste si pe aceast
a cale.
Bibliografie
1. N. Papacu - Asupra unui sir recurent, Arhimede, 5-6/2000, 1-4.
2. Gh. Oprisan - On a class of real sequences defined by recurence, Gazeta Matematic
a
(seria informare stiintific
a si perfectionare metodic
a), XIV(XCIII), 2/1996, 82-90.
3. C. Tig
aeru - Recurenta an = 2a2n1 1 si problema XI.3 a Concursului Revistei
"Sinus", Sinus, II.1(4)/2006.
106

Criterii de congruenta
a triunghiurilor
Marius APETRII 1 si Cristian-Catalin BUDEANU 2
n RecMat - 2/2005, elevilor de clasa a VII-a le este propus
a urm
atoarea problem
a:
VII.63. Fie triunghiurile ABC si A0 B 0 C 0 cu AB = A0 B 0 , AC = A0 C 0 si bisectoarele [AD], [A0 D0 ] congruente. Sa se arate ca 4ABC 4A0 B 0 C 0 (enunt adaptat).
Petru Asaftei, Iasi
n cele ce urmeaz
a, dorim s
a stabilim conditii suficiente pentru congruenta a doua
triunghiuri care au dou
a bisectoare congruente. Vom considera mereu ndeplinit
a
ipoteza
A
(I) Triunghiurile ABC si A0 B 0 C 0 au bisectoarele [AD]
si [A0 D0 ] congruente, cu D (BC), D0 (B 0 C 0 ), iar
0 D 0 B 0 ) 90 .
\ 90 , m(A\
m(ADB)
0 A0 C 0 s
\ B\
Propozitia 1. n ipoteza (I), daca BAC
i
0 0 0
0
0
0
\
\
BDA B D A , atunci 4ABC 4A B C .
Demonstratie. Din congruenta triunghiurilor ADB si
0 B 0 D0 ,
\ A\
A0 D0 B 0 (U.L.U.) rezult
a c
a AB = A0 B 0 si ABD
de unde concluzia urmeaz
a conform U.L.U.

0 A0 C 0 s
\ B\
Propozitia 2. n ipoteza (I), daca BAC
i
0 0
0 0 0
D
B
C
AB = A B , atunci 4ABC 4A B C .
0 0 0
\
Demonstratie. Obtinem c
a 4ABD 4A B D (U.L.U.), de unde ABC
0 B 0 C 0 . Folosind U.L.U., deducem c
a 4ABC 4A0 B 0 C 0 .
A\
0 A0 C 0 s
\ B\
Propozitia 3. n ipoteza (I), daca BAC
i BD = B 0 D0 , atunci
0 0 0
4ABC 4A B C .
0 A0 D 0 , AD = A0 D 0 , BD = B 0 D 0
\ B\
\
Demonstratie. Deoarece BAD
si ADB,
0 0 0
0
0
0
\
a c
a 4ADB 4A D B si
A D B sunt ambele neobtuze, conform L.L.U. rezult
acum concluzia este imediat
a.
0 B 0 C 0 , atunci
\ A\
Propozitia 4. n ipoteza (I), daca AB = A0 B 0 si ABC
4ABC 4A0 B 0 C 0 .
0B0C 0
\ = A\
\
Demonstratie. Deoarece AB = A0 B 0 , AD = A0 D0 , ABC
si ADB,
0
0
0
0
0
D B sunt ambele neobtuze, conform L.L.U. rezult
a c
a 4ADB 4A D B 0 , de
A\
unde concluzia anuntat
a.
0 D0 B 0 s
\ A\
i BC = B 0 C 0 , atunci
Propozitia 5. n ipoteza (I), daca ADB
0 0 0
4ABC 4A B C .
Demonstratie. Consider
am punctul A00 n acelasi semiplan cu A fata de BC,
00
0 0 0
00 C, D 00 (BC), iar
\
astfel nct 4A BC 4A B C . Fie [A00 D00 bisectoarea lui BA
00 D 00 B rezult
\ A\
{E} = A00 B AD, {F } = A00 D00 AC. Din ADB
a c
a AD k A00 D00
1
2

Asist. drd., Facultatea de Matematic


a, Univ. Al. I. Cuza, Iasi
Profesor, C. N. Emil Racovita, Iasi

107

si cum AD = A00 D00 , obtinem c


a ADD00 A00 este paralelogram. Astfel, AA00 k BC si
00
00 CB) m(ACB)
00 BC) sau
\ si m(ABC)
\ m(A\
deci A Ext ABC, de unde m(A\
invers; ne plas
am n prima situatie.
[
A
A
Din AD k A00 D00 obtinem c
a m(EAC)
=
00
00
00
\
\
\
m(AF A ) = si m(AEA ) = m(EA F ) = .

\ = 180 (180 ) = si
Atunci m(ABE)
E F
00 CF ) = 180 (180 ) = . Cum
\
m(A
00
\
0, deducem c
a = . Astfel, m(A
CA) = 0,
00
00
\
deci C, A , A sunt coliniare si m(ABA ) = 0, deci
C
D D
B, A00 , A sunt coliniare. n concluzie, A00 = A, de B
0 0 0
unde 4ABC 4A B C .

Propoztia 6. n ipoteza (I), daca AB = A0 B 0 si AC = A0 C 0 , atunci 4ABC


4A0 B 0 C 0 . (Problema VII.63 din RecMat - 2/2005)
C
C
Demonstratia 1. Consider
am punctul C 00 n ace00
lasi semiplan cu C fata de AB, astfel nct 4ABC
\00 , D00
4A0 B 0 C 0 . Fie [AD00 bisectoarea lui BAC
D
D
BD
AB
00
a
(BC ). Din teorema bisectoarei avem c
=
CD
AC
00
AB
BD
00
si cum AC = AC, deducem c
a
si 00 00 =
C D
AC 00
B
A
00
BD
BD
= 00 00 , deci DD00 k CC 00 . Triunghiurile ACC 00 si ADD00 sunt isoscele si
CD
C D
atunci A se afl
a att pe mediatoarea lui [DD00 ], ct si pe cea a lui [CC 00 ]. ns
a cele
dou
a mediatoare sunt paralele sau coincid; r
amne cea de-a doua variant
a si astfel
rezult
a c
a A apartine medianei din B n 4BCC 00 , fals. Deducem c
a C = C 00 , adic
a
4ABC 4A0 B 0 C 0 .
Observatia 1. n aceast
a demonstratie nu folosim explicit faptul c
a AD si A0 D0
BD
B 0 D0
sunt bisectoare, ci doar egalitatea rapoartelor
= 0 0 . Astfel, autorul probleDC
DC
mei, Petru Asaftei, observ
a c
a r
amne adev
arat
a concluzia dac
a nlocuim n ipotez
a
a ceviene ce determin
a rapoarte egale pe latura pe
bisectoarele [AD] si [A0 D0 ] cu dou
care cad. n particular, concluzia r
amne adev
arat
a pentru [AD], [A0 D0 ] mediane
sau simediane.
Observatia 2. Am putea denumi rezultatul Propozitiei 6 cazul de congruenta
Latura-Bisectoare-Latura. Claudiu-
Stefan Popa ridic
a problema function
arii unor
cazuri Bisectoare-Latura-Bisectoare sau Bisectoare-Bisectoare-Bisectoare; propunem
cititorului demonstrarea sau invalidarea acestora. Pentru o eventual
a justificare, mai
promitatoare pare calea urmat
a n
2bc
2b0 c0
A0
A
Demonstratia 2. Cu notatiile uzuale, la =
cos
cos si la0 = 0
.
b+c
2
b + c0
2
0
A
A
Cum la = la0 , b = b0 , c = c0 , obtinem cos
= cos , de unde A = A0 . Concluzia
2
2
rezult
a din L.U.L.
0B0C 0 s
\ A\
i BC = B 0 C 0 , atunci
Propozitia 7. n ipoteza (I), daca ABC
108

4ABC 4A0 B 0 C 0 .
Demonstratie. Mai nti, s
a observ
am c
a:

\
(i) m(BDA) < 90 AB < AC. ntr-adev
ar:

ac
a2 + c2 b2
>

b+c
2a
h
i
2
2
2a2 c > a2 (b + c) + (c b) (b + c) (b c) (b + c) a2 > 0 b > c.
\ < 90 BD > c cos B
m(BDA)

(ii) n 4ABC, AB < AC, consider


am A00 cu A (BA00 ) si fie [AD, [A00 D00
00 C, D, D 00 (BC); pe segmentul (BC)
\
\
bisectoarele unghiurilor BAC, respectiv BA
00
avem atunci ordinea B D D C. ntr-adev
ar, dac
a AC = b, BC = a, AB = c,
a
a
\ = , avem c
A00 B = c00 , A00 C = b00 si m(ABC)
a + 00 > 2 cos . Relatia este
c
c
evident
a pentru 90 , iar pentru < 90 , folosind teorema cosinusului si ipoteza
c < b, avem:
a
a
a
b2 = a2 + c2 2ac cos a2 > 2ac cos > 2 cos + 00 > 2 cos .
c
c c
Atunci
a
a a
a a
a
00
+ 00 >
00 2 cos
c
c
c
c
c
c
a2
b002
b
D00 C
b00
DC
> 002 > 00 00 <
D (BD00 ) .
c2
c
c
c
D B
DB
A
Revenim la demonstratia propozitiei; putem pre0 0
E
supune f
ar
a a restrnge generalitatea c
a AB A B .
A
Consider
am A00 [BA astfel nct 4A00 BC 4A0 B 0 C 0
(deci A00 B = A0 B 0 ). Cum A (BA00 ), conform observatiei (ii), pe (BC) avem ordinea BDD00 C si atunci
00 C), adic
00 A00 ).
\ > m(BA
\
\ < m(BD
\
m(BAC)
a m(BDA)
00 00
\
C
Construim DE k A D , E AB. Cum m(BAD) < 90 , B D D
DE

00 00
\
a c
a 00 00 =
atunci m(EAB) > 90 si deci ED > AD. Din DE k D A rezult
D A
BE
00 00
00 00
<
1,
de
unde
DE
<
D
A
.
Ob
t
inem
astfel
c
a
AD
<
A
D
,
fals.
R
a
mne c
a
BA00 00
0 0 0
AB = A B, prin urmare 4ABC 4A B C .
Propozitia 8. n ipoteza (I), daca AB = A0 B 0 si BC = B 0 C 0 , atunci 4ABC
4A0 B 0 C 0 .
0 B 0 C 0 ), deci AC <
\ < m(A\
Demonstratie. Presupunem prin absurd c
a m(ABC)
0 0
A C . Cu notatiile uzuale, avem:
i
h
"
#

2 bc (b + c)2 a2
2bc
a2
A
2
=
= bc 1
<
AD =
cos
b+c
2
(b + c)2
(b + c)2
h
i
#
"
2
2
b0 c (b0 + c) a2
a
2b0 c
A0
=
=
< b0 c 1
cos
= AD02 ,
2
2
b0 + c
2
(b0 + c)
(b0 + c)

deci, AD < AD0 , ceea ce contrazice ipoteza. Presupunerea f


acut
a este fals
a, prin
urmare 4ABC 4A0 B 0 C 0 .
109

Asupra problemei 3639


din Gazeta Matematic
a, v. XXXIII (19271928)
1
D. M. B
ATINETU-GIURGIU

n volumul XXXIII (1927-1928) al Gazetei Matematice, la pagina 280, D. V.


Ionescu2 a propus problema 3639 cu urm
atorul enunt:
Sa se demonstreze ca


m
e
1
e
= ,
1+
lim m
m
m
2

e
1
11e
e +
lim m m
1+
=
,
m
m
2
24

e
1
11e
21e
e +
1+
lim m m m

=
,
m
m
2
24
48
e fiind baza logaritmilor neperieni.

(1)
(2)
(3)

Solutia acestei probleme a fost dat


a de d-nii G. G. Constantinescu, Stefan

E.
Olteanu, George Silas n Gazeta Matematic
a, vol. XLII (1936-1937), pag. 36-37.
Mai nti se arat
a c
a:

m
1
1
21
am
11
=e 1

+
,
+
em = 1 +
m
2m 24m2 48m3 m4
am
= 0. Din (4) rezult
a limitele din enunt.
unde lim
m m4

(4)

n aceast
a Not
a ne propunem s
a generaliz
am limita (1). Fie (xn )n1 un sir connot

vergent de numere reale cu lim xn = x = D0 (xn ) R.


n

a dac
a exist
a
Definitie [1]. Spunem c
a sirul (xn )n1 admite o derivat
not

a D (xn ) R, spunem c
a sirul (xn )n1 este
lim (n (xn x)) = D (xn ) R. Dac
derivabil.

r s
Propozitia 1. Sirul

(un )n1 , un = 1 +
, unde r R+ , s R+ este
n
derivabil.
Demonstratie. Dac
a s = 0, atunci un = 1, n N; deci n (un 1) = 0,
n N , deci lim (n (un 1)) = 0.
n

r s
Dac
a s R+ , atunci D0 (un ) = lim un = lim 1 +
= 1. Deci
n
n
n

un 1
ln unn = 1 lim (ln unn ) = ln lim unn =
lim (n (un 1)) = lim
n
n
n
n
ln un

n rs

r r
r sn
= ln lim 1 +
1+
= ln ers = rs R,
= ln lim
n
n
n
n
n

1
2

Profesor, Colegiul National "Matei Basarab", Bucuresti


Matematician romn (19011985)

110

ceea ce demonstreaz
a propozitia.
Propozitia 2. Daca (xn )n1 este un sir convergent cu D0 (xn ) = lim xn =
n
x n
n

= x R , atunci sirul (xn )n1 este derivabil daca si numai daca exista lim
=
n x

= d R+ .
Demonstratie. Este evident c
a

x xx ! n(xnxx)
x n
1
n
x

x
lim (n(xn x))
n
n
lim
= lim
= e x n
.
(5)
1+
n
n
x
x
x n
1
lim (n(xn x))
n
= d R+ , atunci d = e x n
.
Relatia (5) ne arat
a c
a, dac
a exist
a lim
n
x
Prin urmare exist
a si D (xn ) = lim (n (xn x)) = x ln d R, adic
a sirul (xn )n1
n
este derivabil.
Reciproc, relatia (5) ne arat
a c
a, dac
a sirul (xn )n1 este derivabil, atunci exist
a
x n
1
lim (n(xn x))
n
x n
D (xn ) = lim (n (xn x)) R, deci exist
a si d = lim
=e
=
n
n
x
1
e x D(xn ) R+ . Cu aceasta propozitia este demonstrat
a.
n continuare vom nota lim xn = D0 (xn ) = x R si lim yn = D0 (yn ) = y R.
n

Propozitia 3 [2]. Daca sirurile (xn )n1 , (yn )n1 sunt derivabile, atunci sirul
(xn yn )n1 este derivabil si D (xn yn ) = D (xn ) y + xD (yn ).
Demonstratie. Conform enuntului avem:
D0 (xn ) = x = lim xn ,

D0 (yn ) = y = lim yn R

si, de asemenea,
D (xn ) = lim (n (xn x)) ,

D (yn ) = lim (n (yn y)) R.

Prin urmare, trebuie de ar


atat c
a
D (xn yn ) = lim (n (xn yn xy)) R.
n

ntr-adev
ar,
n (xn yn xy) = n (xn yn xyn + xyn xy) = yn n (xn x)+xn (yn y) ,
de unde, prin trecere la limit
a cu n , deducem:

n N ,

D(xn yn ) = lim (n (xn yn xy)) = lim (n (xn x)) lim yn + x lim (n (yn y)) =
n

= D (xn ) D0 (yn ) + D0 (xn ) D (yn ) .


Cu aceasta propozitia este demonstrat
a.
n continuare, fie r R+ , s R+ si vom adopta notatiile:

r n+s
r n
, en (r) = en (r, 0) = 1 +
,
en (r, s) = 1 +
n
n

r s
1
, en = 1 +
, n N .
un = 1 +
n
n
111


r n
Se observ
a c
a lim en (r, s) = er , lim en (r, 0) = lim 1 +
= er , lim un = 1,
n
n
n
n
n
lim en = e.
n

Propozitia 4. Sirul

(en (r))n1 este derivabil si


r2 er
.
2
Demonstratie. Conform inegalit
atilor Bencze-Tth din [3] avem
r
er
e
2
< er en (r) <
, n N , unde a = 2 ; b, c R+ .
an + b
an + c
r
Prin nmultire cu n, deducem c
a
er n
er n
< n (er en (r)) <
, n N .
an + b
an + c
De aici, prin trecere la limit
a cu n , rezult
a c
a
er
r2 er
D (en (r)) = lim (n (en (r) er )) = =
.
n
a
2
D (en (r)) =

(6)

(7)

(8)

Teorem
a. Sirul

(en (r, s))n1 este derivabil si


rer
(2s r) .
(9)
n
2
Demonstratie. S
a observ
am c
a en (r, s) = en (r) un , n N . Deoarece sirurile
(en (r))n1 si (un )n1 sunt siruri derivabile, conform Propriet
atii 3 rezult
a c
a sirul
(en (r, s))n1 = (en (r))n1 (un )n1 este un sir derivabil si
D (en (r, s)) = lim (n (en (r, s) er )) =

D (en (r, s)) = D (en (r) un ) = D (en (r)) lim un + lim en (r) D (un ) =
n

2 r

r e
re
1 + er rs =
(2s r) .
2
2

Observatie. n particular, dac


a r = 1, s = 0 obtinem relatia (1) din Problema
3639 a lui D. V. Ionescu, iar dac
a r = s = 1 obtinem c
a
!!

n+1
e
1
lim n
e
= .
1+
n
n
2
Bibliografie
1. M. B
atinetu-Giurgiu, D. M. B
atinetu-Giurgiu, M. Bencze - Siruri

derivabile,
Gazeta Matematic
a, nr. 9/2005, 410-420.
2. M. B
atinetu-Giurgiu, D. M. B
atinetu-Giurgiu, M. Bencze - Derivable Sequences, Octogon Mathematical Magazine, 13 (2005), no. 2, 936-945.
3. M. Bencze, L. Tth - O generalizare a inegalitatii Plya-Szeg, Arhimede, nr.
3-4/2005, 7-10.

112

O problem
a despre suma cifrelor unui num
ar natural
n baze de numeratie oarecare
Adrian ZAHARIUC 1
1. Introducere. Studiul cifrelor puterilor unui num
ar ntreg este o problem
a
dificil
a de teoria numerelor. Pentru ultimele progrese n acest sens, cititorul poate
consulta [2]. Exist
a multe probleme, evidente din punct de vedere intuitiv, care
nc
a sunt departe de a fi rezolvate. Chiar si faptul c
a suma cifrelor lui an tinde la
infinit nu este att de evident, fiind un rezultat binecunoscut al lui Schnirelman.
Printre ntreb
arile ce apar, una pare adev
arat
a dincolo de orice ndoial
a, dar aproape
imposibil de atacat.
Propozitia 1. a si b sunt numere naturale nenule astfel nct
s(an ) = s(bn )

(1)

pentru orice n. Atunci lg a lg b Z. (s (N ) noteaz


a suma cifrelor num
arului N .)
Observatie. Desigur, lg a lg b = lg a/b, deci lg a lg b Z nseamn
a de fapt c
a
a = b sau c
a num
arul mai mare dintre a si b este obtinut din cel
alalt prin ad
augarea
ctorva 0-uri la sfrsit. Este evident c
a reciproca acestui fapt este adev
arat
a.
Propozitia 2. a si b sunt numere naturale nenule astfel nct
s(an) = s(bn)

(2)

pentru orice n. Atunci lg a lg b Z.


Este clar c
a aceasta este o variant
a mai slab
a a problemei anterioare. De fapt,
(2) este mult mai restrictiv
a dect (1); n caz c
a (2) este adev
arat
a, avem
s(an ) = s(an1 b) = s(an2 b2 ) = = s(abn1 ) = s(bn ),

deci (1) este de asemenea adev


arat
a. Astfel, am devenit interesati de aceast
a ntrebare. Am crezut c
a nu avem de verificat dect un lucru simplu, dar adev
arul s-a
dovedit a fi altul.
2. Observatii preliminare. O modalitate natural
a de a ataca problema este s
a
vedem care termeni ai sirurilor s(an) si s(bn) pot fi calculati usor. Aceast
a metod
a
nu se va dovedi eficient
a, dar ne va da o imagine asupra problemei. Este cunoscut
a
urm
atoarea
Lem
a. Daca N 10k 1, atunci

s 10k 1 N = 9k.

Demonstratie. Putem presupune c


a 10 nu divide N . Fie N = b1 b2 . . . bk , unde
primele cifre pot fi si nule. Este mai convenabil si mai usor s
a calcul
am o diferenta
1

Elev, cl. a XI-a, Colegiul National "Ferdinand I", Bac


au

113

dect un produs:

s 10k 1 N = s 10k N N = s(b1 b2 . . . bk 00


. . 00} b1 b2 . . . bk ) =
| .{z
k cifre

= s b1 b2 . . . bk1 (bk 1)(9 b1 )(9 b2 ) . . . (9 bk1 )(10 bk ) = 9k,

deci lema este demonstrat


a.

Ce se ntmpl
a dac
a folosim aceasta n problema noastr
a? Dac
a 10k 1 > a si b,
atunci


s 10k 1 a = 9k si s 10k 1 b = 9k,
deci, de fapt, s(an) = s(bn) pentru toti n = 10k 1 suficient de mari, ignornd
ipoteza noastr
a. Asadar, din aceast
a observatie, mpreun
a cu observatia c
a ad
augarea
0-urilor n numerele n nu are efect, concluzion
am c
a toate sirurile de forma s(an) au
structuri similare si chiar valori comune pentru anumite valori ale lui n. Mai mult,
este clar c
a alegerea unor valori "frumoase" ale lui n nu este o tehnic
a util
a.
Dar ce s-ar ntmpla dac
a am reusi s
a g
asim forme simple pentru an sau bn?
Aceasta este prima idee util
a.
Desigur, putem t
aia 0-urile de la sfrsitul numerelor a si b, deci 10 nu divide nici
a nici b, si ne ramne s
a ar
at
am c
a a = b. Presupunem c
a nu este adev
arat si lu
am
a > b. Atunci, evident, pentru orice n, an > bn. Vrem s
a g
asim un num
ar an care
are suma cifrelor mai mare dect cea a tuturor numerelor mai mici dect el. Evident,
aceasta va contrazice s(an) = s(bn) si problema este rezolvat
a. Dar care sunt aceste
numere? Sunt numerele care contin doar cifra 9, eventual, cu exceptia primei cifre.
Nu este greu s
a vedem c
a a are un astfel de multiplu dac
a si numai dac
a (a, 10) = 1.
Deci, n acest caz, problema este rezolvat
a. Din p
acate, nu putem rezolva pe aceeasi
idee cazul n care a sau b sunt divizibile cu puteri mari ale lui 2 sau 5. Oricum,
dac
a am fi folosit o baz
a prim
a n locul lui 10, problema ar fi fost rezolvat
a. Din
acest motiv consider
am potrivit s
a generaliz
am problema pentru o baz
a de numeratie
oarecare.
Propozitia 3. Fie b 2, iar m si n doua numere naturale nenule astfel nct
sb (km) = sb (kn),
pentru orice k. Atunci logb m logb n Z.
3. Solutia problemei. Pasul 1. Vom ar
ata ca fractia n/m are un num
ar finit
de cifre dup
a virgul
a. Pentru fiecare i, exist
a un ki Z astfel nct
bi ki m bi + m 1.

Atunci, deoarece functia sb este subaditiv


a,
sb (ki m) sb (bi ) + s(ki m bi ) = 1 + sb (ki m bi ) 1 + ki m bi m.
S
a fix
am j N, foarte mare. Vom demonstra c
a pentru orice i suficient de mare,
primele j 1 cifre ale lui ki n coincid cu primele j 1 cifre ale lui n/m. Prima cifr
a
114

a lui n/m este prima cifr


a din stnga diferit
a de 0 indiferent dac
a este nainte sau
dup
a virgul
a. Fie
jn k
bj .
C = bj
m
Avem

n
1
ki n
1
C
=
< C + j ki mC ki n < ki m C + j
m
ki m
b
b

i
i

1
b
1
i
i
()
b C ki n < b + m C + j = b C + j + m C + j
b
b
b
Ca s
a stabilim faptul c
a primele j 1 cifre ale lui ki n sunt aceleasi cu primele
j 1 cifre ale lui n/m, trebuie s
a ar
at
am c
a
bi C ki n < bi C +

bi+1
.
bj

Conform (), este suficient s


a ar
at
am c
a

1
bi (b 1)
m C+ j <
,
b
bj

care este adev


arat
a pentru i suficient de mare. Atunci, pentru orice j, exist
a termeni
ai sirului ki n care au suma cifrelor cel putin egal
a cu suma primelor j 1 cifre ale
lui n/m.
Cum sb (ki m) m, pentru orice i rezult
a c
a sirul sb (ki n) este m
arginit. De aici
deducem c
a toate cifrele n baza b ale lui n/m sunt 0, eventual cu exceptia unui
num
ar finit.
Pasul 2. Vom ncepe cu un rezultat simplu, dar important. Nu-l vom demonstra,
fiind binecunoscut.
Lem
a. A si B au proprietatea ca A/B are un numar finit de cifre dupa virgula
n baza b. Atunci, toti factorii primi ai lui
B
(A, B)
se numara printre factorii primi ai lui b. Altfel spus, daca p este prim, (p, b) = 1,
atunci expp m expp n. (expp A noteaz
a exponentul num
arului p n descompunerea
lui A.)
Am v
azut c
a n/m are un num
ar finit de cifre dup
a virgul
a. Dar, desigur, putem
demonstra analog c
a si m/n are un num
ar finit de cifre dup
a virgul
a. Atunci, conform
lemei, pentru orice p care nu l divide pe b, expp m = expp n. Cu alte cuvinte,
dac
a b=

t
Y

i
p
i , atunci m = N

t
Y

pri i si n = N

cu (N, b) = 1. S
a fix
am f N suficient de mare si s
a lu
am
k1 =

t
Y

pfi i si .

115

t
Y
1

psi i ,

nlocuind k din ipoteza sb (km) = sb (kn) cu kk1 , obtinem


!
t
Y f s +r

i
i
i
pi
N = sb kbf N = sb (kN ),
sb (k1 km) = sb (k1 kn) sb k
1

k N

Pentru simplitate, fie

K=

t
Y

pfi i si +ri .

Atunci, sb (kN K) = sb (kN ) pentru toti k N, care reduce problema cu mult. Este
important s
a tinem minte c
a (N, b) = 1.
Pasul 3. S
a ar
at
am c
a logb K Z. Deoarece sb (kN K) = sb (kN ) pentru toti
k N, deducem c
a
sb (N ) = sb (KN ) = sb (K 2 N ) = sb (K 3 N ) = . . .

n particular, aceasta nseamna c


a multimea {sb (K u N ) : u N} este marginit
a.
Dac
a logb K R \ Q, vom ar
ata c
a pentru orice nsiruire de cifre, exist
a un u
astfel nct K u N ncepe exact cu acea secventa de cifre, ceea ce ar conduce la o
contradictie. Pentru aceasta, este suficient s
a ar
at
am c
a multimea
{{logb K u N } : u N},

unde {x} = x bxc, este dens


a n [0, 1]. Dar, deoarece logb K u N = u logb K + logb N ,
aceasta rezult
a imediat din lema lui Kronecker. Amintim c
a lema lui Kronecker
afirm
a faptul c
a pentru orice R \ Q, multimea {{n} : n N} este dens
a n
(0, 1). Asadar, am ar
atat c
a logb K Q.
S
a presupunem, prin absurd, c
a logb K Q\Z. Este foarte interesant s
a observ
am
c
a acest lucru nu este posibil dect atunci cnd b este o putere netrivial
a si c
a
demonstratia este f
acut
a n toate celelalte cazuri. Observ
am c
a logb K Q nseamn
a
de fapt c
a, pentru orice p, expp K = logb K expp b. Fie r = blogb Kc; atunci expp K
r expp b, deci br | K. Fie K 0 = K/br ; este clar c
a K 0 < b, deci K 0 este o cifr
a n
0
baza b. Deoarece K este obtinut din K prin stergerea 0-urilor n baza b, rezult
a c
a
sb (kN K 0 ) = sb (kN ), pentru orice k. Deoarece (N, b) = 1, nu este greu s
a ar
at
am c
a
N are un multiplu kN de forma 11 . . . 11 n baza b (exist
a dou
a numere de aceast
a
form
a congruente modulo K; deci K divide diferenta lor, care, dup
a omiterea 0-urilor
de la sfrsit este de forma 11 . . . 11). Deci sb (kN ) = a si sb (kN K 0 ) = K 0 a, de unde
K 0 = 1. Asadar, K = br , deci logb K Z si, deoarece logb K = f +(logb mlogb n)
Z, va rezulta c
a afirmatia din Propozitia 3 este adev
arat
a.
Bibliografie
1. G. Dospinescu, A. Zahariuc - Suma cifrelor unui numar natural, Arhimede.
Revist
a de cultur
a matematic
a (Bucuresti), 2004, nr. 3-4, 2-15.
2. R. Blecksmith, M. Filaseta, C. Nicol - A result on the digits of an , Acta Arith.
64(3), 1993, 331-339.
3. http://www.mathlinks.ro

116

Generaliz
ari ale unor inegalit
ati din RecMat
Alexandru NEGRESCU 1
Ne propunem s
a generaliz
am urm
atoarele inegalit
ati ap
arute n revista "Recreatii
Matematice":
X.53. (nr. 2/2004, p. 155). Fie a, b, c (1, ) astfel nct a + b + c = 9. Sa se
arate ca

loga 2b3 + c3 + logb 2c3 + a3 + logc 2a3 + b3 12.


Angela Tig
aeru

IX.48 (nr. 1/2004, p.77). Fie a, b, c (0, ) cu a + b + c + abc = 4. Sa se


arate ca
a2
b2
3
c2
+
+
.
2
a + bc b + ca c + ab
Cezar Lupu
VII.17 (nr. 1/2001, p. 74). a) Fie x, y, z [2, ). Aratati ca

2
x + y y 2 + z z 2 + x 27xyz.

b) Fie x, y, z [3, ). Aratati ca x2 + y y 2 + z z 2 + x 64xyz.


Lucian Tutescu
Solutii ale acestor probleme pot fi g
asite n numerele 2/2005, 1/2005 si respectiv
1/2002 ale revistei.
Problema 1. Date numerele a1 , a2 , . . . , an (1, ), n 2, fie S = a1 + a2 +
+ + an . Sa se arate ca

loga1 (2an2 + an3 + + ann ) + loga2 (an1 + 2an3 + + ann ) + +

n
+ logan 2an1 + an2 + + ann1
+ n2 ,
logn S 1
S
cu egalitate daca si numai daca a1 = a2 = = an = .
n
Solutie. Not
am cu E membrul stng al inegalit
atii. Utiliznd inegalitatea mediilor, avem:

E loga1 na22 a3 an + loga2 na1 a23 an + + logan na21 a2 an1 =


n
X

logai n + 2 loga1 a2 + loga2 a3 + + logan a1 +


=
i=1

+ loga1 (a3 an ) + loga2 (a1 a4 an ) + + logan (a2 an1 )


n
q
X

logai n + 2n + n n loga1 (a3 an ) logan (a1 an1 ).


i=1

Dar

ln a3 + ln a4 + + ln an
(n 2) (ln a3 ln an ) n2
loga1 (a3 an ) =

.
ln a1
ln a1
1

Elev, cl. a XI-a, Colegiul National "A. T. Laurian", Botosani

117

Cu aceast
a inegalitate si analoagele ei, vom avea
n
n
X
X
n2
1
logai n + 2n + n (n 2) =
+ n2 Pn
+ n2 =
E
log
a
log
a
i
i
n
n
i=1
i=1
i=1

n2
n
n + n2 =
+ n2
logn S 1
a1 + a2 + + an
logn
n
si astfel inegalitatea este dovedit
a. Cazul n care are loc egalitate se obtine cu usurinta.
=

n2
+ n2
logn (a1 a2 an )

Observatie. Inegalitatea din Problema X.53 se obtine pentru n = 3 si S = 9.


Problema 2. Date numerele a1 , a2 , . . . , an (0, ), n 3, fie S = a1 + a2 +

+ + an + n1 a1 a2 an . Sa se arate ca
a2
+
1
a2 a3 an a2 + k

a2
+ +
2
a1 + k
a1 a3 an
1+ n1

S
S n+1
a2n

+
,

an + k n1 a1 a2 an1
k+1
cu egalitate daca si numai daca a1 = a2 = = an = 1 si S = n + 1.
Solutie. Notnd cu E primul membru al inegalit
atii si utiliznd inegalitatea
Cauchy-Buniakowski-Schwarz si cea a mediilor, obtinem
n1

n1

(a1 + a2 + + an )
(a + a2 + + an )
1

=
E P

P
a2 + a3 + + an
a1 + k n1 a2 a3 an
a1 + k
n1
Pn
Pn
2
ai
( i=1 ai )
Pn
= i=1 .
()
=
k+1
(k + 1) i=1 ai

Pe de alt
a parte, pornind de la S = a1 + a2 + + an + n1 a1 a2 an si folosind
din nou inegalitatea mediilor, vom avea
v
s
un
uY
n
Q
n+1
t
n1
a
a,
S (n + 1)
i

de unde

Ca urmare,

S
n+1

n+1

n
Y

i=1

i=1

i=1

ai

! n
n1

v
un

n + 1
uY
S
n
n1
ai = S t ai S
.
n
+
1
i=1
i=1

n
X

Introducnd n (), vom obtine inegalitatea din enunt. Egalitate avem dac
a si numai
S

, adic
a ai = 1, i N si S = n + 1.
dac
a a1 = a2 = = an = n1 a1 an =
n+1
118

Observatii. 1) Mai general, putem considera a1 , a2 , . . . , an (0, ), n 3, si

S = a1 + a2 + + an + p a1 a2 an , p 3; se va obtine

p(n+1)
p+1
1
S
.
E
S
k+1
n+1
2) Pentru n = 3 si k = 1 reg
asim inegalitatea din Problema IX.48.

n privinta Problemei VII.17, vom spune mai nti c


a n nr. 1/2002, la pag. 66,
sunt prezentate dou
a demonstratii pentru urm
atoarea generalizare a acesteia:

3
Fie x, y, z [n, ), n N. Aratati ca x2 + y y 2 + z z 2 + x (n + 1) xyz.
O inegalitate si mai general
a este dat
a de

Problema 3. Fie x1 , x2 , . . . , xn [k, ), k, n N si n 2. Aratati ca


k(n2)+1

n1
n
+ x3 xn1
+ x1 (k + 1) (x1 x2 xn ) k+1 ,
x1 + x2 xn1
n
2

Cu egalitate daca si numai daca xi = k = 1, i = 1, n, sau xi = k, n = 3, i = 1, 3.


Solutie. S
a not
am din nou cu E membrul nti al inegalit
atii de demonstrat.
k2
Observ
am c
a x1 k implic
a xn1

kx

s
i
1
1
xn1
+ x2 kxn2
+ x2 = xn2
+ xn2
+ + xn2
+ x2 .
1
1
1
1
|1
{z
}
k
q
k+1
k(n2)
n2
Deci x1 + x2 (k + 1)
x1
x2 . Utiliznd aceast
a inegalitate si analoagele
ei, obtinem c
a
q
k(n2)+1
k(n2)+1
n k+1 k(n2)+1 k(n2)+1
E (k + 1)
x1
x2
xn
= (k + 1)n (x1 x2 xn ) k+1 ,
cu egalitate asa cum este specificat n enunt.

Observatie. Inegalit
atile din Problema VII.17 se obtin pentru k = 2, n = 3 si
respectiv k = 3, n = 3, iar generalizarea lor, dat
a n revist
a, pentru n = 3.

Cnd se naste, omul zice AA. . . . Cnd moare, zice MOR. . . . Asadar, intervalul
vietii este [A, M OR], adic
a AMOR.

Vizitati pe Internet revista "Recreatii Matematice" la adresa

http://www.recreatiimatematice.uv.ro
119

Cea mai bun


a inegaliate de acest tip . . .
Marian TETIVA1
1. Introducere. Ati ntlnit cu siguranta asemenea enunt (mai ales dac
a v
a
preocup
a si probleme mai grele, "de olimpiad
a"). Iat
a trei astfel de exemple:
1. Cea mai buna inegalitate de tipul
(x + y + z)3 27xyz k[(x + y + z)(xy + xz + yz) 9xyz],

se obtine pentru k = 4 (este asa-numita inegalitate a lui Schur ).


2. Cea mai buna inegalitate de tipul

x, y, z > 0

p kR + hr

care are loc n orice triunghi se obtine pentru k = 2 si h = 3 3 4 (se numeste


inegalitatea lui Blundon n acest caz).
3. Cea mai buna inegalitate de forma
a2 + b2 + c2 kR2 + hr2

si care are loc, iarasi, n orice triunghi este cea obtinuta pentru k = 8 si h = 4.
Cum se abordeaz
a o asemenea problem
a? R
aspundem imediat la aceast
a ntrebare
rezolvnd-o pe prima (a se vedea si [2], problema 9.22, un pic altfel formulat
a acolo);
celelalte sunt solutionate n [3], respectiv [1]. Iat
a mai departe solutia problemei 1.
Trebuie s
a ntelegem bine enuntul; observ
am c
a expresia din membrul drept
(x + y + z)(xy + xz + yz) 9xyz,

are numai valori nenegative, conform unei inegalit


ati cunoscute. De aceea
k1 [(x + y + z)(xy + xz + yz) 9xyz] k2 [(x + y + z)(xy + xz + yz) 9xyz]

a problema noastr
a este, de fapt, problema g
asirii
pentru x, y, z > 0 si k1 k2 , asa c
celui mai mare num
ar k astfel nct inegalitatea
(x + y + z)3 27xyz k[(x + y + z)(xy + xz + yz) 9xyz]

s
a fie adev
arat
a pentru orice x, y, z > 0. De obicei ntr-o asemenea chestiune, se caut
a
mai nti cea mai mare valoare posibil
a a lui k (cea mai mic
a, n alte cazuri), dnd
valori particulare variabilelor, apoi se demonstreaz
a inegalitatea rezultat
a pentru
valoarea extrem
a depistat
a pentru k. Cel mai comun (si comod) este s
a egal
am
variabile; n cazul de fata, s
a punem y = z = 1 si x > 0 arbitrar. Obtinem inegalitatea
(x + 2)3 27x k[(x + 2)(2x + 1) 9x] (x 1)2 (x + 8) 2k(x 1)2 ,

care trebuie s
a aib
a loc pentru orice x > 0 (pentru c
a inegalitatea initial
a vrem s
a
aib
a loc pentru orice x, y, z > 0). De fapt asta nseamn
a x + 8 2k, pentru orice
x 6= 1 si, n ultim
a instanta, pentru orice x > 0. Aici are loc de obicei un proces
de trecere la limit
a, pentru a obtine cel mai bun k. Limita se calculeaz
a, uneori, n
puncte care sunt capete ale intervalelor pe care rezult
a inegalitatea final
a; n cazul
de fata, pentru x 0, obtinem k 4. (Evident, aici trecerea la limit
a n 0 nu era
1

Profesor, Colegiul National "Gheorghe Rosca Codreanu", Brlad

120

strict necesar
a, c
aci x, y, z pot fi considerate 0 - nu neap
arat strict pozitive - si
rationamentul functioneaz
a la fel). Astfel am aflat c
a, probabil, cea mai bun
a valoare
a lui k este 4; numai dac
a reusim s
a demonstr
am inegalitatea pentru k = 4 putem
trage concluzia final
a, cum c
a "cea mai bun
a inegalitate de tipul"
(x + y + z)3 27xyz k[(x + y + z)(xy + xz + yz) 9xyz],

x, y, z > 0

se obtine pentru k = 4. Ori, nlocuind aceast


a valoare a lui k, ajungem la
x3 + y 3 + z 3 + 3xyz x2 y + xy 2 + x2 z + xz 2 + y 2 z + yz 2 ,

x, y, z > 0,

inegalitate bine cunoscut


a (numit
a uneori "inegalitatea lui Schur", asa cum am mai
spus), care apare n multe c
arti (inclusiv n [2]), deci nu o mai demonstr
am aici.
2. Din nou despre normare. Initial aceast
a lucrare s-a vrut o continuare a
notei [4]. ntre timp, s-a ivit aceast
a nou
a inegalitate (vom ar
ata la urm
a prin ce
concurs de mprejur
ari am ajuns la ea) care s-a l
asat rezolvat
a cu metoda norm
arii
(nu ezitati s
a o folositi si pentru inegalitatea lui Schur! de fapt, asa e f
acut
a si n
[2]) si, n plus, ilustreaz
a bine si genul de probleme pe care l-am descris sus; iat
a
de ce am decis s
a amn
am publicarea solutiilor exercitiilor din [4] (mai ales c
a, nu-i
asa?, "cititorii nostri sunt la fel de inteligenti ca si noi", ba chiar mai) n favoarea
problemei pe care o prezent
am chiar acum.
Problema 1. Care este cea mai buna inegalitate de tipul
(a2 + b2 + c2 )(a2 + b2 + c2 ab ac bc)

k(a + b + c)(a2 b + ab2 + a2 c + ac2 + b2 c + bc2 6abc),

a, b, c > 0 ?

Cu alte cuvinte, deoarece si aici a doua parantez


a din membrul drept are valori
nenegative si a, b, c sunt strict pozitive, trebuie s
a determin
am cea mai mare valoare
pe care o poate lua k astfel nct inegalitatea de mai sus s
a fie adev
arat
a pentru orice
a, b, c > 0. S
i de aceast
a dat
a ncepem cu particularizarea b = c = 1 care ne conduce
la (a2 + 2)(a 1)2 2k(a + 2)(a 1)2 , deci, pn
a la urm
a a2 + 2 2k(a + 2), care
ar trebui s
a fie adev
arat
a pentru orice a > 0, a 6= 1, adic
a (pe baza continuit
atii)
pentru orice a > 0. Facem iar pe a s
a tind
a la 0 pentru a g
asi 2k 1 dar, surpriz
a!
Pentru k = 1/2 si b = c = 1 inegalitatea devine a(a 1)3 0 si, prin urmare, este
departe de a fi adev
arat
a pentru orice a, b, c > 0. Trebuie deci s
a rafin
am evaluarea
lui k, observnd c
a, de fapt, am obtinut
a2 + 2
, a > 0;
a+2
ceea ce nseamn
a c
a, n mod necesar, 2k este cel mult egal cu valoarea minim
a a
x2 + 2
functiei f (x) =
pe intervalul (0, ). (De aici si deosebirea fata de cazul
x+2
studiat anterior: minimul nu se atinge n cap
atul intervalului.) Aceasta se dovedeste
a fi ceva mai mic
adect limita functiei n origine, adic
a
a dect 1; valoarea minim
a pentru x = 6 2), astfel c
a ar trebui s
a
a functiei f este 2 6 4 (se realizeaz
avem k 6 2 (f
ar
a s
a fie deloc clar deocamdat
a dac
a aceasta este, ntr-adev
ar,
valoarea care produce
cea mai bun
a inegalitate). Oricum, putem considera mai

departe k [0, 62] (n mod clar valorile negative ale lui k nu pot filuate n discutie
drept candidate pentru "cel mai bun" k); n particular, avem k 6 2 < 1/2
2k

121

1 2k > 0. Mai departe ncerc


am, cu metoda norm
arii, s
a vedem ce se ntmpl
a cu
inegalitatea noastr
a pentru asemenea k si a, b, c numere pozitive oarecare. Datorit
a
simetriei putem presupune c = min{a, b, c}; not
am atunci
a
b
= 1 + x,
= 1 + y,
c
c
unde x si y trebuie s
a fie nenegative. Inegalitatea mai poate fi scris
a n forma
X
X
X
X
a2 b2 + (4k 1)
a2 bc (k + 1)
(a3 b + ab3 )
a4 + (2 2k)
a
b
(toate sumele sunt ciclice) sau nlocuind cu 1 + x si cu 1 + y,
c
c

(1 + x)4 + (1 + y)4 + 1 + (2 2k) (1 + x)2 (1 + y)2 + (1 + x)2 + (1 + y)2 +

+(4k 1) (1 + x)2 (1 + y) + (1 + x)(1 + y)2 + (1 + x)(1 + y)

(k + 1) (1 + x)3 (1 + y) + (1 + x)(1 + y)3 + (1 + x)3 + (1 + y)3 + 1 + x + 1 + y .

Acum urmeaz
a partea nepl
acut
a (dar inevitabil
a cnd folosim metoda norm
arii!)
n care trebuie s
a facem calculele; pe care nu le vom reproduce aici (dar v
a sf
atuim
s
a le verificati!. . . ). Inegalitatea de demonstrat cap
at
a forma
x4 (k + 1)x3 y + (2 2k)x2 y 2 (k + 1)xy 3 + y 4 +

+(2 2k)x3 3x2 y 3xy 2 + (2 2k)y 3 + (3 6k)(x2 xy + y 2 ) 0,

sau nc
a

1
1
(x y)4 + (x2 y 2 )2 + (1 k)xy(x y)2 + 2(1 2k)x2 y 2 +
2
2
+2(1 k)(x + y)(x y)2 + (2 5k)xy(x + y) + 3(1 2k)(x y)2 + 3(1 2k)xy 0.

a
Acum putem demonstra inegalitatea pentru k 6 2; oricum, ea este evident
pentru k < 0, deci putem considera k 0.
S
a observ
am mai nti c
a, pentru aceste valori ale lui k, avem
2(1 2k)t2 2(2 5k)t + 3(1 2k) 0,

pentru orice t R, deoarece discriminantul trinomului este

4(2 5k)2 24(1 2k)2 = 4(k2 + 4k 2) = 4(k + 6 + 2)(k 6 + 2) 0;

n plus, cum am mai ar


atat, k este mai mic dect 1/2, deci 2(1 2k) > 0.
n membrul stng al inegalit
atii de demonstrat toti termenii sunt n mod evident
nenegativi, cu exceptia lui (2 5k)xy(x + y) care poate fi luat n grupul
2(1 2k)x2 y 2 + (2 5k)xy(x + y) + 3(1 2k)xy =
= xy [2(1 2k)xy + (2 5k)(x + y) + 3(1 2k)] ,

pe care-l mai putem scrie

2
x+y
x+y
1 2k
xy 2(1 2k)
+ 2(2 5k)
+ 3(1 2k)
xy(x y)2 .
2
2
2
Dar paranteza mare este nenegativ
a, conform observatiei referitoare la trinomul de
gradul al doilea, iar ultimul termen este "anihilat" de (1 k)xy(x y)2 :
1 2k
1
(1 k)xy(x y)2
xy(x y)2 = xy(x y)2 0
2
2
122

si inegalitatea
este demonstrat
a; valoarea cea mai bun
a (maxim
a) a lui k este, ntr
adev
ar, 6 2.
3. Poate c
a nu e lipsit de interes s
a vedem cum am ajuns la aceast
a inegalitate: desigur, din ntmplare (multe ies astfel . . . )! Am vrut s
a vedem ce se obtine
dac
a aplic
am inegalitatea lui Jensen functiei radical n felul urm
ator:
p
p
p
b
c
a
(b + c)2 +
(a + c)2 +
(a + b)2
a+b+c
a+b+c
a+b+c
r
b
c
a

(b + c)2 +
(a + c)2 +
(a + b)2 .
a+b+c
a+b+c
a+b+c
(Asta mai rezult
a si folosind inegalitatea Cauchy-Buniakowski-Schwarz ). Dup
a ridicare la p
atrat si nc
a cteva calcule ajungem la
P
P
30abc 4 a3 3 a(b2 + c2 )
4
4(ab + ac + bc)2
(a + b + c)3 +
a+b+c
9
9
sau, totuna,
4(a2 + b2 + c2 + 5ab + 5ac + 5bc)(a2 + b2 + c2 ab ac bc) =

X
X

= 4 (a + b + c)4 9(ab + ac + bc)2 (a + b + c) 4 a3 + 3 a(b2 + c2 ) 30abc .

Mai tinem cont acum de dou


a lucruri: prima parantez
a din membrul stng nu dep
aseste 6(a2 + b2 + c2 ), si a doua este nenegativ
a conform inegalit
atii
a2 + b2 + c2 ab + ac + bc,

pe cnd cea de-a doua parantez


a din membrul drept se poate minora (pe baza inegalitatii lui Schur ) cu
X

X
X
a(b2 + c2 ) 30abc = 7
a(b2 + c2 ) 6abc ;
4
a(b2 + c2 ) 12abc + 3
astfel suntem condusi la inegalitatea

24(a2 + b2 + c2 )(a2 + b2 + c2 ab ac bc)

7(a + b + c)(a2 b + ab2 + a2 c + ac2 + b2 c + bc2 6abc), a, b, c > 0


si ntrebarea din problema 1 vine n mod natural (dup
a ce am demonstrat inegalitatea
7
pentru k =
). Putem vedea acum c
a valoarea lui k de la care am pornit este destul
24
de departe de cel mai bun k; avem, totusi, o demonstratie f
ar
a prea multe calcule a
unei inegalit
ati de acest tip, care ne poate da sperante c
a s-ar putea face asa si "cea
mai bun
a dintre aceste inegalit
ati". De asemenea, v
a propunem spre rezolvare
Problema 2. Gasiti cea mai buna inegalitate de forma
(x + y)2 (x + z)2 (y + z)2 + (27k 64)x2 y 2 z 2 kxyz(x + y + z)3 ,

x, y, z > 0.

Bibliografie
1.
2.
3.
4.

L. Panaitopol - O inegalitate geometrica, Gazeta Matematic


a seria B, 4/1982.
L. Panaitopol, V. B
andil
a, M. Lascu - Inegalitati, Editura GIL, Zal
au, 1996.
V. Vjitu - Asupra unei inegalitati, Gazeta Matematic
a seria B, 7/1984.
M. Tetiva - Metoda normarii, RecMat - 1/2006, 30-34.
123

Societatea de S
tiinte Matematice din R. Moldova
Anul 2006 aduce cu sine dou
a date marcante n cercetarea si n nv
atamntul
superior din Republica Moldova. Dou
a institutii ce constituie imaginea Basarabiei
postbelice n stiinta contemporan
a - Academia de S
tiinte a Moldovei si Universitatea de Stat din Moldova - mplinesc 60 de ani de la fondare. S
coala
matematic
a din R. Moldova datoreaz
a n mare parte acestora evolutia sa. Date relevante despre istoria acestor institutii pot fi g
asite n volumele: Institutul de Matematica si Informatica. File din istorie, editat de Acad. de S
t. a Moldovei n 2004, si
Istoria Universitatii de Stat din Moldova (1946-1996), editat de USM n 1996.
La afirmarea scolii matematice basarabene au contribuit un sir de personalit
ati de autentic
a valoare, fondatori de scoli stiintifice: Vladimir Andrunachievici,
Valentin Belousov (algebr
a), Constantin Sibirschi (ecuatii diferentiale), Izrael Gohberg (analiz
a functional
a), Alexandru Zamorzaev (geometrie), Alexandru Cuznetov
(logic
a matematic
a) s.a. S
colile stiintifice create n Republica Moldova au reusit s
a
se impun
a n lumea matematic
a prin realiz
ari valoroase, devenind centre de preg
atire
a specialistilor din republic
a si de peste hotare timp de cteva decenii.
Societatea Matematic
a a fost prezent
a n Basarabia n diferite forme de organizare, reunind n rndurile sale cercet
atori, profesori de matematic
a din universit
ati
si scoli si chiar elevi. n actualul s
au statut Societatea Matematic
a din Republica
Moldova (SMM) activeaz
a din anul 1993. n prezent SMM ntruneste 180 de membri, incluznd matematicieni consacrati, tineri cercet
atori, profesori de la institutii
superioare de nv
atamnt, licee si scoli. n anul 2004 Adunarea General
a a SMM l-a
reales n calitate de presedinte pe academicianul Mitrofan Ciobanu, iar vicepresedinti
pe profesorii universitari Nicolae Jitarasu si Alexei Casu.
Activitatea de baz
a a SMM tine de promovarea cercet
arilor stiintifice n domeniul
matematicii si al informaticii, de monitorizarea sistemului de nv
atamnt matematic
la toate nivelele si de consolidarea comunit
atii matematice.
Un element important n consolidarea comunit
atii matematice din Republica
Moldova, pe parcursul anilor, au fost conferintele stiintifice. n ultimii 50 au fost organizate peste 50 conferinte stiintifice, inclusiv editii ale unor conferinte internationale
ex-sovietice si europene. La organizarea acestor manifest
ari stiintifice aportul Societ
atii Matematice a fost esential, cu toate acestea prima conferinta SMM a fost
organizat
a abia n anul 2001. Beneficiind de sprijinul financiar al UNESCO, prima
conferinta a Societ
atii Matematice din Republica Moldova a reusit s
a ntruneasc
a
peste 160 de participanti, inclusiv 40 de oaspeti de peste hotare, specialisti n domeniul matematicii si al informaticii din Romnia, Franta, Germania, Rusia, Polonia,
Ucraina, Belarus, Serbia si Muntenegru. La conferinta au fost prezentate 148 de
rapoarte iar ulterior au fost publicate materialele conferintei n dou
a numere ale revistei Buletinul Academiei de Stiin
te a Republicii Moldova. Matematica, un num
ar
al revistei Computer Science Journal of Moldova si o culegere de articole n domeniul didacticii matematicii si al informaticii. n anul 2004 a fost organizat
a a doua
conferinta SMM, fiind dedicat
a aniversarii a 40-a de la fondarea Institutului de
Matematic
a si Informatic
a al A.
S. R.M. La aceast
a conferinta au participat
124

peste 170 de specialisti n matematic


a si informatic
a, inclusiv 30 de invitati de peste
hotare (SUA, Romnia, Germania, Ucraina, Rusia, Israel, Canada s.a.).
n anul 2002, sustinnd initiativa societ
atilor matematice din Grecia, Romnia si
Bulgaria, SMM a participat la fondarea Societ
atii Matematice din Sud-Estul
Europei (MASSEE) cu sediul la Atena, Grecia. Fondarea MASSEE a constituit o
nou
a etap
a n cooperarea regional
a a matematicienilor din Balcani.
Criza extins
a prin care a trecut R. Moldova n primii s
ai ani de existenta a avut
repercursiuni negative asupra tuturor sferelor de activitate si n primul rnd asupra
dezvolt
arii stiintelor fundamentale. Astfel, SMM si-a propus drept scop identificarea
unor modalit
ati de dep
asire a dificult
atilor ce afecteaz
a cercetarea si educatia n
R. Moldova. Prima conferinta SMM a adresat conducerii R. Moldova un apel n
care au fost formulate propuneri concrete viznd probleme de stringent
a actualitate
n domeniile cercetare-dezvoltare si nv
atamnt. Initiativele SMM, incluse n acest
apel, au influentat politica Guvernului si a altor structuri statale abilitate, avnd
drept urmare anuntarea unor concursuri nationale de granturi, inclusiv granturi de
cercetare n matematic
a si n informatic
a, granturi pentru organizarea conferintelor
si a simpozioanelor stiintifice, pentru editarea revistelor de specialitate etc.
n anul 2004 a intrat n vigoare Codul cu privire la stiinta si inovare al R. Moldova
ce reglementeaz
a raporturile juridice ce tin de elaborarea si promovarea politicii de
stat n sfera stiintei si inov
arii, de acreditarea organizatiilor din sfera stiintei, de
atestarea cadrelor s.a. Codul admite participarea comunit
atii stiintifice la elaborarea politicii de stat n domeniul stiintei si inov
arii. S
i nv
atamntul, n special cel
universitar si postuniversitar, la etapa actual
a este n proces de reformare n conformitate cu cerintele Programului de la Bologna, la care recent a aderat R. Moldova.
n contextul celor mentionate conducerea SMM si propune monitorizarea procesului de reformare n domeniul matematicii si al informaticii, promovnd valorile
acceptate de comunitatea matematic
a international
a.
Comunitatea matematic
a din Republica Moldova marcheaz
a n anul 2006 organizarea editiei a 50-a a Olimpiadei republicane de matematica pentru elevi. Acest
eveniment este deosebit de semnificativ si gratie rezultatelor Lotului olimpic al Republicii Moldova, care a obtinut doar n ultimii cinci ani 66 de medalii la competitii
internationale, inclusiv trei medalii de aur la olimpiadele internationale de matematic
a si nou
a medalii de aur la olimpiadele balcanice. n anul 2006 Chisinaul va fi
gazda a Olimpiadei Balcanice de Matematica pentru juniori.
Evolutia stiintei si a nv
atamntului n Basarabia dup
a 1990 a fost puternic influentat
a de deschiderea cultural
a c
atre Romnia. Un loc aparte l ocup
a contributia
dezinteresat
a a Romniei n oferirea de burse pentru liceeni, studenti, masteranzi si
doctoranzi din R. Moldova. Participarea larg
a la conferinte stiintifice, inclusiv organizarea unora din ele n comun, a extins si a concretizat cadrul de colaborare, avnd
drept urmare sustineri de teze de doctorat, schimb de profesori invitati, preg
atirea
lotului olimpic, tabere de var
a, editare de carte si multe alte activit
ati deosebit de
benefice pentru cultura, cercetarea si educatia din Basarabia.

Florin DAMIAN, Parascovia SRBU


Fac. de Mat. si Inf., Univ. de Stat din Moldova
125

Concursul de Matematic
a Al. Myller
Editia a IV-a, Iasi
Clasele IV-VI, aprilie 2006
Not
a. Toate subiectele sunt obligatorii. Timp efectiv de lucru: 90 min. Se acord
a din
oficiu 30 de puncte, cte 6 puncte pentru problemele 1-5, cte 8 puncte pentru problemele
6-10 si cte 10 puncte pentru problemele 11-15.

Clasa a IV-a
1. Ce num
ar trebuie sc
azut din 9 pentru ca diferenta obtinut
a nmultit
a cu 8 s
a
devin
a 40?
2. ntr-o clas
a sunt b
aieti si fete. Num
arul b
aietilor este cu 3 mai mare dect
num
arul fetelor. Dac
a n clas
a ar veni 4 b
aieti si ar pleca 4 fete, atunci num
arul
b
aietilor ar fi de 2 ori mai mare dect num
arul fetelor. Cti elevi sunt n clas
a?
3. Se dau numerele: a = 1 + 2 + 3 + + 9; b = 11 + 22 + 33 + + 99;
c = 111 + 222 + 333 + + 999. S
a se calculeze ctul mp
artirii lui a + b + c la 123.

4. Suma dintre un num


ar si succesorul s
au este cu 2006 mai mare dect predecesorul s
au. Care este num
arul?
X
5. Fie p
atratul magic (sumele elementelor de pe linii, coloane
si diagonale sunt egale). Aflati ce num
ar trebuie nscris n c
asuta
10 12
marcat
a cu X?
9
7
6. mp
artind num
arul natural a la num
arul natural b, obtinem ctul 6 si restul
31. S
a se afle a stiind c
a a b < 196.

7. ntr-un cos sunt 28 fructe: mere, pere si caise. Cte fructe sunt de fiecare fel,
dac
a mere sunt de 6 ori mai multe dect pere, iar n cos se afl
a cel putin un fruct de
fiecare fel?
8. Cte numere de 3 cifre se mpart exact la 21?
9. Aflati cel mai mic num
ar natural cu suma cifreler 56.
10. S
a se afle 3 numere naturale stiind c
a: produsul primelor dou
a este 84,
produsul ultimelor dou
a este 192, iar suma dintre primul si ultimul este 46.

11. Un tat
a, dorind s
a-si ncurajeze fiul s
a rezolve probleme, i promite c
a i va
da 8 monede pentru fiecare problem
a bine rezolvat
a, dar pentru problema pe care
nu a rezolvat-o sau a rezolvat-o gresit, fiul va trebui s
a-i pl
ateasc
a 5 monede. Dup
a
26 de probleme, fiul nu trebuie s
a pl
ateasc
a nimic, dar nici s
a primeasc
a ceva. Cte
probleme a rezolvat corect?
12. Ioana culege o l
adita de c
apsuni n 40 minute, iar Luiza culege o l
adita n 2
ore. n ct timp vor culege mpreun
a 3 l
adite de c
apsuni?
13. Mama are cu 14 lei mai mult dect Paul si cu 10 lei mai mult dect Tudor.
Cti lei va da fiec
aruia din b
aieti pentru a avea toti 3 aceeasi sum
a?
126

14. M-am n
ascut n secolul XX. Dac
a n 1999 am avut o vrst
a egal
a cu suma
cifrelor anului meu de nastere, ce vrst
a am acum?
15. Sony car
a n fiecare zi cte o piatr
a din vrful muntelui. n prima zi a petrecut
7 ore urcnd si cobornd, a doua zi a petrecut 8 ore urcnd si cobornd. n fiecare zi
urc
a de dou
a ori mai ncet dect n ziua precedent
a, dar coboar
a de 2 ore mai repede.
Ct timp va munci n cea de-a treia zi?

Clasa a V-a
1. Care este cel mai mare num
ar natural impar de 4 cifre distincte?
2. Determinati x N pentru care 5 3x 2 3x = 81.
3. Cti termeni are suma 26 + 32 + 38 + + 2006?

4. Cte p
atrate perfecte contine multimea {x N | x 1000}?

5. Cte multimi X verific


a {1, 2} X = {1, 2, 3, 4, 5}?

6. Cte numere xy scrise n baza 10 verific


a relatia x + xy = y + yx + 50?
7. Dac
a a b + a c = b c + c2 , b + c = 1003, calculati a + 2b + c.

8. S
a se determine un num
ar de 4 cifre al c
arui produs cu 9 se termin
a cu 3755.
9. Dac
a restul mp
artirii lui 2a + 3b + 5c la 13 este 7, aflati restul mp
artirii lui
34a + 51b + 85c la 13.
10. Cte numere care se scriu cu 4 cifre n baza 10 au suma cifrelor 3?
11. Se consider
a 5 numere naturale distincte avnd suma 60. G
asiti numerele
dac
a suma diferentelor dintre cel mai mare si fiecare dintre celelalte 4 numere este
10.
12. Care sunt ultimele dou
a cifre ale lui 52006 + 72006 ?
o
na
| a, b N , a + b = 2006 . Care este produsul elementelor lui
13. Fie M =
b
M?
artind un num
ar la
14. Suma a dou
a numere naturale este 3n + 5, n N . mp
dublul celuilalt obtinem ctul n si restul 0. Care sunt numerele?
15. Se scriu n ordine cresc
atoare toate numerele nenule din baza 10 n a c
aror
scriere nu apar alte cifre afar
a de 0, 1, 2 si 3. Care este cel de-al 2006-lea num
ar?

Clasa a VI-a
1. Care este cel mai mic num
ar natural de nou
a cifre divizibil cu trei?
2. Fie a, b, c, d m
asurile a 4 unghiuri formate n jurul unui punct. S
a se afle
a
b
c
d
aceste m
asuri dac
a = = = .
1
2
3
4
3. Pretul unui obiect se micsoreaz
a cu 20%. Cu ct la sut
a trebuie s
a se m
areasc
a
noul pret pentru a se ajunge la pretul initial?
4. Calculati m
asura unui unghi stiind c
a triplul complementului s
au este cu 42
30 mai mare dect jum
atatea suplementului s
au.
0

127

5. Care este cea mai mare valoare pe care o poate lua num
arul n = a1 a2 + a3
a4 + + a9 a10 , unde a1 , a2 , . . . , a10 sunt numere naturale distincte din multimea
{1, 2, 3, . . . 10}?
AC
6. Fie A, B, C, D puncte pe o dreapt
a, n aceast
a ordine, astfel nct
= 1,
BD
7
BC
AD
= . Calculati
.
BD
5
AD
7. Se dau dou
a vase cu ap
a astfel nct dac
a turn
am jum
atate din primul n al
doilea si jum
atate din cantitatea de ap
a ce se afl
a acum n al doilea o turn
am n
primul si lund apoi jum
atate din cantitatea aflat
a n primul si o turn
am n al doilea
obtinem (n al doilea vas) 10 litri ap
a. Aflati cti litri de ap
a se afl
a n fiecare vas,
stiind c
a ambele cantit
ati sunt numere ntregi.
8. Ar
atati c
a num
arul a = 3 + 32 + 33 + + 34n se divide cu 120, n N .

9. S
a se calculeze a 2007-ea zecimal
a a num
arului n = 0, 00 (300)+0, 00 (3000)+1.

10. S
a se afle x, stiind c
a



1
2
3
n
1
%din
%din
%din . . .
%din x . . .
=
.
2
3
4
n+1
100n (n + 1)

222 . . . 23
11. Fie n un num
ar natural, n > 2. Num
ar
atorii si numitorii fractiilor
333 . . . 34
555 . . . 56
si
au cte n cifre. Care este fractia mai mare?
666 . . . 67
12. Cte unghiuri cu m
asurile numere naturale consecutive se pot forma n jurul
unui punct?
13. M
asurile unghiurilor unui triunghi sunt direct proportionale cu trei numere
invers proportionale cu trei numere direct proportionale cu trei numere puteri consecutive ale lui 3. G
asiti m
asurile unghiurilor.
14. Fie S = 1 + 2 + 3 + + 102006 . La ce putere apare 2 n descompunerea n
factori primi a lui S?

2006!
15. S
a se afle cardinalul multimii A = (x, y) N N | x
N , unde
7 11y
n! = 1 2 3 n.

Clasele VII-XII, 19 aprilie 2006


Juniori
1 1
1
1
+ +
= are o infinitate de solutii n (N )3 .
a b ab
c
BD
2. Fie 4ABC dreptunghic n C si D (BC), E (CA) astfel nct
=
AC
AE
\ = 60
= k. Dreptele BE si AD se intersecteaz
a n O. S
a se arate c
a m(BOD)
CD

dac
a si numai dac
a k = 3.
1. S
a se arate c
a ecuatia

128

3. Se dau n plan 5 puncte cu proprietatea c


a oricare dintre cele 10 triunghiuri
cu vrfurile n aceste puncte are aria cel mult egal
a cu 1. S
a se arate c
a exist
a un
trapez de arie cel mult 3 ce contine n interior sau pe laturi cele 5 puncte.

Seniori
kp k
p+1

(mod p).
p
2
k=1
2. Fie p 5 num
ar prim. Determinati num
arul polinoamelor de forma X p +
k
l
pX + pX + 1, k > l, k, l {1, 2, . . . p 1} care sunt ireductibile n Z [X].
3. Mediana AM a triunghiului ABC taie cercul nscris n K si L. Dreptele duse
prin K si L paralele la BC taie a doua oar
a cercul nscris n X si Y . Fie P , Q
intersectiile dreptelor AX si AY cu BC. Ar
atati c
a BP = CQ.
1. Fie p 3 num
ar prim. Demonstrati c
a

8
3
9
10
1

11
6
2
5
2

p1
P

ARITMOGRIF
1
6
3
3
6

12
2
1
1
13

9
2
8
8
2

14
12
4
12
12

?
8
5
13
15
16

7
6
1
13
17

18
6
15
2
8

19
4
10
8
8

8
5
10
1
7

12
2
6
19
15

9
4
7
2
15

6
4
3
13
20

4
1
5
6
17

21
15
2
6
17

7
6
9
13
3

12
1
2
19
2

nlocuind numerele cu litere veti obtine numele a 18 matematicieni.


Pe orizontal
a A-B: numele unei reviste de matematic
a pentru elevi si profesori.
Prof. Valeriu Brasoveanu
C. N. "Gh. Rosca Codreanu", Brlad
(Rezolvarea aritmogrifului se g
aseste la pag. 162)

Vizitati pe Internet revista "Recreatii Matematice" la adresa

http://www.recreatiimatematice.uv.ro
129

Concursul de matematic
a Florica T. Cmpan
Etapa judetean
a, 18 februarie 2006
Clasa a IV-a
1. S
a se mpart
a la trei persoane 24 sticle de suc identice ca m
arime, din care 5
sunt pline, 11 umplute pe jum
atate si 8 goale, nct fiecare s
a aib
a acelasi num
ar de
sticle, dar si aceeasi cantitate de suc.
nv. Lina Huzum
2. ntre cele 9 numere de mai jos exist
a un "intrus". Acesta nu respect
a relatia
dintre cifre ce exist
a la fiecare din celelalte opt numere. Descoper
a si scrie relatia,
precum si num
arul "intrus": 9334, 4862, 6148, 5132, 7835, 3524, 9963, 9782, 8133.
nv. F
anic
a Dragnea
3. Cntarul pe care vor s
a se cnt
areasc
a trei copii nu m
asoar
a mase mai mici de
40 kg. Fiecare din cei trei copii cnt
aresc ntre 25 si 30 kg.
Cum a reusit fiecare copil s
a se cnt
areasc
a?
Inst. Iulian Cristea

Clasa a V-a
1. Cifrele care alc
atuiesc vrsta bunicului reprezint
a vrstele celor doi nepoti.
Dac
a mp
artim vrsta bunicului la suma vrstelor nepotilor, se obtine ctul 4 si
restul 12. Aflati vrsta bunicului si vrstele nepotilor.
Mihaela Cianga
2. Se consider
a nmultirea urm
atoare, unde literele nu reprezint
a
a3b
obligatoriu cifre distincte:
cd
a) Determinati e.
ef 3g
b) Ar
atati c
a bd = 63.
hik
c) Reconstituiti nmultirea.
2np3
Gabriel Popa
3. Se consider
a multimea A = {2, 3, 4, . . . , 13}.
a) Determinati B, C disjuncte astfel nct B C = A si suma elementelor din B
este egal
a cu sum
a elementelor din C.
b) Ar
atati c
a nu exist
a M , N disjuncte cu M N = A si produsul elementelor
din M egal cu produsul elementelor din N .
c) G
asiti dou
a multimi X, Y disjuncte cu X Y = A, X avnd dou
a elemente,
cu produsul elementelor lui X egal cu suma elementelor lui Y .
Ionel Nechifor

Clasa a VI-a
1. Pe o tabl
a s-au scris trei numere naturale. Cnd n locul lor s-au scris: suma,
produsul si suma produselor cte dou
a, s-a v
azut c
a pe tabl
a au ap
arut aceleasi
numere ca si cele initiale. Care este produsul lor? Explicati!
2. Pe o tabl
a este scris num
arul 12. La fiecare minut num
arul se nmulteste sau
se mparte f
ar
a rest fie la 2, fie la 3, iar rezultatul se scrie pe tabl
a n locul num
arului
initial. S
a se arate c
a num
arul scris pe tabl
a dup
a exact o or
a nu poate fi 54.
\ si BOC
\ sunt adiacente suplementare. Fie [Ox si [Oy
3. Unghiurile proprii AOB
[ N si m(COx)
[ = p m(BOy),
[ unde p este
bisectoarele acestora. Dac
a m(BOy)
130

num
ar prim, aflati num
arul p.

Vasile Nechita

Clasa a VII-a
1. Fie a un num
ar natural arbitrar, divizibil prin 9, avnd 2007 cifre. Not
am cu
s (a) num
arul care reprezint
a suma cifrelor lui a. G
asiti s (s (s (a))).
Gabriel Mrsanu
2. Vrfurile unui cub se noteaz
a cu 8 numere ntregi consecutive, iar centrul
fiec
arei fete se noteaz
a cu media aritmetic
a a vrfurilor fetei respective.
a) S
a se demonstreze c
a suma centrelor oric
aror dou
a fete opuse este aceeasi si s
a
se afle ct este aceasta n functie de cel mai mic dintre numerele din vrfuri.
b) S
a se g
aseasc
a n ce conditii centrul unei fete este num
ar ntreg. S
a se scrie
vrfurile fetelor ale c
aror centre sunt numere ntregi.
c) Ar
atati c
a, dac
a centrele a trei fete care au un vrf comun sunt notate cu
numere ntregi, atunci si centrele celorlaltor fete sunt notate tot cu numere ntregi.
Julieta Grigoras
3. n tara TI a triunghiurilor isoscele era mp
arat, firesc, triunghiul echilateral.
El decretase c
a este singurul care binemerit
a numele de Prearostogolibil ; supusii s
ai
trebuiau s
a fie numiti teposi dac
a au o latur
a mai scurt
a dect cele egale, respectiv
turtiti dac
a au o latur
a mai lung
a dect cele egale. (Vorba congruent era socotit
a de
ocar
a pe acele meleaguri.) Niste unghiuri umblau venetice prin TI c
autnd fiecare
triunghi isoscel la al c
arui vrf s
a slujeasc
a.
Teposule,

zise un unghi . Eu si vecinii mei de pribegie blbitul de si


nem
asuratul de ne c
aut
am st
apni n TI. Ne-ai fi de mare folos dac
a ai binevoi s
a
ne spui dac
a nu cumva ai o bisectoare interioar
a a ta exact att de lung
a ca o latur
a.
Dup
a vorbire se cunoaste c
a veniti de pe coclauri unde lucrurile nu sunt f
acute
din linii drepte bine limitate. ntrebi de lucruri la care nu gndeste nimeni fiindc
a
nu sunt de niciun folos. Dar, pn
a cercetez pentru r
aspuns, fii bun m
ai cr
ac
anatule si spune-mi dac
a asa se obisnuieste pe la voi: s
a-ti ponegresti colegii cu vorbe
necuviincioase?
Nu e necuviinta, prea-limitatule. Eu, , m
a exprim frumos n grade, de aceea
sunt purt
ator de cuvnt; nu cunoaste fractii ordinare ci doar zecimale si se blbie
grozav cnd ncearc
a s
a spun
a cte grade are; nc
a nu stie dac
a este m
asurabil n
grade. Dar bag seam
a c
a ntrzii cu r
aspunsul la ntrebarea mea; o fi capul t
au mai
mult ascutit dect nc
ap
ator?
Bine, m
ai vorb
aretule. Am cercetat si r
aspund precis: am exact dou
a bisectoare
interioare exact asa de lungi ca laturile mele egale.
Am nteles. Te rog s
a m
a ierti c
a ti-am zis tepos; nteleg c
a esti turtit. Mie
personal nu mi esti de folos, dar iat
a c
a pentru esti bun de st
apn. Dac
a bisectoarele tale egale erau ct latura ta scurt
a, te recunosteam de st
apn. Dac
a o singur
a
bisectoare a ta era ct latura ta scurt
a, te-ar fi slujit cu credinta.
a) Exprimati cu fractii ordinare gradele lui si .
b) Exprimati cu fractii zecimale num
arul de grade, minute si secunde ale lui .
c) Argumentati c
a triunghiurile isoscele care convin lui , respectiv , sunt tepoase, respectiv turtite.
131

d) Desenati un triunghi isoscel cu unghiurile de la vrf . Este el tepos sau turtit?


Dan Brnzei

Clasa a VIII-a
1. ntr-o clas
a sunt 20 de elevi. Fiecare fat
a ofer
a fiec
arui b
aiat trei flori si fiecarei
fete o floare, iar fiecare b
aiat ofer
a cte trei flori fiec
arei fete si cte o floare fiec
arui
b
aiat.
a) Ar
atati c
a num
arul maxim de flori oferite este 780.
b) Cte fete ar trebui s
a fie n clas
a, astfel nct s
a fie oferite exact 780 de flori?
Monica Nedelcu, Iasi
2. Pe fiecare fata a unui cub este scris cte un num
ar natural nenul, iar fiec
arui
vrf i corespunde produsul numerelor de pe cele trei fete adiacente acestuia. Dac
a
suma numerelor corespunz
atoare tuturor vrfurilor este 2006, ar
atati c
a exist
a cel
putin dou
a fete pe care este scris acelasi num
ar.
3. Fie paralelipipedul dreptunghic ABCDM N P Q si punctele E (BN ), F
(DQ) astfel nct suma AE + AF + P E + P F este minim
a. Ar
atati c
a EF AP
dac
a si numai dac
a ABCDM N P Q este prism
a regulat
a.
Valentina Blendea si Gheorghe Blendea, Iasi

Faza interjudetean
a, 20 mai 2005
Clasa a IV-a
1. Lungimea laturii unui p
atrat este de 17 m. O persoan
a pleac
a dintr-un vrf al
p
atratului si, mergnd n acelasi sens pe laturile acestuia, parcurge o distanta de 637
m. Din punctul n care a ajuns se ntoarce si parcurge 773 m. Aflati la ce distanta
se va situa n final persoana, fata de punctul de plecare.
RecMat - 1/2005, P.90
2. Avem mai multe vase identice. S
tim c
a 50 vase pline cu ap
a cnt
aresc 600 kg,
iar 10 vase goale cnt
aresc cu un kilogram mai putin dect apa dintr-un vas plin. S
a
se afle ct cnt
aresc 100 de vase goale.
Petru Asaftei
3. n cte moduri diferite pot fi scrise numerele 1, 2, 3, 4 n p
atratele din
figura al
aturat
a, cte unul n fiecare p
atrat, astfel nct s
a nu existe dou
a
p
atrate al
aturate n care suma s
a fie 5? Justificati. (P
atratele care au doar
un vrf comun nu sunt considerate al
aturate, iar p
atratul mare este fix.)
Petru Asaftei

Clasa a V-a
1. Din produsul tuturor numerelor naturale de la 1 la 2006 se exclud toate
numerele care se divid cu 5. n ce cifr
a se termin
a produsul celorlaltor numere?
2. Pe o tabl
a sunt scrise numerele 1,3,4,6,8,9,11,12,16. Doi copii au sters cte
patru numere si s-a observat c
a suma numerelor sterse de unul este de trei ori mai
mare dect suma numerelor sterse de cel
alalt. Ce num
ar a r
amas scris pe tabl
a?
3. Fie num
arul 123456789. O operatie nseamn
a s
a alegem dou
a cifre al
aturate
c
arora s
a li se scad
a o unitate si s
a li se schimbe locurile (de exemplu: 123456789
123436789 ...). Care este cel mai mic num
ar ce se poate obtine ca rezultat al
acestor operatii? Dup
a cte operatii se obtine cel mai mic num
ar?
132

Clasa a VI-a
1. Se pot aseza pe muchiile unui cub numerele 1, 2, 3, . . . , 12 (cte un num
ar pe
fiecare muchie) astfel nct suma numerelor aflate pe cele trei muchii care pleac
a din
acelasi vrf s
a fie aceeasi pentru fiecare vrf al cubului?
Constantin Chiril
a

\
2. Se consider
a triunghiul ABC cu [AB] [AC] si m(BAC) = 20 . Fie
\ ) = m(CBP
\) = 20 si N (AB) astfel nct
M, P (AC) astfel nct si m(ABM

\
\
m(BCN ) = 50 . S
a se afle m(AM N ).
3. Numerele naturale 22, 23, 24 au urm
atoarea proprietate: descompunerile n factori primi ale numerelor din sir au exponentii factorilor numere impare (22 = 21 111 ,
23 = 231 , 24 = 23 31 ). Care este cel mai mare num
ar de numere naturale consecutive
care au aceast
a proprietate?
Cristian - C
at
alin Budeanu

Clasa a VII-a
1. a) Ar
atati c
a din oricare 3 numere naturale, putem alege dou
a astfel nct
suma lor s
a fie un num
ar par.
b) Fiind date sapte numere naturale, ar
atati c
a putem alege patru dintre ele astfel
nct suma lor s
a fie divizibil
a cu 4.
Cristian Laz
ar
2. Fie ABC isoscel cu BC = 2a, AB = AC = b, a, b N . S
a se determine
toate triunghiurile ABC, dac
a a = 2r, unde r este raza cercului nscris n ABC, si
apoi g
asiti ABC cu aria minim
a.
Dan Brnzei
3. Un trapez ABCD are baza mare [AB] si [AC] [BD] = {O}. Linia mijlocie
a trapezului intersecteaz
a pe AC n E si pe BD n F .
a) Demonstrati c
a ABCD este trapez isoscel dac
a si numai dac
a [OE] [OF ].
b) Vrfurile trapezului si punctul O reprezint
a 5 orase, iar laturile si diagonalele
sale sunt sosele de leg
atur
a. Dou
a masini pleac
a din D, respectiv C pe ruta cea mai
scurt
a spre A, respectiv spre B si alte dou
a masini pleac
a din A respectiv B spre D,
respectiv C, trecnd prin O pe ruta cea mai scurt
a. Cele 4 masini au aceeasi vitez
a,
constant
a, pe ntreg parcursul. Demonstrati c
a primele 2 masini ajung simultan n
D, respectiv C. Pot ajunge, toate patru, n acelasi timp la destinatie?
Claudiu-
Stefan Popa

Clasa a VIII-a
1. Aflati perechile de numere ntregi cu proprietatea c
a diferenta cuburilor lor
este egal
a cu p
atratul diferentei lor.
2. Un poliedru are 17 muchii si 9 fete.
a) Desenati dou
a astfel de poliedre diferite.
b) Dac
a fetele poliedrului sunt doar triunghiuri echilaterale sau p
atrate, iar toate
muchiile sunt de lungime 1 cm, calculati aria sa total
a.
Gabriel Popa

3. Un cub cu latura de n cm (n N ) se mparte n cuburi cu latura de 1 cm


si se coloreaz
a toate cuburile situate pe diagonalele fetelor cubului initial. Aflati n
astfel ca num
arul cuburilor colorate s
a fie 2006.
Julieta Grigoras
133

Solutiile problemelor propuse n nr. 2 / 2005


Clasele primare
P.94. Aflati numerele de doua cifre cu proprietatea ca diferenta dintre numar si
rasturnatul lui este egala cu cel mai mare numar scris cu o singura cifra.
(Clasa I )
Mara Neicu, elev
a, Hrl
au
Solutie. Dac
a un num
ar de dou
a cifre ndeplineste conditia din problem
a, atunci
diferenta dintre num
arul reprezentat de cifra zecilor si num
arul reprezentat de cifra
unit
atilor este egal
a cu 1. Numerele care verific
a cerinta sunt: 21, 32, 43, 54, 65, 76,
87, 98.
P.95. La ora de educatie fizica, fetele unei clase sunt aliniate de la cea mai nalta
la cea mai scunda, iar baietii dupa ele, n aceeasi ordine. Ana este cea mai nalta.
Ene este cel mai nalt, iar Sorin cel mai scund. ntre Ana si al doilea baiat sunt 15
copii, iar ntre Ene si Sorin sunt 10 baieti. Cti elevi sunt aliniati la educatia fizica?
(Clasa I )
nv. Constanta Cristea si Inst. Iulian Cristea, Iasi
Solutie. Num
arul fetelor este 151+1 = 15, iar num
arul b
aietilor este 10+1+1 =
12. La ora de Educatie fizic
a sunt aliniati 27 elevi.
P.96. Descopera
2 5
6
a)
3 8
4
9 7
8
b)
2 5
1
(Clasa a II-a)

regula
10
14
7
6

si completeaza casutele libere n cazurile:


5 7
1
3
;
2
6 13
4
3 3
5 3
.
0
1
2 0
Inst. Maria Racu, Iasi

a b
, atunci regula este dat
a de
c d
a + c = b si b + c = d. n tabelul al treilea complet
am c
asuta liber
a cu 2 + 7 = 9, n
al patrulea cu 13 6 = 7, iar n ultimul tabel cu 3 + 4 = 7, respectiv 4 + 7 = 11.
Solutie. a) Dac
a tabelul este de forma

b) n acest caz regula este dat


a de a c = b si b c = d. Tabelele se completeaz
a
cu 3 0 = 3; 5 3 = 2; b = 0 + 2, a = b + 2 = 2 + 2 = 4.
P.97. n cte moduri putem forma un sir indian compus din 4 baieti si 3 fete
astfel nct doua fete sa nu stea una lnga alta, iar sirul sa nu nceapa cu o fata?
(Clasa a II-a)
Andrei Burdun, elev, Iasi
Solutie. S
irul poate ncepe cu cel mult doi b
aieti. Avem posibilit
atile: bfbfbfb;
bfbbfbf; bfbfbbf; bbfbfbf. S
irul poate fi format n 4 moduri.

P.98. La Craciun, copii au mpodobit bradul cu 35 globuri albe, galbene si rosii.


Andrei a observat ca, daca mparte numarul globurilor albe la cele galbene obtine
ctul 3 si restul 2, iar daca mparte numarul globurilor rosii la cele galbene obtine
ctul 2 si restul 3. Cte globuri de fiecare fel au mpodobit bradul?
(Clasa a III-a)
nv. Rica Buc
ataru, Iasi
Solutie. Folosim metoda figurativ
a.
134

Num
arul globurilor galbene este
[35 (2 + 3)] : 6 = 5.
35
Num
arul globurilor albe este 5 3 + 2 = 17.
3
Num
arul globurilor rosii este 5 2 + 3 = 13.
P.99. La concursul de alergare organizat de clasa a II-a, cei mai buni baieti sunt
Radu, Cezar, Tudor, Dan si Mihai. Care a fost clasamentul final, daca: 1) Radu
nu a luat locul nti, 2) Cezar s-a clasat n urma lui Mihai, 3) Radu s-a clasat
naintea lui Mihai, 4) Tudor nu s-a clasat al doilea, 5) Dan este al treilea dupa
Tudor.
(Clasa a III-a)
nv. Constanta Cristea si Inst. Iulian Cristea, Iasi
Solutie. Deoarece Tudor nu s-a clasat al doilea si Dan este al treilea dup
a Tudor,
deducem c
a Tudor s-a clasat pe locul I. Tinnd

cont si de ordinea RMC, rezult


a
clasamentul final TRMDC.
P.100. Trei numere naturale au suma 60. Sa se afle numerele stiind ca mpartindu-l pe primul la al doilea obtinem ctul 4 si restul 3, iar al treilea este cu 1 mai
mare dect dublul celui de-al doilea.
(Clasa a III-a)
Vasile Solcanu, Bogd
anesti, Suceava
Solutie. Utiliz
am metoda figurativ
a.
Al doilea num
ar este
3
a
b = [60 (3 + 1)] : 7 = 8.
b
60 Primul numar este a = 48+3 = 32+3 = 35.
1
Al treilea num
ar este c = 2 8 + 1 = 17.

P.101. Exista numere naturale care mpartite la 12 sa dea ctul 7, iar mpartite
la 15 sa dea restul 2?
(Clasa a IV-a)
Alexandru-Gabriel Tudorache, elev, Iasi
Solutie. Fie n = 12k + 7 = 15p + 2. Atunci n = 3 (4k + 2) + 1 = 3 5p + 2,
adic
a pe de o parte n d
a la mp
artirea prin 3 restul 1, pe de alt
a parte d
a restul 2,
imposibil.
P.102. Sa se arate ca patratul de latura 36 poate fi acoperit cu piese de forma
1
.
1
(Clasa a IV-a)
Andrei Burdun, elev, Iasi
Solutie. Prin mbinarea a dou
a piese putem obtine o pies
a drep1
tunghiular
a de forma al
aturat
a. Pentru acoperirea p
atratului sunt
1
necesare (36 : 4) (36 : 2) = 162 piese dreptunghiulare. P
atratul
1
poate fi acoperit cu 162 2 piese initiale.
P.103. Un dreptunghi are perimetrul de 624 cm, iar lungimea este dublul latimii.
Poate fi mpartit dreptunghiul ntr-o retea de patrate egale astfel nct suma perimetrelor lor sa fie 6656 cm?
(Clasa a IV-a)
Petru Asaftei, Iasi
Solutie. L
atimea dreptunghiului este 624 : 6 = 104 cm. Dac
a mp
artim dreptunghiul n n p
atrate egale, atunci n poate fi 2, 8, 32, 128, 512, . . . . Suma perimetrelor este 4nl = 6656 cm, de unde nl = 1664 cm. Pentru n = 2 obtinem l = 832 cm

a
b

135

> 104 cm, fals. Pentru n = 8 obtinem l = 208 cm > 104 cm, fals. Pentru n = 32
obtinem l = 52 cm, ceea ce nseamn
a c
a l
atimea poate fi acoperit
a cu de dou
a ori
latura unui p
atrat, ceea ce este fals, deoarece dreptunghiul trebuie acoperit cu 32
p
atrate. Pentru n = 128 obtinem l = 13 cm. Atunci dreptunghiul poate fi acoperit
cu (104 : 13) (208 : 13) = 8 16 = 128 p
atrate. Pentru n > 128 nu avem solutii.

Clasa a V-a
V.61. Determinati x, y, z N n fiecare din cazurile:
25
25
a) x y =
;
b) x2 + y 2 =
;
2z + 1
2z + 1
Vasile Solcanu, Bogd
anesti, Suceava
Solutie. a) Deoarece xy N, atunci 2z + 1 | 25, adic
a 2z + 1 {1, 5, 25}, prin urmare z {0, 2, 12}. Dac
a z = 0, atunci xy = 25, deci (x, y) {(1, 25) ; (5, 5) ; (25, 1)}.
Dac
a z = 2, atunci xy = 5, deci (x, y) {(1, 5) ; (5, 1)}. Dac
a z = 12, atunci xy = 1,
deci (x, y) = (1, 1). n concluzie,
(x, y, z) {(1, 25, 0) ; (5, 5, 0) ; (25, 1, 0) ; (1, 5, 2) ; (5, 1, 2) ; (1, 1, 12)} .

b) Obtinem tot z {0, 2, 12} si, analiznd toate cazurile, g


asim c
a

(x, y, z) {(0, 5, 0) ; (5, 0, 0) ; (3, 4, 0) ; (4, 3, 0) ; (1, 2, 2) ; (2, 1, 2) ; (0, 1, 12) ; (1, 0, 12)} .
S
a not
am c
a a) si b) nu pot avea loc simultan.

V.62. Sa se scrie numarul 12321 ca diferenta a doua patrate perfecte.


Andrei-Sorin Cozma, elev, Iasi
Solutie. Observ
am c
a 12321 = 32 372 . nmultim relatia 32 + 42 = 52 cu 372 si
obtinem c
a 32 372 + 42 372 = 52 372 , de unde 32 372 = 52 372 42 372 , deci
12321 = 34225 21904.
3
3
V.63. Sa se scrie n ordine crescatoare numerele 33 ; 333 ; 333; 333 ; 33 .
Ion Visan, Craiova
Solutie. Observ
am c
a
3
3 3
3
333 > 327 = 33 = 39 = 813 > 333 > 273 = 33 > 32 = 93 = 729 > 333,
3
3
deci 333 < 33 < 333 < 33 < 333 .

V.64. Sa se determine x, y, z N, z 6= 0, nct 5x +5y = z!, unde z! = 123. . .z.


Doru Turbatu, Iasi
Solutie. Not
am U2 (A) ultimele dou
a cifre ale num
arului A. Dac
a x 2, y 2,
atunci U2 (5x + 5y ) = 50, n timp ce U2 (z!) 6= 50, z N. Dac
a x = 1, y 2, atunci
U2 (5x + 5y ) = 30, n timp ce U2 (z!) 6= 30, z N. Analog se arat
a c
a nu convine
cazul x 2, y = 1. R
amn de studiat cazurile (x, y) {(0, 0) , (1, 0) , (0, 1) , (1, 1)}.
Analizndu-le pe rnd, obtinem solutiile (x, y, z) {(0, 0, 2) , (1, 0, 3) , (0, 1, 3)}.

V.65. Vom numi "num


ar p" un numar natural care are exact 4 divizori.
a) Dati exemplu de trei numere p consecutive.
b) Sa se arate ca nu exista trei numere p consecutive astfel nct primul dintre ele
sa fie par.
Ovidiu Pop, Satu Mare
Solutie. a) De exemplu, putem considera numerele 33, 34, 35.
136

b) Presupunem prin absurd c


a exist
a a = 2k, b = 2k + 1, c = 2k + 2 = 2 (k + 1),
k N, trei numere p consecutive. Cum 2k si 2 (k + 1) au fiecare exact 4 divizori,
numerele k si k + 1 trebuie s
a fie ambele prime. Singurele numere prime consecutive
sunt 2 si 3, deci k = 2, apoi a = 4, b = 5, c = 6. Evident ns
a c
a 5 nu este numar p.

Clasa a VI-a
VI.61 Daca a = x9 y 3 z 4 t10 , b = xy 5 z 6 t8 , c = x5 y 7 z 10 t12 si |a| + a = 0, sa se
arate ca |b| + b = |c| + c = 0.
Cristian - C
at
alin Budeanu, Iasi

2
Solutie. Evident c
a |a| + a = 0 a 0. Deoarece a = xy x4 yz 2 t5 0,
rezult
a c
a xy 0. Atunci

b = xy y 2 z 3 t4 0,
c = xy x2 y 3 z 5 t6 0,

prin urmare |b| + b = |c| + c = 0.

VI.62. Fie a1 , a2 , . . . , an {1, 1} astfel nct


an1 an a1
an a1 a2
a1 a2 a3 a2 a3 a4
+
+ +
+
= 0.
a4
a5
a2
a3

Sa se arate ca n se divide cu 4.

Ioana Olan, elev


a, Iasi
Solutie. Suma dat
a are n termeni, fiecare dintre ei fiind +1 sau 1. Rezultatul
fiind 0, num
arul termenilor egali cu +1 este acelasi cu al celor egali cu 1, prin
urmare n = 2k. Produsul celor n fractii este, pe de o parte, (1)k (+1)k = (1)k , pe
.
de alta este a2 a2 a2 = +1; rezult
a k = 2l, l N. n concluzie, n = 4l, deci n .. 4.
1 2

S
a remarc
am c
a pentru n = 4 nu exist
a numere a1 , a2 , a3 , a4 care s
a verifice
ipotezele problemei, ns
a pentru n = 8 putem considera numerele 1, 1, 1, 1, 1, 1,
1, 1.

VI.63. Se poate completa un patrat 10 10 cu numerele de la 1 la 100, asa nct


pentru fiecare coloana sa se poata forma (cu numerele din acea coloana) trei grupe,
astfel ca sumele numerelor din fiecare grupa sa fie egale?
Bogdan Andrei Ciacoi, elev, Gherla
100 101
Solutie. Suma tuturor numerelor din p
atrat este
= 5050. Dac
a s-ar
2
putea completa p
atratul n acest fel, atunci datorit
a faptului c
a pe fiecare coloan
a
suma numerelor este multiplu de 3, suma numerelor de la 1 la 100 ar fi multiplu de 3.
Cum 5050 nu este multiplu de 3, rezult
a c
a p
atratul nu se poate completa n modul
dorit.
VI.64. Pentru fiecare n N, n 4, aratati ca interiorul oricarui patrulater
convex se poate descompune n reuniune de n triunghiuri dreptunghice cu interioarele
disjuncte.
Gabriel Popa, Iasi
Solutie. Observ
am nti c
a interiorul oric
arui triunghi poate fi mp
artit n dou
a
triunghiuri dreptunghice, ducnd n
altimea din vrful unghiului cel mai mare (precautie necesar
a dac
a triunghiul initial este obtuzunghic). Dac
a n = 4, o diagonal
a
mparte interiorul patrulaterului n dou
a triunghiuri, iar dup
a procedeul de mai sus
137

obtinem 4 triunghiuri dreptunghice. Dac


a se poate face o descompunere n k triunghiuri dreptunghice, putem realiza descompunerea n k + 1 triunghiuri, ducnd
n
altimea ipotenuzei n unul din primele k si cu aceasta rezolvarea este ncheiat
a.
VI.65. Fie 4ABC, DE k BC, cu E (AC) si D (AB), iar F (BD)
si {G} = F E DC. Demonstrati ca, daca doua dintre urmatoarele afirmatii sunt
adevarate, atunci este adevarata si a treia:
(i) BD = 2 BF ;
(ii) AC = 2 CE;
(iii) EF = 2 F G.
Claudiu - S
tefan Popa, Iasi
Solutie. Fie H mijlocul lui [CE], iar {L} = BE
A
F H, {K} = CD F H.
(i), (ii) (iii). Folosim n mod repetat teoremele
liniei mijlocii ntr-un triunghi/ trapez si reciprocele lor.
D
E
BC
G
Cum AE = EC si DE k BC, atunci DE =
. Apoi,
2
K
F
H
din DF = F B si EH = HC, obtinem F H k BC,
L
deci [F L], [F K] vor fi linii mijlocii n 4BDE, respectiv
DE
BC
4DBC. Atunci F L =
=
, LK = F K F L = B
C
2
4
BC
BC
BC

=
, adic
a F L = LK = KH. n 4F EC, F H este median
a si
2
4
4
2
FK
= , deci K va fi centru de greutate; rezult
a c
a CK este median
a, adic
a
KH
1
F G = GE.
(i), (iii) (ii). n 4CEF , F H si CG sunt mediane, deci K va fi centru de
BC
DE
greutate. Rezult
a c
a F K = 2 KH si cum F K =
, KH =
, atunci BC =
2
2
2 DE. Conform unei a doua reciproce a teoremei liniei mijlocii n triunghi, mai putin
cunoscut
a, rezult
a c
a [DE] este linie mijlocie n 4ABC (altfel [D0 E 0 ] ar fi acea linie
BC
a DEE 0 D0 ar fi
mijlocie si cum DE = D0 E 0 =
, DE k D0 E 0 k BC, am obtine c
2
paralelogram, adic
a DD0 k EE 0 si atunci AB k AC, fals!).

(ii), (iii) (i). Presupunem prin absurd c


a F nu este mijlocul lui [BD]; fie atunci
F 0 acest mijloc, iar {G0 } = F 0 E DC. Din (ii), EF 0 = 2 F 0 G0 , deci [GG0 ] este linie
mijlocie n 4EF F 0 , prin urmare GG0 k BD, adic
a DC k BD, absurd. R
amne c
a
are loc (i).

Clasa a VII-a
VII.61. Fie n N fixat si p, k N, p < k. Sa se arate ca

1
1
2 (k p + 1)
1
+
+ +
>
.
n+p n+p+1
n+k
2n + k + p

(n leg
atur
a cu V.31 din RecMat - 2/2002).

Gigel Buth, Satu Mare

Solutie. Avem:
(n + p) + (n + p + 1) + + (n + k) =

(n + k) (n + k + 1) (n + p 1) (n + p)

=
2
2
138

(2n + k + p) (k p + 1)
.
2
Inegalitatea dintre media armonic
a si cea aritmetic
a arat
a c
a
=

(n + p) + (n + p + 1) + + (n + k)
>
kp+1

1
n+p

kp+1
1
+ n+p+1
+ +

(1)

1
n+k

(2)

Din (1) si (2) obtinem inegalitatea dorit


a. Problema V.31 se obtine pentru n =
100, p = 1, k = 100.
VII.62. Fie a R astfel nct a10 a6 + a2 = 4. Sa se arate ca 7 < a12 < 16.
Alexandru Negrescu, elev, Botosani
1
10
6
2
a a 6= 0 si a 6= 1, de unde a2 + 2 > 2
Solutie. Faptul c
a a a + a = 4 implic
a
4
8

1
a4 + 1 10
4
12
4
6
a a + a2 =
si a + 4 > 2. Din a + 1 = a + 1 a a + 1 =
2
a
a

1
4 a2 + 2 , avem a12 + 1 > 4 2, deci a12 > 7. Din a10 + a2 = a6 + 4, avem
a
1
4
4
4
a + 4 = 1 + 6 , de unde 1 + 6 > 2, deci a6 < 4, prin urmare a12 < 16.
a
a
a
VII.63. Fie triunghiurile ABC si A0 B 0 C 0 cu AB = A0 B 0 , BC = B 0 C 0 si bisectoarele [BE], [B 0 E 0 ] congruente. Sa se arate ca 4ABC 4A0 B 0 C 0 .
Petru Asaftei, Iasi
Solutie. A se vedea n acest num
ar nota Criterii de congruenta a triunghiurilor,
pag. 107-109.
VII.64. n triunghiul echilateral ABC consideram cevienele AM , BN si CP
concurente ntr-un punct O, interior triunghiului. Aratati ca, daca 4M N P este
echilateral, atunci O este centrul 4ABC.
Temistocle Brsan, Iasi
Solutie. Unghiurile marcate sunt congruente. ntrA
b = 60 implic
\
\
adev
ar, m(A)
a m(AN
P ) = 120 m(AP
N );
\
\
\
m(N
P M ) = 60 implic
a m(BP
M ) = 120 m(AP
N ),
\
\
\
\
deci AN P BP M . Analog ar
at
am c
a BP N CM N ;
P
N
\
\
\
vom avea si egalit
atile AP
N BM
P CN
M . Se arat
a
O
usor c
a 4AN P 4BP M 4CM N , de unde AP =
BM = CN si AN = BP = CM . Conform teoremei lui
MB NC P A
M
C
Ceva, are loc relatia

= 1, care revine B
MC NA P B

3
MB
la
= 1 sau M B M C. Rezult
a c
a M , N , P sunt mijloacele laturilor
MC
triunghiului ABC, deci O este centrul acestuia.
VII.65. Fie ABCD paralelogram. O dreapta variabila ce trece prin A taie
dreptele BC si CD n F , respectiv G si taie paralela prin C la BD n E. Sa
se arate ca AE 2 AF AG.
C
at
alin Calistru, Iasi
139

A
D
Solutie. Fie {O} = AC BD, {P } = AE BD;
cum [OP ] este linie mijlocie n 4ACE, vom avea c
a
O
AE = 2 AP . Din asem
an
arile evidente 4AP D
P
4F P B, 4ABF 4GCF , 4GCF 4GDA,
1
BF
1
CF
obtinem respectiv
=
,
=
,
F
B
C
AP
P F BC AF
GF BF
E
1
CF
=
. De aici,
G
AG
GF BC

1
CF
1
1
CF BF + BC
1 P F BC
1
+
=
+
=

=
AF
AG
GF BF
BC
GF BF BC AP
BF
1 CF BF + BC P F
=

.
AP F G
BF
BF
PF
BC + BF
AF
FB
=

=
, deci
Din 4DP A 4BP F obtinem
BC + F B
AP + P F
BF
PF
1
1 CF AF P F
1
2
1
+
=

=
=
AF
AG
AP F G P F BF
AP
AE
si de aici concluzia rezult
a imediat, aplicnd inegalitatea mediilor.

Clasa a VIII-a
VIII.61. Rezolvati n numere naturale ecuatia x! = y 2z + 1.
Denisa Floric
a si Lucian Tutescu, Craiova
Solutie. Pentru x {0, 1}, obtinem y = 0, z N. Pentru x = 2, g
asim y = 1,
z N sau y N, z = 0. Pentru x 3 nu mai avem solutii, deoarece x! se divide cu
3, pe cnd y 2z + 1 nu (dac
a y = 3k, atunci y 2z + 1 = M 3 + 1, iar dac
a y = 3k 1,
2
atunci y = M 3 + 1, deci y 2z + 1 = M 3 + 2).

VIII.62. If a, b, c > 0 and a + b + c 3, prove that


2 (a + b + c)2 3 (ab + bc + ca) + 3 (a + b + c) .

Babis Stergiou, Chalkida, Greece


Solutie. Inegalitatea de demonstrat revine la

2 a2 + b2 + c2 + (ab + bc + ca) 3 (a + b + c) .
(1)

Conditia din ipotez


a si inegalitatea Cauchy - Schwarz arat
a c
a

2
a + b + c a + b + c 3.
3 (a + b + c)

Atunci:

2 a2 + b2 + c2 + (ab + bc + ca) a2 + b2 + c2 + 2 (ab + bc + ca) =


= (a + b + c)2 = (a + b + c) (a + b + c) 3 (a + b + c) ,

deci (1) este adev


arat
a. Egalitatea se atinge pentru a = b = c = 1.
VIII.63. Let a, b be distinct nonzero real numbers. Find all solutions (x, y) R2
4ab
of the equation (x 1)2 + y 2 + 2
(x 1) y = 0.
a + b2
Jos Luis Daz-Barrero, Barcelona, Spain
140

Solutie. Ecuatia devine succesiv:


2

a + b2
x 12 + y 2 + 4ab (x 1) y = 0

2 2
a x + b2 y 2 + a2 + 2abxy 2a2 x 2aby +

+ b2 x2 + a2 y 2 + b2 + 2abxy 2b2 x 2aby = 0


(ax + by a)2 + (bx + ay b)2 = 0.

Evident atunci c
a ax + by a = bx + ay b = 0, de unde obtinem x = 1, y = 0.
VIII.64. Sa se afle numarul de patrate perfecte p astfel nct 2n p < 2n+1 .
Marian Pantiruc, Iasi
Solutie. Dac
a n = 2k, k N, atunci primul p
atrat perfect va fi chiar 2n .
Dac
a not
am cu m num
arul de p
atrate perfecte cutat, atunci acesta verific
a dubla

2
inegalitate 2k + m 1 < 22k+1 < 2k + m , de unde m 1 < 2k 2 2k < m,

a ntre
ceea ce ne arat
a c
a m 1este
partea ntreag
a a num
arului 2k 2 2k , adic

22k si 22k+1 se g
asesc 2k 2 2k + 1 p
atrate perfecte. Analog trat
am cazul n =

2k + 1, k N, obtinnd m = 2k+1 2k 2 .
VIII.65. Fie M un punct interior tetraedrului ABCD, iar dA , dB , dC , dD
distantele de la M la planele (BCD), (ACD), (ABD), respectiv (ABC). Sa se
MA MB MC
MD
arate ca
+
+
+
12.
dA
dB
dC
dD
D.M. B
atinetu-Giurgiu, Bucuresti
Solutie. Fie MA , HA proiectiile punctelor M , respectiv
A
A, pe planul (BCD); evident c
a dA = M MA , ha = AHA
si analoagele. Mai not
am sA = SBCD , sB = SACD etc.
Observ
am c
a:
M
AM + M MA AMA AHA
sA M A + sA dA sA hA = 3VABCD
B
HA D
sA M A + sA dA sA dA + sB dB + sC dC + sD dD
M
A

sB dB
sC dC
sD dD
MA
>

.
C
dA
sA dA
sA dA
sA dA
Grupnd convenabil termenii, obtinem:

X M A X sA dA
sB dB
6 2 = 12,

dA
sB dB
sA dA
cu egalitate cnd tetraedrul este regulat, iar M este centrul s
au de greutate.

Clasa a IX-a
IX.61. The radii of the three escribed circles of a triangle ABC are ra = 4,
rb = 6, rc = 12. Find the lenghts of the sides of the triangle.
Jos Luis Daz-Barrero, Barcelona, Spain
Solutie. Folosim succesiv binecunoscutele relatii
1
1
1
1
+
+ ; S 2 = rra rb rc ; S = ra (p a) = rb (p b) = rc (p c) .
=
r
ra
rb rc
Obtinem r = 2, apoi S = 24, prin urmare a = 6, b = 8, c = 10.
141

IX.62. Rezolvati sistemul n necunoscutele x1 , x2 , . . . , xn R , n N :


2 (n 1)

n
Y

k=1

n
X

X
n1
xk = 1 + x21
x2k = 1 + x2n
x2k .
k=2

k=1

Silviu Boga, Suceava


Solutie. Considernd o solutie (x1 , x2 , . . . , xn ), din prima egalitate obtinem
n
n
!
!
n
X
Y
X
2
2
xk x1 2 (n 1)
xk x1 +
x2k = 0.
(1)
k=2

k=2

k=2

2 n

n
Q
P 2
Discriminantul este 4 = (n 1)
atii medixk
xk si, conform inegalit
k=2

k=2

ilor, 4 0. ns
a x1 R prin urmare trebuie s
a avem 4 0, fapt care se realizeaz
a
2
2
cnd x2 = x3 = = x2n . Analog obtinem c
a x21 = x22 = = x2n1si atunci
(n 1) tn1
asim c
a x1 =
=
x21 = x22 = = x2n = t R+ . nlocuim n (1) si g
(n 1) t2
tn3 , de unde t = 1. n concluzie, solutiile sistemului au forma (x1 , x2 , . . . , xn ) cu
|xk | = 1, k 1, n.
IX.63. Determinati x, y Z pentru care x3 + y 3 x + y x5 + y 5 .
Romeo Ilie, Brasov
Solutie. Not
am x + y = s, xy = p. Dac
a s > 0, avem:

(x + y) x2 xy + y 2 x + y (x + y) x4 x3 y + x2 y 2 xy 3 + y 4

(x + y)2 3xy 1 x2 + y 2 x2 y 2 xy x2 + y 2

s2 3p 1 s2 2p p s2 2p p2 .

Atunci s2 2p 1 + p, deci

2
1 s2 2p p s2 2p p2 (1 + p) p (1 + p) p2 =

2
1
5
5
= p2 + p + 1 = p
+ .
2
4
4
5
Deoarece 1 p2 +p+1 si p2 +p+1 Z, atunci p2 +p+1 = 1, deci p {0, 1}.
4
Ne amintim c
a s > 0 si s2 3p 1 si obtinem solutiile (x, y) {(0, 1) ; (1, 0) , (1, 1)}.
Analog, dac
a s < 0 g
asim solutiile (x, y) {(0, 1) ; (1, 0) , (1, 1)}. n sfrsit,
dac
a s = 0, evident c
a sirul de inegalit
ati din ipotez
a este adev
arat cu egal, deci orice
pereche (n, n), n Z constituie solutie a ecuatiei.

IX.64. Fie ABCD un patrulater, iar HA , HB , HC , HD ortocentrele triunghi

urilor BCD, ACD, ABD, respectiv ABC. Demonstrati ca AC = HC HA daca si



numai daca BD = HD HB .
Ioan S
ac
aleanu, Hrl
au
Solutie. Not
am cu OA , OB , OC , OD centrele circumscrise triunghiurilor BCD,
ACD, ABD, respectiv ABC. Tinnd

cont de relatia Sylvester, se obtine succesiv:



OA HA + HC OC = OA B + OA C + OA D + AOC + BOC + DOC
142


OA C + CHA + HC A + AOC = OA C + AOC + OA B + BOC + OA D + DOC


CHA + HC A = 2OA OC 2OC OA = AHC + HA C.

Analog se deduce c
a 2OB OD = DHB + HD B. Atunci


AC = HC HA AHC = CHA AHC + HA C = 0 OC = OA



ABCD patrulater inscriptibil OB = OD DHB + HD B = 0 BD = HD HB .


IX.65. Fie a, b R astfel nct sin x + sin a cos x cos b, x R. Sa se arate
ab
ca
Z.

Adrian Zanoschi, Iasi


3
relatia dat
a devine 1 + sin a 0
Solutie. Dac
a sin a < 1, pentru x =
2

sin a < 1, contradictie. R


amne c
a sin a = 1, adic
a a = + 2k, k Z. Inegalitatea
2
din enunt devine:
sin x + 1 cos x cos b sin x cos x cos b 1
cos b
1
1

sin x
cos x

2
2
1 + cos b
1 + cos b
1 + cos2 b
1
1
sin x cos sin cos x
sin (x )
,
2
1 + cos b
1 + cos2 b
1
pentru orice x R (am notat [0, 2) num
arul pentru care cos =
,
1 + cos2 b
cos b
3
sin =
). Lund x = +
n aceast
a inegalitate, obtinem 1
2
2
1 + cos b

ab
1

cos2 b 0 cos b = 0, deci b =


+ l, l Z. Astfel,
=
2
2

1 + cos b
2k l Z.

Clasa a X-a

x
X.61. If x 0, prove that log3 (1 + 3x ) > log4 4x + 3 2
.
Oleg Faynshteyn, Leipzig, Germany
Solutie. Ineglitatea dat
a se scrie echivalent

!x !

x
3
2
1
x
x
log3 3 1 +

> log4 4 1 +
3
4

!x !

x
3
1
2
log3 1 +
.
> log4 1 +
3
4

3
2
1
Aceast
a ultim
a inegalitate rezult
a din observatia c
a
>
, iar log3 a = log3 4
3
4
a ce a > b.
log4 a > log4 a > log4 b, de ndat
X.62. Se da sirul (zn )nN C cu proprietatea ca zn+2 = zn + izn+1 , n N,
si se noteaza cu V multimea termenilor
sai. Daca U3 V , sa se arate ca V = U12 .

(Am notat Uk = z C | z k = 1 .)
Monica Nedelcu, Iasi
143

Solutie. Observ
am c
a zn+3 = zn+1 + izn+2 = zn+1 + i (zn + izn+1 ) = izn , prin
urmare zn+12 = izn+9 = i2 zn+6 = i3 zn+3 = i4 zn = zn , deci sirul este periodic de
perioad
a 12. n plus, iz V pentru orice z V .
Pentru a ar
ata c
a V = U12 , este suficient s
a demonstr
am c
a U12 V . Fie
U12 = {zk | zk = cos

2k
2k
+ i sin
, k = 0, 11}.
12
12

Evident c
a {z0 , z4, z8 } = U3 V , apoi z3 = iz0 V , z6 = iz3 V , z9 = iz6 V ,
z7 = iz4 V , z10 = iz7 V , z1 = iz10 V , z11 = iz8 V , z2 = iz11 V ,
z5 = iz2 V si demonstratia este ncheiat
a.
X.63. Un numar de n jucatori arunca succesiv o moneda avnd fetele s si b;
jocul este cstigat de acela care obtine primul fata s. Sa se afle probabilitatea ca
jucatorul de pe locul k (1 k n) sa cstige jocul n primele np + k aruncari ale
monedei, unde p N este dat.
Petru Asaftei, Iasi
Solutie. Problema extinde, folosind aceeasi idee, cazul n = 2 prezent n unele
manuale. Not
am cu Ai evenimentul ca juc
atorul de pe locul k s
a cstige jocul n
exact ni + k arunc
ari ale monedei; pentru realizarea lui Ai , fetele monedei trebuie s
a
ni+k
1
. Evenimentele A1 , A2 , . . . , Ap
apar
a n ordinea bb
. . . }b s, deci P (Ai ) =
| {z
2
ni+k1ori

sunt incompatibile n totalitatea lor, iar A1 A2 Ap reprezint


a evenimentul a
c
arui probabilitate este cerut
a de problem
a. Avem:
P (A) = P (A1 ) + P (A2 ) + + P (Ap ) =

1
1
1
+ k+m + + k+np =
k
2
2
2

1
1
1 2np
1 2np 1
= n(p1)+k
.
= k
1
2
1 2n
2

1
2n
2005 |x|
3
2005
X.64. Fie f : R (0, ), f (x) =
+ 3 . Sa se
sin
+ 2005 |x|4
2004

x
|x|

1
9
1

, x R .
arate ca f (x) + f (x)
2
2
1 + |x|3
Ioan S
erdean, Or
astie
Solutie. Construim functia g : R (0, ),

1 + |x|3
1
1

g (x) =
f (x) + 2 f (x) 2 , x R ;
9

a
este suficient s
a ar
at
am c
a g (x) 1, x R . Vom demonstra acest lucru n dou
etape:
i) Dac
a |x| > 2, atunci
0 < f (x)

2005 |x|

2005 |x|

144

3
3

|x|

4
|x|

<

4
1
= ,
8
2


1 1
= f (x). Rezult
a c
a
2 2
!

3
3
8
8 1
1 + |x|
4
1
1 + |x|
+
1

2f (x)
2 3 =
+
1
= 1.
g (x) +
9
9
9 |x|3
9 8
|x|

deci f (x) +

1
1
f (x)
=
f
(x)
+
,
iar

2
2

ii) Dac
a 0 < |x| 2, folosind faptul c
a |a| |b| |a b|, avem:

3
1 + |x|
1
1 1 + |x|3
1+8
g (x)
f (x) + f (x) + =

= 1.
9
2
2
9
9

X.65. Fie C un cerc de raza 1 si fie P1 , P2 , . . . , Pn puncte ale discului corespunzator. Aratati ca exista un semicerc nchis S C astfel nct M P1 +M P2 + +M Pn
n, M S.
Adrian Zahariuc, elev, Bac
au
Solutie. Fie O centrul cercului, iar G centrul de greutate al sistemului de puncte
P1 , P2 , . . . , Pn . Vom demonstra c
a M P1 + M P2 + + M Pn nM G, pentru orice
punct M din plan. Raport
am planul complex la un reper cu originea n M ; fie g
afixul lui G, pi afixele punctelor Pi , i = 1, n. Atunci
p1 + p2 + + pn
g=
n |g| = |p1 + p2 + + pn | |p1 | + |p2 | + + |pn |
n
n M G M P1 + M P2 + + M Pn .

R
amne de demonstrat c
a exist
a un semicerc S C astfel nct M G 1, M S.
Dac
a G = O, inegalitatea este adev
arat
a pentru orice M C. Dac
a G 6= O, fie [AB]
diametrul lui C perpendicular pe OG, iar S semicercul delimitat de acesta, "opus"
\
lui G. Atunci m(M
OG) 90 , M S, deci M G M O = 1, ceea ce ncheie
rezolvarea.

Clasa a XI-a
XI.61. Fie a R\{1} si A Mn (C). Daca exista k N astfel nct
Ak+1 = aAk , sa se arate ca In A este inversabila.
Gheorghe Iurea, Iasi
Solutie. Fie X0 Mn,1 (C) o solutie a sistemului (In A) X = O. Atunci
AX0 = X0 , deci Ak X0 = Ak+1 X0 = X0 , de unde X0 = aX0 si cum a 6= 1 rezult
a
c
a X0 = O. Astfel, sistemul omogen (In A) X = O are doar solutia banal
a si
atunci det (In A) 6= 0.
C
D
XI.62. Fie ABCD un patrat de centru O, iar M un
punct variabil pe [AB]. Notam {S} = CM AD, {E} =
SO M D. Se cere locul geometric al punctului E.
O
Petru R
aducanu, Iasi
Solutia 1 (analitic
a). Raport
am planul la un reper
cu originea n A, avnd dreptele AB si AD axe de coordoE
nate. Putem considera A (0, 0), B (2, 0), C (2, 2), D (0, 2),
O (1, 1), iar M (a, 0), cu a (0, 2). Intersectnd
dreapta A L
M
B

2a
CM cu AD, obtinem c
a S 0,
. Atunci ecuatia
a2
145

a+2
(x 1) + y 1 = 0 si cum DM : 2x + a (y 2) = 0, prin
a2
eliminarea parametrului a ntre cele dou
a ecuatii g
asim ecuatia locului geometric:
a ecuatie reprezint
a cercul de diametru [AD], din care
x2 + y 2 2y = 0. Aceast
trebuie retinut doar semicercul interior p
atratului (f
ar
a capete), deoarece a (0, 2).
Solutia 2 (sintetic
a). n 4AM D cu transversala ELS, conform teoremei lui
EM LA SD
Menelaus obtinem c
a

= 1. Apoi, n 4AM C cu transversala OLS


ED LM SM
SC
SD
CD
EM
AM 2
LA OC SM
. De

= 1. Cum
=
=
, atunci
=
obtinem
LM OA SM
SM
SA
AM
ED
AD2
aici rezult
a c
a AE M D, adic
a AEOD este patrulater inscriptibil, deci E apartine
cercului de diametru [AD].
atratului apartine
R
amne s
a ar
at
am c
a orice punct E 0 de pe semicercul interior p
0
locului. Fie {M } = DE 0 AB, {S 0 } = CM 0 DA, {T } = AC M 0 D. Atunci:
dreptei SO este

OA
OD
\
\
=
= tg(OT
D) = tg(AT
E 0 );
OT
OT
\
ctg(AT
E0)
AM 0
E0T E0D
E0T
\
:
=
=
=
ctg(AT
E 0 ).
E0D
AE 0 AE 0
AD
\0 )
ctg(ADM

AD
OA E 0 T S 0 D
S0D
CD
=
,
rezult
a
c
a
=

= 1. Din reciproca teoremei


S 0A
AM 0
AM 0
OT E 0 D 0 S 0 A
0
lui Menelaus n 4AT D urmeaz
a c
a punctele O, E , S sunt colinare, ceea ce ncheie
demonstratia.
Cum

XI.63. Fie f, g : [a, b] R functii continue astfel nct g(a)+f (b) < 1+g(a) f (b).
Sa se arate ca exista c (a, b) pentru care

3
3
3
g (c)
2c a b
3 (2c a b) f (c) g (c)
f (c)
+
+
=
.
bc
ac
(b c) (a c)
(b c)2 (a c)2

Valeriu Brasoveanu, Brlad


Solutie. Florin Popovici, Brasov, d
a urm
atoarea generalizare:

Fie f, g, h : [a, b] R trei functii continue pe [a, b], astfel nct (g (a) 1)
(f (b) 1) > 0 si h (a) = h (b) = 0. Daca n N este impar, exista c (a, b) astfel
nct f n (c) (a c)n + g n (c) (b c)n + (2c a b)n = h (c).

am
ntr-adev
ar, din ipotez
a rezult
a c
a (g n (a) 1) (f n (a) 1) > 0. Consider
n
functia u : [a, b] R, u (x) = f (x) (a x)n +g n (x) (b x)n +(2x a b)n h (x).
Evident c
a u este continu
a pe [a, b], iar
u (a) = (g n (a) 1) (b a)n ;

u (b) = (1 f n (b)) (b a)n

u (a) u (b) = (g n (a) 1) (1 f n (b)) (b a)

2n

< 0.

Urmeaz
a c
a exist
a c (a, b) astfel nct u (c) = 0, de unde concluzia.
Problema se obtine pentru n = 3 si h (x) = 3 (2x a b) (a b) (b x) f (x) g (x).
f (x)
XI.64. Fie f : (0, 1) (0, ) o functie crescatoare cu lim
= 1 si f (x) x,
x0 x
1
x (0, 1). Definim sirurile (an )n1 , (xn )n1 prin an =
(a + b) (2a + b) (na + b)
146

1/n

(a > 0, b > 0) si xn = [f (an+1 ) + f (an+2 ) + + f (a2n )] . Sa se calculeze


lim xn .
n
Gheorghe Costovici, Iasi
Solutie. S
a observ
am nti c
a problema este consistent
a, n sensul c
a exist
a
functii f ca n enunt; de exemplu f1 (x) = x, f2 (x) = sin x, f3 (x) = arctg x, f4 (x) =
ln (1 + x) etc.
Not
am bn = f (an+1 ) + f (an+2 ) + + f (a2n ) si observ
am c
a bn 0:
1
0 < bn nf (an ) n
0.
(a + b) (2a + b) (na + b)
ln bn

am de
Avem c
a xn = eln xn = e n . Pentru a calcula limita exponentului, ne ocup

raportul (Stolz pentru


)

f (a2n+1 ) + f (a2n+2 ) f (an+1 )


bn+1
ln bn+1 ln bn
.
= ln 1 +

= ln
(n + 1) n
bn
bn
bn
Dar

a2n+1 + a2n+2
f (a2n+1 ) + f (a2n+2 )
1
a2n
a2n+1 a2n+2

+
0
=

bn
nf (a2n )
n f (a2n )
a2n
a2n
f (x)
si tinnd seama de faptul c
a lim f (x) = lim
x = 0,
x0
x0 x
bn
bn+1 bn
f (a2n+1 ) + f (a2n+2 ) f (an+1 )
lim
= lim
= lim
=
n f (an+1 )
n f (an+2 ) f (an+1 )
n
f (a2n+2 ) f (an+1 )

f (a2n+2 ) a2n+2 f (an+1 )


f (a2n+1 ) a2n+1

:
lim
n
a2n+1
an+1
a2n+2
an+1
an+1

f (an+2 ) an+2 f (an+1 )


:

=1
an+2
an+1
an+1
0
ln bn
(am folosit Stolz pentru ). Rezult
a lim
= si, prin urmare, lim xn = 0.
n n
n
0

XI.65. Pentru x1 , x2 , . . . , xn R+ , x1 + x2 + + xn = nA, aratati ca



x
x
x
x2 3
xn 3
x1 3
1
2
n
+3
+3
+3
+2
+ 2 ...
+ 2 6n .
A
A
A
A
A
A
Sa se obtina de aici inegalitatea dintre media aritmetica si cea geometrica.
Dumitru Mihalache, Brlad
x
x2
x3
Solutia 1 (a autorului). Este cunoscut
a inegalitatea ex 1 + +
+ ,
1!
2!
3!
x R, egalitate pentru x = 0. (Inegalitatea se poate obtine ignornd restul n
dezvoltarea Taylor a lui ex , sau direct prin deriv
ari repetate). Trecem pe x n x 1
si g
asim c
a

x 1 (x 1)2
x3 + 3x + 2
(x 1)3
+
+
ex1
, x R,
1
2
6
6
x1 x2
xn
cu egalitate pentru x = 1. D
am lui x valorile
, ,...
si nmultim membru cu
A A
A
x1 + x2 + xn
membru inegalit
atile obtinute; tinnd seama de ipoteza
n = 0,
A
ex1 1 +

147

x1
x2
xn
obtinem concluzia. Egalitatea se atinge pentru
=
= =
= 1 x1 =
A
A
A
x2 = = xn .
Vom demonstra c
a x3 + 3n + 2 6x, x R+ ; ntr-adev
ar, acest fapt revine la

2
n
(x 1) (x + 2) 0, evident adev
arat. Cu notatia G = x1 x2 . . . xn , obtinem:


x
x

3
x1
x1
x2 3
xn 3
2
n
6n
+3
+3
+3
+2
+ 2
+2
A
A
A
A
A
A
x x x
n
n
G
G
1
2
n
6
6
6
= 6n n n 1 G A,
A
A
A
A
A
adic
a tocmai inegalitatea mediilor.
Solutia 2 (Marian Tetiva, Brlad). Fie x1 = a1 A, x2 = a2 A, . . . , xn = an A, cu
x1 + x2 + + xn
a1 + a2 + + an =
= n; inegalitatea de demonstrat devine
3
3
A 3

a1 + 3a1 + 2 a2 + 3a2 + 2 . . . an + 3an + 2 6n P (a1 )P (a2 ) P (an ) 6n ,

unde P (x) = x3 + 3x + 2. ns
a
P (a1 ) P (a2 ) P (an ) 6n ln P (a1 ) + ln P (a2 ) + + ln P (an ) n ln 6

ln P (a1 ) + ln P (a2 ) + + ln P (an )


a1 + a2 + + an

ln P
,
n
n

a1 + a2 + + an
deoarece P
= P (1) = 6. Ultima inegalitate ar fi adev
arat
a
n
dac
a functia ln P (x) ar fi concav
a. Cum


2
2
00
0
P (x) P (x) [P (x)] = 6x x3 + 3x + 2 3x2 + 3 =

= 3 x4 4x + 3 = 3 (x 1)2 x2 + 2x + 3 0,
demonstratia este ncheiat
a.
Not
a. Cum domeniul de definitie al functiei ln P (x) este de forma (x0 , ), cu
x0 (1, 0), putem spune c
a inegalitatea din enunt are loc pentru x1 , x2 , . . . , xn
(x0 , +), cu x1 + x2 + + xn 6= 0.

Clasa a XII-a
XII.61. Sa se determine functiile derivabile f : (0, ) (0, ) care admit
o primitiva F : (0, ) (0, ) pentru care lim F (x) = 2 lim f (x) = 1 si
x&0

F (x) f 0 (x) = F (x) f (x) f 2 (x) + x 1, x (0, ).

x&0

Mihai Haivas, Iasi


1
Solutie. (dat
a de Florin Popovici, Brasov). Cu notatia u = F 2 , relatia
2
din enunt se scrie sub forma u00 (x) u0 (x) = x 1, x (0, ), care este o
ecuatie diferential
a liniar
a de ordin 2, cu coeficienti constanti, neomogen
a. Solutia
x2
x2
1 2
x
sa general
a este u (x) = c1 + c2 e , x (0, ), adic
a F (x) = c1 + c2 ex ,
2
2
2
1
a c1 + c2 = . Apoi, derivnd n
x (0, ). Din conditia lim F (x) = 1 obtinem c
x&0
2
1
ambii membri si folosind din nou conditia initial
a, g
asim c
a c2 = , deci c1 = 0. n
2
concluzie, F 2 (x) = ex x2 , x (0, ).
148

Pentru x (0, 1], avem ex x2 ex 1 > 0. Pentru x (1, ), avem


x

00
0
e x2 = ex 2 > e 2 > 0. Apoi, ex x2 > 0, x (1, ) si cum e 1 > 0,

deducem c
a ex x2 > 0, x (1, ). Astfel, F (x) = ex x2 , x (0, ), prin
ex x2
urmare f (x) = F 0 (x) =
, x (0, ) este unica solutie a problemei.
2 ex x2
XII.62. Fie f : R R+ o functie continu
se anuleaza
a, iar F primitiva
sa care
n
P
1
1
F .
; b) lim
n origine. Sa se calculeze: a) lim xF
x
n k=1
x
k
Dan Popescu, Suceava
Solutie. a) Avem:

1
F (y)
F (y) F (0)
lim xF
= lim
= lim
= F 0 (0) = f (0) .
x
y0
y0
x
y
y0
y>0

y>0


1
= f (0), pentru orice 0 < a < f (0), exist
a n1 N
b) Deoarece lim nF
n
n

1
1
a
pentru care n F
> a, n > n1 , altfel spus F
> , n > n1 .
n
n
n
Atunci
X

n1
n1
n1
n
n
n
X
X
X
X
1
1
1
1
1
1
a
>
=
+
>
F
F
F
F +a
k
k
k
k
k
k
k=1
k=1
k=n1 +1
k=1
k=1
k=1

n
n
n
1
1
X
X1
X 1
1
, n > n1
>
+a
F
a
k
k
k
k=1
k=1
k=1

n
n 1
P
P
1
= +.
si cum lim a
F
= +, rezult
a c
a lim
n k=1 k
n k=1
k
XII.63. Fie f, g : R R doua functii surjective. Daca g (g f ) este descrescatoare si exista L > 1 astfel nct |g (x) g (y)| L |x y|, x, y R, sa se arate ca
f are un unic punct fix.
Sorin Puspan
a, Craiova
Solutie. Din |g (x) g (y)| L |x y|, x, y R, rezult
a c
a g este injectiv
a
si, cum g surjectiv
a, rezult
a g bijectiv
a; atunci g 1 este o contractie de constant
a
1
< 1, deci continu
a. Rezult
a c
a g este continu
a si cum este si injectiv
a, atunci g
L
este strict monoton
a. Folosind acum faptul c
a g (g f ) este descresc
atoare, rezult
a
c
a g f este monoton
a, iar g si g f au monotonii diferite. Distingem situatiile:
i) Fie g descresc
atoare, iar g f cresc
atoare; dac
a x, y R, x < y, atunci
(g f ) (x) (g f ) (y) g (x) g (y) f (x) f (y)
|f (x) f (y)| |g (x) g (y)| |f (x) f (y)| L |x y| , x, y R.
ii) Fie g cresc
atoare, g f descresc
atoare; dac
a x, y R, x < y, atunci
(g f ) (x) (g f ) (y) f (y) f (x) g (y) g (x)
|f (x) f (y)| L |x y| , x, y R.
149

n ambele cazuri am obtinut c


a |f (x) f (y)| L |x y| ,

x, y R. Cum f
1
a
surjectiv
a, ca si n cazul lui g, obtinem c
a f 1 este contractie de constant
< 1,
L
deci are un unic punct fix, acesta fiind unicul punct fix si pentru f .

XII.64. Fie M multimea functiilor f : R R derivabile de doua ori si cu


proprietatea ca exista R astfel nct f 00 = f + 1. Aratati ca exista o familie de
grupuri {(G , ); R}
ce verifica simultan:
1) (G , ) ' R2 , + , R;
2) {G ; R} este o partitie a multimii M .
Temistocle Brsan, Iasi
Solutie. Pentru R fixat not
am cu G multimea solutiilor ecuatiei diferent
iale
2
f 00 = f + 1. Ecuatia caracteristic
a
asociat
a
este

=
,
cu
r
a
d
a
cinile:

,
1,2

dac
a > 0, 1,2 = i , dac
a < 0 si 12 = 0 (dubl
a), dac
a = 0. Mai
1
observ
am c
a o solutie particular
a a acestei ecuatii este f (x) = , dac
a 6= 0, si

2
x
, dac
a = 0. Ca urmare, avem:
f0 (x) =
2
n
o

a > 0;
G = ae x + be x 1/; a, b R , dac

a < 0;
G = a cos x + b sin x 1/; a, b R , dac

2
x
G0 = ax + b + ; a, b R , dac
a = 0.
2
S

a
{G ; R}. Multimile G sunt nevide si G1 G2 = , dac
n
P
1 6= 2 (se aplic
a rezultatul: dac
a
Pk (t) ek t = 0, t R, unde 1 , 2 , . . . , n
Evident, M =

k=1

C sunt distincte, atunci polinoamele Pk (t) sunt identic nule pe R, i = 1, n), ceea ce
ncheie demonstratia punctului 2).
Pe multimea G , cu 6= 0, definim operatia prin (f g) (x) = f (x) +
1
x2
g (x) + , x R, iar pe G0 consider
am 0 dat
a de (f 0 g) (x) = f (x) + g (x) ,

2
a cu usurinta
x R. Stabilitatea operatiilor , R, si axiomele de grup se verific
(elementele neutre sunt functiilef si f0 definite
mai sus).

n sfrsit, dac
a fi (x) = aie x + bi e x 1/, i = 1, 2, avem (f
1 f2 ) (x) =
(a1 + a2 ) e x + (b1 + b2 ) e x 1/ si deducem c
a (G , ) ' R2 , + pentru
> 0. La fel se procedeaz
a n cazurile < 0 si = 0.
XII.65. Fie p1 < p2 < < pn numere prime, iar P multimea
polinoamelor de
1
grad n cu termenul liber 1 si ceilalti termeni distincti n multimea
| i = 1, 2, . . . , n .
pi
Sa se arate ca pentru orice x N \ {1} relativ prim cu toate numerele pi exista
37xn + 42
q, r P astfel nct 0 |q (x) r (x)| <
. Daca, n plus, x pn 1,
42n!
inegalitatea de mai sus este stricta.
Marius Pachitariu, elev, Iasi
Solutie. Avem:
150

1 n
1
1
1
1
x + xn1 + + 1 xn
+
+ + n1
+1
kP
p1
p2
p1 2p2
2
pn

1
1
1
1
1
xn
+
+ 2
+
+
+ ...
+1<
2 23 2 5
78 89

1 1
1
1
37 n
< xn
+ +
+
+1<
x + 1.
2 6 20 7
42
(Am utilizat inegalitatea evident
a pn 2n1 n (n 1), precum si faptul c
a
1
1
1
1
1
1
+
+ +
=
< ).
78 89
m (m + 1)
7 m+1
7
Cum multimea P contine n! polinoame, putem g
asi dou
a q, r distincte pentru care
s
a aib
a loc inegalitatea din enunt.
Pentru partea a doua, fie q 6= r si x N \ {1} relativ prim cu toate numerele pi
si astfel nct x pn 1. S
a presupunem prin absurd c
a q (x) = r (x), deci
1 n1
1 n
1 n1
1 n
x +
x
+ + 1 =
x +
x
+ + 1,
pi1
pi2
pj1
pj2
unde (i1 , i2 , .
. . , in ) si (j1
, j2 , . . . , j
ale mul
n ) sunt permut
ari distincte

timii {1, 2, . . . , n}.


1
1
1
1
1
1
Atunci xn1
+xn2
+ +
= 0 si, nmultind

pi1
pj1
pi2
pj2
pin pjn
p1 p2 pn (pjn pin )
aceast
a relatie cu p1 p2 pn , obtinem c
a x divide
, deci n mod
pin pjn
a c
a pik = pjk , k = 1, n, deci q = r si
necesar pin = pjn . Analog se demonstreaz
astfel ajungem la o contradictie.
max k (x) =

IMPORTANT
n scopul unei leg
aturi rapide cu redactia revistei, pot fi utilizate urm
atoarele
adrese e-mail: tbirsan@math.tuiasi.ro, profgpopa@yahoo.co.uk . Pe
aceast
a cale colaboratorii pot purta cu redactia un dialog privitor la materialele trimise acesteia, procurarea numerelor revistei etc. Suger
am colaboratorilor care trimit probleme originale pentru publicare s
a le numeroteze si
s
a-si retin
a o copie xerox a lor pentru a putea purta cu usurinta o discutie
prin e-mail asupra accept
arii/neaccept
arii acestora de c
atre redactia revistei.
La problemele de tip L se primesc solutii de la orice iubitor de matematici
elementare (indiferent de preocupare profesionala sau vrsta ). Fiecare dintre
solutiile acestor probleme - ce sunt publicate n revist
a dup
a un an - va fi
urmat
a de numele tuturor celor care au rezolvat-o.
Adres
am cu insistenta
amintea ca materialele trimise revistei
rug
s
a nu fie (s
a nu fi fost) trimise si altor publicatii.
Rug
am ca materialele tehnoredactate s
a fie trimise pe adresa redactiei nsotite de fisierele lor (de preferinta n LATEX).
151

Solutiile problemelor pentru preg


atirea concursurilor
din nr. 2 / 2005
A. Nivel gimnazial
G86. Fie n 3 un numar natural impar, iar A N o multime cu n2 2n + 2
elemente. Sa se arate ca putem alege n numere din multimea A cu proprietatea ca
suma lor se divide cu n.
Titu Zvonaru, Com
anesti
n1
S
Solutie. Scriem A =
Ai , unde Ai contine toate elementele lui A care dau
i=0

restul i la mp
artirea prin n. Dac
a cel putin o multime Ai este vid
a, cum n2 2n+2 =
(n 1) (n 1) + 1, din principiul cutiei rezult
a c
a exist
a m
acar o multime Aj care
contine cel putin n elemente, iar suma acestora se divide evident cu n. Dac
a toate
multimile Ai , i = 0, n 1, sunt nevide, alegem cte un element din fiecare multime.
n (n 1)
Suma acestora, modulo n, este 0 + 1 + + (n 1)
(mod n) si, cum n
2
n (n 1)
0 (mod n).
este impar, n 1 este par si rezult
a c
a
2
G87. Aratati ca exista o infinitate de numere
naturale n pentru care putem gasi
P
ai {1, 1}, i = 1, n astfel nct 2005 =
ai aj . Aflati valoarea minima a lui n.
1i<jn

Gheorghe Iurea, Iasi


Solutie. Putem scrie:
X

4010 = 2
ai aj = (a1 + a2 + + an )2 a21 + a22 + + a2n
1i<jn

si cum 4010 = 642 86, putem alege a1 + a2 + + an = 64, a21 + a22 + + a2n = 86.
Aceste dou
a egalit
ati sunt realizate pentru n = 86, a1 = a2 = = a75 = 1,
a76 = a77 = = a86 = 1.
Binenteles, exist
a o infinitate de posibilit
ati de alegere ale lui n. De exemplu,
scriind 4010 = 652 109, ar trebui ca a1 + a2 + + an = 65, a21 + a22 + + a2n = 109,
deci putem considera n = 109, a1 = a2 = = a88 = 1, a89 = a90 = = a109 = 1.
Se poate continua!
Demonstr
am c
a valoarea minim
a a lui n este 86. Fie p numere egale cu 1 si q
numere egale cu 1, p + q = n. Atunci

4010 = (a1 + a2 + + an )2 a21 + a22 + + a2n = (p q)2 p q

2
(2p n) n = 4010 4p2 4np + n2 n 4010 = 0.

Cum p N, trebuie ca discriminantul ecuatiei de grad II n p de mai sus s


a fie p
atrat
perfect. Avem c
a = 16 (n + 4010) si cel mai mic p
atrat perfect de aceast
a form
a
se obtine pentru n = 86, cnd = 16 4096 = 2562 .
G88. Spunem ca o multime M R+ are proprietatea (P ) daca orice element
din M este media geometrica a doua elemente distincte ale lui M .
a) Sa se arate ca exista o infinitate de multimi cu proprietatea (P ).
b) Gasiti multimile cu 2005 elemente avnd proprietatea (P ).
Gabriel Popa si Paul Georgescu, Iasi
152

Solutie. a) Pentru orice x R+ \ {1}, multimea M = {xn | n Z} are propri

etatea (P ), deoarece xn = 1 x2n , iar 1 = xn0 xn0 , cu n0 Z arbitrar.


b) Fie M o multime finit
a cu proprietatea (P ), iar a M cel mai mare element
al acesteia; atunci a este media geometric
a a elementelor b, c M , cu b < c. Rezult
a
c
a b < a < c, deci c ar fi un element al lui M strict mai mare dect a! n concluzie,
nici o multime finit
a nu are proprietatea (P ).
G89. Pentru a, b R+ , sa se demonstreze inegalitatea

1
1
1 1 1
1
1
+
+

+ +
.
a2 + ab + b2 a2 + a + 1 b2 + b + 1
3 a b ab
Marius Pachitariu, elev, Iasi
Solutie (semnalat
a de Titu Zvonaru, Com
anesti). Inegalitatea evident
a
1
1
2
2
2

(a b) 0 conduce la a + ab + b 3ab, deci 2


. Analog
a + ab + b2
3ab
1
1
1
1
obtinem c
a 2

si prin sumare obtinem concluzia


a +a+1
3a b2 + b + 1
3b
problemei. Egalitatea se atinge pentru a = b = 1.
Din p
acate, aceast
a rezolvare evident
a nu a fost observat
a n momentul select
arii problemelor, fapt care a condus la includerea ei n sectiunea Probleme pentru
pregatirea concursurilor.
G90. Fie n 1 un numar natural. Sa se arate ca multimea A = {n+1, n+2, . . . , 2n}
se poate partitiona n doua submultimi, fiecare cu aceeasi suma a elementelor, daca
si numai daca n = 4k, k N sau n = 4k + 1, k N \ {1}.
Marian Tetiva, Brlad
Solutie. Dac
a A se poate partitiona cum cere enuntul, suma elementelor lui A:
.
n (3n + 1)
(n + 1) + (n + 2) + + 2n =
trebuie s
a fie num
ar par, deci n (3n + 1) .. 4.
2
Cum n si 3n + 1 au parit
ati diferite, unul dintre ele este multiplu de 4, deci n = 4k,
k N sau n = 4k + 1, k N . n al doilea caz, nu putem avea k = 1, altfel
A = {6, 7, 8, 9, 10} si suma elementelor fiecarei submultimi din partitie ar trebui s
a
fie 20; submultimea care-l contine pe 10 nu poate avea ns
a suma elementelor 20.
Reciproc, fie nti n = 4k, k N. Atunci A = {4k + 1, 4k + 2, . . . , 8k} poate fi mp
artit
a n 2k submultimi cu aceeasi sum
a a elementelor: {4k + 1, 8k}; {4k + 2, 8k 1};
. . . ; {6k, 6k + 1}. Reuniunea a k dintre aceste submultimi constituie una dintre
clasele partitiei, iar cele r
amase cealalt
a clas
a.
Fie acum n = 4k + 1, k 2; atunci A = {4k + 2, 4k + 3, . . . , 8k + 2}. Fie
A = B C partitia dorit
a. n B punem, pentru nceput, numerele 4k + 2, 4k + 3,
4k + 4, 4k + 6, 4k + 8, iar n C numerele 4k + 5, 4k + 7, 4k + 9, 8k + 2; pn
a acum,
elementele din B si cele din C au aceeasi sum
a 20k + 23. Numerele r
amase, de la
4k + 10 la 8k + 1, le mp
atim n k 2 grupe de cte 4 numere consecutive, de forma
{p, p + 1, p + 2, p + 3}. Numerele p si p + 3 din fiecare grup
a le punem n B, iar p + 1
si p + 2 le punem n C, realiznd astfel partitia dorit
a.
G91. Gasiti cel mai mic numar natural k pentru care, oricum am colora o tabla
de sah 8 8 cu k culori astfel nct fiecare culoare sa fie folosita cel putin o data,
exista 6 casute de culori diferite care se afla pe aceeasi linie sau coloana.
Adrian Zahariuc, elev, Bac
au
153

Solutie. R
aspunsul este 35. D
am mai nti un exemplu de colorare cu 34 de
culori care nu are proprietatea dorit
a: mp
artim tabla n 4 table 4 4, pe cea din
stnga-sus o color
am n albastru, pe cea din dreapta-jos o color
am n rosu, iar restul
c
asutelor, n num
ar de 32, vor fi colorate fiecare cu cte una dintre cele 32 de culori
r
amase. n acest fel, pe fiecare linie si pe fiecare coloan
a vom avea cte 5 culori
diferite.
Demonstr
am n continuare c
a num
arul k = 35 verific
a cerintele problemei. Presupunem prin absurd c
a ar exista o colorare cu 35 de culori care nu are proprietatea
dorit
a. Numerot
am culorile de la 1 la 35. Fie li , respectiv ci , num
arul de linii/coloane
pe care se afl
a cel putin o c
asuta de culoare i. Cum intersectia a li linii cu ci coloane
35
P
are li ci elemente, rezult
a c
a
li ci 64 (). Din faptul c
a pe fiecare linie/coloan
a
i=1

avem cel mult 5 culori diferite, rezult


a c
a

35
P

i=1

li 40,

35
P

i=1

ci 40 si, desigur, li 1,

ci 1. Fie xi = li 1 0, yi = ci 1 0 si atunci
35
P

i=1

yi =

35
P

i=1

35
P

xi =

i=1

i=1

ci 35 5. Din () obtinem:
35
35
X
X
(xi yi + xi + yi + 1) 64
xi yi 19.
i=1

35
P

li 35 5,

i=1

ati pe care le
Putem presupune, din simetrie, c
a max xi max yi . Cele trei inegalit
avem la ndemn
a nu sunt suficiente pentru a obtine o contradictie, dar nu las
a dect
3 cazuri:
i) xi = yi = 5 si xk , yk = 0 pentru k 6= i (pentru un i fixat); atunci exist
a o
culoare care ocup
a (nu n ntregime!) 6 linii si 6 coloane, iar restul culorilor apar
exact o dat
a, deci vor r
amne dou
a linii si dou
a coloane care contin c
asute de culori
diferite dou
a cte dou
a, contradictie cu presupunerea asumat
a.
ii) xi = 5, yi = 4 si xk , yk = 0 pentru k 6= i (pentru un i fixat); acest caz este
evident mai defavorabil dect primul.
iii) xi = 5, yi = 4, yj = 1, xk = 0 pentru k 6= i, yk = 0 pentru k 6= i, j; nici acum
nu putem evita existenta unei linii sau coloane alc
atuit
a numai din c
asute de culori
diferite, deoarece 5 + max {1, 2} = 7 < 8.
Rezult
a c
a presupunerea f
acut
a este fals
a si r
amne adev
arat
a concluzia problemei.
G92. Fie 4ABC, cu [AB] cea mai scurta latura. Bisectoarea unghiului C
intersecteaza paralela dusa prin B la AC n M , iar latura AB n C 0 . Paralela prin
C 0 la AC intersecteaza M A n P si BC n Q. Aratati ca naltimile 4ABC pot fi
laturi ale unui triunghi daca si numai daca 2AB > P Q.
Valentina Blendea si Gheorghe Blendea, Iasi
2S
M
P
A
Solutie. Cu notatiile uzuale, ha =
, hb =
a
2S
2S
C
, hc =
. Cum c min {a, b}, atunci hc
b
c
este cea mai lung
a n
altime, prin urmare ha , hb ,
hc pot fi laturi ale unui triunghi dac
a si numai
B

154

dac
a hc < ha + hb . ns
a
hc < ha + hb

1
1 1
< + 2c >
c
a b

1
a

2
+

1
b

2 AB > P Q.

Pentru ultima echivalenta, am folosit faptul cunoscut c


a segmentul ce se spijin
a pe
laturile neparalele ale unui trapez, paralel cu bazele si care trece prin punctul de
intersectie al diagonalelor, are ca lungime media armonic
a a bazelor.

b
G93. Fie 4ABC cu m(A) 90 , M mijlocul lui [BC], iar N punctul de
\M
\
contact al cercului nscris cu BC. Daca BAN
AC, sa se arate ca 4ABC este
isoscel.
Doru Buzac, Iasi
Solutie. Dreptele AN si AM fiind izogonale, putem utiliza relatia lui Steiner:
BN BM
BN
AB 2
c2
, de unde
a parte, BN = p b si CN = p c,

=
= 2 . Pe de alt
2
NC MC
AC
NC
b
deci
pb
c2
= 2 b2 (a + c b) = c2 (a + b c)
pc
b

a b2 c2 + bc (b c) b3 c3 = 0

(b c) a (a + b) + bc b2 + bc + c2 = 0 (b c) a (b + c) b2 + c2 = 0.

b 90 , atunci b2 + c2 < a2 , iar a2 < a (b + c), prin urmare paranteza


Cum m(A)
p
atrat
a este strict pozitiv
a. R
amne c
a b c = 0, deci 4ABC este isoscel.

b = 30 . Se considera DE mediatoarea
b
G94. Fie 4ABC cu m(A) = 105 , m(B)
\ F AB, iar {I} =
lui [BC], D BC, E AB, [CF bisectoarea lui BCE,
CF DE, {G} = CE AI. Sa se arate ca 4DF G este echilateral.
Gabriel Mrsanu, Iasi
\
Solutie.
Evident c
a m(BCF ) =
A
\
[ = 15 . Apoi, 4EBC
E
m(F
CE) = m(ECA)
G
b =
este isoscel cu axa de simetrie ED si m(B)
F
\ = m(DEC)
\ = 60 , de
30 , deci m(BED)
I
[ = 60 . Rezult
unde m(CEA)
a c
a 4AEC
C
D
4IEC (U.L.U), prin urmare EA = EI si B
AC = IC. n 4EAI isoscel, EG este bisectoare, deci va fi si median
a: AG = GI. (1)
1
Aplicnd teorema bisectoarei n 4EBC, cum ED = BE (4DEB dreptunghic,
2
b = 30 ), obtinem:
m(B)
EF
EC
2 ED
ED
=
=
=
.
FB
BC
2 BD
BD
\) =
Reciproca teoremei bisectoarei n 4EDB arat
a c
a DF este bisectoare, deci m(EDF

\
\
\
m(F DB) = 45 . Cum BDF ACB, atunci F D k AC. Punctul D este mijlocul
1
lui [BC], prin urmare DF va fi linie mijlocie n 4BAC: DF = AC (2) si
2
b = 105 .Din (1) si (3) rezult
\
a c
a FG
AF = F B (3). n plus, m(BF
D) = m(A)
1
[
[ = 15 . Punctul
este linie mijlocie n 4ABI, deci F G = BI si m(AF
G) = m(ABI)
2
155

I se afl
a pe mediatoarea lui [BC], deci BI = IC si am ar
atat c
a IC = AC; rezult
a
1
c
a F G = AC. Conform (2), urmeaz
a c
a F G = F D, adic
a 4F GD este isoscel. n
2
plus,
\
[
\
D) m(AF
G) = 180 105 15 = 60 ,
m(DF
G) = 180 m(BF
deci 4F GD este echilateral.

G95. Fie 4ABM dreptunghic n B. Fie C pe [M A] astfel nct M C = AB.


Sa se arate ca n 4 ABC, bisectoarea AD, mediana BE si naltimea CF sunt
concurente.
Dan Brnzei, Iasi
Solutiea 1 (a autorului). Fie {K} = ADCF ;
A
vom ar
ata c
a punctele B, K, E sunt coliniare, de
E
unde rezult
a concluzia. Conform reciprocei teoreK
mei lui Menelaus n 4ADC, coliniaritatea punctelor F
C
KA
BC
0
B, K, E revine la
=
(1). Fie {D } =
D
KD
BD
AD BM ; atunci
M
B
D

KA
KA KD0
CA
DD0

=
=
1
+
=
KD
KD0 KD
CM
KD

0
BD
AC
BD0 D0 M
AC
1+
=
1+ 0
=
=
AB
KC
AB
D M KC

AC
AB AM
AC
AB
AM
=
1+

=
1+
=
.
AB
AM AC
AB
AC
AB
Pe de alt
a parte,

BC
DC
AC
AM
= 1+
= 1+
=
, ceea ce ncheie justificarea
BD
BD
AB
AB

egalit
atii (1).
Solutia 2 (dat
a de Marius Tiba, elev, Iasi). Not
am {T } = EB CF , {U } =
ET
EC
EBAD. Cum CF k BM , cu teorema lui Thales n 4EBD obtinem c
a
=
.
TB
CM
EU
AE
Din teorema bisectoarei n 4AEB avem c
a
=
. ns
a EC = AE, CM = AB
UB
AB
si atunci T = U , de unde concluzia problemei.

A. Nivel liceal
L86. Cercurile de centre Ia , Ib , Ic exnscrise 4ABC sunt tangente laturilor
\
[BC], [AC] respectiv [AB] n D, E, F . Bisectoarea interioara a unghiului BI
aC
intersecteaza latura BC n M ; fie {P } = F E AM , iar Q F D, S DE analog
definite. Aratati ca dreptele DP , EQ si F S sunt concurente.
Neculai Roman, Mircesti, Iasi
Solutie. Cu notatiile uzuale, avem: BD = AE = p c, CD = AF = p b,
BF = CE = p a. Teorema bisectoarei si teorema sinusurilor n 4BIa C dau
cos C2
IA B
MB
. Aplic
am acum relatia (R1 ) din nota Rapoarte determinate
=
=
MC
Ia C
cos B2
156

de o ceviana si o secanta ntr-un triunghi din RecMat 1/2005; obtinem:


AF M B AC P E
PE
AB M C AE

=1
=

AB M C AE P F
PF
AF M B AC
cos B2 p c
cos C2
PE
c
PE
c(p c) cos C2 cos2 B2
PE

.
=

=
PF
p b cos C2
b
PF
b(p b) cos B2 cos2 C2
PF
cos B2
cos A2 SD
cos B2
P E QF SD
QF
,
, prin urmare
Analog,
=
=

= 1 si concluzia
C
A
QD
P F QD SE
cos 2 SE
cos 2
rezult
a din reciproca teoremei lui Ceva.
Not
a. Vlad Emanuel, elev, Sibiu, evalueaz
a rapoartele necesare pentru aplicarea reciprocei teoremei lui Ceva prin considerente de arii.
L87. Sa se arate ca un tetraedru cu muchiile n progresie geometrica si n care
perechile de muchii opuse sunt perpendiculare, este regulat.
Marius Olteanu, Rm. Vlcea
Not
a. Ulterior public
arii n revista noastr
a, problema a ap
arut cu nr. O:1088 n
G.M. 6/2005. Solutia ei poate fi g
asit
a n G.M. 12/2005, pp. 640.
Solutie corect
a s-a primit de la Vlad Emanuel, elev, Sibiu.
L88. Fie 4ABC de arie S, perimetru2p, avnd razele cercurilor circumscris si
8+3 3
3
nscris R, respectiv r. Notam k =
p + r. Fie A, B, C discuri de centru
12
4
A, B, C si aceeasi raza < r. Pentru punctele D A, E B, F C, fie T aria
4DEF . Sa se arate ca, daca = k, atunci |T S| < .
Dan Brnzei, Iasi
Solutie. n
altimea din A taie cercul frontier
a a
A
lui A n A0 , A00 , iar paralelele la ea prin B, C taie
A
celelalte dou
a cercuri frontier
a B, C n B 0 , B 00 , C 0 ,
00
0 00 0 00
0 00 0 00
C , notate asa nct A B B A si A C C A s
a fie
A
paralelograme. n
altimea din D a 4DEF , notat
a
B
C
hD , satisface hA 2 hD hA + 2. Avem si
c
a a 2 EF a + 2. nmultind aceste siruri
C
B
de inegalit
ati ntre numere pozitive si notnd La =
B
C
a + hA , obtinem:
(hA 2)(a 2) < 2T < (hA + 2)(a + 2)
S La < T < S + La |S T | < La .

R
amne de demonstrat c
a La K; de fapt, ar trebui ar
atat c
a unul dintre
numerele La , Lb sau Lc (Lb , Lc construindu-se analog cu La ) poate fi ales K.
Avem:

1 1 1
L = La + Lb + Lc = a + b + c + hA + hB + hC = 2p + +2S
+ +
=
a b
c

2S
p2 + r2 + 4rR
p
r
= 2p +
(ab + bc + ac) = 2p +
= 2+
p+ 2+
r.
abc
2R
2R
2R
Vom fi asigurati c
a putem face alegerea anuntat
a dac
a ar
at
am c
a L 3K,
ine
p
3 3
galitate la care se ajunge usor folosind cunoscutele 2p 3 3R (

) si
2R
4
157

r
1
).
2R
4
L89. Cte drumuri de la A la B exista, daca din orice
A
C
punct de pe traseu ne putem deplasa doar spre stnga sau
E
F
n jos?
D
n etaje
Irina Mustata
a, Iasi
, elev
Solutie. Not
am cu an num
arul drumurilor de la A
la B si ncerc
am s
a g
asim o relatie de recurenta pentru
an . Not
am cu bn1 num
arul drumurilor de la E la B; B G
num
arul drumurilor de la C, F , respectiv D, pn
a la B va fi an1 , an2 , respectiv
bn2 . De la A la C putem ajunge n dou
a moduri, iar de la A la D putem ajunge
ntr-un singur mod f
ar
a a trece prin C, prin urmare
R 2r (

an = 2an1 + bn2 .

(1)

Cum din E putem pleca fie spre C, fie n jos, apoi stnga, spre D, atunci
bn1 = 2an1 + bn2 .

(2)

Din (1) si (2) obtinem


an = 2n1 + bn2 = 2an1 + an2 + bn3 = . . . = 2an1 + an2 + . . . + a1 + 1.
(Acel 1 din final reprezint
a singurul drum posibil de la G la B.) Trecem n n + 1
si sc
adem egalit
atile obtinute; g
asim:
an+1 an = 2an an1 an+1 = 3an an1 .

a omogen
a de
Se verific
a usor c
a a1 = 2, a2 = 5, obtinnd astfel o recurenta liniar
ordin II pentru an . Mai mult, se demonstreaz
a imediat prin inductie c
a an = F2n+1 ,
unde Fk este al k-lea termen din sirul lui Fibonacci.
Not
a. Solutie corect
a s-a primit de la Vlad Emanuel, elev, Sibiu.
L90. Fie n 1 un numar natural. Sa se arate ca multimea A = {n+1, n+2, . . . , 2n}
se poate partitiona n trei submultimi, fiecare cu aceeasi suma a elementelor, daca si
numai daca n este multiplu de 3, n 6.
Marian Tetiva, Brlad
Solutie. Pentru ca A s
a poat
a fi partitionat
a n trei clase avnd aceeasi sum
a
n(3n + 1)
a elementelor, trebuie ca suma elementelor sale, care este
, s
a fie multiplu
2
de 3, ceea ce conduce la 3 | n. Pentru n = 3, evident c
a A = {4, 5, 6} nu se poate
partitiona cum cere enuntul, deci n 6.
Fie acum n = 3k, k 2; dac
a k este par, k = 2p, p 1, atunci A = {6p + 1,
6p + 2, . . . , 12p}. Scriem
A = {6p + 1, 12p} {6p + 2, 12p 1} {9p, 9p + 1},

num
arul de submultimi fiind 3p, fiecare cu aceeasi sum
a a elementelor. Clasele
partitiei se obtin reunind p dintre aceste submultimi (prima clas
a), apoi alte p si la
urm
a cele p care au r
amas.
Pentru k impar, k = 2m + 1, m 1, avem de partitionat multimea A = {6m + 4,
6m + 5, . . . , 12m + 6}. Remarc
am c
a suma tuturor elementelor lui A este 3(2m + 1)
(9m + 5), iar suma elemntelor lui A care dau restul 2 la mp
artirea cu 3 este
(6m + 5) + (6m + 8) + + (12m + 5) = (2m + 1)(9m + 5),
158

prin urmare o clas


a a partitiei poate fi construit
a din numerele care dau restul 2
mp
artirea cu 3; fie P aceast
a multime. Mai not
am cu M multimea elementelor lui
A ce dau restul 1 la mp
artirea cu 3 si cu N multimea elementelor lui A ce sunt
multipli de 3. Suma elementelor lui M este (2m + 1)(9m + 5) (2m + 1), iar suma
elementelor lui N este (2m+1)(9m+5)+(2m+1); vom ncerca s
a schimb
am ntre ele
elemente din N cu elemente din M astfel nct s
a egaliz
am sumele. nti, efectu
am
schimbarea 12m + 1 12m + 6, care creste suma elementelor lui M cu 5. Apoi,
efectu
am m 2 schimb
ari, fiecare m
arind suma elementelor lui M cu 2 (si micsornd
corespunz
ator suma elementelor lui N ):
12m 2 12m; 12m 5 12m 3; . . . ; 9m + 7 9m + 9.
Cu aceasta, solutia problemei este ncheiat
a.
Not
a. O frumoas
a solutie prin inductie matematic
a s-a primit de la Vlad
Emanuel, elev, Sibiu.
L91. Punctele planului se coloreaza n 3 culori astfel nct fiecare culoare sa
fie folosita cel putin o data. Spunem despre un triunghi ca este aproape echilateral
daca masurile unghiurilor sale sunt cel mult 60, 001 . Aratati ca exista un triunghi
aproape echilateral cu vrfurile de culori diferite.
Adrian Zahariuc, elev, Bac
au
Solutie. S
a numim albastru, rosu si verde cele 3 culori. Vom scrie M (a), M (r)
sau M (v) dup
a cum este punctul M albastru, rosu sau verde. Fie A(a), B(r) si C(v)
si D1 un disc de centru O1 si raz
a r > 0 astfel nct A, B, C D1 . Fie d > 1000000r.
Fie D discul de centru O1 si raz
a d + r.
nu este monocromatic
S
a lu
am mai nti cazul cnd multimea de puncte D
a, unde

de culori
am notat cu T complementara multimii T (fata de plan). Fie P, Q D
diferite. Este clar c
a exist
a o nsiruire finit
a de puncte P = M0 , M1 , . . . , Mn = Q
si |Mi Mi+1 | 2r, i = 0, n 1. Cum M0 si Mn au culori
astfel nct [Mi Mi+1 ] D
diferite, rezult
a c
a exist
a i {0, 1, 2, . . . , n 1} astfel nct Mi si Mi+1 au culori
cu culori diferite, s
diferite. n concluzie, exist
a T, S D
a zicem T (a), S(r) astfel

nct [T S] D si |T S| 2r. Fie D2 discul de raz


a r si centru O2 , unde O2 este
mijlocul lui [T S]. Este clar c
a T, S D2 si c
a |O1 O2 | d. Fie un punct O3 astfel
nct triunghiul O1 O2 O3 s
a fie echilateral. S
a demonstr
am acum urm
atoarea
Lem
a. Fie O1 , O2 si O3 vrfurile unui triunghi echilateral de latura l 1000000r
si D1 , D2 si D3 , 3 discuri de raza r si centre O1 , O2 , respectiv O3 . Daca X D1 ,
Y D2 si Z D3 , atunci masurile unghiurilor 4XY Z sunt cel mult 60, 001 .
Demonstratie. Avem XY XO1 + O1 O2 + O2 Y l + 2r si O2 O3 O2 Y +
Y Z + ZO3 Y Z + 2r, sau Y Z l 2r. Atunci
XY
l + 2r
1000002
sin Z

< 1, 00001
< 1, 00001.
YZ
l 2r
999998
sin X
Relatiile analoage se demonstreaz
a ntr-o manier
a asem
an
atoare etc.
S
a revenim la problem
a. Fie D3 discul de centru O3 si raz
a r. Fie W D3 ,
oarecare. Dac
a W este albastru, conform lemei, triunghiul CSW este aproape echilateral. Dac
a W este rosu, triunghiul CT W este aproape echilateral. n fine, dac
aW
este verde, triunghiul ASW este aproape echilateral si demonstratia pentru primul
caz se ncheie.
159

este monocromatic
S
a lu
am acum cazul cnd multimea de puncte D
a, s
a zicem c
a
toate punctele ei sunt albastre. Fie L = {|RV | : R(r), V (v) R2 }. Din presupunerea
sunt albastre, rezult
c
a toate punctele din exteriorul discului D
a c
a multimea L este
m
arginit
a. Atunci exist
a l L astfel nct 1, 000001l > l0 pentru orice l0 L. Fie
punctele R1 (r), V1 (v) astfel nct |R1 V1 | = l. Fie punctul Q astfel nct QR1 =
QV1 = 1, 000001l. Dac
a Q este rosu, OV1 = 1, 000001l contrazice alegerea lui l.
Analog, Q verde ne conduce la o contradictie. Rezult
a c
a Q este albastru. Se verific
a
imediat c
a triunghiul QR1 V1 este aproape echilateral si problema este rezolvat
a.
L92. Daca ai (1, ), i = 1, n, n 3, iar m, p N cu m > p 1, sa se
demonstreze inegalitatea
m
m
m
am
am
2 + + an
1 + a3 + an
+ +
loga1 mp
mp + loga2 mp
mp
a2
+ + an
a1
+ a3
+ amp
n
m
am
1 + + an1
+ logan mp
np.
a1
+ + amp
n1

Gheorghe Molea, Curtea de Arges


ar
a
Solutie. Consider
am o ordonare a numerelor ai (1, ); putem presupune f
a restrnge generalitatea c
a a1 a2 an . Atunci ap1 ap2 apn si
amp
amp
amp
, m > p 1. Aplic
am inegalitatea lui Cebsev si
n
1
2
inegalitatea mediilor:
m
m
am
ap3 + + amp
apn
amp ap2 + amp
2 + a3 + + an
n
3
= 2

n1
n1
+ + amp
amp + amp
ap + ap3 + + apn
n
3
2
2

n1
n1
q
amp + a3mp + + amp
n
2
n1 ap2 ap3 apn
n1
q
m
m
am
p
n1
2 + a3 + + an

(a2 a3 an ) .
mp
mp
anp
+
a
+

+
a
n
2
3
Dac
a not
am cu S membrul stng al inegalit
atii din enunt, folosind aceast
a inegalitate
si analoagele obtinem:

p
S
loga1 (a2 a3 an ) + loga2 (a1 a3 an ) + logan (a1 a2 an1 )
n1

p X
logai aj + logaj ai .
S
n1
i6=j

arul grupelor de forma


Cum logai aj > 0, atunci logai aj + logaj ai 2. Num
n(n 1)
(logai aj + logaj ai ) este
si concluzia problemei urmeaz
a imediat.
2
Not
a. Pentru n = 3, m = 4, p = 3, obtinem problema 22814 din G.M. 5-6/1993.
Pentru n = 3, m = 2, p = 1 obtinem problema C:1605 din G.M. 11/1994.
Principial aceeasi solutie a dat Vlad Emanuel, elev, Sibiu.
L93. Exista vreun polinom f (X, Y ) n doua nedeterminate astfel nct

{f (m, n) | m, n Z} N = x2004
|k1 ,
k
160

n
n

1+ 2 + 1 2
unde xn =
?
2

Gabriel Dospinescu, Paris


Solutie. R
aspunsul este afirmativ! De exemplu, putem considera f (X, Y ) =
X 2004 (2 (X 2 2Y 2 )2 ) si atunci, pentru m, n Z,
f (m, n) > 0 2 > (m2 2n2 )2 |m2 2n2 | = 1.

Obtinem astfel dou


a ecuatii Pell; rezolvndu-le, obtinem c
a, de fapt,
{m2004 | exist
a n a.. |m2 2n2 | = 1} = {x2004
| k 1},
k

ceea ce arat
a c
a {f (m, n) | m, n Z} N = {x2004
| k 1}.
k

L94. Fie
A Mn(C) o matrice cu coeficienti ntregi, inversabila si astfel nct
multimea Ak | k N este finita. Sa se demonstreze ca aceasta multime are cel mult
2
3n elemente. Ramne rezultatul adevarat daca suprimam conditia ca elementele
matricei sa fie ntregi?
Gabriel Dospinescu, Paris
Solutie. Din ipotez
a rezult
a c
a exist
a p 1 astfel nct Ap = In . S
a lu
am p cel
2
mai mic cu aceast
a proprietate si s
a observ
am c
a dac
a p 3n atunci am terminat
prima parte a problemei. S
a presupunem deci contrariul si s
a consider
am matricele
din multimea H = {I, A, . . . , Ap1 }. Ele sunt n mod evident diferite oricare dou
a
si, mai mult, fiecare element al lui H are proprietatea c
a X p = In . S
a observ
am c
a
2
din faptul c
a H are cel putin 3n + 1 elemente, exist
a dou
a elemente n H a c
aror
diferenta este o matrice cu toate elementele multipli de 3 (principiul lui Dirichlet,
2
observnd c
a exist
a 3n matrice cu elemente 0, 1 sau 2). Deci exist
a B Mn (Z) si
1 i < j p 1 astfel nct Aj Ai = 3B. Avem atunci Aj1 In = 3BAp1 .
Notnd s = j i, X = BAp1 avem X M3 (Z) si (In + 3X)p = In . Fie 1 , . . . , n
2
valorile proprii ale lui X. Avem (1 + 3i )p = 1 de unde |i | ; atunci |tr X m | =
3
n

m
P m
i n 2
< 1 de la un rang m0 . Tinnd

cont c
a tr X m este ntreg, obtinem

3
i=1
n
P
m
c
a de la acel rang el este 0 si deci
i = 0 de la acel rang. Din formulele lui
i=1

a parte, din (In + 3X)p = In


Newton rezult
a 1 = = m =0. Pe de alt
a imediat
p 2c
p p1
obtinem
X 3pI
a i = 0 rezult
a
= On . Din faptul c

a c
n + 2 3 X + + 3 X
n
det 3pIn + p2 32 X + + 3p X p1 = (3p) 6= 0, deci matricea 3pIn + p2 32 X +
+ 3p X p1 este inversabil
a si cum X 3pIn + p2 32 X + + 3p X p1 = On , avem
n mod necesar X = On , adic
a Aj1 = In , contradictie cu minimalitatea lui p. Am
justificat astfel prima parte a problemei.
Pentru a doua parte, mai mult o glum
a, raspunsul este evident nu. Este suficient
s
a lu
am o matrice diagonal
a ce are ca elemente pe diagonal
a o r
ad
acin
a primitiv
a de
2
ordinul 3n + 2 a unit
atii.
L95. Fie f : R R periodica, marginita si astfel nct exista x0 R pentru
care ls (x0 ), ld (x0 ) exista, sunt finite si distincte. Determinati a R pentru care nu
Rx
exista lim (f (t) + a) dt.
x 0

Paul Georgescu si Gabriel Popa, Iasi

161

Rx
Solutie. Fie T R+ o perioad
a a lui f si not
am F (x) = (f (t) + a)dt; atunci
0
Z x+(n+1)T
Z T
F (x + (n + 1) T ) F (x + nT ) =
(f (t) + a) dt =
f (t) dt + aT
x+nT
0
Z
!
T

F (x + nT ) = F (x) + n

f (t) dt + aT

n N ,

x [0, T ).

RT
Cum f este m
arginit
a, rezult
a c
a F este m
arginit
a pe [0, T ], iar dac
a f (t)dt + aT 6=
0

!
RT
f (x) dt + aT .
0, rezult
a imediat c
a lim F (x) = (+) sgn
x

1 RT
f (t) dt; atunci
Fie acum a =
T 0
Z x+T
(f (t) + a) dt = F (x) ,
F (x + T ) =
0

x R,

deci F este periodic


a de perioa
a T . Presupunem prin absurd c
a F este constant
a.
Rx2
Atunci F (x1 ) = F (x2 ), x1 , x2 R, deci (f (t) + a) dt = 0, x1 , x2 R. Pentru
x1

> 0 fixat, putem g


asi > 0 astfel ca
Z x2
(ls + a) (x2 x1 )
(f (t) + a) dt (ls + + a) (x2 x1 )
x1

pentru x0 x1 < x2 < x0 , deci ls + a 0 ls + + a. Cum era arbitrar,


obtinem c
a ls (x0 ) + a = 0. Analog, ld (x0 ) + a = 0, deci ls (x0 ) = ld (x0 ) = a,
contradictie cu ipoteza. R
amne c
a F este periodic
a si neconstant
a si deci nu are
RT
limit
a la + n cazul n care f (t) dt + aT = 0.
0

Rezolvarea aritmogrifului de la pag. 129.


S
C
H
U
A R

V
I R
E I F
T C E H
E C R E
I R S
O S
E

ARITMOGRIF

P
F
A
F
F

S
T
O
L
Z

M W
G
I I * M A
R L
U
O E
S
N S
S

162

M F
K I R
H
A L H E
S F A I R E O G
T E M A T I C E B
U I E C I N
R G L O N
M L
B

Probleme propuse1
Clasele primare
P.114. n piramida al
aturat
a unele numere s-au sters de-a
lungul timpului. Putem s
a le punem la loc?
(Clasa I )
Ionela B
ar
agan, elev
a, Iasi

20 28
12 16

P.115. Dac
a din prima lad
a iau 2 mere si le pun n lada a doua, din a doua lad
a
iau 3 mere si le pun n lada a treia, iar din lada a treia iau 4 mere si le pun n prima
lad
a, atunci n fiecare lad
a voi avea cte 34 mere. Cte mere au fost n fiecare lad
a?
(Clasa I )
Mariana Nastasia, elev
a, Iasi
P.116. Luni, mama pune ntr-un cos cteva mere. Joi, ea g
aseste n cos doar 20
de mere. Cte mere a pus mama n cos, stiind c
a, n fiecare zi din acea s
apt
amn
a,
Mihai, cel mai mare dintre frati, mparte fratilor mai mici cu un m
ar mai mult ca n
ziua precedent
a si c
a joi el mparte 5 mere? Cte mere mai r
amn n cos smb
at
a,
dup
a mp
artirea merelor?
(Clasa II-a)
Inst. Maria Racu, Iasi
P.117. n exercitiul 1111111 = fiecare c
asuta poate fi nlocuit
a cu +
sau . Ct poate fi rezultatul acestui exercitiu?
(Clasa II-a)
Diana T
an
asoaie, elev
a, Iasi
P.118. Calculati a + b + c + d, stiind c
a a b = 5 si c d = 15. G
aseste toate
posibilit
atile.
(Clasa III-a)
nv. Rica Buc
atariu, Iasi
P.119. Produsul a 10 numere naturale este 40. Aflati cea mai mic
a si cea mai
mare valoare a sumei celor 10 numere.
(Clasa III-a)
nv. Mirela Buburuzanu, Tomesti (Iasi)
P.120. Se consider
a numerele 1, 2, 3, 4, 5, . . . , 49. Care este cel mai mare
num
ar de numere pe care putem s
a alegem dintre acestea astfel nct suma oric
aror
trei dintre ele s
a se mpart
a exact la 9.
(Clasa III-a)
nv. Felicia-Petronela Leanu, Ceplenita (Iasi)
P.121. Ceasul lui Andrei o ia nainte cu 20 secunde pe or
a. El a potrivit ceasul
luni la ora 8 si a citit din nou ceasul lunea urm
atoare la aceeasi or
a. S
tiind c
a n
aceast
a durat
a ceasul nu a functionat permanent, iar la ultima citire ar
ata ora 8 h
50 min, s
a se afle ct nu a functionat ceasul.
(Clasa IV-a)
Paula Borsanu, elev
a, Iasi
P.122. La Concursul de matematic
a "F.T.Cmpan", etapa judetean
a, au participat 100 elevi de clasa a IV-a, care au avut de rezolvat 3 probleme. Dac
a 70 elevi au
rezolvat bine prima problem
a, 69 a doua problem
a si 64 a treia problem
a, s
a se arate
c
a m
acar 3 elevi au rezolvat corect toate cele trei probleme.
(Clasa IV-a)
Anca Cornea, elev
a, Iasi
P.123. ntr-o cutie sunt 34 bile, din care unele cnt
aresc cu 1 g mai mult. Dac
a
fiecare bil
a cnt
areste un num
ar natural de grame, iar masa tuturor bilelor este 113 g,
1

Se primesc solutii pn
a la data de 1 iunie 2007.

163

s
a se afle cte bile sunt mai grele.
(Clasa IV-a)

Petru Asaftei, Iasi

Clasa a V-a
V.71. Comparati numerele 3300003 si 2450004 .

Lucian Tutescu, Craiova

V.72. Fie multimile A, B astfel nct A B, |P (A)| 60, |P (B)| 260. S


a se
determine |A| si |B|. (Prin |X| am notat cardinalul multimii X.)
Petru Asaftei, Iasi
V.73. S
a se scrie num
arul 20062005 ca o sum
a de sase p
atrate perfecte nenule.
Ionel Nechifor, Iasi
V.74. Se consider
a sirul de numere naturale 1, 1, 2, 5, 12, 27, 58, . . . . Calculati
suma primilor 100 de termeni ai sirului.
Marius Damian, Br
aila
V.75. Fie x, k N, x 2, k < x. S
a se arate c
a
(x 1) 12 . . . k (x) + k + 1 = 11
. . . 1} (x) .
| {z
k+1 cifre

Doru Buzac, Iasi

Clasa a VI-a
VI.71. Fie x, y, z N astfel nct 100x 2006y 2 + 15z = 0. S
a se arate c
a
..
y (x + z) . 85.
Dan Nedeianu, Dr.-Tr. Severin
VI.72. Fie p N un num
ar prim. S
a se determine x, y N astfel nct
p
y

+ N .
x2 x
D. M. B
atinetu-Giurgiu, Bucuresti
. . . 14}, n N . S
a se determine
VI.73. Fie An = 14 + 1414 + 141414 + + 1414
| {z

2
An 7n 7n
multimea M = n N |
N .
9

2n cifre

Valeriu Brasoveanu, Brlad

VI.74. Consider
am dou
a axe perpendiculare Ox si Oy, precum si cele dou
a bisectoare ale unghiurilor drepte care se formeaz
a. Fie A oarecare, iar B, C simetricele
sale fata de prima bisectoare, respectiv fata de Ox. Rotim segmentul [OC] n jurul
lui O cu 90 n sensul acelor de ceasornic si not
am cu D extremitatea segmentului
rotit. S
a se arate c
a:
a) B, O, D sunt coliniare si O este mijlocul lui [BD];
b) D este simetricul lui A fata de a doua bisectoare.
Adrian Corduneanu, Iasi
VI.75. Fie ABCD un patrulater convex, O intersectia diagonalelor, M mijlocul
lui [AB], iar N mijlocul lui [OD]. S
a se arate c
a 2PBCN M < PBDC + PABC .
Bogdan Posa si Marius Dr
agoi, elevi, Motru
164

Clasa a VII-a
VII.71. Fie 4ABC, AB < AC si D (AC). Fie AE bisectoarea lui \
BAC,
E (BD), F mijlocul lui [AD], {O} = AE BF , {G} = DO AB. S
a se arate c
a
GD k BC AB = CD.
Carmen Daniela Tamas, Brlad
VII.72. Fie ABCD paralelogram, E (CD), {M } = AE BD, {N } = BE
AC, {O} = AC BD. S
a se arate c
a AMEN = 2AMON .
Mirela Marin, Iasi
VII.73. Fie a < b c razele a trei cercuri tangente ntre ele si tangente la o
1
1
1
aceeasi dreapt
a n trei puncte distincte. S
a se arate c
a = + .
a
c
b
Dan Radu, Bucuresti
VII.74. Fie M multimea multiplilor lui 36 n a c
aror scriere n baza 10 nu apar
alte cifre dect 4, 6 sau 9. Cte numere cel mult egale cu 100 000 contine M ?
Gabriel Popa, Iasi
VII.75. Fie m 3 un num
ar natural impar si a1 , a2 , . . . , am Z astfel nct
|a1 a2 | = |a2 a3 | = = |am1 am | = |am a1 | .
Demonstrati c
a a1 + a2 + + am se divide cu m.
Maria Mihet, Timisoara

Clasa a VIII-a
VIII.71. Pe planul 4ABC se ridic
a perpendiculara AM . Fie P proiectia lui A
pe planul (M BC), iar E, F proiectiile punctului P pe M B, respectiv M C. S
a se
\
\
arate c
a M EF M CB.
Otilia Nemes, Ocna Mures
VIII.72. Fie a, b, c numere reale distincte. S
a se afle partea ntreag
a a num
arului
a2 + bc
b2 + ca
c2 + ab
A=
+
+
.
(a b) (a c) (b c) (b a) (c a) (c b)
Mihail Bencze, Brasov
VIII.73. S
a se demonstreze c
a a N este ipotenuz
a a unui triunghi dreptunghic
cu laturile exprimate prin numere naturale dac
a si numai dac
a exist
a n N astfel
atrate perfecte.
nct a2 n si a2 + n sunt p
C
at
alin Calistru, Iasi

xy
xy

1, n
VIII.74. Fie (0, 1]; s
a se arate c
a +
:
1 xy
1 xy
fiecare din situatiile:
x+
.
a) x, y [0, );
b) < x 0 y <
1 + x
Gheorghe Costovici, Iasi
2n+1 + 3
VIII.75. Determinati valorile lui n N pentru care fractia
este re3 2n + 1
ductibil
a.
Gheorghe Iurea, Iasi

Clasa a IX-a
IX.71. Fie a < b numere reale; s
a se determine x, y, z R pentru care 2z 2 + b =
165

= 2yz + a, iar

x a + b x = y.

Andrei Nedelcu si Lucian L


adunc
a, Iasi
x
y
z
IX.72. Fie x, y, z [1, +) asa nct
=
=
. S
a se arate c
a
[x]
[y]
[z]
q
q
p
2
2
2
2
2
2
[x] + [y] + [z] + {x} + {y} + {z} = x2 + y 2 + z 2 .

Ce egalitate se obtine pentru x, y, z (, 1]?

Dan Pl
aesu, Iasi
IX.73. Fie a1 , a2 , . . . , an (0, ) cu a1 a2 an = 1. Pentru orice m N , s
a se
arate c
a are loc inegalitatea
m1
m
m
+ a2m1 + + am1
.
am
1 + a2 + + an a1
n
Marius Tiba, elev, Iasi
IX.74. Fie a, b, c lungimile laturilor 4ABC si m, n (0, +). Consider
am
punctele A0 , B 0 , C 0 astfel nct C (AA0 ), A (BB 0 ), B (CC 0 ) si CA0 = ma + n,
AB 0 = mb + n, BC 0 = mc + n. S
a se arate c
a 4ABC si 4A0 B 0 C 0 au acelasi centru
de greutate dac
a si numai dac
a 4ABC este echilateral.
Dumitru Mihalache, Brlad
\ = 60
IX.75. Fie ABCD patrulater inscriptibil, {O} = AC BD, m(AOD)
MA
NC
OA
si punctele M (AB), N (CD). Not
am k =
, r =
, p=
. Dac
a
MB
ND
OB

2 1
\
p 2k, ,
, s
a se arate c
a m(M
ON ) 6= 90 .
r 2r
Mihai Haivas, Iasi

Clasa a X-a
X.71. Fie a, b, c lungimile laturilor unui triunghi. S
a se arate c
a

4
4
4
a + 2 bc
b + 2 ac
c + 2 ab
+

> 3.
+
a
+
c
b+ c
a+ b
Lucian Tutescu, Craiova
X.72. S
a se rezolve sistemul
x2 log2 15 + y 2 log3 10 + z 2 log5 6 = 2 (xy + yz + zx) ,
x + y + z = 5.
Marius Damian, Br
aila
X.73. Determinati functiile f : N
R cu proprietatea
c
a pentru orice x, y N

f (x) + f (y)
x+y
=
.
astfel nct 3 | x + y, are loc egalitatea f
3
3
Eugenia Rosu, elev
a, Iasi
X.74. Fie d1 , d2 dou
a drepte perpendiculare si l1 , l2 dreptele suport ale bisectoarelor celor dou
a perechi de unghiuri opuse formate de ele. Determinati 4ABC
cu A d1 , B d2 , C l1 si O, H l2 (O, H cu semnificatiile uzuale).
Temistocle Brsan, Iasi
X.75. a) Fie C . Dac
a z1 , z2 , z3 sunt trei numere complexe diferite astfel nct Re (
z1 ) = Re (
z2 ) = Re (
z3 ), atunci punctele de afixe z1 , z2 , z3 sunt
coliniare. Not
am cu d dreapta pe care sunt situate aceste puncte.
166

b) Dac
a z10 , z20 , z30 sunt trei numere complexe diferite cu proprietatea c
a Im (
z10 ) =
Im (
z20 ) = Im (
z30 ), atunci punctele cu afixele z1 , z2 , z3 sunt coliniare. Fie d0
dreapta ce le contine.
c) S
a se arate c
a dreptele d si d0 sunt perpendiculare.
Constantin Cocea, Iasi

Clasa a XI-a
XI.71. Fie sirul (xn )n1 , unde x1 > 1, xn+1 =
lim xnn .

xn 1
, n N . Calculati
ln xn

Dan Popescu, Suceava

XI.72. Este posibil ca o functie f : R R, care verific


a 1+f (x)+f (x) f (1+x) = 0,
x R, s
a fie continu
a pe R?
Dorin M
argidanu, Corabia
XI.73. Fie f : [0, 1] R o functie nenul
a de clas
a C k+1 pe [0, 1], k N , astfel
ca f (0) = f (1) = 0. Dac
a pentru orice 1 j k exist
a aj {0, 1} astfel nct
f (j) (aj ) = 0, atunci exist
a x1 , x2 (0, 1) astfel ca f (k+1) (x1 ) f (k+1) (x2 ) < 0.
Gheorghe Morosanu si Paul Georgescu, Iasi
XI.74. Fie a, b, c R, a 6= 0, n 2N . Urm
atoarele afirmatii sunt echivalente:
(1) b2 4ac 0,
(2) det aA2 + bA + cIn 0, A Mn (R).
Marian Urs
arescu, Roman

XI.75. Fie A, B M2 (C). S


a se arate c
a dac
a AB BA comut
a cu A si cu B,
atunci AB = BA.
Dorel Mihet, Timisoara

Clasa a XII-a
XII.71. Fie f : [a, b] R o functie derivabil
a cu derivata continu
a. S
a se arate
Rb
c
a lim a f (x) sin nx dx = 0.
n
Dan Radu, Bucuresti

XII.72. Fie functia f : [0,


a pe [0, 1], cu proprietatea
1] R,
indefinit derivabil
c
a exist
a M > 0 astfel nct f (n) (x) M , x [0, 1] si n N. Ar
atati c
a:
a) Pp
k+1
(n + p) (n + p 1) (n + p k + 2) f (n+pk) (1)
k=1 (1)
lim
= 0, p N ;
p+1
n
(1)
(n + p) (n + p 1) (n + 1)
b)

lim

Pp

k=1

(1)k+1 (n + p) (n + p 1) (n + p k + 2) f (n+pk) (1)


(1)

p+1

(n + p) (n + p 1) (n + 1)

1
0

xn f (n) (x) dx, n N.


Ovidiu Pop, Satu Mare

XII.73. S
a se arate c
a
Z 1
Z 1
(1 + x)n + (1 x)n

1
ln (1 + x)
ln
dx
=
n
dx = n ln 2, n N .
2
n
2
1+x
1+x
8
0 1+x
0
167

S
a se calculeze apoi

/4
R
0

ln

1 + tg3 x
dx.
1 + 3 tg2 x

Gabriel Necula, Plopeni


XII.74. Fie (G, ) un grup comutativ cu proprietatea c
a exist
a n N astfel nct
din xn = y n rezult
a x = y, cux, y G. Dac
a f , g sunt dou
a endomorfisme ale
lui G, atunci ecuatia f (x) = g x1 are solutie unic
a dac
a si numai dac
a functia
h : G G, h (x) = f (xn ) g (xn )este injectiv
a.
D. M. B
atinetu-Giurgiu, Bucuresti
XII.75. Fie A Mn (C) de rang r si S = {B Mn (C) | ABA = On }. Ar
atati
c
a S este subspatiu vectorial n Mn (C) si c
a dim S = n2 r2 .
Adrian Reisner, Paris

Premiu pe anul 2006 acordat


de ASOCIA
TIA RECREA
TII MATEMATICE
Se acord
a un premiu n bani n valoare de 100 lei elevului
ZAHARIUC Adrian Colegiul National "Ferdinand I", Bacau
pentru nota O problema despre suma cifrelor unui numar natural n baze de numeratie oarecare ap
arut
a n acest num
ar la pagina 113.
Acest premiu este acordat de Asociatia "Recreatii Matematice" ca urmare a
contractului de sponsorizare cu Fundatia Cultural
a "Poiana" (director d-l Dan
Tiba).

Premii pe anul 2006 acordate unor elevi


din Republica Moldova participanti la faza final
a
a ONM, Iasi, 2006
Se acord
a cte un premiu n valoare de 100 lei elevilor
GUZUN Ion
BOREICO Iurie
din lotul Republicii Moldova, care au obtinut cele mai bune rezultate.
Premiile sunt acordate de Asociatia "Recreatii Matematice", iar suma de
bani provine din contractul de sponsorizare cu Fundatia Cultural
a "Poiana"
(director d-l Dan Tiba).
168

Probleme pentru preg


atirea concursurilor
A. Nivel gimnazial
G106. Fie m N \ {0, 1} si k N, k m. Pentru fiecare x N, consider
am
propozitiile: x > 1; x > 2; . . . ; x > m. Aflati x N pentru care k din cele m
propozitii sunt adev
arate, iar celelalte m k sunt false.
Maria Mihet, Timisoara

G107. Multimea A N de cardinal n N are proprietatea c


a, oricare ar fi
patru elemente ale sale, putem alege dou
a cu suma 22006 + 1. Aflati valoarea maxim
a
a lui n.
Dan Nedeianu, Dr. Tr. Severin

G108. Fie m, n N . S
a se arate c
a multimea numerelor ntregi de modul cel
mult egal cu n poate fi partitionat
a n m submultimi cu aceeasi sum
a a elementelor,
dac
a si numai dac
a n + 1 m.
Marian Tetiva, Brlad
G109. La un concurs se dau sase probleme evaluate cu 1, 2, 3, 4, 5, repectiv
6 puncte. Dac
a un elev nu rezolv
a o problem
a, primeste 1 punct; dac
a o rezolv
a,
primeste punctajul corespunz
ator. Fiecare elev obtine m
acar 11 puncte. S
a se arate
c
a o problem
a a fost rezolvat
a de cel putin o treime dintre elevi.
Gabriel Dospinescu, student, Paris

G110. Fie multimile A = {k + n | k Z, n N} si B = (0, 1/10). Ar


atati c
a
A B este infinit
a.
Petru Asaftei, Iasi
G111. Fie 0 < a < b numere reale date
a s = x + y, p = xy,
si x,y [a, b]. Dac
ab s2 + ab
s
a se afle maximul expresiei E = p +
.
p
Vlad Emanuel, elev, Sibiu
G112. Dac
a a, b, c sunt lungimile laturilor triunghiului ABC, atunci
q
q
q
(a + b)2 c2 + (b + c)2 a2 + (c + a)2 b2 < 2 (a + b + c) .

Zdravko Starc, Vr
sac, Serbia si Muntenegru
G113. Fie segmentul [AB] de mijloc O si semicercurile C1 si C2 de diametru [AB],
respectiv [AO] situate n acelasi semiplan fata de AB. Perpendiculara n C (AO)
pe AB intersecteaz
a C1 n E si C2 n D. Dac
a AD C1 \ {F }, s
a se arate c
a AE este
tangent
a cercului circumscris 4DEF .
Alexandru Negrescu, elev, Botosani
\ = 60 . Dac
G114. Fie ABCD un paralelogram care nu este romb cu m(BAD)
a
M, N (AC), P (BC) si Q (CD) sunt astfel nct [BM , [DN , [AP si [AQ
\ ADC,
\ BAC
\ si respectiv DAC,
\ atunci M P este
sunt bisectoarele unghiurilor ABC,
perpendicular
a pe N Q.
Andrei Nedelcu, Iasi
G115. Fie p
atratul M N P Q nscris n p
atratul ABCD, M (AB), N (AD),
P (CD), Q (BC) si fie {E} = P N AB, {F } = P Q AB. Not
am cu S1 , S2 ,
S3 ariile p
atratului ABCD, p
atratului M N P Q, respectiv 4P EF . S
a se arate c
a:
169

p
a) S1 S2 = 4 SAEN SBF Q ;

b) S3 S1 ;

1
1
1
=

.
S3
S3 S3
Claudiu S
tefan Popa, Iasi
c)

B. Nivel liceal
L106. Fie I centrul cercului nscris n 4ABC. Dreptele AI, BI si CI intersecteaz
a a doua oar
a cercurile circumscrise triunghiului BCI, CAI si ABI n A0 , B 0 ,
0
respectiv C . Dac
a not
am cu |XY Z| perimetrul 4XY Z, s
a se demonstreze c
a
CA
AB
BC
+
+
= 1.
|BCA0 | |CAB 0 | |ABC 0 |
Titu Zvonaru, Com
anesti
a puncte situate n interiorul 4ABC, avnd distantele pn
a
L107. Fie M1 N dou
la laturile AB, BC, CA egale cu 3, 2, 7, respectiv 92 , 5, 52 . Dac
a raza cercului
circumscris 4ABC este R = 8, s
a se calculeze M N .
Vlad Emanuel, elev, Sibiu
L108. S
a se arate c
a n orice 4ABC are loc inegalitatea

3A
3 3
(sin A + sin B + sin C) 4 3 sin2
.
2
12
Marian Tetiva, Brlad
L109. Se dau numerele reale pozitive subunitare a1 , a2 , . . . ,a2n2 n , n N\{0, 1}.
S
a se demonstreze inegalitatea (sumarea se face prin permut
ari circulare)"
2n1
X
2n 1
a1
<
.
2n1
2n1
2
n+1
a2n1
+
a
+
.
.
.
+
a
+
2n
+
1
2
3
2n2 n
Ioan S
erdean, Or
astie
L110. Fie a, b, c (0, ) si n, k N. Demonstrati identitatea
bn+k
cn+k
4n(a b)2
an+k
k
k
k
+
+

a
+
b
+
c
+
.
bn
cn
an
k(a2k + b2k + c2k )

(n leg
atur
a cu o problem
a propus
a la OBM 2005).
Titu Zvonaru, Com
anesti and Bogdan Ionita
, Bucuresti
L111. Se dau m numere naturale distincte din multimea {1, 2, . . ., n}. S
a se arate
c
a putem alege cteva dintre ele, cu suma S, astfel nct

m(m + 1)
0S
n + 2n m.
2
Adrian Zahariuc, elev, Bac
au
L112. Pentru n N, not
am cu a(n) num
arul modurilor n care n se poate scrie
ca sum
a a unui num
ar par de puteri ale lui 2 si cu b(n) num
arul modurilor n care
n se poate scrie ca sum
a a unui num
ar impar de puteri ale lui 2. S
a se arate c
a
a(n) = b(n), n 2.
Adrian Zahariuc, elev, Bac
au
L113. Determinati numerele reale a, b pentru care multimea A = {an + bn |
n N } este finit
a.
Gheorghe Iurea, Iasi
170

L114. Consider
am o parabol
a si dou
a drepte secante parabolei, paralele ntre ele,
dar neparalele cu axa de simetrie a parabolei. Folosind doar rigla negradat
a, s
a se
construiasc
a tangenta la parabol
a care este paralel
a cu dreptele date.
Titu Zvonaru, Com
anesti
L115. Determinati P R[X], grad P 2, astfel nct functia f : R R,
f (x) = p({x}) + {p(x)} s
a fie periodic
a (unde p este functia polinomial
a atasat
a lui
P , iar {} desemneaz
a partea fractionar
a).
Paul Georgescu and Gabriel Popa, Iasi

Training problems for mathematical contests


Junior highschool level
G106. Let m N\ {0, 1} and k N, k m. For any x N, consider the
assertions: x > 1; x > 2; . . . ; x > m. Find x N such that k out of the m
assertions are true while the other m k are false.
Maria Mihet, Timisoara
G107. The set A N of cardinal number n N has the property that, from any
four of its elements, we can choose two elements whose sum is 22006 + 1. Find the
maximum value of n.
Dan Nedeianu, Drobeta-Turnu Severin

G108. Let m, n N . Show that the set of integer numbers of absolute value
at most equal to n can be partitioned into m subsets with the same sum of their
elements if and only if n + 1 m.
Marian Tetiva, Brlad
G109. Six problems are proposed for a school contest and they are respectively
evaluated with 1, 2, 3, 4, 5, 6 points. If a schoolchild does not solve a problem he
receives 1 point for it; if he (she) solves it he gets the corresponding points. Every
schoolchild receives at least 11 points. Show that a problem was solved by at least
one third of the participating schoolchildren.
Gabriel Dospinescu, Paris

G110. Let us consider the sets A = {k + n | k Z, n N} and B = (0, 1/10).


Show that A B is infinite.
Petru Asaftei, Iasi
G111. Let 0 < a < b be two given real numbers and x, y [a, b] . If s = x + y,
ab(s 2 + a b)
p = x y, determine the maximum value of the expression E = p +
.
p
Vlad Emanuel, highschool student, Sibiu
G112. Let a, b, c be the sides of the triangle ABC. Prove that
p
p
p
(a + b)2 c2 + (b + c)2 a2 + (c + a)2 b2 < 2(a + b + c).

Zdravko Starc, Vrac, Serbia and Montenegro


G113. Let [AB] be a line segment of midpoint O and let C1 and C2 be the
halfcircles of diameters [ AB], respectively [ AO] situated in the same halfplane with
171

respect to line AB. The perpendicular line at C (AB) on AB meets C1 at E and


C2 at D. If AD C1 = {F }, show that AE is tangent to the circle circumscribed to
DEF .
Alexandru Negrescu, highschool student, Botosani
\ =
G114. Let ABCD be a parallelogram that is not a rhomb with m(BAD)

60 . If M, N (AC), P (BC) and Q (CD) are such that [BM , [DN ,


\ ADC,
\ BAC
\ and respec[AP and [AQ are the bisectrix lines of the angles ABC,
\ then M P is orthogonal on N Q.
tively DAC
Andrei Nedelcu, Iasi
G115. Let M N P Q be the square inscribed in the square ABCD, with M (AB),
N (AD), P (CD) and Q (BC) and let {E } = P N AB, {F } = P Q AB.
We denote by S1 , S2 , S3 , the areas of the square ABCD, of the square M N P Q, and
of P EF , respectively. Show that:
p
1
1
1
a) S1 S2 = 4 SAEN SBF Q ;
b) S3 S1 ;
c)
=

.
S3
S3 S3
Claudiu-
Stefan Popa, Iasi

Highschool level
L106. Let I be the centre of the circle inscribed in the arbitrary triangle ABC.
The straight lines AI, BI, CI meet for the second time the circles circumscbribed
to the triangles BCI, CAI and ABI at the points A0 , B 0 and respectively C 0 . If we
denote by |XY Z| the perimeter of the triangle XY Z, show that
BC
CA
AB
+
+
= 1.
|BCA0 | |CAB 0 | |ABC 0 |
Titu Zvonaru, Com
anesti
L107. Let M , N be two points situated in the interior of ABC such their
9
5
distances up to the sides AB, BC, CA are respectively equal to 3, 2, 7 and , 5, .
2
2
If the radius of the circumscribed circle to ABC is R = 8, determine the distance
from M to N .
Vlad Emanuel, highschool student, Sibiu
L108. Show that the following inequality holds in any ABC :

3A
3 3
(sin A + sin B + sin C) 4 3 sin2
.
2
12
Marian Tetiva, Brlad
L109. The real positive and less than 1 numbers a1 , a2 , . . . ,a2n2 n , n N \ {0, 1}
are given. Prove the following inequality (with summation being performed by circular permutations):
X
2n 1
a2n1
1
<
.
2n1
2n1
2
n+1
a2
+ a3
+ . . . + a2n1
+
2n
+
1
2n2 n
Ioan S
erdean, Or
astie
L110. Let a, b, c (0, ) and n, k N. Prove the inequality
bn+k
cn+k
4n(a b)2
an+k
k
k
k
+
+

a
+
b
+
c
+
.
bn
cn
an
k(a2k + b2k + c2k )
172

(A problem connected with another problem proposed at OBM 2005).


Titu Zvonaru, Com
anesti and Bogdan Ionita
, Bucuresti
L111. We are given m distinct natural numbers contained in the set {1, 2, . . ., n}.
Show that it is possible to select a couple of them, with their sum equal to S, such

m(m + 1)
that 0 S
n + 2n m.
2
Adrian Zahariuc, highschool student, Bac
au
L112. For n N, we denote a(n) the number of ways under which the number
n can be written as a sum of an even number of powers of 2 and let b(n) denote
the number of ways under which n can be written as the sum of an odd number of
powers of 2. Show that a(n) = b(n), n 2.
Adrian Zahariuc, highschool student, Bac
au
n
n
L113. Determine the real numbers a, b such that the set A = {a + b | n N }
is finite.
Gheorghe Iurea, Iasi
L114. We consider a parabola and two parallel straight lines that meet the
parabola but are not parallel to its axis of symmetry. Using a non-measuring rule,
build the tangent to the parabola which is parallel to the given lines.
Titu Zvonaru, Com
anesti
L115. Determine the polynomials P R[X] of degree 2 so that the function
f : R R, f (x) = p({x}) + {p(x)} be periodical (where p is the polynomial function
corresponding to P , and {} denotes the franctional part function).
Paul Georgescu and Gabriel Popa, Iasi

ASOCIA
TIA RECREA
TII MATEMATICE
La data de 14.02.2005 a luat fiinta ASOCIATIA

RECREATII

MATEMATICE, cu sediul n Iasi (str. Aurora, nr. 3, sc. D, ap. 6), avnd ca scop sprijinirea activitatilor de matematica specifice nvatamntului preuniversitar, organizarea si desfasurarea de activitati care sa contribuie la dezvoltarea gustului pentru
matematica n rndurile elevilor, profesorilor si iubitorilor de matematica si stimularea preocuparilor si cercetatorilor originale.
Obiectivele majore pentru atingerea scopului propus sunt:
1. editarea unei reviste destinat
a elevilor si profesorilor revista "Recreatii
Matematice";
2. fondarea unei biblioteci de matematic
a elementar
a biblioteca "Recreatii
Matematice";
3. alc
atuirea unei colectii de c
arti de matematic
a elementar
a, c
arti de referinta
si aflate la prima aparitie Colectia "Recreatii Matematice".
Poate deveni membru al Asociatiei, printr-o simpla completare a unei cerei tip,
orice perosana care adera la obiectivele acesteia si sprijina realizarea lor.
173

Pagina rezolvitorilor
BRA
SOV
Liceul "N. Titulescu". Clasa a X-a. ANTONACHE Alexandru: VIII(61,62,64),
Dan:
IX(61,70), X.67; BARBU Cosmin: VIII(61-64), IX(61,70), X.69; BARDA
VIII(61-64), IX(61,70), X.69; BRNZEA Alexandru: VIII(61-64), IX.61; BUTNARIU Anda: VIII(61-64), IX.61; DAVID Dana: VIII(61,64), IX(61,70), X.69; FZI
Sandor: VIII(61-64), IX.61; GHICA Alexandra: VIII(61,63,64), IX(61,63), X.67;
ILIE Andreea: VIII(61,62,64), IX(61,70), X.67; MURESIANU Raul: VIII(61-64),
IX.61; OBANCEA Dragos: VIII.64, IX(68,70), X(67,69); SZCS Daniel: VIII(6164), IX(61,70), X.69; S
EITAN Marius: VIII(61-64), IX.61; S
OIM Mihaela: VIII(6164), IX(61,68,70), X.67; VASILE C
at
alin: VIII(61-64), IX.61; VRDOL Radu:
VIII(61-64), IX.61. Clasa a XI-a. DEMNY Ana-Maria: IX(63,65,68,70), X(67,69);
MUNTEAN Vlad: IX(61,63,65), X(62,69).
CARANSEBE
S
Liceul teoretic "Traian Doda". Clasa a IV-a. SZALO Ildiko: P(94-101).
Liceul pedagogic "G. D. Loga". Clasa a VIII-a. VLAD Adina Alexandra:
VI(61-65), VII(61,64), VIII(61,62), IX.63.
CRAIOVA
Colegiul National "Carol I". Clasa a VI-a. STANCIU Ioan: V(66-70), VI(6670), VIII.68, G(96,97).

HRLAU
Scoala

"P. Rares". Clasa a III-a (nv. BUDACEA Maria). NEICU Mara:


P(100,104,105,108,110).
Ioana: VI(66,67,69,70),
Liceul "Stefan

cel Mare". Clasa a VII-a. CURCA


VII(68,70); PINTILII Anda: VI(66,67,69,70), VII(68,70).
IA
SI
Scoala

nr. 3 "Al. Vlahuta". Clasa a III-a (inst. MAXIM Gabriela). BACIU


Ionela-Lavinia: P(94,96,98,100,103-105,108); CELMARE Raluca: P(94,96,98,100,
103-105,108); GHEMU Laura: P(94,96,98,100,103-105,108); NEAGU Ramona: P(94,
96,98,100,103-105,108); POPOVICI Ionut: P(94,96,98,100,103-105,108); RUSU Ioana
Andreea: P(94,96,98,100,103-105,108); RUSU M
ad
alina Andreea: P(94,96,98,100,
103-105,108); SAVA Vlad: P(94,96,98,100,103-105,108); VECHIU M
ad
alina: P(94,96,
98,100,103-105,108), ZANCHI Georgiana-Alexandra: P(94,96,98,100,103-105,108).

Valentina). BRLADEANU

Clasa a III-a (nv. MARIU


T
A
Andrei: P(98,100,103105,108); CULEA Alina: P(98,100,103-105,108); HADARAG Ana-Maria: P(98,100,

103-105,108); NASTASE
Cosmin: P(98,100,103-105,108); POPA Iulian: P(98,100,
103-105,108); PROCA Ancuta-Ioana: P(98,100,103-105,108); TNCU Alexandra:
P(98,100,103-105,108).
Scoala

nr. 13 "Alexandru cel Bun". Clasa a II-a (inst. COJOCARIU Ana).


Andra: P(104-107,109);
AGAFITEI

Elena-Roxana: P(104-107,109); CARAMALAU

CALIN
Andreea-Claudia: P(104-107,109); COJOCARIU Andreea: P(104-107,109);
DUDUMAN Luisa-Stefania: P(104-107,109); LELEU Alexandrina-Stefana: P(104107,109); LUPASCU Diana-Maria: P(104-107,109); MANOLACHE M
ad
alina-Andre174

A
Narcisa-Lorena: P(104-107,109); PASCU Gabriela:
ea: P(104-107,109); MIHAIL

P(104-107,109); PADURARU
Tiberiu-Stefan: P(104-107,109); RADUCEA
MarinAndrei: P(104-107,109); SAVIN Cristina-Simona: P(104-107,109); S
TEFAN BogdanVasile: P(104-107,109); S
TIUBEI Cosmin-Ionut: P(104-107,109).
Scoala

nr. 26 "G. Cosbuc". Clasa I (inst. RACU Maria). APACHITEI

AuraGeorgiana: P(94-97,105,110); BURA Emma-Andreea: P(94-97,105,110); FILIP In


grid-Stefania: P(94-97,105,110); GRADINARIU
Georgiana: P(94-97,105,110); HRISCU Ovidiu-Constantin: P(94-97,105,110); HUZA M
ad
alina: P(94-97,105,110); MAXIM Alexandra-Camelia: P(94-97,105,110); VASILE Bogdan-Andrei: P(94-97,105,
110). Clasa a IV-a (nv. BUCATARIU Rica). AELENEI George-Ciprian: P(104,
105,108-111); SCUTARU Ionela-Cristina: P(104-112).

Colegiul National "Emil Racovita". Clasa a II-a (inst. CALINESCU


Rodica).
PETREA M
ad
alina: P(104-107,110); UNGUREANU George: P(104,105,107,108,
110). Clasa a V-a. ENEA Maria: V(66-70); TURCANU

Andreea Bianca: V(6670); Clasa a VI-a. TUDORACHE Alexandru Gabriel: V(66-70), VI(66-70).
ALOI

Colegiul National "C. Negruzzi". Clasa a V-a. PAV


Alexandru: V(61-65).
Clasa a VII-a. TIBA Marius: VII(66-68,70), G(96,97,100,103).
PETRO
SANI
T
Florian: VIII(66,68,69), IX.68,
Liceul de informatica. Clasa a X-a. PALI
A
X.68, G(97,102,103).
SIBIU
Colegiul National "Gh. Lazar". Clasa a XI-a. VLAD Emanuel: X(61,63-65),
XI(62,63), L(86,87,89,90,92).
SUCEAVA
Scoala

generala nr. 3. Clasa a II-a (nv. TABARCEA Silvestru). FECHET


S
tefan: P(94-97,99).

Premii acordate rezolvitorilor


ASOCIATIA

"RECREATII

MATEMATICE" n colaborare cu redactia


revistei RECREATII
MATEMATICE acord
a cte o diplom
a si un premiu n
c
arti pentru trei aparitii la rubrica Pagina rezolvitorilor urm
atorilor elevi:
Liceul "N. Titulescu", Brasov
Dan (cl. a X-a): 1/2005(5pb), 2/2005(5pb), 2/2006(7pb);
BARDA
BUTNARIU Anda (cl. a X-a): 1/2005(6pb), 2/2005(5pb), 2/2006(5pb);
SZCS Daniel (cl. a X-a): 1/2005(6pb), 2/2005(5pb), 2/2006(7pb);
Liceul "Stefan

cel Mare", Hrl


au

CIUCA Ioana (cl. a VII-a): 2/2005(5pb), 1/2006(5pb), 2/2006(6pb);


PINTILII Anda (cl. a VII-a): 2/2005(5pb), 1/2006(5pb), 2/2006(6pb);
175

Colegiul National "Gh. Lazar", Sibiu


VLAD Emanuel (cl. a XI-a): 2/2005(5pb), 1/2006(7pb), 2/2006(12pb);
Scoala

nr. 3 "Al. Vlahuta", Iasi


RUSU M
ad
alina-Andreea (cl. a III-a): 2/2005(6pb), 1/2006(5pb), 2/2006(8pb);
SAVA Vlad (cl. a III-a): 2/2005(6pb), 1/2006(5pb), 2/2006(8pb);
Scoala

nr. 13 "Alexandru cel Bun", Iasi


AGAFITEI

Elena-Roxana (cl. a III-a): 2/2005(5pb), 1/2006(5pb), 2/2006(5pb);


Andra (cl. a III-a): 2/2005(5pb), 1/2006(5pb), 2/2006(5pb);
CARAMALAU

CALIN
Andreea-Claudia (cl. a III-a): 2/2005(5pb), 1/2006(5pb), 2/2006(5pb);
COJOCARIU Andreea (cl. a III-a): 2/2005(5pb), 1/2006(5pb), 2/2006(5pb);
DUDUMAN Luisa-
Stefania (cl. a III-a): 2/2005(5pb), 1/2006(5pb), 2/2006(5pb);
LELEU Alexandrina-
Stefana (cl. a III-a): 2/2005(5pb), 1/2006(5pb), 2/2006(5pb);
LUPA
SCU Diana-Maria (cl. a III-a): 2/2005(5pb), 1/2006(5pb), 2/2006(5pb);
MANOLACHE M
ad
alina-Andreea (cl. a III-a): 2/2005(5pb), 1/2006(5pb),
2/2006(5pb);
PASCU Gabriela (cl. a III-a): 2/2005(5pb), 1/2006(5pb), 2/2006(5pb);

PADURARU
Tiberiu-
Stefan (cl. a III-a): 2/2005(5pb), 1/2006(5pb), 2/2006(5pb);

RADUCEA
Marin-Andrei (cl. a III-a): 2/2005(5pb), 1/2006(5pb), 2/2006(5pb);
SAVIN Cristina-Simona (cl. a III-a): 2/2005(5pb), 1/2006(5pb), 2/2006(5pb);
S
TEFAN Bogdan-Vasile (cl. a III-a): 2/2005(5pb), 1/2006(5pb), 2/2006(5pb);
S
TIUBEI Cosmin-Ionut (cl. a III-a): 2/2005(5pb), 1/2006(5pb), 2/2006(5pb);

ASOCIA
TIA RECREA
TII MATEMATICE
Lista membrilor asociatiei - continuarea listei din nr. 1/2006.
40
41
42
43
44
45
46
47
48
49
50
51

Numele si prenumele
Locul de munca
Data nscrierii
Calistru C
at
alin
Univ. Tehn. "Gh. Asachi", Iasi
22.07.2005
Nechita Vasile
Col. Nat. "C. Negruzzi", Iasi
08.08.2005
Kern Edgar
S. C. SELGROS, Brasov
11.08.2005
B
atinetu-Giurgiu Maria Acad. Tehnic
a Militar
a, Buc.
11.08.2005
B
atinetu-Giurgiu Dumitru Col. Nat. "Matei Basarab", Buc.
11.08.2005
Picioroag
a Gabriel
Univ. of Southern, Denmark
27.08.2005
Anisca R
azvan
Lakehead University, Canada
08.09.2005
Chetan Corneliu
CITRIX, INC., USA
02.10.2005
Maftei V. Ioan
Col. Nat. "Sf. Sava", Buc.
09.10.2005
Gheorhita V. Vitalii
Univ. Tehn. "Gh. Asachi", Iasi
20.01.2006
Ionescu Marius Vasile
Dartmounth College, USA
25.03.2006
Ciacoi Bogdan Andrei
Lic. T. "Ana Ip
atescu", Gherla, (elev) 27.03.2006
176

Revista semestrial RECREAII MATEMATICE


este editat de
ASOCIAIA RECREAII MATEMATICE. Apare la datele de 1 martie i
1 septembrie i se adreseaz elevilor, profesorilor, studenilor i tuturor celor
pasionai de matematica elementar.
n atenia tuturor colaboratorilor
Materialele trimise redaciei spre publicare (note i articole, chestiuni de
metodic, probleme propuse etc.) trebuie prezentate ngrijit, clar i concis; ele
trebuie s prezinte interes pentru un cerc ct mai larg de cititori. Se recomand ca
textele s nu depeasc patru pagini. Evident, ele trebuie s fie originale i s
nu fi aprut sau s fi fost trimise spre publicare altor reviste. Rugm ca materialele tehnoredactate s fie nsoite de fiierele lor.
Problemele destinate rubricilor: Probleme propuse i Probleme pentru
pregtirea concursurilor vor fi redactate pe foi separate cu enun i demonstraie/rezolvare (cte una pe fiecare foaie) i vor fi nsoite de numele autorului, coala i localitatea unde lucreaz/nva.
Redacia va decide asupra oportunitii publicrii materialelor primite.
n atenia elevilor
Numele elevilor ce vor trimite redaciei soluii corecte la problemele din
rubricile de Probleme propuse i Probleme pentru pregatirea concursurilor
vor fi menionate n Pagina rezolvitorilor. Se va ine seama de regulile:
1. Pot trimite soluii la minimum cinci probleme propuse n numrul
prezent i cel anterior al revistei; pe o foaie va fi redactat soluia unei singure
probleme.
2. Elevii din clasele VI-XII au dreptul s trimit soluii la problemele
propuse pentru clasa lor, pentru orice clas mai mare, din dou clase mai mici i
imediat anterioare. Elevii din clasa a V-a pot trimite soluii la problemele propuse
pentru clasele a IV-a, a V-a i orice clas mai mare, iar elevii claselor I-IV pot
trimite soluii la problemele propuse pentru oricare din clasele primare i orice clas mai mare. Orice elev poate trimite soluii la problemele de concurs (tip G i L).
3. Vor fi menionate urmtoarele date personale: numele i prenumele,
clasa, coala i localitatea.
4. Plicul cu probleme rezolvate se va trimite prin pot (sau va fi adus
direct) la adresa Redaciei:
Prof. dr. Temistocle Brsan
Str. Aurora, nr. 3, sc. D, ap. 6,
700 474, Iai
Jud. IAI
E-mail: tbirsan@math.tuiasi.ro

CUPRINS
Mendel Haimovici i coala matematic din Iai ........................................................... 89
Florica T. Cmpan ................................................................................................................ 91
Eclipsele de Soare.................................................................................................................. 93
Olimpiada 57 .......................................................................................................................... 96

ARTICOLE I NOTE
A. REISNER Pseudoinvers i invers generalizat ale unei aplicaii liniare............... 97
C. IGERU O soluie parial a unei probleme a lui N. Papacu ............................ 102
M. APETRII i C.-C. BUDEANU Criterii de congruen a triunghiurilor ...................... 107
D. M. BTINEU-GIURGIU Asupra problemei 3639
din Gazeta Matematic, v. XXXIII (1927-1928)............................ 110

NOTA ELEVULUI
A. ZAHARIUC O problem despre suma cifrelor unui numr natural
n baze de numeraie oarecare......................................................... 113
Al. NEGRESCU Generalizri ale unor inegaliti din RecMat...................................... 117

CHESTIUNI METODICE
M. TETIVA Cea mai bun inegaliate de acest tip

.............................................. 120

CORESPONDENE
Societatea de tiine Matematice din R. Moldova ............................................................ 124

CONCURSURI I EXAMENE
Concursul de Matematic "Al. Myller", Ediia a IV-a, Iai............................................. 126
Concursul de matematic "Florica T. Cmpan" ...................................................................... 130

PROBLEME I SOLUII
Soluiile problemelor propuse n nr. 2/2005........................................................................ 134
Soluiile problemelor pentru pregtirea concursurilor din nr. 2/2005 ............................... 152
Probleme propuse................................................................................................................... 163
Probleme pentru pregtirea concursurilor ............................................................................ 169
Training problems for mathematical contests ..................................................................... 171
Pagina rezolvitorilor ............................................................................................................ 174
ASOCIAIA RECREAII MATEMATICE
5 lei

Anul IX, Nr. 2

Iulie Decembrie 2007

RECREAII
MATEMATICE
REVIST DE MATEMATIC PENTRU ELEVI I PROFESORI

e i = 1

Asociaia Recreaii Matematice


IAI - 2007

Semnificaia formulei de pe copert:


i
ntr-o form concis, formula e = 1 leag cele patru ramuri fundamentale
ale matematicii:
ARITMETICA
GEOMETRIA
ALGEBRA
ANALIZA MATEMATIC

reprezentat
reprezentat
reprezentat
reprezentat

de
de
de
de

i
e

Redacia revistei :
Petru ASAFTEI, Dumitru BTINEU-GIURGIU (Bucureti), Cornelia - Livia BEJAN,
Temistocle BRSAN, Dan BRNZEI, Ctlin - Cristian BUDEANU, Alexandru
CRUU, Constantin CHIRIL, Eugenia COHAL, Adrian CORDUNEANU, Mihai
CRCIUN (Pacani), Gabriel DOSPINESCU (student, Paris), Paraschiva GALIA, Paul
GEORGESCU, Mihai HAIVAS, Gheorghe IUREA, Lucian - Georges LDUNC, Mircea
LUPAN, Gabriel MRANU, Andrei NEDELCU, Gabriel POPA, Dan POPESCU
(Suceava), Florin POPOVICI (Braov), Maria RACU, Neculai ROMAN (Mirceti), Ioan
SCLEANU (Hrlu), Ioan ERDEAN (Ortie), Dan TIBA (Bucureti), Marian TETIVA
(Brlad), Lucian TUESCU (Craiova), Adrian ZAHARIUC (Bacu), Adrian ZANOSCHI,
Titu ZVONARU (Comneti).

Adresa redaciei:
Catedra de Matematic Universitatea Tehnic Gh. Asachi Iai
Bd. Carol I, nr.11, 700506, Iai
Tel. 032 213737 / int. 123
E-mail: recreatii.matematice@gmail.com
http://www.recreatiimatematice.uv.ro
COPYRIGHT 2007, ASOCIAIA RECREAII MATEMATICE
Toate drepturile aparin Asociaiei Recraii Matematice. Reproducerea integral sau
parial a textului sau a ilustraiilor din aceast revist este posibil numai cu acordul prealabil
scris al acesteia.
TIPRIT LA SL&F IMPEX IAI
Bd. Carol I, nr. 3-5
Tel. 0788 498933
E-mail: simonaslf@yahoo.com

Anul IX, Nr. 2

Iulie Decembrie 2007

RECREAII
MATEMATICE
REVIST DE MATEMATIC PENTRU ELEVI I PROFESORI

e i = 1
Revist cu apariie semestrial
publicat de

ASOCIAIA RECREAII MATEMATICE

IAI - 2007

Al VI-lea Congres International


al Matematicienilor Romni
n zilele de 28 iunie 4 iulie, 2007, s-au desf
asurat la Bucuresti (Universitate) lucrarile celui de-al VI-lea Congres International al Matematicienilor Romni,
sub auspiciile Academiei Romne si ale Universitatilor din Bucuresti, Pitesti si Timisoara. Un rol central n organizarea Congresului l-a avut Institutul de Matematica
"Simion Stoilov" al Academiei Romne.
Amintim ca precedentele congrese, similare acestuia, s-au tinut la Cluj (1929),
Turnu Severin (1932), Bucuresti (1945), din nou Bucuresti (1956) si Pitesti (2003).
Ideea organiz
arii unor astfel de congrese s-a datorat lui Petre Sergescu, fost profesor la Universit
atile din Cluj si Bucuresti, precum si la Politehnica din Bucuresti
(unde a avut si functia de rector). Petre Sergescu va deveni, dupa stabilirea sa n
Franta (1946), presedinte al Academiei Internationale de Istorie a S
tiintei. El a fost
teica si Dimitrie Pompeiu, reprezensprijinit si ncurajat de c
atre Gheorghe Ti
tanti proeminenti ai matematicii romnesti n prima jum
atate a sec. al XX-lea. Participarea matematicienilor straini a fost din ce n ce mai numeroasa, nregistrndu-se
un record de peste 100 la prezentul congres. De asemenea, numarul matematicienilor romni din diaspor
a a fost considerabil, remarcndu-se n special tinerii care-si
preg
atesc doctoratul n diverse tari.
Mentionam ca toate tarile europene, din est si din vest, au avut participanti. Un
numar de peste 20 participanti au prezentat celelalte continente ale globului (toate).
n cele opt sectii ale Congresului au fost prezentate peste 400 comunic
ari. n
deschiderea Congresului au fost prezentate trei rapoarte, ntocmite de colective
desemnate de Comitetul de organizare: I Cercetarea matematica n Romnia; II
nvatamntul matematic n Romnia; III Diaspora matematica romneasca. Au
raportat Vasile Brnzanescu (Director al Institutului de Matematic
a "S. Stoilov"),
Constantin Niculescu (Universitatea Craiova), Constantin Corduneanu (Academia
Romna) si Dan Timotin (Institutul de Matematica).
Dintre participantii din strainatate (incluznd Diaspora), mentionam ctiva reputati matematicieni ca H. T. Banks (SUA), Daniel Tataru (SUA), Doina Cioranescu (Franta), Izu Vaisman (Israel), Vicenzo Cappasso si Mimo Iannelli (Italia),
Jean Mawhin (Belgia), Mitrofan Cioban (R. Moldova), Tudor Ratiu (Elvetia), Eugen
Grebenikov (Rusia), Alexandra Bellow (SUA), Preda Mihailescu (Germania), Florin
Diacu (Canada).
Multi participanti tineri au prezentat comunic
ari, din cele mai noi domenii de
cercetare matematica. Nici informatica nu a fost neglijata, precum si matematicile
aplicate la diverse alte domenii de cercetare: Mecanica, Fizica matematica, Probabilitate si Statistica, Cercetare operationala.
Un alt fapt demn de remarcat si foarte mbucur
ator pentru noi este participarea
cadrelor didactice din (practic) toate universitatile romnesti n care matematica
apare ca disciplina de studiu.
Congresul s-a bucurat de atentia forurilor r
aspunz
atoare pentru stiinta romneasc
a, ncepnd cu Presedintia (mesaj trimis la deschidere), continund cu Academia
Romna (prin presedintele ei, acad. Ionel Haiduc), cu Universitatea Bucuresti (prin
77

rector, care a oferit si o receptie mpreun


a cu Ministerul de Externe).
O excursie a fost organizata n ziua de duminica 1 iulie, pentru mai bine de o
suta de participanti. S-au vizitat obiective turistice precum orasul Sinaia si Castelul
Bran, iar la sfrsit Universitatea din Pitesti, unde a avut loc si banchetul Congresului. Muzica si dansul (cu specific romnesc), costumele nationale si ambianta
placuta ce a fost prezenta n timpul excursiei (minunatele privelisti de pe traseul
Bran-Cmpulung-Muscel), au contribuit n mare masura la reusita acestei actiuni,
menit
a s
a aminteasc
a oaspetilor de frumusetile tarii noastre.
Sper
am ca urm
atorul congres, Al VII-lea Congres International al Matematicienilor Romni, sa aiba loc n anul 2011, daca se va respecta intervalul de patru
ani ntre congrese consecutive.
Constantin CORDUNEANU
University of Texas at Arlington

78

Conjectura lui Poincar


Termenul conjectura nseamna n matematica presupunere, ipoteza, n sensul
unei afirmatii nedemonstrate, care poate fi adev
arat
a cu o probabilitate destul de
mare (spre exemplu, este adev
arat
a n mai multe cazuri particulare, ca n cazul inductiei incomplete). Conjectura lui Poincar se refera la caracterizarea sferei
3-dimensionale cu ajutorul unor proprietati topologice usor de intuit si a fost propus
a de Poincar n 1904. Jules Henri Poincar (29 aprilie 1854 -17 iulie 1912)
a fost unul dintre cei mai mari matematicieni francezi; a fost, n acelasi timp, un
mare fizician teoretic si un filosof al stiintei. Poincar este descris adesea ca Ultimul
matematician universal, deoarece a excelat n toate domeniile matematice, cu adev
arat importante, ce existau n perioada vietii sale (relativ scurte). La moartea sa,
a fost caracterizat ca matematician, geometru, filosof si om de litere; a fost un poet
al infinitului, un fel de bard al stiintei.
Pn
a n momentul formul
arii conjecturii, existau suficiente informatii si rezultate
legate de caracterizarea suprafetelor (variet
ati 2-dimenionale) orientabile, m
arginite
si nchise din spatiul euclidian 3-dimensioanal. Aceste suprafete pot fi caracterizate
de genul lor. Acesta este un numar ntreg nenegativ (g 0) care poate fi descris
intuitiv ca num
arul de gauri ale suprafetei. Spre exemplu, sfera uzual
a, definit
a ca
locul geometric binecunoscut n geometria euclidin
a clasic
a (sau ca frontiera bilei)
are genul zero pentru c
a nu are nici o gaur
a. Torul, asimilat cu suprafata unui covrig
are genul 1 pentru ca are o gaura. Mai departe, se pot imagina covrigi cu mai multe
g
auri, iar suprafetele lor ne furnizeaz
a suprafete de genuri mai mari. A rezultat
destul de usor c
a doua suprafete orientabile, nchise si marginite (compacte) avnd
acelasi gen pot fi puse n corespondenta biunivoca si bicontinua (sunt homeomorfe).
n particular, sfera apare ca singura suprafata orientabila, compacta de gen zero.
Problema ce apare n mod natural, este dac
a exist
a caracteriz
ari de acelasi fel
pentru sfera 3-dimenional
a, gndit
a ca frontiera unei bile 4-dimensionale. Poincar
a imaginat o operatie intuitiv
a deosebit de fructuoas
a pentru dezvoltarea ulterior
aa
topologiei algebrice, o disciplina matematica noua, extrem de importanta n momentul
actual. Este vorba despre deformarea continua n interiorul unei anumite multimi,
a unei curbe continue sau diferentiabile din acea multime. O astfel de deformare
poart
a numele de homotopie. n particular, este important cazul cnd curba este
simpla nchisa si deformarea se face catre un punct al ei. Intuitiv, putem sa ne
imagin
am procesul realizat cu un lasou lansat de un paznic de vite. Cnd acest lasou
nu prinde gtul niciunei vite, el se strnge napoi n mna paznicului. Cnd lasoul
prinde o vit
a (sau un ciot de copac), a ntlnit o singularitate (o gaur
a) si paznicul este
nevoit sa deschida lasoul (adica sa renunte la continuitate) ca sa-l poata desprinde.
Acum putem formula mai precis continutul conjecturii lui Poincar:
Daca o varietate M 3 , neteda, compacta, de dimensiune 3, are proprietatea ca
orice curba simpla nchisa situtata n aceasta varietate poate fi deformata continuu
la un punct, rezulta ca M 3 este homeomorfa cu o sfera?
Chiar Poincar a notat oarecum prev
az
ator: Mais cette question nous entranerait
trop loin. Conjectura lui Poincar a inspirat multi matematicieni si tentativele de a o
demonstra au condus la multe progrese n ntelegerea topologiei varietatilor de dimen79

siune trei si nu numai. O extindere natural


a a acestei conjecturi a fost formulat
a chiar
de Poincar, care a afirmat, n mod eronat, ca: orice varietate poliedrala compacta,
avnd omologia unei sfere n-dimensionale este homeomorfa cu sfera n-dimensionala.
Notiunea de omologie a fost abordat
a, la nceput, n contextul topologiei combinatorii,
disciplin
a ce studiaz
a complexele simpliciale sau, mai general, celulare, apoi, aceast
a
notiune a fost studiata n conditii mai generale, obtinndu-se invarianti topologici
interesanti.
Pe la sfrsitul anilor 50 si nceputul anilor 60 s-au obtinut rezultate consistente
n studiul conjecturii lui Poincar, realizndu-se c
a studiul variet
atilor de dimensiuni
mai mari era mai usor de facut dect al celor de dimensiune 3. Conjectura lui
Poincar a fost demonstrata n cazul unei dimensiuni mai mari dect 4, n 1960, de
c
atre S. Smale. Alte contributii au fost aduse de c
atre J. Stallings, E. Zeeman si
A. Wallace. 20 de ani mai trziu, M. Freedman a folosit cup-produsul si invariantul
Kirby Siebenmann, pentru a demonstra conjectura lui Poincar n dimensiunea 4.
Pentru dimensiunea 3, toate tehnicile dezvoltate anterior nu au dat rezultate. In
mod curios, la fel ca n problema g
asirii structurilor diferentiabile pe spatiile euclidiene (unde s-au folosit asa numitele teorii de etalonare (gauge), specifice fizicii) solutia
a venit din partea geometriei diferentiale. In geometria diferentiala problematica este
centrat
a pe studierea propriet
atilor diverselor structuri geometrice pe variet
ati si mai
putin pe probleme de tipul conjecturii lui Poincar. A fost R. Hamilton care a propus
pentru studiu fluxul Ricci, pentru care a oferit si unele interpret
ari fizice. El a reusit
sa construiasca, n 2003, o metrica de curbura constanta pe orice 3-varietate avnd
curbura Ricci pozitiv
a.
Ceva mai trziu, G. Perelman de la Sankt Petersburg a oferit o solutie a conjecturii lui Poincar, n cteva articole postate pe Internet. Aceast
a solutie a trezit
intersul mai multor grupuri de cercetatori care au nceput sa aprofundeze detaliile
tehnice ale demonstratiilor propuse de Perelman.
Oricum, comunitatea matematic
a international
a s-a ar
atat convins
a de argumentele lui G. Perelman si, la Congresul International al Matematicienilor din august
2006 de la Madrid, lui G. Perelman i s-a oferit medalia Fields (un premiu asemanator cu premiul Nobel, care nu exist
a pentru domeniul matematicilor). Trebuie spus
c
a G. Perelman a refuzat, pentru prima dat
a n lumea matematicienilor, medalia
Fields din diverse motive. n momentul actual, G. Perelman trece printr-o perioad
a
extrem de rea din viata sa, adoptnd o atitudine de respingere a oricarei tentative
de apropiere din partea confratilor s
ai (o atitudine asem
an
atoare a adoptat sahistul
american R. Fisher n anii 70, dup
a obtinerea titlului de campion mondial). G. Perelman a refuzat si un premiu al Societatii Europene de Matematica (EMS ) si este pe
cale sa refuze si Premiul Mileniului. S-a retras de la Institutul Steklov, unde lucra,
si tr
aieste izolat, al
aturi de mama sa.
ntr-o ierarhie a celor mai importante descoperiri stiintifice din anul 2006, realizat
a
de prestigioasa revist
a Science, solutia dat
a de G. Perelman s-a situat pe primul loc,
devansnd o alta descoperire stiintifica extrem de importanta din domeniul geneticii.

Prof. dr. Vasile OPROIU


Univ. "Al. I. Cuza", Ia
si
80

Tipurile subgrupurilor finite din GL2 (Z)


Gabriel DOSPINESCU 1
Studiul care urmeaza este o continuare a celui nceput n [1]. Cu acel prilej am
demonstrat o serie de rezultate care ne-au permis s
a g
asim major
ari pentru ordinele
subgrupurilor finite ale lui GLn (Z).
Ne-am bazat pe o teorema de mare profunzime, cunoscuta sub numele de Lema
lui Serre. De fapt, rezultatul a fost obtinut de Minkowski si extins de Selberg, drept
pentru care l vom numi n cele ce urmeaz
a Lema lui Selberg.
Ne-a mai r
amas, din planul nostru, s
a demonstr
am c
a n GL2 (Z) exist
a exact
noua tipuri de grupuri finite (pna la un izomorfism). Am adus deja n discutie
marea teorema Jordan-Zassenhaus, care asigura ca n GLn (Z) exista un numar finit
de clase de conjugare ale subgrupurilor finite, rezultat mult mai puternic si mai greu
de demonstrat dect ceea ce am numit noi "versiunea slab
a" a teoremei JordanZassenhaus. De asemenea, pentru n 3, studiul subgrupurilor lui GLn (Z) devine
foarte laborios si complicat; de exemplu, in [2] se demonstreaza ca exista 73 de clase
de conjugare ale subgrupurilor finite din GL3 (Z). Chiar studiul claselor de conjugare
ale subgrupurilor finite din GL2 (Z) este dificil (de altfel, in finalul articolului vom
discuta problema conjugarii subgrupurilor ciclice ale lui GL2 (Z)).
Teorema 5. Exista exact noua clase de izomorfism ale subgrupurilor finite ale
lui GL2 (Z).
Demonstratie. Desigur, aici consider
am si subgrupurile triviale. Deja stim (din
teorema 2) ca ordinul oricarui subgrup finit al lui GL2 (Z) divide pe 24; prin urmare,
ordinul unui subgrup finit al lui GL2 (Z) poate fi doar unul dintre numerele: 1, 2, 3,
4, 6, 8, 12 sau 24.
S
a observ
am c
a sapte clase de izomorfism se g
asesc destul de repede. ntr-adev
ar,
subgrupuri cu un element
sau doua se gasesc fara probleme, cu trei elemente putem
0 1
lua subgrupul generat de
, pentru patru elemente putem alege subgrupul
1 1

0 1
generat de
(izomorf cu Z4 ) si subgrupul format din matricile I2 , I2 ,
1 0

1 0
1 0
si
(izomorf cu Z2 Z2 ), iar pentru sase elemente putem lua
0 1
0 1

0 1
subgrupul generat de
(izomorf cu Z6 ) si subgrupul generat de matricile


1 1
0
1
0 1
si
(izomorf cu grupul S3 al permutarilor de grad 3). Mai mult,
1 1
1 0
teoremele de structur
a ale grupurilor cu cel mult sase elemente arat
a c
a acestea sunt
singurele posibilit
ati pentru subgrupurile lui GL2 (Z) cu cel mult 6 elemente. Ne mai
ramne sa dovedim ca exista cte o singura clasa de izomorfism pentru subgrupuri
din GL2 (Z) cu 8 elemente si tot una pentru cele cu 12 elemente.
Un subgrup abelian cu opt elemente al lui GL2 (Z) (ca, de altfel, orice subgrup
abelian cu opt elemente) ar trebui s
a fie izomorf cu Z8 , cu Z2 Z4 , sau cu Z2 Z2 Z2 .
1

Student, cole Normale Suprieure, Paris

81

Cum orice element din GL2 (Z) are ordinul 2, 3, 4, sau 6 (nu si 8), nu exist
a subgrup
al lui GL2 (Z) izomorf cu Z8 .
Acum sa facem cteva observatii generale despre matricile de ordin 2, 3 sau 4
din GL2 (Z). n primul rnd se poate stabili (chiar cu mijloace elementare, f
ar
a a
folosi adic
a notiuni ca polinom caracterisitic,
valori
proprii,
etc)
c
a
orice
matrice
de

x y
ordin 2 este fie I2 , fie are forma
, cu x, y, z numere ntregi astfel nct
z x
x2 + yz = 1. O matrice A de ordinul patru trebuie sa verifice polinomul X 4 1,
deci (fiind cu elemente numere ntregi) trebuie s
a aib
a valorileproprii 1 sau i;
x y
2
2
cum A 6= I2 , ramne a doua varianta, deci A = I2 si A =
, cu x, y, z
z x
numere ntregi astfel nct x2 + yz = 1. n sfrsit, argumente asem
an
atoare conduc
la concluzia c
a, dac
a A are ordinul 3, atunci valorile sale proprii sunt si 2 ( fiind
o rad
acina cubicadiferita de 1 a unitatii), deci A are urma 1 si determinantul 1 si
x
y
A=
, cu x, y, z numere ntregi astfel nct x2 + x + 1 + yz = 0.
z 1 x
Acum s
a presupunem c
a exist
a n GL2 (Z) un subgrup izomorf cu Z2 Z2
Z2 si sa privim elementele lui ca matrici complexe. Fiind de ordinul al doilea, ele
sunt diagonalizabile si, deoarece ele comut
a dou
a cte dou
a, exist
a o baz
a comun
a
de diagonalizare. n acea baz
a, matricile din G sunt diagonale si au pe diagonala
principal
a valorile lor proprii care sunt 1 (c
aci toate, cu exceptia identit
atii, au
ordinul 2). Astfel s-ar obtine existenta a opt matrici de ordinul al doilea cu 1
pe diagonala principal
a si 0 n rest, evident absurd (acest argument functioneaz
a n
general: ordinul maxim al unui subgrup al lui GLn () care are toate elementele de
ordinul 2 desigur, cu exceptia elementului neutru este 2n ; ceea ce furnizeaz
a si
o demonstratie eleganta a faptului ca, pentru m 6= n, GLm (Z) nu este izomorf cu
GLn (Z), o problem
a greu de rezolvat altfel).
Existenta unui subgrup
al
lui GL2 (Z) izomorf cu Z2 Z4 ar

implica
si existenta
a b
x y
a dou
a matrici A =
, de ordinul al doilea si B =
, de ordinul
c a
z x
4, cu a, b, c, x, y, z numere ntregi, a2 + bc = 1 si x2 + yz = 1, care comuta:
AB = BA. Conditia aceasta (de comutativitate) ne da sistemul bz = cy, ay = bx,
az = cx. Presupunerea c
a un element, oricare, al celor dou
a matrici este nul conduce
la o contradictie (de exemplu, a = 0 implic
a bc = 1 si yz = 1 si acestea contrazic
bz = cy). Daca sunt nenule, a si b sunt prime ntre ele, la fel x si y; de aceea, din
ay = bx rezulta a = x si b = y, apoi obtinem si c = z (semnele corespund),
deci A = B, evident o contradictie. De altfel, se constat
a usor c
a, tot pe aceast
a
cale, se poate obtine si afirmatia demonstrat
a mai sus: orice subgrup al lui GL2 (Z)
format numai din elemente de ordin 2 (cu exceptia elementului neutru) are cel mult
patru elemente; desigur, n cazul general nu se poate proceda asa.
Un subgrup necomutativ cu opt elemente este izomorf fie cu grupul diedral, fie cu
cel al cuaternionilor. Un subgrup al lui
izomorf cu grupul
GL2 (Z)
cuopt elemente,

0 1
0 1
si
diedral, este cel generat de matricile
(omitem de fiecare dat
a
1 0
1 0
verific
arile, acestea fiind imediate). Ne mai r
amne s
a demonstr
am c
a GL2 (Z) nu
82

are subgrupuri izomorfe cu grupul cuaternionilor.


Presupunem ca exista un asemenea subgrup; asta ar nsemna, de fapt, ca exista
matricile A, B GL2 (Z), astfel nct AB = B 3 A, A2 = B 2 , B 4 = I2 si ordinul
lui B este 4; cum am v
azut, asta nseamn
a c
a A2 = B 2 = I2 , ceea ce duce si
la AB = BA. Rezult
a existenta unor numere ntregi
nct
a,b, c,
x, y, z astfel

a
b
x
y
2
2
si B =
a + bc + 1 = x + yz + 1 = 0 si astfel nct A =
.
c a
z x
Egalitatea AB = BA implica si 2ax + bz + cy = 0; eliminnd c, z din primele doua
relatii si folosind-o pe a treia, rezult
a (bx ay)2 + b2 + y 2 = 0, deci b = y = 0, ceea
ce, evident, conduce la contradictie.
S
a ne ndrept
am acum atentia asupra subgrupurilor cu 12 elemente ale lui GL2 (Z);
consideram mai nti un asemenea subgrup G neabelian. Sa presupunem ca printre
numerele x2 , . . . , xq nu apare si 1; cum (2 (1))(2 0) = 6 si cum 12 | (2
x2 ) (2 xq ), trebuie s
a avem xq = 2. Deci exist
a a, b , cu a + b = 10 astfel nct
12|2k + a 0k + b (1)k + (2)k , oricare ar fi k num
ar natural. Este clar c
a trebuie
sa avem atunci b = 0, iar alegerea k = 2 conduce iarasi la o contradictie. Astfel ca
trebuie s
a existe o matrice A G cu urma egal
a cu 1.
Fie u, v valorile proprii ale matricii A (care verific
a u + v = 1). Cum A12 = I2 ,
1
1
1
u si v au modulul 1, deci
= + = u + v = u + v = 1. Din u + v = uv = 1
uv
u
v
rezult
a u3 = v 3 = 1, deci A3 = I2 , adic
a A are ordinul 6. S
a mai consider
am
B G {Ak | 0 k 5} si sa observam ca vom avea atunci G = {Ak | 0 k
5} {BAk | 0 k 5} si ca AB = BA5 . ntr-adevar, este clar (din modul n care
l-am ales pe B) c
a exist
a k 0 astfel nct AB = BAk , ceea ce nseamn
a c
a matricile
k
1
A = B AB si A sunt similare, deci au aceleasi valori proprii. Atunci multimile
{uk , u1k } si {u, u1 } coincid, ceea ce duce imediat la posibilitatile k 1 (mod 6) sau
k 5 (mod 6). Cum G este neabelian ramne k 5 (mod 6) si AB = BA5 . Aceasta
se mai scrie (tinnd cont de A2 A + I2 = 02 , adic
a de teorema Cayley-Hamilton)
AB + BA = B, iar de aici obtinem tr(B) = 2 tr(AB). Cum matricile de ordin 3 sau
6 au urma impara, B trebuie sa aiba ordinul 2 sau 4. Sa presupunem ca B ar avea
ordinul 4; ca mai sus va rezulta ca B 2 = I2 . Un scurt moment de reflectie arata
2
2
existenta unor numere
ntregi

a, b,c, x, y, z astfel nct a + bc + 1 = x + yz + 1 = 0


x
y
a b
si A =
,B=
. Atunci, din AB + BA = B, dupa un mic calcul
z 1x
c a
y 2
ay
obtinem relatia u2 u + 1 +
= 0 (u =
x), egalitate evident imposibil
a.
b
b
Astfel rezulta ca B are ordinul 2, deci singurul subgrup necomutativ de ordin 12 din
i nici nu e greu s
a ar
at
am c
aexist
GL2 (Z) ar putea fi cel diedral. S

a n GL
2 (Z) un
1 0
1 1
si
subgrup izomorf cu D6 : este cel generat de matricile:
.
1 1
1 0
n fine, s
a presupunem c
a G ar fi un subgrup abelian cu 12 elemente al lui GL2 (Z).
Dup
a cum am amintit (v. mai sus problema de olimpiad
a), ordinul oric
arei matrici
din GL2 (Z) poate fi doar 1, 2, 3, 4 sau 6; prin urmare G ar putea fi izomorf (din cele
doua tipuri de grupuri comutative, adica Z12 si Z2 Z2 Z3 ) doar cu Z2 Z2 Z3
(deoarece Z12 este generat de un element de ordin 12).
83

S
a zicem c
a ar fi n GLn (Z) un subgrup izomorf cu Z2 Z2 Z3 ; acesta contine
mai mult de un element de ordin 2 (prin urmare si o matrice de ordin 2
diferita
de
a b
I2 ) , precum si elemente de ordin trei, deci atunci ar exista matricile A =
,
c a

x
y
de ordin 2 (a, b, c Z, a2 + bc = 1) si B =
de ordin 3 (x, y, z Z,
z 1 x
x2 +x+1+yz = 0) astfel nct AB = BA. Obtinem relatiile bz = cy, 2ay = b(2x+1)
si 2az = c(2x + 1); vedem imediat c
a de aici s-ar obtine, dac
a am presupune a = 0,
b(2x + 1)
si
c
a b = c = 0, imposibil. De aceea a 6= 0 si putem exprima y =
2a
c(2x + 1)
; nlocuim n x2 + x + 1 + yz = 0, mai tinem seama si de bc = 1 a2
c=
2a
si ajungem iar la o evident
a contradictie: 3a2 + (2x + 1)2 = 0; deci GL2 (Z) nu are
subgrupuri comutative cu 12 elemente.
Ne-a mai r
amas s
a demonstr
am c
a nu exist
a subgrupuri cu 24 de elemente n
GL2 (Z). Din pacate, orict ne-am straduit, nu am reusit sa gasim o astfel de demonstratie care s
a utilizeze ideile de mai sus. Se poate ns
a ar
ata acest fapt, trecnd la
un nivel superior al edificiului matematic. Avem, mai nti
Teorema 6. Orice subgrup finit al lui GLn (Z) este conjugat cu un subgrup al lui
On (R).
Demonstratie. Prin On (R) ntelegem grupul matricilor ortogonale din GLn (Z).
G fiind subgrupul finit despre care este vorba n enunt, vom defini un nou produs
scalar pe Rn (unde elementele se considera ca vectori coloana) prin
X
hgx, gyi,
hx, yi =
gG

unde h, i este produsul scalar obisnuit din Cn . Deoarece fiecare matrice din G este
inversabila, se verifica usor faptul ca si h, i este un produs scalar, precum si ca
hgx, gyi = hx, yi ,

x, y Cn ,

adica orice element din G este izometrie fata de acest nou produs scalar. Prin urmare, matricile din G sunt matrici ortogonale ntr-o baza ortonormala relativ la acest
produs, ceea ce reprezint
a (reformulat
a) concluzia lemei. Acum putem demonstra
Teorema 7. Orice subgrup finit al lui GL2 (Z) este fie ciclic, fie diedral.
Demonstratie. Fie, iar, G un subgrup finit al lui GL2 (Z), pe care, conform
teoremei anterioare l putem considera direct subgrup al lui O2 (R), si fie H intersectia
lui G cu SO2 (R), adic
a cu grupul (matricilor) rotatiilor lui R2 (matrici ortogonale
cu determinantul 1). Indicele lui H n G este cel mult egal cu 2 (deoarece, pentru
orice x, y G, xy 1 este o matrice ortogonala care are determinantul 1 sau 1). n
plus, H este un grup ciclic, deoarece este un grup finit de rotatii si se arat
a usor c
a
orice asemenea grup este generat de o rotatie a sa de unghi minim (tot asa cum se
arata ca orice subgrup al lui Z este generat de un element al sau de modul minim).
Acum, daca intersectia H are indicele 1 n G, evident vom avea G = H, deci G
este ciclic. Dac
a indicele este 2, consider
am un element s G H, care trebuie s
a fie
o simetrie fata de o dreapt
a ce trece prin origine. Se verific
a atunci cu usurinta (fie
prin calcul, fie recurgnd la interpretarea geometrica pe un desen) ca srs = r1 ,
84

r fiind generatorul lui H. Cum s2 este identitatea si G este generat de s si de r


(indicele lui H n G fiind 2) obtinem imediat ca, n acest caz, G este diedral.
n fine, Teorema 5 rezulta din nou, sub forma:
Teorema 8. Orice subgrup finit al lui GL2 (Z) are cel mult 12 elemente si exista
doar o clasa de izomorfism a subgrupurilor lui GL2 (Z) cu 12 elemente.
Demonstratie. Totul rezulta din teorema anterioara si vesnica observatie ca
matricile din GL2 (Z) au ordin 1, 2, 3, 4 sau 6. Evident, teorema 7 implic
a si faptul
c
a exist
a doar 9 tipuri de clase de izomorfism pentru subgrupurile lui GL2 (Z), dar
noi ne-am straduit mai sus (si nu am reusit n totalitate) sa aratam ca se poate obtine
acelasi rezultat si pe cale "elementara".
Terminam acest articol tinndu-ne o promisiune: stabilirea claselor de conjugare
ale matricilor de ordin 3, 4, 6 din GL2 (Z). Nu vom trata cazul matricilor de ordin 2,
din simplul motiv c
a necesit
a o cu totul alt
a metod
a. Vom folosi f
ar
a demonstratie
un rezultat clasic de geometria numerelor, anume faimoasa teorem
a a lui Minkowski:
orice multime convexa, simetrica n raport cu originea si de arie strict mai mare dect
4 din R2 contine macar un punct laticial nenul. Vom ncepe cu urmatoarea aplicatie
direct
a a teoremei lui Minkowski, care se va dovedi crucial
a n studiul claselor de
conjugare ale matricilor de ordin finit:
Lema
a) Daca a,b, c sunt numere ntregi astfel nct bc = a2 + 1, atunci
1.
2

ecuatia cx 2axy + by 2 = 1 are solutii n numere ntregi.

b) Daca bc = a2 + a + 1, atunci ecuatia cx2 (2a + 1)xy + by 2 = 1 are solutii


ntregi.
Demonstratia este usoar
a dac
a folosim teorema lui Minkowski si foarte grea
altfel. Ideea este urmatoarea (vom rezolva doar a), punctul b) fiind
absolut identic):

dac
a consider
am A multimea punctelor (x, y) din plan pentru care cx2 2axy+by 2 <
2, un calcul imediat arata ca A are aria strict mai mare decat 4. ntr-adevar, putem
n mod evident presupune b, c > 0 si atunci conditia se scrie z 2 + t2 < 2, unde

a
y
z = x c y si t = . Deci A este imaginea cercului de arie 2 prin aplicatia
c
c
z + at
a ariile, c
aci matricea asoliniara (z, t) ( , ct). Or, aceasta aplicatie conserv
c
ciat
a are determinantul 1. Deci A are aria 2 > 4 si totul rezult
a acum din teorema
lui Minkowki: A contine un punct laticial nenul (x, y) si pentru
acest punct, mereu

n ipoteza b, c > 0, avem n mod evident cx2 2axy + by 2 = 1. Cum am spus, b)


urmeaz
a exact aceeasi cale de demonstratie, deci r
amne n seama cititorului.
Acum putem ncepe sa studiem conjugarea matricilor de ordin 3, 4, 6. Mai precis,
vom demonstra urmatoarea:

Teorema 9. Exista exact o clasa de conjugare in GL2 (Z) pentru matricile de


ordin 3 din GL2 (Z). Aceeasi concluzie este valabila pentru matricile de ordin 4 si
pentru cele de ordin 6.
Demonstratie. Dac
a A GL2 (Z) este de ordin 3, respectiv 6, am v
azut n
timpul studiului tipului de izomorfism al subgrupurilor finite c
a A2 + A + I2 = O2
si A2 A + I2 = O2 , respectiv. Sa consideram o matrice A de ordinul 3. Am vazut
85

a
b
unde a, b, c Z verifica a2 + a + 1 = bc.
c 1 a
Aplicnd lema, rezulta ca exista un vector nenul e = (x, y) astfel nct |det(e, Ae)| = 1
(aici (e, Ae) este matricea care are prima coloan
a egal
a cu e si pe cea de-a doua cu
un calcul imediat arat
a c
a de fapt conditia |det(e, Ae)| = 1 este echivalent
a cu
Ae):
cx2 (2a + 1)xy + by 2 = 1. Aceasta nseamna ca (e, Ae) este o baza a lui Z2 , n
2
sensul ca matricea (e, Ae) este n GL2 (Z).
Or, A(Ae) = A e = Ae e, deci n
0 1
aceasta baza matricea lui A este exact
. Am aratat astfel ca elementele
1 1
de ordin 3 sunt toate conjugate ntre ele. Cu exact aceleasi argumente se arat
a c
a
elementele de ordinul 4, respectiv 6 au aceeasi proprietate.
Cititorul ar putea sa se ntrebe n acest moment: de ce am inclus ultimul rezultat
n acest articol? Raspunsul este simplu: cum n tot articolul am oscilat ntre algebra
si teoria numerelor si cum am nceput cu algebra, prefer
am s
a termin
am cu teoria
numerelor. S
i, ntr-adevar, din ultima teorem
a obtinem cteva rezultate foarte frumoase din acest domeniu. Sa luam, de exemplu, iarasi cazul
matricilor
de ordin 3.
a
b
Am v
azut ca pentru o astfel de matrice, scris
a sub forma
, putem g
asi
c 1
a

0 1
x y
P GL2 (Z) astfel nct A = P 1
P . Scriind P =
, un scurt calcul
1 1
z t
arat
a c
a exist
a = det P {1, 1} astfel nct
ca A se poate scrie sub forma

a = (yz xy zt), b = (y 2 yt + t2 ) si
c = (z 2 xz + x2 ), d = (zt xt + xy),

unde xt yz = . Aceasta este deci solutia general


a a ecuatiei a2 + a + 1 = bc
n numere ntregi. ncercati sa demonstrati aceasta prin alte mijloace si veti vedea
avantajele acestei metode. Sa mai subliniem un rezultat, deloc banal, care se poate
obtine de aici. S
a presupunem c
a p este un num
ar prim de forma 3k + 1 si s
a lu
am
p1
u o r
ad
acin
a primitiv
a modulo p. Notnd x = u 3 , obtinem imediat c
a x 6= 1 si
x3 = 1, deci x2 + x + 1 = 0 (lucr
am in Z/pZ). Aceasta arat
a existenta unui num
ar
ntreg a pentru care p | a2 + a + 1. Din cele observate anterior, rezulta ca p se poate
scrie sub forma x2 xy + y 2 pentru niste numere ntregi x si y. Am ar
atat astfel c
a
orice num
ar prim de forma 3k + 1 poate fi exprimat ca x2 xy + y 2 (cu x, y ntregi).
Evident, lucrnd cu matricile de ordin 4, ajungem cu aceleasi argumente la frumoasa
teorema a lui Fermat: orice numar prim de forma 4k + 1 se scrie ca suma a doua
p
atrate de numere ntregi.
ncheiem aici scurta noastr
a vizit
a n lumea subgrupurilor lui GL2 (Z), nu nainte
de a sugera cititorului temerar un studiu al claselor de izomorfism ale subgrupurilor
finite ale lui GL3 (Z).
Bibliografie
1. G. Dospinescu - Cteva proprietati ale subgrupurilor finite din GLn (Z), Recreatii
Matematice 1/2006.
2. Ken-Ichi Tahara - On the finite subgroups of GL3 (Z), Nagoya Math. Journal.
86

O problem
a cu cifrele unui num
ar
Titu ZVONARU 1
Problema G.116, propus
a de Maria Mihet n RecMat - 1/2007, are urm
atorul
enunt:
Aflati toate numerele naturale N de patru cifre nenule distincte cu proprietatea
ca diferenta dintre cel mai mare numar obtinut prin permutarea cifrelor lui N si cel
mai mic asemenea numar este tocmai N .
Ne propunem sa rezolvam urmatoarea problema mai generala:
Aflati toate numerele naturale N cu cifre dictincte cu proprietatea ca diferenta
dintre cel mai mare numar obtinut prin permutarea cifrelor lui N si cel mai mic
asemenea numar este tocmai N .
Fie n num
arul cifrelor lui N . Avem de analizat urm
atoarele cazuri:
A. n = 2. Fie a > b cifrele lui N . Cel mai mare numar scris cu cifrele lui N este
ab, iar cel mai mic este ba.
Avem posibilit
atile:
i) ab ba = ab ba = 0,
ii) ab ba = ba 8a = 19b
si nu obtinem solutii.
B. n = 3. Fie a > b > c cifrele lui N . Avem abccba = xyz, cu {a, b, c} = {x, y, z}
si obtinem z = 10 + c a, y = 9 + b b = 9, x = a 1 c.
Cum a este cea mai mare cifra, rezulta a = 9 si {b, c} = {8 c, c + 1}, adica
8 c = c, b = c + 1. Obtinem solutia 954 459 = 495.
C. n = 4. Fie a > b > c > d cifrele lui N . Avem ca abcd dcba = xyzt si notam
C = {a, b, c, d} = {x, y, z, t}. Obtinem:
t = 10 + d a, z = 9 + c b, y = b 1 c, x = a d.
Observ
am c
a y + z = 8, x + t = 10 si deducem c
a
{y, z} {{0, 8} , {1, 7} , {2, 6} , {3, 5}} , {x, t} {{1, 9} , {2, 8} , {3, 7} , {4, 6}} .
Avem de analizat urmatoarele posibilitati:
{y, z} {x, t} (a, b, c, d)
{0, 8} {1, 9} (9, 8, 1, 0) b 1 c = 6
/C
{0, 8} {3, 7} (8, 7, 3, 0) 9 + c b = 5
/C
{0, 8} {4, 6} (8, 6, 4, 0) b 1 c = 1
/C
{1, 7} {2, 8} (8, 7, 2, 1) b 1 c = 4
/C
{1, 7} {4, 6} (7, 6, 4, 1)
{2, 6} {1, 9} (9, 6, 2, 1) b 1 c = 3
/C
{2, 6} {3, 7} (7, 6, 3, 2) a d = 5
/C
{3, 5} {1, 9} (9, 5, 3, 1) a d = 8
/C
{3, 5} {2, 8} (8, 5, 3, 2) a d = 6
/C
{3, 5} {4, 6} (6, 5, 4, 3) b 1 c = 0
/ C.
1

Com
anesti, e-mail: tzvonaru@hotmail.com

87

Obtinem o singur
a solutie: 7641 1467 = 6174.
D. n = 5. Fie a > b > c > d > e cifrele lui N . Avem c
a abcde edcba = xyztu
si notam C = {a, b, c, d, e} = {x, y, z, t, u}. Obtinem u = 10 + e a, t = 9 + d b,
z = 9, y = b 1 d, x = a e. Rezult
a c
a a = 9 si e 6= 0 (dac
a e = 0 ar urma x = 9,
deci x = z). Observ
am c
a y + t = 8, x + u = 10 si, cum 0
/ C, deducem c
a:
adic
a

{y, t} {{1, 7} , {2, 6} , {3, 5}} ,

{y, t}
{1, 7}
{1, 7}
{2, 6}
{3, 5}
{3, 5}
Nu obtinem solutii.

{x, u}
{2, 8}
{4, 6}
{3, 7}
{2, 8}
{4, 6}

{x, u} {{2, 8} , {3, 7} , {4, 6}} ,

(a, b, c, d, e)
(9, 8, 7, 2, 1)
(9, 7, 6, 4, 1)
(9, 7, 6, 3, 2)
(9, 8, 5, 3, 2)
(9, 6, 5, 4, 3)

b1d=4
/C
ae=8
/C
9+db=5
/C
ae=7
/C
b1d=2
/ C.

E. n = 6. Fie a > b > c > d > e > f cifrele lui N . Avem c


a
abcdef f edcba = xyztuv

si notam C = {a, b, c, d, e, f } = {x, y, z, t, u, v}. Obtinem


v = 10+f a,

u = 9+eb,

t = 9+dc,

z = c1d,

y = be,

Observam ca z + t = 8, y + u = 9, x + v = 10 si deducem ca
{z, t} {{0, 8} , {1, 7} , {2, 6} , {3, 5}}
{y, u} {{0, 9} , {1, 8} , {2, 7} , {3, 6} , {4, 5}}
{x, v} {{1, 9} , {2, 8} , {3, 7} , {4, 6}} .

Avem de analizat urm


atoarele posibilit
ati:
{z, t} {y, u} {x, v} (a, b, c, d, e, f )
{0, 8} {2, 7} {1, 9} (9, 8, 7, 2, 1, 0)
{0, 8} {2, 7} {4, 6} (8, 7, 6, 4, 2, 0)
{0, 8} {3, 6} {1, 9} (9, 8, 6, 3, 1, 0)
{0, 8} {4, 5} {1, 9} (9, 8, 5, 4, 1, 0)
{0, 8} {4, 5} {3, 7} (8, 7, 5, 4, 3, 0)
{1, 7} {0, 9} {2, 8} (9, 8, 7, 2, 1, 0)
{1, 7} {0, 9} {4, 6} (9, 7, 6, 4, 1, 0)
{1, 7} {3, 6} {2, 8} (8, 7, 6, 3, 2, 1)
{1, 7} {4, 5} {2, 8} (8, 7, 5, 4, 2, 1)
{2, 6} {0, 9} {3, 7} (9, 7, 6, 3, 2, 0)
{2, 6} {1, 8} {3, 7} (8, 7, 6, 3, 2, 1)
{2, 6} {4, 5} {1, 9} (9, 6, 5, 4, 2, 1)
{2, 6} {4, 5} {3, 7} (7, 6, 5, 4, 3, 2)
{3, 5} {0, 9} {2, 8} (9, 8, 5, 3, 2, 0)
{3, 5} {0, 9} {4, 6} (9, 6, 5, 4, 3, 0)
{3, 5} {1, 8} {4, 6} (8, 6, 5, 4, 3, 1)
{3, 5} {2, 7} {1, 9} (9, 7, 5, 3, 2, 1)
{3, 5} {2, 7} {4, 6} (7, 6, 5, 4, 3, 2)
88

c1d=4
/C
be=5
/C
be=7
/C
be=7
/C
10 + f a = 2
/C
cd1=4
/C
9+eb=3
/C
be=5
/C
c1d=0
/C
be=5
/C
be=5
/C
af =8
/C
c1d=0
/C
be=6
/C
9+dc=8
/C
af =7
/C
af =8
/C
c1d=0
/ C.

x = af.

Nu obtinem solutii.
E. n = 7. Fie a > b > c > d > e > f > g cifrele lui N . Avem
abcdef g gf edcba = zyxtuvw

si la fel ca n cazul n = 5 deducem ca a = 9 si g 6= 0.


Diferenta dintre un numar si rasturnatul sau este multiplu de 9; cum suma cifrelor
din baza 10 este de asemenea multiplu de 9, deducem c
a suma celor trei cifre nefolosite
la scrierea lui N trebuie s
a fie multiplu de 9.
Deoarece stim ca cifra 0 nu este folosita, ramnd de analizat doar urmatoarele
posibilitati:
cifre nefolosite
0,1,8
9765432 2345679 = 7419753
0,2,7
9865431 1345689 = 8519742
0,3,6
9875421 1245789 = 8629632
0,4,5
9876321 1236789 = 8639532
si nu obtinem solutii.
F. n = 8. Din acelasi motiv ca n cazul n = 7, avem doar posibilitatile:
cifre nefolosite
0,9
87654321 12345678 = 75308643
1,8
97654320 2345679 = 95308641
2,7
98654310 1345689 = 97308621
3,6
98754210 1245789 = 97508421
4,5
98763210 1236789 = 97527421
cu solutia 98763210 1236789 = 97527421.
G. n = 9. Deoarece n este impar, rezult
a c
a singura cifr
a nefolosit
a la scrierea
lui N este 0. Obtinem solutia 987654321 123456789 = 864197532.
H. n = 10. Obtinem solutia 9876543210 123456789 = 9753086421.

ERATA
n num
arul 1/2007 al revistei Recreatii Matematice s-au strecurat urm
atoarele
greseli:
1. La pag. 2, r. 1 n loc de 1866 se va citi 1766.
2. n enuntul problemei XII.76 se va considera c
a functia f : [a, b] R este
continua, conditie care a fost omisa.

89

O problem
a de constructie a unui triunghi
Temistocle BRSAN 1
Problemele de constructie cu rigla si compasul au un specific si un farmec aparte.
Pe aceasta linie, mentionam remarcabila monografie Probleme de constructii geometrice cu rigla si compasul a lui Gh. Buicliu.
Problema de care ne vom ocupa este de tipul urm
ator:
Se porneste de la o figura F (un triunghi, de exemplu) si prin diverse constructii
auxiliare se obtine o configuratie F 0 . Retinnd cteva elemente ale lui F 0 (adica
stergnd o buna parte din configuratia F 0 ), cum putem reconstrui cu rigla si compasul figura initiala F.
Vom da cteva exemple cunoscute de acest fel:
1. S
a se construiasc
a un triunghi ABC cunoscndu-i centrele cercurilor circumscrise triunghiurilor HBC, HCA si HAB, unde H este ortocentrul acestuia ([1],
Problema 624 ).
2. Sa se construiasca un triunghi cunoscnd punctele de intersectie a bisectoarelor
interioare cu cercul circumscris triunghiul ([4], p. 105, Problema 88 ).
3. S
a se construiasc
a triunghiul ABC cunoscnd pozitiile vrfurilor A0 , B 0 , C 0 ale
triunghiurilor echilaterale construite pe laturile lui si n exterior ([1], Problema 637 ).
4. S
a se construiasc
a triunghiul ABC cunoscnd centrele , , ale cercurilor
adjuncte (CA), (AB), (BC) sau centrele 0 , 0 , 0 ale cercurilor adjuncte (BA),
(CB), (AC) (se numeste cerc adjunct (CA) cercul care trece prin B si este tangent
n C laturii CA) ([1], Problema 640 ).
5. S
a se construiasc
a un triunghi cunoscndu-i punctele O, H, I (cu semnificatii
uzuale); n ce conditii exist
a un astfel de triunghi? ([3])
S
a ncepem prin a preciza notatiile (de altfel uzuale, v. [2]) utilizate mai jos
(fig. 1):
I, Ia , Ib , Ic centrele cercurilor nscris si exnscrise triunghiului;
H, Ha , Hb , Hc ortocentrul si proiectiile lui pe laturile BC, CA si respectiv
AB;
D, E, F punctele de contact ale cercului nscris I (I, r) I cu laturile BC,
CA si respectiv AB;
Da , Ea , Fa etc. punctele de contact ale cercului exnscris Ia (Ia , ra ) Ia etc.
cu dreptele suport ale acelorasi laturi;
A0 , B 0 , C 0 punctele date de {A0 } = Db Fb Dc Ec , {B 0 } = Ec Dc Ea Fa ,
0
{C } = Fa Ea Fb Db .
Ne propunem s
a rezolv
am urm
atoarea

Problem
a. Sa se construiasca cu rigla si compasul triunghiul ABC cunoscnd
pozitiile punctelor A0 , B 0 , C 0 .
1

Prof. dr., Univ. Tehnic


a "Gh. Asachi", Iasi

90

Vom face analiza problemei, adic


a pentru configuratia construit
a, format
a din
4ABC dat, 4Ia Ib Ic al centrelor cercurilor exnscrise si 4A0 B 0 C 0 al punctelor de
contact exterioare (Fa , Ea ; Db , Fb si Ec , Dc ) vom indica un numar de proprietati.
Ca urmare, constructia triunghiului A0 B 0 C 0 va decurge cu usurinta.

A
Ec
Fb

Ic

A
Ib

Eb

Fc
I
Dc

B
Da

Ha

Db

B
Fa
Ea
Ia

Fig. 1

Propozitia 1. Triunghiul A0 B 0 C 0 are proprietatile:

c0 ) = 90
1) m(A

A
,
2

2) B 0 C 0 si Db Dc sunt antiparalele, 3) B 0 C 0 k Ib Ic , ca si analoagele acestora.


Demonstratie. 1) Deoarece BDb si BFb sunt tangente la cercul Ib , rezulta ca
B

\
4BDb Fb este isocel si, deci, m(BD
. La fel, considernd 4CDc Ec
b Fb ) = 90
2
C

\
isoscel, deducem ca m(CD
. Ca urmare, n 4A0 Db Dc avem ca
c Ec ) = 90
2
c0 ) = 90 A . Analog, se stabileste c
c0 ) = 90 B si m(C
c0 ) = 90 C .
m(A
a m(B
2
2
2
91

0 D D ), dreptele B 0 C 0
c0 ) = 90 B = m(A\
2) ntruct m(B
si Db Dc sunt antiparb c
2
0 0 0
alele n raport cu 4A B C .
3) Consideram dreapta d determinata de punctele B si Ib ca o secanta a dreptelor
B 0 C 0 si Ib Ic si aratam ca se formeaza unghiuri alterne interne egale.
A
si analoagele (m(Iba ) = 180
Se stie c
a n 4Ia Ib Ic avem m(Iba ) = 90
2
[ = B + C = 90 A ). Atunci, din faptul ca d Ia Ic deducem ca
m(BIC)
2
2
2

C
C

b
\
a
m(d, Ib Ic ) = 90 m(Ic ) = 90 90
= , iar din d A0 C 0 avem c
2
2
c0 ) = C .
m(d,\
B 0 C 0 ) = 90 m(C
2

aci au laturile paralele dou


a
Observatie. 4A0 B 0 C 0 si 4Ia Ib Ic sunt omotetice, c
cte dou
a. Centrul lor de omotetie este punctul de concurenta a dreptelor A0 Ia , B 0 Ib ,
C 0 Ic , pe care-l notam cu K 0 .
Propozitia 2. Dreptele AA0 , BB 0 , CC 0 sunt concurente n punctul H.
Demonstratie. Fie A punctul n care AA0 intersecteaza dreapta BC. Pentru
a arata ca AA0 trece prin H, vom demonstra ca A coincide cu Ha .
Conform teoremei lui Menelaus aplicat
a 4BDb Fb si transversalei AA0 , avem relatia
AB A0 Fb
A B
=

.
A Db
AFb A0 Db
Cum AB = c si AFb = BFb AB = pc (p fiind semiperimetrul 4ABC), r
amne s
a
calcul
am A0 Db si A0 Fb . S
a mai observ
am c
a Db Dc = BDb + CDc BC = p + p a =
b + c. Acum, cu teorema sinusurilor n 4A0 Db Dc si tinnd cont de punctele 1) si 2)
ale Propozitiei 1, obtinem relatia
cos C2
Db Dc
A0 Db
0
=

A
D
=
(b
+
c)
,
b
cos C2
cos A2
cos A2
cos C2
B
2p sin
(din 4BDb Fb ).
2
cos A2
Ca urmare, relatia de mai sus se scrie

iar A0 Fb = A0 Db Db Fb = (b + c)

A B
c
=

A Db
pc
A
Utiliznd formele de tipul cos =
2

cos C2
B
2p sin
A
2
cos 2
.
C
cos 2
(b + c)
cos A2

(b + c)

p (p a)
, dupa calcule de rutina rezulta ca
bc

a2 b2 + c2
A B
=
,
A Db
(b + c) (a + b c)
92

de unde
a2 b2 + c2
A B
a2 b2 + c2
A B
=

=
A B + A Db
(b + c) (a + b c) + a2 b2 + c2
BDb
a 2p
A B
a2 b2 + c2

=
A B = c cos B,
p
2 2p
adic
a A este piciorul n
altimii AHa . Deci AA0 trece prin H. Analog se arat
a c
a si
0
0
BB , CC trec prin H, ceea ce ncheie demonstratia.
Observatie. 4A0 B 0 C 0 si 4ABC sunt omologice, H fiind centru de omologie. Se
remarc
a usor c
a ele sunt si ortologice.
Propozitia 3. Punctul H este centrul cercului circumscris 4A0 B 0 C 0 , iar raza
R a acestui cerc este data de R0 = HA + ra = HB + rb = HC + rc .
0 C 0 ) = m(HB
0E ) =
\
\
Demonstratie. Deoarece B 0 H AEa , avem c
a m(HB
a
A

0
0
\
\
90 m(AEa Fa ) = . Din CH AFa si procednd la fel, deducem c
a m(HC B 0 ) =
2
A
a HB 0 = HC 0 . n mod analog, se arat
a c
a
. Deci 4HB 0 C 0 este isoscel, adic
2 0
0
0 0 0
HB = HA . Rezulta ca H este centrul cercului circumscris 4A B C .
Pentru partea a doua din enunt s
a ar
at
am c
a AA0 = ra . ntr-adev
ar, n 4A0 AFb
B
0F ) =
0 F A) =
\
\
avem: AFb = p c, m(AA
(analoaga uneia de mai sus) si m(A
b
b
2
B
90 +
(unghi exterior 4BDb Fa isoscel). Cu teorema sinusurilor, obtinem ca
2
r
r
B pc
S
p (p b)
ac
0
AA = cos
=
(p

c)
=
= ra .
2 sin B2
ac
(p a) (p c)
pa
Atunci, R0 = A0 H = HA + AA0 = HA + ra etc.
0

Consecinta
. Dreptele A0 Da , B 0 Eb , C 0 Fc sunt concurente n ortocentrul H 0 al
0 0 0
4A B C .
Demonstratie. Segmentele [AA0 ] si [Ia Da ] au lungimi egale cu ra si sunt paralele
(ca fiind perpendiculare pe BC). Prin urmare, patrulaterul A0 AIa Da este paralelogram si, deci, A0 Da k AIa . Din aceasta si faptul ca AIa B 0 C 0 , rezulta ca A0 Da
B 0 C 0 , adic
a A0 Da este n
altimea n 4A0 B 0 C 0 . n acelasi fel, B 0 Eb , C 0 Fc sunt n
altimi
0 0 0
n 4A B C si demonstratia este complet
a.

Propozitia 4. Dreptele A0 Ia , B 0 Ib , C 0 Ic trec prin mijloacele laturilor [BC], [CA]


si repsectiv [AB] ale 4ABC.
Demonstratie. Vom dovedi numai afirmatia relativ
a la A0 Ia . Fie {U } = A0 Ia
a
BC. Revine la a ar
ata c
a BU = .
2
Din faptul ca 4Ia Da U 4A0 Ha U rezulta ca
U Da
ra
U Da
ra
a
cb
cb
=

=
U Da =
p
=
,
U Ha
ra + ha
Da Ha
2ra + ha
2p
a
2
B
cb
c
aci Da Ha = BHa BDa = a cos B (p c) = 2c cos2 p = p
, iar
2
a

.
.
1
a ha
pa
2S p a
a
ra
=
=
1
2
+

=
1
2
+

=
.
2ra + ha
(p a) ra
a
S
a
2p
2 + hr a
a

93

cb
a
= si demonstratia este ncheiata.
2
2
Consecinta
. A0 Ia , B 0 Ib , C 0 Ic sunt simediane att n 4A0 B 0 C 0 ct si n 4Ia Ib Ic .
Demonstratie. Mijlocul U al segmentului [BC] este si mijlocul lui [Db Dc ], c
aci

CDb = BDc = p a. Tinnd


cont de punctul 2) al Propozitiei 1, n 4A0 B 0 C 0
avem c
a A0 Ia trece prin mijlocul antiparalelei Db Dc la B 0 C 0 . Cum n orice triunghi
o simediana este locul mijloacelor antiparalelelor la latura opusa [2], p.55, deducem
c
a A0 Ia este simediana prin A0 a 4A0 B 0 C 0 . n sfrsit din omotetia observat
a mai sus
a triunghiurilor A0 B 0 C 0 si Ia Ib Ic , rezult
a c
a A0 Ia este simediana prin Ia a 4Ia Ib Ic
(faptul rezulta si observnd ca BC si Ib Ic sunt antiparalele n 4Ia Ib Ic ).
n sfrsit, BU = BDa + U Da = (p c) +

Revenim la problema de constructie propus


a la nceput. Rezultatele precedente
fac posibila constructia 4ABC, atunci cnd se da 4A0 B 0 C 0 . ntr-adevar, cu rigla si
compasul putem parcurge fiecare dintre pasii urmatori:
1) se construieste centrul H al cercului circumscris triunghiului A0 B 0 C 0 (care va
fi ortocentrul triunghiului ABC, conform Propozitiei 3, iar conform Propozitiei 2 va
trebui ca A, B, C s
a se afle pe A0 H, B 0 H, C 0 H, respectiv);
2) n 4A0 B 0 C 0 se construieste simediana corespunz
atoare vrfului A0 ;
3) se ia n mod arbitrar un punct C1 (HC 0 ) si prin el construim perpendiculara
pe A0 H care intersecteaza (B 0 H) ntr-un punct B1 ;
4) unim H cu mijlocul segmentului [C1 B1 ] pentru a avea locul mijloacelor segmentelor ce se sprijin
a pe HC 0 si HB 0 si sunt perpendiculare pe A0 H;
5) intersect
am acest loc geometric cu simediana prin A0 construit
a la pasul 2)
(conform Propozitiei 4 si Consecintei sale, acest punct va fi mijlocul laturii [BC]
cautate);
6) obtinem vrfurile C si B ale triunghiului de construit ca intersectii ale perpendicularei pe A0 H dusa prin punctul construit la 5) cu (HC 0 ) si respectiv (HB 0 );
7) se construieste vrful A ca intersectie cu (HA0 ) a perpendicularei prin B pe
C 0 H.
Bibliografie
1. Gh. Buicliu - Probleme de constructii geometrice cu rigla si compasul, Ed. Tehnica,
Bucuresti, 1957.
2. T. Lalescu - Geometria triunghiului, Ed. Tineretului, Bucuresti, 1958.
3. F. Lo Jacomo - Enonc 245, APMEP, Buletin no. 408, 1997, 57-79.
4. D. Smaranda, N. Soare - Transformari geometrice, Ed. Acad. R.S.R., Bucuresti,
1988.

94

Un sir strns legat de sirul lui Wallis


Adrian CORDUNEANU 1 , Gheorghe COSTOVICI 2
Scopul propus este studiul sirului (an )n1 definit prin
an = Cn0 Cn1

1!!
3!!
5!!
(2n 1)!!
n
+ Cn2
Cn3
+ + (1) Cnn
.
2!!
4!!
6!!
(2n)!!

(1)

Vom stabili c
a este monoton descresc
ator si convergent la zero si vom pune n evidenta
legatura acestuia cu sirul lui Wallis (wn )n1 dat de

2
1
(2n)!!
wn =
(2)
(2n 1)!! 2n + 1

(despre care stim c


a lim wn = ).
n
2
Lem
a ([1], p. 124 sau [2], p. 350). Pentru orice k N avem

Z /2
Z /2
(k 1)!! , k par
k
k
k!!
2
.
cos x dx =
sin x dx =

0
0
(k 1)!! ,
k impar
k!!
Demonstratie. Dac
a n una din cele dou
a integrale efectu
am schimbarea t =

a
a; deci integralele sunt egale. Fie Ik valoarea comun
x , vom obtine pe cealalt
2
lor. Integrnd prin parti, gasim:
Z /2
Z /2
cosk x dx =
(sin x)0 cosk1 x dx =
Ik =
0

= (k 1)
= (k 1)
de unde

/2

0
/2

sin x cosk2 x ( sin x) dx =

1 cos2 x cosk2 x dx = (k 1) (Ik2 Ik ) ,


Ik =

k1
Ik2 ,
k

Aceast
a relatie de recurenta si faptul c
a I0 =

k 2.

si I1 = 1 conduc la rezultatul dorit.


2

Propozitie. Are loc identitatea


1!!
3!!
(2n 1)!!
(2n 1)!!
= Cn0 Cn1
+ Cn2
+ + (1)n Cnn
.
(2n)!!
2!!
4!!
(2n)!!

(3)

a moduri. Mai nti,


Demonstratie. Obtinem (3) exprimnd integrala I2n n dou
1
2

Prof. dr., Catedra de matematic


a, Univ. "Gh. Asachi", Iasi
Conf. dr., Catedra de matematic
a, Univ. "Gh. Asachi", Iasi

95

conform Lemei, avem I2n =


I2n =

/2
2n

cos

x dx =

/2

(2n 1)!!
. Apoi, folosind formula binomului, g
asim:
(2n)!! 2
/2

1 sin2 x dx =

0
n
Cn Cn1 sin2 x + Cn2 sin4 x Cn3 sin6 x + + (1) Cnn sin2n x dx =

= Cn0 I0 Cn1 I2 + Cn2 I4 Cn3 I6 + + (1)n Cnn I2n =

n n (2n 1)!!
0
1 1!!
2 3!!
3 5!!
=
Cn Cn
+ Cn
Cn
+ + (1) Cn
.
2
2!!
4!!
6!!
(2n)!!

Egalnd aceste doua valori ale lui I2n , obtinem (3).


Corolarul 1. Sunt adevarate afirmatiile:
1) an = (2n 1)!! , n N ;
(2n)!!
2) (an )n1 este strict descrescator.

Demonstratie. Punctul 1) decurge din (1) si (3), iar 2) din faptul ca


2n + 1
< 1, n N .
2n + 2
Corolarul 2. Sunt adevarate afirmatiile urmatoare:
1
1
1) an =

, n N ;
wn
2n + 1
2) (an )n1 converge la zero.

(4)
an+1
=
an

(5)

Demonstratie. Punctul 1) rezulta din (2) si (4), iar 2) prin trecere la limita n
(5) pentru n .
Observatie. Faptul c
a lim an = 0 poate fi stabilit si utiliznd dubla inegalitate
n

2
3
(2n 1)!!
1
1

<

<
, n > 1 ([3], p. 48).
2
(2n)!!
2
2n
2n
Bibliografie.
1. G. M. Fihtenholt - Curs de calcul diferential si integral, vol. II, Ed. Tehnica,
Bucuresti, 1964.
2. Gh. Siretchi - Calcul diferential si integral, vol. I, Ed. S
tiintifica si Enciclopedica,
Bucuresti, 1985.
3. *** - Probleme de matematica traduse din Kvant, vol. I, E. D. P., Bucuresti, 1983.

96

Asupra r
ad
acinilor polinomului X 3 + pX + q Q [X]
Adrian REISNER1

I. Consider
am polinomul P = X 3 + pX + q Q [X], avnd r
ad
acinile complexe
1 , 2 , 3 . Not
am, ca de obicei, cu r
ad
acina primitiv
a de ordin 3 a unit
atii
3
2
2
1
= cos
+ i sin
).
( = + i
2
2
3
3
Propozitia 1. Pentru , C, urmatoarele afirmatii sunt echivalente:
(i) 3 = p si P ( + ) = 0;
p3
(ii) 3 si p3 sunt radacinile polinomului Q = X 2 + qX .
27
Demonstratie. Cum P ( + ) = 3 + 3 + (3 + p) ( + ) + q, avem c
a

3
p
(i) + 3 + q = 0, 3 = p 3 + 3 = q, 3 3 =
(ii) .
27
Consecinta 1. Daca , C satisfac (i) sau (ii), atunci radacinile polinomului
P sunt + , + 2 si 2 + .
Demonstratie. Dac
a si sunt r
ad
acinile cubice cu produs
real
2
ale celor
2

dou
a r
ad
acini ale polinomului Q, atunci perechile
,

s
i

au aceleasi
propriet
ati, deci are loc (ii). Deducem c
a P + 2 = P 2 + = 0, de unde
concluzia.
4p3
Observatie. Daca = q 2 +
este discriminantul polinomului Q atunci:
27
1) Daca > 0,
atunci Q admite
doua radacini reale si distincte, fie acestea
A

si
B. Notnd cu = 3 A, = 3 B r
ad
acinile cubice reale, atunci + = 3 A + 3 B
este unica r
ad
acin
a real
a a lui P .

2) Daca = 0, atunci Q admite radacina real


a dubla A. Notnd = = 3 A

R, radacinile polinomului P vor fi + = 2 3 A si + 2 = 2 + = 3 A


(r
ad
acin
a dubl
a), toate reale.
3) Dac
a < 0, atunci Q admite dou
a r
ad
acini complexe conjugate A si A.
Notnd cu una dintre radacinile cubice ale lui A si cu conjugatul lui , radacinile
lui P vor fi + , + si 2 + 2 , toate reale.
II. n cele ce urmeaza, vom studia submultimile lui C definite prin:
Ai = Q (i ) = {R (i ) | R Q [X]} , i = 1, 2, 3;
A = Q (1 , 2 , 3 ) = {R (1 , 2 , 3 ) | R Q [X1 , X2 , X3 ]} .
Propozitia 2. n raport cu operatiile de adunare si nmultire a numerelor complexe, Ai se structureaza ca un corp comutativ. n plus, Ai este un Q-spatiu vectorial
de dimensiune cel mult 3.
Demonstratie. Faptul c
a Ai este un inel integru, unitar si Q-spatiu vectorial se
verific
a imediat. Dac
a R Q [X], din teorema mp
artirii cu rest rezult
a c
a R (X) =
P (X) C (X) + + X + X 2 si, cum P (i ) = 0, deducem ca

Ai = + i + 2i | , , Q ,
1

Cercet
ator, Centrul de Calcul E. N. S. T., Paris

97

prin urmare 1, i , 2i constituie un sistem de generatori pentru Q-spatiul vectorial


Ai ; astfel , dimQ Ai 3. n sfrsit, pentru orice a Ai \ {0}, aplicatiile liniare
Ai Ai definite prin x 7 ax si x 7 xa sunt injective (Ai fiind integru), deci
surjective (Ai fiind spatiu vectorial de dimensiune finita) si atunci exista a0 , a00 Ai
pentru care aa0 = a00 a = 1. Urmeaza usor ca a0 = a00 , deci Ai este corp.
Propozitia 3. Multimea A se structureaza canonic drept corp comutativ si ca
Q-spatiu vectorial de dimensiune cel mult 6.
Demonstratie. Cum 1 +2 +3 = 0, deducem ca A = Q (1 , 2 ) = Q (1 ) (2 ).

P (X)
= X 2 + X + 21 + p A1 [X], prin
Dar 2 este radacina a polinomului
X 1
urmare A va fi un A1 -spatiu vectorial de dimensiune cel mult 2. Rezulta ca dimQ A =
dimA1 A dimQ A1 6. Faptul ca orice element nenul al lui A este inversabil se
demonstreaz
a ca n Propozitia 2.
III. Automorfismele algebrei A.
S
a observ
am c
a orice endomorfism al spatiului vectorial A cu propriet
atile c
a
u (1) = 1, u (xy) = u (x) u (y), x,
y

A
este
n
mod
necesar
bijectiv:
dac
a
u
(x)
=
0,

cu x 6= 0, atunci 1 = u (1) = u xx1 = u (x) u x1 = 0, contradictie, deci u este


injectiv, iar surjectivitatea urmeaz
a din faptul c
a dim A este finit
a.
Un astfel de endomorfism (de algebre) u va fi numit automorfism, iar multimea
tuturor acestor automorfisme o vom nota Aut A. n raport cu compunerea functiilor,
Aut A se structureaza n mod evident ca grup abelian.
Propozitia 4. n cazul n care P este un polinom reductibil peste Q, atunci
dim A {1, 2}; mai mult exista doi indici distincti i, j astfel nct A = Ai = Aj .
Daca dim A = 1, atunci |Aut A| = 1, iar daca dim A = 2, atunci |Aut A| = 2.
Demonstratie. Cum P este reductibil peste Q, atunci toate cele trei r
ad
acini
sunt rationale si atunci A = A1 = A2 = A3 = Q, iar dimQ A = 1, sau o singura
radacina, sa zicem 1 , este rationala, caz n care A1 = Q, A = A2 = A3 este
A1 -spatiu vectorial de dimensiune 2, deci dimQ A = 2.
Pentru orice u Aut A si x A, avem c
a u (xn ) = [u (x)]n , deci u (R (x)) =
R (u (x)), R Q [X]. n particular, P (u (i )) = u (P (i )) = u (0) = 0, deci
i {1, 2, 3}, j {1, 2, 3} a. . u (i ) = j ; altfel spus, un automorfism u realizeaza
o permutare a r
ad
acinilor.
Dac
a dim A = 1, deci cnd A = Q, atunci pentru orice x A avem c
a u (x) =
u (x 1) = xu (1) = x, deci Aut A = {1A }. Daca dim A = 2, fie i Q, 2 , 3
/ Q,
2 6= 3 . Pentru u Aut A vom avea c
a u (1 ) = 1 . Dac
a u (2 ) = 2 , u (3 ) = 3 ,
iar x = y + z2 , y, z Q, este un element al lui A, atunci u (x) = y + zu (2 ) = x,
deci u = 1A . Dac
a u (2 ) = 3 , u (3 ) = 2 , atunci u (x) =
y+ z3 . Trebuie
2
s
a avem u (xx0 ) = u (x) u (x0 ), x, x0 A, ceea ce revine la u 22 = [u (2 )] , iar
aceast
a egalitate este realizat
a: am v
azut
tia Propozitiei 3 c
a 2 , 3
n demonstra

sunt r
ad
acinile polinomului X 2 +1 X + 21 + p A1 [X], deci 22 = 1 2 21 p,

prin urmare u 22 = 1 3 21 p = 23 = [u (2 )]2 . Astfel, n acest caz avem ca
Aut A = {1A , u}.
Propozitia 5. Daca P este ireductibil peste Q, atunci:
a) dim A {3, 6};
98

b) Daca dim A = 6 si u Aut A, atunci exista S3 astfel nct u (i ) = (i) ,


i {1, 2, 3}, iar |Aut A| = |S3 | = 6;
c) Daca dim A = 3, atunci |Aut A| = 3 si singurele elemente invariante pentru
orice automorfism sunt elementele lui Q.
Demonstratie. a) P fiind ireductibil, admite trei r
ad
acini distincte irationale
(o eventuala radacina dubla ar anula P si P 0 , deci (P, P 0 ) Q [X] ar divide P , care
astfel nu ar fi ireductibil).

Cum i nu este radacina a unui polinom din Q [X] de


grad 2, atunci 1, i , 2i este sistem liniar independent si astfel dimQ A = 3. Dac
a
2 A1 , atunci 3 = 1 2 A1 , deci A = A1 = A2 = A3 si dimQ A = 3. Dac
a
2
/ A1 , atunci dimA1 A = 2, deci dimQ A
= dimA1 A dimQ A1 = 6.
b) Daca dimQ A = 6, o baza a lui A fiind 1, 1 , 21 , 2 , 1 2 , 21 2 (v. Propozitia
3), nseamn
a c
a un automorfism u este bine determinat prin cunoasterea elementelor
u (1 ) si u (2 ), pentru c
a u realizeaz
a si aici o permutare a r
ad
acinilor lui P (v.
Propozitia 4). Din comportarea lui u fata de baza, rezulta ca

u (1) = 1, u 21 = [u (1 )]2 , u (1 2 ) = u (1 ) u (2 ) , u 21 2 = [u (1 )]2 u (2 ) ;


aceste conditii sunt suficiente pentru a demonstra ca u este automorfism al lui A.
Mai
nti,
sa aratam ca u este automorfism
al lui A1 : 3o baza a acestui spat4iu fiind
1, 1 , 21 , ar fi destul s
a ar
at
am c
a u 31 = [u (1 )] si u 41 = [u (1 )] . ntradev
ar, avem:

3
u 31 = u (p1 q) = pu (1 ) q = [u (1 )] ;
4

u 1 = u p21 q1 = p [u (1 )]2 qu (1 ) = [u (1 )]4 .


Fie acum x = y + z2 , x0 = y 0 + z 0 2 , cu y, z, y 0 , z 0 A1 ; trebuie sa verificam
ca u (xx0 ) = u (x) u (x0 ). Cum
are loc daca x A2 sau x0 = 2 ,
2 aceasta proprietate
2
r
amne s
a justific
am c
a u 2 = [u (2 )] . Avem:

2
2
2 = 1 2 21 p u 22 = 2 3 22 p = 23 = [u (2 )] ,

P (X)
ad
acin
a pentru
= X 2 + X + 22 + p caz n care
dat fiind c
a 3 este r
X

1 2 3
permutatrea asociat
a lui u este =
.
2 3 1
Obtinem astfel toate automorfismele lui A ca fiind asociate cte unei permut
ari
din S3 , deci |Aut A| = |S3 | = 6.
c) Conform celordemonstrate
la a), daca dim A = 3, atunci A = A1 = A2 = A3 ,

baze a lui A fiind 1, i , 2i , i = 1, 2, 3. Fie u Aut A. Daca exista i astfel


elementele unei baze, deci
a
nct u (i ) = i , atunci u invariaz

a toate
u = 1A . Dac
u 6= 1A , avem c
a u (1 ) = 2 si u 21 = 22 sau u (1 ) = 3 si u 21 = 23 . Aceste
automorfisme u sunt astfel unic determinate, deci |Aut A| = 3 (automorfismele sunt
asociate ciclurilor de lungime 3 din S3 ).
C
aut
am acum elementele invariante pentru orice automorfism. Dac
a x = x1 +
x2 1 + x3 21 A, atunci pentru u 6= 1A avem c
a u (x) = x1 + x2 2 + x3 22 si
u2 (x) = x1 + x2 3 + x3 23 . Egalitatea x = u (x) = u2 (x) conduce, dupa calcule, la
x2 + x3 (2 + 1 ) = 0, x2 + x3 (3 + 1 ) = 0. Daca x2 si x3 nu sunt ambele nule,
atunci 2 + 1 = 3 + 1 , deci 2 = 3 , absurd. Rezult
a c
a singurele elemente
invariante pentru orice automorfism sunt elementele lui Q.
99

Inegalit
ati stabilite cu un procedeu de reducere
a num
arului de variabile - Mixing variables
Iurie BOREICO 1 , Andrei CIUPAN 2
Prezent
am n cele ce urmeaz
a un procedeu mai recent de rezolvare a inegalit
atilor
folosit adesea pentru rezolvarea inegalitatilor de tip olimpiada. Ideea ce sta la baza
acestuia este urmmatoarea: cnd avem de-a face cu inegalitati, este convenabil sa
transform
am o problem
a ce comport
a trei variabile ntr-una numai cu dou
a variabile,
sau s
a reducem inegalitatea la cazul cnd unul din numere este egal cu 0. Vom ilustra
acest procedeu prin cteva probleme rezolvate n acest fel. Pornim cu o inegalitate
simpla si binecunoscuta:
1
Exemplul 1. Daca a + b + c = 1, sa se arate ca a2 + b2 + c2 .
1 3
Solutie. S
a consider
am functia f (a, b, c) = a2 + b2 + c2 si s
a ar
at
am c
a
3

a+b a+b
,
, c au aceeasi sum
a
f (a, b, c) 0. Observ
am c
a tripletele (a, b, c) si
2
2
si evalu
am diferenta

a+b a+b
,
,c .
D = f (a, b, c) f
2
2

a+b a+b
1
2
a f (a, b, c) f
,
, c , deci ar
Se vede usor c
a D = (a b) 0, adic
2
2
2

a+b a+b
fi de ajuns sa demonstram ca f
,
, c 0, ceea ce este mai convenabil.
2
2
(a + b)2
1
Acest lucru este echivalent cu
+ c2 . Fiindc
a a + b + c = 1, rezult
a c
a
2
3

2
1
a+b = 1c, deci ne r
amne s
a demonstr
am c
a (1c)2 +2c2 (c 3 )2 0,
3
3
evident adevarata. Mentionam ca inegalitatea de mai sus are multe alte solutii.
S
a ne ocup
am de probleme mai dificile, care au solutii surprinz
ator de directe
prin aceast procedeu.
b
c
3
a
+
+
.
Exemplul 2. Daca a, b, c 0, sa se arate ca
b+c c+a a+b
2
a
b
c
3
Solutie. Fie f (a, b, c) =
+
+
. Vom ar
ata c
a f (a, b, c)
b+c
c+a
a+b
2

b+c b+c
,
. Explicitnd si apoi desf
acnd parantezele, se obtine inegalitatea
f a,
2
2
3
3
2
2
echivalenta b + c + ab + ac 2abc + b2 c + bc2 , inegalitate imediata datorita
inegalitatii mediilor: b3 + c3 b2 c + bc2 si a(b2 + c2 ) a 2bc. Prin urmare, ramne
b+c
sa aratam ca f (a, t, t) 0, unde t =
. Aceasta relatie este echivalenta cu
2
1
2

Elev, Chisin
au
Elev, Bucuresti

100

2t
3
a
arat. Deci problema este rezolvat
a.
+
(a t)2 0, adev
2t a + t
2
Exemplul 3. Daca a, b, c > 0 si abc = 1, atunci a2 +b2 +c2 +3 2(ab+bc+ca).
Solutie. Fie f (a, b, c) = a2 + b2 + c2 + 3 2(ab + bc + ca). Consider
am un
triplet
convenabil,
care
s
a
p
a
streze
condi
t
ia
abc
=
1.
Un
astfel
de
triplet
poate
fi

( ab, ab, c). Avem f ( ab, ab,


c) =
ab + ab + c2 + 3 2(ab+ 2 c) = c2 4 c + 3.
Mai departe, D= f (a, b, c) f ( ab, ab, c) = a2 + b2 + 4 c 2(ab + bc + ca) =
(a b)2 + 2c(2 ab a b), deci

(a b)2
.
D = (a b)2 2c( a b)2 = (a b)2 2c
( a + b)2
Fiindc
a relatia din
enunt este simetric
a, putem presupune c
a c = min{a,
b, c}.Atunci
deducem ca c < 2. Asta nseamna ca c2 2 2ab c ( a + b)2
(a b)2

2c (a b)2 2c
0, deci D 0. Asadar, r
amne s
a ar
at
am c
a
( a + b)2

f ( ab, ab, c) 0 c2 + 3 4 c, adevarata din inegalitatea mediilor pentru


patru numere.
Exemplul 4. Daca a, b, c > 0 si abc = 1, atunci a2 + b2 + c2 + 3 a + b + c +
ab + bc + ca.
P
P
P
Solutie. Sa consideram functia f (a, b, c) = a2 a ab + 3 si sa demonstr
am c
a f (a, b, c) 0. Vom alege un triplet convenabil pe care s
a-l intercal
m ntre
a

f (a, b, c) si 0. Un
astfel
de
triplet,
la
care
s
i
produsul
e
invariant
este
(
ab,
ab, c).

Se vede c
a f
( ab, ab, c) = c2 + ab 3 2 ab c(1 + 2 ab). Atunci D
=
f (a, b, c) f ( ab, ab, c) = a2 + b2 (a + b) (bc + ca) 2ab+ 2 ab + 2c ab.
2
Prin grupare convenabil
a a termenilor, se obtine D = (a b)2 ( a b)
(c + 1) =
2
(a b)
c+1
(c + 1), de unde D = (a b)2 1

. Fiindca
(a b)2
2
( a + b)
( a + b)2
inegalitatea din enunt este simetrica, putem presupune ca c =
min{a, b,
c}. Deaici,
c 1 si ab 1, de unde obtinem succesiv c + 1 < 4 4 ab ( a + b)2 ,

c+1

1 < 0 D 0. Deci, s
a demonstr
am c
a f ( ab, ab, c) 0
deci
( a + b)2

2
2
c. Dar aceasta relatie din urma este adevarata, deoarece
c +(1/ c 1) +2 c+2

c2 + 2 2c + 1 c + 2 c. n concluzie, problema este rezolvat


a.

Exemplul 5. Se dau a, b, c 0, cu a + b + c = 3. Atunci a + b + c


ab + bc + ca.
P
P
(Indicatie. Consideram f (a, b, c) =
a ab. Atunci
2
p

b+c b+c
(b c)
f (a, b, c) f (a,
=
,
) = b + c 2(b + c) bc +
2
2
2
!

2
( b + c)2 ( b + c + 2(b + c)) 4
b c
p

2
b + c + 2(b + c)

Cu presupunerea a = min{a, b, c} si dup


a efectuarea calculelor, se obtine f (a, b, c)
b+c b+c
f (a,
,
) 0.)
2
2
101

Exemplul 6 (Inegalitatea lui Schur ). Daca a, b, c 0 si r 0, atunci


ar (a b)(a c) + br (b a)(b c) + cr (c a)(c b) 0.

Solutie. S
a consider
am f (a, b, c) = ar (ab)(ac)+br (ba)(bc)+cr (ca)(cb)
si sa demonstram ca f (a, b, c) 0. Se poate observa ca f (a, a, c) = cr (c a)2 0.
R
am
ane s
a ar
at
am c
a f (a, b, c) f (a, a, c) f (a, b, c) f (a, a, c) 0. Dar, un mic
calcul ne arat
a c
a f (a, b, c) f (a, a, c) = (a b) [(a c)(ar cr ) + br (c b)] 0,
daca consideram ordonarea a c b, permisa de simetria inegalitatii din enunt.
Prin urmare, f (a, b, c) f (a, a, c) 0 si inegalitatea este demonstrata.
4
Exemplul 7. Daca a, b, c 0, a + b + c = 1, atunci a2 b + b2 c + c2 a
.
27
Solutie. Se observa ca alegerile de triplete folosite n exemplele precedente nu
mai duc la rezolvarea problemei; este nevoie de alegerea unui triplet inspirat. Pentru
2
1
am
c
a "ghicim" si verific
am c
a egalitatea are loc dac
a a = 0, b = , c = , ncerc
3
3
un alt triplet care s
a ilustreze acest caz de egalitate. Mai nti, consider
am functia
4
2
2
2
am diferenta D = f (a, b, c) f (0, a + b, c).
f (a, b, c) = a b + b c + c a , apoi evalu
27
Obtinem D = a (bc (c a)(c b)). Atunci cnd c este ntre a si b, se vede c
a
D 0 (putem face aceasta presupunere, deoarece inegalitatea din enunt este ciclica
n cele trei variabile). Deci ar fi de ajuns sa demonstram ca f (0, a + b, c) 0.
4
Deoarece a + b + c = 1, relatia precedent
a este echivalent
a cu (a + b)2 c

27
8
2c(1 c)(1 c)
, care rezult
a direct din inegalitatea mediilor.
27
Exemplul 8. Daca a, b, c 0 si a + b + c = 3, sa se afle maximul expresiei
E(a, b, c) = ab(a + b) + bc(b + c) + ca(c + a).
Solutie. Daca a = b = c = 1, atunci expresia are valoarea 6, care nu este maximul
3
ei. ns
a vedem c
a, dac
a lu
am un numar egal cu 0 si celelalte dou
a egale cu , atunci
2
27
expresia ia valoarea
. Vom arata ca acesta este maximul expresiei. Consideram
4
27
functia f (a, b, c) = ab(a + b) + bc(b + c) + ca(c + a) . Cu presupunerea a b c,
4
obtinem f (a, b, c)f (a+b, c, 0) = a2 (b+c)+b2 (c+a)+c2 (a+b)(a+b)2 cc2 (a+b) =
27
9
ab(a + b 2c) 0. Cum f (a + b, c, 0) 0 (a + b) c(a + b+ c)
(a + b)c
4
4
(adevarata din inegalitatea mediilor!), problema este rezolvata.
Exemplul 9 (Inegalitatea lui Turkevici ). Daca a, b, c, d 0, atunci

a4 + b4 + c4 + d4 + 2abcd a2 b2 + a2 c2 + a2 d2 + b2 c2 + b2 d2 + c2 d2 .
P 4
P
(Indicatie. Notam f (a, b, c, d) =
a + 2abcd a2 b2 . Presupunnd ordinea
a b c d, se arat
a c
a f (a, b, c, d) f (a, b, c, c) f (a, b, b, b) f (a, a, a, a) = 0.)

5
Exemplul 10. Daca a, b, c 0, sa se arate ca (a + b + c) 81abc a2 + b2 + c2 .
Solutia 1. Inegalitatea din enunt fiind omogen
a, putem presupune c
a a+b+

c = 3. Astfel, ramne sa demonstram ca abc a2 + b2 + c2 3. Fie f (a, b, c) =


102

b+c b+c
abc a2 + b2 + c2 3. Se arata ca f a,
,
f (a, b, c) 0 si apoi ca
2
2

b+c b+c
,
f a,
0 n urma unor calcule de rutin
a.
2
2
Solutia 2. Se tine cont din nou de omogenitatea inegalitatii din enunt si se
presupune abc = 1. Pentru a demonstra ca (a+ b
+ c)6 81(a2 + b2 + c2 ) se
procedeaz
a astfel: ar
at
am c
a f (a, b, c) f (a, bc, bc) 0, unde f (a, b, c) =
(a + b + c)6 81(a2 + b2 + c2 ).
Solutia 3. Vom demonstra inegalitatea din enunt fara a folosi metoda "mixing
variables", pentru a arata ca, desi aceasta metoda este eficienta pentru un numar
foarte mare de probleme, pot exista si solutii mai simple. Folosind binecunoscuta
(ab + bc + ca)2
inegalitate (ab + bc + ca)2 3abc(a + b + c), deducem ca abc
, deci
3(a + b + c)

6
2
ar fi de ajuns s
a demonstr
am c
a (a + b + c) 27 (ab + bc + ca) a2 + b2 + c2 . Am
putea finaliza rezolvarea tot prin mixing variables, dar alegem inegalitatea mediilor:
q
X
X
X
3
2
2
(a + b + c) =
a2 +
ab +
ab 3 (a2 + b2 + c2 ) (ab + bc + ca) ,
de unde, prin ridicare la puterea a treia, rezult
a chiar inegalitatea c
autat
a.

Exemplul 11. Se dau a, b, c 0, astfel nct ab+bc+ca = 1. Sa se afle minimul


1
1
1
expresiei
+
+
.
a+b b+c c+a
5
(Indicatie. Minumul este , si se obtine cnd doua din numere sunt egale cu 1
2
si celalalt este 0. Putem considera doua triplete
p pentru compararea cu f (a, b, c)
si cu 0: a) f (a,
(a + b)(a + c) a; b) f (a, b, c)
b, c) f (a, t, t) 0, t =
1
f 0,
, a + b 0.)
a+b

Aplicatii
1. Se dau x, y, z 0 si x + y + z = 3; s
a se arate c
a x2 + y 2 + z 2 + xyz 4.
2. Se dau a, b, c > 0, abc = 1; sa se arate ca 3(a2 + b2 + c2 ) + 23
4 (a + 1) (b + 1) (c + 1) .
1 1 1
6
3. Dac
a a, b, c > 0 si abc = 1, s
a se arate c
a + + +
5.
a b
c a+b+c
4
4. Fie a, b, c 0, a + b + c = 1. S
a se arate c
a a2 b + b2 c + c2 a + abc
.
27

1
1
1
(ab+bc+ca)
+
+
5. Dac
a a, b, c 0, s
a se arate c
a
(a + b)2 (b + c)2
(c + a)2
9
(Olimpiada Iran, 1996 ).
4
1
a
b
c
7
6. Dac
a a, b, c [ ; 3], s
a se arate c
a
+
+
.
3
a+b b+c c+a
5
7. Fie a, b, c > 0 si abc = 1. Sa se arate ca 2(a2 +b2 +c2 )+18 3(a+1)(b+1)(c+1).
8. Numerele reale a, b, c satisfac relatia a2 + b2 + c2 = 9. Sa se arate ca
2 (x + y + z) xyz 10 (Test de selectie, Vietnam).

9. Daca a, b, c 0, sa se arate ca (a + b + c)4 16 a2 b2 + b2 c2 + c2 a2 .


103

10. Dac
a a, b, c 0 si a + b + c = 1, atunci s
a se arate c
a
2

2
a + b2 b2 + c2 c2 + a2 8 a2 b2 + b2 c2 + c2 a2 .

11. Dac
a a, b, c 0 cu a + b + c = 3 si k 0, s
a se afle valoarea maxim
a a
expresiei
Ea,b,c = ab(a + b + k) + bc(b + c + k) + ca(c + a + k) (Andrei Ciupan, 2007 ).
12. Se dau numerele reale pozitive a1 , a2 . . . an astfel nct a21 +a22 + +a2n = 1. Sa
se afle maximul expresiei E = (1a1 )(1a2 ) (1an ) (Baraj,Seniori,Romnia,2007 ).
Bibliografie
1. V. Crtoaje - Algebraic Inequalities, Editura GIL, 2006.
2. M. O. Drmbe - Inegalitati - Idei si Metode, Editura GIL, 2003.
3. Site MathLinks: www.mathlinks.ro .

Din partea redactiei


De-a lungul celor noua ani de aparitie a Recreatiilor Matematice, am avut bucuria
de a urm
ari evolutia multor elevi de exceptie. Probleme si note semnate Gabriel
Dospinescu, elev, Onesti, au fost urmate de altele semnate de acelasi, dar student,
Bucuresti, si apoi student, Paris. Alti tineri, precum Oana Crj
a, Cezar Lupu,
Irina Mustata
, Marius Pachitariu (pentru a-i aminti macar pe premiantii revistei) si-au f
acut ucenicia n paginile Recreatiilor, fiind acum studenti eminenti ai unor
universit
ati prestigioase.
Parc
a ns
a nicio promotie nu ne-a adus attia colaboratori ca aceea care tocmai
si-a ncheiat studiile liceale n 2007. Ne facem o placuta datorie de a aminti numele
acestor proaspeti studenti (majoritatea la facult
ati de matematic
a):
Adrian Zahariuc, Bac
au (o not
a matematic
a, 15 probleme propuse);
Alexandru Negrescu, Botosani (dou
a note matematice, 10 probleme propuse);
Iurie Boreico, Chisinau (o nota matematica, 2 probleme propuse);
Vlad Emanuel, Sibiu (6 probleme propuse, cel mai bun rezolvitor);
Bogdan Ciacoi, Gherla (dou
a note matematice, 1 problem
a propus
a).

Al
aturi de acestia, au mai publicat o not
a Anca Timofte si Alexandru Turcanu (Botosani), au propus probleme originale Cristian S
avescu (Focsani), Ruxandra Vlcu, Iulia Zanoschi, Florin As
avoaie (toti din Iasi), au fost mentionati
cu solutii deosebite ale unor probleme Dana Timofte, Diana Prodan, Adrian
Hamciuc (Iasi), au primit premii acordate rezolvitorilor Ciprian Costin, Radu
Ciucanu, Mihai D
an
ail
a, Mircea Avram, Andrei Tofan, C
alin Turliuc, S
tefana Br
anisteanu, Alina Andriescu si Irina Pruteanu.
Tuturor le multumim, le dorim succes n viitor si pe toti i astept
am n continuare
al
aturi de Recreatii Matematice!
104

Asupra unei probleme de concurs


Julieta GRIGORAS
POPA2
1 , Claudiu-Stefan

n cele ce urmeaz
a vom vedea cum se face o Nota plecnd de la o problem
a simpl
a
si comun
a si utiliznd mijloace elementare. Asadar, un ndemn la creativitate. . .
Punctul de pornire este cea de-a doua problema propusa la Concursul "Florica T.
Cmpan", etapa judeteana, clasa a VIII-a, al carei enunt poate fi gasit n acest numar
al revistei la pag. 116
Cerinta de la primul punct al acestei probleme este, dup
a cum au observat si
ctiva candidati, o consecinta imediata a faptului ca, n ipoteza problemei, piramida
considerata este una regulata. Vom indica si alte conditii, asemanatoare celei din
problema surs
a, care s
a implice o piramid
a regulat
a. Mai nti ns
a, vom reformula
si vom rezolva problema initial
a.
Problema 1. Fie V ABC o piramida triunghiulara regulata cu V A V B
V C V A. Notam x = V A, y = V B, z = V C, a = BC, b = AC si c = AB. n
ipoteza ca

xa = yb = zc,
(1)
piramida V ABC este regulata.p

Solu
Cum a =
y2 + z2, b =
x2 + z 2 ,
a
ptie.
z

2
2
2
2
2
c =
x + y
(),
condi
t
ia
(1)
devine
x
+
z
y
=

b
y 2 x2 + z 2 = z 2 x2 + y 2 . Dupa desfacerea parantezelor
V
y
2 2
B
si reducerea lui x y , din prima egalitate obtinem c
ax=y
x
si analog y =z, prin urmare V A = V B = V C. Apoi,
c
a = b = c = x 2 si deducem c
a V ABC este piramid
a reguA
lata.
Problema 2. Daca n ipoteza Problemei 1 nlocuim conditia (1) cu
x
y
z
(2)
= = ,3
a
b
c
atunci V ABC este piramida regulata.
Solutie. Din (2) obtinem, folosind () si proprietatea sirului de rapoarte egale,
x2
y2
z2
x2 + y 2 + z 2
1
ca 2
=
=
=
= , de unde 2x2 = y 2 + z 2 ,
y + z2
x2 + z 2
x2 + y 2
2 (x2 + y 2 + z 2 )
2
2y 2 = x2 + z 2 si 2z 2 = x2 + y 2
. Scaznd doua cte doua aceste relatii, rezulta ca
x = y = z si apoi a = b = c = x 2.
Problema 3. Daca n ipoteza Problemei 1 nlocuim conditia (1) cu
x + a = y + b = z + c, 3

(3)

atunci V ABC este piramida regulata.


1
2

Profesor, Liceul Teoretic "V. Alecsandri", Iasi


Profesor, S
coala "A. Russo", Iasi

3 Nota redac
t iei. Un tetraedru (arbitrar) care verific
a conditia V A V B V C V A
se numeste tridreptunghic, care verific
a (2) izodinamic, iar care verific
a (3) Crelle (D.Brnzei, S.
Anit a, C. Cocea - Planul si spatiul euclidian, pp. 245, 255, 251).

105

Solutie. Observ
am, tinnd seama de (), c
a
2
2
2
2
(3) x + a + 2xa = y + b + 2yb = z 2 + c2 + 2zc
2

2
x + y + z 2 + 2xa = y 2 + x2 + z 2 + 2yb = z 2 + x2 + y 2 + 2zc (1)
si astfel Problema 3 se reduce la Problema 1.
Problema 4. Daca n ipoteza Problemei 1 nlocuim conditia (1) cu
|x a| = |y b| = |z c| ,
(4)
atunci V ABC este priramida regulata.
Solutie. Calculele sunt analoage cu cele din solutia Problemei 3.
Apare ntrebarea: ce se ntmpl
a dac
a nlocuim a, b, c cu o permutare oarecare a
lor? Vom ar
ata c
a si n aceast
a situatie piramida este regulat
a.
0
Problema 1 . Daca n ipoteza Problemei 1 nlocuim conditia (1) cu
xb = yc = za,
(10 )
priramida V ABC este regulata.
Solutie. Observam nti ca, daca doua dintre numerele a, b, c sunt egale, rezulta
usor ca sunt egale toate trei; prin urmare, a, b si c sunt fie distincte doua cte doua,
fie egale ntre ele. Presupunem prin absurd c
a ne putem situa n primul caz si vom
ordona numerele a, b, c; fie a < b < c. Atunci:
a2 < b2 < c2 z 2 + y 2 < x2 + z 2 < x2 + y 2 z < y < x.
Pe de alta parte, din (10 ) rezulta ca z > x > y si ajungem astfel la o contradictie.
R
amne c
a a = b = c, apoi x = y = z, deci V ABC este piramid
a regulat
a.
Problema 20 . Daca n ipoteza Problemei 1 nlocuim conditia (1) cu
x
y
z
= = ,
(20 )
b
c
a
atunci V ABC este piramida regulata.
Solutie. Avem succesiv:
x2
y2
z2
x2
y2
= 2
= 2
2
= 2
=
(20 ) 2
2
2
2
2
2
x +z
x +y
y +z
x +z x
x + y2 y2
x2
y2
z2
x2 + y 2 + z 2
z2
2 = 2 = 2 = 2
= 1,
= 2
2
2
y +z z
z
x
y
x + y2 + z2
de unde x = y = z, apoi a = b = c.
x
y
z
astreaz
a
Observatie. Dac
a nlocuim (20 ) cu
=
=
(200 ), concluzia se p
c
a
b
x
y
z
0
000
cu aceleasi calcule. Daca nsa nlocuim (2 ) cu
=
=
(2 ), concluzia se
a
c
b
pastreaza, cu o justificare putin diferita:
x2
y2
z2
x2 + y 2 + z 2
1
(2000 ) 2
=
=
=
=
2
2
2
2
2
2
2
2
z +y
x +y
x +z
2 (x + y + z )
2
2x2 = y 2 + z 2 , 2y 2 = x2 + y 2 , 2z 2 = x2 + z 2 x = y = z.
L
as
am n seama cititorului formularea problememlor 30 si 40 ; solutiile lor sunt
similare cu cele ale Problemei 10 .
Concluzionam ca, daca x u = y v = z t, unde "" este una dintre operatiile
aritmetice +, , , :, iar u, v, t rezprezint
a o permutare oarecare a numerelor a, b, c,
atunci piramida este regulat
a. Se obtin n acest fel 24 de probleme distincte.
106

Intuirea propriet
atilor operatiilor aritmetice
utiliznd metodele figurative
Petru ASAFTEI 1
Este cunoscut faptul c
a n clasele primare se evidentiaz
a, f
ar
a utilizarea terminologiei, unele propriet
ati ale operatiilor lor. Cunostintele clare despre aceste propriet
ati
se obtin daca se porneste de la un suport material intuitiv. n cazul de fata, materialul intuitiv va fi format din retele de puncte n plan, structurate n asa fel nct
elevii s
a sesizeze propriet
atile operatiilor, binenteles, cu ajutorul explicatiilor date
de nvatator. Vom construi modele figurative pentru urmatoarele proprietati: comutativitatea nmultirii, asociativitatea nmultirii, distributivitatea nmultirii fata de
adunare si distributivitatea nmultirii fata de sc
adere.

3x4

1. Comutativitatea nmultirii
Aceast
a configuratie de puncte poate fi privit
a n dou
a moduri:
primul, de jos n sus, care pune n evidenta c
a se repet
a3
linii cu cte 4 puncte n fiecare linie, ceea ce nseamna 3 4;
al doilea de la stnga la dreapta, care pune n evidenta ca
4x3
se repet
a 4 coloane cu cte 3 puncte n fiecare coloan
a, ceea ce nseamn
a 4 3.
Generalizarea se face n mod natural, tot pe baza unei retele de puncte, care este
formata din a linii si b coloane. Prin analogie cu rationamentul precedent, obtinem
a b = b a.

(2x3)+(2x4)

Profesor, S
coala Normal
a "Vasile Lupu", Iasi

107

3+4
3+4

2x(3x4)

3x4

2x4

2x3

3. Distributivitatea nmultirii fata de


adunare
Se constat
a c
a 2 (3 + 4) si (2 3) +
(2 4) reprezinta tot att si scriem 2
(3 + 4) = (2 3) + (2 4).

3x4

2x(3x4)

2. Asociativitatea nmultirii
Rationamentul matematic are la
baz
a mp
artirea retelei n dou
a submultimi de puncte, fiecare format
a din 3
linii si 4 coloane. Prin analogie cu cazul
2x3 2x3
2x3 2x3
precedent, constatam ca 2 (3 4) si
(2 3) 4 reprezint
a tot att si scriem
4x(2x3) sau (2x3)x4
2 (3 4) = (2 3) 4.
Generalizarea acestei proprietati nu este evidenta. Produsul 2 3 trece n a b,
ceea ce nseamna un tablou cu a coloane si b puncte n fiecare coloana. Repetam acest
tablou de c ori si obtinem c (a b) = (a b) c. Pe de alt
a parte, s
a consider
am
din fiecare tablou o singur
a coloan
a; form
am n acest fel a tablouri cu b linii si c
puncte n fiecare linie. Ca urmare, obtinem a (b c). Numarul total de puncte
nu s-a schimbat, ceea ce nseamna ca (a b) c si a (b c) reprezinta tot att si
scriem (a b) c = (a b) c.

4. Distributivitatea nmultirii fata de scadere


Expresiile 2 (4 3) si (2 4) (2 3) reprezint
a tot att si scriem 2 (4 3) = (2 4) (2 3).
Pentru generalizare trecem 2 4 n a b si 2 3 n
a c, cu b c. n continuare, rationamentul este
analog. Obtinem 2 (4 3) = (2 4) (2 3).

4-3
4-3

2x(4-3)

Pentru generalizare trebuie s


a trecem 23 n ab, iar 24 n ac si vom observa c
a
(a b)+(a c) si a(b + c) reprezinta tot att. Scriem a(b + c) = (a b)+(a c).

(2x4)-(2x3)

Observatie. Utilizarea modelelor figurative, de exemplu cazurile 3 si 4, contribuie la ntelegerea algoritmului de rezolvare a exercitiilor care contin paranteze.

ASOCIA
TIA RECREA
TII MATEMATICE

La data de 14.02.2005 a luat fiinta ASOCIATIA


RECREATII
MATEMATICE, cu sediul n Iasi (str. Aurora, nr. 3, sc. D, ap. 6), avnd ca scop sprijinirea activitatilor de matematica specifice nvatamntului preuniversitar, organizarea si desfasurarea de activitati care sa contribuie la dezvoltarea gustului pentru
matematica n rndurile elevilor, profesorilor si iubitorilor de matematica si stimularea preocuparilor si cercetarilor originale.
Obiectivele majore pentru atingerea scopului propus sunt:
1. editarea unei reviste destinata elevilor si profesorilor revista "Recreatii
Matematice";
2. fondarea unei biblioteci de matematic
a elementar
a biblioteca "Recreatii
Matematice";
3. alcatuirea unei colectii de carti de matematica elementara, carti de referinta
si aflate la prima aparitie Colectia "Recreatii Matematice".
Poate deveni membru al Asociatiei, printr-o simpla completare a unei cerei tip,
orice perosana care adera la obiectivele acesteia si sprijina realizarea lor.
Membri de onoare ai Asociatiei, academicienii:

Constantin Corduneanu
Radu Miron

Vizitati pe Internet revista "Recreatii Matematice" la adresa

http://www.recreatiimatematice.uv.ro
108

Jensens inequality for non-convex functions


Holger STEPHAN 1
Jensens inequality is well known: Let P
f (x) be a convex function on x I R,
x1 , ..., xn I and qi 0 are weights with i qi = 1. Then, Jensens inequality
n
X
i=1

qi f (xi ) f

n
X

qi xi

i=1

holds.
This inequality is in some sense equivalent to the definition of convexity. Thats
why, it is not well known that Jensens inequality holds even for non-convex functions
(moreover, this is believed to be false).

Lets start with an easy contest problem (from the journal Die wurzel, Jena,
05/2005, problem 19):
Let x1 ,. . . , xn be positive real numbers with 0.075 h(x)
0.05
x1 + +xn = n. Prove the following inequality.
n
X
i=1

0.025

X 2
1

1 + xi
3 + xi
i=1

(1)

We use Jensens inequality with the function


h(x) =

10

20

30

40

-0.025
-0.05
-0.075

2
1x
1

=
.
1+x 3+x
(1 + x)(3 + x)

and the weights qi = n1 . We get


X
n
n
n
1 X

2
1
1
1X

xi = h(1) = 0
=
h(xi ) h
n i=1 1 + xi
3 + xi
n
n i=1
i=1

and therefore
(1). The
result is true but the proof is false, because h(x) is not convex
for x > 1 + 3 2 + 3 4 = 6.69464 . . . (see the picture). Why does the inequality
holds, nevertheless? To investigate this question, we try to prove Jensens inequality
n
P
deriving an identity for it. We fix x0 =
qi xi and define a function
i=1

g(x) =

f (x) f (x0 )
x x0

(for x = x0 we define g(x0 ) = f 0 (x0 )). Now, we set


n
n
n
X
X
X
X=
qi f (xi ) f
qi xi =
qi f (xi ) f (x0 )
i=1

i=1

i=1

and check, when does X 0 hold. Using easy calculations, we obtain


1

Cercet
ator dr., Institutul Weierstrass, Berlin (e-mail: stephan@wias-berlin.de)

109

(2)

n
X

n
n
X
X

f (xi ) f (x0 )
X=
qi f (xi ) f (x0 ) =
qi f (xi ) f (x0 ) =
qi
(xi x0 ) =
xi x0
i=1
i=1
i=1

n
X

qi g(xi ) (xi x0 ) =

i=1
n
X

i,j=1

qi qj xi g(xi )

1j<in

n
X

n
X
i=1

n
n
X

X
g(xi )qi xi
qj
qj xj =

qi qj xj g(xi ) =

i,j=1

qi qj (xi xj ) g(xi ) g(xj ) .

Thus, we get the identity


n
n
X

X
X=
qi f (xi ) f
qi xi =
i=1

j=1
n
X

i,j=1

i=1

j=1

qi qj xi g(xi ) xj g(xi ) =

1j<in

qi qj (xi xj ) g(xi ) g(xj )

(3)

We see that X 0 holds, if the function g(x) is monotone increasing. The function
g(x) is called slope function. g(x) is the slope of the secant through the points in
x0 and x. If f (x) in convex, g(x) is increasing for any point x0 . But this is not the
only case. Function g(x) is increasing, if looking from the point (x0 , f (x0 )) on the
graph of f (x), no point of f (x) "lies in the shadow" of the graph. This can happen
for some points x0 even if the function is not convex. The function h(x) above is an
example (here is x0 = 1).
An other amazing example are polynomials of
fourth order typical non-convex functions. We
e1
f(x)
consider such a function f (x) and its inflection
points (the x-coordinates are x1 and x2 ). The
e2
inflection point tangents e1 and e2 intersect the
graph of f (x) in points with x-coordinates x3 and
x4 . Now, Jensens inequality holds for x0 x3 or
x
x3
x0 x4 .
x4
x2
x1
The typical proof of Jensens inequality starts from the inequality for convex
functions
(xi x0 )f 0 (x0 ) f (xi ) f (x0 ) (xi x0 )f 0 (xi )
Multiplying by qi and adding up over i we get
n
n
n
n
X
X
X
X
qi (xi x0 )f 0 (x0 )
qi f (xi ) f (x0 )
qi
qi (xi x0 )f 0 (xi )
i=1

i=1

The left hand side in zero because of

i=1

n
P

qi = 1 and

i=1

i=1

n
P

qi xi = x0 . The middle is X

i=1

and transforming the right hand sind in a similar way as above (3) we obtain.
n
n
X

X
X

0
qi f (xi ) f
qi xi
qi qj (xi xj ) f 0 (xi ) f 0 (xj )
i=1

i=1

1j<in

This is similar to identity (3), but holds only for convex functions.
Moreover, identity (3) is very useful, if we are not only interested in Jensens
inequality, but if we want to estimate the dierence on the left hand side of (3).
110

Concursul de Matematic
a Al. Myller
Editia a V-a, Iasi, martie 2007
Clasa a IV-a
1. Daca a = 2 si b + c = 5, calculati a b + a c.
2. Suma a doua numere este 75. Daca din primul scadem 45 si la al doilea adunam
10, obtinem numere egale. S
a se afle cele dou
a numere.
3. Aflati a b c, unde b este dublul lui a, a este dublul lui c, iar a + b + c = 21.
4. Produsul a 7 numere naturale este 7. Aflati suma numerelor.
5. Pentru numerotarea paginilor unei reviste sunt necesare 135 cifre. Cte pagini
are revista?
6. Numerele naturale nenule distincte a si b sunt cele mai mici pentru care
mp
artirile a : 2 : 2 : 2 : 2 si b : 2 : 2 : 2 : 2 se efectueaz
a exact. Aflati a + b.
7. Aflati ce numar se mareste cu 2007 cnd adaugam la dreapta lui cifra 0.
8. Un profesor are un numar de caiete si jumatate din numarul acestora creioane.
Distribuind cte 3 caiete fiec
arui elev mai r
amn 3 caiete, iar distribuind cte 3
creioane fiec
arui elev, r
amn 3 elevi f
ar
a creioane. Care este num
arul elevilor?
9. Alba ca Zapada si cei sapte pitici au suma vrstelor 185 de ani. S
tiind ca Alba
ca Zapada este cu patru ani mai tnara dect cel mai tnar pitic, iar vrstele piticilor
sunt numere naturale consecutive, aflati vrsta Albei ca Z
apada.
10. Setil
a bea la o mas
a obisnuit
a 5 butoaie de ap
a, iar cnd este nsetat bea 7
butoaie. Daca a baut 39 de butoaie, la cte mese a fost Setila nsetat?
11. S
tefan trebuie sa nmulteasca numarul 223 cu un numar format din 2 cifre
consecutive. Din greseal
a, el a schimbat ordinea acestor cifre consecutive si a obtinut
alt produs. Care este diferenta celor dou
a produse?
12. Primul termen al unui sir de numere naturale este 1 + 2 + 3, al doilea este
2 + 3 + 4 + 5, al treilea este 3 + 4 + 5 + 6 + 7 si asa mai departe. Aflati valoarea celui
de-al 50-lea termen al sirului.
13. Suma a 2 numere A si B este 150. Dac
a stergem una din cifrele lui A se
obtine B. G
asiti toate numerele A si B cu aceast
a proprietate.
14. O minge se ridica la trei sferturi din distanta de la care cade. Daca prima
data i se da drumul de la 64 metri, care este distanta totala parcursa de minge pna
cnd atinge p
amntul a patra oar
a?
15. Pe o tabl
a sunt scrise numerele 1, 2, 3, . . . , 100. Lucian si Dana sterg pe
rnd numerele de pe tabla astfel: mai nti Lucian sterge numerele de pe locurile
impare, apoi Dana sterge numerele de pe locurile pare din sirul ramas. Lucian sterge
din nou numerele de pe locurile impare din noul sir si asa mai departe. Care este
ultimul num
ar r
amas pe tabl
a?

Clasa a V-a
1. Care sunt elementele multimii A = {2x N | 100 x < 103}?
111

2. Care este paritatea num


arului N = (3n + 2)(2n + 3), dac
a n este un num
ar
natural impar?
3. Determinati suma celor mai mari doua patrate perfecte impare de trei cifre.
4. G
asiti num
arul natural de dou
a cifre egal cu triplul sumei cifrelor sale.
5. Care este cardinalul multimii M = {x N | 2x < 1025}?
42 4242 424242
6. Calculati suma S =
+
+
.
43 4343 434343
7. La un concurs se dau 30 de probleme. Pentru fiecare raspuns corect se acorda
5 puncte, iar pentru fiecare r
aspuns gresit se scad 3 puncte. Cte r
aspunsuri corecte
a dat un elev care a obtinut 94 de puncte?
8. Scrieti num
arul 200 ca sum
a de puteri ale lui 2.
9. Determinati toate perechile de numere naturale pentru care m2 (n + 1) = 80.
10. Care este cea mai mare fractie de forma

9
ab
, cu x3y
mai mica dect
13
x3y

multiplu de 10?
11. Restul mp
artirii num
arului 3a + b la 23 este 11, iar ctul este c. Stabiliti
paritatea ctului mp
artirii lui 3a + b c la 11.
12. Fie numarul n = 9 + 99 + 999 + + |99999{z 99}. Cte cifre de 1 se folosesc

2007
la scrierea lui n n baza 10?
2005
2006
,b=2
, c = 22007 . Care este ultima cifr
a
13. Consider
am numerele a = 2
a num
arului N = 2a + 3b + 4c?
14. S
tiind ca numerele naturale a si b dau la mpartirea prin 2001 resturile 1001
si respectiv 1999, determinati restul mpartirii la 667 a numarului 2a + 3b.
15. Se d
a sirul de numere naturale 1, 2 3, 4 5 6, 7 8 9 10, . . . . Care este al
50-lea termen al sirului?

Clasa a VI-a
1. Num
arul x pentru care x2 + x y 4 = 0, unde x, y N si y num
ar prim,
este . . .
2. Raportul dintre un numar natural si inversul sau este 9; atunci suma dintre
numar si invers este . . .
3. Dintre numerele 729100 si 246150 mai mare este . . .
4. Trei frati vor s
a-si mpart
a ntre ei 240 de nuci. Ei hot
ar
asc s
a le mpart
a
n parti direct proportionale cu numerele 2, 3 si 5 si apoi sa dea cte 25 din nucile
primite surorii mai mici, care astfel a primit . . . nuci.
5. Num
arul de trei cifre egal cu de 15 ori suma cifrelor sale este . . .
6. Cel mai mic multiplu de 5 cu suma cifrelor 44 este . . .
7. Un muncitor lucreaza piese trei zile astfel nct 20% din productia fiecarei zile
1
este ct din productia zilei urmatoare. Astfel, a treia zi a lucrat cu 55 piese mai
6
mult ca n prima zi. A treia zi a lucrat . . . piese.
112

8. S
tiind c
a raportul dintre suplementul sumei a dou
a unghiuri adiacente si suma
1
suplementelor lor este , masura unghiului format de bisectoarele lor este . . .
3
9. Cel mai mic num
ar de forma xyz (x < y < z) divizibil cu 12 este . . .
o
n
n+80
, n N este . . .
10. Cardinalul multimii M = x N | x = n10

11. Numarul natural care are exact trei divizori naturali, iar suma divizorilor
este 183 este . . .
12. Cel mai mic numar natural de trei cifre care mpartit pe rnd la 2, 3, 4, 5, 6
d
a resturi diferite nenule este . . .
13. Un unghi XOY cu masura de 179 este mpartit de 17 semidrepte n 18
unghiuri, cu m
asurile exprimate prin numere naturale nenule, distincte. Cel mai
mare unghi posibil dintre cele 18 este . . .
14. Fie triunghiul ABC, (AM bisectoarea unghiului BAC si AD BC, unde
\ = m(DAM
\ ) si m(BAC)
\ + m(AM
\
D, M (BC). Daca m(ABC)
D) = 180 , atunci
\ are m
BAC
asura . . .
15. Fie multimea A = {1, 4, 7, 10, 13, . . . , 100} si B o submultime a sa cu n
elemente avnd proprietatea c
a, la orice alegere a celor n elemente din A, s
a existe
cel putin doua cu suma 104. Valoarea minima a lui n este . . .

Juniori (Clasele VII-VIII)


1. a) Ar
atati c
a oricare ar fi n N, num
arul n2 +2n+2007 nu este p
atrat perfect.
b) Fie k un numar natural par, k 4. Sa se arate ca exista un numar natural n
a fie p
atrat perfect.
astfel nct num
arul n2 + 2n + k s
2. Consideram n drepte concurente n punctul P . Dreptele determina n jurul
punctului 2n unghiuri cu interioare disjuncte, fiecare unghi avnd m
asura de 7 sau
de 17 .
a) Sa se afle n.
b) Sa se arate ca cel putin doua dintre cele n drepte sunt perpendiculare.
3. Fie ABC un triunghi dreptunghic n A si fie M mijlocul laturii BC. Pe
perpendiculara n A pe AM consideram un punct D astfel nct segmentele DM si
AB s
a aib
a un punct comun, notat P . Fie E proiectia punctului D pe dreapta BC.
\
[
Sa se arate ca BP
M = EAC.
4. La un concurs de matematic
a particip
a n elevi, n 5, iar proba contine 5
probleme. Fiecare elev a rezolvat exact 3 probleme. Pentru orice grup de 5 elevi
exista o aceeasi problema rezolvata de fiecare elev din grup. Sa se arate ca exista o
aceeasi problem
a rezolvat
a de toti concurentii.

Seniori (Clasele IX-XII)


1. Rezolvati n multimea numerelor ntregi ecuatia x3 y 3 = 2xy + 7.
2. Fie a 2 un num
ar natural. Consider
am sirul (xn )n1 , unde xn = 1 + a+
+a2 + + a2n . Demonstrati c
a exist
a o infinitate de numere prime care nu divid
nici un termen al sirului.
113

\
\ =
3. Pentagonul convex ABCDE are propriet
atile: AB = BC, m(ABE)+m(
CBD)

\
\
\
m(DBE) si m(AEB) + m(BDC) = 180 . Demonstrati c
a ortocentrul triunghiului
BDE se afl
a pe dreapta AC.
4. Fie n 2 un numar ntreg. Demonstrati ca, oricum am colora cu doua culori
n3 + 5n
o multime de
numere ntregi consecutive, exist
a o submultime monocolor
a
6
{a1 , a2 , . . . , an } astfel nct 1 a2 a1 a3 a2 an an1 .

Concursul de matematic
a Florica T. Cmpan
Etapa judetean
a, 17-18 februarie 2007
Clasa a IV-a
1. Suma a patru numere este 282. Primul num
ar, p
atrimea celui de-al doilea si
dublul celui de-al patrulea sunt cu 4 mai mari dect al treilea num
ar. Aflati numerele.
2. Gasiti cel mai mic si cel mai mare numar cu cifre distincte, stiind ca suma
cifrelor fiecaruia este 17.
3. Koallo este un copil care locuiste n dr
agutul or
asel Oloko din Nigeria. Cum el
iubeste matematica, recent a observat ca daca atribuie cte o alta cifra fiecareia dintre
literele K, O, A, L si nmulteste numarul de 5 cifre corespunzator numelui oraselului
cu 11, obtine num
arul corespunz
ator numelui s
au. Care sunt cifrele atribuite fiec
arei
litere?

Clasa a V-a
1. a) Fie num
arul 1234567891011121314...200520062007. S
a se suprime 100 de
cifre astfel nct num
arul r
amas s
a fie cel mai mare posibil.
b) Sa se determine cel mai mic numar natural de forma a1 a2 ...ak , k 1, care
verifica relatia 7a1 a2 ...ak = 5 a1 a2 ...ak 7.
2. Pentru rezolvarea temei de vacanta, bunica i d
a Anei cte o surpriz
a Barbie
imediat ce termin
a de rezolvat o nou
a problem
a. Ana constat
a de fiecare dat
a c
a,
adunnd cifrele numarului de surprize primite pna atunci cu cifrele numarului de
probleme care i-au r
amas de rezolvat, obtine 11. Cte surprize Barbie va avea Ana
la terminarea temei?
Mihaela Cianga
E
3. n figura alaturata avem un sistem de drumuri care F
leag
a localit
atile A, B, C, D, E, F , G. Fiecare drum existent
G
D
ntre dou
a localit
ati vecine are lungimea un num
ar ntreg de
A
kilometri. O localitate se numeste "nod impar" daca suma
lungimilor drumurilor care pleaca din ea este un numar impar
de kilometri. S
a se arate c
a localit
atile nu pot fi toate "noduri
B
C
impare".
Petru Asaftei
114

Clasa a VI-a
1. O reprezentanta a unui constructor de autoturisme a vndut n 2005 mai putin
de 200 de masini. n 2006 vnzarile au crescut cu 28%, iar n 2007 este preconizata o
sc
adere cu 15% fata de 2006. Cte autoturisme se prognozeaz
a a fi vndute n 2007?
Gabriel Popa
2. n figura alaturata este desenat un pentagon ABCDE
n care DAC DBE, ACE BEC si [AC] [BE].
Fie {I} = AD CE si {J} = BD CE.
a) Aratati ca AIC BJE.
b) Demonstrati c
a [AD] [BD].

C
J

A
I

3. a) Cte numere naturale a1 a2 ...an (n 2), formate din cifre nenule, au proprietatea ca toate numerele a1 a2 , a2 a3 , ..., an1 an sunt patrate perfecte?
Adrian Zanoschi
b) Alin arunca doua zaruri, iar Vlad arunca trei zaruri. Fiecare vrea ca produsul
numerelor obtinute de el sa fie patrat perfect. n care dintre cele doua situatii,
probabilitatea atingerii obiectivului este mai mare?
Monica Nedelcu

Clasa a VII-a
1. a) Se dau dou
a numere ntregi x si y. Cu ajutorul lor se formeaz
a un sir de
numere n felul urm
ator. Primul num
ar este egal cu x. Al doilea num
ar este egal cu
x + y. Al treilea numar este egal cu diferenta dintre al doilea si primul numar. Al
patrulea numar este egal cu diferenta dintre al treilea si al doilea numar. Al cincilea
num
ar este egal cu diferenta dintre al patrulea si al treilea num
ar si asa mai departe.
S
a se afle primele 12 numere ale sirului si al 2007-lea num
ar.
b) Alegeti n fata fiecaruia dintre numerele 1, 2, 3, ..., 2006 unul dintre semnele "+"
sau "" astfel nct numarul |1 2 3 ... 2006| sa ia cea mai mica valoare.
Determinati aceast
a valoare.
2. Un iaht trebuie s
a parcurg
a un traseu sub form
a de triunghi echilateral ABC,
plecnd din A. Pe iaht se afla Sam, Bob si John, care ncearca sa nregistreze
viteza vasului, dar avnd toti trei r
au de mare, nu reusesc s
a retin
a dect informatii
incomplete. Astfel, Sam a observat c
a iahtul a parcurs primele trei sferturi ale cursei
n 3 ore si jum
atate, John si-a notat doar c
a ultimele trei sferturi ale drumului au
fost parcurse n 4 ore si jumatate, iar Bob a observat ca pentru a parcurge distanta
de la B la C au fost necesare cu 10 minute mai mult dect pentru distanta de la A la
B. Presupunnd c
a, pe fiecare latur
a a triunghiului, iahtul a avut vitez
a constant
a,
s
a se determine durata parcurgerii ntregului traseu.
3. Consideram un patrat ABCD cu latura de 9 cm si punctele E AD , F BC,
astfel nct A (ED), C (BF ), AE = 9 cm, CF = 3 cm. O furnic
a str
abate
cel mai scurt drum de la E la F care traverseaz
a p
atratul dup
a o paralel
a la BC.
Construiti drumul pe care l parcurge furnica si aflati lungimea lui.

Clasa a VIII-a
1. Consider
am 9 puncte dispuse ca n figura al
aturat
a:
115

O furnic
a pleac
a din A si ajunge n B trecnd prin fiecare punct
B
o singura data, pe un drum fara autointersectie si mergnd pe laturile sau diagonalele patratelor mici care se pot forma cu punctele
din retea. Dac
a lungimea laturii p
atratului mic este 1, ar
atati c
a
lungimea minim
a
a drumului str
ab
atut de furnic
a este 8 si cea A
maxima este 4 + 4 2.
Gheorghe Iurea
2. Fie piramida triunghiular
a V ABC astfel nct AV BV , BV CV , CV AV
si care are produsul oricaror doua muchii opuse egal cu P . Asociem fiecarei muchii
a piramidei cea mai mica dintre ariile triunghiurilor care au drept baza muchia respectiv
a si vrful pe muchia opus
a a piramidei.
a) Demonstrati c
a V este egal dep
artat de muchiile bazei ABC.
b) Daca suma muchiilor piramidei este S si distanta de la V la una dintre muchiile
bazei este d, calculati n functie de S si d suma celor sase arii asociate muchiilor
piramidei.
Julieta Grigoras
3. Fie
n
o

A = a R | a = a1 3 + a2 ( 3)2 + + a2007 ( 3)2007 ; ai {1, 1} , i = 1, 2007 .

a) Determinati num
arul de elemente rationale din multimea A.
b) Determinati num
arul elementelor multimii A.
Cristian Laz
ar si Claudiu-
Stefan Popa

Etapa interjudetean
a, 1-3 iunie 2007
Clasa a IV-a
1. Suma unor numere naturale consecutive este 90. Unul dintre numere este 10.
Care sunt celelalte numere? Justificati r
aspunsul!
2. Se d
a sirul de numere naturale 5, 10, 15, 20, 25, . . . Care este primul num
ar
din sir cu suma cifrelor 27?
3. Lucian-Georges ia dintr-o cutie de fiecare data mai multe bomboane dect
luase data precedent
a si, n 5 d
ati, a luat n total 31 bomboane. Cte bomboane a
luat a patra oar
a, dac
a prima dat
a a luat de trei ori mai putine bomboane dect a
cincea oara?

Clasa a V-a
1. Un dreptunghi cu n linii si m coloane este mpartit n patratele 11. Pe prima
linie coloram primul patratel, pe a doua linie coloram primele doua patratele, pe a
treia linie primele patru p
atr
atele, pe a patra linie primele opt p
atr
atele s. a. m. d.,
pn
a cnd pe a n-a linie se vor colora toate p
atr
atelele. S
tiind c
a num
arul total de
patratele colorate este 1023, aflati cte linii si cte coloane are dreptunghiul.
Mihaela Buc
ataru
2. Pe o tabl
a de sah se g
asesc 31 de pioni. Ar
atati c
a pe tabl
a exist
a o portiune
avnd una dintre formele din figura de mai jos, pe care nu se afla niciun pion.
116

Tamara Culac
3. Un tlhar mparte prada cu tovarasul sau de rautati. Dintr-un saculet plin cu
monede de 10 bani, el scoate pe rnd cte o moned
a, num
arnd: "Una la tine, una la
mine; a doua la tine, una, dou
a la mine; a treia la tine, una, dou
a, trei la mine; . . . "
si la fiecare num
ar rostit, aseaz
a cte o moned
a n fata sa sau a tovar
asului s
au. Dac
a
n saculet sunt 6000 de monede, aflati ce suma (n lei) revine fiecarui tlhar.
Gabriel Popa

Clasa a VI-a
1. Dou
a coli de hrtie care au aceleasi dimensiuni, L = 9 dm si l = 5 dm, se
taie n dreptunghiuri, prima prin drepte paralele cu lungimea, iar a doua prin drepte
paralele cu latimea, numarul si dimensiunile acestor dreptunghiuri fiind arbitrare. Sa
se arate c
a exist
a cel putin o situatie n care suma perimetrelor tuturor dreptunghiurilor obtinute prin t
aiere din prima coal
a este egal
a cu suma perimetrelor tuturor
dreptunghiurilor obtinute din a doua coala.

Petru Asaftei
2n + 7
2. a) Fie multimea A = n N |
fractie reductibil
a . S
a se g
aseasc
a
3n + 1
trei elemente din multimea A.
Enache P
atrascu
b) Cum planteaza un pomicultor 10 copaci pe 5 rnduri, astfel nct sa fie 4 copaci
pe fiecare rnd?
3. Un evantai are 37 de spite, astfel nct unghiul dintre oricare dou
a spite al
aturate are m
asura de 5 . S
a se arate c
a, dac
a se aleg oricare 12 spite, exceptnd-o pe
prima, se formeaza cel putin trei unghiuri de masuri egale.
Doru Buzac

Clasa a VII-a
1. Dan are trei jetoane pe care sunt scrise trei numere reale pozitive, iar Ana are
alte trei pe care sunt scrise inversele lor. Dan i da Anei un jeton pe care aceasta
l asociaz
a cu un altul dect inversul s
au. Apoi, Ana i d
a lui Dan un jeton pe
care acesta l asociaz
a cu un altul dect inversul s
au. S
tefan primeste de la cei doi
copii jetoanele ramase si constata ca nici numerele sale nu sunt inverse unul celuilalt.
Fiecare anunta suma numerelor pe care le are pe cele doua jetoane, respectiv 4, 1 si
7
. Ar
atati c
a produsul numerelor scrise pe cele trei jetoane detinute initial de Dan
3
este 1.
Mihaela Cianga
2. Fie I punctul de intersectie al diagonalelor trapezului ABCD, AB k CD,
AB = a cm, CD = b cm, a > b. Paralela prin I la AB intersecteaza pe AD si BC
a simultan, pe drumul
n punctele P , respectiv Q. Trei mobile M1 , M2 si M3 pleac
cel mai scurt, din punctele B, Q, C spre punctele A, P respectiv D, astfel: M1 cu
viteza constanta b cm/s, M2 cu viteza constanta a cm/s pna n I si apoi, fara oprire,
117

cu viteza constant
a b cm/s de la I la P , iar M3 cu viteza constant
a a cm/s. Aflati
ordinea n care ajung cele trei mobile n punctele A, P , respectiv D.
Claudiu S
tefan Popa
3. Se considera n plan noua puncte, astfel nct din oricare trei se pot alege cel
putin doua cu distanta dintre ele mai mica sau egala cu 1. Aratati ca exista un disc
de raz
a 1 care contine cel putin 5 puncte din cele considerate.
Gheorghe Blendea

Clasa a VIII-a

1. Pe tabla sunt scrise numerele 3 1, 3 + 1, 2. Se sterg cele trei numere si


se scriu n locul lor cele trei medii geometrice a cte doua dintre numere. Se repeta
procedeul cu noile numere. Este posibil ca dup
a mai multi pasi pe tabl
a s
a fie scrise
numerele:
p
p

a) 2 3 2, 2, 2 3 + 2;
b) 2 3, 2 + 3, 4?
Monica Nedelcu
2. Triunghiul alaturat este considerat fix. n cte moduri putem
aseza numerele 1, 2, 3, 4, 5, 6 n cerculete, astfel nct suma numerelor de pe fiecare latur
a a triunghiului s
a fie aceeasi?
Petru Asaftei

3. Exist
a o tar
a K a cubarzilor. Un cubard are corpul de forma unui cub, o
anten
a ce porneste dintr-un vrf al cubului, o coad
a ce este diagonal
a a unei fete a
cubului si o gur
a care este exact la mijlocul unei muchii a cubului. Un cubard se
poate rostogoli dupa voie, se poate umfla sau strnge dupa plac, si poate roti antena
cum vrea. Nu exist
a doi cubarzi pe care i-am putea suprapune nct sa le coincid
a
cozile, gurile si antenele. Care este num
arul maxim al cubarzilor?
Dan Brnzei

Olimpiada Balcanic
a de Matematic
a Juniori
(JMBO)
Editia a XI-a, S
umen (Bulgaria), 23-30 iunie 2007
La aceasta editie a concursului au participat 11 tari: Albania, Bosnia & Hertegovina, Cipru, Macedonia, Grecia, Moldova, Muntenegru, Romnia, Serbia, Turcia,
Bulgaria 1 si trei echipe n afara concursului: Bulgaria 2, Kazahstan si Sumen
&

Varna.
Delegatia Romniei a fost condusa de prof. Dinu Serb
anescu si prof. Mircea
Fianu. Echipa tarii noastre a fost compusa din urmatorii elevi: Chindea Filip si
Bumbacea Radu (din Bucuresti) medaliati cu aur (ambii au acumulat maximum
de puncte), Tiba Marius (Iasi), Ciolan Emil Alexandru (Slatina), Muntean
Alexandru (Bucuresti) si Filip Laurian (Arad) medaliati cu argint.
118

Elevul Tiba Marius a fost primul din medaliatii cu argint, printre toti concurentii
ce au luat aceasta medalie.
Se preconizeaza ca Albania sa gazduiasca urmatoarea editie a JMBO.

Enunturile problemelor
1. Fie a un num
ar real pozitiv astfel nct a3 = 6 (a + 1). S
a se arate c
a ecuatia
2
x + ax + a 6 = 0 nu are solutii reale.
2. Fie ABCD un patrulater convex avnd DAC = BDC = 36 , CBD = 18
si BAC = 72 . Diagonalele AC si BD se intersecteaz
a n punctul P . S
a se
determine m
asura unghiului AP D.
3. Se consider
a 50 de puncte n plan, oricare trei necolineare. Fiecare dintre
aceste puncte este colorat folosind una dintre patru culori date. Sa se arate ca exista
o culoare si cel putin 130 de triunghiuri scalene cu vrfurile n puncte de aceasta
culoare.
4. S
a se arate c
a dac
a p este un num
ar prim, atunci 7p + 3p 4 nu este p
atrat
perfect.
2

Not
a. Timp de lucru: 4 ore si 30 minute. Fiecare problem
a este notat
a cu 10
puncte.

Solutiile problemelor
prezentate de elevul Marius Tiba

6
1. Observam ca a3 = 6 (a + 1) a a2 6 = 6 a2 6 = (a este pozitiv).
a
Atunci, ecuatia din enunt se scrie sub forma
6
(1)
x2 + ax + = 0
a
24
si are discriminantul D = a2
. S
a presupunem, prin reducere la absurd, c
a
a
ecuatia dat
a sau ecuatia (1) are solutii reale. Acest fapt este echivalent cu conditia
D 0. Avem

Atunci,

D 0 a3 24 0 6 (a + 1) 24 0
6 (a 3) 0 a 3.

adic
a 6 9, absurd.

6 = a a2 6 3 (9 6) = 9,

119

2. Fie un punct Q pe dreapta AC ce respect


a or
\
dinea A C Q si este astfel nct m(CBQ) = 18 . n\ QBD,
\ patrulaterul ABCD este inscriptibil.
truct QAD
\ = m(QAB)
\ = 72 . Deducem c
Ca urmare, m(QDB)
a

\ = 36 = m(CDB).
\ Asadar, C este centrul cercum(QDC)
\ ) = 1 m(DQB)
\ = 36 .
lui nscris n 4DBQ si, deci, m(DQP
2
\
n sfrsit, m(AP
D) = 108 (unghi exterior triunghiului
P DQ).

D
P

3. Se constat
a usor c
a exist
a cel putin 13 puncte colorate la fel (dac
a de fiecare
culoare ar exista cel mult 12 puncte colorate cu aceasta, atunci n plan am avea cel
mult 48 puncte). Vom arata ca exista 130 triunghiuri scalene cu vrfurile dintre
aceste 13 puncte.
ntr-adev
ar, cu cele 13 puncte putem forma 13 12 11 : 6 = 286 triunghiuri (n
num
arul celor 13 12 11 triunghiuri, un triunghi este socotit de 6 ori). Observ
am
acum ca, daca consideram un segment [AB], exista cel mult doua puncte astfel nct
[AB] sa fie baza de triunghi isoscel formate cu ele. Cum cu 13 puncte putem avea
13 12
= 78 segmente, rezult
a c
a exist
a cel mult 78 2 = 156 triunghiuri isoscele. Ca
2
urmare, exist
a cel putin 286 156 = 130 triunghiuri scalene cu vrfurile n puncte
de aceast
a culoare.
4. Daca p este par, atunci p = 2 si se verifica direct ca numarul din enunt nu-i
p
atrat perfect.
Dac
a p este impar, atunci p este fie de forma M4 + 1, fie M4 + 3. n cazul n care
p = M4 + 1, rezulta ca numarul 7p + 3p 4 = M4 + 2 (ultima cifra a lui 3p fiind 3) si
nu este patrat perfect. Daca, nsa, p = M4 + 3, sa notam k 2 = 7p + 3p 4. Deducem
imediat c
a k2 = Mp 1 (se utilizeaz
a mica teorem
a a lui Fermat), echivalent cu
2
p | k + 1. De aici deducem c
a p | 1 (stim c
a p prim si p | a2 + b2 p | a si p | b),
ceea ce este absurd. Deci, nici n acest caz numarul din enunt, nu-i patrat perfect.

Vizitati pe Internet revista "Recreatii Matematice" la adresa

http://www.recreatiimatematice.uv.ro

120

Solutiile problemelor propuse n nr. 2 / 2006


Clasele primare
20 28
P.114. n piramida alaturata unele numere s-au sters de-a
8 12 16
lungul timpului. Putem sa le punem la loc?
3
9
(Clasa I )
Ionela B
ar
agan, elev
a, Iasi
5
1
Solutie. 20 = 8 + 12, 28 = 12 + 16 etc.
P.115. Daca din prima lada iau 2 mere si le pun n lada a doua, din a doua lada
iau 3 mere si le pun n lada a treia, iar din lada a treia iau 4 mere si le pun n prima
lada, atunci n fiecare lada voi avea cte 34 mere. Cte mere au fost n fiecare lada?
(Clasa I )
Mariana Nastasia, elev
a, Iasi
Solutie. n prima lad
a erau 34 4 + 2 = 32, n a doua 34 2 + 3 = 35, iar n a
treia 34 3 + 4 = 35 (mere).
P.116. Luni, mama pune ntr-un cos cteva mere. Joi, ea gaseste n cos doar 20
de mere. Cte mere a pus mama n cos, stiind ca, n fiecare zi din acea saptamna,
Mihai, cel mai mare dintre frati, mparte fratilor mai mici cu un mar mai mult ca n
ziua precedenta si ca joi el mparte 5 mere? Cte mere mai ramn n cos smbata,
dupa mpartirea merelor?
(Clasa a II-a)
Inst. Maria Racu, Iasi
Solutie. Merele luate pn
a joi: 5 + 4 + 3 + 2 = 14. Merele existente la nceput
n cos: 20 + 14 = 34. Merele mpartite vineri si smbata: 6 + 7 = 13. Merele ramase
smbata n cos: 20 13 = 7.
P.117. n exercitiul 1111111 = fiecare casuta poate fi nlocuita cu +
sau . Ct poate fi rezultatul acestui exercitiu?
(Clasa a II-a)
Diana T
an
asoaie, elev
a, Iasi
Solutie. Deoarece avem un numar impar de 1, putem obtine ca rezultat 1 sau 3
sau 5 sau 7. Exemplu: 1 1 + 1 1 + 1 1 + 1 = 1; 1 1 + 1 1 + 1 + 1 + 1 = 3;
1 1 + 1 + 1 + 1 + 1 + 1 = 1; 1 + 1 + 1 + 1 + 1 + 1 + 1 = 1.
P.118. Calculati a + b + c + d, stiind ca a b = 5 si c d = 15. Gasiti toate
posibilitatile.
(Clasa a III-a)
nv. Rica Buc
atariu, Iasi
Solutie. a + b + c + d poate fi: 1 + 8 + 1 + 15 = 25, 1 + 8 + 5 + 3 = 17,
2 + 4 + 1 + 15 = 22, 2 + 4 + 3 + 5 = 14.
P.119. Produsul a 10 numere naturale este 40. Aflati cea mai mica si cea mai
mare valoare a sumei celor 10 numere.
(Clasa a III-a)
nv. Mirela Buburuzanu, Tomesti (Iasi)
Solutie. 40 = 2 2 2 5 = 2 2 10 = 2 20 = 4 10. Dup
a caz, suma
numerelor este 9 1 + 40 = 49, 6 1 + 2 3 5 = 17, 7 1 + 2 2 + 10 = 21,
8 1 + 2 + 20 = 30, 8 1 + 4 10 = 22. Suma minim
a este 17, iar cea maxim
a este
49.
P.120. Se considera numerele 1, 2, 3, 4, 5, . . . , 49. Care este cel mai mare numar
de numere pe care putem sa alegem dintre acestea astfel nct suma oricaror trei

121

dintre ele sa se mparta exact la 9?


(Clasa a III-a)
nv. Felicia-Petronela Leanu, Ceplenita (Iasi)
Solutie. Numerele 3, 12, 21, 30, 39, 48 satisfac conditia ceruta la problema. Daca
mai introducem cel putin un num
ar, de exemplu 17, atunci putem g
asi o sum
a de
trei numere care nu se mparte exact la 9, de exemplu 3 + 12 + 17. Num
arul maxim
de numere este 6.
P.121. Ceasul lui Andrei o ia nainte cu 20 secunde pe ora. El a potrivit ceasul
luni la ora 8 si a citit din nou ceasul lunea urmatoare la aceeasi ora. Stiind
ca n

aceasta durata ceasul nu a functionat permanent, iar la ultima citire arata ora 8 h
50 min, sa se afle ct nu a functionat ceasul.
(Clasa a IV-a)
Paula Borsanu, elev
a, Iasi
Solutie. De luni, ora 8, pna lunea urmatoare, ora 8, sunt 168 ore. Daca ceasul
ar fi functionat permanent, atunci ar fi avut un avans de 168 20 : 60 = 56 (minute).
n realitate, avansul este de 50 minute. Ceasul nu a functionat (56 50)60 : 20 = 18
(ore).
P.122. La Concursul de matematica "Fl.T.Cmpan", etapa judeteana, au participat 100 elevi de clasa a IV-a, care au avut de rezolvat 3 probleme. Daca 70 elevi au
rezolvat bine prima problema, 69 a doua problema si 64 a treia problema, sa se arate
ca macar 3 elevi au rezolvat corect toate cele trei probleme.
(Clasa a IV-a)
Anca Cornea, elev
a, Iasi
Solutie. Dac
a fiecare elev a rezolvat bine numai cte dou
a probleme, atunci
numarul de rezolvari corecte este 100 2 = 200. n realitate numarul de rezolvari
corecte este 70 + 69 + 64 = 203. Deoarece 203 200 = 3, nseamna ca macar 3 elevi
au rezolvat corect toate cele trei probleme.
P.123. ntr-o cutie sunt 34 bile, din care unele cntaresc cu 1 g mai mult. Daca
fiecare bila cntareste un numar natural de grame, iar masa tuturor bilelor este 113 g,
sa se afle cte bile sunt mai grele.
(Clasa a IV-a)
Petru Asaftei, Iasi
Solutie. Daca o bila mai grea cntareste 3g , atunci masa maxima este 2g 1 +
3g 33 = 101g < 113g. Dac
a o bil
a mai usoar
a cnt
areste 4g, atunci masa minim
a
este 4g 33 + 5g 1 = 132g > 113g. nseamn
a ca o bil
a mai grea cnt
areste 4g.
Daca fiecare bila cntareste 3g, atunci masa tuturor bilelor este 3g 34 = 102g;
113g 102g = 11g; 4g 3g = 1g. Numarul bilelor mai grele este 11 : 1 = 11.

Clasa a V-a
V.71. Comparati numerele 3300003 si 2450004 .

Lucian Tutescu, Craiova

Solutie. Avem:
3300003 = 3 3300002 = 3 9150001 > 3 8150001 = 3 2450003 > 2 2450003 = 2450004 .
V.72. Fie multimile A, B astfel nct A B, |P (A)| 60, |P (B)| 260. Sa
se determine |A| si |B|. (Prin |X| am notat cardinalul multimii X.)
Petru Asaftei, Iasi
Solutie. Cum A B, atunci |A| |B|, de unde 60 2|A| 2|B| 260. Obtinem ca 6 |A| |B| 8, deci (|A| , |B|) {(6, 6) ; (6, 7) ; (6, 8) ; (7, 7) ; (7, 8) ; (8, 8)}.
122

V.73. Sa se scrie numarul 20062005 ca o suma de sase patrate perfecte nenule.


Ionel Nechifor, Iasi
Solutia 1. Avem:

20062005 = 2006 20062004 = 12 + 52 + 72 + 92 + 252 + 352 20062004 =


2
2
2
2

= 20061002 + 5 20061002 + 7 20061002 + 9 20061002 +


2
2

+ 25 20061002 + 35 20061002 .
Solutia 2 (Emanuel Petrescu, elev, Iasi).

20062005 = 20061002 42 + 52 + 102 + 112 + 122 + 402 =


2

= 20061002 4 + + 20061002 40 .

V.74. Se considera sirul de numere naturale 1, 1, 2, 5, 12, 27, 58, . . . . Calculati


suma primilor 100 de termeni ai sirului.
Marius Damian, Br
aila
Solutie. Adunnd 0 la primul termen, 1 la al doilea, 2 la al treilea etc, obtinem
sirul puterilor lui 2. Deducem ca termenii sirului sunt de forma 2n n, n N. Suma
primilor 100 de termeni va fi

S = 20 0 + 21 1 + 22 2 + + 299 99 =

99 100
= 20 + 21 + 22 + + 299 (1 + 2 + + 99) = 2100 1
= 2100 4951.
2
V.75. Fie x, k N, x 2, k < x. Sa se arate ca
. . . 1} (x) .
(x 1) 12 . . . k (x) + k + 1 = 11
| {z
k+1 cifre

Solutie. Avem:

Doru Buzac, Iasi

(x1) 12 . . . k (x) + k + 1 = (x1) xk1 + 2xk2 + + (k1) x + k + (k+1) =


= xk xk1 + 2xk1 2xk2 + + (k 1) x2 (k 1) x + kx k + k + 1 =
. . . 1} (x) .
= xk + xk1 + xk2 + + x + 1 = 11
| {z
k+1 cifre

Clasa a VI-a

VI.71. Fie x, y, z N astfel nct 100x 2006y 2 + 15z = 0. Sa se arate ca


.
y (x + z) .. 85.
Dan Nedeianu, Drobeta-Tr. Severin
.
.
Solutie. Cum 2006y 2 = 5 (20x + 3z) si (5, 2006) = 1, atunci y 2 .. 5, deci y .. 5.

.
Apoi, 15 (x + z) = 17 118y 2 5x si cum (15, 17) = 1, obtinem c
a x + z .. 17. De.
.
ducem c
a y (x + z) .. 5 17, adic
a y (x + z) .. 85.
VI.72. Fie p N un numar prim. Sa se determine x, y N astfel nct
p
y
+ N .
x2 x
D. M. B
atinetu-Giurgiu, Bucuresti
123

p
y
+ = a N ; atunci p + xy = ax2 , deci p = x (ax y). Cum p
x2 x
este prim, atunci x {1, p} si corespunz
ator g
asim solutiile
Solutie. Fie

(x, y) = {(1, a p) ; (p, ap 1) | a N } .

VI.73. Fie An = 14 + 1414 + 141414 + + 1414


. . . 14} , n N . Sa se determine
| {z

2
An 7n 7n
multimea M = n N |
N .
9

2n cifre

Valeriu Brasoveanu, Brlad


.
Solutie. Notnd cu s (k) suma cifrelor num
arului natural k, avem k s (k) .. 9.
.
.
.
Deducem c
a 14 5 .. 9, 1414 5 2 .. 9, . . . , 1414
. . . 14} 5n .. 9, prin urmare An
| {z
2n cifre

5n (n + 1)
n (n + 1) ..
9n (n + 1) ..
5
. 9. Rezulta de aici ca An

. 9, de unde
2
2
2
An 7n (n + 1)
N, n N . n concluzie, M = N .
9
VI.74. Consideram doua axe perpendiculare Ox si Oy, precum si cele doua bisectoare ale unghiurilor drepte care se formeaza. Fie A oarecare, iar B, C simetricele
sale fata de prima bisectoare, respectiv fata de Ox. Rotim segmentul [OC] n jurul
lui O cu 90 n sensul acelor de ceasornic si notam cu D extremitatea segmentului
rotit. Sa se arate ca:
a) B, O, D sunt coliniare si O este mijlocul lui [BD];
b) D este simetricul lui A fata de a doua bisectoare.
Adrian Corduneanu, Iasi
[
[
Solutie. a) Din 4AOC isoscel avem ca AOx COx,
A
n y
\0 COm
\0 (aceeasi diferenta pn
de unde AOn
a la 135 ).
m
\ si DOm
\0 au complementele egale,
Atunci unghiurile AOm
\
B
deci sunt congruente. Din 4AOB isoscel avem ca AOm
0
x
\ adic
\ prin urmare DOm
\ BOm,
a punctele B, O,
BOm,
O
D
D sunt coliniare. Cum AO = BO = CO = DO, deducem
n
m
si c
a O este mijlocul lui [BD].
C
0
\
\
b) Din relatia deja demonstrat
a AOm DOm rezult
a
\0 DOn
\0 , deci [On0 este bisectoare n 4AOD isoscel. Obtinem ca On0 este
ca AOn
mediatoarea lui [AD], adic
a D este simetricul lui A fata de a doua bisectoare.
VI.75. Fie ABCD un patrulater convex, O intersectia diagonalelor, M mijlocul
lui [AB], iar N mijlocul lui [OD]. Sa se arate ca 2PBCN M < PBDC + PABC .
Bogdan Posa si Marius Dr
agoi, elevi, Motru
Solutie. Fie T mijlocul lui [OB]. Cum M , N , T nu pot fi coliniare, avem
AO + BD
c
a MN < MT + T N =
. n 4CDO, [CN ] fiind median
a, rezult
a c
a
2
CO + CD
CN <
. Atunci:
2
124

PBCN M = BC + CN + N M + M B <
CO + CD AO + BD AB
< BC +
+
+
=
2
2
2
BC + AC + AB BC + BD + CD
=
+
=
2
2
1
= (PABC + PBCD ) .
2

A
M

D
O

Clasa a VII-a
\
VII.71. Fie 4ABC, AB < AC si D (AC). Fie AE bisectoarea lui BAC,
E (BD), F mijlocul lui [AD], {O} = AE BF , {G} = DO AB. Sa se arate ca
GD k BC AB = CD.
Carmen Daniela Tamas, Brlad
Solutie. Cu teorema bisectoarei n 4ABD, obtinem c
a
BE
AB
AG BE
=
. Folosind apoi teorema lui Ceva,

DE
AD
GB DE
DF
AG
AD

= 1, deducem c
a
=
. Atunci GD k BC
FA
GB
AB
AD
AD
AD
AG
=

=
AB = CD.
GB
DC
AB
DC

A
F
G

D
E

VII.72. Fie ABCD paralelogram, E (CD), {M } = AE BD, {N } = BE


AC, {O} = AC BD. Sa se arate ca AMEN = 2AMON .
Mirela Marin, Iasi
Solutie. Cu teorema lui Menelaus n 4M EB
D
E
C
(O N A transversal
a) si 4AEN (M O B
M
N E OB AM
transversal
a) obtinem c
a

= 1,
N
N B OM AE
M E OA BN
O
respectiv

= 1. nmultind aceste
M A ON BE
OA OB
relatii membru cu membru, rezulta ca
A
B
OM ON
NE ME
AAOB AMEN

= 1. ns
a evident c
a d (E, AB) = 2d (O, AB),

= 1, deci
AE BE
AMON AAEB
adica AAEB = 2AAOB si de aici urmeaza concluzia.
VII.73. Fie a < b c razele a trei cercuri tangente ntre ele si tangente la o
1
1
1
aceeasi dreapta n trei puncte distincte. Sa se arate ca = + .
a
c
b
Dan Radu, Bucuresti
Solutie. Fie T1 , T2 , T3 punctele de tangenta ale celor trei cercuri cu dreapta.
Un calcul imediat arata ca T2 T32 = 4bc
T1T32 = 4ac, T1 T22 = 4ab. Scriind
si analog

c
a T2 T3 = T1 T2 + T1 T3 , rezult
a c
a bc = ac + ab si, prin imp
artire cu abc,
obtinem egalitatea dorit
a.
Evident, concluzia nu are loc n cazul n care T1 = T2 = T3 .
125

VII.74. Fie M multimea multiplilor lui 36 n a caror scriere n baza 10 nu apar


alte cifre dect 4, 6 sau 9. Cte numere cel mult egale cu 100 000 contine M ?
Gabriel Popa, Iasi
Solutie. Elementele lui M se divid cu 4 si cu 9, deci se termin
a n 44, 64 sau
96 si, cum au cel mult 5 cifre, au suma cifrelor 9, 18, 27 sau 36. Not
am cu s suma
tuturor cifrelor afara de ultimele doua ale unui numar din M .
Daca un numar se termina n 44, atunci s {10, 19}. Suma 10 se formeaza din
4 + 6, iar suma 19 din 4 + 6 + 9. Obtinem astfel 2 + 6 = 8 elemente ale lui M : 4644,
6444, 46944, 49644, 64944, 69444, 96444, 94644. Dac
a un num
ar se termin
a n 64,
atunci s {8, 17}. Suma 8 se formeaza din 4 + 4, iar suma 17 din 4 + 4 + 9 si gasim
alte 1 + 3 = 4 elemente ale lui M . n sfrsit, daca un numar se termina n 96, atunci
s {12, 21}. Avem c
a 12 = 4 + 4 + 4 = 6 + 6, iar 21 = 9 + 6 + 6 si astfel g
asim nc
a
1 + 1 + 3 = 5 elemente ale lui M .
n total, M contine 8 + 4 + 5 = 17 elemente cel mult egale cu 100000.
VII.75. Fie m 3 un numar natural impar si a1 , a2 , . . . , am Z astfel nct
|a1 a2 | = |a2 a3 | = = |am1 am | = |am a1 | .

Demonstrati ca a1 + a2 + + am se divide cu m.

Maria Mihet, Timisoara


Solutie. Fie d valoarea comun
a a modulelor din enunt; atunci fiecare dintre
numerele a1 a2 , a2 a3 , . . . , am1 am , am a1 este d sau d, deci suma lor este
un multiplu impar de d. Pe de alta parte, suma celor m numere este 0, deci d = 0.
Rezult
a c
a a1 = a2 = = am , de unde a1 + a2 + + am = ma1 .

Clasa a VIII-a
VIII.71. Pe planul 4ABC se ridica perpendiculara AM . Fie P proiectia lui A
pe planul (M BC), iar E, F proiectiile punctului P pe M B, respectiv M C. Sa se
\
\
arate ca M
EF M
CB.
Otilia Nemes, Ocna Mures
M

Solutie. Conform teoremei celor trei perpendiculare,


obtinem c
a AE M B si AF M C. Cu teorema catetei n
4M AB si 4M AC, deducem ca M A2 = M E M B = M F
ME
MF
M C, deci
=
. Rezult
a c
a 4M EF 4M CB,
MC
MB
\
\
de unde M
EF M
CB.

E
A

P
C

B
VIII.72. Fie a, b, c numere reale distincte. Sa se afle partea ntreaga a numarului
b2 + ca
c2 + ab
a2 + bc
+
+
.
A=
(a b) (a c) (b c) (b a) (c a) (c b)
Mihail Bencze, Brasov
Solutie. Dup
a calcule de rutin
a, se arat
a c
a A = 2, deci [A] = 2.

VIII.73. Sa se demonstreze ca a N este ipotenuza a unui triunghi dreptunghic


cu laturile exprimate prin numere naturale daca si numai daca exista n N astfel
nct a2 n si a2 + n sunt patrate perfecte.
C
at
alin Calistru, Iasi
126

Solutie. Fie b, c N cu a2 = b2 + c2 ; considernd n = 2bc, avem c


a a2 n =
(b c)2 si a2 + n = (b + c)2 , Reciproc, daca a2 n = x2 , a2 + n = y 2 , cu x, y N,
atunci 2a2 = x2 + y 2 , deci x, y au n mod necesar aceeasi paritate. Obtinem ca

2
2
x+y
xy
x+y xy
a2 =
+
, cu
,
numere naturale.
2
2
2
2

xy
xy

1,
VIII.74. Fie (0, 1]; sa se arate ca +
:
1 xy
1 xy
n fiecare din situatiile:
x+
a) x, y [0, );
b) < x 0 y <
.
1 + x
Gheorghe Costovici, Iasi
Solu
t
ie.
Este
suficient
s
a
demonstr
a
m,
n
fiecare
dintre
ipoteze, ca numitorul

xy
este strict pozitiv.

1 xy
a) F
ar
a a restrnge generalitatea, s
a presupunem c
a x y; atunci

xy
x

y
>0
> 0 ( x) + y (1 x) > 0.
(1)

1 xy
1 xy
nsa x < x < 2 1 1 x > 0 y (1 x) 0, de unde (1) este
imediata.
x+
b) Mai nti, sa verificam ca 0 <
, adica ipoteza nu este contradictorie.
1 + x
Avem:
x+
1 < x 1 2 < x 1 + x > 0
> 0;
1 + x

x+
.
2 1 x 2 1 0 x 2 x x + (1 + x)
1 + x

Acum:
xy
> 0 y x > 0 (1 xy) y + x > 0

1 xy
1 xy
+x
( + x) y (1 + x) > 0 y <
,
1 + x
adev
arat conform ipotezei.
x
Not
a. Concluzia are loc si dac
a x, y (, 0] sau
< y 0 x < .
1x
2n+1 + 3
este reVIII.75. Determinati valorile lui n N pentru care fractia
3 2n + 1
ductibila.
Iurea, Iasi
Gheorghe

Solutie. Daca d | 2n+1 + 3 si d | 3 2n + 1, atunci d | 3 2n+1 + 3 2 (3 2n + 1),


adic
a d | 7. Cum fractia este reductibil
a, ea se simplific
a prin 7. Vom ar
ata c
a acest
lucru se realizeaz
a dac
a si numai dac
a n = M3 + 1.
Dac
a n = M3 , atunci 2n+1 + 3 = 2 23k + 3 = 2 (7 + 1)k + 3 = 2 (M7 + 1) + 3 =
M7 + 5. Daca n = M3 + 2, atunci 2n+1 + 3 = 23k+3 + 3 = (7 + 1)k+1 + 3 =
(M7 + 1) + 3 = M7 + 4. n sfrsit, pentru n = M3 + 1, avem c
a 2n+1 + 3 =
k
3k+2
n
2
+ 3 = 3 (7 + 1) + 3 = 4 (M7 + 1) + 3 = M7 si 3 2 + 1 = 3 23k+1 + 1 =
3 2 (7 + 1)k + 1 = 6 (M7 + 1) + 1 = M7 .
127

Clasa a IX-a
2
IX.71. Fie
a < b numere
reale; sa se determine x, y, z R pentru care 2z + b =
= 2yz + a, iar x a + b x = y.
Andrei Nedelcu si Lucian L
adunc
a, Iasi
Solutie. Din a doua ecuatie deducem c
a x [a, b], iar y 0. Cu inegalitatea
Cauchy-Schwarz, obtinem

2
2

y2 =
xa+ bx
xa +
bx
12 + 12 = 2 (b a) . (1)

nmultind prima relatie cu 2 si adumnd y 2 , g


asim c
a

y 2 + 4z 2 + 2b = 4yz + 2a + y 2 (2z y) + 2b 2a = y 2

(2)
y 2 = 2 (b a) + (2z y)2 2 (b a) .
p
Din (1) si (2) deducem c
a y = 2z = 2 (b a) si, cum avem egalitate n (1), atunci
1
x a = b x, deci x = (a + b).
2
x
y
z
IX.72. Fie x, y, z [1, +) asa nct
=
=
. Sa se arate ca
[x]
[y]
[z]
q
q
p
2
2
2
2
2
2
[x] + [y] + [z] + {x} + {y} + {z} = x2 + y 2 + z 2 .

Ce egalitate se obtine pentru x, y, z (, 1]?

x2
2

[x]

y2
2

[y]

Dan Pl
aesu, Iasi

x
y
z
=
=
= k [1, 2); atunci
[x]
[y]
[z]

Solutie. Fie

z2
2

[z]

x2 + y 2 + z 2
2

[x] + [y] + [z]

= k2

q
p
x2 + y 2 + z 2 = k [x]2 + [y]2 + [z]2
(1)

y [y]
z [z]
x [x]
=
=
=k1
[x]
[y]
[z]
{x}2

{y}2
2

[y]

{z}2
2

[z]

{x}2 + {y}2 + {z}2

= (k 1)
2
2
2
[x] + [y] + [z]
q
q
2
2
2
2
2
2
{x} + {y} + {z} = (k 1) [x] + [y] + [z] .

[x]

(2)

Scaznd membru cu membru relatiile (1) si (2) obtinem concluzia. Cnd x, y, z


(, 1], vom avea k (0, 1] si, relund rationamentul, gasim ca
q
q
p
2
2
2
2
2
2
[x] + [y] + [z] {x} + {y} + {z} = x2 + y 2 + z 2 .

IX.73. Fie a1 , a2 , . . . , an (0, ) cu a1 a2 an = 1. Pentru orice m N , sa


se arate ca are loc inegalitatea
m1
m
m
am
+ a2m1 + + am1
.
1 + a2 + + an a1
n

Marius Tiba, elev, Iasi


128

Solutia 1 (Titu Zvonaru, Com


anesti). Conform inegalit
atii mediilor, avem:
q
mn
m(mnn+1) m
m
m
m
a1
a2 am
am
n =
1 + + a1 + a2 + + an mn
q

mn
mn(m1)
m
n
= mn
a1
(a1 a2 an ) = mnam1
a1 a2 an .
1

Scriind nc
a n 1 inegalit
ati similare, dup
a adunare si mp
artire cu mn, obtinem
m1
n
m1
m
m
m
m1
a1 + a2 + + an a1
+ a2
+ + an
a1 a2 an ,
(1)

inegalitate echivalenta cu cea dorita, ntruct a1 a2 an = 1.

Solutia 2. Putem presupune f


ar
a a restrnge generalitatea c
a a1 a2 an ;
atunci am1
am1
anm1 si aplicam inegalitatea lui Cebsev:
1
2

m1
m
m
+ a2 am1
+ + an am1

am
1 + a2 + + an = a1 a1
n
2
m1

1
+ am1
+ + am1
.
(a1 + a2 + + an ) a1
n
2
n
1

a concluzia. Egalitatea
ns
a (a1 + a2 + + an ) n a1 a2 an = 1 si astfel rezult
n
se atinge pentru a1 = a2 = = an = 1.

Not
a. Demonstratia inegalitatii (1) ca n solutia a doua (folosind Cebsev) este
prezenta n RecMat 2/2006, pg. 160, n cadrul solutiei problemei L92.
IX.74. Fie a, b, c lungimile laturilor 4ABC si m, n (0, +). Consideram
punctele A0 , B 0 , C 0 astfel nct C (AA0 ), A (BB 0 ), B (CC 0 ) si CA0 = ma+n,
AB 0 = mb + n, BC 0 = mc + n. Sa se arate ca 4ABC si 4A0 B 0 C 0 au acelasi centru
de greutate daca si numai daca 4ABC este echilateral.
Dumitru Mihalache, Brlad
0 ma + n + b 0 mb + n + c


Solutie. Observam ca AA =
AC, BB =
BA, CC 0 =
b
c

mc + n + a mc + n + a
CA + AB . Atunci 4ABC si 4A0 B 0 C 0 au acelasi
CB =
a
a
centru de greutate dac
a si numai dac
a
0 0 0

AA + BB + CC = 0

mc + n + a mb + n + c
ma + n + b mc + n + a

AB +

AC = 0
a
c
b
a
mb + n + c
ma + n + b
mc + n
mb + n
ma + n
mc + n + a
=
=

=
=
. (1)
a
c
b
a
c
b
Dac
a a = b = c, relatia (1) este evident
a. Reciproc, s
a presupunem (1) ndeplinit
a.
Dac
a dou
a dintre numerele a, b, c sunt egale, din (1) rezult
a imediat c
a a = b = c.
n caz contrar, putem considera ca a < b < c si din (1) deducem ca
(mc + n) (mb + n)
(mb + n) (ma + n)
cb
ba
=

=
,
ac
cb
ac
cb
unde primul raport este negativ, iar al doilea pozitiv, ceea ce constituie o contradictie.
\ = 60
IX.75. Fie ABCD patrulater inscriptibil, {O} = AC BD, m(AOD)
MA
NC
OA
si punctele M (AB), N (CD). Notam k =
, r =
, p=
. Daca
MB
ND
OB
129

2 1
\
2k, ,
, sa se arate ca m(M
ON ) 6= 90 .
r 2r

Mihai Haivas, Iasi


Solutie. Cum ABCD este inscriptibil, avem c
a OA OC = OB OD. Atunci:

\
m(M
ON ) = 90 OM ON = 0

k
1
r
1
OA +
OB
OC +
OD = 0
k+1
k+1
r+1
r+1

OA OC + r OA OD cos 60 + k OB OC cos 60 kr OB OD = 0
1
(1 + rk) OA OC = (r OA OD + k OB OC)
2
OD
OB
k
2 (1 + rk) = r
+k
2 (1 + rk) = rp + .
(1)
OC
OA
p

2 1
\
Dac
a p 2k, ,
, evident c
a relatia (1) nu este verificat
a, deci m(M
ON ) 6= 90 .
r 2r

Clasa a X-a
X.71. Fie a, b, c lungimile laturilor unui triunghi. Sa se arate ca

a + 2 4 bc
b + 2 4 ac
c + 2 4 ab

> 3.
+
+

a+ c
b+ c
a+ b
LucianTutescu,
Craiova


Solu
t
ie.
Se
observ
a
c
a
a
<
b
+
c
(deoarece
a
<
b
+
c+
2
bc),
b
<
c+ a

si c < a
+
b. Prin urmare, dac
a a, b, c sunt lungimile laturilor unui triunghi,
atunci a, b, c sunt de asemenea lungimile
unui triunghi. Repetnd
laturilor

rationamentul anterior, deducem ca s


i 4 a, 4 b, 4 c sunt lungimile laturilor unui

a + 4 bc
ar, (1) a +
triunghi. De aici, rezult
a c
a
> 1 (1). (ntr-adev
b+ c

4
4
4
bc > b + c a >
b c 4 a > 4 b 4 c). Acum, nu ne ramne
dect sa remarcam faptul ca inegalitatea din enunt se obtine adunnd relatia (1) cu
inegalit
atile derivate din ea prin permut
ari circulare ale laturilor a, b si c.
X.72. Sa se rezolve sistemul
x2 log2 15 + y 2 log3 10 + z 2 log5 6 = 2 (xy + yz + zx) ,
x + y + z = 5.

Marius Damian, Br
aila
Solutie. Adunnd x2 + y 2 + z 2 n ambii membri ai primei ecuatii, obtinem
x2
y2
z2
2
+
+
=
x2 log2 30 + y 2 log3 30 + z 2 log5 30 = (x + y + z) , sau
log30 2 log30 3
log30 5
2
(x + y + z) . Pe de alta parte, conform inegalitatii lui Cauchy-Schwarz, avem

x2
y2
z2
(log30 2 + log30 3 + log30 5)
+
+
(x + y + z)2
log30 2 log30 3 log30 5
x2
y2
z2
2
+
+
(x + y + z) .
log30 2 log30 3 log30 5
130

Egalitatea are loc dac


a si numai dac
a x, y, z sunt proportionale cu log30 2, log30 3,
log30 5. Astfel, putem scrie
y
z
x+y+z
x
=
=
=
= 5,
log30 2
log30 3
log30 5
log30 2 + log30 3 + log30 5
de unde rezulta ca solutia sistemului este x = 5 log30 2, y = 5 log30 3, z = 5 log30 5.
X.73. Determinati functiile f : N
R cu proprietatea
ca pentru orice x, y N

x+y
f (x) + f (y)
astfel nct 3 | x + y, are loc egalitatea f
=
.
3
3
Eugenia Rosu, elev
a, Iasi
Solu
t
ie.
Fie
f
o
func
t
ie
cu
proprietatea
considerat
a
.
Pentru
x
=
y
=
0,
avem

f (0) + f (0)
0+0
f
=
, deci f (0) = 0. Dac
a lu
am pe rnd x = 2, y = 1 si
3
3
apoi x = 3, y = 0 n egalitatea dat
a, obtinem f (2) = 2f (1) si f (3) = 3f (1). Vom
demonstra prin inductie propozitia P (n): f (n) = nf (1), n N. Am ar
atat deja
ca P (0), P (1) si P (2) sunt adevarate. Presupunem acum ca P (k) este adevarata
oricare ar fi k {0, 1, . . . , m}, m N. Sa demonstram valabilitatea propozitiei
P (m + 1). Distingem trei cazuri.
I. Dac
a m + 1 = 3l + 1, l N, atuncilund x = 3l +
1 si y = 3l 1 n egalitatea
3l + 1 + 3l 1
f (3l + 1) + f (3l 1)
considerat
a (pentru c
a 3 | x+y), avem f
=
,
3
3
deci 3 (2l) = f (3l + 1) + f (3l 1). Cum 2l 3l m si 3l 1 m, rezulta, conform
ipotezei de inductie, c
a f (3l + 1) = 3f (2l) f (3l 1) = 6lf (1) (3l 1) f (1) =
(3l + 1) f (1) = (m + 1) f (1).
II. Daca m + 1 = 3l + 2, l N,
consideram x = 3l
+ 2 si y = 3l + 1 n egalitatea
3l + 2 + 3l + 1
f (3l + 2) + f (3l + 1)
din ipotez
a (3 | x + y) si obtinem f
=
, deci
3
3
f (3l + 2) = 3f (2l + 1)f (3l + 1) = 3 (2l + 1) f (1)(3l + 1) f (1) = (3l + 2) f (1) =
(m + 1) f (1).
III. Daca m + 1 = 3l
+ 3, l N, atunci
pentru x = 3l + 3 si y = 3l (3 | x + y),

3l + 3 + 3l
f (3l + 3) + f (3l)
relatia initial
a devine f
=
, de unde deducem c
a
3
3
f (3l + 3) = 3f (2l + 1) f (3l) = (3l + 3) f (1) = (m + 1) f (1).
Cu aceasta demonstratia propozitiei P (n) este terminata. Cum functia f (n) =
nf (1), n N satisface proprietatea ceruta, rezulta ca aceasta este solutia problemei.

X.74. Fie d1 , d2 doua drepte perpendiculare si l1 , l2 dreptele suport ale bisectoarelor celor doua perechi de unghiuri opuse formate de ele. Determinati 4ABC
cu A d1 , B d2 , C l1 si O, H l2 ( O, H cu semnificatiile uzuale).
Temistocle Brsan, Iasi
Solutie. Fata de reperul cartezian xOy cu d1 luat
a ca ax
a Ox si d2 ca Oy, fie
A (a, 0), B (0, b), C (c, c) (nu este esential faptul c
a l1 este luat
a ca prima bisectoare
b
a
a
a reperului). Mediatoarele laturilor [AB] si [BC] au ecuatiile y =
x
,
2
b
2
b+c
c
c
respectiv y
=
x
. Punnd conditia ca punctul lor comun s
a
2
bc
2
2

apartina dreptei l2 : x + y = 0, obtinem relatia (a + b) 2c ab = 0 (1). Apoi,


131

c
a
n
altimile din A si C au ecuatiile y =
(x a), respectiv y c = (x c), iar
b

c
b

conditia H l2 conduce la c a2 + b2 = 0 (2). Distingem patru cazuri:


(i) a + b = 0, c = 0, adic
a b = a, c = 0; atunci 4ABC este dreptunghic isoscel,
situat n cadranele II sau IV, avnd catetele pe axele reperului.
(ii) a + b = 0, a2 + b2 = 0, adica a = b = 0; atunci 4ABC este degenerat.
(iii) 2c2 ab = 0, c = 0; din nou 4ABC este degenerat.
(iv) 2c2 ab = 0, a2 + b2 = 0, adic
a tot triunghi degenerat.
X.75. a) Fie C . Daca z1 , z2 , z3 sunt trei numere complexe diferite astfel nct Re (
z1 ) = Re (
z2 ) = Re (
z3 ), atunci punctele de afixe z1 , z2 , z3 sunt
coliniare. Notam cu d dreapta pe care sunt situate aceste puncte.
b) Daca z10 , z20 , z30 sunt trei numere complexe diferite cu proprietatea ca Im (
z10 ) =
0
0
Im (
z2 ) = Im (
z3 ), atunci punctele cu afixele z1 , z2 , z3 sunt coliniare. Fie d0
dreapta ce le contine.
c) Sa se arate ca dreptele d si d0 sunt perpendiculare.
Constantin Cocea, Iasi
Solutie. a) Avem:

z1 +

z2 +

z3 +
z1
z2
z3
Re (
z1 ) = Re (
z2 ) = Re (
z3 )
=
=

2
2
2
z3 z1

z2 z1
=
= .
(1)
z2 z1
z3 z1

z2 z1
z2 z1
z2 z1
=
R, ceea ce nseamn
a
, adic
a
Din relatia (1), obtinem
z3 z1
z3 z1
z3 z1
ca punctele de afixe z1 , z2 , z3 sunt coliniare.
b) Deoarece

z0

z0

z0
z10
z20
z30
Im (
z10 ) = Im (
z20 ) = Im (
z30 ) 1
= 2
= 3

2i
2i
2i
z 0 z10

z20 z10
= 30
= ,
(2)
0
0
z2 z1
z3 z10

z 0 z10
R. Prin urmare, punctele cu afixele z10 , z20 , z30 sunt coliniare.
rezult
a c
a 20
z3 z10

z2 z1
z2 z1
z 0 z10
z2 z1
c) Din relatiile (1) si (2) deducem ca
,
= 20
sau
=

z2 z1
z2 z10
z20 z10
z20 z10
z 0 z10
iR+ , asadar dreptele d si d0 sunt perpendiculare.
de unde rezulta ca 20
z3 z10

Clasa a XI-a
XI.71. Fie sirul (xn )n1 , unde x1 > 1, xn+1 =
lim xnn .

xn 1
, n N . Calculati
ln xn

Dan Popescu, Suceava


x1
Solutie. Din inegalitatea 1 <
< x, x (1, ), rezulta ca (xn ) este
ln x
h
in(xn 1)
1
. Folosind
strict descresc
ator si are limita 1. Scriem xnn = (1 + xn 1) xn 1
132

criteriul StolzCesaro,

1
1
1
xn ln xn xn + 1
=
lim
= lim

= lim
n n (xn 1)
n xn+1 1
n (x 1)2 (x 1) ln x
xn 1
n
n
n
x ln x x + 1

= ). Urmeaz
a c
a xnn e0 = 1.
(x 1)2 (x 1) ln x
XI.72. Este posibil ca o functie f : R R, care verifica 1+f (x)+f (x)f (1+x) = 0,
x R, sa fie continua pe R?
Dorin M
argidanu, Corabia
Solutie. R
aspunsul este negativ. Vom demonstra c
a f nu are proprietatea lui
Darboux. Fireste f (x) 6= 0, x R. Daca f ar avea proprietatea lui Darboux,
atunci ar fi negativ
a pe R (pozitiv
a pe R nu poate fi!): f (x) < 0, x R. Deducem
imediat c
a 1 + f (1 + x) > 0, x R, deci f (x) > 1, x R. Pe de alt
a parte,
din 1 < f (x) < 0, x R, rezult
a c
a 0 < f (x) + 1 < 1, x R, adic
a 0 <
f (x) f (x + 1) < 1, x R, contradictie cu f (x) < 0, x R.
XI.73. Fie f : [0, 1] R o functie nenula de clasa C k+1 pe [0, 1], k N , astfel
ca f (0) = f (1) = 0. Daca pentru orice 1 j k exista aj {0, 1} astfel nct
f (j) (aj ) = 0, atunci exista x1 , x2 (0, 1) astfel ca f (k+1) (x1 ) f (k+1) (x2 ) < 0.
Gheorghe Morosanu si Paul Georgescu, Iasi
Solutie. Vom tine seama c
a, dac
a g : [0, 1] R este de clas
a C 1 pe [0, 1], g 0 are
semn constant pe (0, 1), iar g (0) = 0 sau g (1) = 0, atunci g pastreaza semn constant
pe [0, 1] si n plus este monoton
a pe [0, 1]. Dac
a presupunem c
a f (k+1) (x) 0,
(k)
x (0, 1), aplicnd succesiv rezultatul precedent pentru g = f , g = f (k1) , . . . ,
g = f obtinem c
a f este monoton
a pe [0, 1]. Cum f (0) = f (1) = 0 rezult
a c
af 0
pe [0, 1], contradictie; deci exista x1 (0, 1) astfel ca f (k+1) (x1 ) < 0. Analog exista
x2 (0, 1) astfel ca f (k+1) (x2 ) > 0, de unde concluzia.
XI.74. Fie a, b, c R, a 6= 0, n 2N . Urmatoarele afirmatii sunt echivalente:
(1) b2 4ac 0,
(2) det aA2 + bA + cIn 0, A Mn (R).
Marian Urs
arescu, Roman
Solutie. Pentru (1) (2) avem succesiv:

2
b
c
n
2
det aA + bA + cIn = a det A + A + In =
a
a
"
#
2

b
4ac b2
= an det A + In +
In = an det X 2 + Y 2 0,
2
2a
4a

4ac b2
b
unde X = A + In , iar Y =
In ; X, Y Mn (R), XY = Y X.
2a
2a
Pentru (2) (1) sa presupunem ca b2 4ac > 0. Notnd cu x1 , x2 radacinile
reale si distincte ale ecuatiei ax2 +bx+c = 0, vom considera dou
a numere (x1 , x2 )
si (, x1 ) (x2 , ). Fie

a + b + c
0

0 ... 0
2

a + b + c
0
... 0

; aA2 + bA + cIn =
A=
.

..
. . . . . . . . . . . .

.
0
0 ...
0
a 2 + b + c
(din lim

x1

133

n1

Acum det aA2 + bA + cIn = a2 + b + c a 2 + b + c
< 0, tinnd seama
de alegerea numerelor si , de semnul functiei de gradul al doilea si de faptul ca n
este par. Avem deci contradictie, ceea ce ncheie demonstratia.
XI.75. Fie A, B M2 (C). Sa se arate ca daca AB BA comuta cu A si cu
B, atunci AB = BA.
Dorel Mihet, Timisoara
Solutie. Vom utiliza urmatorul rezultat: "Fie A M2 (C) si multimea C (A) =
{X M2 (C) | AX = XA}. Atunci:
a) Dac
a A = kI2 , k C, atunci C (A) = M2 (C);
b) Daca A 6= kI2 , k C atunci C (A) = {A + I2 | , C}".
Sa presupunem ca AB BA nu este de forma I2 . Atunci , , , C astfel
nct A = (AB BA) + I2 si B = (AB BA) + I2 . Dac
a = 0 atunci
1

A = I2 , deci AB BA = O2 , absurd. Deci 6= 0 si atunci AB BA = A I2 ,

2
deci B =
a AB = BA = aA + bA,
A I2 + I2 = A + bI2 , ceea ce implic

adica AB BA = O2 , absurd. Astfel, exista C astfel nct AB BA = I2 .


Cum tr (AB BA) = 0, obtinem = 0, adic
a AB = BA.

Clasa a XII-a
XII.71. Fie f : [a, b] R o functie derivabila cu derivata continua. Sa se arate
Rb
ca lim a f (x) sin nx dx = 0.
n
Dan Radu, Bucuresti
Solutie. Integrnd prin parti, ajungem la relatia
Z b
Z
f (a) cos na f (b) cos nb
1 b 0
f (x) sin nx dx =
f (x) cos nx dx.
+
n
n a
a
R

Daca M = sup |f (x)|, M 0 = sup |f 0 (x)|, obtinem ca 0 a f (x) sin nx dx


x[a,b]

x[a,b]

2M + M 0 (b a)
b

. Rezult
a c
a lim a f (x) sin nx dx = 0, de unde concluzia.
n
n
XII.72. Fie functia f : [0,
1] R,
indefinit derivabila pe [0, 1], cu proprietatea
ca exista M > 0 astfel nct f (n) (x) M , x [0, 1] si n N. Aratati ca:
a) P
p
k+1
(n + p) (n + p 1) (n + p k + 2) f (n+pk) (1)
k=1 (1)
lim
= 0, p N ;
p+1
n
(1)
(n + p) (n + p 1) (n + 1)

b)

lim

Pp

k=1

(1)k+1 (n + p) (n + p 1) (n + p k + 2) f (n+pk) (1)


(1)

p+1

(n + p) (n + p 1) (n + 1)

1
0

xn f (n) (x) dx, n N.


Ovidiu Pop, Satu Mare
134

Solutie. Integrnd prin p


arti, obtinem succesiv:
Z 1
Z 1
1

xn+p f (n+p) (x) dx = xn+p f (n+p1) (x) (n+p) xn+p1 f (n+p1)(x) dx =


0
0
0

Z 1
1

= f (n+p1) (1)(n+p) xn+p1 f (n+p2) (x) (n+p1) xn+p2 f (n+p2) (x) dx =


0

(n+p1)

(n+p2)

(1) (n + p) f
(1) + (n + p) (n + p 1)

Z 1
1

n+p2 (n+p3)
n+p3 (n+p3)
f
(x) (n + p 2)
x
f
(x) dx
x
=f

si asa mai departe. Deducem c


a
Z 1
xn+p f (n+p) (x) dx =
0

p
X

(1)

k+1

k=1

(n + p) (n + p 1) (n + p k + 2) f (n+pk) (1) +

+ (1)p (n + p) (n + p 1) (n + 1)
Aratam ca
lim

xn f (n) (x) dx,


0

n, p N, p 6= 0. (1)

xm f m (x) dx = 0.

(2)

R1
M
a c
a
Din f (m) (x) M , x [0, 1], m N, rezult
0 xm f (m) (x) dx
m+1
M
, m N si trecnd la limit
a, obtinem relatia (2).
m+1
Din (1) si (2) rezult
a concluzia problemei.
XII.73. Sa se arate ca
Z 1
Z 1
n
n
(1 + x) + (1 x)

1
ln (1 + x)
ln
dx
=
n
dx = n ln 2, n N .
2
n
2
1+x
1+x
8
0 1+x
0
Sa se calculeze apoi

/4
R
0

ln

1 + tg3 x
dx.
1 + 3 tg2 x

Gabriel Necula, Plopeni


1y
Solutie. Cu substitutia x =
avem succesiv
1+y
Z 1
Z 1
n
n
(1 + y) + (1 y)
1
2
n
ln
(1
+
x
)
dx
=
ln
dy =
n
2
2
2
(1 + y)
0 1+x
0 (1 + y) + (1 y)
Z 1
Z 1
1
1
n
n
n
=
ln
((1
+
y)
+
(1

y)
)
dy

ln (1 + y) dy
2
2
1
+
y
1
+
y
0
0
Z 1
Z 1
1
1
ln ((1 + x)n + (1 x)n ) dx
ln (1 + xn ) dx =
2
2
0 1+x
0 1+x
Z 1

ln (1 + x)
dx = n ln 2
=n
2
1+x
8
0
135

Z 1
(1 + x)n + (1 x)n

1
ln (1 + x)
ln
dx
=
n
dx = n ln 2.
2
n
2
1
+
x
1
+
x
1
+
x
8
0
0
R1 ln (1 + x)

dx =
ln 2.) Pentru n = 3 avem
(Presupunem cunoscut rezultatul
1 + x2
8
0

3
3
R1 1
R1 1
2 1 + 3x2
(1 + x) + (1 x)

ln
dx = 3 ln 2, sau
ln
dx = 3 ln 2,
2
3
2
3
1
+
x
1
+
x
8
1
+
x
1
+
x
8
0
0
adic
a
Z 1
Z 1
1 + 3x2

1
ln 2
ln
dx
=
3
dx = 3 ln 2 ln 2 = ln 2.
ln
2

2
3
2
1
+
x
1
+
x
8
1
+
x
8
4
8
0
0
Z

/4
R

1 + tg3 x
2 dx = 8 ln 2.
1 + 3 tg x
0
XII.74. Fie (G, ) un grup comutativ cu proprietatea ca exista n N astfel nct
din xn = y n rezulta x = y, unde x, y G. Daca f , g sunt doua endomorfisme ale
lui G, atunci ecuatia f (x) = g x1 are solutie unica daca si numai daca functia
h : G G, h (x) = f (xn ) g (xn )este injectiva.
D. M. B
atine
tu-Giurgiu, Bucuresti
Solutie. Sa presupunem ca ecuatia f (x) = g x1 are solutie unica (evident
aceasta va fi x = e) si s
a ar
at
am c
a h este injectiv
a. Succesiv avem:
Cu substitutia x = tg t rezulta ca

ln

h(x) = h(y) f (xn ) g(xn ) = f (y n ) g(y n ) g(xn ) (g(y n )) = (f (xn ))

g(xn ) g y n = f xn f (y n )

1
1
g xn y n = f xn y n = f y n xn
= f xn y n
,

f (y n )

de unde deducem ca xn y n = e xn = y n x = y, adica h este imjectiva.


Reciproc,
dac
a h este injectiva, sa presupunem ca exista a G \ {e} astfel nct
f (a) = g a1 . Dar h (an ) = f (an ) g (an ) = g (an ) g (an ) = g (an an ) = g (e) = e,
prin urmare h (an) = h (en ), ceea ce contrazice injectivitatea lui h. Urmeaz
a c
a
ecuatia f (x) = g x1 are numai solutia x = e.
XII.75. Fie A Mn (C) de rang r si S = {B Mn (C) | ABA = On }. Aratati
ca S este subspatiu vectorial n Mn (C) si ca dim S = n2 r2 .
Adrian Reisner, Paris
Solutie. Faptul c
a S este subspatiu vectorial n Mn (C) este o simpl
a verificare.

I 0
Matricea A fiind de rang r, exist
a P, Q GLn (C) astfel nct P 1 AQ = r
.
0 0

Acum
= On P 1ABAQ = On P 1AQ Q1BP P1AQ = On
B S ABA

I 0


Ir 0
r
, si fiind
= On , unde am notat Q1 BP =
0 0

0 0

Ir 0

Ir 0
matrice p
atratice de ordin r, respectiv n r. ns
a
=
0 0

0 0

Ir 0
0
0
a B S Q1 BP =
, de unde = Or . Deducem c
0
0

0
0
0

0
, adica dim S = n2 r2 .

136

Solutiile problemelor pentru preg


atirea concursurilor
din nr. 2 / 2006
A. Nivel gimnazial
G106. Fie m N \ {0, 1} si k N, k m. Pentru fiecare x N, consideram
propozitiile: x > 1; x > 2; . . . ; x > m. Aflati x N pentru care k din cele m
propozitii sunt adevarate, iar celelalte m k sunt false.
Maria Mihet, Timisoara
Solutie. Pentru k = 0, toate propozitiile sunt false. n particular, x > 1 este
falsa si daca x {0, 1}. Pe de alta parte, pentru x = 0 si x = 1 evident ca toate cele
m propozitii sunt false, deci pentru k = 0 r
aspunsul este x {0, 1}.
Pentru 1 k m 1, cele k propozitii adev
arate trebuie s
a fie x > 1; x > 2; . . . ;
x > k, iar cele false vor fi x > k + 1; . . . ; x > m. Avem asadar x > k si x k + 1,
deci x = k + 1.
Cnd k = m, toate propozitiile trebuie s
a fie adev
arate. n particular, x > m va
fi adev
arat
a, deci x {m + 1, m + 2, . . . }.

G107. Multimea A N de cardinal n N are proprietatea ca, oricare ar fi


patru elemente ale sale, putem alege doua cu suma 22006 + 1. Aflati valoarea maxima
a lui n.
Dan Nedeianu, Dr. Tr. Severin
Solutia 1 (a autorului). Pentru m = 6, putem considera A = {1, 2, 3, 22006 2,
22006 1, 22006 }. Sa presupunem prin absurd ca ar exista multimi A cu m 7.
Conform principiului cutiei, exist
a o submultime B A de cardinal 4 cu toate
elementele de aceeasi paritate. Suma oric
aror dou
a numere din B va fi par
a, deci
diferita de 22006 + 1.
Solutia 2 (Marius Tiba, elev, Iasi). Pentru a demonstra ca nu putem avea
m 7, putem proceda astfel: dac
a n 7, exist
a n A fie 4 elemente cel mult egale
cu 22005 , fie 4 elemente cel putin egale cu 22005 + 1. Alegnd dou
a numere din cele
patru, obtinem o suma fie mai mare, fie mai mica dect 22006 + 1.

G108. Fie m, n N . Sa se arate ca multimea numerelor ntregi de modul cel


mult egal cu n poate fi partitionata n m submultimi cu aceeasi suma a elementelor,
daca si numai daca n + 1 m.
Marian Tetiva, Brlad
Solutie. Sa presupunem mai nti ca n+1 m. Sa notam M = {x Z | |x| n} =
{x Z | n x n} si Mk = {k, k} pentru k = 0, n. Clasele partitiei pot fi
atunci multimile M0 , M1 , . . . , Mm2 , Mm1 Mm Mn ; este evident c
a suma
elementelor din fiecare astfel de multime este aceeasi si anume 0.
Reciproc, sa presupunem ca avem partitia M = A1 A2 Am unde suma
elementelor din fiecare multime Aj este aceeasi, j = 1, m. Se obtine f
ar
a dificultate
c
a aceast
a sum
a trebuie s
a fie 0 (deoarece si suma elementelor multimii M este 0).
Observam atunci ca fiecare multime a partitiei, cu exceptia celei care-l contine pe 0,
are cel putin doua elemente. (Orice element nenul al multimii respective trebuie sa
fie adunat cu un element nenul pentru a da suma zero). Avem atunci c
a 2n + 1 =
m
P
card M =
card Ak 2 (m 1) + 1, de unde deducem ca n + 1 m.
k=1

137

G109. La un concurs se dau sase probleme evaluate cu 1, 2, 3, 4, 5, repectiv


6 puncte. Daca un elev nu rezolva o problema, primeste 1 punct; daca o rezolva,
primeste punctajul corespunzator. Fiecare elev obtine macar 11 puncte. Sa se arate
ca o problema a fost rezolvata de cel putin o treime dintre elevi.
Gabriel Dospinescu, student, Paris
Solutie. Fie n numarul elevilor; punctajul total este cel putin 11n. Fixam o
problema, de punctaj a, si notam cu A (a) multimea elevilor care rezolva aceasta
problem
a. Astfel, exact |A (a)| elevi primesc a puncte si exact n |A (a)| primesc 1
punct, deci punctajul obtinut la aceast
a problem
a va fi (a 1) |A (a)| + n. Punctajul
total va fi
X
[(a 1) |A (a)| + n] max |A (a)| (1 + 2 + 3 + 4 + 5) + 6n,
a

deci max |A (a)| 15 + 6n 11n, de unde concluzia problemei.


a

G110. Fie multimile A = {k + n | k Z, n N} si B = (0, 1/10). Aratati ca


A B este infinita.
Petru Asaftei,

Iasi
Solutia 1. Consider
am C = x R | x = m2 + 1 m, m N, m 10 . Evi1
1
1
<
dent c
a C A si, dac
a x C, atunci 0 < x =

, prin
2
m
10
m +1+m
urmare C A B. S
a mai ar
at
am c
a multimea
a presupunem
a. Dac
C este infinit
contrariul, exista s 6= t, s, t 10 astfel nct s2 + 1 s = t2 + 1 t. Obtinem
succesiv:
p
p
p
t s = t2 + 1 s2 + 1 s2 2st + t2 = t2 + s2 + 2 2 (t2 + 1) (s2 + 1)
p

(t2 + 1) (s2 + 1) = st + 1 t2 + 1 s2 + 1 = (st + 1)2 s2 + t2 2st = 0,


prin urmare t = s si ajungem astfel la o contradictie. R
amne c
a C este infinit
a,
prin urmare A B este infinit
a.

Solutia 2 (Marius Tiba, elev, Iasi). Pentru fiecare k ntreg negativ divizibil cu

k
1
5, consideram n = k 2 ; ar fi suficient sa aratam ca 0 < k + n <
. Avem:
5
10
r
r
k
k
k
k2 + k > 0 k2 > k k 2 > k 2 k < 0;
5
5
5
r
2
k
k
1
(1 10k)
k2 + k <
k2 <
100k 2 20k < 100k 2 20k + 1,
5
10
5
100
ambele relatii la care am ajuns fiind evident adev
arate.

G111. Fie 0 < a < b numere reale date


si x,y [a, b]. Daca s = x + y, p = xy,
ab s2 + ab
sa se afle maximul expresiei E = p +
.
p
Vlad Emanuel, elev, Sibiu
Solutie. Cum x [a, b], avem ca (x a) (x b) 0 x2 (a + b) x + ab 0,
deci (a + b) x x2 + ab. Scriem o inegalitate analoag
a n y si nmultind membru cu
membru, obtinem:

(a + b)2 xy x2 + ab y 2 + ab (a + b)2 p p2 + ab s2 2p + a2 b2
138

ab s2 + ab
a2 + b2 + 4ab E a2 + b2 + 4ab.
p
Pentru a se atinge egalitatea, trebuie ca x2 (a + b) x + ab = y 2 (a + b) y + ab = 0,
fapt care se realizeaz
a pentru (x, y) {(a, a) ; (a, b) ; (b, a) ; (b, b)}. Rezult
a c
a Emax =
a2 + b2 + 4ab.
G112. Daca a, b, c sunt lungimile laturilor triunghiului ABC, atunci
q
q
q
2
2
2
2
2
(a + b) c + (b + c) a + (c + a) b2 < 2 (a + b + c) .
Zdravko Starc,
Vr
sac, Serbia si Muntenegru
q
q
p+

a dou
a
Solutie. Evident c
a (a + b) c2 < (a + b) = a + b; scriind nc
inegalit
ati similare si sumnd, obtinem concluzia.
Not
a. O mai bun
a evaluare a membrului stng al inegalit
atii, este indicat
a de
Titu Zvonaru:
Daca a, b, c sunt lungimile laturilor unui triunghi, atunci
q
q
q

(a + b)2 c2 + (b + c)2 a2 + (c + a)2 b2 3 (a + b + c) .

G113. Fie segmentul [AB] de mijloc O si semicercurile C1 si C2 de diametre


[AB], respectiv [AO] situate n acelasi semiplan fata de AB. Perpendiculara n
C (AO) pe AB intersecteaza C1 n E si C2 n D. Daca AD C1 = {F }, sa se
arate ca AE este tangenta cercului circumscris 4DEF .
Alexandru Negrescu, elev, Botosani
\ + m(DF
\
E
Solutie. Cum m(DCB)
B) = 90 + 90 =
180 , rezult
a c
a punctele B, F , D, C sunt conciclice.
F
Scriind puterea punctului A fata de cercul determinat
D
de ele, obtinem ca AC AB = AD AF . Pe de alta
2
parte, AC AB = AE din teorema catetei n 4AEB.
Deducem c
a AE 2 = AD AF , de unde concluzia.
A
CO
B
\ = 60 . Daca
G114. Fie ABCD un paralelogram care nu este romb cu m(BAD)
M, N (AC), P (BC) si Q (CD) sunt astfel nct [BM , [DN , [AP si [AQ
\ ADC,
\ BAC
\ si respectiv DAC,
\ atunci M P este
sunt bisectoarele unghiurilor ABC,
perpendiculara pe N Q.
Andrei Nedelcu, Iasi
Solutie. Lu
am R (AD) astfel nct [CR
C
B
P
\ adica cea de a
este bisectoarea unghiului ACD,
S N
treia bisectoare interioara a triunghiului ACD.
Deoarece triunghiurile ABC si ADC sunt conQ
gruente si simetrice fata de centrul O al paralelM O
ogramului ABCD, rezulta ca [AP ] este paralel si
congruent cu [CR]. De asemenea, din congruenta A
D
R
si simetria triunghiurilor AM P si CN R rezult
a
c
a P M este paralel
a cu RN . Prin urmare, dac
a vom ar
ata c
a N Q RN , atunci
rezulta ca N Q M P . Punctul R este centrul cercului exnscris, tangent laturii [N D]
\
n triunghiul CDN . Prin urmare, [N R este bisectoarea unghiului AN
D. Analog, Q
este centrul cercului exnscris, tangent laturii [N D] n triunghiul AN D; rezulta ca
139

\
\
\
[N Q este bisectoarea unghiului CN
D. Deoarece unghiurile AN
D si CN
D au vrful
comun N si laturile [N A si [N C n prelungire, rezulta ca bisectoarele lor [N Q si [N R
sunt perpendiculare, ceea ce ncheie rezolvarea.
G115. Fie patratul M N P Q nscris n patratul ABCD, M (AB), N (AD),
P (CD), Q (BC) si fie {E} = P N AB, {F } = P Q AB. Notam cu S1 , S2 ,
S3 ariile patratului ABCD, patratului M N P Q, respectiv 4P EF . Sa se arate ca:
p
1
1
1
b) S3 S1 ;
c)
=

.
a) S1 S2 = 4 SAEN SBF Q ;
S3
S2 S1
Claudiu S
tefan Popa, Iasi
Solutie.
a) Din congruenta triunghiurilor
D P
C
AM N , BQM , CP Q si DN P (I.U.) rezulta ca AN =
Q
BM = CQ = DP si AM = BQ = CP = DM .
Not
am cu S aria 4AM N ; evident c
a 4S = S1 S2 .
N
Apoi, din asem
an
arile evidente 4AEN 4DP N si

2
SAEN
AN
EA
M B
F
4CP Q 4BF Q, obtinem ca
=
=
S
DN

2
CQ
S
=
, deci S 2 = SAEN SBF Q . Concluzia de la a) este imediata.
BQ
SBF Q
S
CQ
AN
=
; atunci SAEN = k 2 S, SBF Q = 2 , prin urmare
b) Fie k =
ND
BQ
k

2
S
1
2
.
S3 = S2 + 2S + SAEN + SBF Q = S2 + 2S + k S + 2 = S2 + S k +
k
k
1
Cum k + 2, rezulta ca S3 S2 + 4S, adica S3 S1 .
k
c) Cum S3 = SP EM + SP F M , avem succesiv:

1
P M P E sin 45
P M P F sin 45
2P E P F
PE PF =
+
PM =
;
2
2
2
PE + PF

2
PM
P E2 P F 2
=
BQ2 + BM 2 = M Q2 =
2
2
(P E + P F )

2
EF 2 + 2P E P F
1
EF
2
1
=

=
+

2
2
2
2
2
2
BQ + BM
PE PF
AM + BM
PE PF
PE PF

2
1
1
1
+

=
AD
S3
(AM + M B)2 2AM M B
1
1
1
1
1
1
1
1
1
=
+

2
AM

AN
AD
S
S
S

4S
S
S
S
S
3
3
1
1
3
2
1
AB 2 4
2

B. Nivel liceal
L106. Fie I centrul cercului nscris n 4ABC. Dreptele AI, BI si CI intersecteaza a doua oara cercurile circumscrise triunghiului BCI, CAI si ABI n A0 ,
B 0 , respectiv C 0 . Daca notam cu |XY Z| perimetrul 4XY Z, sa se demonstreze ca
140

CA
AB
BC
+
+
= 1.
0
0
|BCA | |CAB | |ABC 0 |
Titu Zvonaru, Com
anesti
Solutie. Not
am cu R raza cercului circumscris 4ABC. Patrulaterul BA0 CI este
b Aplicnd teorema sinusurilor
[ = 1 m(B).
inscriptibil, deci m(\
CA0 I) = m(IBC)
2
n 4AA0 C si n 4ABC, avem:
B
B
A
4R sin cos
AC
2R sin B
B
A0 C
2
2
=
=
=
= 4R cos ,
A
B
B
B
2
sin
sin
sin
sin
2
2
2
2
I
A
B
deci A0 C = 4R sin cos . Atunci:
B
C
2
2
A
B
A
C
|BCA0 | = 4R sin cos + 4R sin cos + a =
2
2
2

2
A
B
C
A
cos + cos + cos

= 4R sin
2
2
2
2
A
A
A
A
4R sin cos
cos
BC
2
2
2

.
=
=
A
B
C
A
A
B
C
|BCA0 |
cos + cos + cos
4R sin
cos + cos + cos
2
2
2
2
2
2
2
Egalnd analog celelalte doua rapoarte din concluzie, se obtine cerinta problemei.
Observatie. 1) Dac
a 4ABC este ascutitunghic si n locul punctului I consider
am punctul O, relatia r
amne valabil
a si constituie o parte a problemei 3103 din
Crux Mathematicorum 1/2006.
2) Relatia este adev
arat
a si pentru ortocentrul H n cazul triunghiului ascutitunghic.
Not
a. n aceeasi maniera a rezolvat problema Vlad Emanuel, elev, Sibiu.
L107. Fie M , N doua puncte situate n interiorul 4ABC, avnd distantele
pna la laturile AB, BC, CA egale cu 3, 2, 7, respectiv 92 , 5, 52 . Daca raza cercului
circumscris 4ABC este R = 8, sa se calculeze M N .
Vlad Emanuel, elev, Sibiu
Solutie. Fie A0 , B 0 , C 0 si A00 , B 00 , C 00 proiectiA
ile punctelor M , respectiv N pe laturile AB,
BC, CA. Consideram punctul I [M N ] astMI
C
fel
= 2 si fie M Q k BC, Q N A00 ,
B
IN
N B
0
0
I = PrBC I, {P } = II M Q. Avem c
a NQ =
IP
MI
2
I
00
0
N A M A = 5 2 = 3 si
=
= ,
C
NQ
MN
3
deci IP = 2, apoi II 0 = 2 + 2 = 4. Daca
M P
Q
I 00 = PrAC I, I 000 = PrAB I, analog se arata ca
00
000
II = II = 4, prin urmare I este centrul cerA I A
C
cului nscris n 4ABC, iar r = 4. Rezult
a c
a n B
inegalitatea lui Euler R 2r se atinge egalitatea si astfel 4ABC este echilateral.
141

Not
am x = A0 B, y = C 0 B. Din teorema lui Pitagora, BM 2 = BC 02 + C 0 M 2 =
BA02 + A0 M 2 . Apoi, cu teorema cosinusului, A0 C 02 = M C 02 + M A02 2M C 0 M A0
cos 120 = BC 02 +BA02 2BC 0 BA0 cos 60 . Obtinem
x2 +4 = y 2 +9,
astfel sistemul

8
7
3
3
,y=
.
x2 + y 2 xy = 19, cu unica solutie admisibila x =
3
3

0
Latura 4ABC echilateral este l = 8 3. Cum
lui BC, vom avea
I este mijlocul
0 0

8
MI
4
A
3
3
I
=
. S
tim ca 0 00 =
= 2, deci
ca BI 0 = 4 3, de unde A0 I 0 = 4 3
3
3
I A
IN

0 00
0 00
A A = 2 3. Din trapezul dreptunghic M A A N gasim M N = 21.
Not
a (Mihai Haivas). Dac
a M, N Int ABC au distantele pn
a la laturile
triunghiului egale cu a, b, c, respectiv a0 , b0 , c0 , iar raza cercului circumscris 4ABC
2 (ab0 a0 b)
, atunci conditia
este R =
a b a0 + b0
(ab0 a0 b) (b c b0 + c0 ) = (bc0 b0 c) (a b a0 + b0 )

este suficienta pentru ca triunghiul sa fie echilateral. n cazul nostru, cele doua relatii
sunt verificate.
L108. Sa se arate ca n orice 4ABC are loc inegalitatea

3 3
3A
(sin A + sin B + sin C) 4 3 sin2
.
2
12
Marian Tetiva, Brlad
Solutia 1 (Vlad Emanuel, elev, Sibiu). Deoarece

BC
A
A

cos
2 cos ,
sin B + sin C = 2 sin
2
2
2
2

A
cu egalitate cnd B = C = , r
amne s
a demonstr
am c
a
2
2

3A
3 3
A
sin A 2 cos 2 3 1 cos

2
2
6

3 3

A
A
A
A

A
2 sin cos 2 cos 2 3 2 3 cos cos + sin sin
.
2
2
2
2
6
2
6
2

A
A
Notam x = cos (0, 1), 1 x2 = sin ; avem de aratat ca
2
2

!
p
p

3
3
3 p
1 x2 .
3x 3 1 x2 x
2 x
2x 1 x2 2x
2
2
2

3
3
2
Daca x >
ramne ca 1 2 1 x , evident, iar daca x <
ramne 1
2
2

asi evident. Egalitatea se atinge n cazul triunghiului echilateral.


2 1 x2 , iar

Solutia 2 (a autorului). Mai general, vom demonstra c


a oricare ar fi x, y, z
(0, ), are loc inegalitatea
x + y 2z
x+y+z
sin x + sin y + sin z
8
x+y+z
sin

sin
sin2
.
(1)
3
3
3
3
12
142

Plec
am de la egalitatea imediat
a
x + y sin x + sin y
x+y
xy
sin

= 2 sin
sin2
,
(2)
2
2
2
4
pe care o rescriem sub formele
z + t sin z + sin t
z+t
zt
sin

= 2 sin
sin2
,
(3)
2
2
2
4
x+y
z+t
x+yzt
x + y + z + t sin 2 + sin 2
x+y+z+t
sin

= 2 sin
sin2
.
4
2
4
8
(4)
Din relatiile (2), (3) si (4) deducem c
a
x + y + z + t sin x + sin y + sin z + sin t
x+y
xy

= sin
sin2
+
4
4
2
4
x+y+z+t
zt
x+yzt
z+t
sin2
+ 2 sin
sin2
.
+ sin
2
4
4
8
Pentru x, y, z, t (0, ), primii doi termeni din membrul drept sunt nenegativi (egali
cu 0 cnd x = y, z = t), deci are loc inegalitatea
sin

x+yzt
x + y + z + t sin x + sin y + sin z + sin t
x+y+z+t

2 sin
sin2
.
4
4
4
8
x+y+z
, obtinem exact relatia (1). Egalitatea se atinge pentru
nlocuind aici t =
3
x+y+z
x = y, z =
, deci cnd x = y = z.
3
Cerinta problemei se obtine din (1) pentru x + y + z = . Egalitatea se atinge n
cazul triunghiului echilateral.

sin

L109. Se dau numerele reale pozitive subunitare a1 , a2 , . . . , a2n2 n , n N\{0, 1}.


Sa se demonstreze inegalitatea (sumarea se face prin permutari circulare)
X
2n 1
a2n1
1
<
.
2n1
2n1
2
n+1
a2
+ a3
+ . . . + a2n1
+
2n
+
1
2n2 n

Ioan S
erdean, Or
astie

, atunci
Solutie. Cum 1 > a2n+1
1
X
a2n+1
2

<

a2n+1
3

a2n1
1
<
+ + a2n+1
2n2 n + 2n + 1

a2n1
1
2n+1
2n+1
+ a3
+ + a2n+1
2n2 n + 2n + a1
+ a2n1
+ + a2n1
a2n1
1
2
2n2 n
.
2n+1
2n+1
a2n+1
+
a
+

+
a
1
2
2n2 n + 2n

a2n+1
2
=

Pe de alta parte, din inegalitatea mediilor M G M A, avem ca


q
2n+1 2n+1 2n1
(2n 1) a2n+1
+
1
+
1

(2n
+
1)
1 1 = (2n + 1) a2n1
a1
1
1
143

(1)

si nc
ati analoage pentru a2 , a3 , . . . , a2n2 n . Prin adunarea
a 2n2 n 1 inegalit
acestora membru cu membru, obtinem ca

2
2n+1
2n+1
(2n 1) a2n+1
+
a
+

+
a
2
1
2
2n n + 2 2n n

(2n + 1) a2n1
+ a2n1
+ + a2n1
1
2
2n2 n

a2n1
+ a2n1
+ + a2n1
1
2
2n2 n

a2n+1
+ a2n+1
+ + a2n+1
1
2
2n2 n + 2n

2n 1
.
2n + 1

(2)

Din (1) si (2) rezult


a inegalitatea dorit
a, care este strict
a ntruct (1) este strict
a.
L110. Fie a, b, c (0, ) si n, k N. Demonstrati inegalitatea

bn+k
cn+k
4n(a b)2
an+k
k
k
k
+
+

a
+
b
+
c
+
.
bn
cn
an
k(a2k + b2k + c2k )

(n leg
atur
a cu o problem
a propus
a la OBM 2005).
Titu Zvonaru, Com
anesti si Bogdan Ionita
, Bucuresti
Solutie. Consideram polinomul P (t) = ktn+k (n + k) tk + n, t > 0. Folosind
schema lui Horner, obtinem c
a

2 n+k2
+ + 2nt + n n (t 1)2 .
P (t) = (t 1) kt
a
n inegalitatea P (t) n (t 1)2 facem t = ; obtinem
b
an+k
ak
n (a b)2
an+k
n (a b)2
k
k
k n+k (n + k) k n +

(n
+
k)
a

nb
+
.
b
b
b2
bn
b2k
b
c
Scriem nca doua relatii analoage pentru t = si t = si le sumam:
c
a
!

n+k
2
2
2
k

a
bn+k
cn+k
(b

c)
(c

a)
(a

b)
+ n + n
+ 2k +
.
k a + bk + ck +n
k
bn
c
a
b2k
c
a2k
ns
a, din inegalitatea Cauchy-Schwarz, avem c
a
2

(b c)
(c a)
(a b + c b + a c)
4 (a b)
(a b)
+ 2k +

= 2k
,
b2k
c
a2k
a2k + b2k + c2k
a
+ b2k + c2k
de unde concluzia problemei.
L111. Se dau m numere naturale distincte din multimea {1, 2, . . . , n}. Sa se
arate ca putem alege cteva dintre ele, cu suma S, astfel nct

m(m + 1)
0S
n + 2n m.
2
Adrian Zahariuc, elev, Bac
au
Solutie. Fie a1 < a2 < < am cele m numere date. Notam cu j indicele
m (m + 1)
si cu i indicele maxim pentru
minim pentru care a1 + a2 + + aj
2
m (m + 1)
care ai + ai+1 + + aj
. Pentru S = ai + ai+1 + + aj este atunci
2
m (m + 1)
ndeplinit
a conditia 0 S
.
2
144

m (m + 1)
, deci
2
m (m + 1)
S ai +
1.
(1)
2
Din minimalitatea lui j, deducem ca
m (m + 1)
ai + ai+1 + + aj aj > a1 + a2 + + ai1 .
a1 + a2 + + aj1 <
2
nsa ak k, k N , deci

i (i 1)
n 1 a1 + a2 + + ai1 1 + 2 + + (i 1) =
i 2n + 1.
2
Apoi, ak n m + k, k {1, 2, . . . , m}; n particular

ai n m + i n m + 2n + 1.
(2)

m (m + 1)
Din relatiile (1) si (2) obtinem c
aS
+ n m + 2n, ceea ce ncheie
2
rezolvarea.
L112. Pentru n N, notam cu a(n) numarul modurilor n care n se poate scrie
ca suma a unui numar par de puteri ale lui 2 si cu b(n) numarul modurilor n care
n se poate scrie ca suma a unui numar impar de puteri ale lui 2. Sa se arate ca
a(n) = b(n), n 2.
Adrian Zahariuc, elev, Bac
au
Solutia 1. Fie N multimea tuturor sirurilor de numere naturale care au un
num
ar finit de termeni nenuli. Pentru fiecare n N, definim
Din maximalitatea lui i, avem S ai

Mn = {(a0 , a1 , . . . ) N | n = a0 + 2a1 + 4a2 + } .

Atunci se observ
a c
a
a(n) = |Mn (A)| ,
b(n) = |Mn (B)| ,

Evident ca

unde Mn (A) = {(a0 , a1 , . . . ) Mn | a0 + a1 + = par} ;


unde Mn (B) = {(a0 , a1 , . . . ) Mn | a0 + a1 + = impar} .

a (2n + 1) = a (2n) si b (2n + 1) = b (2n) .


(1)
S
a evalu
am a (2n). Este clar c
a a0 este tot timpul par. Cardinalul lui Mn (A) cu
a0 = 2k, k {0, 1, 2, . . . , n}, este b (n k). Atunci
a (2n) = b (n) + b (n 1) + + b (0) a (2n) = a (2n 2) + b (n) .

(2)

Analog se arata ca

b (2n) = b (2n 2) + a (n) .


(3)
Folosind relatiile (1), (2) si (3), se arat
a prin inductie c
a a (n) = b (n), n 2.
Not
a. Asemanator a rezolvat problema Vlad Emanuel, elev, Sibiu.
Solutia 2 (a autorului). Vom folosi functii generatoare. Fie
1
f (x) = 1 x + x2 x3 + =
.
1+x
Atunci, din definitia numerelor a (n) si b (n) dat
a n prima solutie,

X

1
1
1
(a (n) b (n)) xn = f (x) f x2 f x4 =

= 1 x,

2
1 + x 1 + x 1 + x4
n=0
145

de unde a (n) b (n) = 0, n 2, ceea ce trebuia demonstrat.


L113. Determinati numerele reale a, b pentru care multimea A = {an + bn |
n N } este finita.
Gheorghe Iurea, Iasi
Solutia 1 (a autorului). Not
am Sn = an + bn , n N . Deoarece A este finit
a,
ea este marginita: exista m = min A, M = max A, deci m Sn M , n N .
Pentru |a| > 1, avem c
a

M 1
S2n = a2n + b2n a2n = 1 + a2 1 1 + n a2 1 > M, n > 2
.
a 1
Din contradictia obtinut
a rezult
a c
a |a| 1. La fel se arat
a c
a |b| 1.
Dac
a |a| < 1, a 6= 0 si |b| 1, atunci a2n+2 < a2n si b2n+2 b2n , deci S2n+2 <
S2n , n N . Rezulta astfel ca multimea A este infinita, fals. Analog se obtine
contradictie daca |b| < 1, b 6= 0. Deducem ca a, b {1, 0, 1} si pentru aceste valori
se verific
a imediat c
a A este finit
a.
Solutia 2 (Vlad Emanuel, elev, Sibiu). Daca A, B sunt multimi finite cu
|A| = m, |B| = n, atunci AB = {x y | x A, y B} si AB = {xy | x A, y B}
sunt tot finite, avnd cel mult
Fie xn = an +bn ; cum A este finit
2 mn elemente.

a,

multimile {x2n | n N } si xn |nn N sunt finite.

x
|
n

N
Atunci
x
=
2n
n
o
2

{2an bn g | n N } este finit


a, apoi (an bn ) | n N este finit
a, deci {an bn | n N }
(an + bn ) + (an bn )
este finita. Cum an =
, deducem ca {an | n N } este finita,
2
de unde se deduce imediat ca a {1, 0, 1}. Analog obtinem ca b {1, 0, 1} si
pentru aceste valori se verific
a imediat c
a A este finit
a.
L114. Consideram o parabola si doua drepte secante parabolei, paralele ntre ele,
dar neparalele cu axa de simetrie a parabolei. Folosind doar rigla negradata, sa se
construiasca tangenta la parabola care este paralela cu dreptele date.
Titu Zvonaru, Com
anesti
Solutie. Fie P : y 2 = 2px parabola data, iar d si d0 cele doua drepte. Fie
P (x1 , y1 ), Q (x2 , y2 ) intersectiile lui d cu P, cu y12 = 2px1 , y22 = 2px2 , iar M (a, b)
punctul c
autat n care tangenta la parabol
a este paralel
a cu dreapta P Q. Tangenta
p
n M are ecuatia by = px + pa si panta , iar dreapta P Q are panta
b
2p (y1 y2 )
2p (y1 y2 )
2p
y1 y2
=
=
=
.
x1 x2
2px1 2px2
y12 y22
y1 + y2
y1 + y2
Din egalitatea pantelor obtinem ca b =
, deci M are aceeasi ordonata cu
2
mijlocul segmentului [P Q].
Folosind teorema lui Ceva, se arata ca putem construi mijlocul unui segment daca
avem dat
a o dreapt
a paralel
a cu el, apoi putem construi o nou
a paralel
a la segment
printr-un punct dat, ambele constructii efectundu-se doar cu sigla negradat
a (vezi,
de exemplu, A.Tth - Notiuni de teoria constructiilor geometrice, E.D.P., 1963).
Cu segmentul [P Q] si dreapta d0 , construim mijlocul M al lui [P Q]. Daca d0
taie parabola n P 0 si Q0 , construim mijlocul M 0 al lui [P 0 Q0 ]. Dreapta M M 0 taie
parabola n punctul T (ea fiind paralel
a cu axa de simetrie a parabolei) si acum
paralela prin T la d este tangenta dorita.
146

Not
a. Solutie corect
a s-a primit de la Vlad Emanuel, elev, Sibiu.
L115. Determinati P R[X], grad P 2, astfel nct functia f : R R,
f (x) = p({x}) + {p(x)} sa fie periodica (unde p este functia polinomiala atasata lui
P , iar {} desemneaza partea fractionara).
Paul Georgescu si Gabriel Popa, Iasi
Solutie. Vom arata ca nu exista polinoame P cu proprietatile din enunt. Presupunem contrariul si fie T R+ o perioada a lui f . Cum discontinuitatile posibile
ale lui f sunt numere ntregi si solutiile ecuatiei p (x) = n, n Z, rezult
a c
a f are n
[0, T ] un num
ar finit de puncte de discontinuitate fie acesta N . Functia f va avea
acelasi numar de puncte de discontinuitate n orice interval [kT, (k + 1) T ], k Z.
Fie n = grad P ; atunci g : R R, g (k) = p ((k + 1) T ) p (kT ) este functie
polinomial
a de grad n 1 n k, deci lim g (k) = . Pentru fixarea ideilor, s
a
k

presupunem c
a lim g (k) = +. Exist
a atunci k Z pentru care g (k) N +[T ]+2.
k

Cum p ((k + 1) T ) p (kT ) N + [T ] + 2, ntre p (kT ) si p ((k + 1) T ) se vor afla


minim N + [T ] + 2 valori ntregi, iar din continuitatea functiei polinomiale p pe R
rezult
a c
a p ia toate aceste valori pe [kT, (k + 1) T ]. n intervalul [kT, (k + 1) T ] se
afl
a cel mult [T ] + 1 numere ntregi, posibile puncte de discontinuitate pentru p ({x})
si cum p ({x}) este continu
a pe restul intervalului, rezult
a c
a f are cel putin N + 1
puncte de discontinuitate pe [kT, (k + 1) T ], contradictie.

IMPORTANT
n scopul unei legaturi rapide cu redactia revistei, pot fi utilizate urmatoarele
adrese e-mail: tbirsan@math.tuiasi.ro sau t_birsan@yahoo.com si
profgpopa@yahoo.co.uk . Pe aceast
a cale colaboratorii pot purta cu redactia un dialog privitor la materialele trimise acesteia, procurarea numerelor
revistei etc. Sugeram colaboratorilor care trimit probleme originale pentru
publicare s
a le numeroteze si s
a-si retin
a o copie xerox a lor pentru a putea
purta cu usurinta o discutie prin e-mail asupra accept
arii/neaccept
arii acestora de c
atre redactia revistei.
La problemele de tip L se primesc solutii de la orice iubitor de matematici
elementare (indiferent de preocupare profesionala sau vrsta ). Fiecare dintre
solutiile acestor probleme - ce sunt publicate n revist
a dup
a un an - va fi
urmata de numele tuturor celor care au rezolvat-o.
Adres
am cu insistenta
amintea ca materialele trimise revistei
rug
s
a nu fie (s
a nu fi fost) trimise si altor publicatii.
Rug
am ca materialele tehnoredactate s
a fie trimise pe adresa redactiei nsotite de fisierele lor (de preferinta n LATEX).
147

Probleme propuse1
Clasele primare
P.134. De la apartamentul meu cobor 7 etaje, apoi urc 4 etaje si observ ca sunt
la etajul 9. La ce etaj locuiesc?
(Clasa I )
Dragos Iacob, elev, Iasi
P.135. n trei vase sunt 36 nuci. Daca din primul vas se iau 3 nuci si din al
treilea o nuc
a si se pun n al doilea vas, atunci n fiecare vas va fi acelasi num
ar de
nuci. Cte nuci au fost la nceput n fiecare vas?
(Clasa I )
nv. Rica Buc
atariu, Iasi
P.136. Aflati vrsta tatalui meu stiind ca este un numar cuprins ntre 35 si 41,
dublul lui ntre 73 si 77, iar triplul lui este cuprins ntre 112 si 118.
(Clasa a II-a)
Iurie Juc, elev, Iasi
P.137. Dorin, Oana si Claudia se preg
atesc pentru Concursul "Florica T. Cmpan". Oana a rezolvat 15 probleme. Dorin a rezolvat un numar de probleme n plus
fata de Oana, egal cu num
arul de probleme rezolvate n plus de Oana fata de Claudia.
Cte probleme au rezolvat mpreun
a cei trei copii?
(Clasa a II-a)
Inst. Maria Racu, Iasi
P.138. Doi tati si trei fii au mpuscat fiecare cte un iepure. Cnd i-au numarat,
au v
azut c
a au doar patru iepuri. De ce?
(Clasa a III-a)
Inst. Elena Nita
, Iasi
P.139. Mutati un singur chibrit pentru a obtine o egalitate:

(Clasa a III-a)

Nicolae Iv
aschescu, Craiova

P.140. Descoper
a regula de completare a jetoanelor
10
0

11
1

12
2

13

14

...

98
72

99
81

si calculeaz
a cte numere diferite sunt scrise pe aceste jetoane pe locul de jos.
(Clasa a III-a)
Lenuta Zaharia, elev
a, Iasi
P.141. Fiul observa ca, atunci cnd i mai trebuia un an pna la jumatatea
vrstei din prezent, tat
al avea vrsta de 12 ori mai mare dect a sa, iar cnd va avea
11 ani, vrsta lui va fi de 4 ori mai mic
a dect a tat
alui. S
a se afle vrsta fiului n
prezent.
(Clasa a IV-a)
Petru Asaftei, Iasi
P.142. Paginile unei c
arti sunt numerotate de la 1 la 336. Din aceast
a carte se
rup, la ntmplare, 111 foi. S
a se arate c
a:
a) suma numerelor de pe foile ramase nu se mparte exact la 10;
b) produsul numerelor de pe foile ramase se mparte exact la 3.
(Clasa a IV-a)
Maria Frangoi, elev
a, Iasi
1

Se primesc solutii pn
a la data de 1 iunie 2008.

148

P.143. Asezati numerele 2, 3, 4, . . . , 10 n p


atratul al
aturat astfel
nct, pe fiecare linie, suma numerelor din primele doua casete sa fie egala
cu numarul din ultima caseta. n cte moduri pot fi asezate aceste numere?
(Clasa a IV-a)
Ionela B
ar
agan, elev
a, Iasi

Clasa a V-a
V.81. Demonstrati ca putem completa cu numere naturale ntr-o infinitate de
moduri casutele libere din figura de mai jos, astfel nct sa se poata efectua corect
operatiile indicate:

Amalia Cantemir, elev


a, Iasi
V.82. ntr-o ferma sunt gaini, oi si vaci, n total 324 de picioare si un numar
impar de capete:
a) S
a se arate c
a n ferm
a nu pot fi 101 g
aini.
b) Sa se arate ca numarul oilor nu poate fi egal cu numarul vacilor.
Petru Asaftei, Iasi
V.83. Sa se demonstreze ca 13 | abc daca si numai daca 13 | 3 ab c.
Otilia Nemes, Ocna Mures
V.84. Determinati cel mai mic si cel mai mare numar natural de 90 de cifre,
divizibile cu 90 si avnd suma cifrelor 90.
Carmen Daniela Tamas, Brlad
V.85. Fie a, b N; s
a se arate c
a dac
a ultima cifr
a a num
arului a2 + b2 este 9,
2
atunci ultima cifr
a a lui (a + b) este tot 9. Reciproca este adev
arat
a?
Ioan S
ac
aleanu, Hrl
au
V.86. a) S
a se rezolve n numere naturale ecuatia x2 + y 2 = 625.
b) S
a se arate c
a ecuatia x2 + y 2 = 2007 nu are solutii n N2 .
Valerica Benta, Iasi
51
89
V.87. S
a se arate c
a7 >3 .
Nela Ciceu, Bac
au

Clasa a VI-a
VI.81. S
tiind c
a 13 | 2a + 3b + 4c + 5d, ar
atati c
a 13 | 43a + 45b + 47c + 49d si
13 | 46a + 30b 64c 54d (a, b, c, d N).
Norbert-Traian Ionita
, elev, Iasi
m n
VI.82. Fie A = 3 5 , m, n N. Not
am cu a, b, c num
arul divizorilor numerelor
A, 3A, respectiv 5A. S
tiind ca a si b sunt direct proportionale cu 3 si 4, iar b si c
sunt invers proportionale cu 15 si 16, s
a se determine A.
Mihai Haivas, Iasi
VI.83. Dac
a p este num
ar prim, iar n N , s
a se arate c
a p4n 3 nu este p
atrat
perfect.
Mirela Marin, Iasi
149

VI.84. Pentru n N , consider


am An = 1 + 11 + 111 + + |11 {z
. . . 1}. Ar
atati c
a:
n

a si numai dac
a 3 - n 1;
a) 3 | An dac
9n
10n
A
2n
< 10n + 1 +
b) 10n + 1 +
<
, n 3.
10
An
11

Temistocle Brsan, Iasi

VI.85. Pe latura Ox a unghiului drept xOy consider


am un punct A, iar pe
bisectoarea unghiului consider
am un punct B. Perpendiculara n B pe AB taie
dreapta Oy n C. S
a se arate c
a AB = BC.
Petru Asaftei, Iasi
VI.86. a) Fie 4ADC si M (AC). S
a se arate c
a PADM < PADC .
b
b) Fie 4ABC si (AD) bisectoarea unghiului A, D BC. Dac
a PABD = PACD ,
sa se arate ca 4ABC este isoscel.
Gheorghe Iurea, Iasi
VI.87. n figura alaturata sunt desenate 6 puncte, care unite doua
cte doua dau nastere la 9 drepte. Avem voie sa stergem unul dintre
puncte si s
a-l desen
am oriunde n alt
a parte.
a) Efectuati operatia descris
a astfel nct, prin unirea cte dou
aa
noilor puncte, sa se obtina 11 drepte.
b) Care este numarul minim si cel maxim de drepte care se pot obtine ntr-o
configuratie permis
a?
Gabriel Popa, Iasi

Clasa a VII-a
VII.81. Se consider
a abc si xyzt numere r
naturale scrise n baza 10. S
a se compare
q p
q p

numerele naturale A = 6 2 abc si B = 3 2 xyzt.


Bogdan Chiriac, student, Iasi
VII.82. Fie a, b numere reale strict pozitive. S
a se arate c
a:
a) dac
a a3 b3 = a + b, atunci a2 + b2 > 1;
b) dac
a a3 + b3 = a b, atunci a2 + b2 < 1.
Ionel Nechifor, Iasi
VII.83. Determinati numerele ntregi a, b, c, d pentru care ac + bd = 1, iar
ad + bc = 2.
Gheorghe Iurea, Iasi
VII.84. Fie patratul ABCD cu latura de lungime a, iar E, F , G puncte pe
a
a
a
laturile [BC], [CD], respectiv [AB] astfel nct CE = , CF = , iar BG = . S
a
4
3
2
se arate c
a dreptele AE, BF si CG sunt concurente.
Claudiu S
tefan Popa, Iasi
VII.85. Fie O intersectia diagonalelor patrulaterului ABCD. Daca AABD =
CD
AB
AABC = ACOD , sa se arate ca

= 1.
AB
CD
Doru Buzac, Iasi
VII.86. Fie A un punct pe manta unei mese de biliard circulare cu raza de 1 m.
O bila pleaca din A si ajunge napoi n A lovind manta de cel putin trei ori; reflexia
150

bilei se face considernd c


a aceasta loveste un perete plan tangent la cerc n punctul
de contact. Sa se arate ca exista o infinitate de traiectorii posibile si sa se determine
traiectoria de lungime minima.
Cristian Laz
ar, Iasi
VII.87. O tabla are forma unui dreptunghi 4 5, format din 20 de patratele
1 1. Avem la dispozitie doua jetoane, fiecare putnd acoperi cte un patratel. n
cte moduri putem aseza jetoanele pe tabl
a, astfel nct ele s
a nu se afle nici pe
aceeasi linie, nici pe aceeasi coloan
a? Generalizare.
Gabriel Popa, Iasi

Clasa a VIII-a
VIII.81. Consider
am fixate numerele a, b Z , m, n N , m 6= n si fie functia
f : N Z, f (x) = ax +b. Dac
a f (1) +f (2)+ + f (m) = f (1) + f (2) + +f (n),
sa se calculeze suma S = f (1) + f (2) + + f (m + n).
Dan Nedeianu, Dr.Tr.Severin
VIII.82. S
a se arate c
a |3xy + x + y| 1, x, y [0, 1].
Ovidiu Pop, Satu Mare
VIII.83. Sa se arate ca nu exista x, y Z pentru care 147x2 = 1 + 4y 3y 2 .
Mihai Cr
aciun, Pascani
8

VIII.84. Laturile a, b, c ale unui triunghi verific


a egalitatea 2 a + b8 + c8 =
4
2
a + b4 + c4 . Sa se arate ca triunghiul este dreptunghic.
Corina Elena Visan, Craiova
VIII.85. Dac
a a, b, c sunt numere reale pozitive, s
a se arate c
a

q
p
p

1
2
3
a2 + b2 ab + b2 + c2 bc 3 = a2 + c2 +
= .
a
c
b
Liviu Smarandache, Craiova

VIII.86. O piramid
a hexagonal
a regulat
a V ABCDEF are muchia bazei AB = 4

cm si n
altimea V O = 4 2 cm. Fie M mijlocul lui V D, {P } = AD BF , iar
{Q} = P M (V CF ). S
a se arate c
a:
a) dreptele V P si DQ sunt concurente; b) DQ (V BF ).
Gabriel Popa, Iasi
VIII.87. Consider
am prisma triunghiular
a regulat
a ABCA0 B 0 C 0 si cubul
AM CN A0 M 0 C 0 N 0 , unde M este punct interior triunghiului ABC. Fie E, F , E 0 ,
F 0 mijloacele muchiilor [AB], [BC], [A0 B 0 ], respectiv [A0 C 0 ].
a) Aflati m
asura unghiului dintre dreptele EF 0 si E 0 F .
b) Aflati m
asura unghiului format de planele (M CC 0 ) si (BCC 0 ).
Claudiu S
tefan Popa, Iasi

Clasa a IX-a
IX.81. Fie a, b R, n N . Daca ecuatia x2 + ax + b + 2 = 0 are ambele radacini
ntregi, ar
atati c
a num
arul 2a2 + b2 este natural compus.
Dorin M
arghidanu, Corabia
IX.82. Determinati functiile f : R R pentru care
151




f x4 + y 3 + z 2 + t = f (x) + f y 2 + f z 3 + f t4 , x, y, z, t R.
Lucian Tutescu si Liviu Smarandache, Craiova
IX.83. Pentru a 9, s
a se demonstreze c
a are loc inegalitatea
q
q

3 + 3a + 9 1 + 1 + a.
Marian Tetiva, Brlad
IX.84. Fie ABC un triunghi. Determinati numerele ntregi a, b, c nenule, prime
ntre ele doua cte doua, astfel nct punctele M , N , P sa fie coliniare, unde M , N ,

P sunt determinate prin conditiile AM = aAB; CN = bCA; CP = cBC.


Ioan S
ac
aleanu, Hrl
au
IX.85. Fie ABC un triunghi ascutitunghic si D = prBC A, E = prCA B, F =
prAB C. Demonstrati echivalenta afirmatiilor urmatoare:
(i) 4ABC este isoscel;
(ii) DB + EC + F A = DC + EA + F B;
1
1
1
1
1
1
(iii)
+
+
=
+
+
.
DB EC
FA
DC
EA F B
Examinati cazurile n care 4ABC este obtuzunghic sau dreptunghic.
Temistocle Brsan, Iasi

Clasa a X-a
X.81. S
a se rezolve n R R R sistemul
x y 2/3 = z 1/3 ; x4/3 y = z 2/3 ;

z 5/3 y 4/3 = z.
Vasile Chiriac, Bac
au

X.82. Solve the equation

aex + b ex 3 = ax3 + b x3 2 + a, a > b > 0.


Zdravko Starc, Vrac, Serbia
X.83. n exteriorul triunghiului ABC se construiesc triunghiurile isoscele BM A,
\
AN C si CP B de baze AB, AC si respectiv BC, astfel nct m(M
AB) = 15 ,

\
\
\
m(N
AC) = 45 , iar m(P
BC) = 30 . S
a se arate c
a m(M
P N ) = 60 .
aeru, Suceava
Angela Tig
X.84. Fie ABC un triunghi n care (tg B 1) (tg C 1) = 2. Daca M si N sunt
picioarele n
altimilor din B, respectiv C, s
a se arate c
a segmentele BM , CN si M N
se pot constitui n laturi ale unui triunghi.
C
at
alin Calistru, Iasi
X.85. Se prelungeste diametrul [M N ] al unui cerc C cu segmentul [N P ] congruent
cu [M N ]. Fie d perpendiculara n P pe M N si R d oarecare. Tangentele duse prin
R la C intersecteaza tangenta n M la C n S si T . Sa se arate ca centrul de greutate
al 4RST este un punct fix.
Adrian Reisner, Paris

Clasa a XI-a
{x}
XI.81. Dac
a m Z, s
a se studieze existenta limitei lim
.
xm sin x
Dan Popescu, Suceava
152

XI.82. Considerul sirul (an )nN , definit prin a0 = 0, a1 = 1 si an+2 = 3an+1 an ,


3 5
n N. Pentru x 0,
, aratati ca sirul xn = a0 + a1 x + a2 x2 + + an xn ,
2
n N, este convergent si calculati limita sa.
Vlad Emanuel, elev, Sibiu
XI.83. Fie f : [0, ) [0, ) pentru care are loc relatia f (f (x)) + 9x = f (6x),
x [0, ). Aratati ca f (x) 3x, x [0, ).
Bogdan Posa si Marius Dr
agoi, elevi, Motru
XI.84. Determinati numerele a R pentru care exist
a o functie continu
a
f : R R astfel nct (f f ) (x) = a2 f (x) 2a4 x, x R.
Andrei Nedelcu, Iasi
XI.85. Fie A = (aij )20072007 o matrice patratica n care aij {1, 0, 1}, i, j =
1, 2007. S
a se arate c
a determinantul matricei 2008I2007 + A este nenul.
Paul Georgescu si Gabriel Popa, Iasi

Clasa a XII-a
XII.81. Dintre toate parabolele y = ax2 + bx + c, s
a se determine aceea care
trece prin punctele A (0, 1), B (1, 2), satisface conditia y 0 pentru 0 x 1 si
realizeaza minimul ariei determinata de graficul parabolei, Ox si dreptele x = 0,
respectiv x = 1.
Adrian Corduneanu, Iasi
XII.82. Determinati primitivele functiei f : (1, ) R,

x3 5x4 + 3 (ln x 1)
f (x) =
.
3
(x4 1)
Dan Nedeianu, Dr. Tr. Severin
XII.83. S
a se determine functiile continue f : R R pentru care
Z x
et f (x t) dt, x R.
f (x) = |x| +
0

Dumitru Mihalache, Brlad


XII.84. Fie polinomul f Z [X], f = a0 X
+ a1 X 2n + + a2n X + a2n+1
pentru care n este impar, a0 a2n+1 este impar, iar a1 a2 este par. Sa se arate ca, daca
f are toate r
ad
acinile reale, cel putin una este irational
a.
Mihai Haivas, Iasi
XII.85. Fie n N, n 2. Ar
atati c
a exist
a P Z [X] de grad n astfel nct
toate multimile Ak = {P (i) (mod k) | i Z}, k N, k 2, sa aiba cardinalul strict
mai mic dect k.
Vlad Emanuel, elev, Sibiu
2n+1

153

Probleme pentru preg


atirea concursurilor
A. Nivel gimnazial
G126. Sa se determine numerele naturale care au proprietatea ca media geometric
a a tuturor divizorilor lor este un num
ar natural.
Petru Minut, Iasi
G127. Dac
a a, b, c, x, y, z, t sunt numere reale pozitive, s
a se demonstreze
inegalitatea

1
1
1
1
8 3
p
.
+
+
+
>
ax+by+cz ay+bz+ct az+bt+cx at+bx+cy
a2 +b2 +c2 x2 +y 2 +z 2 +t2

D. M. B
atinetu-Giurgiu, Bucuresti

G128. Fie a, b, c numere reale pozitive astfel nct abc = 1 si fie t [1, 5]. S
a se
arate c
a
a
b
c
3
+
+

.
a2 + t b2 + t c2 + t
t+1
Titu Zvonaru, Com
anesti si Bogdan Ionita
, Bucuresti

1
1
= {xy} + , x R.
G129. Sa se determine y R pentru care {x} + x +
y
y
(Cu {} am notat partea fractionara.)
Alexandru Negrescu, elev, Botosani
G130. Fie a, b, c lungimile laturilor unui triunghi ABC. Dac
a a2007 + b2007 >
2007
2007
b
2
+1 c
, s
a se arate c
a unghiul C este ascutit.
Lucian Tutescu, Craiova

G131. Fie n, k 2 numere naturale si multimea M = { (n 1) , . . . , 2, 1, 1,


2, . . . , n}. Sa se arate ca M se poate partitiona n k submultimi avnd fiecare aceeasi
sum
a a elementelor dac
a si numai dac
a n se divide cu k.
Marian Tetiva, Brlad

G132. n fiecare cmp unitate al unei livezi m n se afl


a cte un m
ar. Un
numar de k arici pornesc, pe rnd, din cmpul stnga-sus al livezii si se misca spre
cmpul din dreapta-jos. La fiecare miscare, un arici se poate deplasa cu un cmp,
spre dreapta sau n jos, f
ar
a a iesi din livad
a. Ariciul poate s
a culeag
a m
arul din
cmpul pe care l viziteaz
a, dac
a nu a fost cules deja de alt arici. Care este num
arul
minim k, pentru care k arici pot sa culeaga toate merele?
Iurie Boreico, elev, Chisin
au
\ =
G133. Fie 4ABC echilateral si D un punct astfel nct BD = DC, m(BDC)

\
30 , iar BC separ
a A si D. Dac
a E (BD) cu m(BAE) = 15 , s
a se arate c
a
CE AC.
Enache P
atrascu, Focsani

G134. Se considera patrulaterul convex ABCD nscris ntr-un cerc de raza 6


b = 60 si m(B)
b = 45 . S
cm, avnd m(A)
a se arate c
a aria patrulaterului este cel

2
mult egal
a cu 3 6 cm .
Constantin Apostol, Rm. S
arat

154

G135. Fie tetraedrul ABCD cu AB = CD, AC = BD, AD = BC. S


a se arate
ca cel putin doua dintre unghiurile diedre formate de fata (ABC) cu fetele (BCD),
(ACD), (ABD) sunt ascutite.
Dan Brnzei, Iasi

B. Nivel liceal
L126. Fie ABC un triunghi ascutitunghic. Madiatoarea laturii AB intersecteaz
a
latura AC n T , iar mediatoarea laturii AC intersecteaza latura AB n S. Sa se arate
ca paralela prin T la AB, paralela prin S la AC si simediana din A sunt concurente.
Titu Zvonaru, Com
anesti
a se arate c
a
L127. Fie A1 A2 A3 A4 A5 A6 un hexagon inscriptibil. S
rA1 A2 A3 + rA4 A5 A6 + rA1 A3 A6 + rA3 A4 A6 = rA3 A4 A5 + rA1 A2 A6 + rA2 A3 A6 + rA3 A5 A6 ,
unde rXY Z este raza cercului nscris n 4XY Z.

C
at
alin Calistru, Iasi
L128. Sa se arate ca ntre medianele unui triunghi are loc inegalitatea
ih X
X
hY
i
Y
X
m2a m2b
m4a .
m2a m2b
(ma + mb ) 2
8
ma

Dorel B
aitan si I.V.Maftei, Bucuresti
L129. n planul raportat la un reper cartezian xOy consideram vectorii legati
n O: v1 (a1 , b1 ), v2 (a2 , b2 ), v3 (a3 , b3 ). S
a se arate c
a exist
a un tetraedru OABC

regulat, de muchie 1 si astfel nct OA, OB, OC se proiecteaza pe planul xOy n v1 ,
v2 , respectiv v3 daca si numai daca se verifica simultan relatiile:

3 2
3 2
a + a22 + a23 a1 a2 a1 a3 a2 a3 =
b + b22 + b23 b1 b2 b1 b3 b2 b3 = 1;
2 1
2 1
3
(a1 b1 + a2 b2 + a3 b3 ) (a1 b2 + a2 b1 + a1 b3 + a3 b1 + a2 b3 + a3 b2 ) = 0.
2
Irina Mustata
a, Bremen
, student
L130. S
a se arate c
a pentru orice x, y 1 are loc inegalitatea

(xy x y)2 + 6 3 10 xy 6 3 9.

Gabriel Dospinescu, Paris si Marian Tetiva, Brlad


L131. S
a se afle valoarea minim
a a num
arului real k astfel nct, oricare ar fi a,
b, c reale pozitive cu a + b + c = ab + bc + ca, sa aiba loc inegalitatea

1
1
1
(a + b + c)
+
+
k k.
a+b b+c c+a
Andrei Ciupan, elev, Bucuresti
L132. Fie a, b, c, x, y, t R si A = ax+by +cz, B = ay +bz +cx, C = az +bx+cy.

Daca |A B| 1, |B C| 1 si |C A| 1, aratati ca a2 + b2 + c2 x2 + y 2 + z 2 .
3
Adrian Zahariuc, elev, Bac
au
L133. Determinati functiile f : N N pentru care
2f (n + 3)f (n + 2) = f (n + 1) + f (n) + 1, n N.
Gheorghe Iurea, Iasi
155

L134. Avem un colier cu n m


argele, numerotate consecutiv 1, 2, . . . , n, unde
n 3. n cte moduri putem sa le coloram cu trei culori, astfel nct oricare doua
margele consecutive sa aiba culori diferite?
Iurie Boreico, elev, Chisin
au
L135. Se consider
a un poligon cu 3n laturi, n 2, nscris ntr-un cerc de raz
a 1.
Aratati ca cel mult 3n2 dintre segmentele
avnd capetele n vrfurile poligonului au

lungimea strict mai mare dect 2.


Bianca-Teodora Iordache, elev
a, Craiova

Training problems for mathematical contests


Junior highschool level
G126. Determine the natural numbers such that the arithmetic mean of all their
divisors is a natural number as well.
Petru Minut, Iasi
G127. If a, b, c, x, y, z, t are positive real numbers, prove the inequality

1
1
1
1
8 3
p
.
+
+
+
>
ax+by+cz ay+bz+ct az+bt+cx at+bx+cy
a2 +b2 +c2 x2 +y 2 +z 2 +t2
D. M. B
atinetu-Giurgiu, Bucuresti
G128. Let a, b, c be positive real numbers such that abc = 1 and let t [1, 5].
Show that
b
c
3
a
+
+

.
a2 + t b2 + t c2 + t
t+1
Titu Zvonaru, Com
anesti and Bogdan Ionita
, Bucuresti

1
1
G129. Determine y R such that {x} + x +
= {xy} + , x R. (with
y
y
{} denoting the fractional part.)
Alexandru Negrescu, highschool student, Botosani
G130.
Let
a,
b,
c
be
the side lengths of a triangle ABC. If a2007 + b2007 >
2007

b is acute.
2
+ 1 c2007 , show that the angle C
Lucian Tutescu, Craiova
G131. Let n, k 2 be natural numbers and consider the set M = { (n 1) , . . . ,
2, 1, 1, 2, . . . , n}. Show that M can be partitioned into k subsets with the same
sum of the elements in each of them if and only if n is divisible by k.
Marian Tetiva, Brlad
G132. A rectangular garden has m n unit squares. On each of them there is
an apple. A number of k hedgehogs start successively from the first left top square,
moving to the right bottom square. At each step, any hedgehog may move one square
right or below (still remaining in the garden) and picking up the apple on the square,
unless the apple was not earlier picked up. Which is the least number k of hedgehogs
able to pick up all the apples?
Iurie Boreico, highschool student, Chisin
au
156

G133. Let 4ABC be an equilateral triangle and D a point such that BD = DC,
\
\ = 15 ,
m(BDC) = 30 , and BC separates A and D. If E (BD) with m(BAE)
show that CE AC.
Enache P
atrascu, Focsani
G134. Itis considered the convex quadrilateral ABCD inscribed into a circle
b = 60 and m(B)
b = 45 . Show that the area of the
of radius of 6 cm, with m(A)

2
quadrilateral is at most equal to 3 6 cm .
Constantin Apostol, Rm. S
arat
G135. Let ABCD be a tetrahedron with AB = CD, AC = BD, AD = BC.
Show that at least two of the angles between the surface (ABC) with the surfaces
(BCD), (ACD), (ABD) are acute.
Dan Brnzei, Iasi

B. Highschool level
L126. Let ABC be an acute-angled triangle. The mid-perpendicular line of
side AB intersects the side AC at point T , and mid-perpendicular line of side AC
intersects the side AB at point S. Show that the parallel line to AB, through T , the
parallel line to AC, through S and the simedian from A are three concurrent lines.
Titu Zvonaru, Com
anesti
L127. Let A1 A2 A3 A4 A5 A6 be an inscriptible hexagon. Show that
rA1 A2 A3 + rA4 A5 A6 + rA1 A3 A6 + rA3 A4 A6 = rA3 A4 A5 + rA1 A2 A6 + rA2 A3 A6 + rA3 A5 A6 ,
where rXY Z is the radius of the circle inscribed in XY Z.
C
at
alin Calistru, Iasi
L128. Show that the following inequality involving the bisectors of the sides (or
medians) of a triangle holds:
Y
ih X
X
hY
i
X
8
ma
m2a m2b
m4a .
m2a m2b
(ma + mb ) 2

Dorel B
aitan and I.V.Maftei, Bucuresti
L129. In the Cartesian system xOy, let three vectors v1 (a1 , b1 ), v2 (a2 , b2 ),
v3 (a3 , b3 ) be sitting with their origin in the point O. Prove that there exist a regular

tetrahedron OABC with the edges of length 1, such that the projections of OA, OB,

OC (onto the plane xOy) are equal with v1 , v2 , v3 , respectively, if and only if the
equations below are simultaneously satisfied:

3 2
3 2
a + a22 + a23 a1 a2 a1 a3 a2 a3 =
b + b22 + b23 b1 b2 b1 b3 b2 b3 = 1;
2 1
2 1
3
(a1 b1 + a2 b2 + a3 b3 ) (a1 b2 + a2 b1 + a1 b3 + a3 b1 + a2 b3 + a3 b2 ) = 0.
2
Irina Mustata
, student, Bremen
L130. Show that the following inequality holds for any x, y 1:

(xy x y)2 + 6 3 10 xy 6 3 9.

Gabriel Dospinescu, Paris and Marian Tetiva, Brlad


157

L131. Find the minimum value of the real number k such that any would be the
real positive numbers a, b, c with a + b + c = ab + bc + ca, the following inequality
holds :

1
1
1
(a + b + c)
+
+
k k.
a+b b+c c+a
Andrei Ciupan, highschool student, Bucuresti
L132. Let a, b, c, x, y, t R and A = ax+by+cz, B = ay+bz+cx, C = az+bx+cy.

4
If | A B | 1, | B C | 1 and | C A | 1, show that a2 + b2 + c2 x2 + y 2 + z 2 .
3
Adrian Zahariuc, highschool student, Bac
au
L133. Determine the functions f : N N satisfying the equation
2 f (n + 3) f (n + 2) = f (n + 1) + f (n) + 1,

n N.

Gheorghe Iurea, Iasi


L134. A necklace consist of n pearls consecutively numbered 1, 2, . . . , n, where
n 3. How many ways exist to colour these pearls using three colours so that any
two neighbor pearls have dierent colours?
Iurie Boreico, highschool student, Chisin
au
L135. It is considered a polygon with 3n sides, such that n 2 and the polygon
is inscribed into a circle of radius 1. Show that at most 3n2 line segments
with their
ends at pairs of polygons vertices have their length strictly greater than 2.
Bianca-Teodora Iordache, highschool student, Craiova

Premiu pe anul 2007 acordat


de ASOCIA
TIA RECREA
TII MATEMATICE
Se acord
a un premiu n bani n valoare de 100 lei elevului
CIACOI Bogdan Liceul teoretic "Ana Ipatescu", Gherla
pentru nota O propozitie echivalenta cu conjectura lui Goldbach aparuta n numarul
1/2007 al revistei Recreatii Matematice, la pagina 27.
Se acord
a, de asemenea, un premiu n bani n valoare de 100 lei elevilor
BOREICO Iurie, elev, Chisin
au si CIUPAN Andrei, elev, Bucuresti
pentru nota Inegalitati stabilite cu un procedeu de reducere a numarului de variabile
- Mixing variables ap
arut
a n acest num
ar la pagina 100.
Aceste premii sunt acordate de Asociatia "Recreatii Matematice" ca urmare
a contractului de sponsorizare cu Fundatia Cultural
a "Poiana" (director d-l Dan
Tiba).

158

Pagina rezolvitorilor
BRA
SOV
Colegiul National de Informatica "G. Moisil". Clasa a IX-a. AMBRONO Andra: VII(71,73,77,80), VIII(72,73,74), IX.80; BACIU Dan: VII(71,80), VIII(73,74),

IX.80; BAUM Bianca: VII(78,79), VIII(77,78), IX.80; DAINEANU


Catalina: VII(71,
77-80), IX.80; FARSCH Cristina: VII(79,80), VIII(77,78), IX.80; FINTOIU Diana: VII(77,78,80), VIII(71-74,77,78), IX.72; HERMENEANU Horia: VII(77,80),
VIII(77,78), IX.80; MAN Andrada: VII(77,79,80), VIII(77,78), IX.80; MARIAN
Laurentiu: VII(71,73,77), VIII(72,74); NEGREA Andrei: VII.80, VIII(71-74), IX.80;
Sorin: VII(71,73), VIII(72,73),
OANCEA Mihai: VII.80, VIII(71-74); OGLINDA
IX.80; POTEC Ionela: VII(77,79), VIII(77,78), IX.80; SIMION Adriana: VII.73,
VIII(71-73), IX.80; TURTUREANU Veronica: VII(71,79), VIII(71-73,77,78), IX.80.

BOGDANE
STI (SUCEAVA)
Scoala
de
arte
si meserii. Clasa a VI-a. SOLCANU M
ad
alina Vasilica: VI(71,74,

76,78,79), VII.75, VIII.75.

HRLAU
Larisa Ionela: P(121-123), V.73,
Liceul "Stefan
cel Mare". Clasa a V-a. BARAU

Andreea: P(121,122), V(72,73), VI.71; CEUCA


Razvan: P(121,123),
VI.71; BUZILA

Andreea Simona: P(121-123), V.73, VI.71; JITARIU


A
V.73, VI(71,72); IVANU
T
Adina-Diana: P(121-123), V.72, VI.72.
IA
SI

Valentina). CULEA
A
Scoala
nr. 3 "Al. Vlahuta". Clasa a IV-a (nv. MARIU
T

Theodor: P(125-129,131); HADARAG AnaA


Alina: P(125-129,131); DIACONIT
Maria: P(125-129,131); NASTASE Cosmin: P(125-129,131); POPA Iulian: P(125129,131); PROCA Ancuta: P(125-129,131); TNCU Alexandra Ioana: P(125-129,
131). Clasa a IV-a (inst. MAXIM Gabriela). BACIU Ionela-Lavinia: P(124128); CELMARE Raluca-Iuliana: P(124-128); POPOVICI Ionut: P(124-128); RUSU
M
ad
alina-Andreea: P(124-128); SAVA Vlad: P(124-128); VECHIU M
ad
alina: P(124128).
Scoala
nr. 13 "Alexandru cel Bun". Clasa a III-a (inst. COJOCARIU Ana).

Andra: P(124,125,

AGAFITEI
Elena-Roxana: P(124,125,127-129); CARAMALAU

127-129); CALIN
Andreea-Claudia: P(124,125,127-129); COJOCARIU Andreea:
P(124,125,127-129); DUDUMAN Luisa-Stefania: P(124,125,127-129); LELEU Alexandrina-Stefana: P(124,125,127-129); LUPASCU Diana-Maria: P(124,125,127-129);
A
Narcisa-Lorena:
MANOLACHE M
ad
alina-Andreea: P(124,125,127-129); MIHAIL

P(124,125,127-129); PASCU Gabriela: P(124,125,127-129); PADURARU


Tiberiu
S
tefan: P(124,125,127-129); RADUCEA
Marin-Andrei: P(124,125,127-129); SAVIN
Cristina-Simona: P(124,125,127-129); S
TEFAN Bogdan-Vasile: P(124,125,127-129);
S
TIUBEI Cosmin-Ionut: P(124,125,127-129).
Elena). BACIU TuA
Scoala
nr. 14 "Gh. Mrzescu". Clasa a II-a (inst. NUT

George-Stefan: P(114-118, 124-128); GHEdor: P(114-118, 124-128); CHIRILU


TA
M
A
ORGHIT
ad
alina-Gabriela: P(114-118; STOICA Adriana: P(114-118, 124-128);

TMBALARIU
Ioana-Vasilica: P(114-118).
159


Scoala
nr. 26 "G. Cosbuc". Clasa a II-a (inst. RACU Maria). APACHITEI

Aura-Georgiana: P(124-129); BURA Emma-Andreea: P(124-129); CIORNEI Alexan


dra: P(124-127,129); CRACIUN
Ioana-Daniela: P(124-127,129); FILIP Ingrid-Stefa Narcis-Eugen: P(124-127,129); GRADINARIU

A
nia: P(124-129); GHEORGHIT
Georgiana: P(124-129); HRISCU Ovidiu-Constantin: P(124-129); HUZA M
ad
alina:
P(124-129); MARICIUC Dragos-Claudiu: P(124-127,129); MAXIM Alexandra-Came Cosmin: P(124-127,129); VASILE Bogdan-Andrei: P(124
lia: P(124-129); TUC
A
129).

Colegiul National, locatia "Gh. Asachi". Clasa a III-a (inst. CALINESCU


Rodica). BERECHET Alexandru: P(126-130); GHENARU Br
adita: P(126-130);
CHIVULESCU Alexandru: P(126-130); PETREA M
ad
alina: P(126-130); UNGUREANU George: P(126-130).
Razvan: V(71-75), VI(71-73,75),
Colegiul National. Clasa a VII-a. CEUCA
VII(71-75), VIII(72,73,75); MOCANU Dan Mihai: V(76-79), VI(76,78,79), VII(76,77);
PETRESCU Emanuel: V(76-79), VI.76.
ALOI

Colegiul National "C. Negruzzi". Clasa a VI-a. PAV


Alexandru: P(121123), VI(71-74).
Colegiul National "Emil Racovita". Clasa a VII-a. TUDORACHE Alexandru:
VII(76-80), G(117,118).
SUCEAVA
Scoala
cu clasele I-VIII, nr. 3. Clasa a II-a (nv. TABARCEA Silvestru). FE
CHET S
tefan: P(114-118). Clasa a III-a (inst. NECHITA Daniela). FECHET
Mircea: P(114,115,117-119).

Premii acordate rezolvitorilor

ASOCIA
TIA "RECREATII
MATEMATICE" n colaborare cu redactia
MATEMATICE acord
revistei RECREATII
a cte o diplom
a si un premiu n
c
arti pentru trei aparitii la rubrica Pagina rezolvitorilor urm
atorilor elevi:
Scoala
cu clasele I-VIII, nr.3, Suceava

FECHET S
tefan (cl. a II-a): 2/2006(5pb), 1/2007(5pb), 2/2007(5pb);
FECHET Mircea (cl. a III-a): 2/2006(9pb), 1/2007(6pb), 2/2007(5pb);
Scoala
nr. 3 "Al. Vlahuta", Iasi

HADARAG Ana-Maria (cl. a IV-a): 2/2006(6pb), 1/2007(5pb), 2/2007(6pb);


NASTASE Cosmin (cl. a IV-a): 2/2006(6pb), 1/2007(5pb), 2/2007(6pb);
TNCU Alexandra-Ioana (cl. a IV-a): 2/2006(6pb), 1/2007(5pb), 2/2007(6pb);
BACIU Ionela-Lavinia (cl. a IV-a): 2/2006(8pb), 1/2007(6pb), 2/2007(5pb);
RUSU M
ad
alina-Andreea (cl. a IV-a): 2/2006(8pb), 1/2007(6pb), 2/2007(5pb);
160

SAVA Vlad (cl. a IV-a): 2/2006(8pb), 1/2007(6pb), 2/2007(5pb);


Scoala
nr. 26 "Gh. Cosbuc", Iasi

APACHITEI
Aura-Georgiana (cl. a II-a): 2/2006(6pb), 1/2007(5pb), 2/2007(6pb);
BURA Emma-Andreea (cl. a II-a): 2/2006(6pb), 1/2007(5pb), 2/2007(6pb);
FILIP Ingrid-
Stefania (cl. a II-a): 2/2006(6pb), 1/2007(5pb), 2/2007(6pb);

GRADINARIU
Georgiana (cl. a II-a): 2/2006(6pb), 1/2007(5pb), 2/2007(6pb);
HRISCU Ovidiu-Constantin (cl. a II-a): 2/2006(6pb), 1/2007(5pb), 2/2007(6pb);
HUZA M
ad
alina (cl. a II-a): 2/2006(6pb), 1/2007(5pb), 2/2007(6pb);
MAXIM Alexandra-Camelia (cl. a II-a): 2/2006(6pb), 1/2007(5pb), 2/2007(6pb);
VASILE Bogdan-Andrei (cl. a II-a): 2/2006(6pb), 1/2007(5pb), 2/2007(6pb);
Colegiul National, locatia Scoala
"Gh. Asachi", Iasi

PETREA M
ad
alina (cl. a III-a): 2/2006(5pb), 1/2007(7pb), 2/2007(6pb);
UNGUREANU Georghe (cl. a III-a): 2/2006(5pb), 1/2007(7pb), 2/2007(6pb);
Colegiul National, Iasi
R
CEUCA
azvan (cl. a VII-a): 1/2005(6pb), 2/2005(5pb), 1/2006(5pb);
Colegiul National "C. Negruzzi", Iasi
ALOI

PAV
Alexandru (cl. a VI-a): 2/2006(5pb), 1/2007(5pb), 2/2007(7pb);
Colegiul National "Emil Racovita", Iasi
TUDORACHE Alexandru (cl. a VII-a): 2/2006(10pb), 1/2007(10pb), 2/2007(7pb).

Vizitati pe Internet revista "Recreatii Matematice" la adresa

http://www.recreatiimatematice.uv.ro

161

Revista semestrial RECREAII MATEMATICE


este editat de
ASOCIAIA RECREAII MATEMATICE. Apare la datele de 1 martie i
1 septembrie i se adreseaz elevilor, profesorilor, studenilor i tuturor celor
pasionai de matematica elementar.
n atenia tuturor colaboratorilor
Materialele trimise redaciei spre publicare (note i articole, chestiuni de
metodic, probleme propuse etc.) trebuie prezentate ngrijit, clar i concis; ele
trebuie s prezinte interes pentru un cerc ct mai larg de cititori. Se recomand ca
textele s nu depeasc patru pagini. Evident, ele trebuie s fie originale i s
nu fi aprut sau s fi fost trimise spre publicare altor reviste. Rugm ca materialele tehnoredactate s fie nsoite de fiierele lor.
Problemele destinate rubricilor: Probleme propuse i Probleme pentru
pregtirea concursurilor vor fi redactate pe foi separate cu enun i demonstraie/rezolvare (cte una pe fiecare foaie) i vor fi nsoite de numele autorului, coala i localitatea unde lucreaz/nva.
Redacia va decide asupra oportunitii publicrii materialelor primite.
n atenia elevilor
Numele elevilor ce vor trimite redaciei soluii corecte la problemele din
rubricile de Probleme propuse i Probleme pentru pregatirea concursurilor
vor fi menionate n Pagina rezolvitorilor. Se va ine seama de regulile:
1. Pot trimite soluii la minimum cinci probleme propuse n numrul
prezent i cel anterior al revistei; pe o foaie va fi redactat soluia unei singure
probleme.
2. Elevii din clasele VI-XII au dreptul s trimit soluii la problemele
propuse pentru clasa lor, pentru orice clas mai mare, din dou clase mai mici i
imediat anterioare. Elevii din clasa a V-a pot trimite soluii la problemele propuse
pentru clasele a IV-a, a V-a i orice clas mai mare, iar elevii claselor I-IV pot
trimite soluii la problemele propuse pentru oricare din clasele primare i orice clas mai mare. Orice elev poate trimite soluii la problemele de concurs (tip G i L).
3. Vor fi menionate urmtoarele date personale: numele i prenumele,
clasa, coala i localitatea.
4. Plicul cu probleme rezolvate se va trimite prin pot (sau va fi adus
direct) la adresa Redaciei:
Prof. dr. Temistocle Brsan
Str. Aurora, nr. 3, sc. D, ap. 6,
700 474, Iai
Jud. IAI
E-mail: tbirsan@math.tuiasi.ro sau t_birsan@yahoo.com

CUPRINS
Al VI-lea Congres Internaional al Matematicienilor Romni...................................... 77
Conjectura lui Poincar ........................................................................................................ 79

ARTICOLE I NOTE
G. DOSPINESCU Tipurile subgrupurilor finite din GL2 (]) ........................................... 81
T. ZVONARU O problem cu cifrele unui numr .......................................................... 87
T. BRSAN O problem de construcie a unui triunghi ................................................. 90
A. CORDUNEANU i Gh. COSTOVICI Un ir strns legat de irul lui Wallis .......... 95
A. REISNER Asupra rdcinilor polinomului X 3 + pX + q _[ X ] ................................ 97

NOTA ELEVULUI
I. BOREICO i A. CIUPAN Inegaliti stabilite cu un procedeu de reducere
a numrului de variabile Mixing variables ................................ 100

CHESTIUNI METODICE
J. GRIGORA i C.-t. POPA Asupra unei probleme de concurs ................................ 105

MATEMATICA N CLASELE PRIMARE


P. ASAFTEI Intuirea proprietilor operaiilor aritmetice
utiliznd metodele figurative ............................................................ 107

CORESPONDENE
H. STEPHAN Jensen's inequality for non-convex functions ......................................... 109

CONCURSURI I EXAMENE
Concursul de Matematic "Al. Myller", Ediia a V-a, Iai .............................................. 111
Concursul de matematic "Florica T. Cmpan", 2007 ....................................................... 114
Olimpiada Balcanic de Matematic Juniori, Ediia a XI-a, umen (Bulgaria) .......... 118

PROBLEME I SOLUII
Soluiile problemelor propuse n nr. 2/2006........................................................................ 121
Soluiile problemelor pentru pregtirea concursurilor din nr. 2/2006 ............................... 137
Probleme propuse................................................................................................................... 148
Probleme pentru pregtirea concursurilor ............................................................................ 154
Training problems for mathematical contests ..................................................................... 156
Pagina rezolvitorilor ............................................................................................................ 159
5 lei

Anul X, Nr. 2

Iulie Decembrie 2008

RECREAII
MATEMATICE
REVIST DE MATEMATIC PENTRU ELEVI I PROFESORI

125 de ani de la apariia


revistei Recreaii tiinifice
(1883 1888)

e i = 1
Asociaia Recreaii Matematice
IAI - 2008

Semnificaia formulei de pe copert:


i
ntr-o form concis, formula e = 1 leag cele patru ramuri fundamentale
ale matematicii:
ARITMETICA
GEOMETRIA
ALGEBRA
ANALIZA MATEMATIC

reprezentat
reprezentat
reprezentat
reprezentat

de
de
de
de

i
e

Redacia revistei :
Petru ASAFTEI, Dumitru BTINEU-GIURGIU (Bucureti), Temistocle BRSAN, Dan
BRNZEI, Ctlin - Cristian BUDEANU, Constantin CHIRIL, Eugenia COHAL, Adrian
CORDUNEANU, Mihai CRCIUN (Pacani), Paraschiva GALIA, Paul GEORGESCU,
Mihai HAIVAS, Gheorghe IUREA, Mircea LUPAN, Gabriel MRANU, Alexandru
NEGRESCU (student, Iai), Gabriel POPA, Dan POPESCU (Suceava), Florin POPOVICI
(Braov), Maria RACU, Neculai ROMAN (Mirceti), Ioan SCLEANU (Hrlu), Ioan
ERDEAN (Ortie), Dan TIBA (Bucureti), Marian TETIVA (Brlad), Lucian TUESCU
(Craiova), Adrian ZANOSCHI, Titu ZVONARU (Comneti).

COPYRIGHT 2008, ASOCIAIA RECREAII MATEMATICE


Toate drepturile aparin Asociaiei Recreaii Matematice. Reproducerea integral sau
parial a textului sau a ilustraiilor din aceast revist este posibil numai cu acordul prealabil
scris al acesteia.
TIPRIT LA SL&F IMPEX IAI
Bd. Carol I, nr. 3-5
Tel. 0788 498933
E-mail: simonaslf@yahoo.com
ISSN 1582 - 1765

Anul X, Nr. 2

Iulie Decembrie 2008

RECREAII
MATEMATICE
REVIST DE MATEMATIC PENTRU ELEVI I PROFESORI

e i = 1
Revist cu apariie semestrial

EDITURA RECREAII MATEMATICE

IAI - 2008

O sut
a de ani de la nasterea lui Gheorghe Gheorghiev
Gheorghe Gheorghiev a fost elev al
ilustrilor savanti A. Myller si O. Mayer si a devenit, pentru o buna perioada de timp, liderul
scolii de geometrie de la Universitatea din Iasi. A
fost un reprezentant al unei generatii de dasc
ali
ieseni care au vegheat la destinele matematicilor romnesti, ntr-o perioada de mari schimbari
politice. n acelasi timp, a cautat sa reia legaturile
ntrerupte cu matematicile din lumea ntreag
a. A
fost un adev
arat constructor de scoal
a stiintific
a
la Facultatea de matematica din Iasi, dovedinduse un merituos continuator al eforturilor si straduintelor profesorului s
au A. Myller.
S-a n
ascut la 27 iunie 1907 la Bolgrad, judetul
Ismail. A urmat liceul din Bolgrad n anii 19161925, dupa care a sustinut examenul de bacalureat
la Ismail n iunie 1925. Imediat a devenit student
la Facultatea de stiinte din Iasi, sectia matematic
a, unde a urmat cursuri predate
de profesorii A. Myller, Vera Myller Lebede, O. Mayer, Simeon Sanielevici, Petre
Culianu, Constantin Popovici, Mendel Haimovici, Gheorghe Vranceanu s.a. si l-a
avut asistent pe Ilie Popa, cu care a r
amas prieten pentru toat
a viata. n timpul
ultimului an de studiu si dup
a examenul de licenta, n martie 1929, a devenit asistent
si s-a mprietenit cu mai multi tineri matematicieni: I. Schoenberg, P. Cazanachi,
D. Mangeron, E. Stihi, si cu studentul A. Climescu. Tot n 1929 a urmat cursuri de
specializare la Universitatea din Hamburg, cu cunoscutul profesor W. Blaschke.
n 1929 s-a declansat criza economica pe plan mondial, care a afectat si tara
noastra. n perioada 1929-1938, Gh. Gheorghiev a fost profesor de matematici la
liceele din Aiud, Ismail, Chilia Noua, Cetatea Alba si Iasi. In perioada 1938-1946 a
fost nscris la doctorat la Universitatea din Iasi. n aceeasi perioad
a a fost concentrat
si mobilizat timp de peste 5 ani. A participat la r
azboi ca ofiter de geniu si, timp de
circa un an si jumatate, a fost prizonier. A existat chiar pericolul sa fie mpuscat n
momentul n care a fost luat prizonier, pentru ca era ofiter.
n 1946 a sustinut teza de doctorat cu titlul Suprafete ale caror familii remarcabile
de curbe sunt asemenea (comisia de doctorat, fiind compus
a din A. Myller, O. Mayer,
V. Myller, I. Popa).
A functionat la Universitatea din Iasi ca lector (1946-1948), conferentiar onorific
de matematici generale (1946-1948), profesor suplinitor de geometrie analitic
a (19471954), profesor titular de geometrie (1955-1975).
ntre anii 1953-1968 a fost decan al Facultatii de matematica din Iasi iar, ntre
1960-1975, a fost seful catedrei de geometrie. n aceste posturi si pozitii de conducere,
Gh. Gheorghiev a avut o influenta hot
artoare asupra dezvolt
arii facult
atii n ce
priveste stabilirea unor leg
aturi cu directii de cercetare stiintific
a existente att n
scolile de matematica din occident ct si cu cele existente n Uniunea Sovietica.
95

A colaborat la elaborarea unui tratat de geometrie intitulat simplu Curs de geometrie analitica (1951); recunoasterea meritelor acestui tratat s-a realizat prin acordarea Premiului de Stat. A mai colaborat la realizarea unei carti Geometrie analitica
si diferentiala, ap
arut
a n dou
a volume n 1968 si 1969 la Editura Didactic
a si Pedagogic
a. n aceast
a carte ncep s
a se reflecte schimb
arile de conceptie si de tehnic
a
de investigare n ce priveste studiul geometriei diferentiale. Ideile au fost finalizate
n alte doua carti, elaborate n colaborare, Varietati diferentiabile finit si infinit dimensionale, ap
arut
a n dou
a volume n 1976 si 1979 la Editura Academiei Romne
si Geometrie diferentiala ap
arut
a la Editura Didactic
a si Pedagogic
a.
A scris circa 180 de lucrari stiintifice n domeniul geometriei diferentiale si numeroase articole de popularizare a matematicii. A conferentiat la mai multe univer teica" al Academiei Romne,
sitati de prestigiu din lume. A primit premiul "Gh. Ti
a primit titlul de profesor emerit, mai multe diplome si medalii ale unor asociatii
profesionale si numeroase ordine si medalii, din partea statului romn.
Dupa pensionare, n 1975, a ramas profesor consultant la Facultatea de matematic
a (cu unele ntreruperi) pn
a la deces, n 28 iunie 1999.
A abordat teme de cercetare stiintific
a din geometria diferential
a euclidian
a (retele
pe suprafete, cmpuri de vectori pe suprafete) geometria diferentiala afina si proiectiv
a (cmpuri de conuri, configuratii Myller), geometria diferential
a a varietatilor
modelate de spatii Banach, teoria grupurilor Lie, teoria G-structurilor si generaliz
ari
ale acesteia. Circa 30 de tineri studiosi si-au elaborat tezele de doctorat sub conducerea stiintifica a profesorului Gheorghiev. A colaborat, n elaborea cartilor, cu ctiva
colegi apropiati: R. Miron, D. Papuc, V. Oproiu.
A fost profesor vizitator si a conferentiat la numeroase universitati din Europa:
Jena, Padova, Sofia, Londra, Moscova, Debrecen etc.
A predat cursuri de geometrie analitica, diferentiala, teoria grupurilor Lie etc. la
diverse nivele, la Facultatea de matematic
a a Universit
atii din Iasi. A mai predat
si cursuri de matematici generale pentru studentii de la facult
atile Institutului Politehnic din Iasi. Lectiile sale erau renumite prin densitatea informatiilor transmise si
pentru solicitarea intelectuala extrema la care erau supusi studentii ce le urmareau.
n perioada ct a fost decan, n facultate s-a petrecut o extraordinar
a schimbare a
argumentelor predate la cursuri si a celor abordate n cercetare, realizndu-se un salt
de la aspectele traditionale existente n matematicile din perioada interbelic
a, c
atre
matematicile moderne, cercetate si predate n stilul structuralist, promovat de catre
grupul Bourbaki. n ultimii ani ai vietii era preocupat de istoria matematicii si de
probleme matematice ale filosofiei fizicii.
Meritele la catedr
a, n cercetarea stiintific
a, de om care, pe tot parcursul vietii
sale, a practicat cultul muncii si al cinstei au facut din profesorul Gh. Gheorghiev
o personalitate n domeniul matematicilor, respectat
a att n facultate si universitate
ct si pe plan national si international.
Activitatea sa n slujba scolii romnesti si n servirea matematicii a fost r
asplatit
a
prin mai multe distinctii, titlul de Profesor Emerit (1964), Premiul de Stat (1951),
teica" al Academiei Romne (1981), Ordinul Muncii (1960), Ordinul
premiul "Gh.Ti
Meritul S
tiintific (1960), Medalii ale Muncii etc.

Prof. dr. Vasile OPROIU


96

Ilie Popa - 100 de ani de la nastere


Ilie Popa a ramas n constiinta noastra ca
unul dintre matematicienii remarcabili ai scolii de
geometrie din Iasi, cunoscut si recunoscut si peste
hotarele tarii, ca un subtil si profund cercetator
n domeniul istoriei matematicii, ca un profesor
care, cu lectiile sale expuse cu tact, claritate si
farmec, a format numeroase generatii de profesori
si viitori cercetatori, care, de la nivelul functiilor
de conducere asumate, a militat cu daruire si pricepere pentru un nv
atamnt de calitate.
Intreaga viata si activitatea sa variat
a si rodnica sunt legate de orasul Iasi.
n Iasi s-a nascut, la 20 iulie 1907. A urmat scoala primar
a "C. Negri". Apoi, n perioada 1919-1927, Liceul Internat ast
azi Colegiul
National "C. Negruzzi" -, avnd ca profesor de
matematica pe Ion Raianu (pe care-l va evoca
mai trziu cu recunostinta si pretuire). Pe placa de onoare a sefilor de promotie ai
liceului, n dreptul anului 1927, este scris numele s
au (pe aceeasi plac
a, peste ani, va
aparea si numele fiului sau, Eugen, la rndul lui distins matematician). Era mndru
cu faptul ca absolvise Liceul Internat si o afirma cu placere cnd avea prilejul.
A continuat studiile la Facultatea de stiinte (sectia matematic
a) din Iasi si a sustinut examenul de licenta n 1931. A sustinut doctoratul n matematici n 1934, sub
conducerea lui O. Mayer, cu teza Contributii la geometria centro-afina diferentiala.
La aceasta nsirare de date se impune sa mentionam rolul avut n formarea sa de
ambianta Seminarului matematic din Iasi, de conditiile propice de munc
a din biblioteca acestuia, ct si de nrurirea favorabil
a exercitat
a de A. Myller si O. Mayer,
conducatorii de atunci ai scolii matematice iesene. Ilie Popa s-a ncadrat n directiile
de cercetare ale scolii de geometrie din Iasi, contribuind cu rezutatele obtinute la
constructia geometriei centro-afine, considerat
a mai trziu o creatie pur romneasc
a.
ntre 1936 si 1938, Ilie Popa a f
acut stagii de specializare la Roma, cu Enrico Bompiani, si la Hamburg, cu Wilhelm Blaschke, doi renumiti matematicieni ai timpului.
Prin munca si merit a urcat treptele ierarhiei universitare, ocupnd diverse pozitii
si prednd cursuri variate la Universitatea "Al. I. Cuza" si la Politehnica "Gh. Asachi"
din Iasi. n 1942, n urma plec
arii prof. Gr. Moisil la Bucuresti a fost titularizat, pe
postul ramas vacant, ca profesor la Facultatea de stiinte, catedra de calcul diferential
si integral. A fost seful catedrei de analiza matematica din 1948 si pna n anul 1973,
an n care, prin retragere la pensie, a devenit profesor consultant.
La 26 iulie 1983 a ncetat din viata dup
a o lung
a si grea suferinta, fiind nhumat
n cimitirul Eternitatea.
Nu vom prezenta nici macar sumar contributia adusa de Ilie Popa n geometria
centro-afin
a diferential
a, contributie care l-a consacrat si f
acut cunoscut n comunitatea matematic
a; vom observa doar colaborarea n acest domeniu cu geometrul
iesean Gheorghe Gheorghiev si prietenia durabila care s-a legat ntre acestia. Con97

sider
am c
a pentru cititorii prezentei reviste sunt mai accesibile, mai interesante si cu
un folos mai mare alte aspecte ale activitatii variate pe care a desfasurat-o Ilie Popa.
Avnd un spirit patrunzator, o gndire profunda si o mare putere de sinteza si
beneficiind de o bogat
a cultur
a general
a, Ilie Popa a abordat teme diverse din istoria
matematicii romnesti aducnd clarific
ari ntr-un material faptic din trecut, care era
destinat cu precadere uitarii. A scos la lumina meritele unor precursori ai matematicii romnesti ca Dimitrie Asachi fiul cel mare al lui Gh. Asachi , primul romn
ce public
a n str
ain
atate (Mnchen, 1841) o lucrare de matematici superioare sau
N. St.
a n 1872 o formul
a pentru o bucat
a a seriei armonice, ce a atras
Botez, care d
atentia matematicianului belgian E.-Ch. Catalan. Un alt studiu este destinat lui
Amfilochie Hotiniul, care tipareste n Iasi, la 1795, prima aritmetica romneasca Elementi aritmetice aratate firesti, o compilatie dup
a cteva c
arti italienesti. ntreprinde
o munc
a dificil
a de restaurare, atunci cnd reconstituie manuscrisul de geometrie pierdut al lui Gh. Lazar. A pus n evidenta rolul jucat de sotii Alexandru si Vera Myller
n crearea scolii matematice iesene, cu numele generic de Seminarul matematic. Cu
prilejul centenarului Universit
atii din Iasi (18601960), public
a, n al doilea tom
consacrat evenimentului, studiul Dezvoltarea matematicii o ampl
a si documentat
a
contributie privind realizarile scolii matematice iesene pe parcursul unui secol.
S
a mai spunem cititorilor c
a Ilie Popa a publicat n 1955 studiul Recreatii Sti
intifice precursoare a Gazetei matematice n care evidentiaz
a rolul acestei reviste
prima de acest fel din tar
a, str
abun
a a actualei Recreatii Matematice n dezvoltarea
nvatamntului si netezirea drumului ce duce la cercetari originale n matematica.
Ca profesor, Ilie Popa a fost un maestru al artei comunic
arii. Lectiile sale erau
oficiate dup
a un ritual neab
atut. ncepeau cnd profesorul punea ceasul de buzunar
pe mas
a si se terminau cnd acesta era repus la locul lui. Era omis
a pauza dintre
ore, dar timpul acordat prelegerii era respectat cu strictete. Cu un timbru placut, o
vorbire blnd
a, moldoveneasc
a, cu un ritm al expunerii care antrena dar nu obosea,
profesorul reusea imediat s
a captiveze auditoriul. Ideile si argumentele erau expuse
ntr-o nl
antuire usor de urm
arit. Erau scoase n evidenta semnificatiile conceptelor
si rezultatelor. Fata magistrului se acoperea de o lumina discreta, cea a bucuriei
de a d
arui. Cte un zmbet involuntar, retinut cu grij
a, marca un moment cheie al
lectiei sau nsotea un rezultat de o rar
a frumusete. Calmul, echilibrul, tactul erau
ntotdeauna prezente. George St.
Andonie spune inspirat: si alege detaliile cu grija
si obiectivitate si lasa impresia ca-si suna n prealabil cuvintele, spre a-si da seama
de rezonanta lor (Istoria matematicii n Romnia, vol. III).
A avut numeroase functii de conducere n diverse structuri ale nv
atamntului superior: prorector al Universitatii din Iasi, director al Seminarului Matematic
"Al. Myller", director general n Ministerul nvatamntului, rector al Institutului
Pedagogic de 3 ani din Iasi s.a. Cu devotament si competenta, a folosit ntreaga sa
capacitate de munc
a n scopul ridic
arii nv
atamntului romnesc.
La mplinirea unui secol de la nasterea sa, ne gndim cu profund
a recunostinta
la cel ce a fost omul si profesorul Ilie Popa si cu toata admiratia la opera sa de
geometru, de istoriograf al matematicii si nv
atamntului matematic si de dasc
al al
nv
atamntului din tara noastr
a.

Prof. dr. Temistocle BRSAN


98

Simpozion
dedicat revistei "Recreatii S
tiintifice" (1883-1888)
Academia Romna - filiala din Iasi a gazduit, n ziua de 15 martie a.c., simpozionul "125 de ani de la aparitia revistei Recrea
tii stiin
tifice". Festivitatea s-a
desf
asurat sub auspiciile Academiei Romne, Facult
atii de matematic
a a Universit
atii
"Al. I. Cuza", Catedrei de matematic
a a Universit
atii Tehnice "Gh. Asachi" si ale
Asociatiei "Recreatii matematice".
Cuvntul de deschidere a apartinut acad. Viorel Barbu, presedintele filialei Iasi a
Academiei Romne, care a subliniat faptul c
a revista Recreatii stiintifice este prima
publicatie din tar
a cu acest profil adresat
a tineretului. Al
aturi de alte evenimente
iesene remarcabile ale anului 1883 aparitia editiei T. Maiorescu a Poeziilor lui
Mihai Eminescu, inaugurarea statuii lui Stefan
cel Mare s.a. aparitia Recreatiilor

stiintifice reprezint
a un moment important al culturii si spiritualit
atii romnesti.
Fondatorii, distinsi profesori ai Universit
atii din Iasi sau ai scolilor iesene, ct si
colaboratorii revistei au asigurat o nalta tinuta stiintifica acesteia. Revista a circulat
n ntregul Regat al Romniei de atunci si se remarca prin varietatea subiectelor
abordate, rigoare, frumoasa limb
a romn
a folosit
a si o grafic
a excelent
a pentru acele
timpuri.
Acad. V. Barbu mentioneaza ca n cadrul simpozionului este lansata colectia
integrala a revistei Recreatii stiintifice, reeditata n forma originara, nemodificata.
Realizarea acestui proiect de reeditare se datoreste sprijinului material entuziast al
doamnei Marinela Ghigea, director al firmei Kepler Systmes dInformation, precum
si muncii depuse de dr. Dan Tiba, cercetator la Institutul de matematica al Academiei
Romne si de prof. dr. Temistocle Brsan, Universitatea Tehnic
a "Gh. Asachi".
Cuvnt
arile tinute de acad. Radu Miron, prof. dr. Vasile Oproiu, prof. dr. Dorin
Iesan, m.c. al Academiei, prof. dr. Teodor Precupanu, n aceast
a ordine, sunt prezentate mai jos. Programul simpozionului se ncheie cu proiectia unor documente
privind revista Recreatii stiintifice si epoca n care a ap
arut aceasta prezentate de
prof. dr. Temistocle Brsan.

Recreatii S
tiintifice
125 ani de la aparitia primului num
ar
Acad. prof. dr. doc. Radu MIRON
La 15 ianuarie 2008 s-au mplinit 125 de ani de la publicarea numarului 1 din
primul volum al revistei Recreatii stiintif ice. Menita a face educatie stiintifica
tineretului din Regatul Romniei, revista, care a avut o existenta de numai sase ani, a
dep
asit granitele n toate zonele locuite de romni. nfiintat
a de zece oameni nv
atati
din vechea capitala a Moldovei, ea avea sa imprime n constiinta locuitorilor acestui
99

pamnt primele capitole elevate de istorie a stiintelor din tara si cele nti lectii
pentru un nv
atamnt modern n domeniul stiintelor exacte. Peste veac s-a v
azut
nrurirea covrsitoare a ideilor vehiculate n cuprinsul acestei reviste, n S
coala de
toate gradele si n cercetare, conducnd la integrarea noastr
a n rndul tarilor care
aveau deja traditii seculare.
Datorit
a continutului preponderent matematic se poate afirma cu deplin temei
c
a revista a deschis prima pagin
a a matematicilor romnesti. Asa cum s-a remarcat
mai trziu, dac
a publicatia s-ar fi numit Recreatii matematice, ea ar fi constituit
prima publicatie din lume n domeniu, care se adreseaza tineretului. Este adevarat
c
a, la sapte ani de la disparitia "Recreatiilor stiintifice", n 1895 a fost nfiintat
a
Gazeta Matematica cu adres
a special
a pentru tineretul romn. Dar aceast
a revist
a
este considerat
a a doua din lume ca profil si destinatie.
teica a fost impulsioEvident, aparitia "Gazetei", asa cum sublinia Gheorghe Ti
nat
a de "Recreatiile stiintifice".
Acum, la 125 de ani de la nfiintare a celebrei reviste se cuvine s
a exprim
am
omagii profunde memoriei fondatorilor: N. Culianu, C. Climescu, I. Melic de la
Universitatea "Al. I. Cuza", G. I. Lucescu, V. Paladi, G. I. Rosiu, I. D. Rallet,
G. Zarifopol, I. V. Praja si I. M. Dospinescu din nv
atamntul preuniversitar
iesean. Prin competenta, pasiune si sacrificii personale f
acute cu generozitate ei au
reusit s
a trezeasc
a interesul pentru stiinta n general si s
a stimuleze gustul pentru
matematici n special.
Sunt emotionante cuvintele scrise ntr-un editorial al revistei: Credem ca noi am
tras cea ntai brazda care conduce catra lucrari originale. Brazda-i mica si ngusta,
dar exista!
Personalitatea fondatorilor este bine cunoscuta. Ei sunt prezentati de George St.

Andonie n volumul I din Istoria Matematicii n Romnia. Majoritatea lor sunt


oameni de stiinta cu studii nalte f
acute n Franta, Italia, Olanda si Germania. Un
gnd de recunostinta colaboratorilor, nu mai putin celebri: M. Tzony, V. Costin,
P. Tanco, C. Gogu si rezolvitorilor pasionati, elevi pe vremea aceea, E. Pangrati
si D. Pompeiu.
Nu trebuie s
a-i uit
am pe oamenii de stiinta care au sustinut peste timp importanta
revistei si impactul ei n cultura romneasca. Citam doar ctiva dintre ei: Alexandru
Myller, Octav Mayer, Ilie Popa, Gheorghe Gheorghiev, Gheorghe Bantas, Gheorghe
Ti
teica, G. St.
Andonie, N. N. Mihaileanu etc.
Conditiile istorice n care a ap
arut n 1883 revista "Recreatii stiintifice" nu erau
dintre cele mai favorabile. Unirea Principatelor abia se nfaptuise, Regatul Romniei
era abia ntemeiat, Razboiul de Independenta din 1877 lasase urme adnci n constiinta romnilor, alfabetul chirilic fusese nlocuit cu cel latin, limba romn
a literar
a
abia si definitivase procesul de unificare, romnii si afirmau n mod decisiv aspiratia spre o societate moderna. n atari conditii, desi aparusera cu 23 de ani nainte
universitatile din Iasi si Bucuresti, scoala de toate gradele trebuia profund recladita.
Era nevoie imperioas
a de regndit programarea curricular
a, de preg
atit personalul
100

didactic, de scris manuale bune n limba romna, de construit scoli etc.


n atari conditii spirituale, materiale si sociale dure, a pune bazele unei reviste
de cultur
a stiintific
a era un act de curaj, de patriotism. El constituia o important
a
realizare destinat
a poporului nostru. Soliditatea acestui edificiu este dat
a de calitatea
stiintifica, didactica si educationala a subiectelor publicate, de limbajul stiintific
adoptat, de grafica de exceptie utilizat
a n acea vreme. Am prezentat aceste aspecte
n articolul "Centenarul revistei Recreatii stiintif ice", Probleme de istoria si filozofia
stiintei, vol. X, 1984, Filiala Iasi. Valabilitatea afirmatiilor f
acute atunci si p
astreaz
a
temeiul si astazi. Din acest motiv reproduc o parte din text.
Tonul ntregii productii matematice, cuprinznd mai bine de 90% din cele 1920
de pagini ct nsumeaza aceasta revista, a fost dat n primul rnd de fondatorii ei,
care, prin prestigiul lor, au atras foarte curnd valorosi colaboratori: Miltiade Tzony
profesor de mecanica teoretica la Universitatea din Iasi, Candide (probabil Victor
Costin, pe atunci student la Paris), Iacob Solomon inginer, Paul Tanco profesor de matematica si f izica la Gimnaziul Superior din Nasaud, Constantin Gogu
profesor de geometrie analitica la Universitatea din Bucuresti, Vasile Butureanu
profesor de mineralogie si petrografie la Facultatea de stiinte din Iasi s. a.
n paginile revistei sunt publicate articole, note, probleme si solutii din domenii
ca: aritmetica, algebra, geometrie elementara, geometrie analitica si diferentiala, calcul diferential si integral, mecanica, astronomie, istoria matematicii, chimie, f izica,
geograf ie. Apare prima traducere a cartii nti din celebrele "Elemente" ale lui Euclid. Miltiade Tzony tipareste n coloanele ei o remarcabila culegere de probleme
de mecanica teoretica. Geometria proiectiva, domeniu de mare actualitate n acea
vreme, este prezenta prin traducerea primelor opt paragrafe din vestita lucrare "Geometria de pozitie" a lui Criristian von Staudt. ntile elemente din istoria matematicilor n antichitate sunt transpuse n limba romna de Iacob Solomon.
La succesul binemeritat al Recreatiilor stiintif ice a contribuit si prezentarea grafica
excelenta. Scrisa ntr-o limba literara elevata, revista are, cu exceptia unor termeni
matematici n formare, ceva din culoarea si prospetimea revistelor actuale.
Privita global, ca act de cultura stiintif ica, revista rivalizeaza cu cele mai bune
publicatii de acest gen tiparite acum un secol pe plan mondial.
nchei aici relatarea din articolul amintit. Dar ultima fraza trebuie corectata cu
"acum un secol si un sfert pe plan mondial".
Subliniez faptul c
a n tot cuprinsul celor sase volume ale revistei impresioneaz
a
grija pentru rigoarea prezent
arii, acurateta exprim
arii n limba romn
a, actualizarea
expunerilor, informatia de ultima ora, profunzimea rationamentelor si, nu n ultimul
rnd, atentia acordata contributiilor personale ale tinerilor rezolvitori sau autori ale
problemelor propuse spre publicare.
A fost realizat
a astfel n premier
a, o revist
a romneasc
a extrem de important
a
pentru nvatamnt si cercetare n stiintele exacte, de acelasi nivel cu reviste similare
cosacrate si vestite din lume. Dupa sapte ani de la stingerea activitatii acestei reviste,
ideea de a r
aspndi n rndul tineretului pasiunea pentru matematic
a va fi preluata
101

de "Gazeta Matematica".
La 125 de ani de la aparitia revistei "Recreatii stiintifice", generatiile de ast
azi
omagiaz
a acest eveniment ca semn de adnc
a recunostinta adus
a naintasilor nostri
pentru contributia lor inestimabil
a la tezaurul stiintei si culturii romnesti.
S
i un scurt adaos: Centenarul aparitiei revistei "Recreatii stiintifice" a fost organizat n 1983 de matematicieni ieseni n Seminarul Matematic "Alexandru Myller",
iar s
arb
atorirea celor 125 de ani de la aparitie a fost initiat
a tot n cadrul acestui
Seminar pregatit
a fiind de Asociatia "Recreatii Matematice", Facultatea de matematica si Institutul de Matematica "Octav Mayer" de la Filiala din Iasi a Academiei
Romne. Asociatia "Recreatii Matematice" si face un titlu de onoare prin reeditarea integral
a, exclusiv prin grij
a proprie, a colectiei revistei "Recreatii stiintifice".
Acest fapt l dator
am prof. univ. Temistocle Brsan de la Universitatea Tehnic
a
"Gheorghe Asachi" din Iasi, cercetatorului dr. Dan Tiba de la Institutul de Matematic
a al Academiei Romne din Bucuresti si doamnei Marinela Ghigea director al
firmei Kepler Systmes d Information. i asigur
am de toat
a pretuirea si gratitudinea
noastr
a.

Rolul si ponderea geometriei


n revista Recreatii S
tiintifice
Prof. dr. Vasile OPROIU
n revista Recreatii stiintif ice, scrisa si editata de un grup de oameni de stiinta
si cultura inimosi (N. Culianu, C. Climescu, I. Melik, G. I. Lucescu, V. Paladi,
G. I. Rosiu, I. D. Rallet, G. Zarifopol, I. V. Praja, si I. M. Dospinescu),
s-au adunat si publicat diferite materiale din domeniile matematicii, fizicii, chimiei,
geografiei, cosmografiei, topografiei, mineralogiei, istoriei matematicii etc. Revista
se adresa elevilor din clasele de gimnaziu si liceu, dar si altor categorii de persoane
interesate de cunoastere: profesori, studenti, functionari, militari etc.
Geometria, ca ramura a matematicilor are o pondere destul de nsemnata n
paginile revistei. Trebuie sa mentionam, de la nceput, ca G.I. Rosiu publica ntre anii 1883-1885 prima carte a Elementelor lui Euclid (traducere dupa o editie
italian
a). Am reg
asit cu o anumit
a emotie si nostalgie multe formul
ari pe care le
ntlnisem cnd eram student si apoi le citisem n c
artile lui Efimov (editia n limba
romna si cea n franceza) si n cartea lui I.Vaisman. Astfel (n volumul II al Recreatiilor), printre definitiile lui Euclid am regasit formulari precum: Punctul este aceea ce
nu are parti, adeca nu are nici o marime, Linia este lungime fara largime, Suprafata
plana este aceea care este asezata egal n respectul tuturor liniilor sale drepte. Postulatele si axiomele au fost publicate anterior, n primul volum, ntr-o ordine diferita
102

de cea cu care suntem obisnuiti. Astfel, faimosul postulat V al lui Euclid apare ca
axioma XII. Autorul prezint
a si definitiile si axiomele, asa cum au fost prelucrate
de Legendre n Geometria sa, editia V, Paris, 1804. Legendre nu defineste punctul
si, n legatur
a cu definitia, formuleaz
a urm
atoarea asertiune: Def initia unui lucru
este exprimarea raporturilor sale catra lucruri cunoscute. Dupa care, se ncumeta sa
definesc
a dreapta: Linia dreapta este drumul cel mai scurt de la un punct la altul.
Mai sunt prezentate comentarii critice ale diversilor matematicieni relativ la aceste
definitii, inclusiv noi definitii: Cea mai simpla din toate liniile este linia dreapta a
caria notiune este familiara la toti si despre care ni da o idee un f ir ntins. Apoi
sunt prezentate propozitiile de la I la XXII. Cum spuneam, prezentarea Elementelor
continu
a n volumul III cu propozitiile r
amase si cu exercitiile la cartea I.
n revist
a sunt prezentate numeroase aspecte ale geometriei elementare, utile
elevilor, profesorilor si altor persoane interesate: maxime si minime geometrice, media
si extrema ratie, calculul lungimilor unor linii importante din triunghi, calcule pentru
patrulaterul inscriptibil (n primul volum), proprietati ale poligoanelor si dreapta lui
Simson (n vol. II), calcularea volumelor piramidei trunchiate si al conului trunchiat,
proprietati sintetice ale elipsei (n vol. III), teoria transversalelor, diviziunea armonic
a, fasciculul armonic, poli si polare, geometria de pozitie a lui Staudt (n vol. IV);
aceasta din urma se continua si n volumul V. Chestiunile de geometrie analitic
a sunt
considerate separat si se refer
a la: sectiuni plane n conul drept, constructii de curbe,
cu exemplificari din clasele curbelor celebre, tratarea acestora n coordonate polare,
plane principale la suprafetele de gradul al doilea.
O sectiune important
a n revist
a este cea a problemelor propuse (de regul
a, n
jur de 10 probleme la fiecare num
ar), la care se adug
a, pe parcurs, cea cu rezolv
arile
si listele de rezolvitori. Trebuie sa mentionam ca, n Regatul Romniei de atunci,
existau cteva zeci de gimnazii si licee (oricum, sub 30) si c
a num
arul celor care
rezolvau probleme era destul de mic. Moda rezolv
arilor de probleme la reviste de
matematica nu prinsese nca. Mentionam ca existau si colaborari venite de la elevi
din Transilvania, Banat si alte regiuni ale viitoarei Romnii Mari.
Ca o apreciere cu caracter general, continutul revistei Recreatii S
tiitifice era
destul de ridicat din punct de vedere al nivelului chestiunilor de geometrie tratate.
Subiectele erau interesante si atractive pentru numerosi cititori. Cred ca, n redactia
revistei, erau persoane care doreau sa faca revista ct mai atractiva.
R
asfoind cele sase volume am dat si peste un articol fascinant de la sectia cosmografie, scris de G.I. Lucescu, n care se explica n ce manier
a au fost concepute
calendarele iulian si gregorian si ca motivul pentru care s-a facut trecerea de la unul
la altul a fost legat de ideea ca, n acord cu hotarrea Conciliului de la Niceea din
anul 325, punctul de plecare pentru fixarea zilei de Pasti trebuia s
a fie echinoctiul
de prim
avar
a si acesta trebuia s
a fie mereu la 21 martie. Dup
a aceea, se asteapta
prima noapte cu luna plina si Pastele se fixa n duminica imediat urmatoare (astfel,
n 2008, noaptea cu luna plina cade exact n 21 martie si Pastele catolic este fixat n
23 martie; fixarea Pastelui ortodox este mult mai complicat
a si tine de niste date din
103

calendarul iudaic). De la data Conciliului de la Niceea pna n 1582 calendarul iulian


r
am
asese n urm
a cu 10 zile fata de calendarul real (anul din calendarul iulian era
putin mai lung dect anul real). Acest lucru influenta foarte mult diverse activitati
practice, de exemplu, unele lucr
ari agricole ce se f
aceau n strns
a leg
atur
a cu s
arbatorile religioase. n 1582, papa Grigore al XIII-lea a dat o bula prin care se decidea
avansarea calendarului iulian, existent, cu 10 zile si c
a anii multipli de sute (mai
putin cei multipli de 400) nu sunt bisecti. Acest calendar mai are o mica eroare care
const
a n r
amnerea n urm
a cu o zi n circa 3300 ani, eroare considerat
a rezonabil
a
si care va fi corectata n viitor. n tara la noi, calendarul grigorian a fost adoptat n
1923 (s-a trecut la stilul nou!), dat
a cnd ntrzierea calendarului iulian fata de cel
real, sau cel grigorian, ajunsese la 13 zile.
Revenind la revista Recreatii stiintifice, apreciez c
a un cititor interesat poate s
a
gaseasca n cuprimsul ei lucruri incitante, att n domeniul geometriei, ct si n alte
domenii ale matematicilor si ale altor stiinte.

Despre problemele de mecanic


a
Prof. dr. Dorin IESAN,
m.c. al Academiei

Pe lnga alte chestiuni interesante, n revista Recreatii stiintif ice au aparut


si un num
ar de probleme de mecanic
a rational
a, semnate de Miltiade Tzony. Pe
vremea cnd a publicat aceste probleme, M. Tzony era profesor de mecanic
a teoretic
a
la Universitatea din Iasi (a functionat n aceasta calitate n perioada 23.X.1869
15.III.1898).
Miltiade Tzony este autorul unui curs de mecanic
a, prezentat n manuscris, n
dou
a volume. Primul volum a fost scris n 1869, iar al doilea volum n 1881. Pe
prima pagina a acestui curs autorul scrie "Curs
u de Mecanica rationale si aplicata;
Profesat la Universitatea de Jasy; Dupa cei mai buni autori francezi: Delaunay,
Sturm, Duhamel, Bellanger, Bresse, Bour, Collignon, Mesat, Chasles; de Miltiade
Tzony, Licentiat n stiinte matematice de la Sorbona din Paris, Ingineriu al scoalei
de poduri din Paris, Vechiu elevu al scoalei politechnice din acestu orasu, Profesor al
Universitatii de Jasy si a Lyceului Nou din Jasy". Lectiile de mecanica rationala de
la Universitatea din Iasi se f
aceau dup
a acest curs.
In perioada 1885-1888, M. Tzony public
a Un curs de probleme n revista "Recreatii
stiintifice". El marturiseste ca acest lucru l face "n scopul de a usura studentilor
Universitatilor noastre completa pricepere a cursului de mecanica rationala si nimerita ntrebuintare a principiilor acestei nsemnate stiinti" (vol. III, pp. 77-78). S
a
vedem care este originea problemelor si a rezolv
arilor date. M. Tzony ne spune c
a
"problemele sunt lucrate dupa diversi autori ntre care figureaza n primul loc abatele
104

Jullien, a carui carte n aceasta materie a devenit clasica". Am constatat ca toate


cele 98 de probleme publicate de Tzony n "Recreatii stiintifice" sunt luate din cartea
c
alug
arului P.M. Jullien Problmes de mcanique rationnelle, ap
arut
a la Paris n
anul 1855. Cartea lui P.M. Jullien contine att probleme originale ct si probleme ale
altor autori. In "Recreatii stiintifice" M. Tzony prezinta 36 probleme ale carui autor
este P.M. Jullien, 24 de probleme datorate lui W. Walton si 28 probleme ale altor autori (printre care Euler, Bernoulli, Leibniz, Laplace, Gauss, Mbius). Mention
am c
a
problemele datorate lui W. Walton se g
asesc n cartea acestuia A Collection of Problems in Illustration of the Principles of Theoretial Mechanics, aparuta la Cambridge
n anul 1842.
La nceputul prezent
arii problemelor, M. Tzony afirm
a c
a "de cte ori ne va
f i posibil vom indica la f inea f iecarei probleme autorul caruia se datoreste". Problemele publicate de Tzony sunt si rezolvate si cu "toate indicatiunile necesare pentru
a putea f i cuprinse cu usurinta de tnarul public cetitoriu caruia este n special
destinat". M. Tzony sustine, pe drept cuvnt, c
a "opera abatelui Jullien este scrisa
ntr-un mod att de laconic nct cetirea ei de ncepatori este foarte laborioasa si n
unele puncte aproape cu totul nenteleasa". Dintre problemele publicate de Tzony
n "Recreatii stiintifice" un num
ar de 51 sunt rezolvate n cartea lui P.M. Jullien.
Celelalte sunt probleme pe care Jullien le-a propus spre rezolvare. O parte dintre
problemele nerezolvate de Jullien sunt ns
a nsotite de figuri si r
aspunsuri n cartea
lui W. Walton. Am comparat aceste rezolvari si figuri cu cele date de M. Tzony n
"Recreatii stiintifice". Se poate spune cu certitudine c
a Tzony nu s-a inspirat din
cartea lui W. Walton.
Fiecare capitol din culegerea de probleme este prefatat cu "o scurta amintire a
rezultatelor f inale ale teoriei, ntovarasite de cteva notiuni istorice pe care, n lipsa
altor documente, le vom mprumuta pentru cea mai mare parte din opera stiintif ica
de care facem mentiune". Este usor de v
azut c
a aceste comentarii sunt traduse din
cartea lui P.M. Jullien.
Mentionam ca rezolvarile prezentate de M. Tzony sunt clare, iar figurile sunt
ngrijite si binevenite. Un lucru remarcabil este faptul c
a si n cazul problemelor
rezolvate de Jullien, M. Tzony face figuri suplimentare si adaug
a explicatii. In problemele publicate de Tzony n "Recreatii stiintifice" sunt 88 de figuri, dintre care 66
nu se afla n cartea lui Jullien.
Referitor la repartitia pe ani a problemelor se constat
a c
a n anul 1885 sunt
publicate 18 probleme, n anul 1886 apar 33 probleme, n anul 1887 sunt publicate
23 probleme, iar n anul 1888 apar 18 probleme. In anul 1888 revista "Recreatii
stiintifice" si nceteaza aparitia. Acest lucru a curmat publicarea fireasca a altor
probleme.
Mention
am c
a problemele ap
arute se refer
a doar la partea de Static
a a cursului
predat de Tzony la Universitate. Dintre acestea, 18 se refera la echilibrul punctului
material, 39 la echilibrul corpului rigid, 6 trateaza echilibrul unui sistem de bare
articulate, 27 sunt dedicate echilibrului firelor, iar 8 se refer
a la principiul "lucrului
105

mecanic virtual".
Cursul lui M. Tzony (n manuscris) si problemele publicate de el n "Recreatii
stiintifice" au stat la baza nv
atamntului Mecanicii din tara noastr
a.
n afar
a de activitatea de profesor, Miltiade Tzony s-a remarcat prin munca sa
depusa n vederea propasirii Romniei. A fost senator, secretar de stat la Ministerul Constructiilor Publice, director al C.F.R. Printre altele, orasul Iasi i datoreaz
a
pavarea str
azilor.
Despre M. Tzony se pot spune multe lucruri. Un fost elev de-al s
au, Petru N. Culianu, care a urmat cursurile de mecanica de la Universitatea din Iasi n anii 1890,
l descrie pe Miltiade Tzony astfel: "Cu mintea agera, cu f igura frumoasa (a fost
luat ca model de pictorul Grigorescu pentru unele f iguri din biserica de la manastirea
Agapia), ce corespund ntru totul nobletei caracterului sau, el a fost cu totul devotat
datoriei si naltei misiuni a profesorului".

Problematica de algebr
a si analiz
a matematic
a
n revista Recreatii S
tiintifice
Prof. dr. Teodor PRECUPANU
Pentru matematica romneasca, aparitia revistei Recreatii stiintif ice din initiativa unei elite de profesori ai universit
atii si ai liceelor din Iasi, reprezint
a un
moment important, de nceput, pentru crearea unei atmosfere propice dezvolt
arii stiintelor matematice, de atragere a tinerilor, stimulndu-i si amplificndu-le pasiunile,
calauzindu-i spre problemele moderne ale acelei perioade.
Este de remarcat faptul c
a initiatorii revistei erau la curent cu multe din preocup
arile existente n matematica european
a, avnd o bun
a informare, facilitat
a de
accesul la o serie de reviste importante, ndeosebi din Franta, Italia si Germania. Sunt
semnalate nu numai aparitia unor rezultate importante, ci si diverse evenimente ale
comunit
atii stiintifice, cum ar fi, spre exemplu, aparitia revistei Acta Matematica
fondat
a de Mittag-Le ler prin casa regal
a suedo-norvegian
a, solemnitatea retragerii
din nvatamnt la vrsta de 70 de ani a marelui matematician Eugne-Charles
Catalan, aparitia unor tratate importante de matematica, discutiile generate de
proiectul Turnului Eiel, ce urma s
a se construiasc
a n Paris.
Adresndu-se n primul rnd elevilor din nv
atamntul secundar, revista a stimulat, de asemenea, preocuparile profesorilor pentru modernizarea nvatamntului
matematic, gazduind n paginile sale dezbateri interesante cu caracter metodic asupra
programelor analitice si a metodelor prin care s
a fie atrasi elevii pentru studiul
matematicii, s
a se asigure o ct mai bun
a accesibilitate, punnd pe primul plan
intuitia si dezvoltarea abilitatilor de rezolvitori de probleme.
106

Sa remarcam faptul ca n acea perioada, sfrsitul secolului al XIX-lea, unele discipline componente ale matematicii nu erau nc
a bine conturate, n acord cu felul n
care ele erau concepute la marile universit
ati europene, influentate de c
artile importante de matematica din acei ani. Abia aparuse cursul de analiza matematica al lui
Sturm (1880) si cel de calcul diferential si integral al lui Catalan (1878), autor si al
unei c
arti despre serii, unic
a la acel moment, c
arti ce urmau tratatelor celebre scrise
de Leibniz, Bernoulli sau Serret.
Algebra si analiza matematica este prezenta n paginile revistei att prin articole
cu caracter teoretic informativ asupra unor chestiuni importante, ct si printr-o gama
variat
a de exercitii si probleme ce vizau si pe studentii anilor preg
atitori pentru scolile
politehnice din tar
a sau din str
ain
atate. Semnal
am astfel mai mult articole scrise de
C. Climescu dedicate numerelor complexe (numite cantitati imaginare), numere care
nca erau evitate de multi matematicieni. ntlnim, de asemenea, demonstratia teoremei fundamentale a algebrei precum si unele consideratii asupra dreptelor imaginare
sau asupra unor functii complexe, date ns
a sub form
a implicit
a prin relatii polinomiale. Mai mult, n unele probleme apar integrale ale unor functii complexe, stiindu-se
a se depasi aspectele dificile de multivocitate si determinndu-se cu claritate valorile
lor principale.
In cadrul algebrei sunt cuprinse si seriile numerice, dezvolt
arile tayloriene, concepute ca sume (numerice sau polinomiale) infinite. Convergenta acestora nteleasa
intuitiv se reduce de fapt la extinderea operatiilor numerice algebrice si pentru
(nu reiese dac
a se folosea si ), fiind cunoscute limitele fundamentale. Se foloseau
ns
a pentru convergenta criterii fine, aproape toate cele cunoscute ast
azi. Autori ai
articolelor respective sunt C. Climescu, I.D. Rallet si I.V. Praja. Erau folosite n
mod uzual dezvoltarile n serii de puteri ale functiilor elementare. Tot ca facnd parte
din algebr
a, sunt prezentate cunoscutele formule Lagrange si Newton de interpolare
ntr-un articol foarte interesant scris de Alex. Sadoveanu.
In primul volum, din 1883, problema dezvoltarii functiilor n serie este data ca
facnd parte din Analiza algebrica iar Calculul integral era considerat separat de
Analiza matematic
a. De fapt, analiza matematic
a era conceput
a la acel moment
numai ca teorie a derivabilit
atii avnd ca principale rezultate teorema de medie a
lui Lagrange si conditiile suficiente de extrem la functii de una sau mai multe variabile. Problemele de calcul de arii sau volume erau considerate ca facnd parte din
geometrie sau intervenind n probleme de mecanic
a.
Notiunea de derivat
a era acceptat
a prin interpret
arile ei: geometric
a - de tangent
asau cele din mecanica. Mai mult, trebuie avut n vedere ca nsasi notiunea de functie,
fundamentala pentru ntreaga matematica, era conceputa n acele timpuri n acceptia
eulerian
a, ceea ce corespunde ast
azi functiilor elementare.
Mention
am c
a ecuatiile diferentiale sunt frecvent ntlnite n partea de mecanic
a si
de geometrie a curbelor plane (probleme concrete de aflare a unor curbe daca se dau
anumite proprietati metrice legate de tangente) si nu sunt prezentate ca apartinnd
disciplinei de ast
azi Ecuatii diferentiale, ceea ce este normal, ntruct aceast
a disci107

plina avea sa se contureze n matematica mult mai trziu. Este nsa prezentata n
cadrul Analizei matematice problema schimb
arii de variabil
a cu suportul oferit de
ecuatiile diferentiale si schimb
arile de coordonate (ndeosebi polare si sferice) deosebit
de importante n mecanic
a, geometrie si astronomie.
Problemele de algebra si analiza matematica prezente n cele sase volume ale
revistei sunt deosebit de frumoase si ilustrative, de mare diversitate, oferind o imagine
exact
a a continutului disciplinelor de matematic
a din acea perioad
a. Aproape toate
pot fi reg
asite de fapt n culegerile de probleme de ast
azi.
n ncheiere, subliniem nca odata rolul remarcabil avut de revista Recreatii stiintif ice n dezvoltarea si impulsionarea nv
atamntului matematic romnesc n concordanta cu cel european, ce era urm
arit ndeaproape.
Nu aveau s
a treac
a multi ani dup
a ncetarea brusc
a a aparitiei acestei reviste si
la universitatea ieseana un fost rezolvitor al Recreatiilor stiintifice elabora primele
lucr
ari originale de matematic
a. Este vorba de marele matematician romn Dimitrie
Pompeiu, ale c
arui rezultate privitoare la teorema cresterilor finite, obtinute n perioada cnd functiona ca profesor al Universit
atii din Iasi, sunt citate si ast
azi, impresionnd prin profunzimea si eleganta lor. Cercetarile sale de analiza matematica
sunt de fapt primele cercet
ari stiintifice originale de matematic
a la Universitatea din
Iasi.
Invocnd anterior numele lui Catalan, matematician cu vaste preocup
ari matematice (analiza, algebra, geometrie, teoria numerelor), se cuvine a aminti numele unuia dintre primii autori romni ai unor cercet
ari stiintifice originale de matematic
a,
N.St.
a identitate legat
a de seria armonic
a, cunoscut
a
Botez, care stabileste o frumoas
azi ca identitatea Catalan-Botez, rolul lui Catalan fiind acela de a o mentiona n unul
din articolele sale cu precizarea lui Botez ca autor.
Revista Recreatii stiintif ice constituie pasul premerg
ator aparitiei Analelor
stiintif ice ale Universitatii "Al.I. Cuza", n 1900, revist
a dedicat
a cercet
arilor originale ale profesorilor universitatii iesene, dar care a publicat nca din primele numere si
articole ale unor recunoscuti matematicieni straini ca Lucien Godeaux, Mauro Picone,
Kentaro Yano, T.J. Willmore s.a.

Semnalam cititorilor reeditarea colectiei complete a revistei

RECREA
TII S
TIIN
TIFICE (1883-1888),
la 125 de ani de la aparitia primului numar, cu respectarea formei n care a fost
publicata initial. Revista prezinta si astazi interes prin culoarea limbii romne si
terminologiei folosite, prin continutul interesant si de un nalt nivel stiintific, precum
si prin forma grafic
a frumoas
a. Cei interesati pot consulta site-ul revistei

http://www.recreatiistiintifice.ro
de unde se poate prelua gratuit.
108

Despre calendar
Filip REICHER1
Omagiu adus revistei Recreatii Stiin
tifice

la 125 de ani de la aparitie

Nevoia m
asur
arii timpului. Din cele mai vechi timpuri oamenii simteau nevoia
de a m
asura trecerea timpului, pentru a sti ct timp a trecut de cnd a avut loc
un anumit eveniment. Prima posibilitate de a m
asura timpul, a fost de a observa
fazele succesive ale Lunii. Astronomii antichitatii au reusit sa determine cu oarecare
exactitate timpul dintre fazele de acelasi fel ale Lunii (Luna noua, primul patrar,
Luna plin
a, al treilea p
atrar). De cea mai mare importanta era ns
a de a se putea
determina aparitia anotimpurilor succesive.
Calendarul egiptenilor din antichitate. n Egipt se ntmpla un fenomen al
naturii la nceputul fiec
arei veri, care era de cea mai mare importanta pentru agricultura: revarsarea fluviului Nil pe o parte a pamntului, care facea terenul roditor.
Egiptenii au observat, spre marea lor surprindere, ca trei fenomene ale naturii se ntmplau cu regularitate la acelasi timp: nceputul verii, rev
arsarea Nilului si o pozitie
anumit
a a stelei Sirius pe cer. (Aparitia stelei la orizont, naintea r
as
aritului soarelui). Astfel, preotii egipteni au putut sa determine lungimea anului; a fost adoptat
un calendar, care cuprindea 365 de zile.
S-au stabilit 12 luni cu cte 30 de zile si suplimentar 5 zile, dup
a cele 360 de zile ale
celor 12 luni. Primul calendar egiptean, bazat pe miscarea Soarelui pe cer, inclusiv
a pozitiilor stelelor, a fost adoptat n anul 4241 .Hr. Mai trziu a fost determinat si
anul cu lungimea de 365, 25 zile, prin adaugarea unei zile odata la 4 ani. Aceasta a
apropiat lungimea anului egiptean de lungimea exact
a a anului astonomic, care a fost
determinat
a mai trziu de astronomii moderni, anume de 365 zile, 5 ore, 48 minute
si 46 secunde (ca numar zecimal: 365, 2422 zile.)
Calendarul iulian. Acesta este denumit dup
a mp
aratul roman Caius Iulius
Caesar si are 365 de zile ntr-un an ordinar si, o data la 4 ani, 366 zile anul bisect.
Abaterea calendarului iulian. n raport cu anul adevarat, anul astronomic, numit
si anul tropic, care cuprinde 365, 2422 zile, calendarul iulian are o diferenta (abatere),
care face ca, n conformitate cu acest calendar, anotimpurile s
a se schimbe succesiv
dupa o data ntrziata.
Pentru a calcula abaterea, luam valoarea medie a anului, conform acestui calendar:
(4 365 + 1) : 4 = 365, 25. Diferenta fata de anul adev
arat (astronomic) este de
365, 25 365, 2422 = 0, 0078 zile ntr-un an. Dup
a cti ani acest calendar r
amne n
urma cu o zi? Rezultatul se obtine din mpartirea 1/0, 0078 = 128, 2 ani.
Deoarece dupa calendarul iulian era pierduta (ntrziata) o zi la fiecare perioada
de 128 ani si nceputul prim
averii 21 martie era indicat mereu mai trziu, ap
area
o piedic
a n a stabili din timp s
arb
atorile, mai ales Pastele.
Acest lucru s-a remediat prin adoptarea unui calendar nou, calendarul gregorian.
1

Inginer dr., Facultatea de chimie, Univ. Tehnic


a "Gh. Asachi", Iasi

109

Calendarul gregorian. Acesta este denumit dup


a papa Grigore al XIII-lea.
Deoarece abaterea calendarului iulian era prea mare, s-a adoptat un calendar nou,
la care abatarea fata de anul astronomic este mult mai mica.
Conform acestui calendar anii de fine ai secolelor, la care num
arul nu este divizibil
prin 400, nu sunt ani bisecti. Deci, anii 1700, 1800, 1900, 2100, 2200 etc. nu sunt
ani bisecti, n timp ce anii 1600, 2000, 2400, etc. sunt ani bisecti.
Abaterea calendarului gregorian. n intervalul de 400 de ani, acest calendar nu
are 100 de ani bisecti o dat
a la 4 ani ci numai 97 de ani bisecti. Valoarea medie
a anului este de aceea (303 365 + 97 366) : 400 = 365, 2425 zile. Abaterea fata de
anul astronomic este de 365, 2425 365, 2422 = 0, 0003 zile. n cti ani ramne acest
calendar n urma cu o zi? Raspunsul rezulta din mpartirea 1 : 0, 0003 = 3333 ani.
Calendarul gregorian este cel mai exact dintre calendare (mai exact dect calendarul iudaic, aceasta o vom vedea mai jos).
Calendarul iudaic. Ca si alte popoare, evreii au ales sa masoare timpul potrivit
misc
arii Lunii n jurul P
amntului, anume, potrivit fazelor Lunii. nceputul fiec
arei
luni Lunare era determinat prin aparitia secerii subtiri a Lunii pe cer, imediat dup
a
Luna Noua. n fiecare luna se ntrunea "Consiliul Calendarului" si aparitia secerii
subtiri a Lunii pe cer numita "Moled", adica nasterea Lunii trebuia sa fie confirmat
a de c
atre doi martori credibili. "Consiliul Calendarului" se ntrunea dup
a 30 de
zile; dac
a secera subtire a Lunii nu ap
area, ziua a 31-a era stabilit
a drept prima zi
a lunii urmatoare. Apoi erau trimisi oameni n tara si se aprindeau focuri pe coline,
pentru ca toti oamenii sa ia la cunostinta ca a nceput noua luna.
ncepnd din sec. IV .Hr., calendarul Lunar a fost nlocuit n mod treptat cu
calendarul Luni-Solar. Deoarece n biblie este prev
azut c
a eliberarea evreilor din
robia egipteana a avut loc primavara, evreii au dorit sa potriveasca astfel calendarul,
ca sarbatoarea Pastelui Pesah , care aminteste de acest eveniment, sa aiba loc
ntotdeauna n prim
avar
a. Deci, trebuia construit un calendar care s
a potriveasc
a
anul iudaic cu anul astronomic. O lun
a astronomic
a dureaz
a 29 zile, 12 ore, 44 minute
si 2,8 secunde (sau, ca numar zecimal: 29, 53059 zile). Dar 12 luni Lunare dureaza
mai putin dect un an astronomic: 12 29, 53059 = 354, 36708 zile, asa ca trebuia
s
a fie intercalat
a o lun
a suplimentar
a, din cnd n cnd.
Aceast
a problem
a a fost rezolvat
a n antichitate de astronomul Meton din Atena.
Solutia sa se cheama Ciclul lui Meton, care cuprinde 19 ani, n care, pe lnga 12 luni
pe an, se intercaleaza 7 luni suplimentare. Se are n vedere calculul:
durata unui an, conform calendarului cunoscut n acea vreme: 365, 25 zile;
19 ani solari = 19 365, 25 zile = 6939, 75 zile;
(19 12) + 7 luni = 235 luni Lunare; 235 29, 53059 zile = 6939, 68865 zile.
Este o diferenta foarte mica, deci acceptabila. n ciclul lui Meton, urmatorii 7 ani
cuprind a 13-a lun
a: cele cu num
arul de ordine 0, 3, 6, 8 11, 14, 17 din cadrul ciclului.
Calendarul iudaic a adoptat exact aceast
a succesiune de ani, cnd se include o
luna suplimentara. Aceasta luna se cheama ADAR II sau VE-ADAR.
Calendarul iudaic are doua particularitati. Lungimea unui an poate avea numai
353, 354 sau 355 zile ntr-un an obisnuit (ordinar) si 383, 384 sau 385 zile ntr-un an
cu 13 luni (an bisect). A doua particularitate a calendarului iudaic const
a n aceea,
ca data de 1 Tisri (Anul Nou iudaic) nu poate fi niciodata ntr-o duminica, ntr-o
110

miercuri sau ntr-o vineri.


Este usor de stabilit cnd avem un an bisect. Se mparte numarul anului din
calendarul iudaic prin 19. Daca restul este unul din numerele de mai sus, adica 0,
3, 6, 8, 11, 14 sau 17, acel an este bisect. De exemplu: anul 5744 (1983 1984
n caledarul gregorian) este un an bisect, deoarece mp
artirea prin 19 d
a ctul 302
si restul este 6. Mult mai dificil este de a se stabili lungimea unui an iudaic (353,
354 sau 355 de zile ntr-un an ordinar, respectiv 383, 384 sau 385 de zile ntr-un an
bisect). Calendarul iudaic are o ntziere de o zi n 216, 4 ani.
Addendum. La catolici si protestanti, data Pastelui este aceeasi. Ea se stabileste
dupa o regula simpla: Duminica Pastelui este prima duminica de dupa prima luna
plina ce apare dupa echinoxiul de primavara.
2005
2006
2007
2008

Luna plina
05.03
13.04
02.04
21.03

Data Pastelui
27.03
16.04
08.04
23.03

2009
2010
2011
2012

Luna plina
09.04
30.03
18.04
06.04

Data Pastelui
12.04
04.04
24.04
08.04

Data Pastelui pentru ortodocsi se stabileste dup


a un algoritm mult mai complicat.
Interesant este faptul c
a acest algoritm se bazeaz
a tot pe ciclul de 19 ani, ca si pentru
calendarul iudaic. De aceea, nu ntmpl
ator, Duminica Pastelui pentru ortodocsi este
ntotdeauna n ultima zi, putin naintea ultimei zile sau imediat dupa ultima zi din
cele 8 zile ale s
arb
atorii Pastelui (PESAH) din calendarul iudaic.

2005
2006
2007
2008

Ultima zi
de PESAH
dum. 01.05
joi 20.04
marti 10.04
dum. 27.04

Pastele
ortodox
01.05
23.04
08.04
27.04

2009
2010
2011
2012

Ultima zi
de PESAH
joi 16.04
marti 06.04
marti 26.04
smb. 14.04

Pastele
ortodox
19.04
04.04
24.04
15.04

Bibliografie
1.
2.
3.
4.

G. Petrescu - Astronomie elementara, Bucuresti, 1962


G. St
anil
a - Sisteme calendaristice, Bucuresti, 1980
www.jewfaq.org: Judaism 101
LUAH 5743 (Calendar 1982 -1983), editat de Federatia Comunitatilor evreiesti din
Romnia
Nota Redactiei. n revista Recreatii Stiin
asurarea timpu tifice probleme ca: m
lui, alcatuirea unui calendar, stabilirea datei Pastelui au fost ndelung dezbatute si
aprofundate. ntr-un ciclu de noua articole (aparute n vol. I(1883), vol. II(1884) si
vol. III(1885)), G. I. Lucescu, profesor la Liceul National din Iasi, face un istoric al
calendarului, din cele mai vechi timpuri si pn
a la adoptarea calendarului gregorian.
n cuprinsul a cinci scrisori publicate n vol. VI(1888), Constantin Gogu, profesor la Universitatea din Bucuresti, se ocupa de regulile pentru gasirea zilei Pastelui.
Paul Tanco din N
as
aud, primul romn cu titlul de doctor n matematici, ridic
a
problema periodicit
atii cu care Pastele cade a doua zi dup
a Sf. Gheorghe.
111

Cteva probleme de teoria numerelor a c


aror
rezolvare se bazeaz
a pe identit
ati
Marian TETIVA1
Cnd vorbim despre utilizarea identit
atilor n teoria numerelor, probabil c
a ne
gndim n primul rnd la ecuatii diofantice, n special la demonstrarea existentei
solutiilor unor asemenea ecuatii. De exemplu, identitatile
(m2 n2 )2 + (2mn)2 = (m2 + n2 )2

si

(m2 2mn n2 )2 + (m2 + 2mn n2 )2 = 2(m2 + n2 )2


arata ca ecuatiile x2 + y 2 = z 2 si, respectiv, x2 + y 2 = 2z 2 au, fiecare, o infinitate
de solutii ntregi. Totusi, n cele ce urmeaza, vom rezolva alte tipuri de probleme de
teoria numerelor cu ajutorul identit
atilor.
Avem n minte mai ales dou
a identit
ati, binecunoscute cititorilor. Este vorba de
n1

si de

(x y)(x + y)(x2 + y 2 ) (x2

n1

+ y2

n1

(x2 + xy + y 2 )(x2 xy + y 2 )(x4 x2 y 2 + y 4 ) (x2 x2


n+1

= x2

n+1

+ x2 y 2 + y 2

) = x2 y 2

n1

y2

(1)
n

+ y2 ) =
(2)

Ambele sunt valabile pentru orice numere complexe x si y (dar, desigur ca pe noi ne
vor interesa pentru numere ntregi) si orice num
ar natural n 1. Se justific
a la fel,
prin aplicarea repetat
a a formulei (a b)(a + b) = a2 b2 .
Problema cea mai cunoscuta care utilizeaza (1) este probabil
Problema 1. Fie n 1 un numar natural. Sa se arate ca numarul N = 11 . . . 1
scris n baza 10 cu 2n cifre de 1 are cel putin n divizori primi distincti.
Solutie. ntr-adev
ar, avem
n

n1

N = (102 1)/9 = (10 + 1)(102 + 1) (102

+ 1)

deci (1) ne permite s


a scriem pe N ca produsul a n numere; dac
a reusim s
a ar
at
am
c
a acestea sunt prime ntre ele dou
a cte dou
a problema ar fi rezolvat
a, deoarece
fiecare din aceste n numere ar aduce (cel putin) un factor prim n descompunerea lui
N ca produs de numere prime.
i
j
Avem, pentru 0 i < j, c
a 102 + 1 divide pe 102 1 (tot pe baza identit
atii
i
2
2j
(1)), deci daca d este un divizor comun pentru 10 + 1 si 10 + 1, atunci d este
j
j
divizor si al lui 2 = 102 + 1 (102 1). Cum d este impar, rezulta d = 1 si solutia
se ncheie.
Problema care urmeaz
a a ap
arut acum ceva vreme n Recreatii matematice, nr.
1/2005, pag. 42, Problema 2, cl. a X-a, Lucian Tutescu.
Problema 2. Exista o infinitate de numere n N astfel nct n2+n+1 divide pe n!.
Solutie. Sa pornim de la un caz particular al identitatii (2):
m8 + m4 + 1 = (m2 + m + 1)(m2 m + 1)(m4 m2 + 1)
1

Profesor, Colegiul National "Gheorghe Rosca Codreanu", Brlad

112

si s
a observ
am c
a avem
m2 m + 1 < m2 + m + 1 < m4 m2 + 1 < m4
pentru orice numar natural m 2. Aceste inegalitati arata ca (m4 )! se divide cu
(m2 + m + 1)(m2 m + 1)(m4 m2 + 1), deci cu (m4 )2 + m4 + 1 pentru orice m 2
num
ar natural. Problema este asadar rezolvat
a: putem alege n = m4 , m 2. (De
2
exemplu, 16! se divide cu 16 + 16 + 1 = 7 3 13; de fapt, n = 16 este cel mai mic
cu proprietatea din enunt.)
O problem
a asem
an
atoare a ap
arut mai demult n American Mathematical Monthly:
Problema 3. Exista o inf initate de numere n N astfel nct n2+1 divide pe n!.
Solutie. De data asta ne vom folosi de identitatea
4m4 + 1 = (2m2 2m + 1)(2m2 + 2m + 1),
unde vom ncerca s
a mai descompunem si cel de-al doilea factor. Pentru asta s
a
observ
am c
a
4m2 + 4m + 2 = (2m + 1)2 + 1 = 2p2 2m2 + 2m + 1 = p2 ,
daca 2m + 1 si p sunt solutii ale ecuatiei x2 2y 2 = 1.
Dar acest lucru este cunoscut: ecuatia men
a
are, ntr-adev
ar, o infinitate
tionat
de solutii. Mai precis, dac
a not
a
m xk + yk 2 = (1 + 2)2k+1 , cu xk si yk numere
naturale, atunci avem si xk yk 2 = (1 2)2k+1 , deci

x2k 2yk2 = (xk + yk 2)(xk yk 2) = ((1 + 2)(1 2))2k+1 = (1)2k+1 = 1


pentru orice k; de exemplu, primele trei solutii (care corespund lui k = 0, k = 1,
respectiv k = 2) sunt (1, 1), (7, 5) si (41, 29). n plus, se arata destul de usor ca xk
este impar pentru orice k.
Atunci, este suficient s
a alegem perechea (2m + 1, p) ca fiind una dintre solutiile
(xk , yk ) ale ecuatiei x2 2y 2 = 1, pentru a avea
(2m2 )2 + 1 = (2m2 2m + 1)p2 .
n aceste conditii,
4m4 + 1 = (2m2 2m + 1)p2 > 17p2 16p2 + 1,
dac
a m este suficient de mare (ceea ce se poate, deoarece sirul (xk ) tinde la ;
de fapt, pentru k 2, avem m = (xk 1)/2 20, ceea ce asigura valabilitatea
inegalitatii care ne trebuie), de unde obtinem
m2 > 2p 2m2 2m + 1 m2 > 2p.
Astfel c
a 2m2 > 2m2 2m + 1 > 2p > p, deci produsul (2m2 )! contine factorii
(distincti) 2m2 2m + 1, 2p si p, deci se divide cu (2m2 2m + 1)p2 = (2m2 )2 + 1;
prin urmare sunt solutii toate numerele n = 2m2 , unde m = (xk 1)/2, k 2.
Se poate vedea prin calcul direct c
a solutia cea mai mic
a este n = 18 (18! se divide
cu 182 + 1 = 52 13). Aceast
a solutie face parte din sirul de mai sus; se obtine pentru
k = 1, cnd m = 3 si p = 5 (inegalitatea 2m2 2m + 1 > 2p are loc si n acest caz,
chiar daca nu are loc m2 > 2p).
Problema 4. Pentru un numar natural n 2 notam cu h(n) cel mai mare
divizor prim al lui n. Sa se arate ca exista o inf initate de numere n astfel nct
h(n) < h(n + 1) < h(n + 2).
113

Solutie. De ast
a dat
a, pe lng
a identitatea (2), vom folosi si o idee ceva mai
subtila. Anume, sa fixam un numar prim impar p si sa observam ca, la fel ca la
Problema 1, oricare doua dintre numerele
k

p + 1, p2 + 1, . . . , p2 + 1, . . .
au cel mai mare divizor comun 2. De aceea exist
a unul dintre ele care are m
acar
un factor prim mai mare dect p. S
a consider
am primul dintre aceste numere, adic
a
k
j
fie k acel num
ar natural pentru care h(p2 + 1) > p si h(p2 + 1) < p pentru j =
s
1, 2, . . . , k 1 (clar, nu putem avea h(p2 + 1) = p, deoarece nici unul dintre aceste
k
numere nu poate avea factorul p). Numarul n = p2 1 are atunci proprietatea din
k
enunt. ntr-adevar, h(n + 1) < h(n + 2) nseamna p < h(p2 + 1), deci rezulta din
alegerea lui k; si tot din alegerea lui k rezult
a si cealalt
a inegalitate, deoarece
k

k1

h(p2 1) = h((p 1)(p + 1)(p2 + 1) (p2

+ 1)) =
k1

= max{(h(p 1), h(p + 1), h(p2 + 1), . . . , h(p2

+ 1)} < p.

Astfel vedem ca pentru fiecare numar prim impar p exista un numar ntreg pozitiv
k
k astfel nct n = p2 1 sa aiba proprietatea h(n) < h(n + 1) < h(n + 2), q.e.d.
Exercitii pentru cititor.
Problema 5. Fie a, b, c numere ntregi astfel nct ab nu este p
atrat perfect si
a, b sunt pozitive. Ar
atati c
a dac
a ecuatia ax2 by 2 = c are o solutie n multimea
numerelor ntregi, atunci ea are o infinitate de asemenea solutii.
Problema 6. Sa se dea o alta solutie Problemei 3 folosind identitatea
(4x2 2x + 1)2 + 1 = 2(4x2 + 1)(2x2 2x + 1).

Verificati aceasta identitate!


De asemenea, puteti obtine o solutie a problemei folosind identitatea
(x2 + 1)((x + 1)2 + 1) = (x2 + x + 1)2 + 1.
Problema 7. Fie p > 0 un num
ar prim. S
a se arate c
a din oricare 2p 1 numere
ntregi x1 , . . . , x2p1 se pot alege p a caror suma se divide cu p.
Indicatie. Aceasta nu e o problema usoara. De fapt ea este valabila pentru orice
num
ar natural n (adic
a din oricare 2n 1 numere ntregi x1 , . . . , x2n1 se pot alege
n a c
aror sum
a se divide cu n) si n aceast
a form
a se numeste teorema ErdsGinzburg-Ziv (a fost pentru prima data demonstrata de cei trei matematicieni n
1961). Se poate arata ca acest enunt are proprietatea de multiplicativitate, n sensul
c
a dac
a este adev
arat pentru n = a si n = b, atunci este adev
arat si pentru n = ab
(a se vedea cartea lui Horea Banea de Probleme traduse din revista Kvant); prin
urmare demonstrarea sa pentru n numar prim i asigura valabilitatea pentru orice n.
S
i ajungem acum si la indicatia promisa: utilizati identitatea
X
X p1
X
(x1 + + xp )p1
x1 = 0
(x1 + + xp1 )p1 + + (1)p1

pentru care se poate consulta, de exemplu, Ioan Tomescu, Probleme de combinatorica si teoria grafurilor, E. D. P., Bucuresti, 1981. Sumele se fac dup
a toate
posibilitatile de a alege din cele 2p 1 numere cte p, p 1, . . . , respectiv cte unul.
114

O caracterizare a punctului Mathot


1

C
at
alin TIG
AERU

Punctul lui Mathot (sau anticentrul) unui patrulater inscriptibil este definit ca
punctul de intersectie al perpendicularelor duse din mijlocul fiec
arei laturi ale patrulaterului pe latura opusa. Numeroase proprietati ale acestui punct au fost puse n
evidenta, cele mai spectaculoase fiind n leg
atur
a cu cele patru triunghiuri formate
de cte dou
a laturi adiacente ale patrulaterului si cte o diagonal
a.
n acest
a not
a punem n evidenta o caracterizare a punctului Mathot care se refer
a
la cele patru triunghiuri formate de cte o latura si cte doua segmente determinate
pe diagonale de punctul lor de intersectie. Mai precis, demonstr
am
Teorema 1. Se considera patrulaterul inscriptibil ABCD, nscris n cercul de
centru O, n care se noteaza cu E intersectia diagonalelor AC si BD si cu H1 , H2 ,
H3 , H4 ortocentrele triunghiurilor AEB, BEC, CED si respectiv DEA. Atunci:
(a) Patrulaterul H1 H2 H3 H4 este paralelogram.
(b) Intersectia diagonalelor paralelogramului H1 H2 H3 H4 coincide cu punctul Mathot al patrulaterului ABCD.
Pentru demonstratie folosim
Lema 1. Daca O1 , O2 , O3 , O4 sunt centrele cercurilor circumscrise triunghiurilor AEB, BEC, CED si respectiv DEA, atunci
(a) Patrulaterul O1 O2 O3 O4 este paralelogram.
(b) Daca este punctul de intersectie a diagonalelor paralelogramului O1 O2 O3 O4 ,
atunci punctele O, , E sunt coliniare, punctul fiind mijlocul segmentului [OE].
Demonstratie. Cititorul poate verifica imediat faptul c
a patrulaterul O1 O2 O3 O4 este paralelogram. Not
am
cu F piciorul perpendicularei din E pe AB si unim E cu
\ pe de
O3 . Pe de o parte avem m(F[
EA) = 90 m(BAE);

\
\
\ =
alt
a parte m(CEO3 ) = 90 m(CDE); cum m(BAE)
\ rezult
\3 ), adic
m(CDE),
a c
a m(F[
EA) = m(CEO
a F,
E, O3 sunt coliniare, deci EO3 AB, deci O3 E k OO1 ;
analog se demonstreaza ca O1 E k OO3 , O4 E k OO2 ,
O2 E k OO4 , de unde rezulta ca patrulaterele O1 EO3 O,
O2 EO4 O sunt paralelograme. Cum diagonalele paralelogramelor se njum
at
atesc, rezult
a c
a punctul este mijlocul segmentului [OE].
Demonstratia Teoremei 1. Vom folosi si urmatoarele rezultate:
(A) Dac
a U V W Z este un paralelogram, S este intersectia diagonalelor sale si M

este un punct oarecare din plan, atunci 4M S = M U + M V + M W + M Z ;
(B) (Sylvester) Daca M este centrul cercului circumscris triunghiului U V W si

dac
a S este ortocentrul triunghiului, atunci M S = M U + M V + M W ;
(C) Daca ABCD este inscriptibil, daca O este centrul cercului circumscris si daca
este punctul Mathot al patrulaterului, atunci

OA + OB + OC + OD = 2O .
(1)
1

Lect. dr., Univ. "Stefan cel Mare", Suceava

115


Se demonstreaz
a imediat c
a H1 H2 H3 H4 este paralelogram. Putem scrie: OA =

OO4 + O4 A = OO1 + O1 A, OB = OO1 + O1 B = OO2 + O2 B, OC = OO2 + O2 C =

OO3 + O3 C, OD = OO3 + O3 D = OO4 + O4 D, de unde, prin nsumare, obtinem:


2 OA + OB + OC + OD = 2 OO1 + OO2 + OO3 + OO4 +

+ O1 A + O1 B + O1 E + O2 B + O2 C + O2 E + O3 C + O3 D + O3 E +
(B)
+ O4 D + O4 A + O4 E + EO1 + EO2 + EO3 + EO4 =
4
4
4
4
4
4
X
X X X X X
= 2 OOi + Oi Hi + EOi =
OOi + Oi Hi + OOi + EOi =
i=1

i=1

= (A) + lema =

i=1

4
X

i=1

i=1

i=1

OHi + 4O + 4E.

i=1


Dac
Conform lemei, rezult
a c
a O + E = 0.
a
notam cu 0 punctul de intersectie a diagonalelor paralelogramului H1 H2 H3 H4 si, tinnd cont din nou de (A),
n
P
OHi = 4O0 . Ca urmare, obtinem
avem
i=1

2 OA + OB + OC + OD = 4O0 .

(2)

Din (1) si (2) obtinem ca 4O = 4O0 , de unde deducem


c
a 0 si teorema este demonstrat
a. (Cele spuse se
pot urm
ari pe figura al
aturat
a.)
O consecinta imediat
a a teoremei este si relatia vectorial
a
4
X

Oi Hi = 4,

(3)

i=1

care se deduce imediat folosind (A).

Este posibil ca rezultatul notei sa nu fie nou, dar sigur nu este trecut, de exemplu,
printre proprietatile punctului Mathot, demonstrate n capitolul consacrat subiectului, din monografia "Problems in plane and solid geometry", scris
a de Viktor Prasolov, care este accesibil
a pe Internet. Preciz
am c
a autorul nu a g
asit rezultatul nici
n cartile citate n bibliografie, nici n alte carti clasice de geometrie, scrise n limba
romna.
Bibliografie
1. D. Mihalcea, I. Chitescu, M. Chirita
- Geometria patrulaterului, Ed. Teora,
Seria Bacalaureat-Admitere, nr. 24, 1998.
2. C. Mihalescu - Geometria elementelor remarcabile, Bibl. Soc. S
t. Matematice a
S.S.M.R., Ed. Tehnica, Bucuresti, 2007.
116

Unsprezece p
atrate perfecte
Dan POPESCU 1
Scopul acestei note este determinarea numerelor naturale n baza zece de
forma aa
. . . a} b |cc {z
. . . }c d, n N , care, pentru orice num
ar natural n, sunt
| {z
n cifre

n cifre

p
atrate perfecte.
Se va vedea ca rezultatul obtinut are drept consecinte un numar mare de probleme
publicate n reviste de specialitate destinate elevilor (de gimnaziu).
Din modul cum s-a formulat problema, rezult
a c
a p
atratele perfecte c
autate se
g
asesc printre cele ce corespund unei valori particulare a lui n. Pentru n = 4 se
gasesc, cu ajutorul calculatorului urmatoarele 11 patrate perfecte de forma dorita:
1)
2)
3)
4)
5)
6)
7)
8)
9)
10)
11)

1111022224 = 333322 ;
1111088889 = 333332 ;
1111155556 = 333342 ;
1111222225 = 333352 ;
4444222225 = 666652 ;
4444355556 = 666662 ;
4444488889 = 666672 ;
4444622224 = 666682 ;
9999400009 = 999972 ;
9999600004 = 999982 ;
9999800001 = 999992 ;

a = 1, b = 0, c = 2, d = 4,
a = 1, b = 0, c = 8, d = 9,
a = 1, b = 1, c = 5, d = 6,
a = 1, b = 2, c = 2, d = 5,
a = 4, b = 2, c = 2, d = 5,
a = 4, b = 3, c = 5, d = 6,
a = 4, b = 4, c = 8, d = 9,
a = 4, b = 6, c = 2, d = 4,
a = 9, b = 4, c = 0, d = 9,
a = 9, b = 6, c = 0, d = 4,
a = 9, b = 8, c = 0, d = 1.

Vom arata ca exista exact 11 numere de forma aa


. . . a} b |cc {z
. . . }c d care sunt patrate
| {z
n

perfecte pentru orice n N , anume, acelea ce se scriu cu sistemele de cifre (a, b, c,


d) pe care le-am ntlnit mai sus (n cazul n = 4), adic
a
(1)

11
. . . 1} 0 22
. . . 2} 4 = 33
. . . 3} 2 ,
| {z
| {z
| {z
n

(2)

(9)

n+1

n+1

n+1

Profesor, Colegiul National "Stefan cel Mare", Suceava

117

99
. . . 9} 6 00
. . . 0} 4 = 99
. . . 9} 8 ,
| {z
| {z
| {z
n

99
. . . 9} 8 00
. . . 0} 1 = 99
. . . 9} .
| {z
| {z
| {z
n

44
. . . 4} 3 55
. . . 5} 6 = 66
. . . 6} ,
| {z
| {z
| {z

(11)

99
. . . 9} 4 00
. . . 0} 9 = 99
. . . 9} 7 ,
| {z
| {z
| {z
n

(10)

44
. . . 4} 6 22
. . . 2} 4 = 66
. . . 6} 8 ,
| {z
| {z
| {z
n

44
. . . 4} 22
. . . 2} 5 = 66
. . . 6} 5 ,
| {z
| {z
| {z
n

(8)

11
. . . 1} 22
. . . 2} 5 = 33
. . . 3} 5 ,
| {z
| {z
| {z
n

(6)

n+1

11
. . . 1} 55
. . . 5} 6 = 33
. . . 3} 4 ,
| {z
| {z
| {z
n

(5)

44
. . . 4} 88
. . . 8} 9 = 66
. . . 6} 7 ,
| {z
| {z
| {z
n+1

n+1

(4)

11
. . . 1} 0 88
. . . 8} 9 = 33
. . . 3} ,
| {z
| {z
| {z
n

(3)

(7)

n+1

n scopul propus, s
a not
am xn = aa
. . . a} b |cc {z
. . . }c d, n N . Putem scrie
| {z
n

xn = aa
. . . a} 10n+1 a 10n+1 + b 10n+1 + cc
. . . }c + d c =
| {z
| {z
n+1

n+1

. . . a} 10n+1 1 + (b a) 10n+1 1 + aa
. . . a} + cc
. . . }c + d c + b a,
= aa
| {z
| {z
| {z
n+1

n+1

n+1

adica

. . . 1} + (9b + c 8a) 11
. . . 1} + b + d a c,
xn = 9a11
| {z
| {z
n+1

n+1

n N .

()

Acum, s
a observ
am c
a n () coeficientii 9a, 9b + c 8a si b + d a c sunt aceiasi
a trinomul
pentru orice n N (ei reflectnd numai forma lui xn ). Ca urmare dac
din membrul doi este patrat perfect pentru o valoare particulara a lui n, atunci va
avea aceast
a proprietate pentru orice n N . Acest fapt verificndu-se direct pentru
n = 4, vom deduce c
a numerele (1) (11) sunt cele c
autate.
Observatia 1. i) Cititorul poate observa c
a, pentru 1 n 3, exist
a p
atrate
perfecte de forma aa
.
.
.
a
b
cc
.
.
.
c
d
care
nu
apar
printre
cele
unsprezece.
Un
exemplu
| {z } | {z }
n

pentru n = 2: 3442 = 118336. Mai precis, pentru n = 2, exista 18 patrate perfecte


de forma enuntat
a, iar pentru n = 3, num
arul lor este 12. n cazul n = 1, num
arul
lor este mult mai mare, c
aci problema se reduce la identificarea p
atratelor perfecte
cu patru cifre ale sistemului zecimal.
ii) Elevul Aursulesei Tudor, c
aruia i multumim si cu acest prilej, a verificat prin
intermediul calculatorului faptul c
a, pentru 4 n 14, singurele p
atrate perfecte
de forma aa
.
.
.
a
b
cc
.
.
.
c
d
sunt
exact
cele
unsprezece
prezentate
mai
sus.
| {z } | {z }
n

. . . a} bb
. . . }b c, n N
Observatia 2. i) Singurele p
atrate perfecte de forma aa
| {z
| {z
n

n+1

sunt: 11
. . . 1} 55
. . . 5} 6, n N si 44
. . . 4} 88
. . . 8} 9, n N .
| {z
| {z
| {z
| {z
n+1

n+1

ii) Singurul patrat perfect de forma aa


. . . a} bb
. . . }b c, n N este 44
. . . 4} 22
. . . 2} 5,
| {z
| {z
| {z
| {z

n N .

n+1

n+1

Aplicatii
. . . 1} 22
. . . 2} 5 este patrat perfect (OBM 1. Sa se arate ca numarul N = 11
| {z
| {z
1997

1998

juniori, Atena, 1998) [1].

. . . 3} 5 .
Este un caz particular al rezultatului (4); N = 33
| {z
1997

2. Sa se determine cifrele x si y, x 6= 0, daca xx


. . . x} 6 yy . . . y 4 este patrat
| {z
| {z }
n

perfect, pentru orice n N .


Rezolvarea problemei decurge din (8) si (10), deci avem (x, y) {(4, 2) , (9, 0)}.
118

3. Nu exista patrate perfecte n baza zece de forma aa


. . . a}; a 6= 0, n 2.
| {z
n

Rezult
a din cele prezentate mai sus; o alt
a abordare poate fi g
asit
a n [3].
4. Rezultatele de la (3) si (7) sunt prezente n [3].
5. Sa se arate ca numerele a = 11
. . . 1} 22
. . . 2} si b = 44
. . . 4} 88
. . . 8} sunt
| {z
| {z
| {z
| {z
2n

2n

patrate perfecte, n N, n 2.
. . . 1} 0 88
. . . 8} 9 si se aplic
a (2), iar b = 44
. . . 4} 3 55
. . . 5} 6 si se
Se arat
a c
a a = 11
| {z
| {z
| {z
| {z
n1

n1

n1

n1

aplic
a (6).
6. Sa se arate ca exista o infinitate de numere cu terminatia 0004 care sunt
patrate perfecte.
Se poate utiliza egalitatea (10).

7. Este numarul a = |4 4{z


. . . 4}3 55
. . . 5} 6 natural? (P. B
atrnetu - ONM (lista
| {z
2007

2007

scurta), Pitesti, editia 2007 [6]).

Observatia 3. i) n lista scurta cu problemele propuse la Olimpiada Nationala


de Matematica, editia 2005 [6], E. Velcea a propus problema care face obiectul
rezultatului de la (6).
ii) Rezultatul de la (4) a constituit o problem
a de la Concursul Interjudetean
"Gh.Ti
teica", editia 2004.
iii) Autorul acestei note nu a identificat enunturi legate de rezultatele de la (1),
(9) si (11).
n final, propunem urmatorul exercitiu (poate cu o alta abordare):
Sa se arate ca nu exista numere n baza zece cu scrierea pozitionala aa
. . . a} bb
. . . }b,
| {z
| {z
n

care, pentru ficare numar natural nenul n, sa fie patrate perfecte.


Bibliografie

1. D. Brnzei s.a. - 10 ani de Olimpiade Balcanice ale Juniorilor, Paralela 45, 2007.
2. N.B. Vasiliev, A.A. Egorov - Zadaci vsesoiuzni matematiceskih olimpiad-ebvisa,
Nauka, Moscova, 1988.
3. A.P. Ghioca, L.A. Cojocaru - Matematica gimnazala dincolo de manual, Gil,
Zal
au, 2005.
4. I. Cucurezeanu - Patrate si cuburi perfecte de numere ntregi, Gil, Zal
au, 2007.
5. Gazeta Matematica, Seria B, nr. 12/2005, Problema E:13095.
6. Romanian Mathematical Competitions, Theta, Bucuresti, 2005.
7. Romanian Mathematical Competitions, Theta, Bucuresti, 2007.

119

Cercuri seminscrise
si puncte de tip Gergonne sau Nagel
Temistocle BRSAN 1
Fie ABC un triunghi oarecare. Pentru cercurile circumscris, nscris, A-exnscris
etc. folosim notatiile uzuale: C (0, R), C (I, r), C (Ia , ra ) etc. Punctele de tangenta
a dreptei BC cu cercurile C (I, r) si C (Ia , ra ) se noteaz
a D si D0 ; cu E, E 0 si F , F 0
not
am punctele cu semnificatii similare relativ la dreptele CA si, respectiv, AB.
Este cunoscut faptul ca dreptele AD, BE si CF sunt concurente (ntr-un punct
punctul lui Gergonne) si, de asemenea, faptul c
a dreptele AD0 , BE 0 si CF 0 sunt
concurente (ntr-un punct N punctul lui Nagel ).
Se asociaz
a triunghiului ABC trei cercuri seminscrise: C (J1 , 1 ), C (J2 , 2 ),
C (J3 , 3 ) (C (J1 , 1 ) fiind cercul tangent dreptelor AB si AC si tangent interior cercului circumscris triunghiului etc.), precum si trei cercuri ex-seminscrise: C (Ja , a ),
C (Jb , b ), C (Jc , c ) (C (Ja , a ) fiind cercul tangent dreptelor AB si AC si tangent
exterior cercului C (O, R) etc.). Observ
am c
a avem un singur cerc nscris, dar trei
cercuri seminscrise; pe de alta parte, numarul cercurilor exnscrise este egal cu cel al
celor ex-seminscrise. Privitor la cercurile seminscrise, un num
ar de propriet
ati ale
lor sunt date n [3] si [1].
Ne propunem n aceast
a Not
a s
a
"trecem" cele doua rezultate mai sus
mentionate la cercurile seminscrise
si ex-seminscrise.
n scopul propus, s
a not
am D1 si
Da punctele de tangenta a cercurilor
C (J1 , 1 ) si, respectiv, C (Ja , a ) cu
C (O, R); E1 , Eb si F1 , Fc au semnificatii analoage.
Odata cu trecerea de la cercul
C (I, r) la cele trei cercuri seminscrise C (Ji , i ) (i = 1, 2, 3), este
firesc sa consideram n rolul cevienelor Gergonne AD, BE si CF
cevienele AD1 , BE1 si, respectiv,
CF1 . Similar, n locul cevienelor
Nagel AD0 , BE 0 si CF 0 sa consideram cevienele ADa , BEb si, respectiv, CFc legate de cercurile exseminscrise C (Ja , a ) etc.
Vom arata ca rezultatelor clasice de mai sus le corespund cele din urmatoarea
Teorem
a. a) Cevienele AD1 , BE1 si CF1 sunt concurente n centrul S al
omotetiei directe a cercurilor C (O, R) si C (I, r).
1

Prof. dr., Universitatea Tehnic


a "Gh. Asachi", Iasi

120

b) Cevienele ADa , BEb si CFc sunt concurente n centrul S 0 al omotetiei inverse


a cercurilor C (O, R) si C (I, r).

k
Demonstratie. a) Evident, omotetia HA
, cu k = 1 , transforma cercul C (I, r)
r
R
k0
0
,
cu
k
=
,
transform
a C (J1 , 1 ) n C (O, R). Ca
n C (J1 , 1 ), pe cnd omotetia HD
1
1
R
R
0
k
k
urmare, produsul HD
HA
are centrul pe AD1 si raportul kk 0 = 1
= . Cum
1
r 1
r
transforma C (I, r) n C (O, R), acest produs coincide cu omotetia directa a acestor
cercuri. n consecinta, AD1 trece prin S centrul omotetiei directe a cercurilor
R
C (I, r) si C (O, R) (situat pe OI si definit de relatia SO = SI). Similar se arata
r
c
a dreptele BE1 si CF1 trec prin S.

t
b) Se procedeaz
a la fel. HA
, cu t = a , transform
a C (I, r) n C (Ja , a ), iar
r
R
t0
0
t0
t
HDa , cu t = , transforma C (Ja , a ) n C (O, R). Omotetia produs HD
HA
,
a
a
R
cu raportul tt0 = , coincide cu omotetia inversa a cercurilor C (I, r) si C (O, R).
r
a omotetii (situat pe OI si determinat de
ADa contine centrul S 0 al acestei din urm
R
S 0 O = S 0 I). Se arat
a similar c
a si BEb , CFc trec prin S 0 . Q.e.d.
r
Observatia 1. Demonstratia standard pentru concurenta cevienelor Gergonne
(sau Nagel) se bazeaza pe reciproca teoremei lui Ceva. Acest instrument poate fi
utilizat si pentru stabilirea afirmatiilor a) si b), dar cu pretul unor calcule laborioase.
BX
c2 p b
Astfel, dac
a not
am X = BC AD1 , se g
aseste c
a
= 2
(2p = a + b + c).
XC
b pc
Aceasta relatie si cu analoagele ei fac posibila aplicarea reciprocei teoremei lui Ceva
si, deci, dovedirea concurentei dreptelor AD1 , BE1 , CF1 . Faptul ca S este punctul
lor de concurenta devine o chestiune de rutin
a, care cere noi calcule; de exemplu, se
poate utiliza Propozitia 2 din [2] si lista de coordonate triliniare din [4]. n concluzie,
este preferabila demonstratia data pe baza produsului a doua omotetii.
Observatia 2. n [3], sub form
a de problem
a propus
a cititorilor spre rezolvare,
este afirmata concurenta dreptelor AD1 , BE1 , CF1 (cu alte notatii), fara a fi facuta
vreo precizare asupra punctului lor de concurenta.
Observatia 3. n [5], ntr-o interesanta Nota de geometria triunghiului, centrele
de omotetie S si S 0 apar ca puncte de concurenta ale altor dou
a triplete de drepte
asociate unui triunghi dat.
Bibliografie
R. Bairac - Cercuri seminscrise n triunghi, Delta, 1/2006, 12-15.
T. Brsan - Ceviene izogonale si puncte de concurenta remarcabile, 9/2002, 321-326.
A. Girici - Cteva probleme despre triunghiuri si cercuri, Kvant, 11/1990, 46-48.
C. Kimberling - Centrul Points and Central Lines in the Plane of a Triangle,
Mathematics Magazine, 67(1994), no.3, 163-187.
5. I. V. Maftei - Doua puncte remarcabile ntr-un triunghi, G.M. (B) 1/2008, 1-4.

1.
2.
3.
4.

121

O rafinare a inegalit
atii lui Jensen
Florin POPOVICI 1
Cu o demonstratie simpla, stabilim un criteriu de monotonie a functiilor; ca aplicatie, prezent
am o rafinare a inegalit
atii lui Jensen, despre care credem c
a este nou
a.
1. Preliminarii. Fie a, b R, cu a < b. Se stabileste n mod obisnuit

Propozitia 1 (de tip Fermat). Fie f : [a, b] R o functie data. Daca x0 [a, b)
este un punct de maxim local (respectiv de minim local) al functiei f si f are derivata
0
0
la dreapta n R n punctul x0 , atunci f+
(x0 ) 0 (respectiv f+
(x0 ) 0).

Propozitia 2 (de tip Rolle). Daca f : [a, b] R este o functie continua pe [a, b],
care are derivata la dreapta n R pe [a, b) si
f (a) = f (b) ,

(1)

atunci exista c1 , c2 [a, b), astfel nct

0
0
f+
(c1 ) 0 f+
(c2 ) .

(2)

Demonstratie. Presupunem ca f nu-i constanta (cazul contrar fiind banal).


Conform teoremei de m
arginire a functiilor continue a lui Wieirstrass, exist
a c1 , c2
[a, b], nct f (c1 ) = max {f (x) | x [a, b]} si f (c2 ) = min {f (x) | x [a, b]}. Din (1)
rezulta ca putem alege c1 , c2 [a, b). Conform Propozitiei 1, rezulta ca are loc (2).

Propozitia 3 (de tip Lagrange). Daca f : [a, b] R este o functie continua pe


[a, b], care are derivata la dreapta n R pe [a, b), atunci exista c1 , c2 [a, b) nct
f (b) f (a)
0
0
(c1 )
(c2 ) .
f+
f+
ba
Demonstratie. Aplicam Propozitia 2 functiei g : [a, b] R, definita prin
f (b) f (a)
x, x [a, b] .
g (x) = f (x)
ba
Propozitia 4 (criteriu de monotonie). Daca f : [a, b] R este o functie continua
pe (a, b], care are derivata la dreapta n R pe [a, b) si
0
f+
(x) 0,

x [a, b),

(3)

atunci functia f este crescatoare.


Demonstratie. Fie x1 , x2 (a, b], cu x1 < x2 . Conform Propozitiei 3 , aplicate

f (x2 ) f (x1 )

0
, exist
a c1 [x1 , x2 ), astfel nct f+
(c1 )
, de unde
restrictiei f

x2 x1
(a,b]

a f
este cresc
atoare. Urmeaz
a c
a lim f (x) = l
0 f (x2 ) f (x1 ), adic
x&a

(a,b]

0
R {}. Conform ipotezei (3), avem f+
(a) 0. Rezult
a c
a f (a) l. Urmeaz
a
c
a avem f (a) f (x), x (a, b], deci functia f este cresc
atoare.

2. Rezultatul principal. Putem acum stabili urm


atoarea
Teorem
a. Fie I R un interval dat. Daca f : I R este o functie convexa,
atunci pentru orice a1 , . . . , an I, cu a1 a2 an si pentru orice p1 , . . . , pn
1

Prof. dr., Colegiul National "Gr. Moisil", Brasov

122

(0, ), are loc inegalitatea lui Jensen rafinata:

p1 a1 + . . . pn an
p1 f (a1 ) + + pn f (an )
f

p1 + + pn
p1 + + pn

(p1 + p2 ) f (a2 ) + + pn f (an )


(p1 + p2 ) a2 + + pn an

f
(4)
p1 + + pn
p1 + + pn

(p1 + + pn1 ) f (an1 ) + pn f (an )


(p1 + + pn1 ) an1 + pn an

f
0.
p1 + + pn
p1 + + pn
Demonstratie. Stabilim prima inegalitate din (4). Dac
a a1 = a2 , atunci prima
inegalitate din (4) are loc cu egalitate. Dac
a a2 = an , atunci prima inegalitate din
(4) rezulta direct din definitia convexitatii. Consideram cazul a1 < a2 < an . Fie
g : I R functia definita prin

p1 x + p2 a2 + + pn an
p1 f (x) + p2 f (a2 ) + + pn f (an )
f
, x I.
g (x) =
p1 + + pn
p1 + + pn
Deoarece functia f este convexa rezulta (a se vedea [1], 1.3) ca functia f este cona la dreapta pe (a1 , a2 ), are derivat
a la dreapta n a1 ,
tinu
a pe (a1 , a2 ], este derivabil
0
0
f+
(a1 ) R {} si derivata f+
: [a1 , a2 ) R {} este functie cresc
atoare.
Urmeaza ca functia g este continua pe (a1 , a2 ], este derivabila la dreapta pe (a1 , a2 ),
are derivata la dreapta n a1 (n R) si avem

p1
p1 x + p2 a2 + + pn an
0
0
0
f+ (x) f+
, x [a1 , a2 ).
g+ (x) =
p1 + + pn
p1 + + pn
Deoarece
p2 a2 + + pn an
p1 x + p2 a2 + + pn an
x [a1 , a2 ) x <
x<
,
p2 + + pn
p1 + + pn
0
rezult
a ca g+
(x) 0, x [a1 , a2 ). Conform Propozitiei 4, aplicata restrictiei

, rezult
a c
a functia g
este descresc
atoare. Urmeaz
a c
a avem g (a1 )
g
[a1 ,a2 ]

[a1 ,a2 ]

g (a2 ), care este prima inegalitate din (4).


Prin inductie finit
a descendent
a se obtin si celelalte inegalit
ati din (4); ultima
inegalitate din (4) se obtine direct din definitia convexit
atii.

Observatia 1. n particular, din (4) se obtine inegalitatea lui Jensen

p1 f (a1 ) + + pn f (an )
p1 a1 + + pn an
.

f
p1 + + pn
p1 + + pn
Observatia 2. Criteriul de monotonie de mai sus (Propozitia 4) este eficient n
diferite situatii. De exemplu, pe baza lui poate fi obtinuta o rafinare a inegalitatii
lui Tiberiu Popovici (a se vedea [2]).
Bibliografie
1. C. P. Niculescu, L. E. Persson - Convex Functions and Their Applications. A
Contemporary Approach, CMS Books in Mathematics, vol. 23, Springer-Verlag, New
York, 2006.
2. C. P. Niculescu, F. Popovici - A Refinement of Popoviciu s Inequality, Bull. Soc.
Sci. Math. Roum. 49 (97), No.3, 285-290.
123

Asupra unor inegalit


ati geometrice
Gheorghe IUREA1
Rezultatul principal al notei [1] este urm
atoarea
Propozitie. Fie a, b, c lungimile laturilor unui triunghi; atunci, pentru orice
x 0, au loc inegalitatile:
3

(a + b + c) (x + 1) 27 [(ab) (1x) + c (1+x)] [(bc) (1x) + a (1+x)]


[(c a) (1 x) + b (1 + x)] ,
(1)

(a+bc) x + b+ca (b+ca) x + c+ab (c+ab) x + a+bc


p
+ p
+ p
3,
(ax + b) (bx + c)
(bx + c) (cx + a)
(cx + a) (ax + b)
(b + c) x + a + c (c + a) x + b + a (a + b) x + c + b

+
+
cx + a
ax + b
bx + c

2
ax + b + bx + c + cx + a ,

(2)

(3)

n cele ce urmeaz
a, vom demonstra c
a inegalit
atile (1), (2) si (3) au loc pentru
orice a, b, c numere reale pozitive.
Cu substitutiile = (a b) (1 x) + c (1 + x), = (b c) (1 x) + a (1 + x)
si = (c a) (1 x) + b (1 + x), observnd ca + + = (a + b + c) (1 + x),
3
inegalitatea (1) se scrie sub forma ( + + ) 27 (10 ). Dac
a < 0,
0
atunci (1 ) este evident
a. Dac
a 0, cum + , + si + sunt nenegative,
rezulta ca , , sunt nenegative si atunci (10 ) urmeaza imediat din inegalitatea
mediilor (M A M G). Egalitatea se atinge pentru a = b = c si x [0, ) oarecare.
Notnd ax + b = 2 , bx + c = 2 , cx + a = 2 , cu , , > 0, inegalitatea (2)
devine
2 + 2 2 2 + 2 2 2 + 2 2
+
+
3

care, dup
a calcule, poate fi scris
a sub forma
( ) ( ) + ( ) ( ) + ( ) ( ) 0.

Aceasta este ns
a cunoscuta inegalitate Schur. Egalitatea se atinge cnd a = b = c.
Folosind aceleasi substitutii, inegalitatea (3) este echivalent
a cu
2 + 2 2 + 2 2 + 2
+
+
2 ( + + ) ,

2 2 2
2 2 2
+
+
+ + si
+
+

+ + . Egalitatea are loc pentru a = b = c.


care rezult
a prin sumarea inegalit
atilor

Bibliografie
1.

I. V. Maftei, M. Haivas - Tehnici de stabilire a unor inegalitati geometrice,


Recreatii Matematice 1/2008, 22-23.
1

Profesor, Liceul Teoretic "Dimitrie Cantemir", Iasi

124

Metoda deligament
arii si rafinarea unor inegalit
ati
Titu ZVONARU 1
Scopul acestei note este de a prezenta demonstratii elementare pentru unele inegalitati, ca si obtinerea unor rafinari ale acestora. Descrierea metodei deligamentarii
poate fi g
asit
a n [2].
Pentru nceput, o demonstratie prin metoda deligament
arii a unei inegalit
ati
cunoscute:
a
b
c
3
1.
+
+
, a, b, c > 0.
b+c c+a a+b
2
Nesbitt
1
2a b c
ab
ac
a
si, analog,
=
=
+
Solutie. Avem
b+c 2
2 (b + c)
2 (b + c) 2 (b + c)
b
1
bc
ba
c
1
ca
cb
=
+
,
=
+
. Grupnd fractiile
c+a 2
2 (c + a) 2 (c + a) a + b 2
2 (a + b) 2 (a + b)
n functie de numaratorii lor, obtinem:
ba
(a b) (c + a b c)
(a b)2
ab
+
=
=
;
2 (b + c) 2 (c + a)
2 (b + c) (c + a)
2 (b + c) (c + a)
mpreuna cu relatiile similare:
cb
(b c)2
ca
ac
(c a)2
bc
+
=
,
+
=
,
2 (c + a) 2 (a + b)
2 (c + a) (a + b)
2 (a + b) 2 (b + c)
2 (a + b) (b + c)
deducem valabilitatea inegalitatii de demonstrat.
Metoda deligamentarii, folosita n demonstratia urmatoarelor inegalitati, duce si
la obtinerea unor rafinari ale acestora. Chiar daca sunt necesare unele calcule, acestea
sunt usor de condus c
atre rezultatul dorit.
a

3 (a + b + c)
, a, b, c > 0.
4 (ab + bc + ca)
(b + c)
(c + a)
(a + b)
Darij Grinberg si Cezar Lupu
Solutie. Demonstratia din [1] face apel la inegalitatea lui Cebsev si la inegalitatea lui Gerretsen. Avem

a 4ab + 4bc + 4ca 3b2 3c2 6bc


3a
a
=
2 4 (ab + bc + ca) =
2
(b + c)
4 (ab + bc + ca) (b + c)
2.

=
si, analog,
b

(3ab + ac) (a b)

4 (ab + bc + ca) (b + c)

(3ac + ab) (a c)

4 (ab + bc + ca) (b + c)

3b
(3ab + bc) (b a)
(3bc + ab) (b c)
+
,
=
4 (ab + bc + ca)
4 (ab + bc + ca) (c + a)2 4 (ab + bc + ca) (c + a)2
3c
(3bc + ac) (c b)
(3ac + bc) (c a)
.
2 4 (ab + bc + ca) =
2 +
(a + b)
4 (ab + bc + ca) (a + b)
4 (ab + bc + ca) (a + b)2

(c + a)2
c

Com
anesti, e-mail: tzvonaru@yahoo.com

125

Grupnd convenabil, obtinem


(3ab + ac) (a b)

4 (ab + bc + ca) (b + c)

(3ab + bc) (b a)

4 (ab + bc + ca) (c + a)2

ab
(3ab + ac) (c + a) (3ab + bc) (b + c)
,

2
2
4 (ab + bc + ca)
(b + c) (c + a)

si cum
(3ab + ac) (c + a)2 (3ab + bc) (b + c)2 = 3a3 b + a3 c + 3abc2 + ac3 + 6a2 bc+

+2a2 c2 3ab3 b3 c 3abc2 bc3 6ab2 c 2b2 c2 =

2
= 3ab a b2 + c a3 b3 + c3 (a b) + 6abc (a b) + 2c2 a2 b2 =

= (a b) 3a2 b + 3ab2 + a2 c + abc + b2 c + c3 + 6abc + 2ac2 + 2bc2 =

= (ab) a2 b+ ab2 + b2 c+ bc2 + a2 c+ ac2 + 3abc+ 2a2 b+ 2ab2 + 4abc+ c3 + bc2 + ac2 =

= (a b) (a + b + c) (ab + bc + ca) + 2a2 b + 2ab2 + 4abc + c3 + bc2 + ac2 ,

deducem ca

(3ab + ac) (a b)

4 (ab + bc + ca) (b + c)2

(3ab + bc) (b a)

4 (ab + bc + ca) (c + a)2

(a b)2 (a + b + c) (ab + bc + ca) + 2a2 b + 2ab2 + 4abc + c3 + bc2 + ac2


4 (ab + bc + ca) (b + c)2 (c + a)2

(a b) (a + b + c) (ab + bc + ca)
4 (ab + bc + ca) (b + c) (c + a)2

(a b) (a + b + c)
4 (b + c)2 (c + a)2

Prin permut
ari circulare obtinem nc
a dou
a relatii similare. Rezult
a urm
atoarea
rafinare a inegalitatii date:
b
c
3 (a + b + c)
a
+
+

(b + c)2
(c + a)2
(a + b)2 4 (ab + bc + ca)
!

2
2
2
a+b+c
(b c)
(c a)
(a b)

+
+
.
4
(b + c)2 (c + a)2 (c + a)2 (a + b)2 (a + b)2 (b + c)2
b2
c2
a
a2
b
c
+ 2
+ 2

+
+
, a, b, c > 0.
2
2
+c
c +a
a + b2
b+c c+a a+b
Vasile Crtoaje
Solutie. Avem
3.

b2

a
ab (a b)
ac (a c)
a2

=
+
,
2
2
2
+c
b+c
(b + c) (b + c ) (b + c) (b2 + c2 )
b
bc (b c)
ab (b a)
b2

=
+
,
2
c + a2 c + a
(c + a) (c2 + a2 ) (c + a) (c2 + a2 )
c2
c
ac (c a)
bc (c b)

=
+
2
a + b2 a + b
(a + b) (a2 + b2 ) (a + b) (a2 + b2 )
b2

si mai departe
126

ab (a b)
ab (b a)
ab (a b)
+
=

2
2
2
2
(b + c) (b + c ) (c + a) (c + a )
(b + c) (c + a) (b2 + c2 ) (c2 + a2 )

c3 + ac2 + a2 c + a3 b3 b2 c bc2 c3 =

ab (a b)2 c2 + ac + bc + a2 + ab + b2
ab (a b)2 c2 + ac + bc + ab
=

=
(b + c) (c + a) (b2 + c2 ) (c2 + a2 )
(b + c) (c + a) (b2 + c2 ) (c2 + a2 )
2

ab (a b)
ab (a b) (b + c) (c + a)
= 2
.
2
2
2
2
(b + c) (c + a) (b + c ) (c + a )
(b + c2 ) (c2 + a2 )
Obtinem urm
atoarea rafinare a inegalit
atii n discutie:
2
2
2
b
c
a
b
c
a
+ 2
+

b2 + c2
c + a2 a2 + b2 b + c c + a a + b
2
2
2
ab (a b)
bc (b c)
ca (c a)
2
+
+
.
(b + c2 ) (c2 + a2 ) (c2 + a2 ) (a2 + b2 ) (a2 + b2 ) (b2 + c2 )
=

n ncheiere, propunem cititorilor demonstrarea si, eventual, rafinarea urm


atoarelor
inegalit
ati:
y n+1
z n+1
xn + y n + z n
xn+1
+
+

, x, y, z > 0, n N.
y+z z+x x+y
2
5. a, b, c fiind laturile unui triunghi, are loc inegalitatea
bc
ca
ab
+
+
a + b + c.
a+bc b+ca c+ab
Gabriel Dospinescu
ab
a+b
Indicatie.

= . . . etc.
a+bc
2
6. a, b, c fiind laturile unui triunghi, avem
c2 + a2
a2 + b2
1
1
1
b2 + c2
+ 3
+ 3

+
+ .
3
a + abc b + abc c + abc
ab bc ca
Titu Zvonaru si Bogdan Ionita

b2 + c2
1
1
Indicatie. 3

= . . . etc.
a + abc 2ab 2ac
4.

Bibliografie
1. C. Lupu - Asupra inegalitatii lui Gerretsen, R.M.T., 4/2006, 3-100.
2. T. Zvonaru - Inegalitati ligamentate si neligamentate, Arhimede, 5-6/2003, 8-16.
Semnal
am cititorilor reeditarea colectiei complete a revistei

RECREA
TII S
TIIN
TIFICE (1883-1888),
la 125 de ani de la aparitia primului numar, cu respectarea formei n care a fost
publicata initial. Revista prezinta si astazi interes prin culoarea limbii romne si
terminologiei folosite, prin continutul interesant si de un nalt nivel stiintific, precum
si prin forma grafic
a frumoas
a. Cei interesati pot consulta site-ul revistei

http://www.recreatiistiintifice.ro
127

O problem
a si . . . nou
a solutii
Gheorghe IUREA1 , Gabriel POPA2
n num
arul 2/2007 al Recreatiilor Matematice, Enache P
atrascu a propus spre
rezolvare problema
\ =
G133. Fie 4ABC echilateral si D un punct astfel nct BD = DC, m(BDC)

\
30 , iar BC separa A si D. Daca E (BD) cu m(BAE) = 15 , sa se arate ca
CE AC.

Solutia autorului problemei (prezentata mai jos) recurge la o constructie ajutatoare interesant
a, dar greu de g
asit. ncerc
arile de a aborda problema ntr-un mod
diferit au fost ncununate de succes ntr-o m
asur
a mai mare dect ne asteptam; n
cele ce urmeaz
a pot fi g
asite nou
a solutii ale problemei, iar cititorul probabil c
a va
mai observa si altele.
\0 ) =
Solutia 1. Not
am cu A0 simetricul lui A fata de BC. Observ
am c
a m(EBA

0 BC) = 75 60 = 15 , prin urmare EBA


\ m(A
\
\0 EAA
\0 . Deducem ca
m(EBC)
0
0
0 B) = 15 .
\
\
\
patrulaterul ABEA este inscriptibil, de unde EA B EAB, adic
a m(EA
0
0
Obtinem astfel ca 4EBA este isoscel cu EB = EA si de aici rezulta ca E se afla
pe mediatoarea segmentului [BA0 ], deci CE BA0 . Este nsa clar ca BA0 k AC,
prin urmare CE AC (fig. 1).

Fig. 1
Fig. 2
Fig. 3
Solutia 2 (Sergiu Prisacariu, Cristian Laz
ar). Fie F (BD) astfel nct
\) = 90 60 = 30 si cum m(CBF
\) = 75 , deducem
CF AC; atunci m(BCF
\
ca m(CF
B) = 75 . Rezulta ca CB = CF , de unde CA = CF , adica 4CAF este
1
2

Profesor, Liceul Teoretic "Dimitrie Cantemir", Iasi


Profesor, Colegiul National, Iasi

128

[ ) = 45 . Astfel, m(BAF
\) = 60 45 = 15 , prin
dreptunghic isoscel, cu m(CAF

\
urmare m(BAF ) = 15 si astfel F = E, de unde concluzia problemei (fig. 2).
Solutia 3 (Enache P
atrascu). Consider
am punctul S pentru care AB = BS,
\ = m(SBE)
[ = 135 , deducem ca
AB BS, iar AB separa C si S. Cum m(ABE)
[ = 45 + 15 = 60 ,
4ABE 4SBE (L.U.L.), de unde AE = SE. ns
a m(SAE)
deci 4ASE este echilateral. Rezulta ca 4ABS 4ACE (SA = AE, AB = AC,
[ = m(EAC)
[ = 45 ), prin urmare m(ACE)
[ = m(ABS)
[ = 90 (fig. 3).
iar m(SAB)
Solutia 4 ( dup
a o idee dat
a de C
at
alin Budeanu). Vom calcula laturile
4ACE n functie de a = AB.
Mai
nti,
observ
am ca, daca {M } = AD BC, avem

a 3
a 3
a
a
= a 1 + 3 . Apoi,
=
ca AD = AM + M D =
+
+

2
2 tg 15
2
2 2 3

BE
AB
31
si, pe de alta parte,
cum AE este bisectoare n 4ABD, atunci
=
=
EDp AD
2

2a2 1 + 3
2+ 3
2 AB AD

cos 15 =
= a 2. Folosind relatia lui
AE =
AB + AD
2
a 2+ 3
2
2
2
Stewart n 4BCD, obtinem
c
a
CE
BD
=
BC
DE+CD
BEBEDEBD. Am

31
ED; dupa calcule de rutina, deducem ca CE = a.
vazut mai sus ca BE =
2

n concluzie, CA = CE = a si AE = a 2 si, din reciprova teoremei lui Pitagora,


rezult
a c
a EC AC (fig. 4).

Fig. 4
Fig. 5

Solutia 5. Fie {N } = AD

CE
s
i
G
centrul
triunghiului ABC; ca n solutia

BE
31
precedenta, avem ca
=
, de unde, folosind proportiile derivate, obtinem
ED
2

DE
ca
= 31. Aplicam teorema lui Menelaus n 4BM D cu transversala E N
DB

BE DN M C
DN
C; deducem c
a
a calcule de

= 1, de unde
= 2 3 + 1 . Dup
ED N M CB
NM
129

a 3+ 3
a 3+2 3
DN
rutina, rezulta ca DN =
, DG =
, si atunci
= 3 1.
3
2
DG
DE
DN
si din reciproca teoremei lui Thales obtinem c
n concluzie,
=
a N E k BG,
DB
DG
prin urmare N E AC, tocmai concluzia problemei (fig. 5).

Solutia 6. Folosim teorema sinusurilor n triunghiurile ABE si BCD, obtinnd


BE
AB
BC
BD
c
a
=
, respectiv
=
. Se verific
a prin calcul faptul c
a
sin15
sin 30
sin 30
sin 75
sin 15
sin 30
BE
BC
\ CBD,
\ urmeaz
=
, deci
=
. Cum AB = BC si CBE
a c
a
sin 30
sin 75
AB
BD

[ = 90 (fig. 4).
\ = 30 , de unde m(ACE)
4CBE 4DBC, prin urmare m(BCE)
Solutia 7. Raportam planul
la un reper cu originea n M , unde M

= AD

a 3
a
a
a 2 + 3
BC; dac
a a = AB, atunci A 0,
, B ,0 , C
, 0 , D 0,
.
2
2
2
2
\ = m(EDA)
\ = 15 , prin urmare
Observ
am c
a 4EAD este isoscel, deoarece m(EAD)
a
ordonata lui E va fi media aritmetica a ordonatelor punctelor A si D, adica yE = .
2
xE xB
yE yB
=
, de unde xE =
Cum punctele B, E, D sunt coliniare, avem ca

yD yB
x xB
D
a 1 3
yA yC
= 3, iar panta dreptei CE
. Panta dreptei AC este mAC =
2
xA xC
yC yE
1
este mCE =
= si, cum mAC mCE = 1, rezulta ca CE AC (fig. 4).
xC xE
3
Solutia 8. Folosim reperul din solutia precedent
a, lucrnd ns
a cu numere
com
plexe. Pentru simplitate, vom considera c
a AB = 2; atunci C (1), B (1), A 3i ,


31
BE
zB + kzD
=
= k, avem zE =
= 1 3 i.
D 2 + 3 i . Cum
ED
2
1+k
zE zC
= i, prin urmare CE AC (si, n plus, CE = AC) (fig. 4).
Rezulta ca
zA zC


CD
CB + 31
BE
31
2
Solutia 9. Din
=
=
, obtinem c
a CE =
31
ED
2
1
+

2


3 1 CD
2CB +

1

=
2CB CA + 3 1 CD CA .
, deci CE CA =
3+1
3+1


a2

ns
a CB CA = CB CA cos 60 =
, CD CA = CA + AD CA = CA2
2


3
+
1
2
\=
AD AC cos CAD
a . Astfel, CE CA = 0, de unde CE CA (fig. 4).
2
Not
a. Solutia 6 sugereaz
a urm
atoarea extindere:
Se considera 4ABC isoscel ( AB = BC) si triunghiul BCD cu A, D n semi\ = 2m(BDC).
\
plane opuse fata de BC, iar m(ABC)
Daca E (BD) are prosin x2
sin
\ = m(CBD)
\ si

, unde = m(BAE),
prietatea ca
=
sin ( + + x)
sin + x2
\ atunci CE AC.
x = m(ABC),
Problema G133 se obtine pentru = 15 , = 75 si x = 60 .
130

Sur les matrices magiques


Adrien REISNER1
Toutes les matrices considres ici appartiennent lespace vectoriel rel M3 (R).
Cet espace est de dimension 9. Nous nous proposons dtudier certaines proprits
de lensemble des matrices 3 3 dites magiques dont la dfinition est la suivante:
Dfinition. Une matrice A = (aij ) M3 (R) est dite magique si les huit sommes
ai1 + ai2 + ai3 , a1j + a2j + a3j , a11 + a22 + a33 , a31 + a22 + a13 sont gales pour
i, j : 1, 2, 3. On appelera somme magique cette somme commune.
Considrons les trois matrices suivantes (videmment magiques):

1 1 1
1 2 1
0
0 , N = MT .
L = 1 1 1 , M = 0
1 1 1
1 2 1
Proposition 1. Une matrice quelconque A est somme dune matrice symtrique
A0 et dune matrice antisymtrique A00 , la dcomposition A = A0 + A00 tant unique.

1
Dmonstration. On a de faon unique: A = A0 + A00 , o A0 =
A + AT et
2
0
1
00
T
00
00T
A =
A A ; A est symtrique et A est antisymtrique (i.e. A = A00 ).
2
Proposition 2. La somme de deux matrices magiques est une matrice magique.
La transpose dune matrice magique est magique. Enfin le produit dune matrice
maqique et dun scalaire est une matrice magique. Si A est une matrice magique les
deux matrices A0 et A00 dfinies plus haut sont elles mmes magiques.
On se propose
de construiretoutes les matrices magiques antisymtriques. En
0
0 , alors: + = + = + = =
eet, si A00 =


0
0
1 1
1 , o
= = 0. On en dduit la solution gnrale: A00 = 1 0
1 1 0
est un scalaire arbitraire.
On se propose de construire toutes les matrices magiques symtriques. De mme
que prcdemment toutes les matrices symtriques A01 =
(aij ) vrifiant:
s = tr A01 =
1 1 0
1 , o est
a11 + a22 + a33 = 0 sont de la forme suivante: A01 = 1 0
0
1 1
un scalaire arbitraire. Si s 6= 0, la forme gnrale des matrices symtriques magiques
s
sobtient en ajoutant aux lments aij de la matrice prcdente. On en dduit la
3
solution gnrale:

1 1 0
1 1 1
1 + 1 1 1 ,
A0 = 1 0
0
1 1
1 1 1
1

Centre de Calcul E.N.S.T., Paris; e-mail: adrien.reisner@enst.fr

131

o , sont des rels arbitraires. Compte tenu de la Proposition 1, on obtient


immdiatement la forme gnrale des matrices magiques:

0
1 1
1 1 0
1 1 1
1 + 1 0
1 + 1 1 1 =
A = 1 0
1 1 0
0
1 1
1 1 1
1
1
= ( ) M ( + ) N + L.
2
2
Les trois matrices M , N et L tant linairement indpendentes on a, donc, le
Thorme 3. Lensemble des matrices 33 magiques est un sous-espace vectoriel
de M3 (R). Ce sous-espace vectoriel a pour dimension 3, une base tant forme par
les trois matrices L, M et N .
Etant donn les deux matrices A = M + N + L et B = 0 M + 0 N + 0 L
nous avons:
Proposition 4. Le produit AB est une matrice magique si et seulement si on a
0 = 0 = 0. La matrice L est la seule matrice ( un facteur scalaire prs) qui
soit magique et produit de deux matrices magiques.
Dmonstration. Les relations videntes: M 2 = N 2 = M L = LM = N L =
LN = 0, L2 = 3L et M N + N M = 12I 4L conduisent immdiatement :

AB = (M + N + L) 0 M + 0 N + 0 L =

20 + 6 0 40 20 6 0
80
40
= 3 0 L + 40
20 6 0 40 20 + 6 0

et par suite lquivalence: AB est une matrice magique 0 = 0 = 0. On


obtient les quatre cas suivants:

= = 0 L 0 M + 0 N + 0 L = 3 0 L (1)
= 0 = 0 (N + L) 0 N + 0 L = 3 0 L (2)
= 0 = 0 (M + L) (0 M + 0 L) = 3 0 L (3)
0 = 0 = 0
0 L (M + N + L) = 3 0 L
(4)
Les deux cas (1), (4) nen forment quun seul. Les cas (2), (3) sont identiques
un change de matrices prs. On en dduit immdiatement la deuxime partie de la
Proposition 4.
Proposition 5. Le produit dune matrice magique par une combinaison linaire
de L et I est une matrice magique.
Dmonstration. De faon vidente la matrice:
(M + N + L) (0 I + 0 L) = 0 M + 0 N + (0 + 3 0 ) L
est magique.
Proposition 6. A = M + N + L est inversible si et seulement si 6= 0.
De plus, dans le cas o 6= 0 la matrice A1 est elle mme magique.
Dmonstration. La proposition est immdiate puisque
det (M + N+ L) =
3
1
3
4
36. Dans le cas o 6= 0 il vient: A1 =
M + N + L qui est
36

bien une matrice magique.


132

Cette proposition se gnralise avec le thorme suivant:


Thorme 7. Soit A = M + N + L une matrice magique. Alors, pour tout
n N la matrice A2n+1 est magique. Si 6= 0, pour tout p N la matrice
A(2p+1) est magique.

Dmonstration. Avec A = M +N +L on obtient A2 = 12I+ 3 2 4 L


et la Proposition 5 permet alors par une rcurrence vidente sur n N de dmontrer
les implications suivantes: A2n1 est une matrice magique A2n = kI + k0 L
A2n + 1 est une matrice magique. On conclut alors grce la proposition prcdente

2p+1
puisque A(2p+1) = A1
.
Remarques. Mis part le cas o = 0 voir la Propozition 4 les matrices
A2n ne sont pas magiques.
Si = 0 les matrices A et An = 3n1 n L avec n > 1 sont magiques.
Si 6= 0 on obtient immdiatement par rcurrence sur n N:
i
1h
n
2n
n
A2n = (12) I +
(3) (12) L
3
nest pas magique.
n
2n
A2n+1 = (12) (M + N ) + (3) L
est une matrice magique.
Si 6= 0 ces mmes formules sont vrifies aussi pour n < 0.
Gnralisation. Les rsultats prcdents peuvent tre gnraliss pour des matrices magiques appartenant lespace vectoriel rel Mn (R). On considre alors les
deux sous-espaces vectoriels S0 = Mg0 Symn (R) et A0 = Mg0 Asymn (R) o Mg0
est lensemble de matrices magiques ayant une somme magique nulle et Symn (R)
[resp. Asymn (R)] est lensemble des matrices symtriques [resp. antisymtriques] de
Mn (R). Mgn (R) tant le sous-espace vectoriel des matrices magiques de Mn (R),
on dmontre que Mgn (R) = Mg0 , o est la droite engendre par la matrice
(cij ) avec cij = 1 (i, j : 1, . . . , n) voir le Thorme 3. De plus les sous-espaces vectoriels S0 et A0 admettent chacun une base forme des matrices dont les coecients
appartiennent lensemble {0, 1, 2}.

Carrs magiques. Un cas particulier des matrices magiques A = (aij )


Mgn (R) est le "carr magique". Pour construire
un carr magique on simpose

la condition supplmentaire: aij 1, 2, . . . , n2 . Exemples clbres des carrs magiques:


Le carr magique "lo-shu"attribu au
philosophe chinois Kung Tzu (Confucius)
4 9 2
(VI-me sicle avant J.C.): 3 5 7 est un carr magique connu probablement
8 1 6
bien avant Confucius. (voir: http://membres.lycos.fr/fusionbfr/JHM/CM/CM1.html)

16 3 2 13
5 10 11 8

Un autre carr magique clbre est:


9 6 7 12 qui apparat sur un
4 15 14 1
clbre tableau (eau-forte) de Albrecht Drer (1471 1528) intitul Melancolia
(voir: http://users.skynet.be/litterature/lecture/melancholia.htm).
133

Concursul de matematic
a Al. Myller
Editia a VI-a, Iasi, martie 2008
Clasa a VII-a
1. Numerele reale distincte x, y, z au propietatea c
a x3 x = y 3 y = z 3 z.
Sa se arate ca x + y + z = 0.
2. a) S
a se arate c
a, dintre cinci numere naturale oarecare, se pot alege trei
numere cu suma divizibil
a cu 3.
b) Sa se arate ca, dintre 17 numere naturale oarecare, se pot alege noua numere
cu suma divizibila cu 9.
3. Fie AD naltimea triunghiului ascutitunghic ABC. Consideram multimea M
\ = ACX.
\
a punctelor X (AD) cu proprietatea c
a ABX
a) S
a se arate c
a multimea M este nevid
a.
b) Daca M contine cel putin doua elemente, sa se demonstreze ca multimea M
contine o infinitate de elemente.
Cristian Laz
ar

4. Fie segmentul AB si semidrepta (Ox, unde O (AB) si A, B


/ (Ox. Perpendicularele n A si B pe dreapta AB intersecteaza bisectoarele (Oy si (Oz ale
[ si BOx
[ n punctele M , respectiv N . Perpendiculara din A pe (Oy
unghiurilor AOx
intersecteaza perpendiculara din B pe (Oz n punctul P . Sa se arate ca punctele M ,
N , P sunt coliniare.
Mircea Fianu

Clasa a VIII-a
1. Consideram cubul ABCDA0 B 0 C 0 D0 si M , N , P mijloacele muchiilor AB, AD,
respectiv AA0 . Sa se determine masura unghiului dintre dreapta A0 C 0 si dreapta de
intersectie a planelor (M N P ) si (BCC 0 ).
2. Fie a, b numere ntregi distincte cu proprietatea ca exista n numar real astfel
nct a3 a = b3 b = n. S
a se arate c
a n = 0.
3. Se dau sase puncte n plan, oricare trei necoliniare. Consideram zece segmente,
fiecare avnd capetele n cte doua dintre aceste puncte. Sa se arate ca exista cel
putin un triunghi avnd ca laturi trei dintre cele zece de segmente.
4. Fie SABC un tetraedru regulat. Punctele A1 , B1 , C1 apartin muchiilor (SA),
(SB), (SC), respectiv, astfel nct A1 B1 = B1 C1 = C1 A1 . S
a se arate c
a planele
(A1 B1 C1 ) si (ABC) sunt paralele.

Clasa a IX-a
1. Determinati numarul solutiilor ecuatiei

[x]
2007x
=
.
{x}
2008

Mihail B
alun
a
2
2. Rezolvati n multimea numerelor ntregi ecuatia x + x + 4 = y .
Ioan Cucurezeanu
6

134

3. Fie ABCDE un pentagon convex. Demonstrati c


a
aria(ABC)
aria(CDE)
+
< 1.
aria(ABCD) aria(BCDE)
Dan Ismailescu
4. Fie C1 , C2 doua cercuri concentrice distincte si [AB] un diametru al cercului
C1 . Consider
am dou
a puncte variabile M C1 , N C2 , nesituate pe dreapta AB.
a) Ar
atati c
a exist
a si sunt unic determinate punctele P , Q, situate pe dreptele
M A, respectiv M B, astfel nct N sa fie mijlocul segmentului [P Q].
b) Aratati ca suma AP 2 + BQ2 este constanta, unde P , Q sunt definite la a).
Mihai Piticari, Mihail B
alun
a

Clasa a X-a
1. Fie O centrul cercului circumscris triunghiului ABC si A1 punctul de pe cerc
diametral opus lui A. Not
am cu G, G1 centrele de greutate al triunghiurilor ABC si
PG
2
A1 BC si cu P intersectia dreptelor AG1 si OG. Sa se arate ca
= .
PO
3
Gabriel Popa, Paul Georgescu

2. Sa se arate ca nu exista numere ntregi a, b, c astfel nct (a+bi 3)17 = c+i 3.


Dorin Andrica, Mihai Piticari
3. Sa se determine poligoanele convexe, inscriptibile, cu proprietatea ca orice
triunghi determinat de trei dintre vrfurile acestora este isoscel.
Gheorghe Iurea

4. Fie r un numar real cu proprietatea ca 2n r 14 , 2n r + 14 Z 6= , pentru


orice n N. Sa se arate ca r este numar ntreg.
Ciprian Baghiu

Clasa a XI-a
1. Fie A M4 (R) astfel nct det(A2 I4 ) < 0. Sa se arate ca exista R, cu
|| < 1, astfel nct matricea A + I4 sa fie singulara.
Mihai Haivas
a nct B = S 1 AS. S
a se
2. Fie A, B, S M3 (C), S fiind o matrice nesingular
2

2
arate ca tr(B ) + 2 tr(B ) = (tr(A)) .
Mihai Haivas
3. Fie a > 1 un num
ar real. Pentru fiecare num
ar natural nenul n, k(n) este cel
k(n)
k
a se calculeze lim
.
mai mic num
ar natural k pentru care (n + 1) ank . S
n n
Neculai Hrtan

4. Fie f : R R o functie continu


a pe Q, cu proprietatea c
a f (x) < f x + n1
pentru orice x R si n N . Sa se demonstreze ca f este strict crescatoare pe R.
Gabriel Mrsanu, Mihai Piticari

Clasa a XII-a
1. Se consider
a sirul (an )nN , an =

Rn
1

dx
(1+x2 )n .

135

Calculati lim n 2n an .
n
Bogdan Enescu

2. Determinati numerele n N, n 3 si a R pentru care polinomul X n +aX 1


are un divizor de foma X 2 + X + cu , Z.
B
alun
Mihail
a
R1

3. Determinati functiile crescatoare f : [0, 1] R pentru care f (x) enx dx


0

2008, pentru orice n N.

Mihai Piticari
4. Fie A un inel finit n care numarul elementelor inversabile este egal cu numarul
elementelor nilpotente. S
a se arate c
a num
arul elementelor inelului este o putere a
lui 2. (Un element x A se numeste nilpotent dac
a exist
a k natural cu xk = 0.)
Dinu S
erb
anescu

Concursul de matematic
a Florica T. Cmpan
Etapa judetean
a, 16-17 februarie 2008
Clasa a IV-a
1. a) G
asiti regula de formare a sirului 3, 8, 13, . . . si scrieti termenul de pe locul
31.
b) Dupa un concurs de matematica, un elev nu si-a amintit rezultatul unei probleme. Totusi, si-a adus aminte c
a num
arul are sase cifre, ncepe cu 1 si dac
a prima
cifr
a se mut
a la sfrsit, atunci num
arul obtinut va fi de trei ori mai mare dect cel
initial. Care a fost rezultatul problemei?
2. Mergnd cu masina, un sofer observa la ora 09:10 ca pe kilometrajul de la
bord apare num
arul 12921. La ora 11:00, pe kilometraj apare urm
atorul num
ar care
coincide cu r
asturnatul s
au. La ce or
a va observa soferul din nou un astfel de num
ar,
presupunnd ca se deplaseaza cu viteza constanta?
Gabriel Mrsanu, Recreatii Matematice 1/2001
3. Pe o foaie este scris num
arul A = 1234xy. Cinci elevi joac
a urm
atorul joc:
fiecare dintre primii patru citeste numarul, si fixeaza cte o regula de transformare
a lui si scrie pe tabla numarul transformat. Al cincilea, care cunoaste doar primele
patru cifre ale lui A, trebuie s
a ghiceasc
a regula fiec
aruia dintre colegi si s
a ncerce
s
a afle num
arul. S
tiind c
a primii patru au scris pe tabl
a numerele 123500, 123470,
123460, 120000, se cere:
a) Care sunt regulile de transformare observate de al cincilea elev?
b) Poate al cincilea elev s
a afle cu exactitate num
arul? Care sunt valorile posibile
ale num
arului A?
Petru Asaftei

Clasa a V-a
1. La un concurs se acord
a cinci puncte pentru premiul I, trei puncte pentru al
doilea si doua puncte pentru al treilea. Aflati numarul de premii primite de elevii
unei scoli, stiind ca au obtinut n total 25 de puncte si cel putin cte doua premii din
fiecare categorie.
2. Un num
ar natural se numeste simpatic dac
a este format din cifre distincte
nenule, a caror suma se divide cu 10.
136

a) Determinati cel mai mic si cel mai mare num


ar simpatic.
b) Precizati cte numere de trei cifre sunt simpatice si divizibile cu 4.
3. Dac
a a N si b N , not
am ab = ab +ba (de exemplu, 32 = 32 +23 = 17).
a) Determinati numarul n N astfel nct 1 1 + 2 1 + 3 1 + + n 1 = 54.
b) Comparati numerele 3 18 si 2 27.
c) Aflati ultima cifr
a a num
arului 2 (2 2008).
Adrian Zanoschi

Clasa a VI-a
1. Un parinte si mparte averea astfel: primul copil primeste 10 000 lei plus
o cincime din rest; al doilea copil primeste 20 000 lei plus o cincime din noul rest;
al treilea copil primeste 30 000 lei plus o cincime din noul rest si asa mai departe.
S
a se afle suma mp
artit
a de p
arinte, precum si num
arul copiilor, stiind c
a toti au
mosteniri egale.
Mihai Grtan, Recreatii Matematice 1/2002
2. C
at
alin este faiantar si trebuie s
a paveze podeaua unei nc
aperi n form
a de
dreptunghi avnd lungimea de 3 metri si latimea de 2 metri, folosind dale patratice
cu latura de 50 centimetri. El are la dispozitie 6 dale rosii, 6 albastre, 6 galbene si 6
verzi, iar cerinta este ca orice dou
a dale de aceeasi culoare s
a nu se ating
a.
a) Indicati un exemplu de pavare corect
a.
b) Catalin sparge o dala rosie si primeste n loc una verde. Poate acum proceda
n asa fel nct s
a respecte cerinta? Justificati r
aspunsul.
Doru Buzac
3. Un numar natural N se scrie n baza 10 folosind 6 cifre nenule si distincte.
Se stie c
a, oricum am schimba ordinea cifrelor num
arului N , num
arul N precum si
numerele obtinute sunt toate multipli de p, unde p este un num
ar prim.
a) Determinati cte numere se pot obtine din N prin schimbarea ordinii cifrelor.
b) Daca p = 3, dati exemplu de un numar N care sa verifice conditiile din enunt.
c) Dac
a p 6= 3, demonstrati c
a nu exist
a numere N care s
a verifice conditiile din
enunt.
Radu Sava

Clasa a VII-a
1. Se considera sirurile definite prin: a1 = 91204; a2 = 9012004; a3 = 900120004;
. . . ; b1 = 91504; b2 = 9015004; b3 = 900150004; . . .
a) Aflati num
arul de cifre al sumei an + bn , unde n N ;

b) Aratati ca an + bn
/ Q, n N ;

c) Demonstrati ca an Q, n N , nsa bn
/ Q, n N .
Constantin Chiril
a, Recreatii Matematice 1/2001
2. Doua blocuri de locuinte care au naltimea de 21 m fiecare sunt situate pe
un teren plat. La ora 10, umbra primului bloc proiectat
a pe cel de-al doilea are
n
altimea de 15 m, iar umbra celui de-al doilea bloc pe p
amnt are lungimea de 42
m. Ce naltime are umbra primului bloc pe cel de-al doilea la ora 11, daca umbra
celui de-al doilea bloc pe pamnt este de 31,50 m?
3. Pardoseala unei b
ai de dimensiuni L = 45 dm, l = 35 dm este acoperit
a cu pl
aci
de gresie n forma de patrat cu latura de 1 dm. Daca se ndeparteaza cte o placuta
137

din cele patru colturi, se poate acoperi suprafata r


amas
a cu pl
aci dreptunghiulare de
dimensiuni 1 dm 2 dm?

Clasa a VIII-a
1. a) Fie suma

1
1
1
+p
+ + p
.
S=p

2
2
1+ 1 1
3+ 3 1
2007 + 20072 1

Aflati cel mai mic numar natural nenul n pentru care numarul S n este natural.
b) Dac
a a este lungimea ipotenuzei si b, c lungimile catetelor unui triunghi dreptunghic, demonstrati c
a 2a > b + c + ha , unde ha este lungimea n
altimii corespunzatoare ipotenuzei.
Claudiu S
tefan Popa
2. Cte plane pot fi duse la egala distanta de patru puncte necoplanare date?
Justificati r
aspunsul.
3. n cetatea N N a numerelor naturale se organizeaz
a o mare petrecere n cinstea
num
arului 0. La poarta castelului bate unul din locuitorii cet
atii.
Sunt numarul 83. mi permiteti sa intru la petrecere? ntreaba acesta.
La petrecere sunt invitate doar numerele fantastice, i r
aspunse o voce de partea
cealalt
a.
Dar ce nseamn
a num
ar fantastic? ntreab
a num
arul 83.
Sa va explic, spune vocea stranie. Daca n este un numar natural mai mare
dect 1 si not
am An = {x N | (x, n) 6= 1}, num
arul n se numeste fantastic dac
a
pentru orice dou
a numere x, y apartinnd multimii An , suma lor x + y este tot un
element al multimii An . Ati priceput?
Am nteles, raspunde lamurit vizitatorul.
a) Stabiliti voi dac
a num
arul 83 este invitat la petrecere. Aceeasi cerinta si pentru
num
arul 2008.
b) G
asiti toate numerele pare invitate la petrecere.
Alexandru Negrescu

Etapa interjudetean
a, 22-23 martie 2008
Clasa a IV-a
1. George este mai mic dect Andrei cu o p
atrime din vrsta lui Andrei. Peste
un an, Andrei va fi mai mare dect George cu o p
atrime din vrsta lui George. Ce
vrsta au acum Andrei si George?

2. Se considera mpartirea:

2 1
a) Dati un exemplu de astfel de mpartire.
=
=

b) Cte mp
artiri de acest tip se pot efectua? Justifi
cati r
aspunsul!
=
=
3. Un elev de clasa a IV-a are n total 100 de fructe,

nuci si mere. El schimba cu un prieten cte noua nuci


=
pentru doua mere, terminnd toate nucile dupa un numar
de schimburi si r
amnnd n final cu 44 de mere.
a) Cte nuci a avut initial elevul?
b) Cte schimburi s-au facut si cte mere a primit de la prietenul sau?
138

Clasa a V-a
1. Un numar se numeste fiul unui alt numar daca este format cu doua dintre
cifrele numarului initial, numit tata. Dintre numerele de trei cifre cu ultima cifra 0,
aflati toti tatii cu 891 mai mari dect unul dintre fiii lor.
2. Se considera urmatorul tablou cu 200 de linii.
a) Ce num
ar se afl
a n mijlocul ultimei linii a
tabloului?
b) Cte numere contine tabloul?
c) De cte ori apare num
arul 100 n acest tablou?

2
2 4 2
2 4 6 4 2
2 4 6 8 6 4 2
...........................

3. La concursul "Florica T. Cmpan", etapa locala, au luat parte toti elevii de


clasa a V-a dintr-o scoal
a. Elevii din clasa a V-a D au obtinut urm
atoarele rezultate:
prima problem
a au rezolvat-o 9 elevi, a doua problem
a au rezolvat-o 7 elevi, a treia
problema au rezolvat-o 5 elevi, a patra problema au rezolvat-o 3 elevi, iar a cincea
problema a rezolvat-o un singur elev. Toti elevii clasei, n afara de Petrica, au rezolvat
acelasi num
ar de probleme, n timp ce Petric
a a rezolvat cu una mai mult dect colegii
s
ai. Poate s
a fie el premiant al concursului, dac
a premiantii concursului au fost elevii
care au rezolvat 4 sau 5 probleme?

Clasa a VI-a
1. Se considera multimea A =
multimii A N.

2008 2009 2010


,
,
, . . . . Determinati cardinalul
7
8
9

2. Fie dreapta AB, O un punct ntre A si B si, de aceeasi parte a dreptei,


\1 ) = a,
semidreptele [OA1 , [OA2 ,. . . , [OAn , n aceasta ordine, astfel nct m(AOA
\
\
m(A1 OA2 ) = a + 2,. . . , m(An OB) = a + 2n, unde a N, n N, n 2. Determinati
num
arul unghiurilor si m
asura fiec
aruia dintre ele.
3. a) Ar
atati c
a, oricum am alege cinci numere naturale, exist
a printre ele trei
cu suma divizibila cu 3.
b) Aratati ca, oricum am alege 25 de numere naturale, exista printre ele noua cu
suma divizibil
a cu 9.

Clasa a VII-a

1. a) Se considera numerele 1, 3 2, 3 + 2 si 5. Dupa un pas, fiecare numar


se nlocuieste cu media aritmetic
a acelorlaltor
trei. Este posibil ca, dup
a un num
ar
de pasi, s
a obtinem numerele 5 2 2, 3, 3+ 2
2 si 2?

b) Sa se arate ca daca numerele a, b si a + b sunt rationale, atunci a si b


sunt rationale.
2. Fie ABC, AB < AC si D (AC). Fie AE bisectoarea unghiului \
BAC,
E (BD), F mijlocul lui [AD], {O} = AE BF , {G} = DO AB. S
a se arate c
a
GD k BC AB = CD.
Daniela Tamas, Recreatii Matematice 2/2006
3. Ionel si Gigel au trasat cu creta pe parchet, n dou
a colturi diferite ale unei
camere, cte un segment de dreapta cu capetele la marginea peretilor, obtinnd
139

dou
a triunghiuri dreptunghice. Dac
a cele dou
a triunghiuri au aceeasi arie si acelasi
perimetru, sa se arate ca ele sunt congruente.

Clasa a VIII-a

x+y
, stiind ca 0 < x < y si (x y)(3x 2y) = 2xy.
xy
2. Doi pusti au un cornet transparent care, ca si lichidul omogen pentru nghetat
a
continut n el, au forme de piramide patrulatere regulate de vrf V si baze X1 X2 X3 X4 ,
respectiv A1 A2 A3 A4 , Ai (V Xi ), i = 1, 4. Deoarece, din motive care nu intereseaz
a aici, nu-l pot ngheta n pozitie vertical
a, ei nclin
a cornetul, f
ar
a a-l v
arsa,
astfel nct noua suprafata a lichidului este A01 A02 A03 A04 , A0i (V Xi ), i = 1, 4 si
(A1 A2 A3 ) (A01 A02 A03 ) = M1 M2 , unde M1 (A2 A3 ), M2 (A1 A4 ), iar M1 M2 k
A1 A2 .
a) Ar
atati c
a patrulaterul A01 A02 A03 A04 este trapez isoscel.
b) Cum vor controla cei doi pusti nclinarea cornetului, nainte de a-l pune la
nghetat, pentru ca, dupa aceea, planul (V M1 M2 ) sa mparta nghetata n cantitati
egale, stiind ca pot sa masoare doar ariile suprafetelor A1 A2 A02 A01 si A3 A4 A04 A03 ?
Claudiu S
tefan Popa
1. Sa se afle valoarea fractiei

3. Se dau zece numere naturale nenule care au suma egala cu 100. Demonstrati
c
a putem alege trei dintre numerele date care s
a poat
a fi lungimile laturilor unui
triunghi.
Adrian Zanoschi, Recreatii Matematice 1/2002

Concursul "Student pentru o zi"


1

prezentare de C
at
alin TIG
AERU
ncepnd din acest an, Universitatea "Stefan cel Mare" din Suceava organizeaz
a,
n lunile martie si aprilie, suita de concursuri care poart
a denumirea generic
a Student
pentru o zi. Concursurile se adreseaza elevilor de clasa a XII-a si se desfasoara la
acele discipline al c
aror studiu este aprofundat n facult
atile Universit
atii. Proba de
matematic
a a avut loc n ziua de 9 martie si a constat n rezolvarea a patru probleme
n timp de trei ore, programa fiind anuntat
a n prealabil. Au participat 28 de elevi
de la cteva colegii renumite din judetele Neamt, Botosani si Suceava. Premiile I, II
si III au fost n valoare de 500, 300, respectiv 200 de lei. Cstig
atorii acestei prime
editii sunt urm
atorii:
Premiul I Cepoi Alexandru, C.N. "Stefan cel Mare", Suceava
Premiul II Bozianu Rodica, C.N. "A. T. Laurian", Botosani
Premiul III Br
aescu Lucian, C.N. "Mihai Eminescu", Botosani
Problemele date la concurs au fost urm
atoarele:
ln x
Problema 1. Se consider
a functia f : (0, ) R, f (x) = .
x
1

Lector dr., Universitatea "Stefan cel Mare", Suceava

140


a) Ar
atati c
a functia F : (0, ) R, F (x) = 2 x (ln x 2), este o primitiv
a
pentru functia f .
b) Demonstrati ca orice primitiva G a functiei f este crescatoare pe [1, ).
c) Aflati aria suprafetei cuprinse ntre graficul lui f , axa Ox si dreptele de ecuatii
1
x = si x = e.
e
Subiect propus pentru Bacalaureat-2008
Problema 2. Fie (A, +, ) un
a 3 (3x = 0, x A ) si a A
inel de caracteristic
a
un element oarecare. Fie M = x A | x3 = ax = xa si b M cu proprietatea c
b comuta cu orice element din A.
a) Demonstrati ca b x M , oricare ar fi x M .
b) S
a se arate c
a, dac
a a A are n plus proprietatea c
a exist
a n N pentru
2n+1
n
care a = 1, atunci x + (b x)
M , oricare ar fi x M .
Ion Bursuc, Suceava
Problema 3. Dac
a f : [0, 1] R este functia continu
a, atunci
Z 1
Z 1
x (f (x) + f (1 x)) dx =
f (x) dx.
0

Dumitru Cr
aciun, F
alticeni
Problema 4. Se consider
a numerele prime distincte p si q, astfel nct pq = 2r,
cu r 3 prim.
a) Aratati ca p q | pq q p daca si numai daca p q | q pq 1.
b) Demonstrati ca p q | pq q p daca si numai daca q 2 1 (mod (p q)).
aeru, Suceava
C
at
alin Tig
Prima problem
a fiind una cunoscut
a, prezent
am doar solutiile celorlaltor trei.
Problema 2. a) Observam ca ( x)3 = 3 32 x + 3x2 x3 = 3 x3 =
3
3 ax. Cum M , atunci 3 = a = a, de unde deducem c
a ( x) =
a ax = a xa = a ( x) = ( x) a.
b) Demonstram prin inductie ca x2m+1 = am x, pentru orice x M si m N .
Pentru m = 1, relatia este adev
arat
a. Presupunem c
a, dac
a x M , atunci x2k1 =
k1
2k+1
2k1 2
k1
2
k1 3
a
x; atunci x
= x
x = a
xx = a
x = ak1 ax = ak x, ceea ce
ncheie demonstatia afirmatiei initiale.
Deoarece x M , atunci pentru orice numar m N , rezulta ca ( x)2m+1 =
2n+1
2n+1
a ( x)
= x, de unde x+( x)
=
am ( x). Lund m = n, obtinem c
M , ceea ce trebuia demonstrat.
Problema 3. F
ac
and o schimbare de variabil
a, se arat
a c
a

Rb

f (x) dx =

Rb
a

f (a + b x) dx, f : [a, b] R continu


a. Aplicnd de dou
a ori acest rezul-

tat, obtinem c
a

R1
0

R1
0

f (x) dx =

R1
0

f (1 x) dx, respectiv

R1
0

x (f (x) + f (1 x)) dx =

R1
R1
(1 x) (f (x) + f (1 x)) dx, de unde 2 x (f (x) + f (1 x)) dx = f (1 x) dx +
0

141

R1
0

R1
R1
f (x) dx. Deducem ca 2 x (f (x) + f (1 x)) dx = 2 f (x) dx, adica tocmai con0

cluzia problemei.

Problema 4. a) Putem scrie c


a pq q p = (p q) pq1 + qpq2 + + q q1
q q (q pq 1); deoarece (p q, q q ) = 1, deducem ca p q | pq q p daca si numai daca
p q | q pq 1.
b) Rezultatul de la punctul a) se reformuleaz
a astfel: p q | pq q p dac
a si numai
pq
dac
a q
1 (mod (p q)). Dar (p q, q) = 1, deci q (pq) 1 (mod (p q))
(teorema lui Euler); deoarece (p q) = (2r) = r 1, deducem ca q r1 1
(mod (p q)) si q 2r 1 (mod (p q)). Dar (r 1, 2r) = 2, deci p q | pq q p dac
a
si numai dac
a q 2 1 (mod (p q)).
Particulariznd, se pot obtine o serie de probleme interesante; iata cteva exemple:
34 | 67101 10167 , 34 | 137103 103137 , 22 | 881859 859881 , 6 | 20112017 20172011 .

1. La data de 15 ianuarie 1883 a ap


arut primul num
ar al revistei Recreatii Sti
intifice (18831888). n anul acesta, 2008, se mplinesc 125 de ani de la aparitia
acesteia.
Scrieti 2008 folosind num
arul 125 si (numai) operatiile de adunare si mp
artire!
Care este num
arul maxim de operatii cu care puteti face aceast
a scriere? Dar cel
minim? (Nu se accepta termeni nuli!)
2. Se dau trei cifre si un rezultat. Indicati operatiile necesare pentru a restabili
egalitatea!
1
2
3
4
5
6
7
8
9

1
2
3
4
5
6
7
8
9

1=8
2=8
3=8
4=8
5=8
6=8
7=8
8=8
9 = 8.

Not
a. Raspunsurile pot fi gasite la pag. 160.

142

Solutiile problemelor propuse n nr. 2 / 2007


Clasele primare
P.134. De la apartamentul meu cobor 7 etaje, apoi urc 4 etaje si observ ca sunt
la etajul 9. La ce etaj locuiesc?
(Clasa I )
Dragos Iacob, elev, Iasi
Solutie. 9 + (7 4) = 9 + 3 = 12. Eu locuiesc la etajul 12.
P.135. n trei vase sunt 36 nuci. Daca din primul vas se iau 3 nuci si din al
treilea o nuca si se pun n al doilea vas, atunci n fiecare vas va fi acelasi numar de
nuci. Cte nuci au fost la nceput n fiecare vas?
(Clasa I )
nv. Rica Buc
atariu, Iasi
Solutie. Din 36 = 12 + 12 + 12 obtinem: n primul vas erau 12 + 3 = 15 nuci; n
al doilea vas 12 3 1 = 8 nuci, iar n al treilea vas 12 + 1 = 13 nuci.
P.136. Aflati vrsta tatalui meu, stiind ca este un numar cuprins ntre 35 si 41,
dublul lui ntre 73 si 77, iar triplul lui este cuprins ntre 112 si 118.
(Clasa a II-a)
Iurie Juc, elev, Iasi
Solutie. Numerele cuprinse ntre 35 si 41 sunt 36, 37, 38, 39 si 40. Numarul 36
nu verific
a a doua conditie, iar num
arul 37 nu verific
a a treia conditie. Num
arul 38
verific
a toate conditiile, iar dac
a vrsta este mai mare ca 38, atunci dublul vrstei
depaseste pe 77. Tatal are 38 ani.
P.137. Dorin, Oana si Claudia se pregatesc pentru Concursul "Florica T. Cmpan". Oana a rezolvat 5 probleme. Dorin a rezolvat un numar de probleme n plus
fata de Oana, egal cu numarul de probleme rezolvate n plus de Oana fata de Claudia.
Cte probleme au rezolvat mpreuna cei trei copii?
(Clasa a II-a)
Inst. Maria Racu, Iasi
Solutie. Cei trei copii au rezolvat mpreun
a 5 + (5 + a) + (5 a) = 15 probleme.
P.138. Doi tati si trei fii au mpuscat fiecare cte un iepure. Cnd i-au numarat,
au vazut ca au doar patru iepuri. De ce?
(Clasa a III-a)
Inst. Elena Nita
, Iasi
Solutie. Echipa de vnatori este formata din doi feciori, tatal lor si bunicul celor
doi feciori.
P.139. Mutati un singur chibrit pentru a obtine o egalitate:

(Clasa a III-a)
Nicolae Iv
aschescu, Craiova
Solutie. Mut
am chibritul orizontal de la semnul de operatie plus si obtinem:

P.140. Descopera regula de completare a jetoanelor


10 11 12 13 14
98
...
0
1
2
72
143

99
81

si calculeaza cte numere diferite sunt scrise pe aceste jetoane pe locurile de jos.
(Clasa a III-a)
Lenuta Zaharia, elev
a, Iasi
Solutie. Regula este data de: 1 0 = 0, 1 1 = 1,. . . , 9 9 = 81. Numerele
diferite scrise pe locurile de jos sunt generate de produsele:
1 0, 1 1, 1 2, . . . , 1 9
2 5, 2 6, . . . , 2 9
3 5, 3 7, 3 8, 3 9
4 5, 4 7, 4 8, 4 9
5 5, 5 6, . . . , 5 9
6 7, 6 8, 6 9
7 7, 7 8, 7 9
8 8, 8 9
99

10 numere
5 numere
4 numere
4 numere
5 numere
3 numere
3 numere
2 numere
1 num
ar.

n total sunt scrise 37 numere diferite.


P.141. Fiul observa ca, atunci cnd i mai trebuia un an pna la jumatatea
vrstei din prezent, tatal avea vrsta de 12 ori mai mare dect a sa, iar cnd va avea
11 ani, vrsta lui va fi de 4 ori mai mica dect a tatalui. Sa se afle vrsta fiului n
prezent.
(Clasa a IV-a)
Petru Asaftei, Iasi
Solutie. Diferenta dintre vrsta tatalui si vrsta fiului este 4 11 11 = 33 ani.
Figurarea marimilor din problema:
(*) vrsta fiului n prezent
(**) vrsta fiului cnd mai avea un an pna la
jumatatea vrstei din prezent.
Valoarea unui segment este 33 : 11 = 3 ani. Vrsta fiului n prezent este (3 + 1)
2 = 8 ani.
P.142. Paginile unei carti sunt numerotate de la 1 la 336. Din aceasta carte se
rup, la ntmplare, 111 foi. Sa se arate ca:
a) suma numerelor de pe foile ramase nu se mparte exact la 10;
b) produsul numerelor de pe foile ramase se mparte exact la 3.
(Clasa a IV-a)
Maria Frangoi, elev
a, Iasi
Solutie. a) n total cartea are 336 : 2 = 168 foi. Num
arul foilor r
amase este
168 111 = 57. Suma numerelor de pe fiecare foaie este un num
ar impar. Suma
unui numar impar de numere impare este un numar impar, deci nu se poate mparti
exact la 10.
b) n sirul 1, 2, 3,. . . , 336 avem 112 numere care se mpart exact la 3. Acestea
sunt: 1 3, 2 3, 3 3,. . . , 112 3. Pe paginile celor 111 foi putem avea cel mult
111 numere care se mpart exact la 3. Pe paginile ramase vom ntlni cel putin un
numar care se mparte exact la 3, deci si produsul numerelor se mparte exact la 3.
144

P.143. Asezati numerele 2, 3, 4, . . . , 10 n patratul alaturat astfel nct, pe fiecare linie, suma numerelor din primele doua casete sa fie
egala cu numarul din ultima caseta. n cte moduri pot fi asezate aceste
numere?
(Clasa a IV-a)
Ionela B
ar
agan, elev
a, Iasi
Solutie. Suma numerelor de pe primele doua coloane este egala cu
2 6 8
suma numerelor de pe ultima coloana. Suma tuturor numerelor este
4 5 9
54, deci suma numerelor de pe ultima coloan
a este 27. Singura situatie
3 7 10
care satisface conditia de pe ultima coloan
a este 8 + 9 + 10 = 27. Un
exemplu de asezare este prezentat alaturat. Numerele 8, 9 si 10 pot fi asezate n
3 2 1 = 6 moduri pe ultima coloana. Deoarece

2+6=6+2=8
3+5=5+3=8
4+5=5+4=9
si
2+7=7+2=9 ,

3 + 7 = 7 + 3 = 10
4 + 6 = 6 + 4 = 10
nseamn
a c
a avem (8 2) 6 = 96 moduri de asezare a celor 9 numere.

Clasa a V-a
V.81. Demonstrati ca putem completa cu numere naturale ntr-o infinitate de
moduri casutele libere din figura de mai jos, astfel nct sa se poata efectua corect
operatiile indicate:

Amalia Cantemir, elev


a, Iasi
Solutie.

Not
a. Explicatia este aceea ca "schema" se poate descompune n cteva cicluri
"nule", n sensul ca suma numerelor care sunt scrise pe traseu si care se aduna este
egal
a cu suma numerelor care se scad:
I

II

III

V.82. ntr-o ferma sunt gaini, oi si vaci, n total 324 de picioare si un numar
impar de capete:
145

a) Sa se arate ca n ferma nu pot fi 101 gaini.


b) Sa se arate ca numarul oilor nu poate fi egal cu numarul vacilor.
Petru Asaftei, Iasi
Solutie. a) Not
am cu g, o, v num
arul g
ainilor, oilor, respectiv vacilor. Se stie c
a
2g + 4o + 4v = 324, de unde g + 2 (o + v) = 162, deci g trebuie s
a fie num
ar par.
b) Cum g + o + v este numar impar, iar g este par, rezulta ca o + v este numar
impar, prin urmare o si v au n mod necesar paritati diferite.
V.83. Sa se demonstreze ca 13 | abc daca si numai daca 13 | 3 ab c.
Otilia Nemes, Ocna Mures
Solutie. Avem:

13 | abc 13 | 10 ab + c 13 | 13 ab 10 ab + c 13 | 3 ab c.

V.84. Determinati cel mai mic si cel mai mare numar natural de 90 de cifre,
divizibile cu 90 si avnd suma cifrelor 90.
Carmen Daniela Tamas, Brlad
Solutie. Un num
ar cu suma cifrelor 90 este oricum divizibil cu 9; pentru a fi
divizibil si cu 10, el trebuie sa se termine n 0. Cel mai mare numar cu proprietatea
dorita va fi 99
. . . 9} 00
. . . 0}, iar cel mai mic 1 00
. . . 0} 8 99
. . . 9} 0.
| {z
| {z
| {z
| {z
10

80

78

V.85. Fie a, b N; sa se arate ca daca ultima cifra a numarului a2 + b2 este 9,


2
atunci ultima cifra a lui (a + b) este tot 9. Reciproca este adevarata?
Ioan S
ac
aleanu, Hrl
au
Solu
t
ie.
Ultima
cifr
a
a
unui
p
a
trat
perfect
poate
fi
0,
1,
4,
5,
6
sau
9.
Dac
a




U a2 + b2 = 9, obligatoriu U a2 = 0, U b2 = 9 (sau invers) sau U a2 = 4,

U b2 = 5 (sau invers). n primul caz , vom avea U (a) = 0, U (b) {3, 7} (sau
2

invers), deci U (a + b) {3, 7} si atunci U (a + b) = 9. n al doilea caz, vom avea


U (a)
8}, U (b) = 5 (sau invers), deci U (a + b) {3, 7} si din nou obtinem c
a
{2,
2
U (a + b) = 9.
2

Reciproca este fals


a; de exemplu, pentru a = 2, b = 1 avem c
a (a + b) = 9, ns
a
a + b2 = 5.
2

V.86. a) Sa se rezolve n numere naturale ecuatia x2 + y 2 = 625.


b) Sa se arate ca ecuatia x2 + y 2 = 2007 nu are solutii n N2 .
Valerica Benta, Iasi
Solutie. a) Sc
adem din 625, pe rnd, fiecare p
atrat perfect care nu-l dep
aseste;
rezultatul este tot patrat perfect n cazurile 625 49 = 576, 625 225 = 400, 625
400 = 225 si 625576 = 49. Obtinem solutiile (x, y) {(7, 24) ; (15, 20) ; (20, 15) ; (24, 7)}.
b) Un p
atrat perfect d
a la mp
artirea prin 4 fie restul 0, fie restul 1, deci x2 +
2
y poate fi M4 , M4 + 1 sau M4 + 2. Cum 2007 = M4 + 3, ecuatia dat
a nu are solutii
n N2 .
V.87. Sa se arate ca 751 > 389 .
Nela Ciceu, Bac
au
146

Solutia 1 (a autoarei). Avem:


17
= 34317 > 34217 = (9 38)17 > (9 36)17 = 368 234 =
751 = 73
4
4
= 368 22 28 > 368 3 2564 > 369 2434 = 369 35 = 389 .

Solutia 2 (M. Haivas). Inegalitatea se scrie echivalent


4 51
89
51
89
7 4 > 37 7 (2401) 4 > (2187) 7 ,
51
89
ceea ce este evident adev
arat deoarece 2401 > 2187, iar
>
.
4
7

Clasa a VI-a
VI.81. Stiind
ca 13 | 2a + 3b + 4c + 5d, aratati ca 13 | 43a + 45b + 47c + 49d si

13 | 46a + 30b 64c 54d ( a, b, c, d N).


Norbert-Traian Ionita
, elev, Iasi
Solutie. Avem:
13 | 2 (2a + 3b + 4c + 5d) + 13 (3a + 3b + 3c + 3d) 13 | 43a + 45b + 47c + 49d;
13 | 13 (4a + 3b 4c 3d) 3 (2a + 3b + 4c + 5d) 13 | 46a + 30b 64c 54d.
VI.82. Fie A = 3m 5n , m, n N. Notam cu a, b, c numarul divizorilor
numerelor A, 3A, respectiv 5A. Stiind
ca a si b sunt direct proportionale cu 3 si 4,

iar b si c sunt invers proportionale cu 15 si 16, sa se determine A.


Mihai Haivas, Iasi
Solutie. Avem ca a = (m + 1) (n + 1), b = (m + 2) (n + 1), iar c = (m + 1) (n + 2).
a
b
Din = obtinem c
a m = 2, apoi din 15b = 16c deducem c
a n = 3, prin urmare
3
4
2
3
A = 3 5 = 1125.
VI.83. Daca p este numar prim, iar n N , sa se arate ca p4n 3 nu este
patrat perfect.
Mirela Marin, Iasi
Not
a. Domnul Titu Zvonaru atrage atentia asupra faptului c
a ipoteza c
a p este
numar prim nu este importanta, fiind suficient sa consideram p N \ {0, 1}; acest
lucru a fost observat n redactie n momentul selectarii problemei spre publicare, dar
s-a preferat p
astrarea enuntului dat de autor. Prezent
am mai jos solutiile d-lui Titu
Zvonaru.
Solutia 1. Urm
arim ultima cifr
a a num
arului p4n 3:
U (p)
0 1 2 3 4 5 6 7 8 9

U p4
0 1 6 1 6 5 6 1 6 1
U p4n
0 1 6 1 6 5 6 1 6 1
4n
U p 3
7 8 3 8 3 2 3 8 3 8
Deducem astfel ca p4n 3 nu poate fi patrat perfect.
2
2

ar,
Solutia 2. Are loc dubla inegalitate p2n 1 < p4n 3 < p2n . ntr-adev
inegalitatea din dreapta este evident
a, iar cea din stnga se scrie succesiv:
p4n 2 p2n + 1 < p4n 3 2 p2n > 4 p2n > 2,

fapt adev
arat pentru p 2. Astfel p4n 3 este strict cuprins ntre dou
a p
atrate
perfecte consecutive, prin urmare nu poate fi patrat perfect.
147

VI.84. Pentru n N , definim An = 1 + 11 + 111 + + |11 {z


. . . 1}. Aratati ca:
n

a) 3 | An daca si numai daca 3 - n 1;


9n
10n
A2n
< 10n + 1 +
b) 10n + 1 +
<
, n 3.
10
An
11

Temistocle Brsan, Iasi


Solutie. a) Restul mpartirii unui numar prin 3 este acelasi cu restul mpartirii
prin 3 a sumei cifrelor sale. Rezult
a c
a oricum am considera trei termeni consecutivi
arat n aceast
a ordine) si
din An , ei vor fi de forma M3 , M3 + 1 si M3 + 2 (nu neap
atunci suma lor se va divide cu 3.
.
Dac
a n = 3k, grupnd cte trei termenii lui A , deducem c
an=
a A .. 3. Dac
n

3k + 1, atunci An = M3 + |11 {z
. . . 1} = M3 + 1. Dac
a n = 3k + 2, atunci An =
3k+1

M3 + |11 {z
. . . 1} + |11 {z
. . . 1} = M3 + (M3 + 1) + (M3 + 2) = M3 si de aici rezulta concluzia
3k+1

3k+2

de la a).
b) Observam ca

. . . 1} + + |11 {z
. . . 1} = An + 10n + |11 {z
. . . 1} +
A2n =An + |11 {z
n+1

2n

n
. . . 1} + + 11
10
+
11
.
.
.
1
.
.
.
1
+ 11 10n + |11 {z
| {z } =
| {z }
n

=An (10 + 1) + n 11
. . . 1} .
| {z
n

A2n
11 . . . 1
10
9
11 . . . 1
si r
= 1 + 10n + n
<
amne s
a ar
at
am c
a
<
,
An
An
10
An
11
n 3. Scrise dezvoltat, aceste inegalit
ati revin la

Rezult
a c
a

9 + 99 + 999 + + |9 .{z
. . 9} < 10 + 100 + 1000 + + 1 |0 .{z
. . 0};
n

11 + 110 + 1100 + + 11 |0 .{z


. . 0} < 10 + 110 + 1110 + + |11 {z
. . . 1} 0.
n

n1

si sunt adevarate pentru n 3.

VI.85. Pe latura Ox a unghiului drept xOy consideram un punct A, iar pe


bisectoarea unghiului consideram un punct B. Perpendiculara n B pe AB taie
dreapta Oy n C. Sa se arate ca AB = BC.
Petru Asaftei, Iasi
Solutie. Fie E si F proiectiile lui B pe dreptele Ox,
\ CBF
\ (au acelasi
respectiv Oy. Observam ca ABE
complement), BE = BF (deoarece OB este bisectoarea
\) si, conform C.U., rezulta ca 4ABE 4CBF ,
lui EOF
de unde AB = BC.
VI.86. a) Fie 4ADC si M (AC). Sa se arate ca PADM < PADC .
b D BC. Daca PABD = PACD ,
b) Fie 4ABC si (AD) bisectoarea unghiului A,
148

sa se arate ca 4ABC este isoscel.

Gheorghe Iurea, Iasi

Solutie. a) Folosind inegalitatea triunghiului n 4CDM ,

PADM = AD + AM + DM < AD + AM + (DC + M C) =


= AD + DC + (AM + M C) = PADC .

b) Sa presupunem prin absurd ca AB 6= AC; sa zicem ca


AB < AC si fie M (AC) astfel nct AM = AB. Avem ca
4ABD 4AM D (L.U.L), deci PABD = PADM < PACD si
astfel am ajuns la o contradictie.
VI.87. n figura alaturata sunt desenate 6 puncte, care unite doua
cte doua dau nastere la 9 drepte. Avem voie sa stergem unul dintre
puncte si sa-l desenam oriunde n alta parte.
a) Efectuati operatia descrisa astfel nct, prin unirea cte doua a
noilor puncte, sa se obtina 11 drepte.
b) Care este numarul minim si cel maxim de drepte care se pot obtine ntr-o
configuratie permisa?
Gabriel Popa, Iasi
Solutie. a) De exemplu, putem proceda ca n figura 1.

Fig. 1

Fig. 2

Fig. 3

b) Num
arul minim de drepte este 8, deoarece nu vom putea face n asa fel nct
sa obtinem mai mult de 4 puncte coliniare; a se vedea figura 2.
Num
arul maxim de drepte este 13: num
arul maxim posibil de drepte prin 6 puncte
65
este
= 15 si el nu poate fi atins dect dac
a oricare trei puncte sunt necoliniare.
2
La nlocuirea a trei puncte necoliniare cu trei puncte coliniare, num
arul de drepte
scade cu 2 si atunci maximul posibil este 13, atins de exemplu n configuratia din
figura 3.

Clasa a VII-a
VII.81. Se considera abc si xyzt numere naturale
r qscrise n baza 10. Sa se
q p

p
compare numerele naturale A = 6 2 abc si B = 3 2 xyzt.
Bogdan Chiriac, student, Iasi
q p
p

.
Solutie. Fie 6 p2 abc = k N; atunci 6 2 abc = k 2 , iar k2 .. 6, prin urmare

k = 6l. Rezult
a c
a 2 abc = 6l2 , deci abc = 18l4 , de unde abc = 324l8 . Din
100 324l8 999 obtinem c
a l = 1, adic
a A = 6. Cu un rationament asem
an
ator,
gasim ca B = 6 si astfel A = B.
VII.82. Fie a, b numere reale strict pozitive. Sa se arate ca:
149

a) daca a3 b3 = a + b, atunci a2 + b2 > 1;


b) daca a3 + b3 = a b, atunci a2 + b2 < 1.

Ionel Nechifor, Iasi


Solutie. Mai nti, s
a observ
am c
a nu putem avea a = b, altfel din ambele ipoteze
ar rezulta ca a = b = 0. Avem:
a3 b3
a3 + b3
a)
=1
> 1 a2 ab + b2 > 1 a2 + b2 > 1;
a+b
a+b
a3 + b3
a3 b3
b)
=1
< 1 a2 + ab + b2 < 1 a2 + b2 < 1.
ab
ab
VII.83. Determinati numerele ntregi a, b, c, d pentru care ac + bd = 1, iar
ad + bc = 2.
Gheorghe Iurea, Iasi
Solutie. Sc
aznd membru cu membru relatiile din ipotez
a, deducem c
a ad + bc
ac bd = 1, deci (a b) (d c) = 1, de unde a b = d c = 1 sau a b = d c = 1.
n primul caz, substituind a = b + 1 si d = c + 1 n prima dintre relatiile initiale,
avem succesiv:
ac + bd = 1 c (b + 1) + b (c + 1) = 1 2bc + b + c = 1
4bc + 2b + 2c + 1 = 3 (2b + 1) (2c + 1) = 3
(b, c) {(1, 0) ; (0, 1) ; (1, 2) ; (2, 1)} .
Obtinem solutiile (a, b, c, d) {(1, 2, 1, 0) ; (0, 1, 2, 1) ; (1, 0, 1, 2) ; (2, 1, 0, 1)}.
Similar, n al doilea caz gasim solutiile
(a, b, c, d) {(2, 1, 0, 1) ; (1, 0, 1, 2) ; (0, 1, 2, 1) ; (1, 0, 1, 2)} .
VII.84. Fie patratul ABCD cu latura de lungime a, iar E, F , G puncte pe
a
a
a
laturile [BC], [CD], respectiv [AB] astfel nct CE = , CF = , iar BG = . Sa
4
3
2
se arate ca dreptele AE, BF si CG sunt concurente.
Claudiu S
tefan Popa, Iasi
Solutie. Fie {P } = AC BF ; din asemanarea 4CF P
CP
CF
1
CP AG
4ABP , deducem ca
=
= . Atunci

PA
AB
3
P A GB
1 1 3
BE
= = 1 si din reciproca teoremei lui Ceva urmeaza
EC
3 1 1
concluzia.
VII.85. Fie O intersectia diagonalelor patrulaterului ABCD. Daca AABD =
CD
AB
AABC = ACOD , sa se arate ca

= 1.
AB
CD
Doru Buzac, Iasi
Solutie. Notam S1 = AAOD , S2 = AAOB , S = ADOC .
Cum AABD = AABC , rezulta ca ABCD este trapez cu
AB k CD, prin urmare ABOC = S1 . Din ipotez
a vom
CD
si atunci, cum
am c =
avea c
a S = S1 + S2 . Not
AB
S
= c2 . Pe de alt
a parte,
4OAB 4OCD, deducem c
a
S2
150

OD dist (A, BD)


S1
S
S1
=
=
, prin urmare S1 = S S2 . Am
= c si astfel
S2
OB dist (A, BD)
S2
Sr
2

S
S
obtinut c
a S = S S2 + S2 , deci
= 1+
, adic
a c2 = 1 + c. Rezult
a c
a
S2
S2
1
c = 1 + (c este tocmai numarul de aur ), de unde concluzia problemei.
c
VII.86. Fie A un punct pe manta unei mese de biliard circulare cu raza de 1 m.
O bila pleaca din A si ajunge napoi n A lovind manta de cel putin trei ori; reflexia
bilei se face considernd ca aceasta loveste un perete plan tangent la cerc n punctul
de contact. Sa se arate ca exista o infinitate de traiectorii posibile si sa se determine
traiectoria de lungime minima.
Cristian Laz
ar, Iasi
Solutie. Considernd un poligon regulat cu n laturi, n 4, nscris n cercul
dat, vrfurile acestuia pot fi punctele de contact cu manta ale bilei ntr-o traiectorie
dorita; poligonul poate fi chiar unul stelat! Lungimea minima a traiectoriei
se atinge
n cazul triunghiului echilateral nscris n cerc, al carui perimetru va fi 3 3m.
VII.87. O tabla are forma unui dreptunghi 4 5, format din 20 de patratele
1 1. Avem la dispozitie doua jetoane, fiecare putnd acoperi cte un patratel. n
cte moduri putem aseza jetoanele pe tabla, astfel nct ele sa nu se afle nici pe
aceeasi linie, nici pe aceeasi coloana? Generalizare.
Gabriel Popa, Iasi
Solutie. Numaram nti n cte moduri putem aseza jetoanele pe tabla, n absenta restrictiei din enunt. Dac
a jetoanele ar fi numerotate, ar exista 2019 modalit
ati
20 19
de asezare a lor; cum nu conteaz
a ordinea, avem
= 190 modalit
ati de asezare.
2
Dintre acestea, 10 contin cele dou
a jetoane pe prima linie, 10 pe a doua linie etc.,
deci 40 de asez
ari au jetoanele pe aceeasi linie. Apoi, exist
a 6 asez
ari cu jetoanele pe
prima coloana etc., deci 6 5 = 30 de asezari cu jetoanele pe aceeasi coloana.
n final, avem 190 40 30 = 120 asezari ce verifica enuntul.
Generalizare. n cazul unei table m n, avem
mn (mn 1) mn (n 1) mn (m 1)
mn

=
(m 1) (n 1)
2
2
2
2
modalitati de asezare.

Clasa a VIII-a
VIII.81. Consideram fixate numerele a, b Z , m, n N , m 6= n si fie functia
f : N Z, f (x) = ax+b. Daca f (1)+f (2)+ +f (m) = f (1)+f (2)+ +f (n),
sa se calculeze suma S = f (1) + f (2) + + f (m + n).
Nedeianu,
Dr.Tr.Severin

Dan
2
m2 + m a
n +n a
Solutie. Din relatia din ipoteza deducem ca
+mb =
+nb.
2
2
Trecem totul ntr-un membru si simplificam prin mn 6= 0; rezulta ca a (m + n + 1)+
151

2b = 0. Atunci
(m + n) (m + n + 1) a
+ (m + n) b =
2
(m + n) (2b)
=
+ (m + n) b = 0.
2
VIII.82. Sa se arate ca |3xy + x + y| 1, x, y [0, 1].
Ovidiu Pop, Satu Mare
Solutie. Avem de ar
atat c
a 1 3xy + x + y 1, x, y [0, 1]. Prima
inegalitate este echivalent
a cu 0 x (1 y) + y (1 x) + (1 xy), adev
arat
a pentru
x, y [0, 1] (toti cei trei termeni sunt pozitivi). A doua inegalitate este echivalenta
cu 0 (1 x) (1 y) + 2xy, din nou adevarata pentru x, y [0, 1].
VIII.83. Sa se arate ca nu exista x, y Z pentru care 147x2 = 1 + 4y 3y 2 .
h Mihai Cr
i aciun, Pascani
2
Solutia 1. Scriem relatia din enunt sub forma 3 (7x) + y 2 = 4y + 1. Orice
patrat perfect este sau M4 , sau M4 + 1; pentru x, y Z, paranteza patrata este M4 ,
M4 + 1 sau M4 + 2 si atunci membrul stng este M4 , M4 + 3 sau M4 + 2. Cum
membrul drept este M4 + 1, urmeaz
a concluzia.
Solutia 2 (Ioan Stanciu, elev, Craiova). Daca exista x, y cu proprietatile
cerute, n mod necesar 1 + 4y 3y 2 0. Deducem ca y {0, 1}, valori pentru care
x nu este ntreg.

VIII.84. Laturile a, b, c ale unui triunghi verifica egalitatea 2 a8 + b8 + c8 =


4
2
a + b4 + c4 . Sa se arate ca triunghiul este dreptunghic.
Corina Elena Visan, Craiova
Solutie. Avem succesiv:
f (1) + f (2) + + f (m + n) =

a8 + b8 + c8 2a4 b4 2b4 c4 2c4 a4 = 0

4
2
a + b4 c4 4a4 b4 = 0 a4 + b4 + 2a2 b2 c4 a4 + b4 2a2 b2 c4 = 0

2
2
a2 + b2 c4
a2 b2 c4 = 0

a2 + b2 + c2 a2 + b2 c2 a2 b2 + c2 a2 b2 c2 = 0

si de aici concluzia.
VIII.85. Daca a, b, c sunt numere reale pozitive, sa se arate ca

q
p
p

1
2
3
2
2
2
2
2
2
a + b ab + b + c bc 3 = a + c +
= .
a
c
b
Liviu Smarandache, Craiova
Solutie. Fie [OA], [OB], [OC] trei segmente de lungimi
\ = 60 , iar m(BOC)
\ =
a, b, respectiv c, astfel nct m(AOB)

30 . Avem:
q
p
p

a2 + b2 ab + b2 + c2 bc 3 = a2 + c2
p
p
a2 + b2 2ab cos 60 + b2 + c2 2bc cos 30 =
p
= a2 + c2 AB + BC = AC A, B, C coliniare
152

AAOB + ABOC = AAOC

ab sin 60
1
bc sin 30
ac
2
3

+
=
ab 3 + bc = 2ac +
= .
2
2
2
a
c
b
VIII.86. O piramida hexagonal
a regulata V ABCDEF are muchia bazei AB = 4

cm si naltimea V O = 4 2 cm. Fie M mijlocul lui V D, {P } = AD BF , iar


{Q} = P M (V CF ). Sa se arate ca:
a) dreptele V P si DQ sunt concurente; b) DQ (V BF ).
Gabriel Popa, Iasi
Solutie. a) Cum (V AD) (V CF ) = V O, P M (V AD),
iar {Q} = P M (V CF ), nseamna ca Q V O. Astfel
dreptele V P si DQ sunt ambele incluse n planul (V AD),
cu Q Int V P D si de aici urmeaz
a concurenta dorit
a.
b) Observ
am c
a BF AD si BF V O (V O (ABC)),
de unde BF (V AD), prin urmare BF DQ. Vom mai
arata ca DQ V P si atunci va rezulta
ca DQ (V BF ).

Avem c
a P O = 2 cm, deci V P = V O2 + OP 2 = 6 cm, iar
P D = P O + OD = 6 cm. Deducem c
a 4P DV este isoscel,
iar mediana bazei P M va fi si naltime. Astfel, Q va fi ortocentru n 4V P D, prin
urmare DQ V P .

VIII.87. Consideram prisma triunghiulara regulata ABCA0 B 0 C 0 si cubul


AM CN A0 M 0 C 0 N 0 , unde M este punct interior triunghiului ABC. Fie E, F , E 0 ,
F 0 mijloacele muchiilor [AB], [BC], [A0 B 0 ], respectiv [A0 C 0 ].
a) Aflati masura unghiului dintre dreptele EF 0 si E 0 F .
b) Aflati masura unghiului format de planele (M CC 0 ) si (ECC 0 ).
Claudiu
S
tefan Popa, Iasi

0
Solutie. a) Fie l = AB, h = AA
; cum AC = AM 2
0 0 0
si AM = AA0 = h, deducem
al=
c
h 2. n 4A B C echi3
6
l
h
=
, iar din 4CC 0 F dreplateral, avem c
a C 0E 0 =
2
2
h 6
tunghic obtinem ca C 0 F =
. Se observa usor ca EF C 0 F 0
2
este paralelogram, deci EF 0 k F C 0 , EF 0 = F C 0 . Rezulta ca
0 , E 0 F ) = m(F C
0 , E 0 F ) = m(C
0 F E 0 ). Din motive de
\
\
\
m(EF

h
6
si astfel 4F C 0 E 0 este echilateral, prin urmare
simetrie, E 0 F = EF 0 = F C 0 =
2
m
asura unghiului dorit este de 60 .
0 C 0 M 0 , a ca
0 C 0 M 0 ) m(A\
0C 0E 0) =
b) Unghiul planelor este E\
rui masura este m(A\

45 30 = 15 .

Clasa a IX-a
IX.81. Fie a, b R. Daca ecuatia x2 +ax+b+2 = 0 are ambele radacini ntregi,
aratati ca numarul 2a2 + b2 este natural compus.
Dorin M
arghidanu, Corabia
Solutie. Daca x1 , x2 Z sunt solutiile ecuatiei date, atunci a = (x1 + x2 ) si
153

b = x1 x2 2, de unde rezult
a c
a a, b Z, deci 2a2 + b2 N. n plus,

2
2
2a2 + b2 = 2 (x1 + x2 ) + (x1 x2 2) = x21 + 2 x22 + 2

si concluzia se impune.

Not
a. ntr-o forma putin modificata, problema a aparut n RMT 2/2007, cu
num
arul IX.216, sub semn
atura aceluiasi autor.
IX.82.
tiile f : R R pentru
care
Determinati func
f x4 + y 3 + z 2 + t = f (x) + f y 2 + f z 3 + f t4 ,

x, y, z, t R.

Lucian Tutescu si Liviu Smarandache, Craiova


Solu
t
ie.
Pentru
x
=
y
=
z
= t = 0, g
asim f (0) = 0. Dac
ay=
z = 0, atunci


16

asim c
a
f
x
(x),
f x4 + t = f (x) + f t4 , x, t R. Pentru t = x4 , g
= f
4
4
16
4 4
x R. Cum f x = f (x), x R, deducem c
=f x =
af x
=f x
f (x), x R. Prin urmare f (x) = f (x), x R, adic
a f (x) = 0, x R.
IX.83. Pentru a 9, sa se demonstreze ca are loc inegalitatea
q
q

3 + 3a + 9 1 + 1 + a.

Marian Tetiva, Brlad


Solutie. Pentru a = 9 avem egalitate; sa aratam ca are loc strict inegalitatea
din enunt pentru a > 9. Prin ridicare la patrat si cu notatia a = x2 , x > 3, obtinem
succesiv:
q
p

1 + 3a + 9 > 2 1 + a + a 1 + 3x2 + 9 > 2 1 + x + x


p

2x (x 3)
2 (x 3)
3x2 + 9 (x + 3) 2 1 + x 2

>
2
1+x+2
3x + 9 + x + 3
p

x
1

x 1 + x + 1 > 3x2 + 9 + 3.
>
1+x+2
3x2 + 9 + x + 3

Aceasta din
rezulta adunnd x > 3 si x 1 + x > 3x2 + 9 (care
urma inegalitate

revine la x3 + x2 > 3x2 + 9, evident pentru x > 3).


IX.84. Fie ABC un triunghi. Determinati numerele ntregi a, b, c nenule, prime
ntre ele doua cte doua, astfel nct punctele M , N , P sa fie coliniare, unde M , N ,

P sunt determinate prin conditiile AM = aAB; CN = bCA; CP = cBC.


Ioan S
ac
aleanu, Hrl
au

Solutie. Exprimnd vectorii N P si M N n functie de AC si BA, obtinem ca

N P = (b + c) AC + cBA si M N = (1 b) AC aBA. Deoarece punctele M , N , P



sunt coliniare, rezult
a c
a vectorii N P si M N sunt coliniari; folosind relatiile precedente, gasim conditia ab + ac + bc = c. De aici rezulta ca c | ab si cum c este prim
cu a si b, deducem c
a c {1, 1}. Pentru c = 1 g
asim solutiile a = 3, b = 2
sau a = 2, b = 3. Pentru c = 1 g
asim solutiile a = 1, b Z (n acest caz
M = P = B) sau a Z, b = 1 (n acest caz N = A si P = B).
IX.85. Fie ABC un triunghi ascutitunghic si D = prBC A, E = prCA B, F =
prAB C. Demonstrati echivalenta afirmatiilor urmatoare:
(i) 4ABC este isoscel;
154

(ii) DB + EC + F A = DC + EA + F B;
1
1
1
1
1
1
(iii)
+
+
=
+
+
.
DB EC
FA
DC
EA F B
Examinati cazurile n care 4ABC este obtuzunghic sau dreptunghic.
Temistocle Brsan, Iasi
Solutie. Implicatiile (i) (ii) si (i) (iii) sunt triviale. Pentru implicatiile
inverse acestora, utiliz
am relatiile BD = c cos B, DC = b cos C etc. si teorema
cosinusului. Astfel, avem:
X
X 1
X
X 1
2a
2a
=
=

(iii)
2
2
2
c cos B
b cos C
b c a
c2 a2 b2

care, dupa transformari, este echivalenta cu (a b) (b c) (c a) (a + b + c)2 = 0.


Prin urmare, (iii) este echivalent
a cu (i).
Analog se arat
a c
a (ii) este echivalent
a cu (a b) (b c) (c a) (a + b + c) = 0,
deci cu (i).
b este unghiul obtuz, avem echivan cazul n care triunghiul este obtuzunghic si A
lenta conditiilor:
(j) 4ABC este isoscel de vrf A,
(jj) DB + EC F A = DC EA + F B,
1
1
1
1
1
1
(jjj)
+

+
.
DB EC
FA
DC
EA F B
Daca 4ABC este dreptunghic n A, atunci conditia (iii) nu se mai poate formula,
iar (ii) devine DB + b = DC + c, care este echivalent
a cu faptul c
a 4ABC este
dreptunghic si isoscel de vrf A.

Clasa a X-a
X.81. Sa se rezolve n R R R sistemul
x y 2/3 = z 1/3 ;

x4/3 y = z 2/3 ;

z 5/3 y 4/3 = z.

Vasile Chiriac, Bac


au
Solutie. Notnd x1/3 = t, y 1/3 = u, z 1/3 = v, sistemul devine t3 u2 = v;

t4 u3 = v 2 ; t5 u4 = v 3 . Avem c
a t8 = u3 + v 2 = u2 + v u4 + v 3 , de unde
g
asim c
a uv (u v)2 = 0. Prin urmare, u = 0 sau v = 0 sau u = v. Analiznd aceste
cazuri, gasim solutiile (t, u, v) : (1, 0, 1); (1, 0, 1); (0, 0, 0); (1, 1, 0); (0, 1, 1);
1 + 5 1 + 5 1 + 5 1 5 1 5 1 5
si
,
,
,
,
. Corespunzator, obtinem
2
2
2
2
2
2
solutiile (x, y, z) ale sistemului dat.
X.82. Solve the equation

aex + b ex 3 = ax3 + b x3 2 + a, a > b > 0.


Zdravko Starc, Vrac, Serbia
Solutie. Scriem ecuatia sub forma

a ex 2 + b ex 3 = a x3 1 + b x3 2 .

x
Considernd
3
functia f : R R, f (t) = at + b |t 1|, ecuatia devine f (e 2) =
f x 1 . Se verific
a usor c
a f este strict cresc
atoare, deci injectiv
a. Ecuatiei dat
a
se reduce la ex 2 = x3 1, adica x3 ex + 1 = 0. Cum functia g : R R,

155

g (x) = x3 ex + 1 este strict cresc


atoare si g (0) = 0, concluzion
am c
a ecuatia
x3 ex + 1 = 0 are solutia unica x = 0. Prin urmare, ecuatia data are solutia unica
x = 0.
X.83. n exteriorul triunghiului ABC se construiesc triunghiurile isoscele BM A,
\
AN C si CP B de baze AB, AC si respectiv BC, astfel nct m(M
AB) = 15 ,

\
\
\
m(N AC) = 45 , iar m(P BC) = 30 . Sa se arate ca m(M P N ) = 60 .
aeru, Suceava
Angela Tig
Solutie. Vom nota afixul fiec
arui punct cu litera mic
a

ce i corespunde. Deoarece M A se obtine din M B n urma


5
unei rotatii n jurul lui M de unghi
, avem c
a am =
6

5
2a+b 3bi
5

. Analog
, de unde m =
(bm) cos +i sin
6
6
2+ 3i

2b + c c 3i

gasim afixele punctelor P si N , anume p =


,
3 3i

c ai
mp

si de aici
3 1 cos + i sin
respectiv n =
. Prin calcule,
=
1i
np
3
3
concluzia problemei.
X.84. Fie ABC un triunghi n care (tg B 1) (tg C 1) = 2. Daca M si N
sunt picioarele naltimilor din B, respectiv C, sa se arate ca segmentele BM , CN
si M N se pot constitui n laturi ale unui triunghi.
C
at
alin Calistru, Iasi
Solutie. Problema este nrudit
a cu VI.30, publicat
a de acelasi autor n RecMat
b = 45 ; atunci
1/2002. Ca si acolo, cheia rezolvarii este aceea de a arata ca m(A)
triunghiurile ABM si ACN vor fi dreptunghice isoscele, cu BM = AM si CN = AN ,
deci segmentele din enunt se constituie n laturi ale 4AM N .
Deoarece tg A + tg B + tg C = tg A tg B tg C (identitate cunoscut
a), relatia din
ipoteza este echivalenta cu 1+tg B+tg C = tg B tg C, i.e. (tg A 1) (tg B tg C 1) =
0. Se observ
a usor c
a al doilea factor nu se poate anula, deci r
amne c
a tg A = 1,
b = 45 .
adic
a m(A)
X.85. Se prelungeste diametrul [M N ] al unui cerc C cu segmentul [N P ] congruent cu [M N ]. Fie d perpendiculara n P pe M N si R d, oarecare. Tangentele
duse prin R la C intersecteaza tangenta n M la C n S si T . Sa se arate ca centrul
de greutate al 4RST este un punct fix.
Adrian Reisner, Paris
Solutie. Raport
am planul la un reper ortogonal cu originea n M , avnd dreapta M N ca ax
a Ox si tangenta n M
a
la C drept ax
a Oy. Fie
raza cercului C; atunci M (0, 0),
2
N (a, 0), P (2a, 0), iar R (2a, ), cu variabil si fie S (0, s).
Ecuatia dreptei RS este ( s) x 2ay + 2as = 0 si,
a
impunnd conditia c
a d (O, RS) = , obtinem:
2

a
q
( s) + 2as
a
2
q
= | + 3s| = ( s)2 + 4a2 2s2 + 2s a2 = 0.
2
( s)2 + 4a2
156

Dac
a T (0, t), se obtine pentru t ecuatia 2t2 + 2t a2 = 0. Din
relatiile lui Vite, de
2a + 0 + 0 + s + t
ducem c
a t + s = . Centrul de greutate al 4RST este G
,
,
3
3

2a
i.e. G
, 0 , deci este un punct fix.
3

Clasa a XI-a
{x}
XI.81. Daca m Z, sa se studieze existenta limitei lim
.
xm sin x
Dan Popescu, Suceava
[x]
[x]
Solutie. Observ
am c
a sin x = (1) sin (x [x]) = (1) sin {x}, x
m
(1)
1
{x}
{x}
=
= xm
lim
. Pentru calculul limitei
R. Atunci xm
lim
[x]
sin
x
sin

{x}

(1)
x>m
x>m
{x}
1
la stnga, dac
a m = 2k, k Z, obtinem c
a lim
=
= , iar dac
a
x2k sin x
0
x<2k

1
{x}
=
= +. n concluzie, limita dat
a nu
m = 2k + 1, k Z, atunci lim
x2k sin x
0+
x<2k
exista pentru nicio valoare a lui m Z.

XI.82. Considerul sirul (an )nN , definit prin a0 = 0, a1 = 1 si an+2 = 3an+1


3 5
, aratati ca sirul xn = a0 + a1 x + a2 x2 + +
an , n N. Pentru x 0,
2
an xn , n N, este convergent si calculati limita sa.
Vlad Emanuel, elev, Sibiu
sirul (an ), stabilim
Solu
t
ie.
Folosind
ecua
t
ia
caracteristic
a
pentru
c
a a =
"
"
n #
nn #
n
1
3 5
3+ 5
3+ 5

, n N. Rezult
a c
a an 0,
,
2
2
2
5
n N. Evident ca sirul (xn ) este strict crescator si cum

n

3+ 5
1
3+ 5
,
0 xn < 1 +
xn <
x + +
2
2
1 x 3+2 5
(xn ) este marginit. n concluzie, (xn ) este convergent. Fie l = lim xn ; deoarece
n

xn+2 3xxn+1 + x2 xn = a0 + (a1 3a0 ) x = x, prin trecere la limita rezulta ca


x
l= 2
.
x 3x + 1
XI.83. Fie f : [0, ) [0, ) pentru care are loc relatia f (f (x)) + 9x = f (6x),
x [0, ). Aratati ca f (x) 3x, x [0, ).
Bogdan Posa si Marius Dr
agoi, elevi, Motru
9
Solutie. Din ipotez
a deducem c
a f (6x) 9x, de unde f (x) x si f (f (x))
9 2 6
2
+9
9
x, x [0, ). Atunci, conform ipotezei, f (x) 6
x, x [0, ).
6
6
9
Prin inductie se arat
a c
a f (x) un x, x 0, n N , unde u1 = , iar un+1 =
6
157

u2n + 9
. Deoarece (un ) este convergent si lim un = 3, deducem ca f (x) x lim un ,
n
n
6
deci f (x) 3x, x [0, ).
XI.84. Determinati numerele a R pentru care exista o functie continua
f : R R astfel nct (f f ) (x) = a2 f (x) 2a4 x, x R.
Andrei Nedelcu, Iasi
Solutie. Pentru a = 0 exist
a f : R R, f (x) = 0 care verific
a enuntul.
Vom arata ca nu exista astfel de functii pentru a 6= 0. Presupunnd contrariul, din
ipoteza se obtine ca f este injectiva, prin urmare f va fi strict monotona. Rezulta ca
f f este strict cresc
atoare; din x < y deducem c
a (f f ) (x) < (f f ) (y), de unde
2a4 (y x) < a2 (f (y) f (x)) si de aici deducem c
a f este strict cresc
atoare. Ca urmare, si functia f f f este strict crescatoare. Cum (f f f ) (x) = a4 f (x)2a6 x,
x R, n membrul stng am avea o functie strict crescatoare, iar n cel drept o
functie strict descresc
atoare. n concluzie, r
amne c
a a = 0.
XI.85. Fie A = (aij )20072007 o matrice patratica n care aij {1, 0, 1},
i, j = 1, 2007. Sa se arate ca determinantul matricei 2008I2007 + A este nenul.
Paul Georgescu si Gabriel Popa, Iasi
Solutie. Fie B = 2008I2007 + A; atunci bii = 2008 + aii , iar bij = aij pentru
i 6= j. Observam ca
2007
X
|bij | , i = 1, 2007.
(1)
|bii | = |2008 + aii | 2007 > 2006
j=1,j6=i

Presupunem prin absurd ca det B = 0; atunci exista c1 , c2 , . . . , c2007 R, nu toti


nuli, astfel nct
c1 C1 + c2 C2 + + c2007 C2007 = O,
undeC1 , C2 , . . . , C2007 sunt coloanele lui B. Fie k 1, 2007 pentru care |ck | =
max |ci | ; i = 2007 ; atunci
c1
c2
ck1
ck+1
c2007
Ck = C1 C2
Ck1
Ck+1
C2007
ck
ck
ck
ck
ck
c1
c2
ck1
ck+1
c2007
bkk = bk1 bk2
bk,k1
bk,k+1
bk,2007 .
ck
ck
ck
ck
ck
De aici deducem ca

2007
2007
2007
X cj
X

X cj

|bkj |

bkj
|bkj | .
(2)
|bkk | =

j=1,j6=i ck j=1,j6=i ck
j=1,j6=i

Relatiile (1) si (2) intr


a n contradictie, prin urmare r
amne c
a det B 6= 0.

Clasa a XII-a
XII.81. Dintre toate parabolele y = ax2 + bx + c, sa se determine aceea care
trece prin punctele A (0, 1), B (1, 2), satisface conditia y 0 pentru 0 x 1
si realizeaza minimul ariei determinata de graficul parabolei, Ox si dreptele x = 0,
respectiv x = 1.
Adrian Corduneanu, Iasi
Solutie. Din conditiile y (0) = 1 si y (1) = 2, g
asim y = ax2 + (1 a) x + 1,
cu a R . Cum y (0) = 1 > 0, y (1) = 2 > 0, conditia y 0 pentru x [0, 1] se
158

1+x
> a, x (0, 1), deci a sup h (x), unde h : (0, 1) R,
x x2
x(0,1)

1+x
. G
asim cu usurinta a 3 + 2 2, prin urmare y = ax2 + (1 a) x + 1,
h (x) =
x x2

R1
9a
a n enunt este A = y (x) dx =
a , 3 + 2 2 \ {0}. Cum aria cerut
,
6
0

3 2
obtinem ca A este minima pentru a maxim, deci a = 3 + 2 2, iar Amin =
.
3
XII.82. Determinati primitivele functiei f : (1, ) R,

x3 5x4 + 3 (ln x 1)
.
f (x) =
3
(x4 1)
Dan Nedeianu, Dr. Tr. Severin
Solutie. Cu substitutia x4 1 = t, avem de calculat
Z
1
(5t + 8) (ln (t + 1) 4)
dt =
16
t3

Z
Z
1
5t + 8
5t + 8
=
dt
+
ln
(t
+
1)
dt
.
4
16
t3
t3
Calculnd a doua integrala prin parti si nlocuind t cu x4 1, gasim ca
Z

4
x8 + 3x4
1
x4

f (x) dx =
2
2 ln x + 16 ln x 1 + C.
4
4
(x 1)
4 (x 1)
scrie sub forma

XII.83. Sa se determine functiile continue f : R R pentru care


Z x
et f (x t) dt, x R.
f (x) = |x| +
0

Dumitru Mihalache, Brlad


Solutie. Cu schimbarea de variabil
a x t = u, aducem ecuatia functional
a
Rx x+u
Rx u
la forma f (x) = |x| + e
f (u) du. Notnd e f (u) du = F (x), deducem ca
0

ex f (x) = F 0 (x), x R si ecuatia devine ex F 0 (x) (ex F (x) = |x|, x R, sau


2
x2 , x < 0
0
, o primitiv
aa
nc
a (ex F (x)) = |x|, x R. Considernd G (x) =
2
x
2 ,x 0
functiei g (x) = |x|, x R, obtinem c
a(ex F (x) = G (x) + C, C constant
a, de unde
x2
2 ,x < 0
+ C. Cum f (0) = 0, obtinem
f (x) = F 0 (x) ex , deci f (x) = |x| +
x2
2 ,x 0
(
2
x x2 , x < 0
C = 0 si atunci f (x) =
, functie care verifica ecuatia data.
2
x + x2 , x 0

XII.84. Fie polinomul f Z [X], f = a0 X 2n+1 + a1 X 2n + + a2n X + a2n+1


pentru care n este impar, a0 a2n+1 este impar, iar a1 a2 este par. Sa se arate ca, daca
f are toate radacinile reale, cel putin una este irationala.
Mihai Haivas, Iasi
Solutie. Dac
a x1 , x2 , . . . , x2n+1 ar fi r
ad
acini rationale ale lui f , atunci yi =
a0 xi , i = 1, 2n + 1 vor fi radacini rationale ale polinomului g = Y 2n+1 + a1 Y 2n +
159

a0 a2 Y 2n1 + + a2n
a c
a yi Z si yi | a2n
0 a2n+1 . Rezult
0 a2n+1 , i = 1, 2n + 1.
Cum a0 a2n+1 este impar, atunci a2n
a
este
impar,
deci
yi , i = 1, 2n + 1 sunt
2n+1
0
2n+1
P
numere impare. Din prima relatie Vite,
yi = a1 , rezulta ca a1 este impar.
i=1
P
Apoi, din i<j yi yj = a0 a2 , obtinem ca a0 a2 este numar impar, fiind o suma de
2
C2n+1
= n (2n + 1) termeni impari, iar n fiind impar. Deducem ca a2 este impar si
astfel produsul a1 a2 va fi impar, contradictie. Rezult
a c
a f are cel putin o r
ad
acin
a
irational
a.
XII.85. Fie n N, n 2. Aratati ca exista P Z [X] de grad n, astfel nct
toate multimile Ak = {P (i) (mod k) | i Z}, k N, k 2, sa aiba cardinalul strict
mai mic dect k.
Vlad Emanuel, elev, Sibiu
Solutie (Gheorghe Iurea). Deoarece P (mk + i) = P (i) (mod k), m Z
si k N, k 2, i = 1, k 1, multimea Ak contine, cel mult, elementele P (0),
P (1), . . . , P (k 1) (mod k). Trebuie determinat un polinom P Z [X] astfel nct
printre elementele enumerate mai sus, cel putin doua sa fie egale (pentru orice k).
Polinomul P = X n X, verific
a cerintele date, ntruct P (0) = P (1), k Z, k 2.

R
aspunsuri (la recreatiile de la pag. 142)
1. Numarul maxim cerut este 4015:
2008 = |125 : 125 + 125 : 125
{z + + 125 : 125},
2008 termeni

pe cnd cel minim este 24:


2008 = 125 + + 125 + 125 + + 125 : 125.
|
{z
} |
{z
}
16 termeni

2.

8 termeni

(1 + 1 : 1)3 = 8
222=8

32 3 : 3 = 8


4+ 4 4=8

5 + 5!! : 5 = 8
p
(6 + 6)!! : 6! = 8
7+7:7=8
8+88=8
99:9=8

(Pentru orice n N , factorialul se defineste prin n! = 1 2 3 n, iar semifactorialul astfel: (2n)!! = 2 4 6 (2n) si (2n 1)!! = 1 3 5 (2n 1).)
160

Solutiile problemelor pentru preg


atirea concursurilor
din nr. 2/2007
A. Nivel gimnazial
G126. Sa se determine numerele naturale care au proprietatea ca media geometrica a tuturor divizorilor lor este un numar natural.
Petru Minut, Iasi
n n
Solutie. Fie 1 = d1 < d2 < < dk = n divizorii lui n; atunci
,
,
d1 d2
n
...,
este sirul acelorasi divizori, scris descrescator, prin urmare d1 d2 dk =
dk
nk
2
, de unde (d1 d2 dk ) = nk . Media geometric
a a tuturor divizorilor lui
d1 d2 dk

n este k d1 d2 dk = n si este num


ar natural dac
a si numai dac
a n este p
atrat
perfect.
G127. Daca a, b, c, x, y, z, t sunt numere reale pozitive, sa se demonstreze
inegalitatea
1
1
1
1
8
p
.
+
+
+

2
2
2
ax+by+cz ay+bz+ct az+bt+cx at+bx+cy
3 a +b +c x2 +y 2 +z 2 +t2

D. M. B
atinetu-Giurgiu, Bucuresti
Solutie. Observam ca produsul (a + b + c) (x + y + z + t) este, dupa desfacerea
parantezelor, tocmai suma numitorilor din membrul stng ai inegalit
atii de demonstrat. Not
am acest membru stng cu S; folosind inegalitatea dintre media aritmetic
a
si cea armonica, obtinem
16
(a + b + c) (x + y + z + t) S 16 S
.
(a + b + c) (x + y + z + t)
r
r
a2 + b2 + c2 x + y + z + t
x2 + y 2 + z 2 + t2
a+b+c
si

(inegalitatea
nsa
3
3
4
4
dintre media aritmetica si cea patratica) si atunci concluzia problemei urmeaza. Egalitatea se atinge cnd a = b = c si x = y = z = t.
G128. Fie a, b, c numere reale pozitive astfel nct abc = 1 si fie t [1, 5]. Sa
se arate ca
b
c
3
a
+ 2
+ 2

.
2
a +t b +t c +t
t+1
Titu Zvonaru, Com
anesti si Bogdan Ionita
, Bucuresti
Solutie. Deoarece abc = 1, exist
a numerele reale pozitive x, y, z astfel nct
x
y
z
a = , b = , c = . Avem ca
y
z
x
x
xy
xy
x2
a
=
=

a2 + t
x2 + ty
2xy + (t 1) y 2
2x + (t 1) y
2x2 + (t 1) xy
b
c
x2
y2
z2
a
+ 2
+ 2
2
+ 2
+ 2

2
a +t b +t c +t
2x + (t1) xy 2y + (t1) yz 2z + (t1) zx

(x + y + z)2
.
2 (x2 + y 2 + z 2 ) + (t 1) (xy + yz + zx)
161

Prin urmare, este suficient s


a demonstr
am c
a
3
(x + y + z)2

.
2
2
2
2 (x + y + z ) + (t 1) (xy + yz + zx)
t+1
Dup
a efectuarea calculelor, acesta se reduce la

(5 t) x2 + y 2 + z 2 xy yz zx 0,

evident adev
arat
a.

Not
a. Pentru t = 2 se obtine o problem
a propus
a la concursul Baltic Way 2005.

1
1

= {xy} + , x R.
G129. Sa se determine y R pentru care {x} + x +
y
y
(Cu {} am notat partea fractionara.)
Alexandru
Negrescu,
elev,
Boto
sani
1
2
2
1
1
1
Solutie. Pentru x = obtinem c
a
+
= , deci
=
. Cum
y
y
y
y


y y
2
1
2
1
1
[0, 1), iar
Z, deducem c
a
=
= 0, prin urmare (0, 1) si
y
y
y
y
y
2
2
2
N. Astfel, (0, 2) N, deci = 1 si atunci y = 2.
y
y
y

1
1
= {2x} + , x
Pentru y = 2, egalitatea din enunt devine {x} + x +
2
2
R, iar aceasta
este adevarata ntruct revine la cunoscuta identitate a lui Hermite
1
[x] + x +
= [2x], x R.
2
G130. Fie a, b, c lungimile laturilor unui triunghi ABC. Daca a2007 + b2007 >
2007
b este ascutit.
2
+ 1 c2007 , sa se arate ca unghiul C
Lucian Tutescu, Craiova
Solutie. Vom arata ca c este cea mai mica latura a triunghiului, de unde concluzia
este imediat
a. S
a presupunem prin absurd c
a c b; atunci

2007
a2007 + c2007 a2007 + b2007 > 22007 + 1 c2007 a2007 > (2c)
a > 2c,

de unde a > 2c b + c, ceea ce contrazice inegalitatea triunghiului. Analog se


procedeaza daca am presupune ca c a.
G131. Fie n, k 2 numere naturale si multimea M = { (n 1) , . . . , 2, 1, 1,
2, . . . , n}. Sa se arate ca M se poate partitiona n k submultimi avnd fiecare aceeasi
suma a elementelor daca si numai daca n se divide cu k.
Marian Tetiva, Brlad
Solutie. Conditia este necesara: daca M admite o partitie ca n enunt, atunci
suma elementelor sale (care este n) va fi egal
a cu sk (s fiind suma elementelor din
fiecare clas
a a partitiei).
Pentru a demonstra suficienta, vom construi efectiv o partitie n cazul n care
162

n = ks, cu s N. Consider
am multimile:

M1 = {2, 3, . . . , s, s + 1, 1, 2, . . . , (s 1) , s} ;
M2 = {s + 2, . . . , 2s, 2s + 1, (s + 1) , . . . , (2s 1) , 2s} ;
..
..
.
.
Mk1 = {(k 2) s + 2, . . . , (k 1) s, (k 1) s + 1, ((k 2) s + 1) , . . . , (k 1) s} ;
Mk = {1, (k 1) s + 2, . . . , ks, ((k 1) s + 1) , . . . , (ks 1)}

si este evident ca M = M1 M2 Mk , Mi Mj = , i 6= j, iar suma elementelor


oric
arei multimi Mi este s.

G132. n fiecare cmp unitate al unei livezi m n se afla cte un mar. Un


numar de k arici pornesc, pe rnd, din cmpul stnga-sus al livezii si se misca spre
cmpul din dreapta-jos. La fiecare miscare, un arici se poate deplasa cu un cmp,
spre dreapta sau n jos, fara a iesi din livada. Ariciul poate sa culeaga marul din
cmpul pe care l viziteaza, daca nu a fost cules deja de alt arici. Care este numarul
minim k, pentru care k arici pot sa culeaga toate merele?
Iurie Boreico, elev, Chisin
au
Solutie. Numerot
am cmpurile (x, y), cu x {1, 2, . . . , m}; y {1, 2, . . . , n},
ncepnd din coltul stnga-sus. Fiecare miscare a unui arici duce la marirea coordonatei x sau y a cmpului pe care se afla cu 1, adica suma coordonatelor creste
cu 1 la fiecare miscare a unui arici. n particular, un arici poate s
a viziteze cel
mult un cmp de pe diagonala x + y = k. Cea mai lung
a diagonal
a are lungimea
min (m, n) (diagonalele cu aceasta lungime sunt x + y = m + 1, x + y = m + 2, . . . ,
x + y = n + 1 daca, de exemplu, asumam ca m n), prin urmare avem nevoie de cel
putin min (m, n) arici care s
a culeag
a toate merele de pe aceast
a diagonal
a; rezult
a
k min (m, n). Pe de alt
a parte, un num
ar de min (m, n) arici sunt suficienti: n
cazul m n, putem considera ca primul arici merge spre dreapta pna la marginea
livezii, apoi coboara pna la destinatie; al doilea merge o unitate n jos, apoi spre
dreapta pn
a la marginea livezii, dup
a care coboar
a; al treilea merge dou
a unit
ati n
jos s.a.m.d.
Raspunsul este deci k = min (m, n).
\ =
G133. Fie 4ABC echilateral si D un punct astfel nct BD = DC, m(BDC)

\ = 15 , sa se arate ca
30 , iar BC separa A si D. Daca E (BD) cu m(BAE)
CE AC.
Enache P
atrascu, Focsani
Not
a. A se vedea nota O problema si. . . noua solutii din acest num
ar al revistei,
pag. 128.

G134. Se considera patrulaterul convex ABCD nscris ntr-un cerc de raza 6


b = 45 . Sa se arate ca aria patrulaterului este cel
b = 60 si m(B)
cm, avnd m(A)

2
mult egala cu 3 6 cm .
Constantin Apostol, Rm. S
arat
Solutie (Gabriel Popa). Cu
teorema sinusurilor n 4ABC si n 4ABD,

obtinem c
a AC = 2R sin 45 = 2 3 cm, respectiv BD = 2R sin 60 = 3 2 cm.
1
\
Dac
a = m(AC,
BD), aria patrulaterului este S = AC BD sin si este maxim
a
2

163

cnd = 90 . n cazul nostru, cum AC 2 + BD2 > (2R) (relatia revine la 30


> 24),
deducem ca exista un patrulater cu diagonalele perpendiculare si de lungimi 2 3 cm

1
si 3 2 cm, prin urmare maximul ariei se atinge si este Smax = 2 3 3 2 = 3 6
2
cm2 .
G135. Fie tetraedrul ABCD cu AB = CD, AC = BD, AD = BC. Sa se arate
ca cel putin doua dintre unghiurile diedre formate de fata (ABC) cu fetele (BCD),
(ACD), (ABD) sunt ascutite.
Dan Brnzei, Iasi
Solutie (R
azvan Ceuc
a, elev, Iasi). Este evident ca cele trei diedre nu pot fi toate neascutite; sa
presupunem prin absurd c
a dou
a dintre ele sunt neascutite (anume cele de muchii BC si AC, ale c
aror m
asuri vor fi , respectiv , cu , 90 ), iar al treilea,
anume cel de muchie AB, are masura < 90 . Se observ
a usor c
a proiectia O a lui D pe planul (ABC)
apartine interiorului sau laturilor unghiului opus la
\ Deducem ca AABC = AABO AACO ABCO , prin urmare
vrf lui ACB.
AABC = AABD cos AACD cos (180 ) ABCD cos (180 ). Tetraedrul este
echifacial, deci AABC = AABD = AACD = ABCD si, dup
a simplificare, obtinem c
a
1 + cos (180 ) + cos (180 ) = cos . Cum cele trei cosinusuri sunt numere din
intervalul [0, 1), ajungem la o contradictie.

B. Nivel liceal
L126. Fie ABC un triunghi ascutitunghic. Mediatoarea laturii AB intersecteaza
latura AC n T , iar mediatoarea laturii AC intersecteaza latura AB n S. Sa se
arate ca paralela prin T la AB, paralela prin S la AC si simediana din A sunt
concurente.
Titu Zvonaru, Com
anesti
Solutia 1 (n maniera autorului). Not
am cu b, c lungimile laturilor AC,
\ = m(CAQ),
[ unde Q este intersectia paralelei
respectiv AB si fie = m(BAQ),
prin T la AB cu paralela prin S la AC. Not
am, de asemenea, {D} = AQ BC.
Folosind teorema sinusurilor n 4ABD si n 4ACD, gasim ca
c sin
BA
=
.
(1)
DC
b sin
Cum patrulaterul ASQT este paralelogram, folosind triunghiul
ASQ, deducem c
a
c
sin
c
SQ
AT
A
= .
=
=
= 2 cos
(2)
b
sin
AS
AS
b
2 cos A
BD
c2
Din (1) si (2) obtinem ca
= 2 , de unde AD este simediana
DC
b
din A n triunghiul ABC.
Solutia 2 (Vlad Emanuel, student, Bucuresti). Este cunoscut faptul c
a simediana dusa dintr-un vrf este locul geometric al mijloacelor antiparalelelor la latura
164

opus
a (vezi, de exemplu, L. Niculescu si V. Bosko - Probleme practice de geometrie,
Ed. Tehnica, 1990). Cum noi dorim sa aratam ca diagonala AQ a paralelogramului
ASQT este simediana, ar fi destul sa demonstram ca ST este antiparalela la BC,
AT
AS
AB
deci ca
=
. Acest lucru este nsa evident, deoarece AT =
, iar
AB
AC
2 cos A
AC
si astfel rezolvarea este ncheiata.
AS =
2 cos A
L127. Fie A1 A2 A3 A4 A5 A6 un hexagon inscriptibil. Sa se arate ca
rA1 A2 A3 + rA4 A5 A6 + rA1 A3 A6 + rA3 A4 A6 = rA3 A4 A5 + rA1 A2 A6 + rA2 A3 A6 + rA3 A5 A6 ,
unde rXY Z este raza cercului nscris n 4XY Z.
C
at
alin Calistru, Iasi
Solutie. Dac
a R este raza cercului circumscris hexagonului, este cunoscut
a relatia
X
Y
Z
rXY Z = 4R sin sin sin , unde XY Z este un triunghi avnd vrfurile comune
2
2
2
cu hexagonul. Vom demonstra nti urmatoarea
Lem
a. Daca A1 A2 A3 A4 este un patrulater inscriptibil, atunci
rA1 A2 A3 + rA1 A3 A4 = rA1 A2 A4 + rA2 A3 A4 .
ntr-adevar, cu notatiile din figura, vom avea:

+
rA1 A2 A3 + rA1 A3 A4 = 4R sin sin sin
+
2
2
2

+
+ sin sin sin
;
2
2
2

+
rA1 A2 A4 + rA2 A3 A4 = 4R sin sin sin
+
2
2
2

+
+ sin sin sin
2
2
2

+
si dezvoltnd sin
= sin cos + sin cos etc., obtinem concluzia lemei.
2
2
2
2
2
Aplicnd lema patrulaterelor inscriptibile A1 A2 A3 A6 si A3 A4 A5 A6 si sumnd
membru cu membru egalit
atile obtinute, g
asim tocmai relatia de demonstrat.
Not
a. Aceeasi solutie a fost data de Vlad Emanuel, student, Bucuresti.
L128. Sa se arate ca ntre medianele unui triunghi are loc inegalitatea
X
hY
i
ih X
Y
X
m2a m2b
(ma + mb ) 2
m2a m2b
m4a .
8
ma
Dorel B
aitan si I.V.Maftei, Bucuresti
A
a
2
2
Solutie. Avem, folosind cunoscutele x + y + z 2 xy + xz + yz, sin
2
b+c
A
B
C
si 4R sin sin sin = r, ca
2
2
2
A
B
C
2 2
2 2
2 2
a b + b c + c a abc (a + b + c) (b + c) sin (c + a) sin (a + b) sin 2p =
2
2
2
A
B
C
= sin sin sin 2p (a + b) (b + c) (c + a) =
2
2
2
r 2S
S
=
(a + b) (b + c) (c + a) =
(a + b) (b + c) (c + a)
(1)
4R r
2R
165

Apoi, s
a observ
am c
a are loc identitatea

a4 +b4 +c4 +(a + b + c) (a + b + c) (a b + c) (a + b c) = 2 a2 b2 + b2 c2 + c2 a2 ,


de unde obtinem ca

a4 + b4 + c4 + 16S 2 = 2 a2 b2 + b2 c2 + c2 a2

(2)

Pentru triunghiul de laturi a, b, c, exist


a un triunghi dual, avnd laturile ma , mb ,
P 2 2 Se Q
mc . Relatia (1) aplicat
(ma + mb )
a n triunghiul dual ne d
a c
a2
ma mb
e
R
si putem scrie, folosind (2), ca
P 2 2 P 4
2
ma mb ma
Se
16Se2
Se
= ma mb mc =
=
.
e
4ma mb mc
4ma mb mc
R
e
4S
Combinnd aceste relatii se obtine concluzia problemei.

Not
a. Principial aceeasi solutie a dat Marius Olteanu, inginer, Rm. Vlcea.

L129. n planul raportat la un reper cartezian xOy consideram vectorii legati


n O: v1 (a1 , b1 ), v2 (a2 , b2 ), v3 (a3 , b3 ). Sa se arate ca exista un tetraedru OABC

regulat, de muchie 1 si astfel nct OA, OB, OC se proiecteaza pe planul xOy n
v1 , v2 , respectiv v3 daca si numai daca se verifica simultan relatiile:

3 2
3 2
a1 + a22 + a23 a1 a2 a1 a3 a2 a3 =
b1 + b22 + b23 b1 b2 b1 b3 b2 b3 = 1;
2
2
3
(a1 b1 + a2 b2 + a3 b3 ) (a1 b2 + a2 b1 + a1 b3 + a3 b1 + a2 b3 + a3 b2 ) = 0.
2
Irina Mustata
a, Bremen
, student
Solutie. Completam reperul din plan la unul n spatiu Oxyz si fie A (a1 , b1 , c1 ),
B (a2 , b2 , c2 ), C (a3 , b3 , c3 ) astfel nct OABC este tetraedru regulat de muchie 1.
Din OA = OB = OC = 1 deducem c
a a21 + b21 + c21 = a22 + b22 + c22 = a23 + b23 + c23 =
\ = m(BOC)
\ = m(COA)
[ = 60 rezulta, via produs scalar, ca
1, iar din m(AOB)
1
a1 a2 + b1 b2 + c1 c2 = a1 a3 + b1 b3 + c1 c3 = a2 a3 + b2 b3 + c2 c3 = . Aceste egalit
ati
2
pot fi scrise sub forma matriceala astfel:

a1 a2 a3
1 12 21
a1 b1 c1
a2 b2 c2 b1 b2 b2 = 1 1 1 .
(1)
2
2
1
1
a3 b2 c3
c1 c2 c3
1
2
2

1 12 12
12 12
a1 b1 c1
2
3
12 ,
Fie X = a2 b2 c2 , iar A = 21 1 12 ; vom avea A1 = 12
2
1
1
3
1
1
a3 b2 c3
1
2 2
2
2
2
deci

X X T A1 = I3
(2)
Evident, de aici avem ca X T A1 X = I3 si, dupa efectuarea calculelor, se vor obtine
exact cele trei conditii din enuntul problemei.
S
a ar
at
am acum c
a aceste conditii sunt suficiente, adic
a s
a demonstr
am c
a putem
gasi c1 , c2 , c3 care sa dea restul conditiilor din egalitatea (2). Ecuatiile n care apar
166

c1 , c2 , c3 sunt:

3a1 a2 a3
a1
a1 a2
3a2
a3
3a3
c1

+ c2 +

+ c3
+
= 0;
2
2
2
2
2
2
2
2
2

3b1
b1 3b2
b1 b2
b2 b3
b3
3b3
c1

+ c2 +

+ c3
+
= 0;
2
2
2
2
2
2
2
2
2

3 2
c1 + c22 + c23 c1 c2 c1 c3 c2 c3 = 1.
2
Primele doua sunt ecuatii omogene cu 3 necunoscute, despre care se stie ca au o
a
variabil
a liber
a, deci o solutie netrivial
a (ce1 , ce2 , ce3 ). Cum a treia ecuatie este simetric
si omogen
a n c1 , c2 , c3 , putem nmulti ce1 , ce2 , ce3 cu un factor k astfel nct egalitatea
sa fie ndeplinita si rezolvarea problemei este ncheiata.
L130. Sa se arate ca pentru orice x, y 1 are loc inegalitatea

2
(xy x y) + 6 3 10 xy 6 3 9.
Gabriel Dospinescu, Paris si Marian Tetiva, Brlad
Solutie. Lu
am x = a + 1, y = b + 1, cu a, b 0; inegalitatea de demonstrat
devine

a2 b2 + 6 3 10 (a + b + ab) 2ab 0.

Cum a + b 2 ab si 6 3 10 > 0, ar fi suficient s


a ar
at
am c
a

a2 b2 + 6 3 10 2 ab + ab 2ab 0.

a
Cu notatia t = ab 0, am avea de justificat c

t4 + 6 3 10 t2 + 2 6 3 10 t 0 f (t) 0,

unde f : [0, ) R,
f (t) = t3 + 6 3 10 t + 2 6 3 10 . Derivata acestei

2
functiei este f 0 (t) = 3 t2
3 1 , care are ca singur
a r
ad
acin
a pozitiv
a pe

3 1. E usor de v
azut
a acesta
c
este punct de minim pentru f pe intervalul [0, ),
prin urmare f (t) f
3 1 = 0, t 0, ceea ce ncheie demonstratia.

Nota autorilor. De fapt, avem ca f (t) = t 3 + 1


t + 2 3 2 , ceea ce
conduce la concluzia dorit
a f (t) 0, t 0, ns
a aceast
a descompunere este mai
greu de v
azut.
Not
a. Solutii asemanatoare celei prezentate s-au primit de la Vlad Emanuel,
student, Bucuresti, precum si de la dl. Marius Olteanu, inginer, Rm. Vlcea.
L131. Sa se afle valoarea minima a numarului real k astfel nct, oricare ar fi
a, b, c reale pozitive cu a + b + c = ab + bc + ca, sa aiba loc inegalitatea

1
1
1
(a + b + c)
+
+
k k.
a+b b+c c+a
Andrei Ciupan, elev, Bucuresti
Solutii. nparticular,
inegalitatea din enunt trebuie sa aiba loc pentru a = b =
3
9
9
c = 1; astfel, 3
k k k . Vom ar
ata c
a este valoarea minim
a c
autat
a
2
8
8
167

a lui k; pentru aceasta, ar trebui s


a ar
at
am c
a

1
1
1
9
(a + b + c)
+
+
(a + b + c + 1) ,
a+b b+c c+a
8
oricare ar fi a, b, c R+ cu a + b + c = ab + bc + ca. Observam ca

(a + b + c)2 + (a + b + c)
(a + b + c)2 + ab + bc + ca
=
=
a+b+c
a+b+c
(a + b) (b + c) + (b + c) (c + a) + (c + a) (a + b)
=
;
a+b+c
9 (b + c) (c + a)
1

, deoarece
astfel, ar fi suficient s
a demonstr
am c
a (a + b + c)
a+b
8 (a + b + c)
scriind nca doua inegalitati similare si sumndu-le, obtinem chiar ceea ce dorim.
Aceasta ultima inegalitate se scrie succesiv:
2
9 (a + b) (b + c) (c + a) 8 (a + b + c) 9 (a + b) (b + c) (c + a)
8 (a + b + c) (ab + bc + ca) 9 (a + b) (b + c) (c + a)
8 (a + b) (b + c) (c + a) + 8abc (a + b) (b + c) (c + a) 8abc,
fapt care rezulta din inegalitatea mediilor.
Not
a. Solutie corect
a a dat dl. Marius Olteanu, inginer, Rm. Vlcea.
L132. Fie a, b, c, x, y, z R si A = ax+by+cz, B = ay+bz+cx, C = az+bx+cy.

Daca |A B| 1, |B C| 1 si |C A| 1, aratati ca a2 + b2 + c2 x2 + y 2 + z 2 .
3
Adrian Zahariuc, elev, Bac
au
Solutie. Deoarece distanta pe axa real
a ntre oricare dou
a dintre numerele A,
B si C este cel putin 1, distanta dintre cel mai mare si cel mai mic dintre ele este
cel putin 2, deci cel putin unul dintre ele se afla la distanta de cel putin 1 fata de
origine. Putem presupune ca |A| = max {|A| , |B| , |C|} 1. Folosind identitatea lui
Lagrange si inegalitatea CBS, obtinem:

2
2
2
2
2
a + b2 + c2 x2 + y 2 + z 2 = (ax + by + cz) + (ay bx) + (bz cy) + (cx az)
a+b+c+1=

(ax + by + cz) +

(ay + bz + cx bx cy az)2
|B C|2
4
2
= |A| +
.
3
3
3

L133. Determinati functiile f : N N pentru care


2f (n + 3)f (n + 2) = f (n + 1) + f (n) + 1,

n N.
Gheorghe Iurea, Iasi
Solutie. Notam an = f (n), n N; atunci 2an+3 an+2 = an+1 + an + 1. Cum
2an+4 an+3 = an+2 +an+1 +1, deducem c
a 2an+3 (an+4 an+2 ) = an+2 an , n N.
Prin urmare, 2an+3 |an+4 an+2 | = |an+2 an |, n N. Deoarece 2an+3 an+2 =
an+1 + an + 1 1, rezulta ca an 6= 0, n 2. Daca exista n0 N cu an0 +2 6= an0 ,
folosind relatiile anterioare gasim ca an0 +4 6= an0 +2 , an0 +6 6= an0 +4 , . . . , si atunci
|an0 +2 an0 | > |an0 +4 an0 +2 | > |an0 +6 an0 +4 | > > 0,
contradictie. Prin urmare, an+2 = an , n N. Notnd a1 = a3 = a5 = = a N,
a2 = a4 = a6 =
= b N, gasim 2ab = a + b
+ 1, de unde a = 1, b = 2 sau a = 2,
1, n par
2, n par
b = 1, deci f (n) =
sau f (n) =
.
2, n impar
1, n impar
168

L134. Avem un colier cu n margele, numerotate consecutiv 1, 2, . . . , n, unde


n 3. n cte moduri putem sa le coloram cu trei culori, astfel nct oricare doua
margele consecutive sa aiba culori diferite?
Iurie Boreico, elev, Chisin
au
Solutie. Notam cu an numarul modalitatilor de colorare si vom calcula an recursiv. Evident ca a2 = a3 = 6. Fie n 4; putem alege culoarea margelei 1 n
3 moduri, iar culorile m
argelelor 2, 3, . . . , n n cte dou
a moduri, obtinnd astfel
3 2n1 modalit
ati de colorare n care m
argelele 1 si 2, 2 si 3, . . . , n 1 si n au
culori diferite. Mai avem nsa o conditie: ca margelele n si 1 sa aiba culori diferite;
atunci 3 2n1 = an + bn , unde bn este numarul colorarilor de tipul descris mai sus
pentru care m
argelele 1 si n au aceeasi culoare. Observ
am c
a bn = an1 , suprimarea
m
argelei n dnd o corespondenta bijectiv
a ntre num
arul color
arilor corespunz
atoare,
prin urmare an1 + an = 3 2n1 .
Avem ca an + an+1 = 3 2n si, prin scadere, an+1 an1 = 3 2n1 . Deducem ca

22k 1
a2k+1 = 3 22k1 + 22k3 + + 23 + a3 = 6 a2k2 + + 22 + 1 = 6 2
=
2 1
22k+1 2. Cum a2k + a2k+1 = 3 22k , vom avea ca a2k = 22k + 2. Raspunsul poate
n
fi scris sub forma an = 2n + 2 (1) .
L135. Se considera un poligon cu 3n laturi, n 2, nscris ntr-un cerc de raza
1. Aratati ca cel mult 3n2 dintre segmentele
avnd capetele n vrfurile poligonului

au lungimea strict mai mare dect 2.


Bianca-Teodora Iordache, elev
a, Craiova
Solutie. Evident c
a oricum am alege 4 puncte pe cercul
de
raz
a 1, exist
a

dou
a printre acestea situate la o distanta cel mult egal
a cu 2. Consider
am graful G (X, U ), unde X este multimea vrfurilor poligonului initial, iar doua vrfuri vor
fi unite
printr-o muchie daca si numai daca distanta dintre ele este strict mai mare
dect 2. Conform observatiei initiale, oricum am alege 4 vrfuri ale grafului, exist
a
dou
a care nu sunt unite printr-o muchie, deci G nu contine subgrafuri complete de
ordin 4. Aplicam acum urmatorul rezultat:
Teorema lui Turan. Daca G = (X, U ) este un graf neorientat cu n vrfuri ce
nu contine subgrafuri complete de ordin p, iar r este restul mpartirii lui n la p 1,
atunci
p 2 n2 r2
r (r 1)
|U |

+
.
p1
2
2
n cazul nostru avem 3n vrfuri, p = 4, r = 0, prin urmare |U | 3n2 , decicel
mult 3n2 distante formate cu vrfurile poligonului initial sunt strict mai mari ca 2.
Not
a. Solutie asem
an
atoare a dat Vlad Emanuel, student, Bucuresti.

169

Probleme propuse1
Clasele primare
P.154. Dorina are 15 baloane rosii si albastre. Cte baloane rosii poate avea,
dac
a num
arul acestora este mai mic dect num
arul baloanelor albastre si este cel
putin egal cu 3?
(Clasa I )
Inst. Maria Racu, Iasi
P.155. Dintr-o carte lipsesc cteva pagini, de la numarul 71 la numarul 94. Cte
foi lipsesc din aceasta carte?
(Clasa I )
Ionela B
ar
agan, elev
a, Iasi
P.156. La concursul "Desene pe asfalt", elevii claselor I-IV de la S
coala "Otilia
Cazimir" au acumulat 50 de puncte si cel putin 2 premii din fiecare categorie. Care
este cel mare num
ar de premii pe care-l pot primi elevii, dac
a pentru premiul I s-au
acordat 10 puncte, pentru premiul al II-lea s-au acordat 6 puncte, iar pentru premiul
al III-lea s-au acordat 4 puncte?
(Clasa a II-a)
nv. Elena Porfir, Iasi
P.157. Prin golirea unui singur vas, ales dintre cele de mai jos, putem face ca
restul vaselor s
a aib
a cantit
ati egale de lichid. Care vas trebuie golit?

(Clasa a II-a)

Amalia Cantemir, elev


a, Iasi

P.158. Aflati trei numere naturale stiind c


a, adunndu-le dou
a cte dou
a, obtinem
100, 89, respectiv, 141.
(Clasa a III-a)
Inst. Maria Racu, Iasi
P.159. Se consider
a numerele: a = 1 + 4 + 7 + 10 + + 2008, b = 2 + 5 + 6+
+ + 2009, c = 3 + 6 + 8 + + 2010. Aratati ca suma a + b + c se mparte exact
la 3, fara sa calculati aceasta.
(Clasa a III-a)
Iuliana Moldovanu, elev
a, Iasi
P.160. Numarul a este de forma xy0, iar numarul b este de forma uv. Sa se afle
a si b stiind c
a a + b = 22 zeci.
(Clasa a III-a)
Dragos Toma, elev, Iasi
P.161. Fie a si b doua numere naturale astfel nct diferenta lor este de 5 ori mai
mic
a dect suma lor. S
a se arate c
a num
arul cel mai mare se mparte exact la 3, iar
cel mai mic se mparte exact la 2.
(Clasa a IV-a)
Diana T
an
asoaie, elev
a, Iasi
P.162. Maria are 9 s
aculeti cu monede. Cel putin un s
aculet cnt
areste un
kilogram. n orice grupare de 5 saculeti, cel putin 3 saculeti au aceeasi masa, iar n
orice grupare de 6 saculeti, cel mult 5 saculeti au aceeasi masa. Care este cel mai
mare num
ar de s
aculeti de un kilogram pe care l poate avea Maria?
(Clasa a IV-a)
Petru Asaftei, Iasi
1

Se primesc solutii pn
a la data de 1 iunie 2009.

170

P.163. Jum
atatea produsului a dou
a numere naturale consecutive mp
artit
a cu
3, nu poate da niciodata restul 2.
Recreatii S
tiintifice, Anul I (1883), nr. 4, pag. 119

Clasa a V-a
V.95. Doua numere naturale se scriu n baza 10 folosind doar cifrele 1, 4, 6, si 9.
Poate fi unul dintre numere de 2008 ori mai mare dect cel
alalt?
C
at
alin Budeanu, Iasi

V.96. Determinati k, n N astfel nct


(1 + 1 n) + (2 + 2 n) + + (k + k n) = 3 4 5 6.

Petru Asaftei, Iasi


atrat perfect.
V.97. Ar
atati c
a num
arul N = 17n +21n +25n , n N, nu poate fi p
Virginia Grigorescu, Craiova
V.98. Fie n N . S
a se demonstreze c
a num
arul N = 5050 . . . 505 (2n + 1 cifre)
se scrie ca suma a 4n + 2 patrate perfecte distincte.
Veronica Pl
aesu si Dan Pl
aesu, Iasi
V.99. Se consider
a num
arul N = 1 + 11 + 101 + 1001 + + |100 {z
. . . 01}.
n cifre
a) Pentru n N, n 5, aratati ca 5 | N 5 | n.
b) Precizati care dintre propozitiile "3 | n 3 | N " si "3 | N 3 | n"
adev
arat
a pentru orice n 3.
Temistocle Brsan,
V.100. Determinati numerele naturale nenule a si b pentru care exist
a n
a
3n + 2
si 3a + 2b < 100.
astfel nct =
b
7n + 5
Gheorghe Iurea,

este

Iasi
N

Iasi
an + b
V.101. Consider
am fractia
, unde n, a, b, c, d N , astfel nct b si d au
cn + d
paritati diferite, iar a si c au aceeasi paritate. Aratati ca, daca ad bc = 2k , k N ,
atunci fractia este ireductibila.
Cosmin Manea si Dragos Petric
a, Pitesti

Clasa a VI-a

a1 a2
VI.95. Determinati numerele naturale nenule a1 , a2 ,. . . , a2008 , stiind c
a
=
12
a2007 a2088
a2 a3
= =
, iar a1 + a2008 = 2009.
23
2007 2008
Gheorghe Iurea, Iasi
VI.96. Determinati p N pentru care numerele p, p + 12, p + 22, p + 52, p + 72,
p + 102 si p + 132 sunt prime.
Damian Marinescu, Trgoviste

VI.97. a) Daca a, b, c, d, e, f N sunt astfel nct (a, b) = (c, d) = (e, f ) =


a c
e
(b, d) = 1, iat t = + N, aratati ca f = bd.
b
d f
7
4
1
+ N.
b) Determinati a, b N pentru care

2a + 1 2b
6
Cosmin Manea si Dragos Petric
a, Pitesti
171

VI.98. Determinati cel mai mic num


ar natural n cu proprietatea c
a num
arul
zerourilor n care se termina numarul (n + 10)! este cu 2008 mai mare dect numarul
zerourilor n care se termina n! (unde n! = 1 2 3 n).
C
at
alin Budeanu, Iasi
VI.99. Un patrulater convex are doua laturi opuse congruente si diagonalele
congruente. Aratati ca patrulaterul este trapez isoscel sau dreptunghi.
Ioan S
ac
aleanu, Hrl
au

b
b
b
VI.100. Fie 4ABC cu m(A) 90 . Sa se arate ca m(B) = 2m(C) daca si
numai daca exista M [BC] astfel nct AB = AM = M C.
Petru Asaftei, Iasi
b
VI.101. Fie ABC un triunghi dreptunghic cu m(A) = 90 si CD bisectoarea
b D (AB). Perpendiculara din D pe bisectoarea unghiului B
b interunghiului C,
secteaza ipotenuza BC n E. Daca P este punctul de intersectie a bisectoarelor
unghiurilor triunghiului ABC, iar M este punctul de intersectie dintre EP si AC,
\
\
ar
atati c
aM
PA P
BE.
Nela Ciceu, Bac
au si Titu Zvonaru, Com
anesti

Clasa a VII-a
VII.95. Fie ABCD p
atrat, M un punct oarecare pe (AB), iar N (BC) este
astfel nct M N M D. Aratati ca AM AB + CN CB = DM 2 .
Ovidiu Pop, Satu Mare si Gh. Szllsy, Sighetul Marmatiei
VII.96. Fie [AD] median
a n 4ABC, M mijlocul lui [AD], {E} = BM AC,
iar punctul F pe dreapta AB este astfel nct CF k AD. Demonstrati ca punctele
D, E si F sunt coliniare.
Mirela Marin, Iasi
VII.97. Fie C1 (O1 , r1 ) si C2 (O2 , r2 ), r1 < r2 , dou
a cercuri tangente exterior.
Consider
am punctele A0 C1 , B 0 C2 , de aceeasi parte a dreptei O1 O2 , astfel nct
A0 O1 k B 0 O2 . Daca AB este tangenta comuna exterioara a cercurilor (A C1 ,
B C2 ), demonstrati c
a dreptele AB, A0 B 0 si O1 O2 sunt concurente.
Romanta Ghita
si Ioan Ghita
, Blaj
VII.98. Sa se determine numerele naturale nenule a si b, stiind ca sunt direct
proportionale cu b 6 si a si invers proportionale cu a + 12 si b.
Constantin Apostol, Rm. S
arat
VII.99. Fie a, b Z si numerele A = 119a5 + 5b3 4a si B = 119b5 + 5a3 4b.
Sa se arate ca A se divide cu 120 daca si numai daca B se divide cu 120.
Dan Nedeianu, Dr. Tr. Severin
VII.100. Aratati ca 2a2 + 15b2 + 7c2 10ab 6ac + 20bc, a, b, c R.
Alexandru Negrescu, student, Iasi
VII.101. Pentru n N , not
am cu d (n) num
arul divizorilor primi ai lui n.
a) Determinati cardinalul multimii A = {n N | n 208, d (n) = 3}.
b) Aflati cel mai mic si cel mai mare element al multimii
B = {k N | n N , n 2008, a.. d (n) = k} .

Gabriel Popa, Iasi

172

Clasa a VIII-a
VIII.95. Pentru a, b, c R , notam =
numarul x =

b3
c3
a3
+ 3 + 3 n functie si .
3
b
c
a

a b
c
a c
b
+ + , = + + . Calculati
b
c a
c
b a

Elena Nicu, Malu-Mare (Dolj)


VIII.96. Rezolvati n numere naturale ecuatia x2 + y 2 + xy = x2 y 2 .
Mihail Bencze, Brasov
VIII.97. Fie d1 , d2 , d3 , d lungimile diagonalelor fetelor, respectiv diagonalei unui
2d2 d2
paralelipiped dreptunghic. Daca d21 = 2 2 32 , sa se arate ca paralelipipedul are o
d2 + d3

d 3
.
muchie de lungime cel putin egal
a cu
3
Gheorghe Molea, Curtea de Arges
\
VIII.98. Fie VABCD piramid
a patrulater
a regulat
a. Not
am u = m((V BC),(ABC)),
\
\
v = m((V BC)
, (V CD)) si t = m((V BC)
, (V AD)). Ar
atati c
a u + v + t > 180 .
Claudiu S
tefan Popa, Iasi

VIII.99. Pentru n N , consider


am A = 12 , 22 , 32 , . . . , n2 . Determinati n,

stiind c
a exist
a o functie f : A A astfel nct f (x) f (y) = x y, x, y A.
Cristian Laz
ar, Iasi
2
2
n
VIII.100. Rezolvati n n N ecuatia x 8 + 1287 = 0.
Mihai Cr
aciun, Pascani
VIII.101. Se calculeaza suma cifrelor pentru fiecare dintre numerele de la 1 la
n, n > 10. Pentru fiecare suma dintre cele n se calculeaza din nou suma cifrelor,
repetndu-se aceast
a operatie pn
a cnd obtinem n numere formate din cte o singur
a
cifr
a. S
a se afle n, stiind c
a n multimea astfel obtinut
a cifrele 1, 2, 3 si 4 se repet
a
de cte 101 ori fiecare, iar cifrele 5, 6, 7, 8 si 9 de cte 100 ori fiecare.
Mihai Haivas, Iasi

Clasa a IX-a

2IX.91. Fie
a, b, c, p R, p > 0. Daca ax + bx + c p, x [1, 1], atunci
cx + bx + a 2p, x [1, 1].
Dorin M
arghidanu, Corabia
IX.92. Fie n N, n 3, iar , R astfel nct n + + 6= 0. Ar
atati c
a
n2
X
(1 + ) (n + )
i+1
(1) (1 + ) (n i 1 + )
(1 + ) (n 1 + ) +
n++
i=1

( + 1) ( + n 1)
.
n++
Gheorghe Costovici, Iasi
AB
3
b = 90 si
IX.93. Fie 4ABC dreptunghic cu m(A)
= , iar D mijlocul lui
AC
2
[AC]. Notam cu E punctul de intersectie a cercurilor C1 (A, AD) si C2 (B, BC), aflat
[
de aceeasi parte a dreptei AB ca si punctul C. Determinati m
asura unghiului CAE.
aeru, Suceava
C
at
alin Tig
n

( + 1) ( + i 1) + (1) ( + n 2) = (1)

173

IX.94. n 4ABC, I este centrul cercului nscris, iar {M } = AI BC. Demon\


strati c
a bisectoarea unghiului AM
C, BI si AC sunt trei drepte concurente dac
a si

b
numai dac
a m(A) = 120 .
Vlad Emanuel, student si Andrei Cozma, elev, Bucureti
a
IX.95. Dac
a xi [0, a], i = 1, n si xn+1 = x1 , demonstrati c
n
X
na2
xi+1 (a xi ) <
.
4 sin2 4
i=1
Gigel Buth, Satu Mare

Clasa a X-a
X.91. Ar
atati c
a

3 2 2
1 4
3 4
4
1 2
+ arctg
+
= 2 arctg
+ arctg
+
arctg
.
7
4
16
7
4
256

D. M. B
atinetu-Giurgiu, Bucuresti

X.92. Fie a, b C. Demonstrati ca ecuatia z 2 az


+ b = 0 are ambele solutii de
2
modul 1 dac
a si numai dac
a |b| = 1 si |a| + a2 4b = 4. (n legatura cu X.77 din
RecMat - 1/2007.)
Marian Tetiva, Brlad
X.93. Dac
a a1 , a2 , . . . , an (0, 1) sau a1 , a2 , . . . , an (1, ), iar f, g : {1, 2, . . . , n}
{1, 2, . . . , n} sunt functii injective, sa se arate ca
X
X

n
n
logak af (k)
ak n2 .
ag(k)
k=1
k=1
Dan Popescu, Suceava
X.94. a) S
a se arate c
a
p
p
p
x2n + y 2n + xn y n + x2n + z 2n + xn z n y 2n + z 2n + y n z n , x, y, z R , n N.

b) Demonstrati ca, daca n este par, inegalitatea este stricta, iar daca n este impar,
atunci exista x, y, z R pentru care se atinge egalitatea.
Bogdan Victor Grigoriu, F
alticeni
X.95. Consideram functia f : R3 R3 ,

f (x, y, z) = sin x + sin y + sin z + sin (x y) + sin (y z) + sin (z x) .

Determinati maximul si minimul functiei f .

C
at
alin Calistru, Iasi

Clasa a XI-a
XI.91. Fie matricele A, B, C, D Mn (R) astfel nct AC + BD = In , iar
AD = BC. Demonstrati c
a CA + DB = In si DA = CB.
I. V. Maftei, Bucuresti si Mihai Haivas, Iasi

1 1
2
.
XI.92. Determinati matricele X Mn (R) pentru care X + X =
1 1
Adrian Reisner, Paris
174

XI.93. Studiati convergenta sirului (un )n1 definit prin u1 0, un+1 =

n N .

un + 1
,
u2n + 1

Gheorghe Costovici si Adrian Corduneanu, Iasi


XI.94. S
a se demonstreze c
a pentru orice n N , exist
a numerele distincte
4 n
.
x1 , x2 , . . . , xn (1, 2), asa nct x1 x2 xn =
e
Dan Pl
aesu, Iasi

1 n1
1
1 21
+ + 1 +
n , unde
XI.95. Calculati lim 1 + + 1 +
n
n
n
n
1 este fixat. (n legatura cu L83 din RecMat-1/2005.)
Marius Olteanu, Rm. Vlcea

Clasa a XII-a
XII.91. Prove that

R1
0

(1 + x) e(1+x)e dx = ee 1.

Zdravko Starc, Vr
sac, Serbia
XII.92. Fie b > a > 0, iar f : [a, b] R o functie continu
a pe [a, b] si derivabil
a
Rc
pe (a, b); s
a se arate c
a exist
a c (a, b) astfel nct b f (x) dx = c (b c) f (c).
a

Dan Nedeianu, Dr. Tr. Severin

R2 sin x
2 cos 1
dx
.
x
c
1
Constantin Micu, Melinesti (Dolj)

XII.93. Demonstrati c
a exist
a c (2, ) pentru care
2n
R

xa + b

dx, unde a (0, ) si b R.


n n
x2a+4 + 1
Liviu Smarandache, Craiova
XII.95. Fie (A, +, ) un inel n care 0 6= 1 si 1 + 1 + 1 + 1 + 1 = 0. Sa se arate
c
a, dac
a x3 y 2 = y 2 x3 , x, y A, atunci inelul este comutativ.
I.V. Maftei, Bucuresti si Mihai Haivas, Iasi
XII.94. Calculati lim n

Semnalam cititorilor reeditarea colectiei complete a revistei

RECREA
TII S
TIIN
TIFICE (1883-1888)
la 125 de ani de la aparitia primului num
ar, cu respectarea formei n care a fost
publicat
a initial. Revista prezint
a si ast
azi interes prin culoarea limbii romne si
terminologiei folosite, prin continutul interesant si de un nalt nivel stiintific, precum
si prin forma grafica frumoasa. Cei interesati pot consulta site-ul revistei

http://www.recreatiistiintifice.ro
de unde se poate prelua gratuit.

175

Probleme pentru preg


atirea concursurilor
A. Nivel gimnazial
G146. Fie x, y, z (0, ) astfel nct xyz = 1. Aratati ca
yz 3
zx3
xy 3
+ 4
+ 4
1.
4
x +y+z
y +z+x z +x+y
Liviu Smarandache si Lucian Tutescu, Craiova
G147. Fie n N, n 2, fixat, iar a, b, c sunt numere naturale astfel nct
hn 1i
a + b + c n.
na + (n + 1) b + 2nc = n2 + 1. Aratati ca n
2
Gheorghe Iurea, Iasi
a se arate c
a orice num
ar natural are un multiplu
G148. Fie a1 a2 . . . ap N. S
de forma a1 a2 . . . ap a1 a2 . . . ap . . . a1 a2 . . . ap 0 . . . 0.
Marian Pantiruc, Iasi
G149. a) Determinati dou
a numere prime p, q astfel nct p < q, iar p2 1 are
mai multi divizori naturali dect q 2 1.
b) Determinati toate numerele prime p pentru care p2 1 are exact opt divizori
naturali.
Dan Popescu, Suceava
G150. Fie m si n numere naturale nenule cu proprietatea c
a m 1 + 2 + + n.
S
a se arate c
a m poate fi scris ca suma ctorva numere distincte dintre 1, 2,. . . , n.
Marian Tetiva, Brlad
G151. Bazele unei prisme sunt poligoane cu 2008 vrfuri. Numerot
am cu 1,
2,. . . , 2008 vrfurile bazei inferioare si, corespunz
ator, cu a1 , a2 ,. . . , a2008 vrfurile
bazei superioare, unde {a1 , a2 , . . . , a2008 } = {1, 2, . . . , 2008}.
a) Demonstrati ca putem gasi o numerotare pentru baza superioara astfel nct
.
i + ai .. 8, i {1, 2, . . . , 2008}.
b) Demonstrati ca nu putem gasi o numerotare pentru baza superioara astfel nct
.
i + ai .. 9, i {1, 2, . . . , 2008}.
Gabriel Popa si Gheorghe Iurea, Iasi
altiG152. n triunghiul isoscel ABC (AB = AC) not
am cu B 0 , C 0 picioarele n
0 0
milor din B, respectiv C. Dac
a AB = 2 B C , s
a se determine unghiurile triunghiului.
Nela Ciceu, Bac
au si Titu Zvonaru, Com
anesti
\ = 30 si
G153. n triunghiul ABC, M este mijlocul laturii [BC], m(ABC)
\ = 105 . Perpendiculara din C pe AM taie AB n Q. Calculati valoarea
m(ACB)
QA
raportului
.
QB
Neculai Roman, Mircesti (Iasi)
G154. Fie D mijlocul laturii [BC] a triunghiului echilateral ABC de latur
a 1,
iar P un punct mobil pe [CD]. Not
am cu M si N proiectiile pe AP ale punctelor B,
respectiv C. Aflati aria locului geometric descris de segmentul [M N ].
Marius Olteanu, Rm. Vlcea
G155. Fie C cercul circumscris 4ABC ascutitunghic. Not
am cu P punctul de
intersectie al tangentelor duse la cerc n B si C, {D} = AP C, iar M si N sunt
176

mijloacele arcului mic BC, respectiv arcului mare BC. S


a se arate c
a dreptele AM ,
DN si BC sunt concurente.
Gabriel Popa, Iasi

B. Nivel liceal
L146. n plan se considera dreptele d1 , d2 ,. . . , dn+1 , oricare doua neparalele.
Not
am cu k = m(dk\
, dk+1 ), k 90 , k = 1, n. Pe d1 se consider
a un segment
de lungime 2 care se proiecteaza pe d2 , apoi segmentul obtinut se proiecteaza pe
d3 si tot asa, pna cnd p
tiind ca
pe dn+1 se obtine un segment de lungime 1. S

n
tg min i | i = 1, n =
4 1, determinati unghiurile k , k = 1, n.
Cristian S
avescu, student, Bucuresti

L147. Se consider
a un poligon convex cu n laturi, n 4, avnd proprietatea c
a
oricare dou
a diagonale nu sunt paralele si oricare trei nu sunt concurente n puncte
diferite de vrfurile poligonului. Se noteaz
a cu ni num
arul punctelor de intersectie a
diagonalelor interioare poligonului si cu ne cel al punctelor de intersectie exterioare
poligonului.
a) S
a se arate c
a exist
a exact opt poligoane care verific
a relatia ni > ne .
b) S
a se arate c
a exist
a exact trei poligoane pentru care ni + ne = kn2 , k N .
Mihai Haivas, Iasi
L148. Pe latura (AB) a triunghiului ABC consider
am punctul D astfel nct
AB = 4 AD. De aceeasi parte a laturii AB ca si punctul C, lu
am un punct P
\
\ si P B = 2 P D. Demonstrati ca patrulaterul ABCP este
astfel nct P
DA ACB
inscriptibil.
Nela Ciceu, Bac
au si Titu Zvonaru, Com
anesti
L149. S
a se determine pozitia punctului P pe directoarea parabolei P, astfel
nct aria triunghiului P T1 T2 sa fie minima, unde T1 si T2 sunt punctele de contact
cu parabola ale tangentelor duse din P la P.
Adrian Corduneanu, Iasi
L150. Fie tetraedrul A1 A2 A3 A4 , iar P un punct n interiorul s
au. Not
am cu
Aij (Ai Aj ) proiectiile ortogonale ale lui P pe muchiile Ai Aj ale tetraedrului.
Demonstrati ca
1
VP A12 A13 A23 + VP A12 A14 A24 + VP A13 A14 A34 + VP A23 A24 A34 VA1 A2 A3 A4 .
4
Cnd se atinge egalitatea?
Marius Olteanu, Rm. Vlcea
S
a se demonstreze ic
a nu exist
a numere naturale n si k astfel nct
h L151.
2n+1 i h
k
2+ 3
= 4 + 15 .
Cosmin Manea si Dragos Petric
a, Pitesti
L152. Pentru a, b, c R si x R+ , demonstrati inegalitatea
2

3 (x + 1) (a + b + c)
1 1 1
9
i
h
2+ 2+ 2.
2
2
2
2
2
a +b +c
a b c
3(x2 + 1)(a2 + b2 + c2 ) + 2x(a + b + c) (ab + bc + ca)
I. V. Maftei si Dorel B
aitan, Bucuresti
177

L153. G
asiti toate functiile f : R R cu proprietatea c
a

2
2
f x + xy + yf (y) = xf (x + y) + f (y) , x, y R.

Adrian Zahariuc, student, Princeton


L154. Fie P R [X] un polinom de gradul n si p : R R functia polinomial
a asociat
a. S
tiind c
a multimea {x R | p (x) = 0} are k elemente (distincte), iar
functia f : R R, f (x) = |p (x)| este derivabila pe
h nR,
i aratati ca numarul maxim de
radacini complexe nenule ale lui P este egal cu 2
2k.
2
Vlad Emanuel, student, Bucuresti
a matrice astfel nct matricea AB BA s
a fie
L155. Fie A, B M2 (C) dou
inversabila. Sa se arate ca urma matricei (I2 + AB) (AB BA)1 este egala cu 1.
Florina Crlan si Marian Tetiva, Brlad

Training problems for mathematical contests


A. Junior highschool level
G146. Let x, y, z (0, ) such that xyz = 1. Prove that

xy 3
yz 3
zx3
+ 4
+ 4
1.
+y+z
y +z+x z +x+y
Liviu Smarandache and Lucian Tutescu, Craiova
G147. Let n N, n 2 be a fixed number and let a, b, c be natural numbers
hn 1i
such that na + (n + 1) b + 2nc = n2 + 1. Show that n
a + b + c n.
2
Gheorghe Iurea, Iasi
G148. Let a1 a2 . . . ap N. Show that every natural number has a multiple of
the form a1 a2 . . . ap a1 a2 . . . ap . . . a1 a2 . . . ap 0 . . . 0.
Marian Pantiruc, Iasi
G149. a) Determine two prime numbers p, q so that p < q, and p2 1 has more
natural divisors than q 2 1.
b) Determine all the prime numbers p such that p2 1 has exactly eight natural
divisors.
Dan Popescu, Suceava
G150. Let m and n be nonzero natural numbers with the property that m
1 + 2 + + n. Show that m may be written as the sum of a couple of distinct
numbers among 1, 2,. . . , n.
Marian Tetiva, Brlad
G151. The bases of a prism are polygons with 2008 vertices. We number by
1, 2, . . . , 2008 the vertices of the lower basis and by a1 , a2 ,. . . , a2008 the vertices of
the upper basis, where {a1 , a2 , . . . , a2008 } = {1, 2, . . . , 2008}.
.
a) Show that we can find a numbering for the upper basis so that i + a .. 8,
x4

i {1, 2, . . . , 2008}.
178

.
b) Show that we cannot find a numbering for the upper basis so that i + ai .. 9,
i {1, 2, . . . , 2008}.
Gabriel Popa and Gheorghe Iurea, Iasi
G152. In the isosceles triangle ABC (AB = AC), B 0 , C 0 denote the feet of
the altitudes from B, respectively C. If AB = 2 B 0 C 0 , determine the angles of the
triangle.
Nela Ciceu, Bac
au and Titu Zvonaru, Com
anesti
\
G153. In the triangle ABC, M is the midpoint of the side [BC], m(ABC) = 30
\ = 105 . The perpendicular from C on AM cuts AB at Q. Calculatre
and m(ACB)
QA
the value of the ratio
.
QB
Neculai Roman, Mircesti (Iasi)
G154. Let D be the midpoint of the side [BC] in the equilateral triangle ABC
of side length 1, and let P be a moving point on [CD]. Denote by M and N the
projections of the points B, respectively C on AP Find the area of the geometric
locus described by the segment [M N ].
Mariu Olteanu, Rm. Vlcea
G155. Let C be the circumcircle of the acute-angled triangle 4ABC. Denote by
P the intersection point of the tangents to the circle at B and C, {D} = AP C,
while M and N are the midpoints of the small arc BC, respectively of the big arc

BC. Show that the straight lines AM , DN and BC meet at a point.


Gabriel Popa, Iasi

B. Highschool level
L146. The straight lines d1 , d2 ,. . . , dn+1 , are considered in the plane such that
any two lines are not parallel. We denote by k = m(dk\
, dk+1 ), k 90 , k = 1, n.
A segment of length 2 is cosidered on d1 that is projected on d2 , then the obtained
segment is projected on d3 and so on, until a segment
of length 1 is obtained on

p
n
4 1, determine the angles k ,
dn+1 . Knowing that tan min i | i = 1, n =
k = 1, n.
Cristian S
avescu, student, Bucuresti
L147. A convex polygon with n sides, n 4, is considered such that any pair of
diagonals are not parallel and and any three diagonals do not meet at other points
except the vertices of the polygon. Let us denote by ni the number of intersection
points of the diagonals inside the polygon and by ne the number of intersection points
of the diagonals outside the polygon.
a) Show that exactly eight polygons exist such that the inequality ni > ne is
satisfied.
b) Show that exactly three polygons exist such that ni + ne kn2 , k N .
Mihai Haivas, Iasi
L148. A point D is considered on the side (AB) of the triangle ABC such that
AB = 4 AD. In the same halfplane as point C with respect to the side AB, we take
\
\ and P B = 2 P D. Prove that the quadrilateral
a point P such that P
DA ACB
ABCP is inscriptible, that si it admits a circumscribed circle.
Nela Ciceu, Bac
au and Titu Zvonaru, Com
anesti
179

L149. Determine the position of the point P on the directrix line of the parabola
P, so that the area of the triangle P T1 T2 be minimum, where T1 and T2 are the
contact points with P of the tangents drawn from P to P.
Adrian Corduneanu, Iasi
L150. Let us consider the tetrahedron A1 A2 A3 A4 , and a point P inside it.
Denote by Ai j (Ai Aj ) the orthogonal projections of P on the edge(s) Ai Aj of the
tetrahedron. Prove that
1
VP A12 A13 A23 + VP A12 A14 A24 + VP A13 A14 A34 + VP A23 A24 A34 VA1 A2 A3 A4 .
4
When the equality is attained?
Marius Olteanu, Rm. Vlcea
h
2n+1 i
L151. Prove than no natural numbers n and k exist such that 2 + 3
=
h
k i
4 + 15 .
Cosmin Manea and Dragos Petric
a, Pitesti
L152. For a, b, c R and x R+ , prove the inequality

3 (x + 1)2 (a + b + c)4
1 1 1
9
h
i

2+ 2+ 2.
2
2
a2 + b2 + c2
a
b c
3(x2 + 1)(a2 + b2 + c2 ) + 2x(a + b + c) (ab + bc + ca)
I. V. Maftei and Dorel B
aitan, Bucuresti
L153. Find all functions f : R R with the property that

f x2 + xy + yf (y) = xf (x + y) + f 2 (y) , x, y R.

Adrian Zahariuc, student, Princeton


L154. Let P R [X] a polynomial of degree n and p : R R its associated
polynomial function. Knowing that the set {x 6 R | p (x) = 0} consists of k (distinct)
elements, and the function f : R R, f (x) = | p (x)| is dierentiable
h n i on R, show
that the maximum number of nonzero complex roots of P equals 2
2k.
2
Vlad Emanuel, student, Bucuresti
L155. Let A, B M2 (C) be two matrices such that the matrix A B B A is
invertible. Show that the trace of the matrix (I2 + AB) (AB BA)1 is equal to 1.
Florentina Crlan and Marian Tetiva, Brlad

180

Concursul omagial "Recreatii S


tiintifice"
Acest concurs este organizat cu prilejul mplinirii a 125 de ani de la aparitia revistei "Recreatii S
tiintifice" (1883-1888), prima revist
a stiintific
a (predominant matematic
a) din tar
a adresat
a tineretului.
Organizatorii concursului: Asociatia "Recreatii matematice".
Premiile prev
azute de concurs:
Premiul I
200 lei (un premiu)
Premiul II
100 lei (dou
a premii)
Premiul III 50 lei
(trei premii)
Participantii la concurs: orice elev al scolilor de orice nivel.
Obligatiile concurentilor:
se cere rezolvarea celor cinci probleme enuntate mai jos, selectate din revistele
Recreatii Stiin
tifice si Recreatii matematice;
elevii vor trimite solutiile prin post
a (plic simplu timbrat) pe adresa:
Asociatia "Recreatii matematice"
str. Aurora, nr. 3, sc. D, ap. 6
700474, Iasi
cu mentiunea (pe plic): Concursul "Recreatii Stiin
tifice";
data limit
a de participare este 1.02.2009;
concurentii vor redacta ngrijit solutiile problemelor trimise (cte o singura
problema pe foaie, cu enunt, figura etc.).
Acordarea premiilor se face pe baza punctajelor obtinute de concurenti:
fiecare problema este notata maxim cu 10 puncte;
se acorda cte 2 puncte suplimentare pentru alte solutii ale problemei, generaliz
ari etc;
se depuncteaz
a solutiile incomplete sau redactate nengrijit;
concurentii trebuie sa obtina
minim 42 puncte, pentru premiul I,
minim 35 puncte, pentru premiul II,
minim 25 puncte, pentru premiul III.
Sponsor: Fundatia culturala "Poiana" (director, Dan Tiba).
Premiantii concursului vor fi anuntati n nr. 1/2009 al revistei Recreatii matematice (ce va ap
area n martie 2009).
Problemele concursului
1. Ion si Constantin merg la cumparaturi cu sotiile lor, Maria si Elena. Fiecare
din aceste patru persoane cumpara un numar de obiecte ce le plateste pe fiecare cu
attia lei cte obiecte a cumparat. Ion cumpara noua obiecte mai mult dect Elena
si fiecare sot cheltuieste cu 21 lei mai mult dect sotia sa. Care este sotia lui Ion si
care este a lui Constantin? Care este numarul de obiecte cumparate de fiecare dintre
aceste persoane? Care este suma cheltuita de fiecare dintre ele?
181

2. Sa se rezolve sistemul de ecuatii


(x + 2y) (x + 2z) = a,
(y + 2x) (y + 2z) = b,
(z + 2x) (z + 2y) = c
(0 < a < b < c) .
3. Fie O, I, I 0 centrele cercului circumscris triunghiului ABC, cercului nscris
acestuia si, respectiv, al cercului exnscris tangent laturii BC. Sa se demonstreze ca
[0 = 2 |sin B sin C| .
tg IOI
2 cos A 1
4. Sa se taie o sfera cu un plan astfel nct diferenta volumelor conurilor drepte
ce au ca baze sectiunea planului cu sfera si vrfurile pe sfera sa fie maxima.
5. Fie M un punct exterior cercului C de centru O si raza R. Notam cu T1 , T2
punctele de contact ale tangentelor duse din M la C si cu A punctul de intersectie a
dreptei OM cu cercul C care verifica conditia A
/ [OM ]. Sa se determine multimea
punctelor M pentru care se poate construi un triunghi cu segmentele [M T1 ], [M T2 ]
si [M O], dar nu se poate construi un triunghi cu [M T1 ], [M T2 ] si [M A].

IMPORTANT
n scopul unei leg
aturi rapide cu redactia revistei, pot fi utilizate urm
atoarele
adrese e-mail: t_birsan@yahoo.com si profgpopa@yahoo.co.uk . Pe
aceasta cale colaboratorii pot purta cu redactia un dialog privitor la materialele trimise acesteia, procurarea numerelor revistei etc. Sugeram colaboratorilor care trimit probleme originale pentru publicare s
a le numeroteze si
s
a-si retin
a o copie xerox a lor pentru a putea purta cu usurinta o discutie
prin e-mail asupra acceptarii/neacceptarii acestora de catre redactia revistei.
La problemele de tip L se primesc solutii de la orice iubitor de matematici
elementare (indiferent de preocupare profesionala sau vrsta ). Fiecare dintre
solutiile acestor probleme - ce sunt publicate n revist
a dup
a un an - va fi
urmata de numele tuturor celor care au rezolvat-o.
Adres
am cu insistenta
amintea ca materialele trimise revistei
rug
s
a nu fie (s
a nu fi fost) trimise si altor publicatii.
Rug
am ca materialele tehnoredactate s
a fie trimise pe adresa redactiei nsotite de fisierele lor (de preferinta n LATEX).
Pentru a facilita comunicarea redactiei cu colaboratorii ei, autorii materialelor sunt rugati s
a indice adresa e-mail.
182

Pagina rezolvitorilor
BRA
SOV
Colegiul National de Informatica "Gr. Moisil". Clasa a IX-a (prof. POPOVICI
Florin). DARIE Flavius: VII.88, VIII(88-90), IX(88,89).
CRAIOVA
Colegiul National "Carol I". Clasa a VIII-a (prof. STANCA Monica). STANCIU Ioan: VII(81,82), VIII(84-86), G(126,129,130,133).
IA
SI

Scoala
nr. 11 "Otilia Cazimir". Clasa a III-a (inst. HUZUM Lina). ANICAI

Simina: P(144-148);
Laura: P(144-148); HUZUM Andrei: P(144-148); MARIU
TA
STOIAN Ioana: P(144-148).
Elena). BACIU TuA
Scoala
nr. 14 "Gh. Mrzescu". Clasa a III-a (inst. NUT

A George-Stefan: P(144-148); POSTUDOR Georgianador: P(144-148); CHIRILUT


M
ad
alina: P(144-148); STOICA Adriana: P(144-148).

Scoala
nr. 26 "G. Cosbuc". Clasa a III-a (inst. RACU Maria). APACHITEI

Aura Georgiana: P(144-149); BURA Emma-Andreea: P(144-149); FILIP Ingrid Narcis-Eugen: P(144-149); HRISCU OvidiuA
S
tefania: P(144-149); GHEORGHIT
Constantin: P(144-150); HUZA M
ad
alina: P(144-150); LESOVSCHI AlexandraIoana: P(144-149); LUPU Roxana-Elena: P(144-149); MARICIUC Dragos-Claudiu:
P(144-149); MAXIM Alexandra-Camelia: P(144-149); TUDOSE Ema-Alina: P(144 Cosmin: P(144-149); VASILE Bogdan-Andrei: P(144-149). Clasa a III
150); TUC
A
Andrei-Alexandru: P(144-150); DUMBRAVA

a (nv. HRIMIUC Valeria). BRUMA


Bianca: P(144-150); HARAPCIUC Eduard-Gabriel: P(144-150); MANTALEA AlexAdrian: P(144-150); OLARU Alexandra: P(144-150).
Colegiul National. Clasa a V-a (prof. POPA Gabriel). STOLERU Georgiana
Ingrid: V(88-90,93,94).

A
Colegiul National "C. Negruzzi". Clasa a VII-a (prof. SAVA Radu). IONIT
Norbert Traian: VII(81-86), G126.
SUCEAVA
Scoala
cu clasele I-VIII, nr. 3. Clasa a III-a (nv. TABARCEA Silvestru).

FECHET S
tefan: P(136-139,141,143); Clasa a IV-a (inst. NECHITA Daniela). FECHET Mircea: P(134-141,143).

Premii acordate rezolvitorilor


Scoala
nr. 14 "Gh. Mrzescu", Iasi

BACIU Tudor (cl. a III-a): 2/2007(10pb), 1/2008(6pb), 2/2008(5pb),


George-
A
CHIRILUT
Stefan (cl. a III-a): 2/2007(10pb), 1/2008(6pb), 2/2008(5pb),
STOICA Adriana (cl. a III-a): 2/2007(10pb), 1/2008(6pb), 2/2008(5pb).
Scoala
nr. 26 "G. Cosbuc", Iasi

Narcis-Eugen (cl. a III-a): 2/2007(5pb), 1/2008(6pb), 2/2008(5pb).


A
GHEORGHIT
183

Revista semestrial RECREAII MATEMATICE


este editat de
ASOCIAIA RECREAII MATEMATICE. Apare la datele de 1 martie i
1 septembrie i se adreseaz elevilor, profesorilor, studenilor i tuturor celor
pasionai de matematica elementar.
n atenia tuturor colaboratorilor
Materialele trimise redaciei spre publicare (note i articole, chestiuni de
metodic, probleme propuse etc.) trebuie prezentate ngrijit, clar i concis; ele
trebuie s prezinte interes pentru un cerc ct mai larg de cititori. Se recomand ca
textele s nu depeasc patru pagini. Evident, ele trebuie s fie originale i s
nu fi aprut sau s fi fost trimise spre publicare altor reviste. Rugm ca materialele tehnoredactate s fie nsoite de fiierele lor.
Problemele destinate rubricilor: Probleme propuse i Probleme pentru
pregtirea concursurilor vor fi redactate pe foi separate cu enun i demonstraie/rezolvare (cte una pe fiecare foaie) i vor fi nsoite de numele autorului, coala i localitatea unde lucreaz/nva.
Redacia va decide asupra oportunitii publicrii materialelor primite.
n atenia elevilor
Numele elevilor ce vor trimite redaciei soluii corecte la problemele din
rubricile de Probleme propuse i Probleme pentru pregatirea concursurilor
vor fi menionate n Pagina rezolvitorilor. Se va ine seama de regulile:
1. Pot trimite soluii la minimum cinci probleme propuse n numrul
prezent i cel anterior al revistei; pe o foaie va fi redactat soluia unei singure
probleme.
2. Elevii din clasele VI-XII au dreptul s trimit soluii la problemele
propuse pentru clasa lor, pentru orice clas mai mare, din dou clase mai mici i
imediat anterioare. Elevii din clasa a V-a pot trimite soluii la problemele propuse
pentru clasele a IV-a, a V-a i orice clas mai mare, iar elevii claselor I-IV pot
trimite soluii la problemele propuse pentru oricare din clasele primare i orice clas mai mare. Orice elev poate trimite soluii la problemele de concurs (tip G i L).
3. Vor fi menionate urmtoarele date personale: numele i prenumele,
clasa, coala i localitatea.
4. Plicul cu probleme rezolvate se va trimite prin pot (sau va fi adus
direct) la adresa Redaciei:
Prof. dr. Temistocle Brsan
Str. Aurora, nr. 3, sc. D, ap. 6,
700 474, Iai
Jud. IAI
E-mail: t_birsan@yahoo.com

CUPRINS
O sut de ani de la naterea lui GHEORGHE GHEORGHIEV ..................................... 95
ILIE POPA 100 de ani de la natere.................................................................................... 97
Simpozion dedicat revistei "Recreaii tiinifice" (1883-1888) ........................................... 99

ARTICOLE I NOTE
F. REICHER Despre calendar........................................................................................... 109
M. TETIVA Cteva probleme de teoria numerelor a cror rezolvare
se bazeaz pe identiti ......................... 112
C. IGERU O caracterizare a punctului Mathot ......................................................... 115
D. POPESCU Unsprezece ptrate perfecte ...................................................................... 117
T. BRSAN Cercuri seminscrise i puncte de tip Gergonne sau Nagel..................... 120
F. POPOVICI O rafinare a inegalitii lui Jensen ......................................................... 122
Gh. IUREA Asupra unor inegaliti geometrice ............................................................. 124
T. ZVONARU Metoda deligamentrii i rafinarea unor inegaliti.............................. 125

CHESTIUNI METODICE
Gh. IUREA, G. POPA O problem i nou soluii.................................................. 128

CORESPONDENE
A. REISNER Sur les matrices magiques......................................................................... 131

CONCURSURI I EXAMENE
Concursul de matematic "Al. Myller", ed. a VI-a, 2008.................................................. 134
Concursul de matematic "Florica T. Cmpan", 2008 ....................................................... 136
Concursul "Student pentru o zi", Suceava........................................................................... 140

PROBLEME I SOLUII
Soluiile problemelor propuse n nr. 2/2007........................................................................ 143
Soluiile problemelor pentru pregtirea concursurilor din nr. 2/2007 ............................... 161
Probleme propuse................................................................................................................... 170
Probleme pentru pregtirea concursurilor ............................................................................ 176
Training problems for mathematical contests ..................................................................... 178
Concursul omagial "Recreaii tiinifice" ....................................................................... 181
Pagina rezolvitorilor ............................................................................................................ 183
ISSN 1582 1765

6 lei

You might also like